ſ!.,* * .:-: ·~ ſººſ wriae****** • • • • • • • • • • • • • • • • • • • • • • •-º : *., , ºſi . . . . . . . … :º și , , , , ) » ™~|- •. &-:· . . '،|-·- ----º.|- :| ------, -, as ºs º-º-º: - ºzº...a - a --~~~...~. * & 5-ar-rº, e.g. . … sº-º-º-º-º-º: -------~~~~-º-º-º-º-º-º-º-º-º----------- ~~~~~~ - - >{º ** |-~: ~!|- º , z. * · · · · · · · · · · · · · · · · · · · · · · * * * * * *** •، -، ***********įºstºries,,,,,...}}. (, * * * * *ſ ≡≡ºsºſ; $;':·----~--~!} ,;---&#### , , , .| -2 .: - - '' ''(.*''? !! !! * * * * · * * * *Jº.--. » .vses: , f{{! •-·-º:irae ≡º, §§§) №j ∞rj (*), §§§ ºr EĻĻĽĮŁ sae #№|№]; =; # \\$$$$ ,Nº Ņ . . .6%. ºilº &rºſſ Fº º -ek - ºr sº nº ºzº.… . . Lº sº.-- =z-z-z- …- …-a ºr -º \02, Sºn x^- \& O ST SOUT Eß’s PHILOSOPHIC PRACTICAL MATHEMATICS DESIGNED FOR THE USE OF ACCOUNTANTS, MERCHANTS, BUSINESS MEN, PRIVATE LEARNERS, HIGH GRADE COMMERCIAL COLLEGES AND NORMAL SCHOOLS. THE PHILOSOPHIC SYSTEM Is Presented and Applied to the Solution of COMMERCIAL, FINANCIAL, MECHANICAL and PROFESSIONAL CALCULATIONS and PROBLEMS, covering the various fields of Business Life. { IT SOLVES AND DISCUSSES HUNDREDS OF PRACTICAL QUESTIONS-COMPLEX AND COMPOUND–NEVER BEFORE PRESENTED IN ANY SIMILAR TREATISE. It Sparkles with New Thought, Advanced Methods, and Higher Work. It presents the Rarest Gems of the Science of Practical Mathematics, and it Teaches that a NEW TRUTH is better than an OLD ERROR, however ancient or renowned. By GEO. souT.E. Tractical and Consulting JAccountant, Lecturer on Commercial Sciences, Tresident of Soule Commercial College and Literary Institute, Fellow of the Institute of Accounts of JNew York, &Member of the JAssociated JAccountants of JNew Orleans, and Director in and President of various Corporations. JAuthor of Soule’s “Intermediate Philosophic Arithmetic,” “Contractions in Ö(umbers,” “Thilosophic Drill Problems,” “Analytic and Philosophic, Commercial and Exchange Calculator,” “&Manual of JAuditing,” and “JNew Science and Practice of JAccounts.” FIFTH EDITION, REWISED 1905. PUBLISHED BY THE AUTHOR. NEW ORLEANS. * ENTERED ACCORDING TO ACT OF CONGRESS IN THE YEARS 1872-1895-1898-1901-1905 By GEO. Soulé. IN THE OFFICE OF THE LIBRARIAN OF CONGRESS, AT WASHINGTON ALL RIGHTS RESERVED, COMPOSITION ELECTROTYPED BY BY MALUS & HOFELINE. T. A. SLATTERY. PRINTED AND BOUND BY L. GRAHAM & SON, LTD. PREEYACE. Twenty-four centuries ago the renowned Grecian mathematician, Pythagoras, an– nounced to the world his first great discovery in Mathematics, and since that eventful time, the science of Mathematics has been by master minds enriched with new discoveries until, as a science, it stands the equal of any, and is as unerring in its results as the laws which govern the planets in their revolutions through the realms of space. But, while as a science, Mathematics stands without a peer, and in general usefulness, without a superior, the first, and by far the most generally used subdivision of the science, Arithmetic, is uniformly pre- sented in the text books of the day, by methods which are general'y admitted to be more or less defective in logical reasons for the operations, and in clear elucidations of the subjects. In addition to these defects, the various treatises on the subject, almost without an exception, are deficient in practical problems, and are by far too brief and of too low a grade to meet the commercial and business demands of this age of progress. With a view, therefore, to remedy in a measure the imperfections named, the author presents a thousand page work on Philosophic Practical Mathematics, replete with all known abbreviated methods and with practical problems covering the various fields of trade, finance, mechanics and business all of which are fully elucidated by operations, philosophic reasoning and pertinent discussions. Thus the work supplies a long-felt want, and is of great value as a book of reference and authority for all classes of business men and practical book-keepers. For private learners, teachers, commercial colleges and literary institutions, it is believed to have no equal and to possess the crown of superiority. The Science of Numbers, or Practical Arithemetic, is one of the queenly products of the human mind. With man, it has come down the evolving centuries, compounding itself with his thought and occupation, and unfolding its beauties as the human race advanced on the planes of civilization. Receiving new inspirations and contributions from profound thinkers as the ages passed by, it has attained to the dignity of a queenly science before whose throne the logic of mind and the industries of the world worship. The erudite Hindoo and the learned Egyptian communed at its infant shrine. Before it Thales and Pythagoras reasoned with uncovered heads; at its feet Plato laid the genius of his great mind; and Archimedes and Aristotle employed their eloquence and philosophy in unfold- ing its mysteries. Newton and La Place adorned it with modern thought, and Pestalozzi leavened it with the great principle of analytic reason. But while the mathematicians of ancient fame were enthusiastic regarding numbers, and spent much time in fanciful contemplation and speculation upon their properties, they did but little for the advancement of practical arithmetic. Their numerical recreation and labors were largely with magic squares and circles, while their philosophic talents were bestowed largely upon the noble science of geometry. This science was so highly esteemed by Plato that over the door of his lecture hall he wrote: “Let no one enter here who is ignorant of geometry.” Thus, until the time of Recorde, I500 to 1545; Ramus, I517 to 1572; Stevinus, I550 to I585; Newton, I642 to 1727; Leibnitz, I646 to 1716; and later Pestalozzi, 1746 to 1827, arithmetic did not receive the attention due to its merits, and which modern business methods now demand. The Pestalozzian process of analytic reasoning has been, during the past seventy years, utilized by more than a hundred authors. Colburn, Emerson, Hobart, Stewart, McCormick, Ray, Greenleaf, Dodd, Perkins, Brooks, Davies, Robinson, Thompson and all authors since 1810 have used in various modified forms, more or less, the analytic methods of Pestalozzi, and thus they have moved on with the evolving world and performed a measure of efficient missionary work in the pagan realms of practical mathematics. The analytic method of Pestalozzi is the foundation of the golden Philosophic Sys— tem presented in this and the author's other works on the Science of Numbers. In the higher evolution of this system, the author of this work employs and combines the three great processes of acquiring knowledge and eliciting truth, viz.: comparison, analysis and synthesis. These three processes constitute the trinity of mathematics, and by an ingenious use of them, the author is believed to have advanced the science to loftier planes and to more rational methods than were ever before achieved. In no ancient or modern work on numbers have comparison, analysis and Synthesis been woven into such an extended chain of logical and philosophic reasoning as is herein presented. | 4:4556 IV PREFACE. ... By the Philosophic System, all the difficult scientific statements of true proportion are avoided, all the objections raised against the purely analytic method are obviated, the some— what entangling cause and effect method is surpassed, and that monstrosity of a process which “considers whether the answer is to be greater or less than the third term " is anathematized and consigned to the shades of the dark ages of arithmetic. By the Philosophic System, all the arbitrary rules that overload the faculty of memory and prevent the expansion of the higher faculties of causality and comparison are abandoned, and the reasoning organs of the mind are brought into action, thereby capaci- tating the learner not only to produce the results of problems, but to observe fine distinctions, reason logically, and deduce correctly. Thus like geometry, it serves to enlarge the pow- ers of the mind and to qualify for high planes of usefulness, not only in the fields of mathe- matics, but in all the vocations of life. & The Philosophic System is believed to be the most valuable improvement yet made to impart a thorough knowledge of the principles of numbers and to capacitate the student to utilize the same in the practical affairs of business life. But notwithstanding , its superiority and the fact that its advocates include many of the most profound mathematical minds, yet, like every other improvement or discovery in education, commerce, art, or science, it has some opponents and is regarded with indifference by those who are satisfied with the non-progressive and non-reasoning methods of past ages. For forty-eight years the author has labored with tongue and pen in the develop- ment of the Philosophic System of Arithmetic, and has tested its superior merits in the school room and lecture hall with over 18,000 students; and from a full knowledge of its advantages, he conscientiously assures his co-laborers in the mathematical field of edu– cation, that a more thorough knowledge of the science of numbers can be imparted, and in far less time, by this system than by the usual methods and systems of work. © It stands without a peer in the annals of mathematics, and it is believed that the day is not far distant when its banner will wave in triumph from every spire, pinnacle and dome of the Temple of Practical Mathematics. . The method of disseminating or imparting arithmetical knowledge, that is in almost universal use by the great army of authors and teachers, is the rule method, or in other words, the non-reasoning method. By this method the principles involved are generally lost sight of, and results without reasons are sought for. Being fully convinced of this fact and knowing that the best results are to be attained only by thoroughly understanding the principles of the science, the author has in this treatise labored most earnestly to dethrone the venerable absurdities that have, as he believes, obscured the science of num- bers for ages. The entire work sparkles with the rarest gems of the science of numbers and teaches that a new truth is better than an old error, and that facts and reasons are better than fallacious theories, however ancient or renowned. To obviate a formal introduction to the work, the subject matter usually classed under that head has been presented in remarks and discussions throughout the work, in connection with the various topics treated. By reason of this arrangement, the remarks and discussions have been made more pertinent, and will be read at the time when the learner most needs them. The special features of the work and its many points of superior merit are so num- erous and the number of subjects treated is so great as to preclude the propriety of giving them space in the preface. . A cursory look through the alphabetical index following the preface, will give full information thereof. In the preparation of the work, the author has been guided by an experience of forty-eight years as a teacher and lecturer on the science of numbers and as a practical accountant and commercial lawyer in daily professional intercourse with all classes of busi- ness men; and many of the intricate computations that have been . Submitted to him for adjustment are herein presented and fully elucidated. . These problems and the solution thereof constitute a special and most valuable feature of the work. In concluding the preface, the author desires to extend his thanks to: I. Miss Carrie McGuigin, his faithful amanuensis, for six years of efficient service in the preparation of the copy. s 2. Mr. J. M. Buchee, thirty-five years his sincere friend and co-worker in the cause . of practical education, for his cheerful assistance rendered in proof-reading and in work- ing problems. To him is to be credited the preparation of Involution and Evolution, Square and "Cube Root, Arithmetical and Geometrical Progressions and Annuities. His discus. sions and criticisms on various matters in connection with the work have been valuable and will be kindly remembered. r To his sons, Albert Lee Soulé, Edward Everett Soulé and Frank Soulé, teachers in the Soulé College, for assistance in proof-reading and in working problems. To A. L. Soulé, is to be credited the work of preparing Continued Fractions, Savings Banks, Per— mutations, Combinations. Probabilities and Chances. PREFACE. V In presenting this work to the public, the great tribunal that is to pass judgment upon it, the author, , with the deepest convictions of the superior merit and value of the philosophic system, indulges the hope that it may prove acceptable and he asks, that in behalf of justice, in the interest of progress and the science of numbers, a decision may not be rendered until a thorough examination shall have been made and its merits contrasted with those of other systems. & GEO. SOULE. PREFACE TO EDITION OF 1905. * A A ~–wr-w In 1872 the Author published a work entitled Soulé's Analytic and Philosophic Com— mercial and Exchange Calculator, which comprised about one-half the matter contained in the present work. Five years’ labor was devoted to this treatise and it was received with So much favor by Business Men, Accountants and Learners, that the Author was encour— aged to enlarge the work and to include, elucidate and discuss many practical subjects and problems from various fields of business mathemathics, that were not presented in the first work. The new and enlarged work was given to the public in 1895 under the name of Soulé's Philosophic Practical Mathematics. This work was revised and a new edition pub— lished in 1898. In extent of subjects, in material, in original and in philosophic solutions and discussions of problems, covering the whole field of Practical Mathematics, this work is believed to be far in advance of any similar publication. In 1901 another edition was published with several new problems. The present edition, 1905, has been revised and enriched with the presentation and solution of many new practical problems drawn from different lines of trade, finance, mechanics, agriculture, general business, etc. Large numbers of these problems have been presented to the Author, for solution, by accountants from all sections of the country, and, as they are more or less complex coming from all kinds of business, they are gems of varied hue in the crown of Practical Mathematics. The brilliant conquests of commerce, the amazing magnitude of finance and exchange and the higher demands of business, have produced a need for more contracted and rapid systems for ordinary computations, and for a higher knowledge of the more intricate and complex practical mathematical problems that are daily occurring in business affairs. To meet this demand this work has been prepared and revised from time to time. It is now tendered to the public in its improved and complete form, with the confident hope that it will prove valuable to those for whom it has been written and published. GEO. SOULF New Orleans, January 2, 1905. ALFF/AEET/CAL //VDEX. A. PAGES Abbreviations, list of, ................... . . . . . . . . . . . . . . . . . . . . . . . . . . . . . e e e o e e e e e º e º e º e & Q & Q & P 38–39 Abstract number, def, of,................ . . . . . . . . . . . . . . . . . . . . . . . . . . . .................. to e º 'º º 26 , reasons for multiplying, . . . . . . . . . . . . . . . . . . . . . . . . . . . . . . . . . . . . . . . . . . . . . . . . . 188 , reasons for dividing, . . . . . . . . . . . . . . . . . . . . . . . . . . . . . . . . . . . . . . . . . . . . . . . . . . . . 200 Account sales, def of, . . . . . . . . . . . . . . . . . . . . . . . . . . . . . . . . . . . . . . . . . . . . . . . . . . . . . . . . . . . . . . . . . . . . 480 , equation of . . . . . . . . . . . . . . . . . . . . . . . . . . . . . . . . . . . . . . . . . . . . . . . . . . . . . . . . . . . . . . . . 643 Accounts current and interest accounts, . . . . . . . . . . . . . . . . . . . . . . . . . . . . . . . . . . . . . . . . . . . . . . . . . . . 653–665 showing interest method, product method, and equation method. g Accounts current and interest accounts, in English money, ....... . . . . . . . . . . . . . . . . . . . . . . . . . 507, 661 , With brokers, . . . . . . . . . . . . . . . . . . . . . . . . . . . . . . . . . . . . 705 , in German money, . . . . . . . . . . . . . . . . . . . . . . . . . . . . . . . 664 Actuaries’ table for life insurance, ......................................................... 536–539 4ddition, civil service method of, .................................................... • * * * * * * * * 46 def of . . . . . . . . . . . . . . . . . . . . . . . * * * * * * * * * * e s s e e s e e s e e s e º e º e s e e s e s e e s s e e s e e s a c e s a e º 'º' 40 grouping the figures in, . . . . . . . . . . . . . . . . . . . . . . . . . . . . . . . . . . . . . . . . . . . . . . . . . . . . . . . . . 48 horizontal, . . . . . . . . . . . . . . . . . . . . . . . . . . . . . . . . . . . . . . . . . . . . . . . . . . . . . . . . . . . . . . . . . . . . . . 49 of decimals, ......... * . . . . . . . . . . . . . . . . . . . . . . . . . . . . . . . . . . . . . . . . . . . . . . . . . . . . . . . . . . . 223 of denominate numbers, ... . . . . . . . . . . . . . . . . . . . . . . . . . . . . . . . . . . . tº ſº º e º ſº e º e º ſº ºn tº gº © tº & © tº 267–269 of dollars and cents, . . . . . . . . . . . . . . . . . . . . . . . . . . . . . . . . . . . . . . . . . . . . . . . . . . . . . . . . . . . 52 of fractions, . . . . . . . . . . . . . . . . . . . . . . . . . . . . . . . . . . . . . . . . . . . . . . . . . . . . . . . . . . . . . . . . . . . . 175–180 of metric numbers, . . . . . . . . . . . . . . . . . . . . . . . . . . . . . . . . . . . . . . . . . . . . . . . . . . . e ſº tº e º e º ºs º ºs º 936 of several columns at once, . . . . . . . . . . . . . . . . . . . . . . . . . . . . . . . . . . . . . . . . . . . . . . . . . . . . . . 53 • Principle of . . . . . . . . . . . . . . . . . . . . . . . . . . . . . . . . . . . . . . . . . . . . . . . . . . . . . . . . . . . . . . . . . . . . 40 Proof of . . . . . . . . . . . . . . . . . . . . . . . . . . . . . . . . . . . . . . . . . . . . . . . . . . . . . . . . . . . . . . . . . . . . . . . . 47 , tables, . . . . . . . . . . . . . . . . . . . . . . . . . . . . . . . . . . . . . . e = s. sº tº e s e º ºs e º 'º e º ſº º e º e º e º ſº e º 'º e º 'º e º 'º 41, 42 Vicenary system of . . . . . . . . . . . . . . . . . . . . . . . . . . . . . . . . . . . . . . . . . . . . . . . . . . . . . . . . . . . . . . 56 Adjustment of interest on partners’ accounts, .............................................. 845 “ the calendar, history of . . . . . . . . . . . . . . . . . . . . . . . . . . . . . . . . . . . . . . . . . . . . . . . . . . . . 237 “ losses in insurance, . . . . . . . . . . . . . . . . . . . . . . . . . . . . . . . . . . . . . . . . . . . . . . . . . . . . 512, 523, 525 “ insurance under the # loss clause, ........................................... 524 4 ( & & & 4 “ # Value clause, . . . . . . . . . . . . . . . . . . . . . . . . . . . . . . . . . . . . . . . . . 525 & 4 “ “ “ 75% co-insurance clause,.............................. 524 & & & 4 & 4 “ average clause, . . . . . . . . . . . . . . . . . . . . . . . . . . . . . . . . . . . . . . . . 524 & 4 4 & marine loss, . . . . . . . . . . . . . . . . . . . . . . . . . . . . . . . . . . . . . . . . . . . . . . . . . . . . . . 524 ( & & & mdse. loss, full insurance, . . . . . . . . . . . . . . . . . . . . . . . . . . . . . . . . . . . . . . . . 525–526 Admission of new partners, . . . . . . . . . . . . . . . . . . . . . . . . . . . . . . . . . . . . . . . . . . . . . . . . . . . . . . . . . . . . . 858 Ad-Valorem duty, def. of, . . . . . . . . . . . . . . . . . . . . . . . . . . . e e º e e s is e e s e e º sº e e s a e s is is e a e e s tº e º ºs e º sº * * * 500 Aliquots, table of . . . . . . . . . . . . . . . . . . . . . . . . . . . . . . . . . . . . . . . . . . . . . . . . . . . . . . . . . . . . . . . . . . . . . . . . . 94, 139 Alligation, alternate and medial by several methods, . . . . . . . . . . . . . . . . . . . . . . . . . . © tº gº & s tº tº e º ºs e & 789–803 American experience table in insurance, . . . . . . . . . . . . . . . . . . . . . . . . . . . . . . . . . . . . . . . . . ... . .536–538, 541 Angles, def of . . . . . . . . . . . . . . . . . . . . . . . . . . . . . . . . . . . . . . . . . . . . . . . . . . . . . . . . . . . . . . . . . . . . . . . . . . . . 337 Angular measure, . . . . . . . . . . . . . . . . . . . . . . . . . . . * * * * * * * * * * * * * * * * * * * * * * * * * * * * * * * * * * * * * * * * * * * * * 252 Annual interest, . . . . . . . . . . . . . . . . . . . . . . . . . . . . . . . . . . . . . . . . . . . . . . . . . . . . . . . . . . . . . . . . . . . . . . . . . . . 607 ALPHABETICAL INDEX, VII PAGES. Annuities, ....... * @ e º 'º 6 e e º e º e & o e º e º & G & o e - to e º e º e e e º e º e e º e © e º e º 6 & e s is © & © e > * > 0 ºr a e s a e e s e s e e º e e º e 920–928 Apothecaries fluid measure, .................. & © tº c & © e º e º e o e * * * @ 9 • e º e e s e o e º e e'• * - - - - - - - - - 251 Weight, . . . . . . . . . . . . . . . . . . . . . . . . . . . . . . . . . . . . . . . . . . . . . . . . . . . . . . . . . . . . . . . . . . . . . 250 Arabic notation, . . . . . . . . . . . . . . . . . . . . . . . . . . . . . . . . . . . . . . . . . . . . . . . . . . . . . . . . . . . . . . . . . ....... 30 Arbitration of exchange, . . . . . . . . . . . . . . . . . . . . . . . . . . . . . . . . . . . . . . . . . . . . . . . . . . . . . . . . . . . . . . 736–743, 774 Area, def of . . . . . . . . . . . . . . . . . . . . . . . . . . . . . . . . . . . . . . . . . . . . . . . . . . . . . . . . . . . . . . . . . . . . . . . . . . . . . . 338 Argentine republic exchange, . . . . . . . . ...................................................... 771 Arithmetic, def. of . . . . . . . . . . . . . . . . . . . . . . . . . . . . . . . . . . . . . . . . . . . . . . . . . . . . . . . . . . . . . . . . . ....... 27 , philsophic system of, .......... • * * * - - - - - - - - - - - - - - - - - - - - - - - - - - - - - - - - - - - - - - . . . . . . 26 Arithmetical complement, . . . . . . . . . . . . . . . . . . . . . . . . . . . . . . . . . . . . . . . . . . . . . . . . . . . . . . . . . . . . . . . . 26 supplement, . . . . . . . . . . . . . . . . . . . . . . . . . . . . . . . . . . . . . . . . . . . . . . . . . . . . . . . . . . . . . . . . 26 units of order,. . . . . . . . . . . . . . . . . . . . . . . . . . . . . . . . . . . . . . . . . . . . . . . . . . . . . . . . . . . . . 26 Progression, . . . . . . . . . . . . . . . . . . . . . . . . . . . . . . . . . . . . . . . . . . . . . . . . . . . . . . . . . . . . . . . . 901–908 Articles of agreement, . . . . . . . . . . . . . . . . . . . . . . . . . . . . . . . . . . . . . . . . . . . . . . . . . . . . . . . . . . . . . . . . . . . . . 806 Artificiers' work in surfaces,............................................................... 360 Assayers' weight, ..., . . . . . . . . . to º e º 'º e g º º & © e º e º 'º e º e º 'º e s e e º e e s e e s e s - e. e. e. e. e. e. e. e s e e o e s e e o e º e e s e e e 251 Assessment roll, def. of . . . . . . . . . . . . . . . . . . . . . . . . . . . . . . . . . . . . . . . . . . . . . . . . . . . . . . . . . . . . . . . . . . 489, 677 Assets, def of . . . . . . . . . . . . . . . . . . . . . . . . . . . . . . . . © e º 0 & 9 e º 'º e º 'º e º 3 s • e s e e s e e º ºs e s e < e e s e s sº e e º e e s e e 487 Assignee, def of, . . . . . . . . . . . . . . . . . . . . . . . . . . . . . . . . . . . tº gº e º e º e º e º 'º - e. e º e º e º e º e s e º e • e s tº e º e º is º e º e 487 Assignment, def. of . . . . . . . . . . . . . . . . . . . . . . . . . . . . . . . . & © tº e º 'º e º e o dº º e s e e º ºs e º º v c e e º e s e e s e º e º e s a s 487 Austrian exchange, . . . . . . . . . . . . . . . . . . . . . . . . . . . . . . © e o 'º e e º e e e s e e º e e e s e e º ºs e e e s e e e º e s e o a e e º e s e e 767 Average adjuster in insurance, ... . . . . . . . . . . . . . . . . . . . . . © e o e º e º 'º e e g º e º e º e s e º e º a c e s e e e o e s e o e 527 “Average Clause” in insurance, . . . . . . . . . . . . . . . . . . . . . . '..................................... 512 Average of cotton, . . . . . . . . . . . . . . . . . . . . . . . . . . . . . . . . . . . . . . . . . . . . . . . . . . . . . . . . . . . . . . . . . . . . . . 868 of payments or accounts, . . . . . . . . . . . . . . . . . . . . . . . . . . . . . . . . . . . . . . . . . . . . . . . . . . . . . .627—653 , simple and compound partnership, . . . . . . . . . . . . . . . . . . . . . . . . . . . . . . . . . . . . . . . . . . . . . . . 861 storage, . . . . . . . . . . . . . . . . . . . . . . . . . . . . . . . . . . . . . . . . . . . . . . . . . . . . . . . . . . . . . . . . . . . . . . . . . 778 Averaging sales in commission, . . . . . . . . . . . . . . . . . . . . . . . . . . . . . . . . . . . . . . . . . . . . . . . . . . . . . . . . . . . . 867 Axiom, def of . . . . . . . . . . . . . . . . . . . . . . . . . . . . . . . . . . . . . . . . . . . . . . . . . . . . . . . . . . . . . . . . . . . . . . . . . . . . 26 E Balance of trade, . . . . . . . . . . . . . . . . . . . . . . . . . . . . . . . . . . . . . . . . . . . . . . . . . . . . . . . . . . . . . . . . . . . . . . . . . . 711 Bankers’ interest on daily balances, . . . . . . . . . . . . . . . . . . . . . . . . . . . . . . . . . . . . . . . - - - - - - - - - - - - - - - - 665 method of interest, . . . . . . . . . . . . . . . . . . . . . . . . . . . . . . . . . . . . . . . . . . . . . . . . . . . . . . . . . . . . . . 571 and merchants' discount, . . . . . . . . . . . . . . . . . . . . . . . . . . . . . . . . . . . . . . . . . . . . . . . . . . . . . . . . . 576 Bankruptcy, . . . . . . . . . . . . . . . . . . . . . . . . . . . . . . . . . . . . . . . . . . . . . . . . . . . . . . . . . . . . . . . . . . . . . . . . . . . . . . 487–489 Base, def. of . . . . . . . . . . . . . . . . . . . . . . . . . . . . . . . . . . . . . . . . . . . . . . . . . . . . . . . . . . . . . . . . . . . . . . . . . . . . . . 434 Basis of life insurance, . . . . . . . . . . . . . . . . . . . . . . . . . . . . . . . . . . . . . . . . . . . . . . . . . . . . . . . . . . . . . . . . . . . . 536 Belgium exchange, . . . . . . . . . . . . . . . . . . . . . . . . . . . . . . . . . . . . . . . . . . . . . . . . . . . . . . . . . . . . . . . . . . . . . . . . 764 Benevolent insurance associations, . . . . . . . . . . . . . . . . . . . . . . . . . . . . . . . . . . . . . . . . . . . . . . . . . . . . . . . . . 535 Bill for storage, . . . . . . . . . . . . . . . . . . . . . . . . . . . . . . . . . . . . . . . . . . . . . . . . . . . . . . . . . . . . . . . . . . . . . . . . . . . 784 Bills, short sight, bankers, commercial, bill of lading, a 1 bills, prime bills, . . . . . . . . . . . . . . . . 716 and invoices, . . . . . . . . . . . . . . . . . . . . . . . . . . . . . . . . . . . . . . . . . . . . . . . . . . . . . . . . . . . . . . . . . . . . . . . . 283–296 of exchange, forms and defs. of, . . . . . . . . . . . . . . . . . . . . . . . . . . . . . . . . . . . . . . . . . . . . . . . . . . . . . 713 Binary scale, . . . . . . . . . . . . . . . . . . . . . . . . . . . . . . . . . . . . . . . . . . . . . . . . . . . . . . . . . . . . . . . . . . . . . . . . . . . . . . 27 “Black” method of interest, . . . . . . . . . . . . . . . . . . . . . . . . . . . . . . . . . . . . . . . . . . . . . . . . . . . . . . . . . . . . . . 574 Boards, lumber, timber and logs, measured, . . . . . . . . . . . . . . . . . . . . . . . . . . cº-oº º e s e º 'º - e & a tº e º e º e º e e 384, 388 Bonded Warehouses, . . . . . . . . . . . . . . . . . . . . . . . . . . . . . . . . . . . . . . . . . . . . . . . . . . . . . . . . . . . . . . . . . . . . . . . 501 Bonds and stocks fully treated (see stocks and bonds), . . . . . . . . . . . . . . . . . . . . . . . . . . . . . . . . . . . . . 676–710 Books and paper table, . . . . . . . . . . . . . . . . . . . . . . . . . . . . . . . . . . . . . . . . . . . . . . . . . . . . . . . . . . . . . . . . A. e. e. e. 254 VIII ALPHABETICAL INDEX. PAGES. Porrowers and lenders, .................................................................... 546 Borrowing method in subtraction, ......................................................... 66 Bottomry and respondentia,............................................................... 512 Brazilian exchange, ............................... . . . . . . . . . . . . . . . . . . . . . . . . . . . . . . . . . . . . . . . . 768 invoice of coffee, . . . . . . . . . . . . . . . . . . . . . . . . . . . . . . . . . . . . . . . . . . . . . . . . . . . . . . . . . . . . . . . . 505–506 Brokerage and commission, . . . . . . . . . . . . . ................ . . . . . . . . . . . . . . . . . . ................. 480–487 on “cash notes”. . . . . . . . . . . . . . . . . . . . . . . . . . . . . . . . . . . . . . . . . . . . . . . 584 Budget, def. of . . . . . . . . . . . . . . . . . . . . . . . . . . . . . . . . . . . . . . . . . . . . . . . . . . . . . . . . . . . . . . . . . . . . . . . . . . . 489 Buying and selling exchange, ...... © tº e º 'º º e º e s e º 'º e º 'º e s e º e º 'º e º e s e º e s a s e e s a e s e s e e s s a s a e s is a e e s e 717 Buying exchange on own account, ... . . . . . . . . . . . . . . . . . . . . . . . . . . . . . . . . . . . . . . . . . . . . . . . . . . . . . . 717 & 4 through agent, . . . . . . . . . . . . . . . . . . . . . . . . . . . . . . . . . . . . . . . . . . . . . . . . . . . . . . . . . . 718 é & & & “ and broker, . . . . . . . . . . . . . . . . . . . . . . . . . . . . . . . . . . . . . . . . . . . . . . 718 Building and loan associations, . . . . . . . . . . . . . . . . . . . . . . . . . . . . . . . . . . . . . . . . . . . . . . . . . . . . . . . . . . . 977–1004 Building and loan associations, history and definitions; kinds of members; issuing of shares; premiums or discount and net loans; dividends and withdrawals; four leading plans—the Terminating, Serial, Permanent, and Dayton plans, defined and discussed; gross, met, installment, and interest in advance plans of charging interest and premiums on loans discussed; State and United States leagues; National building associations; plans of loaning money; plans of with- drawal; plans of distribution of profits; problem by the gross plan, worked in full; different methods of some leading New Orleans associations. Problems worked and explained to illustrate the following important points: 1. To find the rate per cent of interest received by a non-borrowing member on maturing shares. 2. To find the rate per cent of interest paid by a borrower, on the net plan. 3. To find the gain per cent realized by building and loan associations on investment of dues. at simple interest. 4. To find the per cent realized by building and loan associations on investment of dues at simple interest weekly. 5. To find average annual gain per cent realized on weekly payments at simple interest, and annual amounts to be put at compound interest to close of period. 6. To find total cost of a loan to a borrower at 6 per cent simple interest, Gross plan, with monthly payments of $1 per share. 7. To find actual premium charged on loans and refunded on their payment. 8. To find average annual gain per cent realized on monthly payments for each year, and the annual amounts put at compound interest to close of period. Four solutions given, embracing the compound interest table, annuity table, and logarithms. r 9. To find the length of time required by a purchaser to pay for a piece of property by monthly payments including the interest. Also required to find the number of payments to be made, total amount of interest, average per cent paid, and gain made by the company. 10. To find the face of mortgage notes, payable in successive months, to pay for a certain loan. 11. Miscellaneous problems. ALPHABETICAL INDEX. IX C: PAGES Calendar, adjustment of, ............. gº tº tº e º 'º ſº tº e º 'º - e º & Cº tº gº tº e º 'º º º tº dº º ſº tº e º e o 'º e º e º º e º º º e e e g º ºs e e º e 237 def of . . . . . . . . . . . . . . . . . . . . . . . . . . . . . . . . . . . . . . . . . . . . . . . . . . . . . . . . . . . . . . . . . . . . . . . . . 239 *money, ............................................................................ 234 *tion, .............................................................................. 161–164 9°pital stock dividend,.................................................................... 678 Carlisle table in life insurance, .......................................... • - - - - - - - - - - - - - - - - - 536–539 9*Peting floors, . . . . . . . . . . . . . . . . . . . . . . . . . . . . . . . . . . . . . . . . . . . . . . . . . . . . . . . . . . . . . . . . . . . . . . . . . . 368 Cash balances, . . . . . . . . . . . . . . . . . . . . . . . . . . . . . . . . . . . . . . . . . . . . . . . . . . . . . . . . . . . . . . . . . . . . . . . . . . . . 653 dividend, . . . . . . . . . . . . . . . . . . . . . . . . . . . . . . . . . . . . . . . . . . . . . . . . . . . . . . . . . . . . . . . . . . . . . . . . . . . . 678 storage, . . . . . . . . . . . . . . . . . . . . . . . . . . . . . . . . . . . . . . . . . . . . . . . . . . . . . . . . . . . . . . . . . . . . . . . . . . . . . 777 “Cash notes,” with commission and brokerage, ............................................ 582–589 and exchange combined, . . . . . . . . . . . . . . . . . . . . . . . . . . . . . . . . . . . . . . . . . . . . . . . . . . . . 732 Certificates of stock,...................................................................... 676 Chain measure, . . . . . . . . . . . . . . . . . . . . . . . . . . . . . . . . . . . . . . . . . . . . . . . . . . . . . . . . . . . . . . tº e e º 'º e º e º ei e º gº 244. Character and face of foreign bills,..... . . . . . . . . . . . . . . . . . . . . . . . . . . . . . . . . . . . . . . . . . . . . . . . . . . . 715 Check figure, use of . . . . . . . . . . . . . . . . . . . . . . . . . . . . . . . . . . . . . . -- . . . . . . . . . . . . . . . . . . . . . . . . . . . . . . 152 Chilian exchange, . . . . . . . . . . . . . . . . . . . . . . . . . . . . . . . . . . . . . . . . . . . . . . . . . . . . . . . . . . . . . . . . . . . . . . . . . 771 Chinese exchange, . . . . . . . . . . . . . . . . . . . . . . . . . . . . . . . . . . . . . . . . . . . . . . . . . . . . . . . . . . . . . . . . . . . . . . . . . 774 Circle, def of . . . . . . . . . . . . . . . . . . . . . . . . . . . . . . . . . . . . . . . . . . . . . . . . . . . . . . . . . . . . . . . . . . . . . . . . . . . . . 338 Circular measure, with deſs......................................... tº º º tº e º 'º tº e º 'º º º ſº e º e º 'º º © Cº tº 252 Clandestine stock, . . . . . . . . . . . . . . . . . . . . . . . . . . . . . . . . . . . . . . . . . . . . . . . . . . . . . . . . . . . . . . tº º ſº e º ſº e º 'º e 679 Classification of fractions,. . . . . . . . . . . . . . . . . . . . . . . . . . . . . . . . . . . . . . . . . . . . . . . . . . . . . . tº º tº e º e º e º e ºs 165. Clearance, def. of . . . . . . . . . . . . . . . . . . . . . . . . . . . . . . . . . . . . . . . . . . . . . . . . . . . . . . . . . . . . . . . . . . . . . . . . . 501 Cloth measure, . . . . . . . . . . . . . . . . . . . . . . . . . . . . . . . . . . . . . . . . . . . . . . . . . . . . . . . . . . . . . . . . . . . . . . . . . . . . 243 “Co-insurance clause,”. . . . . . . . . . . . . . . . . . . . . . . . . . . . . . . . . . . . . . . . . . . . . . . . . . . . . . . . . . . . . . . . . . . . 513 Cold storage, . . . . . . . . . . . . . . . . . . . . . . . . . . . . . . . . . . . . . . . . . . . . . . . . . . . . . . . . . . . . . . . . . . . . . . . . . . . . . . 777 Colombian exchange, . . . . . . . . . . . . . . . . . . . . . . . . . . . . . . . . . . . . . . . . . . . . . . . . . . . . . . . . . . . . . . . . . . . . . . 771 Collateral note with margin, . . . . . . . . . . . . . . . . . . . . . . . . . . . . . . . . . . . . . . . . . . . . . . . . . . . . . . . . . . . . . . 958 Combinations and permutations and chance, ... . . . . . . . . . . . . . . . . . . . . . . . . . . . . . . . . . . . . . . . 943, 944, 945 Commercial instruments of Writing, forms of, . . . . . . . . . . . . . . . . . . . . . . . . . . . . . . . . . . . . . . . . . . . . . . 551–556 Commission, averaging sales in, . . . . . . . . . . . . . . . . . . . . . . . . . . . . . . . . . . . . . . . . . . . . . . . . . . . . . . . . . . . 867 on “cash notes,”. . . . . . . . . . . . . . . . . . . . . . . . . . . . . . . . . . . . . . . . . . . . . . . . . . . . . . . . . . . . . 584 and brokerage fully treated, . . . . . . . . . . . . . . . . . . . . . . . . . . . . . . . . . . . . . . . . . . . . . . . . . . 480–487 Common fractions, def. of . . . . . . . . . . . . . . . . . . . . . . . . . . . . . . . . . . . . . . . . . . . . . . . . . . . . . . . . . . . . . . . . . 165 Companies, kinds of insurance, . . . . . . . . . . . . . . . . . . . . . . . . . . . . . . . . . . . . . . . . . . . . . . . . . . . . . . . . . . . . 511 Comparative weights, measures and values, . . . . . . . . . . . . . . . . . . . . . . . . . . . . . . . . . . . . . . © e s e e º e s a e 254 Comparison, def. of . . . . . . . . . . . . . . . . . . . . . . . . . . . . . . . . . . . . . . . . . . . . . . . . . . . . . . . . . . . . & © e o e s & e e e e 26 of simple, annual and compound interest, . . . . . . . . . . . . . . . . . . . . . . . . . . . . . . . . . . . . 615. of the foot of different nations, . . . . . . . . . . . . . . . . . . . . . . . . . . . . . . . . . . . . . . . . . . . . . . . 253 of thermometers complete, . . . . . . . . . . . . . . . . . . . . . . . . . . . . . . . . . . . . . . . . . . . . . . . . . . . 274–277 Complement of number, def. of, . . . . . . . . . . . . . . . . . . . . . . . . . . . . . . . . . . . . . . . . . . . . . . . . . . . . . . . . . . . 26 ( & “ multiply by, . . . . . . . . . . . . . . . . . . . . . . . . . . . . . . . . . . . . . . . . . . . . . . . . . . . . . . 108 Complex fraction, def. of . . . . . . . . . . . . . . . . . . . . . . . . . . . . . . . . . . . . . . . . . . . . . . . . . . . . . . . . . . . . . . . . . 166 Composite factor, def. of, . . . . . . . . . . . . . . . . . . . . . . . . . . . . . . . . . . . . . . . . . . . . . . . . . . . . . . . . . . . . . . . . . . 146 number, def. of . . . . . . . . . . . . . . . . . . . . . . . . . . . . . . . . . . . . . . . . . . . . . . . . . . . . . . . . . . . . . . . . 146 Compound denominate numbers complete, . . . . . . . . . . . . . . . . . . . . . . . . . . . . . . . . . . . . . . . . . . . . . . . . . 229–278 equation, . . . . . . . . . . . . . . . . . . . . . . . . . . . . . . . . . . . . . . . . . . . . . . . . . . . . . . . . . . . . . . . . . . . . . . . 627 fraction, . . . . . . . . . . . . . . . . . . . . . . . . . . . . . . . . . . . . . . . . . . . . . . . . . . . . . . . . . . . . . . . . . . . . . . . 165 interest, . . . . . . . . . . . . . . . . . . . . . . . . . . . . . . . . . . . . . . . . . . . . . . . . . . . . . . . . . . . . . . . . . . . . . 609, 917 interest tables, . . . . . . . . . . . . • . . . . . . . . . . . . . . . . . . . . . . . . . . . . . . . . . . . . . . . . . . . . . . 560, 561, 576 Check figure proof • e º e º e º ºs e º e º e º 'º - e º e º e º ºs º gº tº gº tº e º 'º º e o e º c . . . . . . . . . . . . . . . . . . . . . . . . . . . . . . . .1005-1008 X ALPHABETICAL INDEX. PAGES Compound number, . . . . . . . . . .… tº e º s e º e 26 Proportion, . . . . . . . . . . . . . . . . . . . . . . . . . . . . . . . . . . . . . . . . . . . . . . . . . . . . . . . . . . . . . . . . . . . . . 312 Computing interest, universal formula for, ...................................... A c s e s e e s e s a 561 99ncrete number, . . . . . . . . . . . . . . . . . . . . . . . . . . . . . . . . . . . . . . . . . . . . . . . . . . . . . . . . . . . . . . . . . . . . . . . . . 26 99ne, def of . . . . . . . . . . . . . . . . . . . . . . . . . . . . . . . . . . . . . . . . . . . . . . . . . . . . . . . . . . . . . . . . . . . . . . . . . . . . . . 370 Conjoined proportion, . . . . . . . . . . . . . . . . . . . . . . . . . . . . . . . . . . . . . . . . . . . . . . . . . . . . . . . . . . . . . . . . . . . . . 312, 333 Connecticut system of partial payments, .................................................. 618 Consignee, . . . . . . . . . . . . . . . . . . . . . . . . . . . . . . . . . . . . . . . . . . . . . . . . . . . . . . . . . . . . . . . . . . . . . . . . . . . . . . . 480 Consignment, . . . . . . . . . . . . . . . . . . . . . . . . . . . . . . . . . . . . . . . . . . . . . . . . . . . . . . . . . . . . . . . . . . . . . . . . . . . . . 480 Consignor, . . . . . . . . . . . . . . . . . . . . . . . . . . . . . . . . . . . . . . . . . . . . . . . . . . . . . . . . . . . . . . . . . . . . . . . . . . . . . . . . 480 Consols. . . . . . . . . . . . . . . . . . . . . . . . . . . . . . . . . . . . . . . . . . . . . . . . . . . . . . . . . . . . . . . . . . . . . . . . . . . . . . . . . . . . 680 Continued fractions,. . . . . . . . . . . . . . . . . . . . . . . . . . . . . . . . . . . . . . . . . . . . . . . . . . . . . . . . . . . . . . . . . . . . . . . 949 Contractions in division, . . . . . . . . . . . . . . . . . . . . . . . . . . . . . . . . . . . . . . . . . . . . . . . . . . . . . . . . . . . . . . . . . . 138–142 in multiplication, ... . . . . . . . . . . . . . . . . . . . . . . . . . . . . . . . . . . . . . . . . . . . . . . . . . . . . . . . . . 88–121 in multiplication of fractions,................................................ 190–197 in practical problems, . . . . . . . . . . . . . . . . . . . . . . . . . . . . . . . . . . . . . . . . . . . . . . . . 96–101, 278–283 in subtracting dollars and cents, . . . . . . . . . . . . . . . . . . . . . . . . . . . . . . . . . . . . . . . . . . . . . 68 Contracting methods of vicenary addition, ... . . . . . . . . . . . . . . . . . . . . . . . . . . . . . . . . . . . . . . . . . . . . . 58 Conventional interest, . . . . . . . . . . . . . . . . . . . . . . . . . . . . . . . . . . . . . . . . . . . . . . . . . . . . . . . . . . . . . . . . . . . . . 547 Co-operative insurance associations, . . . . . . . . . . . . . . . . . . . . . . . . . . . . . . . . . . . . . . . . . . . . . . . . . . . . . . . 535 Corporation, def. of . . . . . . . . . . . . . . . . . . . . . . . . . . . . . . . . . . . . . . . . . . . . . . . . . . . . . . . . . . . . . . . . . . . . . . . 676 Cost of imported goods, . . . . . . . . . . . . . . . . . . . . . . . . . . . . . . . . . . . . . . . . . . . . . . . . . . . . . . . . . . . . . . . . . . . 752 Costa Rican exchange, . . . . . . . . . . . . . . . . . . . . . . . . . . . . . . . . . . . . . . . . . . . . . . . . . . . . . . . . . . . . . . . . . . . . . 771 Cotton average, . . . . . . . . . . . . . . . . . . . . . . . . . . . . . . . . . . . . . . . . . . . . . . . . . . . . . . . . . . . . . . . . . . . . . . . . . . . 868 Coupon bonds, . . . . . . . . . . . . . . . . . . . . . . . . . . . . . . . . . . . . . . . . . . . . . . . . . . . . . . . . . . . . . . . . . . . . . . . . * * * * 679 Country clause in insurance policy, . . . . . . . . . . . . . . . . . . . . . . . . . . . . . . . . . . . . . . . . . . . . . . . . . . . . . . . . 514 Credit storage, . . . . . . . . . . . . . . . . . . . . . . . . . . . . . . . . . . . . . . . . . . . . . . . . . . . . . . . . . . . . . . . . . . . . . . . . . . . . 777 Creditor, . . . . . . . . . . . . . . . . . . . . . . . . . . . . . . . . . . . . . . . . . . . . . . . . . . . . . . . . . . . . . . . . . . . . . . . . . . . . . . . . . . 487 Cross multiplication, . . . . . . . . . . . . . . . . . . . . . . . . . . . . . . . . . . . . . . . . . . . . . . . . . . . . . . . . . . . . . . . . . . . . . . 88 Cuban exchange, . . . . . . . . . . . . . . . . . . . . . . . . . . . . . . . . . . . . . . . . . . . . . . . . . . . . . . . . . . . . . . . . . . . . . . . . . . 771 Cube, def. of . . . . . . . . . . . . . . . . . . . . . . . . . . . . . . . . . . . . . . . . . . . . . . . . . . . . . . . . . . . . . . . . . . . . . . . . . . . . . 370 root, . . . . . . . . . . . . . . . . . . . . . . . . . . . . . . . . . . . . . . . . . . . . . . . . . . . . . . . . . . . . . . . . . . . . . . . . . . . . . . . 891 Cubic measure, . . . . . . . . . . . . . . . . . . . . . . . . . . . . . . . . . . . . . . . . . . . . . . . . . . . . . . . . . . . . . . . . . . . . . . . . . . . . 247 Currency, gold, silver, and uncurrent money, . . . . . . . . . . . . . . . . . . . . . . . . . . . . . . . . . . . . . . . . . . . . . . 493–500 Customhouse business, . . . . . . . . . ‘. . . . . . . . . . . . . . . . . . . . . . . . . . . . . . . . . * * * - - - - - - - - - - - - - - - - - - - - - - - 500–508 Customs, def. of, . . . . . . . . . . . . . . . . . . . . . . . . . . . . . . . . . . . . . . . . . . . . . . . . . . . . . . . . . . . . . . . . . . . . . . . . . . 500 in interest and discount, . . . . . . . . . . . . . . . . . . . . . . . . . . . . . . . . . . . . . . . . . . . . . . . . . . . . . . . . . 556 Cylinder, def. of, . . . . . . . . . . . . . . . . . . . . . . . . . . . . . . . . . . . . . . . . . . . . . . . . . . . . . . . . . . . . . . . . . . . . . . . . . . 370 mensuration of . . . . . . . . . . . . . . . . . . . . . . . . . . . . . . . . . . . . . . . . . . . . . . . . . . . . . . . . . . . . . . . . . 374 I O. Daily balances, interest on, . . . . . . . . . . . . . . . . . . . . . . . . . . . . . . . . . . . . . . . . . . . . . . . . . . . . . . . . . . . . . . . 665 Danish exchange, . . . . . . . . . . . . . . . . . . . . . . . . . . . . . . . . . . . . . . . . . . . . . . . . . . . . . . . . . . . . . . . . . . . . . . . . . 773 Days of grace, . . . . . . . . . . . . . . . . . . . . . . . . . . . . . . . . . . . . . . . . . . . . . . . . . . . . . . . . . . . . . . . . . . . . . . . . . . . . 549 Debentnre insurance or investment, def. of, . . . . . . . . . . . . . . . . . . . . . . . . . . . . . . . . . . . . . . . . . . . . . . . . 535 stock, . . . . . . . . . . . . . . . . . . . . . . . . . . . . . . . . . . . . . . . . . . . . . . . . . . . . . . . . . . . . . . . . . . . . . . . . . 677 Debtor, . . . . . . . . . . . . . . . . . . . . . . . . . . . . . . . . . . . . . . . . . . . . . . . . . . . . . . . . . . . . . . . . . . . . . . . . . . . . . . . . . . . 487 Decimal fractions, def. of, . . . . . . . . . . . . . . . . . . . . . . . . . . . . . . . . . . . . . . . . . . . . . . . . . . . . . . . . . . . . . . . . . 165, 216 fractions treated in full, . . . . . . . . . . . . . . . . . . . . . . . . . . . . . . . . . . . . . . . . . . . . . . . . . . . . . . . . . . 216–229 scale changed to other scales, . . . . . . . . . . . . . . . . . . . . . . . . . . . . . . . . . . . . . . . . . . . . . . . . . . . . . 28, 29 ALPHABETICAL INDEX. XI PAGES. Definition, * * * * * * * * * * * * * * * * * * * * * * * * * * * * * * s a e e s a e s e e s s e a s • * * * * * * * * * * * * * * * * * * * * a e º e º e s e < * * * * * 25 Demonstration, def. of . . . . . . . . . . . . . . . . . . . . . . . . . . . . . . . . . . . . . . . . . . . . . . . . . . . . . . . . . . . . . . . . . • * * 26 Denominate number, def. of, . . . . . . . . . . . . . . . . . . . . . . . . . . . . . . . . . . . . . . . . . . . . . . . . . . . . . . . . . . . . . . 26, 229 numbers treated in full, . . . . . . . . . . . . . . . . . . . . . . . . . . . . . . . . . . . . . . . . . . . . . . . . . . . . . . .229–278 Penominator, . . . . . . . . . . . . . . . . . . . . . . . . . . . . . . . . . . . . . . . . . . . . . . . . . . . . . . . . . . . . . . . . . . . . . . . . . 165 Derivation of names of months, . . . . . . . . . . . . . . . . . . . . . . . . . . . . . . . . . . . . . . . . . . . . . . . . . . . . . . . . . . 240 of year, month, week, day, etc. . . . . . . . . . . . . . . . . . . . . . . . . . . . . . . . . . . . . . . . . . . . . . 239 Determining gain or loss of business, . . . . . . . . . . . . . . . . . . . . . . . . . . . . . . * * * * * * * * * * * * * * * * * * * 809 Diagonal, def. of . . . . . . . . . . . . . . . . . . . . . . . . . . . . . . . . . . . . . . . . . . . . . . . • * * * * * * * * * * * * * * * * * * * * * 337 rods, use of . . . . . . . . . . . . . . . . . . . . . . . . . . . . . . . . . . . . . . . . . . . . . . . a s a s tº e º e º & © tº e º e º e e 413 Diagram of United States lands divided into townships, . . . . . . . . . . . . . . . . . . . . . . . . . . . . . . . . . . t 246 Diamond Weight, . . . . . . . . . . . . . . . . . . . . . . . . . . . . . . . . . . . . . . . . . . . . . . . . . . . . . . . . . . . . . . . . . . . . . . . . . 251 Difference or remainder, def. of, . . . . . . . . . . . . . . . . . . . . . . . . . . . . . . . . . . . . . . . . . . . . . . . . . . . . . . . . . . . 64, 434 between true discount and bank discount, . . . . . . . . . . . . . . . . . . . . . . . . . . . . . . . . . . . . 604 K of latitude, . . . . . . . . . . . . . . e e s tº e e s e e - © º º is e º e º e º 'º - - e º s e a tº 4 s - º º e º s a e < * * * * * * * * * * * * * * * 301 of longitude, . . . . . . . . . • e º 'º e º e º e º e º e º e e s e s e s • * * * * * * * * * * * * * * * * * * * * * * * * * * * * * * * * * * * 302 Discharge, def. Of, . . . . . . . . © º e º e e & e º 'º - e º e g º ºs e e º a tº w e s = e º is a n e º 'º e º is a e º 'º e º 'º e s ∈ e º 'º e o e º s e º 'º e º 'º & 487 Discount day, . . . . . . . . . . © o º 'o e º tº º te e º 'º - © tº e º is ſº tº e º 'º - © e º a s a tº tº º w tº ºr s a s - - - - e º e º e - - Q - e º e = ** - - - - - - 551 on time drafts, . . . . . . . . . . . © e º 'º e & © e º 'º • * - e. e. e º tº e º 'º - * * * * s e s - a - e. e. g. a s e º 'º - - - a a s e º s - - - - - 732 ID1scounting notes, . . . . . . . • * * * e o s * @ e º 'º tº s e º ºs e e º e º e º e e s e s is a e e s e s e e s a e º 'º e a s s at e º e º 4 º e s e º e < e < * * * 576 notes bearing interest, . . . . . . . . . . . . . . . . . . . . . . . . . . . . . . . . . . . . . . . . . . . . . . . . . . . . . . . 580 Dissolution of partnerships, . . . . . . . . . . . . . . . . . . . . . . . . . . . . . . . . . . . . . . . • e e s e - e. e º a s e a e e º e s e e 806 Dividends, defs. Of, . . . . . . - e = e s e e g º m - - - e, e º e º e - - - - e º e s tº a s - - - - e º s a s a - - - e. e. e - - - - - - . . . . 123, 487, 678 J”visibility of numbers, . . . . . . . . . . . . . . . . . . . . . . . . . . . . . . . . . . . . . . . . . . . . . . . . . * * * * * * * * * * * * * * 147 * . Vision, Whole numbers, . . . . . . . . . . . . . . . . . . . . . . . . . . . . . . . . . . . . . . . . . . . . . . . . . . . . . . . . . . . . . . . . 122–143 , fractions, . . . . . . * * * * * * * * * * * * * * * * * * * * * * * * * * * * * * * * * * * * * * * * * * * * * * * * * * * * * * * * * is s a s e - 4 a º 197—213 , contractions in, . . . . . . . . . . . . . . . . . . . . . . . . . . . . . . . . . . . . . . . . . . . . . . . . . . . . . . . . . . . . . . . 138–142 def, of . . . . . . . . . . . . . . . . . . . . . . . . . . . . . . . . . . . . . . . . . . . . . . . . . . . . . . . . . . . . . . . . . . . . . . . . . 122 , logic of . . . . . . . . * * * * * s tº e º & w is e is a s s s e º ſº º e º e > * * * * * * * * * * * * * * * * * * * e s a s a m + e º ºs e e s p → * ~ * 122 of decimals, . . . . . . . . . . . . . . . . . . . . . . . . . . . . . . . . . . . . . . . . . . . . . . . . . . . . . . . . . . . . . . . . . . . . . 226 of denominate numbers, . . . . . . . . . . . . . . . . . . . . . . . . . . . . . . . . . . . . . . . . . . . . . . . . . . . . . . . . . 273 of metric numbers, . . . . . . . . . . . . . . . . . . . . . . . . . . . . . . . . . . . . . . . . . . . . . . . . . . . . . . . . . . . . . . . 939 principles of . . . . . . . . . . . . . . . . . . . . . . . . . . . . . . . . . . . . . . . . . . . . . . . . . . . . . . . . . . . . . . . . . . . . 123 proof of . . . . . . . . . . . . . . . . . . . . . . . . . . . . . . . . . . . . . . . . . . . . . . . . tº @ tº e º e º e º e º - - & © e º e º 'º - - e º 'º 123 Divisors, deſs. of . . . . . . . . . . . . . . . . . . . . . . . . . . . . . . . . . . . . . . . . . . . . . . . . . . . . . . . . . . . . . . . . . . . . . . . . . 123, 147 Dollars and cents, addition of . . . . . . . . . . . . . . . . . . . . . . . . . . . . . . . . . . . . . . . . . . . . . . . . . . . . . . . . . . . . . 52 { { “ subtraction of . . . . . . . . . . . . . . . . . . . . . . . . . . . . . . . . . . • * * g e º º ſº º ºſ e º & tº gº tº sº e e º 'º e º 'º 67, 68 Doyle's rule in lumber measure, . . . . . . • * * * * * * * * * * * * * * * * * * * * * * * * * * * * * * e º e º e º a s e º 'º e º e s - e. e. e. e. 389 Draft, def. of . . . . . . . . . . . . . . . . . . . . . . . . . . . . . . . . . . . . . . . . . . . . . . . . . . . . . . . . . . . . . . . . . © e º ºs s > e - e º 'º e 502 Drawback, def. of, . . . . . . tº e º e s sº tº e º e < * * * * * * * * * * * * * * * * * * * * * * * * * * * * s s e º e º ºs e e s = e º 'º e º e º a s a s e e º s e 502 Drawing notes, points in, . . . . . . . . . . . . . . . . . . . . . . . . . . . . . . . . . . . . . . . . . . . . . . . . . . . . © tº ſº e º 'º º e º 'º e º 'º e 559 Drill exercises, addition, . . . . . . . . . . . tº ~ * * * * * * * * * * * * * * * * * * * * * * * * * * * * * * * * e o e e º a 43, 44, 45, 48, 50, 55, 58 ( & mensuration of solids, . . . . . . . . . . . . . . . . . . . . . . . . . . . . . . . . . . . . * * * * * * * * * * * * * * * * 4.17, 418 problems in contracted methods, . . . . . . . . . . • * s s tº is e º a e < * s e e s e e º e < e < e < * tº e º º e º dº º ºs e º e º tº e º º 115 & 4 “ simultaneous multiplication, . . . . . . . . . . . . . . . . . . . . . . . . . tº gº tº e º 'º - tº º tº gº tº e º 'º - e º e 90 Dry Imeasure, - - - - - e a e s e e º e º 'º e º e º e º e º e º ºs e e º e º e º ºs e e s is a e s e º e s e s - sº e s s s a e e s a e * * * * * * * * * * * * * * * * * * * 248 Due date in equation, . . . . . . . . . . . . . . . . . . . . . . . . . . . . . . . . . . . . . . . . . . . . . . . . . . . . . . . . . . . . . . . . . . . . . 627 Duodecimal scale, tº s e e is * * * * * * * * * * * * * * * * * * * * * 0 e o e º e º is e e º e º e º e º e s e e e º e o e º e s e º e o e g c e s e e o e e s e s 27 XII ALPHABETICAL INDEX, PAGES. Plevens, properties of ....................................... . . . . . . . . . . . . . . . . . . . . . . . . . . . . . 150–158 *lipse, def of . . . . . . . . . . . . . . . . . . . . . . . . . . . . . . . . . . . . . . . . . . . . . . . . . . . . . . . . . . . . . . . . . . . . . . . . . . . 339 Plucidation of vicenary addition,.................................... . . . . . . . . . . . . . . . . . . . . . 56 *gineers' measure, . . . . . . . . . . . . ............................. . . . . . . . . . . . . . . . . . . . . . . . . . . . . . . 244 *nglish consols, . . . . . . . . . . . . . . . .................. . . . . . . . . . . . . . . . . . . . . . . . . . . . . . . . . . . . . . . . . . . 680 exchange, . . . . . . . . . . . . . . . . . . . . . . . . . . . . . . . . . . . . . . . . . . . . . . . . . . . . . . . . . . . . . . . . . . . . . . . . 740–745. life insurance table No. 3, . . . . . . . . . . . . . . . . . . . . . . . . . . . . . . . . . . . . . . . . . . . . . . . . . . . 536, 537, 538 money, freight in . . . . . . . . . . . . . . . . . . . . . . . . . . . . . . . . . . . . . . . . . . . . . . . . . . . . . . . . . . . . . . . . . 7:53 “ interest on, . . . . . . . . . . . . . . . . . . . . . . . . . . . . . . . . . . . . . . . . . . . . . . . . . . . . . . . . . . . . . . . . 569 “ reduced to decimal of a pound, ... . . . . . . . . . . . . . . . . . . . . . . . . . . . . . . . . . . . . . . . . . 450 “ table, . . . . . . . . . . . . . . . . . . . . . . . . . . . . . . . . . . . . . . . . . . . . . . . . . . . . . . . . . . . . . . . . . . . . . 680 System of numeration, . . . . . . . . . . . . . . . . . . . . . . . . . . . . . . . . . . . . . . . . . . . . . . . . . . . . . . . . . . . 31 Equador exchange, . . . . . . . . . . . . . . . . . . . . . . . . . . . . . . . . . . . . . . . . . . . . . . . . . . . . . . . . . . . . . . . . . . . . . . . . 771 Equal annual payments of principal and interest, ... . . . . . . . . . . . . . . . . . . . . . . . . . . . . . . . . . . . . . . . 588 Equation method of account current and interest accounts, . . . . . . . . . . . . . . . . . . . . . . . . . . . . . . . 653, 656 Equation of accounts or payments, ... . . . . . . . . . . . . . . . . . . . . . . . . . . . . . . . . . . . . . . . . . . . . 627-653-1009-1012 Treating equation of sums due at different times, with different credits, from earliest and latest dates; time counted forward and backward; simple and compound equations; equation of account sales and on joint account; equation of notes bearing interest; rent Inotes; notes maturing at different times, etc. Equilateral triangle, def. of . . . . . . . . . . . . . . . . . . . . . . . . . . . . . . . . . . . . . . . . . . . . . . . . . . . . . . . . . . . . . . . 338 Equivalency of numbers, . . . . . . . . . . . . . . . . . . . . . . . . . . . . . . . . . . . . . . . . . . . . . . . . . . . . . . . . . . . . . . . . . . 311 Equivalent single discount of a series, ... . . . . . . . . . . . . . . . . . . . . . . . . . . . . . . . . . . . . . . . . . . . . . . . . . . 459 Even numbers, def. of . . . . . . . . . . . . . . . . . . . . . . . . . . . . . . . . . . . . . . . . . . . . . . . . . . . . . . . . . . . . . . . . . . . . 25, Evil effects of usury laws,. . . . . . . . . . . . . . . . . . . . . . . . . . . . . . . . . . . . . . . . . . . . . . . . . . . . . . . . . . . . . . . . 545. Evolution, . . . . . . . . . . . . . . . . . . . . . . . . . . . . . . . . . . . . . . . . . . . . . . . . . . . . . . . . . . . . . . . . . . . . . . . . . . . . . . . . 885–900 Exchange, . . . . . . . . . . . . . . . . . . . . . . . . . . . . . . . . . . . . . . . . . . . . . . . . . . . . . . . . . . . . . . . . . . . . . . . . . . . . . . . . 710–777 Treating of balance of trade; origin and importance of exchange; bills of exchange; letters of credit; travelers’ checks; character of foreign bills; par of exchange; rate of exchange; buying, selling, and investing on account, through agent, and through agent and broker; time exchange; exchange and cash notes combined; discount on time drafts; arbitration of exchange; history of English monetary pound; cost of imported goods; freight in English money; filling orders of English merchants, exchange between London and other countries; arbitration of foreign exchange; unification of the money of the world; exchange with Argentine Republic, Austria, Belgium, Brazil, China, Cuba, Colombia, Costa Rica, Chili, Denmark, England, Equador, Finland, France, Germany, Greece, Guatemala, Honduras, Italy, Japan, Mexico, Nicaragua, Norway, Peru, Portugal, Russia, Salvador, Spain, Sweden and Switzerland. Excise duties, . . . . . . . . . . . . . . . . . . . . . . . . . . . . . . . . . . . . . . . . . . . . . . . . . . . . . . . . . . . . . . . . . . . . . . . . . . . . . 502 Exercises in addition, . . . . . . . . . . . . . . . . . . . . . . . . . . . . . . . . . . . . . . . . . . . . . . . . . . . . . . 43, 44, 45, 48, 50, 55, 58 in notation and numeration, . . . . . . . . . . . . . . . . . . . . . . . . . . . . . . . . . . . . . . . . . . . . . . . . . . . . . 33, 34 Expectancy of life table, . . . . . . . . . . . . . . . . . . . . . . . . . . . . . . . . . . . . . . . . . . . . . . . . . . . . . . . . . . . . . . . . . . 537 JF Face and character of foreign bills, . . . . . . . . . . . . . . . . . . . . . . . . . . . . . . . . . . . . . . . . . . . . . . . . . . . . . 744 of note, . . . . . . . . . . . . . . . . . . . . . . . . . . . . . . . . . . . . . . . . . s º e g º e º g º a tº a gº tº e V & e e º 'º t e º ºs e º e g º º º ſº tº º 549 Factoring, . . . . . . . . . . . . . . . . . . . . . . . . . . . . . . . . . . . . . . . . . . . . . . . . . . . . . . . . . . . . . . . . . . . . . . . . . . . . . . . . 157 Factors, def. of . . . . . . . . . . . . . . . . . . . . . . . . . . . . . . . . . . . . . . . . . . . . . . . . . . . . . . . . . . . . . . . . . . . . . . . 75, 84, 146. 146. , prime and composite, . . . . . . . . . . . . . . . . . . . . . . . . . . . . . . . . . . . . . . . . . . . . . . . . . . . . . . . . . . . . . ALPHABETICAL INDEX. XIII PAGES. PAGES. Fictitious dividend, . . . . . . . . . • * * * * * * * * g ∈ 678 | Fractions, decimal, . . . . . . . . . . . . . . . . . . . . .216–229 Figures, def. of, . . . . . . . . . . . . . . . . . . . . . . . . 27 , def of . . . . . . . . . . . . . . tº gº tº e º & a tº 164 , local value of, . . . . . . * * * * * * * * * * * e 27 , division of, . . . . . . . . . . gº tº º & & tº ſº tº 197—213 , order of . . . . . . . tº e < * * * * * * * * * * * * * 29 , G. C. D. of, . . . . . . . . . . tº t e º e º e & 213 , simple value of, . . . . . . . . . . . . . . . . 27 , L. C. M. of . . . . . . . . . . . . . . . . . . 214 Finland exchange, . . . . . . . . . . . . . . . . . . . . . . 764 , multiplication of, . . . . . . . . . . . . 183–197 Fire losses adjusted, . . . . . . . . . . . . . . . . . . . . 525 , Teduction of, . . . . . . . * @ e º ſº tº º gº a 167–175 Floating policy in insurance, . . . . . . . . . . . . 514 , Subtraction of, . . . . . . . . . . . . . . 180–183 Focal date in equations, . . . . . . . . . . . . . . . . 627 | Framing, building, laying floors, wain- l'Orced cash dividend. . . . . . . . . . . . . e e º 'º e 678 Scoting, etc., . . . . . . . . . tº e º º gº tº dº e º & ºt 361 Foreign coins, table of, . . . . . . . . . . . . . . . . . 743 | Free list, def. of, . . . . . . . . . . . . . . § º g º º G & © tº 501 Forfeited stock, . . . . . . . . . . . . . . . . . . . . . . . . 677 | Freight in English money, . . . . . . . . . . . . . . 753 Formation of partnership, . . . . . . . . . . . . . . 805 | French currency, interest on, . . . . . . . . . . . 570 Formula, def. of, . . . . . . * * * * * * * * * * * * * * * * * * 26 exchange, . . . . . . . . . . . . . . . . . . . . . . 759–764 Formulae for complement and supple- money table, . . . . . . . . . . . . . . . . . . . 235 ment numbers, . . . . . . . . . . . . . * * * * * * 114 Tentes, . . . . . . . . . . . . . * * * * * * * * * * * * 680 Fractions, addition of, . . . . . . & sº e º ºs e g º ºs e º s 175–180 system of numeration, . . . . . . . . . . 30 , continued, . . . . . . . . . . . . . & © tº º ºs ºs 949 | Frustum, def. of, . . . . . . * * * * * * * * * * * * * * * * * 370 Gauging, . . . . . . . . . . . . . . . . . tº gº tº dº e º $ tº * * * g e º a tº e º ºs º ºs º e º is $ sº * * * * * * * 413 General and particular average, (liscussion and problems, . . . . . . . . . . . . . . . . . . . . . . . . . . . . . . . . 527 Geographical mile, . . . . . . . . . . . . . . . . . . . . . . . . . . . . . . . . . . * e s tº e º e s ∈ s e e s s is s e s m = < e < n e a s e e s ∈ e º e a e 24.2 Geometrical progression. . . . . . . . . . . . . . . . . . . . . . . . . . . . . . . . . . . . . . . . . . . . . . . . . . . . . . . . . . . . . . . . . . 908-920 German currency, interest on . . . . . . . . . . . . . . . . . . . . . . * * * * * * * * a s = * * * * * * * * * * * * * * * * * * * * * * * * * g e 571–664 exchange, . . . . . . . . . . . . . . . . . . . . . . . . . . . . . . . . . . . . . . . . . . . . . . . . . . . . . . . . . . . is tº e º $ tº gº tº gº 764-767 money table. . . . . . . . . . . . . * * * * * * * * * * * * * * s a 4 s e º e a tº a º e & e º e º $ sº º e < * * * * * e s tº * * * * * * * 236 Glazing and painting, . . . . . . . . . . . . . . * * * * * * * * * * * * * * * * * * * * * * * * * * * * is e e º is e s e s is a tº e a º e º ºs e e s ∈ s 366 Gold, silver, currency and uncurrent money, . . . . . . . . . . . . . . . . . . . . . . . . . . . . . . . . . tº e º ºr e º 'º gº 493-500 Goods, cost of importing, . . . . . . . . . . * * * * * * * * * * * * * * * * * * * * * * * * * * * * * * * * * * * * * * * * * * * * * * * * * * * * * = 752 , insuring. . . . . . . . . . . . . . . . . . . . . . . . . . . . . . . . . . . . . . . . . . . . . . . . . . . . . . . . * * * * * * * * * * * 526 Greatest common divisor, . . . . . . . . . . . . . . . . . . . . . . . . . . . . . . . . . . . . . . . . . . . . . . . . . . . . . . . . . . . . 158 of fractions, . . . . . . . . . . . . . . . . . . . . . . . . . . . . . . . . . . . . . . . . . . . . . . . . . . . . . . . . . . . 213 German exchange, . . . . . . . . . . . . . . . . . . . . . . . . . . . . . . . . . . . . . . . . . . . . . . . . . . . . . . . . . . . . . . . . . . . . . 764 Gross and Inet earnings, . . . . . . . . . . . . . . . . . . . . . . . . . . . . . * * * * * * * * * * * * * * * * * * * * * * * * 677,678 Weight, . . . . . . . . . . . . . . . . . . . . . . . . . . . . . . . . . . . . . . . . * • * * * * * * * * * * * 502 Guarantee, def. of . . . . . . . . . . . . . . . . . . . . . . . . . . . . . . . . . . . . . . . . . . . . . * * * * * * * * * * * * * * * * * * * * * * 480 Guaranteed stock, . . . . . . . . . . . . . . . . . . . . . . . . . . . . . . . . . . . . . . . . . . . . . . . . . * * * * * * * * * * * * * * * * 677 Guatemala exchange. . . . . . . . . . . . . . . . . . . . . . . . . . . . . . . . . . . . . . . . . . . . . . * * * * * * * * * * * * * * * * * * * * * * 771 Hamburg exchange on other cities, . . . . . . * * * * * * * * * * * * * * * * * * * * * * * * * * * * * * * * * * * * * * * s e º a s tº e © tº º 766 History of English monetary pound. . . . . . . . . . . . . . * * * * * * * * * * * * * * * * * * * * * * * * * * * * * * * * * * * * * * g e 740 of interest, . . . . . . . . . . . • * * * * * * * * * s • * * * * * * * * * * * * * * * * * * * * e º e º s e s e s e i s s sº e s = e º e < * * * * * 543 of the year and adjustment of the calendar, . . . . . . . . . . . * & © e º 8 s tº g º e º a e º is $ $ s tº e º & 237 Homestead associations, to find rate of interest made by, . . . . . . . . . . . . . . . . . . . . . . . . . . . . . . . . . 592-595 See Building and Loan Associations, for full and extended treatment of methods in use by Homestead Associations. Honduras exchange, . . . . . . . . . . . . . . . . . . • * * * * * * * s is is e º e º a tº e s a s = n < n < e a e e s is e a e º e a e s e e tº º e º is a s e e is 771 Horizontal addition, . . . . . . . . . . . . . . . . . . tº gº tº e º tº € $ tº º e s tº e º 'º º e * * * * * * * * * * * * * * * s & e º e s tº a º e º 3 e º 'º e º sº. 49 68 subtraction, . . . . . . . . . . . . . . . . . . . . . . . . . . . . . . . . . . . . . . . . . . * * * e º e e º 'º e * e e º tº º ſº tº tº tº e º º ºs XIV ALPHABETICAL INDEX. PAGES. Importance of exchange, . . . . . . . . . . . . . . . . . . . . . . . . . . . . . . . . . . . . . . . . . . . . . . . . . . . . . . . . . . . . . . . ... 712. of insurance, . . . . . . . . . . . . . . . . . . . . . . . . . . . . . . . . . . . . . . ; . . . . . . . . . . . . . . . . . . . . . . . . . . 514 of life insurance, . . . . . . . . . . . . . . . . . . . . . . . . . . . . . . . . . . . . . . . . . . . . . . . . . . . . . . . be e s & a s 532 of multiplication, . . . . . . . . . . . . . . . . . . . . . . . . . . . . . . . . . . . . . . . . . . . . . . . . . . . . . . . . . . . . . 77 Imported goods, cost of . . . . . . . . . . . . . . . . . . . . . . . . . . . . . . . . . . . . . . . . . . . . . . . . . . . . . . . . . . . . . . . . . . . 752 Improper fractions, def. of, . . . . . . . . . . . . ... • e º e < * * * * * * * * * * * * * * * * * * * * * * * * * * s • * * * * * * * * * * * * * * * * * * 165 Inclined plane, use of . . . . . . . . . . . . . . . . . . . . . . . . . . . . . . . . . . . . . . . . . . . . . . . . . . . . . . . . . . . . . . . . . . . . 427 Indorsements on notes and bills, . . . . . . . . . . . . . . . . . . . . . . . . . . . . . . . . . . . . . . . . . . . . . . . . . . . . . . . . . . . 550 Installment, def. of . . . . . . . . . . . . . . . . . . . . . . . . . . . . . . . . . . . . . . . . . . . . . . . . . . . . . . . . . . . . . . . . . . . . . . . 677 dividend, . . . . . . . . . . . . . . . . . . . . . . . . . . . . . . . . . . . . . . . . . . . . . . . . . . . . . . . . . . . . . . . . . . . . 678 Installments, payments by, . . . . . . . . . . . . . . . . . . . . . . . . . . . . . . . . . . . . . . . . . . . . . . . . . . . . . . . . . . . . . . . . 616–627 Insurance, . . . . . . . . . . . . . . . . . . . . . . . . . . . . . . . . . . . . . . . . . . . . . . . . . . . . . . . . . . . . . . . . . . . . . . . . . . . . . . . . 508–543 defs. of property, fire, river, marine, transit, live stock, personal, life, health, accident, fidelity and co-operative; policies and rates defined; kinds of insurance companies; adjustment of losses; bottomry and respondentia ; average and co-insurance clauses; lightning clause, # loss, # value, etc.; unearned premiums; general and particular average; life insurance fully treated. Integer, def. of, ... . . . . . . . . . . . . . . . . . . . . . . . . . . . * * * * * * * * * * * * * * * * * * is e º e º e º s tº e º is a e s is º e º is is as 146 Interest, . . . . . . . . . . . . . . . . . . . . . . . . . . . . . . . . . . . . . . . . . . . . . . . . . . . . . . . . . . . . . . . . . . . . . . . . . . . . . . . . . . 543–616, full definitions; history of usury laws; simple interest; annual, semi-annual, and quarterly interest; compound interest; table of interest laws; promissory notes and negotiable paper; interest divisors; interest on United States bonds; interest on English money; interest for 360 and 365 days to a year; interest on French and German currency; different methods of calculating interest; discounting notes in full ; “cash notes”; renewing notes; notes bearing interest; interest on installment plan ; true discount; maturity values. Interest divisors, . . . . . . . . . . . . . . . . . . . . . . . . . . . . . . . . . . . . . . . . . . . . . . . . . . . . . . . . . . . . ... • . . . . . . . . . . . . 559 method of account current and interest accounts, . . . . . . . . . . . . . . . . . . . . . . . . . . . . . . . . . 653 on daily balances, . . . . . . . . . . . . . . . . . . . . . . . . . . . . . . . . . . . . . . . . . . . . . . . . . . . . . . . . . . . . . . . . 665 term in Savings banks, . . . . . . . . . . . . . . . . . . . . . . . . . . . . . . . . . . . . . . . . . . . . . . . . . . . . . . . . . . . . 669 Internal revenue, def. of . . . . . . . . . . . . . . . . . . . . . . . . . . . . . . . . . . . . . . . . . . . . . . . . . . . . . . . . . . . . . . . . . . 502 Interval between dates, . . . . . . . . . . . . . . . . . . . . . . . . . . . . . . . . . . . . . . . . . . . . . . . . . . . . . . . . . . . . . . . . . . . 296 Intrinsic value of stock, . . . . . . . . . . . . . . . . . . . . . . . . . . . . . . . . . . . . . . . . . . . . . . . . . . . . . . . . . . . . . . . . . . . 677 Investing in exchange, . . . . . . . . . . . . . . . . . . . . . . . . . . . . . . . . . . . . . . . . . . . . . . . . . . . . . . . . . . . . . . . . . . . . 721 in exchange through agent and broker, . . . . . . . . . . . . . . . . . . . . . . . . . . . . . . . . . . . . . . . . . 722, 724 See also Homesteads and Building Associations for extended treatment. Investment and loan companies, . . . . . . . . . . . . . . . . . . . . . . . . . . . . . . . . . . . . . . . . . . . . . . . . . . . . . . . . . . . 675 Invoice or manifest, def. of . . . . . . . . . . . . . . . . . . . . . . . . . . . . . . . . . . . . . . . . . . . . . . . . . . . . . . . . . . . . . . 500 Invoices and bills, . . . . . . . . . . . . . . . . . . . . . . . . . . . . . . . . . . . . . . . . . . . . . . . . . . . . . . . . . . . . . . . . . . . . . . . . . 283–296 Involution, . . . . . . . . . . . . . . . . . . . . . . . . . . . . . . . . . . . . . . . . . . . . . . . . . . . . . . . . . . . . . . . . . . . . . . . . . . . . . . . . 883 Isosceles triangle, def. of . . . . . . . . . . . . . . . . . . . . . . . . . . . . . . . . . . . . . . . . . . . . . . . . . . . . . . . . . . . . . . . . . 338 Italian exchange, . . . . . . . . . . . . . . . . . . . . . . . . . . . . . . . . . . . . . . . . . . . . . . . . . . . . . . . . . . . . . . . . . . . . . . . . 764 J Japanese exchange, . . . . . . . . . . . . . . . . . . . . . . . . . . . . . . . . . . . . . . . . . . . . . . . . . . . . . . . . . . . . . . . . . . . . . . . 772 Jettison, . . . . . . . . . . . . . . . . . . . . . . . . . . . . . . . . . . . . . . . . . . . . . . . . . . . . . . . . . . . . . . . . . . . . . . . . . . . . . . . . . . 528 Jºº. Key figure, use of . . . . . . . . . . . . . . . . . . . . . . . . . . . . . . . . . . . . . . . . . . . . . . . . . . . . . . . . . . . . . . . . . . . . . . . . 152 a t t e º s & a s a º ºs e º & tº 242 Knot, def. of . . . . . . . . . . . . . . . . . . . . . . . . . . . . . . . . . . . . . . . . . . . . . . . . . . . . . . . . . . . . ALPHABETICAL INDEX. PAGES. Land measure. . . . . . . . . . . . . . . . . . . . . . . . . . . . . . . . . * * * * * > 0 s e e e s e o e s s e s e s e e s a e s s a e s e e s e e s • * * * * 272 Latitude, difference of . . . . . . . . . . . . . . . . . . . . . . . . . . . . . . . . . . . . . . . . . . . . . . . . . . . . . . . . . . . . . . . º, º e º 'º 301 Leakage and breakage, . . . . . . . . . . . . . . . . . . . . . . . . . . . . . . . . . . . . . . . e e s e e e s a e e s e º e s e º e º e º e = * * * * * * 502 Leap years, . . . . . . . . . . . . . . . . . . . . . . . . . . . . . . . . . . . . . . . • e o e s e e s e e º e º e s a e e º e º e º e º e s e e < * * * * * * * * * * 236 Least common multiple, . . . . . . . . . . . . . . . . . . . . . . . . . . . . . . . . . . . . . . . . . . . . . . . . . . . . . . . . . . . . . . . . . . . I59 common multiple of fractions, . . . . . . . . . . . . . . . . . . . . . . . . . . . . . . . . . . . . . . . . . . . . . . . . . . . . . . 214 Legal interest, . . . . . . . . . . . . . . . . . . . . . . . . . . . . . . . . . . . . . . . . . . . . . . . . . . . . . . . . . . . . . . . . . . . . . . . . . . . . 547, 548 Length of day or night, to find, . . . . . . . . . . . . . . . . . . . . . . . . . . . . . . . . . . . . . . . . . . . . . . . . . . . . . . . . . . . 297 Letters of credit, . . . . . . . . . . . . . . . . . . . . . . . . . . . . . . . . . . . . . . . . . . . . . . . . . . . . . . . . . . . . . . . . . . . . . . . . . 714. Levers, kinds and application of . . . . . . . . . . . . . . . . . . . . . . . . . . . . . . . . . . . . . . . . . . . . . . . . . . . . . . . . . . 419 Liabilities, def of . . . . . . . . . . . . . . . . . . . . . . . . . . . . . . . . . . . . . . . . . . . . . . . . . . . . . . . . . . . . . . . . . . . . . . . 487 Licenses and taxes, . . . . . . . . . . . . . . . . . . . . . . . . . . . . . . . . . . . . . . . . . . . . . . . . . . . . . . . . . . . . . . . . . . . . . . . . 489–493 Life insurance, . . . . . . . . . . . . . . . . . . . . . . . . . . . . . . . . . . . . * & © tº e s e º e º ſº e º e º 'º & º 'º - * * * * * * tº e º e º 'º e º a sº e º e e 532–543 defs. of and basis of, with tables. . . . . . . . . . . . . . . . . . . . . . . . . . . . . . . . . . . . . . . . . . . . . . . . . . . . 536–541 Lightning clause in insurance, . . . . . . . . . . . . . . . . . . . . . . . . . . . . . . . . . . . . . . . . . . . . . . . . . . . . . . . . . . . 513. Like fractions, def. of . . . . . . . . . . . . . . . . . . . . . . . . . . . . . . . . . . . . . . . . . . . . . . . . . . . . . . . . . . . . . . . . . . . 175 numbers, def. of . . . . . . . . . . . . . . . . . . . . . . . . . . . . . . . . . . . . . . . . . . . . . . . . . . . . . . . . . . . . . . . . . . . . . 25 Linear measure table, . . . . . . . . . . . . . . . . . . . . . . . . . . . . . . . . . . . . . . . . . . . . . . . . . . . . . . . . . . . . . . . . . . . . . 242–243 Lines, defs, of . . . . . . . . . . . . . . . . . . . . . . . . . . . • * * * * * * * * * * * * * * * * * * * * * * * * * * * e a e < e < e < e < e º e s e e 336 Liquid measure. . . . . . . . . . . . . . . . . . . . . . . . . . . . . . . . . . . . . . . . . . . . . . . . . . . . . . . . . . . . . . . . . . . . . . . . . 247 Liquidation value of stocks, . . . . . . . . . . . . . . . . . . . . . . . . . . . . . . . . . . . . . . . . . . . . . . . . . . . . . . . . . . . . . . 677 List prices and trade discounts, . . . . . . . . . . . . . . . . . . . . . . . . . . . . . . . . . . . . . . . . . . . . . . . . . . . . . . . . . . 457–466 Loan and investment companies, . . . . . . . . . . . . . . . . . . . . . . . . . . . . . . . . . . . . . . . . . . . . . . . . . . . . . . . . . . 675 Local value of figures, . . . . . . . . . . . . . . . . . . . . . . . . . . . . . . . . . . . . . . . . . . . . . . . . . . . . . . . . . . . . . . . . . . . . 27 Logic of division, . . . . . . . . . . . . . . . . . . . . . . . . . . . . . . . . . . . . . . . . . . . . . . . . . . . . . . . . . . . . . . . . . . . . . . . . . 122 Logs, lumber, etc., measured, . . . . . . . . . . . . . . . . . . . . . . . . . . . . . . . . . . . . . . . . . . . . . . . . . . . . . . . . . . . . 384 London exchange on New York, . . . . . . . . . . . . . . . . . . . . . . . . . . . . . . . . . . . . . . . . . . . . . . . . . . . . . . . . 757 & 4 on Calcutta, . . . . . . . . . . . . . . . . . . . . . . . . . . . . . . . . . . . . . . . . . . . . . . . . . . . . . . . . . . . 758 & 4 on Paris, . . . . . . . . . . . . . . . . . . . . . . . . . . . . . . . . . . . . . . . . . . . . . . . . . . . . . . . . . . . . . . . . 757 { { on Shanghai, . . . . . . . . . . . . . . . . . . . . . . . . . . . . . . . . . . . . . . . . . . . . . . . . . . . . . . . . . . . . 758 Long division, . . . . . . . . . . . . . . . . . . . . . . . . . . . . . . . . . . . . . . . . . . . . . . . . . . . . . . . . . . . . . . . . . . . . . . . . . . . . 129 OD, . . . . . . . . . . . . . . . . . . . . . . . . . . . . . . . . . . . . . . . . . . . . . . . . . . . . . . . . . . . . . . . . . . . . . . . . . . . . . . . . 502 Longitude, difference of, . . . . . . . . . . . . . . . . . . . . . . . . . . . . • * > * * * * * * * * * * * * * * * * * * * * * * * * * * * * * * * * * * * 302 and time, . . . . . . . . . . . . . . . . . . . . . . . . . . . . . . . . . . . . . . . . . . . . . . . . . . . . . . . . . . . . . . . 303 Loss or gain of a day in traveling, . . . . . . . . . . . . . . . . . . . . . . . . . . . . . . . . . . . . . . . . . . . . . . . . . . . . . . . . 306, 307 Losses in insurance adjusted, . . . . . . . . . . . . . . . . . . . . . . . . . . . . . . . . . . . . . . . . . . . . . . . . . . . . . . . . . 512, 523, 525 Lumber, logs, timber and boards measured, . . . . . . . . . . . . . . . . . . . . . . . . . . . . . . . . . . . . . . . . . . . . . . . . 384 Lumber and board measure, Doyle & Scribner Rules. . . . . . . . . . . . . . . . . . . . . . . . . . . . . . . . . . . . . . 388 Manifest or invoice, def. of . . . . . . . . . . . . . . . . . . . . . . . . . . . . . . . . . . . . . . . . . . . . . . . . . . . . . . . . . . . . . . . 500 Manufacturing on shares and tollage, . . . . . . . . . . . . . . . . . . . . . . . . . . . . . . . . . . . . . . . . . . . . . . . . . . . . . 785–789 Margin on collateral notes, . . . . . . . . . . . . . . . . . . . . . . . . . . . . . . . . . . . . . . . . . . . . . . . . . . . . . . . . . . . . . . . . 958 to carry stock, . . . . . . . . . . . . . . . . . . . . . . . . . . . . . . . . . . . . . . . . . . . . . . . . . . . . . . . . . . . . . . . . . . . 704 Mariners’ measure, . . . . . . . . . . . . . . . . . . . . . . . . . . . . . . . . . . . . . . . . . . . . . . . . . . . . . . . . . . . . . . . . . . . . . . . . 242 Market value of Stocks and bonds, . . . . . . . . . . . . . . . . . . . . . . . . . . . . . . s s e s a s • * * * * * * * * is a e s - e. e. 677 Marking goods, . . . . . . . . . . . . . . . . . . . . . . . . . . . . . . . . . . . . . . . . . . . . . . . . . . . . . . . . . . . . . - - - - * * * * * - - - 451–457 Maturity of note, . . . . . . . . . . . . . . . . . . . . . . . . . . . . . . . . . . . . . . . . . . . . . . . . . . . . . . . . . . . . . . . . . • * * 549 values in Series of notes or payments, . . . . . . . . . . . . . . . . . . . . . . . . . . . . . . . . . . . . . . . . . . . . 595 Measures, def. of . . . . . . . . . . . . . . . . . . . . . . . . . . . . . . . . . . . . . . . . . . . . . . . . . . . . . . . . . . . . . . . . . . . . . . . . 229 of time, . . . . . . . . . . . . . . . . . . . . . . . . . . . . . . . . . . . . . . . . . . . . . . . . . . . . . . . . . . . . . . . . . . . . . 236 tables, . . . . . . . . . . . . . . . . . . . . . . . . . . . . . . . . . . . . . . . . . . . . . . . . . . . . . . . . . . . . . . . . . . . . . . . . . . 231 Measures of extension, . . . . . . . . . . . . . . . . . . . . . . . . . . . . . . . . . . . . . . . . . . . . . . . . . . . . . . . . . . . . . . . . . . . . 241 of Weight, . . . . . . . . . . . . . . . . . . . . . . . . . . . . . . . . . . . . . . . . . . . . . . . . . . . . . . . . . . . . . . . . . . . . 249 Measuring oil tanks, . . . . . . . . . . . . . . . . . . . . . . . . . . . . . . . . . . . . . . . . . . . . . s tº a º is s a 3 & 4 ~ * * * * * * * * * * * 398 Mechanical powers, . . . . . . . . . . . . . . . . . . . . . . . . . . . . . . . . . . . . . . . . . . . . . . . . . . . . . . * * * * * * * * * * * * * * 419–434 Mechanics’ work in surfaces, . . . . . . . . . . . . . . . . . . . . . . . . . . . . . . . . . . . . . . . . . . . . . . . . . . . . . . . . - - - - 360 Medial proportion, . . . . . . . . . . . . . . . . . . . . . . . . . . . . . . . . . . . . . . . . . . . . . . . . . . . . . . . . . . . . . . . . . . . . . 333 { { or alligation, . . . . . . . . . . . . . . . . . . . . . . . . . . . . . . . . . . . . . . . . . . . . . . . . . . . . . . . . . 789 Mensuration of surfaces and solids with diagrams and cuts, . . . . . . . . . . . . . . . . . . . . . . . . . . . . . . . 336–419 MENSURATION –LINEAR MEASURE. To find diameter and circumference of circle, . . . . . . . . . . . . . . . . . . . . . . . . . . . . . . . . . . . . . . . . . . . . . . 340 hypotenuse, . . . . . . . . . . . . . . . . . . . . . . . . . . . . . . . . . . . . . . . . . . . . . . . . . . . . . . . . . . . . . . . . . . . . . . . 341 base or altitude, . . . . . . . . . . . . . . . . . . . . . . . . . . . . . . . . . . . . . . . . . . . . . . . . . . . . . . . . . . . . . . . . . . . 341 XVI ALPHABETICAL INDEX. PAGES. To find diameter or circumference from area, . . . . . . . . . . . . . . . ................................ 341 side of Square equal to given circle, ...... & e º e º e º e º e º ºs e º e º e º e º ºs e º 'º º e º 'º - e º ºs e e º ºn tº e * @ e > 342 “ of inscribed square, . . . . . . . . . . . . . . . . . . . . . . . . . . . . . . . . . . . . . . . . . . . . . . . . . . . . . . . . . . . 342 height of tree, or to a certain point, . . . . . . . . . . . . . . . . . . . . . . . . . . . . . . . . . . . . . . . . . . . . . . 343 length of part of tree broken at top, . . . . . . . . . . . . . . . . . . . . . . . . . . . . . . . . . . . . . . . . . . . . . . . 344 distance between two points when one is accessible, . . . . . . . . . . . . . . . . . . . . . . . . . . . . . . . . 345 distance between two inaccessible objects,. . . . . . . . . . . . . . . . . . . . . . . . . . . . . . . . . . . . . . . . . . 345 MENSURATION.—SURFACES. To find area of parallelograms, . . . . . . . . . . . . . . . . . . . . . . . . . . . . . . . . . . . . . . . . . . . . . . . . . . . . . . . . . . . . 346 of trapezoid, . . . . . . . . . . . . . . . . . . . . . . . . . . . . . . . . . . . . . . . . . . . . . . . . . . . . . . . . . . . . . . . . . 347 of trapezium, . . . . . . . . . . . . . . . . . . . . . . . . . . . . . . . . . . . . . . . . . . . . . . . . . . . . . . . . . . . . . . . . 347 of triangle, . . . . . . . . . . . . . . . . . . . . . . . . . . . . . . . . . . . . . . . . . . . . . . . . . . . . . . . . . . . . . . . . . . 348 of circle. . . . . . . . . . . . . . . . . . . . . . . . . . . . . . . . . . . . . . . . . . . . . . . . . . . . . . . . . . . . . . . . . . . . . . 349 of circular ring, . . . . . . . . . . . . . . . . . . . . . . . . . . . . . . . . . . . . . . . . . . . . . . . . . . . . . . . . . . . . . 349 of ellipse, . . . . . . . . . . . . . . . . . . . . . . . . . . . . . . . . . . . . . . . . . . . . . . . . . . . . . . . . . . . . . . . . . . . . 350 of irregular polygon, . . . . . . . . . . . . . . . . . . . . . . . . . . . . . . . . . . . . . . . . . . . . . . . . . . . . . . . . . 351 convex surface of prism or cylinder, . . . . . . . . . . . . . . . . . . . . . . . . . . . . . . . . . . . . . . . . . . . . . . . 351 area of segment of a circle, . . . . . . . . . . . . . . . . . . . . . . . . . . . . . . . . . . . . . . . . . . . . . . . . . . . 352, 355, 356 chord of segment of a circle, . . . . . . . . . . . . . . . . . . . . . . . . . . . . . . . . . . . . . . . . . . . . . . . . . . . . . . 356 radius of inscribed circle, . . . . . . . . . . . . . . . . . . . . . . . . . . . . . . . . . . . . . . . . . . . . . . . . . . . . . . . . . 357 Paving yards and walks, . . . . . . . . . . . . . . . . . . . . . . . . . . . . . . . . . . . . . . . . . . . . . . . . . . . . . . . . . . . . . . . . . . 360 Framing, building, laying floors, wainscoting, etc., . . . . . . . . . . . . . . . . . . . . . . . . . . . . . . . . . . . . . . . . 361 Plumbers' work, . . . . . . . . . . . . . . . . . . . . . . . . . . . . . . . . . . . . . . . . . . . . . . . . . . . . . . . . . . . . . . . . . . . . . . . . . . 363 Slating and shingling roofs, . . . . . . . . . . . . . . . . . . . . . . . . . . . . . . . . . . . . . . . . . . . . . . . . . . . . . . . . . . . . . . . 364 Plasterers' work, . . . . . . . . . . . . . . . . . . . . . . . . . . . . . . . . . . . . . . . . . . . . . . . . . . . . . . . . . . . . . . . . . . . . . . . . . . 365 Painting and glazing, . . . . . . . . . . . . . . . . . . . . . . . . . . . . . . . . . . . . . . . . . . . . . . . . . . . . . . . . . . . . . . . . . . . . . 366 Carpeting floors, . . . . . . . . . . . . . . . . . . . . . . . . . . . . . . . . . . . . . . . . . . . . . . . . . . . . . . . . . . . . . . . . . . . . . . . . . . 368 MENSURATION.—SOLIDS. To find solidity of a prism or cube, . . . . . . . . . . . . . . . . . . . . . . . . . . . . . . . . . . . . . . . . . . . . . . . . . . . . . . . 371 surface of a prism or cube, . . . . . . . . . . . . . . . . . . . . . . . . . . . . . . . . . . . . . . . . . . . . . . . . . . . . . . . . . 371 solidity of square or rectangular pyramid, . . . . . . . . . . . . . . . . . . . . . . . . . . . . . . . . . . . . . . . . . 371 surface of Square or rectangular pyramid, . . . . . . . . . . . . . . . . . . . . . . . . . . . . . . . . . . . . . . . . . . 372 solidity of hexagonal or other pyramid, . . . . . . . . . . . . . . . . . . . . . . . . . . . . . . . . . . . . . . . . . . . . 372 surface of octagonal or other pyramid, . . . . . . . . . . . . . . . . . . . . . . . . . . . . . . . . . . . . . . . . . . . . . 372 solidity of frustum of square pyramid, . . . . . . . . . . . . . . . . . . . . . . . . . . . . . . . . . . . . . . . . . . . . . 373 surface of frustum of Square pyramid, . . . . . . . . . . . . . . . . . . . . . . . . . . . . . . . . . . . . . . . . . . . . . 373 solidity of prisms, . . . . . . . . . . . . . . . . . . . . . . . . . . . . . . . . . . . . . . . . . . . . . . . . . . . . . . . . . . . . . . . . 374 solid contents of cylinders, . . . . . . . . . . . . . * * * * * * * & & ſº tº gº tº £ tº $ ſº º ºs & & ſº tº º e º e s s e º 'º a gº e º gº tº º e º 'º ºf 374 surface contents of cylinders, . . . . . . . . . . . . . . . . . . . . . . . . . . . . . . . . . . . . . . . . . . . . . . . . . . . . . . 374 solid contents of cone, . . . . . . . . . . . . . . . . . . . . . . . . . . . . . . . . . . . . . . . . . . . . . . . . . . . . . . . . . . . . . 375 surface of cone, . . . . . . . . . . . . . . . . . . . . . . . . . . . . . . . . . . . . . . . . . . . . . . . . . . . . . . . . . . . . . . . . . . . 375 solid contents of frustum of cone, . . . . . . . . . . . . . . . . . . . . . . . . . . . . . . . . . . . . . . . . . . . . . . 375 surface contents of frnstum of cone, . . . . . . . . . . . . . . . . . . . . . . . . . . . . . . . . . . . . . . . . . . . . . 377 solidity of a sphere, . . . . . . . . . . . . . . . . . . . . . . . . . . . . . . . . . . . . . . . . . . . . . . . . . . . . . . . . . . . . . . . 377 surface of a sphere, . . . . . . . . . . . . . . . . . . . . . . . . . . . . . . . . . . . . . . . . . . . . . . . . . . . . . . . . . . . . . . . . 377 solidity of hemisphere, . . . . . . . . . . . . . . . . . . . . . . . . . . . . . . . . . . . . . . . . . . . . . . . . . . . . . . . . . . . . 378 & 4 of segment of sphere and cylinders . . . . . . . . . . . . . . . . . . . . . . . . . . . . . . . . . . . . . . . . 378-398. & 4 of prolate spheroid, . . . . . . . . . . . . . . . . . . . . . . . . . . . . . . . . . . . . . . . . . . . . . . . . . . . . . . . 379 surface of prolate spheroid, . . . . . . . . . . . . . . . . . . . . . . . . . . . . . . . . . . . . . . . . . . . . . . . . . . . . . . . * 379 solidity of oblate spheroid, . . . . . . . . . . . . . . . . . . . . . . . . . . . . . . . . . . . . . . . . . . . . . . . . . . . . . . . * 37 surface of oblate Spheroid, . . . . . . . . . . . . . . . . . . . . . . . . . . . . . . . . . . . . . . . . . . . . . . . . . . . . . . . . . 380 solidity of semi prolate spheroid, . . . . . . . . . . . . . . . . . . . . . . . . . . . . . . . . . . . . . . . . . . . . . . 380 { % of segment of prolate spheroid, . . . . . . . . . . . . . . . . . . . . . . . . . . . . . . . . . . . . . . . . . . * 381 4 & of segment of oblate spheroid, . . . . . . . . . . . . . . . . . . . . . . . . . . . . . . . . . . . . . . . . . . gº. 381 ‘‘ of middle frustum of prolate Spheroid, . . . . . . . . . . . . . . . . . . . . . . . . . . . . . . . . . . . . . 383 “ of middle frustum of elliptic spindle, . . . . . . . . . . . . . . . . . . . . . . . . . . . . . . . . . . . . . 383 4 & of casks or barrels, . . . . . . . . . . . . . . . . . . . . . . . . . . . . . . . . . . . . . . . . . . . . . . . . . . . . . . . . 383 4 & of cylindrical ring, . . . . . . . . . . . . . . . . . . . . . . . . . . . . . . . . . . . . . . . . . . . . . . . . . . . . . . . 384 4 : of Wedge. . . . . . . . . . . . . . . . . . . . . . . . . . . . . . . . . . . . . . . . . . . . . . . . . . . . . . . . . . . . . . . . . . 384 & 4 of round timber or logs, . . . . . . . . . . . . . . . . . . . . . . . . . . . . . . . . . . . . . . . . . . . . . . . . . . . 385 To reduce round timber to square. . . . . . . . . . . . . . . . . . . . . . . . . . . . . . . . . . . . . . . . . . . . . . . . . . . . . . . . . . 386 ‘‘ find solidity of four sided timber, . . . . . . . . . . . . . . . . . . . . . . . . . . . . . . . . . . . . . . . . . . . . . . . . . . . . . . 387 “ “ solidity of four sided timber that tapers regularly, . . . . . . . . . . . . . . . . . . . . . . . . . . . . . . . . . 387 “ “ length of timber to obtain given Solidity, . . . . . . . . . . . . . . . . . . . . . . . . . . . . . . . . . . . . . . . . . . 388 ** reduce round timber to board measure, . . . . . . . . . . . . . . . . . . . . . . . . . . . . . . . . . . . . . . . . . . . . . . . . . 388–390 ** find board feet in planks, girders, joists, etc., . . . . . . . . . . . . . . . . . . . . . . . . . . . . . . . . . . . . . . . . . . . sº-º: * * * * * * * * * * * * tº º º & 272 ALPHABETICAL INDEX. XVII PAGES. MENSURATION.—PRACTICAL PROBLEMS. To find cubic feet and inches in solids, . . . . . . . . . . . . . . . . . . . . . . . . . . . . . . . . . . . .................. 393 cost freight per cubic feet, . . . . . . . . . . . . . . . . . . ....................................... 394 bushels in boxes, bins, etc., . . . . . . . . . . . . . . . . . . . . . . . . . . . . . . . . . . . . . . . . . . . . . . . . . . . . . . . . 394 cubic yards in levees or excavations, . . . . . . . . . . . . . . . . . . . . . . . . . . . . . . . . . . . . . . . . . . . . . . . 395 gallons in vessels, cisterns, etc., . . . . . . . . . . . . . . . . . . . . . . . . . . . . . . . . . . . . . . . . . . . . . . . . . . . 396 “ in oil tanks, . . . . . . . . . . . . . . . . . . . . . . . . . . . . . . . . . . . . . . . . . . . . . . . . . . . . . . . . . . . . . . 398 “ in horizontal cylinder, partly filled, . . . . . . . . . . . . . . . . . . . . . . . . . . . . . . . . . . . . . . . 398–400 barrels in a cistern, . . . . . . . . . . . . . . . . . . . . . . . . . . * * * * * * * * * * * * * * * * * * * * * * * * * * * * * * * * * * 400 loads of earth to fill lots, . . . . . . . . . . . . . . . . . . . . . . . . . . . . . . . . . . . . . . . . . . . . . . . . . . . . . 401 Squares of earth in grading, etc., . . . . . . . . . . . . . . . . . . . . . . . . . . . . . . . . . . . . . . . . . . . . . . . 402 cords of Wood in a rank, . . . . . . . . . . . . . . . . . . . . . . . . . . . . . . * * * * * * * * * * * * * * * * * * 402 Inumber of cars to carry Specified amount, . . . . . . . . . . . . . . . . . . . . . . . . . . . . * * * * * * * 403 & 4 of bricks in walls and foundations, . . . . . . . . . . . . . . . . . . . . . . . . . . . . . . . . . . . . . . 403 4 & perches in a stone wall, . . . . . . . . . . . . . . . . . . . . . . . . . . . . . . . . . . . . . . . . . . . . * - - - - 404 cubic contents of various vessels and solids, . . . . . . . . . . . . . . . . . . . . . . . . . . . . . . . . . . . . . . . . 405 diameter of cylindrical vessels to contain specified gallons, . . . . . . . . . . . . . . . . . . . . . . . 408 To 8*8° coal, . . . . . . . . . . . . . . . . . . . . . . . . . . . . . . . . . . . . . . . . . . . . . . . . . . . . . . . . . . . . . . . . . . . . . . . . . . . 409 To find tons of hay in stacks, mows, loads and windrows, . . . . . . . . . . . . . . . . . . . . . . . . . . . . . . . . . 110, 411 “ of hay in circular stacks, . . . . . . . . . . . . . . . . . . . . . . . . . . . . . . . . . . . . . . . . . . . . . . . . . . . . . 410 “ of hay in rectangular, stacks, . . . . . . . . . . . . . . . . . . . . . . . . . . . . . . . . . . . . . . . . . . . . . . . . . 411 approximate bushels in cribs, . . . . . . . . . . . . . . . . . . . . . . . . . . . . . . . . . . . . . . . . . . . . . . . . . . . . . . 412 bushels of grain in regular heaps, . . . . . . . . . . . . . . . . . y - e s • * * * * * * * * * * * * * * * * * * * * * * * * * * * * 413 To gauge barrels, casks, etc.,. . . . . . . . . . . . . . . . . . . . . . . . . . . . . . . . . . . . . . . . . . . . . . . . . . . . . . . . . . . . . . 413–416 To find tonnage of vessels, . . . . . . . . . . . . . . . . . . . . . . . . . . . . . . . . . . . . . . . . . . . . . . . . . . . . . . . . . . . . . . . . 416 Merchants and bankers’ discount, . . . . . . . . . . . . . . . . . . . . . . . . . . . . . . . . . . . . . . . . . . . . . . . . . . . . . . * - e. 576 Merchants’ system of partial payments, ... . . . . . . . . . . . . . . . . . . . . . . . . . . . . . . . . . . . • - - - - - - - - - - 617 Metric system of weights and measures, . . . . . . . . . . . . . . . . . . . . . . . . . . . . . . . . . . . . . . . . . . . . . . . . . 928–942 Mexican exchange, . . . . . . . . . . . . . . . . . . . . . . . . . . . . . . . . . . . . . . . . . . . . . . . . . . . . . . . . . . . . . . . . . . . . . . . . 771 Minuend, . . . . . . . . . . . . . . . . . . . . . . . . . . . . . . . . . . . . . . . . . . . . . . . . . . . . . . . . . . . . . . . . . . . . . . . . . . . . . . . . . . 64 Mint Weight, . . . . . . . . . . . . . . . . . . . . . . . . . . . . . . . . . . . . . . . . . . . . . . . . . . . . . . . . . . . . . . . . . . . . . . . . . . . . 249 Miscellaneous problems of all kinds, . . . . . . . . . . . . . . . . . . . . . . . . . . . . . . . . . . . . . . . . . . . . . . . . . . . . . . 951 Mixed numbers, . . . . . . . . . . . . . . . . . . . . . . . . . . . . . . . . . . . . . . . . . . . . . . . . . . . . . . . . . . . . . . • * * * * * * * * 165 Monetary units of different ages and nations, . . . . . . . . . . . . . . . . . . . . . . . . . . . . . . . . . . - * * * * 230 Mortality table, . . . . . . . . . . . . e e o e º 'º e s e e e º e e s is e e s a • * * * * * * * * * * * * * * * * * * e e s s e < * * * * - - s e = . .538, 541 Mortgage, def. of . . . . . . . . . . . . . . . . . . . . . . . . . . . . . . . . . . . . . . . . . . . . . . . . . . . . . . . . . . . e e < * * * * * * * * * 680 Multiples, def. of . . . . . . . . . . . . . . . . . . . . . . . . . . . . . . . . . . . . . . . . . . . . . . . . . . . . . . . . . . . . . . . . . . . . . - 147 Multiplicand, . . . . . . . . . . . . . . . . . . . . . . . . . . . . . . . . . . . . . . . . . . . . . . . . . . . . . . . . . . . . . . . . . . . . . . . . . . . . . 75 Multiplier. . . . . . . . . . . . . . . . . . . . . . . . . . . . . . . . . . . . . . . . . . . . . . . . . . . . . . . . . . . . . . . . . . . . . . . . . . . . 75 Multiplication, . . . . . . . . . . . e e s e º e º e º e s a e º e º e º ºs e e s e a s a s e s = e s e e s = e s - e. e. e. e s e s - e e s e s e e s - e. e. e. e. e. e. e. e. 75–122 , by aliquots, . . . . . . . . . . . . . . . . . . . . . . . . . . . . . . . . . . . . • e º sº e º sº. 4 e º a sº e • & © tº tº gº tº sº 94–101 , by factors, . . . . . . . . . . . . . . . . . . . . . . . . . . . . . . . . . . . . . . . . . . . . . . . . . . . . . . . . . . . . . . . 84 , contractions in, . . . . . . . . • * * * * * * * * * c e s e e s tº e º e a e e s a tº e º e s e e º 'º - e. e. e. e. e. g. e. e. e. e. n e s a e s - 88–121 , def. of . . . . . . . . . . . . . . . . . . . . . . . . . . . . . . . . . . . . . . . . . . . . . . . . . . . . . . . . . . . . . . . . 75, 183 , directions for, . . . . . . . . . . . . . . . . . . . . . . . . . . . . . . . . . . . . . . . . . . . . . . . . . . . . . . . . . . . . 85 importance of . . . . . . . . . . . . . . . . . . . . . . . . . . . . . . . . . . . . . . . . . . . . . . . . . . . . . . . . . . . . 77 of abstract numbers, with reasons, . . . . . . . . . . . . . . . . . . . . . . . . . . . . . . . . . . . . . . . . . 8() of decimals, . . . . . . . . . . . . . . . . . . . . . . . . . . . . . . . . . . . . a s a 5 - - a tº s tº e º O - © tº e º 'º - 225 of denominate numbers, . . . . . . . . . . . . . . . . . . . . . . . . . . . . . . . . . . . . . . . . . . . . . . . . . . . 271 of dollars and cents, . . . . . . . . . . . . . . . . . . . . . . . . . . . . . . . . . . . . . . . . . . . . . . . . . . . . .... • 84 of metric numbers, . . . . . . . . . . . . . . . . . . . . . . . . . . . . . . . . . . . . . . . . . . . . . . . . . . . . . . . . 938 , philosophic system of . . . . . . . . . . . . . . . . . . . . . . . . . . . . . . . . . . . . . . . . . . . . . . . . . . . . 77 , simultaneous or cross, . . . . . . . . . . . . . . . . . . . . . . . . . . . . . . . . . . . . . . . . . . . . . . . . . . . . 88 table to 25 × 25, . . . . . . . . . . . . . . . . . . . . . . . . . . . . . . . . . . . . . . . . . . . . . . . . . . . . . . . . . . 76 With naughts on right, . . . . . . . . . . . . . . . . . . . . . . . . . . . . . . . . . . . . . . . . . . . . . . . . . . 8.3 Nautical mile, def. of, . . . . . . . . . . . . . . . . . . . . . . . . . . . . . . . . • * * * * * * * * * * * * * * * * * * * * * * * * * * * * * * * 242 Negotiable paper, . . . . . . . . . . . . . . . . . . . . . . . . . . . . . . . . . . . . . . . . . . . . . . . . . . . . . . . . . . . . . . . . * * * - - 549 New Ham Sphire system of partial payments, . . . . . . . . . a e s a s s e s - e. e. e s a - - - e º e < * * * * * * * * * 618 Nicaraguan exchange, . . . . . . . . . . . . . . . . . . . . . . . . . . . . . . . . . . • , s , = e s a • * * * * * * * * * * * * * * * * * * * * * * 771 Nines, property of . . . . . . . . . . . . . . . . . . . . . . . . . . . . . . . . . . . . . . . . . . . . . . . . . . . . . . . . . . . . . . . . . . . . . . 113, 150 Non-participating policies in insurance, . . . . . . . . . . . . . . . . . . . . . . . . . . . . . . . . . . . . . . . . . . . . . . . . . . 511 Norwegian exchange. . . . . . . . . . . . . . . . . . . . . . . . . . . . . . . . . . . . . . . . . - * s - - - - - e º º is - - - - - e º 'º - - - 773 Notation, Arabic. . . . . . . . . . . . . . . . . . . . . . . . . . . . . . . . . . . . . . . . . . . . . . . . . . . . . . . . . . . * * * * * * - - 30 , def. of . . . . . . . . . . . . . . . . . . . . . . . . . . . . . . . . . . . . . . . . . . . . . . . . . . . . . . . . . . . . . . * - - - & Gº tº e - 30 XVIII ALPHABETICAL INDEX, PAGES. Netation, exercises in. . . . . . . . . . . . . . . . . . . . . . . . . . . . . . . . . . . . . . . . . . . . . . . . . . . . . . . . . . . . . . . . . . . . . . 33, 34 of decimals, . . . . . . . . . . . . . . . . . . . • * * * * * * * * > * > * > * * * * tº s º & e º º te e º ºs e g º º e º ºs e tº e º e ºs e º e e s is a e 216 , TOIll all. . . . . . . . . © tº 6 tº e º 'º e s e - e º us tº - • 6 e s tº e - © tº e e - is º ºs e º 'º tº tº 4 - - - 8 & 9 & 3 tº g º & © e º a tº - e g º e - ... ... 30, 31 Notes, discounting, . . . . . . . . . . . & © tº gº tº 3 º' tº e º e º 'º e º 'º º * @ e º 'º e º e e º e © tº e º e o e º ſº tº e º 'º e s tº e º 'º a s º e º e º e º e º e º 'º 576 , equation of interest bearing, . . . . . . . . . . . . e e e º 'º - © e º 'º e s to e º 'º tº tº ſº e º 'º e º 'º - e s e s tº e º e º º • e º e s a sº 649 & “ of maturing at different times, etc., . . . . . . . . . . . . . . . . . . . . . . . . . . . . . . . . . . . . . . . 651 { { of Tent, . . . . . . . . . . . . . . . . . . . . . . . . . . . . . . . . . . . . . . . . . . . . . . . . . . . . . . . . . . . . . . . . 648 , interest bearing, to collect and discount. . . . . . . . . . . . . . . . . . . . . . . . . . . . . is e - a s gº - - - e s - - 557, 580 , partial payments on, . . . . . . . . . . . . . * * * * * * * * * * * * * * * e a e e s a e a e s = s. s = < * * * * * * * * * * * * * * * * * * * 624 points in drawing, . . . . . . . . . . . . . . . . . . . . . . . . . . . . . . . . . . . . . . . . . . . a * * * * * * * * * * * * * * * * * * 559 , promissory, . . . . . . . . . . . . . . . . • * 0 e º e º 'º - e º 'º - - e s tº a tº º 6 e º 'º - e s tº e º 4 & a tº 4 - e. e º 'º - e º 'º e - © e º e º - s e º e - 549 , with collaterals on margin, . . . . . . . . . . . . . . . . . . © e º e o e º e º e º e s ∈ e º sº a ºn s a a t t t w e º s ºn tº t e o 'º º º 958 Numbers, defs. Of all kinds, . . . . . . . . . . . . . . . . . . . . . . . . • e s e º e º 'º e º e a s e e º a e s ºf s - s • * * * * * * * * * * * 25, 26, 146 including abstract, concrete, odd and even, perfect and imperfect, prime and composite, like and unlike, complement and supplement, etc. Numeration, def. Of, . . . . . . . . tº º e º e º 'º - - - tº e º 'º e º e s - - e º is º - e º & e º 'º e © & a - 4 º' tº e º 'º º e • * * * * * * * * * * e s e e º a 30 , English system of, . . . . . . . . . . © 3 & 4 º º o e º e º e e s sº e º 6 e º e º e º e s a e s & e º e s - e º e a º 'º tº € 8 & a 31 , exercises in, . . . . . . . . . . . . . . . . tº e º e º e º e º e º 'º - e º e - a tº e º 'º - • e s - tº s - e. e. e. * * * * * * * * * * * * * 33, 34 , French system of, . . . . . . . . . . . . . . . . • e e s e º e s a • * * * * * * * * * * - • * * * g e º e º 'º & w tº a 30 of decimals, . . . . . . . . . . . . . . . . . . . . . . . . . . . . . . . . . . . . . . . . . . . . . . . . . . . . . . . . . . . . . . . . 219 Numerator, def. of . . . . . . . . . . . . . . . . . . . . . . . . . . . . . . . . . . . . . . . . . . . . . . . . . . . . . - e. e - - - - - - - - - - - - 165 Numerical equation, def. of . . . . . . . . . . . . . . . . . . . . . . . . . . . . . . . . . . . . . . . . . . . . . . . . e - - - e < * - e. e. e - - 40 Oblate spheroid, . . . . . . . . . . . . . . . . . . . . . . . . . . . . . . . . . . . . . . . . . . . . * * * * * * * * * * * * * * * * * * * * * * * * 370 & 4 mensuration of . . . . . . . . . . . . . . . . . . . . . . . . . . . . . . . . . . . . . . . . . . . . . . . . . . . . . . . . . 379, 380 Odd numbers, def. of, . . . . . . . . . . . . . . . . . . . . . . . . . . . . . . . . . . . . . . . . . . . - - - e º - - - e. e - - - e. e. e - - - iº & - - 25 Oil tank measurements, . . . . . . . . . . . . . . . . . . . . . . . . . . . . . . . . . . . . . . . . . . . . . . . . . . . . . . . . . . . . . . . . . . . 398 Old French and Spanish lineasures, . . . . . . . . . . . . . . . . . . . . . . . . . . . . . . . . . . . . . . . . . . . . . . . . . . . . . . . 253 Open policy, def. of . . . . . . . . . . . . . . . . . . . . . . . . . . . . . . . . . . . . . . . . . . . • * * * * * * * s e º e e º 'º w s & © - * * * * * * 510 Operation, def. of . . . . . . . . . . . . . . . . . . . . . . . . . . . . . . . . . . . . . . . . . . . . . . e - - - ºr e - - * * * * * * ~ * * * - - 26 Order of figures. . . . . . . . . . . . . . . . . . . . . . . . . . . . . . . . . . . . . . . . . . . . . . . . . . . . . . . . . . . . . . . . . . . . . . 29 Origin of exchange, . . . . . . . . . . . . . . . . . . . . . . . . . . . . . . . . . . . . . . . . . . . . . . . . . . . . . . . . . . . . . . . . . . . . . • 711 Painting and glazing, . . . . . . * * * * * * * * tº e º is e º e º e º 'º º ºs e º a º e º e º 'º º is e º 'º - e º s º e < e e * - © s tº e º a tº e tº & B tº ſº tº ſº a w 366 Par of exchange, . . . . . . * * * * * * * * * * s e a e º e s e tº e e º e - © tº e º 'º tº º s tº e º e º a tº dº e º e º 'º - a s & © to e º e º 'º - e º 'º tº º e º e 4 o e 716 of Stock, . . . . . . . . . . . . . . . . • * * * * * * * * * * * * e s e e º e s is a • * * * * * * * * * * * * * * * * * * * * * * * * * * * e e a e e s e s - e. 677 Parallelogram, def. Of, . . . . . . . * * * * * * * * * * * * * * * * * * * * * * * * * * * * * * * * * * * * * * * * * * * * * * * * * * * * * * * * * * * * * 338 to find area of . . . . . . . . . . . . . . . . . . . . . . . . . . . . . . . . . . . . . . . . . . . . . . . . . . . . . . . . . . . . . 346 Parenthesis and Vinculum, use of, . . . . . . . . . . . . . . . . . . . . . . . . . . . . . . . . . . . . . . . . . . . . . . . . . . . . . . . 134 Paris exchange on other cities, . . . . . . . . . . • * * * * * * * is a e < * * * * * * * * * * * * * * * * * * * * * * * * * * * * * * * * * * * * * * 764 Partial payments, . . . . . . . . . . . . . . . . . . . . . . . . . . . . . . . . . . • e s e º e s e º 'º e s a s e e s s a s e o e e s s e s e º e s e s e e s s , 616–627 Five systems defined; problems worked in full. Particular and general average, . . . . . . . . . . . . . . . . . . . . . . . . . . . . . . . . . . . . . . . . . . . . • * * * * * * * * * * * * * * 527 Parties to notes, bills and drafts, . . . . . . tº e - © e º 'º e º ºs e º 'º - e º e s - - e. e. e. e. • * g e º - - - tº e º 'º e º e < e - e. e. e. * e º e e s - 549, 554 Partitive proportion, . . . . . . . . . © e º ºs s tº º ſº º º ºs e º e º 'º º ºs º º º © º e º e º & © tº dº tº tº º e º e º e º & tº e º e • * * * * * * * * * * * * • * 332 Paving yards and walks, . . . . . . . . * @ & E * : * ~ 3 - e. e. e. e. e. e ſº e º O & e º e º & º e s - a e º e tº - e º 'º e º o * * * * * * * * * > e s sº e - e. 360 Percentage, . . . . . . . . . . . . . . . . . . . . tº tº º º & © w is e - * g e º - - tº º e s - © tº e º e º s º e º 'º - a s e - e º a tº e º ºs e e º 'º' tº e & s • * * e º ºn a 434–480 definitions; tables of aliquots; to find percentage, base, rate per cent, amount or difference in all combinations; per cent on English money; marking goods; impor- ting English goods; trade discounts and list prices; series of discounts reduced to one equivalent; discounts on bills; tables of trade discounts; value of sugar cane per ton; quantitative chemical analysis; to find sucrose multiplier; practical questions. Permutations and combinations and chance, . . . . . . . . . . . . . . . . . . . . . . . . . . * * * * * * * * * * * * * * * * * * * * * 943 Peruvian exchange, . . . . . . . . . . . . . . . . . . . . . * * * * * * * * * * * * * * e s - e. e. e. e. e. e. e. e s = e º e s = • * * * * * * * * * * * * * * * * * 771 Philosophic solution, def, of, . . . . . . . . . . . . . . . . . . . . . . . . • e s tº a s e s e º 'º e º 'º e - e. e º e º e a s e º e º e s e e e e º 'º e e 26 system, def. of . . . . . . . . . . . . . . . . . & © tº 0 ° tº º & © tº e º & - - e º e º - 4 º' s e < * * * * * * tº e º 'º e º e e º 'º - e. e. e. e. e. 26 ‘‘ of contracting interest, . . . . . . . . e - e s e - e. e s s a s e e s - 4 e 4 e º e º 'º - e s s e º 'º e º ºs e - 566 & 4 of multiplication, . . . . . . . tº º 0 e º a 9 • * * * * * * * * * * * * * * * * * * * * * * * * * * * * * * * * * * * * * * 77 ( & of proportion, . . . . . . . . . . . . . . . . . . • e e s e º a s e e s e a s a • * * * * * * * * * * * * * * * * * * * * * * 315 Phrases and words used by bankers and brokers, . . . . . . . . . . . . . . . . . . . . . . . . . . . . . . . . . . . . . . . 682 Plane figure, def. of . . . . . . . . . . . . . . . . . . . . . • * g º e º 'º a ſº e g º a e g º e s a s a 4 s a tº a s a s is e º e s s a e º e º e º º e º e º e - 337 Plasterers’ work, . . . . . . . . • * * * * * * * * * * ~ * * * ~ * c e e e e º e s e e e a e s e e e º a 4 • e s e s a s a s = e º s s e s a e s a e s a e e s - 365 Plumbers’ work, . . . . . . . . * * * * * * * * * * * * * * * * * * * g e º s e e s e º e s e a º e º e º s - s = - * g is - - - - - - - - - * * - - - - - - - 363 Points in drawing notes, . . . . . . . . . . . . . . . . . . . . . © tº t e º e tº º e º ſº tº e e e s a e º me tº e - sº e e º tº © tº e º is - © e g tº tº º ºr º e º 'º 559 ALPHABETICAL INDEX. XIX PAGES. Policies, insurance, defs, of . . . . . . . . . . . . . . . . . . . . . . . . . . . . . . . . . . . . . . . . . . . . . • * * * * * * * c tº e º & 509, 510, 533 Polygon, def. of, . . . . . . . . * * * * * * * * * * * * * * * * * * * * * * * * * * * * * * * * * * * * * * * * * * * * * * * * * * * * * * * * * * * * * * * * * * 337 Ports of entry, def. of . . . . . . . . . . . . . . . . . . . . . . . . . . . . . . . . . . . . . . . . . . . . . . . . . . . . . . . . . . . . . . . . . . . . 501 Portuguese exchange, . . . . . . . . . . . . . . . . . . . . . . . . . . . . . . . . . . . . . . . . . . . . . . . . . . . . . . . . . . . . . . . . . . . . . 770 Powers of numbers, . . . . . . . . . . . . . . . . . . . . . . . . . . . . . . . . . . . . . . . . . . . . . . . . . . . . . . . . . . . . . . . . . . . . . . . 147 Practical operations with aliquots and other numbers, . . . . . . . . . . . . . . . . . . . . . . . . . 96–101, and 278–283. problems in mensuration of surfaces and solids, . . . . . . . . . . . . . . . . . . . . . . 357–360 and 393–417 { { Ill percentage, . . . . . . . . . . . . . . . . . . . . . . . . . . . . . . . . . . . . . . . . . . . . . . . . . . . . . . . . . 470–479 & 4 in stocks and bonds, . . . . . . . . . . . . . . . . . . . . . . . . . . . . . . . . . . . . . . . . . . . . . . . . . . . 698. Preferred creditors, def. of, . . . . . . . . . . . . . . . . . . . . . . . . . . . . . . . . . . . . . . . . . . . . . . . . . . . . . . . . . . . . . . . . 487 stock, . . . . . . . . . . . . . . . . . . . . . . . . . . . . . . . . . . . . . . . . . . . . . . . . . . . . . . . . . . . . . . . . . . . . . . . . . . . 677 Premises of problems, . . . . . . . . . . . . . . . . . . . . . . . . . . . . . . . . . . . . . . . . . . . . . . . . . . . . . . . . . . . . . . . . . . . 26 Price of stocks from English and French quotations, . . . . . . . . . . . . . . . . . . . . . . . . . . . . . . . . . . . 708, 709 Prime factor, . . . . . . . . . . . . . . . . . . . . . . . . . . . . . . . . . . . . . . . . . . . . . . . . . . . . . . . . . . . . . . . . . . . . . . . . . . . . . 146 number, . . . . . . . . . . . . . . . . . . . . . . . . . . . . . . . . . . . . . . . . . • a • * g º e a 9 - e º s e s tº e s a e º 'º - tº e º ºs e s tº e º 'º. 146 Principles of decimals, . . . . . . . . . . . . . . . . . . . . . . . . . . . . . . . . . . . . . . s e s - e. e. e. e. e. e s m = * * * * * * * * * * * * * * 220 of fractions, . . . . . . . . . . . . . . . . . . • * * * g e º º a s a e s a 4 e º e º a s e º 'º - - - - e º e º a tº e º 'º º 166 Problem, def. of, . . . . . . . . . . . . . . . . . . . . . . . . . . * * * * * * * * > * * * * * * * * * * * * * * * * * * * * * * * * * * * * * * * * * 26 • PſelhlSeS OT, . . . . . . . . . . . . . . . . . . . . . . . . . . . . . . . . . . . . . . . . . . . . . . . . . . . . . . . . . . . . . . . . . . 26. , solution of . . . . . . . . . . . . . . . . . . . . . . . . . . . . . . . . . . . . . . . . . . . . . . . . . . . . . . . . . . . . . . . . . . . 26 Product, def. of . . . . . . . . . . . . . . . . . . . . . . . . . . . . . . . . . . . . . . . . . . . . . . . . . . . . . . . . . . . . . . . . . . . . . . . . 75 method of account current and interest accounts, . . . . . . . . . . . . . . . . . . . . . . . . . . . . . . . 653 Profit and loss, . . . . . . . . . . . . . . . . . . . . . . . . . . . . . . . . . . . . . . . . . . . . . . . . . . . . . . . . . e is a e s º e s is e º e º & © 2 450 Progression, arithmetical, . . . . . . . . . . . . . . . . . . . . . . . . . . . . . . . . . . . . . . . . . . . . . . . . . . . . . . . . . . 901–908 , geometrical, . . . . . . . . . . . . . . . . . . . . . . . . . . . . . . . . . . . . . . . . • * * * * * * * * * * * * 908–920 Prolate spheroid, def. of . . . . . . . . . . . . . . . . . . . . . . . . . . . . . . . . . . . . . . . . . . . • * e s tº e º e º 'o 370 * { mensuration of, . . . . . . . . . . . . . . . * * * - - - - - e. • a • * * * * * * * * * * * * * * * 379 Promissory notes and negotiable paper, . . . . . . . . . . . . . . . . . . . . . . . . . . . . . . . . . . . . . . . . . . . . . . . . 549–554 Proof of addition, . . . . . . . . . . . . . . . . . . . . . . . . . . . . . . . . . . . . . . . . . . . . . . . . . . . . . . . . . . . . . 47, 152, 1005, 1008 of division, . . . . . . . . . . . . . . . . . . . . . . . . . . . . . . . . . . . . . . . . . . . . . . . . . . . . . . . . . . . . . . . . . . . . . . 123, 154 of multiplication, . . . . . . . . . . . . . . . . . . . . . . . . . . . . . . . . . . . . . . . . . . . . . . . . . . . . . . . . . . . . . . . . . . 75, 154 of subtraction, . . . . . . . . . . . . . . . . . . . . . . . . . . . . . . . . . . . . . . . . . . . . . . . . . . . . . . . . . . . . . . . . . . . . . 64, 153 of transfers and postings, . . . . . . . . . . . . . . . . . . . . . . . . . . . . . . . . . . . . . . . . . . . . . . . . . . . . . . . . . . . 156 Proper fraction, def. of . . . . . . . . . . . . . . . . . . . . . . . . . . . . . . . . . . . . . . . . . . . . . . . . . . . . . . . . . . . . . . . . . . . 165 Properties of nines and elevens, . . . . . . . . . . . . . . . . . . . . . . . . . . . . . . . . . . . . . . . . . . . . . . . . . . . . . . . . . . . 150–157 Proportion, . . . . . . . . . . . . . . . . . . . . . . . . . . . . . . . . . . . . . . . . . . . . . . . . . . . . . . . . . . . . . . . . . . . . . . . . . . . . . . . 310–336 definitions and terms; general principles; relationship and equivalency of numbers; Solutions by proportion, analysis, and the philosophic systems compared; partitive proportion; medial proportion; conjoined proportion; reciprocal proportion. Proportional division, . . . . . . . . . . . . . . . . . . . . . . . . . . . . . . . . . . . . . . . . . . . . . . . . . . . . . . . . . . . . . . . . . . . . . 871–876. Pulleys, kinds and use of . . . . . . . . . . . . . . . . . . . . . . . . . . . . . . . . . . . . . . . . . . . . . . . . . . . . . . . . . . . . . . . . . 424–427 Purchasing French exchange from distant cities, . . . . . . . . . . . . . . . . . . . . . . . . . . . . . . . . . . . . . . . . . . 762 Pyramid, def. of . . . . . . . . . . . . . . . . . . . . . . . . . . . . . . . . . . . . . . . . . . . . . • * * * * * * * * * * * * * * * * * * * * * * * * * * * * 37(). PARTNERSHIP-SETTLEMENTS. All problems worked in full. Partnership, definition of, . . . . . . . . . . . . . . . . . . . . . . . . . . . . . . . . . . . . . . . . . . . . . . . . . . . . . . . . . . . . . . . . . 803 , sharing gains and losses of, . . . . . . . . . . . . . . . . . . . . . . . . . . . . . . . . . . . . . . . . . . . . . . . . . . 803 , commercial , ordinary | º!. | In Louisiana, . . . . . . . . . . . . . . . . . . . . . . . . . . . . . . . . . . . . . . . . . . . . . . . 803–804 , in commendam | , limited , trading or commercial \ , non-trading | 3 ºr ~~~ | yº In common law States, . . . . . . . . . . . . . . . . . . . . . . . . . . . . . . 804–805. , particular | , limited , formation of three methods, . . . . . . . . . . . . . . . . . . . . . . . . . . . . . . . . . . . . . . . . . . . . . . . . . . 805 , dissolution of . . . . . . . . . . . . . . . . . . . . . . . . . . . . . . . . . . . . . . . . . . . . . . . . . . . . . . . . . . . . . . . . 806 Partners, nominal or ostensible, silent, secret, sleeping or dormant partners and sub-partners, 805 Partnership, articles of . . . . . . . . . . . . . . . . . . . . . . . . . . . . . . . . . . . . . . . . . . . . . . . . . . . . . . . . . . . . . . . . . . . 806–807 , settlements, discussion of, . . . . . . . . . . . . . . . . . . . . . . . . . . . . . . . . . . . . . . . . . . . . . . . . . . . . 808 Resources, assets or effects, defined. . . . . . . . . . . . . . . . . . . . . . . . . . . . . . . . . . . . . . . . . . . . . . . . . . . . . . . . 808 Liabilities, defined, . . . . . . . . . . . . . . . . . . . . . . . . . . . . . . . . . . . . . . . . . . . . • * * * * * * * * * * * * * * * s tº e º e º e º e 808. XX ALPHABETICAL INDEX. PAGES. Investment, net investment, average investment, defined, , , , . . . . . . . . . . . . . . . . . . . . . . . . . . . . . . . 809 Capital, net capital, insolvency, net gain, and net loss, defined, . . . . . . . . . . . . . . . . . . . . . . . . . . . . 809 Two methods of determining gain or loss, . . . . . . . . . . . . . . . . . . . . . . . . . . . . . . . . . . . . . . . . . . . . . . . . . 809–810 Oral exercises, . . . . . . . . . . . . . . . . . . . . . . . . . . . . . . . . . . . . . . . . . . . . . . . . . . . . . . . . . . . . . . . . . . . . . . . . . . . . 810 Settlement, involving net investments, . . . . . . . . . . . . . . . . . . . . . . . . . . . . . . . . . . . . . . . . . . . . . . . . . . . . 811 y * { “ Withdrawals, . . . . . . . . . . . . . . . . . . . . . . . . . . . . . . . . . . . . . . . . . . . . . . . . . . . 811 % * { & & { { and net investments, . . . . . . . . . . . . . . . . . . . . . . . . . . . . . . 811 from statement of resources and liabilities, and gains and losses, by two methods, With Variations, . . . . . . . . . . . . . . . . . . . . . . . . . . . . . . . . . . . . . . . . . . . . . . . . . . . . . . . . . 812 made in practical accounting form, . . . . . . . . . . . . . . . . . . . . . . . . . . . . . . . . . . . . . . . . . . . . 813 from trial balance, by resource and liability method, . . . . . . . . . . . . . . . . . . . . . . . . . . 814 & 4 { { by loss and gain method, . . . . . . . . . . . . . . . . . . . . . . . . . . . . . . . . . . . 815 between physicians, in practical form, . . . . . . . . . . . . . . . . . . . . . . . . . . . . . . . . . . . . . . . . . 815–816 where no contract was made regarding gains and losses, . . . . . . . . . . . . . . . . . . . . . . 817–818 by note, from stated conditions, . . . . . . . . . . . . . . . . . . . . . . . . . . . . . . . . . . . . . . . . . . . . . . . 818 from trial balance, with conditions, . . . . . . . . . . . . . . . . . . . . . . . . . . . . . . . . . . . . . . . . . . 819 between partners, with salary allowances, . . . . . . . . . . . . . . . . . . . . . . . . . . . . . . . . . . . . . 819 with steamboat owners, with conditions, . . . . . . . . . . . . . . . . . . . . . . . . . . . . . . . . . . . . . . 819–820 to adjust current accounts with the entries, . . . . . . . . . . . . . . . . . . . . . . . . . . . . . . . . . . . 820–821 * { cost of driving logs, . . . . . . . . . . . . . . . . . . . . . . . . . . . . . . . . . . . . . . . . . . . . . . . . 821–822 between manufacturer and planter (on shares), . . . . . . . . . . . . . . . . . . . . . . . . . . . . . . . . 822 { { merchant and planter, to gin, . . . . . . . . . . . . . . . . . . . . . . . . . . . . . . . . . . . . . . . . 822–82 * * son and daughter, . . . . . . . . . . . . . . . . . . . . . . . . . . . . . . . . . . . . . . . . . . . . . . . . . . . 823 • { children, from a will, . . . . . . . . . . . . . . . . . . . . . . . . . . . . . . . . . . . . . . . . . . . . . . . . 823 to find gain from resources and liabilities, . . . . . . . . . . . . . . . . . . . . . . . . . . . . . . . . . . . . . 823 { { net capital of partners at closing, . . . . . . . . . . . . . . . . . . . . . . . . . . . . . . . . . . . . . 824 from unequal investments and unequal shares in gains and losses, . . . . . . . . . . . . . . 824–825 to find net capital at commencing, . . . . . . . . . . . . . . . . . . . . . . . . . . . . . . . . . . . . . . . . . . . . 825 * * original investments, . . . . . . . . . . . . . . . . . . . . . . . . . . . . . . . . . . . . . . . . . . . . . . . . . . 825–826 net insolvency at commencing and capital or insolvency at closing, . . . . . 826 k . net capital, from irregularly kept books, . . . . . . . . . . . . . . . . . . . . . . . . . . . . . . . 826–827 * { respective net gains, with salaries allowed, . . . . . . . . . . . . . . . . . . . . . . . . . . . . . 827 & 4 gain and capital from trial balance, when goods and books are burned, . 828 { { value of merchandise from trial balance, . . . . . . . . . . . . . . . . . . . . . . . . . . . . . . 829 { { met investment after reinvestment of profits, . . . . . . . . . . . . . . . . . . . . . . . . . . . 829–830 & 4 amount of cash stolen by dishonest book-keeper, . . . . . . . . . . . . . . . . . . . . . . . 830 to adjust accounts of partners, without entries in books, . . . . . . . . . . . . . . . . . . . . . . . 830 • { debit balances of partners, with unequal interests in gains and losses, 830 { { cost of cotton factory between contractors, . . . . . . . . . . . . . . . . . . . . . . . . . . 831 between cattle dealers, from stated conditions, . . . . . . . . . . . . . . . . . . . . . . . . . . . . . . . 831–832 to adjust gain proportionately, with allowance for reserve fund, . . . . . . . . . . . . . . 832–833 between principal and agent, . . . . . . . . . . . . . . . . . . . . . . . . . . . . . . . . . . . . . . . . . . . . . . . . . 833–83.1 to find cash balance, net gain and present worth of a branch store—three solu- tions, . . . . . . . . . . . . . . . . . . . . . . . . . . . . . . . . . . . . . . . . . . . . . . . . . . . . . . . . . . . . . . . . . . 834–837 to find gain and capital of proprietor of branch store; also condition of mana- ger's account. . . . . . . . . . . . . . . . . . . . . . . . . . . . . . . . . . . . . . . . . . . . . . . . . . . . . . . . . . . . 838 with agent of branch house, to determine cash on hand, net gain and net worth, .838–839 With branch store—two solutions, . . . . . . . . . . . . . . . . . . . . . . . . . . . . . . . . . . . . . . . . . . . . . 840–841 between temperance town and druggist agent, . . . . . . . . . . . . . . . . . . . . . . . . . . . . . . . . 842—843 of interests in proportion to amounts invested, . . . . . . . . . . . . . . . . . . . . . . . . . . . . . . . . 843 to find net gain of a manufacturing agency, . . . . . . . . . . . . . . . . . . . . . . . . . . . . . . . . . . . 84.4 ( & net Sales, list price, and cost, . . . . . . . . . . . . . . . . . . . . . . . . . . . . . . . . . . . . . . . . . . 845 to adjust 1nterest on partners’ accounts, . . . . . . . . . . . . . . . . . . . . . . . . . . . . . . . . . . . . . . . 845–846 to find partners' balances, . . . . . . . . . . . . . . . . . . . . . . . . . . . . . . . . . . . . . . . . . . . . . . . . . . . . 846–818 between book-keeper and firm, with entry, . . . . . . . . . . . . . . . . . . . . . . . . . . . . . . . . . . . . 848 & 4 partners, with salary allowance, . . . . . . . . . . . . . . . . . . . . . . . . . . . . . . . . . . . . . 848 to adjust interest balances, with entries . . . . . . . . . . . . . . . . . . . . . . . . . . . . . . . . . . . . . . . 849 & f & 4 in case of dissolution, . . . . . . . . . . . . . . . . . . . . . . . . . . . . . . . . . . . . . 849 between mechanics, for lost time, with entry, . . . . . . . . . . . . . . . . . . . . • * * * * * * * * * * * 850 to adjust gain on admission of new partner, . . . . . . . . . . . . . . . . . . . . . . . . . . . . . 850 { { anticipated loss, . . . . . . . . . . . . . . . . . . . . . . . . . . . . . . . . . * = e is e e s s • * * * ‘. . . . . 850 ( & commission, charges and interest, . . . . . . . . . . . . . . . . . . . . . . . . . . . . . . . . . . . 851 ( & coin and currency accounts, . . . . . . . . . . . . . . . . . . . . . . . . . . . . . . . . . . . . . . . . . . 851 to reduce coin balances to currency, and vice versa, . . . . . . . . . . . . . . . . . . . . . . . . . . . . . 851—852 between partners of a note, considered doubtful, with entries, . . . . . . . . . . . . . . . . . 852–853 of partners' balances after division of resources, with entry, . . . . . . . . . . . . . . . . . . ; o changing single entry to double entry, using single entry ledger, . . . . . . . . . . . . . . ALPHABETICAL INDEX. XXI PAGES. Settlement to open new books from irregular set, . . . . . . . . . . . . . . . . . . . . . . . . . . . . . . . . . . . . . . . . 855 between partners from given resources and liabilities, .........................855–856 e & 4 “ , one of whom withdraws,....................................856-858 Admission of a new partner into a commercial firm, (four cases), . . . . . . . . . . . . . . . . . . . . . . . . . . .858–860 Partnership and partnership average, ...... * * * * * * * * * g e º e - • e º e º 'º e º e º a e e º e e e º e º e º 'º e & © e º e º ºs e º e 861 Simple partnership average problems, solved, ...................... • * r * e s a s e º e s e e º e º sº e s e - 861–863 Compound “ & 4 { { { { * * * * * * is a e e º e a e s a s e tº ſº tº a 6 - - e º is e º e . . . . . . . . . . . . .863–867 Averaging Sales on commission, ....................... * * * * * * * * * * * * * * * * * * * * © º ºr e º 'º e º ſº tº e º tº e 867 Cotton average problems, solved, . . . . . . . . . . . . . . . . * * * * * * * * * * * * * * e s a e e º e s a a e s a © e º a º ºs e e s is e º 'º a 868 Division and proportional division, . . . . . . . . . . . . . . . . . . . . . . . . * * * * * * * * * * * * * e s a e º e º e º a © tº e º 'º tº e º & 871 Settlements, with Special conditions, . . . . . . . . . . . * * * * * * * * * * * * * * * * * * * * * * * * * * * * * * * * * * * * * * * * * * * 872 Division of proceeds of partial interest sold, in order to equalize remaining investments, ... .872–873 Division of proceeds of a dinner in proportion to the quantities furnished. . . . . . . . . . e e º s º e - 873 To find average rate per cent of insurance on cotton for different times and rates, . . . . . . . . . . 873 Settlement with the different grades of cotton, . . . . . . . . . . . . . . . . . . . . tº e º 'º e º is is e e º e º e º º is a s & tº e º e 874 Settling estates, . . . . . . . * * * * * * * * * * * * * * * * * * * * * * * * * * * * e e s e e o e s - e º e s e e tº e º e º e º 'º e g º e º e º 'º º ºs e º 'º º sº º º 874 Division of remainder of an estate in reciprocal proportion to ages, . . . . . . . . . . . . . . . . . . . . . . . .874–875 Division between two surviving heirs, in certain proportions, . . . . . . . . . . . . . . . . . . . • e s e e º e º e º 4 875 Division of an estate, dependent upon return of certain heirs, . . . . . e - e º s tº e º e tº e º e º e º e e e s & 6 tº 876 To find amount of an estate, when shares are given, . . . . . . . . . . . . . . º e < e < e < * * * * * • * g º e º is tº s e s 6 tº 876 Proportional division, . . . . . . . . . . . . . . . . . . . . . . . . . . . . . . . . . . * * * * * * * * * * * • ‘º e º e º e º e º 'º º . . . . . . . . . . .876–877 The land problem, . . . . . . . . . . . . . . . . . . . . . . . . . . . . . . . . . . . . . . 6 * * * * * * * * * * * * * * * c e is e e s & e º 'º tº º e º 'º e º 'º & 878 The ditch problem, . . . . . . . . . . . . . . . . . . . . to e º e º e º 'º - e º e g º ºs e º 8 tº º º * * * * * * * * * * * * * * * * e º e º e º e º e tº e º 'º º º 878 Division of work and funds, . . . . . . . . . . . . . . . . . e - e º e s e e o e • * * * * * * * * * * * * * * * * e s e e s e e s e º e s a e e s e - To find number of stolen articles, when shares are given, . . . . . . . . tº e º e º 'º - e. g. g º ºs e º 'º - - - - . . . . . . . .879–880 Division of land in shares, . . . . . • * * * e º e º e º e º e º e º 'º e º e º e e tº e e º e º 'º e * 9 & & © tº tº 6 tº tº e º 'º e º 'º - º ºr tº º º tº º tº e º ſº. 880 To find interest in mining company, . . . . . . . . . . . . . . * * * * * * * * * * * * * * tº e º e º e º e º te e º e º 'º e ‘º e º 'º º e s e º sº º 880 with portion of gain to be credited to Division of gain dependent upon fixed investments, . . . . . . . . . . . . . . . . . . . . . . . . . . . . . . . . . . . . . . . .881-882 TěSèI'We fund, . . . . . . . . . . . . . . . . . . . . . . . . . . . . . . . . C. Quantitative chemical analysis, . . . . . . . . . . . . . . . . . . . . . . . . . . . . . . . . . . . . . . . . . . . . . . . . . . . . . . . . . . . . Quantity, def. of . . . . . . . . . . . . . . . . . . . . . . . . . . . . . . . . . . . . . . . . . . . . . . . . . . . . © º e º e s ∈ e s a e - © tº e º e º 'º e º & * 607 • * * * * c e e º e º 'º - e < * * * * * * * * * * * * * * * * * * * * * * * * * s e tº º º e e Quarterly interest, . . . . . . . . . . . . . . . Quarternary scale reduced to decimal, . . . . . . . . . . . . . . . . . . . . . . . . . . . . . . . . e - © e º e º a º e º 'º - 4 e º & e º 'º º 28 Quinary scale reduced to decimal, . . . . . . . . . . . . . . . . . . . . . . . . . . . . . . . . . . . . - - - © tº e º g º º e e º º te tº © e º e e 28 © º e tº e º ſº º ſº 123 Quotient, . . . . . . . . . . . . . . . . . . . . . . . . . . . . . . . . . . . . . . . . . . . . . . . . . . . . . . . tº ſº e º ſº º tº e º e tº e º 'º a tº e J R Radix, def of . . . . . . . . . . . . . . . . . . . . . . . . . . . . . . . . . . . . . . . . . . . . . . . . . . . . . . . . . . . . tº e e e e º e e º e 434 Rate, def. of . . . . . . . . . . . . . . . . . . . . . . . . . . . . . . . . . . . . . . . . . . . . . . . . . . . . . . . . . . . . . . • e º ſº e º 'º - of commission and brokerage, . . . . . tº e s e º e º a tº e º e º 'º e º e º 'o • * * * c e a e º e • * e s e e s e º e - tº º e º e e e s e e 480 of exchange, . . . . . . . . . . . . . . . . . . . . . . . . . . . . . . . . . . & e º e º 'º e º sº e º e s s a tº e © tº º º & © tº e e ... 716, 727, 729, 744 of insurance premium, . . . . . . . . . . . . . . . . . . . . . . . . . . . . . . e - © tº tº º gº e º 'º e © tº o e º ſº tº º e º 'º - . . . . .510, 515, 534 of interest in sale by installment plan, . . . . . . . . . . . . . . • ‘º e º e º e s e º sº e < e < * * * * * e º ºs e º e º e º º e º e 592 of storage, . . . . . . • * * * * * * * * * * * * * * * * * * * * * * * * * * * * * * * * * * * * e < * * * * * tº e º e º is e º 4 s e s a e & © - s e º 'º e º e 777 Ratio, defs, of . . . . . . . . . . . . . . . . . . . . . . . . . . . . • e e º 'º - e. e. e. e. © e º e º 0 & © tº e º s a a s e º a • * * * * * * * * * * * * * * * 308 , principles of . . . . . . . . . . . . . . . . . . . . . . . . . . . . . . . * * * tº a º e º 'º - e s tº e tº dº tº e > © e º º s º º & © tº 3 º' tº a tº g º g g tº e 309 of diameter and circumference, . . . . . . . . . . . . tº tº ſº e º e & © e º 'º e º ºs e e s º º e º e º º * is e º 'º º tº ºn e º a tº e º is e - 338 of sphere and cube, . . . . . . . . . . . . . . . . © e º ºs e s - e. * * * * * * * * * * * * * * * * * * * * * * * e º e º e tº e º 'º e º sº tº e e s = 377 of square and circle, . . . . . . . . . . . . . . . . . . . © e º e º 'º e º 'º e º 'º e º e º tº e g g g tº e º a • * * * * a s e s - e. e. e. e. e. e. e. g. a e 33 Receipts, forms of, . . . . . . . . . . . . e & © tº e º e e º o e e º e º 'º * - - - - e º e º 'º - tº - - - 6. © e. e. e º s - - - - e s e º e º e s - - - - tº º e e º e 551 , storage or Warehouse, . . . . . . . . . . . . . . . . . . . . . . . . . © e º s a e º e s e < e < e < e e s a s e = * * * * * * * * * * * * 777 Reciprocal of a number, . . . . . . . . . . . . . . . . . . . . . . • * e º e º & tº e º & e • * * * * * * * * * * * * * * * * * * * * * * * * * * * * * e 147 of a fraction, . . . . . . . . . . . . . . . . . . . . . . . . . . . . . . . . . . . . . . . . . . . . . . . . . . . . . . . . . . . . . . . . . . . 166 proportion, . . . . . . . . . . . . . . • * * * * * * * c e s = e º e º e e s • * * * * * * * * * * * * * * * * * * * * * * * * * 6 g º a º is s tº e 334 Rectangle, def. of . . . . . . . . . . . . . . . . . . . . . . . . . . . . . . . . . . . . . . . . . . . . . . . . . . . . . . . . . . . . . . . . ºn tº e s s a s 337 Reduce shillings and pence to decimal of a pound, . . . . . . . . . . . . . . . . . . . . . . . . . . . . . . . . . . . . . . 450 Reduction of decimals, . . . . . . . . . . . . . . . . . . . . . . . . . . . . . . . * c e s - e. e. e. e. a = e º º e s s e a s = * * * * * * * * * * * * 220–223 of denominate numbers. . . . . . . . e e s e º a s a s a s s e º e º 'º - e < * s e º a e < * * * * * * * * * * * * * * * * * * * * * 256–267 of fractions, . . . . . . . . . . . . . . . . & e e º ºs e º 's e º e º 'º e º 'º º tº a º * @ e º 'º e tº e º 'º e º 'º - e º ºs e º 'º e º 'º - © 4 & 8 g º e & 167–175 of metric numbers . . . . . . • * * * * e s a e e < * * * * * * * * * * * * * * * * * * * • e º e º 'º e º s e e º e º ºs tº s º dº º is tº e a 935 of “ ( 4 to American equivalents and vice versa, . . . . . . . . . . . . • * * * * * * * * * 940 Registered bonds, . . . . . . . . . . . . e e º e s - e. e s e e º f * * * * * * * * * * * * * * * * - e s e º e g is e º e º 'º e º s e º e < e s - © e º e º e º a tº 680 Reinsurance, . . . . . . . . . . . . . . . . . • * * * * * * * * * is e s a s = e º e º e º 'º e s is a • * * * * * * g e º 'º e º e º e º 'º e a tº e º e º 'º e º a tº e º e º 511 Relationship of numbers, . . . . . . . . . . . . . . . . . . . . . . . . . . . . . . . . © e º a tº & XXII AL PHABETICAL INDEX, PAGES. Remarks on measurement of segments, . . . . . . . . . . . . . . . . . . . . . . . . . . . . . . . . . . . . . . . . . . . . . . . . . . . . 382 Rene Wing notes, . . . . . . . . . . . . . . . . . . . . . . . . . . . . . . . . . . . . . . . . . . . . . . . . . . . . . . . . . . . . . . . . . . . . . . . 589 Rent notes, equation of, . . . . . . . . . tº tº 4 & © - tº & e º 'º & * & ſº & e º 'º e º 'º e º e º e g * * * * * * * * * * * * * * * * * © tº - tº e º 'º º e º s e a 648 Repetends, def. of, . . . . . . . . . . . . . . . . . . . • * tº e º 'º e g tº e º e º e a * * * * * * * * * * * * * * * * * * * is º a s s s a º a s e s a e s = n e s 217 Reserve fund, . . . . . . . . . . . . . . . . . . . . . . © & © tº ſº e º ºs * * * * * * * * * * * * * * * * * * * * * * * * * * * * = e s a s • e a e º a a s = e a e s , 678 Resources, def. of, . . . . . . . . . . . . * * * * * * * * * * * * * * * * * * * * * * * * * * * * * * s s a s a s is a e e s = e s s a e s e a < * * * * * * * 487 Respondentia and bottomry, . . . . . . . . * * * * * * * * * * * * * * * * * * * * * * * * * * * * * * * * * * * * * * * * * * * * * * * * * * * * * 512 Return premiums, def. of . . . . . . . . . . . . . . . . . . . . . . . . . . . . . . . . . . . . . . . . . . . . . . . . . . . . . . . . . . . . . . . . 510 Rhombus, def. of . . . . . . . . . . . . . . . . . . . . . . . . . . . . . . . . . . . . . . . . . . . . . . . . . . . . . . . . tº 4 s tº e º 'º - tº * * * * * * 338 Right angled triangle, def. of, . . . . . . . . . . . . . . . . . . . . . * * * * * * * * * * * * * * * * * * * * * * * s e a ºn a a e s = < e < e e s s 337 Risks, def. of . . . . . . . . . . . . . . . . . . . . . . . . . . . . . . . . . . . . . . . . . . . . . . . . . . . . * * * * * e º e º tº º a ºn tº e º 'º e & 4 & 6 tº 6 e 51() Rods, diagonal and ullage, use of . . . . . . . . . . . . . . . . . . . . . . . . . . . . . . . . . . . . . . . . . . . . . . . . . . . . . . . . 413 Roman system of notation, . . . . . . . . . . . . . . . . . . . . . . . . . . . . . . . . . . . . . . . . . . . . . . . . . . . . . . . . . . . . . . . . 30, 33 Bussian exchange, . . . . . . . . . . . . . . . . . . . . . . . . . . . . . . . . . . . . . . . . . . . . . . . . . . . . . . . . . . . . . . . . . . . . . . . . . 771 Sales on joint account, . . . . . . . . . . . . . . . . . . . . . . . . . . . . . . . . . . . . . . . . . . . . . . . . . . . . . . . . . . . . . . . . . . . . 662 Salvadorean exchange, . . . . . . . . . . . . . . . . . . . . . . . . . . . . . . . . . . . . . . . . . . . . . . . . . . . . . . . . . . . . . . . . . . . . 771 Salvage, def. of . . . . . . . . . . . . . . . e & tº e º e º 'º - e. e. e. e. e. e. e º 'º e º e º 'º e º e Savings banks and savings bank accounts, fully illustrated, . . . . . . . . . . . . . . . . . . . . . . . . . . . . . . . . 669–675 Scale beam, def. of . . . . . . . . . . . . . . . . . . . . . . . . . . . . . . . . . . . . . . . . . . . . . . . . . . . . . . . . . . . . . . • * tº e a tº e º a s 420 of numbers, . . . . . . . . . . . . . . . . . . . . . . . . . . . . . . . . . . . . . . . . . . . . . . . . . . tº e e º e e * tº e º 'º - tº e º 'º e º e º e e 27 Scaline triangle, def. of . . . . . . . . . . . . . . . . . . . . . . . . . . . . . . . . . . . . . . . . . . . . . . . . tº gº tº 9 tº a 6 - - - - - © e º 'º is is e 338 Schedule, def. of . . . . . . . . . . . . . . * * * * * * * * * * * * * e º e º º e º e º 'º e º e s e e s e º e s e e º e s e e e s e e s s a e e e - © e º t e º e e 487 Science, def. of . . . . . . . . . . . . . . . . . . . . . . . . . . . . . . . . . . . . . . . . . . . . . . . . . . . tº º & & & e º e tº © e º 'º - e º º ſº tº e º e º e 25 Screw, use of the, . . . . . . . . . . . . . . . tº & © º 'º - e º tº tº º te e º tº e º º º & © & Cº tº ºn tº tº e º & © tº a º º e º 'º e º e º e º 'º e º e º & tº º ſº tº e º e s 430–434 Scrip insurance, . . . . . . . . . . . . . . . . . . . . . . . . . . . . . . . . . . . . tº e º 'º & © tº e º e & e º 'º © tº º e º 'º e º ºs is tº gº tº e º 'º º e º s tº tº e º e 5 12 Selling exchange on OWn account, . . . . . . . . . . . . . . . . . * * * * * * * * * * * * * * * * * * * * * * * * * g e º e - e. * * * * * 719 & 4 through agent, . . . . . . . tº º t e de e º e º e s e - - - - - e º ſº tº e º e - © tº e º a tº e a tº e s e s a 4 - - - - - * * * * * * * 719 & 4 { { “ and broker, . . . . . . . . - - - - - tº e º e º 'º e º 'º - e. e. e. e. tº e s e s e e < * * * * * * * * * * * * 720 Semi-annual interest, . . . . . . . . . . * * * * * * * * * * * * * e = c is e e º e - e º e º e º e º e º e = e s º s e e s e e s e º 'º - - - - a e s ∈ e e a e 607 Set of foreign B. E. . . . . . . . . . . . . . . . . . . . . . . . . . . . . . tº e º e º 'º tº dº º tº e º º º & º 'º e º sº s ºr e º 'º - tº e º e º e s s sº e 713 Settling estates, . . . . . . . . . . . . . . . . . . . . . . . . . . . . . . . . . . . . . . . . . . . . . . . . . . . . . . . . . . * * * * * * * * * * * e º 'º is 874 Settling with insurance companies, . . . . . . . . . . . . . . . . . . . . . . . . . . . . . . . . . . . . . . . . . . . . . . . . . . . . . . . 525 Shares, manufacturing on, . . . . . . . te e º 'º e º 'º e a s e º e s is tº e tº e º e º ºs e º e º 'º e º e e º e - * * * * * * * * * * * * * c e s is tº e. 785–789 Sharing gains and losses, . . . . . . . . © e º 'º e º 'º & e º e º ſº e º & © - - - - e º e º e º e º e - - - - e. e. e. e. e. e. e. e. e. a 4 - * * * * * * * & e - 861 Shingling and slating roofs, . . . . . . . . • e º ºs e º e º e s e º 'º e > * > * * * * * e º e º e º e º e - e. e. e. e. e. e. e. e. e. e. e. e. e. * * * * * e s s e 364 Shoemakers’ measure, . . . . . . . . . . . . . . . • * > * * * * * * * * e e e s • * * * * * * * * * * * * * tº e º 'º - - - - - © tº e & Short cuts in division, . . . . . . . . . . . . . . . • * c e º 'º & ſº gº tº e º 'º - © º 'º e º 'º º tº $ tº dº e º 'º - e º 'º e º is a e º is tº is a 4 - © tº º tº s º e . 138–142 “ in multiplication, . . . . . . . . . . . . . . . . . . . . . . . . . . . • * * * * * * * * * * * * * * * * * * * * * * * * * * * * . . . . S8–121 ‘‘ in & 4 of fractions, . . . . . . . . . . . . . . . . . . . . e tº & © º e º sº º e s ∈ & e s a s a tº 4 - e º s is e e s ∈ s 190–197 division, . . . . . . . . . . . . . . . . • e s - c e ºs tº e º 'º e s tº e e tº e º e º 'º tº e e º e º e º 'º - e tº © e º 'º e - - - - - - a s * * * * 128 Signs and symbols, . . . . . . . . . . . . . . . . . . . . . . . . . . . . . . . . . . . . . & e º e º e s - a tº e º ſº s e s m tº ºt & e º sº - - - tº e e s tº e < 36–39 Silver, gold, currency and uncurrent money, . . . . . . . . . . . . . . . . . . . . . . . . . . . . . . . . . . . . . . . . . . . . 193—500 Simple equation, . . . . . . . . . . . . . . . . . . . . . . . . . . . . . . . . . . . . . . . . . e is e o e º 'º - - - a s a e e s tº a e - a - - e º 'º e º e º sº e 627 fraction, . . . . . . * * * * * * * * * * * * * * * * * * e e s e º e a tº e e s a 4 - e. e e g º e s r. º. º ºs s ºn e º e s e s a e s ∈ a • * • * * * * * * * 165 value of figures, . . . . . . . . . . . . . tº º e º 'º º e º ºs e º º º ºs e - © e º g º e s e º 'º - 6 * * * * * * * * * * * * * * * * * * * * * * * * * * 27 Simultaneous multiplication, . . . . . . . . . . . . . . . . . . . . . . . . . . . . . . . . . . . . . . . . . . . . . . . . - - - - - tº e º 'º º º 88 Single discount from a series, . . . . . . . . . . . . . . . . . . . . . . . . . . . . . . . . . . . . . . . . . . . . . . . . . . . . . . . . . . . . 459 Slating and shingling roofs, . . . . . . . . . . . . . . . . . . . . . . . . . . . e e e s is a e s - e º e e s a s s s s e º a • * * * * * * * * * * * 364 Smuggling, def. of . . . . . . . . . . . . . . . . . . * * * * * * * * * * * * * * * * * * * * * a e º e º e - • * e s e e g = e s - - - e º ºs e º e º º º 501 Solar days and years, . . . . . . . . . . . . . . . . . . . . . . . . . . . . . . tº e º e º e º 'º e º 'º - * = c → - - - - - • tº e º t t e s is 236 Solid, def. of . . . . . . . . . . . . . . . . . . . . . . . . . . . . . . . . . . . . . . . • e º e º e s - © tº e º º e º ºs e º e º 0. e - e º s s tº s & s is 370 Inleasure, . . . . . . . • * * * * * * * * * * * * * * * * * * * * * * * * * * * * * * * * * * * * * * * * * & a e º º e º e 4 & 3 - a tº tº a sº * 2.47 Solution of problems, def. of . . . . . . . . . . . . . . . . * @ e º e º 'º e º e º s a e s is a s s a • * * * * g e º e º e º 'º e º a e s e º e e s a s is 26 philosophic, def. of, . . . . . . . . . . . . . . . • * * * * * * * * * * * * * * * * * * * * * * tº gº gº e º e s tº º e º º s a tº º º & 26 statement, def. of, . . . . . . . . . . . . . . . . . . . . • * * * * * * * g º e s e e a e s ∈ e e s e º e s e s is a m e º sº a • * * * * * * * * 26 Spanish exchange. . . . . . • e e º e º e e s tº 0 - - - - - - e. e º ºs tº e & e º 'º - - - - e e s tº a e g º g c → • - e. e. a g º e e e s a 4 - - - - - - * * * * * * * 764 Special laws and business customs in interest and discount, . . . . . . . . . . . . . . . . . . . . . . . . . . . . . . . . 556 names given to partners, . . . . . • * * * * * * * * * * e e s e e o e & © e º ſº & e e º 'o e • e e s s e e s s = * * * * * * * * * * * * * * * 805 Specific duty, def. of . . . . . . . . . . . . . . . . . . . . . . . . . . . . . . . . . . . . . . . . . . . . © º e º e º 'º s e s ∈ e. e. e. e s = n < e < * * * * 500 Sphere, def. of . . . . . . . . © º & © - e º º * * * * * * * * * * * e e tº tº º tº a tº º & * * * * * * * * * e e s m e º e a tº a s a e e º ºs e º 'º e º 'º a º º º s e tº 370 , ratio of, to cube, . . . . . . . . . . . . . . . . . . © & © tº e º 'º e º 'º e º e º e º 'º e s e º e - e º 'º e º $ tº a “* * * * * * * * * * e e s & 377 Square, def. of . . . . . . . . . . . . . . . . . . . . . . - - - - - e º s e e e • * * * * * * * * is e s is a e º a e º e o e a s e s e s a e < * * * * * * * * * * * * 337 , ratio of, to circle, . . . . . . . . . . . • * * > 0 tº e e s m e º e - e - e. e. e. e. a e s a e e e s e e s e e s e e s is e º 'º - e. e. e. e. e. e s is a tº e 338 square measure. . . . . . . . . . . . . . . - * * * * * * * * * * * * * * * * * * * * * e s - e º a tº e - e - e º e º ºs e º 'º - - - - - e. e. e. e º e º º 243 e º s 885 tº ſº tº gº tº tº dº e º & ſº tº e º e º ſº tº e º tº e º 'º º tº e º 'º e º is e e 512 I’OOt. • * * * * * e e s - a e º 'º º e º 'º e º e o ºs e e º 'º e º e g º e º ºs e s is e e º e e s - e s e e s • * * * * * * * * * * * * * * * * * * * * * * * * * ALPHABETICAL INDEX. XXIII PAGES Squaring numbers, . . . . . . . . . . . . . . . . . . . . . . . . . . . . . . . . . . . . . . . . . . . . . . . . . . { } & © tº $ $ ſº tº & © tº e s & ſº tº g g g g g & 118 Statement line, . . . . . . . . . . . . * * * * * * * * * * * * * * * * * * * * * * * * * * s e º e o e s a s g g g º e º a s s * tº $ tº a tº ſº gº º g g tº e º gº tº e & 161, 184 solution, def. of . . . . . . . . . . . . . . . . . . . . . . . . . . e & a 2 tº e s e º is e º s º tº & # tº tº gº tº dº º tº $ tº º tº # 6 º' tº ę & © e & 26 Steelyard, def. of, . . . . . * * * * * * * * * > tº e º 'º e º 'º e s e º e º ºs e e ſº * * * * * * * * * * * * * * * * * * * * * tº º sº e º e º is ſº e º s is e º 'º e 421 Stocks and bonds, . . . . . . . . . . . • * * * * * * * * * * * * * * * * * * * * * a se e is tº e º 'º º e s tº dº º ºs e s tº 8 º' & © e º is tº º is & & G e º e . . . . . 676–710 definitions of all kinds; dividends; words and phrases of brokers; to buy, sell, and invest in stocks and bonds; to buy, sell, and invest through a broker; to find gain per cent on ; price of stocks; amount to be invested in ; par value of; price to realize certain rate; value at maturity; present worth of; value of bonds under varying conditions; value of stocks after dividends; watering stocks; interest accounts with brokers; margin to carry stock; stock exchange; United States stocks and bonds in Europe; operations in French and English quotations; practical problems in all phases of stocks and bonds. Stock dividend, . . . . . . . . . . . . . . & e º e º a g º e º º is º º ſº tº tº e º 'º º e º º ſº tº º ºs º 'º e º ſº tº e º 'º e º e º 'º e º tº º tº e º e º a tº e © tº e g g is e. 678 exchange, . . . . . . . . . . . . . . . . . . . . . . . . . . . . . . . . . . . . . . . . . . . . . . . . . . . . . . . . . . . . . . . . © e º e e ſº tº e º & 681 Storage, . . . . . . . . . & º º 'º C & ſº & © tº e º 'º º e º º te e º 'º * † tº º ſº tº º º gº tº ſº º º ſº tº º tº gº º º tº º tº g º ſº º . . . . . . 777–785 defs. and kinds of; storage per month; for average period; for average period with deliveries; storage when received and delivered on different dates; storage on grain. Subtraction, . . . . . . tº e º ſº tº º & tº dº e º º © tº e º º ſº e º & © tº $ tº e º ſº ſº tº tº tº dº ſº e s tº tº e º G & º e º ºs e g º sº º e s tº º tº e º e º e g g = * * * * g º & 64–7:5 , borrowing method improper, . . . . . . . . . . tº ºs º º t e º º & © tº ſº. tº g º ºs tº e º 'º $ tº s is tº t e s tº e g & tº £ tº º e º 'º 66 , by addition, . . . . . . . . . . . . . . . . . . . e e º te e º ºs e e º e e º 'º is tº a º ºs tº tº gº g º ºn tº e s tº $ tº dº º tº º sº º e º sº º & e g º & º 66 , by commencing on the left, . . . . . . . . . . . . . . . . . . . . . . . . . . . . . . . . . . . . . . . . . . . . . . . . . . . 69 , by complement of ten, . . . . . . . . . . . . . . . . . . . . . . . . . . . . . . . . . . . . . . tº y º e º a 2 & e s tº gº tº g tº gº gº 69, 70 horizontal, . . . . . . . tº e º sº e º 'º tº e º 'º & s is º º e º 'º a tº e º 'º e º dº º e s tº e e º º tº e º tº tº & E tº dº e º ºs e e g º s & © e º 'º e ge 68 of decimals, . . . . . . . . . . . . * e e º 'º e º e º 'º e º 'º º e º e º e e e e g g º e e º ſº e º º º e s tº gº º tº gº tº g tº e º º sº e º º ºs º e ºs 224 of denominate numbers, . . . . . . . ë º e º º º q & & & © tº a tº e º tº e º $ tº & º ºs e º is º gº tº e º 'º e e º e e s tº e º ºs & e & e 269 of dollars and cents, . . . . . . . . . . . . . . . . . . . . . . . . . . . . tº g g tº 9 & © tº e e g º º tº $ tº gº & & 9 º' g º gº tº a tº t. tº 67 § of fractions, . . . . . . . . . . . . . . . . . . . * & e º ºs e º e º e º 'º a º e º e º 'º e < * * * * * * * * * * * * * * * * * * * * * * * * * * 180–183 of metric numbers, . . . . . . . . . . tº dº º e º ſº º ſº º e º º º e º e & tº dº ſº º tº ſº. © º º tº $ tº º sº a tº 8 tº $ a tº a tº e s & © tº tº ſº & º 937 , proof of, . . . . . * @ e º ſº e º e º e e º 'º e • * e º 'º e º e < * * * * * * * * * * * * * * * * * * * * * * * * * * * * * * * * * * * * * * * * * 64 table, . . . . . . . . . . . . . . . . . . . . . . . . . . . . . . . . . . . . . . . . . . . . . . . . . . . . . . . . . . . . . . . . . . . . . . . . 64 Subtrahend, . . . . . . . e e º sº e º se º 'º e º ºs e e º e s e s e º ºs e e s a e e s a e º e a s e e s = e e s tº e e s s = * * * * * * * * * * * * * * * * * * * * * * * 64 Sucrose multiplier, . . . . . . . . . se g is e º e º ºs e º e º ºs e e s e º e s a e < e < e < e < e s = e s a e s sº e s e º e º 'º º º is a s n e s a s ∈ s & s & e º º 467 Sugar cane, to find value perton, . . . . . . . . . . . . . . . . . . . . . . . . . . . . . . . . . . . . . . . . . . . . . . . . . . . . . . . . . . 466 Supplement of number, . . . . . . . . . . . . . . . . . . . . . . . . . . . . . . . . . . . . . . . . . . . . . . . . . . . . . . . . . . . . . . . . . . 26 of number, multiply by, . . . . . º, e = e º ºs e º e s e º is a s e º e e s s e º e º º e º e s a # & © e º s a sº s º º tº e º e º ºs & 108 Surfaces, def. of, . . . . . . . . e e º e e g g g g º e e º a e s e a º ºs e e a s e s e s e s = e e s e e s e º e e s a s a sº e º sº * * * * * * * * * * * * * * * * * 337 Surface measure, . . . . . . . . . . . . . . . tº e º e s e e s tº e a tº º e s tº e s tº $ tº e e º e º e s e º sº e s e a tº e s e º ºs e º e s e e s s e º e º e º ºs e s e 243 Surveyors’ measure, . . . . . . . . . . . . * * * * * * * * * a s e s e º e s = e tº e s tº º º e º e º e º e º 'º a ſº * * © it tº ſº tº $ tº e º t e tº tº $ tº $ tº e º ſº s 244 Sweden exchange, . . . . . . . . . . e e º e s s a e º e a e s e a e s e e s is e e s e e s e º 'º e º 'º s º e º 'º º º * g º e g º e º º $ $ tº $ tº $ tº e º ſº tº º ſº & 773 Swiss exchange, . . . . . . . . . . . . . . . . • * g e º 'º e º e s is s e º e º e º e º & © e º e g * * * * * * * * * * * * * tº e º º ºs e º e º 'º º s º e s a s is e 764 Symbols and signs, . . . . . . . . . . . . . . . . . . . . . . . . . . . . . . . . . . . . . . . . . . tº e º e º is e º e e is tº e º e º us tº t tº e º s tº $ tº e s s & 36–39 Synopsis for review, . . . . . . . . . . . . . . . . . . . . . . . . . . . . . . . . . . . . . . . . . . .35, 63, 74, 121, 142, 149, 160, 163, 166 System, check or key figure, . . . . . . . . . . . . . . . . . . . . . . . . . . . . . . . . . . . . . . . * e g g s e º e º e º ſº e s tº e & & º e tº $ w e 155 of addition, Vicenary, . . . . . . . . . . . . . . . . . . . . . . . . . . . . . . . . . . tº dº º ºs e e º 'º e a tº e º ºs tº º e º ºs e e s & e º º 56 of multiplication, philosophic. . . . . . . . . . . . . . . . . . . . . . . . . . . . . . . . . . . & e g º ºs e e º e e s a s tº e º e e 77 of notation, . . . . . . . . . . . . . . . . . . . . . . . . . . . . . . . . . . . . . . . . . . . . . . . . . . . . . . . . . . . . . . . . . . . . . 31 of numeration, . . . . . . . . . . . . . . . . . . . . . . . . . . . . . . . . . . . . . . . . . . tº e º e g º º ºs e º E tº e º & e º ſº º 'º & a tº € e 30, 31 of numbers, . . . . . . . . . . . . . . . . . . . . . . . . . . . . . . . . . . . . . . . . . . . © tº $ tº e º 'º & e º e º s = e s e e s is a tº * * * * * 27 Table of addition, . . . . . . . . . . . . . . . . . . . . . . . . . . . . . . . . . . . . . . . . . . . . . . . . . º, e º is e º 'º c e s e º ºs º º e º $ tº e & e e 41, 42 of aliquots, . . . . . . . . . . . . . . . . . . . . . . . . . . . . . . . . . . . . . . . . . e & s e º e e s = e tº tº g º e º ºs e & e º ſº e º e º is s 94, 139, 436 of annuities, . . . . . . . . . . . . . . . . . . . . . . . . . . . . . . . . . g e s s e e s is is e e s is a e s s e º e º e º e s s º ºr a º & © tº 925, 926, 927 of areas of segments of a circle, . . . . . . . . . . . . . . . . . . . . . . . . . . . . . . . . . . . . . . . . . . . . . . . . . . . . 353, 354 of casting unearned premiums, . . . . . e e = s e º e e º s e º e º s is e º s a e s & e º is s = e º 'º e s a e s = s a tº ſº tº ſº a g º 'º gº 522 of compound denominate numbers, . . . . . . . . . . . . . . . . . . . . . . . . . . . . . . . . . . . . . . . . . . . . . . . . . .231-256 of compound 111terest, . . . . . . . . . s e e s s e s o e e º e s e e s e e º e s e s s e s s a s e º e º e s = e º e s s & e s e e º e = * * * 611, 612 of decimals, . . . . . . . . . . . . . . . . . . . . . . . . . * @ º & & & © e & • e s = e < e < e < e < e e s e e s a • * * * * * * * * * * * * * * * * * * 218 of equivalents of metric and American weights and measures, . . . . . . . . . . . . . . . . . . . . . . . Q33 of foreign coins, . . . . . . . . . . . . . . . . . • e o e s e e s e e º e s e s e e s s a e º 'º & e s s a e s is e º e s e e s s e e s a e º 'º * * * > * 743 of interest divisors, . . . . . . . . . . . . . . . . . . . . . . . . . . . * * * * * * * * * * * * * * * * * * * * * * * * * * * * * * * tº tº e º 'º º 560, 561 of Interest laws of states and territories, . . . . . . . . . . . . . . . . . . . . . . . . . . . . . . . . . . . . . . . . . . . . 548 of life insurance, . . . . . . . . tº e e is e º ºs e º s • e o e s e e o e s a • * * * * * * * * * * * * * * * * * * * * * * * * * * * * 537, 538, 539, 541 of marking goods. . . . . . . . . . . . s e e s e e s e s is a e s e e s = * * * * * * * • * g e º e º e e s a e s e s e º 'º e º e º e s e e e s tº e s is 454 of metric systems, . . . . . . . . . . . . . . . . . . . . . . . . . . . . . . . . . . . . . . . . . . . . . . . . . . . . . . . .931, 932, 933 XXIV INDEX. ALPHABETICAL Table of multiplication, . . . . . . of multiples in mensuration, of permutations, of principal to double itself, of regular polygons, of roman characters, . . . . . . . . . . . . . . . . . . . . . . of subtraction, of time, between two dates, of trade discount. . . . . . . . . . . . . . . . . . . . . . . . . . Tare, def. of, Tariff, def. of, Taxes and licenses, Ternary scale, Theorem, def. of, Thermometers, comparison of, Thirty-six per cent method of interest, . . . . . . . . . . Timber, logs, etc., measured, Time exchange, & 4 , discount on, measure table, Storage, table, between two dates, . . . . . . . . . . . . . . . . . . Tollage, Tonnage duty, def. of, . . . . . . . . . . . . . . . . . . . . . . . . . . . of vessels, to find, Trade discounts and list prices, Trapezium, def. of, mensuration of, Trapezoid, def. of, Imensuration of, Travelers’ checks, Triangles, def. of, Troy weight, True discount, remainders, . . . . . . . . . . . . . . . . . . . . . . . . . . . . . . PAGES. Ullage rods, use of, . . . . . . . . . . . . . . . . . . . . . 413 Uncurrent money, currency, gold and silver, . . . . . . . . . . . . . . . . . . . . . . . . . . . . . . . . 493–500 Unearned premiums, table for, . . . . . . . . . . 522 Unification of the money of the world, .. 776 Unit, def. of . . . . . . . . . . . . . . . . . . . . . . . . . . . 25, 164 , fractional, . . . . . . . . . . . . . . . . . . . . . . . . 25, 164 of a number, . . . . . . . . . . . . . . . . . . . . . . . 25 United States bonds outstanding, . . . . . . . 680 & 4 coin table, . . . . . . . . . . . . . . . 232 & 4 money table. . . . . . . . . & º ºs 231. 4 & government lands, . . . . . . . . 245 & 4 system of partial pay- ments, . . . . . . . . . . . . . . . . 616 Universal formula for computing interest, 561 Unlike fractions, . . . . . . . . . . . . . . . . . . . . . . . 175 numbers. . . . . . . . . . . . . . . . . . . . . . . . 25 Use of compound interest table, . . . . . . . . 613 Usury laws, evil effects of . . . . . . . . . . . . . . 545 Value, def. of . . . . . . . . . . . . . . . . . . . . . . . . . . 229 of English money in U. S. . . . . . . . 235 of figures, . . . . . . . . . . . . . . . . . . . . . . . 27 of gold and silver, . . . . . . . . . . . . . .233, 234 tº º ſº e º 'º º e º e º s a s e e º e = & e is a s e s is e e º 'º e ºs * * * * * * * * * e º ºs e a e º s a t e e º e º s & * * * * * * * s & e º s e º º & 8 & # * * * * * * * * * * * * g g g g º ºs tº º e º 'º e º º e º e º e s tº º e º is s e e s º e º s e * e s 3 & s º ºs e e º s tº $ is * * * * * * * * * * * * * * * * * & & e e * * * * * > e º 'º tº º is is a e e º is º º e º e is e < * * * * g e º sº e e * * * * * * * * * * * * * * * * is a s tº e º e s e º a s e s e s e e e & © e º & gº e º ſº ſº tº e º & 9 º' tº 9 & © & © s e º e º s s e is sº tº gº & e g g g = e º 'º gº e º ſº a s & e º & tº º º & © & # tº ſº e º e º & e º 'º e º ºs e g º 'º e º 'º º e º 'º e s s e º & e * * * * * * * * * * e º 'º s e º 'º e º 'º $ tº $ sº tº e s e º e º e º a s is a º e e e º 'º e º E & E; e º º e º s º e º e o 'º & “ e tº ſº e º e g º e g º ºs e º 'º a dº e s s tº tº 4 tº & e º 'º e º & e º e º 'º e s e g * * * * * * * e º ºs e e º ºs e e º º e e º 'º e º e º e = e º s e g º e s e as a sº tº w tº e º e º e º e º 'º e s e s is * * * * * * * * * * * * * e º s º e = * * * * * * * * * * * * * * * * tº e º º º ºs e º 'º e º e s tº $ tº $ tº º º te s tº gº e º e g º dº e º e º e º e º ºs e g º ºs e º ºs e e º ſº e s e º e º e s 6 s e s s e º sº tº $ & & sº tº º is © tº * * * * tº dº ſº tº e º ſº e º 'º º ºs e º ſe e º e s ſº e º e º e e e g º º tº e º ºs e º e º is is e º s is is a us tº e º ſº º 'º e g is e s is e º e º is e e s s = e º e º 'º tº e º 'º e º tº º e º O & e º & e º 'º e º e º is tº s = e s tº gº º ê º $ ſº tº ſº gº tº º $ tº e º e º is gº tº t e º & * @ Tº e º 'º ſº º ºs º gº & © tº e º e s tº e º 'º g is s ſº º is tº e e tº e º ºs º ºs e º e º e º e º e º e s e e s sº e s e º a s a s e º e s e e s e s a e s e s e. g. a e s a s a wº e º º sº e < * * * * * * tº ºp º ºs º º º Gº tº º ºs º 'º e º e º e de e º 'º e s s as e s tº e º e º e s tº • e º e º e e º ºs e º e º is is e º e s e e e s m e º s e < * * * * * * * * * * * * * * * * * * * g e e s = e º e s e s tº e º sº * * * * * * * * * * * * * • * g e º & © e º e s e º a tº gº º is a e º e & © tº e º ºs º is tº º is 6 & © & © tº tº tº tº * * * e e º e º e e º e s tº e s is s = e e º e e s ∈ n e º e º e s e º is e s e s a e is e s is e s e e e º e º e º e s e a tº e º ºs º is e g º e s p & & e º tº * * * * * * * tº e s tº s e e º e º e s tº e s tº t is g a £ tº º e º ºs e º & & © tº e º g tº e º ºs e º 'º e º s & e º e º ſº e s tº a tº º s e º e s m & e º sº e º e º º e º e º e s tº e º e º 'º' tº e º e s tº s º is e º º e º gº tº $ is tº t tº $ ſº º e s & 6 & 8 tº e º e º ſº º sº tº e º & • * * * * * g e e º 'º g g g º e e g º e º sº gº w tº e s a s e º ſº e º ºr e º e º 'º e º • * * * * * * * * * * * * * * * e < e º e º e º ºs s = e º e s s e º e s º is e e º & * tº & e º e s º is t w e s tº tº s = e s tº e º ſº tº & tº º & º º $ tº tº ſº º # * * * s tº tº * * * is a tº e g º gº e º e º e s e < * g º e º ºs º a ſº e s is tº e º ºs e & © & © tº & B * * * * * g g g g º º is º ºs e º gº tº gº & tº gº tº sº º ſº is g g & G - tº gº tº # 8 º 'º & ſº tº * * * g º e s is º is a º º a 2 m º º e º e º s & e & 6 º e º ſº is º º ºs e º tº º e & & a g º g g g g g g g g g g & & 4 & e º º & tº tº $ tº º º e = 8 s g º e º e º i e º º gº tº & s º º is tº $ 4 & 8 tº º ſº & a tº 8 & 8 * e e s tº e s e s a & E 9 & e < * * * * * * * * * * * * * * * * * * * * * * * * * s = e s e e s e e s sº º ºs e e s w e º e s & e e s m e º 'º is 4 e = * * * * * * * * * * * * * * * * * * * * e s e s a e º e º ºs e s e ºs e º & tº e s e º 'º e s e º ºs tº s = e º is a e g g g e s sº a e º e < e < e º ºs & & & & & a tº * * * * * * * * * * * e º e º 'º & e s sº º e g º e g º e º ſe a e g º e s s e e s s tº tº * * * * * * * * * * * s s is s is º gº a g º is a g º e º e º e º a s s a w e º 'º e º sº e º ºs e º 'º & & * e º e º a tº e º e º e s ∈ is & e º is tº a tº º sº e g º ºs e º e e s & = < * * * s e s e º e s is s a tº e º is tº e º ºs º & © e º a º s & e e s e º ºs e º g e º e º a s e e s is e º is e e º e s tº e º e º ſº we tº e & © º ºs e e e s a s is a e e s a s a sº e s s e º a s a s is e º ºs e e is e º 'º º º e º 'º e º 'º * * * * * * * * tº G & © e s º e # e e s s a e s e e s e º a e s e s e º e e s e s is e s e s e e s ∈ e e s e º e s a “ e = e g º e s s a e g º sº e s a e s = ºr = g is a e e s ∈ s & & © tº e º 'º e º is a e • e. e. g. a s a s = e s = e e s is s a º e º sº a tº e º 'º e s is s & e º ºs e º 'º e º 'º ºf tº gº tº º ſº e e º e = e s = e s e e s e s a s e e s is e e º 'o e º e º 'º - * * * * * * * * * * * e e a tº e s e e º e s e º ºs s e º e s tº gº º e º º & © º º º tº e º e < * * * * * * * Value of insurance, & © tº of notes bearing interest and mat- uring in consecutive months, of stock after declaring dividends, . of stock having specified time to run, of United States stocks and bonds in London, of watered stock, Valued policy, def. of, Various methods of interest, Vermont system of partial payments, . . . Vicenary scale changed to decimal, system of addition eludicated, Vinculum, use of, . . . . . . . . e e º 'º - Wr Warehouses, grain and bonded, Watered stock, Wedge, use of, cubical contents of . . . . . . . . . . . . . Weight, def. of, . . . . . . . . . . . . . . . . . . . . . . . . & of coin, Wheel and axle, use of, Wine measure, . . . . . . . . . Words and phrases used by brokers and bankers, .. * g e º e º e s e º e º º e & e g º a dº e º 'º is e e s ∈ tº e º is tº gº e g º ºs e º 'º º & tº $ tº tº º e e º e e º sº e º a e º 'º º 'º e º 'º tº e º g º e º e º ºs e tº e º e º e g o ºs e º sº e º e s is tº e º º ºs e º e • e a e s , s s e º e s e s is e s is © & 8 e º 'º' & sº e º ºs e º 'º e º e º ºs e º º sº 502 416 457–466 338. 347 338 347 56–58 134 P- ºr a y { { { 678 429. 384 229 233 422 247 • * e o e o e o e < * * * * * * * * * * * * * * * * * * 682, Sl's Pilsit Pºdiul Millimits. *1|efinitions. º =N 1. A Definition is the meaning or import of a word or words expressed by Other words. 2. Science is classified knowledge. 3. Art Is the practical application of the principles of science, according to prescribed methods. 4. Quantity is anything that can be increased or diminished. 5. A Unit is a single thing of whatsoever denomination or nature, as one Orange, one pound, etc. 6. A Number is a unit or a collection of like units. 7. Like Numbers are those which express units of the same kind. Thus: Five apples and six apples, seven pounds and nine pounds, are like numbers. 8. Unlike Numbers are those which express units of different kinds. Thus: Four pounds, seven hours, ten peaches, are unlike numbers. 9. Odd and Even Numbers. An ODD NUMBER is one that cannot be divided by 2 without a remainder, as 1, 3, 5, 7, 35, 479, etc. An EVEN NUMBER is one that can be divided by 2 without a remainder, as 2, 6, 18, 54, 536, etc. 10. The Unit of a Number is one of the collection of units forming that number. Thus the unit of twelve hats is one hat; of five is one ; of four pounds, one pound. 11. The Unit is the universal basis of numbers and the foundation of arithmetic. From unity arise two distinct classes of number: 1. Integers, 2. Frac- tions. The first class, Integers, has its origin in the multiplication of the unit: and the second class, Fractions, results from the division of the unit. The first is synthetical, the second is analytical. 12. A Fractional Unit is one of the equal parts into which any integral unit is divided. If the integral unit is divided into two equal parts, each is called a half; if into three, each is called a third; if into four, each is called a fourth ; and so on according to the number of parts into which the integral unit is divided. See Fractions, page 164. 13. Numbers are expressed by words, figures, or letters. (25) 26 SOULE's PHILOSOPHIC PRACTICAL MATHEMATICs. 4× 14. An Abstract Number is one in which the kind of unit or quantity is not designated. Thus: Three, four, five, etc. 15. A Denominate or Concrete Number is one in which the kind of unit is designated. Thus: Two pounds, five yards, nine dollars, etc 16. A Compound Number is a denominate number expressed in two or more denominations. Thus: Five years, four months, and eight days; two miles, five furlongs, and ten rods; two yards, two feet, and five inches. 17. The Arithmetical Units of Orders are 1, 10, 100, 1000, 10000, etc. 18. An Arithmetical Complement of a Number is the difference between the number and a unit of the next higher order. Thus: 3 is the arithmetical com- plement of 7; 26 is the arithmetical complement of 74; 19 is the arithmetical com- plement of 981. 19. An Arithmetical Supplement of a Number is the difference between the number and a unit of the same order. Thus: 7 is the arithmetical supplement of 17; 12 is the arithmetical supplement of 112. 20. A Problem is a question proposed or given for solution, or it is a ques- tion requiring some unknown result from stated conditions. 21. The Premises or Conditions of a problem are the known facts and truths, therein given as a basis or predicate, for the solution. 22. A Solution of a problem is the process of determining the required result or demonstration. 23. A Philosophic Solution is, in this work, a full numerical statement showing, step by step, how the result of a problem is obtained, with a logical reason for each conclusion reached in the solution. This process of solution and reasoning from the premises and conditions of all problems constitutes the Philosophic System of Arithmetic. * 24. An Axiom is a self-evident truth. Aacioms are the laws which govern the logical mind in processes of reasoning. - 25. A Theorem is a statement, the truth or falsity of which is to be deter- mined by reasoning. 26. Demonstration is a process of reasoning by which the truth of a theorem is proved. 27. Comparison is considering the relations between numbers; the rela- tions between the premises and conditions of problems and the required result. NOTE. – Comparison is the true method of investigation—it is the royal road to the fields of unknown truth and knowledge. The comparison of two numbers as to their unit value, gives ratio; the comparison of two ºratios gives proportion ; and the comparison of several numbers which differ by a common ratio con- stitutes progression. 28. A Solution Statement, or an Operation, is a statement of the figures employed in solving a problem. 29. A Formula is the expression, by symbols, of general principles appli- cable to the operations of particular problems. 30. Philosophy is the knowledge of phenomena as explained by, and resolved into, causes and reasons, powers and laws. 3r DEFINITIONS. 27 31. Arithmetic is the science of numbers; or to define it more extendedly, it is that branch of mathematics which treats of the principles, properties and rela- tions of numbers when expressed by the aid of figures, either singly or combined. These principles and relations of numbers, combined with the facts relating to problems, are applied, by the reasoning powers of man, to the solution of all numer- ical problems of business affairs and of practical life. 32. Figures.—Figures in arithmetic, are characters used to represent num- bers. The ten Arabic figures which we use, are Naught or Cipher. One Two Three Four Five Six Seven Eight Nine O 1 2 3 4 5 6 7 8 9 By properly combining these ten figures, all possible numbers may be repre- sented. The 1, 2, 3, 4, 5, 6, 7, 8, and 9 are sometimes called digits. They are also called the significant figures, because each signifies a number when alone. The naught (0) is so called, because by itself it does not signify or express any number. It expresses number only when used in connection with other figures. 33. Walue of Figures.—The value of a figure is its power to express quantity. FIGURES have two values: 1. A simple value. 2. A local value. 34. The Simple Walue of a figure is the quantity expressed when standing alone, or in the unit's place. Thus, when we write 3, independently of other figures, or in the unit column of numbers, it has only a simple value expressing 3 units. 35. The Local Walue of a figure is the quantity expressed when standing to the left of other figures. Thus, 3 in 34 expresses a quantity of thirty units; and 3 in 345, expresses three hundred units. The local value depends upon the scale or system of numbers employed, and the location in the Scale. 36. The Radix is the number of units of one order which it takes to make one of the next higher order; thus the radix of the common system is 10; of the quinary system (not in use), it is 5. 37. A Scale or System of Numbers, in Arithmetic, is a succession of units, increasing and decreasing according to some established custom in the opera- tions of numbers. Thus there is the binary, the trinary, etc., etc., and the decimal, the duodecimal, etc., etc. 38. In these four named different scales, the value of four ones (1111) would be as follows: In the binary scale or system, the first 1 on the right is 1; the second 1 is 2; the third 1 is 4; and the fourth 1 is 8; making 15 altogether. 39. In the trinary system or scale, the first 1 on the right is 1; the second 1 is 3; the third 1 is 9; and the fourth 1 is 27; making 40 altogether. 40. In the decimal scale, the first 1 is 1; the second 1 is 10; the third 1 is 100; and the fourth 1 is 1000; making in all 1111. 41. In the duodecimal scale, the first 1 is 1; the second 1 is 12; the third 1 is 144; and the fourth 1 is 1728; making in all 1885. From the foregoing, we see that the system of scale derives its name from 28 SOULE's PHILOSOPHIC PRACTICAL MATHEMATICS. M. the Radia or Ratio of value given to each succeeding figure from the right toward the left. 42. The Decimal Scale or System is one in which the Radix or law of increase and decrease is always ten. This system is in general use, and derives its name from the Tatin word decem, which means ten. In any scale, the number of figures or characters required, including the 0, is the same as the radix. TO CHANGE ONE SCALE TO ANOTHER. 43. The following elucidations will show the manner of reducing one scale to the notation of another: 1. Reduce 2436 of the Decimal Scale to the notation of the Quinary Scale. OPERATION. 2436 – 5 = 487 fives and 1 unit over. 487 fives – 5 = 97 fives of fives or 25's and 2 fives over. 97.25's -i- 5 = 19 fives of fives of fives or 125's and 225's over. 19 125's -- 5 = 3 625's and 4 125's over. Hence 2436 of the Decimal Scale = 34221 of the Quinary Scale. There is no conventional method of reading numbers written in other scales than the decimal. This number 34221, Quinary Scale, would be read, 3.625's, 4 125's, 225's, 25's and 1 unit. 2. Reduce 34221 of the Quinary Scale to the Decimal Scale. FIRST OPERATION. SECOND OPERATION - BY SIMPLE REDUCTION. 1. Hait of the Quinary Scale = 1 unit of the Decimal Scale. 625's 125's 25's 5’s units. C 5 S & 4 & & & 4 & 4 - 10 units & 4 & 4 & & 3 4 2 2 1 2 25’s & & & 4 { { & 4 - 50 4 & & 4 & 4 & & 4 125's & & • { & 4 & 4 - 500 & 4 & 4 & & & & 5 : 5 & 4 & & ( & 4. - & 4 & 4 4 & 4- 3 625's * =1875 “ 19 125's Hence 34221 of the Quin- 5 ary Scale - - - =2436 units of the Decimal Scale. ºm- - 97 25's 5 487 5's 5 2436 units. 3. Reduce 2031 of the Quaternary Scale to the notation of the Decimal Scale. FIRST OPERATION. SECOND OPERATION BY SIMPLE REDUCTION. 1 unit of the 4 scale = 1 unit of the 10 scale. 64's 16’s 4's units. 3 fours ** * * 4 “ — 12 units “ 10 “ 2 0 3 1 0 sixteens ** * * 4 ** = 0 units “ 10 “ 4 2 sixty-fours “ “ 4 “ =128 units “ 10 “ - Hence 2031 “ “ 4 ** =141 units ** 10 “ 8 16's. 4 35 4’s 4. 141 units. .# DEFINITIONS. 29 4. Reduce 141 of the Decimal Scale to the Quaternary Scale. OPERATION. 141 – 4 = 35 4's and 1 unit over. 35 4’s — 4 = 8 16's and 3 4's over. 8 16’s — 4 = 2 64’s and 0 16's over. Hence 141 of the Decimal Scale = 2031 of the Quaternary Scale. This 2031 of the Quaternary Scale is read, 2 64's, 0 16's, 3 4's and 1 unit. 5. Reduce 4893 of the Decimal Scale to the notation of the Vicenary or Vigesimal Scale and reverse the work. Before this reduction can be performed, we must adopt 20 characters to repre- sent 20 figures in the Vicenary Scale. To do this in the plainest manner, let us use the 10 figures of the Decimal Scale and the first 10 letters of the English alphabet, for the required additional numbers. 10 11 12 13 14 15 16 17 18 19 Thus, 1 2 3 4 5 6 78 9 a b c d e f g h i j 0 Here we have 20 characters, each, except the naught, representing a single number or integer in the Vicenary Scale. The numerical power of the letters is shown by the small figures placed directly above them. Having the required figures, we make the reduction by the following OPERATION. 4893 – 20 = 244 20's and 13 (d) units over. 244 20's — 20 = 12 (c) 400's and 4 20's over. Hence 4893, Decimal Scale, – 12 400's 4 20's and 13 units. Or, expressed in the figures adopted as above, it equals c 4 d. REVERSE OPERATION. (12) (13) c 4 (l 20 244 20's or, thus: d 2– 13 units. 20 4 20’s – 80 “ c 400's == 4800 “ 4893 units. *====º 4893 “ 44. Order of Figures.—The successive places occupied by figures are called orders. Thus in the Decimal System, a figure in the first place is called a figure of the first order, or of the order of units ; a figure in the second place is a figure of the second order, or of the order of tens ; in the third place, of the third order, or of the order of hundreds, and so on; each figure next to the left belonging to a distinct order, the unit of which is tenfold the size or value of a unit of the order of the figure on its right. 3O SOULE's PHILOSOPHIC PRACTICAL MATHEMATICS * 45. From the above, we see 1st: That ten units of any order in a number, in the Decimal System, make one unit of the next higher order. 2d. That moving a figure one place to the left, increases its representative value tenfold. 3d. That moving a figure one place to the right, decreases its representative value tenfold. 46. Notation is a method of writing numbers. There are two Systems, the Arabic and the Roman. 47. By the Arabic notation, numbers are expressed or Written by figures. This system is in general use and is so called because it was introduced into Europe by the Arabians, in the 10th century. 48. By the Roman notation, numbers are expressed or written by letters. This system is now used chiefly to number chapters and divisions of books. It is so called because it was used by the ancient Romans. 49. Numeration is naming the places which figures Occupy. 50. Reading Numbers is expressing their value orally. 51. There are two systems of numeration, or reading numbers, the French and the English. The French system is the one in general use in the United States and on the Continent of Europe. The English system is that generally used in England and in English Provinces. one septillions, three hundred thirty-eight sextillions, one hundred seventy- nine quintillions, six hundred four quadrillions, nine hundred thirty-two trillions, four hundred eighty-seven billions, two hundred sixty-three millions, one hundred ninety-six thousand, five hundred sixty-eight. JD --> h = # # 3 • *ſ ad CO 3 : ‘f © : cº CD 2- - O .S. g g r: § 3 ; e • F-4 º C C := E = 2 o, tº at: P = 3 & 5 SP E F 2 + 2 + 3 + 2* .3 C C 2 tº # E = E. : z = z = # 5 # E 5 § 3 E . E . = . 5 tº 3 E = E 5, & # 5 § 3 ; 3 =# #5 E # = z = z = É # & 5 * pºss * ºt p- • *=d 3 = + c = 0: # 02::= Gº & E H 5 ºn 5 = 5 F : # g’. ſº to ~ - E + E * : * = 2 = 3; 3 = F = < * E = 3 3 # = < * > . . ; ; 2 º 3 × 2 = 3 S ; # 3 F 2: 3: ; E 2 # = 3 e L' C/2 . ſº "25 - : § – 3 C : 302 = 3 dº #3 & 3.33.33E- 23 ° .32 , 3 F : 3 § 3 ; º Q # = 5 ; * = # = 3; a + š + F : *- : * * * * * * : *- : ; # # * - ºn CŞ # 53 : 3 E = 3 - 3 3 E : 3 °F. : 3 5 : 3 + 3 & F : 5 ; ; s o E = c Gº Gº Tº J. F. "E. J. F. Tº A F ºf . º 'º A rººt /, … 'E 2, o Tº c r- : "E 2, 2 3 + 35 C/2 = 2 E = 2 + = 2 +5 = 2 = = 2'3 = 2 = E Z = E Z = E Z E E 2:5 .c. : : E - 3 ... E + 3 E → * E * : E = E E * : E F = E = E E F = E = 3 E = E 5, & 3. 5 §2 3 * = 3; 3 = 5 & E 35 E. E. 35 = P 35 ; E 3: E → E + 35 = P 35 F # ſº 5 § E3 = 5.5H5% = 5% = 5  ºr 55FF as #355,5555 ; ; ; * .E & 8 #: @## 2 8 4, 5 6 1, 3 3 8, 179,604,932, 48 7, 2 G 3, 19 6, 5 6 8. § = &O CO. \__N/~/ \-e-Na-> \->N/~/ \->N/~ \->~~~~~\_ºvº-Z \->-SA-Z\->TV-Z \->~~ § 3 ; := -s. 3 Period Period of Period of Period of Period of Period Period Period Period of Period 5 £ : a $2 : O ep- Sex- uin- uad- of of Of Thou- of g = 3 2. 5 : Octillions tillions. tillions. tillions. rillions. Trillions. Billions. Millions. sands. Units. 2 aſ .: ſº sº. § 3 ; : H 5 The number is ‘read two hundred eighty-four octillions, five hundred sixty- 3 3 & c5 ºr = QD a $3 'S &Q Q ##. 3 ## ; : 5 3. C. § * T â, # 3 & H 3 ºf 3rd E. # 3 § i * DEFINITIONS. 3 I Q Ú) 9 * g : # E É ##### + 3 .S. e wº :- -> *** = . a.º. *- JO g: r CD GD # a 5 S2 cº F = C 3 C . ...+,+,+: tº º £ 3 := cº • * +: ##### F. cº - . & E := aff c º T= 2, 3 #5 cº QD (10 × 2– H - • F- 2/2 m{ ºš'g'53 : â, a 5 & 5 H.3 à = = 3 ### 3. # £5 # 2 o *:::= ** = * = ##### ă ă ă ă ă ă ă ă ă c E Ž e ####g E- L; - s CAE e CAE CO UD 9 3 ####### #: ; ; # 2, #: 2, # ##### * • * e -á r- Q ; : , ; ; ; EF 3 ; ; 5 § 3 ; # 3 ; ; ; ; ; . # 4 g #5 := 3 (2 + Žiž Tº E 5 3.2.É3 2 to .3 & É,2: .3 Éſ. * #### # * * = E * : E 3 - E = 2; 5 § H = 5 Tº F = . 5"; . Tº P iſ o : É's # 3 #5, # 5 #### 5 #3; # 5 #3; # = É ă ăgă CD . º e p- • *t Úº s sº 's 3 × 3 × 33 3:35 s : 335 s : ; # = 3, ##### = ## = #4255.25:42.É .2.É.33: 2.É3.25 Zä33 ### § 3. * : *- : * – F : *- : * * * * *- : 5 - 2 : * : * ~ Z * * = 3 Q = * : 3 5: ; Pºo : 5 ; ; 3 F : 3 ; ; 3 = % cº º c - 3 × = : 3 ; ;- 3 + c : ; (2 32 15 × 2 = 2.5 + ... à 5 × 3.5 ° à 5 ° 55 ° à 5 & 3 + ... à 5 , º, .33% > P - ric o E = a TE = to E'E 2: E to # = 0.3 E 2 : E a 3 E. z ř; E ºf 3 ####3 2 º' 6' 35 B # 3 F # & F # 5 E É E E # 35 = # E = É 3 E É H = É 3 E É E 5: * g : - Z # = AE = E.: E.B.: # = 3.5 E3 = E 5: 5.35 E.; F = <= 5.3 F ##### # 3 #; F ============================F #### ; : # 2 s.45 ° tº 3 s.1799 o 49 32.4 S T 2 & 3, 1965 6 sº: Ǻ 5 §§ \ -N- /N A *—y- a -N- ~~ —’á5 3 ºf 3 * C § Perºof Quad- Period of Trillions. Period of Billions. Period of Millions. Period of Units. ##### I’ll]OIl S. +º c + 2 f 33 . as a E- .F. 3 The number is read two hundred eighty-four thousand five hundred sixty- a #35 ââ 2 : one quadrillions, three hundred thirty-eight thousand one hundred seventy- #3; #: 3 : -> - e !", * e º * º 3 2 5.35 % 5 §2 So nine trillions, six hundred four thousand nine hundred thirty-two billions, ... à .3? ce B.: e º * * * * * #3: ... à S㺠E Z four hundred eighty-seven thousand two hundred sixty-three millions, one º żºłż ż Öſ) dº e - t º - q := 8 & ep rº hundred ninety-six thousand five hundred sixty-eight. ###3's THE ROMAN SYSTEM OF NOTATION. 54. In the Roman system of notation the letter I represents one ; V, five : X, ten ; L, fifty; C, one hundred ; D, five hundred, and M, one thousand. The Intermediate and succeeding numbers are expressed according to the following principles: 1st. Every time a letter is repeated, its value is repeated; thus II represents two ; XX represents twenty; CCCC represents four hundred. 2d. When a letter is placed after one of greater value, the sum of their values is the number expressed. Thus, VI expresses sia: ; XVII expresses seven- teen. 3d. When a letter is placed before one of greater value the difference of their values is the number expressed., Thus, IV expresses four ; XL expresses forty. 4th. When a letter is placed between two letters of a greater value, it is combined with the one following it. Thus, XIX expresses nineteen ; CXLVI expresses one hundred forty-sia. 5th. A dash or bar placed over a letter multiplies its value by 1000. Thus, V expresses 5000; X expresses 10,000; L expresses 50,000; C expresses 100,000; D expresses 500,000; M expresses 1,000,000. 6th. In like manner, a double or a treble dash placed over a letter multiplies its value by 1000 two times for the double dash, and three times for the treble dash. Thus, Cexpresses 100,000,000; Mexpresses 1,000,000,000; Cexpresses 100,000,000,000; M expresses 1,000,000,000,000. 32 SOULE's PHILOSOPHIC PRACTICAL MATHEMATICS. * TABLE OF ROMAN CHARACTERS. I OD18. XXV twenty-five. II tWO. XXVI twenty-six. III three. XXVII twenty-seven. IV four. XXVIII twenty-eight. V five. XXIX twenty-nine. VI six. XXX thirty. VII SeWOIA, XL forty. VIII eight. L fifty. IX nine. LX sixty. X ten. LXX seventy, XI eleven. LXXX eighty. XII twelve. XC ninety. XIII thirteen: C one hundred. XIV fourteen. CC two hundred. XV fifteen. CCC three hundred. XVI sixteen. CCCC four hundred. XVII seventeen. D five hundred. XVIII eighteen. DC Six hundred. XIX nineteen. DCC Seven hundred. XX twenty, DCCC eight hundred. XXI twenty-one. DCCCC s nine hundred. XXII twenty-two. M one thousand. XXIII twenty-three. MM two thousand. XXIV twenty-four. MDCCCXCIV=1894. MDCLXVII = 1000667. TMCCCCXXVMDXLII = 1426542. MMMMDCCXXMCLMCXIV = 1003721151114. NOTE.4–It may be proper to remark that the methods used by the ancient Romans to express large numbers were varied and somewhat different from the method now used, and as here explained. Because of the complex system of the Roman numerals and the great difficulty in using them, in the study of arithmetic and in business calculations, the Romans themselves used a mechanical contri- vance called an Abacus. This Abacus consisted of a table or board with compartments or grooves representing a different value to be given to the counters or pebbles placed therein. In later years, the Abacus was made with wires on which were placed movable counters, each wire representing a unit of a different value. And in this form, the Abacus is still used by some Europeans. The Roman notation is chiefly used in numbering chapters and sections in books, marking dials of clocks and watches, and in writing physicians' prescriptions. -º-º-º-º-º: 2 & Q) C) C) Q) 3 ºotation and Numeration, — OR — WRITING AND READING NUMBERS. 55. To Write, or Notate Numbers, begin at the left and write the figures of each period in their proper places, filling the vacant orders, if any, with naughts. 56. To Read or Numerate Numbers, begin at the right and point off the Inumber into periods of three figures each. Then commence at the left and read in Succession each period with its name. 57. To Werify the Notation, or Writing, numerate the number and see if it agrees with the number given. EXERCISES IN NOTATION AND NUMERATION. 58. Write seven ; nine; ten; fifteen; thirty-two; eighty-nine; ninety-nine; One hundred nine. Write six hundred twenty-two; twenty; one hundred twenty-two; thirty- SOVCI). Write four; forty; forty-four; four hundred; four hundred four; four hun- dred forty-four. Write two; ten; two hundred eight; two hundred twelve; three hundred; four hundred four; five hundred fifty-four; six hundred; seven hundred; eight hundred; nine hundred; ten hundred ten. Write one thousand one; two thousand two hundred one; three thousand thirty-one; forty-four hundred forty; five thousand five hundred five; sixty hun- dred sixty; eight thousand nine hundred eighty-eight; twelve thousand two; twenty-three thousand forty-five; thirty thousand fifty-four. 59. Read the following numbers: 307 13:28 34546 142462 815621.11 220 8123 19703 770077 40404040 170 2813 10010 555055 27624555 202 7890 88888 809010 3333333333333 200 3218 40.101 (306060 11111110 322 9087 90909 100107 1201050607082 60. Write the following numbers: 1. Write 1 unit; 1 ten; 1 hundred; 1 unit and 1 ten; 1 hundred, 1 ten and 1 unit. 2. Write 1 unit, 2 tens, and three hundreds; 4 units, 5 tens, and 6 hundreds; 7 hundreds, 8 tens, and 9 units. 3. Write 0 units, 1 ten, and 0 hundreds; 4 units, 0 tens, 0 hundreds and 4 thousands. º (33) 34 SOULE's PHILOSOPHIC PRACTICAL MATHEMATICS. +& 4. Three hundred forty-one thousand, twenty-two. 5. Sixty-five million, one hundred thirty-two thousand, three hundred eighty- SeVéll. 6. Twelve billion, sixteen million, forty-three thousand, one hundred eleven. 7. Nine hundred thousand, three hundred fifty. 8. Six million, one hundred sixty-nine thousand, four hundred thirty-seven. 9. Twenty-two billion, one hundred three million, five hundred seventy-six thousand, one hundred two. t 10. One hundred two trillion, one hundred twenty-five million, four hundred three. 11. Write 208 million, 18 thousand, 1 unit. 12. Write 10 billion, 8 million, 103 thousand, eleven. 13. Write 200 sextillion, 1 quintillion, 100 quadrillion, 10 trillion, 111 billion, 1 million, 10 thousand, 10 units. 14. Write 97 million, 14; 5 thousand, 5. 15. Write eleven thousand, 11 hundred, 11; 16 thousand, 16 hundred, 16. 16. Five billion, fourteen million, nine. 17. Two hundred one million, twenty thousand. 18. Seventy-seven trillion, seventy-six. 19. Eight thousand, nine hundred, ninety. 20. Four hundred twenty million, one. 21. Thirty thousand million, thirty. 22. Three hundred eleven Octillion. 23. Four hundred two vigintillion. 24. Fill all the orders of figures with 9's from the order of units to the order of octillions, both inclusive, point off, and read the same according to the French and the English systems of numeration. 61. Write in figures the following numbers, and numerate them according to the English system of numeration: 1. Twelve hundred forty-one million, one hundred twenty-three thousand five hundred fourteen. 2. Four hundred nineteen thousand one hundred fifty-two million, twenty-one thousand forty-Seven. 3. Fifty-three billion, two hundred twelve thousand twenty-six million, seventy-five thousand three hundred eighty-four. Ans. 53,212026,075384. 4. 1342 trillion, 11122 billion, 14 million, 19 units. pºse 5. 1 quadrillion, 1 trillion, 1 million, 1. 62. Read the following numbers: XXXIV. LI. C. CCC. DC. MMDCLI. XXXV. LXI. CI. CD. DLXIX. MMMXC. |XXXVI. LXIX. CX. CDXIV. M. MMCCX. XLIX. XC. CL. D. MC. MMCMMDX. Write, in the Roman System of Notation, the following numbers: 9, 12, 14, 37, 49, 83, 108,519, 1519, 14704, 88976, 13140363, 1001001001. SYNOPSIS FOR REVIEW. 1. : i 19. 20. 21. 22. 23. 24. 25. 26. 27. 11 SYNOPSIS FOR REVIEW. Cº-º- 63. Define the following words and phrases: A Definition. Science. Art. Quantity. A Unit. A Number. Like Numbers. Unlike Numbers. Odd and Even Numbers. The Unit of a Number. The Unit. A Fractional Unit. Numbers. An Abstract Number. A Denominate or Concrete Number. A Compound Number. The Arithmetical Units of Orders. An Arithmetical Complement of a Number. An Arithmetical Supplement of a Number. A Problem. The Premises or Conditions. A Solution. A Philosophic Solution. An Axiom. A Theorem. Demonstration. Comparison. 28. 29. 30. 31. 32. 33. 34. 35. 36. 37. 38. 39. 40. 41. 42. 43. 44. 45. 46. 47. 48. 49. 50. 51. 52 - e 53. 54. A Solution Statement, or an Opera- tion. A Formula. Philosophy. Arithmetic. Figures. Value of Figures. The Simple Value. The Local Value. The Radix. f A Scale or System of Numbers. The Binary Scale. The Trinary Scale. The Decimal Scale. The Duodecimal Scale. The Decimal Scale or System. To Change one Scale to Another. Order of Figures. Decimal System of Figures. Notation. Arabic Notation. Roman Notation. Numeration. Reading Numbers. Two Systems of Numeration. French System of Numeration. English System of Numeration. The Roman System of Notation. gºigns and Syrrhboſs. signs and s>yrnees: _-. *—ºº--A 64. Signs and Symbols are used to abridge arithmetical operations. They also indicate some relationship existing among numbers, and what operation is to be performed. The signs in general use in Arithmetic are as follows: +, −, X, +, = , (), or , , :, ; :, ..., 16°, v', ', 19, 29, 3°, ,- 65. The perpendicular or Greek Cross, (+) is the sign of Addition ; it is called plus, and is read plus, or and. It means more, and indicates that the num- bers between which it is placed are to be added. Thus, 7 + 9 is read 7 plus 9, and means that 7 and 9 are to be added. When used after a number, thus, 5 +, which is read 5 plus, it means 5 and a small excess. 66. The horizontal line (–) is the sign of Subtraction, and is called minus. It means less, and indicates, when placed between two numbers, that the one that follows it is to be taken from the one before it. Thus, 7–3 equals 4. 67. The oblique or Saint Andrew's Cross, (x) is the sign of Multiplication. It is read multiply by, or times. It indicates that the numbers between which it is placed are to be multiplied together. Thus, 7 × 5 is read, 7 multiplied by 5, or 5 times 7. 68. The horizontal line with a point above and a point below it, (--) is the sign of Division. It is read divided by. It indicates, when placed between two numbers, that the one before it is to be measured or divided by the one after it. Thus 12-i-4 equals 3. 69. The parallel horizontal lines (=) are the sign of Equality. It is read equals or is equal to. It indicates that the quantities between which it is placed are equal. Thus, 5+3=12—4. A statement of this kind is called an equation, because the quantity of 5+3 is equal to 12–4. 70. The () or is the sign of Aggregation. The first is the Parenthesis, the second is the Vinculum. They are both used for the same purpose. They indicate that the numbers within the parenthesis or below the vinculum, are to be considered as one quantity. Thus, 16–(5+4)=7, or 16—5+4=7. See page 134. 71. The single point (...) is the Decimal sign. It indicates that the num- bers which follow it are tenths, hundredths, etc. Thus, .5, .05 are read 5 tenths, 5 hundredths. Af 72. The (: ) is the sign of Ratio. It is read, is to or the ratio of. 73. The (::) is the sign of Proportion. It is read, as, or equal. Thus, 3: 6 :: 5: 10 is read 3 is to 6 as 5 is to 10; i.e. 3 is such a part of 6 as 5 is a part of ten. (36) Yºk SIGNS AND SYMBOLS. 37 74. The (16*) is the sign of Involution. The small figure to the right and the top of the number indicates the power to which the number is to be raised. Thus, 8* indicates that 8 is to be raised to the second power or taken as a factor twice. Thus, 8×8=64. 8* indicates that the third power of 8 is required. Thus, 8 × 8 × 8–512. 75. The radical sign (V) is the sign of Evolution. It indicates that some root of a number is to be found. Thus, V64 indicates that the square root of 64 is to be found. V512 indicates that the cube root of 512 is required. V4096 indicates that the fourth root of 4096 is to be extracted. The small figure within the branches of the radical sign is called the index, and indicates what root is required. Other signs of evolution, now often used, are as follows: Thus, 256* indicates that the square root is to be extracted. 625* indicates that the cube root is to be extracted. 1463 indicates that the square root of the cube of the number is required. 76. The (...) is the sign of Deduction. It is read, therefore, hence, or con- 8èquently. 77. The Interrogation (?) is the sign meaning what, or how many. It signi- fies that the answer to the question asked is to be found. 78. The expressions 19, 20, 30, denote first, second, third, etc. 79. The sign of the comma (,) is the sign of Numeration. It is used to separate large numbers into periods, to facilitate the reading of them. SIGNS PAND DABBREVIDTIONS, _*. -r Aº- wº- 80. The following are the principal signs and abbreviations in general use among merchants and business men, and are used in this work: Abbreviation, contraction of a word or phrase by omitting letters or substituting certain charac- ters. Ø, at, to, per, for, or for each. % or acct., account. A. D., the year of our Lord. Agt., Agent. . M., Forenoon. amt., amount. ans., answer. Apr., April. ass’t'd, assorted. Aug., August. bal., balance. bbl., or brl., barrel. bgs, bags. bdls., bundles. bls., bales. B. L., Bill of Lading. bkts., baskets. bot. bought. B. M. the gay world. B. Pay., Bills Payable. B. Rec., Bills Receiv- able. To O., buyer's option. |bgue., barque. br., brig. bu., bushel. bXs., boxes. c. or ſº, cents. Cap., Capital. chts., chests. cks., checks, casks. Co., Company. C. O. D., Cash on De- livery. C. B., Cash Book. com., commission. const., consignment. Cr., Creditor. C.S., Cases. cwt., hundred weight. (l. pence. D. or d., or dol., or $, dollar. do., or “the same. dft., draft. NotE.—The origin of the dollar sign, $, has been variously accounted for: some to be an abbreviation of the letters U. S., the initials for United States. Dec., December. doz., dozen. Dr., Debtor. ds., days. dwt. or pwt., penny- weight. ea., each. E. E., Errors excepted. E. & O. E., Errors and omissions excepted. emb’d., embroidered. Eng., English. En Ville, or E. W., in the town or city. ergo, therefore. Et als., and others. Ex Rel., on the relation or information of the aggrieved party. ex., example. exch., exchange. exp., expenses. f., farthing. fav., favor. Feb., February. fig'd, figured. f. o. b., free on board. fol., folio. fr., franc. ft., feet or foot. fwd. or for’d, forward. frt., freight. gal., gallon. gr., grain, grOSS. hf., #. § hf chts., half chests. hhd., hogshead. in., inches. I. B., Invoice book. i. e., that is. ins., insurance. inst., present month. int., interest. inv., invoice, inventory I. O. U., I owe you. Jan., January. Ibs., pounds. Led., Ledger. L. F., Ledger folio. M., Noon. m/a., months after date. Mar., March. mdse., merchandise. MS., manuscript. mo., month. No., number. Nov., November. N. B., note well. N. P., Notary Public. Oct., October. O. I. B., Outward In- voice Book. O. K., all right. Oz., ounce. p., Dage. p])., pages. pay’t., payment. pd., paid. pkgS., packages. pcs. Or ps., pieces. pr., pair. per or #3, by. plts., plates. pwt. or dwt., weight. P. M., afternoon. P. O. Post Office P. P. C., to take leave. Prox., Next month. P. S. Postscript. pun., puncheon. pts., pints. qr., quarter. qts., quarts. R. R., Railroad. rec'd, received. rec’t., receipt. R. S. V. P., answer if you please. s. or / shilling. penny- S. B. Sales Book. Sept., September. Schr., Schooner. Sh., Ship. Shpt., Shipment. Sunds., Sundries. S. o., seller's option. Sr., Senior, Str., Steamer. Supt., Superintendent. T. O., Turn. Over. tres., tierces. ult., last month. U. S., United States. ves., vessel. W. I., West Indies. wt., weight. yd., yard. yr., year. # or L., pound. %, per cent. , number. +, sign of addition. —, sign of subtraction. ×, sign of multiplica- tion. --, sign of division. =, sign of equality. 1", one and one-fourth. 1*, one and one-half. 1°, one and three- fourths. Mk., Marks, the Ger- man monetary unit. V, Check mark; cor- rect; approved. GB or $, Cifrao, used to separate the mil- reis from the reis in Brazil money. 17 doz. $, ºr, $1%, $º = 17 doz., 4 of which are Ø $10 per doz., 6 @ $12 and 7 (2 $15. 8 doz. 3 (a) 5 ſ : a 4/6, = 2 doz. No. 4 @ 5 shillings per doz., and 6 doz. No. 5 (2 4 shillings six- pence per dozen. It is thought by But it was in use long before the adoption of the Federal currency, and it is probably a modified figure 8, denoting a piece of eight, i. e., eight reals—an old Spani The character £, pound sterling, the initial letter of the Latin word Librae, pound. sh coin, the value of which was a dollar. . is but a capital L with a line drawn across it, and represents (38) X- SIGNS AND ABBREVIATIONS. 39 The sign ſ, ºr Ş. is believed to have been originally a capital S (written thus, / ), the initial letter for the word shilling. The sign fö., pound weight, is formed from the first and third letters of the Latin word Libra, and connected with a horizontal line. The sign (3) is a modification of the Latin word ad, which means at or to. The sign ºl, fºr penny, one-twelfth of an English shilling, is a coin corresponding in value to the gºd of the old Saxon pound, which was the same in value as the Roman demarius, the go of the Libra. Hence used to denote penny. The character 36 is a modification of +, the sign of division. Thus eight per cent. may be expressed by Iño, or 8 - 100, or by omitting the denominator and writing it thus, 8-i-, or in rapid writing thus, 8 °ſ, or as we now write it, thus 8%. The character #3 is another form of p, the initial letter of the Latin word Per. The abbreviation Q. K. is used by business men and telegraph operators, and signifies “all right,” or “inspected and approved.” The abbreviation Fr. is a contraction of the word France, which is derived from the German Word Franke, a name of the German tribe who, in the fifth century, overran and conquered Gaul, and established the Kingdom of France. S. L. & C. means Shippers Load and Count. These letters are used on Railroad Bills of Lading and restrict the responsibility of the Railroad Company regarding the correctness of the articles shipped. C. I. F. or Cif. These letters are used by buyers when making offers for goods, and mean Cost, Insurance and Freight, and all charges to the wharf of the buyer. g Shippers’ Order Notify. When these words are indorsed on a Bill of Lading, they mean that the consignee cannot take possession of the goods until he has paid the Draft drawn on him by the shipper and received the Bill of Lading for the goods. The Draft and Bill of Lading are sent by the shipper to a Bank which notifies the consignee that they hold a Draft on him, which by paying he will be given the Bill of Lading for the goods. SSG) @2%) * *-*-es--- 3.3 Fº */ :------------Rs 81. Addition—Increasing—is the process of uniting two or more numbers of the same name or kind, into one equivalent number. 82. The number obtained by this process is called the sum or amount. 83. The Sign of Addition is a perpendicular cross, +, called plus; it means more; thus, 7 -i- 9 is read, 7 plus 9, and indicates that 7 and 9 are to be added. When used after a number, thus, 5 +, which is read 5 plus, it means 5 and a Small excess. 84. The Sign of Equality is =. It is read equals or is equal to, and denotes that the numbers between which it is placed, are equal to each other; thus, 7 H-9 = 16, means that 7 and 9 added, are equal to 16. The expression is read, 7 plus 9 equals 16. 85. A Numerical Equation is an equality between two numerical expres- sions, which, though differing in form from each other, are equivalent. Each expres- Sion is called a term of the equation. Thus, 5 + 8 = 13 is a numerical equation in which the 5 + 8 is called the first member of the equation and 13 the second member, and both are called the terms of the equation. 86. Principle of Addition. Numbers of the same kind, order, or character only, can be added. Thus we cannot add 2 apples and 3 oranges; or 5 pounds of sugar and 6 boxes of peaches; or 6 units and 5 hundreds; or ; and #, etc. We can only add apples to apples, Oranges to oranges, sugar to Sugar, peaches to peaches, units to ultiis, hundreds to hundreds, halves to halves, fourths to fourths, etc. We can collect together things of different kinds, apples, peaches, oranges, etc., but by collecting them together we do not increase the number or sum of either, and hence there is no addition. Addition is the basis of all numerical operations, and being so constantly used in all departments of business life, therefore it is essentially necessary that the accountant and business man should be both accurate and rapid in the various methods of adding numbers. By due attention to the principles and combinations of numbers, as herein presented, aided by practice, the learner can, in a few weeks, acquire a degree of proficiency that will place him in the front rank of experts, and doubly capacitate him for service to himself or to others for whom he may labor. To assist in disciplining the mind and to point out the best and most rapid systems of adding columns, we present the following: - (40) ADDITION TABLES. 4I 87. 5 7 apply accurately, the result of two or more figures. ADDITION TABLES. NOTE.—In learning these tables and in handling all numbers, all intermediate words and thoughts that occur between the numbers to be combined, should be omitted. Thus, instead of saying or thinking that 2 and 2 are 4, 3 and 5 are 8, etc., say or think 4; 8; etc. *|1 ; ; }. . }. i. . ee}#. i ; : i ; * "a,Tole INTo- L- Explanation.—In this table, we show 20 different combinations of the 9 significant figures, to pro- duce results from 1 to 9. It may be said that three 1’s make 3, three 2's make 6, etc., and that they are regular combinations; but we see by the table that two 1’s are 2, and that two 2's are 4, etc. Hence, though the table does not contain all the possible combinations, it does contain all that are essential and of value in this connection. 10 11 13 14 16 17 1 8 * "a Tole * : $. 3. 3. º | . ; 3. ; INTc. 22. Eacplanation.—In this table, we show the 25 different combi- nations of the 9 significant fig- ures, the sum of which equals ten or more. To attain rapidity in adding, it is absolutely neces- sary that the learner should be so familiar with these combina- tions that he can instantly See the result without adding, i. e. he must know the result by the combination, just as he knows the value of 4 or 5, by the com- bination of lines forming the figure, or as he knows the pro- nunciation of a word without, spelling it. The rapid increasing and decreasing operations in the Science of numbers, depend largely upon the capacity of the calculator to apprehend instantly and to is to aid the learner in acquiring the desired rapidity. And the object of these tables 88. ADDITION AND SUBTRACTION TABLE. * Talole INTc- 3- 1 & 3–9 1 & 3–8 || 1 & 3–7 || 1 & 3–6 || 1 & 3–5 & 3–4 || 1 & 2–3 || 1 & 3–2 2. “ “ 9 2 4 4% 8 2 * { 4 7 2 : {{ 6 2 “ {{ 5 2 44 4 4 2 “ 44 3 3 “ “ 9 || 3 “ “ 8 || 3 “ “ 7 || 3 “ “ 6 || 3 “ “ 5 || 3 “ “ 4 4 (4 “ 9 4 44 “ 8 || 4 44 4 7 || 4 44 (4 6 || 4 44 (4 5 5 (€ 4 9 || 5 4 “ 8 || 5 4 “ 7 || 5 4: “ 6 6 44 4 9 || 6 4' 48 || 6 4' 4 7 7 * 4 9 || 7 44 (+ 8 8 4 “ 9 the difference between each is to be supplied by the learner, Explanation.—In this table, we present the 36 combinations of the significant figures, in which This is a very important table for rapid work in subtraction, by the addition method, and should receive careful attention. 42 SouLE's PHILOSOPHIC PRACTICAL MATHEMATICS. 89. ADDITION AND SUBTRACTION TABLE. "T"a Tole INTo- 4:- 1 & 3–100 26 & 3–100 51 & 3–100 76 & 2–100 2 * 44 100 27 44 44 100 52 “ “ 100 77 44 44 100 3 ** {{ 100 28 4 44 100 53 44 44 100 78 “ “ 100 4 “ 44 100 29 4 44 100 54 ($ 44 100 79 (4 tº 100 5 “ 44 100 30 46 (£ 100 55 “ 44 100 80 ($ 44 100 6 4' 44 100 31 4 44 100 56 4' 44 100 81 tº 44 100 7 “ (: 100 32 “ “ 100 57 “ “ 100 82 “ “ 100 8 “ (: 100 33 44 4 100 58 44 44 100 83 44 44 100 9 ** {{ 100 34 “ 4 100 59 “ (: 100 84 44 44 100 10 * $ 44 100 35 “ 4 100 60 “ “ 100 85 ($ 44 100 11 ** {{ 100 36 44 °4 100 61 (< 44 100 86 4' 44 100 12 4° 44 100 37 44 44 100 62 “ (: 100 87 “ “ 100 13 ** 44 100 38 44 44 100 63 44 44 100 88 “ 44 100 14 “ “ 100 39 4 44 100 64 “ “ 100 89 “ 44 100 1 5 “ 44 100 40 “ 44 100 65 “ 44 100 90 46 “ 100 16 “ “ 100 41 4 44 100 66 ($ 44 100 91 4 44 100 1 7 “ 44 100 42 4° 44 100 67 tº 4 100 92 “ “ 100 18 “ 44 100 43 44 44 100 68 44 44 100 93 44 44 100 19 “ 44 100 44 44 44 100 69 “ 44 100 94 44 44 100 20 ($ 44 100 45 % ($ 100 70 (£ 44 100 95 44 44 100 21 “ (4 100 46 (£ 44 100 71 44 44 100 96 “ (: 10() 22 “ “ 100 47 44 44 100 72 4° 44 100 97 (£ 44 100 23 & 4 100 48 (£ 44 100 73 44 44 100 98 “ 44 100 24. 44 44 100 49 44 44 100 74 44 44 100 99 44 44 100 | 25 “ (: 100 50 44 44 100 75 (£ 44 100 Ea:planation.—We present this table to aid the learner in instantly seeing the difference between 100 and any number from 1 to 99. It is of special value in addition and subtraction, and all who expect to become rapid Calculators must be proficient in this character of work. These tables constitute the alphabet of numbers, and render obsolete the dis- gusting and mind weakening practice of counting fingers, birds ponds, apples on trees, or rabbits in the yard, etc., which is so often seen in primary arithmetics. on limbs, ducks in : : & i i 90. With one look at each of the following groups of numbers, name the result and repeat the operation until you can call the amounts at sight. 4 t; : i i i i 4-: | : ! 9. * 7 # : i * -; f 3 : 1 2 4 1 6 3 7 1 8 3 4 3 3 2 5 2 6 2 2 1 8 4 5 6 7 3 4 1 4 2 9 3 4 3 2 5 9 5 6 3 5 0 2 8 9 8 9 8 9 8 8 7 9 5 6 8 6 6 4 4. 2 2 9 8 7 9 7 3 8 7 6 8 5 5 6 9 4 7 5 9 3 3 4 4 7 5 6 1 7 7 8 3 4 5 1 6 9 8 2 3 6 5 5 2 2 3 3 2 1 5 7 4 7 7 9 8 7 8 2 4 6 8 9 3 7 1 8 4 6 5 4 9 7 9 7 4 8 2 7 7 8 5 6 8 8 5 5 8 9 9 8 9 8 8 5 6 9 DRILL EXERCISE NO. 2. 6 7 8 9 4 7 2 4. 1 7 8 6 7 5 8 3 2 9 8 0 2 4 6 3 2 8 9 7 3 7 9 1 2 3 4 3 7 4 4 5 6 8 4 1 3 9 3 9 3 1 5 0 9 8 7 6 5 4 3 9 5 5 4 4 3 3 2 2 1 1 6 7 8 9 0 3 1 8 2 4 () 6 0 9 6 8 2 5 2 7 5 8 — — — — — — — — — — — (43) 44 SOULE's PHILOSOPHIC PRACTICAL MATHEMATICS. 4% DRILL EXERCISE No. 2.—Continued. 9 0 8 7 5 3 4 2 1 8 6 4 9 4 1 7 2 8 9 l 2 3 4 5 6 7 8 9 0 9 1 8 2 7 3 7 4 5 8 1 2 7 0 4 7 8 9 9 8 8 3 2 5 1 9 0 1 2 7 8 1 9 2 6 7 4 5 1 2 3 4 5 6 7 8 9 7 1. 8 2 0 3 7 4 6 5 0 9 8 7 5 6 4 3 2 1 2 3 4 5 6 7 8 9 0 8 4 7 2 9 1 4 8 2 9 0 9 8 8 7 7 6 6 5 5 5 4 3 3 2 1 0 6 5 6 9 8 5 4 8 2 9 3 8 4 1 5 2 6 9 5 2 8 2 2 9 9 8 8 7 7 6 1 0 6 8 2 4 9 0 3 1 0. 1 2 3 4 5 6 7 8 9 0 9 8 7 6 5 4 3 2 0 1 3 4 7 5 6 8 2 1 9 3 0 9 6 7 4 3 2 8 1 9 1 2 3 4 5 6 7 8 9 3 8 4 9 5 2 1 6 8 9 8 7 6 5 3 2 1 7 0 4 9 1 2 8 9 0 3 3 5 8 9 1 0 3 8 2 6 4 9 0 7 8 9 1 8 4 5 4 5 4. 5 4 5 4 5 4 5 4 5 4 5 4 5 9 8 7 6 5 4 3 2 1 8 5 6 4 3 8 1 6 3 1 4 2 5 9 6 0 2 5 6 8 9 1 3 9 0 2 5 9 7 3 5 7 9 1 8 3 8 6 0 4 8 2 8 6 3 9 1 7 4 1 9 2 8 3 8 4 7 5 6 0 9 1 8 3 6 2 5 8 1 1 2 6 3 5 4 8 9 1 9 2 8 3 7 4 6 5. 8 9 0 6 3 9 2 8 1 0 6 2 4 5 9 6 8 2 9 6 7 8 9 0 1 2 3 4 5 6 7 8 9 3 8 5 9 3 2 1 4, 9 8 1 4 0 3 6, 8 9 1 6 3 8 2 6 9 3 2 8 7 8 9 7 8 3 9 3 9 8 5 2 8 2 7 7 4 5 6 7 4 8 8 9 4 7 8 9 8 8 9 3 2 1 5 7 6 5 8 3 7 9 7 5 8 4 8 7 2 7 5 2 6 5 1 2 3 4 5 6 7 8 9 () 1 2 3 4 5 6 7 8 9 DRILL EXERCISE NO. 3. 91. 1. Write all the combinations of two figures that make 2, 3, 4, 5, 6, 7, 8 and 9. 2. Write all the combinations of two figures, that make 10, 11, 12, 13, 14, 15, 16, 17 and 18. Repeat the following exercises several times, naming the results as rapidly as you can, and continue the exercise for several consecutive days: 3. Commence with 1 and orally add thereto 2, and continue to add 2 to the successively occurring sums, until you produce 101. Thus, 3, 5, 7, 9, 11, 13, etc. DRILL EXERCISE. 45 DRILL EXERCISE No. 3.-Continued. 4. Commence with 1 and in like manner add 3 until you produce 100. Thus, 4, 7, 10, 13, etc. 5. Commence with 1 and in like manner add 4 until you produce 101. Thus, 5, 9, 13, 17, etc. 6. Commence with 1 and in like manner add 5 until you produce 101. 7. : 11. Orally add by 2’s until you produce 20. 12. 13. 14. 15. 16. 17. 18. 19. 20. 21. 22. 23. 24. {{ {{ {{ 4% {{ {{ {{ 6é & 4 { % 3'S {{ 4’s {{ 5’S 4% 6'S 4% 77S {{ 8’S 44 9'S ** 107S 46 11?S << 127S {{ 137S {{ 14?S {{ 15'S 4% {{ {{ 4% 4% {{ {{ 4% 30. 44 40. {{ 50. 44 60. 44 70. & 4 80. {{ 90. {{ 100. 4 110. 44 120. {{ 130. ** 140. 4, 150 Commence with 1 and in like manner add 6 until you produce 61. Commence with 1 and in like manner add 7 until you produce 71. Commence with 1 and in like manner add 8 until you produce 81. Commence with 1 and in like manner add 9 until you produce 91. 25. Commence at 1 and orally add 3 and 5, alternately, until you produce 100. 26. Commence at 1 and orally add 4 and 7, alternately, until you produce 100. 30. Add by 2's, 3's and 4's alternately from 0 to 99. 92. Add the following problems orally: 5-H6+8=? 7+4+9=} 6-H7+5=} 9-H8+8=? 4-H 7–H9–? 8-H 9-H6=? 14+ 7+ 3=} 21–H 0–1– 2=? 7+12+ 0=? 26-1- 9-H12=? 8-H23–H 5=} 10–1–15–H16=} 22–H 8+ 1=? 12+ 9-H 7=? 4+ 9-H 8=? 20–H40–H 3=? 22+14+ 4 =? 17+19.--18=? What is the sum of 8 peaches and 10 Oranges } What is the amount of four 0's plus three 0's? Add twelve thousand, twelve hundred and twelve. What is the sum of twenty blackbirds, twenty sick birds and twenty-six red birds? 24+16+ 5+ 8=? 7+ 9-1-30+ 5=? 18-H 12–1–15+ 6=? 43+ 7+14+ 9=? 72-H21+ 9-H 14=? 69-H15+12+36=? 46 SOULE's PHILOSOPHIC PRACTICAL MATHEMATICS. Aſ. WBITTEN PROBLEMS IN ADDITION. 93. Add the following numbers, 6376, 564, 309, 485, and 5092: OPERATION. : : | 3 () 9 485 5092 Sum, 12,826 Carrying figure8, 132 Ea:planation.—In all addition problems, we first write the numbers so that units of the same order stand in the same column, i. e. whits in the units’, or first column; tens in the tens', or second column; hundreds in the hundreds', or third column, and so on through the numbers. We then begin at the units’ or first column and add the columns separately. In adding the first column, we commence with the 2 and 5, and name only the successive results; thus, 7, 16, 20, 26, which is 2 tens and 6 units; the 6 we write in the first place, or column of units, and place the 2 tens which is to be carried to the column of tens, directly below the 6 in a small figure. Then adding the 2 tens to the tens' column, we say, 11, 19, 25, 32—which is 3 hundreds and 2 tens; the 2 tens we write in the column of tens, and place the 3 hundreds which is to be carried to the hundreds' column, directly under it. Then adding the 3 hundreds to the hundreds' column, we say, 7, 10, 15, 18, which is I thousand and 8 hundreds ; the 8 hundreds we write in the hundreds' column, and the carrying figure, 1 thousand, directly under. Then adding the 1 thousand to the fourth, or thousands' column, we say, 6, 12, which is 1 ten thousand and 2 thousands, and this being the last col- umn to add, we write the figures in their respective columns and produce 12826 as the 8wm of all the numbers. NOTE.--When adding, set the result in pencil figures, being careful to place the carrying fig- ure or figures directly beneath the answer figure of each column added, as shown in the preceding problem. Different methods of using the carrying figure, elucidated by the following examples: THIRD METHOD. FIRST-REGULAR METHOD. SECOND METEIOD. $ 5462.48 $ 5462.48 37 $ 5462.48 848.19 848.19 31 848.19 761.78 761.78 25 761.78 81484.52 81484.52 33 81484.52 7948.93 7948.93 40 7948.93 852.57 852.57 13 852.57 smºmºmºsºm-mººms *=ºmºsºmºmºsºms 8 **E= $97358.47 .37 OI’ smm mºsº, sºmmºns 8 1432,33 3.1 $97358,47 13 * > 25. 40. 33 33 40 25 13 3.1 S .37 $97358.47, $97358.47 Explanation.—By the first or Regular Method, which is preferable for accountants, the carry- ing figure is written in a small figure under the resultant figure of the column added. By the second method, we commence on the right and add each column independently of the other columns, and set the results under the numbers added, or to the right of them, as is shown in the operation; then these several results are added, and produce the sum or amount. By the third method, we commence on the left and add each column independently of the other columns, and set the results as shown in the operation; then we add these several results and produce the sum or amount. It will be observed that in the second and third methods the units, tens, hundreds, thousands, etc., are written in their respective columns. NotE.-In the second and third methods, the results of the separate columns may be written to the right or left of the numbers added, or on a separate piece of paper, and the final result thereof written beneath the numbers added. NOTE 2. The second method, shown above, is sometimes called the Civil Service Method, because it is used by many of the Civil Service Clerks. x- GENERAL DIRECTIONS FOR ADDITION. 47 GENERAL DIRECTIONS FOR ADDITION. 94. From the foregoing elucidations, we derive the following general direc- tions for addition : 1. Write the numbers so that units of the same order stand in the same column. See explanation, page 46. 2. Begin at the units’, or first column on the right, and add each column sepa- rately, writing the unit result under the column added, and carrying the tens, if any, to the tens’ column. At the last column, write the full sum of the column. See explan- ation, page 46. 3. To add horizontally, the numbers are not written in columns of like orders; and the result, or sum, is written to the right of the numbers. PROOE OF ADDITION. 95. The best proof of the correctness of addition is for the calculator to be proficient in his work, and then re-add the columns in the reverse direction. Cast. ing out the 9's, as is sometimes done, is not positive proof of correctness, as the figures may be transposed, or replaced by wrong ones having the same sum, or mistakes may be made that balance each other, and the work may appear to prove right when it is wrong; though it will never prove wrong when it is right; hence many accountants in verifying their calculations, prefer to repeat, or go over the addition in a reverse direction. When a column is added in the reverse direction and the result disagrees with the first addition, then omit the bottom figure of the column when adding it up and the top figure of the column when adding it down, until all the other figures of the column have been added, and then add the omitted figures. By this means, the combination in which the error has been made is changed, and the error, which might be repeated indefinitely, is avoided. See page 152 for a system of proof of addition, subtraction, multiplication, and division, and also of the proof of the correctness of the transfer of numbers. WRITTEN PROBLEMS. 96. Add the following numbers, and verify the work by reverse addition: 359 Ea:planation.—Commence at the top of the right hand column and add, naming ; results only, thus: 13, 20, 22, 30, 37, 42,49; then add the 4 tens to the second column 892 and proceed in the same manner, 9, 10, 13, 22, 28, 32, 34, etc. 768 In adding, the thought of words and the repetition of results should be ; banished from the mind. Never allow yourself to spell your way or crawl up or 587 down a column, as is too often done by many teachers and their pupils. Thus: 9 --> and 4 are 13, 13 and 7 are 20, 20 and 2 are 22, 22 and 8 are 30, 30 and 7 are 37, 37 and 5429 5 are 42, 42 and 7 are 49. 48 soule's PHILOSOPHIC PRACTICAL MATHEMATICs. (2) 47 72 12 45 68 21 33 64 25 35 (3) 63 28 34 98 92 32 99 55 67 23 97. WRITTEN PROBLEMS.–Continued. (4) 124 690 658 67 623 489 109 327 432 399 ADDITION This is a very rapid system of adding and consists in grouping two or . (5) 845 324 97 8 326 72 216 123 78 666 (6) 4072 1867 504 1090 68 507 92 417.3 8293 8902 (7) 6913 2456 4379 308 6547 7085 2123 3458 6565 4567 (8) 540314 123456 342189 67310 102938 125439 334455 121212 304958 123456 BY GROUPING THE FIGURES. (9) 687.659 789021 90876 . 427893 608091 623784 989790 602548 553241 789021 more figures in a column and naming the result of the group, in the same manner as we group and name the result of several letters in words, when we read. The following problems will illustrate this method: 11; 31} 37; º ; 69 #18 22 8 6 34 4 41 9 61 6 ſi 9 1 | Explanation.—Commence at the top of the first column and add thus: 13, 22, 34, 41, 49, 61. Then add the 6 tens or carrying figure to the second column and continue as before, 17, 31, 37, 49, 61, 69. NOTE.--To attain a high degree of skill in this method, the student should know perfectly the alphabet of numbers, as given on page 41. 3 DRILL EXERCISE. 98. In the second, third and fourth examples following, the columns are grouped, as indicated by the braces, as they Would naturally be grouped mentally, in practical addition : |} ( ( jºr HORIZONTAL ADDITION. 49 In adding long columns, the rapidity may be often increased by grouping and adding such numbers as make 10 or 20, independently of other groupings. Where the same figure occurs more than once, multiply it instead of adding. Thus, if the figure 7 occurs 4 times, multiply it by 4 and use the product instead of adding. Where figures occur in regular order and the number is odd, thus, 5, 6, 7, or 19, 18, 17, 16, 15, then as many times the middle number as there are numbers in regular order, will be their sum. When the number in regular order is even, thus, 2, 3, 4, 5, or 17, 16, 15, 14, 13, 12, then one-half as many times the sum of the extremes as there are numbers in regular order, will be their sum. Accountants generally add down the column so that the sum, when found, is at the foot of the column where it is to be written, and to prove the result, they re-add the column upward. EHORIZONTAL ADDITION. 99. This method of addition consists in adding numbers from right to left, or from left to right, when they are written in horizontal lines or when they are not written in vertical columns of like orders. The method is very valuable in business and is in general use by the most efficient clerks and accountants. NOTE 1.--When using this method great care is required to add, only FIGURES OF LIKE ORDER. NOTE 2.—The method of adding two figures of the numbers at a time may be used when adding horizontally. Add horizontally the following numbers: (1) 456, 248, 3947, 1084, 2681, 89020 = 97436. NOTE.--When adding horizontally, add toward the side at which the total result is to be written. Thus in the above we commence on the left and say 6, 14, 21, 25, 26; then (carrying the 2 to the 5) 7, 11, 15, 23, 31, 33; then (carrying the 3) 7, 9, 18, 24; then (carrying the 2) 5, 6, 8, 17; then (carrying the 1) 9, which completes the result. \ Add the following numbers horizontally: (2) 245, 362,2581, 409, 687, 5498, 64, 791062 = (3) 79, 508, 309, 84, 506, 1742, 18, 164307, 4235, 78215 = (4) 516, 4027, 684, 97, 68, 1493, 54, 352085, 781, 5638, 721409 = Add the following numbers horizontally and vertically: (5) 4345 1542 5136 6362 3809 3908 2621 # $: ; ; ; ; 2214 214 136 1043 2132 1025 47 # # 3&#: ; 3625 2153 214 256 3429 836 215 # #35:5: ; 5625 2101 9876 2123 1215 3724. 215.2 § 3; :::::: 1212 3.120 215.2 1312 21.21 3002 82 # $ $35: ; 1198 28 73 2006 7849 2150 6 $ $: ; ; ; ; ; 4621 982 4215 2982 1215 1098 4635 # * : * : *: ; 5731 4842 1563 2842 1563 4910 3125 #$: ; ; ; ; ; #::::::::::A; ####:}; ***** | ***** ## 8:3# $$$$$$$. :*:::::::::::$ *:::::::: ; 5o 'SOULE's PHILOSOPHIC PRACTICAL, MATHEMATICS. * DRILL EXERCISES. 100. What is the sum of each of the following Columns of numbers? (1) (2) (3) (4) (5) (6) 4304 780 890 777 9040 984.75 291 1261 706 888 1288 8697 643 537 73 999 9907 46789 98 309 4009 666 6543 57698 1400 6987 8888 645 2018 94.576 2099 7286 5679 388 5281 49567 8877 1896 2834. 777 6745 84685 101. Add the following numbers horizontally: (1) 248, 3936, 409, 1278, 97, 563, 9210, 45682 = (2) 335, 1468, 87, 911, 1809, 19068, 54, 17401 = (3) 72, 13615, 41848, 1905, 8, 9763, 691, 8625 = 102. Drill daily on the following columns of numbers with and without grouping, until you acquire great rapidity: (1) (2) (3) (4) (5) (6) (7) (8) (9) (10) 818 412 582 328 809 98.1 864 677 595 849 390 297 578 346 523 350 363 305 249 283 970 318 757 386 605 269 629 420 463 327 276 824 420 672 848 789 145 982 830 651 752 932 731 793 945 696 174 217 221 543 843 373 542 864 397 136 144 326 232 502 865 576 853 965 684 169 176 111 151 113 129 876 684 448 976 295 767 871 387 4.38 768 444 743 404 666 468 644 512 516 455 904 102 915 151 217 687 747 814 247 328 972 814 686 148 879 825 156 376 331 633 114 331 637 263 516 951 106 468 281 624 346 554 917 295 259 784 872 189 828 581 545 161 650 161 896 122 694 177 986 491 622 197 411 461 864 440 788 885 817 888 749 490 237 874 565 450 866 264 918 992 717 876 349 898 150 414 944 294 289 202 222 902 489 769 514 654 922 896 259 548 234 396 698 243 446 789 116 597 381 365 166 484 228 174 576 458 911 814 329 208 365 235 433 952 489 747 866 277 678 662 272 386 949 683 394 636 179 476 640 764 729 624 687 574 407 241 129 716 821 287 955 897 762 956 812 477 659 802 457 848 177 477 849 658 798 681 778 584 587 255 tºm- tº mºmmasºme - - tº- - -** *- -*m-ſº 12978 TWOTE.-By prefixing 1 to the second line from the top, and affixing the unit figure of the third line to the second line, the footing or sum of each problem is shown. Yºr ADDITION OF FRACTIONAL NUMBERS 5 I TO ADD THE FRACTIONAL NUMBERS, 4 or g. 103. In business, the measurements of some lines of goods are recorded to the accuracy of 4 and # of the unit of measurement, and such parts or fractions are Written in Small figures to the right and at the top of the whole number thus, 41*, 39°, 40° would be read, in cases where the small figures represent FOURTHs, FORTY-ONE AND ONE-FOURTH, THIRTY-NINE AND THREE-FourTHs, and FORTY AND TWO-FOURTHS. In case the small figures indicated EIGHTHs, the numbers would be read 41 and ONE-EIGHTH, 39 and THREE-EIGHTHs, and 40 and Two-EIGHTHS. NOTE:-This work is presented in advance of fractions, on account of its importance in busi- ness. It is believed that it can be readily understood by the practical sense of students who have not yet studied fractions. Add the following numbers vertically and horizontally: 41*, 39°, 40°, 418, 391, 388, 38°, 40°, 418, 398. OPERATION wºux. OPERATION HORIZONTALLY. 398 402 41*, 39°, 40°, 418, 39%, 38°, 38°, 40°, 41*, 39* = 4013. 413 391 388 Explanation.—Commencing at the top of the vertical column or at the ; left of the horizontal line and adding the small figures indicating FOURTHS, we 4.18 obtain 23 FourTHs. Then, since 4 fourths equal on E, in 23 fourths we have 5 393 and 3 fourths. Then adding the 5 to the other whole numbers, we produce 401 401 # and 3 fourths. NotE.—In business, where there are many bales, bags, gallons, yards, pounds, etc., to add, it is usual to set the numbers to be added in columns of 10, as shown in the following examples: What is the total weight of the following 78 bales of cotton ? (Add the footings of the columns horizontally). (1) (2) (3) (4) (5) (6) (7) (8) 425 465 525 546 499 500 458 519 465 519 423 424 426 427 428 425 501 502 503 501 504 506 507 508 509 510 511 512 513 514 515 516 430 431 423 433 444 445 . 446 447 517 518 519 520 523 524 525 526 448 449 450 451 452 453 454 455 527 546 537 536 534 532 529 544 528 529 530 531 532 533 534 546 545 543 540 538 458 455 * * * * + 3& M. Jº 3: 39 ºf jº. # * * * * * * * % # # * + 3 + +& * * * * – ##### lbs. NoTE.—When weighing bales of cotton, it is not the custom to consider the fractions of pounds. Find the total weight of the following 96 bags of coffee : (Add the footings of the columns horizontally). (1) (2) (3) (4) (5) (6) (7) (8) (9) (10) 160 161 162 163 164 165 163 160 162 161 161 ,165 164 160 163 162 161 165 163 163 165 164 163 162 161 160 161 162 161 162 164 163 162 161 160 165 164 163 161 162 163 162 161 160 165 164 163 162 162 160 162 161 160 165 164 163 162 160 160 161 165 160 161 162 163 164 165 162 162 160 161 162 1:63 164 165 160 164 161 165 164 163 162 161 160 165 161 160 160 161 162 163 164 165 163 160 163 * =º tº-º — = ***** lbs. 52 SOULE's PHILOSOPHIC PRACTICAL MATHEMATICS. Weekly Report of Mail Matter Received at a City Post Office. CLASS OF MAIL. Mos. TUEs. | WED. | THURs. FRI. SAT. | TOTAL. -- Qrdinary Letters . . . . . . . . . . . . . . . . . . . . . 29246 || 28561 29203 || 27693 27367 39985 | ***** Registered Letters. . . . . . . . . . . . . . . . . . . . 1657 1823 1654 1236 1342 394 | ***** Postal Cards. . . . . . . . . . . . . . . . . . . . . . . . . . 5129 4214 3942 4870 5986 4652 | * * * * * Book Packets . . . . . . . . . . . . . . . . . . . . . . . 1844 1297 1329 1656 894. 773 | * * * * * Farcels * * * * * * * * * * * * * * * * * s • * * * * * * * * a e s a 950 720 604 898 471 504 | ***** Newspapers . . . . . . . . . . . . . . . . . . . . . . . . . . 41695 || 42322 || 44.275 || 46889 573.18 40207 | ***** Total * * * * * * * * * * * * * * * * e = * * * * * * * * * * * * * * + 2 + # # tº * * * * + ºf # * * % ºf 34, 3- 36. * * * * * # * * * $6. Weekly Report of a Circulating Library. CLASS OF BOOKS. MON. | TUES. WED. THURS. FRI. SAT. ToTAL. Fiction. . . . . . . . . . . . . . . . . . . . . . . . . . . . . . . 4761 3857 1935 4560 2976 4085 | ***** History. . . . . . . . . . . . . . . . . . . . . . . . . . . . . . . 1322 1421 1202 2121 1322 1382 | ***** Biography. . . . . . . . . . . . . . . . . . . . . . . . . . . . 1215 1110 1313 1214 1124 1174 ***** Science . . . . . . . . . . . . . . . . . . . . . . . . . . . . . . 2424 1324 1387 1216 1317 1566 | ***** Poetry. . . . . . . . . . . . . . . . . . . . . . . . . . . . . . . . 1316 1218 1400 878 420 817 | ***** Religion. . . . . . . . . . . . . . . . . . . . . . . . . . . . . . 198 408 108 135 308 297 | ***** Total . . . . . . . . . . . . . . . . . . . . . . . . . . # 4t 36 & # * * * + 3 + ºf * ºf 34 × * * * * 2::::kx sk:::::::::: |ADDITION OF DOLLARS AND CENTS. 104. Dollar and Cent Signs.—The dollar sign is $, and the cent sign is 2. When the dollar sign is placed before numbers, they are read as dollars. Thus, $45 is read 45 dollars. When the cent sign is placed after numbers, they are read as cents. Thus, 14g is read 14 cents. When dollars and cents are written together, the cents are separated from the dollars by a decimal point (...) and the sign of cents is omitted. Thus, $16.45 is read 16 dollars and 45 cents. Since there are 100 cents in 1 dollar, cents always occupy two places and only two, in connection with dollars. When the number of cents is less than 10, a naught must be used to fill the tens’ column, or the first place at the right of the point. Thus, 8 dollars and 5 cents are written $8,05. When cents only are written, they are expressed as follows: 25 cents or 25g or $.25. - When writing numbers representing dollars and cents for the purpose of addition, they must be set so that dollars will be under dollars, and cents under cents, in the regular order of units, tens, hundreds, etc., and the decimal points (...) that separate dollars and cents must be in a vertical line. The dollar sign ($) and the point (...) should never be omitted when writing dollars and cents, except when writing in books, or on paper, which contains dollar and cent columns. 105. The United States Monetary Units are as follows: 10 mills (m) = 1 cent, 2. 10 dimes = 1 dollar, $. 10 cents = 1 dime, d. 10 dollars = 1 eagle, E. * : PROBLEMS. 53 PROBLEMS. (1) (2) (3) (4) (5) (6) 106. Add $321. $521.16 $ 9.45 $ 431. $194.15 $684,22 640.80 83.25 80. 124. 8.05 708.10 9.13 19.30 17. 381. 73.75 276.14 75.20 S. .65 569. 6.13 582.98 100.05 4,07 6.10 827. .95 928.35 $1146.18 $635.78 $113.20 $2332- $283.03 $3179.79 7. Add $108, $97.16, $84.12, $ .75, $8, $6.40, 25g, $18. Ans. $322.68. 8. Add $580.10, $671.23, $794.98, $88, 45%, 5%, $3.10. Ans. $2137.91. 9. Add $999.99, $888.88, $777,77, $666.66, $555,55, $444.44, $333.33, $222.22, $111.11, and 12. Ans. $4999.96. 10. Add $987.65, $876.54, $765.43, $654.32, $543.21, $123.45, $234,56, $345.67, $456.78, $567.89, $678.90, and $789. Ans. $7023.40. 11. James has $420; Conrad has $130 more than James; and Henry has as much as James and Conrad together. What sum have the three ? Ans. $1940. 12. A lady paid $42.75 for a dress, $80 dollars for a shawl, $21 for a bonnet, and $8.75 for a pair of shoes. What was the total cost 3 Ans. $152.50. 13. Add the following columns vertically and horizontally, and verify the work by adding the results: Totals. $468|24 || $306|50 || $ 48|21 || $516|18 || $233|40 || $108|96 || $410 |25 || $337|68 123 25 789 |06 437 |52 213|78 478 || 09 890 43 289 |67 869 || 30 371 || 60 930 || 48 270 |78 418 55 407 || 54 173| 26 157 |87 687 || 23 987|25 270 45 103 |48 682 |76 823 47 527 | 28 193 |16 184 || 50 818|25 390 || 31 970 |87 276|43 752 |34 843| 57 865 |12 976 | 89 972|11 434 || 65 545 |62 274|90 717|30 222 |34. 166||36 527 | 72 729 |55 217|41 297 ||31 318|82 249 || 32 576 87 554 |43 161 || 19 235 |45 490 || 78 684 || 44 582|57 757 |42 543 | 85 684 |68 560 || 43 328|80 809 | 66 981 |32 346|24 523 |45 350 | 67 386 |45 605 || 78 848|77 789 |55 169 |45 123|45 234 56 456 || 78 890 |71 256 |87 ADDITION OF SEVERAL COLUMNS IN ONE OPERATION. 107. In many cases, the addition of several columns at one operation will greatly expedite the accountants' work; but in long columns of solid figures, the addition by grouping, as above explained, is by far the most rapid and practical. The following will illustrate the primary work of adding several columns at ODCO : e d : 1 º Explanation.—Commence at the top and proceed by adding to the upper 71 number 47, first the tens of the next number below, then the units, and in like 62 manner continue till all are added, thus: 47 -- 10 – 57 –– 8 = 65 + 70 = 135 + . 1 = 136 -- 60 = 196 -- 2 = 198 + 90 = 288 + 4 = 292 + 20 = 312 + 6 = 318, the sum of the two columns. 318 54 soul E's PHILOSOPHIC PRACTICAL MATHEMATICS. º: By naming results only, which in practice should be done, we have 47, 57, 65, 135, 136, 196, 198, 288, 292, 312, 318. 2. Add : ; Explanation.—Commencing at the top, we have : 1587 . 1 3691 2 5691 3 5791 4 5796 4210 5 6796 6 7296 7 7376 8 7383 11593 9 11383 10 11583 ** 11593, the answer. These operations show the basis of adding several columns simultaneously, and though the method is too laborious to be of material advantage in long columns, it should be practiced until the mind can easily and readily retain large and vary- ing numerical results. In practice, we shorten the work very much by combining and adding whole amounts at once. Thus, in the second problem, by combining, as we would naturally do in practice, we would have in the two upper numbers 5796, then adding the hundreds' and thousands' figures (15) of the third number, we have 7296; then add the units' and tens’ figures of the third number, we have 7383, to which add the fourth number at once, and we have 11593, the correct result. To show clearly how to contract, and to practicalize the above system, we present the following combinations, which should be carefully studied. ADD THE FOLLOWING NUMBERS : (3) (4) (5) (6) (7) (8) (9) (10) (11) (12) (13) (14) 23 52 84 68 98 88 97 96 95 89 99 89 16 36 95 72 59 46 89 99 88 87 98 98 39 88 179 140 157 134 186 195 183 176 197 187 Explanation.—In the third, fourth and fifth problems, the sum of the units' figures being less than 10, the whole sum is instantly seen. In problems 6, 7, 8, 9, 10, 11, 12, 13 and 14, we instantly see that the sum of the units' figures is 10, or an excess of 10, and hence we know that the sum of the tens is to be increased by 1, and without thinking what the excess is, we write the result of the tens’ figures, which, according to the combi- nations, shown in tables 1 and 2, we know without adding, and while writing this result, we give an Instantaneous thought to the exact sum of the units' figures. ADD THE FOLLOWING NUMBERS : (15) (16) (17) (18) (19) (20) (21) (22) (23) (24) (25) (26) 74 86 88 68 126 342 485 1627 178 526 395 2653 98 95 97 91 75 92 304 211 201 308 296 1210 172 1st 185 159 201 434 739 138s 379 s54 691 3863 Explanation.—In these problems, from 15 to 20 inclusive, we show how to facilitate and expe- dite the work when one number approximates 100. Thus, in problem 15, we first mentally add 100 to the 74 which makes 174, and then, because the 100 added is 2 more than the 98, which should have been added, we mentally deduct 2 from 174, and write 172, the correct sum. In problems 16, 17, 18, 19, and 20, we mentally add to each upper number, 100, and then from the several sums thus produced, we mentally deduct, respectively, 5, 3, 9, 25, and 8. In problems 21 and 22, we first mentally add, respectively, 300 and 200, and then to the respective sums thus obtained, we mentally add 4 and 11. The other numbers are added in like manner. NOTE.-Combinations similar to the foregoing problems, 3 to 26, must be practiced until the student can easily and rapidly perform the work, otherwise, proficiency in addition and multipli- cation cannot be attained. Rapid work in simultaneous multiplication requires the instantaneous sum of two pairs of numbers, and hence, again, the importance of proficiency in this work. NOTE.—In using numbers mentally or orally, the calculator should always think or speak the result in pairs, thus, 2476 and 158, should be read mentally or orally, 24, 76, and 1, 58. DRILL PROBLEMS. 55 DRILL PROBLEMS. 108. Add the following columns at one operation, and drill on the Same, at different sittings or lessons, until you achieve rapidity and accuracy: 87 146 184 64 85 240 264 429 793 843 798 96 95 72 98 132 193 187 367 241 427 524 64 27 92 58 42 321 284 316 475 762 27 81 36 21 88 406 343 509 666 533 18 45 47 76 98 463 220 704 988 487 74 47 362 432 567 4807 463 65342 123456 21 88 406 234 405 621 247 37286 906342 36 92 29 444 219 1282 5002 22406 421362 48 81 391 843 924 1096 1603 1321 187367 19 67 132 777 633 2761 4120 74350 8899 27 41 254 96 111 7932 2031 64275 364819 86 78 345 123 456 101 501 11111 300405 32 72 789 202 404 8763 1004 24359 87.1460 59 33 876 463 99 5832 803 606 463220 - 63 12 101 508 399 1847 160 10009 768 58 92 999 387 555 1234 4512 56789 102938 ADDITION BY ONE OF TEIE PROPERTIES OF 9 109. 1. What is the sum of 5 lines of numbers, the first being 467 ? Ans. 2465° OPERATION: 467 1st line. 382 26 % 815 3d 44 617 difference between 26 line and 9. 184 {{ {{ 3d line and 9. 2465 Ans. Explanation.-In all problems of this character, with any number of odd lines, the answer may be produced as soon as the FIRST line of numbers is given, by writing the first line minus the num- ber of 9's to be produced, and then prefixing the number of 98 to this result. Then, when the other lines are set, to insure the result, every two lines must make 9, or, in other words, one-half of the other lines must be such numbers as will, when added to the remaining half alternately, produce 9 in each column of the two lines added. In this problem, we subtract 2 from the first line 467, and write 465, to which we prefix the 2 and thus produce the answer 2465. It should be borne in mind that 5 lines give two 9's, 7 lines three 9's, 9 lines four 9’s, etc. 2. What is the sum of 9 lines of numbers, the first being 10644? Ans. 410640. OPERATION: 10644. 1st line. 23456 2.d 4% 13482 3d 44 96.780 4th {{ 25621 5th €4 76543 difference between 26 line and 9. 865.17 44 {{ 3d “ and 9. 03219 4% {{ 4th “ and 9. 74378 4% 4% 5th 4 and 9. What is the sum of 7 lines of numbers, the first being 86021? Ans. 386018. 56 soul E's PHILOSOPHIC PRACTICAL MATHEMATICs. A. TEIE VICENARY SYSTEM OF ADDITION 110. The Vicenary System combines the grouping method of adding, with a method of recording the 20's on the joints of the fingers as fast as they are pro- duced by the operation of adding. NotE.—This system was elucidated and published by the author in 1873. All expert calculators admit that the most rapid system of adding long col- umns of numbers is by grouping two, three or more numbers and mentally naming the result of the group. Btut such a system requires a constant strain on the mind to retain the large and varying results and to make the several additions of the different groups, that with many it is wholly impracticable. To avoid this mental strain, several different methods of operation have been suggested by different calculators, but all have had some demerit that has rendered them more or less objectionable. With a view, therefore, to obviate the objections and difficulties of the usual methods and render the work simple, comprehensible and practical, we present a new system, which, from the character of the work, we name the Vicenary System. The leading pecu- liarity of this system consists in recording on the different joints of the fingers and thumb of the left hand, the 20's as soon as they are produced, and thus freeing the mind from all effort to retain large results, or of making the rather difficult addi- tions resulting from the several groupings. THE WICENARY SYSTEM ELUCIDATED. 111. We have before stated that the leading characteristic or peculiarity of the Vicenary Sys. tem of addition consists in recording the 20's as Soon as produced, on the joints of the fingers of the left hand; and in order to render this part of the operation clear, we present a cut of a hand with the different joints of the fingers numbered, so as to represent the values that we record on them in adding. The manner of making the record of the 20's on these joints is, by simply placing the end of the thumb on the finger, or the end of the first finger on the thumb, at such joint as represents the value to be recorded. The manner of making the record, and the value of the several joints, should be well understood before the operation of adding is commenced. The addition tables, on preceding pages, should also be well under- stood. In making the addition, we first mentally group enough figures to produce a result not less than 10, and then to this result add the result of enough other fig- ures to produce a second result not less than 20; then having 20 or an excess of 20, we record the 20 on the hand, by placing the end of the thumb on the first joint of the first finger which represents the value of 20, then we add the excess, if any, to the next group, and continue to produce and record the 20's until the column is added. If it is desired, enough figures to produce a result in excess of 20, may be grouped at once, and the record made and the eaccess carried on as above eaplained. THE VICENARY SYSTEM OF ADDITION, 57 112. To better elucidate the vicenary system, we present the following problems and explanations: 97 76 89 64 38 36 92 74 53 45 27 84 65 36 87. 1045 (1) Add the following numbers: Explanation.—Commencing at the bottom of the units' column, we first mentally see 13, (7,6), to which we add 9 (5,4), which makes 22; we then record the 20 by placing the end of the thumb on the first joint of the first finger. Next, we see 14 iTººl (7, 5, and 2 in excess of 20), to which we add 9 || || º i_-_º (3, 4, 2), and obtain 23; then we record the 20 by */ lº, / / ! ; ; ; 3. passing the end of the thumb to the end of the W *. f. Agi #_i_3 º A * § § |g | gets fº t> gº # ; second finger. We then see 17 (6, 8, and 3 in ex- cess of 20), to which we add 4 and obtain, 21; then we record the 20 by passing the end of the thumb to the end of the third finger. Next we see 16 (9, 6, and 1 in excess of 20), tº which we add 9, (7,2), and obtain 25, the 20 of which we record by passing the end of the thumb to the first joint of the fourth finger, and the 5 we write in the units' place of the answer. We now have the first column added, and by inspecting the position of the thumb and fingers of the left hand, we find 80 recorded, which, with the 5, the last excess of 20, make 85, the sum of the column. Then, to add the second, or tens' column, we first mentally see 16 (8 and the 8 tens from the units' column) plus 9 (3, 6) = 25; wº then record the 20 on the first joint of the first finger, as above directed. Next we see 15 (8, 2, and the 5 in excess of 20), plus 9 (4, 5), - 24, then recording the 20, by moving,the end of the thumb to the first joint of the sécond finger, we next see 20 (7, 9, and the 4 in excess), which we record by passing the end of the thumb to the first joint of the third finger." Then we see 12 (3, 3, 6), plus 8 = 20, this we record by placing the end of the thumb on the first joint of the fourth finger. Then we see 16 (7,9), plus 8 = 24; the 20 We record by placing the end of the first finger on the first joint of the thumb, and as the addition of the col- umn is now completed, we inspect the position of the fingers and thumb of the recording hand and from the position last above named, find the record to be 100, which, with the 4 last excess, makes 104, the result of the column. This 104 we write in the total result, and obtain 1045, as the Bum of all the numbers. - 5 ! | 1 6 2 4. 01.61.| ||; 2 7 2 1 2 8 ſ 3 (3) Add the following numbers: 2 1. Explanation.—Having made the explanation of the preceding example very full, we will therefore, in this, omit some of the details. To make the operation clearer, we have linked together, with brackets, the numbers used to produce the various intermediate results. Commencing at the bottom of the column, we pro- ceed thus: 11, 14, 25; then recording the 20 and adding the 5 excess to the next 2 1 group, we have 15, 7, 22; then, recording the 20 and adding the 2 excess we have 15, 13, 28; then, recording the 20 and adding the 8 excess to the next group, we have 23; then, recording the 20 and adding the 3 excess to the next group, we have 18, 10, 28; then, recording the 20 and adding the 8 excess to the next group, we have 21; then, recording the 20 and adding the 1 excess to the next group, we 23 have 10, 11, 21; then recording the 20 and adding the 1 excess to the next group, 28 we have 16, 11, 27; then recording the 20 and adding the 7 excess to the next group, we have 17, 7, 24; then recording the 20 and adding the 4 excess to the next group, we have 21; then recording the 20 and adding the 1 excess to the next group, *sº 2 1 6 we have 16. This completes the addition, and by inspecting the recording hand, we 25 find the end of the first finger on the second joint of the thumb, which shows that 200 has been recorded, which, with the last obtained 16 makes 216, the total sum of the column. 58 SOULE's PHILOSOPHIC PRACTICAL MATHEMATICs. * DRILL PROBLEMS, Add, separately, the following columns of numbers: (3) (4) (5) }}º 1 s 6 4. 11 8 8 8 3 ) 7 ) 24 : 13 : #: 4 8 7 5 g ; : }} 7 #} º '7 ) 23 7 3 2 ) 4 §§ § #} 3 ) 11 s 28 3 ) 1 9 5 : | : {. 9 2 2 9 9 ) 17 9 ) 7 9 § { | 9 7 3 7 3 9 1. Explanation.—In the third example, we added in the same manner as in the preceding example. In the fourth and fifth examples, in stating the results of the grouped figures, and also in the result in excess of 20, we have set only the excess figure, or figures, which, in practice, are the only figures that should be mentally noticed. In the fourth example, we grouped differently from the bracketing of the third example in order to show that no particular form of grouping is necessary, and that the more figures grouped, the more rapidly the addition is made. In the fifth example, we find five figures alike, and hence, by multiplying, we instantly see 45; we then record the 40, and continue in the usual manner, until we come to three figures that are alike; we then multiply them, and to the product, add the 3 excess, and 4, the last figure in the column, and produce 11 in excess of 20. CONTRACTING METEIODS. 113. To be observed when adding by the Vicenary System: 1. In mentally naming the results of the various groups, the unit figure only should be named; thus, in the addition of the fourth example, instead of mentally naming or thinking 17, 11, etc., name or think only 7, 1, etc. Remember, in prac- tice, these results are never set on paper. It is here done to elucidate the work. 2. In adding the two results of the grouped figures to produce an excess of 20, add only the unit figures, and only mentally name the excess of 20. These points were elucidated in adding the fourth and fifth examples. 3. Whenever the same figure occurs connectedly several times, the sum should be obtained by multiplying instead of adding. This principle was elucidated in example 5. DRILL PROBLEMS, 59 Add the following columns of numbers by the Vicenary Method until you acquire rapidity and accuracy: (6) (7) 63 25 23 93 83 72 43 72 11 63 15 63 00 92 31 41 57 23 35 21 00 51 24 33 49 75 17 35 23 87 15 23 95 64 53 68 72 19 47 x== eighty-seven thousand five hundred sixty-two. 2. Find the sum of 4888765, 4100000808707 and 222222333333444444. 68 (8) 24 63 75 82 29 76 84 57 38' 92 14 68 74 99 76 57 81 92 25 63 48 78 97 62 54 38 91 56 72 85 39 24 76 58 49 62 14 17 27 19 ( 9 ) * ( 1 0 ) *- (11) 49 83 27 51 43 02 13 38 55 28 33 24 81 91 88 92 02 48 65 08 62 64 87 48 55 82 78 57 20 31 42 53 84 43 90 18 22 37 40 50 (12) 39 77 62 35 56 50 46 34 40 19 58 49 68 62 93 82 55 23 21 30 74 28 84 25 62 49 89 98 28 33 49 87 62 49 77 83 92 11 13 14 ( ) (14) 7563 28325 461523 6393 21783 151672 3243 72 MISCELLANEOUS PROBLEMS IN ADDITION. 114. 1. Add seven million four thousand ninety-six, and three hundred Ans. 7391658. 92238, 1600084, 8888888, 99999999999, (15) 1963 2345 1215 1872 7312 910 2311 617 99 313 4632 516 3313 88 200 3915 617 3129 28 319 4615 313 3239 272 99 333 8917 222 6717 516 817 999 1322 400 620 189 4002 841 264. 974. 311 3263 70015 8063 1000 56792 107331 2441 3457 67423 87635 297521 80000 6751 324 233 49 275 3617 81535 71.23 60737 3015 40123 891.395 206 1482 5014 382 1571 932.176 35489 Ans. 2222265.33349723125. 6O soul.E's PHILOSOPHIC PRACTICAL MATHEMATICs. 4× 3. A merchant bought at one time 768 barrels flour for $4032; at another, 273 barrels for $1774.50; and at another, 410 barrels for $2921.25. How many bar. rels did he buy and what was the total cost? Ans. 1451 bbls. $8727.75 cost. 4. From New Orleans to MacDonoughville is 1 mile; thence to Algiers,1; thence to Old Spanish Fort St. Leon, 16; thence to Poverty Point, 18; thence to Point Celeste, 7; thence to Pointe-à-la-Hache, 3; thence to Sixty Mile Point, 15; thence to Quarantine, 9; thence to Bolivar Point,3; thence to Forts St. Phlip and Jackson, 2; thence to The Jump, 10; thence to Head of Passes, 11; thence to Pilot Town, 10; thence to Port Eads, 1. How many miles from New Orleans to Port Eads? Ans. 107 miles. 5. From New Orleans to Algiers Depot is 1 mile; thence to Gretna, 3; thence to Jefferson, 9; thence to St. Charles, 6; thence to Boutte, 6; thence to Bayou des Alemendes, 8; thence to Raceland,8; thence to Ewing's, 6; thence to Lafourche, 6; thence to Terrebonne, 3; thence to Chucahoula, 6; thence to Tigerville, 5; thence to L'Ourse, 4; thence to Bayou Boeuf, 3; thence to Ramos, 3, thence to Morgan City, 4; thence to Galveston, 240. How many miles to Galveston 3 Ans. 321 miles. 6. During the fiscal year ending September 1, 1893, the sugar production? in the different sugar producing parishes of Louisiana, Was as follows: PARISHES IN POUNDS. | PARISHES. IN POUNDS. PARISHES. IN POUNDS. Ascension . . . . . . . . . , 39,754,703 || Lafourche. . . . . . . . . . 39,875,463 | St. John the Baptist 21,196,526 Assumption . . . . . . . . 49,939,541 Orleans . . . . . . . . . . . . 2,626,000 || St. Landry......... 4,282,245 Avoyelles. . . . . . . . . . . 2,574,891 |Plaquemines. . . . . . . . 18,943,010 | St. Martin. . . . . . . . . . 7,024,291 East Baton Rouge. . 4,324,213 || Pointe Coupee. . . . . . 7,058,314 || St. Mary ..... tº e o 'º e e 79,641,726 Iberia. . . . . . . . . . . . . . 18,247,813 || Rapides . . . . . . . . . . . . 4,567,404 || Terrebonne. . . . . . . . . 68,456,522 Iberville . . . . . . . . . . . 38,755,524 || St. Bernard. . . . . . . . . 2,666,908 || Vermilion... . . . . . . . 3,452,639 Jefferson . . . . . . . . . . . 6,661,494 || St. Charles. . . . . . . . . 13,693,265 | West Baton Rouge... 17,424,725 Lafayette. . . . . . . . . . . 323,438 St. James . . . . . . . . . . 30,554,329 || Other small parishes 2,640,000 How mamy pounds were produced in the State during the fiscal year. Ans. 484,684,984 pounds. 7. During the fiscal year ending September 1, 1893, the different COuntries of the world produced the following quantities of Sugar : COUCJINT* tº R.I.E.H.S. CANE SUGAR. IN TONS. CANE SUGAR. IN TONS. BEET SUGAR. IN TONS. Cuba. . . . . . . . . . . . . . . . . 850,000 || Java. . . . . . . . . . . . . . . . . 430,000 || Germany . . . . . . . . . . . . 1,225,000 Porto Rico. . . . . . . . . . . 70,000 || British India......... 50,000 || Austria-Hungary..... 780,000 Trinidad. . . . . . . . . . . . . 50,000 | Brazil. . . . . . . . . . . . . . . . 200,000 || France. . . . . . . . . . . . . . . 590,000 Barbados . . . . . . . . . . . . 70,000 | Natal . . . . . . . . . . . . . . . . ,000 || Russia.... . . . . . . . . . . . . 460,000 Jamaica. . . . . . . . . . . . . . 28,000 | Fiji. . . . . . . . . . . . . . . . . . 23,000 || Belgium . . . . . . . . . . . . . 180,000 Antigua and St. Kitts, 30,000 || Phillipine Tslands. ... 225,000 | Holland . . . . . . . . . . . . . 70,000 Martinique. . . . . . . . . . . 30,000 || Manila, Cebu, Hoilo... 250,000 | United States, short Guadeloupe... . . . . . . . . 50,000 | United States. . . . . . . . 246,094 tons . . . . . . . . . . . . . . . 13,284 Lesser Antilles. . . . . . . 27,000 | Peru. . . . . . . . . . . . . . . . 40,000 | Other countries . . . . . . 97,000 Demerara... . . . . . . . . . . 110,000 | Egypt . . . . . . . . . . . . . . . 65,000 Reunion . . . . . . . . . . . . . 35,000 | Sandwich Islands . . . . 125,000 Mauritius . . . . . . . . . . . . 75,000 How many tons each, of cane and beet sugar, were produced in the world during the fiscal year, and how many tons of both kinds 3 Ans. 3,087,094 tons of cane sugar, 3,415,284 tons of beet sugar, and 6,502,378 tons of both. * MISCELLANEOUS PROBLEMS IN ADDITION. 61 8. From New Orleans to the mouth of Red River, is 210 miles; thence to Black River, 40; thence to Alexandria, 110; thence to Grand Ecore, 120; thence to Grand Bayou, 95; thence to New Hope, 60; thence to Waterloo, 30 ; thence to Shreveport, 35. How many miles to Shreveport by river ? Ans. 700, miles. 9. From New Orleans to Carrollton, is 7 miles; thence to Donaldsonville, 71; thence to Plaquemines, 32; thence to Baton Rouge, 20; thence to Port Hudson, 23; thence to Bayou Sara, 12; thence to mouth of Red River, 40; thence to Nat- chez, 72; thence to Rodney, 45; thence to Grand Gulf, 18; thence to Vicksburg, 61; thence to the Louisiana Line, 97; thence to Helena, 230; thence to Columbus, 329; thence to Cairo, 20; thence to Cape Girardeau, 50; thence to St. Louis, 151. How many miles to St. Louis by river. Ans. 1278 miles. 10. According to the most reliable information, the following figures show the area, in Square miles, and the population of the different countries of the world: Square Miles. Population. Square Miles. Fopulation. North America. . . . . . . . . 9,349,741 88,005,695 || Africa. . . . . . . . . . . . . . . . . . 11,514,985 168,497,091 South America ........ 6,887,794 33,565,882 || Australasia and Poly- Europe. . . . . . . . . . . . . . . . . 3,942,530 360,580,788 nesia. . . . . . . . . . . . . . . . . 3,456,103 5,683,968 Asia, including Malaysia 16,956,284 823,155,251 | South Polar Regions ... 253,678 . . . . . . . . . What is the entire area and the whole population of the world? Ans. 52,361,115 square miles area, 1,479,488,675 population. . 11. Bought at one time 343 yards of calico and 132 yards of silk; at another, 104 yards of calico and 24 yards of silk; at another, 96 yards of calico and 48 yards of silk. How many yards of each kind did I buy? Ans, calico, 543; silk, 204. 12. Newman, Soulé, and Lynch, form a copartnership; Newman invests $5400, Soulé, $5000, and Lynch, $1000 more than both Newman and Soulé. What is the capital of the firm 3 13. The cotton crop of the Southern States, from 1885 to Sept. 1, 1893, was as follows: º dº 1885-6. 6,550,215 bales. . . . . . . . . . . . . . New Orleans received 1,764,013 bales. 1886-7. 6,513,263 “ . . . . . . . . . . . . . . 1,762,798 “ 1887-8. 7,017,707 “ ..... .* * * * * * * * * 1,780,375 “ 1888-9. 6,935,082 “ . . . . . . . . . . . . . . 1,697,376 “ 1889-0. 7,311,322 “ . . . . . . . . . . . . . . 1,973,571 “ 1890-1. 8,652,597 “ .............. 2,077,744 “ 1891-2. 9,035,379 “ .............. 2,503,251 “ 1892-3. 6,700,365 “ . . . . . . . . . . . . . . 1.602,079 “ How many bales were produced in the eight years, and how many of them did New Orleans receive? Ans. 58,715,930 bales. N. O. rec'd 15,161,207 bales. 14. The Mayor of the City of New Orleans, receives a yearly salary of $3500; the treasurer, $3500; the Commissioner of Public Works, $3500; the Comptroller, $3500; the Commissioner of Police and of Public Buildings, $3000; the City Attorney, $3500; the City Surveyor, $2500; the City Superintendent of Public Schools, $3000; the City Superintendent of Fire Alarm Telegraph, $1800. What are the total salaries of these officers ? Ans. $27800. 15. The Governor of Louisiana, receives a salary of $4000 per annum; the 62 soul E's PHILOSOPHIC PRACTICAL MATHEMATICS. X. Lieutenant Governor, $8 per day during the 60 days' session of the Legislature; the Secretary of State receives $1800 per annum; the Auditor of Accounts, $2500; the State Treasurer, $2000; the Attorney General, $3000; the five Justices of the Supreme Court, $5000 each; the two Judges of Criminal Court in N. O., $4000 each; the two Judges of the Court of Appeals in N. O., $4000 each; the five Judges of the Civil District Court, Parish of Orleans, $4000 each; the State Super- intendent of Education, $2000. What is the total salary of all these officers, includ- ing the per diem of the Lieutenant Governor ? Ans. $76780. 16. A father gave to his son seven thousand eight hundred dollars; to his daughter, nineteen hundred and fifty dollars more than he gave to his son; and to his wife, three thousand five hundred more than he gave to both the son and the daughter. What sum did he give away ? Ans. $38600. 17. The length of the Mississippi River is 4200 miles; of the Nile, 4000; Amazon, 3750; Yenisei, 3400; Obi, 3000; Yang-tse-Kiang, 3320; Niger, 3000; Lena, 2700; Amoor, 2650; Volga, 2000; Ganges, 1600; Brahmapootra, 2300; La Plata, 2300; Mackenzie, 2300; St. Lawrence, 2000; Saskatchewan, 1900; Orinoco, 1550; Columbia, 1020; Colorado, 600; Yukon, 1600; Red River, 1500. What is the combined length of all? AnS. 50690. 18. The following statement compiled from the New Orleans Cotton Exchange Report of September 1st, 1893, shows the consumption of cotton by the Southern States, from Sept. 1, 1892, to Sept. 1, 1893: * No. of Mills | Looms Spindles Bales Pounds Average ST A T E S . in in in Weight Operation. Operation. Operation. Consumed. Consumed. Per Bale. Alabama. . . . . . . . . . . . . . . . . . . . . . . 22 2,875 159,912 50,222 24,449,891 486.83 Arkansas. . . . . . . . . . . . . . . . . . . . . . . 1 108 3,000 765 382,500 500.00 Georgia. . . . . . . . . . . . . . . . . . . . . . . . . 57 11,243 || 511,336 | 184,403 || 86,931,608 || 471.45 Kentucky . . . . . . . . . . . . . . . . . . . . . . 6 688 50,543 || 17,004 || 8,339,968 || 490.47 Louisiana . . . . . . . . . . . . . . . . . . . . . . 4 1,518 52,800 || 14,408 || 6,862,896 || 476.32 Mississippi. . . . . . . . . . . . . . . . . . . . . 7 1,812 53,692 | 15,955 7,479,433 468.78 Missouri. . . . . . . . . . . . . . . . . . . . . . . 1 240 10,500 1,931 942,115 487.89 North Carolina. . . . . . . . . . . . . . . . . 122 11,108 || 524,770 | 181,996 || 82,913,899 || 455.50 South Carolina. . . . . . . . . . . . . . . . . 53 13,537 525,457 203,533 94,013,304 461.90 Tennessee . . . . . . . . . . . . . . . . . . . . . 26 2,621 128,215 33,443 16,239,159 485.58 Texas . . . . . . . . . . . . . . . . . . . . . . . . . . 7 1,362 44,436 12,396 6,478,066 522.59 Virginia. . . . . . . . . . . . . . . . . . . . . . . . 12 3,109 106,486 27,792 12,961,202 466.36 How many each, of mills, looms and spindles were in operation, and how many each, of bales and pounds of cotton were consumed during the year by the twelve states? Ans. 318 mills, 50,221 looms, 2,171,147 spindles. 743,848 bales, 347,994,041 pounds. 19. A man made a will and bequeathed $4000 to each of three sons; and to each of two daughters, $500 more than to each son; and to his wife, $1000 more than to a son and a daughter together; and the remainder of his estate, which was $1000 more than he bequeathed to his wife and children, he left to benevolent institutions. What was the sum received by the wife and by each daughter; what was the sum given to benevolent institutions, and what was the amount of the whole estate? Ans. Wife, $9500; each daughter, $4500; Benevolent Institutions, $31500; whole estate $62000. 64. 65. 66. 67. 68. 69. 70. 71. 72. 73. 74. 75. 76. 77. 78. 79. SYNOPSIS FOR REVIEW, 117. Define the following words and phrases: Signs and Symbols. What are they used for ? Sign of Addition. Of Subtraction. Of Multiplication. Of Division. Of Equality. The Parenthesis and Winculum. Decimal Point. Sign of Ratio. Sign of Proportion. Sign of Involution. Sign of Evolution. Sign of Deduction. Interrogation Point. The Signs for First, Second, etc. Sign of the Comma. 80. 81. 82. 85. 86. Signs and Abbreviations in Busi- Il CSS, Addition. Sum, or Amount. Numerical Equation. Principle of Addition. 87, 88, and 89. The Alphabet of Num- 96. 99. 103. 104. 105. 107. 109. bers. Proof of Addition. Horizontal Addition. Adding Fractional Numbers, 4 or $. Adding Dollars and Cents. U. S. Monetary Units. Addition of Several Columns in- One Operation. Addition by one of the Proper- ties of 9. (63) *. subtraction. º---------------------N (DECREASI NG). *—e-º-Lº- 118. Subtraction is the process of finding the difference between two num. bers of the same kind. 119. The result obtained by subtraction is called the Difference, or Remainder. 120. The greater number is called the Minuend, which means a number to be decreased. - 121. The lesser number is called the Subtrahend, which means the number to be subtracted. 122. The Sign of Subtraction is a horizontal line —. It is read minus, and means less. When this sign is placed between two numbers, it indicates that the number after it is to be subtracted from the number before it. Thus, 8–3 is read, 8 minus 3 = 5, or 8 less 3 = 5. 123. The Principle governing all problems in subtraction is, that like num- bers and units of the same order only, can be subtracted, the one from the other. 124. To Prove the operation of subtraction, add the remainder to the sub- subtrahend; if the sum is equal to the minuend, the work is correct. 125. Subtraction is the reverse of addition, and by it we find what number added to the lesser of two numbers will produce the greater. 126. SUBTRACTION TABLE. 2 — 2 = 0 || 3 — 3 = 0 || 4 — 4 = 0 || 5 – 5 = 0 || 6 – 6 = 0 || 7 – 7 = 0 || 8 — 8 = 0 || 9 — 9 = 0 3 — 2 = 1 || 4 — 3 = 1 || 5 — 4 = 1 || 6 — 5 = 1 || 7 – 6 = 1 || 8 — 7 = 1 || 9 — 8 = 1 || 10 — 9 = 1 4 – 2 = 2 || 5 — 3 = 2 || 6 — 4 = 2 || 7 — 5 = 2 | 8 — 6 = 2 | 9 – 7 = 2 | 10 — 8 = 2 | 11 — 9 = 2 5 — 2 = 3 || 6 — 3 = 3 || 7 — 4 = 3 || 8 — 5 = 3 || 9 – 6 = 3 || 10 – 7 = 3 | 11 — 8 = 3 | 12 — 9 = 3 6 — 2 = 4 || 7 — 3 = 4 || 8 — 4 = 4 || 9 — 5 = 4 || 10 – 6 = 4 || 11 — 7 = 4 || 12 – 8 = 4 || 13 — 9 = 4 7 — 2 = 5 || 8 — 3 = 5 9 — 4 = 5 | 10 — 5 = 5 | 11 – 6 = 5 | 12 — 7 = 5 || 13 — 8 = 5 || 14 — 9 = 5 8 — 2 = 6 || 9 — 3 = 6 || 10 — 4 = 6 || 11 — 5 = 6 | 12 – 6 = 6 || 13 — 7 = 6 || 14 – 8 = 6 | 15 – 9 = 6 9 — 2 = 7 || 10 — 3 = 7 || 11 — 4 = 7 | 12 — 5 = 7 || 13 — 6 = 7 || 14 – 7 = 7 || 15 – 8 = 7 | 16 — 9 = 7 10 — 2 = 8 || 11 — 3 = 8 || 12 — 4 = 8 || 13 — 5 = 8 || 14 – 6 = 8 || 15 — 7 = 8 || 16 — 8 = 8 || 17 — 9 = 8 11 — 2 = 9 || 12 – 3 = 9 || 13 — 4 = 9 || 14 — 5 = 9 || 15 – 6 = 9 | 16 – 7 = 9 || 17 – 8 = 9 | 18 – 9 = 9 127. ORAL EXERCISES. 1. Commence at 50 and orally count to 0 by continually subtracting 1, thus: 49, 48, 47, 46, 45, etc. 2. Commence at 50 and orally count to 0 by continually subtracting 2, thus: 48, 46, 44, 42, etc. : 3. Commence at 51 and orally count to 0 by successively subtracting 3, thus: 47, 44, 41, 38, etc. (64) Y ORAL EXERCISES. 65 4. In like manner, commence at 50 and subtract respectively 4, 5, 6, 7, 8, 9, 10, until you produce 1, thus: 46, 41, 35, 28, etc. 5. Commence at 50 and subtract alternately 2 and 5 until you produce 1, thus: 48, 43, 41, 36, etc. 6. Commence at 50 and subtract alternately 8 and 3 until you produce 6, thus: 42, 39, 31, etc. 7. 60 — 2 + 12 + 6 — 2 — 2 — 3 + 5 – 8 + 11 – 6 + 9 — 3 equals & 8. 75 + 5 — 20 — 20 — 5 + 8 + 7 – 9 + 6 – 5 + 4 — 3 + 2 – 45 = } 128. To subtract one number from another, when any figure of the subtrahend is less than the corresponding figure of the minuend. 1. From 897 Subtract 641. OPERATION. 897 641 Explanation.—First write the numbers with the lesser wºnder or 641 897 over the greater, so that units of the same order stand in the same column. Then commence with the units’ figure and Subtract each -º- o-e order separately, thus: 1 from 7 leaves 6; 4 from 9 leaves 5; 6 from 8 256 256 leaves 2. By this work we obtain the difference, or remainder, 256. Subtract the following : 278 843 384 978 425 9876 6203 499 521 762 655 679 3456 7021 --as -- -º- - smºs-- 129. Demonstration—to prove that the difference between two numbers is the same as the difference between the two numbers when equally increased. OPERATION. 6 6 + 3 = 9 5 5 + 2 = 7 23 23 + 10 = 33 4 4 + 3 = 7 2 2 + 2 = 4 11 11 —H 10 = 21 – The difference is — T The difference is T - The difference i ºmme 2 the same. 2 3 the same. 3 12 * . IS 12 Ea:planation.—Here we see that the difference between 6 and 4 is 2, and that the difference between 6 and 4 equally increased by 3, is also 2. The operations with 5 and 2, and 23 and 11 show similar results. Hence the law that the difference between two numbers is the same as the difference between the two numbers when equally increased. The application of this numerical law is shown in the following problem, and it governs all operations in subtraction, where the subtrahend figure of any order exceeds the minuend figure of the same order. 130. To subtract one number from another, when any figure of the subtrahend is greater than the corresponding figure of the minuend. 1. From 4173 Subtract 2346. FIRST OPERATION. #### #### ÉÉÉÉ ÉÉÉÉ Minuend - - - 417 3 Subtrahend - - - 2346 subtrahend - - 2346 ° Minuend - - - - 4173 Difference - - 1827 1827 Explanation.—Having written the numbers with the lesser under or over the greater, so that units of the same order stand in the same column, we commence at the right hand to perform the operation. º © We first observe that 6 units cannot be taken from 3 units. We therefore, according to the 66 SOULE's PHILOSOPHIC PRACTICAL MATHEMATICS. foregoing numerical law, mentally increase the 3 units by 10 units, making 13 units; from this, we subtract the 6 units and set the remainder, 7 units, in the line of difference. Then, as we added 10 units to the minuend, we now. to compensate therefor, mentally add 1 ten, the equivalent of 10 units }. the tens’ figure of the subtrahend, and say 5 from 7 leaves 2, which we write in the line of dif- €I'ellC6. We next observe that 3 hundreds cannot be taken from 1 hundred ; and, therefore, for reasons above given, we mentally add 10 hundreds to the 1 hundred, making 11 hundreds, and then say 3 from 11 leaves 8. Then having added 10 hundreds to the hundreds' figure of the minuend, we now men- tally add 1 thousand, the equivalent of the 10 hundreds, to the thousands' figure of the subtrahend, and say 3 from 4 leaves 1. This completes the operation and gives 1827 as the difference between the two numbers. 131. TELE BORFOWING METELOD IMPROPER. The foregoing is the only rational and true method of subtraction, and it should be universally substituted for the absurd and unmathematical “borrowing ” method, which is given by nearly all the authors of arithmetics now before the public. In the language of Shakespeare: “Neither a borrower nor a lender be.” Subtract the following: 7843 582 6213 98761 1234 1407.28 34007 5907 427 7302 5496 87.601 60893 65100 132. TO SUBTRACT BY ADDITION. 1. From 4173 Subtract 2346. OPERATION, 4173 2346 Explanation.—We will here perform the operation by addition, 2346 ° 4173 which consists simply in adding to the subtrahend such a number as will make it equal to the minuend. Thus, commencing with the units' figure of the subtrahend, or smaller number, we say, 6 and 7 1827 1827 make 13; and write the 7 in the units' place of the difference; then carrying 1, we Say 5 and 2 make 7, and write the 2 in the tens’ column of the difference; then we say, 3 and 8 make 11, and write the 8 in the third column, or hundreds' place of the difference; then carrying 1, we say 3 and 1 make 4, and Write the 1 in the fourth place of the difference. This com- pletes the operation. 2. From 73245 Subtract 1228. OPERATION BY SUBTRACTION. *:::: Explanation.—Here we say 8 from 15 leaves 7; 3 from 4 leaves 1; 2 from 2 leaves 0; 1 from 3 leaves 2; 0 from 7 72017 leaves 7. OPERATION BY ADDITION. 73245 1228 Ea:planation.—Here we say 8 and 7 make 15; 3 and 1 make 4; 2 and 0 make 2; 1 and 2 make 3; 0 and 7 make 7. 72017 3. From 56802 Subtract 50531. OPERATION BY SUBTRACTION. 56802 50531 Explanation.—Here we say 1 from 2, 1; 3 from 10, 7; 6 * from 8, 2; 0 from 6, 6; 5 from 5,0; which being the last 6271 figure on the left, has no value, and hence is not written. OPERATION BY ADDITION. 56802 50531 Ea:planation.—Here we say 1 and 1 = 2; 3 and 7 = 10; tº-ºº-e 6 and 2 = 8; 0 and 6 = 6; 5 and 0 = 5. The naught is not 6271 written for the reason given in the first solution. Y GENERAL DIRECTIONS FOR SUBTRACTION. 67 Subtract the following by both methods: 428 284 2087 4892 78091 5647832 537 192 5301 1309 60742 4909271 GENERAL DIRECTIONS FOR SUBTRACTION. 133. From the foregoing elucidations, we derive the following general directions for subtraction: 1. Write the numbers so that units of the same order stand in the same column. 2. Begin at the units' figure and take successively each figure of the subtrahend from the figure of the corresponding order of the minuend. 3. When a figure of the subtrahend is greater than the figure of the same order in the minuend, add 10 to the minuend figure, perform the subtraction, and then add 1 —the equivalent of the 10—to the meat subtrahend figure. 4. To subtract by the addition process, proceed as elucidated in problem 1 above. PROBLEMS. 134. Write the following groups of numbers as they are here written, and Subtract the lesser from the greater of each group : (1) (2) (3) (4) (5) (6) (7) 467 1807 3842 607 3001 6879 12004 342 4251 1291 8013 1009 9640 17862 Find the difference between the following numbers: 8. 624 and 507. Ans. 117. 12. 754 and 621. Ans. 133. 9. 1148 and 5162. Ans. 4014. | T3. 41074089 and 1875429. AnS. 39198660. 10. 7013 and 904. * * 14. 9876543210 and 1234567sgo 11. 237 and 894. Ans. 657. Ans. 864.1975320. TO SUBTRACT DOLLARS AND CENTS. 135. What is the difference between $483 and $51.65% Ans. $431.35. OPERATION. $483.00 Explanation.—In all problems of this kind, we first write the 51.65 numbers in the same manner as when adding dollars and cents, with dollars under dollars, and cents under cents, so that units of the same $431.35 order stand in the same column, and the points in a vertical line. When there are no cents in the minuend, we fill the place of cents with naughtS. The operation of subtraction is performed with dollars and cents, the same as with other numbers. What is the difference between the numbers in each of the following groups: (1) (2) (3) (4) (5) (6) (7) $16.25 $8.00 $.75 $41.04 $10.50 $1.93 $408.76 9.38 3.75 .50 6.61 4.78 .47 291.85 *=mº-ºmmº 68 MATHEMATICS. * SOULE's PHILOSOPHIC PRACTICAL (8) $681.85 90.38 (14) $576.00 132.85 (9) (10) (11) $127.05 $248.00 $49.11 105.50 181.15 9.89 (15) (16) (17) $87.45 $ 482.68 $14.00 19.38 2246.10 68.25 (12) (13) $8529.09 $560784290.15 2798.17 91842531.83 (18) (19) $279107.16 $4764000.24 91020.48 7150831.50 BUSINESS CONTRACTIONS IN SUBTRACTING DOLLARS AND CENTS. 136. What is the “Change” (difference) to be given to the purchasers in the following transactions? NOTE.-Make the subtractions mentally, using pen or pencil, only to write the difference. Repeat the work until you acquire rapidity and accuracy. Amount of Bills Amount of Bills Amount of Bills Purchase. Presented. Purchase. Presented. Purchase. Presented. $ 1.45 $ 3.00 Ans. $ $ 2.75 $ 5.00 Ans $ $ 4.65 $ 10.00 Ans. $ .65 2.00 ($ $ 1.63 5.00 % $ 43.40 60.00 ($ $ 4.27 10.00 ($ $ 23.42 50.00 “ $ 72.20 100.00 ($ $ 1.85 5.00 ($ $ 3.85 10.00 (€ $ 1.15 5.00 % $ 12.20 15.00 ($ $ 38.25 50.00 “ $ 67.75 100.00 € $ 16.63 20.00 “ $ 14.10 20.00 (€ $ 25.10 40.00 % $ 7.18 10.00 “ $ 6.38 10.00 (€ $ Explanation.—In all cases like the above, first add (mentally) to the amount of purchase enough cents to make even dollars; then make the (mental) subtraction, and to the difference add (mentally) as many cents as you added to the purchase. Thus, in the first example, 55c. added to the $1.45, makes $2.00; and $2.00 from $3.00, leaves $1.00; and 55c. added to $1.00, makes $1.55 change. With a little practice, these mental additions and subtractions may be performed instantly and without any mental labor. See page 42, for table to qualify to see instantly the difference between any number and 100. NOTE.-In business practice, the book-keepers, salesmen and “change ’’ clerks, generally count out to the purchaser, when the amount of purchase embraces dollars and cents, sufficient change to make with the purchase even dollars, and then count even dollars to the amount of money presented by the purchaser. Thus, if the purchase were $2.85 and the money presented was $5.00, the party making change would count out 15 cents, thus making $3.00, and then count out $2.00 more, thus making, with the purchase, $5.00. EIORIZONTAL SUBTRACTION. 137. Accountants and business men frequently find it convenient to sub- tract horizontally, i.e., when the minuend and subtrahend are on the same line. The process or operation in this case, is the same as explained in the General Direc- tions, except that neither number is written under the other. Find the difference, by horizontal subtraction, between the following numbers: (1) (2) 84 – 52 = 32. 143 – 62 = 81. (5) (6) 87 — 39 = ? 643 – 214 = ? (4) (3) 4 721 – 95 = 626. 62 — 18 = ? - (7) (8) $486.48 – $91.35 = ? $327.15 – $168.08 = ? (11) $2476.40 and $6871.55 = } (9) (10) $386.14 — $169.87 = ? $480.23 and $291.45 = ? 㺠(13) $84936.14 and $57170.28 = ? $4238.64 and $56789.34 = ? +x SUBTRACTION BY THE METHOD OF COMMENCING ON THE LEFT 69 SUBTRACTION BY THE METHOD OF COMMENCING ON THE LEFT. 138. In the operations of addition and subtraction, we generally commence to add and to subtract on the right, but we have shown in adding several columns at once, that we may commence to add on the left; and in like manner, we may commence on the left in the operations of subtraction, and perform the work towards the right with the same rapidity and ease as in the ordinary way of work. It is often convenient, and sometimes necessary, to know almost instantly the difference between the hundredth, thousandth or millionth figures independent of the difference of the lesser orders of figures, and by this new method of work, this important result is obtained. The following example and the solution of the same will render the operation clear: From 43827 subtract 17495. OPERATION. Explanation.—We commence on the left at the 1, and because the adjoining 43827 subtrahend figure (7) is greater than the minuend figure of the same order, we 17495 add 1 to the first subtrahend figure and say 2 from 4 leaves 2, then because. We added 1 to the subtrahend figure of the fifth order, we now add its equiva- * lent, 10, to the minuend figure of the next lower order, and say 7 from 13 leaves 6; 26332 then passing to the next lower order, for the reason that the subtrahend figure of the next lower order is greater than the minuend figure of the same order, we add 1 to the 4 and say 5 from 8 leaves 3; then, because we added 1 to the subtrahend figure of the third order, we now add its equivalent, 10, to the minuend figure of the second order, and say 9 from 12 leaves 3; then 5 from 7 leaves 2, and thus we have the correct remainder. Working by this method, what is the difference between 389210 and 190768? Ans. 198442. 2487 and 568; 4784 and 4819; 9621 and 6876; 534672 and 270819. SUBTRACTION BY THE COMPLEMENT OF 10. 139. In many lines of business, banking especially, and in the operation of closing accounts, it is often convenient, and a saving of time, to be able to write the difference between the sums of several unadded debit and credit numbers, without first finding the amount of the respective debit and credit numbers of which the difference or balance is desired. 1. What is the difference or balance of the following account? Ans. 6790. I)R. C.R. Explanation.—Here we add the units' figures of the smaller, or the *==mºsºs credit side, and obtain 22; this we subtract from the next higher number 8472 || 3876 of tens (30) and obtain 8, which we add to the units' figures of the debit. 2168 || 4589 side and produce 40. The 0 we write as the first figure of the differ- ence. Here we observe that there were three tens in the credit side and 4607 || 764 four in the debit, and hence we have 1 ten to add to the tens of the debit, or to 596 || 1483 subtract from the tens of the credit. We will subtract it from the credit. 1659 We now add the second column and obtain, minus the 1 ten, 28; this taken 6790 Bal from 30, leaves 2, which, added to the second column of the debit, gives 29; * the 9 we write as the second figure of the balance. Here we observe that fººms. there is one more ten in the credit side than in the debit. This 1 ten we add to the third column of the credit and obtain 25, and this taken from 30 leaves 5, which we add to the third column of the debit and produce 27; the 7 is written as the third figure of the balance. We now observe that there is an excess of one ten in the credit, which We add to the fourth column of credit and obtain 9, this taken from 10 leaves 1, which is added to the fourth column of debit and produces 16; the 6 is written as the fourth figure of the difference; and as the tens are now equal in the debit and credit numbers, the difference or balance is complete. 7 O SOULE's PHILOSOPHIC PRACTICAL MATHEMATICs. 2. What is the balance of the following account ? DR. CR. DIRECTIONS: First. 7 -- 3 +8 = 18; 20 — 18 = 2; 2 + 1 = 3, the first *-*mºs figure of the balance — 2 tens excess on the debit. 948 || 4351 Second, 2+2+9+ 4 = 17; 20–17 = 3. ... 3 + 5=8, the second fig- ure of the balance — 2 tens excess on the debit. 793 Third, 2-H4+ 7+ 9 = 22. 30–22 = 8; 8+3= 11. This gives 1 as 427 the third figure of the balance, and as there were 3 tens in the debit and 1 ten in the credit, there are two tens excess in the debit, which, sub- tracted from the fourth figure of the credit, gives 2 as the fourth and final B alance, 2183 figure of the balance. ? 3. A depositor has a credit balance of $7206. He draws the following checks: $527, $1318, $98, and $1642. What is the credit balance? Ans. $3621. IXIRECTIONs. First. Add horizontally the units’ figures of the checks = 25; 30–25 = 5. Then 5 +6= 11. (2 tens excess on the checks). Šćcond. Add, horizontally the tens' figures of the checks and the 2 tens excess, = 18, 20–18 = 2. Then 2+0=2, the second figure of the balance. (2 tens excess in the check numbers). In the same manner, proceed with the remaining orders. 4. What is the debit balance of the following account” DR. CR. Directions. First 10–6–4 ; 4 + 2 + 5 = 11. 1375 || 356 Second. 10–5 = 5; 5 + 9 +7= 21. 8692 Third. (3–1 =2) 10–2 = 8; 8 + 3 + 6 = 17. ºmºsºm- | 9711 Balance. Fourth, 8-1-9. 5. Find the difference between the following numbers by the same process 38071 and 934068. Ea:planation. 9 and 8 = 17; 3 and 6 = 9; 9 and 0 = 9; 1 and 4 = 5; 6 and 3 =9; 9–1 = 8. 6. Find the balance of the following accounts: IDR. . CR. • DR. C.R. DR. CR. $ 482.45 $345.36 571 | 6416 4123.14 5421.45 1243.91 || 92.85 899 342.37 562.87 223 1691.52 7. Find the balance of the following accounts: Rohlman, Levy & Hiller. 1891 1891 Jan. 1 To Merchandise. . . . . . . . 4 || 184|50 Jan. 15|By Cash. . . . . . . . . . . . . . . 21 || 7500 Feb. 18 || “. “. . . . . . . . 10 || 75|65 Feb. 21 | “ “ . . . . . . . . . . . . . . . 28 || 225/55 March 29 || “ “. . . . . . . . 46 || 212||13 April 17 | “ “ . . . . . . . . . . . . . . . 40 || 65|25 April 4 || “. “ . . . . . . . 54 || 478|14 May 1 | Balance. . . . . . . . . . . . . . Bolton, Ricks & Schwamacher. 1894 1894 Aug. 3 |S. B. . . . . . . . . . . . . . . . . . . 14 ||1468|15 July 6 II. B. . . . . . . . . . . . . . . . . . . 31 ||2476/58 * | 17 S. B. . . . . . . . . . . . . . . . . . . 46 || 527|28 Aug. 10 |C. B. . . . . . . . . . . . . . . . . . 23 ||3823|47 Sept. 4 loſ. . . . . . . . & s e e s e e s - e e s • 74 || 193|16 & 4 24 J.. . . . . . . . . . . . . . . . . . . . 35 ||1068|75 Oct. 14 IS. B. . . . . . . . . . . . . . . . . . . 98 ||2809|75 Oct. 5 C. B. . . . . . . . . . . . . . . . . . . 67 || 483|22 Nov. 18 S. B. . . . . . . . . . . . . . . . . . . 116 || 79.4|88 Dec. 1 | Balance . . . . . . . . . . . . . z- NOTE.-If it is desired to balance and foot the account, as in the work of closing Ledgers, first add the greater side of the account, and set the amount. Then add the lesser side and find the difference by the method shown in problem 2, above. This method of subtraction should be practiced by all students who aspire to be accountants, until they acquire accuracy and rapidity. Some bankers will not employ book-keepers who cannot subtract in this manner. +). MISCELLANEOUS PROBLEMS IN SUBTRACTION. 71 8. A depositor has a credit balance of $4208.10, he deposits $684.50 and draws checks for the following amounts: $528.00, $476.40, $78.00, $287.44. What is his credit balance 3 * Ans. $3522.76. 9. My cash balance in bank was $1846.15; I drew out $920.00, then depos. ited in currency, $1200.00, and in checks, $1719.25; I then drew out $408.50. How much is my present cash balance in bank 2 Ans. $3436.90. MISOELLANEOUS PEROBLEMS IN SUBTRACTION. 140. 1. Bought a lot of flour for $2225, and sold the same for $2800. What was the gain } Ans. $575. 2. It is 700 miles to Shreveport and 321 miles to Galveston. How much farther is it to Shreveport than to Galveston ? Ans. 379 miles. 3. The ant has fifty eyes, and the dragon fly 12000. How many more has the dragon fly than the ant? Ans. 11950 eyes. 4. The Old Testament, King James' edition, contains 39 Books, 929 chapters, 23214 verses, 592439 words, and 2738100 letters. The New Testament contains 27 Books, 260 chapters, 7950 verses, 182253 words, and 933380 letters. How many more of each, Books, chapters, verses, words and letters, does the Old Testament Contain than the New Ans. 12 Books, 669 chapters, 15264 verses, 410186 words, and 1804720 letters. 5. Mercury, the smallest planet of the Solar system, is 36,000,000 miles dis- tant from the Sun. Neptune, the most distant, but not the largest planet, is 2,746,000,000 miles distant from the Sun. How much farther from the Sun is Nep- tune than Mercury Ż Ans. 2,710,000,000 miles. 6. Physicists have determined that to produce the color, dark red, 395,000,- 000,000,000 ethereal waves strike the eye per second ; and to produce violet, 760,000,000,000,000 ethereal waves strike the eye per second. How many more waves per second are required to produce violet than dark red 3 Ans. 365,000,000,000,000. 7. Sound travels through the air at the rate of 1118 feet per second, and a bullet fired from a rifle, travels 1750 feet per second. How much faster does the ball travel than sound 3 Ans. 632 feet per Second. 8. Physiologists have determined, with the aid of the microscope, that the lungs of a man contain not less than 600,000,000 air cells; they have also deter- mined that a single drop of human blood contains more than 4,000,000,000 of corpuscles. How many more corpuscles in one drop of blood than air cells in the lungs? Ans. 3,400,000,000. 9. General George Washington was born in 1732, and died in 1799; General Robt. E. Lee was born in 1807, and died in 1870. How much older was General Washington than General Lee, when he died ? Ans. 4 years. 10. What is the difference between 23222 and 11 thousand 11 hundred 11 ? Ans. 11111. 72 SOULE's PHILOSOPHIC PRACTICAL MATHEMATICS. 11. What number must be added to 68741 to make a million ? Ans. 93.1259. 12. Hubbard has $500 which is $150 more than I, and I have $75 more than Reiffer. How much has Keiffer, and how much have I? Ans. Keiffer has $275. I have $350. 13. There are two parties who owe me $8000, and one of them owes $4250. The other wishes to pay me $1700 on account. How much will he then owe ? Ans. $2050. 14. A speculator bought a lot of apples for $215, and sold them at such a price, that if he had gotten $22.50 more, he would have gained as much as they cost him. How much did he sell them for 3 Ans. $407.50. 15. I gave a $20 bill for a purchase of $8.15. How much change should I receive 3 Ans. $11.85 16. The common house fly makes 330 beats per second with his wings, and the honey bee makes 190 beats per second. How many more beats will the fly make in One minute than the bee ? AnS. 8400. NOTE.-There are sixty seconds in a minute. 17. From New Orleans to Vicksburg is 401 miles, and to Natchez, 277 miles. How far is it from Natchez to Vicksburg 3 Ans. 124 miles. 18. What is the difference between one million, seventeen thousand seven, and one thousand, sixteen hundred sixteen 3 Ans. 1014391. 19. The sum of two numbers is 1463, and one of the numbers is 628. What is the other ? Ans. 835. 20. The velocity of our earth on its yearly voyage through space, around the sun, is 99733 feet per second; the velocity of a 12-pound cannon ball, fired from a gun, with an average charge of powder, is 1734 feet per second. How many feet farther does the earth travel, in each second, than a cannon ball ? Ans. 97999 feet, or 18 miles and 2959 feet. 21. What number is that, to which, if 17821 be added, the sum will be 3790.7% AnS. 20086. 22. At an election, the defeated candidate received 23742 votes; but had he received 5112 votes more from new voters, he would have been elected by 1000 majority. How many votes did the elected candidate receive? Ans. 27854. 23. How many years have elapsed since the birth of the following named persons: Zoroaster, according to Aristotle, was born B. C. 5429; Abraham, B. C. 2000; Menes, B. C. 2000; Moses, B. C. 1570; Solomon, B. C. 1033; Homer, B. C. 1000; Lycurgus, B. C. 850; Thales, B. C. 640; Solon, B. C. 638; Pythagoras, B. C. 600; Confucius, B. C. 551; Buddha, or Guatama, B. C. 500; Sophocles, B. C. 495; Socrates, B. C. 470; Hyppocrates, B.C. 460; Plato, B.C. 429; Aristotle, B.C. 384; Demosthenes, B.C. 382; Praxiteles, B. C. 360; Alexander, B. C. 356; Euclid, B. C. 323; Cicero, B. C. 106; Seneca, B. C. 5; Plutarch, A. D. 50; Justinian, A. D. 483; Bacon, A. D. 1561. 24. A father divided his plantation, consisting of 4500 acres, among his * MISCELLANEOUS PROBLEMS IN SUBTRACTION. 73 five sons: Albert, Edward, William, Frank, and Robert. To Albert, he gave 800 acres; to Edward, he gave 150 acres more than he gave Albert; to William, he gave 100 acres less than he gave Edward; to Frank, he gave as much as he gave Edward; and the remainder he gave to Robert. How many acres did Robert receive 3 Ans. 950 acres. 25. The Equatorial diameter of the earth is 7925.65 miles, and the Polar diameter is 7899.17 miles. How much greater is the Equatorial diameter than the Polar 3 Ans. 26.48 miles. 26. Find the difference between 1001734 and MMDCCXLIV. Ans. 10. 27. A carriage, a horse, 4 mules, and 25 sheep, are worth $1425. The car- riage is worth $450, the horse, $150, mules, $175 a piece. What are the sheep worth 3 Ans. $125. 28. The cash on hand in the morning was $278.50. Received during the day from sales, $482.10, and from other sources, $675. There is on hand at the close of the day, $330.50. How much was paid out during the day ? Ans. $1105.10. 29. The cash balance in bank, in the morning, was $2860.15. Deposited during the day, $472. Drew checks in favor of A. $95, of B. $106, and of C. $400. There remains on hand undeposited, $150. What is the balance in bank and on hand, at the close of the day's business? Ans. $288.1.15. 30. December 1, 1893, a merchant had cash in his safe, $2340, and on deposit in bank, $6800. During the month he received $4520, and paid out $5350. What is the balance in Safe and bank & Ans. $8310. 31. January 1, 1894, a retailer deposited $35 change in his cash drawer, in the morning, as per ticket: paid out during the day, $320; received from J. Jones, during the day, $410; from S. Smith, $80. At night there is $328 in the drawer. What have the cash sales been 3 Ans. $123. 32. There is $22.40 change in the drawer in the morning, as per ticket: during the day, cash was received from A. $60, from B. $40.50, from C. $121, and from bills receivable, $600. There was paid out for expenses, $34.50, and to D. $28. There was $996.15 on hand in the drawer at night. What were the cash sales? Ans. $214.75. 33. A merchant purchased merchandise to the amount of $65,421.10, and sold merchandise to the amount of $42,412.90. He then took an account of stock and found that he had $31,018.40 on hand. How much did he gain 3 * Ans. $8010,20. 74. SOULE's PHILOSOPHIC PRACTICAL MATHEMATICS. º: 118. 119. 120. 121. 122. 123. 124. 129. syNOPSIS FOR REVIEw. Subtraction. Difference, or Remainder. Minuend. Subtrahend. Sign of Subtraction. Principle of Subtraction. To Prove Subtraction. What is the Numerical Law re- garding the Difference between TWO Numbers ? * The Borrowing Method Impro- º: O 131. per. 132. 133. 135. 136. 137. 138. 139. 141. Define the following words and phrases: To Subtract by Addition. General Directions for Subtrac. tion. To subtract Dollars and Cents. Business Contractions in Sub- tracting Dollars and Cents. To Subtract Horizontally. Subtraction by the Method of Commencing on the Left. Subtraction by the Complement of 10. sultiplication. -------------------------Rs ( INCREASING ). 142. Multiplication is the process of increasing one of two numbers as many times as there are unitſ, in the other. Or, differently explained, it is the pro- cess of finding the product of two numbers. 143. The number to be multiplied is called the Multiplicand. 144. The number which shows how many times the multiplicand is to be increased, or repeated, is called the Multiplier. 145. The result obtained by multiplying is called the Product. 146. The multiplicand and multiplier are called Factors. The meaning of the word factor is maker or producer. 147. The Sign of Multiplication is an oblique cross, x. It shows that the numbers, between which it is placed, are to be multiplied together, and is read multiplied by, or times. Thus, 8 × 3, is read 8 multiplied by 3, or 3 times 8. Changing the order of the factors does not change the product or result. Thus, 8 x 3 may be read 3 times 8, or 8 times 3. 148. To Prove the operations of multiplication, repeat the work, or multiply the multiplier by the multiplicand. If the result is the same as the first, the work is probably correct. NoTE.—See page 145 for préof of multiplication by casting out 9's and 11's. PRINCIPLES OF MOILTIPLICATHON. 149. 1. In all multiplication operations, the product is the same, in name or kind, as the multiplicand. 2. In all cases, the multiplier must be regarded as an abstract number. Two denominate numbers cannot be multiplied together as denom- inate numbers. Thus, we cannot multiply 5 apples by 5 apples, 10 cents by 10 cents. 12 yards by 8 pounds, or 6 boxes by $2. All such questions are absurd and inSolvable. To illustrate this principle, we present the following problem: 1. What will 8 pounds of rice cost at 11 cents a pound 7 OPERATION. Explanation.—Here we have 11c., the price of 1 pound, to be increased - * * > 8 times, and our reasoning is, since 1 pound costs 11c.18 pounds will cost 8 Multiplicand, 112 times as much, or 8 times 11c., which is 88c. To use the 8 as a denominate Multiplier, 8 number and say 8 pounds times 11c., would show a deficiency of head sense on the part of the calculator. It is true that in this problem the Product 88 multiplicand and multiplier are both denominate numbers; but since the roduct, g multiplier shows the number of times the multiplicand is to be increased or taken, it must always be considered and used as an abstract number. ... Thus, in this example, we do not multiply 11c. by 8 pounds, but we take or increase 11c. 8 times, for the reason that 8 pounds will cost 8 times as much as 1 pound. e º To our mind, nothing in connection with the Science of numbers is more absurd or meaning- less than to speak of multiplying 11c. by 8 pounds, or $2, by 5 gallons, or 50c., by 500. To propose to multiply 4 oranges by 5 peaches would be equally good sense. We 'know what is meant y tak- ing or increasing lic. or 4 oranges a certain number of times; but we cannot understand or com- prehend the taking of anything so many cent times, gallon times, or peach times. (75) 76 SOULE's PHILOSOPHIC PRACTICAL MATHEMATICS. 150. MTULTIPLICATION TAET,E. * 3) 10 15 20 25 30 35 40 45 50 55 60 65 70 75 80 85 90 95 20| 21 40 42 60 63 80| 84 100,105 12012; 140,147 160|168 180|189 210 231 252 273|2 294 315 336 357 378 399 420 441 16 32 48 64 80 96 112 128 144 160 176 192 208 224 240 256 272 288 304 320 336 18 36 54 72 90 108 126 144 162 180 198 216 234. 252 270 288 306 324 ºAL9 tº tºº ºded 360 378 4 8 12 16 20 24 28 32 36 40 44 48 52 56 60 64 68 72 76 80|100 84|105 9 18 27 36 45 54 63 72 81 90 99 108 117 126 135 144 153 162 171 180 Tsº 10 20 30 40 50 60 70 80 90 100 110 120 130 140 150 160 170 180 190 200 º 12 24 36 48 60 72 84 96 108 120 132 144 156 168 180 192 204 216 228 240 252 15 30 45 60 75 90 105 120 135 150 165 180 195 210 225 240 255 270 285 300 315 : 10 12 14 16 18 20 22 24 26 28 30 32 34 36 38 40 42 91 104 117 130 143 156 169 182 195 208 221 234 247 260 2732 42 48 54 60 66 72 78 84 90 96 102 108 1 2 299 322|33 345 368 391 414 437 460 104 112 120 128 136 144 114|133|152 120 140|160 Tºmºrºs 1 3 1 4 105 112 119 126 : : 2 0 168 37 2 2 44 46 48 50 : 72 75 88||110 92||115 96|120 132|154 138|16.1 144|168 100|125 150|175 176 184 192 198 207 216 200 220 230 240 225 250 264 276 288 286 299 330 345 312 300 325 360 375 352 368 384 400 396 414 432 450 456 475 462 460i483 480504 500|525 |2 5 We here present the Multiplication table of Pythagoras, which extends to 25 times 25. If the commercial student excuses himself from learning the Whole table, he should at least learn from 1 to 20 times 10, or that part bounded by the heavy lines. This portion of the table is far more valuable than the 12 times 12 table which is given in school arithmetics, and should be thoroughly learned. When learning this table, or when using it in the solution of problems, be careful not to use any intermediate words orally or mentally, but simply note the figures and pronounce the product at once; thus, do not say or think 7 times 3 are 21; 14 times 8 are 1.12; 17 times 9 are 153, etc.; merely look at the numbers to be multiplied and say, or think, 21, 112, 153, etc. In reading, we do not stop to spell orally or mentally the words that compose the sentences; from the combination of the letters we see what the words are, without looking specially at each individual letter; and to read or operate with rapidity in the combination of numbers, we must omit all Superfluous talk or thought. EXAMPLES, WITH PHILOSOPHIC SOLUTIONS. 77 THE IMPORTANCE OF MULTIPLICATION. 151. Next to a thorough knowledge of addition stands multiplication, as an important acquisition to be possessed by business aspirants. All who are experts in these two subjects of practical arithmetic, combined with a neat and rapid hand- Writing, hold passports to well paying positions. No young man or lady who is ambitious to achieve success and honor in the fields of business can afford to be non- efficient in these topics. Such proficiency is worth more than unnumbered testi- monials of competency from honored merchants, noted bankers, railroad kings, political leaders, or popular governors, THE PEIILOSOPHIC OR LOGICAL SYSTEM OF MULTIPLICATION. 152. At this point of our work, we introduce, and throughout the book shall continue to use the logic of numbers—the Philosophic System of Solving problems. (See page 26 Article 23). By this system, the reasoning faculties of the mind are brought into action, invigorated, strengthened, and capacitated to see fine distinctions, to consider Con- ditions, to investigate facts, to reason logically, and to deduce correct conclusions, from not only the premises and conditions of problems, but upon all matters and questions that the changing affairs of this world's life may present for consider- ation. EXAMPLES, WITH PHILOSOPHIC SOLUTIONS. 153. 1. One orange costs 5 cents. What will 6 oranges cost 3 Ans. 30 cents. SOLUTION STATEMENT. 5% 6 Reason.—One orange costs 5 cents. Since 1 orange costs 5 cents, --> 6 oranges will cost 6 times as much, which is 30 cents. 30g Ans. 2. If one hat costs $2, what will 4 hats cost? Ans. $8. SOLUTION STATEMENT. $2 4 Reason.—One hat costs $2. Since 1 hat costs $2, 4 hats will cost --> 4 times as much, which is $8. $8 Ans. 3. At 8 cents per yard, what will 5 yards cost 7 Ans. 402. SOLUTION STATEMENT. 8g 5 Reason.—One yard costs 8 cents. Since 1 yard costs 8 cents, 5 ºmº yards will cost 5 times as much, which is 40 cents. 40% Ans. (Continued.) 78 soul E's PHILOSOPHIC PRACTICAL MATHEMATICS. * THE REASON, WHY, AND WHEREFORE, CONTINUED. Question.—How do you know, that if 1 yard costs 8 cents, 5 yards will cost 5 times as much 3 Answer.—By the exercise of my judgment—by the use of the reasoning faculties of the mind. Question.—What do you mean, in this connection, by judgment? Answer.—The conclusion arrived at by the operations of the mind, after duly considering the premise, the facts, and the conditions of the problem. Question.—What do you mean by premise or premises 2 Answer.—The proposition, declaration, truth, or fact, which is asserted as the basis or predicate of a question. In this problem, the premise is, one yard costs 8 cents. Question.—Why will 5 yards cost 5 times as much as 1 yard 3 Answer.—Because 5 is five times as much as 1. Question.—What kind of reasoning is the foregoing? Answer.—Analogical and axiomatical. Analogical, because there is analogy, relationship or likeness existing between the cost of 1 yard and the cost of 5 yards. Axiomatical, because the premise and question considered, the conclusion is self. evident. Question.—What is reason ? Answer.—The faculty or power of the human mind, by which truth is dis- tinguished from falsehood, right from wrong, and by which correct conclusions are reached by considering the logical relationship which exists between the premises, the facts, and the conditions of particular statements and questions. 4. What will 4 books cost at 40 cents each 3 SOLUTION STATEMENT. 40g Questións.—1. How do you 4 Reason.—One book costs 40c. know that 4 º will gºt 4 times as much as 1 book # 2. ºmºsºs Since one book costs 40 cents, 4 What do you mean by judgment? $1.60 Ans. books will cost 4 times as much. 3. Why will 4 books cost i times -- as much as 1 book? NOTE.—The answers given to like questions in the preceding problems, are the proper answers to these and similar questions. 5. At 18 cents per dozen, what will 5 dozen cost 3 SOLUTION STATEMENT. 18g Questions.—1. How do you 5 Reason.—One dozen costs 18c. 1-now that 5 º Y. *; & times as much as OZell º *º-> Since 1 dozen costs 18 cents, 5 Why will 5 dozen cost 5 times as 902 Ans. dozen will cost 5 times as much, much as 1 dozen # 3. What do you mean by judgment 3 6. If 1 hat costs $4, what will 6 hats cost? SOLUTION STATEMENT. $ 4 º Questions.—1. How do you 6 Reason—One hat costs º º: know they will ? 2. Why will 1 hat costs $4, 6 hats will cost they # 3. What is judgment in $24 Ans. times as much. this connection ? +): EXAMPLES, WITH PHILOSOPHIC SOLUTIONS. 79 7. Flour is $7 per barrel, what will 20 barrels cost 3 SOLUTION STATEMENT. W. Questions.—1. How do you $ 7 Reason.—One barrel costs $7. e - 20 Since 1 barrel costs $7, 20 barrels know this? 2. Why will they --sº will cost 20 times a h 3. What do you understand by $140 Ans. S Illll CIſl. judgment in this connection? 8. Bought 12 pounds of sugar at 72 per pound. What was the cost of the Whole % SOLUTION STATEMENT. 72 Reason.—One pound costs 7c. º 12 Since 1 pound costs 7 cents, 12 Questions.—1. How do you tº-º- pounds will cost 12 times as know this 2. Why will it? 84% Ans. much. 9. At $7 per cord, what will 123 cords cost 7 SOLUTION STATEMENTS Ea:planation.—In the second statement, for convenience, the lSt. 2d. multiplicand is used as the mul- e $ 7 123 tiplier. Questions.—1. How do you Reason--One cord costs $7. ſº * * * * * 123 7 Since 1 cord of wood costs $7, know this? 2. Why will it * 4- m-m- 123 cords will cost 123 times as $861 Ans. $861 Ans. much. 10. If an employé receives $4 per day for services and he works 22 days, how much money has he earned ? SOLUTION STATEMENT. Reason.—One day's service is 22 worth $4. Since 1 day's service Questions.—1. How do you is worth $4, 22 days' services are know this ; 2. Why will he 4 worth 22 times as much, or since 3. What d e *- he receives $4 for 1 day’s work, * at do you mean by judg- $88 Ans. for 22 days' work he will receive ment # 22 times as much. 11. There are 60 minutes in an hour. How many minutes are there in a day of 24 hours? Ans. 1440 minutes. SOLUTION STATEMENT. - 60 Reason.—In 1 hour are 60 min- Questions.—1. How do you 24 utes. Since there are 60 minutes know this What do you mean - in 1 hour, in 24 hours there are by judgment # 1440 Ans. 24 times as many. The reasoning of these problems and the answers to the questions following the reasoning, should be repeated until the mind is fully capacitated to solve, in like manner, all similar problems. DRILL PROBLEMS. 154. Solve the following problems in like manner as the foregoing, and write the reason for each : 1. What will 6 books cost, at 40g each Ans. $2.40. 2. At 152 per dozen, what will 5 dozen cost Ans. 752, 3. If 1 box costs $2, what will 24 boxes cost? Ans. $48. 8o soule's PHILosophic PRACTICAL MATHEMATICS. 4× 4. At $6 per cord, what will 41 cords of wood cost? Ans. $246. 5. Paid $4 per barrel for potatoes and bought 36 barrels. What did they cost ºf Ans. $144. 6. What will 9 yards cost at 162 per yard 3 Ans. $1.44. 7. Flour is worth $6 per barrel. What are 25 barrels worth ? Ans. $150. 8. 12 inches make a foot. How many inches in 15 feet 3 Ans. 180 inches. 9. 4 quarts make a gallon. How many quarts in a barrel that holds 41 gallons ? Ans. 164 quarts. 10. If you buy 17 boxes of peaches @ $2 per box, what will they cost? Ans. $34. 11. If you buy 9 pencils at 5 cents each, and hand to the seller 50g, how much change should you receive 3 Ans. 52. 12. A merchant bought 43 barrels of apples at $4 per barrel, and paid $120 on account. How much does he still owe ? Ans. $52. TO MULTIPLY ABSTRACT NUMBERS AND GIVE REASONS THEREFOR. 155. 1. Multiply 7 by 6. OPERATION. 7 Ea:planation and Reason.—Plato tells us that one is the basis of all 6 things; but whether this statement be true or false, 1 (one) is certainly the basis of all numbers. Multiplication is the process of repeating one * number as many times as there are units—ones—in another. Considering 42 Ans, these facts, we first multiply the 7 by 1, and in the product obtain a pre: mise for our argument. Thus, axiomatically 1 time 7 is 7. Since 1 time 7 is 7, 6 times 7 is 6 times as many, which is 42. This is the long looked for reason for the multipli- cation of abstract numbers. Multiply the following numbers, and write the reason: 6 × 4; 8 × 5; 17 × 12; 23 × 16; 234 × 157; 341 × 256. TO MULTIPLY, WHEN THE MULTIPLIER CONSISTS OF ONLY ONE FIGURE. 156. 1. What is the product of 947 multiplied by 6% OPERATION. Ea:planation.—In all problems of this kind, write the multiplier under the units' figure of the multiplicand; then commence with the units’ figure, and say, 6 times 7 are 42, which is 4 tens and 2 units; # g the 2 units write in the units’ place of the product and retain in the # §§ mind the 4 tens to add to the column of tens ; next say 6 times 4 tens #5 are 24 tens plus the 4 tens retained in the mind, are 28 tens, which is 2 Multiplicand 94.7 hundreds and 8 tens; the 8 tens write in the tens’ column of the product, Multipli 6 and retain in the mind the 2 hundreds to add to the column of hundreds. Ipſler Then say 6 times 9 are 54 hundreds, plus 2 hundreds are 56 hundreds, emºsºmeºmº which is 5 thousand and 6 hundreds, which write respectively in the Product 5682 thousands' and hundreds' columns of the product. This completes the operation and gives a product of 5682. In practice, instead of saying 6 times 7 are 42, 6 times 4 are 24, etc., we should only name the result of the combination. Thus, 42, 24, etc. In handling figures, we show.ld always pronounce the result of the combinations without naming the figures that make the result, just as we pronounce words without spelling or naming the letters that make the words. * MULTIPLICATION ELUCIDATED. 8I f Perform the following multiplications: • (2) (3) (4) (5) (6) Multiplicand, 543 983 2769 76895 81.453 Multiplier, 7 8 5 9 6 Product, 3801 7864. 1384.5 692055 488718 7. At $85 each, what will 7 wagons cost 3 Ans. $595. 8. What will 8 lots of ground cost, at $1875 each 3 Ans. $15000. 9. At $6 per barrel, what will be the cost of 245 barrels of flour? SOLUTION STATEMENT. Reason.—One barrel of flour costs $6. Since 1 barrel of flour Multiplier, 245 costs $6, 245 barrels will cost 245 times as much. The $6 is the Multiplicand, $ 6 real multiplicand, but in the operation we use it as the multiplier. p 7 This is done for convenience, in all problems where the multipli- cand is less than the multiplier. The result is the same which- $1470 AllS. ever factor is used as a multiplier. 10. What will 42 dozen boxes cost, at $9 per dozen? Ans. $378. 11. At $7 a piece, what will 48 chairs cost? Ans. $336. 12. A clerk receives $75 per month. If he spends $40 per month, how much can he save in one year, or 12 months? y Ans. $420. 13. Multiply 1, 2, 3, 4, 5, 6, 7, 0, 8, 9, and 10 together. Ans. 0. 14. Bought 44000 pounds of cotton at 8% per pound and sold it at 9% per pound. What was the gain 3 Ans. $440. 15. What is the difference in the cost of 150 sheep at $4 a head, and 80 head of cattle at $12 a head. Ans. $360. TO MULTIPLY, WHEN THE MULTIPLIER CONSISTS OF MORE THAN ONE FIGURE. 157. 1. What is the product of 397 multiplied by 653? OPERATION. F.3 g . # 'E 2 #"> 23 & E #: ##### Multiplicand, 397 Multiplier, 653 First partial product by 3 units, 1191 = 3 times the multiplicand. Second “ 4% by 5 tens, 1985 = 50 “ {{ 4% Third {{ é & by 6 hund's, 2.382 = 600 ($ {{ {{ Total product, 25924.1 = 653 {{ 44 & 4 Explanation.—In all problems of this kind, write the multiplier under the multiplicand, so that units of the same order stand in the same column, and then multiply by one figure at a time. 82 - SOULE's PHILOSOPHIC PRACTICAL MATHEMATICs. * First multiply by the units’ figure, then by the tens', hundreds', and so on, in regular order, through the multiplier; then add the several partial products together and thus obtain the required product. In this problem, first multiply by 3, the units’ figure, in the same manner as explained in the first problem where there was but one figure in the multiplier and obtain 1191 as the first partial product. Write this below the multiplier so that units of the same order stand in the same column. Next multiply by the 5 tens; say 5 times 7 are 35, which is 3 hundreds and 5 tens ; write the 5 tens in the tens’ column directly below the multiplying figure, and reserve in the mind the 3 hundreds to add to the hundreds’ column. Then say 5 times 9 are 45 + 3 hundreds, which were reserved, mak- ing 48 hundreds which is 4 thousands and 8 hundreds ; write the 8 hundreds in the column of hundreds, and reserve the 4 thousands to add to the thousands’ column; then say 5 times 3 are 15 plus 4 thous- º reserved, are 19 thousands which is 1 ten thousand and 9 thousands, which write in their respective COLUIDOll].S. Then, in like manner, multiply by the 6 hundreds in the multiplier, being careful to write the first figure obtained (2) in the hundreds' column, directly under the 6 of the multiplier, and the other figures in their respective columns, thousands, ten thousands, and hundred thousands ; then add the partial products together, which gives 259241 as the whole product of 397 multiplied by 653, NoTE.—In practice, remember to name or think only the results of the numerical combina- tions, when adding or multiplying. EXAMPLES. 2. Multiply 3426 by 457. OPERATION. * É ... F & . gºš # tº 3 ; ; • * ###### © sº; e. >|r. HP tº #5 Multiplicand, 3.426 Multiplier, 4 57 First partial product by 7 units, 23982 = 7 times the multiplicand. Second “ “ by 5 tens, 1713 () — 50 % “ {{ Third 4 “ by 4 hund's, 13704 = 400 “ “ {{ Whole product, 1565 682 = 457 46 “ 4% 3. Multiply 28433 4. Multiply 989769 5. Multiply 21794 by 4172 by 248.193 by 2365 Multiply the following numbers: 6. 483 by 569 8. 581 by 76 10. 671 by 508 12. 1683 by 328 7. 924 by 237 9. 1847 by 84 11. 8765 by 2046 13. 2346 by 127 Operation of the 10th problem. 671 508 Ea:planation.—In all problems where there are naughts in 5368 the multiplier, multiply by the significant figures only, for the 3355 reason that the product of any number by 0 is 340868 Ans. X- MULTIPLICATION. 83. º TO MULTIPLY, WHEN EITHER FACTOR IS 1 WITH NAUGHTS a' ANNEXED, AS 10, 100, 1000, ETC. EXAMPLES. 158. 1. Multiply 465 by 100. OPERATION. Explanation.—In all problems of this kind, we annex as many naughts to the multiplicand as there are naughts in the multiplier. In this problem, we 46500 annex two 0's and produce 46500, the answer. The basis or reason for this is in accordance with the Principles of Notation, (Arts. 44 and 45) where it was º shown that annexing one naught to a number removes or changes the unit of such number into tens, the tens into hundreds, the hundreds into thousands, and so on, and hence mul- tiplies by 10, 100, 1000, etc., for the higher powers of ten. 2. Multiply 500 by 10. Ans. 5000 || 4. Multiply 1850 by 1000. Ans. 1850000 3. Multiply 1000 by 100. Ans. 100000 || 5. Multiply 7020 by 10000. Ans. 70200000 TO MULTIPLY, WHEN EITHER THE MULTIPLICAND OR MULTIPLIER, OR BOTH, HAVE NAUGHTS ON THE RIGHT. 159. Multiply 463 by 200. OPERATION. 463 463 Explanation.—In all problems of this kind, we first multiply 2 Of 200 together the significant figures and to the product annex as many º º, thºs are on the right of the multiplier or multiplicand, *- or of both. In writing the numbers to be multiplied, place the 92600 Ans. 92600 Ans. right hand significant figures of the multiplicanã and muftiplier in º e the same column, omitting the naughts or extending them to the right. In this problem, we first use the multiplier 2 hundred as 2 units; hence the first partial pro- duct, 926, was 100 times too small. We then, by annexing the two naughts, multiplied it by 100, º ºned 92600 as the correct product. For further explanation and reason, see problem 1, rt. 158. 2. Multiply 940 by 4700. OPERATION. 94 940 Explanation.—94 multiplied by 47 equals 4418 as the 47 4700 first partial product; then, since the 94 was used as 94 -ºm- OI’ units instead of 94 tens, we annex one naught to 4418 658 658 making 44180; then, since the 47 was used as 47 units 376 376 instead of 47 hundreds, we annex two naughts to 44180, **- e-mºmº-mº- making 4418000, the correct answer. See Problem 1. Art. 4418000 Ans. 4418000 Ans. 158. 3. Multiply 3400 by 26. Multiply 5020 by 420. 4. Multiply 82000 by 483. OPERATION. OPERATION. OPERATION. 3400 5020 82000 r 483 26 420 483 O 82000 204 1004 246 966 68 2008 656 3864 *-ºs- - 328 4- 884.00 Ans. 2108.400 Ans. ——— 39606000 Ans. 39606000 Ans. 5. Multiply 842 by 600. 8. Multiply 23500 by 12030. 6. Multiply 1208 by 1020. 9. Multiply 1000 by 6208. 7. Multiply 9900 by 707. 10. Multiply 81000 by 90200. 84 SOULE's PHILOSOPHIC PRACTICAL MATHEMATICs. TO MULTIPLY BY TEIE FACTORS OF A NUMBER. 160. Factors of a number are such numbers as will, when multiplied together, produce the number. Thus, 6 and 6 are the factors of 36; 7 and 8 are the factors of 56. PRINCIPLE. The product of any number of factors will be the same, in whatever order they may be multiplied. 1. Multiply 2435 by 42. OPERATION. *: Explanation.—In all problems of this kind, separate the multiplier into two or more factors; then multiply the multiplicand by one of 17045 the factors, the resulting product by another factor, and so on, until 6 all the factors have been used. The last product will be the correct product. 102270 Ans. Use the factors and multiply the following numbers: 2. 480 by 361. 3. 1756 by 125. 4. 3281 by 128. 5. 781 by 63. 6. 3140 by 36. 7. 588 by 81. 8. 473 by 432. 9. 5107 by 336. To MULTIPLY, WHEN THE MULTIPLICAND OR MULTIPLIER CON- TAINS DOLLARS AND CENTS. 161. 1. Multiply $342.15 by 6. OPERATION. $342.15 Eacplanation—In all problems of this kind, multiply in the regular manner, then prefix the dollar sign ($) and place the point (. ) two places from the right. The answer is then Product, $2052.90 in dollars and cents. 2. What will 1682 pounds of sugar cost, at 9% per pound 7 Ans. $151.38. 3. A merchant's monthly expenses are $1342.75. What are they for 12 months & Ans $16113.00. 4. It costs a family $2.30 a day for marketing. What will be the expense for 30 days? Ans. $69.00. 5. What will 37 boxes of oranges cost, at $3.75 per box 3 Ans. $138.75. 6. At 16 cents per pound, what is the value of 23780 pounds of cotton ? Ans. $3804.80. 7. If it costs $17500 to construct one mile of railroad, what will be the cost to build 364 miles 3 Ans. $6370000. 8. What will 875 tons of railroad iron cost at $55 per ton ? Ans. $48125. 4× MISCELLANEO US PROBLEMS IN MULTIPLICATION. 85. GENERAL DIRECTIONS FOR MULTIPLICATION. 162. From the foregoing elucidations, we derive the following general directions for multiplication: 1. Write the multiplier under the multiplicand, so that units of the same order stand in the same column, and draw a line beneath. 2. When the multiplier consists of one figure, begin at the units’ figure and multiply each figure of the multiplicand by the multiplier. Write in the product line the units of each result, and add the tens, if any, to the newt result. 3. When the multiplier consists of two or more figures, begin at the units’ figure and multiply successively, each figure of the multiplicand by each figure of the multiplier, placing the right hand figure of each partial product under that figure of the multiplier which produced it. 4. Draw a line beneath the several partial products and add them together; the sum will be the required product. If there are any decimals in the factors, point off as many figures from the right of the product as there are places of decimals in the multiplicand and multiplier. PROOF.—1st. Carefully review the work. 2d. Multiply the multiplier by the multiplicand; if the results are the same, the work is probably correct. NOTE.-See proof of multiplication by casting out the 9's and 11's, page 154. MISCELLANEOUS PROBLEMS IN MULTIPLICATION. 163. 1. What is the value of the following numerical expressions: (43 —6)+(8 × 2). Ans. 53. 2. What is the product of 8 + 7, multiplied by 204—101? Ans. 1545. 3. What is the product of 16 + 18–10 by 12 × 23 Ans. 576. 4. What is the difference between 50—(5 × 4) and 25 + 4–83 Ans. 9. 5. Multiply 240 – 50+ 22 by 14 × 16 – 112–65. Ans. 29736. 6. Multiply 16 thousand 16 hundred 16, by 11 thousand 11 hundred forty and 11. Ans. 214052016. 7. Multiply together the following numbers: 9 × 8 × 7 × 6 × 5 × 4 × 3 × 0 × 2 × 1. Ans. 0. 8. What will 6 dozen dozen boxes cost, at one half a dozen dozen cents per box 3 Ans. $622.08. 9. The pulse of a healthy middle-aged person beats 72 times per minute, how many times does it beat in 1440 minutes? * Ans. 103680. 10. Multiply one million twenty-six, by nineteen thousand seven hundred ten. Ans. 19710512460. 11. One cubic foot contains 1728 cubic inches. How many cubic inches in 324 cubic feet 3 Ans. 55.9872. 86 SOULE's PHILOSOPHIC PRACTICAL MATHEMATICS. * 12. One square foot contains 144 square inches. How many square inches in 95 square feet? Ans. 13680. 13. One gallon contains 231 cubic inches. How many cubic inches in a cis- tern that holds 3500 gallons 2 AnS. 808500. 14. One bushel contains 2150.42 cubic inches. How many cubic inches in 20 bushels 3 Ans. 43008.40. 15. One mile contains 5280 feet. How many feet in 25 miles? Ans. 132000. 16. The human heart beats 4200 times an hour. How many times does it beat in 10 years, there being 24 hours in one day, and allowing 365 days in each year 3 AnS. 3679.20000. 17. Sound travels 1118 feet per second. How far will it travel in 10 minutes, there, being 60 seconds in a minute 3 Ains. 670800 feet. 18. A railroad train runs 25 miles an hour. How far will it go in 3 days, allowing 3 hours for lost time in stoppages } Ans. 1725. 19. Light travels 192500 miles per second. How many miles will it travel in 1 day, there being 24 hours in a day, 60 minutes in an hour, and 60 seconds in a minute % Ans. 1663.2000000. OPERATION INDICATED. 24 hours, x 60 minutes, x 60 seconds, x 192500 miles per second, = the answer. 20. If a person respires 17 times in a minute, how many times will he breathe in a day? Ans. 24480. 21. At $17 per ounce, what is the value of 9 pounds of gold, there being 12 ounces in a pound, Troy or Mint weight? Ans. $1836. 22. What will 27893 pounds of tobacco cost, at 56 cents per pound 3 Ans. $15620.08. 23. What will 1870 acres of land cost, at $18 per acre 2 Ans. $33660. 24. The Senate and the House of Representatives of the State of Louisiana, consist of 137 members, who receive $4 per day. The regular session continues 60 days. What is the yearly expense for the salaries of the State's law makers ? Ans. $32880. 25. How many hours, minutes, and seconds in the month of July 2 NOTE.—See Problem. 19. Ans. 744 hrs. 44640 min. 2678400 sec. 26. What will 1463 ounces of silver cost, at 87.2 per ounce? Ans. $1272.81. 27. A contractor has 865 men employed at $1.50 per day. What are the weekly wages of all, for 6 days’ labor? Ans. $7785. 28. What will it cost to build 37428 cubic yards of levee, at 23 cents per cubic yard & Ans. $8608.44. 29. A steamboat arrives with 3840 bales of cotton, 1320 sacks of cotton Seed, and 580 barrels of molasses. Her freight charges are $2 per bale for Cotton, 25% per +& MISCELLANEOUS PROBLEMS IN MULTIPLICATION. 87 Sack for Cotton seed, and 50% per barrel for molasses. What is the amount of her freight bill? Ans. $8300. 30. A drayman charges 75 cents a load, and he has hauled 63 loads. How much is due him & Ans. $47.25. 31. What will it cost to slate the roof of a house containing 52 squares, at $13.25 per square 2 Ans. $689. 32. The walks around a dwelling contain 129 square yards. What will it cost to flag them with German flags, at $3.10 per square yard 2 Ans. $399.90. 33. What will it cost to pave a street containing 20000 square yards, with stone, at $4.75 per square yard 2 Ans. $95000. 34. Bought 2180 barrels of coal at 48% per barrel. What was the cost? t Ans. $1046.40. 35. Multiply 5 billion 16, by 5 million 1 thousand. All S. 25005000080016000. 36. A hogshead of sugar contains 1085 pounds. How many pounds in 107 hogsheads of equal weight? Ans. 116095. 37. A planter produced 68 bales of cotton. If the average weight of the bales was 460 pounds, and the cotton sold for 13 cents per pound, how much money would it bring ? * Ans. $4066.40. 38. What will 3 cases, containing 2 dozen pairs each, of shoes cost, Ø $2.90 per pair 3 Ans. $208.80. 39. If it costs $1.50 a day to support one person, what will it cost to sup- port a family of 13, for one year or 365 days? Ans. $7117.50. 40. A merchant sold three dozen dozen ladies’ hose at one-quarter of a dozen dozen cents a pair. How much did he receive for them 2 Ans. $155.52. 41. The pressure of the atmosphere is 15 pounds on every Square inch of surface. The exterior surface of a man of average size is about 2500 square inches. How many pounds weight does he sustain } Ans. 37500 pounds. 42. How many dollars are 375 $10 gold pieces worth ? Ans. $3750. 43. What is the value of 2146 dimes 3 Ans. $214.60. 44. What is the value of 1010 quarter dollars? Ans. $252.50. 45. What is the value of 728 nickles 2 Ans. $36.40. 46. What is the value of 1612 half dollars? Ans. $806.00. 47. 4875 is the thirteenth part of a number. What is the number 3 Ans. 63375. 48. The sun is 1384500 times as large as the earth; the earth is 45 times as large as the moon. How many times is the Sun as large as the moon? Ans. 62302500. 49. A man's receipts are $1800 a year, and his disbursements are $1125 a. year. How much are his net receipts in three years? Ans. $2025. 50. The population of a city is 250,000. If the average expense of each inhabitant is $1 per day, what is the total yearly expense for an ordinary year? Ans. $91,250,000. 51. A merchant sold 52 barrels of molasses, each containing 41 gallons, at 38g per gallon. How much did he receive for it? Ans. $810.16. 88 SOULE's PHILOSOPHIC PRACTICAL MATHEMATICS Yºr 52. Light travels 192500 miles a second, and it requires 100000 years to travel to us from some of the fixed stars that are seen with the telescope. Allowing 365 days, 5 hours, 48 minutes, and 49 seconds to a year, and remembering that there are 24 hours in a day, 60 minutes in an hour, and 60 seconds in a minute, how far distant are such stars 3 AnS. 607470883250000000 miles. SIMULTANEOUS, OR CROSS MULTIPLICATION. 164. This system of multiplication is of inestimable value. It is, so to speak, the accountants' and calculators' magic wand, by which they may produce results in multiplication operations with almost lightning rapidity. No one can be proficient in the handling of numbers without a thorough knowledge of this system of work in addition to the several other contracted methods, which follow this work. The operations of this system are based upon the following principles: Hundredsxtens “ thousands. Tens × hundreds 66 thousands. 10. Units X thousands “ thousands. J 11. Ten thousands x units produce ten *:::::::) 1. Units × units produce units. 2. Tens × units { % ten S. 3. Units X tens {{ tenS. } 4. Hundreds × units produce hundreds. 5. Tens × tens “ hundreds. | 6. Units x hundreds {{ hundreds. 7. 8. 9 Thousands × units “ thousands. l 12. Thousands X tens 44 ten thousands. 13. Hundreds × hundreds “ ten thousands. } 14. Tens × thousands 44 ten thousands. 15. Units x ten thousands 6% ten thousands. 16. Hundred thousands × units produce hundred thousands.) 17. Ten thousands X tens { % hundred thousands. 18. Thousands × hundreds {{ hundred thousands. 19. Hundreds x thousands “ hundred thousands. } 20. Tens × ten thousands {{ hundred thousands. 21. Units × hundred thousands 4% hundred thousands. and so on for higher numbers. J PROBLEMIS. 1. Multiply 54 by 37. OPERATION. Explanation. First multiply together in the ordinary manner, the units' fig- 54 ures, thus, 7 times 4–28, and write the 8 in the first place of the product, and 37 - carry 2. Next multiply the tens' figure of the multiplicand by the units' figure of the multiplier, thus, 7 times 5 (+2 to carry)=37, which retain in the mind and 1/ add thereto the product of the units’ figure of the multiplicand by the tens, *º-º-º: figure of the multiplier, thus, 3 times 4 = 12, H-37 = 49. Write the 9 in the 1998 tens' place of the product and retain the 4 hundreds in the mind to be added to the column of hundreds. The product of the multiplicand by the units' figure of the multiplier, and also of the units' figure of the multiplicand by the tens' figure of the multi- plier is now produced in the two figures (98) of the final product, and hence we have no further use for the units’ figure of either factor, therefore check and pass the unit column and proceed to mul- tiply the tens’ figures of the two factors, thus, 3 times 5 are 15, plus the 4 retained in the mind, makes 19, which write to the left of the two product figures first obtained, and complete the product. SIMULTANEOUS OR CROSS MULTIPLICATION. 89 To elucidate the operation by figures only, we present the following; OPERATION. *|† 54 Baplanation.— 5 ** >~~ g 3. 7 x 4 = 218 units. *== 7 × 5 + 2 + 3 × 4 = 419 tens. 1998 3 × 5 + 4 = 119 hunds. Multiply the following numbers: (2) (3) (4) (5) (6) (7) (8) (9) 35 62 87 76 93 89 93 86 46 23 42 58 64 97 78 92 10. Multiply 5417 by 62. Ans. 335854. OPERATION. Explanation.—In this, as in the above problems, first multiply the units 5417 figures and set the result in the first place of the product. Thus, 2 times 7 $) are 14; set the 4 and carrying the 1 proceed as follows: 2 times 1 are 2 plus 62 1 makes 3, to which we add 6 times 7 are 42, making 45; set the 5 and carry V 4. Then 2 times 4 are 8 plus 4 makes 12, to which we add 6 times 1 are 6, Tºº-º- making 18; set the 8 and carry the 1. Then 2 times 5 are 10 plus 1 makes 11, 335854 to which we add 6 times 4 are 24 making 35; set the 5 and carry the 3. Hav- ing now as many figures in the partial product produced as we have in the multiplicand, for the reason given in the first problem we check and pass the units’ figure of the multiplier, and with the tens' figure multiply 6 times 5 are 30 plus the 3 to carry makes 33, which completes the product. 11. Multiply 62549 by 53. Ea.planation.— Carrying and product figures. OPERATION. 3 × 9 = 27 or thus: 27; 12, 14, 45, 59; 62549 3 × 4 + 2 + 5 × 9 = 59 15, 20, 20, 40; 1/ 3×2 + 4 + 5 × 5 = 35 18, 21, 10, 31; *=== 3 × 6 + 3 + 5 × 2 = 31 , (check the 3). 3315097 5 × 6 + 3 = 33 30, 33. Multiply the following numbers: (12) (13) (14) (15) (16) (17) 13502 3243 14907 897 52061 298.1453 43 27 64 48 83 39 18. Multiply 7524 by 346. OPERATION. Ea:planation.—In the elucidation of this problem, the principles 7 524 \ used being the same as in the above examples, we will therefore con- 34 6 dense the explanation of the work. 6 times 4 = 24, set the 4 and carry the 2. 1/1/ th ; times 2 = 12 + 2 = 14 + 4 times 4 = 30, write the 0 and carry 2603304 6 6 times 5 = 30 +3 =33 + 4 times 2 = 41 + 3 times 4 = 53, write the 3 and carry the 5. 6 times 7 = 42 +5 = 47 -- 4 times 5 = 67 + 3 times 2 = 73, write the 3 and carry the 7. Having now as many figures in the partial and final product as we have figures in the multi- plicand, we observe that the product of the multiplicand by the units' figure of the multiplier is already produced in the final product, and hence we check and pass the units’ figure and commence with the tens' figure of the multiplier; and as the product of the first three figures of the multiplicand 90 soule's PHILosophic PRACTICAL MATHEMATICs. { has been taken by the tens' figure of the multiplier, we commence with the fourth or last figure of the multiplicand. 4 times 7 = 28 + 7 = 35 + 3 times 5 = 50, set the 0 and carry the 5. * Now, as the product of the multiplicand by the tens' figure of the multiplier is produced in the final product, we check and pass the same and commence with the third or hundreds' figure of the multiplier; and as the product of the first three figures of the multiplicand, by the third or hundreds' figure of the multiplier has already been produced, we commence with the fourth or thousands' figure of the multiplicand. 3 times 7 = 21 + 5 = 26, which we set and complete the product. 19. Multiply 74018 by 45602. OPERATION. f 74 () 18 ing an 45 602 Elaplanation.— º: #. 1/1/v/v/ 2 × 8 = 16 or thus: 1. 8 = wº & A ºn a tº 3.37 53 68836 2 × 1 + 1 + 0 × 3 16; 2, 3; 48; 2× 0 + 0 × 1 + 6 × 8 = 48 8, 12, 6, 18, 40, 58; 2× 4 + 4 + 0 × 0 + 6 × 1 + 5 × 8 = 58 14, 19, 5, 24, 32, 56; 2 X7 -- 5 + 0 × 4 + 6 × 0 + 5 × 1 + 4 × 8 = 56 (Check 2) 5, 24, 29; 0 × 7 + 5 + 6 × 4 + 5 × 0 + 4 × 1 = 33 33; (check 0) 42, 45, 6×7 -- 3 + 5× 4 + 4 × 0 = 65 20, 65; (check 6) 35, 5 × 7 -- 6 + 4 x 4 = 57 41, 16, 57; (check 5) 4 × 7 -- 5 = 33 28, 33. The multiplication sign X, in the above elucidation, should be read “TIMES.” NoTE 1.-it will be observed that the general principles governing this system of multiplication have been strictly used in the operations and explanations. Thus, in the last problem, units are multi- plied by units, (8 by 2); tens by units, (1 by 2); units by tens, (8 by 0); hundreds by units, (0 by 2); tens by tens, (1 by 0); units by hundreds, (8 by 6); thousands by units, (4 by 2); hundreds by tens, (0 by Ö); tens by hundreds, (1 by 6); units by thousands, (8 by 5); ten thousands by units, (7 by 2); thousands by tens, (4 by 0); hundreds by hundreds, (0 by 6); tens by thousands, (1 by 5); units by º (8 by 4); ten thousands by tens, (7 by 0); thousands by hundreds, (4 by 6); hun- dreds by thousands, (0 by 5); tens by ten thousands, (1 by 4); ten thousands by hundreds, (7 by 6); thousands by thousands, (4 by 5); hundreds by ten thousands, (0 by 4); ten thousands by thous- ands, (7 by 5); thousands by ten thousands, (4 by 4); ten thousands by ten thousands, (7 by 4). NOTE. 2.--By these lengthy elucidations, the work seems more tedious than it really is in practice. In performing the operation practically, we name results only, and thereby very much lessen the labor DRILL PROBLEMS, IN SIMULTANEOUS MULTIPLICATION. 165. Students should drill on the following problems, at different times until they can call the intermediate results as rapidly as they can talk. A few weeks of practice, an hour or two per day, will capacitate the learner to see mentally the intermediate results faster than he can talk or write them. 1. Multiply 1234 by 37. OPERATION. 1 2 3 4. * 3 7 Intermediate results.-28; 21, 23, 12, 35; 14, 17, 9, 26; 1/ 7, 9, 6, 15; (check 7) 3, 4. 456 58 SIMULTANEOUS OR CROSS MULTIPLICATION. 9 I 2. Multiply 438 by 395. OPERATION. Intermediate results.-40; 15, 19, 72, 91; 20, 29, 27, 4.38 56, 24, 80; (check 5) 36, 44, sº, 9, 53; (check 9) 12, 17. - or thus: 40; 91; 80; (check 5). 1 73 0 1 0 53; (check 9) 17. 3. Multiply 3602 by 5184. OPERATION. Intermediate results.-8; 16; 24, 25, 2, 27; 12, 14, 360 2 48, 62, 10, 72; (check 4) 24, 5 18.4 31, 6, 37; (check 8).3, 6, 30, 36; 1/1/1/ (check 1) 15, 18, 1867 2 768 or thus: 8; 16; 27; 72; (check 4). 37; (check 8) 36; (check 1) 18. 4. Multiply 90472 by 8706. & OPERATION. Intermediate results.-12;. 42,43; 24, 28, 14, 42. —t – 9 () 4 7 2 49, 4, 53, 16, '69; 54, 60, 8 7 0 6 t 28, 88, 56, 144; (check 6). 1/1/1/ 14, 32, 46; (check 0) 63, 67; (check 7) 72, 78; 78 7 6 49 2 3 2 or thus:... 12; 43; 42; 69; %, 144; (check 6) 46; (check 0) 67; (check 7) 78. 5. Multiply 503102 by 430240. OPERATION. Intermediate results.-8; 4; 4; 12, 2, 14, 6, 20; 5 0.31 02 2, 6, 8, 8, 16; 20, 21, 3, 24; 430 24 0 (check 4) 10, 12, 9, 21, 4, 25; 1/VVV (check 2) 2, 12, 14; (check 0) 15, 16; (check 3) 20, 21; 21 64 54 6 04480 then annex the 0. or thus: 8; 4; 4; 20 ; 16; 24; (check 4) 25; (check 2) 14; (check 0) 16; (check 3) 21; then annex the 0. § $ore.—when there are naughts on the right of either or both of the factors, treat them as explained in Problem 1, Article 159. 6. Multiply 4081700 by 53920. OPERATION. 4081 700 Baplanation.—14; 66; 52; 115; 53% 48; (check 2) 80; (check 9) 20; (check 3) 26; annex 3 naughts. 2 2008 52 640 00 92 SOULE's PHILOSOPHIC PRACTICAL MATHEMATICs. 7. Multiply 240700 by 68100. Multiply by the following problems: (8) (9) (10) (11) (12) (13) 321 2334 4354 270018 554.422 9080706 34 136 809 30402 19345 1122033 10914 (14) (15) (16) (17) (18) (19) 456 4120 5841 16190 2736 88997 123 54 325 2305 2107 11238 56088 (20) (21) (22) (23) (24) (25) 6208 894 2382 6709 4087 19205 340 143 751 284. 2614 41003 21107.20 (26) (27) (28) (29) (30) (31) 14901 789 999 666 5566 5070608 782 567 888 777 4488 107024 11652582 32. Multiply 222333444555 by 219543620324 OPERATION. - 222333444555 21954.3620324 assilssºgliooasſºszo Ans. A DIFFERENT METHOD OF SIMULTANEOUS MULTIPLICATION. 166. Multiply 673 by 452. AnS. 304196. OPERATION. © Ea:planation.— 2 × 3 = 6 "ş, 2 × 7 – 3 x 5 = 29 2 × 6 + 2 carried -- 7 × 5 + 3 × 4 = 61 304196 5 × 6 + 6 carried -- 4 × 7 = 64 4 × 6 + 6 carried = 30 It will be observed that the problem is to repeat 673,452 times; this has been done in the operation, though by reason of reversing the multiplier it appears at first sight that the multiplicand has been repeated only 254 times. By following Yºr SIMULTANEOUS OR CROSS MULTIPLICATION. 93 the different steps of the solution with care, it is clear that the 673 has been repeated 2 times + 50 + 400 times, as the problem required. This method of simultaneous multiplication is preferred by some calculators to the method first presented. We however much prefer the first method, for the following reasons: 1. There are no less figures to make and no less thinking to do in this method than there is in the first method. 2. By reversing the multiplier errors are much more liable to occur, and increased work is required in rewriting the multiplier. 3. When making extensions in bills or invoices, the reversing of the multi- plier would add mental labor to the operation, lead to confusion, and increase the liability to make errors. We present this method more to show its deficiencies, when compared with the regular simultaneous multiplication first presented, than for any merit it pos- $62SSéS. 2. Multiply 6758 by 927. Ans. 6264666. OPERATION BY THE ABOVE METHOD. 67.58 Eacplanation.— 7 x 8 = 56 729 7 × 5 + 5 + 2 × 8 = 56 sºmeºmºmºmºmºmºmºsºme 7 x 7 -- 5 + 2 × 5 + 9 × 8 = 136 6264666 7 × 6 + 13 + 2 × 7 -- 9 × 5 = 114 2 × 6 + 11 + 9 × 7 = 86 9 × 6 + 8 = 62 TO MULTIPLY WHEN THE MULTIPLIER CONSISTS OF TWO FIGURES. 167. 1. Multiply 5234 by 23. orrºs. Explanation.—First. 4 × 23 = 92; set the 2 and carry 9. 5234 Second. 3 × 23 = 69 + 9 = 78; set the 8 and carry 7. 23 Third. 2 × 23 = 46 -- 7 = 53; set the 3 and carry 5. Fourth. 5 × 23 = 115 –– 5 = 120. 120382 + 2. Multiply 6845 by 34. OPERATION. Explanation.— 6845 5 × 34 = 170. 34 4 × 34 = 136 -– 17 = 153. gºmºmºmºmºsº 8 × 34 = 272 + 15 = 287. 232730 6 × 34 = 204 -- 28 = 232. Multiply the following problems in the same manner: 3 4 * (6) (7) (8) (9) § 13; ački 63024 725 309 8523 42 26 54 63 37 86 47 amm amº- mº amme sº-s m-- *-* 94. SOULE's PHILOSOPHIC PRACTICAL MATHEMATICs. A. CONTRACTIONS WHEN TEIE MULTIPLIERS ARE CONVENIENT ALL- QUOTS OF 10, 100, or 1000, OR SOME MULTIPLE THEREOF. NOTE.—Those who are unfamiliar with division and with fractions may pass such problems as involve this knowledge. * 168. TABLE OF ALIQUOTS. To multiply by 1% multiply by 10 and divide the product by 9 { % 1+ {{ 10 {{ {{ {{ 8 {{ # {{ 10 {{ {{ {{ 7 {{ 13 { % 10 4. {{ {{ 6 {{ 24 4% 10 {{ 4% {{ 4 44 3# , 4% 10 {{ {{ 4% 3 44 6# {{ 100 {{ {{ {{ 16 {{ 8% {{ 100 {{ {{ {{ 12 4% 12; * { 100 {{ {{ {{ 8 {{ 14% {{ 100 44 44 {{ 7 {{ 16# {{ 100 {{ {{ {{ 6 {{ 18; {{ 300 44 {{ {{ 16 {{ 25 {{ 100 4% \ {{ {{ 4 {{ 31#. 4% 500 4% {{ {{ 16 {{ 33% 4. 100 {{ {{ “ 3 {{ 37; 44 300 {{ {{ 4 * 8 {{ 624 {{ 500 {{ {{ {{ 8 44 663 44 100 and subtract # of product; or 4% 200 and divide by 3. {{ 75 “ 100 and subtract # of product. {{ 83% {{ 500 and divide by 6, or x 1000 and +12. {{ 87; & 4 700 and divide by 8, or {{ 4% 100 and deduct # of product. {{ 1124 4% 100 and add # of product. {{ 125 44 1000 and divide by 8. {{ 133; 4% 100 and add # of product. {{ 1663 4% 1000 and divide by 6 4% 250 {{ 1000 {{ 4 {{ 33.3% 4% 1000 {{ 3 {{ 375 {{ 3000 { % 8 {{ 625 {{ 5000 {{ 8 {{ 833% 44 5000 {{ 6 {{ 875 {{ 7000 {{ 8 NOTE.-Learn this table perfectly. The reason for the above methods of work will appear plain by a careful examination of the following problems: PROBLEMS. 169. 1. Multiply 289274 by 24. OPERATION. - 4) 2892740 Ea:planation.—In answering the conditions of this simple problem, which is to repeat the multiplicand 24 times, we first observe that 2% is + of 723185. An 10, or that 4 times 2} make 10, and hence to facilitate the work, we first S. multiply by 10, which is done by annexing one naught. This gives us a product of 2892740 which is four times too #". for the reason that 10 is 4 times 2}. To produce the correct result, therefore, we divide by 4, which gives us 723185. NotE.—In practice, we would simply divide by 4 and carry the division one place. * CONTRACTIONS IN MULTIPLICATION BY ALIQUOTS. 95 2. Multiply 79862 by 124. OPERATION., 8) 7986200 Explanation.—The conditions of this problem require that the multi- ... plicand be taken or repeated 124 times; but in performing the operation, 998.275 Ans we first observe that 12} is # of 100, and hence to save time and figures, we { * first take or repeat the multiplicand 100 times, as indicated by the small naughts, which gives a product as many times too great as 100 is times as great as 12%, which is 8 times. Therefore, we divide by 8 to produce the correct product. NOTE.-In practice, the annexing of naughts should be made mentally, or, we would 8imply -- 8 and extend the operation two places. 4. 3. Multiply 937104 by 25. OPERATION. 4) 93710400 Explanation.—We first multiply by 100, which is done by annexing gº two naughts to the multiplicand, and then divide by 4. T e º & & * fº 23427600 Ans. º reason for this work is the same as that given in the preceding NOTE.-In practice, divide by 4 and extend the operation two places. 4. Multiply 94.26 by 663. OPERATION. 3) 94.2600 Explanation.—We first multiply by 100, and then deduct one-third of 314200 the product from itself; the remainder is the correct result or product. The reason for this is, that as 663 is only # of 100, when we multiply by 100, we produce a product #'too great, and hence by deducting # of the product, 6284.00 we have in the remainder the correct result. Multiply the following numbers by the above methods: (5) (6) (7) (8) (9) (10) (11) 32 36 96 168 40 124 238 6# 8% 16# 62% 37; 125 112 200 300 1600 10500 1500 15500 26775 (12) (13) (14) (15) (16) (17) (18) 817 640 264 640 800 696 1215 375 625 875 75 18; 833% 1334 Multiply 321, 192, 248, 576, 643, and 764, by 24, 3}, 84, 124, 14%, 163, 18%, 25, 33%, 373, 62%, 75, 873, 1123, 125, 375, and 625. TO MULTIPLY BY 73, 174, 224, 274, 324, 15, 35, 45, AND 55. 170. 1. To multiply by 73, multiply by 10, and deduct # of the product. The reason of this is 73, being # of 10, multiplying by 10 gives a product # too large, therefore, by deducting 4 of the product by 10, we obtain the correct result. 2. To multiply by 174, multiply by 20, and deduct 4 of the product. The reason is 17; being # of 20, multiplying by 20, gives a product # too large. Hence, by deducting # of the product by 20, we produce the correct result. 3. To multiply by 22%, multiply by 20 and add 4 of the product to itself. 96 soule's PHILOSOPHIC PRACTICAL MATHEMATICs. * Considering the reasons given in the two preceding problems, this operation is clear, from the fact that 224 is ; more than 20. 4. To multiply by 27%, multiply by 30, and deduct fºr of the product. This operation is evident because 27# is +} of 30. 5. To multiply by 32% multiply by 30 and add #3 of the product. NOTE: 1.-324 is T', more than 30. NOTE 2.—There are two 24's in 5; three 24's in 74; four 24's in 10; six 24's in 15; eight 2% in 20; nine 23's in 224; eleven 23's in 27+, and thirteen 23's in 32%. 6. To multiply by 15, multiply by 10 and add 4 of the product to itself, or multiply by 30 and divide by 2. 7. To multiply by 35, 45 or 55, multiply respectively by 70, 90, and 110, and divide the product by 2; or halve the multiplicand and double the multiplier. NOTE.-This principle may be applied to 65, 75, 85, and 95, by parties who know the multi- cation table to 20 times 10. Multiply the following numbers: (1) (2) (3) (4) (5) 247 214 586 637 4826 15 35 45 55 15 74.10 7490 26370 70070 3705 35035 PRACTICAL OPERATIONS WITH ALIQUOTS AND OTHER NUMBERS. NotE.—Students who are not familiar with division and fractions, may omit such problems as involve this knowledge in their solution. 171. 1. What will 462 pounds cost at 25¢. Directions. Divide by 4, carrying the division two places for cents. 2. What will 840 yards cost at 1242 per yard? Directions. Divide by 8, carrying the division two places for cents. 3. What will 563 bushels cost at $1.25 per bushel? Directions. To the number of bushels add 4 of same, carrying the divis- ion two places for cents. Or, divide by 8, carrying the division three places. 4. What cost 1267 gallons at $1.6247 Directions. To the number of gallons add 3 and # of same, carrying the division two places for cents. 5. What cost 4102 bushels at $1.374? Directions. To the number of bushels add + and # of same, carrying the division two places for cents. 6. What cost 65 turkeys at $12.334 per dozen : Directions. Multiply the number of turkeys by 12, add to the product # the number of turkeys and divide the sum by 12, carrying the division two places for cents. PRACTICAL OPERATIONS WITH ALIQUOTS AND OTHER NUMBERS. 97 7. What cost 1183 pounds at $1.16% Directions. Add to the price # and #5 of itself. Or, add to the price # of itself plus # of the #. 8. What cost 375 boxes at $2.35% Directions. Multiply the price by 3 and divide the product by 8, carry- ing the division three places. 9. What cost 76 yards at $1.75% Directions. Multiply the number of yards by 2 and deduct # of product. NOTE. $1.75 is g of $2. 10. What cost 1428 pounds at 95% Directions. From the number of pounds deduct #6, carrying the divis- ion two places for cents. 11. What cost 50 barrels at $7.45? Directions. Divide the price by 2, carrying the division 2 places for cents. 12. What cost 84% gallons at 75% Directions. From the gallons, with the fraction expressed decimally, deduct # of the same. 13. What cost 2347 pounds corn at 374.2 per bushel ? Directions. Multiply the pounds by 3 and divide the product by 8, 7, and 8, carrying the division two places. 14. What cost 87 dozen at $1.20 per dozen” Directions. Add # of the number of dozens to the same, carrying the division two places for cents. * 15. What cost 625 pounds at $1.68 per pound? Directions. Multiply the price per pound by 5 and divide the product by 8, carrying the division three places. NOTE. 625 is # of 1000. • 16. What cost 83% dozen at $6.70 per dozen 7 Directions. Multiply the price by 5 and divide the product by 6, carry- ing the division two places. NotE. 83% is 3 of 100. 17. What cost 94 yards at 74% per yard? Directions. Annex 0 to the yards and deduct # of the product from itself. 18. What cost 95 yards at 1742 per yard 3 Directions. Multiply the number of yards by 20 and deduct 4 of the product from itself. NOTE. 174 is g of 20. 19. What cost 187; gallons at $3.42 per gallon ? Directions. Multiply the price by 200 and deduct # of the product from itself. Or, remove the decimal point two places to the right of the price, and add g of the amount to itself. Or, shorter still, multiply the price by 3 and divide the product by 16, carrying the division three places. NotE. 1874 is ſº of 1000. 20. What cost 250 barrels at $8.15 per barrel. Directions. Divide the price by 4, carrying the division three places. 98 SOULE's PHILOSOPHIC PRACTICAL MATHEMATICS. 4% 21. What cost 139 pounds at $1.16% per pound 3 Directions. To the number of pounds add # of the same, carrying the division two places for cents. 22. What cost 6871 yards at 242 per yard Directions. Divide the number of yards by 4, carrying the division one place and point off two. 23. What cost 433 yards at $1.1633 Directions. To the yards, with the fraction expressed decimally, add # of the result. 24. What cost 87.4 pounds at $2.65% Directions. From the price per pound omit decimal point, deduct # of same, carrying the division two places. Or, from the price deduct # and express the fractional remainder decimally. Thus, 265 – 33} = 231% = $231,874. 25. What cost 235 pounds at $1.80? Directions. Multiply the pounds by 2 and deduct #5 of the product, or # of the multiplicand from the product. 26. What cost 196 pounds at $1.75% Directions. Multiply the pounds by 2 and deduct # of the product from itself. Or, to the number of pounds add 4 and # of the same. 27. What cost 164 yards at 55g 3 Directions. To # the number of yards extended two places, add ºr of the #. 28. What cost 50 pounds at $1.29% Directions. Divide the price by 2, carrying the division two places for CentS. f 29. What cost 58 pounds at 97% Directions. Annex 2 naughts to the 58 and deduct 3 times 58 from the product. 30. What cost 8863 pounds of wheat at $1.15 per bushel ? Directions. Multiply the pounds by 2 and from the product deduct ºr of 8863 × 1.15 60 NOTE. $1.15 - 60 pounds per bushel = 14c. per pound. 31. What cost 6420 pounds corn at 49% per bushel? Directions. From the number of pounds deduct # and point off two fig- 6420 × 49 56 NOTE. # = {c. per pound. 32. What cost 582 gallons at $1.314 3 ' Directions. To the number of gallons add + and ſº (4 of the 4) of the the number of pounds. Or thus, ures. Or thus, gallons. 33. What cost 465 yards at $2.93%g 3 Directions. Multiply the number of yards by 3 and from the product deduct # of the number of yards. Yºr PRACTICAL OPERATIONS witH ALIQUOTS AND OTHER NUMBERS. 99 34. What cost 3840 pounds of hay at $183 per ton ? Directions. sº or point off three figures in the pounds and mul- tiply the pounds by # the price, or the price by 4 the pounds. NOTE.-In this case, when the price is in dollars there will be three decimals in the answer. When the price contains cents, there will be five decimals in the answer. 35. What cost 14675 pounds of oats at 482 per bushelf º e 14 4 Directions. gº tº by 2 and point off two places. 36. What cost 28760 pounds of barley at 52g per bushel? 28760 × 52 48 or multiply the pounds by 3, divide the product Directions. two places. 37. What cost 34 bushels and 24 pounds of corn meal at 75% per bushel? Directions. To # of the bushels add # of 75, thus, *so or to the pounds add +3 of the same and point off or thus, 34 at 75 = $25.50 $25.50 = cost of 34 bushels. .375 = “ “ , { { or 22 lbs. 24×75= .41 35 = ** ** 2 lbs. $25.91 practically. $25.91 practically. 38. What cost 475 lbs. oats at 36% per bushel? Directions. Since there are 32 lbs. in a bushel, mentally separate the price 36% into 32%-i-4g. Point off two places in 475 and then add of it to itself. Thus: 4.75 .593 $5.34% 39. What cost 1663 pounds of beans at $1.30 per bushel ? Directions. lºgº or 1663 + 62 = 26# bus. 26 bus. $1.30 = $33.80 $26. = 26 bus. at $1.00 31 lbs. .65 7.80 – 26 “ .30 15% “ say .33 or thus, .65 = 31 lbs. 4; “ “ . 9 .33 = 15% “ . 9 = 4; “ $34.87 practically. 40. What cost 2781 pounds of Small hominy at 64 cents per bushel ? © 1 & 2781 & º tº Directions. tºº. Or, since there are 50 pounds in a bushel and since the price is equal to 50 + 4 + +", of # of 50, therefore to the number of pounds, add # and #5 of # of the pounds, and point off two places. $34.87 practically. Thus: }) 27.81 Yº) 5.562 2.2248 $35,5968 : IOO SouLE's PHILOSOPHIC PRACTICAL MATHEMATICs. A. 41. What cost 473 bushels and 55 pounds of sweet potatoes at $1.40 per bushel? NOTE. 60 pounds make a bushel of sweet potatoes in New Orleans. Directions. 473x $1.40 = $662.20 to which add # of $1.40 which is $1.28 (# or # of $1.40) = $1.40–12 (fly of 140) = $1.28 + $662.20 = $663.48. Ans. or thus: To the bushels add # of themselves, and to this result add +} of $1.40 thus: 473. 94.60 94.60 1.28 $663.48 42. What cost 53 tons, 1670 pounds of hay at $16.25 per ton? 43. What cost 25 sacks, 3750 pounds corn at 63% per bushel ? Directions. Add to the weight # of itself. NotE. 63 is # more than 56, the number of pounds in a bushel of corn. 44. What cost 25 sacks, 4375 pounds of bran at 874.2 per cwt. ?" Directions. Subtract from the weight # of itself. 87 NotE. 873c. for 100 pounds is #= #c. per pound = #c. less than 1c. per pound. 45. What cost 30 quarter bales, 3214 pounds hay at $15 per ton? Directions. Subtract from the weight # of itself. NOTE. $15 per ton is #33 = #c. per pound. 46. What cost 176 sacks, 35220 pounds oats at 374.2 per bushel ? Directions. Add to the pounds # of itself, # of the # and # of the #. NotE. Since 32 pounds make a bushel, 37} is #, + of the #, and # of the + more than 32. 47. What cost 35220 pounds, 1100% bushels oats at 38% per bushel? Directions. Add to the weight 4 of itself, and 4 of the #. Or, add to the weight (38-32) = 62 for each bushel, and 42 (3%) for 20 pounds (3) 62 per bushel. The above problems should be carefully noted by the student, as they contain many points that will aid him in becoming a rapid calculator. In all problems of multiplication, a quick, critical view should be given to the factors, the price and the quantity, to see what advantages may be taken or what short combinations may be made with them as aliquots, or as approximations to hun- dreds or easy fractions of hundreds, or in consequence of their family properties as tens, complements, supplements, etc. By so doing, the mind is made acute and great facility is acquired in handling numbers. The table of aliquots should be thoroughly understood. There is almost no end to the various combinations that may be used in the operations of multiplication, Particular attention is invited to the methods follow- ing, wherein numbeºs are glassed into families and where each family possesses some e *q → Ç º £ : © *e Yºr CONTRACTIONS IN MULTIPLICATION. IOI individuality of relation to the principles of numbers by which the operations of ‘multiplication may be contracted. It is often convenient to consider the price as the quantity, and the quantity as , the price, before making the multiplication, as indicated in the directions for work- ing several of the above problems. For further work of this character, see the miscellaneous problems following Denominate Numbers. TO MULTIPLY WEHEN TELE MULTIPLIEE IS A MIXED NUMBER, WEHOSE FRACTIONAL PART IS ONE FRACTIONAL UNIT LESS THAN * A WHOLE NUMBER. 172. 1. Multiply 243 by 113. OPERATION. 243 Explanation.—Multiply by 12, and deduct from the product 11% + of the multiplicand. The reason for this is clear, for when we 2916 multiplied by 12 we repeated the multiplicand # time more than the 60; problem requires, and hence the deduction of one-fourth thereof gives the correct product. 2855#. PROBLEMIS. Multiply the following problems in like manner: (2) (3) (4) (5) (6) (7) (8) 358 149 87 246 • 892 675 486 7; 53 9% 14: 24; 68; 37+} TO MULTIPLY BY ANY NUMBER BETWEEN EIGHTY-EIGHT AND ONE HUNDRED, OR NINE HUNDRED AND EIGHTY-EIGHT AND ONE TEIOUSAND. 173. 1. Multiply 216 by 98. OPERATION. Explanation–In all problems of this kind, multiply by 100 and 0 0 then deduct as many times the multiplicand as the multiplier is lºë 216 than 100. In this problem, we first multiply the 216 by 100, 432 which is done by annexing two naughts; this gives us a product 2 times 216 too great, for the reason that 100 is two more than 21168 A. 98. Hence, to produce the correct result, we deduct 2 times IlS. 216 which is 432 from the product by 100, and in the remainder we have the correct product. 2. Multiply 4268 by 997. OPERATION. Explanation.—In all problems of this kind, multiply by 2000 and then deduct as many times the multiplicand as the multiplier is 4268000 less than 1000. In this problem, we first multiply the .4268 by 12804 1000, which is done by annexing three naughts; this gives us a mºmºmºmºmºse product 3 times 4268 too great, for the reason that 1000 is three 4255196 A. more than 997. Hence, to produce the correct result, we deduct IlS. 3 times 4268, which is 12804, from the product by 1000, and in the remainder we have the correct product. I O2 SOULE's PHILOSOPHIC PRACTICAL MATHEMATICS. * Multiply the following numbers in the same manner: 3. 342 by 94 6. 528 by 93 9. 3284 by 995 12. 860 by 89 4, 684 ($ 88 7. 763 “ 91 10. 4009 {{ 989 13. 498 & 92 5. 507 4 91 8. 842 {{ 99 11. 3864 ($ 996 14. 6407 & 988 TO MULTIPLY BY ANY NUMBER BETWEEN ONE HUNDRED AND ONE HUNDRED AND THIRTEEN, AND ONE THOUSAND AND ONE TEIOUSAND AND THIRTEEN. 174. 1. Multiply 5239 by 107. OPERATION. Explanation.—In all problems of this kind, first multiply by 523900 100, then add to the product thus obtained, as many times the 36673 multiplicand as the multiplier is greater than 100. The reason for this is obvious, for the multiplicand is to be 560573 Ans increased 107 times, and by annexing two 0's we increase it 100 times, to which product we add the product of the multiplicand by the 7, and thus repeat the multiplicand 107 times. 2. Multiply 2871 by 1005. OPERATION. Explanation.-In all problems of this kind, first multiply by 28710 00 1000, then add to the product thus obtained, as many times the 14355 multiplicand as the multiplier is greater than 1000. NOTE: 1.-In the foregoing problems, small naughts have been annexed, in order to show more clearly the operations of 2885355 Ans. the work. NOTE: 2. —The operations of the two preceding methods may be extended to the extent of the students’ knowledge of the multiplication table. Multiply the following numbers in the same manner: 3. 2564 by 103 6. 521 by 1001 9. 48071 by 1007 4. 809 44 106 7. 1685 4 1006 10. 2910 & 111 5. 4712 € $ 112 8. 2364 ($ 1012 11. 17036 44 1008 TO MULTIPLY BY ANY NUMBER BETWEEN TWELVE AND TWENTY. 175. 1. Multiply 564 by 13. * OPERATION. 1692 Daplanation.-In all problems of this kind, multipl 3. º OI’ * OT º; 13 by the unit figure of the multiplier and write the . ºmºmºmºmº gº cºmme over or under the multiplicand, one place to the right, and 7332 1692 7332 add same to the multiplicand. *º-º-º-º-º: By this means, we save multiplying by the tens' figure 7332 of the multiplier. NOTE.-This method of arranging the figures may also be used to advantage when the multi- plier is any number similar to the following: 106 or 1004. In such cases, the products would be placed two places to the right for hundreds, and three, for thousands. ¥ CONTRACTIONS IN MULTIPLICATION. IO3 Multiply the following numbers as above: 2. 681 by 14 5. 1269 by 16 8. 728 by 13 11. 286 by 19 3. 754 £6 17 6. 5028 4: 18 9. 1407 4 15 12. 1675 4 17 4. 425 ($ 106 7. 1232 ($ 108 10, 3416 44 1004 13. 4062 ($ 1005 TO MULTIPLY BY ANY NUMBER OF TWO FIGURES WHICH ENDS IN ONE. 176. 1. Multiply 452 by 41. OPERATION. te 1808 Explanation.—In all problems of this kind, multiply 452 or 452 or 452 × 41 by the tens' figure of the multiplier and write the product 41 41 1808 over or under the multiplicand one place to the left and -ºsmºs-smºs add same to the multiplicand. 18532 1808 18532 By this means, we save multiplying by the units' figure gº-º-º: of the multiplier. 18532 NOTE 1.--This method of arranging the figures may also be used to advantage when the multi- plier is any number similar to the following: 601 or 4001. In such cases, the products would be placed two places to the left for hundreds, and three, for thousands. NOTE 2.-In all these contracted methods, the operation should be repeatcd several times on each problem. PROBLEMIS. Multiply the following numbers as above: 2. 648 by 31 5. 4235 by 51 8, 2890 by 61 11. 9027 by 71 3. 872 ($ 41 6. 2806 ($ 81 9. 5075 (€ 91 12. 2364 “ 111 4, 425 ($ 501 7. 1242 {{ 601 10. 6322 4° 4001 13. 4327 4 7001 TO MULTIPLY BY THE FACTOR'S OF THE MULTIPLIER. 177. Multiply 8421 by 64. OPERATION. 8421 Ea:planation.—Instead of multiplying by 64, we use the factors 8 and 64 - 8; first multiplying by 8 we produce a product of 67368, which we multi- e- ply by 8, and thus produce the correct result. By this system of work, we 67368 save one line of figures and the addition of two lines. S age 84 for further explanations. 538944 Ans. ee pag p PROBLEMS. Multiply the following numbers as above: 2. 2407 by 32 4. 796 by 63 6. 527 by 48 8, 2840 by 56 3. 682 4° 45 5, 1428 4 36 7. 290 46 72 9. 7065 (€ 49 IO4. soule's PHILOSOPHIC PRACTICAL MATHEMATICs. 4× TO MULTIPLY ANY NUMBER BY ELEVEN OR ONE EIUNDRED AND ELEVEN. 178. 1. Multiply 62 by 11. OPERATION. Erplanation.—In multiplying any number by 11, it is evident that we have but to add the number to its decuple, i. e. to ten times the 682 A number, hence in this and all operations of two figures we have but IlS. to add the two figures together and place the sum between them. When the sum of the two figures is in excess of 9, carry one to the left hand figure. Thus multiply 87 by 11; 8 and 7 added make 15; place the 5 between the figures and add the 1 to the 8 which gives a product of 957. 2. Multiply 3456 by 11. OPERATION. 3456 Explanation.—In this problem, according to the principle stated 11 in the first problem, we add the number to its decuple, thus 0-H. 6 = 6; 6 + 5 = 11; 5 + 4 + 1 to carry = 10; 4 + 3 + 1 to carry = 8; 38016 then bring down the 3, and we have the correct product. 3. Multiply 3456 by 111. OPERATION. Explanation.-In this problem, we are required to repeat 3456 100 times plus 10 times, plus 1 time. Hence, if we add 3456 to 10 3456 times 3456 and 100 times 3456 we shall have the correct result. This 111 we do by bringing down the first figure 6, and adding as follows: 6 plus 5 = 11; 6 plus 1 to carry, plus 5 plus 4 = 16; 5 plus 1 to & carry, plus 4 plus 3 = 13; 4 plus 1 to carry, plus 3 = 8; and then 383616 bring down the 3. By performing these operations in the usual & manner, the work will appear plain. Multiply the following numbers as above: 4. 463 by 11 6. 245 by 111 8. 823 by 111 10. 2407 by 111 5. 386 ‘‘ 11 7. 741 & 11 9, 568 46 11 11. 14586 “ 1111 TO MULTIPLY BY ANY NUMBER ONE PART OF WHICH Is A FACTOR OF ANOTHER PART. 179. 1. Multiply 3246 by 328. OPERATION. 3246 Ea:planation.—The conditions of this problem are, that we take or 328 repeat the multiplicand 328 times. In performing the operation, we tºº, first multiply by 8, which repeats it 8 times; we yet have it to repeat 25968 320 times, and as 320 is equal to 8 40 times, therefore, 40 times the pro- 1038720 duct by 8 (25968) is the same as 320 times the multiplicand; hence, to eme shorten the operation, we add to the product by 8, 40 times itself. 1064688 The sum of these two products is the correct product. 2. Multiply 7251 by 618. OPERATION. Ea:planation.—In this problem, we first multiply by the 6 hundreds "; which repeats the 7251 600 times, and leaves it unrepeated 18 times. 1 To repeat it 18 times more, we observe that 18 is equal to 6, 3 times, 4350600 and having already repeated the 7251 600 times, we therefore repeat 130518 the one hundredth part of the product by the six hundreds 3 times, which is equal to 18 times 7251, and add the same to the product by 6 hundreds. The result is the correct product. 4481118 * - CONTRACTIONS IN MULTIPLICATION. IO5 3. Multiply 536183 by 27945. FIRST OPERATION. 536183 -- 27945 Eacplanation.—In thi bl re Ob that the first two fi a planation.—In this problem, we observe that the first two figures 48.2564700 45, are equal to the hundreds' figure 5 times, and that the thousands' 1447694.1000 and ten thousands' figures 27, are equal to the hundreds' figure 3 times. 24128235 We therefore multiply first by the hundreds' figure 9, which repeats the 14983 638935 multiplicand 900 times. We have yet to repeat it 27045 times, and SECOND OPERATION. (using the 27000 first), since 27000 is equal to 9 3000 times, we therefore Omitting the naughts. *P* the product by 9 3000 times, instead of the multiplicand 27000 536183 times, which gives us 14476941000; and since 45 is equal to 9 five times, 27945 we therefore repeat the one hundredth part of the product by the nine *=ºmºeºsºmmºn hundreds 5 times, instead of repeating the multiplicand 45 times, which 1.# gives us 24128235. We then add the several products together, the sum 24128235 of which is the true product. 14983633935 TO MULTIPLY BY ANY NUMBER OF TWO FIGURES ENDING WITH NINE. 180. 1. Multiply 563 by 39. OPERATION. 563 563 x 40 Explanation.—In all problems of this kind, in- 39 22520 crease the multiplier by 1 and from the product sub- OI’ e - © º e tract the multiplicand. In this problem we multi- 22520 21957 Ans. ply by 40, which gives a product of 22520, from 21957 Ans. which we subtract 563 and obtain 21957, the answer. PROBLEMIS. Multiply the following numbers as above. 2. 726 by 29 4. 864 by 49 6. 627 by 59 8. 9081 by 69 . 3. 683 (4 79 5. 927 & 89 7. 1074 “ 19 9, 23.45 % 99 TO MULTIPLY ANY TWO NUMBERS OF TWO OR THREE FIGURES EACH, WHEN THE TENS AND HUNDREDS ARE ALIKE AND THE SUM OF THE UNITS IS TEN. 181. 1. Multiply 86 by 84. OPERATION. Explanation.—In all problems of this kind, first multiply the units' 86 figures and write the whole result, then add one to the multiplier of 84 the tens, and multiply the other tens, or tens and hundreds, by it, and prefix the result to the product of the units' figures. In this problem, we first multiply 4 times 6 are 24, which we set. Then add 1 to the 8 7224 of the tens' column, making it 9; we multiply 9 times 8 are 72, which is set to the left of the 24 and produces the correct product. I oé SOULE's PHILOSOPHIC PRACTICAL MATHEMATICS. - * 2. Multiply 132 by 138. OPERATION. Explanation.--We first multiply 8 times 2 are 16, which constitutes 132 the units’ and tens' figures of the product. We then add one to 13, 138 making it 14, and multiply 14 times 13 are 182, which completes the product. Or, after adding the 1 to the 13, we may multiply 14 times 3 are 42; set the 2 and multiply 14 times 1 plus 4 to carry, are 18. 18216 NoTE 1.--When the multiplication of the units' figures does not give a product of two figures, the tens’ place must be filled with a 0. NoTE 2.-The reason for this work is that since the sum of the units’ figures is 10, and the tens' figures are alike, it is plain that the sum of the products of the tens by the units and the units by the tens, is equal to 10 times the tens’ figure, and since its first figure occupying the second place is naught it does not change the original product of the units, and since the second or carrying figure of the units by the tens is always the same as the tens of the factors, we therefore add 1 to either factor and multiply by the other for the final product. Multiply the following numbers as above: (3) (4) (5) (6) (7) (8) (9) (10) 38 61 223 149 326 417 311 208 32 69 227 141 324. 413 319 202 1216 4209 50621 21009 105624. 172221 (11) (12) (13) (14) (15) (16) (17) (18) (19) 29 47 81 91 126 206 164 102 113 21 43 89 99 124 204 166 108 117 essm -sºme mºm a-m mºm- ºmmº- mºmmam -m-m-m-, *- TO MULTIPLY TWO NUMBERS OF TWO FIGURES EACH WHEN THE TENS7 FIGURES ADD TEN AND THE UNITS? FIGURES ARE ALIKE. 182. 1. Multiply 47 by 67. OPERATION. Ea:planation.—In all problems of this kind, first multiply the units’ 47 figures and set the whole result in the product line, then multiply the 67 tens' figures and add to their product one of the units' figures, and prefix the result to the product of the units' figures. In problem 1, we first multiply 7 times 7 = 49; then 6 times 4 = 24, to which we add 7 3149 Ans. and obtain 31. Problems 2, 3, 4, and 5, were similarly worked. NOTE-When the product of the units' figures is but one figure, the tens' place in the product must be filled with a 0. Multiply the following numbers as above: (2) (3) (4) (5) (6) (7) (8) (9) (10) (11) 84 73 96 41 78 24 98 46 42 77 24 33 16 61 38 84 18 66 62 37 *-* * " - - - - - 2016 2409 1536 2501 M. CONTRACTIONS IN MULTIPLICATION. Io? TO MULTIPLY TWO NUMBERS WHEN THE UNITS? FIGURES ARE FIVES AND THE SUM OF THE TENS, FIGURES IS MORE OR LESS THAN TEN. 183. 1. Multiply 65 × 85. 65 OPERATION. Explanation. First, 5 × 5 = 25 units. 85 Second, 8 × 6 = 48 hundreds, to which add — 5 × 5 = 25. 8t. × 6t.= 48h.-- # of 4 of the sum of the tens' figures, 8 + 6 - 2 = 7, 5525 (8+6) = 7h.= 55h. or 5525. = 55 hundreds. - 2. Multiply 45 × 35, 45 OPERATION. Explanation. First, 5 × 5 = 25 units. 35 Second, 3 × 4 = 12 hundreds, to which add — 5 x 5 = 25. 3t. × 4t. = 12h. x 4 + of the sum of the tens' figures, 3 + 4 + 2 = 3 1575 of (3 + 4).3%h.-- 12h. = 1550 + hundreds 50 units, = 1550 + the 25 units first the 25 = 1575. obtained = 1575 product. 45 SECOND OPERATION. Explanation.—In this solution, since the sum 35 of the tens’ figures is odd, we first find the pro- duct of the tens' figures to which we add the pro- 3 x 4 = 12h.-- # of (3 + 4) = 3h., duct of the units figures and thus produce the * 1575 º final product. This method may be used in all 50 units = 1550 + 5 × 5,- 1575. cases, if-desired. PROBLEMIS. Multiply the following problems as above : (3) (4) (5) (6) (7) (8) (9) (10) (11) 85 75 45 105 165 115 175 225 185 25 65 95 85 95 115 15 35 145 a- - - - - - - - - TO MULTIPLY TWO NUMBERS OF TWO OR THREE FIGURES EACH, WHEN THE HUNDREDS AND TENS, OR THE UNITS OR TENS FIGURES ONLY, ARE ALIKE. 184. Thus, multiply the following numbers: (1) . (2) (3) (4) 54 87 124 116 34 82 123 146 1836 Ans. 7134 15252 16936 Ans. Ea:planation.—In all problems of this kind, first multiply the unit figures and write the unit result in the product line, then add the column of units or tens, that is not alike, or both, where three figures are used, and with the sum multiply one of the numbers in the column, where they are I O3 SOULE's PHILOSOPHIC PRACTICAL MATHEMATICs. Yºr alike, and add to it the carrying figure; then multiply the figures in the tens or tens and hundreds, and add the carrying figure; the result will be the correct product. In problem 1, we first multiply 4 times 4 = 16; the 6 we set and carry 1; then 3 plus 5 = 8, and 8 times 4 = 32 plus 1 to carry make 33; the 3 we set and carry 3; then 3 times 5 = 15 plus 3 to carry make 18. In problem 2, we first multiply 2 times 7 = 14; then 7 plus 2 = 9 and 9 times 8 =72 plus 1 to carry = 73; then 8 times 8 = 64 plus 7 to carry =71. Problems 3 and 4 are similarly worked. PROBLEMIS. Multiply the following numbers as above: (5) (6) (7) (8) (9) (10) (11) (12) (13) 73 65 106 45 123 89 154 93 146 23 64 107 85 113 85 154 97 106 sºm-º dº tºº ºmºmº ºmºmºmºm sºmmºn sºmmºn sº- tºmsºmºsº TO MULTIPLY BY THE COMPLEMENTS AND SUPPLEMENTS OF NUMBERS. [See page 26 for a definition of Complements and Supplements of Numbers]. 185. The following operations involving the complements and supplements of numbers are deduced from Algebraic principles, and while these principles are easily applied to Arithmetical Computations, the reasons therefor cannot be fully com- prehended without a knowledge of Algebra, and hence are not given. TO MULTIPLY TWO NUMBERS, BOTH OF WHICH ARE CONVEN- IENT NUMBERS UNDER ONE HUNDRED, ONE THOUSAND, ETC. 186. 1. Multiply 97 by 94. OPERATION. 97 (3 Complement of 97) ..º.º. all problems of this kind, 1st multiply 6 C l t of 94 the complements of the two numbers and write the result in 94 (6 Complement o ) the product line. 2. Subtract the complement of either &=º number from the other number and prefix the remainder to 91.18 the product of the complements. Or, add the multiplicand and multiplier, rejecting 1 at the left hand of the sum. NOTE. 1.--When the product of the complements does not contain as many figures as there are naughts in the order, 100, 1000, etc., to which the numbers belong, prefix naughts to supply the deficiency. See Problems 2 and 3. NOTE 2.—When the number of figures in the product of the complements exceeds the naughts in the order, 100 and 1000, etc., to which the numbers belong, add the excess, or left hand figure to the result obtained by subtracting the complement of either number from the other number. See Problems 4 and 5. * CONTRACTIONS IN MULTIPLICATION. IO9 2. Multiply 98 by 97. 3. Multiply 991 by 989. OPERATION. Eacplanation. 1st. 3 times OPERATION. Ea:planation. 1st. 98 (2 Comp.) 2=6, which we write in the 991 (9 Comp. of 991.) 11 times 9 = 99, 97 (3 Comp.) product line and prefix a 989 (11 Comp. of 989.) which is written Inaught as directed in Note 1. in the product line 2d. 3 from 98 = 95, which and a naught pre- pletes the product. - Note 1. 2d. 9 from 989 = 980, which is prefixed to the 099 and completes the product. 5. Multiply 978 by 940. -ms-ºn- 4. Multiply 89 by 88. OPERATION. Ea:planation. 1st. 12 times OPERATION. Explanation. 1st. 60 89 (11 Comp.) 11 =132, 32 of which is writ- 978 (22 Comp.) times *; = 1320, 320 of 88 (12 Comp.) ten in the product line, as di- 940 (60 Comp.) which is written in the rººd * . º 77-H1 product line as directed in e OIO &j = - Note 2. 7832 Ans. , 78, which is prefixed to the 919320 Ans. 2d. 60 from 978 =918-1- 32 and completes the product. 1 =919, which is prefixed to the 320 - and completes the product. PROBLEMS. Multiply the following numbers as above: (6) (7) (8) (9) (10) (11) (12) (13) (14) (15) 94 92 88 82 99 73 97 74 85 99 93 93 88 91 96 98 62 85 91 99 (16) (17) (18) (19) (20) (21) (22) (23) 996 988 991 969 950 940 965 999 994 997 985 985 988 930 987 999 990024 9.85036 976135 TO MULTIPLY TWO NUMBERS, WHOSE COMBINATION IS SUCH THAT THE PRODUCT OF THE COMPLEMENT OF THE UNITS, FIGURE OF THE MULTIPLIER, BY THE TENS FIGURE OF THE MULTIPLICAND IS THE SAME AS THE PRODUCT OF THE TENS” FIGURE OF THE MULTIPLEER BY THE UNITS’ FIGURE OF THE MULTIPLICAND. 187. 1. Multiply 48 by 34. OPERATION. Explanation.—In this problem, the complement of the unit figure of 48 the multiplier is 6, and 6 times the tens' figure of the multiplicand is 24; 34 which is the same as the product of the tens' figure of the multiplier, and the unit figure of the multiplicand. In all cases of this kind, we multiply gººm-º: the units' figures and write the result in the product line. Then we add 1 1632 Ans. to the tens’ figure of the multiplier, and with the sum multiply the tens' figure, or tens and hundreds of the multiplicand and prefix the result to the product of the units' figures. In this problem, we say 4 times 8 are 32; 4 times 4 are 16. NotE.—Any multiple of 11, may be multiplied as above when the figures of the multiplier add 10. I IO SOULE's PHILOSOPHIC PRACTICAL MATHEMATICS. M. PROBLEMS. 2. Multiply the following numbers as above: (2) (3) (4) (5) (6) (7) (8) (9) (10) (11) (12) (13) 63 32 88 146 128 42 96 147 96 84 44 55 29 94. 73 77 94 86 38 124 66 67 73 82 TO MULTIPLY TWO NUMBERs, BOTH OF WHICH ARE CONVEN- IENT NUMBERS UNDER ANY NUMBER OF EIUNDREDS. 188. 1. Multiply 497 by 294. OPERATION. Explanation. — In this problem, we first multiply 3 C I t; Of 9 the complements of the multiplicand and the 30 500) 497 & Complement of 97 multiplier and set the result 18, in the product line. 9 U (300) 294 6 Complement of 94. (Where this result is but one figure, the tens' place is - filled with a 0. When the result is more than two figures, 39 the hundreds' figure is added to the hundreds' column of the product). 2. We mentally add the complement fig- 146118 ures to their respective numbers, and thus produce 500 and 300 as shown in the parentheses on the left. 3. We multiply the 5 of the 500, by the complement of the multiplier, and the 3 of the 300, by the com- plement of the multiplicand and add the products, which gives 39, as shown by the number within the brace on the left of the operation. 4. We subtract the 39 from 100 and thus obtain 61 which is prefixed to the 18 in the product line. 5. We multiply 5 and 3, the significant figures of 500 and 300, and thus obtain 15, from which we subtract 1, the number of hundreds used when the 39 was subtracted, and prefix the remainder 14, to the 6118, which completes the product. All similar numbers of hundreds and thousands may be multiplied in a similar manner. NOTE: 1.—The figures shown within the brace and parentheses should be mentally made. NOTE: 2.-When the sum of the products of the increased hundreds' figures by the comple- ments, in that part of the operation where the 39 was produced, exceeds 100, then subtract the sum from 200, and from the product of the increased significant figures of the multiplicand and multi- plier, deduct 2. PROBLEMS. 2. Multiply 595 by 393. 3. Multiply 698 by 497. OPERATION. Elaplanation by figures. OPERATION. (6) 595 (5) 7 times 5 = 35. (7) 698 (2) (4) 393 (7) 7 times 6 + 5 times 4 = 62. (5) 497 (3) 62 from 100 = 38. 233835 Ans. 4 times 6 — 1 = 23. 346906 Ans. Multiply the following numbers as above: (4) (5) (6) (7) (8) (9) (10) (11) (9) 891 (9) 291 892 991 394 791 392 488 (6) 592 (8) 295 596 892 592 495 788 692 (12) (13) (14) (15) (16) (17) (18) (19) 493 692 790 392 488 598 691 894 395 597 691 593 489 496 394 389 Yºr CONTRACTIONS IN MULTIPLICATION. I I I TO MULTIPLY TWO NUMBERS, BOTH OF WHICH ARE CONVEN- IENT NUMBERS AROVE, ANY NUMBER OF HUNDREDS. 189. 1. Multiply 704 by 305. OPERATION. Explanation.—In this problem, we first multiply the supplement figures 4 and 5, and set the result 20 in the product line. 2. We multiply the 704. hundreds' figures of each number by the supplement figure of the other, 305 and add the two products thus, 5 times 7 = 35; 4 times 3 = 12; 35 and 12 = 47; this 47 we prefix to the 20 in the product line. 3. We multiply the 2 hundreds' figures and obtain 21, which we write in the product and com- 14,720 plete the operation. All similar numbers of hundreds and thousands may be similarly worked. NOTE. 1.--When the product of the units’ or supplement figures do not give two figures, fill the tens' place with a 0. See Problem 2. When this product exceeds 2 figures, add the hundreds” figure to the hundreds' column of the product. See Problem 3. NOTE: 2. —When the sum of the products of the hundreds' figures by the units' figures is not two figures, fill the thousands' place in the product with a 0. See Problem 4. When this sum exceeds 2 figures, add the ten thousands to the ten thousands' column of the product. See Problem 5. PROBLEMS. (2) (3) (4) (5) (6) (7) (8) 301 312 501 912 813 612 1206 202 209 301 811 206 509 911 60802 6520s 150801 739632 - - (9) (10) (11) (12) (13) (14) (15) 406 612 308 810 511 203 1412 507 109 702 305 406 202 1211 TO MULTIPLY TWO NUMBERS, BOTH OF WHICH ARE CONVEN- IENT NUMBERS OVER ONE HUNDRED, ONE THOUSAND. ETC. 190. 1. Multiply 108 by 104. OPERATION. Explanation—In all problems of this kind, first write the product of the supplement figures in the product line. 2. Cancel, mentally, I08 (8 Supplement.) one of the left hand figures in one of the numbers, add the remain- 104 (4 Supplement.) ing figures and the carrying figure, if any, and prefix the sum to the product of the supplement figures. Thus, 4 × 8 = 32; then cancel 1123 1 in the 108 and add 8 to 104 = 112, which is prefixed to the 32 and 232 Ans. completes the product. Or, first cancel the left hand figure in one of the numbers, then add the remaining figures and annex thereto the product of the supplements of the two numbers. NOTE 1.-In case of hundreds, when the product of the supplement numbers does not contain two figures, fill the tens' place with a 0. In case of thousands, when the product or the supplement numbers does not contain three figures, fill the hundreds' place or the tens' and hundreds' places with 0's. NoTE 2.—In case of hundreds, when the product of the supplement numbers contains more *:::::::: figures, add the hundreds to the hundreds' column, when adding the multiplicand and multiplier. In case of thousands when the product of the supplement numbers contains more than three #; add the thousands to the thousands' column or place, when adding the multiplicand and multiplier. II 2 SOULE's PHILOSOPHIC PRACTICAL MATHEMATICS. X- 2. Multiply 1014 by 1009. OPERATION. Explanation.—Here we first multiply the supplement numbers and write the result in the product line. 2. Cancel, mentally, the left hand figure 1014 in one of the numbers, add the remaining figures, and the carrying fig- 1009 ure, if any, and prefix the sum to the product by the supplement figures. Thus, in this problem, 9 × 14 = 126; then cancel the left hand 1 in the 1023126 Ans 1014, add 14 to 1009, and prefix the sum, 1023, to the product by the Sup- g plement numbers. NOTE: 1.—When the product of the supplement numbers does not contain three figures, fill the hundreds' place, or the hundreds' and tens', with 0s. See Problem 4. NoTE 2. When the product of the supplement numbers contains more than three figures, add the thousands to the thousands' column. For the higher numbers, as 10000, etc., the same principle maintains. See Problem 5. PROBLEMIS. Multiply the following numbers as above: (3) (4) (5) (6) (7) (8) 103 1004. 1036 116 121 1342 102 1001 1040 111 104 1015 10506 1005004 1077.440 12876 12584 1862130 (9) (10) (11) (12) (13) (14) (15) 103 114 109 1008 1024 1135 1037 108 112 112 1007 1016 1010 1040 TO MULTIPLY TWO NUMBERS, ONE OF WHICH IS A CONVENIENT NUMBER OVER, AND THE OTHER A CONVENIENT NUMBER UNDER ONE HUNDRED, ONE THOUSAND, ETC. 191. 1. Multiply 105 by 93. OPERATION. º . problems § this º first multiply the Complement and Supplement numbers and subtract the produ 105 (5 Supplement.) from 100, or 1000, . as the numbers multiplied tºº. to †. 93 (7 Complement.) order of hundreds or thousands, and write the remainder in the pro. duct life, as the first two figures of the product. 2. Cancel the 9765 left hand 1 in the multiplicand, add the remaining figures of the Same, from which sum subtract. 1, or the number of hundreds used in subtracting the product of the complement and supplement fig- ures, and prefix the remainder to the product first obtained. Thus, in this problem, we say, 7x5 = 35; 35 from 100 =63, which is written in the product line. Then cancel, mentally, the left hand 1, we add 93 and 5 = 98, less 1, for the hundred used in subtracting the 35 = 97, which is prefixed to the 65, and completes the product. NoTE 1. . When the remainder, resulting from the subtraction of the products of the comple- ment and supplement numbers does not contain two figures, then fill the tens' place with a 0. See Problem 2. NOTE 2.--When the product by the complement and supplement figures exceeds 100, or 1000, then subtract the same from 200 or 300, or 2000 or 3000, according as the numbers multiplied are of the hundreds' or thousands' orders, and then, from the sum of the added figures of the multiplicand and multiplier, subtract as many as you used hundreds or thousands. See Problems 2 and 3. CONTRACTIONS IN MULTIPLICATION. II 3 Multiply the following numbers as above : (2) (3) MENTAL OPERATION.—6 × 223 = 112 MENTAL OPERATION.—11X12 = 132; 1223 1338; 2000–1338 = 662. Then 4+3 89 200–132=68. Then 9-1-2 = 11; 11–2 994. Tłº, º º e Il "TV E 12. — = 9. Then 8 + 1 = 9. Left hand 1 is *mºnºsmº tº Left hand figure of iºnisand 9968 canceled. 1215662 is canceled. (4) (5) (6) (7) (8) (9) (10) (11) 116 1012 133 1023 125 123 1013 1123 97 988 85 997 96 92 996 988 11252 999856 11305 1019931 12000 11316 1008948 1109524 (12) (13) (14) (15) (16) (17) (18) (19) (20) 108 1012 1104 124 111 109 1018 1201. 1302 95 990 988 96 88 89 940 996 975 *E* **E= *==* *E-º-º: sº *E---> º-º-º-º-º: *==sº * tº-º-º-º-º: A PECULLAR PROPERTY OF THE NUMBER NINE. 192. If we multiply the 9 figures in their order: 1, 2, 3, 4, 5, 6, 7, 8, 9, by 9, or any multiple of 9, not exceeding 9 times 9, the product will be in like figures, except the tens' place, which will be a 0. The significant figure of the product will be the quotient of the multiplier divided by 9. Thus, to multiply by 9, will give a product of 1s; 18, which is two times 9, will give a product of 2s, and so on with the other multiples of 9, up to 81. OPERATIONS. (1) (2) (3) (4) 123456789 123456789 123456789 123456789 9 27 45 72 1111111101 3333333303 5555555.505 88888.88808 If we omit the 8 in the multiplicand the product figures will all be the same. There are many peculiar properties belonging to the figure nine, by reason of its being 1 less than the radix of our system of notation, but being mostly of no practical value, we cannot give them space. There is, however, one property of the nine, that may often be used with advantage by accountants, in the detection of errors in posting or transferring accounts, and we will state it. It is as follows: That the difference between any number, and the figures composing the num- ber reversed in any manner, is a multiple of 9. Thus, the difference between 6871, and any transposition or reversing possible to be made with the same figures, will be a multiple of 9. EXAMPLES. 6871 6871 6871 6871 6871 6871 7861 1786 6718 6781 8671. 8167 9) 990 9) 5085 9) 153 9) 90 9) 1800 9) 1296 110 565 17 10 200 144 NotE.—See Expert Accounting, in Soulé's New Science and Practice of Accounts, for an extended application of the Properties of 9's and 11's, to the finding of Errors in Accounts. Also, see the Properties of 9's and 11's following Division in this book. II.4. SOULE's PHILOSOPHIC PRACTICAL MATHEMATICS. * ALGEBRAIC FORMUL.AE FROM WPHICH ARE DEDUCED THE OPERA- TIONS OF MULTIPLYING COMPLEMENT AND SUPPLEMENT NUMBERS. 193. (See Problem, page 105). (See Problem, page 106). 86 = 90 — 4 47 = 50 — 3 84 = 90 – 6 67 = 50 + 17 7224 90 × 90 – 4 × 90 3149 50 × 50 — 3 × 50 — 6 × 90 + 4 × 6 + 17 × 50 — 3 × 17 8100 — 10 × 90 –H 24 = 2500 + 14 × 50 — 51 = 8100–900 + 24 = 7224 2500 + 700 — 51 = 3149 (See Problem, page 107). (See Problem, page 108). 65 = 60 + 5 97 = 100 — 3 85 = 80 -H 5 94 = 100 – 6 5525 60 × 80′ + 5 × 80 9118 100 × 100 — 3 × 100 + 5 × 60 + 5 × 5 4800 + 400 + 300 + 25 = 5525 (See Problem, page 109). 48 = 50 — 2 - 34 = 50 — 16 1632 50 × 50 — 2 × 50 — 16 × 50 + 2 × 16 2500 — 18 × 50 + 32 = 2500–900 + 32 = 1632 (See Problem, page 111). 704 = 700 + 4 305 = 300 + 5 214720 700 x 300 + 4 × 300 + 5 × 700 + 4 × 5 210000+1200+3500–H20=214.720 (See Problem, page 112). 105 = 100 + 5 93 = 100 – 7 9765 100 x 100 + 5 × 100 — 7 × 100 — 5 × 7 10000–2 × 100 — 35 = 10000 — 200 — 35 = 9765 — 6 × 100 + 3 × 6 10000 – 9 × 100 + 18 = 10000–900 + 18 =. 9118 (See Problem, page 110). 497 = 500 — 3 294 = 300 – 6 146118 500 × 300 — 3 × 300 — 6 × 500–H3× 6 150000–900–3000+18=146118 (See Problem, page 111). 108 = 100 + 8 104 = 100 + 4 11232 100 × 100 + 8 × 100 + 4 × 100 + 8 × 4 10000+ 800 + 400+ 32=11232 Tº2 = 40 – 8 43 = 40 + 3 1376 40 × 40 – 8 × 40 + 3 × 40 – 8 × 3 1600 — 5 × 40 – 24 = 1600 — 200 –24 = 1376 CONTRACTIONS IN 115 MULTIPLICATION. & MISCELLANEOUS DRILL PROBLEMIS IN CONTRACTED METHODS. 194. The student who aspires to rapidity, accuracy, and superior merit as a business calculator, should practice a few minutes on one of the following exercises, each day, before entering upon the regular work of arithmetic. This exercise is the secret road that leads to honor and fame as an expert calculator. NOTE.—Students who do not understand division and fractions, may omit such problems as require such knowledge. Add at one Operation. 36 42 57 28 • 81 79 14 68 94. 86 23 *= 346 481 209 526 140 37 34 93 92 EXERCISE No. 1. MULTIPLY SIMULTANEOUSLY. 46 62 58 - 2416 124 342 26 34 47 23 237 276 MULTIPLY BY ALIQUOTS. 63 432 56 344 1368 4026 25 33%. 12; 62% 125 375 COMPLEMENTS. SUPPLEMENTS. 98 91 991 108 104. 162 97 88 988 105 102 107 COMPLEMENTS AND SUPPLEMENTS. 126 146 1012 497 594 806 92 77 988 294 395 307 TENS’ FAMILY. 61 124 323 87 73 84 109 69 126 327 27 33 24 101 ADD HORIZONTALLY. 4708, 290, 728, 49, 864, 71403. Add b Vicenary 3204 2063 72 1. 1012 1011 2161 1024 3248 721 1809 109 93 44 46 714. 512 68 48 the ystem. 58 34 68 17 45 78 12 34 56 90 13 51 42 67 19 78 55 89 76 90 68 34 79 23 31 84 45 54 55 66 77 89 I I 6 SOULE's PHILOSOPHIC PRACTICAL MATHEMATICS. * 㺠195. DRILL ExeRcise No. 2. 327 SIMULTANEOUS MULTIPLICATION. 114 208 83 68 89 298 689 548 768 987 4067 75 79 97 76 89 98 287 578 876 352 320 * = e {-º-º: &_-ºl Gº-º-º: *= *=== tºº-ºº: emº 612 184 ALIQUOTS. º 1462 at 12; g. 480 at 374 g. 654 at 87.4%. 544 at 18%. 326 at 16%. 562 at 83.4%. * 116 at 25%. 183 at 334%. 275 at $1.25 COMPLEMENTS. 2362 87 85 78 82 . 99 988 985 976 89 91 88 89 99 989 975 974. 1427 ° 5134 * > — —— . * tº 409 SUPPLEMENTS. COMPLEMENTS & SUPPLEMENTS. 1292 104. 123 1014 1126 || 137 1045 148 102 116 1009 1012 || 88 986 95 36021 196. DRI LL EXERCISE No. 3. 12003 MISCELLANEOUS CONTRACTING METHODS. 10400 96 78 593 412 42 68 246 6843 463 5026 94. 38 491 418 29 36 127 729 1624 10820 * = *s * º mºº smºs ºsmºs immºn $º *=ts *=s=== 72035 se 7604 58924. 3456 432 268 2695 7321 964 36945 111 17% 75 1008 996 Subtract by addition: 4867 | 248 3842 | 163 460 | 6892 1261 963 49 588 | 4087 527 1125 | 1.407 123 || 9364 65 OPERATION. 45 OPERATION. 85 35 75 95 5 x 5 = 25. 8t. x 6t.= 48h.-H. — 5 × 5 = 25. 3t. × 4t.= 12h.-- 55 25 5525 - # of (8+6)= 7h.= 55h. or 1575 # of (3 + 4) = 34h.-- 12h.- || – — 55.25. 1550 – the 25 = 1575. 63 32 88 146 42 94 73 77 86 CONTRACTIONS IN MULTIPLICATION. I 17 197. DRI LL EXERCISE No. 4. 1. 62 × 25 6. 974 × 125 11. 328 × 375 2. 87 × 124 7. 65 × 224 12. 261 × 750 3. 140 × 18; 8, 184 × 1874 13. 414 × 35 4. 365 x 33% 9. 212 × 75 14. 807 x 84 5. 740 x 62% 10. 514 × 83% 15. 694 × 16# 198. DRILL EXERCISE No. 5. NEW ORLEANs, Apr. 1, 1894. NEUGASS & THOMPSON, Bot. Of DELERNO & GRANTER. H { 5234 lbs. Butter sº sº a 25 g 136; “ La. Pecans - tº-º * “ 124 g 56 doz. C. Eggs - $º “ 18; g 249 boxes Lobsters * tº “ 374g. 140 jars Pickles º “ 624% 48 lbs. Rice & º wº “ 64%. 83 bush. W. Corn e * “ 87 g 64 “ W. Oats - * * *º “ 44 g 92 bags Salt - sº tº & “ 93 g 84 lbs. B. Tea º tº- “ 97 g 108 gals. R. Whiskey tº sº 4 $1.06 143 doz. Pine Apples - sº * : {{ 1.15 119 lbs. Y. H. Tea, gº “ 96 g 324 boxes Shrimp - me ſº * “ 35 g 240 gals. Fire Proof Coal Oil 4- “ 15 g. 164 lbs. Lard sº ſº tº º “ 7.4% 286 “ Green Peas sº ſº 3- “ 24% 320 “ N. Y. C. Cheese º tº- “ 174g 112 gals. La. Molasses - - - “ 22; ſ! 124 “ B. Whiskey º 4 $1.25 72 bbls. La. Oranges gº {{ 3.75 61 “ N. Potatoes º * {{ 2.50 16; lbs. C. Soap - e- gº = “ 84 g 9+ “ Persian Dates - - - “ 63 g 1 bbl. 50g Sugar, 241-24-217 lbs. “ 54 g 43 lbs. Lard Oil tº gº tº “ 58 g 45 “ Elgin C. Butter R-8 º “ 35 g 34 bush. W. Oats tºº gº sº 4 48 g 425 bbls. Flour gº -> gº 4 $8.75 562070 NOTE 1.—All extensions of the above bill should be made mentally. NotE 2.—See pages 96 to 101 for other Practical Drill Problems. 58, 68, 69, 70, 90, 91, and 95, for Rapid Methods of Handling Numbers. Also, see pages 53, 54, 57, II 8 sou LE's PHILOSOPHIC PRACTICAL MATHEMATICS. º: MULTIPLICATION BY SQUARING NUMBERS. 199. TABLE OF SQUARES AND CUBES. | - Numb. Square. Cube. Numb. Square. Cube. Numb. Square. Cube. Numb. Square. Cube. 1 l 1 26 676 17576 T51 2601 |132651. 76 5776 438.976 2 4 8 27 729 || 19683 52 2704 ||140608 77 5929 || 456533 3 9 27 28 784 || 21952 53 2809 ||148877| 78 || 6084 || 474552 4 16 64 29 841 || 24389 54 2916 |157464 79 6241 || 493039 5 25 125 30 900 27000 55 3025 166375|| 80 || 6400 || 512000 6 36 216 || 31 961 29791 56 3136 |175616 81 6561 || 531441 7 49 343 || 32 || 1024 || 32768 57 || 3249 |185193, 82 6724 || 55.1368 8 64 512 33 1089 || 35937 58 || 3364 |195112| 83 | 6889 || 571787 9 81 729 || 34 || 1156 || 39304 59 || 3481 (205379| 84 || 7056 || 592.704 10 || 100 || 1000 || 35 | 1225 || 42875 60 3600 |216000 85 | 7225 | 614125 11 | 121 | 1331 36 | 1296 || 46656 61 || 3721 |226981| 86 || 7396 || 636056 - 12 || 144 1728 || 37 || 1369 || 50653 62 3844 |238328 87 7569 || 658503 13 | 169 || 2197 || 38 || 1444 || 54872 63 || 3969 |250047| 88 || 7744 | 681472 14 || 196 || 2744 || 39 1521 593.19 64 | 4096 |262144|| 89 || 7921 || 704969 15 225 | 3375 | 40 | 1600 | 64000 65 4225 274625 90 8100 | 729000 16 || 256 | 4096 || 41 || 1681 | 68921 66 4356 |287496. 91 8281 || 753571 17 | 289 || 4913 || 42 || 1764 || 74088 67 || 4489 |300763| 92 | 8464 || 778688 18 324 5832 || 43 | 1849 || 79507 68 4624, 314432| 93 || 8649 || 804357 19 || 361 | 6859 || 44 1936 || 85184 69 || 4761 |328509| 94 || 8836 || 830584 20 | 400 8000 || 45 | 2025 | 91125 70 4900 |343000 95 || 9025 || 857.375 21 || 441 9261 || 46 || 2116 || 97336 71 || 5041 |357911| 96 || 92.16 || 88.4736 22 || 484 |10648 || 47 2209 103823 72 || 5184 |373248 97 || 9409 || 91.2673 23 || 529 |12167 || 48 || 2304 |110592 73 || 5329 |389017| 98 || 9604 || 94.1.192 24 || 576 |13824 || 49 || 2401 117649 74 5476 |405224| 99 || 9801 || 97,0299 25 | 625 15625 | 50 || 2500 125000 75 5625 42.1875|| 100 10000 |1000000 SQUARING NUMBERS. 200. The usual methods of Squaring numbers on the basis of complements and supplements, are attended with so many variations that they are of very little practical value, especially to those who understand the system of simultaneous multiplication, and hence, we shall present but little work of this kind. TO SQUARE NUMBERS BETWEEN TWENTY-FIVE AND FIFTY. 201. 1. What is the square of 26 % OPERATION. Explanation.-In all problems of this kind, mentally subtract 26–25 = 1 25 from the number to be squared, and the remainder from 25; then 25 — 1 = 24 square this second remainder and add to its hundreds' figure the first 242 = 576 remainder. p- * To be proficient in this work, the squares of numbers as high as 576–H199–676 Ans. 25, must be thoroughly iearned from the table. g X- CONTRACTIONS IN MULTIPLICATION. I IQ TO SQUARE NUMBERS BETWEEN FIFTY AND SEVENTY-FIVE. 202. 2. What is the square of 63? OPERATION. 63 — 25 = 38 Explanation.--In all problems of this kind, mentally subtract, 63 – 50 = 13 first 25, and then 50 from the given number; then square the second 132 = 169 remainder and add to its hundreds' figure the first remainder. 169-H3899 = 3969 Ans. TO SQUARE NUMBERS BETWEEN SEVENTY-FIVE AND ONE HUN- DIRED. 203. What is the square of 89? OPERATION. Explanation.—In all problems of this kind, subtract 89 – 75 = 14 75 from the number, and the remainder from 25; then 25 — 14 = 11 square the second remainder and add to its hundreds' fig- 112 121 ure the difference between the number and the second 121+(8999–1199)= 7921 Ans. remainder. PROBLEMIS. What is the square of 28, 42, 51, 67, 78, and 93% To MULTIPLY Two NUMBERS ON THE PRINCIPLE THAT THE SQUARE OF THE MEAN OF TWO NUMBERS MINUS THE SQUARE OF ONE–HALF OF THE DIFFERENCE, IS EQUAL TO THE PRODUCT OF THE TWO NUMBERS. 204. Multiply 15 by 19. OPERATION. 15 + 19 = 34 -- 2 = 17* = 289 – 2* = (4) 285 Ans. Ea:planation.—From the square of the mean, subtract the square of half their difference. 1. Multiply 68 by 72. - OPERATION. (70 — 2) × (70 + 2) = 70° — 2* = 4900 – 4 = 4896 Ans. I 20 SOULE's PHILOSOPHIC PRACTICAL MATHEMATICs. 2. Multiply 81 by 79. OPERATION. 80°–1 = 6400–1 = 6399 Ans. 3. Multiply 92 by 108. OPERATION. 100° — 8° = 10000 — 64 = 9936 Ans. Multiply the following numbers as above: 4, 28 by 34 6. 67 by 69 8. 57 by 63 5. 34 ($ 32 7. 73 44 85 9. 74 44 86 TO MULTIPLY TWO NUMBERS, THE DIFFERENCE OF WHICH IS ONE, THREE, FIVE, OR ANY SMALL NUMBER. 205. 1. Multiply 21 by 22. OPERATION. 21* = 441 + 21 = 462 Ans. Ea:planation.—In all problems of this kind, square the lesser number and add to the product as many times the lesser number as it is less than the greater. The reason is that the lesser number should be repeated as many times as there are units in the greater number; but in squaring the lesser Inumber, you fail to repeat it as many times as there are units difference between the numbers. 2. Multiply 87 by 89. º OPERATION. 87° = 7569 + (2 × 87) = 174 = 7743 Ans. 3. Multiply 236 by 241. OPERATION. 236* = 55696 -- (5 x 236) = 1180 = 56876 Ans. 4. Multiply 96 by 92. Ans. 8832. Multiply 354 by 361. Ans. 127794. 6. Multiply 32 by 43. SYNOPSIS FOR REVIEW OF MULTIPLICATION. A Lº - a v- w wr Note:-The numbers before the words and phrases refer to the articles treating the subjects Ilºſſle (1. 206. Define the following words and phrases: cand or Multiplier contains Dollars and Cents. 142. Multiplication. 162. General Directions for Multiplica- 143. Multiplicand. tion. 144. Multiplier. 163. Miscellaneous Problems in Multi- 145. Product. plication. 146. Factors. 164. Simultaneous or Cross Multiplica- 147. The Sign of Multiplication. tion. - r - | 148. Proof of Multiplication. 165. Drill Problems in Simultaneous 149. Principles of Multiplication. Multiplication. 150. Multiplication Table. 166. A Different Method of Simulta- 151. The Importance of Multiplication. neous Multiplication. 152. The Philosophic or Logical System 167. To Multiply when the Multiplier of Multiplication. consists of Two Figures. 153. Examples with Philosophic Solu- 168. Table of Aliquots. tions. 169. Problems. *154. Drill Problems. 170. To Multiply by 74, 174, 224, 274, 155. To Multiply Abstract Numbers 324, 15, 35, 45 and 55. and give reasons therefor. 171. Practical Operations with Ali- 156. To Multiply, when the Multiplier Quots and other numbers. consists of only one figure. 172. To Multiply, when the Multiplier 157. To Multiply, when the Multiplier * is a mixed number whose frac- consists of more than one fig- tional part is one fractional ll.T0. unit, less than a whole number. 158. To Multiply, when either factor is 173-93. Miscellaneous Contractions in 1 with naughts annexed, as 10, Multiplication. 100, 1000, etc. 193. Algebraic Formulae, for Comple- 159. To Multiply, when either the Mul- ments and Supplements. tiplicand or Multiplier, or both, 194-98. Drill Problems in Contracted have naughts on the right. Methods in Addition and Multi- 160. To Multiply by the Factors of a plication. Number. 199. Table of Squares and Cubes. 161. To Multiply, when the Multipli- 200-5. Squaring Numbers. (121) IVISION. a-a-a-a-a-a-a-a-a-a-/----------El =N **** ( DECREASI NG). *—iº—dº- -sº-sºr—wº- 207. Division is the process of finding how many times one mumber is equal to another, which is used as a unit of measure. Or, differently defined, it is the process of finding one of the factors of a given product, when the other factor is known. 208. Erom the first and the better definition, we see that division is a process of measuring some numbers by other numbers. And that it is not the process of finding “how many times one number is contained in another,” as is taught by nearly all the authors of Arithmetics. One number cannot go into another, how- ever Small the one, or large the other; and the question of division of numbers does not Warrant a definition so unmathematical, indefinite and illogical. THE LOGIC OF DIVISION. 209. The following questions and operations will elucidate the true meaning of division and its philosophy: 1. You have $6, and I have $2. How many times is your money equal to mine * Is it not clear, by the terms of the question, that your sum of money is to be compared with, and measured by, my $2? And the contracted thought to do this is, $6 is equal to $2, 3 times. The full logical thought, recognizing 1, or unity, as the basis of all numerical computations, would be as follows: $6 is equal to $1, 6 times. Since $6 is equal to $1, 6 times, it is equal to $2, one-half the number of times, which is 3. 2. Again—you have 8 yards of cloth, and I have 4 yards. How many times is your quantity of cloth equal to mine? Here it is plain that my 4 yards is made, by the terms of the question, the unit of measure; and your 8 yards is the thing to be measured. And the contracted thought to do the work would be, 8 yards is equal to 4 yards, 2 times. The full logical reason is as follows: 8 yards are equal to 1 yard 8 times. Since 8 yards are equal to 1 yard, 8 times, they are equal to 4 yards instead of 1, the fourth part of the number of times, which is 2. 3. Divide 10 by 5. In this problem, the real question is, how many times is 10 equal to 5, not how many times 5 can go into 10, or how many times is 5 contained in 10. 5 is the unit of measure, and 10 is the number to be measured or divided. jº The contracted reasoning is, 10 is equal to 5, 2 times; not 5 is contained or º (122) * * DIVISION. I 23 goes into 10, 2 times. The full logical reason is as follows: 10 is equal to 1, 10 times. Since 10 is equal to 1, 10 times, it is equal to 5, instead of 1, the fifth part of the number of times, which is 2. This process of reasoning is applicable to every possible question of division, and it is the only logical reasoning that can be given for abstract numbers. Yet strange to say, it has escaped the attention or the approbation of all other authors of Arithmetics. With due modesty, we claim some merit for its first introduction, and earnestly commend it to the thoughtful consideration of authors, of students, and of the public. 210. The Dividend is the number to be measured, or the number to be divided. 211. The Divisor is the number used as the unit of measure or the number by which to divide. 212. The Quotient is the result of the division, and shows how many times the dividend is equal to the divisor. 213. The Remainder is the number left after dividing dividends which are not multiples of the divisor, or which are not equal to the divisor an exact number of times. It must always be less than the divisor. 214. The Sign of Division is a horizontal line with a point above and below, thus -i-. It is read divided by, or is equal to ; and it indicates that the number before it, is to be divided by the number after it: thus 25 -- 5, is read 25 divided by 5, or 25 is equal to 5. 215. Division is also indicated by a horizontal line, a vertical line, or a curved line, when placed between the dividend and the divisor. Thus, the following state- Inents : 36 9 || 36 9) 36 36(9– 9 are all read 36 divided by 9, or how many times is 36 equal to 9, or 36 is equal to 9 how many times 3 PRINCIPLES OF DIVISION. 216. 1. When the divisor and dividend are both denominate or both abstract numbers, the quotient will be an abstract number. 2. When the divisor is an abstract number and the dividend a denominate Inumber, the quotient will be a denominate number. 3. When there is a remainder, it is a part of the dividend, and is therefore the same in name or kind. 4. Multiplying the dividend or dividing the divisor, multiplies the quotient. 5. Dividing the dividend or multiplying the divisor, divides the quotient. 6. Multiplying or dividing both the divisor and dividend by the same num- ber, does not change the quotient. 217. Proof.-Multiply the quotient by the divisor, and to the product add the remainder, if any. If the result is equal to the dividend, the work is correct. See Proof of Division by the Properties of 9's and 11's, at the close of this subject. I 24 SOULE's PHILOSOPHIC PRACTICAL MATHEMATICs. • { 218. The operation of division may be performed, either by addition or sub- traction. Thus, in the following problem: How many times is 43 equal to 12% Ans. 3 times and 7 remainder. OPERATION BY ADDITION. OPERATION BY SUBTRACTION. 43 is equal to 12, 1 time. 43 43 “ “ “ 12, 2 times. 43 is equal to 12, 1 time. 24 31 43 “ “ “12, 3 times. 43 “ “ “ 12, 2 times. 36 19 and 7 remainder. 43 “ “ “ 12, 3 times. 43 and 7 remainder. ORAL EXERCISES. 219. 1. How many times is 0 equal to 1 ? or 0 + 1 = ? no times. 2. 4% {{ 1. “ 0% Or 1 - 0 = } Ans. An infinite number of times. 3. How many times is 1 equal to 13 or 1+1= ? | 12. How many times is 35 equal to 7? or 35— 7= ? 4. & 4 é & 2 & 4 1 ? or 2–1= ? | 13. 4 & & 4 56 ** 83 or 56— 8– ? 5 ( & & 4 3 ( & 1? Or 3–1=3 || 14. & 4 & 4 63 ‘‘ 93 or 63-- 9= ? 6. & 4 & & 4 “ 2% or 4–2–3 || 15. 4 & 4 & 72 ** 93 or 72– 9–? 7. & 4 & 4 8 ‘‘ 23 or 8–2 – ? | 16. & & 4 & 80 ‘‘ 10? or 80–10= ? 8. { { & 4 9 ** 33 or 9–3–? | 17. & 4 & 4 88 ‘‘ 11? or 88–11= ? 9. { { & 4 12 “ 4 3 or 12–4–? | 18. & 4 & 4 96 “ 12% or 96–12– ? ( & & & 2 & & º —5– * #. ( & & 4 ; & & # : *::=; 19. §–? 4)42=? 9)45=? 77-7=? 12 | 84= ? 20. How many times is 36 equal to 3, to 4, to 6, to 9, to 12, to 36% 21. How many times is 42 equal to 2, to 6, to 7, to 42? 14 21 28 64. 8 22. – =? – =1 − =1 − =? 24(?-, 32(ºr 7) 56 =? 9 || 72=? 2 7 4. 8 — ? ~ : FRACTIONAL NUMBERS. 220. When we divide a unit or a number of units of any kind into equal parts, these parts are sometimes called fractions. The name of the equal parts varies according to the number of parts into which the thing or number is divided. When the unit or number is divided into two equal parts, 1 of the parts is called one-half, and is written thus, #. If divided into 4 equal parts, 1 of the parts is called one-fourth, and is written thus, 4; 3 of the parts are called three-fourths, and are written thus, #. In like manner, we obtain fifths, sixths, sevenths, eighths, twelfths, sia teenths, twenty-firsts, etc. * DIVISION. I 25 In Writing fractional numbers in figures, we place the number which shows the name of the parts below a horizontal line as a divisor, and the number which shows how many parts are taken, or used, above the line as a dividend. The following examples will fully elucidate this work: Two-thirds, 3. Five-eighths, #. Seven-twelfths, ". Three-fourths, #. Seven-ninths, #. Nine-tenths, #. Fifteen-sixteenths, #. Eleven-eightieths, #. How do you find #, #, #, #, etc., of any number 2 How do you find 3, #, #, g, etc., of any number 2 What is 4 of 4% What is # of 15% What is 3 of 9% What is # of 15 + “ (: ; “ 6 “ (: ), “ 18% “ (: ; “ 12? “ (: ; “ 633 “ “ # 4 83 “ “ , « 28 “ “g “ 40% “ “ H. “ 80? 221. ORAL AND WRITTEN PROBLEMs. SOLUTION STATEMENT. * 1. Five pounds cost 40%. g Reason—Five pounds cost 40 40c. Since 5 pounds cost 40 5 What was the cost of 1 Cent8, 1 pound will cost the 5th part, which is 8 cents. Q - - pound 4 82 Ans. REASONS, WHY, AND WHEREFORE, CONTINUED. Question 1. How do you know that if 5 pounds cost 40 cents, 1 pound will cost the 5th part 2 Answer. By the exercise of my judgment—by the use of the reasoning faculties of my mind. Question 2. What do you mean in this connection by judgment 3 Answer. The conclusion arrived at by the operations of the mind, after hav. ing duly considered the premise, the facts, and the conditions of the problem. Question 3. What do you mean by the premise or premises 2 Answer. The proposition, declaration, truth or fact which is stated, or asserted, as a basis, or predicate, of a question. In this problem the premise is, 5 pounds cost 40 cents. . Question 4. Why will 1 pound cost one-fifth part as much as 5 pounds 3 Answer. Because 1 is the fifth part of five. Question 5. What kind of reasoning is the foregoing 3 Answer. (See answer to the same question on page 78). Question 6. What do you mean by reason 3 Answer. (See answer to the same question on page 78). I 26 soul E's PHILOSOPHIC PRACTICAL MATHEMATICS. * SOLUTION STATEMENT. Reason.—Twelve yards cost g 2. If 12 yards cost 60%. 60 60c. Since 12 yards cost 60c., What will 1 yard cost 3 12 1 yard will cost the 12th part, which is 5 cents. | 52 Ans. Questions. 1. How do you know this? 2. Why will it? 3. What do you mean by judgment 3 3. Nine barrels of flour cost $72. What was the cost of 1 barrel ? & Ans. $8. NoTE.—Work and reason as in above problems. SOLUTION STATEMENT. - 4. Paid $18 for 6 days' $ . Reason.—Six days’ labor cost - 7 labor. What was the rate | 18 $18. Since 6 days’ labor cost 6 $18, 1 day’s labor will cost the Q per day ? 6th part, which is $3. $3 Ans. Questions. 1. How do you know this? 2. Why will it? SOLUTION STATEMENT. Reason.—The freight on 17 5. The freight on 17 bales $ bales was $51. Since the freight of cotton was $51. What 17 51 on 17 bales, was $51, on 1 bale was the freight per bale 3 - ºm- it would be the 17th part, $3 Ans. which is $3. Questions. 1. How do you know this? 2. Why will it? 3. What do you mean by judgment 3 SOLUTION STATEMENT. Reason.—Eight persons are to 6. If you divide 2 dozen g e Oranges equally among 8 *2, receive 24Oranges. Since 8 per- | sons are to receive 24 oranges, persons, how many oranges s e- 1 Jerson will receive the 8th will you give each 3 | 3 oran ges Ans. part, which is 3 oranges. Questions. 1. How do you know this? 2. Why? SOLUTION STATEMENT. Reason.—In 12 hours 360 miles 7. A rail © IMI. are run. Since 360 miles are run e road train runs 360 in 12 hours, in one hour the 12th 360 miles in 12 hours. * 12 part of the distance would be * Tun, which is 30 miles. Or, since What is the speed per hour? 12. hours' running gives 360 | 3 * º miles, 1 hour's running will give 0 miles Ans the 12th part. g Willg Questions. 1. How do you know this? 2. Why? DIVISION. 127 1st SOLUTION STATEMENT. ſh. Reason.—Seven cents buy 1 p • 1. pound of rice. Since 7 cents 8. Rice is 7 cents per 7 || 35 will buy 1 pound, 1 cent will * buy the 7th part, and 35 cents 5 lbs. Ans. will buy 35 times as much. Or, can you buy for 35 cents? 2d solution statDMENT. since 7 cents buy 1 pound, for 35 cents you can buy as many 7 || 35 pounds as 35 cents are equal to 7 cents. pound. How many pounds *º- 5 lbs. Ans. Questions. 1. How do you know this? 2. Why? 3. What do you mean by judgment? º NOTE--Work and reason the following problems in the same manner. 9. At 9 cents per pound, how many pounds can be bought for 45 cents? Ans. 5 pounds. 10. Flour is worth $8 per barrel. How many barrels can be purchased for $56% Ans. 7 barrels. 11. For 8.95, how many papers can you buy at 5 cents a paper? Ans. 19. 12. At $3 a piece, how many chairs can be bought for $36? Ans. 12 chairs. 13. If the printer charges $2.50 to set one page of this book, how many pages can be set for $75 % Ans. 30 pages. 14. The dividend is 42 and the divisor, 7. What is the quotient? Ans. 6. 15. The divisor is 8, the quotient 3, and the remainder 2. What was the dividend ? Ans. 26. -> TO DIVIDE WEHEN THE DIVISOR DOES NOT EXCEED TWELVE. 222. 1. Divide 3648 by 5. OPERATION. h Ea:planation.—In all problems of this kind, we write tº º t - º the numbers as shown in the operation, and then begin on Divisor 5) 3648 dividend. the left of the dividend to divide, we begin on the left * > in order to carry the remainder, if any, of the higher order Quotient 729, and 3 rem. of units to the next lower order. In this problem, we first take the 3 thousands, and by inspection, we see it is not equal to 5; we therefore unite it with the 6 hundreds, making 36 hundreds, which, by trial multipli- cation and subtraction, mentally performed, we find is equal to 5, 7 hundreds times and 1 remainder; the 7 we write in the hundreds’ column of the quotient line, directly under the 6, the last figure of the dividend used. Then, to the 1 remainder, we mentally annex the 4 tens, making 14 tens, as the second partial dividend, and which, by mental multiplication and subtraction, we find it equal to 5, 2 tens times and 4 remainder; the 2 we write in the tens’ column of the quotient line, and to the 4 we mentally annex the units' figure of the dividend, making 48 units as the third and last partial diyi- dend; this we find, by mental multiplication and subtraction, to be equal to 5, 9 times and 3 remain- der. This remainder is usually expressed fractionally by writing it over the divisor; thus # which expresses the part of a unit of times that the remainder is equal to the divisor. I 28 SOULE's PHILOSOPHIC PRACTICAL MATHEMATICS. X- SHORT DIVISION. 223. Operations in division according to the foregoing method, are called short division, because the multiplication and subtraction work, in finding the remainder of the partial dividends, were mentally performed. 2. How many times is 846 equal to 6% OPERATION. º'. * preceding problem, we gave a full and ; rr; c. awº ivi o explicit, explanation for each step of the operation. In practice Divisor 6) 846 dividend much of the explanation therein given is omitted, and the work e -*- performed thus: Commencing with the left hand figure we say 8 Quotient 141 is equal to 6, 1 time and 2 remainder; 24 is equal to 6, 4 times; tº 6 is equal to 6, 1 time. PROBLEMS. Work the following indicated divisions: (3) (4) (5) (6) (7) (8) 7) 847 8) 12327 9) 1085 11) 2386 2344 7 | 93020 121 1540% 120; 2164? 3 (9) (10) (11) (12) (13) (14) 8) 1471 4) 11899 9) 81018 12) 10824 21345-1-8 9) 76451 Divide the following: 15. 9872 by 4 17. 10286 by 6 19. 1691 by 9 21. 10008 by 9 16, 1483 “ 7 18, 4.8710 ($ 7 20. 4.1070 € $ 8 22. 199999 44 8 23. What is + of $ 528? 25. What are ; of $ 448% 24. “ are # of $1005? 26. “ “ g of $6448? 27. How many apples can be bought for $2.25, at 5 cents a piece 2 Ans. 45 apples. 28. At 15 cents per pound, how many pounds can you buy for $3.15% Ans. 21 pounds. 29. Paid $90 for 10 volumes of Chambers' Cyclopedia. What was the price of one volume 3 Ans. $9. 30. If 8 men are to receive $5791 in equal parts, what will be each man's share 2 Ans. $7.23% 31. The dividend is 63, and the quotient is 9. What is the divisor 3 Ans. 7. 32. The quotient is 36, and the divisor is 6. What is the dividend ? Ans. 216. 33. The dividend is 72, and the divisor is 4. What is the quotient 2 - Ans. 18. 34. The quotient is 15, the divisor is 3, and the remainder is 2. What is the dividend ? Ans. 47. 35. The divisor is 24, the quotient is 6. What is the dividend? Ans. 144. 36. How many pounds of cotton, at 11 cents a pound, will be required to pay for 33 pounds of sugar (a) 8 cents per pound 3 Ans. 24. +): DIVISION. I 29 TO DIVIDE WHEN THE DIVISOR EXCEEDS TWELVE. 224. 1. Divide 7387 by 36. OPERATION. th †W. first write º ii. shown i. tº ge ivri e e Operation, and commence to divide as explained in the Divisor, Dividend, Quotient. first written example. But as the divisor is ; large to be 36) 7387 ( 205; conveniently used mentally, we therefore write the operation 72 of multiplying the divisor by the quotient figures, and sub- * tracting the successive products from the several partial dividends. In performing the division, we first see by 187 comparison, that 7 thousands are not equal to 36, and hence 180 there will be no thousands in the quotient, We then annex to the 7 thousands the 3 hundreds, making 73 hundreds as the 7 ind first partial dividend; this is equal to 36, 2 times, and a re- I’6100.3.IIlCl62I’. mainder; we write the 2 in the hundreds' column of the -- uotient, multiply the divisor by it, write the product under, and subtract the same from the 73 hundreds of the dividend. This work gives us 1 hundred remain- der, to which we annex the 8tems, making 18 tens as the second partial dividend; this partial dividend not being equal to 36, we write 0 (no tens) in the tens’ column of the quotient, and annex to the 18 tems the 7 whits, making 187 units as the third and last partial dividend. This is equal to 36, 5 times, and a remainder; we write the 5 in the quotient, multiply and subtract as we did with the first obtained figure of the quotient, and thus produce 7 remainder, which we write over the divisor as explained in short division. LONG DIVISION. Operations in division, according to the above method, are called long division, for the reason that the multiplication and subtraction work in finding the remain- ders of the partial dividends is written. See Contractions in Division. .GENERAL DIRECTIONS FOR DIVISION. 225. From the foregoing elucidations, we derive the following general directions for the operations of division. For the process of reasoning given in connection With the operations or solution statements, we refer to pages 122 to 129. 1. Draw a vertical or curved line, and write the dividend on the right, and the divisor on the left. * 2. Take the least number of the left hand figures of the dividend that are equal to or greater than the divisor, find how many times the same is equal to the divisor, and write the result in the quotient line. 3. Multiply the divisor by this quotient figure, subtract the product from the partial dividend used, and to the remainder annew the succeeding figure of the dividend and divide as before. 4. Proceed in like manner until all the figures of the dividend have been used. 5. When the partial dividend is not equal to the divisor, write a maught in the quotient, annea, the succeeding figure of the dividend to the partial dividend, and pro- ceed as before. 6. If there is a remainder after the last division, write it in the quotient, draw a line below it, and write the divisor underneath, as a part of the quotient. I 30 SOULE's PHILOSOPHIC PRACTICAL MATHEMATICs. 4× PROOF. Multiply the quotient by the divisor and to the product add the remainder, if any. If the result is equal to the dividend, the work is correct. NOTE: 1. -The product referred to in No. 3, must never be greater than the partial dividend from which it is to be subtracted; if it is larger, the quotient figure is too large, and must be diminished. NOTE: 2-The remainder, after each subtraction referred to in No. 3, must always be less than the divisor; if it is not, the last quotient figure is too small and must be increased. NOTE: 3.-The order of each quotient figure is the same as the lowest order in the partial dividends from which it was obtained. PROBLEMI. How many times is 66804 equal to 53° Ans. 1260}#. OPERATION. Divisor, Dividend, Quotient, Proof. 53) 66804 (1260% 1260 quotient. 53 53 divisor. 138 3780 106 6300 º-s, 24 remainder. 320 -emmºmºm. 3.18 66804 dividend. 24 remainder. PROBLEMIS. Divide the following numbers: 1. 784 by 82. Ans. 9;;. 2. 91070 by 8761. Ans. 10####. 3. 107941 by 396. Ans. 4. 7167901 by 11267. Ans. 5. 37021 by 2002. Ans. 6. 8888888 by 332311. Ans 7. Divide $6805 equally among 5 men. What will be the share of each * Ans. $1361. 8. What is the sixty-fourth part of $44800 % Q Ans. $700. 9. 145 men picked 1305000 pounds of cotton. Suppose each picked an equal quantity, how much did each man pick 3 Ans. 9000 pounds. 10. A father gave his 7 Sons a Christmas present of $353.50 to be shared equally among them. What was each one's share ? Ans. $50.50. TO DIVIDE WHEN TEIERE ARE NAUGHTS ON THE RIGHT OF THE DIVISOR. 226. 1. Divide 2843 by 200. Ans. 14.5%;. OPERATION. Explanation.—Since by our scale of numbers they increase from 2 00)28 43 right to left in a ten-fold ratio, and decrease from left to right in a cor- responding manner, it is clear that the removal of any order of figures from left to right diminishes its value ten times for each place of 14 and 43 rem. removal. And as previously shown on page 83, that the annexing of naughts multiplies numbers, by removing them to places of higher value, so in like manner, cutting figures off from the right of a number removes the remaining orders to the right, and hence decreases them ten-fold for every figure cut off. Hence to cut off one figure is dividing by 10; to cut off two figures divides by 100; to cut off three figures divides by 1000; and SO OI). Considering these principles, in all cases of this kind we cut off the naughts from the right of the divisor and the same number of figures from the right of the dividend; and then divide the remaining figures of the dividend by the remaining figures of the divisor. When there is a remainder, annex the figures cut off, and we obtain the true remainder. DIVISION. I 3 I 3. l 2. Divide 87931 by 1000. Ans. 87% OPERATION. 1|000)87931 Quotient 87 and 931 remainder. 0 O 3. Divide 178 by 10. 4. Divide 6581 by 300. OPERATION. OPERATION. 1|0,178 3|006581 Quotient 17, and 8 remainder. Quotient 21, and 281 remainder. Ans. 17%. Ans. 21}}}. 5. Divide 71468071 by 341000. OPERATION. 341 | 000)71468||071(209344%; Ans. 682 323s 3069 199 6. Divide 88.97600 by 8100. Ans. 1098;#. 7. Divide 1000000 by 10000. Ans. 100. 8. Divide 99999 by 9000. ” Ans. 11%. 9. Divide 33440 by 270. Ans. 123}#}. 10. Divide 140817 by 6800. Ans. 20áš. TO DIVIDE BY THE FACTORS OF A NUMBER. 227. 1. Divide 936 by 24. Ea:planation.—In all problems where the divisor is a com- OPERATION. posite number, we may divide by the factors and thus 4)936 shorten the operation, In this example the factors are 4 and -º-º: 6, and we first divide by 4 which gives a quotient 6 times 6)234 too large, for the reason that 4 is but of 24, the true divisor. --> We therefore divide this quotient by 6 and obtain the true 39 quotient. 2. Divide 588 by 28. The factors are 4 and 7. Ans. 21. 3. Divide 6976 by 32. The factors are 4 and 8. Ans. 218. 4. Divide 2583 by 63. The factors are 7 and 9. Ans. 41. 5. Divide 10206 by 81. The factors are 9 and 9. Ans. 126. 6. Divide 11984 by 56. The factors are 8 and 7. Ans. 214. I 32 SouLE's PHILOSOPHIC PRACTICAL MATHEMATICS. PROBLEMS INVOLVING THE ENGLISH MONEY OF ACCOUNT. 228. 1. What will 13840 pounds of cotton cost, at 8 pence a pound. Ans. £461. 6s. 8d. OPERATION. Explanation.—In this problem, the price is given in one of the subdivisions of the English monetary unit, and hence 13840 we must know what that unit and its subdivisions are, before 8d. we can solve the problem. The English monetary unit is gº-me the pound 8terling, yº,jº divided into 20 shillings; each shilling is divided into 12 pennies, and each penny into 4 12) 110720d. farthings. With this knowledge of English money, we can work all problems of the above character. In this example, 29) 922% 8d. * we first multiply the price of one pound by the number of pounds, and thus produce the value of the whole in pence. fº 461 6S. Then to reduce the pence to shillings, we divide them by 12, and obtain 9226 shillings and a remainder of 8, which being a part of the dividend, is therefore 8d. Then, to reduce the shillings to pounds, we divide them by 20, and obtain 461 pounds and a remainder of 6, which being a part of the second dividend, is there- fore 6s. In the English monetary system the following abbreviations are used: £, represents pounds; s. represents shillings; d. represents pence, and f. represents farthings. 2. What is the value of 483 yards 3. What will 241 boxes cheese cost, of cloth, at 16 shillings per yard 3 at £3 per box” Ans. £386. 8s. Ans. £723. OPERATION. OPERATION. 483 - 241 16 3 20) 7723 shillings. 36 723 # 386 8S. 4. Sold 486 yards of calico at 5 pence a yard. What did it amount to ? * Ans. £10 2s. 6d. 5. Bought 38495 pounds of good middling cotton at 7 pence per pound. E[ow much did it cost 3 Ans. £1122 15s. 5d. 6. What is the value of 850 barrels of flour at 34 shillings a barrel ? Ans. £1445. 7. How much will 1812 tons of iron cost, at £52 4s. per ton ? Ans. £94586 8S. 8. Bought 38421 pounds of cotton at 9 pence per pound. What did it cost 3 Ans. £1440 15S. 9d. Yºr DIVISION. I33 TO FIND THE TRUE REMAINDERS WHEN DIVIDING BY THE FAC- TOR'S OF A NUMBER. 229. Divide 1607 by 72, using the factors 3, 4, and 6, and find the true remainder. Ans. 22 quotient, and 23 remainder. FIRST OPERATION. Explanation.—In this problem, using the factors 3, 4, and 6, we 3) 1607 obtain for remainders 2, 3, and 1. * The first remainder 2, is clearly units of the given dividend, ſº o and hence a part of the true remainder. 4) 535 2, 1st remainder. The second remainder 3, being fourths of the second dividend 535, which are reciprocal thirds of the given dividend, it is hence 6) 133 3, 2d remainder. # of the reciprocal of + of # = 9, of the given, dividend and true 7 remainder. ºp The third remainder 1, being sixths of the third dividend, 133, 22 1, 3d remainder, which are reciprocal twelfths of the given dividend, it is hence # of the reciprocal of # of + of # = 12, of the given divi- dend and true remainder. Therefore 2, the first remainder, plus 9, the unit value of the second remainder plus 12, the unit value of the third remainder = 23, the true remainder. SECOND OPERATION. TEIIRD OPERATION. 3 | 1607 3 | 1607 4 || 535 = 2, 1st remainder 4 | 535 = 2 = 3 of 1. 6 || 133 = 3, 2d remainder. 6 || 133 = 3 = } + 3 = + + 4 = +}. 22 = 1, 3d remainder. tº-º-º-º: First remainder - - E. tº- di - 2 2 3 e Plus 2d remainder 3, Xthe preceding divisor 3=9 mºmes - 2 ll. * -ºº º - ------- e 22 = 1 = }} + +} = }} + 6 = ##, Plus 3d remainder i. all the preceding divisors or 23 the true remainder. 4 and 3- - - - - - - 12 which added, gives the true remainder - 23 Eplanation.—The second operation involves fractions, and hence not regularly in order at this F. In this operation each subsequent remainder is reduced to the fractional unit of the preced- ing fraction. g By the third operation, we see that the true remainder may be obtained without considering the reciprocal relationship of the quotients and divisors. The general method shown in the third operation is as follows: Add to the first remainder, the product of the other remainders by all the divisors preceding the one which produced it. 2. Divide 7851 by 64, using the fac- 3. Divide 17803 by 96, using the fac- tors 8 and 8. a tors 2, 3, 4, and 4. Ans. 185%. Ans. 122 quotient, 43 remainder. OPERATION. OPERATION. Ea:planation.— 2)17803 Elap lanation. 8)7851 Here the 1st re- *- mainder is 3, to 3)8901–1 1st remainder - - º 1 º which we add 8)981—3, 1st remainder. the product of - the 2d remain- 4)2967–0 2d remainder, 0x2 = - - 0 122–5, 2d remainder. Herămultipºl - by the preced- 4)741–3 3d remainder, 3x3x2 = - 18 ing divisor 8, equals 40, making 43, the true re- -- mainder. 185–1 4th remainder 1X4×3×2 = 24 True remainder, - - 43 I 34. soul E's PHILOSOPHIC PRACTICAL MATHEMATICS. Divide 27865 by the factors of 81. Ans. 344sºr or 344 and 1 remainder, Divide 101041 by the factors of 84. Ans. 1202% Divide 899 by the factors of 108. Ans. 81% s. If $4691 are divided equally among 35 men, what will each one receive? Ans. $134;. : TEIE PARENTEHESIS AND WIN CULUM. 230. The parenthesis, (), or vinculum, -, denotes that the numbers within the parenthesis or below the vinculum are to be considered as one quantity. Thus, as shown on page 36, 12 — (4-H3) = 5, or 12 – 4 + 3 = 5. In all operations involving the signs of +, −, x, and -- the following prin- ciples govern : 1. The signs +, and — affect only the number or expression which immedi- ately follows either of them. 2. The signs x and -- indicate an operation to be performed with the numbers or expressions between which either may be placed, and such operation must be performed before that of any + or — which may immediately precede or follow. r" 3. When the parenthesis or vinculum is used, the operations indicated thereby must be performed or simplified into one expression before connecting them with any other numbers in the problem. This is done for the reason that all numbers included within the parenthesis or the vinculum are considered as one number. 4. In the operations of all problems involving these principles, commence with the (), or , if any, then with the expressions indicated by the x and -- signs, and lastly with the numbers connected with the signs of + and —, which should be considered from left to right. 5. In cases where there are double parentheses, the operation indicated by the interior parenthesis must be performed first. PROBLEMIS. 1. Simplify the following expression: 16 + 24 + 8 + 4 — 3 × 5. Ans. 27. 2. What is the value of the following numerical expression ? (740 + 63 — 200) -- 450 — 325 + 76 x (65 – 35 + 10) * OPERATION. 740 + 63 = 803 – 200 = 603; then 450 — 325 = 125 + 76 = 201; then 603 -- 201 = 3; then 65 — 35 = 30 + 10 = 40; then 3 × 40 = 120 Answer. 3. Simplify the following expression: (740 + 63 — 200) + (450 — 325 + 76 ) × (65 – 35 + 10)? † DIVISION. 135 OPERATION. 740 + 63 = 803 – 200 = 603; then 450 — 325 = 125 + 76 = 201; then 603 + 201 = 804; then 65 – 35 = 30 + 10 = 40; then 804 x 40 = 32160 Ans. 4. Simplify the following expression: (740 + 63 – 200) + 450 — 325 + 76 x (65 – 35 + 10). OPERATION. 450 — 325 = 125 + 76 = 201; then 65 — 35 = 30 + 10 = 40; then 201 x 40 = 8040; then 740 + 63 = 803 – 200 = 603; then 603 + 8040 = 8643 Ans. 5. What is the value of the following expression ? (420 – 60) — 576 — 490 + 312 – 10 + 16 × 5 + 8 + 6 – 4. Ans. 584. 6. What is the value of 9500 + 6200 – 9000 -- 3400 + 2100 — 5490 × (6500 – 6400 + 200) * Ans. 15697. 7. What is the value of 2765–1624 × (320 + 96) -- (3645 – 3376). Ans. 1764}{}}. 8. What is the value of 47 –H 1561 -- 7 -- 142 × 20 + 522 -i- 87 –H 207 -- 23 + (342 — 150) × 62. Ans. 15029. 231. MISOELLANEOUS PROBLEMS IN DIVISION. 1. One of the factors of 10800 is 225. What is the other ? Ans. 48. What number multiplied by 137, will give 959137 for the product? Ans. 7001. 3. Multiplying 372 by an unknown number gives 44640. What is the num- ber 3 Ans. 120. 4. What is the quotient of 9126 divided by 9% AnS. 1014. 5. Divide four million eight thousand sixteen by MMDCXLIV. Ans. 1515}}#}. 6. The great church of St. Peter, in Rome, will accommodate 57000 people. The largest church in New Orleans will accommodate 2500. How many times is the capacity of the church of St. Peter equal to that of the largest church in New Orleans ? Ans. 22#. 7. What number is that to which, if sixteen be added, the sum multiplied by 8, and 13 subtracted from the product, the remainder will be 3393 Ans. 28. OPERATION. 339 + 13 = 352 -- 8 = 44 — 16 = 28 Ans. The student will write the full reasoning. 8. There is a number from which, if you subtract 55, and divide the remain- der by 12, your quotient will be 36. What is that number 3 Ans. 487. I 36 SOULE's PHILOSOPHIC PRACTICAL MATHEMATICs. X. OPERATION. The student will write the full reasoning. 9. A merchant owes a debt of $1875, which he agreed to pay by weekly installments of $25. He has made 55 payments. How many more payments has he to make 3 Ans. 20. 10. The velocity of the earth on its yearly voyage around the sun is 99733 feet per second. The velocity of a cannon ball fired from a gun with an average charge of powder is about 1750 feet a second. How many times as fast as the veloc- ity of a cannon ball, is the velocity of our earth ? Ans. 56}#}. 11. About one-siath of a man's weight is blood. How many pounds of blood in a man whose weight is 168 pounds 3 Ans. 28 pounds. 12. A merchant bought 350 barrels of flour at $6 per barrel, and sold it at $7.50 per barrel. The gain he gave, in equal parts, to four worthy boys, to aid them in obtaining an education. What was the cost and the selling price of the flour, and how much money did each boy receive 3 Ans. $2100 cost, $2625 selling price, $131.25 each boy received. 13. The air which surrounds our earth, and of which we each inhale about 600 gallons every hour, is composed of four parts of nitrogen, and one part of oxy- gen. How many gallons of each are there in a room 22 feet long, 21 feet wide, and 10 feet high, and which contains 34560 gallons of air 3 Ans. 6912 oxygen, 27648 nitrogen. 14. An acre contains 160 Square rods. How many acres in a plantation con- taining 123200 square rods 3 Ans. 770 acres. 15. A boy sold 50 oranges at 52 each, and thereby gained $1.50. At what rate did he buy the Oranges } Ans. 2% a piece. 16. How many times 136 will produce 1768 % Ans. 13. 17. Divide the product of 750 and 875, by their difference. Ans. 5250. 18. The diameter of the earth at the equator is 7925 miles. How long would it take a locomotive to travel that distance, at the rate of 25 miles an hour * Ans. 317 hours = 13 days, 5 hours. 19. It is estimated that, by reason of intemperance, the United States loses annually $98400000. How many school houses costing $5000 each, and how many libraries costing $3000 could be established with this amount of money 2 Ans. 12300 of each. 20. The first Atlantic telegraph cable, as originally made, cost $1258250. Ten miles of deep sea cable were made at a cost of $1450 per mile, and 25 miles of shore ends were made at a cost $1250 per mile. The remainder cost $485 per mile. How many miles of cable were made 3 Ans. 2535 miles. ^. 21. The circumference of our earth at the equator is 24899 miles, and the mean diameter of the earth is 79.12 miles. How many times is the circumference as great as the mean diameter ? Ans. 3.}}#} times. 22. A grocer wishes to put 3335 pounds of sugar in three kinds of boxes, containing respectively, 20, 50, and 75 pounds, and use the same number of boxes of each kind or size. How many boxes will he require? Ans. 23 of each size. 4× DIVISION. I37 23. The capacity of steam engines is measured by horse power; and 1 horse power is a force that will raise 33000 pounds, one foot in one minute. How much horse power has a steam engine that possesses a capacity of 1188000 pounds 2 Ans. 36. 24. The average weight of a man is 150 pounds, and about # of this weight is blood. Allowing that the heart throws out two ounces of blood at each pulsation, that it beats 72 times a minute, and that 16 ounces make a pound, how long will it take the heart to circulate all the blood in the body? Ans. 2+}# minutes. 25. Ten freedmen agreed to pick 20000 pounds of cotton and receive for their labor # of the cotton picked. After they had picked 7000 pounds, four freed- men quit, leaving the other six to finish the work. How much cotton is each entitled to When the work is finished ? Ans. 140 pounds each for those who left, and 573% each for those who remained. 26. The Old and New Testaments contain 1189 chapters and 31164 verses. If a person were to read 12 chapters every Sunday, how many Sundays would it require to read the contents of the whole Bible? If he were to read 85 verses a day, how many days would it require to read the contents of the whole book? Ans. 99 tº Sundays; 366 # days. 27. A merchant bought 800 gallons of molasses at 652, and sold # of it at 72% a gallon. From the profit he bought his children a set of Cutter's Anatomical and Physiological Charts, and had $8.20 left. What did the charts cost 3 Ans. $19.80. 28. A man produces, by breathing, at least six gallons of carbonic acid gas every minute; a single burning gas jet, ten gallons; an ordinary stove, sixty gal- lons. How many gallons of carbonic acid gas will two heated stoves, fifty burning gas jets, and an audience of 1000 people, produce in three hours, and how many times would the quantity fill a room 100 feet long, 50 feet wide, and 30 feet high 3 Ans. 1191600 gallons. 1% times. 29. A merchant pays $180 for 12 dozen shirts. At what price per shirt must he sell them to gain $723 Ans. $1.75. 30. A tailor paid $117.60 for a piece of cloth, the price was $2.80 a yard. How many yards did it contain } Ans. 42. 31. Paid $4569 for paving 1523 square yards. What was the price per square yard? Ans. $3.00. 138 SOULE's PHILOSOPHIC PRACTICAL MATHEMATICs. { CONTRACTIONS IN DIVISION. 232. 1. Divide 9746521 by 634. OPERATION. ū34)9746521(15373 ºr Ans. 3.06. Ea:planation.—The operation of this problem is abridged 2365 by performing mentally, the subtraction of the product of 1632 the divisor by the quotient figure, instead of placing it -º-º-ººs under the dividend and then subtracting, as is ordinarily 1941 done. 39 Or, to arrange the figures differently, which we much prefer, as follows: 3241 - 4369 In this problem, the subtraction is performed mentally, 0634. as in the example above, but the remainders are placed in a 634)9746521 º * * * sº tº t o - vertical position over the dividend, so as to avoid repeating 15373 ºr Ans. the figures of the dividend. TO DIVIDE BY THE FACTORS OF THE DIVISOR. 233. Divide 8316 by 36. OPERATION. 36) 8316 Ea:planation.—Here we first divide by 6, which gives *m- e us a quotient 6 times too large, because 6 is but # of 36 1386 guotient by 6 © º 5 y Q. y the true divisor; and hence, to obtain the correct result, 231 Ans. we divide the first quotient by 6. * DIVISION. I 39 CONTRACTIONS. 234. Peculiar to problems when the divisor is an aliquot part of 10, 100, or 1000, or some convenient multiple of 10, 100, or 1000. Thus, to divide by 1% multiply the dividend by 9 and divide the product by 10 1+ {{ {{ {{ 8 {{ 4. {{ 10 1#. 4. $é {{ 7 44 4% « 10 1#. {{ {{ {{ 6 {{ {{ {{ 10 2} {{ \ {{ {{ 4 {{ 4% {{ 10 3} {{ {{ {{ 3 {{ 4% {{ 10 6# {{ {{ {{ 16 4. {{ {{ 100 8% {{ {{ {{ 12 {{ {{ {{ 100 12#. {{ {{ {{ 8 {{ * {{ {{ 100 14% {{ {{ 44 7 {{ {{ {{ 100 16# & 4 & 4 & 4 6 {{ {{ {{ 100 18; {{ {{ {{ 16 {{ 4. 44 300 22} & 4 44 & 4 4 {{ {{ . {{ 90 25 44. {{ {{ 4 {4 {{ {{ 100 31#. {{ & 4 {{ 16 {{ {{ - {{ 500 33% 44 {{ {{ 3 {{ {{ {{ 100 37; {{ * .. {{ 8 {{ {{ {{ 300 624 {{ {{ {{ 8 & 4 {{ {{ 500 66# {{ {{ 46 3 {{ {{ {{ 200 75 4% {{ {{ 4 44 {{ {{ 300 83; {{ {{ {{ 6 { % { % { % 500 87; {{ { % {{ 8 {{ 44 44 700 88; {{ {{ {{ 9 44 {{ 44 800 125 {{ & 4 44 8 {{ {{ 44 1000 133} { % 44 44 3 & 4 & 4 44 400 1663 {{ {{ { % 6 44 {{ & 4 1000 225 {{ { { {{ 4 44 {{ 4. 900 250 {{ 4% {{ 4 4% 4. {{ 1000 333; {{ {{ 44 3 {{ {{ 44 1000 375 {{ {{ {{ 8 {{ 44 {{ 3000 625 44 4% 4% 8 {{ {4 {{ 5000 833% {{ 4% {{ 6 {{ {{ 44 5000 875 {{ 4% 4% 8 {{ {{ {{ 7000 15 {{ 4% 4% 2 {{ {{ {{ 30 35 4% 44 4% 2 {{ 44 {{ 70 45 {{ 44 {{ 2 {{ {{ 44 90 The reasons for these contractions are based upon the fact that in all division operations the result or quotient is not changed by multiplying the divisor and the dividend by the same number. .* I4O SOULE's PHILOSOPHEC PRACTICAL MATHEMATICs. To elucidate this work, we present the following problems: *. 3. Divide 7242 by 24. OPERATION. 2} | 7242 Explanation.—By inspection, we observe that 4 times 2} are 10; we 4 4 therefore, to save time and labor, first multiply it by 4, to produce a **- new and more convenient divisor; and in order not to effect a change 10 28 • or error, in the quotient, we also multiply the dividend by 4 for a new 96.8 Ans. dividend, and then divide in the regular manner which, in this case, is done by simply pointing off one figure. 4. Divide 179801 by 334. OPERATION. Eacplanation.—-In this problem, we also see by inspection that 3 33 #) 179801 times the divisor makes 100, hence for the reasons given above, we mul- tiply the dividend by 3, and divide the product by 100. In practice, we -* *me do not set the figure (3 in this example) by which we multiply the 5394.03 Ans. dividend. 5. Divide 12358 by 373. OPERATION. 373) 12358 Explanation.—We here observe, by inspection, that the divisor 37} -*mºms is § of a hundred, and hence, if we multiply it by 8, we will have for 39 Ø ) 98864 a new divisor 300; therefore, for reasons given above, we multiply the divisor and the dividend by 8, and then divide in the usual manner. 329}# Ans. “ y &, 6. Divide 97450 by 75. OPERATION. 75) 97450 tº - tº - Explanation.—By inspection, we here observe that 4 times the 399) 38.98% divisor makes 300; we therefore multiply the divisor and dividend by 4 and with their products proceed to divide. 1299; Ans. 7. Divide 4307491 by 125. OPERATION. 125) 4307491 Ea:planation.—In this problem, we observe that 8 times the divisor makes 1000; we therefore multi- 1000) 34459928 = 34459 +}} Ans. ply the dividend by 8 and divide the product by 1000. PECULLAR CONTRACTIONS. 235. In the preceding contractions the work was based upon the principle that multiplying the divisor and dividend by the same number does not effect a change in the quotient. In the following contractions the work is based upon the principle that, to add to, or subtract from both divisor and quotient, the same aliquot parts, will effect no change in the result. ¥ DIVISION. I41 By the application of these principles, we can often increase or diminish either divisor or dividend, or both, by aliquot parts, and thus obtain a more con- venient divisor. 8. Divide 426 by 74. OPERATION. Ea:planation.—In this problem, we first divide by 10, and then 73) 42.6 add # of the quotient to itself. This we do because by inspection, 14.2 we observe that 73 is # of 10, or, that it is # of itself less than 10; hence it follows that if we divide by 10, our quotient will be # of 4 itself too small, and to obtain the correct quotient we must add # 56.8 = 56% Ans. of the quotient by 10 to itself. NOTE.-To add to, or deduct from, the dividend before dividing, will produce the same result as adding to, or deducting from, the quotient. 9. Divide 2548230 by 24. OPERATION. # of 24 254823% is 6 — Explanation.—In this problem, we first divide by 30 and then add 39 | #, + of the quotient to itself. The reason for the work is the same as *--> that given in the above problem. 106.1764 Ans. 10. Divide 8316 by 36. FIRST OPERATION. SECOND OPERATION. Explanation.—In the first operation, we de- 1. g duct from the dividend and divisor # of them- 6 Of 36 # 1 36 8316 selves, which decreases each in like proportion, IS 6 = 6 - 6 and then divide. *- : - 39 277 ºr In the second operation we divide by 30, 39 || 6939 46 ºr which gives a quotient # too large, because the 30 with which we divide is # º * 36. the true divisor : hence we deduct # of the 231 Ans. 231 Ans. quotient by 30 from itself, and obtain in the re- mainder the true result. NoTE.—In nearly all contractions like the above, it is easier and shorter to divide by the faç- tors of the divisor. Hence we give but few problems, merely to show the application of the princi- ple. PROBLEMS. Divide the following numbers: 1. 4897.20 by 54. 2. 8725 by 35. 3. 26748 by 48. 4, 8316 by 36. SYNOPSIS FOR REVIEW OF DIVISION. named. 207. 209. 210. 211. 212. 213. 214. 215. 216. 217. 218. 219. 220. 221. -º- -Q. © NOTE.—The numbers before the words and phrases refer to the articles treating the subjects How to Divide when the Divisor does not exceed Twelve. Short Division. To Divide when the Divisor ex- ceeds Twelve. Give the General Directions for Long Division and all the De- tails for the Full Operation. To Divide when there are Naughts on the Right of the Divisor. To Divide by the Factors of the Divisor. Operation in English Money. 236. Define the following words and phrases: Division. 222, The Logic of Division. The Dividend. 223. The Divisor. 224. The Quotient. The Remainder. 225. The Sign of Division. Other Signs of Division. Principles of Division. 226. Proof of Division. How many ways may Division be 227. performed. Oral Exercises. 228. Fractional Numbers. 229. The Philosophy of Division. To Find-the True Remainder. →3' T TO *. (142) MISCELLANEO US PROBLEMS INVOLVING THE PRINCIPLES OF ADDITION, SUBTRACTION, MULTIPLICATION, AND DIVISION. a .44 º A-La y—w Yºrv -w 237. 1. The subtrahend is 216, and the remainder is 184. What is the min- uend ? Ans. 400. 2. A grocer paid $350 for some tea and some coffee, and for the tea he paid $50 more than for the coffee. What did he pay for each 3 Ans. tea, $200; coffee, $150. 3. The sum of two numbers is 787, and one of them is 298. What is the other ? Ans. 489. 4. The difference between two numbers is 97, and the lesser number is 103. What is the greater ? Ans. 200. 5. What number multiplied by 4 will give the same product as 16 multiplied by 127 Ans. 48. 6. If the speed of the steamer J. M. White is 15 miles per hour in still water, and the velocity of the river is 3 miles per hour, how far will she run up the river in 4 hours? How many miles down the river in 4 hours ? How far if she runs in still water 4 hours ? Ans. 48 miles up; 72 miles down; 60 miles in still water. 7. If two men start from the same point and travel in opposite directions, one at the rate of 20 miles per day and the other at the rate of 25 miles per day, how many days will they travel before they are 495 miles apart 3 Ans. 11. 8. The greater of two numbers is 897 and their difference 598; what is the lesser ? Ans. 299. 9. A newsboy sold 20 papers at 5% each, and with the money bought oranges at 4% each. How many oranges did he get? Ans. 25. 10. A boy sold 5 chickens at 25% a piece, and 8 ducks at 50% each. He received in payment 3 pigeons at 30% each, and the balance in money. How much money did he receive 3 Ans. $4.35. 11. The divisor is 187 and the quotient 101, what is the dividend? Ans. 18887. NoTE.–In division operations, the worker must remember, 1. That the divisor, while it is the unit of measure, is, also, one of the factors of the dividend. 2. That the quotient is the other factor of the dividend. 3. That the dividend is the product of the divisor and the quotient, plus the remainder when there is one, and hence, dividing the dividend by either factor will give the other, and the remainder if any. (143) I44 soul E's PHILOSOPHIC PRACTICAL MATHEMATICS. X- 12. The product of two numbers is 42230 and one of them is 205; what is the other ? Ans. 206. 13. A grocer bought 28 barrels of apples at $4 per barrel; at what price must he sell them per barrel to realize a gain of $42? Ans. $54. 14. The dividend is 5484888 and the quotient 81864, what is the divisor 3 Ans. 67. 15. What number divided by 33 will give the quotient 30449, and remain- der 18% Ans. 1004835. 16. J. J. Foley bought a barrel of sirop de batterie containing 43 gallons at 95% per gallon; 4 gallons having leaked out, he sold the remainder at $1.05 a gallon. How much did he gain by the transaction ? Ans. $.10. 17. R. S. Soulé bought 354 barrels of flour for $2478, and sold the same at $7.50 per barrel. How much did he gain 3 Ans. $177. 18. The capital stock of a manufactory is $100000, which is divided into 200 shares. What are 5 shares worth ? Ans. $2500. 19. R. D. T. Sherwood sold to H. Ries 25 barrels of apples at $4 per barrel, and 124 barrels of potatoes at $3.25 per barrel. He received in payment 1 hogshead of Sugar containing 1143 pounds at 82, and the remainder in money. How much money did he receive 3 Ans. $411.56. 20. A speculator bought 528 cords of wood at $6.50 per cord. He re-corded the wood so that it measured 579 cords, which he sold at $6.75 a cord. How much did he gain } Ans. $476.25. 21. The dividend is 7359, the quotient 198, and the remainder 33; what is the divisor Ž Ans. 37. 22. A. and B. Were partners in business, equal in gains and losses; A. managed the business, for which he was to receive from the gains $1200. The gains of the business were $18524 and the losses $7108. What amount of money was due each on settlement 3 Ans. A. $6308, B. $5108. 23. A fruit dealer bought 250 boxes of peaches for $375; he sold 94 boxes at $2 per box, and 147 boxes at $3 per box. The remainder of the peaches spoiled in his store; his expenses in buying and selling were $45. How many boxes were spoiled, and how much did he gain? Ans. 9 boxes; $209 gain. 24. A cistern holding 1500 gallons, has two pipes, one of which will fill it in 10 hours, and the other empty it in 30 hours. If both pipes are left open, how long will the cistern be in filling 3 Ans. 15 hours. 25. A cistern holding 3000 gallons, has three pipes; by one 30 gallons per minute, and by another 20 gallons per minute, are conducted into it, and by the third it may be emptied when full in 24 hours. If all three pipes are left open, in what time will the cistern be filled? Ans. 100 minutes NOTE.-There are 60 minutes in an hour. Yºr MISCELLANEOUS PROBLEMS. I45 26. Edward has $165.00. One-fifth of the amount is in 50-cent pieces, the remainder is in 25-cent, 10-cent, 5-cent, 3-cent, and 1-cent pieces; allowing that he has an equal number of each coin, how many has he 3 Ans. 66 50-cent pieces, and 300 of each of the other coins. 27. A merchant had 200 bbls. of potatoes for which he was offered $4.00 per barrel, which price would have given him a gain of $20. Having declined this offer, on a declining market, he finally sold at a loss of $20. How much per barrel did his potatoes cost, and how much per barrel did he sell them for? Ans, cost $3.90, and sold for $3.80 per bbl. 28. A government ration consists of 1 lb., 4 oz. of beef, and 1 lb., 6 oz. of flour; if beef costs 1032 a lb. and flour 3g2 a lb., what will be the cost of 10840 government rations, and how many lbs. of, beef and flour will there be? - Ans. $1983.38, cost; 14905 lbs. of flour; 13550 lbs. of beef. NOTE.—There are 16 ounces in a pound. OPERATION INDICATED. 1 lb., 4 oz. = 14 lbs. beef (a) 10% = 1 * 6 “ = 1; “ flour à 3%g = Cost of one ration - - - - = $ 1+ x 10840 = lbs. beef. 13 × 10840 = “ flour. 29. A speculator bought an invoice of flour and pork for $297; the flour cost $7 a barrel, and the pork $12; there were 3 times as many barrels of flour as of pork. How many were there of each 3 Ans. 27 bbls. flour, 9 bbls. pork. £roperties of Numbers. D E FINITION S. * 238. The Properties of numbers are those qualities which belong to them. 239. An Integer is a whole number; as 1, 5, 6, 18, etc. Whole numbers are divided into two classes, Prime and Composite. 240. A Prime Number is one that can be divided, without a remainder, only by itself and 1; as 1, 2, 3, 5, 7, 11, 13, 17, etc. 241. A Composite Number is one that can be divided, without a remainder, by some other whole number than itself and 1; as 4, 9, 12, 15, 24, etc. All Composite Numbers are the product of two or more other numbers. Numbers are prime to each other when they have no common factor that will divide each without a remainder; as 6, 13, 29, etc. 242. An Even number is one that can be divided by 2, without a remain- der; as 4, 8, 12, 56, etc. 243. An Odd number is one that cannot be divided by 2, without a remain- der; as 1, 7, 19, 45, 133, etc. 244. The Factors of a number are the numbers which multiplied together will produce it. Thus, 4 and 4, or 2 and 8, are factors of 16; 2, 3, and 4, or 2, 2, 2, and 3 are factors of 24. Every factor of a number is a divisor of it. *NOTE.-For other Definitions of Numbers, see pages 25 to 31. 245. A Prime Factor of a number is a prime number that will divide it without a remainder. Thus, 1, 2, 3, and 5, are the prime factors of 30. 246. A Composite Factor of a number is a composite number that will divide it without a remainder. Thus, 6 and 8, are composite factors of 48. 247. A Perfect Number is one which is equal to the sum of all its divisors, except itself; as 6 = 1 + 2 + 3. 248. An Imperfect Number is one, the sum of whose divisors is more or less than itself. Thus, 14 is greater than 1 + 2 + 7, its divisors; and 18 is less than 1 + 2 + 3 + 6 + 9, its divisors. This last is called an abundant number. (146) Yºk PROPERTIES OF NUMBERS. I47 249. An Aliquot part of a number is such a part as will divide it without a remainder. Thus, 1, 2, 3, 4, 6, and 8, are aliquot parts of 24. All aliquots are factors of the number. 250. The Reciprocal of a number is the quotient of 1 divided by the number. Thus, the reciprocal of 8 is 1 + 8 = }; and the reciprocal of # is 1 + 4 = 4. 251. Powers of a number are as follows: The first power is the number expressed by itself; as 5 is the first power of 5. The second power is the product arising by using the number as a factor twice; as 5 x 5 = 25, which is the second power of 5. The third power is the product obtained by using the number as a factor three times; as 5 × 5 × 5 = 125, which is the third power of 5. And in like manner higher powers of numbers are obtained. * 252. A Multiple of a number is one or more times the number; or it is any product, dividend, or number of which a given number is a factor, or which is exactly divisible by a given number; as 25 is a multiple of 5; 24 is the multiple of 2, 3, 4, 6, 8, and 12. A number may have an indefinite number of multiples; as 2 will divide 4, 6, 8, 10, 12, 14, etc., indefinitely. 253. A Common Multiple of two or more given numbers is a number divisible by each of them without a remainder. Thus, 24 is a common multiple of 1, 2, 3, 4, 6, 12, and 24. 254. The Least Common Multiple of two or more given numbers is the least number that is divisible by each of them without a remainder. Thus, 12 is the least common multiple of 1, 2, 3, 4, 6, and 12. NOTE.-Since every number is divisible by 1 and itself, the factors 1 and the given number aré not usually given when naming the multiples. We shall not hereafter name them as multiples. DIVISIBILITY OF NUMBERS. 255. A Divisor or measure of a number, is any number that will divide it without a remainder. Thus, 4 is a divisor, or measure, of 12, and 5 is a divisor, or measure, of 20. g’ One number is said to be Divisible by another when there is no remainder after dividing. 256. A Common Divisor of two or more numbers is a number that will divide each of them without a remainder. Thus, 2 is a common divisor of 12, 18, and 24. 257. The Greatest Common Divisor of two or more given numbers is the greatest number that will divide each of them without a remainder. Thus, 6 is the greatest common divisor of 12, 18, and 24. - I 48 soule's PHILOSOPHIC PRACTICAL MATHEMATICS. 2} 258. Every number is divisible by 2, whose unit figure is divisible by 2. Thus, 34, 176, 790 are each divisible by 2. * 259. Every number is divisible by 4 when its units' and tens' figures are divisible by 4. Thus, 156, 264, 34512, 56.1308, are each divisible by 4. 260. All numbers are divisible by 3, the sum of whose figures are divisible by 3. Thus, 114, 225, 4101, are each divisible by 3. 261. All numbers ending in 0 or 5 are divisible by 5. Thus, 10, 15, and 35, are each divisible by 5. 262. All numbers whose unit figure is divisible by 2, and the sum of whose figures is divisible by 3, are divisible by 6. Thus, 36, 102, 678, 15936, are each divisible by 6. 263. Every number is divisible by 8, when the units', tens', and hundreds' figures are divisible by 8. Thus, 3824, 12512, 190720, are each divisible by 8. 264. All numbers are divisible by 9, the sum of whose figures are divisible by 9. Thus, 441, 3456, 123453, are each divisible by 9. 265. All numbers ending in naught are divisible by 10. Thus, 20, 380, 11750, are each divisible by 10. 266. A number is divisible by 12 when the sum of its figures is divisible by 3, and the units and tens are divisible by 4. 267. The combination of numbers that are divisible by 7, and 11, are so Varied and intricate that they cannot be conveniently classified or formulated under any brief general law, and being also of very little practical value, they are not presented. SYNOPSIS FOR REVIEW. Inamed. 238. 239. 240. 241. 242. 243. 244. 245. 246. 247. 248. 249. º NOTE.-The numbers before the words and phrases refer to the articles treating the subjects 268. Define the following words and phrases: The Properties of Numbers. An Integer. A Prime Number. A Composite Number. An Even Number. An Odd Number. Factors of a Number. A Prime Factor. A Composite Factor. A Perfect Number. An Imperfect Number. An Aliquot. 252. 253. 254. 255. 256. 257. 358. 259. 250. 251. The Reciprocal of a Number. The Powers of a Number; 1st. 2d, 3d, etc. The Multiple of a Number. A Multiple. A Common Multiple. Least Common Multiple. A Divisor. - A Common Divisor. Greatest Common Divisor. What numbers are divisible by 2, 3, 4, 5, 6, 8, 9, 10, 12. (149) *roperties of Nines and Elevens. ===N *_* a Lºh-ººmm 269. The number nine has been called the “Magical Number.” It pos- sesses a great many peculiar properties, some of which are more curious than practical, and will be but briefly treated here. Among the Ancients, the special properties of 9, and of some other figures, were used in their magic squares and circles, and in their numerical combinations and puzzles, to such an extent as to induce the belief, among the ignorant, that they possessed supernatural power, or that numbers possessed material virtues. On page 55, we presented a very peculiar property of 9, applied to the opera- tion of addition. A curious principle of 9 is shown in the following: Take any number of two places of different figures, transpose the figures, and take the difference between the two numbers, and name one of the figures of the difference, and the other figure is known. Thus, take 73, transpose the figures, and we have 37; then 73–37 = 36 the difference or remainder, in which we see that the sum of 3 and 6 = 9. In all similar cases, the sum of the remainder will be 9, and hence when one of the figures is known the other may be found by subtracting the one known from 9. Another interesting puzzle based upon the same principle is to write any number of three or more figures, divide by 9, and give the remainder; then erase any one of the figures in the number and divide by 9, and give the remainder. The figure erased will be the difference between the remainders, when the second re- mainder is less than the first ; but when the second remainder is greater than the first, the figure erased will be the difference of the remainders subtracted from 9. Thus, 4381 divided by 9, gives 7 remainder. Then cancelling 3 and dividing 481 by 9, gives 4 remainder, and 7–4 = 3 the figure erased. Again, taking 4381 -- 9 gives 7 remainder. Then erasing 8 and dividing 431 by 9, gives 8 remainder. Then 8–7 = 1 and 9 — 1 = 8, the figure erased. Again, let the reader take any number and divide it by 9, and give the remainder. Then multiply the number by any number given, and divide the prod- uct by 9, and the remainder may be told before the work is performed. Thus, 428 –- 9 gives 5 remainder. Now multiply the 428 by 4 and divide the product by 9, and the remainder will be 2. To determine this remainder, we multiply the first remainder by the number used to multiply the first number and divide the product by 9; the remainder thus obtained will be the same as the remainder in the second division. But, as our purpose is now not to present a discussion of these questions except to apply one of the properties of 9 and 11 to the proof of addition, sub- traction, multiplication, and division, we will not occupy space to discuss the plailosophy or reason of these puzzling results. (150) * PROPERTIES OF NINES AND ELEVENS. I5 I The basis or radix of our numerical system being 10, equal to 9 + 1, if we multiply the radix and its equivalent by the same digit, thus 10 x 4 = 40, and 9 + 1 × 4 = 36+ 4 = 40, we find the product of 9 alone to lack just the multiplier of being equal to the product of 10, and as this will hold good of any multiple of 10, we see that if we divide any decuple as 40, 60, 80, etc., by 9, the remainder is always the same as the number of tens divided, viz: 4, 6, 8, etc., and if we multiply these numbers by 10, or any power of 10, as 400, 6000, etc., the remainders will still be the same digits 4, 6, etc. Therefore we may conclude that any number as 467853 can be resolved into as many multiples of 10 as it contains digits, thus: 400000 + 60000 + 7000+ 800 + 50 + 3, and since the excess of 9's in each of these round numbers is the same as its significant figure, it is plain the excess of 9's in the entire number 467853 is equal to the excess of 9's in the sum of its digits, thus: 4 + 6 + 7 + 8 + 5 + 3 = 33; divide this by 9 gives quotient 3, and a remainder or excess of 6. In finding the excess of 9's, it is best to reject the 9's as soon as its sum occurs in the additions, thus: 4 + 6 = 10, dropping 9 = 1 + 7 + 8 = 16, dropping 9 = 7 + 5 = 12, dropping 9 = 3 + 3 = 6 excess. This property is peculiar to 9 by reason of its being 1 less than the radix of Inotation. This property belongs also to any number that will give a remainder 1 when dividing the radix by it, as 3, and 3 and 9 are the only numbers that will divide 10, with 1 for a remainder; if the basis of our system were 9, then 8, 4, and 2, would possess these properties. The number 11 has several special properties, one of which is similar to that of 9 and results from the fact that 11 is 1 more than the radix 10. The usual method of finding the excess of 11's is somewhat different, viz.: by adding the alter- nate figures; thus, to cast the 11's out of 92257458, we first find the sum of the figures in the odd places, beginning at the right hand, thus: 8 + 4 + 5 +2 = 19, casting out the 11's gives us an excess of 8; we then add the figures in the even places, thus: 5 + 7 -H 2 + 9 = 23, giving an excess of 1; subtract this last excess from the first, 8–1 = 7, the excess required; if the first excess was the smaller, we would increase it by 11, then subtract the second. The excess of 11’s may be cast out at the partial additions, as for 9’s. These two properties of 9 and 11 can be used in proving addition, subtrac- tion, multiplication and division, depending on this principle, that any number divided by 9 or 11 leaves the same remainder as the excess obtained by casting out its 9's Or 11's. These methods of proof, however, are not absolutely infallible, as will be shown on the second page following. By reason, therefore, of the liability to error, the great majority of business men, in verifying their calculations, prefer to repeat or go over, in a reversed direction, the first operation. I 52 * SOULE's PHILOSOPHIC PRACTICAL MATHEMATICs. PROOF OF ADDITION BY CASTING OUT THE NINES AND ELEVENS. 270. 1. Add the following numbers and prove the work by casting out the 9's and 11’s. OPERATION AND PROOF BY CASTING OUT THE 9’s. Excess of 9’s. 365592 3 286548 6 64320 6 94094 8 = 23 = 5 excess of 9’s in the numbers. Sum 810554–5 excess of 9's in the sum. NOTE.-Instead of casting the 9's out of each number separately, they may be cast out of all the numbers continuously. Thus we may say 3 + 6 = 9, which is dropped. Then 5-H 5 = 10 — 9 = 1 (omitting the 9) + 2 = 3 = 13 — 9 = 4 + 6 = 10 — 9 = 1 10 — 9 = 1 + 8 = 9 — 9 = 0, 6 1 + 3 = 4 + 2 = 6 (omitting 9) = 1, (omitting 9) + 4 = 5 excess numbers. + + of OPERATION AND PROOF BY CASTING OUT THE 11’s. Excess of 11's. 365592 7 286548 #) 64320 3 94094 0 y = 19 = 8 excess of 11’s in the numbers. Sum 810554–8 excess of 11’s in the sum. NOTE.-Instead of casting the 11's out of each number separately, they may be cast out of all the numbers collectively. Thus we may add the figures in the odd places of all the numbers, making 56 = 1 excess of 11; then add the figures in the even places of all the numbers, making 48 = 4 excess of 11's ; then subtract the last ex- cess from the first increased by 11, thus 1 + 11 = 12 – 4 = 8 excess of 11’s in all the numbers. Explanation.—To prove problems of addition by casting out 9's or 11's, first find the excess of the 9's or 11's in all the numbers; then find the excess of 9’s or 11's in the sum of the numbers; if the two excesses are the same the work is correct, as far as this proof can determine. THE CHECK OR KEY FIGURE. 271. The excess of 9’s or 11's is called the check or key figure by account- ants, and is used not only to prove the operations of addition, subtraction, multipli- cation and division, but to prove other operations of accounting. CONTRACTED METHODS OF FINDING THE CHECK OR KEY FIGURE. 272. 1st CONTRACTED METHOD, Add the following numbers and prove by the check figure system: 365592 286548 64320 94094 = 104 = 5 check figure of all the sºmmemºmºsºms numbers. 810554 = 23 = 5 check figure of the Sum. Explanation.—By the first contracted method, add the figures in all the numbers as units = 104; then add these figures as units = 5 the check fig- ure of all the numbers. Then add the figures in the sum as units = 23, which add as units = 5 the check figure of the sum. The addition of the several sums is to be con- tinued as long as any succeeding sum has more than one figure. The check figure of all the nunabers, and the check figure of the sum being equal, proves the work as far as the method can. | 2d CONTRACTED METHOD. Add the following numbers and prove by the check figure system: 365592 = 30 = 3 286548 = 33 = 6 64320 = 15 94094 = 26 * * Gº ſº- ; — 23 = 5 check figure. 810554 = 23 = 5 check figure. Explanation—By the second contracted method, add the figures of each number as units and then add the figures of the result, when it has more than one figure. The results are the check fig- ures of each number. Then add the check figures and the sum of the same, and thus produce 5 as the check figure of all the numbers. Then add the figures of the sum of all the numbers as ' units, and also add this sum which gives 5, the check figure. The first of these contracted methods of finding the check figure, or the earcess of 9's, is new, and is now published for the first time. Y PROOF BY CASTING OUT THE 9's OR II's. 153 ERRORS NOT DETECTED BY CASTING OUT THE NINES, OR BY THE NINE CHECK FIGURE METEIOD. 273. 1. When figures have been transposed. 2. When the value of one figure is as much too great as that of another is too small. 3. When a figure or figures have been omitted, and other figures having the same sum have been sub- stituted. 4. When 9 or 9’s have been omitted. ERRORS NOT DETECTED BY CASTING OUT THE ELEVENS, OR BY TEIE ELEVEN OEIECK FIGURE METEIG) D. 274. When alternate figures, i. e. figures in the odd, or figures in the even places, have been transposed, thus, 234567 = 3 excess; then 234765 = 3 excess; or 236547 = 3 excess. 2. When two figures of the same unit value in the odd and even places have been transposed for two other contiguous figures, whose value is the same but differ- ent from the value of the other two, thus, 335644 = 1 excess; then 445633 = 1 excess, or 564433 = 1 excess. 3. When figures of the same unit value in the odd and even places have been omitted, thus, 45567 = 5 excess; then 467 = 5 excess. e By the foregoing, we see that the method of proof by casting out the 11's, is more reliable than the method by casting out the 9’s. PROOF OF SUBTRACTION BY CASTING OUT THE NINES OR ELEVENS. 275. From 1748272 take 1451604, and prove the work by casting out the 9's and 11's. OPERATION AND PROOF BY CASTING OPERATION AND PROOF BY CASTING: OUT THE 9’s. OUT THE 11's. Excess of 9's. Excess of 11's. Minuend, 1748272 4 1 = difference 1748272 = 9 Subtrahend, 1451604 3 } of excess. 1451604 0 | 9=difference of excess. Remainder, 296668 = 1 excess. 296668 = 9 excess in remainder. Explanation.—-To prove problems in subtraction by casting out the 9's or 11's, first find the excess of 9’s or 11's in the minuend and subtrahend, and take the difference. Then find the excess of 9’s or 11's in the remainder; if the excess in the remainder is the same as the difference of the excesses of the minuend and subtrahend, the work is correct, as far as this proof can determine. I54 SOULE's PHILOSOPHIC PRACTICAL MATHEMATICs. XF PROOF OF MULTIPLICATION BY CASTING OUT THE NINES OR ELEVENS. 276. and 11's. , OPERATION AND PROOF BY CASTING OUT THE 9’s. Excess of 9's. Multiplicand, 350412 6 ) 6 × 3–18– Multiplier, 29595 3 } 0 excess. Broduct, 10370443140=0 excess. Multiply 350412 by 29595, and prove the work by casting out the 9's OPERATION AND PROOF BY CASTING OUT THE 11’S. Excess of 11's. 350412 7 7 JR_35_o a 10370443140 = 2 exceSS. Explanation.—To prove multiplication by casting out the 9’s or 11's, first find the excess of 9’s or 11's in the multiplicand and multiplier, and then find the excess of the product of these two eXCeSSéS. Then find the excess in the product; if the excess in the product equals the excess of the product of the excesses of the factors, the work is correct, as far as this proof can determine, PROOE OF DIVISION BY CASTING OUT THE NINES OR ELEVENS. 277. Divide 65358547823 by 2789, and prove the work by the excess of 9's • and 11's. OPERATION AND PROOF Ol]T THE 9’s. Dividend, 65358547823 = 2 excess of 9’s. BY CASTING Divisor, 2789 = 8 excess of 9’s. Quotient, 23434.402 = 4 excess of 9’s. Itemainder, 645 = 6 excess of 9’s. 8 × 4 = 32 = 5 excess of 9’s. 5 + 6 = 11 = 2 excess of 9’s. OPERATION AND PROOF BY CASTING OUT THE 11’s. Dividend, 65358547823 = 8 excess of 11's. Divisor, 2789 = 6 excess of 11’s. Quotient, 23434.402 = 2 excess of 11's. Remainder, 645 = 7 excess of 11’s. 6 × 2 = 12 = 1 excess of 11’s. 1 + 7 = 8 = 8 excess of 11’s. Explanation.—To prove division by casting out 9's or 11's, first find the excess of 9’s or 11's in the dividend, divisor, quotient, and remainder; then multiply together the excesses in the divisor and quotient, and find the excess of 9’s or 11's of the product; then, to this excess, add the excess of the remainder and find the excess of 9's or 11's in this sum; if the excess is the same as the excess in the dividend, the work is correct, as far as this method of proof can determine. EXCESS OF ELEVENS, OR THE CHECK OR KEY FIGURE. 278. The excess of 11's in a number may be found in various ways. We here indicate three of the methods: First method. By dividing the number by 11. Second method. By casting the 11’s out of the sum of the figures in the odd places, then casting the 11's out of the sum of the figures in the even places, and 7 : PROOF BY CASTING OUT THE 9's OR II's. I55 subtracting the excess of 11's in the even places from the excess of 11’s in the odd places. In case the first excess is smaller than the second, increase it by 11 and then subtract. Third method. From the sum of the figures in the odd places, subtract the sum of the figures in the even places. In case the first sum is less than the second, add 11 or some multiple of 11 to the first sum and then subtract; or when the first sum is less than the second, subtract 11 or some multiple of 11 from the second suin, and then take the remainder from the first sum. t TEIE CEHECK OR KEY FIGURE SYSTEM. 279. The excess of 11’s is used largely by accountants in locating errors, proving transfers, etc., and in books of accounts as above stated, is known as the 11 Check or Key Figure System. EXERCISES IN FINDING THE CEIECK OR REY FIGURE. 280. Find the check or key figures of the following numbers: 1. 64752. Sum of figures in odd places, 15. Sum of figures in even places, 9. 9 from 15 = 6, check figure. NOTE.-See three methods above for finding check figure. 2. 134091. Sum of figures in odd places, 4. Sum of figures in even places, 14. 14 from (4 + 11) = 1, check figure. Or, (14 – 11) = 3 from 4 = 1 check figure. 3. 47. Mentally think 4 from 7, 3, check figure. 4. 52. Mentally think 5 from 13, 8, check figure. 5. 368. Mentally think 6 from 11, 5, check figure. 6. 193. Mentally think 9 from 15, 6, check figure. 7. 507. Mentally think 0 from 12, 1 from 2, 1, check figure, or mentally think 11 from 12, 1, check figure. 8. 20908. Mentally think 0 from 19, 1 from 9, 8, check figure, or mentally think 11 from 19, 8, check figure. 9. 50. Mentally think 5 from 11, 6, check figure. 10. 7060. Mentally think 13 from 22, 9, check figure, or mentally think 2 from 11, 9, check figure. 11. 34567. Mentally think 7, 12, 15, and 6, 10, 5, check figure. 12. $ 7428.95 Mentally think 5, 13, 17, and 9, 11, 18, 7 from 17, 10, check figure. 13. $20000.00 Mentally think 0, 0, 0, 2, 0, 0, 0, 2, check figure, or mentally think 0 from 2, 2, check figure. 14. $52978.10 Mentally think 0, 8, 17, 22; and 1, 8, 10, from 22, 12; 1 from 2, 1, check figure, or mentally think 0, 8, 17, 22 and 1, 8, 10, from 11, 1 check figure. 15. $ 4872.27 16. 25607.14 17. 134560.05 18. 289176.40 19. 10806.70 20. 100000.00 21. 100708,05 22. 50000.20 23. 899887,69 I56 soul E's PHILOSOPHIC PRACTICAL MATHEMATICS. º: APPLICATION OF THE ELEVEN CHECK FIGURE METHOD OF LOCATING ERRORS AND OF PROVING TRANSFERS AND POSTING.S. 281. Post or transfer the following sales book items: LEDGEER ACCOUNTS. SALES BOOK. JONES. & SMITH. tº C ºffſ - * - - * - - ##|g 7| || 487|59 8| ||16541 § 3. ' Jones bot. Mdse. 7 487.59 Smith ** ** 6|| ||16541120 BROWN. WOOD. Brown. “ “ 7| || 90.7815 — - ,- - Wood “ “ 10| ||10862|05 º 9087|51 | 0|| ||10862 d | Ea:planation.—Before posting the item of Jones' purchase, find and write the check figure to the left of the folio column in the Sales Book, or whatever book is used as a posting medium. Then when the item has been posted, find and write the check figure of the posted item to the left of the folio column in the Ledger, or whatever book the item was transferred to. If the check fig- ures are the same, the posting is correct. If they are not the same, there is an error, and it has been located in this posting. In the first posting above, the check figures are the same (7’s) and the work is correct. In the second item (Smith's purchase) the check figure in the Sales Book is 6, and in the Led- ger account it is 8. This shows an error and locates it in this posting. In the third item (Brown’s purchase) the check figure in the Sales Book is 7, and in the Led- ger account it is 8, hence an error in this posting. * The check figure in the Sales Book of the fourth item is 10, while it is 0 in the Ledger posting. This difference in the check figures shows an error, and locates it in the item posted. In the above manner, all postings and transfers may be proved. For further work on the application of the properties of 9 and 11, to the detection of errors in trial balances, transfers, postings, etc., see Soulé's New Science and Practice of Accounts. --> | | f O actoring. 282. Factoring consists in separating or resolving a composite number into its factors. The operation is performed by division. NotE.-Before entering upon this work of factoring, and the two subjects following, the learner should thoroughly understand what are prime and composite numbers, factors and multiples of num- bers, and the divisibility of numbers as shown on pages 146 to 150. 283. OPERATION. 2 5460 2|2730 1365 5 3| 273 7 91 13 WRITTEN EXERCISES. What are the prime factors, or divisors, of 5460? Ea:planation.—In all problems of this kind, we first divide the given number by any prime factor, and the successive quotients by prime factors, or divisors, until we obtain a quotient that is a prime number. In this problem, our last quotient is 13, which not being divisible, is a prime number. Hence the divisors 2, 2, 5, 3, 7, and the quotient 13, are all prime factors, or divisors, of 5460. PROBLEMS. 284. Find the prime factors of the following numbers: 1. 84 2. 376 3. 864 4. 6105 7. 25600 10, 32460 5, 1683 8. 10376 11. 13532 6. 3560 9. 71460 12, 96033 285. What are the common prime factors of 28, 64, and 72% OPERATION. 2 ) 28 64. 72 2 || 14 32 36 7 16 18 Explanation.—In all problems of this kind, we divide the given numbers by any common prime factor of all the numbers, and the quotients thus obtained are divided in the same manner, till they have no common factor or divisor. The several divisors will be the common prime factors of the numbers. NOTE.—A number that is a factor, or divisor, of two or more numbers is called a common factor of these numbers. PROBLEMS, 286. Find the prime factors common to the following numbers: 1. 18, 24, and 36 4. 2. 54, 72, and 84 3. 506, 436, and 308 44 and 280 7. 5. 148, 256, and 320 6. 325, 635, and 550 28 and 64 8. 112 and 250 9. 526, 400, and 780 (157) I 58 SOULE's PHILOSOPHIC PRACTICAL MATHEMATICs. X- $º. GREATEST COMMON DIVISOR. 287. For the definition of a divisor, a common divisor, and the greatest common divisor, see page 147. G. C. D. is the abbreviation for the greatest common divisor. 1. What is the greatest common divisor of 42, 56, and 210 OPERATION. 2 |42, 56, 210 Explanation.—In all problems of this kind, we first divide by any prime factor that will divide all the numbers; then we divide in 7| 21, 28, 105 like manner the successive quotients thus obtained, until we obtain | *E===mEºmºmºmºmº quotients that have no common factor, or are prime to each other; 3 4 15 then we multiply all the divisors together, and in the product we have the greatest common divisor. 2× 7 =14 Ans. NOTE 1.--When there is no number greater than 1, that will divide all the numbers without a remainder, then 1 is the greatest common divisor. NOTE: 2.—When there are two large numbers, the operation may be more easily performed by first dividing the larger number by the smaller, and if there is a remainder divide the preceding divisor by it, and thus continue until there is no remainder. NOTE: 3.—When there are more than two numbers, proceed as with two, and then, with the atest common divisor of the two and one of the other numbers, and thus continue until through with all the numbers. The last divisor will be the greatest common divisor. Problems 2 and 3 elucidate this operation: 2. What is the greatest 3. What is the greatest common divisor of 88 and common divisor of 195, 24 % Ans. 8. 285, and 315? Ans. 15. OPERATION. OPERATION. 24)88(3 285)315(1 15)195(13 72 285 15 1624(1 30)285(9 45 16 270 45 8)16(2 & 15)30(2 16 30 f GENERAL DIRECTIONS FOR FINDING THE GREATEST COMMON DIVISOR. 288. From the foregoing elucidations, we derive the following general direc- tions for finding the G. C. D. 1. Write the numbers on a horizontal line and divide by any prime number that will divide all without a remainder, and write the quotients in a line below. 2. Continue this process of dividing the successive quotients, until quotients are obtained which have no common factor, or divisor. FACTORING, I59. 3. Multiply together all the divisors, and their product will be the G. C. D. NOTE.-When there are two or more large numbers, it is often more convenient to work by successive divisions, as elucidated in problems 2 and 3. PROBLEMIS. 289. What is the greatest common divisor of the following numbers? 4. Of 441 and 567 ? Ans. 63. 5. Of 90, 315, and 810? Ans. 45. 6. Of 654, 216, and 108? Ans. 6. 7. Robinson has 25, and Blaise, 45 dimes. How shall they arrange them in packages, so that each will have the same number in each package 3 Ans. 5 in each package. 8. A planter has 697 bushels of corn and 204 bushels of rough rice, which he wishes to put into the least number of bins containing the same number of bushels, without mixing the two kinds. How many bushels must each bin hold? Ans. 17 bushels. 9. A commission merchant has 2490 bushels of wheat, 1886 bushels of corn, and 8438 bushels of oats, which he wishes to ship in the least number of sacks of equal size, that will exactly hold either kind of grain. How many sacks will he require? Ans. 6407. OPERATION INDICATED. Find the greatest common divisor as above, (it is 2). 2)2490 1886 8438 1245-H 943-H4219–6407 Ans. LEAST COMMON MULTIPLE. 290. For the definition of a multiple, a common multiple, and the least common multiple, see page 147. L. C. M. is the abbreviation for the least common multiple. 1. What is the least common multiple of 5, 6, 8, 21, 28? OPERATION. 2) 5, 6, 8, 21, 28 Explanation.—In all problems of this kind, we first arrange the numbers on a horizontal line, and then divide by 2) 5 3 4 21 14 the smallest prime number that will divide two or more 3) 5. 3 2 21 7 without a remainder, and write the quotients and undivided 4./ numbers in a line below; this process of dividing we cont 7) 5 1 2 7 7 tinue until there are no two numbers that can be divided by the same number without a remainder; then we multiply the divisors and the numbers in the last line together, and 2 × 2 × 3 × 7 × 5 × 2–840 Ans. the product is the least common multiple. 5 1 2 1 1 NoTE.—When there is any number of the problem that will divide any of the others without a remainder, it may be cancelled before commencing to divide. I6O SOULE's PHILOSOPHIC PRACTICAL MATHEMATICS. GENERAL DIRECTIONS FOR FINDING THE LEAST COMMON MULTIPLE. 291. From the foregoing elucidations, we derive the following general direc- tions for finding the L. C. M. 1. Write the numbers in a line and then divide by the smallest prime number that will divide two or more without a remainder, and write the quotients and undi- wided numbers in a line below. 2. Continue this process of dividing, until there are no two numbers, with common factors, or that can be divided by any number greater than 1. 3. Find the continued product of the divisors and the numbers in the last line, and it will be the L. C. M. g PROBLEMIS. 2. What is the least common multiple of 4, 9, 12, 15, and 24? Ans. 360. What is the least common multiple of the following numbers: 3. Of 8, 4, 9, and 30? Ans. 360. 4. Of 50, 27, 3, 45, and 63? Ans. 9450. 5. Of 21, 36, 11, and 22% Ans. 2772. 6. Of 800, 600, 10, 40, and 12% Ans. 2400. 7. Of 8, 18, 20, and 70 ? Ans. 2520. 8. A drayman has 2 drays and 2 floats. On 1 dray he can haul 9 barrels of flour, and on the other 12 barrels; on 1 float he can haul 18 barrels, and on the other 21 barrels. What is the least number of barrels that will make full loads for either of the drays or the floats? Ans. 252. 9. A fruit dealer desires to invest an equal amount of money in oranges, peaches, and grapes, and to expend as small a sum as possible. The price of oranges is $2.40 per box; peaches, $1.60; and grapes for a medium article, 90g., and for first quality, $1.20; of these two qualities the fruit dealer took the cheaper. How much more money did he invest than he would have done had he taken the grapes at $1.20 per box 3 Ans. $28,80. PARTIAL OPERATION. L. C. M. of $2.40, $1.60, .90 = $14.40. $14.40. × 3 (kinds of fruit) = $43.20 spent by purchasing grapes a 90g. L. C. M. of $2.40, $1.60, $1.20 = $4.80. $4.80. × 3 = $14.40, what he would have spent by taking grapes (a) $1.20. $43.20–$14.40 = $28.80, Ans. MSYNOPSIS FOR RE VIE W OF FA CTORING! NOTE.-The numbers before the words and phrases, refer to the articles treating the subjects named. 292. Define the following words and phrases: 282. What is Factoring 3 285. A Common Factor. 288. General Directions for Finding Greatest Common Divisor. 291. General Directions for Finding Least Common Multiple. 293. Cancellation is the process of shortening the operations of division, or of the indicated result of multiplication and division operations combined, by rejecting equal factors from both dividend and divisor, or from both increasing and decreasing numbers, of an indisated result. The operation is performed by drawing a line across each factor cancelled, or cut out; thus, 3, 7, 25. 294. The Principles of Cancellation, are : º 1. Rejecting, or cancelling a factor from any number, is in effect dividing the number by that factor. 2. Rejecting, or cancelling equal factors from both dividend and divisor, or from both increasing and decreasing numbers in an indicated result, does not change the quotient or result. NoTE.—A good understanding of factors, multiples, and the divisibility of numbers, will greatly facilitate the student in the operations of cancellation. STATEMENT LINE. 295. A statement line, as used in this work, consists of a vertical line upon whose right and left sides are placed respectively the increasing and decreasing numbers of the problem. This statement is made to facilitate the operation, as will be shown in the problems following. In the solution of all practical problems, when writing the numbers on the statement line, whether increasing or decreasing, a reason must be mentally given for each number written, as is elucidated in the 12th Problem of Article 297, follow- ing. See pages 77 to 81, and 125 to 128, for a full elucidation of the reasoning given in the solution of problems, and of the philosophic system presented in this work. PROBLEMIS. Divide 7 × 3 × 4 by 7 × 4. Operation by Cancellation. Explanation.—In all problems where we have both mul: tiplication and division operations to perform, we use a vertical ë or perpendicular fine, which we call the statement line. This 43 line is used to facilitate the work by separating the dividends 4. and the divisors, or the increasing and the decreasing numbers. The dividends, or the increasing numbers, are always placed * upon the right hand side of the line, and the divisors or 3, Alls. decreasing numbers, are always placed upon the left hand side. In this example, having written the numbers that constitute the dividend and the divisor, respectively upon the right and left hand sides of the statement line, we cut out, or cancel, the equal factors 7's and 4’s in the numbers constituting the dividend and the divisor, and thus obtain 3 as the answer to the problem. To perform the work without the aid of cancellation, we would be obliged to make the following figures: 7 × 3 = 21, which x 4 = 84, the dividend; then 7 × 4 = 28, the divisor; then 28)84(3, Ans. 84 amº (161) I62 soul E's PHILOSOPHIC PRACTICAL MATHEMATICS. x- 2. Multiply 25, 48, and 88 together, and divide the product by the product of 10, 36, and 8. Operation by Cancellation. Explanation.—In this example, we write the numbers 3 1% 25 5 2 on the line as above directed and then cancel 10 and 25 by º ; # - 5; then 36 and 48 by 12; then 8 and 88 by 8; then 4 and 2 by 2. This is all that can be cancelled, and we then multiply 110 together 5, 2, and 11, divide the product by 3, and thus 36; A. obtain the true result, 36%. Il S. 7 3. Divide the product of 32 × 3 by 8 × 9 × 16. Operation by Cancellation. Explanation.—Having written the numbers on the § |32 4 statement line, we first cancel 8 and 32 by 8; then 9 and 3 3 g 3 by 3; then 16 and 4 by 4. Now having no more numbers on 4 I6 the increasing side of the line to cancel, we multiply - together the remaining numbers on the decreasing side of 12| 1 = T's, Ans. the line and thus produce the correct result, Tºg. NOTE.—In all cases where, after cancelling, no factor appears on either side of the statement line, the factor 1 is always understood as being there. Its non-appearance is in consequence of not having written it when we cancelled a number by itself, as in the following problem. 4. Multiply 14, 5, and 3 together, and divide the product by the product of 2, 15, and 7. OPERATION. 2 | 1.4 7 1 2 || 14 7 1 Ea:planation.—In this problem, we first 3 15 # 13 Iš # 1 cancelled 2 and 14; then 5 and 15; then the 3 1. 1. and 3; and lastly the 7 and 7. The work might 7 | 3 OI’ 7 || 3 have been abridged by multiplying the 3 and 5 and then cancelling both 15's, and then multi- 1. Ans. 1. 1 = 1, Ans. plying the 7 and 2 and cancelling both 14's. 7 GENERAL DIRECTIONS FOR, CANCELLATION. 296. From the foregoing elucidations, we derive the following general direc- tions for cancellation : 1. Cancel all the factors common to both dividend and divisor, or to the increas- ing and decreasing numbers. *- 2. Divide the product of the remaining factors of the dividend by the product of the remaining factors of the divisor. NOTE.--When a factor cancelled is equal to the number itself, the unit 1 remains, since a number divided by itself gives 1 as a quotient. If the 1 is in the dividend it must be retained ; if in the divisor, it may be disregarded, since dividing by 1 does not change the quotient. 297. Cancel and work the following line statements: (1) (2) (3) (4) (5) 2 || 6 12 || 45 31 || 124 9 || 76 10 | 16 3 || 9 9 || 56 5 || 17 20 | 91 135 | 45 18 || 54 7 | 84 51 || 10 70 || 140 24 || 60 27 16 || 4 4 25 *- 1, Ans. 70, Ans. #, Ans. 1921%, Ans. 14, Ans. jºr CANCELLATION. I63 6. Divide the product of 6, 7, 12, and 22 by the product of 11, 3, 14, and 8. Ans. 3. 7. What is the quotient of 28 × 65 × 7 × 78 + 56 x 130 x 42 × 13% Ans. # 8. Multiply 21, 55, and 128 together, and divide the product by 14 x 25 x 64. Ans. 63. 9. How many bushels of corn, at 70% a bushel, will pay for 140 gallons molasses at 65 cents a gallon ? Ans. 130 bushels. 10. Bought 420 pounds of sugar at 6 cents a pound, and gave in payment 360 pounds of rice. What was the price of the rice per pound 7 Ans. 7 cents. 11. Sold to a drayman 64 bushels of oats at 75 cents a bushel, for which he is to pay in drayage at 50 cents a load. How many loads must he haul ? Ans. 96 loads. 12. Paid 65% for 5 yards of calico. What will 27 yards cost at the same rate? Analytic Solution by Explanatºon.—In all practical problems of this kind, Cancellation we give a reason for each step of the operation, and make e the whole statement to indicate the final result without §§ 13 performing any of the intermediate work. In this problem, #|27 we place the 65c. on the increasing side of the statement line as our premise, and reason thus: 5 yards cost 65c. Since 5 e yards cost 65c., 1 yard will cost # part of it, and 27 yards will $3.51, Ans. cost 27 times as much as 1 yard. * j 13. A merchant sold 25 boxes of candles containing 36 pounds each at 162 per pound, and received in payment starch at 6 cents per pound. How many boxes, each containing 30 pounds, did he receive? Ans. 80 boxes. 14. How many pounds of butter at 35% per pound, will pay for 245 pounds of rice at 5 cents per pound 3 Ans. 35 pounds. 15. If 17 barrels of flour cost $110.50, what will 500 barrels cost at the same rate? Ans. $3250. 16. Sold 340 boxes of soap, containing 45 pounds each, at 122 per pound, and received in payment starch at 9% per pound. How many boxes, each contain- ing 24 pounds, did I receive 3 Ans. 850 boxes. a - -aº a w ur-w SYNOPSIS FOR RET/IE W OF CANCELLATION. NoTE.—The numbers before the words and phrases, refer to the articles treating the subjects named. 298. Define the following words and phrases? 293. Cancellation. 294. Principles of Cancellation. 295. Statement Line. 296. General Directions for the Operation. E. ſo N O fractions. 36 S. 3: tº s º ºs e - e º º ºs e º e º e º e Eºcºcº =N —-tº-4(O)-4— 299. According to the general methods prescribed by the text books of Our country, for the various operations of fractions, the subject is correctly considered by both the teacher and the pupil to be the most difficult in the science of numbers. But by our fully evolved, logical, and philosophic system of handling fractional numbers, the subject is simplified, rationalized, and rendered pleasing to the student. By our method of work, fully one-half of the time and figures required by the ordi- nary methods is saved, and all the arbitrary and absurd rules which overload the organ of memory and prevent the expansion of the higher faculties of causality and comparison, are abandoned to the shades of the dead past, and entombed with the ingenious minds which gave them birth. By our system, all the reasoning faculties of the mind are brought into action and exercised in a manner to give logical strength and acuteness to work, not only in the fields of mathematics, but upon all the plains of life. In behalf of truth, education, and humanity, we lament the non-progressive methods of the general school and college arithmetics. The present authors of school arithmetics, with but few exceptions, are timidly following in the obscure paths of arithmetical Science which were marked out ages ago, when the science was in its infancy—too cowardly, non-progressive, and con- tracted in their views to seek and explore more direct and comprehensible routes to the fountains of mathematical knowledge. 300. The Unit is the universal basis of numbers, and the foundation of arith- metic. From unity arise two distinct classes of numbers. 1. Integers. 2. Frac- tions. The first class, integers, has its origin in the multiplication of the unit; and the second class, fractions, results from the division of the unit. The first is synthetical, the second is analytical. I) EFINITIONS. 301. A Fraction is one or more of the equal parts of a unit, or of a collection of units. Or more briefly, it is a part of anything, or a numerical expression of a part of a unit; thus, 1 half and 3 fourths are fractions. 302. A Fractional Unit is one of the equal parts into which any integral unit is divided. If the integral unit is divided into two equal parts, each is called a half; if into three, each is called a third ; if into four, each is called a fourth ; and so on, according to the number of parts into which the integral unit is divided. 303. Fractions are divided into two kinds, Common, or Wulgar, and Decimal Fractions. (164) Y FRACTIONS. I65 304. Common Fractions are expressed by two numbers, one written above the other, with a horizontal line between them. The number below the line is called the Denominator, and the number above the line is called the Numerator. Thus, # (one-half), # (three-fourths), # (five-sixths), (seven-eighths), and +} (thirteen-seven- teenths), are common fractions, the denominators of which are respectively 2, 4, 6, 8, and 17. The numerator and denominator together, are called the terms of the fraction. 305. The Denominator of a fraction shows the number of equal parts into which the unit is divided. Thus in the fraction #, the 8 is the denominator and shows that the unit is divided into 8 equal parts called eighths. 306. The Numerator of a fraction shows the number of equal parts taken to form the fraction. Thus in §, the numerator is 5 and shows that 5 of the 8 equal parts are taken, or expressed, by the fraction. All fractions arise from division and are expressions of unexecuted division in which the numerator is the dividend, the denominator the divisor, and the fraction itself the quotient. 307. Decimal Fractions are those in which the denominators are not generally expressed, but are always 10, or some power of ten; thus, .5, .75, .821, read respec- tively five tenths, seventy-five hundredths and eight hundred twenty-one thousandths, are decimal fractions. To write these fractions as common fractions, they would be written thus, #3, #3, and #5. The point (...) placed before the 5, 7, and 8, in the above decimally expressed fractions, is called the decimal point, and is used to abbreviate the work. CLASSIFICATION OF FERACTIONS. 308. For convenience, fractions are classed under the following heads: Proper Fractions, Improper Fractions, Simple Fractions, Mia!ed Numbers, Compound Frac- tions, and Complea, Fractions. 309. A Proper Fraction is one in which the numerator is less than the denominator; as 3, #, g. * 310. An Improper Fraction is one in which the numerator is equal to or greater than the denominator; as, #, 3, #, and *. 311. A Simple Fraction is one in which both terms are whole numbers, and may be either a proper or an improper fraction; as, #, #, ++, or *. 312. A Mixed Number is a number composed of a whole number and a fraction; as, 24, 5}, and 21+%. 313. A Compound Fraction is a fractional part of a fraction or mixed number; as, # of 3, 4 of +3 of 12#. I 66 SOULE's PHILOSOPHIC PRACTICAL MATHEMATICs. X} 314. A Complex Fraction is one that has one or more of its terms fractional; aS, # 6# 3 # 6# – of —, — of — of — # 5% # 1#. 8 315. The Reciprocal of a Fraction is the result of 1 divided by the fraction. Thus, the reciprocal of # is 1 -- # = # = 14. 316. The Walue of a Fraction is the result of its numerator divided by its denominator. Thus, # = 4, * = 23. GENERAL PRINCIPLES OF FERACTIONS. 317. 1. Multiplying the numerator, or dividing the denominator, multiplies the fraction. 2. Dividing the numerator, or multiplying the denominator, divides the fraction. 3. Multiplying or dividing both numerator and denominator by the same number does not change the value of the fraction. SYNOPSIS FOR RE VIEW OF FIRACTIONS. ºm-E-3 NOTE.-The numbers before the words and phrases, refer to the articles treating the subjects named. 318. Define the following words and phrases: 301. A Fraction. 310. An Improper Fraction, 302. A Fractional Unit. * 311. A Simple Fraction. 303. How Fractions are Divided. 312. A Mixed Number. 304. Common Fractions. 313. A Compound Fraction. 305. Denominator. 314. A Complex Fraction. 306. Numerator. 315. The Reciprocal of a Fraction. 307. Decimal Fractions. 316. Value of a Fraction. 308. Classification of Fractions. 317. General Principles of Fractions. 309. A Proper Fraction. iteduction of Fractions. (3) º *H*=N 319. Reduction of Fractions is the process of changing their form without altering their value. 320. A fraction is reduced to Higher Terms when the numerator and the denominator are expressed in larger numbers. Thus, # = }, or $, or ; ; # = #, or -Ér, Or ##, etc. † wº- 321. A fraction is reduced to Lower Terms when the numerator and the denominator are expressed in Smaller numbers. Thus, # = }, or 3; # = }, or #. 322. A fraction is reduced to its Lowest Terms when its numerator and its denominator are prime to each other, or have no common divisor, Thus, #, #, and *#, are in their lowest terms. 323. Whole Numbers may be reduced to fractions having any desired denom- inator. Whole line. Half lines. Third lines. Fourth lines. ORAL EXERCISES. 324. 1. If a line, an Orange, an apple, or a unit of any kind is divided into two equal parts, what is each part called? Ans. #. 2. If divided into three equal parts, what is each part called ? Ans. #. 3. If divided into four equal parts, what is each part called? Ans. 4. - 4. When divided into four equal parts, what are three of those parts called ? Ans. #. 5. How would you get # of an apple? Ans. Divide it into 4 equal parts and take 3 of the parts. 6. When any number or thing is divided into five equal parts, what is one of those parts called? AnS. #. 7. What are two, three, and four of the parts called respectively? Ans. #, #, and #. 8. 1 unit, abstract, or denominate of any kind, equals how many halves? thirds? fourths? fifths? sixths? sevenths? eighths? ninths? (167) I 68 soul E's PHILOSOPHIC PRACTICAL MATHEMATICS. X- ge 9. 2 = how many halves? thirds? fourths? fifths? sixths? sevenths? eighths? *. 3 = how many halves? thirds? fourths? fifths? sixths? sevenths? eighths? *. 4 = how many halves? thirds? fourths? fifths? sixths? sevenths? eighths? ninths, 5 = how many halves? thirds? fourths? fifths? sixths? sevenths? eighths? 13. 6 = how many halves? thirds? fourths? fifths? sixths? sevenths? eighths g What kind of numerical work is the above called ” 325. 4 = how many fourths? sixths? eighths? tenths? twelfths? fourteenths? # = how many sixths? ninths? twelfths? fifteenths? eighteenths? twenty-firsts? # = how many eighths? twelfths? sixteenths? twentieths? twenty-fourths? # = how many tenths? fifteenths? twentieths? twenty-fifths? thirtieths? # = how many sixteenths? twenty-fourths? thirty-seconds? fortieths? sixty- fourths # & - + = how many thirty-seconds? forty-eighths? sixty-fourths? eightieths? What kind of numerical work is the above called 3 326. Answer the following numerical questions: # = | #iº | #E} | #: | # =}| #: # = } | . . ; ; # =; #F# # =; # = } 12 – s 2 4 - 4 8 - - 6 4 - 9 6 - 7 5 O – *i; } :=}| #E} | #E} | . E! | #: What kind of numerical work is the above called ? 327. How many halves = a unit? How many eighths = a unit # {{ “ thirds = {{ 44 “ ninths = {{ 64 “ fourths = “ 4% “ tenths = ** 44 “ fifthS = “ ‘‘ ‘‘ elevenths = % {{ “ SixthS = “ 6% “ twelfthS = {{ {{ “ Sevenths = “ 64 “ thirteenths = “ 1 unit equals how many eighths? § = ? 13 units equal how many thirds? # =? 1. {{ & 4 {{ twelfths? 43 = ? 13 “ 64 {{ sixths? *=? 3 units equal “ thirds? § = ? 13 “ 4% 44 eighths 3 4.g4–? 4. {{ {{ 4. fourths? 14 = ? 3+ “ {{ é- fourths # 14–? 5 {{ {{ {{ halves? 19 =? 2; “ { % {{ sixteenths?#=? 14 “ 44 & 4 halves? # =? 43 “ & 4 & 4 eighths? §§ – ? 14 “ {{ {{ fourths 7 # =? 328. What is the reciprocal of 1, of 2, of 3, of 4, of #, of 23 329. Analyse the fraction #. Analysis.—# is a proper fraction, since the numerator is less than the denominator; 4 is the denominator, and shows that the unit is divided into 4 equal parts; + is the fractional winit, since it is ONE of the four equal parts into which the unit is divided; 3 is the numerator and shows that three of these equal parts are taken; 3 and 4 are the terms of the fraction, and its value is less than 1, or unity. * - REDUCTION OF FRACTIONS. I 69 TO REDUCE FRACTIONS TO HIGHER TERMS. 330. 1. Change # to a fraction whose denominator is 64. OPERATION. 64 - 16 = 4 Explanation.—In all problems of this kind, we first divide the required denominator by the denominator of the given 13 × 4 = 52 fraction. Then with the quotient thus obtained, multiply both º- — Ans. terms of the given fraction, and in their products we have the 16 × 4 = 64 required fraction. GENERAL DIRECTION FOR REDUCING FRACTIONS FROM LOWER TO HIGHER TERMS. 331. From the foregoing elucidations, we derive the following general direction for reducing fractions from lower to higher terms: Divide the required denominator by the denominator of the given fraction, and multiply both terms of the fraction by the quotient. PROBLEMIS. 332. Change ++ to a fraction whose denominator is 75 {{ # to {{ {{ {{ 176 4% # to & 4 4% 44 384 {{ # to & 4 4. 4é 2180 TO REDUCE FRACTIONS TO THEIR LOWEST TERMS. 333. Reduce # to its lowest terms. FIRST OPERATION. Ea:planation.—In all problems of this kind, we divide 2)# = ##; 4) # = g Ans. both the numerator and the denominator by their common factors. Or, as shown in the second operation, we may SECOND OPERATION. produce the same result with less figures, by dividing 8)# F § Ans. both terms of the fraction by their greatest common divisor. By this reduction we change the form of the fraction ##, but we do not alter, or change, its value, for the fractional unit of the resulting fraction (g) is 8 times as great, while the number taken is ; as great. When the terms of the fraction have no common factor greater than 1, the fraction is in its lowest terms and is called an irreducible fraction. The object of reducing fractions to their lowest terms is to enable us to understand their value more easily and readily. - I7o SouLE's PHILOSOPHIC PRACTICAL MATHEMATICS. GENERAL DIRECTION FOR REDUCING FRACTIONS TO THEIR LOWEST TERMS. 334. From the foregoing elucidations, we derive the following general direction for reducing fractions to their lowest terms: Cancel all the factors common to both the numerator and the denominator; Or, divide both numerator and denominator by their greatest common divisor. PROBLEMIS. 335. Reduce the following fractions to their lowest terms. 2. #4, ##, ##, ##, Ans. #, #, #, #. 7. Hººs, Ans. #. 3. ###, #3, ###, Ans. #3, #, #r. 8. #####, Ans. ####. 4. ####, AnS. #. 9. #####, AnS. #. 5. Hº, Ans. ###. 10. ####, Ans. #. 6. ###, Ans. #. 11. §§§, Ans. ##. TO REDUCE WHOLE OR, MIXED NUMBERS TO IMPROPER FRACTIONS. 336. 1. Reduce 53 to an improper fractions, or to thirds. OPERATION. 5; Explanation.—In all problems of this kind, we reason tº- thus: Since there are 3 thirds in every unit or whole 14. A number, in 5 units there are 5 times as many, which are *ā- Ans. * + the # make *. 2. Reduce 9 to a fraction whose denominator is 6. OPERATION. 9 × 6 = ** Ans. GENERAL DIRECTION TO REDUCE WHOLE OR MIXED NUMBERS TO IMPROPER FRACTIONS. 337. From the foregoing elucidations, we derive the following general direction for reducing whole or mixed numbers to improper fractions: Multiply the whole number by the required denominator and to the product add the numerator of the fraction, and write the required denominator under the result. * REDUCTION OF FRACTIONS. 171 PROBLEMS. Reduce the following numerical expressions to improper fractions: 3. 84 Ans. *. 8. 714 Ans. 4+*. 4. 16# Ans. *. 9. 68% Ans. 44%. 5. 17; Ans. 4. 10. 21833 Ans. ***. 6. 323 Ans. *;1. 11. 23# Ans. ***. 7. 435% Ans. **iš. 12. 108Tºg Ans. 1-#4. 13. Reduce 14 to a fraction whose denominator is 9. 14. {{ 37 44 {{ 44 4% 24. 14. {{ 54; “ {{ & 4 {{ 16. TO REDUCE IMPROPER FRACTIONS TO WHOLE OR MIXED NUMBERS. 338. Reduce 41 to a mixed number. OPERATION. Ea:planation.—In all problems of this kind, we reason thus: + = 44 Ans. Since there are 4 fourths in 1 unit, or whole number, in 17 fourtha OI’ there are as many units as 17 is equal to 4, which is 4 times with 1 17 -- 4 = 44 Ans. remainder, or altogether 4+ as the proper quotient, or answer. *. GENERAL DIRECTIONS TO REDUCE IMPROPER FRACTIONS TO WEIOLE OR MIXED NUMBERS. 339. From the foregoing elucidations, we derive the following general direction for reducing improper fractions to whole or mixed numbers : Divide the Numerator by the IDenominator. PROBLEMS. 340. Reduce the following improper fractions to whole or mixed numbers: 2. ** Ans. 4. 6. * Ans. 33. 3. * * Ans. 53. 7. # Ans. 5+. 4. 1 #4 Ans. 48. 8. 4.4 Ans. 9+. 5. 2.23. Ans. 18%. 9. *###9. Ans. 34&#. l 2 172 soul.E's PHILOSOPHIC PRACTICAL MATHEMATICs. 4× TO REDUCE COMPOUND FERACTIONS TO SIMPLE FRACTIONS. 341. 1. Reduce # of # of £ to a simple fraction. OPERATION. Explanation.—In all problems of this kind, we multiply together 3 % = }}. A all the numerators for a new numerator and all the denominators # × # × # = #& Ans. for a new denominator. When a compound fraction contains whole or mixed numbers, they must first be reduced to improper fractions. When there are common factors in both terms of a compound fraction, they should be can- celled before multiplying. By cancelling the common factors, the work is shortened and the result * for the reason that dividing both terms of a fraction by the same Inumber does not alter its value. 2. Reduce # of 73 of # of 4 of +43 to a simple fraction. OPERATION. 3 3 I; 2 A 1 1 – × — × − x − x – = – Ans § 2 g 1 12 3 3 3 GENERAL DIRECTIONS TO REDUCE COMPOUND FRACTIONS TO SIMPLE FRACTIONS. 342. From the foregoing elucidations, we derive the following general direction for reducing compound fractions to simple fractions: Cancel common factors if they occur in both terms of the fractions ; then multi- ply the numerators together for the new numerator and the denominators together for the new denominator of the fraction. PROBLEMS. 343. Reduce the following compound fractions to simple ones: 1. Reduce # of ; of § to a simple fraction. Ans. Hºg. 2. Reduce # of 74 of # of 4 of Hº to a simple fraction. Ans. #. 3. Reduce # of 24 of # of # of 13 to a simple fraction. Ans. }}. 4. # of +% of Hºr. Ans. #. | 8. ºr of 96. Ans. 54. 5. # of § of #3. Ans. #s. 9. § of +3 of 17*. Ans. 6. 6. # of 3% of 4. Ans. 14. 10. § x #.x #. Ans. #. 7. # of 84. Ans. 24%. 11. 5 × 2 × #. Ans. ##. TO REDUCE FRACTIONS OF DIFFERENT DENOMINATORS TO EQUIV- ALENT FRACTIONS OF A COMMON DENOMINATOR, OR OF TEIE LEAST COMMON DENOMINATOR. 344. A Common Denominator is a denominator common to two or more fractions. 345. The Least Common Denominator of two or more fractions is the least number divisible by each of the denominators. * REDUCTION OF FRACTIONS, I73 846. A Common Denominator of two or more fractions is a common multiple of their denominators; and the least common denominator of two or more fractions is the least common multiple of their denominators, for the reason that all higher terms of a fraction are multiples of its corresponding lower or lowest terms. 347. Reduce #, #, and #, to equivalent fractions having a common denominator. OPERATION. i; #, #, #, 3 × 4 × 8 = 96, common denominator. # of = 32; hence ##, equivalent of #. # of £96=72; hence #, equivalent of #. § of = 84; hence ##, equivalent of £, Explanation.-In all problems of this kind, we obtain the common denominator by multiplying together the denominators of all the fractions. Then to find the respective numerators we take such a part of the common denominator as the respective fractions are parts of a unit, as shown in the operation. Or, divide the common denominator by the denominator of each fraction, and multiply the quotient by its numerator. GENERAL DIRECTIONS TO REDUCE FRACTIONS TO A COMMON DENOMINATOR. 348. From the foregoing elucidations, we derive, the following general direc- tions for reducing fractions to a common denominator: 1. Multiply together the denominators of all the fractions for a common denominator. 2. Then to find the respective numerators, take such a part of the common denominator as the respective fractions are parts of a unit. Or, divide the common denominator by the denominator of each fraction, and multiply the quotient by its 77.0/merator. PROBLEMIS. 349. Reduce the following fractions to equivalent fractions having a common denominator: 1. #, #, and #. Ans. #, #, and #. 2, #3, #, and #. Ans. ###, ###. and #}. 3. *, and #. Ans. ###, and ###. 4. #, #3, #5, ##, and #. Ans. #####, #5, #####, #####, and #. 5. Hº, 3, 4, and 3%. Ans. ###, #1, ###, and ####. I74 SOULE's PHILOSOPHIC PRACTICAL MATHEMATICs. X- 350. Reduce 3, #, and g to equivalent fractions having the least common denominator. OPERATION. 2 || 3. 4. 8 2 || 3. 2. 4 3. 1. 2 º 2 × 2 × 3 × 2 = 24, least common denominator. # of = 8, hence ºr is the eqtivalent of 3. # of $ 24 = 18, hence # is the equivalent of #. § of = 21, hence # is the equivalent of #. Eacplanation.—In all problems of this kind, we first find the least common multiple of the denominators of all the fractions as explained in Article 290, page 159, which is the least common denominator. Then, having the least common denominator, to find the respective numerators, we take such a part of the ieast common denominator as the respective fractions are parts of a unit, as shown in the operation. Or, divide the L. C. D. by the denominator of each fraction and multiply the quotient by its numerator. . Before finding the L. C. D., reduce mixed numbers to improper fractions, and the fractions to their lowest terms. GENERAL DIRECTIONS TO REDUCE FRACTIONS TO A LEAST COMMON DENOMINATOR. 351. From the foregoing elucidations, we derive the following general direc- tions for reducing fractions to the least common denominator. 1. Find the least common multiple of the denominators of all the given fractions. 2. Then to find the respective numerators, take such a part of the least common denominator as the respective fractions are parts of a unit. Or, divide the L. C. D. by the denominator of each fraction and multiply the quotient by its numerator. NOTE.-Mixed numbers must be reduced to improper fractions, and the fractions to their lowest terms before finding the least common denominator. PROBLEMIS. 352. Reduce the following fractions to equivalent fractions having a least common denominator. * 1. 3, #, and #. Ans. ##, #}, and #. 2. Hºs, +, and #. AnS. ##, ##, and #. 3. #, ##, and #. Ans. #6, #3, and #. 4. I'd ##, ##, and #. Ans. #5, #3, #3, and #. 5. 54, #, 14%, and #. Ans. ***, #3, #, and 4g. 6. ##, 8, #}, and 1. e Ans. ##, *, #3, and #. 7. 34, #, #, and #. Ans. ##, #3, #, and 49. 8. Fºr, 3, 4, and #. Ans. ####, ####, #3, and +}#}. e () () (e) (C) 2 GT c-ā-ā-ā-āj-,- Addition of Fractions. ===N *—dº ſº- -*ºr w -uu- 353. Addition of Fractions is the process of adding two or more fractional numbers of the same kind, or of the same denomination. 354. Like Fractions are those which express like parts of like units or things. Thus, # yard and # yard, also # and # are like fractions. * 355. Unlike Fractions are those which express unlike parts of like units or things, or parts of unlike units or things. Thus, # of a pound and # of a pound are unlike parts of like units or things, and # and # are unlike parts of unlike units. In addition of whole numbers, we learned that we could not add apples to oranges, pounds to boxes, or units to tens or hundreds ; that we could only add things that were of the same unit kind. This same principle maintains in the addition of fractional numbers. We cannot add halves to thirds, fourths to fifths, etc. We can only add halves to halves, fourths to fourths, etc. ORAL EXERCISES. 356. 1. Add +, +, and #. Ans. 14. SOLUTION.—Since the fractional parts are alike, we have only to add the numerators to obtain the sum of the fractions. Thus, # -- 3 + # = } = 1%, or 14, Ans. 2. Paid $4 for a grammar and $; for an arithmetic. What did both cost 7 Ans. $1}. SOLUTION.—Since the halves and fourths are unlike parts of the unit dollar, we cannot add them in their present form. We must first reduce the 4 to the fractional unit of the fourths, and by the exercise of our reason and knowledge of numbers, we see that 3 is equal to 3, and 3 added to # equals #, and # equals $1+, Ans. 3. What is the sum of ; and g { SOLUTION.—Since the fractional parts are unlike, we cannot add them in their present form. We must first reduce the # to the fractional unit of eighths; and by the exercise of our reason and knowledge of numbers, we see that # is equal to #, and # added to § equals º and *-equals 13, Ans. 4. What is the sum of #, g, ºr, and #2 Ans. 24;. 5. Add #, #3, ##, and ##, Ans. 2%. Mentally add the following fractions: # -- # = ? # -- # = ? +++3 = ? # + š ++* = ? # -- # = ? § -- # = ? # +# = ? # + 3 ++’s = ? # + # = ? # -- # = ? # + 4 + # = ? # + +º, +% = ? # + # = ? # + 4 = ? # + # ++’s = ? # + 4 ++} = ? # + # = ? # ++’s' = ? (175) 176 soul E's PHILOSOPHIC PRACTICAL MATHEMATICS. 357. What is the sum of 3 and #3 OPERATION. # # 8 9 17 – 1 -5 find the numerical fractional equivalent value of # and # in the unit of twelfths. reason thus: # = +; and # = nº ; (Write the 8 under the # as a memorandum). (write the 9 under the # as a memorandum). Tºg and g = +} = 1.1%, 2. What is the sum of ; and #% 3. What is the sum of # and + 3 4. Add {} and #; 358. Add 3, #, and #, OPERATION. 2 7 # 3 ; 16 MEMORANDUM |UNITS. FRACTIONS. i 3 ºf 1. $º-º-º-ºms § 24 24 2#, Ans. 9 Add the following: 2, #, #, 3, and #, 3. I's, #, #, 4. #, #, and #, 359. Add #, #. #, #, OPERATION. 1 2 3 4 5 7 amºmº ºms ºsmº ºsmºs smºs º-se 2 3 4 5 § 3 UNITS. MEMORANDUMI IFRACTIONS, 1. 1. I I 3 ; 17 1. * *= *-* * * tº 4 2 4 3 40 4}# Ans. 4 . 6 • 4 # and #; # and #, Ans. 1-#. Explanation.—Since the fractional units are unlike, and since neither can be reduced to the unit value of the other, We, there- fore, first reduce both to a common denominator, by multiplying their denominators together, which gives 12, as the least common denominator. Having the least common denominator, we next To do this, we + = *; and # = |* ; Ans. PROBLEMIS. Ans. 1:#. AnS. 1+. 4 1 7 2 ° Ans. 234. 9 5 - 1 2 3 e Ans. 'IOT 3 1#; Ea:planation.—In all problems having more than two terms, we select and add only two terms at a time. Here we select the # and # and reason thus: # = #, which added to # = ** = 13. We set the 1 in the column of units and the # in the column of mem- morandum fractions, and cancel the terms added, } and #. We now have the #, and # to add, and producing the least common denominator, 24, we reason thus: # = # and # =#. (Write the 16 under the # as a memorandum). Then # = ºr and # = #. (Write the 9 under the # as a memorandum). Then # and # = # =1#a. We write the 1 in the column of units and the ºr in the Column of memorandum fractions. We then cancel the ##, add the Column of units and to the sum annex the remaining fraction and obtain 23'ſ, the answer. Ans. Ans. 5 1 7 4}#. # , and g together, Eacplanation.—Selecting the 4 and # to add first, we reason thus: # = }, which added to # = # = 14. We write the 1 in the column of units and the # in the column of memorandum fractions and can- cel the # and #. Next we select the # and 3 and reason thus: # = }, and # = # which added to # = } = 13 = 13. We write the 1 in the column of units and the # in the column of memorandum fractions, and cancel the # and #. Then we select the + and the # (in the col- umn of memorandum fractions) andreason thus: # = } which added to # = #, We set the # in the column of memorandum fractions and cancel the # and #. Next we select the # and the # and reason thus: # = # and # = #, which added to g = i =1#. We write the 1 in the unit column and the # in the column of memorandum frac- tions, and cancel the # and #. # and # remain to be added; and as they are not of the same unit value and neither can be reduced to the unit value of the other, we proceed to reduce both to equivalent fractions of the same unit, or to the least common denominator, which is 40. Having the least common denominator, we then reduce the # and # to their numerical equivalent fractional values in fortieths. To do this, we rea- son thus: # = #, and # =#3; # = f; and # = #3, which added to # = # =1#. We writé the 1 in the column of units and the #6 in the column of memorandum fractions and cancel the # and #. * ADDITION OF FIRACTIONS. 177 Lastly we add the column of units and to the sum annex the remaining fraction, and thus produce 445 as the correct result. NotE.—The foregoing problems illustrate the most easy, rapid, and rational system of adding fractions known, and as fractions are so indispensable and of so frequent occurrence in practical life, the principles involved in the system should be thoroughly understood. In practical work, we would very much shorten the operation by adding several fractions at once, and mentally performing the most, if not all of the reduction and addition work, without stating the results. Thus, in the above problem, we would add the 4, #, and £ at once. We can instantly see that their sum is 'º, or 2+, and without naming or setting the 24, we add to it mentally the result of 3 and #, which we mentally see is #, or 13, making 33, which are the only figures we write. Thus all the fractions, except # and #, are added at one mental operation. Then we men- tally add the sum of # and # by the same process of reasoning as given in the illustration of the above example, and obtain the correct result, 4+5. 3 4. 4, 17 Add #, 32 ºz. 2 T- OPERATION. 3 4. A 17 Eacplanation.—Selecting the # and # to first add, we reason smº sº-ºº ºne *-*. thus: The least common denominator is 20; # of 20 = # and # 4. # 9 24 = }}; then # of 20 = ºr and # = ##, which added to # = ## = 1}, which we set in their proper columns and cancel # and #. We next select š and #4 and reason thus: The L. C. D. is 72; UNITS. MEMORANDUM of 72 = *g and # = }}; then ºr of 72 = **, and #4 = }}; and FRACTIONS. + ## = }} = 1%, which we write in the proper columns, and cancel the # and the #4. We now have ## and ++ to add, and 1. II. If having obtained 360 as the least common denominator, we rea- ^^ T^ 3 #3, son thus:...g. = ##, and ## = ###; * = 3;u, and +} = #3; # 2 #5 3 29 72 3 6 O + $5 = ### which we write in the proper column and cance 2###, Ans. ## and ##. Then adding the units’ column and annexing to the sum the final fraction, we produce the answer, 23#. NOTE.—To contract the work of finding the least common denominator of two fractions, we see, by inspection, what is the greatest common divisor of the denominators, and then divide either denominator by it, and, with the quotient multiply the other denominator the product will be the least common denominator. 2. Add + of ; of 24, 3, #, and # together. OPERATION. Statement showing the reduction of the fractions. Explanation.—Here we have compound frac- 3 e 1 2 g 3 5 9 tions and mixed numbers, and before adding, * , 4 s g wzº we reduce the mixed numbers to improper frac- * > Statement showing the result of the reduction and tions, and the compound fractions to simple the addition of the fractions ones. Then we add the # and #, which are TNITS. MIEMORANDUMI FRACTIONS. equal to 1 and #; then the 3 and #, which are 3 3 # 9 I 23 equal to ##; then the ## and #, which are equal 1. A s 6 2; § 24 to 1 and ###. Then adding the whole numbers, 20 216 3 529 and annexing the fraction, we have 2#33 as the correct result. GENERAL DIRECTIONS FOR THE ADDITION OF FIRACTIONS. 360. From the foregoing elucidations, we derive the following general direc- tions for the addition of fractions: 1. Select such two fractions as can be the most easily reduced to the same frao- 178 SouLE's PHILOSOPHIC PRACTICAL MATHEMATICS. X- tional unit ; find the equivalent value of both the fractions in this fractional whit, and add the numerators together; if the sum equals or eacceeds a unit write the unit in the column of units, and the fractional remainder in the column of memorandum fractions. If the sum is less than a unit, write the fraction in the column of memo- ºrandum fractions. Then cancel the fractions added. 2. In like manner select, reduce, and add two more fractions, and thus proceed wntil all are added. 3. When there are compound fractions, reduce them to simple ones before adding. When there are mia!ed numbers write the whole numbers in the column of wnits, cancel them from among the fractions, and then add the fractional numbers. All fractional ea.pressions should be in their lowest terms before adding. 361. PROBLEMS IN ADDITION OF FERACTIONS. 1. Add #, 3, #, #, #, and Hº, Ans. 4%. 11. $, ºr, and #, Ans. 1%;" 2. Add #, #, and ##, Ans. 23%. 12. 34, 1}, 24, and 4*, Ans. 114. 3. Add #, #, #, and Hº, Ans. 243. 13. #3, #, ##, and ##, Ans. 2 #. 4. Add #, ##, ##, and ##, Ans. 34%, or 3%. 14. 24, 34, 44, and 5, Ans. 1513. 5. Add #, +%, #, and ##, Ans. 3.}}. 15. #, #, #, and #3, * Ans. 14%g. 6. # of # and 2% of § of #, Ans. #4. | 16. 73, 53, and 10}, Ans. 23}}. 7. Add 24, 6%, 5}, and 24, Ans. 17. 17. 44, ##, and ºr, Ans. 14%. 8. §, #, and #5, Ans. 21%. | 18. 14, 34%, 13, and #}, Ans. 21+3. 9. #, #, and #, Ans. 1-#. 19. 3, 1+3, 10%, and 5. Ans. 1834. 10. 11'5, 6%, 18}}, and 25%, Ans. 28%. 20. 1254, 327+, and 25+, Ans. 478+. 21. #3, #, 14%, ##, and ###, Ans. 3+}}. 22. What is the weight of 10 sacks of wheat which weigh respectively: 1544, 1494, 1603, 1573, 1524, 1413, 1633, 1584, 1394, and 1613 pounds? Ans. 15394 lbs. 23. Add + of ; of # and 24 of ºr of 1. Ans. 1 #. 24. Add # of ; of 4 and # of 4 of #. Ans. 23#. 25. How many yards in 8 bolts of domestic, measuring as follows: 40%, 394, 434, 42%, 433, 384; 39%, and 414 yards? Ans. 328}#. 26. 14 bags of coffee weigh as follows: 1624, 1633, 161+, 1644, 1654, 1643, 1653, 1623, 165*, 1643, 1653, 1654, 164++, and 1654 pounds. How many pounds in all ? --- Ans. 2301. 27. A merchant bought 11534 pounds of rice for $924; 871; pounds of sugar for $87%; 5803 pounds of coffee for $115g; 240+ pounds of cheese for $434; and 408; pounds of Graham flour for $183. What was the total number of pounds, and the total cost of all he purchased ? Ans. 32543 pounds; $357; cost. 28. Add #, #, 4, #, and 3% of # of 14. Ans. 5}}. 29. Add #, 3, #, +}, 1}, and 4 of 3. Ans. 5*. * ADDITION OF FRACTIONS, I 79 ADDITION OF FEACTIONS BY A NEW METHOD, 362. The following new method of addition of fractions may be used to advantage when the fractions are large, and the denominators are prime to each other. * The principles involved are novel and should serve to awaken increased interest in the operation of numbers. 363. 1. Add the following fractions: #, #, and #. OPERATION. #, #, and #, added by the Explanation.—In the operation of problems by usual method, gives 2:#. this method, we are governed by the following gen- eral principles: 1. A proper fraction is less than unity. 2. A complement of a fraction is the differ- ence between the fraction and unity, and is found by subtracting the numerator from the denominator. OPERATION BY THE NEW METEIOD. FRACTIONS. UNITS. COMPLEMENTS. T. * = º — 4 e d # = 1 9 3. The sum of several fractions is therefore equal # = 1 — # to as many units as there are fractions, less the sum 4 I of the complements of the fractions. # = 1 — # In this problem the sum of the units is 3, and s=ºs amºe the sum of the complements is ###, and the differ- 3 — ### = 25%, Ans. ence is 2 ºr, the º: 3602 e 02 º SECOND OPERATION BY THE NIEW METHOD, FRACTIONS. UNITS. COMPLEMENTS. # = 1 — # # - 1 — ; ; 360 the C. D. x the 3 units = 1080 the sum of the 4. == 1. tº- l units in the C. D. unit. 5 5 3 Then # = 80 * # of 360 = } = 135 K = 287 the sum of the complements. 5. J - 72 *. 1080 – 287 = 793 = the sum of the fractions. 793 -i- 360 = 2;, Ans. NOTE.-In case some of the denominators are composite and not all prime to each other, the L. C. D. may be found and the operation performed as above. PROBLEMIS. Add the following problems by the above method: 9 1 6 2 3 #5 8 1 O 7 2. Hºr, #, and ##. 4. ##, ##, 3, and ###. 1 2 1 1 3 1 _9.9 205. 3. ##, ##, #, and #}. 5. Hº, and ###. subtraction Of Fractions. =N º-º-º-Lº- 364. Subtraction of Fractions is the process of finding the difference between two fractional numbers of like units and of like parts. 365. The Principle is, that fractions can be subtracted only when they express like parts of like units Or When they have a common denominator. ORAL EXERCISES. NOTE.-See Oral Exercises pages 167 and 168. 366. 1. One unit of any kind equals how many 3's? # taken from # leaves how many 3's 2 One unit of any kind equals how many 3's 2 # from # leaves how many #’s? # from 4 leaves how many #’s? # from 4 leaves how many #’sº # from 4 leaves how many 3's 2 i 367. Answer by mental work the following numerical questions: #—# = ? * — ; = ? # — # = ? # -- #– # = ? # — # = } # —# = ? 1 —# = ? # + 3–4 = ? # —# = ? #—# = ? 34–13 = ? 24 — 3 + 4* = ? PROBLEMIS. 1. What is the difference between +; and ++ 7 Ans. #. OPERATION. ## Explanation.—In this example the denomina- ll. OI’ 4-3 – 44 = + Ans. ** 1 5 T 1 5 57 tions being the same, we have but to take H. #, Ans. from +3 to obtain the correct difference. 2. What is the difference between # and #3 Ans, go. OPERATION. Ea:planation —Here we see that the fractions are not of like # = 15 or ; # units, and that neither can be reduced to the unit of the other, # = 16 15 16 therefore, we must reduce the fractions to a common denomi- nator. By inspection, and in accordance with the principles —l. ºmº- 1 as explained in the first four problems of addition of fractions, 2 0 } Ans. 3 O2 Ans. We see that the least common denominator is 20; then, that # are equal to #3, and that # are equal to #3, and that the differ- ence is go. ** * * (180) ¥ SUBTRACTION OF FKACTIONS. I8 I 3. From 28; take 74. Ans. 214. OPERATION. 28; Ea:planation.—In this problem we first observe that the 7; # may be reduced to 8ths, and by the exercise of our reason we see that it is equal to #, which taken from 3 amºs- leaves #; then we find the difference between the whole 21; Ans. numbers as in simple subtraction. 4. What is the difference between 37; and 12#3 - Ans. 24+}. OPERATION. 72 99 Explanation.—By inspection, we here see that the frac- 373 = 27 tions belonging to the whole numbers are not of like units 12# = 56 or of the same denominator, and that neither can be reduced to an equivalent fraction of the same unit as the -> other, and, therefore, we must reduce both to fractions 24}#, Ans. of like whits or of a common denominator, before we can subtract; and by multiplying together the denominators, we produce 72 as the least common denominator, which, for convenience, we write below the fractions, and by the same reasoning as given in addition of fractions, we see that § are equal to #3 and that § are equal to ##, which, for convenience, we carry to the right of the respective fractions, and to economize time, we write only the numerators. ... We now observe that the upper fraction, belonging to the greater number, is less than the lower fraction, belonging to the lesser number. Therefore, before we can subtract the fractions, we must add 1, reduced to 72ds, to ##, which gives us #3. We now subtract # from ## and have a remainder of # as the fractional part of our answer. . We now add 1 to the subtrahend, to compensate for the 1, previously added to the minuend, making it 13, which we subtract from 37 and have a remainder of 24, which we write below the line and thus complete the operation. GENERAL DIRECTIONS FOR SUBTRACTING FRACTIONS. 368. From the foregoing elucidations, we derive the following general direc- tions for subtracting fractions: 1. Reduce the given fractions to equivalent fractions of like fractional units, or of the least common denominator; then take the difference of their numerators and write the same over the common denominator. 2. When there are mia!ed numbers, subtract the fractional parts first and then the whole numbers. PROBLEMS IN SUBTRACTION OF FRACTIONS. 369. 1. What is the difference between +4 and #3 Ans. #. 2. What is the difference between # and #3 Ans. #. 3. What is the difference between 5% and 3; ? Ans. 23. 4. What is the difference between 7 and 3+6 & * Ans. 34%. 5. What is the difference between 23# and 14? Ans. 9% What is the difference between the following numbers: 6. # and #, Ans. #. 9. § of # and # of #, Ans. #. 7. # and #, Ans. #6. 10. 84 and 3%, Ans. 4}}. 8. § and #, Ans. #'s. 11. 12; and 94, Ans. 2%. I 82 soule's PHILOSOPHIC PRACTICAL MATHEMATICS. XF 12. Hºf and #, Ans. Tºg. 17. 25% and 945, Ans. 16#. 13. 4 and #, Ans. #. 18, 9 and 35%, Ans. 5}}. 14. # and #5, Ans. ##. 19. Hº, and 3Fs, Ans. 3 #. 15. 2; and 13, Ans. 1+. 20. 7+; and 4, Ans. 3+3. 16. 9; and 23, Ans. 6+5. 21. 31; and 173, Ans. 13%. 22. From 6% of + + 13; take # of # of 15—# of #. Ans. 13%. OPERATION INDICATED. * x * = 2; 2 + 13% = 15%; # × 3 × 15 = ** = 3; ; 3%– (#x #) = 2;. Ans. 15%—2; = 13%. 23. From 84 + 63 — ºr take # of # of 3 of 13 + 2}. Ans. 10#3. 24. E. J. Jacquet had $38#. He gave $2} for a pair of Indian clubs, $5; for books, $14 for a drawing board, and $3 for ink and pencils. How much had he left? Ans. $283. 25. W. G. Crena had $74 and his friend gave him $4 more; R. C. Bush had $16; and he spent $5g. How much more has R. C. Bush than W. G. Crena? Ans. $24. 26. M. Gundersheimer bought 2 bags of coffee, each weighing 1634 pounds. He sold 27+ pounds, 503 pounds, 87+ pounds, and 45% pounds. How many pounds has he left? Ans. 1163 lbs. 27. S. Delerno bought 75% gallons of molasses. He used 44 gallons, lost by leakage 23 gallons, and sold 22# gallons. How much has he left? Ans. 46% gallons. 28. What is the difference between a dozen times 6, plus 64, and 6 times a dozen minus one dozen and a half dozen 3 • Ans. 244. 29. J. Astredo bought 6 chests of tea weighing 38%, 42%, 444, 413, 394, and 43% pounds. He sold 120g pounds and used 5% pounds. How many pounds has he on hand? Ans. 123}. 30. J. Birba owned the steamer Isabel, and sold # of it. What is 4 of his present interest ? AnS. #. 31. From the sum of 64 and 83, take the difference between 14; and 94 % Ans. 10%. 32. What number is that to which if 16} be added, the sum will be 44; ? Ans. 27%. 33. P. H. Weiss bought 3 of ; of a vessel and sold # of # of his share. How much of the whole vessel has he left 2 AnS. #. 34. W. Van Benthuysen owned # of the steamer Natchez. He sold to J. Maier + interest in the steamer, and to Leo. Winner, 4 of his remaining interest. What is the present interest of each in the boat Ž Ans. Van Benthuysen, §; Maier, #; and Winner, 3. 35. F. Brinker and Henry Pike, were each # owners of a broom and brush factory. F. Brinker sold 4 of his interest to G. Wagner, and then # of his remaining interest to Henry Pike, who subsequently sold # of # of his whole interest to W. Lacoume. What is the present interest of each ownerº Ans. F. Brinker, $; G. Wagner, #; H. Pike, # and Wm. Lacoume, #. sultiplication of Fractions. =N Aº-sº -*ºr v---wº-w 370. Multiplication of Fractions is the process of multiplying when one or both of the factors contain fractional numbers. `NotE.—See the Philosophy of Multiplication, pages 77 to 80. In the multiplication of simple numbers, we saw that the result of multipli- cation operations was increasing, but in the multiplication of fractions, when the multiplier is less than a unit, the result is decreasing. This is evident from the fact that multiplication is the process of repeating the multiplicand as many times as there are units in the multiplier, and, therefore, when the multiplier is less than a unit, the multiplicand will be repeated only a part of a time, or such a part of itself as the multiplier is a part of a unit. PRACTICAL DEFINITION OF MULTIPLICATION. 371. To elucidate the principles of the subject and render clear the reasoning, We present our first questions in denominate numbers; and to aid still further in comprehending the work, we give the following practical definition of multiplication. 372. Multiplication is that operation in the practical computation of numbers, of finding the cost of either a part of one, or of many pounds, yards, barrels, etc., when the cost of one pound, yard, barrel, etc., is given. On the prin- ciple or fact embraced in this definition, we base our reasoning for the solution of every practical question that can possibly be presented in multiplication, either of Simple numbers or of fractions. METHOD OF REASONING. * 373. Considering the foregoing, we see that in all multiplication questions of a practical nature, we must necessarily reason from one, or unity, to a part of one or many. Thus, if 1 pound cost 50%, 4 of a pound will cost 4 part of it; and if 1 yard cost $2, 3 yards will cost three times as much, or 3 times $2. In the Solution of questions in abstract numbers, we apply the same system of reasoning without naming the factors, and thereby avoid all of the arbitrary rules given in other arithmetics of the day. (183) 184 soule's PHILOSOPHIC PRACTICAL MATHEMATICs. * STATEMENT LINE. 374. In all problems where we have both multiplication and division work to perform, we use a vertical line, to facilitate the operation. The right hand side is the increasing or multiplication side, and the left hand side is the decreasing or division side. At the top of the right hand side, we first write the premise of problems, or the number to be multiplied or divided, or that which the conditions of the question require the answer to be in. Then, continuing with the operation, the increasing and decreasing numbers are written respectively upon the increasing and decreasing sides. But be it remembered, we never place a number on either side after the premise or nature of the answer is written, without giving a reason therefor. In practice, the reason is instantly seen by the mind and directs the placing of the figures. When a problem is fully stated on this line, cancellation is applied and the operation is performed more readily than by any other manner of statement. NoTE.—See page 161 for a full elucidation of the Statement Line. º ORAL EXERCISES. 375. 1. What will 6 pounds cost at 5% per pound 7 Ans. 30%. SOLUTION.—According to Article 152, page 77, we reason as follows: 1 pound costs 5c. Since 1 pound costs 5c., 6 pounds will cost 6 times as much, which is 30c. SOLUTION STATEMENT. e g º 2. What will 6 pounds 2 | 11 Reason.—one pound costs 53c. cost (a) 54% per pound 2 § 3 Since 1 pound COStS 5ie, or *c., * 6 pounds will cost 6 times as Ans. 332. | 33g, Ans, much, which is 33c. SOLUTION STATEMENT. Reason.—One pound costs 5c. 3. What will 64 pounds 5 Since 1 pound costs 5c. 4 pound cost at 52 per pound? 2 13 * will cost # as much, and 4% lbs. Ans. 32.4% - will cost 13 times as much, o º 32% Ans. which is 324c SOLUTION STATEMENT. Explanation and Ireason.—Here 4. One pound costs 54%. ; # we see by considering the ques What Will 6 # pounds COSt tion, that 5% c. is the number to *= - - sº be multiplied. Accordingly, we at the same rate % 4 || 143 place the same at the top of the º line; and to facilitate the work, to free the operation of º e 35% 2 Ans. jractions—we reduce #. 5}c. to halves, making *c. the denominator of which we place on the degreasing side, and the numerator on the increasing side, of the statement line. We then reason as follows: Since 1 pound costs 42-c., # of a pound Will cost 4 part of it, and as this conclusion is a decreasing one, we write the 2 on the decreasing side; then, since 4 of a pound costs the result of the statement thus far made, *, 6% reduced, will cost 13 times as much, which, because the conclusion is an increasing one, we writé on the increasing side, and thus complete the reason and the statement, which worked out, gives the answer, 35% c. * MULTIPLICATION OF FRACTIONS, I85 SOLUTION STATEMENT. g * Reason.—One pound costs 5% 5. One pound costs 54%. } § 4 or *c. Since 1 pound costs lºc. What will 64 pounds cost -** -i tºº + pound will cost the 4th part at the same rate % 3 || 100 and 34 pounds will cost 25 times 33.4%, Ans. as much. THE REASON, WHY, AND wherEFORE, CONTINUED. Question. How do you know that if 1 yard costs * cents, # of a yard will cost the 4th part? Answer. By the exercise of my judgment—by the use of the reasoning faculties of the mind. Question. What do you mean in this connection, by judgment 3 Answer. The conclusion arrived at by the operations of the mind after duly considering the premise, the facts, and the conditions of the problem. Question. What do you mean by premise or premises 3 Answer. The proposition, declaration, truth, or fact, which is asserted as the basis or predicate of a question. In this problem the premise is, one pound costs 5% or * cents. Question. Why will 64 pounds cost 64 times as much as 1 pound 3 Answer. Because 6+ is six and one-fourth times as much as 1, Question. What kind of reasoning is the foregoing? Answer. Analogical and axiomatical. It is Analogical, because there is analogy, relationship, or likeness existing between the cost of 1 pound and the cost of 64 pounds. It is Axiomatical, because the premise and question considered, the conclusion is self-evident. Question. What is reason 3 Answer. The faculty or power of the human mind by which truth is dis- tinguished from falsehood, right from Wrong, and by which correct conclusions are reached by considering the logical relationship which exists between the premises, the facts, and the conditions of particular statements and questions. SOLUTION STATEMENT. - Reason.—One yard costs 163c. 6. At 1632 per yard, 3 || 50 Since 1 yard costs ºc., 4 of a º 2 || 45 ard will cost the # part d what will 224 yards cost 3 y part, an # y * yards will cost 45 times as $3.75, Ans. much. SOLUTION STATEMENT. g Ičeason.—One bushel costs 664c, 7. What will 4; bush- 2 | 133 Since 1 bushel costs 134c., 4 of a els cost at 6632 per bush- 4 || 19 bushel will cost the 4 part, and el ? wº-mº i =ºmºsºme * bushels will cost 19 times as $3.15%, Ans. much. I86 SOULE's PHILOSOPHIC PRACTICAL MATHEMATICS. 4× Question. How do you know this? SOLUTION STATEMENT. $ Reason.—One dozen costs $4. 8. Chickens are selling 4 . Since 1 doz.costs ºf dollars, , of a at $3; per dozen. What 6 || 25 dozen will cost # part, and * will 4+ dozen cost 3 gº | * dozen will cost 25 times as much. $153, Ans. Questions. 1. How do you know this? 2. What do you mean by judgment SOLUTION STATEMENT. Reason.—One gallon costs 42}c 9. What will 8 gallons 2 | 85 Since 1 gallon costs * cents, 8 cost at 424 cents per gal- 8 gallons will cost 8 times as lon 3 | $3.40, Ans. much. SOLUTION STATEMENT. Reason.—One pound costs 12c. 12 Since 1 pound costs 12 cents, # 2 | 29 º of a pound will cost the # part, * - sº and * pounds will cost 29 times $1.74, Ans, as much. SOLUTION STATEMENT. º $ Reason.—One dozen costs $#. 11. What will # of a 4 || 3 * Since 1 dozen costs # of a dol- g 3 || 2 lar, of a dozen will cost the dozen cost at $# per dozº * ? 10. What will 144 lbs. cost at 12 cents per pound? gº tº sº # part, and 3 of a dozen will cost $3, Ans. 2 times as much. . SOLUTION STATEMENT. Reason.—One day’s work is 12. A mechanic Works 4 13 worth $3+. Since 1 day’s work 153 days and receives $3} 2 || 31 is worth ** dollars, # of a day’s per day. How much money s 403 work is worth # part, and ** is due him 3 days’ work is worth 31 times as $503, Ans. much. Questions.—1. How do you know this? 2. What do you mean by judgment 3 3. What is the use of the statement line? 4. What kind of numbers do you place on the right hand side 2 5. What kind on the left # 6. What do you do before you place a number on either side? Answer.—Give the reason for so doing. 7. What do you mean by reason ? 8. What is Cancellation ? 9. Why do you use Cancel- lation ? GENERAL DIRECTIONS FOR MIUILTIPLYING FRACTIONS. 376. From the foregoing elucidations, we derive the following general directions for multiplying fractions? 1. Write on the upper right hand side of the statement line the premise of the problem ; or the number which is to be multiplied or divided ; or the number represen- Yºr MULTIPLICATION OF FRACTIONS. 187 ting the answer. Then, reasoning from ONE, OR UNITY, To A PART OF ONE or to many, write the other numbers upon the multiplication or the division side of the line, according as the conclusion is increasing or decreasing. 2. Mia!ed numbers should be reduced to fractional eacpressions, and the reason given for writing both the numerator and the denominator. PROBLEMIS. 377. What will # of a yard cost, at # 2. What will 584 pounds cost, at 16# of a dollar per yard 3 Ans. $4. per pound 3 Ans. $9.75. OPERATION OPERATION $ 2 4. § Write the reason. .* 25 ; # 39 Write the reason. : | $9.75, Ans. | $#, Ans. 3. What will 3; dozen cost, at $3% per dozen ? Ans. $12}. NOTE.—Make the statement and write the reason. 4. What will 53 bushels cost, at 154% per pint % OPERATION. Explanation.—Reducing and placing the 15% c. on the state- 2 31 ment line, we reason as follows: Since 1 pint costs *c., 2 pints or a quart will cost 2 times as much ; and if 1 quart, 2 costs the result of the statement now made, 8 quarts, or * a peck, will cost 8 times as much; and if a. peck costs the 4 = 124 result of this statement, 4 pecks, or a bushel, will cost 4 8 || 43 times as much ; and if a bushel costs the result of this - º # of a ºhel will cost the # part of it, and 4* ºmº- Cost 43 times as much. $53,32, Ans. 5. What will 50+ pounds of tea cost, at 1032 per ounce? OPERATION. Ea:planation.—Here we reduce and place the 10}c. on the 2 21 line, and reason thus: Since 1 ounce costs *c., 16 ounces or 2 1 pound will cost 16 times as much, and since 1 pound costs 16 the result of this statement, + part of a pound will cost + 4 || 201 part of it, and *%+ will cost 201 times as much. This com- -ºms pletes the reasoning and statement, which worked, gives $84,42, Ans. $84.42, answer. Make the solution statement and write the reason for the following problems: 6. What will 24 pounds cost, at 94% per pound 3 Ans. $2.28. 7. What will 143 dozen cost, at $5 per dozen ? Ans. $73}. - 8. What will 6 dozen and 7 chickens cost, at $4.87; per dozen 3 Ans. $32,093. 9, What will 42 pounds and 11 ounces of butter cost, at 2232 per pound? Ans. $9.60+}. SOLUTION STATEMENT. Explanation and Reason.—As usual, we here place 2 || 45 the premise, the price of one pound, on the statement 6 | 683 line, and reason as follows: 1 pound, or 16 ounces, cost #c. Since 1 pound costs *c., 1 ounce will cost, the 16th part and 683 ounces (which is 42 pounds and $9.60+}, Ans. 11 ounces) will cost 683 times as much. 1. 188 SOULE's PHILOSOPHIC PRACTICAL MATHEMATICs. ¥ 10. What will 193 pounds cost, at 1832 per pound? Ans. $3.60;#. 11. What will 253 yards cost, at 17% per yard 3 Ans. $4.55%. 12. What will 113 yards cost, at 1242 per yard? Ans. $1.42 #. 13. What will 213 yards cost, at 1642 per yard 2 Ans. $3.58%. 14. What will 14+ pounds cost, at 1242 per pound? Ans. $1.74%. 15. What will 31; pounds cost, at 11.3% per pound? Ans. $3.46%. 16. What will 13 pounds and 9 ounces of tea cost, at 87%g per pound? Ans. $11.86%. TO MULTIPLY ABSTRACT FRACTIONAL NUMBERS. 378. 1. Multiply 84 by 33. OPERATION. 25 Ea:planation and Reason.—In this problem, both factors 4 Iš 5 are abstract numbers. Hence we cannot give the same - analogical reasoning as we gave in the foregoing problems 125 º where the factors were denominate, or concrete numbers; although were we to do so, the result, so far as the figures are concerned, would be correct. We therefore reduce and 31%, Ans. place the 8%, the number to be multiplied, on the statement li.e., and reason as follows: Axiomatically 1 time 8; or * is º. Since 1 time * is º. 4 time * is + part of *, and * are 15 times as many. Or, by analysis, thus: 1 time * is %, Since 1 time * is º, + time * is the # part = }}, and * are 15 times #3 = ** = 314, answer. * NOTE.—It will be observed that we found our reasoning upon one, which is the basis of all numbers, as explained in Article 155, page 80. Multiplication is the process of repeating one number as many times as there are units—ones—in another. And it is self-evident that one time any number is equal to the number. This self-evident conclusion is the premise for all questions in multiplica- tion of abstract numbers. 2. Multiply § by 3, Ans. }. SOLUTION STATEMENT. 4 || 3 Reason.—One time # is #. Since 1 time # is #, # time # is the 3 2 # part, and # times # is 2 times as many, which is #. * - Or, by analysis, thus: Since 1 time # is #, time # is the # #, Ans. part of # = 1%, and # times # is 2 times ºf = |*, or #, answer. 3. Multiply +} by $, and write the reason, Ans. Hºs. SOLUTION STATEMENT. 4. Multiply 6 by É, and 6 : pº . 3}. Write the reason. 8 || 5 Ans. 3 SOLUTION STATEMENT. 6 || 5 5. Multiply # by 9, and 9 write the reason. 7 * - Ans. 73. | 6. 5; by ÉÉ, and write the reason. Ans. 24. 7. $4 by ##, and write the reason. AllS. #. 6 * MULTIPLICATION OF FRACTIONS, * 189 MISCELLANEOUS PROBLEMS IN MULTIPLICATION OF FERACTIONS. 379. 1. What will 16 yards cost, at 1432 per yard? Ans. $2.36. 2. What will 23# pounds cost, at 35% per pound? Ans, $8.31#. 3. What will # of a yard cost, at $3 per yard? Ans. $#. 4. What will 3 a yard cost, at $4 per yard? Ans. $4. 5. What will 84 pounds cost, at 74% per pound? Ans. 63%. 6. What will 103 pounds cost, at 94% per pound? Ans. 99 Hºg. 7. Multiply $4 by 12, Ans. 54. 8. Multiply 11 by Tºr, Ans. 1. 9. Multiply fºr by 11, e Ans. 1. 10. Multiply 13 by ºr, Ans. 94%. 11. Multiply 105 by gº, e * Ans. 12. 12. Multiply 136 by #, Ans. 44%. 13. Multiply 12 by 31#, Ans. 382. 14. Multiply # by #, AnS. ##. 15. Multiply 114 by 13, y Ans. 18%. 16. Find the value of #, of $, of #, of #, of 4, Ans. #. 17. What is the product of #, #, #, and #3 Ans, ºs. 18. What is the product of 13, #, 2, and 54% e Ans. 11}. 19. What is the product of # of 2+ by # of 74% Ans. 1 #. 20. What is the product of 124 multiplied by 54 times 63%. Ans. 4641%. 21. At +} of a dollar a pound, what will # of a pound of tea cost 3 Ans. *F of a dollar. 22. What will 5; dozen buttons cost, at gº of a dollar per dozen 3 ºs. Ans. # of a dollar. 23. What will 43 yards cost, at 442 per yard? Ans. 20%. 24. What will 143 dozen cost at $5 per dozen ? Ans. $734. 25. What will 154 pounds cost, at 1042 per pound 7 Ans. $1.62%. 26. What will 40g pounds cost, at 22#g per pound? Ans. $9.19%r. 27. What will 2812; gallons cost, at $4.50 per gal? Ans. $12656.25. 28. What cost 471; gallons, at $33 per gallon ? Ans. $1592:3. 29. Sold 9378524 pounds of cotton at 14% per pound. What did it amount to? Ans. $135695.53%. 30. If a man earns $2; in a day, how much will he earn in 16; days? Ans. $414. 31. A contractor pays $1+ per day for labor, and he has 370 men employed for six days. How much money will it take to pay them 3 Ans. $2775. 32. P. Machray paid +% of a dollar for a book, and for paper # of the cost of the book. How much did he pay for the paper? Ans. $3. I90 soul.E's PHILOSOPHIC PRACTICAL MATHEMATICs. 4× 33. Distillers of the essence of rose have determined, by experience, that it requires 48000 pounds of rose leaves to make or distill one pound of the ottar of roses. How many pounds of rose leaves will it require to distill 50% pounds of the Ottar of roses? - Ans. 2442000 pounds. OPERATION INDICATED. 34. If one pound costs a cent and a half, what will 2 3 e 254 pounds cost? 2 || 51 Write the reason. Ans. 3842. | 38+2. wº 35. G. V. Hooper owned # of the steamer Katie, and sold # of his share to Maschek. What part of the whole steamer did he sell? Ans. §. 36. E. Garner can work the problems in this book in 43 months. How many months would it take him to work 3 of them? Ans. 3% months. OPERATION. MOS. e Reason.—He can work all the problems in 4 || 19 *months. Since he can York all the prob- 3 || 2 * r. l.9. = 34. tº lems in *months, to work"; of the problems Or, "4 x # 3}, Ans he would require's part as many months, aná *- to work #, he would require twice as many 3%, Ans. months. " 37. W. J. Kearney paid $# for 1 gallon of molasses. What is # of a gallon worth at the same rate 3 Ans. $3. 38. What will 74 boxes of raisins cost, at $24 per box : Ans $16%. 39. On one occasion at the New Orleans Opera, 4 of the ladies and gentlemen present were French; # of the remainder, American; # of the remainder, German; and the others were of different nationalities. What part were Americans, what part were Germans, and what part were of different nationalities? Ans. # Americans, #3 Germans, and # of different nationalities. 40. C. Reynolds owned g of a plantation and sold # of his share to D. C. Williams, who sold # of what he purchased to Frank Soulé, who sold # of what he purchased to L. B. Keiffer. What is Keiffer's share in the plantation ? Ans. #'s. 41. W. Weiss owned # of 2000 acres of land and sold # of his share to H. Marsden, who sold # of what he purchased to J. T. Finney. How many acres have each 3 Ans. W. Weiss, 400; H. Marsden, 450; and J. T. Finney, 750 acres. MENTAL CONTRACTIONS IN MULTIPLICATION OF FERACTIONS. NoTE.—See pages 88 to 120, for Contracted Methods of Multiplication 380. In problems of multiplication of fractions in which the factors are small, the operation may be performed mentally, or with but few memorandum figures, without reducing or making the statement as above, and the result produced almost instantly. - 1. Multiply 124 by 8+, Ans. 103}. OPERATION. 12; Ea.planation. 8 × 12 = 96 + (4 of 12 = 3) => 99, -- (# of 8#. 8 = 4) = 103, which we set in the product line. Then, + of # = }, which is annexed to the 103, and completes the an- 103}, Ans. SWeI’. CONTRACTIONS IN MULTIPLICATION OF FRACTIONs. I9 I 2. What will 123 pounds cost, at 9% per pound 3 Ans. $1.23%. OPERATION. 9; 12; Explanation. 12 × 9 = 108 + (# of 9 = 6) = 114, + (# $1.23%, Ans. 3. Multiply 13; by 84, OPERATION. 13; # 8% 1.16%, Ans. 4. Multiply 113 by 84, OPERATION. 11; ; 8# 3 964, Ans. Multiply 124 by 44, Ans. Multiply 94 by 63, Ans. Multiply 54 by 54, Ans. Multiply 11% by 12 ºr, Ans. i Multiply 10% by 15, Ans. of 12 = 9) = 123, -ī- (§ of # = |* = }) = $1.23%, answer. Ans. 116%. Explanation. 8 × 13 = 104, -i- (# of 13 = 6}) = 110, with # reserved; + (# of 8 = 6) = 116, which is written in the prod- uct line. Then # of # = # -- the # reserved = }, which is an- nexed to the 116 and completes the answer. Ans. 96%. Explanation. 8 × 11 = 88, + (; of 11 = 5}) = 93, with 4 reserved; + (# of 8 = 2+) = 95, with # reserved; (then 4 of + = }, + + reserved = # + # reserved = }=1}, which is added to the 95 and completes the answer. PROBLEMIS. 53%. , 10. Multiply 243 by 93, Ans. 2394. 613. 11. Multiply 37% by 25%, Ans. 948#. 160}#. 12. Multiply 714 by 524, Ans. 37274. 28%. 13. Multiply 3704 by 434, Ans. 160244. 142*. º PRACTICAL APPROXIMATIVE CONTRACTIONS. TO MULTIPLY ANY FRACTIONAL NUMBERS TO THE NIEAREST UNIT. 381. 1. Multiply 9% by 84. OPERATION. 9% Explanation.—In this practical system of contraction, we 8; first multiply the whole numbers, and retain in our mind the product, to which we mentally add, first the product, to the nearest unit, of the multiplicand by the fraction of the multi- 77, Ans. plier, and then the product, to the nearest unit, of the multi- plier by the fraction of the multiplicand, and set the result in the product line, the same being the practical answer. - In this example, we see that the product of 9 and 8 is 72; that the product of 9 by the + to the nearest unit is 2; that the product of the 8 by the # to the nearest unit is 3, and that the sum of the three products is 77. 2. Multiply 104 by 7%. OPERATION. * 10} Laplanation.—Here we see that the product of the whole 7; numbers is 70; that of 10 to the nearest unit is 1, which added to the 70 makes 71; then that the # of 7 to the nearest unit is 2, *º which added to the 71 makes 73, the practical answer; 733, is 73, Ans. the exact result. I92 SOULE's PHILOSOPHIC PRACTICAL MATHEMATICS. † 3. What will 15; pounds cost, at 1342 per pound 3 OPERATION. 15; Earplanation. 15 × 13 = 195, -i- + of 15 to the nearest unit 13# is 4, - 199, -i- # of 13 to the nearest unit = 10, − 209, as the $2.09, Ans. practical result. The exact result is $208}}. 4. What will 253 yards cost, at 1732 per yard 2 OPERATION. 25; Explanation. 25 × 17 = 425, -H # of 25 to the nearest unit 17; is 19 = 444; + 3 of 17 to the nearest unit = 11, = $4.55, the practical answer. The exact result is $4.551. $4.55, Ans. NoTE.—In multiplying the 25 by #, we gained #c., and in multiplying the 17 by #, we lost #c.; this lºc, excess of loss, and the loss of #c. by reason of not multiplying the fractions, account for *gc. deficit in the answer. In practice, it is easy to see whether the loss or gain, by reason of the contractions, exceeds a unit; and if so, to increase or decrease the result accordingly. With large numbers, the regular system, as first presented, under the head of multiplication of fractions, is preferable to this approximative method. - - PROBLEMS. 5. Multiply 94 by 64, Ans. 593. 6. What will 113 yards cost, at 12% per yard 3 Ans. $1.42%. 7. What will 213 yards cost, at 1642 per yard 3 Ans. $3.58g. 8. What will 83 pounds cost, at 162 per pound? Ans. $1.40. 382. TABLE OF THIRTY-SECONDS AND THEIR EQUIVALENT VALUES IN DECIMALS. Fractions. Decimal IFractions. Decimal Fractions. Decimal Fractions. Decimal Lquivalent. e Equivalent. IEquivalent. Equivalent. # = .03125 ºr .28125 |}} = .53125 | # = .78125 # = Tº = .0625 # = ſº = .3125 # = # = .5625 }} = +3 == .8125 ºr - .09375 |}} = .34375 |}} = .59375 3% = .84375 32 = .15625 |}} = .40625 |3} = .65625 | }} = .90625 † = * = .1875 # = 15 = .4375 # = +} = .6875 % = }} = .9375 ** = .21875 $3 = .46875 |3} = .71875 |3} = .96875 & = *; - + = .25 #}=#=#=#= .5 #=}}=}=#= .75 #=}{=}=#=}= 1 383. TABLE SHOWING THE PRICE OF A SINGLE ARTICLE WHEN THE PRICE PER DOZEN IS GIVEN. Per Doz. Per Piece. Per Doz. Per Piece. Per Doz. Per Piece. Per Doz. Per Piece. 25c. = 21; c. $3.75 = 31+c. $7.25 = 60% c. $10.75 = 89.3 c. 50c. = 4}c. 4.00 = 33}c. 7.50 – 62}c. 11.00 = 91}c. 75c. = 6+c. 4.25 = 351%, c. 7.75 = 64 ºc. 11.25 = 93#C. $1.00 = 8#c. 4.50 = 37.4c. 8.00 = 66;c. 11.50 = 95;c. 1.25 = 10; c. 4.75 = 39%;c. 8.25 = 68}c. 11.75 = 97}}c. 1.50 = 12; c. 5.00 = 41}c. 8.50 = 70;c. 12.00 = $1.00 1. 75 = 141;c. 5.25 = 43#c. 8.75 = 72}}c. 12.25 = 1.02*, c. 2.00 = 16; c. 5.50 = 45;c. 9.00 = 750. 12.50 = 1.04% c. 2.25 = 18#c. 5.75 = 47}}c. 9.25 = 771, c. 12.75 -- 1.06+c. 2.50 = 20; c. 6.00 = 50c. 9.50 = 79;c. 13.00 = 1.08% c. 2.75 = 22+}c. 6.25 = 52; 2 c. 9.75 — 81+c. 13.25 = 1.10% c. 3.00 = 25c. 6.50 = 54;c. 10.00 = 83; c. 13.50 = 1.12% c. 3.25 = 27 ſºc. 6.75 = 56+c. 10.25 = 851% c. 13.75 = 1.14 'gc. 3.50 = 29;c. 7.00 = 584c. 10.50 = 87}c. 14.00 = 1.163c. º: CONTRACTIONS IN MULTIPLICATION OF FRACTIONS. I93 PECULIAR CONTRACTIONS OF MULTIPLICATION OF FERACTIONS. 384. The preceding problems, and the explanations given, fully illustrate the work, or the general principles of contraction of fractions, without regard to any peculiar combination of numbers. The principles upon which the work is based are so few and simple, the operations So short and easily performed, and the practical advantages of the work So great, that We specially commend it to the earnest attention of all classes of com- mercial men and accountants. In the following methods of contraction, the process depends somewhat on the peculiar combination of numbers, and hence, although shorter than the first contractions, is less valuable. TO MULTIPLY NUMBERS OF TWO OR MORE FIGURES EACH, WHEN ONE OR MORE OF TEIE RIGHT HAND FIGURES CONSIDERED As TENS, HUNDREDs, ETC., MAY BE REDUCED TO FRACTIONS OF HALVES, FOURTHS, OR EIGHTHS. 385. 1. Multiply 425 by 875. OPERATION. 4+ hundreds. Explanation.—In this problem, we reduce the units' and 8; hundreds. tens' figures of both the multiplicand and multiplier to fractions of + and # respectively and thus obtain 4+ and *-mºmº-º-º- 8% to be multiplied together, which operation we perform 371875, Ans. according to the work elucidated in Problem 1, page 190, and obtain a product of 37*. Then we reduce the frac- tional part, º, of this product, to a whole number, and annex it to the integral part of the prod- uct; 3 = 1875, which annexed to 37 gives 371875, the correct product. This nº is reduced to a whole number for the reason that it represents ºr of 10000; and it represents this for the reason that we first reduce four figures, two in each factor, to fractions. NOTE.-See Table of Thirty-Seconds, on page 192, for the decimal equivalents of fractions from ºf to ##. 2. Multiply 850 by 450. OPERATION. 84 Explanation.—By reducing the units’ and tens' figures 4; of both numbers as explained in the preceding problem, * we have 8+ and 44 to be multiplied together. The prod- *-mºmºmº uct of 84 × 44 is 38+. This + represents, for reasons 382500, Ans. given in the preceding problem, 4 of 10000, and hence we reduce it to a whole number and annex the same to the 38, 4 of 10000 = 2500, which annexed gives 382500, the correct product. 3. What will 2812; gallons cost, at $4.50 per gallon ? OPERATION. 284 Explanation.—The reason for the work of this problem 4; is the same as elucidated in the two preceding, and hence $12656.25, Ans. is omitted. t I94. soul E's PHILOSOPHIC PRACTICAL MATHEMATICS. jºr 4. What will 1650 bbls. pork cost, at $13.75 per bbl. OPERATION, 1. Explanation.--In this problem, we use the units', tens’ 16# and hundreds' figures of the price as #, and the units’ and & tens’ figures of the barrels as #, and thus produce a º-º-º-mºmºmºmº-º-º-º-º: product of 22#; the # we reduce and annex as above $22687.50, Ans. directed and obtain $22687.50. Multiply the following numbers: 5. 12250 by 4750. 7. 3456 by 2125. 6. 24250 by 8125. 8. 379 by 425. OPERATIONS. (5) (6) (7) sº 12+ 24+ 3456 7 4; 8} 2} 4+ 58187500, Ans. 197031250, Ans. 6912 1516 432 94; 7344000, Ans. 161075, Ans. 9. Multiply 775 by 425; 1850 by 475; 3250 by 875; 14625 by 16125. TO SQUARE NUMBERS THE SUM OF WHOSE FRACTIONS ADDS ONE. 386. 1. Multiply 94 by 94. OPERATION. 9% Ea:planation.—In this problem we add 1 to 9, making it 94 10; then we multiply 10 times 9 are 90, and 3 of # is +, and thus produce the correct result. We do this because one half of 9 taken twice and added to its square, is the 904, Ans. same as to multiply the 9 by 1 more than itself. This F. and process of work are applicable to the mul- tiplying of any two like numbers whose fractions add unity or 1. PROBLEMS. 2. What will 124 pounds cost, at 1242 per pound? Ans. $1.56}. 3. What will 64 pounds cost, at 6% per pound? Ans. 42*. 4. What will 83 pounds cost, at 85% per pound? Ans. 72%. 4. What will 193 pounds cost, at 193% per pound? Ans. $3.80%}. 6. What will 49 ºr pounds cost, at 49+2 per pound? Ans. $24.50%r. TO MULTIPLY ANY TWO NUMBERS, THE DIFFERENCE OF WHICH IS ONE, AND THE SUM OF WHOSE FRACTIONS IS ONE. 387. 1. Multiply 54 by 43. OPERATION. 5} Explanation.—In all problems of this kind, we add 1 to 4; the larger number, and then multiply the sum by the lesser number, and set the result in the product line; then 1 5 we square the fraction of the larger number and subtract 24}}, Ans. the result from 1, and annex the remainder to the whole number in the product. In this example, we added 1 to 5, which made 6: this we multiplied by 4, and produced 24; we then squared #, which gave us 's; We º ºcted the ſº from 1, and obtained #, which we annexed to the 24 and completed the COTTOC tº TêSUllt. * CONTRACTIONS IN MULTIPLICATION OF FRACTIONS. 195 PROBLEMS. ** 2. What will 84 pounds cost, at 7:42 per pound 7 Ans. 63%. 3. What will 103 pounds cost, at 94.2 per pound? Ans. 994'sſº. 4. What will 193 pounds cost, at 1832 per pound? Ans. $3.60%. TO MULTIPLY ANY TWO LIKE NUMBERS, WHOSE FRACTIONS HAVE LIKE DENOMINATORS, OR, WHOSE DENOMINATORS MAY BE EASILY REDUCED TO THE SAME DENOMINATION. 388. 1. Multiply 84 by 83. OPERATION. 84 Explanation.—In all problems of this kind, we add to 8; the multiplicand the fraction of the multiplier, and then multiply by the whole number of the multiplier and set Lºs the result in the product line; then we multiply the frac- 72%, Ans. tions together and annex the result to the whole numbers, & & & and obtain the answer. In this problem, we added # to 8}, which gave us 9, which we multiplied by 8 and obtained 72; we then multiplied the fractions and obtained º. The reasons given in the first problem, under the head of contractions, cover all these special cases, and hence we here only give the directions for performing the operation. PROBLEMS. 2. What will 43 yards cost, at 442 per yard 3 Ans. 20%2. 3. What will 94 yards cost, at 9% per yard? Ans. 92%. 4. What will 124 yards cost, at 1242 per yard? Ans. $1,60}}. TO MULTIPLY ANY TWO NUMBERS WHOSE FRACTIONS ARE ONE. HALF, ONE THIRD, ONE FOURTH, ETC., OR WHOSE FRACTIONS ARE OF THE SAME VALUE. 389. Multiply 7; by 54. OPERATION. 7; Explanation.—Here we multiply the 7 by 5, and produce 5} 35, to which we mentally add # of the sum of 7 and 5, and obtain 41, which we set in the product line; then we sºmº multiply the fractions together and obtain +, which we 41%, Ans. annex to the 41 and obtain the correct result. The reason * for adding 4, the sum of 7 and 5 is, because # of either number added to # of the other is the same as 4 of their sum. Were the fractions #, +, +, etc., we would add +, +, +, etc., of the sum. I96 SOULE's PHILOSOPHIC PRACTICAL MATHEMATICS. * 2. Multiply 83 by 43. OPERATION. 8; Explanation.—In this problem, we first multiplied the 8 4; by 4, and obtained 32, which we retained in the mind; then we added the 8 and 4, which gave us 12; we then * = multiplied the 12 by #, and obtained 8, which we added 40%, Ans. to the 32, and set the result, 40, in the product line; then we multiplied the fractions and annexed the product to the product of the whole numbers, and thus obtained the correct result. PROBLEMS. 3. What will 94 pounds cost, at 54% per pound? Ans. 49%.g. 4. What will 14+ pounds cost, at 1242 per pound? Ans. $1.74%. 5. What will 31; pounds cost, at 1142 per pound? Ans. $3.46%. 6. What will 93 pounds cost, at 1132 per pound? Ans. $1.14%. 7. What will 154 pounds cost, at 1042 per pound? Ans. $1.62%. 8. What will 405 pounds cost, at 22.Ég per pound? Ans. $9,194. NoTE.—See pages 88 to 120 for other Contractions in Multiplication. _º à livision of Fractions. =N 390. Division of Fractions is the process of dividing when the divisor or dividend, or both, contain fractional numbers. In the division of simple numbers, we saw that the result of division opera- tions was decreasing; but in the division of fractions, when the divisor is less than a unit, the result is increasing. This fact is plain, for the reason that the operation of division is the process of finding how many times the dividend is equal to the divisor, and, hence, when the divisor is less than 1, the dividend will be equal to the divisor as many times itself as 1 is equal to the divisor. *… QUESTIONS IN DIVISION. 391. In practical operations, we usually have the three following cases or questions in division of fractional numbers: 1st. To find the cost of one pound, yard, or article of any kind, when we have the cost of many pounds, yards, or articles of any kind given. 2d. To find the cost of one pound, yard, or article of any kind, when we have the cost of a part of a pound, yard, or article of any kind given. 3d. To find the number of pounds, yards, or articles of any kind that can be bought with a specified sum, when we have the price of one, or a part of one pound, yard, or article of any kind given. METHOD OF REASONING. 392. From these questions, we see that division is the converse of multiplica- tion, and that from the nature of the question, we must reason from many to one or from a part of one to one. Thus: 1st, if 5 pounds cost 50g, 1 pound will cost the # part of it; in the 2d case, if # of a yard cost $2, # of a yard will cost the # part of it, and #, or a whole yard, will cost 4 times as much; and in the third case, if *g buy 1 yard, or any other thing, #2 will buy the # part of it, and #, or a whole cent, will buy 2 times as much. NotE.—See Introductory Remarks and Elucidations of Division on pages 122 and 123. (197) I 98 SOULE's PHILOSOPHIC PRACTICAL MATHEMATICs. 4× 393. ORAL AND WRITTEN PROBLEMs. 5 e 1. If fºr cost x, what will 1 book cost Answer. If 5 books cost x, 1 book will cost the 5th part of x. -*- cost x. What will 1 hat cost? 9. * hats Answer. Since 8 hats cost X, 1 hat will cost the 8th part of x. 3. *... cost x. What will 1 pencil cost 3 pencils 4. —"— cost x. What will 1 orange cost 3 oranges 5. --— cost x. What will 1 chicken cost * chickens 6. g of a yard cost x. What will # of a yard cost 7 7. If 3 of a yard cost 102, what will # of a yard cost 7 Analytic Solution.—If # of a yard cost 10 cts., # of a yard will cost the half part of 10 cts., which is 5 cts. Question. How do you know this? 8. # of a pound cost 15 cts. What will # of a pound cost 3 9. § of a pound cost 20 cts. What is the cost of $ 3 10. # of a pound cost 12 cts. What did 1 pound cost? Analytic Solution.—Since # of a pound cost 12 cts., 4 of a pound will cost # of 12 cts., which is 4 cts., and #, or one pound, will cost 4 times as much, which is 16 cts. 11. § of a yard cost 40 cts. What will be the cost of 1 yard 3 12. § of dozen cost $16. What is the value of ; of a dozen 3 Analytic Solution.—Since # of a dozen cost $16, # of a dozen will cost 4 part, which is $8, and # or a whole dozen will cost 3 times as much, which is $24; and since 1 dozen costs $24, # of a dozen will cost + part, which is $6, and # of a dozen will cost 3 times as much, which is $18. SOLUTION STATEMENT. Reason or the Philosophic $ Solution. 2+ or # yards cost 13. 24 yards cost $64. $6+, or $*#. Since # yards cost : ; * dollars, # of º: will cost º 1. the 5th part, and # or a whole What will 3% yards cost 5 | 16 yard wifi cost 2 times as much; and since 1 yard costs the result of this statement, # of a yard at the same rate? $8, Ans. will cost the 5th part, and ºl will cost 16 times as much. SOLUTION STATEMENT. - Reason.—% pounds cost 14*c. g 14. 84 pounds cost 62; 2 | 125 Since * pounds cost 135 cts., # cents. What was the cost 25 | 3 of a pound will cost the 25th 1. º * - mºmme part, and #, or a whole pound, of 1 pound | 74%, Ans. win cost 3 times as much. Questions. 1. How do you know this? 2. What do you mean by judgment? 3. What kind of reasoning is this? 4. What is the premise in this problem? 5. What do you mean by premise? * I)IVISION OF FRACTIONS, I99 SOLUTION STATEMENT. 15. 33 yards cost 37; 2ſ 75 cents. What did 1 yard 15 || 4 cost 7 tºmºmºmº || s=s=mºmº 102, Ans. SOLUTION STATEMENT. $ 16. A man receives $21; 2 || 43 for 64 days' services. What 25 4 s the rate per day ? tº tº , Wa, |) y $3.44, Ans. Reason.—4 yds, cost #cts. Since "# yards cost ºf cents, # of a yard will cost the 15th part, and # or a whole yard will cost 4 times as much. Reason.—% days' services cost $#. Since he receives * dol- iars for * days' services, for + of a day's service he will receive the 25th part, and for #, or a whole day, 4 times as much. Or, since ºf days' services are worth ** dollars, 4 of a day's service is worth the 25th part, and 4 or a whole day's service is worth 4 times as much. SOLUTION STATEMENT. $ 17. 83 dozen cost $52. 26 º What will 1 dozen cost? sº | $6, Ans. SOLUTION STATEMDENT. $ 18, 9 sheep cost $33}. : 135 What will 1 sheep cost 7 &=º $33, Ans. SOLUTION STATEMENT. $ 19. If ; of a pound cost : : $g, what will 1 pound cost? dº $1%, Ans. 20. Bought 12; dozen for $124. What was the cost per dozen 3 SOLUTION STATEMENT. s fb 21. At 7:4 cents per 1. pound, how many pounds 1. ão can be boughtfor83} cents? tº ºsmºmº 11% lbs., Ans. 22. Chickens cost $# a piece. Reason.—% dozen cost $52. Since ºf dozen cost $52, # of a dozen will cost the 26th part, and 3 or a whole dozen, will cost 3 times as much. Reason.—9 sheep cost $1.3%. Since 9 sheep cost +3* dollars, 1 sheep will cost the 9th part. Reason.—# of a ſb, cost $3. Since # of a pound cost # of a dollar, 4 of a pound will cost the third part, and # or a whole pound, will cost 4 times as much. AnS. $1. Reason.—ºftc. buy 1 pound. Since ºf cents will buy 1 pound, 4 of a cent will buy the 15th part, and # or a whole cent, will buy 2 times as much; and since 1 cent will buy this result, # of a cent will buy the third part and *;0 cents will buy 250 times as much. How many can be bought for $134 3 Ans. 18 chickens. 2OO soul E's PHILOSOPHIC PRACTICAL MATHEMATICS. 2. REASONING FOR THE DIVISION OF ABSTRACT NUMBERS. 394. 1. Divide 6 by 2. STATEMENT. 2 || 6 Remarks.-The real question in this problem is to find *s how many times 6 is equal to 2. Or, in other words, we 3. A are required to measure 6 by the unit of measure, 2. The , Ans. basis or unit of all numbers is 1; and hence in our rea- soning for division of abstract numbers, we use 1 as our unit of first measure. The following is our premise, reasoning, and conclusion: 6 is equal to 1, 6 times. Since 6 is equal to 1, 6 times, it is equal to 2, instead of 1, # as many times, which is 3, 2. Divide g by #. SOLUTION STATEMENT. 8 || 7 Explanation and Reason.—Here we are required to mea: 3 4 sure # by the tunit of measure, #. The basis or unit of all numbers is 1. Hence, as explained on page 122, in our “mºsºme ºmº reasoning for division of whole numbers, we use 1 as our 24 || 28 first unit of measure. The following is our premise, rea- *mmº soning, and conclusion: # is equal to 1, § of a time. Since 1%, Ans # is equal to 1, § of a time, it is equal to #, instead of 1, 4 62 e times as many times; and to #, instead of #, the # part of the number of times. SOLUTION STATEMENT. Reason.—A is equal to 1 * e 4. 15 º Since + : equal to * tº º times, it is equal to #, instea 3. Divide 33 by 23. 8 || 3 of 1, 3 'times as many times, gºe and to #, instead of #, the eighth 1}}, Ans. part of the number of times. SOLUTION STATEMENT. Reason.—Writing 5 on the statement line, we reason thus: 6 # 5 is equal to 1, 5 times. Since 4. Divide 5 by ºr. 5 is equal to 1, 5 times, it is immºns mm sºmmºn equal to Tºr, 11 times as many 9%, Ans. times, and to ºr, the 6th part of the number of times. SOLUTION STATEMENT. Reason.-In this example, we 15 2 write the dividend on the state- tº º 2 8 ment line and reason thus: ºr 5. Divide # by 8. is = to 1, ºr of a time. Since ºmmºns mºmmº *; is == to 1, 1% of a time, it is go, Ans. = to 8, the 8th part of the Inumber of times. NOTE:-The solution of the five preceding problems elucidates the only correct reasoning for dividing abstract fractional numbers. But for practical work we would not advise a change from the reasoning given where the numbers are denominate. 6. Divide 223 by 54, Ans. 45%. 8. Divide 143 by 9, Ans. 13. 7. Divide 3 by 3, Ans. 44. 9. Divide 32 by 9, Ans. 33. * DIVISION OF FRACTIONS. 2OI GENERAL DIRECTIONS FOR DIVISION OF FBACTIONS. 395. From the foregoing elucidations, we derive the following general direc- tions for division of fractions: 1. Write on the upper right hand side of the statement line the number which is to be divided or measured. Then, reasoning from the basis of MANY to ONE, or from a PART OF ONE to ONE, write the other numbers on the division or the multiplication side of the statement line, according as the conclusion is decreasing or increasing. 2. Mia!ed numbers should be reduced to fractional eaſpressions, and the reason given for writing both the numerator and the denominator. MISCELLANEOUS ORAL EXERCISES. 396. 1. If 3 of a yard costs $24, what will 1 yard cost? Ans, $6. 2. If 3 of a yard costs $24, what will # of a yard cost? Ans. $#. 3. # of a number is 15. What is the number 3 Ans. 20. 4. If 3 of a number is 8, what is 13 times the number ? Ans. 21. 5. If 3 of a dozen cost $8, what will # of a dozen cost at the same rate 3 e Ans. $9. 6. What part of 4 is 33 AnS. #. Analytic Solution—Here, by the terms of the question, we have 3 to divide or measure by 4” and by the exercise of our reason, we proceed thus: Since 3 is equal to 1, 3 times, it is equal to 42 + of 3 times, which is #. Or thus: Since 1 is # of 4, 3 is 3 times #, which is #. 7. What part of 5 is #3 AnS. #. Analytic Solution.—Since # is equal to 1, 3 of a time, it is equal to 5 the # part of # of a time, which is ſº. 8. What part of # is 7 ? Ans. 83. Analytic Solution.—Since 7 is equal to 1, 7 times, it is equal to ł, 5 times 7 which is 35, and to # instead of +, + part of 35, which is 8%. 9. What part of ; is ; ? Ans. 13%. Analytic Solution.—Since # is equal to 1, § of a time, it is equal to 4, 8 times ; which is º. and to § instead of , ; part of 49, which is #9, or 14%. * 10. What part of g is #3 Ans. #. 12. What part of 5 is # of 2? Ans. #. 11. What part of 34 is 24% Ans. Hºr. 13. What part of # is # of #3 Ans. #. 14. 9 is ; of what number ? Ans. 72. Analytic Solution.—Since 9 is ; of a number, 3 or the whole number is 8 times 9, or 72. 15. 13 is + of what number 7 Ans, 91. 16. 21% is # of what number 3 Ans. 1064. 17. § is # of what number ? AnS. #. 18. 24 is # of how many times 3? Ans. 10. Analytic Solution.—Since 24 is # of the number, # is + part of 24 which is 6, and # or the whole number is 5 times 6, which is 30; and as 30 is equal to 3, 10 times, therefore, 24 is # of 10 times 3. 19. 32 is 4 of how many times 8? Ans. 7. 20. 28 is is of how many times 12? Ans. 5. 2O2 SouLE's PHILOSOPHIC PRACTICAL MATHEMATICS. X- 21. # of 48 is # of what number? Ans. 54. Analytic Solution.—Since 48 is the whole of a number, 4 of the number is + part of 48, which is 12, and # is 3 times 12, which is 36; and since 36 is # of an unknown number, # of it is # of 36, which is 18, and 3 or the whole number is 3 times 18, which is 54. 22. § of 63 is ºr of what number 3 Ans. 154. 23. § of 3 of 64 is ºr of what number? Ans. 104. 24. # of # of 42 is # of what number ? Ans. 7%. 25. # of 32 is # of 4 times what number? * Ans. 9. Analytic Solution.—Since 32 is the whole of a number, 3 of the number is + part of 32, which is 8, and # is 3 times 8, which is 24; and since 24 is # of 4 times an unknown number, 3 of 4 times the number is # of 24, which is 12, and 3 or the whole of 4 times the number, is 3 times 12, which is 36; and since 36 is 4 times the number, 4 of 36, which is 9, is the required number. 26. # of 40 is # of 7 times what number 3 Ans. 6. 27. # of 56 is # of 6 times what number 7 Ans. 12. 28. § of ; of 66 is 33 of 3 times what number? Ans. 3. 29. What is $ and # of a $, of 3 of 15? Ans. 5. MISCELLANEOUS PROBLEMS IN DIVISION OF FEACTIONS. 397. 1. Bought 4 yards for $144. What was the cost per yard 7 Ans. $33. 2. Sold 84 pounds for $1.87. What was the price per pound? Ans. 22 cents. 3. Paid 37; cents for 64 yards of calico. What was the price per yard? Ans. 6 cents. 4. At $13 per gallon, how many gallons can be bought for $148.4% Ans. 108 gallons. 5. Divide # by 2. Ans. #. 13. Divide 21 by #r. Ans. 33. 6. Divide ºr by 3. Ans. }. 14. Divide 105 by #. Ans. 119. 7. Divide +} by 5. Ans. #. 15. Divide +} by #. Ans. 4. 8. Divide # by 5. Ans. #. 16. Divide # by #. Ans. 12. 9. Divide 7+ by 9. Ans. #. 17. Divide 24 by 3. AnS. 3. 10. Divide 2 by # Ans. 23. 18. Divide ## by ++. Ans. 2*. 11. Divide 3 by ºr. Ans. 7. 19. Divide 34 by 24, Ans. 1%; 12. Divide 5 by +4. Ans. 5+. 20. Divide # of $4 by #. AnS. #. 21. If one pound of tea costs # of a dollar, how many pounds can be bought for $25% Ans. 30 lbs. 22. 6 barrels of flour Were divided among some poor families in such a manner that each received # of a barrel. How many families were there? Ans, 9 families. OPERATION INDICATED. 1 = family, 6 i. Or, 2 || 3 Write the reason. •= ± -º-º: 9 families. 9 families, Ans. DIVISION OF FRACTIONS. 2O3 23. If a boy can earn ºr of a dollar in one day, how many days will it take him to earn $21? Ans. 33 days. 24. Henry walked 25 miles, which was # of the distance Robert walked. Płow many miles did Robert walk? Ans. 30 miles. 25. At the battle of Germantown, the British lost about 600 men; this was # of the number lost by the Americans; and the number lost by the Americans was # of the number they received as re-enforcements just before the battle. How many men did the Americans lose, and how many did they receive as re-enforce- ments? Ans. 1000 men lost, 2500 re-enforcements. 26. A man had his store insured for $9000, which was # of ºr of its value. What was the store Worth 7 Ans. $12375. 27. Sulphur will fuse at 2320 Fahrenheit, which is 7+ times the temperature required to melt ice. At what temperature will ice melt 3 Ans. 32°. OPERATION INDICATED. 2329 -- 7# = 329. 28. A quantity of mercury weighed 320624 lbs., which is 134 times the weight of an equal bulk of water. What would an equal bulk of water weigh 3 Ans. 2375 lbs. 29. A pound of water at 2120 F. was mixed with a pound of powdered ice at 32O. What was the temperature of the mixture? Ans. 1229. OPERATION. 212 -- 32 ## = 122°, Ans. NOTE.-A body in cooling 19 gives out just as much heat as it takes to heat it 1°. In the above mixture one pound gains exactly the temperature that the other loses. * This will not be the case, however, when substances of different natures are mixed. 30. When the air was at the freezing point, a cannon 27613% feet distant from New Orleans was discharged. 25% seconds elapsed after the discharge before the sound reached New Orleans. How many feet per second did the Sound travel ? Ans. 1090 feet. 31. Divide 287; by 5. Ans. 57+}. ion without the Explanation.—We first divide the 287 by the process of org. Line short division and obtain a quotient of 57, and a remain- g der of 2; this remainder we reduce to a fraction whose 5)287 denominator is the same as that of the fraction to be )287; divided, add it to this fraction, and then divide the sum * -º by 5 and annex the result to the quotient 57. Thus 2 = } 57%, Ans. + # = **, and * +5 = ##. 32. Divide 1471 fºr by 9. Ans. 163+. 33. Divide 10443 by 12. Ans. 87 ſº. 34. E. T. Churchill divided 14+; dozen apples among 3 boys and 2 girls; he gave to each girl twice as many as to each boy. How many did each boy and each girl receive? Ans. 24; doz. each boy, 4% doz. each girl. (Operation on other side). 2O4. SouLE's PHILOSOPHIC PRACTICAL MATHEMATICS. OPERATION INDICATED, 3 boys, each receives 1 apple, which makes 3 apples. 2 girls, each receives 2 apples, which makes 4 apples. 3 + 4 = 7, the sum of the proportion of the apples due the 3 boys and 2 girls. Statement to obtain the amount Statement to obtain the amount due each boy. due each girl. IDOZ. DOZ. 12 || 175 12 || 175 7 || 3 7 || 4 3 2 35. Divide 1 by #. Ans. 5. 36. Divide # by 1. Ans. #. 37. If 44 pounds of coffee cost 90 cents, what will 223 pounds cost? Ans. $4.55. 38. R. E. L. Flemming owns 3 of the capital stock of a factory valued at $24000; he gives # of # to educational societies, and the remainder he divides equally among his four children. How much does he give to educational Societies and how much does each child receive? Ans. $1500 to educational societies; $1875 each child receives. 39. S. J. Weis has 65+ yards of cloth, 2 yards wide. How many yards of lining # of a yard wide will be required to line it? Ans. 1963 yards. OPERATION INDICATED. (654 × 2) -- #. 40. Divide 18 oranges between A. and B. so that A. will have 4 more than B. What number will each have 3 Ans. A. 10; B. 8. OPERATION. Statement showing Statement showing what B. rec'd, what A. rec'd. B. receives 1. 18 18 A. receives 14 9 || 4 9 || 4 ſº-º- * I ammº 4 5 A. and B. receive 2#. 8 tºmºg ºmºmº 10 41. Divide 18 oranges between A. and B. so that A. will have # less than B. What number will each have # Ans. A. 7#; B. 10%. 42. A., B., and C. are to receive $26 in proportion to 3, #, and #. What will each receive 3 Ans. A. $12; B. $8; C. $6. 43. Frank can work 100 problems in 4 hours, and Lillie can work the same problems in 5 hours. How many hours will it require for both to work the prob- lems? Ans. 2% hours. SOLUTION.—Since Frank can work the problems in 4 hours, in 1 hour he can work # of them; and since Lillie can work the problems in 5 hours, in 1 hour she can work # of them. Hence, ††: = # of the problems, worked by both in 1 hour; and since ºr of the work required 1 hour, 3% of the work will require # of an hour, and #3, or the whole work, will require 20 times as many hours, or * hours, which is 23 hours. º * DIVISION OF FRACTIONS. 2O5 44. A. and B. can do a piece of work in 14 days, A. can do # as much as B. How many days will it take each to do it, working alone? Ans. ; days fºr B. 23 “ “ A. OPERATION INDICATED. 1 equals the work done by B. # “ {{ {{ ** A. 1#, or #, equals the work done by B. and A. Hence, B. does # of the work in 1 day, and A. does # of the work in 1 day. 14 ds. -- # = 24; days, for B. to do the work alone. 14 ds. -- # = 323 days, for A. to do the work alone. .45. Three persons, A., B. and C., do a piece of work; A. and B. together do # of it, and B. and C. do # of it. What part of the work is done by B.? Ans. ##. SoLUTION.—As A. and B. do of it, it is clear that C. does the remaining #; and as B. and C. do ºr of it, it is clear that A. does the remaining Yºr. Then, as A. does ºr, and C. §, they, together, do nºr-H # = ##; and if A. and C. do #5 of a piece of work done by A., B., and C., it is clear that B. does the difference between #3 and ##, which is...}}. 46. If 6 oranges and 7 lemons cost 33%, and 12 oranges and 10 lemons cost 54%, what was the cost of 1 orange and 1 lemon each? Ans. 1 Orange, 22. 1 lemon, 32. OPERATION. 6 oranges and 7 lemons cost 33%. 12 {{ 10 4% 542. 12 {{ 14 44 66 g. 12 “ 10 44 54%. 4 & 12. 122 + 4 = 3%, cost of 1 lemon. 32 x 7 (lemons) = 21%, cost of 7 lemons. 332 — 21% = 12¢, cost of 6 oranges. 12% -- 6 = 22, cost of 1 Orange. Reason.-Since 6 oranges and 7 lemons cost 33 cents, twice as many, or 12, oranges and 14 lemons will cost twice as much which is 66 cents; and since, by the second condition of the prob- lem, 12 oranges and 10 lemons cost 54 cents, the difference between 66 cents and 54 cents, which is 12 cents, will be the cost of the difference between (12 oranges and 14 lemons) and (12 oranges and 10 lemons) which is 4 lemons. And since 4 lemons cost 12 cents, one lemon will cost the fourth part Which is 3 cents. And since 6 oranges and 7 lemons cost 33 cents, by subtracting the cost of 7 lemons which is 21 cents, we have I2 cents, the cost of 6 oranges. And since 6 oranges cost 12 cents, one orange will cost the sixth part which is 2 cents. 47. A miller invested $54 in grain of which #was barley at 624% per bushel; # was wheat at $1.874 per bu.; and the balance oats (2 374% per bu. How many bushels of grain did the miller buy 3 Ans. 40 bus. OPERATION. Bushels. BU. * @ 13°C. = **c. = .18% proportionate cost of the barley. | 1 # (2) ###c.= *#5c.— 1.12% & 4 & 4 wheat. 1.35 54.00 To (a) ºc. = **c.— .03% & 4 4 & oats. | 40 bus. purchased, Ans. $1.35 proportionate cost of 1 bu. of grain. 48. A. and B. can do a piece of work in 10 days; A. alone, can do it in 15 days. How many days will it take B. to do it? Ans. 30 days. Wiscellaneous Problems, =N INVOLVING THE PRINCIPLES OF ADDITION, SUBTRACTION, MULTIPLICATION, AND DIVISION OF FRACTIONS. P-Jºyr-w 398. Find the difference between # and #; # and #; ; and #; 3% and 2%; 4% and # of 3} + Ans. To last, 3. 2. Find the sum of # of ºr and # of #r. Ans. #. 3. To the quotient of 23 divided by 5}, add the quotient of 33 divided by ++. Ans. 7#. 4. A number was divided by 3, and gave a quotient of 20. What was the number 3 Ans. 15. 5. What number is that, which being multiplied by ºr, gives as a product ## Ans. #. OPERATION INDICATED. # -- + = #, Ans. 6. What number is that, from which, if you take # of itself, the remainder Will be 12% & Ans. 30. OPERATION INDICATED. 1 = } – } = }; if # = 12, # = 6, and # equals 30. # e 7. What number is that, to which, if you add # of itself, the sum will be 40? Ans. 25. OPERATION INDICATED. 1 = } + # = #; if # = 40, # = 5, and # = 25. 8. A. owns # of a store which is worth $25000, and sells # of his share. What part does he still own, and what is it worth ? Ans. A. owns ºr, worth $2500. 9. Smith owns ºr of a cotton mill and sells #5 of his share to Jones for $33000. What is the mill worth at that rate? Ans. $242000. 10. John has 5 cents, and James # of 8 cents. What part of James' money is John’s 7 AnS. #. OPERATION INDICATED. # of 82 = 22 and # = 62; then 5 -- 6 = #, Ans. 11. The sum of four fractions is 1}. Three of the fractions are #, #, and #. What is the fourth” Ans. *r. 12. What number is that, to which if # of # of 14 be added, the sum will be 1; ? Ans. 1. 13. Two boys bought a bushel of oranges, one paying 24 dollars and the other 43 dollars. What part of it should each have 3 Ans. First, #; second, #. (206) jºr MISCELLANEOUS PROBLEMS IN FRACTIONS. 207 14. A farmer sold # of his mules on Monday; on Tuesday he bought # as many as he sold, and then had 40. How many mules had he at first 3 Ans. 56 mules. OPERATION INDICATED. # from # = #. # o # = 0; # # ... } + # = 8 fi 7 * 7 , and # ; # 15. A planter gave 50 bales of cotton at $50% per bale for flour at $7.4 per barrel. How many barrels of flour did he receive 3 Ans. 334 bbls. 16. A merchant bought 20 barrels of flour at $6; per barrel. Having sold 5 barrels at $74 a barrel, and 6 barrels at $83 per barrel, at what price per barrel must he sell the remainder to gain $10 on the whole 3 Ans. $64. 17. If it requires 34 yards of cloth to make a coat, 24 to make a pair of pantaloons, and 13 to make a vest, how many suits may be made out of 1574 yards, allowing # of a yard waste in cutting each suit 3 Ans. 21. 18. A. W. McLellan gave #, #, and # of his money to different benevolent institutions, and had $1000 left. How much had he at first 3 Ans. $20000. 19. C. Manson owning ºr of a rice mill, sold # of his share for $8800. What was the value of the mill? Ans. $24200. 20. A book-keeper worked 914 days, and after paying # of # of his earnings for board and washing, had $438 remaining. How much money did he receive in all, and how much per day ? Ans. $730 in all, $8 per day. 21. Prophet can do a piece of work in 6 days, and Fisher can do the same work in 8 days. How many days will it take both together to do the work 3 Ans. 3.} days. OPERATION INDICATED. # + 4 = #P. 1 day -- # = 3} days. 22. Myers, Levy, and Hoffman can do a piece of work in 10 days; Myers and Levy can do it in 15 days. In what time can Hoffman do it, working alone? Ans. 30 days. 23. A man died and left his wife $14400, which was # of # of his estate. At her death she left # of her share to her daughter. How much money did the daughter receive, and what part was it of her father's estate 3 * Aris. $12000, #4 of her father's estate. 24. A man engaging in trade lost # of the money he invested; he then gained $1000, when he had $3800. What did he have at first, and what was his loss % Ans. $4900 at first, $2100 loss. oppEATION INDICATED. $3800–$1000 = $2800. $2800 -- # = $4900. # of $4900 = $2100. 208 SouLE's PHILOSOPHIC PRACTICAL MATHEMATICs. XF 25. A mule and a dray cost $240; the mule cost 13 times as much as the dray. What did each cost? Ans. $90 dray, $150 mule. OPERATION INDICATED. 1 (cost of dray)+ 1} (cost of mule)=2# = $240. $ 240 240 8 || 3 8 || 3 3 || 5 | $90 cost of dray. | $150 cost of mule. 26. How many bushels of apples at $4 a bushel, will pay for ºr of a barrel of Oranges at $63 a barrel? Ans. 73 bushels. 27. Sweeney paid # of his year's wages for board, 3 of the remainder for clothes, and had $80 left. How many dollars did he receive for labor? Ans. $560. 28. Forcheimer lost 3 of his fish-line, and then added 254 feet, when it was just # of its original length. What was its original length'? Ans. 204 feet. OPERATION INDICATED. 1 = #–# = #. #–3 = } = 254 feet. 254 × 8 = 204 feet, Ans. 29. Purcell, having a certain number of cents, gave one-half of them and half a cent over to one beggar; one-half of what he had remaining and half a cent over to a second beggar; and to a third, one-half of what he then had and half a cent over, and had left 3 cents. How many cents had he at first 3 Ans. 31 cents. OPERATION INDICATED. (3 + 42 over) × 2 = 72, had before making 3rd gift. ( 7 -- #2 over) × 2 = 15%, {{ << 2nd £6 (15 + #2 over) × 2 = 312, {{ 4% 1St 4 30. A housekeeper's weekly purchases from her butcher were as follows: on Monday, 73 pounds of meat; on Tuesday, 64 pounds of meat and 1 duck; on Wednesday, 84 pounds of meat; on Thursday, 63 pounds of meat; on Friday, 3 pounds of meat and 2 fish; and on Saturday, 93 pounds of meat, 1 chicken and 2 dozen crabs. The meat was 12#2 per pound; the fish were 75% each; the chicken $1.40; the duck 802, and the crabs 40% per dozen. What sum of money was due the butcher at the close of the week? Ans. $9.79%. 31. John lives with his parents, but works for Mr. Smith who pays him $210 per year. His parents board him, but he has his clothes to buy. He spends # of his wages for cigars, # of the remainder for theater tickets, # of the remainder for wine, and 4 of what he then has for novels. How much has he remaining at the end of the year to pay for his clothes? Ans. $30. 32. Joseph worked on the same conditions as John, in the problem above. He gave +4 of his wages to the cause of charity, ºr of the remainder for useful books, # of the remainder for evening tuition, paid $100 for clothes, and deposited the balance in the bank. How many dollars did he put in the bank? Ans. $50. * MISCELLANEOUS PROBLEMS IN FRACTIONS. 2O9 33. A. G. Niehues and R. G. Jones have $1899, and Jones has 3} times as much as Niehues. How much has each? Ans. Niehues $422, and Jones $1477. NOTE.-For the operation, see problem 25. 34. J. C. Beals can solve 25 problems in 50 minutes and H. H. Barlow can solve them in 30 minutes. In what time can both solve them? Ans. 18; minutes. 35. U. Burke purchased 200 barrels of flour for $1450, and sold # of it at a profit of $4 per barrel, and the remainder at $74% per barrel. How much did he gain } Ans. $67.50. 36. What is the numerical value of 44–3 2% + 1} AnS. 114. 37. M. Ernst bought 3841; pounds of cotton at 7; pence per pound. What did it cost 3 Ans. £124, 0s, 113d. 38. R. J. Kennedy has 3 dozen oranges which he wishes to divide between Miss Katie and Miss Tillie, so that Miss Katie will receive + more than Miss Tillie. How many will each receive? Ans. Miss K. 20, and Miss T. 16. OPERATION INDICATED 1 = Miss Tillie's proportional share. 36 36 1} = Miss Katie's proportional share. 4 9 || 4 # = sº ſº tº 4 || 5 2} = the sum of the proportional shares. 16 sº I -º-º-º-º: 20 39. A tree 110 feet high, had # of it broken off in a storm. How much of it was left standing ? Ans. 44 feet. 40. What cost 223 pounds of coffee at 21% per pound 7 Ans. $4.94+3. 41. If 18; yards cost $3.37}, what will 34 yards cost 3 Ans. 63 cents. 42. Miss Cora has $600 of which she wishes to give to A. 3, B. 4, C. §, and D. #. How much will each receive? Ans. A. $200, B. $150, C. $120, and D. $100. 43. Miss Mamie has $600 which she wishes to give to A., B., C., and D. in the proportion of #, +, +, and #. How much will each receive? Ans. A. $210+}, B. $157+%, C. $1264%, and D. $105*. OPERATION INDICATED, 600 # + 4 + 4 + 4 = ##. 19 || 20 = $2104% A's share. 3 44. If a yard and a half cost a dollar and a half, what will twelve and a half yards cost 3 Ans. $12#. 45. What part of 6 cents is # of 5 cents? Ans. #. OPERATION INDICATED. (5 × #) + 6. Write the reason. 46. A planter remits his factor $500 to invest in rice and coffee, in equal sums. He pays 94% per pound for rice, and 233% per pound for coffee. How many pounds. of each did he purchase ? Ans. 27023; lbs. rice. 1069+ºf lbs. coffee. 47. If 3 of 6 be 3, what will # of 20 be? Ans. 73. 48. If 3 is the third of 6, what will the fourth of 20 be? Ans. 34. 2 IO SOULE's PHILOSOPHIC PRACTICAL MATHEMATICS. jºr 49. E. L. Hunt owned a quantity of rice, of which he sold # for $99.60. What is § of the remainder worth at the same rate } Ans. $16.60. 50. M. Landman paid $60 for # of an acre of land. What is the value of § of an acre ? Ans. $50. 51. J. J. Hauler bought 937852; pounds of cotton at 14% per pound. What was the cost 3 …” Ans. $135695.53%. 52. E. T. Berwick invested # of his money in sugar, in rice, 3 in coffee, and deposited in bank $2645. How much money had he at first? Ans. $63480. 53. L. Meyer spends # of his time in study, # in labor, $ in rest and recrea- tion, and the remainder in sleep. How many of the 24 hours of a day does he sleep 3 Ans. 7 hours. 54. An industrious young lady spends # of her time in the performance of household affairs, # in reading good books, ſº in physical exercise in the open air and Sunlight, $ in the practice of music, singing and parlor amusements, or Social intercourse, 2 hours per day in eating, and the remainder of the day in sleeping. How many hours per day does she devote to each 3 Ans. 6 hours to household affairs; 4 hours to reading; 2 hours to exercise; 3 hours to music, etc.; 2 hours to eating, and 7 hours to sleeping. 55. A loafer spends 4 hours per day sauntering on street corners, 3 hours Smoking and drinking, # of the day in sleep, + of the day in drunkenness, +'s in eating, Hº in quarreling, and the remainder of the day in gaming. How many hours does he spend in gaming 3 Ans. 3 hours. 56. A fashionable young lady spends # of her time in dressing, painting, and making her toilet, $ in reading novels and papers of senseless fiction, % in making calls and gossiping, ºr in street promenading, ºr in criticising industrious young men and speculating upon the qualities and fortune of an anticipated husband, ºr in making remarks derogatory to the character of those who labor, while her own mother is perhaps cooking or washing, ++, in entertaining young men, and the remainder in eating and sleeping. How many hours does she devote to useful ser- vice, and how many to eating and sleeping 3 Ans. 0 hours to useful service; 8 hours to eating and sleeping. 57. A man willed # of his property to his wife, # of the remainder to his daughter, and the remainder to his son; the difference between his wife's and daughter's share was $8000. How much did he give his son? Ans. $4800. OPERATION INDICATED. # = wife's interest; +* = daughter's interest; # = son’s interest; +”; – # = # = $8000; then $8000 + + = $25600 the whole estate; # of which is $4800, Ans. 58. R. W. Tyler owned a # interest in a factory, and sold to C. Modinger 4 of his interest for $15000. What interest does he still own, and how much is it worth at the rate received for the part sold 2 Ans. He still owns 3, worth $15000. 59. A. and B. own a certain number of oranges; A. owns # and B. #. C. pays A. 50 cents for # of his oranges, and B. 40 cents for # of his oranges. What part of the whole number do A., B., and C. respectively own, and what is the remainder of A. and B. Worth at the same rate? Ans. A. owns #. A’s share is worth $1.00. B. owns #3. B's share is worth $1.60. C. owns ºs. Yºr MISCELLANEOUS PROBLEMS IN FRACTIONS. 2 II 60. J. Cassidy owned 4 of the steamer R. E. Lee. He sold to G. Buesing # interest in the steamer for $20000; and to J. C. Beals + of his remaining interest at the same rate. What did he receive for the last sale, and what is his remaining interest in the boat? Ans. He received $30000; # remaining interest. 61. N. Puech and A. Palacio bought, on joint account, each 4, the New Orleans Cotton Factory. N. Puech sold # of his interest to R. Krone, and subsequently 4 of his remaining interest to A. Palacio, who subsequently sold # of ; of his whole' interest to R. Lynd, for $7500. What is the factory worth at the same rate, and what is each owner's interest ? Ans. $32000 value of factory; Puech owns #; Krone, #; Palacio, #; and Lynd, #. 62. A. has $100; he gives # of it for 5 barrels of flour, and # of the remainder for 4 barrels of potatoes, and with the remainder he buys coffee at 20g per pound. How much coffee did he buy? Ans. 200 lbs. Coffee. 63. A person owns ºr of a stock of goods; he sells # of his share for $5000, and + of the remainder for $5000, and then the balance of his interest for $15000. What part did he sell the last time, and what would the whole stock be worth at that rate % Ans. # sold last. $27,777; value of stock. OPERATIONS IN MIXED NUMBERS, COMPLEX, COMPOUND, AND IMPROPER FRACTIONS, INVOLVING + , —, x AND + SIGNS, AND ALSO THE PARENTHESIS OR VINCULUM. NoTE.—For directions how to reduce or simplify fractional expressions indicated by the + , – , X and — signs, and by parentheses or vinculums, see Article 230, page 134. - 7 6+ 64. Multiply ſº by "#. py; by si OPERATION INDICATED. 3 22 11 || 2 3 22 4 || 25 3 || 4 4 || 25 OT 11 || 2 10 || 3 8 || 15 10 || 3 sº º ºs * º sº * 1#. 1; 24, Ans. Ans. 2; s.sa., 7% 6# 65. Divide 5; by 3# OPERATION INDICATED, 3 22 11 || 2 3 22 4 || 25 3 || 4 25 || 4 OI’ 11 || 2 10 || 3 15 | 8 3 || 10 # = ſº ſº-º-º-º: * = gº º gºmº, sº-º 1; 1; #, Ans. Ans. ## 2 I 2 soul E's PHILOSOPHIC PRACTICAL MATHEMATICS. 66. Multiply # by # Ans. #. 67. Divide # by # Ans. 23#. 68. Reduce. * Of # of 8% -- 7# to a simple fraction. Ans. #. 69. What is the result of 3 + 3 x 3 + # -- # -- #3 Ans. 24%. 70. Find the value of 2; x +} + 7# — 63. Ans. 1%. 71. § -- # -- # – # × #} equals what number ? Ans. 11%. 72. # — ; x # -- # = what number ? Ans. }; 73. What is the value of # + 3 — 3 × (## -- #3) -- #! Ans. # 74. Add + of 3, #, #, #, and 74—53. Ans. 44%. 75. What is the product of ; of 124 by § of 6%—#3 Ans. 34.3%. 76. What is the product of # of 13; +13 by # of (+}-#)? Ans. 4}}. 77. Divide the sum of 4, 3, #, and 3, by the difference between 3; and # of 3. w Ans. 31's. 78. Multiply [60 divided by (# of 74 of 3)] by # of 7. Ans. 12. NOTE.-When there are double parentheses, perform the operation indicatéd by the interior Vºl. first. See page 134, Article 230, for full instructions regarding Parentheses and 1IlCl11UIIITS, 79 Simplif o # + # — # x + i + +, +- # 1 6. º plify the fraction + 1. 2.4 - 2 1 7 Ans. 8+}. # — # X ## -- # – ## 9 63 7 6 3 5 — —4 4 2. – 2. 2. – 5.2 - 5 — 1 1 1 – 81. = #; # -- # = #; * —# = ##; # —## = } *** -- # = 84%, Ans e e 8 .* 9 80. Divide (ś. of #-) by (#- of #) Ans. } 6+ T +} 4} 81. L. Kaiser bought # of ; of 284 barrels of apples, and sold to S. L. Crawford # of 9 barrels for $204, which was $1.50 more than the same cost. What was the cost of the whole, and how many barrels has he unsold 3 Ans. $394, cost; 7; barrels unsold. 82. What is the difference between # of # of eleven thousand eleven hundred and eleven, and a third, plus one half a third of two thirds of 150$ 2 Ans. 7249-##. gºats COIII]]|| Divis]ſ [f Fra(ti)|S. *= tº 399. The Greatest Common Divisor of two or more fractions is the greatest fraction that will divide each of them, without a remainder. 400. One fraction is divisible by another when the numerator of the divisor is a factor of the numerator of the dividend, and the denominator of the divisor is a multiple of the denominator of the dividend. Thus +} is divisible by #; for +} = }}; and #} -- s = 4. 401. The Greatest Common Divisor of several fractions, is a fraction whose numerator is the greatest common divisor of all the numerators, and whose denominator is the least common multiple of all the denominators. Thus the G. C. D. of # and ## is ###. 1. What is the greatest common divisor of 3, 4}, and #1 Ans. Töö. Operation to find the G. C. D. Operation to find the L. C. M. of the numerators : of the denominators : 3 || 3 9 18 2 |4 2 25 l 3 6 2 l 25 3 = G. C. D. 2 × 2 × 25 = 100 L. C. M. GENERAL DIRECTION FOR FINDING THE GREATEST COMMON DIVISOR OF FERACTIONS. 402. From the foregoing elucidations, we derive the following general direc- tion for finding the greatest common divisor of fractions: Find the G. C. D. of the numerators of all the fractions and write it over the L. C. M. of their denominators. NotE.—All the fractions must be in their simplest form before commencing the operation. –2. 2. What is the greatest common divisor of #, #, #, and #3 Ans. Tåg. 3. What is the greatest common divisor of 8% and #3 Ans. Ps. 4. What is the greatest common divisor of 5, 3}, 63, and #53 AnS. #'s. 5. What is the greatest common divisor of #, #, #, #, and #3 AnS. gº. 6. A grocer has three kinds of molasses, which he wishes to ship in the least number of full kegs. Of the 1st quality, he has 115% gallons; of the 2d quality, 1284 gallons; and of the 3d quality, 134% gallons. How many gallons will there be in each keg, and how many kegs will be required? Ans. 64% gals. in each keg ; 59 kegs required. (213) #as Umm Mlik ºf Fridiºs -------------------------------------N Aºa —-ºº- Æa -* vºw ~gº w 403. The Least Common Multiple of two or more fractions is the least number that each fraction will divide without a remainder. NotE 1.-The G. C. D. of several fractions is always a fraction; but the L. C. M. of several fractions may be a fraction or a whole number. 404. A fraction is a multiple of a given fraction when its numerator is a multiple of the given numerator and its denominator is a divisor of the given denominator. Thus # is a multiple of #; for 4 is a multiple of 2, and 5 is a divisor of 15 Eſence # -- # = 6; or thus # = }#; and # -- # = 6. 405. A fraction is a common multiple of two or more given fractions when its numerator is a common multiple of the numerators of the given fractions, and its denominator is a common divisor of the denominators of the given fractions. 406. A fraction is the least common multiple of two or more given fractions. when its numerator is the least common multiple of the given numerators, and its denominator is the greatest common divisor of the given denominators. 1. What is the L. C. M. of 3, #2, and #3 Ans. 63. Operation to find the L. C. M. Operation to find the G. C. D. of the numerators : of the denominators : 2 2 5 4 3 || 3 12 15 1. 5 2 1 4 5 L. C. M. is, 2 × 5 × 2 = 20. G. C. D. is 3. 2 Hence the L. C. M. of the fractions is ºº = 63. GENERAL DIRECTION FOR FINDING THE LEAST COMMON MULTIPLE OF FEACTIONS. 407. From the foregoing elucidations, we derive the following general direction for finding the least common multiple of fractions: Find the least common multiple of the numerators and the greatest common divisor of the denominators, and then divide the L. C. M. of the numerators by the G. C. D. of the denominators. NotE.—The fractions must be in their simplest form before commencing the operation. (214) Yºr LEAST COMMON MULTIPLE OF FRACTIONS. 2 I 5 2. What is the L. C. M. of #, #, and #g Ans. 4}. 3. What is the L. C. M. of 3, 3, #, #, and #3 Ans. 60. 4. What is the L. C. M. of 54, 74, #4, and # Ans. 44. 5. There is an island 15 miles in circuit, around which A. can travel in # of a day, B. in § of a day, and a horse car in ºr of a day. Suppose all start together from the same point to travel around it in the same direction, how long must they travel before coming together again at the place of departure, and how many miles will each have traveled ? Ans. 104 days; A., 210 miles; B., 180 miles; Horse Car, 525 miles. PARTIAL OPERATION. 2) 4 8 10 Denominators. 3) 3 7 3. Numerators. *-i-º-º-º-mºmºmºmº - 2 the Greatest Common Divisor. 1 7 1 3 × 7 = 21 -- 2 = 104 days before they all meet; then the following propor- tional statements give the miles traveled by each : A. B. H. Car. or thus, 15 -- # x 104 = 210 miles 15 15 15 traveled by A. 3 || 4 7 || 8 3 || 10 15.-- g × 10% = 180 miles 2 21 2 21 2 || 21 traveled by B. tº- 15 –- ºr × 10% = 525 miles 525 m., Ans, traveled by Horse Car. 210 m., Ans. 180 m., Ans. 6. What is the Smallest sum of money for which I could purchase a number of bushels of oats, at $º a bushel; a number of bushels of corn, at $3 a bushel; a number of bushels of rye, at $14 a bushel; or a number of bushels of wheat, at $24 a bushel; and how many bushels of each could I purchase for that sum ? Ans. $224; 72 bushels of oats; 36 bushels of corn; 15 bushels of rye; 10 bushels of wheat. PARTIAL OPERATION. 5) 5 5 3 9 Numerators. 2) 16 8 2 4 Denominators. 3) 1 1 3 9 *=º 1 1 1 3 2 the Greatest Common Divisor. 5 × 3 × 3 = 45 the least common multiple of the numerators, which divided by 2, the greatest common divisor of the denominators, gives $224, the smallest sum ; then 22; divided by $4%, $3, $#, and $# gives respectively the number of bushels of each article represented by the different prices. 7. In December 1875, the Earth, Mars, and Saturn, were in conjunction. The period of the revolution of Mars is 13 years, and of Saturn 294 years. When will they be again in conjunction at the same point of their orbits? Ans. In 1003 years. Alecimal Fractions. —sº a Aſſº– -ºr-ºr-º- 408. A Decimal Fraction is one or more of the equal parts of a unit, which is divided into tenths, hundredths, thousandths, etc., according to the decimal scale; hence the denominator of decimal fractions is always 10 or some power of 10. The word decimal is derived from the Latin word decem, which means ten. 409. The Decimal Point (. ) is used to distinguish decimals from whole numbers. When there are mixed numbers, it also separates the whole numbers from the decimals. The following are decimal fractions: #3, Tºo, #3, and Tážg. They are here written as common fractions, but generally the denominator of decimal fractions is omitted and the value is indicated by Writing the decimal point before the numerator. To write the above fractions according to the decimal notation, they would be written thus: fºr decimally expressed is .7 *śr decimally expressed is .137 ºr decimally expressed is .15 T###5 decimally expressed is .0423 410. Notation of Decimals. Whenever decimal fractions are expressed decimally, the numerator must have as many decimal places as there are naughts in the denominator. Thus, #6 = 4; Tº = .16; # = .1456. When the number of naughts in the denominator is greater than the number of figures in the numer- ator, naughts must be prefixed to the numerator until the number of places is equal to the naughts in the denominator. Thus, rão = .04; tºo - .007; rºw = .00125; etc. 411. A Pure or Simple Decimal consists of a decimal fraction, decimally expressed or written. Thus .5, .42, .875, and .1256 are pure decimals, and are read respectively 5 tenths; 42 hundredths; 875 thousandths; and 1256 ten thousandths. 412. A Mixed Decimal consists of a whole number and a decimal. Thus, 24.5 and 41.25 are mixed decimals. They are read respectively, 24 and 5 tenths; 41 and 25 hundredths. NOTE.—A mixed decimal may be read as an improper fraction. Thus, 24.5 = 24%; = 2,4}. 413. A Complex Decimal consists of a decimal with a common fraction annexed. Thus, .15% and .005% are complex decimals. They are read respectively, 15; hundredths; 5; thousandths. 414. A Terminate Decimal is one which ends, as # = .125. And an interminate decimal is one that does not end, as # = .3333+. NOTE.-Decimal fractions were invented by Regiomontanus, in 1464; but they were not in general use until the latter part of the sixteenth century, when the first treatise on decimals was written by Stevinus, in 1585. •. (216) Yºr DECIMAL FRACTIONS. 217 f r CIRCULATING DECIMALS. 415. A Circulating Decimal is one in which a figure or set of figures con- stantly repeats itself. Thus, # = .3333+ , ºr = .181818--, + = .142857+. 416. A Repetend is the figure or set of figures which repeats itself, and it is expressed by placing a dot over the figure repeated, thus: # = .3333+ = 3. When the repetend consists of more than one figure the dot is placed over the first and last figure, thus: ºr = 181818+ = i8; and + = 142857+ = i+2857. 417. A Pure Circulating Decimal is one which contains only the repetend, as # = 6; } = .142857; # = 1. 418. A Mixed Circulating Decimal is one which contains other figures than the repetend, as # = .16; ### = .647. 419. A Simple Repetend contains one figure, as .6. 420. A Compound Repetend contains two or more figures, as i8 and .342. 421. Similar Repetends are those which begin and end at the same decimal place, as .536 and 427. 422. Dissimilar Repetends are those which begin and end at different decimal places, as .205 and 3765. 423. A Perfect Repetend is one which contains as many decimal places, less 1, as there are units in the denominator of the equivalent common fraction; thus, 4 = i+2857. 424. Conterminous Repetends are those which end at the same decimal place, as .50397 and 42618. } 425. Co-originous Repetends are those which begin at the same decimal place, as .5 and 124. Circulating decimals had their origin in reducing common fractions to deci- mals. The general law for the formation of repetends will be seen from the follow- ing: 1. * = .1111+ = i. 5. # = .4444+ = 4. 2. ºs = .01010+ = .0i. 6. # = .2727-- = .27. 3. * = .001001-- = .00i. 7. § = 135135+ = .iáš. 4. nºw = .000100014--.000i. 8. §§§ = .17281728--- iT28. 2 I 8 soul E's PHILOSOPHIC PRACTICAL MATHEMATICs, X- 426. Decimal fractions, like whole numbers, decrease toward the right and increase toward the left in a ten-fold ratio; and hence the prefixing of naughts between the decimal figures and the decimal point, or the removal of the decimal point towards the left diminishes their value ten-fold, or divides the decimal by ten for each order or place removed. Thus: .5 = #; .05 = Hitſ; .005 = Tºro ; etc. 427. The removal of the decimal point to the right, increases the value ten-fold or multiplies the decimal by ten for each place removed. Thus : .005 = Tºo : .05 = Tâû ; .5 = *; ; etc. 428. Annexing naughts to decimals does not change their value, because the significant figures are not thereby removed nearer to or farther from the decimal point. Thus: .5 = #; also .50 = }}; ; .500 = ***, all of which are equal. 429. Decimal orders are also called decimal places, each order being counted as one place. Thus, in .0043 there are four decimal places, although the 3 is of the fifth decimal place from unity, the base of the system. 430. The following table will illustrate more fully the relation of whole numbers and decimals, with their increasing and decreasing orders to the left and right of the decimal point: TABLE. WHOLE NUMBERS. DECIMALS. é tº CD # 5 . E & C 32 zº r: .# * gº tº F. --> := tº F: cº gº *] F. : ſº rº º • C C p F. s'E 3 # E *:::= < 3 ‘E tº 3 F = E E $2.5 ° E 2: . $ 35 ; E 2 = E 2 = 22 S- rº; tº P- := 3 £ l 2.3 te ºr T 2 sº E ºf = . ; ; 3 = 3 E Tº Gº SH 9 *- : SP º: 2, $2 & 23 SP E. F. Sº 3 & F : G 3.3 : E : º, $º ºt. # = } 5 : 3 E. × 2 = 2 & 5 # 5 3.5 ± 3: E # * = É # 5 É # E & E E 5 # F# E É E F = E = F E = P C F. E. F. F. H. E. E. E. 9 8 7 6 5 4 3 2 1 - 2 3 4 5 6 7 8 9 Orders of ascending scale. Orders of descending scale. This number is read 987 million 654 thousand 321, and 23 million 456 thousand 789 hundred millionths. 431. In order to clearly understand decimals, we must bear in mind that one is the basis of all numbers, integral and fractional, abstract and denominate, and that all mathematical operations have this fundamental principle for their origin, and every number is but a multiple, either ascending or descending of unity OT O706. 432. The names of the decimal orders are derived from the names of the orders of whole numbers. Thus the names of the orders in the ascending scale, are, after units, tens, hundreds, etc., and the orders in the descending scale, are, after units, tenths, hundredths, etc., the decimal orders being the reciprocal of the orders of whole numbers equally distant with themselves from the units. DECIMAL FRACTIONS. 2 I9 433. Numeration of Decimals. In reading decimal fractions, the entire decimal is regarded as reduced to units of the lowest order expressed, and the name of this order is given to the entire number of decimal units. Thus, .25 is read twenty-five hundredths. 434. Before reading a decimal, we must determine 1st. How many units are expressed. To do this, we numerate and read the significant figures of the decimal as in whole numbers. 2nd. We must determine the name of the lowest order in the decimal. To do this, we numerate the number decimally. Thus to read .001073, we commence at the 3 and numerate to the 1 thousand, and thus find that 1073 units are expressed; then we commence at the decimal point and numerate decimally to the 3, and thus find that millionths is the lowest order; we then read 1073 millionths. EXERCISES. 435. Read the following numbers: 1. 16.008 reads thus: Sixteen units and eight thousandths. 2. .94; reads thus: Ninety-four and three-eights hundredths. 3. 5067.4005 reads thus: 5067 units and 4005 ten-thousandths. 4. Write and read 197.8; 4.68907; .00073; 48.769146. 5. Write and read 2.491; 10.0101089167; 582.400410905. 6. Write and read 5841.291;; 8000.0000000217; 9876541.1000001. 436. Writing Decimals. In writing decimals, we write down the given number as if it were a whole number; then, to facilitate the operation, we numerate from right to left, beginning the numeration with tenths, and continue until we come to the required place or order, always writing 0's to fill the places not occupied by significant figures. Thus, to write 25 ten thousandths, we first write the 25; then we begin at the right and numerate thus, tenths, hundredths, thousandths, ten- thousandths; by this we find that four places are required and as there are but two figures in the number, we prefix two 0's and obtain the correct result .0025. 1. Write 104 hundred thousandths. Ans. .00104. Ea:planation.—According to the above directions, we write > the 104 and then commence on the right and numerate thus: OPERATION. - *> 00.104 Tenths, hundredths, thousandths, ten-thousandths, hundred- thousandths. This numeration shows that five places are required, and as we have but three, we therefore prefix two 0's. EXERCISES. 437. 1. Write 10101 hundred billionths. Ans. .00000010101. Write decimally, numerate, and read the following: 2. 31.4 millionths. 6. 1205 ten millionths. 10. Po. 3. 12 thousandths. 7. 897 hundred billionths. 11. ºr. 4. 107 billionths. 8. 1 Sextillionth. 12, Tºr. 5. 1 trillionth. 9. 21001 ten vigintillionths. 13 10423 te 100000 14. Taºğgo 16. 1000 dºwo 18. 1#. 15 748#. 100 d 1000000 || 17. Tºo 19. To Jºão 22O SOULE's PHILOSOPHIC PRACTICAL MATHEMATICS. Yºr t PRINCIPLES. 438. From the foregoing work we recapitulate the following principles: . 1. Decimals are governed by the same laws of notation as whole numbers; hefice the value of any decimal figure depends upon the place it occupies. 2. Each removal of the decimal point one place to the right, is equivalent to multiplying the decimal by 10. 3. Each removal of the decimal point one place to the left, is equivalent to dividing the decimal by 10. 4. Annexing or rejecting naughts at the right of any decimal does not change its value. <> <> REDUCTION OF DECIMALS. TO REDUCE DECIMAL FRACTIONS TO A COMMON DENOMINATOR. 439. 1. Reduce .7, .18, .2581, and .045, to a common denominator. OPERATION. .7000 Explanation.—To reduce decimals to a common denom- º: inator, we have but to annex a sufficient number of 0’s to .0450 give each decimal the same number of places. TO REDUCE A DECIMAL TO A COMMON FRACTION. . 440. 1. Reduce .25 to a common fraction. Ea:planation.—In all problems of this kind, we simply write º the decimal as a common fraction and then’ reduce it to its Töö = #, IlS. lowest terms. 2. Reduce .125 to a common fraction. OPERATION. ** = #, Ans. 3. Reduce .593 to a common fraction. FIRST OPERATION. 593 445. Explanation.—To reduce complex decimals — = — and 44%-- #49 = 4% = 4%. Ans. * simple fractions, we first write the decimal 100 oo’ 8 T-T 8 8 O 0 - 3 22 as a common fraction; then we reduce both 8 go. the numerator and the denominator to, the fractional unit of the denominator contained in the numerator term of the fraction, and thus obtain a complex fraction, which we reduce to a simple fraction. * REDUCTION OF DECIMALS. 22 I SECOND OPERATION. Explanation.—Here, when, reducing, the fraction to the 59; fractional unit of the denominator contained in the numera- — = ### = 4%. Ans for term of the fraction, we shorten the work by omitting 8 0 O - 3 27 e the denominator (8) in both terms of the complex fraction, 100 and writing the result as a simple fraction. By this process, we save the operation of division, the result of which is the cancelling of the denominator in both terms of the complex fraction. GENERAL DIRECTION FOR REDUCING DECIMAL FRACTIONS TO COMMON FRACTIONS. 441. From the foregoing elucidations, we derive the following general direc- tion for reducing decimal fractions to common fractions: Write the decimal as a fraction, then reduce the fraction to its lowest terms. Reduce the following decimals to common fractions: 4. .8, AnS. #. 10. .07, Ans. Tºo. 16. .484, Ans. #6. 5. .05, Ans. #5. 11. .005, Ans. #5. 17. .055#, Ans. #. 6. .25, Ans. 12. .1045, Ans. #6. 18. .008%, Ans. 7. .125, Ans. 13. .88, Ans. #. 19. .00054%; Ans. 8. .675, Ans. 14. .909, Ans. 20. .999, Ans. 9. .105, Ans. 15. .00025, Ans. 21. .4007 ºr, Ans. #####. TO REDUCE COMMON FRACTIONS TO EQUIVALENT DECIMALS. 442. 1. Reduce # to a decimal. OPERATION. Explanation.—To reduce common fractions to decimals, we an: nex naughts to the numerator and divide by the denominator, and 8) 3,000 then point off as many spaces for decimals as there were 0's annexed. When a remainder continues beyond four or six places, we discon- tinue dividing and write the sign + to the right of the last figure .375, Ans. obtained, which indicates that the quotient is not complete. The annexing of 0's to the numerator is equivalent to multiplying it by 10 for each naught annexed, consequently the quotient obtained is as many times 10 too great as there were 0's annexed; and hence the reason for pointing off as many places in the quotient as there were 0’s annexed to the numerator. f 2. Reduce # to an equivalent decimal. OPERATION. 7) 5.000000 .714285-H, Ans. 222 SOULE's PHILOSOPHIC PRACTICAL MATHEMATICS. 3. Reduce ### to an equivalent decimal. FIRST OPERATION. SECOND OPERATION. Explanation.—Here, in the first operation, we annex six 725) 3.000000(4137-- 725) 3. (.004137-1-, Ans. 0's and obtain but 4 figures in 2900 .004137-1-, Ans. 30 tenths. the quotient. Therefore, in *E=ºmºte * -º-; order to point off as many dec- 1000 300 hundredths. imal places as we annexed 0's, 725 . we prefix two 0's and thus ob- 3000 thousandths. tain the correct result. The 2750 2900 reason for this will appear clear 2175 if we consider each step of the 1000 ten thousandths. work as performed in the sec- 5750 725 ond operation. We are to di- vide or measure 3 by 725, and 2750 hundred thousandths. we first see that 3 is not equal 2175 to 725 any whole or unit num- ber of times; we, therefore, 5750 millionths. write the decimal point in the 5075 quotient, annex a 0 to the 3 units and thus reduce it to 30 675 remainder. tenths, which we also see is not equal to 725 any tenth times, and hence we write 0 in the tenths' place of the quotient; we then annex another 0 and thereby reduce the 30 tenths to 300 hundredths, which we see is not equal to 725 any hundredths times, and hence we write 0 in the hundredths' place of the quotient ; we then annex another 0 and thereby reduce the 300 hundredths to 3000 thousandths, which we see is equal to 725, 4 times, with a remainder. We have now obtained the first significant figure of the decimal, and we continue the division in the usual manner to the sixth decimal place and annex the + sign to indicate that there is still a remain- der. The sign — is sometimes used to indicate that the last figure is too great. Thus, # = .1666-1-; or, by abbreviating # = .167—. 4. Reduce 6; to a decimal. FIRST OPERATION. SECOND OPERATION. 6# = ** and * = 4) 27.00 63 = 6 and #; and * # = 4) 3.00 6.75, Ans. .75 + 6 = 6.75, Ans. GENERAL DIRECTION FOR REDUCING COMMON FRACTIONS TO EQUIVALENT DECIMALS. 443. From the foregoing elucidations, we derive the following general direc- tion for reducing common fractions to equivalent decimals: Annea, naughts to the numerator and divide by the denominator. Then point off, from the right of the quotient, as many decimal figures as there are naughts anneared. Reduce the following fractions to equivalent decimals not exceeding 6 places: 5. #. Ans. .71875 9. §, Ans. .625 6. ###, Ans. .336 10. Ta, Ans. .076923-H 7. T#5, Ans. .032 | 11. 87%, Ans. .370625 8. § of +, Ans. .1071.42–H | 12. 47.18%, Ans. 47.1875 13. Reduce # to a complex decimal of 3 places. OPERATION. & 3) 2.000 * .6663, Ans. 14. Reduce # to a complex decimal of 4 places. Ans. .4285#. 15. Reduce # to a complex decimal of 6 places. Ans. .2222223 * ADDITION OF DECIMALS, 223 ADDITION OF DECIMALS. 444. Addition of Decimals is finding the sum of two or more decimals. Since decimals increase from right to left, and decrease from left to right in a ten-fold ratio as do simple whole numbers, they may be added, subtracted, mul- tiplied, and divided in the same manner. 1. Add. 785, .93, 166.8, 72.5487, and 4.17. OPERATION. .785 Explanation.—In all problems of this kind, we write the .93 numbers so that units of the same order stand in the same 166.8 column, and the decimal points are in a vertical line; then we e add as in simple whole numbers. When the addition is 72.5487 completed, we point off in the sum, from the right hand, as 4.17 many places for decimals as equal the greatest number of decimal places in any of the numbers added. If there are 245.2337. A complex decimals they must be reduced to pure decimals, as c , AmS. far, at least, as the decimal places extend in the other numbers. GENERAL DIRECTIONS FOR ADDITION OF DECIMALS. 445. From the foregoing elucidations, we derive the following general direc- tions for addition of decimals: 1. Write the numbers so that units or figures of the same order stand in the same column. - 2. Then add as in simple numbers, and point off, in the sum, from the right, as many decimal places as equal the greatest number of decimal places in any of the ww.mbers added. Add the following numbers: 2. 3.25, 42.348, 748.4, and 29.32. Ans. 823.318. 3. .0049, 47.0426, 37.041, and 360.0039. Ans. 444.0924. 4. 1121,6116, 61.87, 46.67, 165.13, and 676.167895. Ans. 2071.449495. 5. .8, .09, 34.275, 562,0785, and 1.01. Ans. 598.2535. 6. 81.61356, 6716.31, 413.1678956, 35.14671, 3.1671, and 31.4.6. Ans. 7564,0052656. 7. 1.013, 240.064, 999.9, 80.6051, and .17. Ans. 1321.7576. 8. What is the sum of the following numbers: twenty-five, and seven millionths; one hundred forty-five, and six hundred forty-three thousandths; one hundred seventy-five, and eighty-nine hundredths; seventeen, and three hundred forty-eight hundred thousandths. Ans. 363,536487. 9. A farmer sold at one time 3 tons and 75 hundredths of a ton of hay; at another time, 11 tons and 7 tenths of a ton; and at a third time, 16 tons and 125 thousandths of a ton. How much did he sell in all? Ans. 31.575 tons. 224 SOULE's PHILOSOPHIC PRACTICAL MATHEMATICs. SUBTRACTION OF DECIMALS. 446. Subtraction of Decimals is finding the difference between two decimals. 1. From 345.3046 subtract 92.1435847. OPERATION. 345.3046 Ea:planation.—In all problems of this kind, we write the 92.1435847 numbers so that units of the same order stand in the same column, and the decimal points are in a vertical line; then we iº subtract as in simple whole numbers, and point off in the 253.1610153, Ans. difference, from the right hand, as many piaces for decimals as equal the greatest number of decimal places in either the minuend or subtrahend. When the decimal places in the subtrahend exceed those in the minuend, naughts are under- stood to occupy the vacant places, and may be filled in if it is desired. 2. From 142.63 subtract 51.1111}. OPERATION. *::::::: Ea:planation.—In all problems of this kind, we reduce the −: complex decimals to pure decimals of equal places and then 91.5555#, Ans. subtract as in subtraction of fractions. GENERAL DIRECTIONS FOR SUBTRACTION OF DECIMALS. 447. From the foregoing elucidations, we derive the following general directions for subtraction of decimals. 1. Write the numbers so that the units of the same order stand in the same column. 2. Then subtract as in simple numbers, and write the decimal point as in addi- tion of decimals. NOTE 1.-If there are complex decimals of unequal places in either or both of the given dec- imals, reduce them to pure decimals of equal places, and then subtract as in subtraction of fractions. NOTE 2.—If there are not as many decimal places in the minuend as there are in the subtra- hend, annex decimal naughts to it, until the decimal places are equal. PROBLEMS. (3) (4) (5) 81.04089 121.25 532.8 14,587 109.05438 9.00451681 66.45389, Ans. 12.19562, Ans. 523.79548319, Ans. 6. From 461.072 take 427.125, Ans. 33.947. 7. From 17.5 take 4.19, Ans. 13.31. 8. From 4000.0004 take 4.3, Ans. 3995.7004. 9. From three million take three millionths, AnS. 29.99999.999997. 10. From 11 take 1 and 9 trillionths, Ans. 9.999999999991. 11. From 24000 subtract 2.078, Ans. 23997.922. 12. From 886.333 subtract 98.5427, Ans. 787.7903. * MULTIPLICATION OF DECIMALS. 225 MULTIPLICATION OF DECIMALS. 448. Multiplication of Decimals is finding the product, when either or both of the factors contain decimals. 1. Multiply 26.58 by 4.3. IFIRST OPERATION. Explanation.—In all problems of this kind, we multiply as in whole numbers, and point off on the right of the product as many 26.58 places for decimals as there are decimal places in both the multipli- 4.3 cand and multiplier. The reason for thus pointing off the 3 deci- mal places in this problem is obvious from the fact, that in the multiplicand, we have 2 decimal places or hundredths, which we 7974 used as whole numbers, and thereby produced a product 100 times 10632 too great; and in the multiplier we have 1 decimal place or tenths which we also used as a whole number, and thereby produced a 114.294. Ans product 10 times too great; and both together, give a product 1000 •- * *-2 © times too great; hence, to obtain the correct product, we divide by 1000, or point off 3 decimal places. SECOND OPERATION. 100 2658 Explanation.—In this operation, we reduce the factors to com- 10 || 43 mon fractions, and then multiplying them together, we obtain a *=sº product of ++$#4, which, written decimally, is 114.294. This pro- 114294 A cess shows, in another way, why we point off on the right of the 1000 g product as many places for decimals as there are decimal places in or decimally written both factors. 114.294, Ans. 2. Multiply 4,024 by .0056. OPERATION. 4.024 Explanation.—In all problems of this kind, where the num- .0056 ber of figures in the product is not equal to the number of decimal places in the two factors, we must prefix a sufficient 24144 number of 0's to supply the deficiency. In this example, we 20120 prefix one 0. The reason of this will appear evident by e- working the example as a common fraction as shown in the .0225344, Ans. second operation of the first problem. GENERAL DIRECTIONS FOR MULTIPLICATION OF DECIMALS. 449. From the foregoing elucidations, we derive the following general direc- tions for the multiplication of decimals: 1. Multiply as in whole numbers and from the right of the product point off as many figures for decimals as there are decimal places in the multiplicand and multiplier. 2. If the product does not contain as many decimal places as both factors, - Supply the deficiency by prefiasing naughts. PROBLEMIS. 3. Multiply 27 by .9, Ans. 24.3. 4. Multiply .38 by 8, Ans. 3.04. 5. Multiply .75 by .42, Ans. .3150. 6. Multiply .006 by .0103, Ans. .0000618. 226 SouLE's PHILOSOPHIC PRACTICAL MATHEMATICS, 7. Multiply 9999. by .9999, Ans. 9998.0001. 8. Multiply 7.0007 by .0303, * Ans. .21212,121. 9. Multiply 340.012 by 61.23, AnS. 20818.93476. 10. Multiply .1234 by 1234. Ans. 152.2756. 11. Multiply 1500 by .00014, Ans. 21. 12. What is the product of one thousand twenty-five multiplied by three hundred twenty-seven ten-thousandths? Ans. 33.5175. 13. What is the product of seventy-eight million two hundred five thousand two, multiplied by fifty-three hundredths? Ans. 41448651.06. 14. Multiply one hundred fifty-three thousandths by one hundred twenty- nine millionths. Ans. .000019737. 15. Multiply 1 thousand by 1 thousandth. Ans. 1. 16. Multiply 2 million by 2 billionths. Ans. .004. 17. What will 37.23 tons of hay cost at $20.75 per ton 2 Ans. $772.52-H.. 18. What will 428,431 bushels cost at $1.125 per bushel? Ans. $481.98--. TO MULTIPLY A DECIMAL OR MIXED NUMBER BY TEN, ONE HUNDRED, ONE THOUSAND, ETC. 450. 1. Multiply 428.375 by 100. OPERATION. Ea:planation.—In all problems where the multiplier is 10, O'Q27 ſº 100, etc., we simply remove the decimal point as many places 42837.5, Ans. to the right as there are naughts in the multiplier, annexing Inaughts if required, as shown in Articles 426 and 438. 2. Multiply 271.32 by 1000, Ans. 271320. 3. Multiply.756 by 100, Ans. 75.6. 4. Multiply .025 by 10, Ans. .25. 5. Multiply 61,052 by 10000, Ans. 610520. A- -- * * r −ww.r-w DIVISION OF DECIMALS. © 451. Division of Decimals is the process of finding the quotient when the divisor or dividend, or both, contain decimals. 1. Divide 17.094 by 8.14. FIRST OPERATION. º lanation.—In all problems of this kind, we divide as in whole numbers, and then point off as many places for decimals from the 8.14) 17,094 (2.1, Ans. right of the quoiº dº jºinial places in the dividend exceed those in 16 28 the divisor, observing to supply any deficiency by prefixing naughts. In this problem, the excess is one, and we therefore point off one dec- 814 ſº imal place in the quotient. The reason for thus pointing off is 814 obvious from the fact that in the dividend we had 3 decimals or *QUSANDTHS, and in the divisor we had 2 decimals or hundredths, Th Eºº ill al and thousandths divided by hundredths give tenths as a quotient. § Fºº, Y}* *ś2 *PPèar plain if we observe that the dividend is the product of the divisor and quotient multiplied together, and hence we point off enough icº" places in the quotient to make the number in the two factors equal to the number in the product or dividend, according to the principles shown in the first problem of multiplication of decimals. DIVISION OF DECIMALS. 227 SECOND OPERATION. 1000 || 17094 Ea:planation.--In this operation, we reduce the decimals to common 814 || 100 fractions and then proceed as in the division of mixed numbers. The reduction of the dividend and divisor to common fractions, 2 and then the mixed numbers to improper fractions, is performed .1, Ans. thus: The dividend 17,094 = 17+3+m = |}}}#; the divisor 8.14 = 8*śr Decimally written 2.1 Ans. =###. This method also shows the reason for pointing off, and may be used for all problems in decimal fractions. PRINCIPLES OF DIVISION OF DECIMALS. 452. From the foregoing elucidations, we derive the following principles: 19. The dividend must contain at least as many decimal places as the divisor; and when both contain the same, the quotient is a whole number. 29. The dividend is the product of the divisor and Quotient, and hence con- tains as many decimal places as both the divisor and quotient, 3°. The quotient must contain as many decimal places as the number of decimal places in the dividend exceeds the number in the divisor. 2. Divide 7898.56 by 2.4683. OPERATION. 2.4683) 7898,5600 (3200, Ans. Ea:planation.—Here we have an excess of decimals in 74049 the divisor, and in all cases of this kind, we first make 49366 them equal by annexing naughts to the dividend, and 49366 the quotient will be a whole number. The reason for annexing the naughts will appear more obvious by 00 solving the problem in the form of a common fraction. 3. Divide 7.0761 by 687. OPERATION. 687) 7.0761 (103 or thus: aft;...º.º.º. 687 .0103, Ans. 10000 || 70761 none in the divisor; hence, according *-*-* 687 to the foregoing instructions, we must 2061 *-i-º- point off 4 decimal places in the quo- 2061 | 0.103. Ans tient, and as there are but 3 figures in ſº 7 • the quotient, we prefix 1 naught. In 4-mºmºs all problems of this kind, 0's are prefixed to supply any deficiency of figures that may occur. 4. Divide 47.789 by 39.27. f OPERATION. Explanation.—In this problem we have a remainder, 39.27) 47.789 (1.2168+, Ans. after dividing the dividend, of 665; to this and the two 39.27 successive remainders, we annex 0's and continue the -- division until we have produced 4 decimal places. The 8519 annexing of 0's reduces the successive remainders to the 7854 next lower order of tenths, and hence all quotient figures **-* * produced by annexing 0's, are decimals. We therefore 6650 point off from the right of the quotient as many places for 3927 decimals as the number of decimals in the dividend exceeds ºm- those of the divisor plus the number of 0's annexed. 27230 This is done in all division problems where 0's are an- 23562 nexed, and a sufficient number of 0's should be annexed to produce 4 or 6 decimal places. When there is a remain- 36680 der after the last division, the plus (+) sign should be 3.1416 annexed to the answer to indicate that the quotient is amº-mºº- incomplete. 228 SOULE's PHILOSOPHIC PRACTICAL MATHEMATICs. 4% * GENERAL DIRECTIONS FOR DIVISION OF DECIMALs. 453. From the foregoing elucidations, we derive the following general directions for dividing decimals: 1. Divide as in whole numbers and point off as many decimal places in the quotient as those in the dividend eacceed those in the divisor. 2. When there is a remainder, annew naughts to the dividend and carry the work as far as may be desired. 5. Divide .112233 by 12, 16. Divide 1.12233 by 12, 11. Divide.0004869 by .0396 OPERATION. OPERATION. FIRST OPERATION. 12) .112233 12) 1.12233 .0396).0004869(122-- 396 =.0122--, Ans. 9352–H =.009352–H, Ans. 9352–H =.09352-H, Ans. | *=º 909 7. Divide 11.2233 by 12. |8. Divide 112.233 by 12. 792 - OPERATION. OPERATION. •º. 1170 12) 11.2233 12) 112.233 792 .9352-H, Ans. 9.3527–H, Ans. 378 SECOND OPERATION. 9. Divide,0004869 by 396.10. Divide,0001809 by 3.96. Sººdºº-oº-, Ans. OPERATION. OPERATION. 96 396).0004869(12–H 3.96).0004869(12-i- 909 396 =.0000012–1–, Ans. 396 = .00012-i-, Ans. 792 909 909 1170 792 792 Afte 792 117 117 378 12. Divide 67.8632 by 32.8. Ans. 2.069. 17. Divide 7,482 by .0006. Ans. 12470. 13. , Divide 983 by 6.6. Ans, 148.939-H | 18. Divide 1 by 999. Ans. .001001-H. 14. Divide 13192.2 by 1947. , Ans, 1260. | 19. Divide 84375 by 3.75. Ans. 22500. 15. Divide 67.56785 by .035. Ans. 1930.51. 20. Divide 1081 by 39.56. Ans. 27.3255+. 16. Divide 00125 by 5. Ans. .0025. 21. Divide 35.7 by 485. Ans. .0736+. 22. If rice costs 8.0775 per pound, how many pounds can be bought for $40,64875? Ans. 524.5 pounds. 23. Sold 14.75 acres of land for $191,75. What was the price per acre 2 Ans. $13. 24. Divide four thousand three hundred twenty-two and four thousand five hundred seventy-three ten-thousandths, by eight thousand and nine thousandths. Ans. .5403+. TO DIVIDE DECIMAL FRACTIONS BY TEN, ONE HUNDRED, on E THOUSAND, ETC., ETC. 454. 1. Divide 48.76 by 10. * OPERATION. Pºplanation.--In all problems of this kind, we simply remove 4,876. A the decimal point as many places to the left" as there are 0's in .ö (0, Ans. the divisor. The reason for this was fully shown in Articles © gº tº 426 and 438. When th g in the dividend to allow this to be tº its ... be †: º, º; º ...” figures 2. Divide 875.25 by 100. Ans. 8.7525. 5. Divide 9.85 by 100. Ans. .0985. 3. Divide 5231 by 1000. Ans. .0005231. 6. Divide 025 by 200. Ans. .000125. 4. Divide 72 by 10000. Ans. .0072. 7. Divide 412.99 by 10. Ans. 41.299. (ſºld Dºmint Nileſ. —-mº-º-o-º: DEFINITIONS. 455. A Denominate Number is a concrete number which expresses a particular kind of unit or quantity, either simple or compound. 456. A Simple Denominate Number is a number which expresses a unit or units or quantities of but one kind or denomination: as 5 dollars, 15 boxes, 8 pounds, 4 days, etc. 457, A Compound Denominate Number is a number which expresses units or quantities of two or more denominations, under one kind of measure: as 8 dollars, 20 cents; 12 pounds, 10 ounces; 7 days, 18 hours, 21 minutes, etc. In whole numbers and in decimals, the law of increase and decrease, between units of lower and of higher orders, is by the uniform scale of 10; but in compound numbers the scale varies according to the kind of measure employed. MEASURES. 458. A Measure is a standard unit established by law or custom, by which quantity, such as extent, dimension, capacity, amount, or value, is measured or estimated. There are eight kinds of measure: º 1st. Length. 2d. Surface, or area. 3d. Solidity, or capacity. 4th. Volume. 5th. Weight, or force of gravity. 6th. Time. 7th. Angles. 8th. Money, or Value. WEIGHT. 459. Weight is that property of matter by virtue of which a body tends toward the centre of the earth; and the resistance required to overcome this centralizing pressure, or gravitating tendency of matter is named weight. Weight varies according to the quantity of matter and its distance from the centre of the earth. VALUE. 460. Walue is the ratio or unit of measure of wealth existing between different commodities with reference to an exchange. It is the sole condition of Wealth and the universal name given to the inherent quality or power of ome thing to command another in eachange. * Briefly expressed, value is the worth of one thing as compared with some other thing. (229) 23O SOULE's PHILOSOPHIC PRACTICAL MATHEMATICS. x- MONEY. 461. Money is stamped metal called coin, or printed bills or notes called paper money, issued by the general government of States or Nations, and supplied to the people to facilitate trade and commerce. It is the standard measure of the Value of things and services, and hence, it is the universal medium of exchange and settlement among all civilized peoples. See Domestic Exchange. 462. Currency (from curro, I run), is a term applied to the money of a nation, whether it be coin or paper money. 463. Bullion is the name given to uncoined gold and silver, and includes bars or ingots. 464. Legal Tender. By this term is meant money which is required by law to be accepted in payment of debts. 465. Gold Coins are a legal tender for any amount, when not below the Standard weight prescribed by law. When gold coins are reduced by natural abrasion, below the standard weight, they are a legal tender at a valuation propor- tional to their actual weight. 466. Standard Silver Dollars are by Act of Congress of 1878, a legal tender for all debts, except in contracts where other money is specified. 467. United States Bank Notes are a legal tender for all debts except duties on imports and the interest on the public debt. 468. Silver Certificates are a legal tender for all debts and are also receivable for customs, taxes and all public dues. * 469. National Bank Notes are not a legal tender; but as they are secured by a deposit with the Comptroller of Currency at Washington, and are redeemable in lawful money by the National Banks, and the Treasurer of the United States, they are perfectly good, and are accepted without question by individuals, firms, corporations, States and the general government, in payment of all kinds of debts, except interest on the public debt, throughout the United States of America. NOTE.-Standard Silver Dollars may be deposited in amounts not less than $10 with the Treasurer or any Assistant Treasurer of the United States, and Silver Certificates received therefor. 470. MONETARY UNITS OF DIFFERENT AGES, NATIONS AND PEOPLES. A great variety of articles and objects have been made to serve as money in different ages and different countries. The money, or measure of value of the ancient Greeks was an ox. We learn from Homer that the armor of Diomede cost nine oxen, and that of Glaucus, one hundred. History tells us, also, that when Pythagoras made his first great discovery in Mathematics— that the square described upon the hypotenuse of a right angled triangle, was equal to the sum of the squares described upon the other two sides—he sacrificed 100 oxen, in honor of the discovery. Evidence is abundant to show that, th the most distant regions and at different times, cattle formed a currency for the early pastoral and agricultural nations. The ancient Germans used cattle; and cattle and slaves, or “living money,” were in common use among the Anglo-Saxons. * MONETARY UNITS OF DIFFERENT AGES, NATIONS AND PEOPLEs. 23 I With the early Italians and Romans, sheep and oxen formed the oldest medium of exchange, ten sheep equalling one ox. In the early English and Irish civilization, the sheep and ox were the units of exchange. The Zulus and Kaffres estimate their wealth, to-day, in the unit of cattle, and use them as money in all transactions which inyolve payment. The Savage tribes of Australia use greenstone, serviceable for making hatchets, and red ochre, Serviceable for painting their bodies, as a general medium of exchange or money. The Indians of British Columbia, use for money haiqua shells; while the Indians of Alaska use the skins of animals. The early Carthagenians and Spartans used for money, pieces of leather and skins, marked with a government stamp. In Chinese Tartary, cubes of tea pressed together; in Hindostan and the Western coast of Africa, shells called cowries, and in Abyssinia, blocks of rock salt are to this day used as money. In Iceland, dried fish, and among the North American Indians, the belt of wampum, or Indian beads made of shells, are used as money. The monetary history of some of the American colonies, from 1620 to 1700, shows that almost every commodity pro- duced or manufactured, was used as money. The early settlers of Virginia made Tobacco a legal tender for the payment of debts; and hundreds of Virginians bought their wives from English wife sellers with tobacco, paying from 120 to 150 pounds per wife. This was undoubtedly the best use ever made of body and brain injuring tobacco. Following these various rude forms of money, iron, lead, copper, tin, and later, silver and gold, were introduced by the ancients, and copper, bronze, silver and gold, and paper bills are used by most modern nations. Russia at one time used platinum for money, but found it unsuitable. Nearly all modern civilized nations have adopted gold and silver as the general standards for money. They are among the most imperishable of all substances, contain great value in small volume, and are easily transported or secreted, which are important considerations in business, and in case of unjust or insecure government. Our own country, and many others, find it convenient to use copper, nickel and bronze for the Subdivisions of the monetary unit in connection with gold, silver, and paper money. MEASURE TABLES. 471. A Measure Table is a regularly arranged Statement showing how many times a higher denomination of a system of measurement equals the next lower denomination of the same system. Or, in other words, it shows how many units of each lower denomination are required to equal one unit of the next higher denomination. 472. TABLE OF UNITED STATES MONEY. 10 Mills (m) = 1 Cent, 2. E. $ d. Ct. II]. 10 Cents = 1 Dime, d. | 1 = 10 = 100 = 1000 = 10000 T0 Dimes = 1 Dollar, $. 1 = 10 = 100 = 1000 10 Dollars = 1 Eagle,” E. 1 = 10 = 100 20 Dollars = Double Eagle. 1 = 10 NOTE 1.-The mill is not coined. It is used only in computations. NOTE: 2.—The word dollar is derived from Thal, a dale or valley; or from Joachim’s Thaler, first coined in the mines of the Bohemian Valley of Saint Joachim, from whence it was borrowed from the Austrians by Spain, during her union with Austria, under the Empire of Charles the W., and which was known as the old Spanish Carolus pillar dollar, which, for ages, had been used by bankers as a standard of value for the English monetary pound, during its many changes. The dime is from the French word disme meaning one-tenth ; the cent is from the Latin centum, a hundred; the mill is from the Latin mille, a thousand; the eagle the $10 coin, is the name of the national bird. See page 38, for the Origin of the Symbol of the Dollar ($). 232 SOULE's PHILOSOPHIC PRACTICAL MATHEMATICs. * 473. United States Money is the legal monetary measure of value in the United States of America. The unit of the measure is the gold and the silver dollar. 474. The U. S. monetary unit, the Dollar, was established by the Continental Congress, August 8, 1786, with the proviso that it should be decimally divided, Which gives great facility in commercial computations. .* 475. The coin of the United States consists of gold, silver, nickel, and bronze. The following table shows the name, value, composition and weight of each coin, as now issued (1894) by the Mints: NOTE.-Paragraph 5 of Section 8, of Article I, of the Constitution of the United States, gives Congress power “to coin money, regulate the value thereof, and of foreign coins, and to fix the standard of weights and measures;” and Section 10 of Article I, says no State shall coin money, emit Bills of Credit, or make any thing but gold or silver coin a tender in payment of debts. By Act of Congress, of February 25th, 1862, the issue of United States Treasury Notes was authorized and made “lawful money and a legal tender in payment of all debts, public and private, § sº United States, except duties on imports and interest on bonds and notes of the Illt;0 3,150S, 476. TABLE OF UNITED STATES COIN. COIN. VALUE. COMPOSITION. WEIGHT. GOLD. Dollar. . . . . . . . . . . . 100 cents. . . . . . . . . 90 parts gold, 10 parts alloy. . . . . . . . . . . . . 25.8 grains Troy. Quarter Eagle. ...|2+ dollars. . . . . . . . . 90 “ “ , 10 “ “. . . . . . . . . . . . . . 64.5 “ & 4 Three Dollars....!3 dollars... . . . . . . . 90 “ , 10 “ “ . . . . . . . . . . . . . 77.4 “ § { Half Eagle. . . . . . . 5 dollars. . . . . . . . . . 90 “ “ , 10 “ “. . . . . . . . . . . . . . 129 & 4 Eagle. . . . . . . . . . . . 10 dollars. . . . . . . . . 90 * * & 4 : 10 * * “. . . . . . . . . . . . . . 258 (« & 4 Double Eagle..... 20 dollars. . . . . . . . . 90 * * “ , 10 “ “. . . . . . . . . . . . . . 516 “ 4 & SILVER. - Dime . . . . . . . . . . . . . 10 cents. . . . . . . . . 90 parts silver, 10 parts alloy. . . . . . . . . . . . 38.58 grains Troy. llar. . . . . 25 ts. . . . . . . . . & 4 << , 10 “ “. . . . . . . . . . . . . .4 & 4 & 4 $º: ; ::::::::::: ; ; ; ; ; ; ::::::::::: tº . . Dollar. . . . . . . . . . . . 100 cents. . . . . . . . . 90 * * “ , 10 “ “. . . . . . . . . . . . . 412.5 & 4 & 4 NICKEL. 3-cent piece...... 3 cents. . . . . . . . . . . 75 parts copper, 25 parts nickel. . . . . . . . 30 grains Troy. 5-cent piece. . . . . . 5 cents. . . . . . . . . . . 75 “ “ , 25 “ “. . . . . . . . . . 77.16 & 4 IBRONZE, One cent.......... 1 cent. . . . . . . . . . . . 95 parts copper, 5 parts tin and zinc.. . . . 48 grains Troy. NoTE. 1.-From 1786 to 1834 the $10 gold piece weighed 270 grs., 4% pure, and all other denominations were in like proportion. From 1834 to the present time 1894, it has weighed 258 grains, ºr pure. NotE 2.—The silver dollar weighed, from 1786 to 1837, 416 grains, # pure, and all other denominations in silver were in like proportion. From 1837 to 1853, it weighed 412.5 grains, Tºo pure, and the 50c, 25c, 10c, and 5c. pieces, in like proportion. In 1851, the 3c. piece was coined, # pure. From 1853 to the present time, the Dollar has weighed 412.5 grains, but the subdivisions thereof were reduced in 1853, to the weights shown in the above table. 477. Silver Coin of the denominations of 58%, 25¢, 10¢, and 5%, are all made under the standard weight to prevent their being shipped abroad and are legal tender only for amounts not exceeding $10. 478. The Nickel 5 and 3-cent pieces, and the bronze 1-cent, are legal tender Only for amounts not exceeding 25 cents. NoTE.—The 5c. nickel weighs 5 grams, or 77.16 grains, and is nearly 3% of a meter in diameter; 48.6, laid side by side, measure one meter. - © See Report of Director of the U. S. Mint of 1893, for all the laws relating to the coinage of U. S. money, and the weights and purity of the various coins from the founding of the government to July 1893. COMPARATIVE VALUES OF GOLD AND SILVER, 233 COMPARATIVE VALUES OF GOLD AND SILVER. As shown in the above table of coins, 25.8 grains of nine-tenths pure gold coin and 412.5 grains of nine-tenths pure silver coin are each $1.00 United States money. Hence the monetary ratio between gold and silver in the United States is as 1 to 15.988. i. e. silver being 1, gold is 15.988. In England this ratio is 1 to 14.287; in France it is 1 to 15.5; and in Germany, 1 to 13.95. ALLOWANCE MADE FOR DEVIATION IN THE WEIGHT OF COIN. 479. In the coinage of the United States money the following allowance is made by law for a deviation in Weight: # grain for the Double Eagles and Eagles; # grain for all other gold pieces; 14 grains in all silver pieces; 3 grains in the nickel 5-cent piece, and 2 grains in the nickel 3-cent piece, and bronze 1-cent piece. 480. The old silver half dime and 3-cent pieces, the bronze 2-cent pieces, and the nickel 1-cent pieces are not now coined. 481. The Trade Dollar was coined for Asiatic commerce, and not for currency. The weight is 420 grains, ſº pure. 482. The ALLOY of a coin is some harder metal mixed with the gold or silver to harden it moderately and thus lessen the wear or abrasion. Gold and silver, in a pure state, being very soft, would rapidly wear away, were they not alloyed. The alloy for American gold coin is composed of about ºf silver and ſº cop- per. The difference in color of our gold coins is because of the different quantity of silver in the alloy. - The alloy for silver coin is pure copper. Coin thus alloyed is called standard. WEIGEIT OF COIN. 483. $10000 gold = 258000 gr. = 44 lbs. 9 oz. 10 pwt. 0 gr. Troy. $1000 standard silver dollars = 412500 gr. = 71 lbs. 7 oz. 7 pwt. 12 gr. Troy. $1000000 gold weighs 53750 ounces Troy or 3685.71 avoirdupois pounds. $1000000 standard silver dollars = 412500000 gr. Troy or = 58928.55+ pounds avoirdupois = 3.68+ car loads of 16000 pounds each. $1000000 silver trade dollars weigh 87.5000 ounces Troy or 60000 pounds avoirdupois. • $1000000 silver, half and quarter dollars, 20-cent pieces and dimes, weigh 803750 ounces Troy or 55114.28 avoirdupois pounds. 234. SOULE's PHILOSOPHIC PRACTICAL MATHEMATICS. Yºr VALUE OF GOLD AND SILVER. e 484. The following values are based upon the present United States mone. tary valuation (1894) of these metals: 1 ounce Troy of pure gold is worth $20.67–H 1 pennyweight Troy of pure gold is worth 1.03+ 1 ounce Troy of pure silver is worth 1.29-H 1 pennyweight Troy of pure silver is worth .0645-H CANADA MONEY. º 485. Canada Money is the legal currency of the Dominion of Canada. It consists of gold, silver, and bronze coin and of paper Imoney. s 486. The silver coins are the 50¢, 25¢, 102, and 52 pieces. The bronze coin is the 12 piece. The gold coins in use are the sovereign and half sovereign. ENGLISH MONEY. 487. English, or Sterling Money, is the legal currency of Great Britain. NOTE.—Sterling is derived from Easterling, the name given to the early German traders whose money was called Easterling money. 488. The Monetary Unit of Great Britain is the pound sterling which is a gold coin weighing 123,274 grains, 4% pure. It is equivalent to $4.8665 U. S. money. NoTE.—For exchange purposes between the United States and England, the pound sterling is valued by bankers at $4.86% and the rate of exchange is quoted in dollars and cents, $4.864, more or less, according as premium is charged or discount is allowed. TABLE OF ENGLISH MONEY. 4 Farthings (far. or qr.) = 1 Penny d. 9. & 12 Pence = 1 Shilling S. 1 = 1; - 21 = 252 = 1008. \ 1 Sovereign sov. 1 = 20 = 240 = 960. 20 Shillings F OI’ 1 = 12 = 48. l 1 Pound £. 1 = 4. 21 Shillings = 1 Guinea. 8. 1 Crown = 5 Shillings C. - NotE.-1st. The Guinea has not been coined since 1816; the term is only used in trade. 2nd. The symbols, £. s. d. qr. or far. are the initials of the Latin words, libra, solidus, denarius and quadrans, signifying respectively, pound, shilling, penny, and quarter. The quarter means the farthing which is derived from a modification of the word “four-things,” the old English penny being marked with a cross so deeply made that it could be broken into two or four pieces, called, respectively, half-penny and four-things. 489. The money of Great Britain consists of gold, silver, copper, and Bank of England notes, or bills. * VALUE OF FOREIGN MONIES. 235 VALUE OF ENGLISH MONEY IN THE U. S. MONETARY UNIT. Present legal value. Value previous to 1834. 1 Pound or Sovereign = $4.8665 $4.44; 1 Shilling == .2433 .22% 1 Penny F. .02024- .01% 1 Crown F. 1.2166-1- 1 Guinea = 5.1098-1- NOTE.-By Act 56, George III., which is still in force, one pound of standard Troy gold (## pure) is coined into £46. 14s. 6d. The full weight of one gold pound or sovereign is therefore 123.274 grains of standard gold, or 113.001 grains of pure gold. But, allowing for the abrasion or wear, a sovereign Weighing but 122.75 grains of standard gold, is in England a legal tender for the payment of debts. A pound of silver ## pure, is coined into 66 shillings. The full weight of the shilling is therefore 87 ºr grains standard silver, or 80 ºr grains of pure silver. A pound of copper is coined into 24 pennies. A pound of bronze, 95% copper, 4% tin, and 1% zinc, is coined into 40 pennies or 80 half pennies, or 160 farthings. Bank of England Notes are a legal tender for any sum exceeding £5. Gold is a legal tender for any amount; silver, not exceeding 40s.; and copper not exceeding 12d., when in pennies or half pennies, and not exceeding 6d., when in farthings. The gold coins are the sovereign and the half sovereign. The silver coins are the crown, half crown, 4s., 2s., 1s., and 6d. See English Exchange for the history of English money, and a full elucidation of the subject. FRENCEI MONEY. 490. French Money is the legal currency of France. It is based on the decimal system, and the Unit is the silver Franc, which equals 19.3 cents U. S. Imoney. NOTE.-In exchange transactions between the United States and France, the rate of exchange is the variable number of francs and centimes allowed for $1. The basis for the rate is 5.20 francs for $1. This rate is the par of ecclvange, and is quoted more or less as premium is declared or discount is allowed. TABLE OF FERENCEI MONEY. fr. do. Ct. Iſl. 10 Millimes (m.)= 1 Centime Ct. --- 1 = 10 = 100 = 1000 10 Centimes = 1 Decime do. 1 = 10 = 100 10 Decimes = 1. Franc fr. 1 = 10 NOTE.-The Millime is not coined; the term simply means the tenth part of a centime. 491. The money of France consists of gold, silver, bronze, and National Bank notes. All French coin is based upon the unit of weight—the gramme. A kilogramme of standard gold, ºr pure, is coined into 155 Napoleons (20 franc pieces), or 3100 francs. The gold coins are the 100, 50, 20, 10, and 5 franc pieces. A kilogramme of silver is coined into 200 francs. The silver coins are the 5, 2, 1, 3, and 4 franc pieces. The copper or bronze coins of the French are the 10, 5, 2, and 1 centimes, which weigh respectively 10, 5, 2, and 1 grammes. The Franc is used in Switzerland and Belgium, and under different names, in Spain, Italy, Greece, and Venezuela. NotE.—See the subject, Foreign Exchange, in this work, for a full elucidation of French Exchange. For the history of French money, see the Metric System, in this work. 236 soul E's PHILOSOPHIC PRACTICAL MATHEMATICS. * GERMAN MONEY. 492. German Money is the legal currency of the German Empire. In 1871 the German Empire established a new and uniform system of money of which the gold “Mark” (Reichsmark), is the Unit. The Mark is equal to 23.8 cents United States money. - 493. The coin of the Empire consists of gold, silver, and nickel. Paper money is also used. TABLE TOF GERMAN MONEY. 100 pfennige, marked Pf., make 1 mark, marked RM. NOTE.-In exchange transactions with the German Empire, for convenience, bankers base the rate of exchange upon the equivalent value of 4 marks expressed in dollars and cents. The exchange par of 4 marks is 954 cents. The rate of exchange is 954, more or less, accor- ding as premium is charged or discount is allowed. See the subject, Foreign Exchange, in this work for further information of German money and a full elucidation of German, English, French, Italian, Austrian, Belgian, Russian, Spanish, Grecial), Portugese, Japanese, Chinese, and Brazilian, and other South American Exchange. MEASURE OF TIME. 494. 1.-Time is a measured portion of duration. 495. 2.-The Unit of measure is the mean solar day. 496. 3.—A Year is the time of the revolution of the earth around the sun. 497. 4.—A Day is the time of the revolution of the earth on its axis. 498. 5.—The Solar Day is the interval of time between two successive passages of the sun across the same meridian of any place, and they are of unequal length on account of the unequal orbital motion of the earth and the obliquity of the ecliptic. 499. 6.—The Mean Solar Day is the mean, or average length of all the Solar days in the year. Its duration is twenty-four hours. 500. 7.—The Civil, or Legal Day used for ordinary purposes, and which corresponds with the Mean Solar Day, commences at midnight and closes at the Inext midnight. e 501. 8.-The Astronomical Day commences at noon, and closes at the Inext in OOn. 502. 9.—The Solar Year is 365 days, 5 hours, 48 minutes, 49.62+ seconds. 503. 10.—The Common, or Civil Year consists of 365 days for 3 successive years, every fourth year containing 366 days, one day being added for the excess of the Solar Year over 365 days. This intercalary day is added to the month of February, which then has 29 days, and the year is called bissextile or leap year. 504. 11.—Leap Year. To determine what years are leap years, the following regulation has been adopted: Every year that is divisible by 4 is a leap year, unless it ends with two naughts, in which case it must be divisible by 400 to be a leap year. Thus, 1884, 1776, and 1600 were, and 2000, 2400, and 2800 will be leap years; but 1885, 1794, and 1800 were not, and 1900, 2100 and 2200 will not be leap years. X- BRIEF HISTORY OF THE YEAR AND ADJUSTMENT OF CALENDAR. 237 505. A BRIEF EIISTORY OF THE YEAR AND OF THE ADJUSTMENT OF THE CALENDAR. The necessity of some division and measurement of time was early felt, and in all ages the subject has received the attention of the most learned men. The phases of changes of the moon supplied a natural and very obvious mode of dividing and reckoning time, and hence the division into months of 29 or 30 days was, perhaps, the earliest and most universal. But it would soon be observed that, for many purposes, the changes of the seasons were more serviceable as marks of division; and thus arose the division into years, determined by the motions of the sun. It was, however, soon discovered that the year, or larger division, did not contain an exact number of the smaller divisions or months, and that an accommodation was neces- sary; and various not very dissimilar expedients were employed for correcting errors that arose. The ancient Egyptians had a year determined by the changes of the seasons, without refer- ence to the changes of the moon, and containing 365 days, divided into 12 months of 30 days each, with 5 supplementary days at the end of the year. The Jewish year consisted, in the earliest periods, as it still does, of twelve lunar months, a thirteenth being from time to time introduced, to accommodate it to the sun and seasons; this was also the case with the ancient Syrians, Mace- donians, etc. The Jewish months have alternately 29 and 30 days; and in a cycle of 19 years there are 7 years having the intercalary month ; some of these years having also one, and some two days more than others have, so that the length of the year varies from 353 to 385 days. The Greeks, in the most ancient periods, reckoned according to real lunar months, twelve making a year; and about 594 B. C., Solon introduced in Athens the mode of reckoning alternately 30 and 29 days to the month, accommodating this civil year of 354 days to the solar year by the occa- sional introduction of an intercalary month. A change was afterwards made, by which three times in eight years a month of 30 days was intercalated, making the average length of the year 365+ days. The divisions of calendars of time now in use in America, and throughout Europe, are the same as those of the Romans. It is supposed that Romulus, the founder of Rome, first undertook to divide the year in such a manner that certain epochs should return periodically after a revolu- tion of the sun; but the knowledge of astronomy was not then sufficiently advanced to allow this to be done with much precision. He placed the commencement of the year in spring, and divided it into 10 months: March, April, May, June, Quintilis, Sextilis, September, October, November, and December. March, May, Quintilis, and October, contained thirty-one days each; the other six contained only thirty. The names Quintilis and Sextilis remained in the calendar till the end of the republic, when they were changed into July and August; the former in honor of Julius Caesar, and the latter, of Augustus. -- The year of Romulus contained only 304 days. Numa Pompilius, 672 B. C., added two months: January to the beginning of the year, and February to the end. About the year 452 B.C., this arrangement was changed by the Decemvirs, who placed February after January; since that time the order of the months has remained undisturbed. In Numa’s year, the months consisted of 29 and 30 days alternately, to correspond with the synodic revolution of the moon. The year would therefore consist of 354 days; but one day was added to make the number odd, as being more lucky. In order to produce a correspondence with the solar year, Numa ordered an intercalary month to be inserted every second year, between the 23d and 24th of February, consisting alternately of 22 and 23 days. Had this regulation been strictly adhered to, the mean length of the year would have been 365+ days, and the months would have continued for a long time to correspond with the $3,1116 SeaSOIlS. But a discretionary power over the intercalary month was exercised by the pontiffs, who fre- quently abused it for the purpose of hastening or retarding the days of the election of magistrates; and the Roman calendar continued in a state of uncertainty and confusion till the time of Julius Caesar, when the civil equinox differed from the astronomical by three months. |Under the advice of the astronomer Sosigenes, Caesar, 46 B. C., abolished the lunar year, and 238 SOULE's PHILOSOPHIC PRACTICAL MATHEMATICs. Y regulated the civil year entirely by the sun. He decreed that the common year should consist of 365 days, but that every fourth year should contain 366. In distributing the days among the different months, he ordered that the odd months, that is the first, third, fifth, seventh, ninth, and eleventh, should contain each 31 days, and the other months 30; excepting February, which in common years was to contain only 29 days, but every fourth year 30 days. This natural and convenient arrange- ment was interrupted to gratify the frivolous vanity of Augustus, by giving August, the month named aſter him, an equal number of days with July, which was named after the first Caes - c. The intercalary day, which occurred every fourth year, was inserted between the 24th and 25th of February. According to the peculiar and awkward manner of reckoning adopted by the Romans, the 24th of February was called the sixth before the calends of March, secto calendas, i. e. six days before March 1st. In the intercalary year, this day was repeated and called bis-secto calendas ; whence the term bissextile, which means two days, both of which are six days before March 1st. The corresponding English term leap year appears less correct, as it seems to imply that a day was leapt-over instead of being thrust in. It may be remarked, that in the ecclesiastical calendar, the intercalary day is still inserted between the 24th and 25th of February. The Julian year consisted of 365+ days, and consequently differed in excess by 11 minutes 10.38 seconds, from the true solar year, which consists of 365 days, 5 hours, 48 minutes, 49.62+-seconds. In consequence of this difference, the astronomical equinox in the course of a few centuries, sensibly fell back towards the beginning of the year. In the time of Julius Caesar, it corres- ponded to the 25th of March; in the sixteenth century, it had retrograded to the 11th. The correction of this error was one of the purposes sought to be obtained by the reforma- tion of the calendar effected by Pope Gregory XIII., in 1582. By suppressing 10 days in the calendar, calling October 5th the 15th, Gregory restored the equinox to the 21st of March, the day on which it fell at the time of the Council of Nice, in 325; the place of Easter and the other mov- able.church feasts in the ecclesiastical calendar having been prescribed at that Council. And in order that the same inconvenience might be prevented in future, he ordered the inter- calation which took place every fourth year to be omitted in centennial years, excepting such as could be divided by 400 without a remainder, as shown above in Article 504. The Gregorian method of intercalation thus gives 97 intercalations in 400 years; consequently, 400 years contain: 400 × 365 + 97 = 146097 days, and therefore the length of one year is 365.2425 days. or 365 d. 5 h. 49 m. 12 sec., which exceeds the true solar year by 22.38 sec., an error which amounts only to one day in 3866 years. The Gregorian calendar was received immediately or shortly after its promulgation by all the Roman Catholic countries of Europe. The Protestant States of Germany and the kingdom of Denmark adhered to the Julian Calen- dar till 1700; and in England and the English American colonies, the alteration was successfully opposed by popular prejudice till 1752. In that year the Julian calendar, or old style, as it was called, was formally abolished by the Act of Parliament, and the date used in all public transactions rendered coincident with that followed in other European countries, by enacting that the day follow- ing the 2d of September of the year 1752, should be called the 14th of that month. When the alteration was made by Pope Gregory, it was only necessary to drop 10 days; the year 1700 having intervened, which was a common year in the Gregorian, but a leap year in the Julian calendar, it was now necessary to drop 11 days. The year thus adjusted is called the new style; and the Julian year, before the adjustment, is called the old style. The old style is still adhered to in Russia and the countries following the com- munion of the Greek church; the difference of date in the present century amounts to twelve days. Thus, when it is January 1st in Russia and Greece, it is January 13th in other countries. The civil or legal year, in England and the English American colonies, formerly commenced on the 25th of March, the day of the Annunciation, though the historical year began on the 1st of January, the day of the Circumcision. By the Act of Parliament for the alteration of the style, in 1752, the beginning of the year was transferred to the 1st of January. X- BRIEF HISTORY OF. THE YEAR AND ADJUSTMENT OF CALENDAR. 239 A CALENDAR. 506. A Calendar is a division of time into certain periods adopted to the purposes of civil life, as hours, days, weeks, months, years, etc. The names and orders of the months, and the number of days contained in each, are now as follows: Names. No. No. days. Names. No. No. days. Names. No. No. days. January, 1st, 31 May, 5th, 31 September, 9th, 30 Rebruary, 2d, 28 June, 6th, 30 October, 10th, 31 March, 3d, 31 July, 7th, 31 November, 11th, 30 April, 4th, 30 August, 8th, 31 December, 12th, 31 The number of days in each, may be readily remembered by committing to memory the following lines: “Thirty days hath September, April, June, and November; And all the rest have thirty-one, Excepting February alone; To which we twenty-eight assign, Till Leap Year gives it twenty-nine.” 507. The Seasons. March, April, and May, are called Spring. June, July, and August, are called Summer. September, October, and November, are called Autumn, and December, January, and February are called Winter. 508. TABLE OF TIME MEASURE. 60 Seconds (sec.) = 1 Minute... . . . . . min. yr. mos. wº. ds. g his. min. S60. º = º º 1 = 12 = }; = 8760 = 525600 = 31536000 7 Days E i Wº......... wi. T † 366 = 8784 – 527040 = 31622400 365 Days = 1 Common year...yr. 1 = | = lº = lſº = *::::: 366 Days = 1 Leap Year. . . . . . yr. * 1 = 60 = 3600 12 Calendar months = 1 Civil Year. . . . . . yT. * i = 60 10 Years = 1 Decade. . . . . . . . . . d. - 10 Decades or 100 Years = 1 Century. . . . . . C. 509. DERIVATION OF THE YEAR, MONTH, WEEK, DAY, ETC. The term Year is derived from the Saxon Gear, and is the time of the earth's revolution around the Sun. * The term Century is derived from the Latin Centuria, and means a collection of a hundred things. * The term Month is derived from the Saxon Monath, from Mona, the Moon, and means the tº of one revolution of the moon around the earth. tº a º Week. The origin of the term week, is not known. It is more ancient than the writings of Moses, and without proper evidence it has been thought by some to have its origin in the phases of the moon, or the number of planets known to the Chaldeans and Egyptians, or to the motion of celestial bodies. Seven days have been used as a division or period of time by most eastern nations from time immemorial. The Greeks, however, did not use it, and it was not introduced into Rome until after the reign of Theodosius. g tº tº The Egyptian week commenced with Saturday. When the Jews took their flight from Egypt, from their hatred to their ancient oppressors, they made Saturday the last day of the week, and the Christians have so continued it. 24O SouLE's PHILOSOPHIC PRACTICAL MATHEMATICS, * * The term Day is derived from the Saxon Daeg, and means the time of the revolution of the earth upon its axis. The term Hour is derived from the Latin hora, and means a definite space of time fixed by natural laws. > The term Minute is from the Latin minutum, (very small). The sixtieth part of an hour of time; also the sixtieth part of a degree of angular space. Second, from the Latin secundus, (the next after the first). The sixtieth part of a minute of time, or of a degree of angular space; that is, the second regular subdivision of the hour or of the degree. 510. DERIVATION OF THE NAMES OF THE MONTHS. January, was so named in honor of Janus, the Roman deity who presided over gates and doors, and who was naturally presumed to have something to do with the opening of the year. This deity was represented with two faces, one looking backward, the other forward, implying that he stood between the old year and the new, with a regard for each. February derives its name from the circumstance that during the month occurred the Roman religious festival called the Lupercalia, and also Februalia, from februare, to purify. March, the third month of the year, but originally the first, was named Martius, in honor of Mars, the god of war and the reputed father of Romulus, the founder of Rome. April. The Romans gave this month the name of Aprilis, from aperire, to open, because it Was the season when the buds began to open and the earth to produce vegetation. May. The common belief is that May was named in honor of Maia, the mother, by Jupiter, Of the god Hermes, or Mercury; but the most reliable authority is that it was named in honor of the majores, or maiores, the senate in the original constitution of Rome. June. The name of this month has been claimed in honor of Juno; but better evidence proves that it was dedicated a junioribus, that is, to the junior or inferior branch of the original legislature of Rome. ' º July was originally designated Quintilis, in reference to its fifth place in the calendar; but having been the month in which Julius Caesar was born, after the death of this emperor, the Romans changed the name to July, in honor of that great warrior. August, in the old Roman calendar was called Sextilis, it being the sixth month in the ten which constituted the year. But it was subsequently named August, in honor of the emperor Augustus, and one day was taken from February and added to August to make 31, the same as July, the month named in honor of Caesar. September, October, November, and December, are named, respectively, from the Latin numerals, Septem, Octo, Novem, and Decem, as when the year consisted of but ten months and began in March, they were the seventh, eighth, ninth and tenth months, as their Latin names indicate. 511. DERIVATION OF TEIE DAYS OF THE WEEK. Sunday is so called because it was anciently dedicated to the sun by the sun worshipers. Romans, Saxons, etc. In the 4th century, A. D., it was made the Christian Sabbath by Constantine, who left the Sun worshipers and became a Christian. Monday, from the A. Saxon, monami-daeg, moon day; a day sacred to the moon as Sunday was to the sun. Tuesday derived from Tuisco, the Saxon god of war. Wednesday derived from the Scandinavian deity, Woden, whose functions corresponded to those of Mercury, in the Greek and Roman Mythology. Thursday, named for the Scandinavian god Thor, the god of Thunder. Thor was equivalent to Jupiter in the Greek and Roman Mythology. Friday, so named in honor of Freya, a Saxon goddess, the wife of Woden or Oden. MEASURES OF EXTENSION. 24. I Saturday is so called from being dedicated by the Romans to Saturn, the god of Agriculture, and who was the father of Jupiter, Neptune, and Pluto, and in whose honor were held the Saturn- alian festivals. Thus we see that the English names of the week are derived from the ancient Saxons, who borrowed the week from some eastern nation, and substituted the names of their own divinities for those of the gods of Greece and Rome. 512. THE FOLLOWING ARE THE NAMES OF THE WEEK IN LATIN, SAxon AND ENGLISH: ~– LATIN.——s SAXON.—, ENGLISH.—N Dies Solos, Dies Mercurii, Sun's day, Woden's day, º Sunday, Wednesday, Dies Lunae, Dies Jovis, Moon's day, Thor's day, Monday, Thursday, Dies Martis, Dies Veneris, Tiw’s day, Friga's day, Tuesday, Friday, Dies Saturni. Seterne's day. Saturday. MEASURES OF EXTENSION. 513. Extension is that property of matter by which it occupies space. It may have one or more of the three dimensions—length, breadth, and thickness. 514. The Yard is the United States and English unit of measure of exten- Sion, whether for a line, surface or solid. A yard is a rod or tape whose length is equivalent to ###### of the length of a pendulum vibrating seconds of mean time in the latitude of London in a vacuum at the level of the sea, when the temperature is 620 Fahrenheit; or differently stated, a pendulum that vibrates seconds under the conditions given, is divided into 391393 equal parts, and 360000 of these parts are taken for a yard, i. e. the second’s pendulum is 39.1393 inches in length of which the yard consists of 36 of the 39 inches. NoTE 1.-The yard was thus determined in England, during the reign of William IV., and has been adopted by, and is now in general use throughout, the United States. e 2.—By joint resolution of Congress, June 14th, 1836, the Secretary of State was directed to furnish to the Governors of each State, for the use of the States, a complete set of all the weights and measures adopted by the United States; this was done and the statutory standards of all the States have been made conformable thereto, except the bushel of New York State, and the barrel of Massachusetts. 3.—The metric system was legalized by the United States iu 1866, but its use was left optional with the people, and is used only by certain professional and Scientific men NOTE.-See Metric System, in this book. 515. A Line has only one dimension—length. 516. A Surface has two dimensions—length and breadth. 517. A Solid, or volume has three dimensions—length, breadth, and thickness. - LINE, OR LINEAR MEASURE. 518. Line, or Linear Measure is used to measure distances, or length, in any direction. & - One inch. Two inches. Three inches. Note.—For the operations of linear, surface, and solid measure, see “Mensuration of Surfaces and Solids,” in this book. 242 SOULE's PHILOSOPHIC PRACTICAL MATHEMATICS. TABLE L. m. fur. rā. yd. . . ft. in. 12 Inches (in.) = 1 Foot. . . . . . . . . . . . . . . . ft. 1–3–24=960=5280=15840=190080 3 Feet = 1 Yard. . . . . . . . . . . . . . . . d. 1= 8=320=1760= 5280= 63360 5+ Yards, or 164 feet = 1 Rod, or Pole. . . . . . . . rd., or P. 1= 40– 220– 660– 7920 40 Rods = 1 Furlong. ... -- . . . . . . fur. (obsolete) 1= 5}= 16}= 198 8 Furlongs (320 rds.)= 1 Mile (Statute Mile). mi. 1= 3= 36 - 3 Miles = 1 League. . . . . . . . . . . . . L. 1= 12 NotE.—The units of length are nearly all derived from different parts of the human body, and from other objects: Thus, the foot was originally the length of the king's foot, and consequently varied as a king with a long or a short foot chanced to reign. The span was the space between the end of the thumb and little finger, when both were extended. A hand was the width of the hand across the palm. The cubit was the length of the arm from the elbow to the end of the middle finger. The fathom was originally the distance between the ends of the two middle fingers, when the arms and fingers were extended to their farthest limit. Henry I., king of England, declared that the ell, or yard, should be the length of his own arm, from the end of the longest finger to the middle of the breast, and that the other measures should be . from this. An acre was originally as much land as could be ploughed in a day by a yoke OI OX62E1, Furlong, now obsolete, is from fur. meaning furrow; and long, i. e. the length of a furrow. Rod (16+ ft.) came from a rod or stick used for measuring. An inch was, with Edward II., in 1324, the length of 3 barley corns, sound and dry; and 12 of such inches was a foot. See Mariners', Cloth, and Shoemakers’ Measures; also Table of Comparative Weights, Measures and Values, and the old French and Spanish Measures, in this book. MARINERS’ MEASURE. 519. Mariners’ Measure is used to measure distances at Sea and also to measure the depths of seas. TABLE. 6 Feet = 1 Fathom. 120 Fathoms = 1 Cable-length. 880 Fathoms, or 7; Cable-lengths = 5280 ft. = 1 mile. 520. A Minute of the earth's circumference is called a geographical, or nautical mile, or knot, which is gº of #6 = grºwd of the circumference of the earth. The circumference of the earth at the equator is 24899 miles, which, divided by 21600, gives 1.15273+ statute miles. The length of a degree at the equator is 1.15273× 60 equals 69.1638 statute miles. wº SEHOEMAIKERS’ MEASURE. 521. Shoemakers’ Measure is used by shoemakers to measure the human feet and in the manufacture of boots and shoes. 522. The Unit of Measure is # of an inch which is the same as the former unit of 1 barley corn, when 3 barley corns made 1 inch. No. 1, small size, is 4; inches; and every succeeding number increases # of an inch to 13. No. 1, large size, is 84+ inches; and every succeeding number increases # of an inch to 15. Y TABLES OF MEASURES, 243 523. MISCELLANEOUS UNITS of LINEAR MEASURE. NOTE:-The barley corn was formerly a unit of measure in England; 3 well matured barley corns placed together, lengthwise = 1 inch. TABLE. *; of an Inch = A Line (American). 9 Inches = A Span. +% of an Inch = A Line (French). 3 Feet = A Pace. 4 Inches = A Hand. 24 Feet (28 in.) = A Military Pace. 3 Inches = A Palm. 18 Inches = A. Cubit. NOTE:-The cubit of the Egyptians, about 3000 B.C. was 20.63 inches. The cubit of Moses varied at different periods from 18 to 25.2 inches. Egypt, Chaldea and other nations had regular though varying systems of weights and measures. CLOTEI MEASURE. 524. Cloth Measure is used to measure all kinds of goods sold by the yard. TABLE. --" yd. qr, na. in. 2# Inches (in.) = 1 Nail - - na. 1 = 4 = 16 = 36 4 Nails (9 inches) = 1 Quarter - qr. 1 = 4 = 9 4 Quarters = 1 Yard - yd. 1 = 2+ This table formerly contained: NoTE 1.-The Flemish ell, or yard, which equaled 3 quarters or 27 inches; the English ell, or yard, which equaled 5 quarters or 45 inches; the French ell, or yard, which equaled 6 quarters or 54 inches. NOTE 2.—All of the above units of measure are now out of use except the yard, which is divided into halves, quarters, eighths, sixteenths, etc., in place of feet and inches. At the Custom- house, the yard is decimally divided. NOTE 3.—The nail of 2+ inches is derived from the six-penny nail of commerce, which is, or was formerly, 2+ inches long. The word penny when applied to nails refers to the size and is a corruption of pound. Thus By a six-penny nail is meant a nail, 1000 of which will weigh 6 pounds. And by the 4, 8, 10 and double 10-penny is meant that 1000 will weigh respectively 4, 8, 10 and 12 pounds. 525. SQUARE, OR SURFACE MEASURE. 3 Feet. One # Squar |e Yard 3 ft. x 3 ft. = 9 sq. ft. = 1 sq. yd. 526. Square, or Surface Measure is used in computing surfaces or areas. 527. A Surface has length and breadth, but no measurable thickness. 528. The Area of a surface is the quantity of surface it contains, and is expressed by the product of the length by the breadth. 529. A Square is a plane figure bounded by four equal sides, and having four right angles. 244 SOULE's PHILOSOPHIC PRACTICAL MATHEMATICS. 530. TABLE. 144 (12 x 12) Square Inches (sq. in.) = 1 Square Foot - - sq. ft. * 9 (3 x 3) Square Feet = 1 Square Yard - sq. yd. 304 (54 × 53) Square Yards GEº { 1 Square Rod - - sq. rā. or 2724 sq. ft. * --> Or Perch - - P. 160 Square Rods = 1 Acre - - - - A. 640 Acres = 1 Sq. Mile, or Section... sq. mi., or sec. sq. mi. 3. sq. rà. Sq. yd. sq. ft. Sq. in. 1 = 640 = 102400 = 3097600 = 278784.00 = 4014.489600 1 = 160 = 4840 = 43560 = 6272640 1 = 30+ = 272} = 39.204 1 = 9 = 1296 1 = 144 NotE.—The term Perch is from the French perche, a pole or rod, used for measuring, and is a surface equal to a square rod. 531. Architects, carpenters, slaters, and some other mechanics frequently measure their work by the square, which is a space 10 ft. by 10 ft., equaling 100 Square feet. 532. A square foot, yard, or mile, is a square each side of which is one foot, yard, or mile. 3 Feet. The number of small squares contained in any large square is equal to the product of the number of units in one side, multiplied by the number of units in the other side. Thus, in figure 2, each side of which is three feet, there are 9 square feet. The difference between 84ware feet and feet 8quare, 8quare miles and miles square, etc., for any unit of measure, is not generally understood, and because of its practical importance we solicit special attention to it. By 3 yards square is meant a square figure, each side of which is 3 yards; but by 3 square yards is meant 3 small Squares, each a yard long and a Fig. 2. yard wide. : Figure 2 is 3 feet square, and contains 9 square feet. The difference between 500 rods square and 500 square rods is 249500 square rods. There is no difference between 1 yard square and 1 square yard, 1 foot square and 1 square foot, etc., of any unit of measure, but increase the measure above 1 unit and there is a difference. NOTE.-Formerly, but now obsolete, 40 square rods or perches = 1 rood. 4 roods = 1 acre. CHAIN MEASURE, OR SURVEYORS AND ENGINEERS, MEASURE. 533. Chain Measure is used by surveyors and topographical engineers, in measuring land, laying out roads, etc. NoTE.—The chain here used is called Gunter's Chain and is named after Edmund Gunter, an eminent English mathematician, born 1581. 534. TABLE. 7.92 Inches = 1 Link - - - 1. 25 Links F. 1 Rod, or Pole - rid., or P. 4 Rods, or te 66 feet” } – 1 Chain - - - ch. 80 Chains = 1 Mile - - - mi. (5280 ft). º: UNITED STATES GOVERNMENT LAND, .. 245 Engineers commonly use another chain or tape line, which consists of 100 links, each 1 foot long. mi. ch. rd. 1. in. 1 = 80 = 320 = 8000 = 63360 1 = 4 = 100 = 792 1 = 25 = 198 1 = 7.92 535. Surveyors’ Square Measure is used by surveyors' in measuring the area or surface of land. The acre is the measuring unit for land. TABLE. 625 Square Links (sq. 1.) 16 Square Rods 10 Square Chains 1 Square Rod, or Pole. sq. rd., or P. 1 Square Chain - - sq. ch. 1 Acre - - - - - A : "...º. Miles (6 1 Square Mile, or Sec. sq. mi., or Sec. T13.1°C WL116S e . es Squar§ } = 1 Township - : - Tp. also 10000 Square Links, sq. L. = 1 Square Chain - Sq. Ch. 10 Square Chains = 1 Acre - - - - A. Tp. sq. mi. A. Sq. ch. Sq. rd. Sq. 1. 1 = 36 = 23040 = 230400 = 3686400 = 2304000000 DNITED STATES GOVERNMENT LANDS. 536. In 1802, Col. Jared Mansfield, the Surveyor General of the North- western territories, instituted a very convenient system of laying out government lands. The system adopted is as follows: FIRST.—A line is run North and South and is called the Principal Meridian, P. M. SECOND.—Other lines are then run 6 miles apart and parallel to the P. M. THIRD.—A line is run on a parallel of latitude East and West which is called the base line, B. L. -> POURTH.—Other lines are then run 6 miles apart and parallel to the B. L., thus dividing the land into squares, 6 miles each way, called townships, each of which contains 36 square miles. A line of townships running North and South is called a Range which is designated by its number East or West of the P. M. Townships are also designated by their number North or South of the B. L. Townships are also divided into square miles, called sections, which are numbered from 1 to 36, beginning on the North-east corner of each township. Each section is subdivided into half sections, quarter sections, half quarter sections, and quarter quarter sections. 537. TABLE. 1 Township = 6 mi. Square = 36 Sq. mi. = 23040 A. 1 Section = 1 “ {{ = 1 {{ = 640 € $ 1 Half Section = 1 “by 3 mi. = } “ = 320 4% 1 Quarter Section = 4 “ “ , “ = + “ = 160 “ 1 Half Quarter Section = 4 “ “ + “ = } “ = 80 “ 1 Quarter Quarter Section = # “ “ # “ = ** “ = 40 “ 246 SouLE's PHILOSOPHIC PRACTICAL MATHEMATICS, Yºr The following diagrams will make clear the foregoing statements regarding .." and sections, and the manner of designating the subdivisions of SęCTIOILS : 538. DIAGRAM SEIOWING THE DIVISION OF THE GOVERNMENT LANDS OF THE UNITED STATES INTO TOWNSEIIPS. Figure 1. N Ea:planation.—In fig- fi ure 1, W. E. is the base º line. N. S. is the prin- cipal meridian. R. 1 E. is range 1, east ; R. 2 E. is range 2, east ; 2 3 R. 3 W. is range 3, E west; T. 3 N. is town- 2 3 T. 1 S ship 3, north; T. 1 S. | is township 1, south ; B. A. is township 2, north, range 2, west; B. is township 3, South, range 2, east. A SECTION ENLARGED, 539. One Mile Square. A TOWNSHIP, Six Miles Square. N N 6 5 4 3 2 1 N. : Sec. 7 8 9 10 11 12 320 Acres. 18 17 16 15 14 13 E 1 5: N. E. 19 20 2 22 23 24 of W. 4. S. E. # * 40 A. 30 29 28 27 26 25 S. W. + Sec. of 160 Acres. S. E. # 31 32 33 34 35 36 80 A. S The above section represents one of the 36 sections of a township. NoTE.—1. In some parts of Louisiana and Missouri, that were settled by the French, the old French arpent is still used, instead of the acre, as the unit in measuring land... 2. An arpent (Paris) contains 100 sq. rods, (Paris) or 36804.12+ sq. ft. (English), and is 191.1844 ft. (English) on each side. 3. An acre contains 160 sq. rods, or 43560 sq. ft., and is 208,7103-H ft. on each side. These figures show that an arpent (Paris) is .8449–H per cent., a little more than } of an acre. See Table of French and Spanish Measures, and Table of Comparative Weights, Measures, and Values, in this book. X- SOLID, OR CUBIC MEASURE. 247 SOLID, OR CUBIC MEASURE. 540. Solid or Cubic Measure is used in measuring the contents, or volume, of Solids. 541. A Solid, or Body, has length, breadth, and thickness. 542. A Cube is a solid, bounded by six equal square sides, or faces; hence its three dimensions are equal to each other. 543. The Contents, or Volume, of a regular body is expressed by the product of its length, breadth, and thickness. 544. TABLE. 1728 Cubic Inches (cu. in.) = 1 Cubic Foot - - - - cu. ft. 27 Cubic Feet (3× 3x3) = 1 Cubic Yard - - - - cu. ya. , 16 Cubic Feet = 1 Cord Foot - - - - cq. ft. 8 Cord Feet, or 128 cubicft. = 1 Cord of Wood - - - ca. 24; Cubic Feet, or 164 feet long, 14 ft. high, and K = 1 Perch of Stone - - - Peh. 1 foot wide 40 Cu. Ft. in United States, and 42 Cu. Ft. in England, 1 Shipping Ton. 100 Cu. Ft., 1 Register Ton, used in estimating the tonnage of vessels. cu. ya... cu. ft. cu. in. cd. c.d. ft. cu. ft. cu. in. = 27 = 46656 1 = 8 = 128 = 221184 tri NOTE.-A cord of wood is so named from the fact that originally it was measured by a cord or String. 545. A Square of earth is a cube 6× 6×6 = 216 cubic feet. 546. In civil engineering, the cubic yard is the unit for measuring excava- tions, embankments, and levees. A cubic yard of earth is called a load. 547. In commerce, the cubic foot is often the unit for computing freight charges. LIQUID, OR WINE MEASURE. 548. Liquid or Wine Measure is used in measuring molasses, wine, oil, etc., and in estimating the capacities of cisterns, reservoirs, etc. 549. The Unit of measure for liquids is the gallon, which contains 231 cubic inches. 550. The Gallon of the United States is the standard or Winchester wine gallon of 231 cubic inches, and contains 8.3388822 avoirdupois lbs., or 58,372.1754 Troy grains of distilled water at 39.83° F., the barometer being at 30 inches. It is equal to 3.785207 litres. The gallon of the State of New York is of the capacity of 8 lbs. of -pure water at its maximum density 39.2° F., or 221.184 cubic inches. It is equal to 3.62436 litres. 551. TABLE. 4 Gills (gi.) = 1 Pint - - pt. gal. qt. pt. gi. 2 Pints = 1 Quart t. 1 = 4 = 8 = 32 4 Quarts = 1 Gallon - gal. = 231 cubic in 1 = 2 = 8 314 Gallons = 1 Barrel - bbl. - 1 = 4 2 Barrels, or 63 gallons 1 Hogshead hind. NOTE.—1. In the old tables, 42 gallons = 1 tierce; 84 gallons = 1 puncheon; 2 hogsheads = 248 SouLE's PHILOSOPHIC PRACTICAL MATHEMATICS. X. 1 pipe or butt; and 2 pipes = 1 ton. But these measures are no longer used as units. They are casks of variable capacity, and in commerce, are usually gauged separately and have their capaci- ties in gallons marked on them. 2. The barrel is used as a unit of measure only in estimating the capacity of Wells, cisterns, vats, etc. '3. In Massachusetts, the barrel contains 32.28 gallons. 4. This standard unit for liquid measure is called the Winchester gallon, from the standard having been kept at Winchester, England. It was called the wine measure, because wine was measured by it, while beer was measured by a larger measure. 5. The gallon is from the Latin gelo, a vessel for holding wine. The quart is derived from the Latin quartus, a fourth; the pint is derived from the Greek pinto, to drink; and the gill is from the Latin gilla, a glass to drink from. NOTE.—The units of the old beer and milk measure, now nearly obsolete, are : 2 pints = 1 quart; 4 quarts = 1 gallon = 282 cubic inches; 36 gallons = 1 barrel; 54 gallons = 1 hogshead. DRY MEASURE. 552. Dry Measure is used to measure grain, fruit, vegetables, etc. 553. The Unit of dry measure is the bushel, which contains 2150.42 (prac- tically 2150.4) cubic inches. NOTE.—This United States standard unit for dry measure is the old English Winchester bushel, so called from the standard being kept at Winchester. It is a cylinder 18% inches in diame- ter and 8 inches deep. It contains 543391.89 standard Troy grains, 2150.42 cubic inches, or 77.6274 ºis pounds of distilled water at the temperature of 39.83° Fahrenheit, and barometer II].C. (162S, TABLE. © * 2 Pints (pt.) = 1 Quart - - - - qt. bu. pk, qt. pt. 8 Quarts = 1 Peck - - - - pk. 1 = 4 = 32 = 64 4 Pecks = 1 Bushel - - - bu. 1 = 8 = 16 8 Bushels (480 pounds) = 1 Quarter - - - qr. 1 = 2 36 Bushels = 1 Chaldron - - ch NotE.—1. The heaped bushel, the cone of which is 6 inches above the brim of the measure, contains 2747.7 cubic inches. 2.—When corn is bought or sold in the shuck, 2% to 4 cubic feet as per agreement, are allowed for a bushel, according to the quantity of shuck on the corn and the density of the corn in the crib. If the corn is shucked or husked, 2 cubic feet are allowed for a bushel. 3.—In many sections of the country, 2 heaped bushels of corn “in the ear” are called 1 bu. of shelled corn. In other places, 2 even bu. of “ears” are allowed for 1 bu. of shelled corn. 4.—In Maryland, 5 bushels of corn make a barrel of corn. 554. The dry gallon, or half peck, contains 268.8 cubic inches; and 10 dry gallons, or 2688 cubic inches make a bushel of coal in Pennsylvania, Ohio, Kentucky, and several other states, when gauging coal barges, or when measuring heaps or piles of coal. - The dry gallon of 268.8 cu. in. is also one-eighth of the American standard bushel. In Louisiana, the custom, when selling coal, is to allow 2.6 bushels of 2688 cu. in. = 6988.8 (6988.865) cubic inches for a barrel. See page 409. Article 3912, of Weights and Measures, Revised Statutes of Louisiana, says: “That Weights and Measures are to be procured by the Governor, like those of the United States, and deposited in the office of the Secretary of State.” Article 3925, says: “There shall be in this State a dry measure to be known under the name of barrel, which shall contain 34 bushels, according to American standard.” MEASURES OF WEIGHT, 249 Article 3926, says: “Coal shall be sold by the barrel or bushel measure.” Accordingly, in Louisiana, a barrel of coal is 34 times 2150.42 cubic inches, = 6988,865 cubic inches. In England, the standard Imperial bushel contains 2218.192 cubic inches; the Imperial quar- ter is 480 pounds; and the Imperial gallon, both dry and liquid measure, contains 277.274 cubic inches, which is one-eighth of the English bushel. The Imperial standard gallon, both for dry and liquid measure, contains 10 pounds avoirdu- pois of distilled water, temperature 62° Farenheit, barometer 30 inches. In volume its capacity is 277.274 cubic inches. See Mensuration of Solids for practical problems in measuring or gauging barges, barrels, etc. MEASURES OF WEIGHT. TROY, OR MIN T W EIGHT. 555. Troy Weight is used in weighing gold and silver, and in physical and chemical experiments. 556. The Standard Unit of weight in the United States is the Troy pownd, which contains 5760 grains. TABLE. º EQUIVALENTS. lb. OZ. pwt. gT. 24 Grains (gr.) = 1 Pennyweight - pvt. or dwt. 1 = 12 = 240 = 576 20 Pennyweights = 1 Ounce - - - - - - oz. 1 = 20 = 480 12 Ounces == = 24 1 Pound - - - - - - lb. 1. NOTE 1.-The Troy pound is identical with the Imperial Troy pound of England, and derives its name from Troy Novant, the ancient name of London. Some derive its name from Troyes, a town in France, where this weight was first used in Europe. It was brought from Cairo, Egypt, in the 12th Century. It is equivalent to the weight of 22.79442 cubic inches of distilled water at its maximum density, or of 22.8157 cubic inches 629 Fahrenheit, barometer at 30 inches in both cases. 2.—The term pound is derived from the Latin word pendo, meaning to bend or weigh. The term ounce is from the Latin word wincia, meaning one-twelfth part of a pound. The pennyweight was the weight of the old English silver penny, which was formerly used as a weight. The term grain is derived from grains of wheat which were formerly used for weighing. At first 32 grains taken from the middle of the ear or head, and well dried, was equal to a pennyweight. But subse- quently the pennyweight was divided into 24 parts, each part being called a grain, though it is heavier than a grain of wheat. 3.—The symbol lb. is from libra, the Latin word for pound; oz. is from the Spanish word onza, meaning ounce; put. is a compound of p. meaning penny and wit, meaning weight; dwt. is from d. in the Latin word denarius, which means penny or the 240th of a pound, and wi. in Weight; gr. is the abbreviation of grains. AVOIRDUPOIS, OR COMMERCIAL WEIGHT. 557. Avoirdupois or Commercial Weight, is used in weighing all coarse articles: as groceries, cotton, iron, etc. 25o SOULE's PHILOSOPHIC PRACTICAL MATHEMATICS. X- 558. TABLE. 27# Grains = 1 Dram - - - - dr. 16 Drams = 1 Ounce - - - - oz. 16 Ounces = 1 Pound - - - - lb. 25 Pounds = 1 Quarter - - - - qr. 4 Quarters, or 100 pounds = 1 EIundredweight - cwt. 20 Hundredweight, or 2000 pounds = 1 Ton - - - - - T. 480 lºounds = 1 Imperial Quarter - Im. qr. 100 Pounds is also called 1 Cental - - - - c. T. CWt. lb. OZ. dr. 1 = 20 = 2000 = 32000 = 512000 1 = 100 = 1600 = 25600 1 = 16 = 256 = 7000 gr. 1 = 16 NOTE.-1. The cwt. in England, and in some cases in the United States, is 112 pounds, or 4 quarters of 28 pounds. The ton English is 2240 pounds. This is called the long ton, and 2000 pounds, the short tom. 2.—The long ton is used in estimating duties at the U. S. Customhouse, and also at the mines in weighing coal, ores, etc. 3.—The avoirdupois pound is equivalent to the weight of 27.7015 cubic inches of distilled water at its maximum density, or of 27.7274 cubic inches at 62°Fahrenheit, barometer in both cases at 30 inches. The avoirdupois pound of England and the United States is the same. 4.—1 ft. avoirdupois = }}#} of 1 fö. Troy, or ## of 1 lb. Troy. Or 144 f5. avoirdupois = 175 Ib. Troy. But the Troy ounce is heavier than the avoirdupois ounce. 175 Troy ounces = 192 avoir- dupois ounces. 5.—The term avoirdupois, is derived from the French word avoir du pois, to have (a fixed or standard) weight. The term ton is derived from the Saxon word tunne, a cask. The origin of the pound, ounce and grains was given in the Troy table. The symbol cwt. is compounded from the words, centum and weight. The term dram is from drachma, originally a small Greek weight; it is now obsolete as the ounce, in commerce, is divided into halves and quarters. See Miscellaneous Table for old English units of Stone, pig, fother, etc. APOTEIECARIES” WEIGHT. 559. Apothecaries’ Weight is used by physicians and apothecaries in weighing and compounding dry medicines. Medicines are bought and Sold by avoirdupois weight. TABLE. lb. OZ. dr. SCI’. gr. 20 Grains (gr.) = 1 Scruple - scr. or 9 1 = 12 = 96 = 288 = 5760 3 Scruples = 1 Dram - dr. “ 3 1 = 8 – 24 = 480 8 Drams = 1. Ounce - oz. “ 3 1 = 3 = 60 12 Ounces = 1 Pound - Ib. 4 ſh 1 = 20 NOTE.-1. The grain, the ounce, and the pound of this weight are the same as those of Troy weight, the ounce being differently divided. 2.--The terms pound, ounce, dram and grains were defined in the Troy and avoirdupois tables. . The term scruple is from the Latin word scrupulus, a small stone. The symbols used in this weight had their origin, according to Champollion, in the Egyptian hieroglyphics. NOTE.-Physicians, when writing prescriptions use the Roman numerals in small letters, prefixing the symbols and writing j for i when it terminates a number. Thus, 22 gr. is written gr. xxij; 11 scruples, 9xj; 6 ounces, 3v.j. - . B. is an abbreviation for recipe, or take; a., aa., for equal quantities; 88, for semi, or half as 3 viss., means 64 drams; gr. for grain; P. for particula, or little part; P. aeq. for equal parts; q. p. quantum placet, as much as you please. - Apothecaries and Physicians, of earlier times designed to conceal from others all knowledge of the ingredients given as medicines, and hence the articles used as medicine were named in Latin, indicated in Latin numerals, and arbitrary signs employed to express the quantity. TABLES OF WEIGHTS. 25 I 560. Apothecaries' fluid measure is used in compounding liquid medicines. ſº TABLE. 60 Minims (m.) = 1 Fluidrachm - - - - f 3 8 Fluidrachms = 1 Fluidounce - - - - f2 16 Fluidounces = 1 Pint - - - - - - O. 8 Pints = 1 Gallon - - - - Cong. Cong. O. f3 f 3 Ill. 1 = 8 = 128 = 1024 = 61440 One M. = about 1 drop of water. 1 = 16 = 128 = 7680 1 = 8 = 480 1 = 60 NotE.—Minim (M.) is from the Latin minimus, the least; the minim is the smallest fluid measure in use; one minim is equal to about a drop of water. Fluidrachms and Fluidounces are formed by prefixing fluid to the terms drachms and ounces of the apothecaries' weight. O. is an abbreviation of octams, the Latin for one-eight; meaning one pint, or the eighth part of a gallon. Cong. is from congiarium, the Latin for gallon. Drops, are indicated in physicians’ prescriptions by Gºtt, which is from the Latin guttae. 561. A single common teaspoonful, or 45 drops, makes about one fluidrachm; 4 teaspoonfuls make 1 tablespoonful, which is about half a fluidounce; a wine glass equals about one and a half fluidounces; a common teacup holds about 4 fluid- ounces; a pint of water weighs 1.0423-H pounds; practically 1 pound. DIAMOND WEIGHT. 562. Diamond Weight is used in weighing diamonds and other precious Stones. TABLE. 16 Parts = 1 Carat Grain = .792 Troy grains. 4 Grains = 1 Carat = 3.168 “ {{ NotE.—The term carat is derived from Kuara, the name of a tree in Abyssinia, which bears a bean which varies but little in weight from the time it is gathered. In early ages, it was used as a weight for gold in Africa. In India, it was used as a weight for diamonds and gold. ASSAYERS’ WEIGHT. 563. Assayers' Weight is used by assayers in determining the Quantity of any particular metal in ores, or metallic compounds. TABLE. 1 Carat grain 2 Pwts. and 12 grains, or 60 grains Troy. 1 Carat 10 Pwts. Troy. 24 Carats 1 Pound Troy. NOTE.—1. This assay carat is entirely different from the carat in diamond weight. 2.—The term carat is also used by assayers to express the proportional purity, or the finenes; of gold, each carat meaning the twenty-fourth part. Thus, gold 16 carats fine has 16 parts gold and 8 parts alloy. 252 SOULE's PHILOSOPHIC PRACTICAL MATHEMATICs. CIRCULAR, OR ANGULAR MEASURE. 564. Circular, or Angular Measure is used in measuring angles, latitude and longitude, the location of vessels at sea, of planets, stars, etc. 565. The Standard Unit for measuring angles is the degree, which varies with the size of the circle. 566. A Degree is the angle measured by the arc of a #0 part of the circum: ference of a circle. 567. A Circle is a plane figure bounded by a curved line, every part of which is equally distant from a certain point within called the center. 568. The Circumference of a circle is the curved A. line by which it is bounded, as A. E. D. B. G. H. F. 569. The Radius of a circle is a line extending from its center to any point in its circumference, as C. E. H and C. D. 570. The Diameter of a circle is a straight line passing through its center and terminating at each end in the circumference, as A. B. 571. An Arc of a circle is any portion of the circumference, as B. D., D. E., etc. - *. 572. A Chord of a circle is a straight line drawn within a circle and ter- minating in the circumference, but not passing through the center, as F. G. 573. A Segment of a circle is any part cut off by a chord, as F. H. G. 574. A Sector of a circle is any part of a circle bounded by two radii and the arc included between them; as the space between C. D., C. B., and B. D. 575. An Angle is the opening or space between two lines or surfaces which meet in a common point, called the vertex. Thus, A. C. E., E. C. D., and D. C. B. are angles, and C. is their vertex. 576. A Semi-Circumference is one-half of a circumference, or 1800. 577. A Quadrant is one fourth of a circumference, or 90°. 578. A Sextant is one-siath of a circumference, or 600. 579. A Sign is one-twelfth of a circumference, or 300. TABLE. 60 Seconds (marked ”) = 1 Minute - '. C. S. O f f / 60 Minutes = 1 Degree - O. 1 = 12 = 360 = 21600 = 1296000 30 Degrees = 1 Sign - - S. 1 = 30 = 1800 = 108000 12 Signs, or 3600 = 1 Circle - - c. 1 = 60 = 3600 1 = 60 * MEASURES OF LENGTH, SURFACE AND SOLID. 253 THE OLD FRENCEL AND SPANISH MEASURES OF LENGTH, SURFACE, AND SOLID. 580. Louisiana having been both a French and a Spanish province, the old French and Spanish units of measure are often met with in private and public records; and to aid in understanding such units, the following table is presented: 581. OLD FRENCH SYSTEM. 1 Point 1 Line = 12 points 1 Inch = 12 lines 1 Foot = 12 inches 1 Ell = 43 inches 10 lines 1 Toise = 6 feet 1 Perch or Rod, (Paris) = 18 feet 1 Perch or Rod, (Royal) = 22 feet TABLE. ENGLISH AND AMERICAN MEASURE. .0074 English inches. .08884 English inches. 1.06577 English inches. 12.7892 English inches. 46.716 English inches. 76.7352 English inches. 19.1838 English feet. 23.447 English feet. 1 League (common)= 25 to a degree = 2280 toises = 14579.688 English feet = 2.761 miles. 1 League (post) = 2000 toises = 12789.2 English feet = 2,422 miles. 1 Fathom (brass) = 5 feet French = 63.946 English inches. 1 Cable length = 100 toises = 639.46 English feet = 106.58 English fathoms. 582. OLD SPANISH SYSTEM. 1 Foot = 11.1284 English inches. 1 Wara = 3 feet = 0.9274 English yard = 33.3864 English inches. . 1 Common League = 19800 Spanish feet. 1 Judicial League = 15000 Spanish feet. 583. OLD FRENCH SQUARE & CUBIC MEASURE. 1 Square inch = 1.13587 English Square inches. *1 Arpent (Paris) = 100 square perches, = 36804,120336 sq. feet, English. - 1 Arpent (Woodland) = 100 sq. perches (Royal) = 54978.994576 sq. feet, English. 1 Cubic inch = 1.2106 cubic inches, English. 1 Cubic foot = 2091.85 cubic inches, English. *Arpent is the old French name for acre. 584. COMPARISON OF THE FOOT OF DIFFERENT NATIONS, AS NOW IN USE. Russian foot (16 vershok)= 28 inches English or U. S. measure. Prussian foot (fuss)= 12.3567 in. English or U. S. measure. Bavarian foot (fuss)= 11.4204 in. English or U. S. measure. Hanoverian foot (fuss)= 11.4504 in. English or U. S. measure. Saxon foot (fuss)= 11.1494 in. English or U. S. Idea,SULT6. Austrian foot (fuss)= 12.4455 in. English or U. S. measure. Spanish foot (pié)= 11.1288 in. English or U. S. IOlea SULT6. French foot, old, (pied de roi)= 12.7892 in. Eng- lish or U. S. measure. NOTE.—France and Spain now use the metric system. Egyptian foot (cubit)= 21.3, 22.667, 25.13, and 26.5. Persian foot (gaz)= 25 common, 37.5 Royal, in. English or U. S. measure. Turkish foot (pic)=26.8 or 27.06 in. English or U. S. measure. Bohemian foot (fuss)= 11.88 in. English or U. S. Illea,SUIT6, Bremerian foot (fuss)= 11.386 in. English or U. S. measure. Danish foot (fod)= 12.3576 in. English or U. S. IOlea,SULT6: . Swedish and Norwegian foot (fot)= 11.6904 in. English or U. S. measure. ... • NOTE.—Sweden and Norway now use the metric system. Hamburgian foot (fuss)= 11.2896 in. English or U. S. measure. Japanese foot (shaku) = 11.948 in. English or U. S. measure. Mexican vara or yard = 32.97 in. English or U. S. measure. Portugese foot (palmo or span)= 8.6568 in, Eng- lish or U. S. measure. Arabian foot (guz)=25 in.; (covid) 19 in. English or U. S. measure. 254 SOULE's PHILOSOPHIC PRACTICAL MATHEMATICS. º: 585. TABLE OF COMPARATIVE WEIGHTS, MEASURES, AND VALUES Avoirdupois. Troy. Apothecaries. 7000 gr. = 1 lb. 5760 gr. = 1 lb. 5760 gr. = 1 lb. 1 lb. = 1+* lbs. – 1+ºr lbs. or 144 lbs. F 175 lbs. - 175 lbs. 1 Oz. - Hº Oz. - ## Oz. Or 192 OZ. F 175 Oz. - 175 oz. A Wine Gallon = 231 cubic inches. | Solidity of Cube = 1, Solidity of Sphere The Old Beer Gallon = 282 ( & & 4 diameter of which = one side of Cube = .5236 A Dry Gallon = 268.8 & 4 { { 1 oz. pure gold. . . . . . . . . . . . . . . . . ... = $20.67–H An Imperial Gallon = 277.274 “ 4 & 1 oz. pure silver. . . . . . . . . . . . . . . . . . = $ 1.29–H U. S. Bushel = 2150.42 cubic inches. | 1 p wit. pure gold.................. = 1.03 U. S. Bushel heaped = 2688. & 4 é & 1 pvt. pure Slver................. F .0645. An English Bushel 2218.192 * * & 4 Diameter of Circle = 1, Circumference = 3.1416 Area of a Square = 1, Area of Circle diameter of which = one side of the Square = .7854 1 pint of water weighs 1.0431 lbs. 1 Gallon. “ & 4 8.338882 lbs. 1 Cubic foot of water weighs 62.425 lbs. at 39.20 F. The Common year has 365 days. The Leap year has 366 days. The Solar year has 365 days, 5hrs., 48 min., 49.7 sec. A Yard. . . . . . . . . . . . = 36 in. 1 Milreis of Brazil... = 54.6 cts. I 1 Drachma of A Vara of Spain... = 33.3864 “ | 1 Rouble of Russia Greece. . . . . . . . . = 19.3 cts. A Vara. Of Mexico.. = 32.97 “ gold . . . . . . . . . . . = 77.2 ** | 1 Peso of Cuba. . . . = 92.2 “ A Meter. . . . . . . . . . . = 39.37 “ | 1 Yen of Japan gold = 99.7 “ | 1 Peso, or dollar of $1. . . . . . . . . . . . . . . . . . =100 cts. 1 Lira of Italy..... = 19.3 ** Mexico Silver... = 57.7 “ 1 Franc of France. . =19.3 “ 1 Peseta of Spain... = 19.3 “ | 1 Piaster of Egypt. = 4.9 “ 1 Mark of Germany. =23.8 “ | 1 Crown of Sweden. = 26.8 “ | 1 Piaster of Turkey = 4.4 “ 1 £ of England..... =$4.8665 “ | 1 Rupee of India 1 Sol of Peru silver. = 53.1 ‘‘ 1 Crown of Austria.=20.3 & 4 silver. . . . . . . . . . = 25.2 * * NOTE.-These monetary values are in terms of United States gold, October 1st, 1893. NoTE--See Table of Values of Foreign Coins. A Statute mile = 5280 ft. A Geographical, or Nautical mile, or knot, - 6086.41 ft. A Statute mile being 1, a Geographical mile is 1.15273–H 586. An acre contains 160 sq. rds., or 43560 sq. ft., and is 208,7103-1- ft. on each side. An arpent (Paris) contains 100 sq. rds. (Paris), or 36804,120336 sq. ft. (English), and is 191.1844 ft. (English) on each side. An arpent (Woodland) contains 100 sq. rās, (Royal), or 54978.994576 sq. ft. English, and is 234.476 English ft. on each side. NOTE.-See Metric System for other comparisons. BOOKS AND PAPER. SIZE OF PAPER. 587. Inches. Flat Letter . . . . . . . . . . . . . . . . . 10 × 16 Law Blank, or Small Cap ... 13 × 16 Flat Cap . . . . . . . . . . . . . . . . . . . . 14 × 17 Crown. . . . . . . . . . . . . . . . . . . . ... 15×19 Demy . . . . . . . . . . . . . . . . . . . . . . . 16 × 21 Folio Post. . . . . . . . . . . . . . . . . . . 17X22 Check Folio . . . . . . . . . . . . . . . . 17X24 Medium—Writing. . . . . . . . . . . 18×23 Medium. . . . . . . . . . . . . . . . . . . . . 18×24 Inches. Extra Size Folio. . . . . . . . . . . . . . . . 19 × 23 Medium—Writing and Printing. 19×24 Royal—Printing. . . . . . . . . . . . . . . . 20X24 Medium–Printing. . . . . . . . . . . . . 20X25 Double Cap . . . . . . . . . . . . . . . . . . . . 17 × 28 Super Royal—Writing.......... 20X28 Card Board & Sup. Royal, Print’g 22×28 Imperial—Writing . . . . . . . . . . . . . 22X30 Imperial—Printing. . . . . . . . . . . . . 22 × 32 NOTE.-Printing paper is made of many sizes, according to the requirements of the printer. In book printing 24 × 38 inches, called double medium, is used very largely. WEIGHT OF GRAIN AND PRODUCE PER BUSHEL. 255 588. In copying legal papers, recording deeds, etc., clerks are usually paid by the folio. Thus: 100 words make 1 folio in New York, and New Orleans; 72 words make 1 folio in Com. Law; 90 words make 1 folio in Chancery. In printing books 240 impressions, or 120 sheets printed on both sides, make 1 token. A sheet (medium) folded in 2 leaves is called folio. & & ( & & & 4 & 4 { { & & 8 & 4 & & & 4 12 & 4 & 4 & 4 16 & 4 4 & & & 18 & 4 & 4 & 4 24 & 4 & & & 4 32 24 Sheets = 1 Quire. 480 Sheets = 20 Quires = 1 Ream. 2 Reams = 1 Bundle; 5 Bundles = 1 Bale. 12 Units = 1. Dozen. 144 Units = 12 Dozen = 1 Gross. 12 Gross = 1 Great Gross. º 20 Units = 1 Score. 56 lbs. = 1 Firken of Butter. 100 lbs. = 1 Quintal of Dried Fish, 196 lbs. = 1 Barrel of Flour. 200 lbs. = 1 Barrel of Flour in California. 200 lbs. = 1 Barrel of Beef, Pork, or Fish. 280 lbs. = 1 Barrel of Salt. 100 lbs. = 1 Cask of Raisins. 14 lbs. Iron or Lead = 1 Stone. A Stone of Fish or Meat = 8 lbs. & & 4 & “ quarto or 4to. octavo or 8vo. 16mo. 18mo. 24mo. 32mo. A Stone of Glass A Seam of Glass 21; Stones 8 Pigs 7 lbs. Wool 2 Cloves 2 Stones 6}. Tods 2 Weys 12 Sacks A Truss of Hay A Truss of Straw 256 Pounds of Soap 25 Pounds of Powder 12 Barrels of Wheat duodecimo or 12mo, 5 lbs. 24 Stones. 1 Pig. 1 Fother. 1 Clove. 1 Stone. 1. Tod. 1 Wey. 1 Sack. 1 Last. 56 lbs. 36 lbs. 1 Barrel. 1 Keg. 7 English Quarters. 589. *WEIGHT OF GRAIN AND PRODUCE PER BUSHEL, AS USED IN NEW ORLEANS WHEN THERE IS NO AGREEMENT TO THE CONTRARY. Wheat. . . . . . . . . . . . . bush. 60 lbs. Clover Seed. . . . . ... bush. 60 lbs. | Dried Peaches. . . . . bahs. 33 lbs. Corn. . . . . . . . . . . . . . . “ 56 “ | Timothy Seed...... “ 45 ‘‘ | Dried Apples. . . . . . ‘‘ 24 Rye. . . . . . . . . . . . . . . “ 56 “ | Barley Malt..... ... “ 34 “ | Onions. . . . . . © e º e o º “ 56 “ Oats. . . . . . . . . . . . . . . “ 32 “ | Peas, Split......... “ 60 “ | Coarse Salt... . . . . . “ 50 * * Barley. . . . . . . . . . . . . “ 48 “ Small Hominy..... “ 50 “ | Fine Salt. . . . . . . . . . “ 50 “ Irish Potatoes... . . . “ 60 “ | Flaxseed. . . . . . . . ... ** 56 ‘‘ | Stone Coal. . . . . . . . ** 80 * * Sweet Potatoes. ... “ 60 ‘‘ | Hempseed. . . . . . . . . “ 44 “ | Corn Meal......... “ 44 “ . Beans. . . . . . . . . . . ... 62 “ | Buckwheat. . . . . . . . “ 52 “ Plastering Hair.... “ 7 “ Bran. . . . . . . . . . . . . . . “ 24 “ | Castor Beans....... “ 46 “ | Blue Grass Seed. ... “ 10 “ tidim ºf Dºminate Numbers. -------------------------------------N *—aſh- A NOTE.—For the Metric System of Weights and Measures, see Appendix. 590. Reduction is the operation of changing an expression in one or more denominations to an equivalent expression in some other denomination or denomin- ations of the same kind of measurement: Thus, $10 may be changed to its equiva- lent 1000 cents; 36 inches may be changed to its equivalent 3 feet, and 5 lbs. 8 oz. avoirdupois may be changed to its equivalent 54 lbs., or 88 ounces. Reduction is of two kinds, descending and ascending. 591. Reduction Descending is changing the forms of denominate quan- tities from a higher to a lower order of units, or denomination; as in changing or reducing dollars to cents, pounds to grains, feet to inches, etc. 592. Reduction Ascending is the converse of reduction descending, and hence it is the changing of the form of denominate quantities from a lower to a higher denomination; as cents to dollars, grains to pounds, inches to feet, etc. REDUCTION DESCENDING. TO REDUCE A SIMPLE DENOMINATE NUMBER TO A LOWER DENOMINATION, (IN THE SAME SYSTEM OF MEASURE). 593. 1. Reduce 6 feet to inches. OPERATION. 6 feet. Ea:planation and Reason.—By considering the conditions 12 inches. of the problem, we see that we are required to reduce 6 — –— feet to inches. i. e. to find the equivalent of 6 feet in the | 72 inches, Ans. 2 Q. unit inches. Before we can perform the operation, we ft. º, must know the units of the lower denominations from 6. 0 feet to inches. And by referring to the Table of Linear 12 Measure, we find that 1 foot = 12 inches. This gives the 6 premise for the solution and from it we reason as follows: *E*- Since 1 foot is - to 12 inches, 6 feet are = to 6 times as 72 inches, Ans. many, which is 72 inches, the answer. 2. Reduce 3 bushels to pints. FIRST OPERATION. bu. pks. Qts. pts. Ea:planation and Reason.—In this problem, we are re- 3 0 0 quired to reduce bushels to pints; but before we can ===s** sºma ººm. perform the operation we must know either the different 4 8 2 units of the lower denominations from bushels to pints, or the equivalent of one bushel in pints. By reference to 3 12 96 the Table of Dry Measure, we see that 1 bushel = 4 pecks; ºme ººm- 1 peck = 8 quarts; and 1 quart = 2 pints. These equivae 12 96 192 pts., Ans. lents furnish our premises, and from them we develop the 7 Solution. We first write the problem as shown in the operation, filling all vacant denominations, from (256) Yºr REDUCTION DESCENDING. 257 bushels to the denomination required, pints, with naughts; then below each denomination we draw a line and write thereunder the number of units of each order which make one of the next higher order. Having thus stated the problem, we reason as follows: Since 1 bushel is = to 4 pecks, 3 bushels are = to 3 times as many, which is 12 pecks; then since 1 peck is = to 8 quarts, 12 pecks are = to 12 times as many, which is 96 quarts; then since 1 quart is = to 2 pints, 96 quarts are = to 96 times as many, which is 192 pints, the answer. i.e. If we work from the basis of the number of pints in a bushel, we would reason thus; 1 bushel †gº º: pints: Since 1 bushel is - to 64 pints, 3 bushels are = to 3 times as many, which is pln US. SECOND OPERATION. 3 bu. 3 bu. Explanation.—In this operation, we reason as follows: 4 tº: In one bushel there are 4 pecks, and in 3 bushels there 12 pks. or 2 pts. are three times as many pecks, which is 12; then in 1 eck there are 8 quarts, and in 12 pecks, there are 12 192 pts, Ans. " quarts, 9 96 QtS. times as many, which is 96; then in 1 quart there are 2 2 pints and in 96 quarts there are 96 times as many, which 192 pts., Ans. is 192 pints. 3. Reduce $7 to mills. Ans. 7000 m. 4. Reduce 3 shillings to farthings. Ans. 144 far. 5. Reduce 4 pounds Troy to pennyweights. Ans. 960 pvt. 6. Reduce 5 bushels to quarts. Ans. 160 qts. 7. Reduce 3 days to minutes. Ans. 4320 m. 8. Reduce 20 to ". AnS. 7200 ". * 9. Reduce 2 square feet to square inches. Ans. 288 sq. in. 10 . Reduce 16 pounds avoirdupois to grains. Ans. 112000 grains. TO REDUCE A COMPOUND DENOMINATE NUMBER TO A LOWER DENOMINATION, (IN THE SAME SYSTEM OF MEASURE). 594. 1. Reduce 3 bu., 2 pks., and 1 pint, to pints. OPERATION. bu. pks. Qts. pts. Ea.planation and Reason.—Here we are required to deter- 3 2 () 1. mine the number of pints in the whole expression. We º t_º ºmº first state the problem as shown in the operation, filling 4 8 2 the vacant units or denominations in the scale with a naught and writing under each term the number of units 3 14. 112 of that order which make one of the next higher order. * *mºmºsºme mºº-ºº: Having thus stated the problem, * º * *: : Since 1 bushel = 4 pecks, 3 bushels are = to 3 times as 14 112 225 pts., Ans. many which is 12 º + the 2. pecks = 14 pecks; then since 1 peck = 8 quarts, 14 pecks are = to 14 times as many, which is 112 quarts; then since 1 quart = 2 pints, 112 quarts are = to 112 times as many, which is 224 pints –H 1 pint = 225 pints, answer. º In all problems of like character to the three preceding, the form of operation and the pro- cess of reasoning here given should be used. - 2. Reduce 3 £, 2s. 8d. 3 far, to farthings. Ans. 3011 far. 3. Reduce 6 lbs. 12 oz. 13 drS. to drams. - Ans. 1741 drs. 4. Reduce 5 lbs. 2 oz. 12 grs. to grains. -- Ans. 29772 grs. 258 SOULE's PHILOSOPHIC PRACTICAL MATHEMATICS, Af. 5. Reduce 3 rods 20 links 6 inches to inches. Ans. 758.4 or 758% in. 6. Reduce 3 f.oz. 4 f.dr.s. 20 m. to minims. Ans. 1700 m. 7. Reduce 3 cable-lengths 5 fathoms to feet. Ans. 2190 ft. 8. Reduce 3 rds. 12 ft. 6 in. to inches. Ans. 744 in. 9. Reduce 6 hbds. 42 gals. 3 qts. 3 gills to gills. Ans. 13467 gills. - TO REDUCE A FRACTIONAL DENOMINATE NUMBER TO A LOWER DENOMINATION, (IN THE SAME SYSTEM OF MEASURE). 595. Reduce # of a gallon to gills. OPERATION. 8 || 3 gallons. Explanation and Reason.—In this problem, we are re. 4 quarts. quired to find the equivalent of § of a gallon, in the unit 2 pint of gills. By referring to the Table of Wine Measure, we pints. see that 4 gills = 1 pint ; 2 pints = 1 quart ; 4 quarts = 4 gills. 1 gallon. Then with this data as our premises, we write º-e mº- the # of a gallon on the statement line and reason as 12 gills, Ans. follows: Since 1 gallon = 4 quarts, there are 4 times as many quarts as gallons; then since 1 quart = 2 pints, ſº e there are two times as many pints as quarts; then since 1 pint = 4 gills, there are 4 times as many gills as pints. This completes the reasoning, and by Working out the statement, we obtain 12 gills as the equivalent of # of a gallon. 2. Reduce # of a dollar to cents. Ans. 60 cts. 3. Reduce # of a pound (£) to farthings. Ans. 800 far. 4. Reduce ºf of a Troy pound to grains. Ans. 1728 grs. 5. Reduce fºr of a yard to inches. Ans. 13+ in. 6. Reduce # of a bushel to pints. Ans. 27# pts. 7. Reduce # of a week to seconds. Ans. 403200 sec. 8. Reduce # of a franc to millimes. Ans. 285% m. 9. Reduce # of a mark to pfenniges. Ans. 75 pf. 10. Reduce # of an acre to sq. in. Ans. 5488560 sq. in. § * * * * * * * * * * * * 607. A—a 4- A ºr r–Wurr-w Addition of Compound Denominate Numbers is the process of uniting two or more compound denominate numbers into one equivalent number. The process of adding is the same as adding simpſe numbers, except that the scale of increase and decrease, by passing from one denomination to another, varies with every system of measurement and with almost every denomination of each system. We shall, therefore, not discuss the subject at length, but shall proceed to illustrate the process by which results in compound addition are determined. 1. Add 5 bu. 3 plºs. 7 qts. 1 pt., 8 bu. 3 qts. 1 pt., and 3 bu. 3 p.ks. 1 pint. OPERATION. bu. pks. Qts. pt. 5 3 7 1 8 O 3 1. 3 3 () 1 17 3 3 1, Ans. 3 pts. -- 2, No. of pts. in a Qt., = 1 qt, and 1 pt. 11 qts. + 8, No. of qts. in a pk., = 1 p.k. and 3 qts. 7 pks. -- 4, No. of pks. in a bu., =1 bu. and 3 pks. Explanation.—In all problems of this kind, we first write the numbers so that units of the same denomina- tion stand in the same column, and begin with the lowest denomination to add. Accordingly, we here first add the pints and find the sum to be 3 pts., which we divide by 2, (since 2 pts. = 1 qt.) and obtain 1 qt. and 1 pt. remainder. The 1 pt. we write under the column of pts., and carry or add the 1 qt. to the column of qts., which added gives 11 qts. and which we divide by 8, (since 8 qts. = 1 pk.) and obtain 1 pk. and 3 qts. remainder. The 3 qts. we write under the column of qts., and add the 1 plc. to the column of plºs., which added gives a sum of 7 plºs. This we divide by 4 (since 4 plºs. = 1 bu.) and obtain 1 bu, and 3 p.ks. The 3 pks. we write under the column of plºs. and add the 1 bu. to the column of bushels, which gives a sum of 17 bu., which we write under the column of bushels. This completes the operation and produces 17 bu., 3 pks., 3 qts., 1 pt. as the complete result. 2. Add 1536. 10s. 9d., 8:6. 9s. 7d., 14%. 12s. 10d., and 13. 18s. 6d. * Ans. 273E. 11s. 8d. 3. Add 12 yds. 3 qrs. 2 na. 2 in., 5 yds. 2 qrs. 1 na. 1; in., 2 q1's. 1. na. 13 in., 8 yds. 2 in. Ans. 27 yds. 0 qrs. 3 na. 4 in. NOTE.-In working the above problem, the student must remember, when dividing the . inches, that whenever the dividend is fourths, the remainder is also fourths. The remainder is always like the dividend. 4. Add 10 lbs. 13. 63.0 9. 10 grs, 103.23. 19. 16 grs, and 13 lbs. 23. 7 3. 3 B. 12 grs. Ans. 24 lbs. 33. 0 3. 2 9. 18 grs. 5. Add 3 yds. 2 ft., 9 in., 4 rās. 2 yds. 1 ft. 11 in., and 5 rids. 4 yds. 2 ft. 8 in. Ans. 11 rds. 0 yd. 1 ft. 4 in. 6. Add 21 gals. 3 qts. 1 pt. 3 gi., 32 gals. 1 qt. 0 pt. 2 gi., and 47 gals. 2 qts. 1 pt. 1 gi. Ans. 101 gals. 3 qtS. 1 pt. 2 gi. 7. What is the sum of the following quantities, Troy weight? 25 lbs. 10 oz. 15 pwt. 18 grs.; 42 lbs. 9 oz. 20 grs.; 26 lbs. 4 oz. 7 pwt.; 11 oz. 19 pwt. 23 grs.; 100 lbs. 6 oz. 14 pvt. 16 grs. Ans. 196 lbs. 6 oz. 18 pwt. 5 grs. (267) 892 ºt "so1.LVINAHLvM TVoILoviſa or HaOSOTIH.I s.3Tnos "su V, OT #I 9T g Of = "upur # 09 19 = "Itſ 3 () 99 6 = "ep # 0 0 9 g = "x{A : *999 ‘puſ “IUI •ep *OIGITVOIOINI NOITVºIGIdſ() *oes 0I 'upul FI ‘silſ 9T ‘ep g suv "I SI p.1 g 'ud gT "Iuſ # "suv & "Io 9; ‘IUI 9T put: “I aſ 'pi I ‘uo T9 put 1.I “I LI "sp. 9 “uo 19 ſul 6 pp V '6 "up "no 9;&T "J no 02 "SpA “no 09 prº “up "no Ig6 aſ "no gT ‘sp/ "no 9T “uſ "no IgEI "ly "no gT "Spä "no SI ppV '8 "up "no gg.T "J no 02 "sp/ "no go ‘suv 'uſui & put 'III 3 'ep # "HA # ppy 'Ol 13 Ig ggz ‘suy IZ If &I OI gT 89 89 89 8 gî 33 89. #2, jºg 8I 99 83 - 73 99 89 & 99 ſº £1. 83 #9 gT •oes ºut 3op •oes ºur 'll *@IºIſl'SVGIW HVTſ) OHIO I I g L, 383 "SUV 9. I g 39 97T I I I 39 paſ 9 0 & § ; º, I g f Tā ‘ºd # "[e: "puu *GI?IſlSWGIWI GINIAA ‘I FI pr 9 ºf giff 'suw I & g zz8I "sury 9 89 99 I 1. 3. #39 f £2 00, I g I 997 9 09 Ojº, 0 9 & º: # ºr ºf * * * * *GISIſlSWGIWI (IWIL #09I ſº iſ 69 I ‘suy 000T g|I 93), I 6 TIII &I #3&I A, 39 g ºf 9 98 #I D, ſº ºup "no “ly ‘no 'ly "o 'O GI*IſlSWGI WI ()I{Iſl{) HO OIITOS *GISIſlSWGIWI X?IOI. 19 9 II g I j92. If "sury 36 & 33 33 & 333 33 97'ſ 8 09 69 g 689 P. #f 3 f6, 99 & 099 7 00I '8 Fº g3 g O&T 9 •up •bs *†J. •bg tº •0S "di "QI ‘V. •UI •bs "GISIſn's WCIWI (ISIVſmöS 9 I f iſ I () 83 °SUV I & # 99T 'Suv # & g g!99 "SuW 8 I ? ), iſ 0 if 03 0 9 P. ØØ 6 & 8, 69 g T 9 O % g fºg & & 3 & 33 TY O 9 9T 0 & 9 & 9 I 919 QI 3 g 8 9T & I g 7% A. I g 3 & 9 #96 g I D, 6 97 9 & 7 gº 9 & 0. T I & 00&I 6T & 9 TT P3 "up qJ ‘pA ‘pt "Ing. "UI “I •up “bu 'lb "pā. 'aff ‘aos ‘ip 20 °g "QIYIſlSWGIWI &IWGINIT 'Innsvºw H.LOTO "LH})IGIAA SGII*IWOGIHALOGIW 33 8 T 6Ig "sury . . . ZI g WI & 6 WIZ "SUV - 9I OI 0 #9 ~ 9, 8 9. § 8 99 II 1.I 8 Q9 6 II 6T & TI ſy 03 f OT 18 f 6 9T 0 8T g/, I 6I 1. 99 ZI 9T fº, 8 gT 09 33 9I 6 jº, “Ip "zo q, "Ib "1Ao ‘J. "EH})I{HAA SIOdiſlolº[IOAW *13 q Ad "ZO "g! "J.H})IGIAA XO'HAL : 5uſAOTIOJ Qū) ppy ālūqin ſºul Dulut Nulls. ------------------------------------------------Rs —ſh_rº añ w-w-w 608. Subtraction of Compound Denominate Numbers is the process of decreasing one compound denominate number by another of the same system of ImeaSurement. As in addition, the scale of increase and decrease varies, otherwise the work is the same as in subtraction of simple numbers. 1. From 1836. 4s. 7d. 3f. subtract 11:6. 9s. 11d. 2f. OPERATION. :8. S. d. f. Ea:planation.-In all problems of this kind, we first 18 4 7 3 write the numbers so that units of the same denomina- 11 9 11 2 tion stand in the same column, and begin with the lowest denomination to subtract. Accordingly, we here commence with the farthings and say 2 far. from 3 far. 6 : 14 8 1 Ans. leaves 1 far. which we write under the column of far- things. We now come to the column of pence and observe that 11d. cannot be taken from 7d., because the 11d. is the greater number; we therefore, according to the law that the difference between two numberg is the same as the difference between the two mumbers when equally increased, as demonstrated in Article 129, add 12d. to the 7d., making 19d. From this we subtract the 11d. and have 8d. remainder, which we write under the column of pence. Then, as we added 12d. to the minuend, we, now, to compensate therefor, according to the above law, add 1s., the equivalent of 12d., to the column of shillings in the subtrahend and thus we have 10s. to subtract from 4s. which we cannot do. Hence, for reasons above given, we add 20s. to the 4s, which makes 24s., from which we take 10s. and have 14s. remainder, which we write under the column of shillings. We now add 1:6, the equivalent of 20s, to the 1136, making 12.É., which we take from 1836 and have a remainder of 636, which we write under the column of pounds. This completes the operation. It will be observed that when we added the 12d. to the column of pence, and the 20s. to the column of shillings, that, in each case, we added such a number of that order as made one of the next higher order. This must always be done in simple numbers or in any of the systems of com- pound numbers, when the subtrahend number or denomination exceeds the minuend number of like denomination. 2. From 45 bbls. 19 gals. 2 qts. take 24 bbls. 24 gals. 3 qts. OPERATION, gº bbl. gal. Qt. Fæplanation.—We proceed in this as in the previous 45 19 2 example, to find the several denominate remainders; 24 24 3 but since one of them, the 25% gals., is partly fraction- al, we reduce the # gallon to quarts, equal to 2 qts., 20 25; 3 which we add to the 3 qts. in the remainder, making 2 5 qts., equal to 1 gallon and 1 quart; we set down the 1 qt, and add the 1 gal. to the 25 gals., giving 26 gals, 20 26 1 for the final result. 3. From 3 mi, 5 fur. 30 rds. take 1 mi. 7 fur. 32 rās. Ans, 1 mi. 5 fur, 38 rids. 4. From 7 lbs. 3 oz. 12 pywt. 20 grs. take 3 lbs. 5 oz. 10 pwt. 15 grs. - Ans. 3 lbs. 10 oz. 2 pwt. 5 grs. (269) 27o SOULE's PHILOSOPHIC PRACTICAL MATHEMATICs. 5. From 30O 25' 32” take 25o 34' 357/. Ans. 4o 50' 57". 6. From 4 lbs. 12 oz. 13 drs. take 2 lbs. 9 oz. 15 drs. - Ans. 2 lbs. 2 oz. 14 drs. 7. From 13 hrs. 24 min. 7 sec. take 10 hrs. 30 min. 12 sec. Ans. 2 hrs. 53 min. 55 sec. 8. From 3 sq. rds. 5 sq. ft. 95 sq. in. take 1 sq. rd. 10 sq. ft. 100 sq. in. Ans. 1 sq. rd. 267 sq. ft. 31 sq. in. SOLUTION. sq. rd. sq. ft. Sq. in. Explanation.—By proceeding as in the explanations 3 5 above given, we obtain 1 sq. rā. 2664 sq. ft. 139 sq. in. 10 100 as a result; but it is not proper to have a fractional expression in any but the lowest denomination of a compound denominate number, so we proceed to reduce 266+ 139 # of a sq. ft. to sq. in.; # of 144 sq. in. = 36 sq. in. ; # = 36 139 sq. in. -- 36 sq. in. = 175 sq. in., or 1 sq. ft. an #: *; àY. #: 31 sq. lº º # ft. #. SCI. Itſ, all Ol IIIl © all SWOT to O6 L SC. TC1, SC - IUe 267 31 31 º in. Q. Q1 9. Subtract the following: A. R. P. sq. yd. sq. ft. sq. in. 40 3 22 30 4 10 16 3 30 35 5 100 23 3 31 24+ 7 54 # = 2+ # = 36 Ans. 23 3 31 25 0 90 10. From # of a hogshead, subtract 3 of a gallon. Ans. 37 gal. 0 qt. 1 pt. 0# gi, OPERATION INDICATED. # of a hind. = 37 gal. 3 qt. 0 pt. 1; gi. # of a gal. = 2 qt. 1 pt. 14 gi. 37 gal. 0 qt. 1 pt. 04; gi. 11. From 2% bu. Subtract # of a peck. Ans. 2 bu. 1 p.k. 4 qt. 143 pt. 12. From 555 A. 3 R. 30 P. take .335 of a square mile. Ans. 341 A. 2 R. 6 P. 13. From 5 m. 4 fur. 24 r. take 2 m. 6 fur. 37 r. 3 yd. 2 ft. 9 in. g Ans. 2 m. 5 fur. 26 rā. 14 yd. 0 ft. 3 in. 14. From the sum of # of 365+ days and # of 23 hours, take # of ºr of 154 minutes. Ans. 156 d. 14 h. 47 m. 25* Sec. 15. From the sum of # of 54 miles and .875 furlongs take # of 24 yards. Ans. 2 m. 2 fur. 34 rā. 34 yd. 0 ft. 4; in. 609. Multiplication of Compound Denominate Numbers is the process of determining the product of two numbers, when the number to be multiplied is a compound denominate number. Compound multiplication differs from multiplication of simple numbers, in that when the product in any denomination equals or exceeds one of the next higher denomination in the same system of measurement, said product must, by a process of division, be reduced to that next higher denomination, before the number in it can be multiplied. 1. Multiply 4 gals. 3 qts. 1 pt. 2 gi. by 9. OPERATION. gal. qts. pt. gi. Explanation.—In all problems of this kind, we write 4 3 1 2 the multiplier under the lowest denomination of the 9 multiplicand and multiply. . We first say 9 times 2 gi. are 18 gi. which, as shown in the operation, equals 4 pts. and 2 gi. The 2 gi. we write under the gills and 44 1 1 2 reserve the 4 pts. to add to the product of pints. We then say 9 times 1 pt. are 9 pts, and 4 pts. added make 18 gi. -- 4 = 4 pts. 2 gi. 13 pts. which reduced to the next higher denomination 13 ot . . 2 = 6 otS. 1 pt equals 6 qts. and 1 pt. The 1 pint we write under the plS. + 2 = 0 qLS. L. pt. unit or denomination pints, and reserve the 6 qts, to 33 qts. + 4 = 8 gals. 1 qt. add to the product of quarts. Then we say 9 times 3 qts. are 27 qts, and 6 qts. added make 33 qts. equal to 8 gals. and 1 qt. The 1 qt. we write under the quarts, and reserve the 8 gals. to add to the product of gallons. Lastly we say 9 times 4 gals. are 36 gals. plus the 8 gals. reserved, are 44 gals, which WO .* under the denomination of gallons. This gives 44 gals. 1 qt. 1 pt. 2 gi. for the entire product. & NOTE. 1.-The multiplier must always be considered an abstract number, Art. 149, page 75. f NOTE. 2.--When the multiplier is large, and a composite number, we may multiply by the actors. 2. Multiply 7 Cong. 3 O. 15 fº by 6. Ans. 44 Cong. 7 O. 10 fº . 3. Multiply 27 cords 34 cu. ft. by 12. Ans. 327 cords 24 cu. ft. 4. Multiply 3 mi. 7 fur. 20 rds. by 7. Ans. 27 mi. 4 fur. 20 rds. 5. Multiply 7 hrs. 30 mi. 24 sec. by 5. Ans. 1 da. 13 hrs. 32 min. 6. Multiply 3 francs, 7 dec. 3 cen. 4 mil. by 5. Ans. 18 fr. 6 dec. 7 cen. 7. Multiply 43. 3s. 4d. by 12. - Ans. £50. 8. In 5 bbls. of pecans, each containing 2 bu. 3 p.k.s. 5 qts. 1 pt., how many bushels? Ans. 14 bu. 2 pks. 3 qts. 1 pt. (271) 272 SOULE's PHILOSOPHIC PRACTICAL MATHEMATICS. ºr 9. Multiply 25 gallons, 3 quarts, 1 pint, by 3.5. OPERATION. 25 gals. 3 qts. 1 pt. = º ptS. 2) 724.5 pt.. 4 3. *-* -*= 4) 362 qts. and .5 or + pt. 103 1035 2 621 90 gals. 2 qts. -* Ans. 90 gals., 2 qts, # pt. 207 724.5 pt. 10. How many bushels in 12 barrels of grain, each barrel containing 4 bus., 3p};S., 5 qts., 1 pt. 2 Ans. 59 bus, 2 qts. 11. How many gallons in 15 casks of wine, each cask containing 42 gallons, 2 quarts, 1 pint, and what will they cost at $2.48 per gallon 3 Ans. Contents: 639 gals., 1 qt., 1 pt. Cost: $1585.65. 12. A man has 7 lots of land, each containing 24 A., 3 R., 25 P. How many acres has he altogether, and what are they worth at $15.65 per acre ; Ans. 174 A., 1 R., 15 P. Cost: $2728.47#. 13. If a boy studies 5 hours 25 minutes each day, how many days of 24 hours each will he study in 54 days 3 Ans. 12 ds., 4 hrs., 30 m. 14. What is the distance around an octagonal or eight sided field, each side measuring 1 fur., 25 ras, 3 yds., 2 ft., 8 in...? . e Ans. 1 mi., 5 fur., 5 rds., 3 yds., 1 ft., 10 in. 15. A man bought 25 pieces of broad, cloth, each containing 32 yds., 3 grs, 2 na. How many yards did he buy, and what did they cost at $5.84 per yard Ans. 821 yds., 3 qrs., 2 na. Cost: $4799.75. A 4 acres. x 28 compasses. B SPECIAL PROBLEM. A farmer has a piece of land that he measures with a pair of compasses whose points are 6 feet apart, and finds the following dimensions of his land: A to B 4 acres 28 compasses; A to C 4 acres 13 compasses; C to D 2 acres 9 compasses. How many acres in the plat of ground 3 Ans. 15 acres, 98 sq. rods, 16 Sq. yds., 8 sq. ft., 5.52 sq. in. REMARKS.–The compasses for measuring land is a primative instrument in the form of compasses, whose points are 6 feet apart. 35 spans or units of the compasses (210 feet) is consid- ered an acre for approximate work on a farm. This is incor- rect, as an acre or square piece of ground containing one acre, ſº º is 208,7103 ft. on each side. See Comparative Table on page #54 of this book, also table on page 244. Seepage 253 for arpent, Whieh is the old French word for acre. SOLUTION. 1: "Eé"*}918 ft. A to c. | i C 2 acres 9 com. D - 2 & & e #"Tºº $47 ft. c. to D x 918–495132 sq. ft. in the main body of land. Then 4 × 210 = 840 ft. *::"E}}#} 1008 feet A to B. Less - - - 474 “ C to D. Gives the base of *=º triangle - - - - 534 “ × to B. Then 534×918 A to C – 2 for the triangle = 245106 sq. feet. Total sq. feet in the plat of land e- 680238 ‘‘ ‘ ‘ 680238 – 43560 sq. feet in an acre, gives 15.616 -i- acres. .616x160 = 98.560 sq. rods. .56×30+ = 16.94 sq. yds. .94× 9 = 8.46 SQ. ft. .46 × 12 = 5.52 sq. in. t MoTE.—See page 254 for Paris and Woodland Arpents. 3) in ºf CII Dºmint Ninters • * > e s tº e º 'º e º e º & e º - º 'º º============N 610. Division of Compound Denominate Numbers is the process of deter. mining any required part of a compound denominate number. It is the reverse of compound multiplication, and we first divide the highest denomination in the number, and if any remainder occurs in the result, it must be reduced to the next lower denomination before the number in that denomination is divided. 1. Divide 32 lbs. 12 oz. 12 drs. by 5. OPERATION. lbs. OZ. dr. Explanation.-In all problems of this 5)32 12 12 kind, we write the quantity to be di- vided in the order of its denominations 6 8 15%, Ans. and place the divisor on the left, as in division ofº numbers. -- h = ſº Having thus stated the problem, we 32 lbs. -- 5 = 6 lbs. and 2 lbs. remainder. . first divide the 32ibs. By 5 and obtain * a quotient of 6 lbs. with 2 lbs. remain- 2 lbs. × 16 = 32 oz. -- 12 oz. = 44 oz. der. We write the 6 lbs. in the quo. 44 oz. -- 5 = 8 oz. and 4 OZ. remainder. tient line under the pounds, and re- duce the 2 lbs. remainder to ounces = 32 oz. –H 12 oz. = 44 oz. which we divide 4 oz. × 16 = 64 drs. -- 12 drs. = 76. drs. by 5 and obtain 8 oz. with a remainder 76 drs. -- 5 = 15 drs. and 1 dr. remainder. of 4 oz. The 8 oz. we write in the quotient line, under the ounces, and 1 dr. -- 5 = + dr. reduce the 4 oz. to drams, = 64 drs. -- 12 drs. = 76 drs. which - 5 = 15 drs. with 1 dr. remainder, or 15% drs. ; this is written in the quotient line under the drams, and completes the operation. NoTE.—When the divisor is large, and a composite number, we may divide by the factors. 2. A box contains 8 bu. 3 p.k.s. 5 qts. How many smaller boxes, each holding 1 pk, 1 qt. 1 pt., can be filled from the larger box 3 Ans. 30 boxes. OPERATION INDICATED. 8 bu. 3 pks. 5 qts. = 570 pts. Explanation.—In all problems of this kind, 1 plc. 1 qt. 1 pt. = 19 pts. we first reduce both dividend and divisor to the same denomination, and then divide as in simple ^ 570 pts. -- 19 = 30 boxes, Ans. numbers. 3. Divide 37 mi. 3 fur. 4 rās. by 4. Ans. 9 mi. 2 fur. 31 ris. Divide 60 24, 32” by 7. Ans. 54' 56”. Divide 24 lbs. 3 oz. 12 pvt. 12 grs. by 12. Ans. 2 lbs. 0 oz. 6 pvt. 1 gr. Divide 25 yds. 3 qrs. 2 nails by 13. Ans, 1 yd. 3 qrs. 3 nails, 14% in. Divide 3 cu. yas. 19 cu. ft. 996 cu. in. by 12. Ans. 8 cu. ft. 659 cu. in. : (273) 274 SOULE's PHILOSOPHIC PRACTICAL MATHEMATICS. Yºr 8. Divide 3 mi. 75 ch. 21 l. by 7. Ans. 45 ch. 3 1. 9. What is 3 of 2 m. 3 fur. 1 yd. 2 ft. Ans. 7 fur. 5 rds. 1 ft. 104 in. 10. If 300 families live in one township, and the land be equally divided, how large a homestead will each have 3 Ans. 76 A. 3 R. 8 P. 11. How long will it take 4 men to excavate a cellar 16 feet square and 12 feet deep, if 1 man removes 3 cubic yards and 15 cubic feet in 1 day ? Ans. 8 days. 12. If a man travels 771 miles in 30 days, what distance does he pass over in each day ? Ans. 25 mi. 5 fur. 24 rol. 13. If one spoon Weighs 3 oz. 12 p wt. 12 gr., how many dozen spoons can be made from 38 lbs. 11 oz. 12 pywt. 12 gr. of silver ? Ans. 10; dozen. 14. If a man steps 2 ft. 94 in., how many steps will he make in going 334 miles? Ans. 63360 steps. 15. If the distance around the earth were 25000 miles, and a man could travel 33% miles each day, how long would it take him to make the circuit of the globe, allowing 365+ days to the year 7 Ans. 2 years 19 days 12 hours. /To TN COMPARISON OF THERMOMETERS. f. Ar c 611. There are three scales in use for registering temperature : ¥ so-ſoo 1st. The Fahrenheit (marked F.) in which the freezing point 200 H of water is marked 329, and the boiling point 2120. Zero is 32° 190 – Hoſſ" below freezing in this scale. 180H —lso NotE.—This F. scale is generally used in the United States for ordinary 170H || |60 R. household and business purposes. \ * º 160 — 170 2nd. The Centigrade (marked C.) in which the freezing point isol—|| of water is marked 09, and the boiling point 100°. 14OH- Ti”9| |60 NOTE.—This C. scale is used in France and several countries in Europe, 130 H r and by scientists, generally. isol |H|40H50 3rd. The Reaumur (marked R.) in which the freezing point of to – |—|40 water is marked 0°, and the boiling point 80°. g 100 H|||—|30ſ NoTE.—This R. scale is used generally in Germany and Spain. 90 – —lsº From the above facts, we make the following statement of 80 H|| ||20 ratios by which one scale may be reduced to any other : 70 H| | | | |30 1st. Since the number of degrees between the freezing and *Tl|lio boiling points in the F. scale is (2120–320 =) 180° and 100° in C., 50 H ‘H10 therefore, (a) 10 F. = +}}o C. = }o C; and (b) 19 C. = }#9 | 39 2| F. = }o F. - *:: 0–H–0– 2nd. Since the C. and R. scales both have the freezing point 20 H at 00 or zero, and since the boiling point is 100° C. and 80° R., "Hº"T "I therefore, (a) 10 C. = #9 = to R.; and (b) 19 R. = ** = }*C., 3rd. Since the number of degrees between the freezing and boiling points in the R. scales is 800 and in the F. scale 180°, (212°–32° = 180°), therefore, (a) 10 R. = lºſo F. = ?? F; and (b) 19 F. = #o R. = #9 R. A. COMPARISON OF THERMOMETERS. 275 PROBLEMIS. 1. Convert 600 F. to its equivalent in Centigrade degrees or Scale. NOTE.-In changing the F. scale to C. or R. scale, or in changing the C. or R. scale to F., it must be remembered that the number of degrees that expresses the temperature on a F. Scale (which marks freezing at 32°) does not express the number of degrees above freezing point, as it. does on the C. and R, scales. Thus, 609 F. is not 600 above freezing point, but 60° — 32° = 28° above it. Hence to reduce a Fahrenheit reading to a Centigrade reading, first subtract 32° from the F. reading or number, and then multiply the remainder by +93 = }; and to reduce F. to R. first subtract 32° and multiply by ºr or $. To reduce a C. to a F, reading or scale, first multiply the C. number by ## = # and then add to the product 320 ; and to reduce R. to F. first multiply the R. reading by *" or ; and add 32° to the product. OPERATION. 280 × 100 280 × 5 600–320 = 280, above the freezing point of F. Then sº OI’ # =15;9 C. Ans. 2. Reduce 350 C. to the F. Scale. OPERATION. 350 × 180 350 ×9 100- or T5 = 630 above freezing of F., or zero of C. Then 630 + 32O = 950 F. Ans. 3. Reduce 600 F. to Reaumur scale. OPERATION. 600 — 320 = 28, above freezing point of F. and above zero of R. 289 × 8 280 × 4 Then ** or º =12;O R. Ans. 4. Reduce 129 R. to F. Scale. OPERATION. 120 ×180 120 ×9 80 4 = 270 above the freezing point of F. Then 279-H 32° = 59°F. Ans. 5. Reduce 50° C. to R. scale. OPERATION. 500 ×80 500 × 4 TDD- or -5 6. Reduce 600 R. to C. scale. OPERATION. = 400 R. Ans. 600 ×100 T 600 × 5 —sº- O T- = 750 C. Ans. 276 SouLE's PHILOSOPHIC PRACTICAL MATHEMATICS. TO EXPRESS TEMPERATURE WEIEN BELOW ZERO. 612. When the temperature is below 0 or zero, in any scale, it is indicated by a minus sign placed before the number of degrees. Thus: — 100 F. indicates 10° below 0, or 420 below freezing point in F. scale. — 15° C. indicates 150 below 0, or zero, according to the C. scale. — 20° R. indicates 200 below 0, or zero, according to the R. scale. 7. Reduce — 16° C. to its equivalent, in the F. scale. OPERATION. —160 × # = — 28.80 F. Then, –28.80 + 320 = 3.20 F. Ans. - NOTE.—In adding or subtracting the 329 where we have minus numbers, it should be done algebraically. * 8. Reduce 229 F. to its equivalent on the C. scale. OPERATION. — 100 × 100 — 109 × 5 220–320 = — 100 ; X * or – 10° x # = — 5:9 C. Ans. Taj- or 9 9. Change — 139 F. to the C. scale. OPERATION. — 130 — 320 = — 450 below freezing of F.; — 45° x 5 = — 250 C. Ans. --~~ 10. Change — 12° C. to the R. scale. OPERATION. — 129 ×80 & *i;- or — 120x # = – 9.60 R. Ans. -> 11. Change — 20° R. to the F. scale. OPERATION. T*.* or—20° x * = — 45° F.; Then, – 45°–322 = –13°F. Ans. 12. Change — 200 R. to the C. scale. OPERATION. — 200 x + = — 250 C. Ans. Reduce each of the following problems: 13. 66O F. to C. 17. — 32O F. to C. 14. 800 C. to F. 18. — 14O C. to F. 15. 212O F. to R. 19. — 18O R. to C. 16. 750 C. to R. 20. – 24O F. to R. iscellaneous Problems, == IN DENOMINATE AND IN COMPOUND DENOMINATE NUMBERS. =s*—dº--º- -sur-Nºr-wº- ſ 613. 1. Paid $2 for sawing 3 cas. 26 cu. ft. of wood. How much could be sawed for $1, at the same rate 3 Ans. 1 ca. 77 cu. ft. ed. cu. ft. OPERATION INDICATED. 2 3 26 1 77 Ans. 2. Henry traded 4 rubber balls for 3 p.ks. 4 qts. 1 pt. of pecans. How much did he get for each ball? Ans. 7 qts. + pt. 3. An English weaver sold 7% webs of cloth of an equal number of yards in each web at 30:6. 8s. 3d. 2 far. a web. How much did he receive for all ? OPERATION INDICATED. £. S. d. far. £. S. d. far. 30 8 3 2 30 8 3 2 7; or, 7; 228 2 1; 3 74 times 30:6 =225 0 0 0 2 7# times 8s. = 60 0 0 74 times 3d. = 22 2 228 2 2 1 Ans. 74 times 2 far. = 15 Ans. 228 2 2 1 4. 38 lbs. 12 oz. 15 drs. of butter cost $74. How much can be bought for $1? * Ans. 5 lbs. 4 oz. 10%; drs. OPERATION INDICATED. fb. OZ. dr. Explanation and Reason.—In this problem, we 38 12 15 -- 7# or *. have the quantity that $7+ = $* will buy, and we reason as follows: Since $2.3 will buy 38 lbs. 12 oz. 15 drs., 4 of a dollar will buy the 22d part and 3, or $1 will buy 3 times as much. In the operation 22)116 6 13 of all problems of this kind, it is best to first multiply the dividend by the denominator of the 5 4 10##, Ans. divisor, and then divide the product by the numer- ator of the divisor, as shown in the above partially worked operation. 5. A farmer sowed 3 bu. 3 p.k.s. 6 qts. of oats on each of 3.5 acres. How much oats did he sow on all ? Ans. 13 bu. 3 pks. 1 qt. 6. Divide 26 tons, 13 cwt. 3 qrs. 12 lbs. of coal equally among 12 families, and find what each will have to pay at $12 per ton. Ans. $26.6935. 7. How many days at $1.25 per day will it take a man to earn 200 lbs. 12 oz. of beef at 6 cents a pound 7 Ans. 9.6 + days. (277) 278 SouLE's PHILOSOPHIC PRACTICAL MATHEMATICS Y 8. A merchant exchanged 3 yds. 2 qrs. of broad cloth worth $4 per yard, for 26 gals. 3 qts. 1 pt. of molasses. What was the molasses worth per gallon ? Ans. 52; a cents. 9. At a certain distance 20 lbs. of steam expanded the mercury in a Fahr- enheit thermometer 3.5 degrees. How many lbs. at the same distance would be required to expand it from zero to the freezing point? Ans. 1824 lbs. NOTE.—There are 32° between zero and the freezing point, F. 10. From 17 lbs. 5 oz. take 2 lbs. 9 oz. Ans. 14 lbs. 12 oz. 11. One side of a square field is 1080 ft. long; how many rods of straight fence will inclose it 2 Ans. 261 ra. 4yd. 1 ft. 6 in. 12. The wheels of a locomotive are 18 ft. 3 in. in circumference and make 44 revolutions a second; what time will be required to run 99 miles? Ans. 1 h. 46 mi. 5 sec. PRACTICAL PROBLEMIS. 614. N. B. See pages 96 to 101, of this book, for various contracted methods of Working business problems containing the elements of per bushel, per hundred (c), per thousand (m), per ton and per dozen, and also problems where one or both of the factors are aliquots or multiples of 10, 100, or 1000. 13. What cost 1521 pounds OPERATION INDICATED. Reason.—One bushel costs 84c. Since 1 bu. or 56 lbs. of corn at 842 per bushel, and 1521 - 56 costs 84c. 1 pound will cost how many bushels are there? = 27 bu. 9 lbs. 56 || $4, the ºthº, an ºl Ans. $22.814; 27 bu. 9 lbs. 1521 pºin cost is times 14. What cost 2842 bu. 16 opBRATION INDICATED. Reason.—One bushel costs $1.22. Since 1 bushel or 60 lbs. of wheat at $1.22 per bushel? $ lbs. costs $1.221 pound will Ans. $3467.56°. 60 | 1.22 cost the 60th part, and 170536 170536 pounds will cost 170536 times as much. SECOND OPERATION. THIRD OPERATION. 2842 @ $1.22 = $3467.24 - 2842 bu.. (2) $1 = $2842. ## = # (a) $1.22 = .32%; (20g) $1 = 568.40 (2) (2 *=mºmºrs mºms mºs @ (2¢) tº of $# 56.84 $3467.56; 16 lbs. (a) * $3467.56% See pages 96 to 101, for work similar to that of the third operation. PRACTICAL PROBLEMS. 279 15. What cost 342506 lbs. Of Wheat at $9.80 per Imperial quarter ? Ans. $6992.831's. 16. Corn is 60¢ per bu. What is it worth per cental * Ans. $1.07%. 17. Wheat is $2.90 per cental. What is it worth per bushel? Ans. $1.74. 18. Cloth is $1.80 per yard. What is it worth per metre ? Ans. $1.968–H. 19. Cloth is $3.937 per metre. What is it worth per yard? Ans. $3.60. 20. Cloth cost in Mexico $2 per vara. What is it worth per yard? Ans. $2.181 + practically. $2.183 + accurately. NOTE. –A vara of Mexico is 32.97 in., prac- tically, 33 Inches. 21. What cost 376 pounds of hay at $15 per ton ? Ans. $2.82. 22. Cloth is worth $1.45 per yard. What is 1t worth per Spanish vara ? Ans. $1.342 + practically. $1.344 + accurately. NoTE.—A Spanish vara is 33.3864 inches, prac- tically, 33% inches. 23. Sold 5294 pounds of hay at $23.75 per ton. How many tons were there, and what was the value of it? Ans. 2 tons, 1294 lbs. $62.86; value. 24. What cost 4 tons 1420 pounds of hay, at $16.25 per ton ? Ans. $76.53%. 2 OPERATION INDICATED. 9.80 480 || 342506 OPERATION INDICATED. g 60 56 || 100 OPERATION INDICATED. g 2.90 100 60 OPERATION INDICATED. 1.80 36 || 39.37 OPERATION INDICATED. 3.937 39.37 || 36 INDICATED. Accurately. $ 2. 32.97 36.00 OPERATION Practically. 2. 33 || 36 OPERATION INDICATED. $ $ 15. 376 15. Or, 2 | .376 2000 OPERATION Practically. $ INDICATED. Accurately. $ 1.45 1.45 36 3 100 36.0000 || 33.3864 OPERATION INDICATED. $ 5294 - 2000 = 2 t. 1294 lbs. 23.75 OI’ $23.75 × 2 = $ 5.294 $23.75 × 1294 = $ * === 9000 $62,863 INDICATED. OPERATION $ 16.25 2 || 9.420 or, as in the preced- ing problem. 28O soule's PHILOSOPHIC PRACTICAL MATHEMATICS. º: 25. What cost 1265 pounds of bran at 80% per cwt.% Ans. $10.12. 26. What is the value of 5790 hoop poles at $18 per M. ? Ans. $104.22. 27. What is the value of 1364 pine apples at $11.4 per C.? Ans. $156.86. 28. What is the cost of 31845 feet of lumber at $22.25 per M. ? Ans. $708.55#. 29. Bought 3 coops of chickens con- taining 2 dozen and 7 chickens each, at $4.35 per dozen. What did they cost? Ans. $33,714. 30. Butter is worth 35g per pound. How much can you buy for 102* Ans. 44 ounces. 31. Sell 4; inches of silk at $2.75 per yard, and state the amount. Ans. $.343. 32. A lady wishes to buy 402 worth of silk which is $3.00 per yard. How much will you sell her ? Ans. 4% in. 33. A clerk commenced work on the 17th of January, and discontinued April 1st. He received $65 per month. How much was due him counting January 17th, but not April 1stº Ans. $160.334. OPERATION INDICATED. g 30, or, 12.65 100 | 1265 “” 80g OPERATION INDICATED. $ 18. OI’ 5.790 1000 || 5790 * 18 OPERATION INDICATED, 11:50 or, 13.64 100 || 1364 ° 11; OPERATION INDICATED. $ 22.25 or, 31.84; 1000 || 31845 " ? 22+ OPERATION INDICATED. $ 4.35 12 || 93 OPERATION INDICATED. OZ. 16 35 | 10 OPERATION INDICATED. $ 2.75 36 2 || 9 OPERATION INDICATED. in. 36 300 | 40 OPERATION INDICATED. $ 65 $65 30 74 ° 244 NOTE.-In computing salaries and rents, all months are considered as containing 30 days. 34. What will 4 bu. 3 pks. 1 qt. 1 pt. cost at 10g per pint? Ans. $30.70. 35. What cost 4 bu. 3 p.ks. 1 qt. 1 pt. at $6 per bushel? 36. What cost 4 bu, 3 pks. 1 qt. 1 pt. at 25g per quart? Ans. $28,784. Ans. $38,373. 37. What cost 4 bu. 3 p.ks. 1 qt. 1 pt. at $1.75 per peck & Ans. $33.57+}. 38. A grocer has 8 jars of butter, each weighing 14 lbs. 7 oz. How many pounds in all, and what is it worth at 32.4% per pound? Ans. $37.533. Aſ. PRACTICAL PROBLEMS. 28 I 39. Bought 3 bales of hay weighing as follows: (1) 421 pounds, (2) 394 pounds, (3) 487 pounds, at $22.50 per ton. What did they cost 3 Ans. $14.64%. 40. What is the cost of 2417 cocoanuts at $8.25 per C. ? Ans. $199.404. 41. What is the value of 8750 shingles at $8.75 per M. ? Ans. $76,564. 42. What is the value of 11428 fence pickets at $9 per M. 3 Ans. $102.852. 43. Maple syrup is worth $1.92 a gallon. How much can be bought for 25% # Ans. 4% gills, or 1 pt. # gi. 44. Tea is worth $.75 per pound. How much will you sell for 20% - Ans. 44; Oz. 45. Bought 4692 pounds of barley at $.88 per bushel. How many bushels were there and what was the cost 3 Ans. 97 bu. 36 lbs. Cost $86,02. 46. Bought 2765 pounds of oats at 76% per bushel. What was the cost, and how many bushels were there ? Ans. $65.66% cost. 86 bu. 13 lbs. 47. What is the cost of 4878 pounds of wheat at $2.45 per cental? Ans. $119,511. 48. What is the cost of 200 sacks of guano each weighing 162 pounds, at $524 per ton ? Ans. $846.45. 49. A planter shipped 6 dozen dozen boxes of peaches to market, but being delayed on the way # a dozen dozen boxes spoiled; the remainder were sold at 70 cents per box. What did they amount to ? Ans. $554.40. 50. A grocer bought a cask of rum containing 84 gals., at $1.25 per gal.; he lost # of it by leakage; at what rate must he retail the remainder per quart to gain $27 on his purchase ? Ans. 55%. 51. A box, 16 in. long, 12 in. wide, and 11.2 in. deep, contains 1 bushel; how many bushels will a proportionally shaped box contain, each dimension of which is as many feet as the first is inches, that is, 12 times as great Ans. 1728 bu. 52. A box 16 in. square and 8.4 in. deep, contains one bushel, how much will a box contain, whose dimensions are g times as great 3 Ans. 0.669921875 bu., or 2 p.ks. 5 qts. . pt. 53. A can or vessel, 11 in. deep, 7 in. long, 3 in. wide, contains 1 gallon; how many gallons will a similar vessel contain, each of whose dimensions is 10 times as great 3 Ans. 1000 gals. The last three questions give convenient dimensions for constructing the bushel and gallon measures; and the given problems are easily solved by cubing the mul- tiple of the dimensions; thus, the 3d problem is equal to 12°, the 4th to (g)”, and the 5th to 108. 54. If cloth costs $6.40 per yard, and it takes 6 yds. 3 qrs. 3 na. to make a suit, and a tailor charges $15 a suit for the making, what will be the entire cost of making up 173 yds. 1 qr. 3 na., and how many suits can be made 3 Ans. $1485. 25 suits. 55. How many bottles, holding 1 qt. 1 gi., will it require to hold the contents of 1 had. of wine 3 Ans. 224 bottles. 282 SOULE's PHILOSOPHIC PRACTICAL MATHEMATICS * 56. A hardware merchant received from Liverpool an invoice of iron weighing 2.T. 2 cwt. 3 q1s. 20 lbs., long ton weight; the invoice price was £12. 17s. 6d. per ton; what did the whole cost in sterling money? FIRST OPERATION. 2 T. 2 cwt. 3 q1s. 20 lbs. = 4808 lbs., and £12 17s. 6d. = 30906. 1 T. = 2240 lbs. ) 14856720d. 12) 6632}#d. mºmºmºmºmº 20) 552s. 833d. Ans. £27 12s. 833d. SECOND OPERATION. 2 T. 2 cwt. 3 qrs. 20 lbs. = T. 2.1464 + and £12 17s. 6d. = £ 12.875 £27,6349000 20 S. 12.6980 12 d. 8.376 Ans. £27 12s. 8.376d. - Eacplanation.—In this and similar questions, we first reduce the inte- gers of the commodity to the lowest denomination given, = 4808 lbs., and then reduce the price to its lowest term or denomination. We multiply these two amounts togeth- er, giving 14856720d., as the cost of 4808 lbs. at 30906. per pound; but since this last number is the cost of one ton instead of one pound, and as there are 2240 lbs. in one ton long weight we must divide by 2240 to reduce it to the rate per ton, which gives us 6632}}d. as the total cost, and this, when reduced, gives £27. 12s. 84%d. for the answer. Ea:planation.—Here we reduce each term to the decimal of its own highest denomination, then multi- ply them together, and reduce the result according to the principles of reduction of denominate decimals. 57. A cargo of castings was received from Wales, amounting to 225 T. 17 cwt. 2 qrs, long ton weight; the invoice price was £10. 12s. 6d. per ton; the freight was 4S. per cwt. Considering the pound sterling to be equal to $4.84, what would be the cost in Federal money, of the invoice and the freight, and also the entire cost of the whole cargo? Ans. Cost of invoice, $11615.62%. Cost of freight, $ 4372.94. Cost of the whole, $15988.56*. 58. What will be the cost of 1450 tons, 2 cwt. 3 qrs. of coal, at $4 per long ton ? Ans. $5800.55 OPERATION BY OUR NEW METHOD. Ea:planation.—1 cwt. = 112 lbs. = % ton, = T. CWT. QRS. 1450 2 3 112 – 2240 = .05 ton. 1 qr. = + of a cwt. or # .05 .014 = .0125 of .05 of a ton, which is .05 - 4 = .0125 of a .10 -H .0375 = . 1375 ton. 1450.1375 × $4 = $5800.55. Ans. NotE,--In calculating the wholesale cost of coal, it is the custom to consider the pounds only to the nealºst quarter- SECOND NEW METHOD. Multiply total number of pounds by the price, divide by 4, 4 – = Ibs. 8 and 7, and point off 1 place in the dollars. Or make the 8 — = price per ton. statement thus, and cancel: 7 Hills and Invoices. * * * * * * * * * * * * * * * * * * * * * * * * sº e º e s tº e e Tº e º a º ºs e º a===N -*. wºr * = -d-. wr vºy 615. Bills, in a general sense, embrace all written statements of accounts and many legal instruments of writing; but in a more common and limited sense they are statements of goods sold or delivered, services rendered or work done, with the price or value, quality or grade of each article or item. Bills or invoices of mer- Chandise should state the place and date of each sale, the names of the buyer and seller, the price, the extra charges or the discount to be allowed, the marks and numbers on the goods, and the terms of the sale. When goods are bought to be sold, or when bills are rendered to the jobber or retailer, or consigned to an agent, the bill is then called an invoice. It is the custom of accountants and merchants, when making bills, to com- Imence the name of each article with a capital. When a charge is made for the box, barrel, jar, etc., containing goods, it is customary to write its price above and to the right of it, and add the same to the cost of the goods it contains. In making extensions, fractions of cents are not used in the product; when they are # or more, they are counted cents, when they are less than $, they are not counted. NOTE.-Some houses count only every other #c. as a whole cent, thus equalizing the matter with their customers. In making the following bills, students should use pen and ink, and give earnest attention to the proper form and spacing, to plain, neat and rapid penman- ship of both words and figures; and above all, to the accuracy of extensions and additions. NOTE.--When notes or drafts are given in payment, the student should draw the same and correctly mature the notes. [No. 1. Groceries]. NEW ORLEANS, Jan'y 2, 1894. H. A. & R. C. SPENCER, Bot. of A. L. dº E. E. Soulé. Terms—Cash. 1893 Dec. 16 || 2 bags Rio Coffee, 325 lbs. *º tº &= * * &- (a) $ 234c. || $ 76.38 50c. 1 bbl. Sugar, 234 lbs. 4- tº º sº * se tºº 4 & 9 C. 21|61 # Chest Black Tea, 35 lbs. - - - - - - - “ 874c. 30|63 1 blol. Rice, 243 – 16 = 227 tº sº sº sº tº * & 4 8 C. 18|16 40 gals. N. O. Molasses * * wº e sº *- $º * , 75 c. 3000 6 doz. Brooms gºe &º *º wº -ºs sº -: •º * * 4.15 24|90 3 bbls. XXX Family Flour sº tº sº - * tº “ 8.12% 24|38 25 lbs. Cream Crackers wº- * es sº * - * º & 4 16 c. 4|00 50 lbs. Graham do. * º wº & sº- sº * & 4 15 c. 7|50 20 lbs. W. Butter s tº & ºt sº º &E, sº 4-8 * ** 30 C. 6|00 Rec’d payment, $ 243 56 A. L. & E. E. SOULE Per F. Richardson. 284 SOULE's PHILOSOPHIC PRACTICAL MATHEMATICs. Yºr ... [No .2. Groceries]. NEW ORLEANs, Apr. 1, 1894. Bot. of P. H. Thoele dé, Co. MAYER & BARRIERE, 5233 lbs. Butter * * tºº &º º * > tºº cº (2) 25 c. 136% “ La. Pecans gº tºº tº dº tº tº º & 4 12; c. 56 doz. C. Eggs tº { } gº sº tº tº e gº “ 18; c. 249 boxes Lobsters tºº gºe * > gº sº gºe tº & & 37+c. 140 jars Pickles & e sº tº tº gº tº gºe “ 62.4c. 48 lbs. Rice dº * & e ; : tºº gº tº tºº ( & 6+c. 83 bu. W. Corn e tº- tºº * > tº º * : ** 87 c. 64 “ W. Oats £º tº tº * * * * > tº e ** 44 c. 92 bags Salt e- * * * * * = ** 93 c. 84 lbs. B. Tea gº tº ſº tº gº g- gº gº ** 97 C. 108 gals. R. Whiskey sº º tº ſº tº gº “ $1.06 143 doz. Pine Apples º wº gº tº gº Eº ** 1.15 119 lbs. Y. H. Tea gº * e º tº gº º tº e ** 96 0. 324 boxes Shrimp tº a tºº º tº º tº * ** 35 C. 240 gals. Fire Proof Coal Oil tº º º * > ** 15 C. 164 lbs. Lard &= - tº {- º º & * * tº & & 7% c. 286 ‘‘ Green Peas - - - tº tº tº wº & 4 2}c. 320 “ N. Y. C. Cheese - * > tº gº tº º “ 173c. 112 gals. La. Molasses * * * * * * “ 22}c. 124 “ B. Whiskey gº * > dº sº tº & Cº. “ $1.25 72 bbls. La. Oranges * * º º sº gº “, 3.75 61 “ N. Potatoes { } tºº º º tº tºº ** 2.50 * 16# lbs. C. Soap * > {º tº º º º sº & & 84c. 9+ “ Persian Dates tº gº tº º gº * sº & & 64 c. 1 bbl. 50c. Sugar, 241-24 = 217 lbs. {º} sº º & & 5}c. 43 lbs. Lard Oi sº tº * tº º tº * ** 58 C. 45 “ Elgin C. Butte * = tº gº sº ** 35 C. 34 bu. W. Oats sº *_º * : gº tº tº- * * “ 48 C. 425 bbls. Flour * * * * = as sº “ $8.75 562070 NotE.—All extensions of the above bill should be made mentally. —-º- [No. 3. Drugs.] Henry L. Landfried, bought of R. S. Winn, of Bayou Tunica Post Office, La., November 2, 1894, the following Drugs: 60 oz. Morphine at $3.15; 20 lbs. Insect Powder at 35%; 28 lbs. Gum Camphor, at 552; 15 lbs. Gum Opium, at 602; 2 lbs. Blue Mass, English, at 902; 46 lbs. Potassium Bromide, at 60g; 8 lbs. Senna Leaves, at 552; 1 Carboy Nitric Acid, 74 pounds, at 11%. What was the amount of his bill? (Make and receipt the bill). Ans. $262.34. —-º- [No. 4. Books]. NEw York, Dec. 8, 1894. H. C. SPENCER & CO., Bot. Of B. D. Rowlee dº Co. TERMS-Cash. (2) $ 1 5 i 20 doz. Missionary Bibles, gº 108 “ small New Testaments, 65 “ Prayer Books, g-> 65 “ Hymn Books, gº 3 “ Bible Dictionaries, # “ Webster’s Dictionary, & & & 4 ; i & & & & 5 Rec’d payment, $ 953 25 B. D. ROWLEE & CO., Per E. Conrad. BILLS AND INVOICES. 285 [No. 5. Tobacco]. NEw ORLEANs, Jan'y 31, 1894, Bot. of L. L. Williams dº Co. WM. MELCHERT & Co., TERMS-Cash. 321 lbs. Tobacco, Low Lugs, – as tº tº * $º sº. * - tº- (a) 6 c. 1140 * * & 4 Med. Lugs, - * sº sº $º gº tº- º “ 74c. 509 & 4 “ Low Leaf, - - sº tº º ºs gº º “ 9+c. 965 “ “ Med. Leaf, - - * * * * * * “ 11}c. 398 “ ‘‘ Good Leaf, - - - - - - - gº “ 13+c. 2416 44 “ Fine Leaf, - - - - - - - - “ 15 c. 713 “ & & Selections - tºs sº {-> gº § { tº sº “ 163c. Rec’d payment, $ 798 49 L. L. WILLIAMS & CO. —-0- [No. 6. Sugar]. NEW ORLEANs, Dec. 17, 1894. Bot. of C. J. Sinnott. F. L. RICHARDSON, JR., TERMS-Dft. 30 days. 1420 lbs. Sugar, Common, sº s sº e * * º tº Ø 5}c, 1927 “ “ Good, agº, * tº- * sº * sº * * “ 7+c. 2810 “ “ Fair, †º gº sº. - - - - - “ 7;c. 902 & 4 “ Prime, gº *- cº-e tº- gº * 4- &=e *- “ 84c. 813 ºf “ Choice, - - - - - - - - - “ 9+c. 2741 * * “ Yellow Centrifugal, sº tº sº tº sº. º “ 104c Rec’d payment by dft. (a) 30 days' sight, $ 878 78 C. J. SINNOTT. -º- [No. 7. Tobacco]. NEW ORLEANs, Dec. 23, 1894. Bot. of Sadler dº Smith. MONTGOMERY & TREPAGNIER, TERMS-3 mos. 7 Gross Chewing Tobacco, - - - - - - - - a $13. 180 lbs. Smoking do., - - - - - - - - ** 1.40 6 M. Havana Cigars - tº e tº º tº- fº wº- *- º ** 70. 2 M. N. O. Manufacture do - tº me *-, sº * . & º “ 30. I Rec’d payment, .# —-3– [No. 8. Pork and Beef 1. Jacob Broders, bought of C. F. Sauter, of St. Louis, Mo., October 15, 1894, the following articles: 15 bbls. of Prime Mess Pork, $13.50; 10 bbls. Extra Mess Beef, $8.75; 152 lbs. Bacon, Shoulders, at 74 g; 75 lbs. Long Clear Sides, 74%; 146 lbs. Sugar Cured Hams, at 1242; 78 lbs. Breakfast Bacon, canvassed, at 1142. Make out a receipted bill. Total, $333.58. 286 SouLE's PHILOSOPHIC PRACTICAL MATHEMATICS. X- [No. 9. Candles]. NEW ORLEANs, Feb. 4, 1894. F. L. & W. P. RICHARDSON, IBot. of P. W. Sherwood dº Co. TERMS-1 mo. 30 box. Sperm Candles, 596 lbs. - - - sº ºn. - º @ .35} 24 do. Adam Extra Candles, 483 lbs. - - - - - sº “ .28 15 do. Silver Gloss Starch 360 lbs. - - 4- - - º “ .10% Rec’d payment, $ 385 52 —-4– [No. 10. Shoes]. NEW ORLEANs, August 8, 1894. M. G. WUERPEL, Bot. of Albert Bohnet. TERMS-Cash. 6 dozen Men's Boston, 17 in. Kip Boots - Ø $50.00 4 “ Ladies' Calf, Pink Lined, Balmorals - & 4 25.00 5 “ Youths' Wax Brogans - - - sº & 4 15.50 12 “ Youths' Buff Congress, Balmorals - & 4 17.45 1 “ Misses’ Lasting Congress, No. 1, sewed & 4 23.75 2# “ Misses' Goad Polish No. 1, sewed &= & & 32.60 | 2 “ . Boys' Buff Balmorals, Cloth Top, tº & 4 25.75 i 35 “ Men's 17 in. French Top Calf Boots - & 4 35.90 l 6 “ Misses' Calf Balmorals No. 1, pegged - & 4 17.00 || 7 “ Misses’ Lasting Congress, No. 2, sewed { { 19.00 20 “ Boys' Split Wax Brogans - * * { { 12.00 2573|65 —º- [No. 11. Domestic]. Eugene Ellis, bought of Wm. Dirker, of New Orleans, La., November 4, 1894, the following Dry Goods: 3704 yds. of Toledo Plaids at 152; 42 yds. Cotton Check at 12.4%; 81 yds. Cotton Duck at 1042; 90 yds. of Hickory Stripes at 1442 ; 67 yds. Tickings at 132; 86 yds. Lowell Sheetings, 10-4, bleached, 26%; 82 yds. Sheeting, Pembroke 10-4, at 294 g; 384 yds. Cotton Drills, Pride of the South, at 7#2; 143 yds., Lonsdale Cambric at 124%; 45 yds. Wamsette at 14%. , Make and receipt the bill. Total, $164,72. —#- [No. 12. Drugs]. NEw ORLEANs, August 7, 1894. Bot. of Henry Palmer. HENRY HIRSCH, TERMS-Note 60 days. 55 lbs. Acetic Acid, No. 8., º, º º sm. - (2) 25c. 123 “ Carbolic Acid, Cryst. * - gº * & & 60c. 98 “ Sulphuric Acid, Carboy, per pound - - & & 6c. 67 “ Citric Acid - sº º sº sº me 4 & 53c. 34 oz. Antipyrine - - - - - - - “ $1.50 78 lbs. Aqua Ammonia - - - - º - 4 & 15c. 88 “ Bismuth * - - * - - - ** 3.50 44 “ Cream Tartar, pure - - - - - & 4 35c. 35 “ Castile Soap, white mº ºme º tº º & & 25c. 2 cans Glycerine, 50 pounds - - - tº- gº & & 35c. 24 lbs. Gum Arabic, French, powdered - º & 4 700. 67 “ Oil, Bergamot - ſº º º dº tº ** 3.00 45 “ Potassium Chlorate - - - - - & 4 35c. ſ 34 “ Potassium Iodide sº sº dº º ºs ** 1.50 | 4. | 82584 BILLS AND INVOICES. 287 [No. 13. Butter and Cheese]. NEw ORLEANs, August 4, 1894. J. A. SABATIER, Bot. of W. E. Slaughter. TERMS-Due bill 1 Imo. 167 lbs. Extra Fancy Creamery, Butter - - (2) 6}c. 112 “ Fancy Creamery { { . - tº- & 4 25 c. 75 “ Choice Creamery & 4 sº - sº- * { 20 c. 55 “ Prime Creamery & 4 & - 4- & 4 17 C. 85 “ Fancy Dairy & & º - * & 4 16 c. 62% “ Choice Dairy é & * - * & s 14 c. 111 “ Prime Dairy & 4 4-> - - { { 12 c. 48 “ Common & 4 tº- - * { { 8#c. 75 “ New York Fancy Cream Cheese - - & 4 12#c. 88% “ Western Chedders, Full Cream Fancy Cheese 11;c. & 74 “ America Full Cream Fancy Cheese 12 c. 90 “ Twins Full Cream Fancy & & - - & 4 11 c. 75 “ Flats Full Cream Fancy § { - * & 4 10}c. 38 “ Best Skims & 4 - º- & 4 8 c. 92 “ Skims & & - * & 4 5}c. 157|40 -º- [No. 14. Wood and Coal]. --- NEW ORLEANs, Jan'y 29, 1894. H. J. CALVERT, In acc’t with L. B. Keiffer. Jan. | 1 || To old balance as per bill rendered, 91|10 6 | 12 cords Ash Wood - - * - - - - ſº - (a) $7. 6 || 4 cords Oak Wood - -> e- - - e- - - - ** 6.50 14 |50 bbls. Pittsburg Coal - - - º - - º - { { 60c. Cr. $231|10 8 || By Cash - sº * * * * * * * * - “ $50 29 || “.6 days’ Labor, at $4, - tº cº- tº º sº tº - ** 24 74|00 Balance due Jan'y 29, 1894, $ 157 10 Settled by note at 60 days. L. B. KEIFFER. [No. 15. Poultry and Eggs]. NEW ORLEANs, August 9, 1894. J. H. McNEELY, w Bought of S. Delerno. TERMS-Note 30 days. 6# doz. Western Grown Chickens, - - - - Ø $ 5.00 7# ‘‘ Louisiana Young Chickens, º - & & & 3.00 41’s “ Ducks, - gº tº - tº - - & & 2.55 6; “ Western Geese, g- º tº gº - - & 4 6.25 2# “ Western Young Turkeys tº ſº e º 'º' ** 15.00 1ł “... Western Old Turkeys, - - - - - ** 18.00 76 “ Texas Eggs, - - - - - - “ .18% 18863 288 SOULE's PHILoSOPHIC PRACTICAL MATHEMATICS. D. S. DRENNAN, [No. 16. Rice]. TERMS-Due Bill, 30 days. Bought of A. J. Duclos. NEW ORLEANs, August 13, 1894. Aug. | 13 | 160 lbs. Rice, No. 2, tºº sº 136 & 4 Common - * 240 & 4 Ordinary - sº 110 & & |Fair - º &- 273 & 4 Good - gº * -> 140 * * * Prime Gº tº- 310 & 4 Choice * > e. 150 & & Head, & •º 285 & 4 Fancy *E= tº 25 bbls. of 162 lbs. Rough Rice FRANK DUMAS, & 4 & 4 & 4 & 4 é & & 4 & 4 & 4 [No. 17. Plantation Sugar, Open Kettle]. Bought of H. P. Knobloch. '84.50 189|22 NEW ORLEANs, August 15, 1894. July | 10 |280 lbs. Choice Sugar - 4- tº º ºs wº - Ø 4}c. 197 ‘‘ Strict, Prime *- -- “º * * e- * ** 5 C. 230 ** Prime tº * gº {º tº- º *- º- “ 4}c. 254 ** Good Fair 4-3 tº º tº tº- s e-º < “ 4#c. 290 ** Good Prime * º tº $º & gº q-, “ 43c. 260 “ Common sº * 3-º º & gº & “ 43c. 425 “ Inferior º gº tº gº * º ſº gº “ 3}c. 85|40 -O- [No. 18. Cotton]. NEW ORLEANs, August 9, 1894. s. ForFT, To J. J. Foley, Dr. TERMS-Cash. 2 bales, 452,461 = 913 lbs. Low Ordinary Cotton - a 53e 1 “ 458 lbs. Ordinary Cotton sº º sº Ǻ & & 6 c. 3 “ 450,455,460 = 1365 lbs. Good Ordinary Cotton “ 6}c. 1 “ 456 lbs. Low Middling, Cotton - - - “ 64c. 1 “ 462 lbs. Middling Cotton tº me gº “ 74c. 1 ‘‘ 455 lbs. Good Middling Cotton - - - “ 73c. 2 “ 452,460 = 912 lbs. Middling Fair Cotton - “ 74c. 1 ‘‘ 456 lbs. Fair Cotton & sº sº gº º ** 8 c. 372.56 * BILLS AND INVOICES. 289 [No. 19. Refined Sugar]. © NEw ORLEANS, August 10, 1894. EDWARD LeBLANC, |Bot. Of E. N. Moore. TERMS-60 days. 252 lbs. Cut Loaf Sugar - - - - - - (a) 6; c 176# “ Powdered Sugar - *- sº sº- - “ 64 c 240 “ Standard Granulated Sugar tº º - “ 6+ c 175 “ Off Granulated Sugar * * * * “ 63 c 252 “ Candy A. Sugar - - - - - - “ 6% c 144 “ Confectioners' A. Sugar sº me m ms “ 6, c 208 “ Extra C. Sugar * * tº sº m s “ 5+ c 197 “ C. Sugar. - & º --> gº tº- - ** 5 C 99.95 -º- [No. 20. Lumber]. NEW ORLEANs, Jan'y 12, 1894. A. & S. H. SOULE, To Jas. Mc. Hugh. Dr. To 4378 feet Common Boards - - - º - º (a) $21.00 per M. “ 1760 feet Dressed Flooring - - - -- - ** 28.50 “ ** 5125 Brick * * - - - - - º ** 14.25 + . “ 9250 Cypress Shingles - - - - - - é & 6.50 ** “Cartage and Labor, 14|25 Received payment, $ 289 51 —-º- [No. 21. Freight Bill]. NEW ORLEANs, Jan’y 3, 1894. GEO. F. BARTLEY & CO., To Steamship Knickerbocker and Owners, Dr. For Freight on 4393 cubic ft. (a) 25c. The same being contents of 8 boxes measuring as follows: Nos. 1, 2 & 3: 5 ft. 4 in. x 4 ft. 6 in. × 2 ft. 8 in. “ 4, 5 & 6: 6 ft. 2 in. x 3 ft. 0 in. x 2 ft. 11 in. “ 7 & 8: 12 ft. 3 in. × 2 ft. 4 in. X 1 ft. 6 in. Received payment, $ 109 91 E —-º- [No. 22. Services]. A carpenter receives $3.25 per day of 9 hours. How much will he earn work- ing each week day in the month of January, 1894, which commenced on Monday? (Make the bill). Ans. $87.75. —º- [No. 23. Services]. A laborer receives 30g per hour, and he works 104 hours per day. How much will he receive for 6 days work? (Make the bill). Ans. $18.90. 29O SOULE's PHILOSOPHIC PRACTICAL MATHEMATICs. * º [No. 24. Services]. A clerk commenced work June 20th, and worked till September the 15th, at $65 per month. He paid $15 per month for board and $3 per month for washing, including June 20th, but not September 15th, how much money has he, not allowing for other expenditures? (Make the bill). Ans. $133.163. NOTE.-In calculating salaries and rents by the month, 30 days are allowed for a month; i. e. the same salary or rent is paid for each month whether of 28, 29, 30 or 31 days. —-3– [No. 25. Rent and Services]. NEw ORLEANs, Jan'y 4, 1894. J. T. O'QUINN, * To W. Hermann, Dr. For Rent of house No. 386 Dryades Street, from Oct. 7, 1894, to Jan. 1, 1895, 1st date included, 23# months at $35 cº- -> - - º- º “ Services as collector from Sept. 19, 1894, to Jan. 4, 1895, both dates included, 3}} months, at $75 tº- -- e sº * - - tº Received payment, $ 363 00 v--— [No. 26. Services]. NEw ORLEANs, Feb. 1, 1894. MISSISSIPPI VALLEY TRANSPORTATION CO., To Buck dº Richardson, Dr. For services rendered in cause No. 55472. “Steamer R. E. Lee, and Owners vs. Miss. W. T. Co.” Received payment, —eſpe— [No. 27. Services]. A mechanic receives $4 per day of 9 hours, and 60g an hour for “overtime.” During the first three days of the week he worked 30 hours, and the last three days of the week he commenced at 6:30 A. M. and worked till 10 P.M., less two hours for dinner and supper. What are his wages for the week's work & (Make the bill). - Ans. $33.90. —cºo- [No. 28. Carpenters' Time Sheet]. Rate M. T. W. T. F. S. ds. p: day. *: R. Hirsh... . . . Foreman . . Oct. 22 10 || 10 || 10 || 10 || 15 10 || 6; $4| 00 F. Smith. . . . . Carpenter. “ 22 10 || 10 || 10 || 10 | 15 || 10 || 64 3| 50 O. Wood. . . . . do. ** 22 10 || 10 || 10 || 10 | X | 10 5 3| 00 P. Murphy ... do. ** 22 10 || 10 || 10 || 10 || 10 || 10 || 6 3| 00 H. Brown.... do. “ [22] 10 || 5 || 5 || 5 || 10 || 10 || 44 3| 00 J. Fitzpatrick do ** 26 × | X | X | X | 10 || 10 || 2 2 50 j. s." Smith |Apprentice “ 23 ió | 10 || 3 || 10 | 16 | 16 || 5 || 2 00 114|00 T- Ans. In this time sheet, what is due to each mechanic, and what is the total amount at the close of the week; 10 hours being a day’s work, and double price being paid for overtime? Ans. $114. Total amount. NotE.—The numbers written after the names represent the number of hours that each worked on the different days of the week. The X signifies no hours. * BILLS AND INVOICES. 29 I [No. 29. Factory Time Sheet]. Rate M. T. W. T. F. S. ds. per day. J. C. Bell... . . . . . . . . . . . . . . . . . . . . . . . . . . . . . . . . . . Oct. | 8 || 10 | 10 | 10 | 10 | 10 | 10 || 6 || 400 Thos. A. Clark. . . . . . . . . . . . . . . . . . . . . . . . . . . . . . . . “ | 8 || 10 || 10 || 10 | X | 10 || 10 || 5 || 350 A. J. Duclos. . . . . . . . . . . . . . . . . . . . . . . . . . . . . . . . . . “ | 8 || 5 || 10 || 10 || 10 || 10 || 10 || 54 || 350 T. J. Fatjo. . . . . . . . . . . . . . . . . . . . . . . . . . . . . . . . . . . “ | 8 || 10 | X | 10 || 10 || 10 || 10 || 5 || 350 H. A. Hornor. . . . . . . . . . . . . . . . . . . . . . . . . . . . . . . . ** | 8 || 10 10 || 10 || 10 || 10 || 10 6 3|50 H. C. Lowry. . . . . . . . . . . . . . . . . . . . . . . . . . . . . . . . . . ** | 8 || 10 || 10 || 10 || 10 || 10 || 10 || 6 2|00 In this time sheet, what is due each workman Saturday night, 10 hours being a day's work, and what is the total amount? FLYNN & [No. 30. Paints, Oils, etc.]. KERWIN, TERMS-30 days. —eº- Bought Ans. To the last, $111.25. NEw ORLEANs, Mar. 11, 1894. of Donelon dº Haight. 2 cases, 12 gals. Green Paint # “ 3 ** Vermilion 12 gals. Roofing Paints 9 “ Carriage “ 6 “ Copper 4 & 50 lbs. White Lead 75 “ Putty tº-º: 12 gals. Coach Warnish 18 ‘‘ West. Linseed Oil 45 lbs. Eng. Vermilion 20 “ Vandyke Brown [No. 31. M. A. TERRY & CO., TERMS Cash. Building Materials]. $1.80 2.40 163|01 NEw GRLEANs, Mar. 9, 1894. Bought of H. M. Thompson dº Son. 750 ft. 14x3+ Clear Pine Flooring 1200 “ No. 1, Weatherboards 550 “ Ceiling tºº gº 3500 Laths tº º ſº s 45 bbls. Lime sºme tº- 18 “ Domestic Cement 800 Paving Brick tº- 25 bbls. Sand * sº 12 “ Small Shells - 2000 Am. Fire Brick º 750 ft. Sewer Pipe § -º 600 “ Chimney Flue Lining Eº* >arssºesº† :tº-ºfºsº*º- per M. $25.00 & 4 & 4 & 4 (2) & 4 per M. (2) & 4 per M. (2) 4 & 13.00 15.50 1.90 46126 292 soule's PHILOSOPHIC PRACTICAL MATHEMATICs. E. WEIDIG, [No. 32. Hardware]. NEw ORLEANs, Dec. 22, 1893. bot. of Reymes dº Carmam. TERMS-Cash. —Days—% Discount—Days—% Interest after Maturity and subject to sight Draft without notice. 18} 16 & 4 49+ doz. Claw Hammers - Lathing Hatchets - Chain Bolts - - . Nails, ea. 6d.-8d.-10d. . Measuring Tapes - Marking Brushes - Butcher Saws - Gem Wrenches - Chisels, ea. § and # in. File Handles - - Curry Combs - - Gas Plyers - - 10"/14" Shelf Brackets Ivory Rules - tº Pointing Trowels - Mortise Locks tº- Rim & & - Shutter Bolts -> Padlocks - - Drawer Pulls - - Dashboard Lanterns [No. 33. McDANIEL & KELLER, sº - - Gº- (2) $ 4.95 º - - º & & 8.49 $º - - tº & 4 1.75% º - - º & & .04% s - -> * > & 4 4.50 gº -- º sº & 4 .95 gº - e- º & 4 5.65 º - - tº º 46 2.35 tº - º tº º & 4 6.80 gº º tº- tº & 4 .88 º -> sº tº º & 4 1.76% º - - tº & 4 8.46 º tº- º « » é & 4.98 gº e- sº tº º & & 10.20 - - º * > & 4 4.75 º º º gº & 4 3.30 * {-> Gº tº & 4 2.85 ge tº- -> & & 4 1.60 - -> - gº & 4 2.38 gº & º gº & 4 1.86% tº e º tº ſº &é 4.25 Grain, Hay, etc.]. NEW ORLEANs, Jan'y 25, 1894. Bot. of Redfield dº Halsnith. TERMS-Note at 60 days. 2144 lbs. bu. Yellow Corn, tº º - ſº tº - @ $ 63c. 1242 “ — “ Texas Wheat, * - sº sº - “ 1.70 852 “ —— “ White Oats, tº sº - we * - & 4 56c. 792 “ —— “ Barley, - tº º - * * - & 4 83c. 1427 “ — Cwt. Bran, - sº m sº sº tº tº & & 75C. 3745 & 4 Tons Timothy Hay, * - sº ºne º “ 18.50 * 1701 * * Tons Clover Hay, * * - sº gº - ** 20. Rec'd pay’t by note at 60 days, $ 150 27 REDFIELD & HALSMITH. BILLS AND INVOICES. 293 [No. 34. Sugar—Centrifugals]. J. HEINTZELMAN, Bot. Of Wm. O’Briem. NEw ORLEANs, August 4, 1894. TERMS-Cash. 244 lbs. Plantation Sugar gº. * sº-s sº (a) 5% c. 355 ‘‘ Granulated 4 & q= - sº sº sº ** 6 C. 197 ‘‘ Off Granulated “ +-º *. º sº wº “ 6+ c. 280 “ Choice White & & * * sº * dº * “ 64 c. 175 ‘‘ Off White { { tº-3 E-> tº- sº sº “ 6; c. 290 “ Gray Off White “ sº & &= sº tº “ 64 c. 178 “ Fancy Yellow & 4 sº * sº tº-: sº “ 6+ c. 230 “ Choice Yellow “ Gº tºº gº sº &- ** 6 C. 312 ‘‘ Prime Yellow 4 & º * sº * &- “ 5}{c. 230 ‘‘ Off Yellow & & gº {- # e. †º tº-e “ 5+ c. 234 “ Yellow Fair & 4 & e -> & tº sº “ 4; c. 226 ‘‘ Seconds & 4 &- &= dº * “ 4; c. 174|34 J. P. POWELL, TERMS-1 mo. [No. 35. Produce]. Bot. NEw ORLEANs, August 7, 1894. of W. E. Richards. 15 6 150 & 4 & 4 { & & ( & & 4 & 4 27 bbls. Neshamnock Potatoes Onions tº- * * 6 Crates Cabbage . . . 12 Strings La. Garlic *-* - *- 30 bbls. Sweet Potatoes, Yams * Western Cucumbers 40 doz. Egg Plants tº 140 lbs. & Choice Northern White Beans Northern Red Kidney & 4 Northern White Kidney “ Green Peas 3-3 e * Dried Peaches & sº Evaporated Peaches * i : i .20 332 26 2.94 soul E's PHILOSOPHIC PRACTICAL MATHEMATICS [No. 36. Character, Virtue, Knowledge, etc.]. NEW ORLEANs, Oct. 1, 1894. * ETHICAL AMBITION, Bot. of Soulé College. TERMS-Fidelity to Duty. 1163 lbs. Energy and Perseverance tºº gº º (a) .87% 45 ‘‘ Self Reliance sº sº tº gº Q = º & 4 .95 100 “ Honesty sº * * * * * sº sº. “ $100.00 17 “ Business Sagacity * * * * * ** 1.15 150 “ Punctuality sº gº tº º gº *...* é & 1.16# 49 “ Cheerfulness es &=º tº º * * tº & & .81 56 ‘‘ Faithfulness & º * gº tº gº é & .94 73 boxes Accuracy tº sº * * = * i e & & .75 38 “ Thoughtfulness tº & ºn tº * > tº & & .66% + yd. Talkativeness *- tº cºs gº º * & 4 .01 25 yds. Economy e- * * is ºs sº * = ( & .83 2} “ Recreation tº * º º tº º & & .33% 50 bu. of Politeness ſº tº º gº tº tº gº & 4 2.00 75 ‘‘ ‘‘ Neatness and Order tº º º sº & 4 1.00 75 “ “ Obedience and Good Deportment º & 4 2.50 + “ “ Attention to other Peoples' Business - & & 1.00 50 “ “ Attention to my own Business - º & & 5.00 500 bbls. Practical English Education * gº ‘‘ 4.00 500 “ Practical Business Education tº- sº & 4 4.00 500 facts of Physiology and Hygiene - º * † & 4 5.00 500 “ “ Phrenology and Sociology tº- tºº ** 5.00 [No. 37. Dry Goods]. NEw ORLEANs, Feb’y 14, 1894. WEHRT & GEORGE, Bot. Of Morris & Childress. TERMS-30 days. 25 yds. Serge e ‘º - * * - * * (Ø .22} 37% “ Hopsacking - tº- wº - * - & 4 .42% 18% “ Diagonal Serge * * * * * & 4 .39 15} “ French Sateens º gº e- * º & 4 .27# 12 “ Brocaded Silk * * *-* e- *g: gº & 4 .62% 47 ‘‘ Colored Satins gº * s * * sº gº * { .43 22 “ Table Damask se *º- tºº tº- * & 4 .47% 310 “ Crash º tº * sº sº tº &= & 4 .06 82% “ Canton Flannel * - sº gº & 4 .09% 64 “ Storm Serge 4- * * - sº & e .35 9 “ Brocaded Cashmere sº º sº tº- & 4 .14+ 53 “ Ladies’ Cloth 3- *- - & * sº { { .87% Lºw 87 “ Figured Satinet º * as * sº ( & .95 20 “ French Velours tº- *-> * sº “ 1.47% 35 “ Surah Silk e-º *- e- sº .75 33+ “ Scotch Plaids «º *- * - *º- * { { .65 28 “ Whipcords *º * -º *E º { % .33% 17+ “ Henriettas tº- sº sº sº & ( & .36 142} “ Silk Ribbon * wº s e-8 & 4 .12% 75 ‘‘ Satin * * * wº wº º * & f .16# 1# doz. Initial Linen Handkerchiefs * '' 4.75 | 382 jºr BILLS AND INVOICES. 295 [No. 38. Invoice of Goods Shipped]. NEW ORLEANs, Jan'y 16, 1894. Invoice of Sundries, purchased by H. Strong & Co., and shipped per Steamer La Belle, for account and risk of Chas. B. Thorn, Shreveport, La. º 87 bbls. Molasses, 3498 gallons * - sº - - - - Ø 60c. 20 hbds. Sugar, 23780 lbs. se smº sº º sm * * * “ 90. 10 bbls. Rice, 2150 lbs. - - - " - - - - - ** 50. Charges. 4346.9 Drayage, º s w = - º - - - - * - Q- 1750 Insurance on $4800.40 sº g- º gº tº «º * > --> <--> Ø #% 30|00 Commission on $4364.00 sº .* * * * * ** º “ 24% 109|10 $ 4503 10 -Ö- [No. 39. Slaters' Bill]. NEw ORLEANs, Jan'y 14, 1894. C. H. REYNOLDs, To H. Marsden, Dr. 36.04}; sqs. - - *- e- * (a) $14.50 For 239 ft. Guttering - - - º * > s & & .90 Received payment, $ 737 77 For slating a roof measuring 72 ft. 4 in. by 49 ft. 10 in., and containing [No. 40. Cistern Makers’ Bill]. NEw ORLEANs, Nov. 4, 1894. NEW ORLEANS, ST. LOUIS, AND CHICAGO R. R., - To W. L. dº H. Jackson, Dr. For 150 Cisterns holding 766782.45 gals. (a 24c. per gal. The inside meas- urement of each cistern is as follows: 11 ft. 3 in. perpendicular height, lower base 9 ft. 2 in. in diameter and upper base 8 ft. 5 in. in diameter. Received payment, § 19169 56 NotE.—For other bills involving discount, rebate, etc. and embracing different lines of busi- ness, see other pages of this book as per index. o Tº Filiit Indlf.Tille Between TWO DâtéS, —N 616. 1. How many years, months, days, hours, and minutes from 4:20 o'clock P. M. June 10, 1889, to 9:15 o'clock A. M. August 14, 1894, not allowing for Teap years, and counting 30 days to the month? OPERATION. Eacplanation.—In all prob- lems of this kind, we write yr. IſlO. d. hr. min. the later date first. since it y 1894 8 14 9 15, expresses the greater period 1889 6 10 16 20 of time, and the earlier date º & e beneath. Then subtract as in O yrs. 2 mos. 3 ds. 16 hrs. 55 min. Ans. compound denominate num- bers. NOTE.—The months are numbered from January, and the hours are counted from 12 o'clock at night. - SECOND OPERATION The time from June 10, 1889, to June 10, 1894, - * - - = 5 yrs. 66 “ June 10, 1894, to Aug. 10, 1894, - - tº - = 2 mos. {{ “ 4:20 P. M. Aug. 10, 1894, to 4:20 P. M. Aug. 13, 1894, - 3 ds. 44 “ 4:20 P. M. Aug. 13, 1894, to 8:20 A. M. Aug. 14, 1894, - 16 hrs. & 4 “ 8:20 A. M. Aug. 14, 1894, to 9:15 A. M. Aug. 14, 1894, = 55 min. NotE.—While both of the above operations are correct according to the conditions of the problem, and conform to the usual method of finding the time between dates, neither operation gives an accurate result, for the reason that some years and some months contain more days than other years and other months. The only accurate way is when the time is less than one year, to count the exact number of days in each month of intervening time, or refer to a time table. When the time is more than one year, find the days for the months, as above, and allow 365 days for com- mon, and 366 for leap years. 2. The Declaration of Independence was ratified July 4, 1776; the battle of New Orleans was fought Jan'y 8, 1815. What is the time between these two dates, by the usual method? Ans. 38 yrs. 6 mos. 4 ds. 3. Washington was born February 22, 1732. How old was he at the Declar- ation of Independence, July 4, 1776? How long did he live after that event, his death occurring December 14, 1799, and what was his age 3 w Ans. 44 yrs. 4 ms. 12 ds. 23 {{ 5 4% 10 { % 67 4 9 4, 22 (4 Jamestown, Va., was first settled May 23, 1607, and the Pilgrims landed at Plymouth, December 22, 1620. What time intervened, by the usual method? Ans. 13 yrs. 6 mos. 29 ds. 5. A note was dated January 10, 1892, and was made payable 2 years after date. When did it become due; how many days did it run, by the usual method; and how many days counting actual time; no allowance to be made for days of grace # Ans. January 10, 1894, it matured; 720 ds. by the usual method; 731 ds. actual time. (296) TO FIND THE LENGTH OF THE DAY AND THE NIGHT, 297 6. A note is drawn Oct. 15, 1894, and made payable 3 months after date. When does it mature, allowing 3 days of grace according to business custom, and how many days does it run, actual time 3 Ans. Matures Jan. 18, 1895; runs 95 ds. 7. A note is drawn, Oct. 15, 1894, and made payable 90 days after date. When does it mature, allowing the customary 3 days of grace, and how many days does it run, actual time? Ans. Matures Jan'y 16, 1895; runs 93 days. NOTE.-It is the custom, when maturing commercial instruments, to be governed strictly by the terms expressed therein. When they are made payable in years or months, they are matured in years or months, counting them as they run from the date of the instrument. When they are made payable in days, they are matured in days, the actual number of days in each month of the intervening time being counted. The day that a note or other instrument is dated, is not counted as one of the days which it has to run. The day it matures is however counted. When discounting notes, bills of exchange, etc., the actual number of days which the instru- ment has to run, is counted, to compute the interest, whether it is drawn in years, months or days. See Banker's Discount in this book for extended work of maturing and discounting commer- cial paper. HOW TO FIND TEIE LENGTH OF THE DAY OR NIGEIT WHEN WE HAVE THE TIME TEIE SUN RISES OR SETS. 617. 1. If the Sun sets at 7h. 5m. 10s., what is the length of the day and the night, and at what time does the Sun rise ? OPERATION. Sun sets 7h. 5m. 10s. × 2 = 14. 10. 20. add 12h. Time after midnight that the Sun Set = 19h. 5m. 10s. 24h. Sun rises 4h. 54m. 50s. 19h. 5m. 10s. Length of day 14h. 10m. 20s. w 24h. Length of night 9h. 49m. 40s. Explanation.--In all problems like this, first add 12h., to the time the Sun sets, which gives the time past midnight that the Sun sets. Then subtract this from 24h. and in the remainder we have the time the Sun rises. Then, since the sun rises 4h. 54m. 50s. A. M. and sets 19h. 5m. 10s. past midnight the difference will be the length of the day. Then the length of the day subtracted from 24h. will give the length of the night. 2. If the Sun rises at 4h. 54m. 50s., what is the length of the night and the day ? OPERATION. Sun rises 4h. 54m. 50s. × 2 = 9h. 49m. 40s. length of night. 24h. 14h. 10m. 20s. length of day. Explanation.—In all problems of this kind, multiply the time the Sun rises by 2 and in the product we have the length of the night; Then, to find the length of the day we subtract the length of the night from 24h. NotE.—The above methods of finding the length of the day and the night, and the rising and the setting of the Sun are not absolutely correct, on account of the obliquity of the earth's orbit, and other astronomical reasons. 3. When the Sun sets 6h. 34m. 15s., what is the length of the day and what time does the Sun rise? Ans. Length of day, 13h. 8m. 30s. | Sun rises 5h. 25m. 45s. 4. When the Sun rises at 5h. 14m. 15s. past 12 midnight, what is the length of the night? Ans. 10h. 28 m. 30S. •oſqº on, Jo Insor oth on peppe eq 1stitti Kep I ‘ĀIbniq9JI sooëique ourſ, eul uoqAA ‘sieok deoI uſ—'ILON 30A, 8. I02 & 001, T tº gº tº ſº tº º 669 08 869 6& A69 86. 969 A& Q69 93 f69 93 969 tº &69 £3 I69 & O69 I& 689 0& 889 61 !89 8T 989 All Q89 91 789 QI 989 WI 889 g|I I89 &I 089 II 6/9 Ol 829 6 ZZ9 8 929 A. 9/9 9 f/9 9 8/9 ſy &A.9 9. IAS) & 0/9 I | 699 IQ 899 09 199 66 999 8& 999 L& j99 98 999 gº 699 jº, I99 £& 099 & 699 I& 899 0& 199 GI 999 8I 999 LI j99 9T 899 QI 699 fºl IQ9 £I 099 &I 679 II 879 0T /j9 6 979 8 gj9 J, fif{} 9 gj9 Q gº f If 9 4: Of 9 & 68.9 I © & © Q & © 899 09 189 6& 989 8& 989 L& j/89 98. 889 93 389 +8. I£9 £& 089 & 6&9 I?. 869 0& 169 61 939 8I 939 A.T W&Q 9] 839 QI &9 pH I29 £I 039 &I 6T9 II 819 OI M.I.9 6 919 8 919 1, #I9 9 919 Q &I9 iſ II9 £ 019 & 609 I 809 Ig 109 08 909 6& 909 8& 709 L& 809 9& &09 96 I09 fº 009 96. 669 & 869. I& 169: 0& 969: 61 Q69 8I j69 J.T 869 9T Č69, 91 I69 WI 06Q 9T 689 &I 889. II 189 OI 989: 6 Q89 8 jSQ 1, 889, 9 Č89 Q I89 p. 089 9. 61.9 & 8/9 l L19. I8 909 Og 919 6& j/9: 86 £1.9 L& &19 9& I/Q 9% 09 ſº. 699 96. 899 & 199 Tº 999 0& Q99 6ſ f09 81 999 LI Č99 91 I99 QI 099 iſ 699 9T 899 &I 199 TI 999 OI QQQ 6 fgg 8 899 L &QQ 9 IQQ 9 099 ſº 6fç 8 879 & Miſº I 979 08 Qj9 6& fig 83 gig 16 96 Iſºg gº Of g ſº, 699 9& 889 & 189 I& 989 06 Q99 6ſ fgG 81 889 LI Č89 91 Igg gl 089 WI 6&9 8I 8&g &I 1&g II 9&g OI Q&Q 6 #89 8 939 1, Zög 9 I&Q 9 0&g f 6IQ 8 8IQ & LIQ I 9Ig Tg g|IQ 08 jLQ 6& 3IQ 8& IIQ 93 0IQ 93 609 7% 809 96. 109 & 909, Tó Q09 0& #09 61 809 8T &09 LI I09 9T 009 g| 667 WI 867 &I 167 &l 967 II Q67 OI j67 6 967 8 &67 / I67 9 06; 9 68; 7 887 9 187 & 987 T e e º O © g87 08 j97 63 88ſ. º: I8p 9& 087 93. 6/j/ fº 817 93 1/f & 9/j/ I’. 91.7 0& fºLi 61 91.7 81 ČLi MI ILſ 9T Olip QI 697 FI 897, 8 I 197 &I 99p II Q97 OI j97 6 897 8 Č97 P. I97 9 097 9 69'ſ ſº 897 9 1917 & 997 I ‘ggſ, Ig j9; 08 89.7 6& 39.7 8& &IQ 13,387 ||3|I9'ſ 1& 09f 9& 677 gº 8ff fa. 1.77 93. 977 & gff. I& pff 0& gfſ 6T 27ſ. 81 If? Ll Off 9 I 68ſ g|I 887 fl 18; 81 98? &l g3? II figj QI 88ſ. 6 38ſ 8 I9'ſ 1, 087 9 6&7 9 8&ſ p 137 9 937 & 937 I ge e e g º e fºſ' 8& 937 13 3&ſ 98. I&# 9& 037 jö 6Iſ, 88 8If & 1.If Iº. 9If 0& GIſ 6T #If 8T 9If DT &If 9T IIf g[ OIf WI 607 8T 807 &I 10; II 907 OI Q0ſ, 6 f0.7 8 90'ſ L 80ſ, 9 IOf Q 007 jº 668, 9 868 & 168 I 969 Ig 968 08 j68 6& 868 8& &68 16 I68 9& 068 9& 683 f6 888 83 188 & 988 Tö 988 07. f$3 6T 888 8T 388 LI I88 9T 088 QI 618 WT 818, 8T 118 &T 919 II Q18 OI f/8 6 818, 8 & 18 1, IL3 9 018 Q 698 y 898 8 198 & 999 I g99 Ig j98 08 898 6& Č98 83 T99 || 3 099 96 698 gº, 893 f6, /.98 8% 999 & 999 TZ j93 03 898 6T 398 8T I98 LI 099 9T 678 QL 878 WI 1.f3 8T 979 &I Qj9 II fj9 OI 879 6  8 If8 1. Ofg 9 689 Q 899 f 188 9 988 & 989 I j93 09 988 66 388 83 I89 13 083 96 639 96 863 jº, 133 33 933 33 Q39 T3 j/39 03, 939 61 338 8T I38 LI 028 9T 6I9 9T 8I9 FI /...I.9 9T 9I9 &I QI9 II #Ig OI 9I9 6 &I9 8 II9 L OI8 9 609, Q 803 jº 108 8 908 & Q08 I f03 Ig 808 08 Č08 66 I08 83 008 L3 663, 93 863, 93 16% 7% 963, 83, Q63, 33 j63, T6 863 03 £63 6T I66 8T O63, LI 683, 9 I 886, QI 183 fºl 983, 8 I ç83 &I #83 II 883 0T £83, 6 I86, 8 086 A. 616, 9 818, 9 LL3 f 918, 8 913 & f/3. T 813 08 £13, 63 IL& 83 016 /3 693, 93 893 93 193 jº, 993, 93 Q96, 36 j93, T3 993 03 £93 6T I93 8T 093, LI 693 9T 893 9T L93 WI 993 8T Q96 &I #93, TI 993 0T 393 6 IQ3 8 093, 1. 6f3 9 8f3, Q 1.f3 iſ 976, 9 Qf6 & jić, I º Ig 37& Og If & 66 07& 83 683, 13 883 96 183 93 993 f6, Q96, 93 fgö, 33 983, T6 £83 03 T83, 61 08& 8T 663 LI 836, 9T 136 QT 936, 71 Q33 81 j6% &I 936 II 333 0T I33, 6 066, 8 6T6 L 8I& 9 J.T.& 9 9IZ iſ QI& 8 fT3 & 8I& T |ZT3 Ig II& 09 OI& 66 60& 83 806 M& I,0& 93 903, g3, Q03 f6, f06, 93, 803, 33 306, IG I03 06 003, 6 I 66T 8T 86T LI 16I 9T 96T QI Q6I jºſ f6I 8T 861 &I 36T II I6I OT 06T 6 68T 8 88T L 18T 9 98T 9 Q8I 7 j9T 9 881 & Č8I I ILI (93 OLT 6T 69T 8T 89T 1..I 19| 9T 99T gT Q9I fi f{}I gT 99T &I 39T. II [9][ OI 09T 6 69 I 8 89. I I, 19T 9 99T 9 QQI f j9I 9 99T & 69. I I Of I 03 68ſ 6T 89T 8I 18T LI 99T 91 ggſ g|I fg|I #1 99T 81 39T &I I£I II 08T OT 6&I 6 8&I 8 16I 1. 9&I 9 Q&T 9 fºL if 9&T 9 3&I & IZI I Igſ Ig I8I 0809 I 09 08I 63.67ſ 63 6/I 83|87T 83 8/I 1317T LZ //I 93/971 93 9/I gågſ. I ga. g|I jøffT pa j/I 83.9 FI 92. 8LI 36|&f I & 31, I IZIf I IC, 0&T 03 6II 63 8II 82 LII 13 9II 93, 9II Q& fºLI jø 8II ga. 3II 33 III IC, 0II 03 60I 61 80I 8T 10T LI 90I 9T Q0I QI #OT #I 80T 91 30I &I IOI II 00I 0I 66 86 16 96 Q6 #6 86 £6 I6 19 & O9 I h # h # "OGICI * AON "J.OO "..I. J.GIS **) 0 V "ATIſ) ſº *GINſ) ſ” * XVIAI *}{dV **IVIN * {{{H}{ "NVſº *Oºſ(I * AON "JLOO "J.J. H.S *9ſ) V * XT[ſ] ſº "HNſ) ſ” *ACWIWI **IJIV **IVIN * {{{IJI "NVſ’ ‘SlueM 0AI, up IJIM. S04E0D uéAIA) 0AL u09A439 SAB(I uſ Qulū, Qū) 5ulpuſh I0J Qiqui, oupſ, º: DIRECTIONS FOR THE OPERATION. 299 TO DETERMINE TEIE DAY OF THE WEEK ON WEHICH AN EVENT DID OR MAY. OCCUR. 619. A convenient method of determining immediately what day of the week any date transpired, or will transpire, from the commencement of the Christian era, for the term of three thousand years. 620. The following table shows the ratio to be added for each month : TABLE OF MONTHS. January, ratio is - 3 || May, ratio is - - 4 || September, ratio is 1. February, “ - - 6 June, * { * = sº | October, {{ - - 3 March, {{ e-e 6 July, {{ - - - 2 | November, “ - - 6 April, {{ º 2 || August, “ - - - 5 || December, “ - - 1 * NotE.—In leap years, the ratio of January is 2 and the ratio of February is 5. There is no change in the ratios of the other months. 621. The following table shows the ratios to be added for each century of the Christian era : TABLE OF CENTURIES. 200, 900, 1800, 2200, 3000, the ratio is tº 300, 1000, the ratio is - gº tº sº tº 400, 1100, 1900, 2300, 2700, the ratio is - - - 500, 1200, 1600, 2000, 2400, 2800, the ratio is - 600, 1300, the ratio is tº- s *- * * 700, 1400, 1700, 2100, 2500, 2900, the ratio is - 100, 800, 1500, the ratio is º e- sa º º i DIRECTIONS FOR THE OPERATION. 622. 1. Add to the given year, (omitting the century figures) one-fourth part of itself, rejecting the fractions, if any. 2. To this sum add, 10, the day of the given month; 29, add the ratio of the month, as per table of months; 30, add the ratio of the century as per table of centuries. 3. Divide this sum by 7. The remainder is the day of the week counting Sunday as the first day, Monday as the second, etc. 3OO SouLE's PHILOSOPHIC PRACTICAL MATHEMATICS. x: PROBLEMIS. 623, 7, The battle of Shiloh was commenced on the 6th of April, 1862. *What was the day of the week? Ans. Sunday. OPERATION. The given year, omitting the centuries, is - 62 One-fourth of same, rejecting fractions, is - 15 The day of the month was the * ºs - 6 The ratio of April is * wº tº- * * - 2 The ratio of 1800 is sº gº sº tºp - 0 This sum, divided by 7, gives * * - 7) 85 12 and 1 remainder tºº gº gº 12+1 The 1 remainder signifies the first day of the week—SUNDAY. 2. The Declaration of Independence was signed July 4, 1776. What was the day of the week 3 Ans. Thursday. OPERATION. Given year is - - - - 76 One-fourth of same is - 19 Day of month is - - - 4 Ratio of July is - - - - 2 Ratio of 1700 is - - - - 2 Divide by 7) 103 * 14–H5 remainder—THURSDAY. 3. Gen. R. E. Lee was born June 19, 1807. What was the day of the week # Ans. Friday. 4. Martin Luther was born Nov. 10, 1483. What was the day of the week? Ans. Monday. " 5. What day of the week will January 1 occur in 2000? Ans. Saturday. NOTE.-When there is no remainder after dividing by 7, the day of the week is Saturday. 6. A note is dated Wednesday, October 24th, 1894, payable in 120 days. In what year and month, and on what day of the month and day of the week, will it become due, allowing 3 days of grace? Ans. Sunday, Feb. 24, 1895. 7. On what day of the week were you born, and if you live to be 150 years old, as we wish you may, on what day of the week will you die ifference of Latitude. 624. Latitude is the distance in degrees, minutes, and seconds, of any place on the globe, North or South of the equator. Latitude is reckoned from the equator to each pole of the earth, and like arcs of all circles, is measured in degrees, minutes and seconds, and can never be greater than a quadrant, or 90 degrees. The earth not being a perfect sphere, but oblate, or flattened at the poles, the degrees vary slightly in length toward the poles. The difference of latitude between two places is found by subtraction or addition, as in compound denominate numbers. 1. The latitude of Washington City is 38° 53' 39" North, and that of New Orleans is 29° 56' 59° North. Required the difference of latitude. OPERATION. Lat. of Washington = 38o 53° 39” N. * “ New Orleans = 29O 56° 59'ſ N. Dif. of latitude = 8o 56° 40” Ans. Pºplanation.-Since both places are on the same side of the equator, that is in North lati- tude, we subtract the lesser latitude from the greater. When the latitude of one place is North, and that of the other South, of the equator, we add the two latitudes together, and the sum will be their difference of latitude. 2. The latitude of Mobile is 30° 41' 26" North, and that of Quebec is 460 48' 17" North. What is their difference? AnS. 16O 6' 51". 3. Philadelphia is in latitude 39° 56' 53' North, and Rio de Janeiro 220 54, 24" South. What is their difference # AnS, 62o 51/17//. 4. New York is in latitude 40 deg. 42 min. North, and Cape Horn is in lati- tude 55 deg. 58 min. South. What difference intervenes? Ans. 96 deg. 40 min. 5. Havana is in latitude 23 deg. 9 min. North, and San Francisco lies 14 deg. 39 min, further North. What is the latitude of San Franciscot Ans. 37 deg. 48 min. (301) 3]ifference of Longitude. 5N aſſº. —-dºh- A -ºr wºr---ºut-w 625. The Longitude of a place is its distance in degrees, minutes, and seconds, East or West, from a given meridian. * A degree of longitude on the equator is 69.1638 statute miles, but since all meridians are drawn through the poles and meet in a point, they gradually converge as we advance from the equator, and vary in length with each degree of latitude, until they meet in the poles, and the longitude becomes nothing. 626. A Meridian is an imaginary circular line on the surface of the earth, passing through the poles and any given place. The meridian from which longitude is reckoned is called the first meridian, or standard meridian, and is marked 0°. All places East of the first or standard meridian, within 1809, are in East longitude; and all places West of the first or standard meridian, within 1800, are in West longitude. The English reckon longitude from the meridian of Greenwich; the French from that of Paris. ' The government of the United States usually reckons longitude from the English standard meridian, Greenwich. In American maps, the meridian of Greenwich is printed at the top and the meridian of Washington, the capital of the U. S., is printed at the bottom. The difference of longitude between two places is found like the difference of latitude, by subtraction or addition, as in compound denominate numbers. 1. The longitude of Galveston is 940 47' 26" West; of Liverpool, 39 4' 16" West. What is their difference # Ans. 919 43' 10". OPERATION. Ezplanation.—Since both places . in West longitude, we subtract the Long. of Galveston = 94° 47' 26". W. iesser from the greater. When one “ “ Liverpool = 39 4' 16" W. place is in West and the other in *-**-*. East longitude, we add the longi- º e O Al Q/ A / tudes of the two places together, and Dif, of longitude 91O 43/ 10 Ans. if the sum is more than iso degrees we subtract it from 360 degrees, because the shortest distance between the places would then be the other way around the world. 2. Cape Flattery, Wash., is in longitude 124° 44' 30" W. Hong Kong, Ch., is in longitude 1140 9' 32° East. What is their difference 7 Ans. 121° 5' 58". OPERATION. Long. of Cape Flattery = 1240 44' 30" W. 3600 01 0// “ “ Hong Kong = 114O 9/ 32// E. 238o 54/ 2// 238o 54/ 2// 121O 5' 58// 3. The longitude of Calcutta is 88.0 20' 11" East, of Trieste, 13° 46' East. "What is their difference # An S. 74O 34' 11". 4. The longitude of Havana is 820 21' 17" West, and of Antwerp 4o 24! 44” East. What is their difference # AImS. 86O 46' 1". 5. St. Petersburg is in longitude 300 19' 22" East. Cedar Keys is in longi- tude 83C 1’ 57’’ West. Find their difference. Ans. 113C 21’ 19//. (302) a a Aſº A- a —w wºr--ºr- 627. The circumference of the earth, being a great circle, is divided into 360 equal parts called Degrees of Longitude. The earth revolves on its axis, from West to East, once in 24 hours—which gives the sun the appearance of passing around the earth from East to West. *. Now, since the earth revolves once in 24 hours, all parts of its surface or circumference—the 360° of longitude—pass under the sun during that space of time. Hence, since 360° are passed under the sun in 24 hours, ºr part of 360°, or 15° of longitude are passed in 1 hour. Since 150 are passed in 1 hour or 60 minutes, ºr part of 15°, or 15' of longitude are passed in 1 minute. Since 15' of longitude are passed in 1 minute, or 60 seconds, ºr part of 15', or 15" of longitude are passed in 1 second. Then since 150 of longitude 1 hour of time, 1O 4 4. = + of 1 hour (60 min.) = 4 min. of time. Then Since 15' “ { % = 1 minute of time, 1/ 4. 4% = #5 of 1 minute (60 sec.) = 4 sec. of time. Then since 15// “ & 4 = 1 second of time, 1// 44 4% = +3 of a second of time; Or, since 1 “ {{ '- 4 seconds of time, 1// 4. § { # of 4 sec. = #5, or # of a second of time. Töö) 628. From the foregoing, we deduce the following: COMPARATIVE TABLE OF LONGITUTE AND TIME. 360° long. make a difference of 24 hours of time. 15O 4. {{ & 4 {{ 1 { % 4% 15/ 4% {{ 64 { % 1. minute 46 15// {{ {{ {{ { % 1 second { % 1O “ {{ { % 46 4 minutes 46 1/ {{ 44 {{ & 4 4 seconds { % 1// { % {{ { % { % *; second { % 629. Longitude and Time give rise to two classes of problems, as follows: 1. To reduce time to longitude, or to find the difference of longitude between two places, when the difference of time is known. * 2. To reduce longitude to time, or to find the difference of time between two places, when the difference of longitude is known. & (303) 3O4. SOULE's PHILOSOPHIC PRACTICAL MATHEMATICS. º: PROBLEMS UNDER THE FIRST CLASS, OR, TO FIND THE DIFFERENCE OF LONGITUDE BETWEEN TWO PLACES, WHEN THE DIFFERENCE OF TIME IS KNOWN. 630. 1. Reduce 11 hrs. 20 min. 40 sec. of time to longitude. OPERATION. 11 hrs. 20 min. 40 sec. Explanation.—Since according to the foregoing eluci- 15 dations, 1 hr. = 15° of longitude, 1 min. = 15' of longi- tude, and 1 second = 15" of longitude there are 15 times as many 9, ', and " of longitude as there are hrs., min., 1700 10/ 0’’ Ans. and sec. of time. Hence in all problems of this kind, we multiply the different units of time by 15, as in multipli- cation of compound denominate numbers, and thus convert them into units of longitude. 2. The difference of time between two places is 3 hrs. 15 min. 24 sec. What is their difference of longitude 3 - AnS. 480 51' 0/. 3. The difference of time between New York and Chicago is 54 min. 19 sec. What is the difference of longitude? Ans. 130 34' 45". 4. When it is 11 o'clock 16 minutes 1 # seconds A. M. at Boston, it is 10 o'clock A. M. at New Orleans. Find the difference of longitude. OPERATION INDICATED. Time at Boston 11 hrs. 16 min. 11%; Sec. ‘‘ ‘‘ New Orleans 10 hrs. 0 min. 0 Sec. Difference of time 1 hr. 16 min. 1 # Sec. 15 190 0/ 19''. AnS. 5. The longitude of Rome, Italy, is 12° 27' E.; the difference of time between Rome and Mobile, Ala., is 6 hrs. 41 min. 57+; sec. What is the longitude of Mobile, West? AnS. 889 2/ 28// W. OPERATION INDICATED. 6 hrs. 41 min. 57+; Sec. = dif. of time. 15 100C 29/ 28' = dif. of longitude. 12O 27, 0” = longitude of Rome, E. 88O 2/ 28' = longitude of Mobile, W. NOTE.-When the Dif. of Long. given is that of two places which are in opposite longi- tudes, and the Long. of one of them is given to find that of the other, we subtract the longitude of the given place from the Dif. of Long. and thereby find the longitude of the other. This is done because the Dif. of Long. between the two places is equal to their sum (one being E. and the other W.) and consequently, when the Long. of one is given, that of the other is found by subtraction. Should the Dif. have been that of two places in the same kind of Long., we would then have added the difference of longitude, if the lesser longitude were given; and sub- tracted, if the greater longitude were given, to find the longitude of the other. * 6. When it is 12 M. in New York, it is 18 min. 2; sec. past 11 o'clock at Cincinnati. What is their difference of longitude 3 AnS. 10O 29' 21". OPERATION INDICATED. 12 hrs. 0 min. 0 sec. 11 hrs. 18 min. 2; Sec. 41 min. 573 sec. × 15 = 100 29' 21". 7. The difference of time between Jerusalem and Baltimore is 7 hr. 28 min. 36 sec.; the longitude of Baltimore is 76 deg. 37 min. West; what is the longitude of Jerusalem, East 3 Ans. 35 deg. 32 min. E. * LONGITUDE AND TIME. 305 PROBLEMS OF TEIE SECOND CLASS. OR, TO FIND THE DIFFERENCE OF TIME BETWEEN TWO PLACES, WHEN THE DIF- FERENCE OF LONGITUDE IS GIVEN. 631. 1. Reduce 300 42' 50" of longitude to time. OPERATION, Explanation.—As shown by the foregoing elu- 15) 300 42/ 50// cidated table of longitude and time equivalents, 159 of longitude = 1 hr. of time, 15' of longi- º tude = 1 min. of time, and 15" of longitude = 2 hrs. 2 min. 51# Sec. Ans. 1 sec. of time; therefore, there will be ºr as many hours, minutes, and 8econds of time as there are degree8, minutes, and seconds of longitude, and hence we divide the 9, ', and '', by 15, as in divis- ion of compound denominate numbers, and thus reduce longitude to time. 2. The longitude of Paris, France, is 2° 20' 15" E., and of Berlin, Germany, 13O 23' 44' E. What is the difference of time 3 Ans. 44 min. 13++ Sec. OPERATION. 13O 23/ 44” = longitude of Berlin, E. 2O 20/ 15'' = & 4 “ Paris, E. 15) 11o 3' 29' = difference of longitude. 44 min. 13+3 sec. = difference of time. 3. The longitude of Bombay, India, is 72° 54’ East; of St. Louis, Mo., 900 15' 15° West. What is the difference of time, and when it is 10 A. M. in St. Louis, what is the time in Bombay ? OPERATION. 72O 54." 0’’ = longitude of Bombay, E. 900 15' 15’’ = % “ St. Louis, W. 15) 1630 9. 15” = difference of longitude. 10 hrs. 52 min. 37 sec. = difference of time, or the length of time it is 10 o'clock in Bombay, before it is 10 A.M. in St. Louis. 10 hrs. = St. Louis time. 20 hrs. 52 min. 37 sec., or 8 hrs. 52 min. 37 sec. P. M. in Bombay. 4. The difference of longitude between St. Paul and Cincinnati is 100 35' 24”. What is the difference of time 3 Ans. 42 min. 21; sec. 5. The longitude of New Orleans is 90o 4' 9". The longitude of San Fran- cisco is 1220 26’ 45". What is the difference in time, and when it is 12 M. in New Orleans, what is the time in San Francisco 3 OPERATION INDICATED. 122O 26% 45'' = longitude of San Francisco. 900 4' 9’’ = {{ “ New Orleans. 15) 320 22, 36” = difference of longitude. 2 hrs. 9 min. 30% sec. = dif. of time.—1st. Ans. 12 hrs. 0 min. 0 sec. = New Orleans time. 2 hrs. 9 min. 30% sec. = dif, of time, or time before 12 M. in San Francisco. 9 hrs. 50 min. 29# sec. = 50 minutes 29; seconds past 9 A.M., San Francisco time.—2d Ans. 3O6 soul E's PHILOSOPHIC PRACTICAL MATHEMATICs. *X 6. The longitude of Boston is 71°3' 30", and the longitude of Chicago is 87° 37' 45”. What is the time in Boston when it is 10 o'clock A. M. in Chicago Ans. 11 o'clock 6 min. and 17 Sec. A. M. OPERATION. 87O 37. 45'' = longitude of Chicago. 71O 3' 30’’ = * { “ Boston. 15) 160 34' 15° = difference of longitude. 1 hr. 6 min. 17 sec. =difference of time, or the length of time it is 10 A. M. in Boston before it is 10 A.M. in Chicago. 10 hrs. = Chicago time added. 11 hrs. 6 min. 17 Sec. A. M. Ans. 7. In traveling from Washington, longitude 770 0 15° W., to New Orleans, longitude 90o 4' 9° W., how much time will an exactly running watch appear to gain } Ans. 52 min. 15; Sec. 8. The longitude of Greenwich is 0, of Astoria, Oregon, 123° 49' 42*. How much earlier does the sun rise in Greenwich than in Astoria, Oreg. 3 Ans. 8 hrs. 15 min. 18; Sec. 9. When it is noon at Pekin, longitude 118 deg. East, what time is it at Paris, longitude 2 deg. 20 min. -15 sec. East? Ans. 4h. 17 m. 21 Sec. A. M. OPERATION INDICATED. 118. 2. 20. 15. 12. 15) 115. 39. º 7. 42. 39. 7. 42. an. 4. 17. 21. 10. When it is midnight at Halifax, longitude 63 deg. 36 min. West, what time is it at Canton, 113 deg. 14 min. East longitude 3 Ans. 11 h. 47 m. 20 Sec. A. M. THE LOSS OR GAIN OF A DAY BY TRAVELING AROUND THE WORLD. 632. Two very curious facts would be noticed, could we start on any given meridian with the sun on a certain date and journey with it around the earth, so to speak: The first fact to be noticed would be that in going around the earth from Blast to West, and upon arriving at our original meridian, we would find that we had lost one day's time, i. e. that we were one day behind the correct date. The jºr LOSS OR GAIN OF A DAY BY TRAVELING AROUND THE WORLD 3O7 Second fact noticed would be that if we should reverse our first direction and go around from West to East, we would gain one day's time, i. e. we would be one day ahead of the correct date. The meridian at Greenwich, Eng., is one of the standards of longitude for the world. When it is midnight on Sunday there, it is Sunday from there to 180° East, and Monday from Greenwich to 1800 West, or to the same meridian. Hence Ship captains change their time at this 180th meridian. Thus in traveling from the West towards the East, and upon crossing the 180th meridian on, say Sunday, the time on shipboard would be changed to Monday. And in traveling from East to West (in the opposite direction) and arriving at the 180th meridian, on Sunday, the time would be changed to Saturday." These changes are always made by nautical men in order that they may have the correct time of all localities East or West of the 180th meridian. There is also another line which may be named an International Date Lºve. This line differs slightly from the 180th meridian. The direction of this line, as indicated by Mr. Olney, the famous Scientist and Mathematician, is that it starts from the Chatham Islands, latitude 440 S., longitude 177° W., and runs north and west, to the East of New Zealand, New Guinea, and Borneo, thence through the Philippines, approaches China near Canton, and goes to the northwest of the Japan Isles, through Behring Sea, and terminates in the North Pole. The date on the east of this line is generally one day behind what it is on the west of it. —º *— /~ “g / tº sº ATIO. 633. Ratio, in the mathematical sense in which it is here used, means the measure of the relation which one quantity bears to another of the same kind, as expressed by the quotient of the first divided by the second; that is, it is the num- ber of times that one quantity (the first) is equal to another, (the second) which is used as a unit of measure. Thus, the ratio of 6 to 2, is 6-2, and is equal to 3. If we ask what is the relation of 6 to 2, the correct answer would be 6 is 3 times 2. We thus see that the ratio three is the number which measures the relation of 6 compared with 2, and therefore, that ratio is not merely the relation of two similar numbers, but the measure of this relation. NOTE 1.-Mathematical writers are not agreed in their definitions of the ratio of one num- ber or quantity to another. Many contend that it is the quotient of the second quantity divided by the first. Thus, the ratio of 6 to 2 is, 2 – 6 is #-- *. This is the preferred definition of Webster and of the Dictionary of Mathematics. But nearly all of the later writers as well as the German, English and most of the French, prefer the definition first above given, and we are in full accord with them. The result of the two systems is the same, although differently obtained. By the first definition We saw that 6 is equal to 2, 3 times; and by the second definition we see that 2 is equal to 6, # of a time, which expresses precisely the same ratio between the numbers. The question is, which number, the first or the second, shall be used as the unit of measure, or the divisor ? The prepon- derance of authority is largely in favor of the second. NOTE 2.-Numbers or magnitudes can have no ratio to each other unless they are of the Same kind. Thus, there can be no ratio between 6 dollars and 2 yards, for 6 dollars are not equal to 2 yards any number of times. 634. The Terms of a Ratio are the numbers compared. The first term of a ratio is the ANTECEDENT, which means going before ; the second term is the CON- SEQUENT, which means following. 635. The Sign of Ratio is the colon (: ), which is the sign of division, with the horizontal line omitted. Thus, the ratio of 6 to 2, is written, 6: 2. Ratio is also indicated by Writing the consequent under the antecedent in the form of a frac- tion. Thus, the ratio of 6: 2 is often written g. 636. An Inverse Ratio is the quotient of the consequent divided by the antecedent. Thus, the inverse ratio of 8:4, is 4. 637. The Walue of a ratio is the quotient of the antecedent divided by the Consequent, and is always an abstract number. 638. A Simple Ratio is the ratio of two numbers, as 8:4. 639. A Compound Ratio is the ratio of the products of the corresponding terms of two or more simple ratios, as follows: § {}= 6 × 8 : 2 × 4 ; or 3 × # is a compound ratio, - 6. 640. The Reciprocal of a ratio is the quotient of 1 divided by the ratio, or it is the quotient of the consequent divided by the antecedent. Thus, the ratio of 6 to 2 is 6: 2 or #, and its reciprocal is, 1 + 3 = }, or $. 641. The Ratio of two fractions is obtained by reducing them to a common denominator and then comparing their numerators. Thus, the ratio of #: # is the Same as 5: 2. 642. The Ratio of Compound Denominate numbers is found by reducing them to the same denomination and then making the comparison. (308) GENERAL PRINCIPLES. 3O9. From the foregoing definitions and elucidations, the following formulas and general principles are deduced: FORMUL.A.S. 643. 1. The Ratio = the Antecedent -- Consequent. 2. The Consequent = Antecedent -- Ratio. 3. The Antecedent = Consequent X Ratio. GENERAL PRINCIPLES. 644. 1. Multiplying the antecedent or dividing the consequent multiplies the ratio. 2. Dividing the antecedent or multiplying the consequent divides the ratio. 3. Multiplying or dividing both terms by the same number does not change the value of the ratio. 645. These general principles may be formulated into one General Law, as follows: Any change in the antecedent produces a LIKE change in the ratio ; but any change in the consequent produces an OPPOSITE change in the ratio. PROBLEMIS. 646. What is the ratio of the following numbers: 1. 10 to 5 3. 4 to 8 5. 54 to 8 7. 4; to #; 2. 18 to 6 4. 7 to 42 6. 11 to 60 8. ## to ## 9. What is the ratio of 5 gallons to 3 quarts 2 Ans. 63. 10. What is the ratio of 20g to $3? Ans. I's. 11. The antecedent is 12 and the consequent 4. What is the ratio ? Ans. 3. 12. The antecedent is 12 and the ratio is 2. What is the consequent ? Ans. 6. 13. The consequent is 16 and the ratio is 8. What is the antecedent ? Ans. 128. 14. The antecedent is 5% and the ratio is 4. What is the consequent ? AnS. 14. 15. What is the reciprocal ratio of 16 to 48? Ans. 3. 16. The reciprocal of the ratio of two numbers is 3 and the antecedent is 16. What is the consequent 3 Ans. 48. 17. What is the ratio of a pound Troy to a pound Avoirdupois? Ans. ##. 18. What is the ratio in grains of a pound of iron to a pound of gold 3 Ans. }}}. 19. The ratio of two numbers is 3, the antecedent is 15. What is the con- sequent? Ans. 5. 20. The ratio of two numbers is 4 the consequent is 14. What is the ante- cedent 3 Ans. 5%. 21. The inverse ratio of two numbers is 34 and the antecedent is 20. What is the consequent? Ans. 70. º 8 : 12 22. What is the value of { 10 : 5 } º Ans. 14. - º 23:4% OPERATION INDICATED. 23. What is the value of { 3} : 1; } º 3 || 8 30 7 24. What is the difference between the direct 3 || 10 and the inverse ratio of 2; and 7# 3 Ans. 24. 8 || 5 Ans. 1#. ºroportion. }) /9 eº --~~~~~~~~~~~~~~~~~~i= Ç' S, \ 647. Proportion arises from the comparison of ratios. It is a comparison of the results of two previous comparisons. Every proportion involves three com- parisons: the first two were those which produced the ratios, and the third, that Which compares or equates the ratios. Proportion is the expression of the equality of equal ratios, or, it is the comparison of two equal ratios. Thus, 6 : 2::15:5 is a proportion, and is read 6 is to 2 as 15 is to 5, or the ratios of 6 to 2 equals the ratio of 15 to 5. Thus, the ratio of 6: 2 as 15:5 is a proportion, i. e. four quantities are in proportion, when the first is the same multiple or part of the second, that the third is of the fourth. 648. The Sign of Proportion is a double colon (::), or the sign of equality (= ). Thus, the above proportion is expressed 6:2::15 : 5, or 6: 2 = 15 : 5. The first is read, 6 is to 2 as 15 is to 5. The second is read, the ratio of 6 to 2 equals the ratio of 15 to 5. 649. The Terms of a proportion are the numbers compared. 650. The Antecedents of a proportion are the first and third terms. 651. The Consequents are the second and fourth terms. 652. The Extremes are the first and fourth terms. 653. The Means are the second and third terms. 654. In the proportion, 3: 6::: 4:8, all the numbers are the proportionals ; 3 and 4 are the antecedents ; 6 and 8 are the consequents ; 3 and 8 are the eactremes; and 6 and 4 are the means. 655. Three numbers are proportional, when the ratio of the first to the second is equal to the ratio of the second to the third. Thus, 4, 8, and 16 are proportional, since 4: 8 : : 8 : 16, each ratio being #. In this kind of proportion, the second term is called a mean proportional between the other two. 656. A Simple Proportion is an expression of the equality between two equal ratios, as above elucidated. 657. A Compound Proportion is an expression of the equality between two ratios, when one or both of which are compound. 658. There are several other classes or divisions of proportion; viz: Con- joined, Partitive, Reciprocal, Medial, etc., but as we shall perform all kinds of proportional problems by our philosophic system, independently of the various terms and the ingenious classifications, or mechanical formulas used by most authors, we will not now occupy space with them. As we present the problems, which are generally classed under these various heads or divisions of proportion, We shall then define each, with our philosophic Solution. 659. The following General Principles are derived from the preceding elucidations: 1. In every proportion, the product of the eatremes is equal to the product of the means. 2. The product of the eatremes divided by either of the means, gives the other mean. 3. The product of the means divided by either eatreme, gives the other eatreme. (310) - ăţillinºiſ all Elliſilly Of NIIIlberS. 660. In defining ratio, we showed that there is a difference between the relationship and the ratio of two numbers; we shall now show in the following work, other differences which were not then indicated. Relationship of numbers is a more general term with a wider significance than ratio. And used as we shall employ it, it includes far more than the numerical measure, ratio, of one number by another. g Through the relationship and the equivalency of numbers, we shall, by comparison, analysis, and synthesis, solve all forms of ratio and proportional questions, simple, compound, reciprocal, etc., without regard to any of the preced- ing mathematical conventional terms, ratios, proportions, general principles, etc. Different kinds of relationship and equivalency of value may exist between things of the same, and things of different kind. Thus, the relationship and equivalency of monetary value, exist when 5 yards cost 50%, 1 hat cost $4, etc.; the relationship and equivalency of numerical value exist when 8 = 5, 2 = 3, etc.; the relationship and equivalency of exchange, or barter value exist when 10 pounds of rice = 24 oranges, 6 days labor = 18 bushels of corn, etc.; the relationship and equivalency of service or force and results exist when 30 men make 50 yards of levee in 20 days; or when a steamboat carries 1000 bales of cotton 500 miles for $2000, etc. And thus some kind of relationship and equivalency exists, like the law of compensation in nature, in every conceivable statement of numbers. The following problems and their solutions will more clearly show the rela- tionship and equivalency of numbers, and at the same time elucidate the philosophic system of work. SIMPLE PROPORTION. PROBLEMIS. 1. 9 pounds cost 45%. What will 3 pounds cost at the same rate 3 PROBLEMI CLASSIFIED. Explanation and IReason.—This is a problem in simple pro- Ib portion, and by our system of reasoning, it is as easily § is solved as a question in division. In all problems of all classes of proportion, there is one 3 number which is of the same name, nature, or denomination, OPERATION. as the Inumber or answer required. This number we use as the mature of the answer, and in the solution of problems, we first 5 write it at the top of the increasing side of the statement line. 4 In this problem, the nature of the answer is 45c. which 3 we Write on the statement line and reason as follows: Since 9 pounds cost 45c. 1 pound will cost the 9th part, 152, Ans which we indicate by writing the 9 on the decreasing side 9 º of the statement line; then since 1 pound costs the result of this statement, 3 pounds will cost 3 times as much, which worked, gives, 15c. the cost of 3 pounds. It will be observed that we classified the problem before stating the operation. This the learner or calculator should always do, in order to see more clearly the relationship and the equivalency of the numbers which are compared with the nature of the answer. C. 9 (311) SOULE's PHILOSOPHIC PRACTICAL MATHEMATICS. COMPOUND PROPORTION. 2. Three boys earned $72 in 12 days. How many dollars can 2 boys earn in 15 days? PROBLEMI CLASSIFIED. Explanation and Reason.—This is a problem in compound proportion, and by the philosophic system of work it is as bºy. #. º easily solved as a problem in simple proportion. By inspec. pº tion and reason, we see that $72 is the nature of the answer. 2 15 We therefore write the same on the iºgºs; º: statement line and reason thus: 3 boys earned $72. Since OPERATION. boys earned $72, 1 boy earned the 3d part, and 2 boys can $ earn twice as much ; then, since $72 are earned in 12 days, 72 in 1 day the 12th part will be earned, and in 15 days, 15 3 || 2 times as much will be earned. This completes the reasoning. 12 | 15 The operation statement is worked out by cancellation, as J explained in cancellation, and in other subjects treated. &= gº-mº- It will be observed that in giving the reason for writing $60, Ans. the terms, boys and days, on the statement line, we did not state all of the conditions or relationships of the syllogistic premises from which we deduced our conclusions of increase and decrease of the dollars. Thus, per example, with the term boys, we said: Since 3 boys earned $72, 1 boy earned the 3d part, etc. To consider the full conditions, or to state the full premises of this term, we would reason thus: Since 3 boys earned $72 in 12 days, 1 boy earned the 3d part, etc. To consider the full conditions of the term days, we would reason thus: Since $72 are earned by 3 boys in 12 days, in 1 day the 12th part will be earned, etc. To name all the conditions at each step of the analysis would require much time, and would rather confuse than aid the learner. For these reasons, and because the analysis is clear, correct, and logical, without naming all the conditions, we omit their useless repetition. CONJOINED PROPORTION. 661. Conjoined proportion is a proportion in which there are several terms of different kinds of which each preceding term or antecedent is of an equivalent value to a succeeding term or consequent. 3. If 4 apples are worth 3 oranges, and 2 oranges are worth 100 grapes, and 200 grapes are worth 6 peaches, and 9 peaches are worth 30 cents, then how many apples can we buy for 90 cents? PROBLEMI CLASSIFIED. Ap. Or. 4 - 3 Gr. 2 = 100 Pe. 9 = 30 90 OPERATION. Explanation and Reason.—This is a problem in conjoined proportion, and by the philosophic method of work it is 3 || 2 solved in the same manner as the two preceding problems. 100 | 200 Writing on the statement line the 4, which represents apples and which is the nature of the answer, we reason thus: 6 || 9 Since 4 apples are worth 3 oranges, conversely 3 oranges are 30 || 90 worth 4 apples; and since 3 oranges are worth 4 apples, 1 orange is worth the # part, and 2 oranges are worth 2 times as many; then since 2 oranges are worth 100 grapes, con- versely 100 grapes are worth 2 oranges; and since 100 grapes are worth two oranges, 1 grape is worth the Tło part, and 200 grapes are worth 200 times as many. Then since 200 grapes are worth 6 peaches, conversely 6 peaches are worth 200 grapes; and since 6 peaches are worth 200 grapes, 1 peach is worth the # part, and 9 peaches are worth 9 times as many. Then since 9 peaches are worth, 30c., conversely 30c. are worth 9 peaches; and since 30c. are worth 9 peaches, 1 cent is worth the ºr part, and 90c. are worth 90 times as many, The above is the full reasoning for the problem, and it gives healthful exercise to the mind of the student—exercise which sharpens and strengthens and logicalizes the mind for service not only in the domain of mathematics, but upon all questions of life. The reasoning may be abbreviated thus: Since 3 oranges = 4 apples, 1 orange = # part, and APPLES. 24 apples, Ans. * PROPORTION, 3 I 3 2 oranges = 2 times as many; then, since 100 grapes = 2 oranges, 1 grape = Tº part, and 200 grapes = 200 times as many; then, since 6 peaches = 200 grapes, 1 peach = }; part, and 9 peaches. = 9 times as many; then, since 30c. = 9 peaches, 1c. = }; part, and 90c. = 90 times as many. NOTE. 1.-The student should repeat the reasoning of the foregoing problems until it is perfectly understood, before advancing to other problems. NOTE. 2.-By this philosophic process of solution, we consider and compare with the nature of the answer but one number at a time, and the conclusion of all comparisons either increases or decreases the nature of the answer at the head of the statement line. Should the conclusion increase or decrease anything else, it is wrong, and the reasoning must be revised. NOTE. 3.-In all the reasoning for each number of every term, it will be observed that we make 1, which is the basis of all numbers, the standard unit of comparison or measurement; and that the reasoning is always from 1 to a collective number, or from a collective number to 1. GENERAL DIRECTIONS FOR PROPORTION 662. From the foregoing problems, explanations, and reasoning, we deduce the following general directions for all proportional work: 1. Classify the problem. 2. Write the nature of the answer at the top of the increasing side of the state- nent line. 3. Commence with such number of any term in the classified statement, that is the equivalent of the nature of the answer, and comparing one by one, each number of all the terms with the nature of the answer, consider and determine the equivalency or relationship easisting between them, and whether from the conditions and relationship the nature of the answer is to be increased or decreased, and then write the number on the increasing or decreasing side of the statement line, according to the conclusion reached by the comparison, the relationship and the conditions of the case. 4. When each number of all the different terms of the classified statement has been compared, considered, and written on the statement line, then work out the problem, canceling as much as possible. REMAIRKS. 663. Before presenting other problems, we desire to place before the student a few more facts pertaining to proportion. We wish to compare true proportion, the analytic system of proportion, and the philosophic system of proportion, and to make a few pertinent remarks regarding the illogical and mechanical systems of proportion taught by nearly all the text books now before the public. The philosophic system of solving proportional problems is not proportion, nor is it analysis. To elucidate this, let us take the following problem and Solve it, 19, by proportion; 29, by analysis; 3°, by the philosophic method: 5 yards of cloth cost $14. What cost 15 yards of cloth at the same rate 3 3I4. soul E's PHILOSOPHIC PRACTICAL MATHEMATICs. * 664. FIRST SOLUTION BY PROPORTION. Cost of 15 yds. : $14: ; 15 yards: 5 yds. Explanation.—This is the state 14 × 15 ment by true proportion, and by = $42, Ans. it we see that the cost of 15 yards Cost of 15 yds. = bears th I $14, th *> g ears the same relation to © Or, 15 : 5, 3 times; and 3 timeS $14 F. $42, Ans. cost of 5 yards, that 15 yards bear to 5 yards. By the arbitrary rules of nearly all the school arithmetics in the land, the principles of proportion are ignored, and in place thereof are substituted illogical and absurd mechanical operations by which answers are obtained. More than one hundred authors say in effect: “Write that number which is like the answer sought as the third term; then if the answer is to be greater than the third, make the greater of the two remaining numbers the second term, and the smaller the first term; ” etc. *, This process will produce the answer, but it is illogical, arbitrary, and absurd. It robs proportion of all claims to a scientific process, and the student who thus learns to solve proportional problems knows no more of true proportion than the Hottentot knows of the Christian religion. In the language of Prof. Brooks, when thus treated: “The whole subject becomes a piece of charlatanism, utterly devoid of all claims to science.” If proportion is used at all, it should be used as a logical process of reasoning, and the solution statements should be made by a truthful proportional comparison of the elements of the problem. For the higher mathematics, proportion is indispensable; but in arithmetic, proportional problems are more easily solved by analysis than by proportion, and still more easily by the philosophic method than by analysis. 665. SECOND SOLUTION BY ANALYSIS. OPERATION. $14 -- 5 = $24; Analysis.-5 yards cost $14. What will 15 yards cost? Since 5 yards cost $14, 1 yard will cost the # part of $14 = $2; Then $24 × 15 = $42, Ans. *..."; $';*. º, Sºhº. $23 = $42. By the analytic system no proportional statement is made, and no false claim thereto is assumed. The process of reasoning is less Scientific than in proportion, but much easier, and far more practical. The only serious objection that can be urged against the analytic system is the labor to produce each intermediate result of the alternate operations of division and multiplication. This objection is entirely obviated by the philosophic system, and all the merit of the axiomatical reasoning is retained. 666. The analytic system originated with Pestalozzi, a Swiss teacher and educational reformer, who was born in 1746, and who wrote and taught from 1781 to 1827. His first methods were quite crude, but yet they served as the germs from which have grown a large part of all modern improved methods of teaching. Yºk SOLUTION BY THE PHILOSOPHIC SYSTEM. 315 He aimed to take nature's method. He first taught his pupils to think, then to write, and then to read. He claimed that “the first one to read must have Written before he had anything to read.” In arithmetic he commenced with the simplest problems and taught his pupils to think, to reason, and to analyze every problem without regard to any rules. In 1805, his disciple, Joseph Neef, came to this country and opened a school on the Pestalozzian system near Philadelphia. But the people were not then pre- pared for the improved methods, and in a few years the school was abandoned. The spirit of the new system had however taken root in the minds of pro- gressive teachers who were pleased to find something better adapted to the wants of the educational world than the old arbitrary system; and from that time to the present, the Pestalozzian system has been growing in popular favor, and it now con. stitutes the basis of progressive educational systems throughout the civilized world. For more than 60 years past, most of the authors of arithmetics have utilized, to some extent, in various modified forms, the system of Pestalozzi. There Was no system of mental arithmetic before his day. e In 1820, Warren Colburn published what he called an “Introduction to Arithmetic on the Inductive or Intellectual System.” In this work he presented many of Pestalozzi's ideas and methods. From Pestalozzi's Analytic System of Arithmetic has been evolved the Philosophic System—a system without a peer in the annals of mathematics, and to extend, to practicalize, to logicalize, and to perfect which, the author of this book has labored with tongue and pen for more than a third of a century. He has taught the system to over 9000 students and has by his lectures and publications advanced the philosphic system to loftier plains and to more rational and logical methods than were ever before achieved. In no ancient or modern work on numbers, have comparison, analysis, and synthesis been woven into such a chain of logical and philosophical reasoning as is presented in this and the author's other works on numbers. The day is not far distant, when the banner of the philosophic system will wave in triumph from every spire, pinnacle, and dome of the Temple of Practical Mathematics. 667. THIRD SOLUTION BY THE PEIILOSOPHIC SYSTEM. OPERATION. *, 4. Premise.—5 yards cost $14. Reason.—Since 5 yards cost $14, 1 # | 15 3 yard will cost the # part, and 15 yards will cost 15 times as - $42, Ans much, which is $42. 7 e By this philosophic system, all the difficult scientific statements of true pro- portion and all the objections to the analytic method are obviated, and solutions are performed with the least possible number of figures, and by the easiest possible process of logical and axiomatical reasoning. 316 SOULE's PHILOSOPHIC PRACTICAL MATHEMATICS. º: * 668. Practical arithmetic has been sadly neglected by mathematicians, ancient and modern. Pythagoras, Plato, Euclid, and Archimedes, of ancient fame, Newton, La Place, Leibnitz, and La Grange, of modern renown, have enriched the literature of arithmetic, but they did not give to it the penetrating thought which they gave to the higher branches of mathematics. And our more recent authors of note, Davies, Peck, Ray, Greenleef, Perkins, Dodd, Brooks, Robinson, Thompson, Wentworth, Hill, and others, have adopted largely the arbitrary non-reasoning methods of their predecessors. They have failed to keep pace with the progress of an ingenious age. The world of thought in other departments of applied science has moved grandly forward and left the mathematicians of the 19th century contemplating and approving many of the monstrous absurdities of the “dark ages” of arithmetic. With the ancients, magic squares and the properties of numbers seem to have been deemed more important than the science or the practical application of arith- metic. With the moderns, mechanical methods to produce answers, without regard to reason, science, or brevity, seem to be the leading characteristic. * And thus arithmetic has been slighted. Only feeble efforts have been made to unfold and evolve its logical relations as a science, and to elucidate the philo- sophic principles which extend throughout all its parts. The brain-entangling cause and effect method of Prof. Robinson and some others, the complicated and confounding methods of Prof. Davies and a hundred others, seem to us a disgrace to the science of numbers, a discredit to the intelli- gence and genius of arithmeticians, and an insult to the common sense of mankind. The damaging effect of the arbitrary rules of these authors, on the minds of the rising generation, cannot be estimated. They fail to present the full meaning of problems, prevent the exercise of the reasoning faculties of the student, and produce a distaste for the science of numbers. TO THE STUDENT. The philosophic system, or the reasoning we employ in solving problems in proportion, is applicable to the solution of all arithmetical questions, and hence efficiency herein is a condition precedent to every other consideration. The reason- ing combines that of multiplication and division of whole and fractional numbers, in alternation, and there is no other exercise that can be given in numbers, that will give so much strength, acuteness, flexibility and comprehensibility to the reasoning organs of the brain. Thus preparing the mind not only to solve easily all practical problems in the most rapid manner, but also capacitating the person for efficient Service in all the positions and relationships of business life where conclusions must be deduced from the facts or premises, or from the circumstances and conditions of things relating to the subject under consideration. Therefore, we have presented a large number of problems in proportion and given the full reasoning in the Solution of many of them. Some of the more intricate problems are presented and solved more as logical drills for the mind than for the practical value of the problems. The student should º: PROBLEMS IN PROPORTION. 317 critically read and re-read the reasoning of such solutions and then Solve the prob- lems and write the reasoning without reference to that in the book. By this means, a high degree of capacity to reason, to conclude and to express thought with the pen, logically and grammatically may be attained. 669. PROBLEMS IN PROPORTION. CLASSIFICATION. Reason.—8 pounds cost 56c. 1. If 8 pounds of sugar Ig 㺠Since 8 pounds cost 56c., 1 pound will cost the art, and 27 cost 562, what will 27 27 the part. OPERATION. pounds will cost 27 times as pounds cost at the same C. much. # 7 NOTE.-The student should rate? Ans. $1.89. 3 work this problem and write | $1.89, Ans. the reasoning. CLASSIFICATION. Reason.—8 pounds cost 56c. lbs. C. Si 8 ds cost 56c. con- 2. If 8 1nce 8 poun © pounds of Sugar S 1; versely 56c. cost or bought 8 cost 562, how many poun pounds, and since 56c. bought Ž, y pounds organos. 8 pounds, 1C. will buy the 'g can be bought for $1.89% 7 ; 27 part, and 189C. will buy 189 | times as many. Ans. 27 pounds. #5 139 Note—The student should 27 lb work this problem and write 7 lbs. Ans. the reasoning. CLASSIFICATION. Reason.—$1.89 bought 27 $1.89 º; pounds. Since 189C. bought 27 3. If $1.89 will buy 27 56 pounds, 1c. will buy Tºg part, OPERATION. and 56c. will buy 56 times as pounds of Sugar, how many lbs many pounds can be bought for 1% ; 8 . NOTE.—The student should 562 % A. 8 *- work this problem and write g nS. 8 pounds. 8 lbs. Ans, the reasoning. CLASSIFICATION. Reason. # of a pound cost lbs $ $14. Since # of a ºn: 4. If # of a pound cost # 1; ; tºº : rº e 23 will cost 4 times as much. Then $1}, what will 23 pounds OPERATION. º 1 º fº. *i; * i. of a pound will cost the # par COSt 3 Ans. $54 2 $ 3 and * pounds will cost 21 times tº Q as much. 3 || 4 We leave this operation un- 8 || 21 cancelled in order that the - * statement may the better . * the student in understanding $54, Ans. & the reasoning. NotE.—The student should work the above problem, and should write the reasoning for it and all other problems that are solved. 3.18 soul E's PHILOSOPHIC PRACTICAL MATHEMATICs. 4% 5. A man walks 320 miles in 25 days, when he walks 6 hours per day. How many miles can he walk in 20 days by walking 8 hours per day? Ans. 341 miles. 6. § of a steamboat is valued at $35700. What is the value of the whole boat at the same ratef Ans. $40800. 7. A merchant borrowed $4000 for 64 days at 6 pr. ct. He now desires to compensate the party from whom he borrowed the money by loaning him $1500 at 8 Dr. Ct. How many days must he loan it? Ans. 128 days. OPERATION INDICATED. ds. 64 º * NOTE.-The student should classify the problem and 1500 | 4000 l Still (16 Dºlj SILOUILOl C y P 8 || 6 write the reasoning. tºms"º " sºmmemº 8. A merchant invested $15000 for 4 months and gained $800. How much investment would be required to gain $1400 in 3 months, at the same rate 3 Ans. $35000. 9. A note broker loaned $5000 for 30 days at 8%. The party to whom he loaned the money wishes to reciprocate the favor by loaning the note broker $2000 at 6%. How long a time should he lend it 3 Ans. 100 ds. 10. A farmer owned # of a farm and sold # of his share for $12400. What is the value of the whole farm at the same rate 3 Ans. $49600. 11. Tea costs 80 cents per pound. How much can you buy for 152, at the same rate % Ans. 3 ounces. CLASSIFICATION: OPERATION INDICATED. C. lb. OZ. 80 1 or 16 16 Oz. Write the 15 80 | 15 reasoning. 12. A baker charges 52 for a loaf of bread when he pays $7.25 per barrel for flour. What ought he to charge when he pays $8.70 per barrel for flour? Ans. 62. 13. If a 52 loaf of bread weighs 15 ounces when wheat is worth $1.40 per bushel, what ought to be the weight of a 1242 loaf when wheat is worth $1.00 per |bushel? Ans. 524 ounces. CLASSIFICATION. OPERATION. C. OZ, $. OZ. The student should write the 5 15 1.40 15 12; 1.00 5 reasoning by which the figures 2 25 are placed on the statement 1.00 | 1.40 g # line. 524 oz., Ans. 14. A baker charges ten cents for a loaf of bread weighing 8 oz., when he pays $6.30 per barrel for flour. How many ounces should the same price loaf weigh when he pays $7.20 per barrel? Ans. 7 oz. 15. If the fore-wheel of a carriage is 7 ft. 6 in. in circumference, and turns round 616 times, how often will the hind wheel, which is 12 ft. 10 in. in circumfer- ence, turn round in going the same distance? Ans. 360 times. PROBLEMS IN PROPORTION. 3 I 9 OPERATION. 16. 13% gallons cost $4}. $ What will # of a gallon cost at the same rate 3 Ans. $4. NOTE.—The student should classify the problem and write the reasoning. sº2 : : 17. If it requires 12 yds. of cloth to make a dress when the cloth is 38 inches wide, what must be the width of a piece 20 yds. long, to answer the same purpose? Ans. 22# inches. 18. A grocer has coffee at 1242 per pound, and a coal dealer has coal at 40% per barrel. If, in exchanging, the grocer puts his coffee at 14%, what should the coal dealer charge for his coal? Ans. 442 per barrel. CLASSIFICATION. OPERATION. g g g 124 14 14 40 25 || 2 Write the 40 reasoning. 44%, Ans. Or thus: (LASSIFICATION. OPERATION. 2 gain. 2 || 3 142 — 12#2 = 14% gain 25 2 40g | 40 NOTE.—The student should write the rea- 4#2 gain. soning for the operations. 40g value of coal. 44% price of coal. 19. A. has rice at 62 per pound, and B. has sugar at 72 per pound, which they wish to barter. Before bartering it is agreed that B. shall value his sugar at 7:42, and that A. shall advance his rice accordingly. What should be the exchange price of A’s rice 2 Ans. 6%. NOTE.-See Miscellaneous Problems in Percentage for full elucidation of this class of problems. 20. If, with $8.10, you can buy 9 yards of cloth, how many yards can you buy for $5.40% Ans. 6 yds. * 21. If 6 yds. cost $7.20, how much will 12 yds. cost 3 * Ans. $14.40. 22. If, for 12 cents, you can buy 3 of a yard, how many yards can you buy for 36 cents 3 Ans. 2 yds. 23. If ; of a ton of hay cost $15, how much will 3000 pounds cost Ans. $30. 24. If 20 pounds of sugar will supply a family of six persons for 7 days, how many pounds will be required to supply the same family 30 days? Ans. 85% lbs. 32O soulE's PHILOSOPHIC PRACTICAL MATHEMATICS. X- 25. If 3 men eat 3 dozen oysters in 3 minutes, how many men will it require to eat 100 dozen oysters in 100 minutes? CLASSIFICATION. OPERATION. M. D. Oys. Min. Men. 3 3 3 3 100 100 3 || 100 Write the 100 || 3 reasoning. 3 men, Ans. 26. If 3 men eat 3 dozen oysters in 3 minutes, in how many minutes can 100 men eat 100 dozen oysters ? s CLASSIFICATION. OPERATION. M. D. Oys. Min. Min. 3 3 3 3 100 100 100 3 Write the 3 100 reasoning. 3, min. Ans. 27. If 3 men eat 3 dozen oysters in 3 minutes, how many dozen oysters can 100 men eat in 100 minutes? CLASSIFICATION. OPERATION. M. D. Oys. Min. D. Oys. 3 3 3 3 100 100 3 100 Write the 3 || 100 reasoning. 3333; doz. Oysters, Ans. 28. If 24 acres of land will produce 550 bushels of potatoes, how many bushels will 8; acres produce at the same rate 3 Ans. 1925 bus. 29. If 13, hens lay 13, eggs in 14 days, how many eggs will 6 hens lay in 7 days? Proportional Answer, 28 eggs, Logical Answer, 24 eggs. OPERATION FOR PROPORTIONAL ANSWER. Reason for the Logical Answer.—Since 1; 2 || 3 = eggs hens lay 13 eggs, in 13 days, the period 3 || 2 7 for laying is hence 36 hours. And as 7 T--> days = 168 hours there will be as many 6 Write the periods for laying as 168 is equal to 36, 3 || 2 reasoning. which is 168 - 36 = 4 periods and 24 7 hours over. Therefore the number of eggs laid will equal the product of the 4 pe- tºmº || “…sºmeºmºsºmºsºms riods by the 6 hens, which is 4 × 6 = 24 28 eggs, Ans. eggs answer. NOTE.-Since 36 hours is the laying period, it is presumed in this solution that the hens do not lay any eggs during the 24 hours. 30. If 14 cats catch 1; rats in 14 minutes, how many cats will it take to catch 200 rats in 50 minutes? Ans. 6 cats. OPERATION. Cats. CLASSIFICATION. 2 || 3 C. R. M. 3 ão 1; 1; 1; 2 3 200 50 50 3. PROBLEMS IN PROPORTION. 32 I 31. A railroad contractor agreed to grade 450000 cubic yards in 90 days. He employed on the work 560 men for 30 days; then, measuring the grading completed, he finds that they have made 120000 cubic yards. How many more men must he employ at the expiration of the 30 days, in order to complete the work in the con- tract time 3 Ans. 210. PROBLEM CLASSIFIED : OPERATION. Yds. D. M. M. 450000 90 569 70 120000 30 560 2 60 || 30 --msºme *= 120999 || 3399% 330000 60 4. 11 770 men required. 560 men employed. 210 men needed. Explanation.—In all problems of this character, we must first find the number of men that will be required to perform the unfinished work in the unexpired time. We therefore place 560, the number representing men, on the increasing side of our statement line; and then to find the proportional numbers, we deduct from the whole work that was to be performed, and the time in which it was to be finished, the work that has been completed and the time consumed in its com- pletion, and in the remainder, as shown in the problem classified, we have the numbers containing the direct relationship with the numbers representing the work performed and the time consumed. Having these proportional numbers, we reason as follows: If it requires 560 men 30 days to grade 120000 yards, to do the work in 1 day instead of 30, it will require 30 times as many men, and to do the work in 60 days instead of 1, it will require the 60th part of the number of men; then, if it requires 560 men to grade 120000 yards, to grade 1 yard instead of 120000, it will require the 120000th part of the number of men, and to grade 330000 yards instead of 1, it will require 330000 times as many men. This completes the reasoning and statement, the result of which is 770 men, as the number required to perform the unfinished work in the unexpired time, and as 560 men are now employed, it is clear, by the exercise of our reason, that as many additional men must be employed as 770 is greater than 560, which is 210. 32. If 125 men, in 45 days, working 8 hours per day, can build 15 walls, each 150 feet long, 3 feet thick, and 12 feet high, how many men will be required to build 21 walls, each 200 feet long, 5 feet thick, and 9 feet high, in 14 days, working 10 hours per day ? Ans. 750. PROBLEM CLASSIFIED : M. D. H. W. F. L. F. T. F. H. Student should make the 125 45 8 15 150 3 12 statement and write the 14 10 21 200 5 9 reasoning. 33. A merchant owning # of a steamboat, sold # of his share for $45000. What was the steamer worth at that rate 3 Ans. $90000. 34. A besieged city, containing a population of 45000, has provisions for 4 weeks. How many citizens must be sent away, that the provisions may support the remaining population 40 days 3 Ans. 13500. 35. If a man walks 50 yards in ; of a minute, how far will he walk in 16# hours? Ans. 66400 yds. 36. Allowing the number of pulsations in a person to be 72 per minute, and 322 soul.E's PHILOSOPHIC PRACTICAL MATHEMATICs. Yºr that the velocity of sound is 1118 feet per second, what will be the distance of the electric fluid when 10 pulsations are counted between seeing the flash of lightning and hearing the sound of the thunder ? Ans. 93163 ft. PROBLEMI CLASSIFIED : STATEMENT : P. M. Ft. Sec. Ft. P. Ft. 72 = 1 1118 or, 1 1118 72 1118 10 72 60 1 m. Or 60 Sec. 10 10 37. A builder contracts to erect a wall 140 feet long, 11 feet high, and 24 feet thick, in 10 days. He employed 8 men who labored on the wall 6 days, during which time they built the wall 6 feet high, the required length and thickness. How many additional men must be employed to complete the work in the required time 3 Ans. 2. 38. In a piece of machinery there are two wheels running into each other: one contains 42 cogs and the other 33 cogs; in how many revolutions of the larger wheel will the smaller gain 10 revolutions Ans. 363. PROBLEM CLASSIFIED : STATEMENT : C. R. R. 42 10 1 33 9 || 33 *º-º-e 10 9 * 39. A contractor engaged to build 480000 cubic yards of levee in 90 days. He employed 500 men who worked 60 days and built 300000 cubic yards. How many additional men must be employed at the end of 60 days to complete the work in the contract time 3 Ans. 100 additional men. 40. A passenger train leaves the depot and runs at the rate of 30 miles per hour. 5 hours after the departure of said train, the fast mail train leaves and runs at the rate of 40 miles an hour. How far will the fast mail train run before it over- takes the passenger train } Ans. 600 miles. OPERATION INDICATED. 30 x 5 = 150 miles = distance of passenger train when fast express starts. 40 miles, speed of fast express train. 30 miles, “ “ passenger train. 10 miles, gain in a run of 40 miles. M. Fast Ex. 40 10 150 NOTE.—The student should write the tº-º reasoning for the operation. 41. A thief steals an article, and in running off with it he takes steps 6 feet long; after he has gone 150 feet, a police officer starts after him and takes steps 64 feet long; allowing that they both step equally fast, how far will the officer run before he catches the thief ? Ans. 1950 feet. PROBLEMS IN PROPORTION. 323. 42. The steamer Katie leaves the wharf at New Orleans, and runs at the average speed of 15 miles an hour. When the Katie had run 25 miles, the steamer R. E. Lee leaves the wharf and runs at the average speed of 18 miles an hour. How far will the Lee run before she overtakes the Katie : Ans. 150 miles. 43. A man has six hands hired, four of them at 75g a day and two of them. at $1.00 a day. He wants to advance them $6.00 on account, each one in the propor- tion of his daily pay. How much will each receive? C. OPERATION. 600 4 men at 752 per day = $3.00 5 3 2 men at $1. per day = 2.00 &ºm º º $3.60 + 4 = 902 to each of 4 $5.00 men at 75% per day. C. 600 $ $ 5 2 6 6 * | sºmºmºse or thus: 5 || 75 5 | 1.00 $2.40 - 2 = $1.20 to each of 2 *E= wºº ºmºsºme men at $1. per day. 90g. $1.20 44. Mr. A. leaves New Orleans on a journey, and travels at the rate of 25 miles a day; 6 days after Mr. B. leaves New Orleans, and travels the same road at the rate of 30 miles a day. How many days will it be before Mr. B. will overtake Mr. A. and how far does each travel? Ans. 30 ds. f A. travels 750 mi. B. travels 900 mi. 45. The interest on $5000, for 135 days, at 8 per cent. is $150. What will be the interest on $4500, for 96 days, at 6 pr. ct. ^ Ans. $72. 46. If a grocer uses a gallon measure that is ; a pint too small, what will be the true measure for 250 gallons of the false? Ans. 2343. OPERATION INDICATED. 1 gal. = 8 pts. – pt. = 73 pts. sold for 8 pts. = of 250 gals. =234 gals. true measure. Or, # pt. Short weight on each of 250 gals. = 125 pts. = 153 gals. short weight. 250 — 153 = 2343 gals. true measure. 47. A. paid B. $50 for goods, bought by the pound. When B. weighed the goods, he used a false weight of 14 ounces. What were A's loss and B's gain, and What was the true amount of goods received by A. ? Ans. A. loses # of weight bought. B. gains # of weight Sold. A. received $43.75 true value of goods. OPERATION. A. lost 2 oz. on each 16 bought = # = } of the weight bought. B. gained 2 oz. on each 14 A. having lost 2 oz. on each 16, he therefore received sold = # = } of weight ++ of $50 worth of goods = $43.75 = true value sold. of goods received by A. 324 SouLE's PHILosophic PRACTICAL MATHEMATICS. Yºr 48. A grocer used a false weight of 15+ ounces, instead of 16 ounces, for a pound; how many pounds did one of his customers lose when he bought 225 pounds just weight? Ans. 11+ lbs. OPERATION INDICATED. * OZ. OZ. 15} = false weight. 4 16 = true weight. 15#. # Oz. loss on each pound of 15% oz. Oz. loss on 154 oz. 3 = 16 225 177 ºr oz. = 11 lbs. 1 ºr oz. or 11:#f lbs. 49. In the above problem, what would the customer have lost had he bought 225 lbs. by the false weight? Ans. 10}} lbs. OPERATION INDICATED. OZ. -- OZ. 16 4 15+ 16 # oz. loss on a pound of 16 oz. 3 = Oz. loss on 16 oz. or 225 1 pound. Or, # oz. x 225 = 1685 oz. -- 16 = 10% lbs. 50. If a mixture of 80 gallons of brandy and water, consisting of ; of brandy and + of water, be worth $400, what would be the value of 100 gallons of pure brandy of the same quality? Ans. $666.663. 51. If a perpendicular post that is 6 feet high produces a shadow, on level ground, of 8 feet, what is the height of a tree, the shadow of which at the same time is 170 feet 3 Ans. 1274. 52. If $# will buy 12 peaches, how many peaches will $24 buy? Ans. 40. 53. If it requires 124 yards of cloth to make a suit, when the cloth is 14 yards wide, how many yards will it require when it is 14 yards wide 3 Ans. 15. PROBLEMI CLASSIFIED : STATEMENT : Yds. YolS. W. Yds. 12% 1; 2 || 25 14 2 || 3 - 5 || 4 54. If a boy can run 320 rods in 84 minutes, how many minutes will it take him to run 24 miles? Ans. 194 min. 55. 4 men can pick 1500 pounds of cotton in 24 days, how many men will it require to pick 84 bales of 450 pounds each, in 14 days? Ans. 20 men. 56. If 5 men, in 64 days of 53 hours each, can build a wall 1494 feet long, 34 feet thick, and 5% feet high, how many men will it require to build 2 walls, each 903 feet long, 4 feet thick, and 54 feet high, in 104 days of 84 hours each 3 Ans. 3. CLASSIFICATION. M. W. D. H. F. L. F.T. F. H. 5 1 6; 5; 1494 # 5% 3 2 104 8% 90; 4 54 PROBLEMS IN PROPORTION. 325 OPERATION. M. Reason.—Since it requires 5 men to build 1 wall, to build 2 walls it will re- 5 quire 2 times as many men; then since it requires 5 men when they work 6: 2 or * days, if they work but 3 day, it will require 13 times as many men, and 2 || 13 if they work # or 1 whole day, it will require # as many men; then since it - requires the number of men shown by the result of this statement when they 21 || 2 work 1 day, to work # day, it will require 2 times as many men, and to work 5 28 * (10% reduced) days, it will require the 21st part of the number of men; then, 17 | 2 since it requires 5 men when they work 5% or * hours per day, if they work 448 || 3 but # of an hour it will require 28 times as many men, and if they work # or 1 O whole hour it will require + as many men; then, since it requires the number 3 || 272 of men shown by the result of this statement when they work 1 hour per day, 13 || 4 to work # an hour, it will require 2 times as many men, and to work 24 (8% 4 reduced) hours, it will require the 17th part of the number of men; then, since 16 || 3 it requires 5 men to build a wall #8 (149% reduced) feet long, to build it # of a foot long will require the 448th part of the number of men, and to build it 3 or 1 4 21 whole foot long, will require 3 times as many men; then, since it requires the number of men shown by the result of this statement to build it 1 foot long, to build it of a foot long, will require # part of the number of men, and to build it 24% feet long will require 272 times as many men; then, since it requires 5 men to build a wall 3+ or '#' feet thick, to build it 4 of a foot thick will require the 13th part of the number of men, and to build it # or 1 whole foot thick, will require 4 times as many men; then, since it requires the number of men shown by the result of this statement to build it 1 foot thick, to build it 4 feet thick will require 4 times as many men; then, since it requires 5 men to build a wall 5% or '#' feet high, to Build it of a foot high will require the 16th part of the number of men, and to build it # or 1 whole foot high, will require 3 times as many men; then, since it requires the number of men. shown by the result of this statement to build it 1 foot high, to build it 4 of a foot high will re- quire + part of the number of men, and to build it ** feet high will require 21 times as many men. This completes the reasoning and the statement. OPERATION. R * sº B º 57. º If 4 boys can work 60 prob º: MOTE.—The student should lems in 3 hours, how many boys & wº * * k 40 bl 60 | 40 classify the problem and write will be required to wor proolems 8 || 3 the reasoning. in 8 hours? Ans. 1 boy. º sº-º-º-º-º-º OPERATION. 58. In what time can 50 men do "25 NOTE.-The student should a piece of work that 2 men can do 50 || 2 classify the problem and write in 25 days? Ans. 1 day. --- the reasoning. 59. If it costs $3750 to supply 250 laborers for 30 days, when they are fur- nished with 28 ounces of provisions per day, what will it cost to supply 175 laborers for 40 days, when they are furnished with 24 ounces of provisions per day ? Ans. $3000. 60. If it requires 126 pieces of paper to cover the walls of a room when the paper is 18 inches wide, how many pieces will it require when it is 21 inches wide? Ans. 108. 61. If the freight on 200 bales of cotton for 450 miles is worth 2400 pounds of sugar, when sugar is worth 1242 per lb., how many pounds of sugar would the freight on 350 bales of cotton for 300 miles be worth, when the sugar is valued 83% per lb 3 Ans. 4000. PROBLEMI CLASSIFIED : STATEMENT : B. M. P. C. f5. 200 450 2400 12; | 2400 350 300 8; 200 || 350 Write the 450 i 300 reasoning. 2 || 25 35 | 4 326 SOULE's PHILOSOPHIC PRACTICAL MATHEMATICS. 4× 62. If 24 men, in 124 days of 94 hours each, dig a ditch, when the earth is estimated at 4 degrees of hardness, 542; feet long, 4% feet wide, and 44 feet deep, in how many days of 74 hours each, will 248 men dig a ditch, when the earth is esti- mated at 74 degrees of hardness, 3873 feet long, 33 feet wide, and 13 feet deep, 'allowing that time is required in proportion to the degrees of hardness of the earth? A Ans. 4% days. 63. How many planks 13 ft. 9 in. long and 3%in. thick are equivalent to 5000 planks, 93 ft. long and 23 in. thick? Ans. 2750 planks. CLASSIFICATION. Reason.—Placing the 5000 planks on the statement line, we P. Ft. L. In. T. reason as follows: Since there are 5000 planks ºf ft. long, if 5000 93 2# they were but # of a foot long there would be 77 times as 131°; 3# many, and if ; or a whole foot long the 8th part; then, since f there is the result shown by this statement, when the planks OPERATION. are 1 foot long, if they were but I's of a foot long there would P. be 12 times as many, and if \# ft. long, there would be the 5000 165th part; then, since there is this indicated result when 8 || 77 the planks are + in. thick, if they were but 4 in. thick there 165 | 12 would be 11 times as many, and if 3 or 1 in. thick the 4th 4 || 11 part; then, since there is this result when they are 1 in. thick, 7 | 2 if they were 3 in. thick there would be 2 times as many, and if in. thick the 7th part. 64. If it costs $59.75 to carpet a room 32 ft. by 12 ft. how much will it cost to carpet a room 48 ft. by 16 ft. with the same kind of material % Ans. $119.50. 65. If 8 men in 6 days of 10 hours each cut 50 cords of wood, how many cords will 15 men cut in 15 days of 8 hours each 3 Ans. 1874. 66. If 3 compositors, in 24 days of 94 hours each, set up 28} pages of type, how many compositors will be required to set 680 pages in 44 days of 10% hours each # Ans. 33; men. PROBLEMI CLASSIFIED : STATEMENT : C. D. H. P. º C. 3 2+ 94 28% -- 3 4}. 10% 680 4 || 9 21 5 Write the 2 | 19 reasoning. 76 7 85 3 680 67. If 53 yards cost $8.20, what will 224 yards cost? Ans. $36. 68. If + of a pound costs $4, what will 1++ pounds cost 3 Ans. $2.124. 69. If 10 men in 6 days earn $240, how much can 6 men earn in 10 days? *. Ans. $240. 70. How many chickens at 70g a piece must be given for 2 barrels of flour at $7 per barrel ? Ans. 20. 71. If it costs a family of 8 persons $4000 a year to live in New Orleans, what will it cost a family of 9 persons to live in St. Louis in the same style for 10 months, supposing the prices to be in St. Louis + less than in New Orleans? Ans. $2812.50. PROBLEMS IN PROPORTION. 3.27. 72. If # of a pint of wine cost 37.4%, what will # of a hogshead cost at the same rate # Ans. $201,60. PROBLEMI CLASSIFIED : STATEMENT : P. C. C. # 37; 2 || 75 3 || 4 H. 2 Write the # 4 reasoning. 63 5 || 4 73. A garrison consisting of 4500 men, has provisions for 9 weeks, allowing each man 24 ounces per day. How many reinforcements may be sent into the garrison, so that the provisions will last but 5 weeks when the daily allowance is reduced to 16 ounces 7 Ans. 7650. 74. If it requires 2 men # of a day to perform # of a piece of work, how many days will it require 3 men to perform # of it? Ans. # of a day. PROBLEMI CLASSIFIED : STATEMENT : M. D. W. DS. 2 # # 4 || 3 3 # 3 | 2 Write the 2 || 3 reasoning. 9 || 8 75. If it requires 5 men # of a day to perform # of a piece of work, how long will it require 3 men to perform # of it 3 Ans. 24 d. 76. If the freight on 85 bales of cotton for 300 miles is worth 1000 pounds of sugar, when the sugar is worth 84% per pound, how many pounds of sugar must be given for the freight on 110 bales of cotton for 200 miles, when the price of sugar is 6#2 per pound? Ans. 11734 pounds. OPERATION INDICATED. lbs. 1000 85 | 110 NoTE.—The student should classify the prob- 300 200 lem and write the reasoning. 2 17 25 || 4 77. If a loaf of bread worth 5% weighs 64 ounces, when wheat is worth $1.80 per bushel, what should be the weight of a 12#2 loaf, when wheat is worth $1.62% per bushel? Ans. 18 oz. 78. If it requires 5 men # of § of a day to do # of § of 4 of a piece of work, what time will it require 4 men to do # of # of the work? Ans. 1; d. PROBLEM CLASSIFIED : STATEMENT : * M. Ds. W. Ds. 5 # x 3 = + 3 × 3 × # = } 4 | 1 - 4 5 Write the 4 # × # = # 8 reasoning. 15 || 8 328 soule's PHILosophic PRACTICAL MATHEMATICs. * 79. If 12 pounds of rice are worth 9 pounds of sugar, how many pounds of rice can be bartered for 217 pounds of sugar 3 Ans. 289; lbs. 80. If 25 oranges are worth 48 peaches, how many oranges can be bought for 240 peaches 2 Ans. 125 Oranges. 81. If it requires 1800 brick, 8 inches long, and 4 inches wide, to pave a side- walk 40 ft. long and 10 ft. wide, how many brick 10 inches long and 5 inches wide, will be required to pave a yard 50 feet long and 30 feet wide Ans. 4320 brick. 82. If # of a gill of Wine costs 12#2, what will 42; gallons cost, at the same rate # Ans. $342. 83. If 4 boys can make 100 hats in 6 days of 10 hours each, how many hats would 6 girls make in 10 days of 8 hours each, allowing that 4 girls can do as much work as 5 boys in the same time CLASSIFICATION, Boys. Hats. Ds. Hr. 4 100 6 10 10 8 Girls. 6 = 7# boys. FIRST OPERATION. Hats. 100 4 2 | 15 6 || 10 10 || 8 250 hats, Ans. SECOND OPERATION, Hats. 100 5 6 10 8 10 : 250 hats, Ans. Ans. 250 hats. Eacplanation.—Since the working capacity of each girl is greater than that of each boy, we must first find the equivalent working capacity of the 6 girls in the working capacity of boys. And since 4 girls can do the work of 5 boys, 1 girl can do # part = 1+ or # times as much as 1 boy, and 6 girls can do 6 times as much, = # x 6 = ** = 73 boys; i. e. the work done by 6 girls is equal to the work done by 7# boys. Explanation to the second operation.—Since 4 boys make 100 hats, 1 boy will make the + part, and 5 boys or 4 girls will make 5 times as many. And since 4 girls make the result of this state- ment, 1 girl will make the + part, and 6 girls will make 6 times as many. A ſhen, since 100 hats are made in 6 days, in 1 day # part would be made, and in 10 days 10 times as many. Then, since 100 hats are made when the hatters work 10 hours a day, to work 1 hour a day they woul." make T'ſ part, and 8 hours 8 times as many. 84. If 20 girls can make 200 shirts in 10 days, how many days would it re- quire 15 boys to make 300 shirts, estimating that 5 girls can do as much work as 6. boys in the same time? PROBLEM CLASSIFIED : G. B. S. D. 20 6 200 10 5 15 300 Ans. 24 ds. OPERATION: D. 10 5 20 Write the 15 | 6 reasoning. 200 300 PROBLEMS IN PROPORTION. 329 85. If it takes 7 men # of a day to do g of a piece of work, how long will it require 5 men to do # of the work? w Ans. # day. 86. A garrison of 3000 men has provisions for 42 days, allowing each man 2 lbs. 4 oz. per day. How many reinforcements may be sent in, so that the provisions Will last 10 weeks, when the allowance is reduced to 1 lb. per day ? Ans. 1050 reinforcements. 87. If 200 pounds of first grade rice cost $12, what will be the cost of 600 pounds of second grade, estimating that 5 pounds of the first grade are equal to six pounds of the second 7 Ans. $30. CLASSIFICATION. lbs. 200 1st grade $12. 600 2d grade, X # = 500 1st grade. FIRST OPERATION. Explanation.—Since 1 pound of the second grade = } of the first grade, 600 pounds of the second grade = 600 × # = 500 pounds of the first grade. Then, since 200 pounds cost $12, 12 1 pound will cost gºt part, and 500 pounds 500 times as much. 200 || 500 $30, Ans. SECONID OPERATION. t $ Ea:planation of the second operation.—Since 200 pounds of 12 first grade cost $12, 1 pound will cost gººd part, and 5 pounds 200 || 5 of first grade or 6 pounds of second grade will cost 5 times as 6 || 600 much. And since 6 pounds of second grade cost this result, tºº is 1 pound of second grade will cost # part, and 600 pounds of $30, Ans. second grade will cost 600 times as much. 88. If 45 pounds of coffee cost $9, what will be the cost of 90 pounds of a different quality, estimating that 4 pounds of the first quality are equal to 5 pounds of the second 3 Ans. $14.40. 89. If 5 oxen or 7 cows eat 3 ºr tons of hay in 87 days, in what time will 2 oxen and 3 cows eat 6 ºr tons ? Ans. 210 days. CLASSIFICATION. Explanation.—Since by the first condition of the problem, Ox C T DS 5 oxen or 7 cows performed the eating, and by the second 5 7 3-4 87 condition 2 oxen and 3 cows performed the eating, it is there- OI’ TT fore necessary to first find the equivalent eating capacity of 2 and 3 6-ºr 1 ox compared with that of 1 cow ; or the equivalent eating capacity of 1 cow compared with that of 1 ox. Hence, since 5 oxen eat as much as 7 cows, 1 ox will eat # part = 1%, or 4 times as much as 1 cow ; or since 7 cows eat as much as 5 oxen 1 cow will eat 4 part = } as much as 1 ox. Having now the equivalent eating capacity of 1 ox and of 1 cow each measured by the capacity of the other, we next find the eating capacity of the 2 oxen in that of cows; or the eating capacity of 3 cows in that of oxen, and since the eating capacity of 1 ox is 1%, or # that of 1 cow, the eating capacity of 2 oxen is 2 times as much = 3 × 2 = ** = 2; cows. This added to the 3 cows = 3 + 2} = 5% cows. Or, since the eating capacity of 1 cow is # that of 1 ox, the eating capacity of 3 cows is 3 times as much = } x 3 = ** = 2+ oxen. This added to the 2 oxen = 2 + 2} = 4+ oxen. Now, from these reductions of the capacity of oxen to cows, or cows to oxen, and the other conditions of the problem, we may solve the problem from either of the following classifications: 33O SOULE's PHILOSOPHIC PRACTICAL MATHEMATICs. CLASSIFICATION. With the eating capacity of oxen ex- pressed in that of cows. C. T. Ds. 7 3++ 87 5% #, OPERATION. Ds. 87 7 29 || 5 37 || 11 11 || 74 210 ds., Ans. CLASSIFICATION. With the eating capacity of the cows reduced to that of oxen. Ox. T. Ds. 5 3+*r 87 4} 6 ºr OPERATION. Ds. 87 5 29 7 37 || 11 11 || 74 210 ds., Ans. 90. If 4 men or 5 women can perform # of a piece of work in 27 days, how long will it require 1 man and 1 woman, working together, to perform the whole work? CLASSIFICATION. 1st OPERATION. Men. Wo. DS. W. Ds. 4 or 5 27 # 27 1 and 1 whole or 4 14 2#. # 9 || 5 3 4 80 ds., Ans. Ans. 80 ds. 2d OPERATION. Ds. Eacplanation.—Since 27 the work of 4 men is 5 = to that of 5 women, 9 4 the work of 1 man is = to that of 14 women; 3 || 4 and the work of 1 man tº-º-º: and 1 woman is = to 80 ds, Ans. 14 + 1 − 24 women's ? work. Or, since the work of 5 women is = to that of 4 men, the work of 1 woman is = to that of # of a man, or to # of the work of 1 man; and the work of 1 woman and 1 man is = to 3 + 1 = 14 men’s work. From these reductions and the , conditions of the problem, the classification and operation statements are made. NotE-The student should write the reasoning for each operation. This problem may also be solved as follows: 4 × 27 = 108 = days for 1 man to do # of the work. 5 × 27 = 135 = days for 1 woman to do # of the work. O l 3 º : #s = work done by 1 man in 1 day. = work done by 1 woman in 1 day. 1. *E=g '- tº t }rº + H+H = Ha = go which is the work done by 1 man, and 1 woman in 1 day. Then, since tº of the # of work is done in 1 day, to do #3 of the # work, will require 60 days. And since it requires 60 days to do # of the work, to do + will require # part of the time, which is 20 days; and to do # or the whole work will require 4 times as many days, which is 80. 91. If 4 of a pound costs 50¢, what will 4 pounds and 11 ounces cost, at the same rate 3 Ans. $9.374. 92. A grocer sold 40 gallons of a compound, consisting of 4 whiskey and 4 water, for $60. What would be the value of 50 gallons of pure whiskey at the same rate % Ans. $150. 34- PROBLEMS IN PROPORTION. 33 I 93. If 3 men and 5 women can do a piece of work in 14 days, 6 women and 8 boys can do the same work in 104 days, and 4 men and 7 boys can do it in 12# days. How long will it take 2 men, 10 boys and 3 women to do it? Ans. 9+} ds. OPERATION. First Step. Men. Women. Boys. Days. Work in 1 day, 3 + 5 E 14 = *r 6 + 8 = 10% = # Classification. 4 + 7 - 12; – # M. W. M. W. Work. (3 5 = +) × 6 = 18 + 30 = # = ºr Subtract upper Equa- W. B. W. B. tion from lower. (6 + 8 = #1) x 5 == 30 + 40 = }} Second Step. M. B. — 18 + 40 = ºr Multiply by 2. 4 + 7 = ºr Multiply by 9. M. B. — 36 + 80 = ºr Add the two Equations. 36 + 63 = }} B. 143 = ### 1 B. = ### Work in 1 day. Third Step. TM. B. M. W. 4 + 7 = #F = ºr 3 + 5 = 1°, (1 = ###)x 7 (1 = Hºg) × 3 = 7 = #F Subtract. = 3 = +}g subtract. M. 5 W. = +}; 4. = #F © 1 W. = ++g Work in 1 day. 1 M. = ++g Work in 1 day. Fourth Step. M. W. B. 1. 1 1 Iłg + T+g + ### M. W. B. 2 3 10 ++, + ++, + ºr = Pºs = work in 1 day; 12138 + 1224 = 944 days, Ans. 3 32 SOULE's PHILOSOPHIC PRACTICAL MATHEMATICS. - PARTITIVE PROPORTION. 670. Partitive Proportion is a proportion where a number is divided into parts which bear certain quantitative relations or ratio to each other. OPERATION. * Ea:planation.—Since A. re- 1. tº e ſº ceives 3 to B's 4, they, together Divide 35 oranges § # _A. § B. receive 3 + 3 + 7. "Hence, A. between A. and B. in the + = { will receive 3 and B. # = % of rati & A. B. the Oranges. And since + are “. O of 3 to 4. How many 35 35 to receive 35 oranges, + will Will each receive? 7 || 3 7 || 4 receive + part and 3, 3 times as Ans. A. 15: B. 20. *= ºsmºs *= ºs many. This gives A's share, y * Similar reasoning with the # 15, A. 20, B. gives B's share. 2. A father divided $66 as a christmas present among his six children in proportion to their ages, which were as follows: Albert 16, Eddie 14, Mamie 12, Frank 10, Robert 8, and Lillie 6. How many dollars did he give to each 3 Ans. Albert $16, Eddie $14, Mamie $12, Frank $10, Robert $8, and Lillie $6. 3. Divide $2222 into 4 parts, which shall be to each other as 1, 2, 3, and 5. How many dollars will there be in each part 3 Ans. 1st, $202; 2d, $404; 3d, $606; 4th, $1010. 4. A business man has $42000 invested in three kinds of stock—bank, insur- ance, and manufacturing. He has three times as much bank stock as insurance Stock, and 24 times as much manufacturing stock as bank and insurance stock together. What is his investment in each kind of stock # Ans. Insurance stock, $3000; bank stock, $9000; * manufacturing stock, $30000. Memorandum.—Insurance stock 1 + bank Stock 3 = 4 × 23 = 10 = manu- facturing stock; then, 1 + 3 + 10 = 14 = $42000. 5. An Egyptian army commander had 1300 camels, which he distributed among three separate general officers, in proportion to #, #, and #. How many did each receive 7 Ans. The 1st, 600; the 2d, 400; and the 3d, 300. 6. A benevolent man has $600, of which he wishes to give to A. §, to B. #, to C. #, and to D. #. What amount will each receive 3 Ans. A., $200 ; B., $150; C., $120; and D., $100. 7. Another benevolent man has $600 which he wishes to give to W., X, Y., and Z., in proportion to #, +, +, and #. What amount will each receive? Ans. W., $210.53; X., $157,89; Y., $126.32; Z., $105.26. NotE.—See Partnership Average for the solution of the two preceding problems. 8. James and Louis, together, have $124, and James has three times as much as Louis. How many dollars has each 3 Ans. James $93., Louis $31. Solution.—Let $1 represent what Louis has; then, since James has 3 times as many, he has $3, and together they have $1 + $3 = $4; then, $124 divided in the ratio of 1 to 3, gives Louis $3). and James $93. PROBLEMS IN PROPORTION. 333 9. Henry and William, together, have $296, and three times Henry's share equals 5 times William's share. How many dollars has each 3 Ans. Henry $185, William $111. Solution.-Since 3 times Henry's share = 5 times William's share, 1 time Henry's share = } of 5 times William's share, − 5 – 3 = 1; or § of William's share; and this added to William's share, which is 1, = } + 3 = , = what both have, or $296. Then, $296 divided in the ratio of # to 3 gives Henry $185 and William $111. * Or, thus: Since 5 times William's share = 3 times Henry's share, 1 time William's share = } of Henry's share which added to # (Henry's share in fifths) = $ the sum of both shares. Then dividing $296 in the ratio of # to #, gives William $111 and Henry $185. Or, thus: To avoid fractions, give Henry 5 shares and William 3, making 8 shares. Then since 8 shares equal $296, 1 share equals the # part, which is $37. Then 5 times $37 equals $185 equals Henry's share; and 3 times $37 equals $111 equals William's share. 10. A. and B. have $1020. § of A's is equal to ; of B's. How much has each 3 Ans. A. $540. B. $480. Solution. # of A's = # of B's, then # of A's = } of B's, which is 3 of # = #, and 3 thirds or the whole of A's = 3 times # or # of B's. Hence 3 + š, B's share = ** which is the sum of A's and B’s shares, or $1020. And ºr of $1020 is $60; and 9 times $60 = $540, A's share; and 8 times $60 = $480, B's share. 11. X. and Y. own a plantation containing 3500 acres, 4 of X’s interest is equal to # of Y’s. How many acres has each 3 Ans. X. 1500; Y. 2000 acres. 12. A pomologist has an orchard containing 1744 trees, apple, peach and pear; there are 3 times as many apple as peach trees, plus 150; and 4 times as many peach as pear trees, minus 120. How many trees are there of each kind? Ans. 122 pear, 368 peach, 1254 apple. NOTE.-Problems of this character belong properly to Algebra; but as they are frequently met with in arithmetics, we have given it place, and for those who have not studied Algebra, we give a brief solution. Solution.—Let pears = 1; then peaches = 4 – 120, (which is 4 times the pears — 120); then apples = 12 — 360 -- 150, (which is 3 times the peaches + 150). Then, since the pears, peaches and apples = 1744, it follows that 1 + (4 — 120) + (12 – 360 + 150) = 1744. Then, transposing or classifying we have 17 = 1744 -- 360 + 120 – 150 = 2074. Then, 17 = 2074, 1 = 122 pear trees. (122 × 4) — 120 = 368 peach trees. (368 × 3) + 150 = 1254 apple trees. MEDIAL PROPORTION. 671. Medial Proportion is a proportion where two or more quantities of different values are combined to find the mean or average value of the combination. This division of proportion is generally, and will be in this book treated under the subjects of Alligation Medial and Alligation Alternate. CONJOINED PROPORTION. 672. For a definition of Conjoined Proportion, see page 312. 1. If 40 oranges are worth 60 apples, and 75 apples are worth 7 dozen peaches, and 100 peaches are worth 1 box of grapes, and 3 boxes of grapes are worth 40 pounds of pecans, how many pounds of pecans can be bought for 100 oranges? Ans. 22# lbs. 334 SOULE's PHILOSOPHIC PRACTICAL MATHEMATICs. * 2. If 40 lbs. of sugar are worth 60 lbs. of rice, and 75 lbs. of rice are worth 500 oranges, and 150 oranges are worth 200 apples, and 250 apples are worth 16 doz. bananas, and 10 doz. bananas are worth 8 doz. peaches, and 16 doz. peaches are worth 50 lbs. of pecans, and 40 lbs. of pecans are worth 8 boxes of grapes, and 3 boxes of grapes are worth 2 boxes of raisins, and 7 boxes of raisins are worth 2 bbls. of flour, and 3 bbls. of flour are worth 6 bbls. of potatoes, and 2 bbls. of . potatoes are worth $7, then how many pounds of sugar can we buy for $40% Ans. 70+'s lbs. RECIPROCAL PROPORTION. 673. Reciprocal Proportion is a proportion where a number is divided into parts which bear certain, reciprocal quantitative relations to each other. 1. Four citizens of a neighborhood, A., B., C., and D., agreed to build a schoolhouse and to pay for the same in reciprocal proportion to the distance that each resides from the selected location. A. resides 2 miles; B., 1 mile; C., # mile; and D., 4 mile from the schoolhouse. The cost of the house was $1500. What does each Owe ? Ans. A. $100; B. $200; C. $400; D. $800. OPERATION. A. 2 miles. The reciprocal of 2 is # Explanation.-In solving problems, of B. 1 mile. 4% { % ** 1 is 1 this kind, we first find the reciprocal of O il {{ { % { % is 2 each number which represents the dis- . 3 mile. # is tance that the several parties named D. # Imile. {{ & 4 “ # is 4 reside from the school, or other object, for which the money is contributed. In * * ... fºr this problem, the reciprocal numbers are The sum of the reciprocals is 73 shown in the operation of the problem, and were obtained in accordance with & Article 250, page 147. 1500 These reciprocals show the respective 2 proportional parts that each party_is liable for, and the sum of the same, (7%) represents the whole cost of the school- smm mºm-mº smºsºms ºmº * -ºº-º-º: house, $1500. Hence to determine each $100 $200 $400 $800 party's liability, or the amount that each has to pay, we write the $1500 on the statement line, commence with A. and reason thus: Since "# (7%) parts are equal to, or have to pay $1500, 4 part equals or will pay the T's part and #, or a whole part, twice as much; and since i part equals or will pay this indicated result, 3 part (which is A's) is equal to, or wili pay, # as much. This shows A's amount to be $100. Similar reasoning is given for the statements of B., C., and D., which are exhibited in the operation. 2. A father left an estate of $81600, to be divided among his 5 children in reciprocal proportion to their ages, which are 3, 8, 10, 12, and 15 years. How many dollars will each receive? Ans. 3 years, $38400. 10 years, $11520. 8 {{ 14400. 12 “ 9600. 15 ($ 7680. A. B. 1500 2 15 500 500 1 15 2 15 2 º 1 º 4 PROBLEMS IN PROPORTION. 335 3. Suppose, in the above problem, the father had ordered that the money should be divided in proportion to their ages, how much would each have received? Ans. 3 years, $ 5100. 10 years, $17000. 8 * $13600. 12 “ $20400. 15 “ $25500. 4. A., B., C., and D., agree to contribute the necessary funds for the erec- tion of a mill at a certain specified place, in reciprocal proportion to the distance that they respectively reside from the location selected for the mill, on condition that they should be joint owners in the mill in proportion to the amount contributed by each in its erection. The distance that each resides from the Selected location of the mill, is as follows: A. 4 of a mile, B. § of a mile, C. 1 mile, and D. 24 miles. The mill cost $3500; what is the interest of each, A., B., C., and D., in the mill? Ans. A. #; B. #; C. #; and D. #. * * * * * * * * * * * * * * * * * > se e º ºs e º e s - e º 'º e º e s sº e º e e º e s e s tº tº e º " 674. To the Reader. Mensuration of surfaces and solids is, by most authors, located in the latter part of arithmetics. But, because of the general importance of the subject in business affairs, We present mensuration in advance of square root and percentage and the several subjects involving per cent. The subject is herein treated far more extendedly, practically and philosophically than in any other treatise. If the student has any special reason for acquiring a knowledge of percent- age, interest, exchange, etc., before he learns mensuration, he may omit this work until he has studied the percentage Subjects. The few problems involving the principles of square root, may be omitted by those unacquainted with that subject, or if preferred they may turn to the subject of square root, and in a few minutes acquire the knowledge thereof necessary to work the problems. It is presumed that those who possess this book are already familiar with the first principles of square root. 675. Mensuration is the process of finding the length of lines, the area of surfaces, and the volume or solidity of solids. The principles that govern the process of work are derived from Geometry, a very important and inter- esting branch of mathematics, but which cannot be fully explained in a treatise of this character. In order, however, to explain Some of the most simple and fre- Quently occurring geometrical terms and figures, we present the following GEOMETRICAL DEFINITIONS AND FIGURES. 676. A Point is that which has position with- O out measurable length, width, or thickness. 677. A Line is that which has length without measurable width or thickness. 678. A Straight Line is one that does not change direction at any point. _Ts 679. A. Curved Line is one that changes direction at every point. 2Ts 2^ 680. A Crooked or Broken Line is one that changes direction at some of its points. (336) * MENSURATION. 337 681. A Surface is that which has length and width, without measurable thickness. 682. A Plane Surface is a surface which lies even throughout its whole . extent. 683. A Curved Surface is a surface that has length and width, without measurable thickness, and like a curved line, changes its direction at every point. 684. A Wolume, Solid or Body, is that which has length, width and thickness. ANGLES. 685. An Angle is the 686. A Right Angle Opening or divergence of is an angle formed by tWO lines from a certain * One Straight line meeting oint _f | \_ another, so as to make p o equal angles. 687. An Acute Angle is | 688. An Obtuse Angle A. less than a right angle. is greater than a right angle. FIGURES. 689. A Plane Figure in geometry is a portion of a plane bounded by lines, straight or curved, or by both combined. When the lines which bound a figure are straight, the space that they inclose is called a polygon, or rectilineal figure. *— 690. A. Polygon is a plane \ figure, or portion of a surface bounded by straight lines. 691. A Square is an equi- lateral rectangle. B F- 692. A Rectangle is a & E 693. A Right-An- quadrilateral polygon which * ăgled Triangle is a tri- has its opposite sides equal *: and parallel, and all its an- # 5 angle that has one of # its angles a right an- gles right angles. * gle. # A. Base, In the right-angled triangle, A B C, the side A C, opposite the right angle B, is called the hypotenuse; the side A B, the base, and the side B C, the perpendic- wla, Upward of 2400 years ago, Pythagoras, a celebrated Grecian geometer, discovered that the square described on the hypotenuse of a right-angled triangle is equal to the sum of the squares described on the other two sides. And on making this discovery, to express his joy and gratitude, we are told that he sacrificed one hundred oxen to the Muses. Pythagoras also discovered that a circle is the great- est of all figures of equal perimeter. A 694. A Diagonal is a straight line connecting the vertices of two opposite angles of a polygon. As the line A B, in this | rectangle. The sum of all the lines bounding a polygon constitutes - its perimeter. B 338 SOULE's PHILOSOPHIC PRACTICAL MATHEMATICS. 4: 695. A Parallelogram 696. A Rhombus is an is a quadrilateral having equilateral oblique-angled its opposite sides parallel. parallelogram. ZLZ 697. A Trapezoid is a quadrilateral having only two of its opposite sides ' parallel. 698. A Trapezium is a quadrilateral having no two sides parallel. A 701. A Sca- lene Triangle # is a triangle f which has allits --J sides unequal. 700. An ISOS- celes Triangle is one having \ only two of its lsides equal. O OOO Pentagon. Hexagon. Heptagon. Octagon. Nonagon. Decagon. 699. An Equi- lateral Triangle is one having its three sides equal. - i º 702. A Circle is a plane figure bounded by a regular curved line, every point of which is equally distant from a point within called the center. For the definition of the Diameter, the Circumference, the Radius, a Chord, an Arc, a Sector, an Angle, and a Segment of a Circle, see Circular Measure, page 252. 703. The Ratio between the diameter and the circumference of a circle has been demonstrated in geometry to be as 1 to 3.1416; i. e., when the diameter of a circle is 1, the circumference is 3.1416, practically. NOTE.-This ratio cannot be exactly expressed in figures. The decimal has been carried by mathematicians to two hundred and fifty places, and yet the exact ratio was not obtained. It is impossible to square the circle. Van Ceulen, a German mathematician, first extended the approximation to 36 places by continually bisecting the arc of a circle. In honor of this achievement, the 36 numbers expressing the ratio of the diameter of a circle to its circumference were engraved on his tomb-stone. The following are the numbers: 3.1415926535897932.38462643383279502884, 704. The Area of a figure, or of a described object, is the measure of its surface in some square unit, as the inch, the foot, the yard, the mile, etc. 100 705. The Ratio between the area of a circle and of a square, one side of which is equal to the diameter too of the circle, has been demonstrated in geometry to be as 1 to .7854; i. e., when the area of a square is 1, the area of the circle is .7854. Or, if we divide a square surface into 10000 smaller squares and then cut off Area 10000 Area 7854. the four corners so as to make a circle, we will thus 10000–2146=7854. cut away 2146 of the smaller squares and have remaining in the circle 7854 of the smaller squares. 3. * MENSURATION. 339 2 706. An Ellipse is a figure bounded by an oval curved line. The transverse diameter, or azis of an ellipse is a line passing through its & 4 center in the direction of its length, as the line A. B. The conjugate diameter, or aſcis, is a line passing through the center of the a' ellipse in the direction of its width, as the line E D. - TABLE OF MULTIPLES. 707. The following table may be often used to advantage in the computation of mechanical and geometrical problems: - TABLE. Diameter of a circle × 3.1416 = Circumference. Radius of a circle × 6.28318.5 = Circumference. Square of the radius of a circle × 3.1416 = Area. Square of the diameter of a circle × 0.7854 = Area. Square of the circumference of a circle × 0.07958 = Area. Half the circumference of a circle × by half its diameter = Area. Circumference of a circle × 0.159155 = Radius. Square root of the area of a circle × 0.56419 = Radius. Circumference of a circle × 0.31831 = Diameter. Square root of the area of a circle × 1.12838 = Diameter. Diameter of a circle × 0.866 = Side of inscribed equilateral triangle. Diameter of a circle × 0.7071 = Side of an inscribed square. Circumference of a circle × 0.2251 = Side of an inscribed square. Circumference of a circle × 0.2821 = Side of an equal square. Diameter of a circle × 0.8862 = Side of an equal square. Base of a triangle X by 3 the altitude = Area. Multiplying both diameters and .7854 together = Area of an ellipse. Surface of a sphere × by # of its diameter = Solidity. Circumference of a sphere x by its diameter = Surface. Square of the diameter of a sphere × 3.1416 = Surface. Square of the circumference of a sphere × 0.3183 = Surface. Cube of the diameter of a sphere × 0.5236 = Solidity. Cube of the radius of a sphere × 4.1888 = Solidity. Cube of the circumference of a sphere × 0.016887 = Solidity. Square root of the surface of a sphere × 0.56419 = Diameter. Square root of the surface of a sphere × 1.7724.54 = Circumference. Cube root of the solidity of a sphere × 1.2407 = Diameter. Cube root of the solidity of a sphere × 3.8978 = Circumference. Radius of a sphere × 1.1547 = Side of inscribed cube. Square root of (; of the square of ) the diameter of a sphere = Side of inscribed cube. Area of its base x by # of its altitude = Solidity of a cone or pyramid, whether round, square, or triangular. Area of one of its sides × 6 = Surface of a cube. Altitude of trapezoid X # the sum of its parallel sides = Area. 34O SouLE's PHILOSOPHIC PRACTICAL MATHEMATICS. º: LINEAR MEMSURE, -º- 708. In the mensuration of lines, surfaces, and solids, we have three kinds of measure—line or linear, surface and solid. The unit of measure for lines is an inch, a foot, a yard, a rod, etc., or some subdivision thereof. PROBLEMIS. 1. The diameter of a circle is 50 feet. What is the circumference? Ans. 157.08 feet. Explanation.—In all problems oper ATION. of this kind, we multiply the O diameter by 3.1416, which is the 50 x 3.1416 = 157.0800 ft., Ans. ratio between the diameter and \ . the circumference. NOTE.—In all problems in Linear, Surface, or Solid Measure, the student should draw on his paper the outline of the figure to be measured, before performing the operation. 2. The circumference of a circle is 40 feet. What is the diameter ? Ans.-12.73 + ft. Explanation.—In all problems of this kind, OPERATION. we divide the circumference by 3.1416, which is the ratio between the diameter and the circum- 40.0000 -- 3.1416 = 12.73 + ft., Ans. ference of a circle. 3. What is the circumference of a circular garden, the diameter being 25 yards and 2 feet? Ans. 241.9032 ft. 4. What is the diameter of a circle whose circumference is 68 feet 9 inches f Ans. 262.605 in. 5. The radius of a circle is 40 feet. What is the circumference # Ans. 251.328 ft. 6. What is the perimeter of a nonagon, each side of which is 61 inches? Ans. 549 inches = 45 ft. 9 in. NOTE.-The perimeter of a body or figure is its outer boundary or the sum of all its sides. 7. What is the perimeter of a hexagon whose sides are each 4 ft. 3 in.? Ans. 25 ft. 6 in. 8. The perimeter of a duodecagon is 27 feet. What is the length of one side? Ans. 2+ ft. = 2 feet 3 inches. º: LINEAR MEASURE. 34. I TO FIND THE HYPOTENUSE OF A RIGHT-ANGLED TRIANGLE. WHEN THE BASE AND THE ALTITUDE ARE GIVEN. PROBLEMIS. 709. 1. The base of a right-angled triangle is 60 feet and the altitude 25 feet, what is the hypotenuse? Ans. 65 feet. NOTE.-Students who have not studied square root may omit this and the two following problems, until they have learned square root. OPERATION. 60° = 3600 Explanation.—In all problems of this kind, we extract = 2 * 25 wº-º 625 the square root of the sum of the squares of the base and 4225 altitude, and the result will be the hypotenuse. V4225 = 65 feet, Ans. 2. If one side of a square park is one mile, what is the distance from the center to either corner ? Ans. 3733.52 + feet. TO FIND THE BASE OR THE ALTITUDE OF A RIGHT-ANGLED TRI- ANGLE, WHEN THE HYPOTENUSE AND ONE OF THE OTHER SIDES ARE GIVEN. 710. 1. The hypotenuse of a right-angled triangle is 50 feet and the base 40 feet, what is the altitude : .* Ans. 30 feet. OPERATION. 50? – 2500 40? – 1600 Explanation.—In all problems of this kind, we extract *mº the 8quare root of the difference between the squares of the Difference, 900 hypotenuse and the given side, and the result will be the required side. V 900 = 30 feet, Ans. 2. The hypotenuse of a right-angled triangle is 410 feet, and the base is 90 feet. What is the perpendicular or altitude : Ans. 400 feet. TO FIND THE DIAMETER OR THE CIRCUMFERENCE OF A CIRCLE, WELEN TELE AREA IS GEVEN. NOTE.—The area of a figure is the quantity of its surface expressed in units of square measure, as an inch, foot, yard, acre, etc. PROBLEMIS. 711. 1. The area of a circle is 47124 square feet, what is the diameter? Ans. 244.94+ feet. 34.2 SOULE's PHILOSOPHIC PRACTICAL MATHEMATICS. º: Explanation.—In all problems of this kind, we first divide the area by OPERATION. .7854, and then extract the square root of .7854 ) 47124.0000 ( 60000 the quotient. To find the circumference, we would multiply the diameter by / V 60000 = 244.94 + ft., Ans. 3.1416, or divide the area by .07958, y and extract the square root of the quotient. 2. What is the diameter of a circle whose area is 5026.56 feet 3 Ans. 80 feet. TO FIND THE SIDE OF A SQUARE THAT IS EQUAL IN AREA TO A GIVEN CIRCLE. PROBLEMS. 712. 1. What is the side of a square equal in area to a circle whose diame- ter is 40 feet 3 Ans. 35.448 feet. OPERATION. .8862 Explanation.—In all problems of this kind, we multi- 40 diameter. ply the diameter by .8862, or where the circumference is *mºnººmºsºsº given, multiply it by .2821; the result in each case is the 35.4480 feet, Ans. side of an equivalent square - NOTE.--The decimal .8862, is the square root of .7854 and is the side of a square equal in area to a circle whose diameter is 1. The decimal .2821 is the square root of .07958, and is the side of a square equal in area to a circle whose circumference is 1 2. The circumference of a race track is one mile or 5280 feet. What is the side of a square field of equal area 3 Ans. 1489.488 feet. 3. The diameter of a circle is 60 feet, what is the side of a square of equal area 3 Ans. 53.172 feet. TO FIND THE SIDE OF A SQUARE INSCRIBED IN A GIVEN CIRCLE. PROBLEMIS. 713. 1. The diameter of a circle is 40 feet, what is the side of the inscribed Square ? Ans. 28.284 feet. OPERATION. Explanation.—In all problems of this * .7071 kind, we multiply the diameter by .7071, or 40 where the circumference is given, multiply it N 2% - by .2251; the result in each case is the side 28.2840 feet, Ans. of the inscribed square. 2. What is the width of one side of the largest square piece of timber that can be sawed from a log 30 inches in diameter Ans. 21.213 inches. 2} LINEAR MEASURE, 343 TO FIND OR MEASURE THE HEIGHT OF A TREE, OR THE HEIGHT TO A CERTAIN POINT OF TEIE SAME WITHOUT CUTTING OR CLIMBING TEIE TREE. PROBLEMIS. 714. The figure A B C D, represents a tree. What is the distance from C to B 3 Ans. 36 feet. NoTE.—The point C is placed 3 feet from the ground to facilitate the observation by the eye. Ea:planation.—First measure from the base of the tree a distance of, say 10 feet, and at this point, vertically erect a pole E F, that has feet and inches marked thereon. Then, place a second pole GH, say 2 feet outside of the first, and at a point I, 3 feet from the ground, place the eye and prolong the sight by the side of the first pole to the point B, on the tree and note at what height the line of sight strikes the first pole. Then, suppose in this example, that the line of sight touches the pole at the height of 9 feet, we have the following proportion of triangles: As 2 feet is to 6 feet, so is 12 feet to the distance from C to B, or, thus: If a triangle of 2 feet base gives 6 feet height, 1 ft. base will give # part, and a triangle of 12 ft. base, (the distance from C to I) will give 12 times as many feet height. STATEMENT. 6 = ft. high. 2 | 12 36 ft., Ans. NOTE 1.-The 6 ft. height is found by subtracting 3 feet from 9; 9 being the pole mark of the line of sight and 3 feet being the distance that the line C I is from the ground. NoTE 2.—The difference in the diameter of the tree at the ground and at the point B, is not considered in the problem. For practical work of this kind this element need not be considered. This problem is of value to lumbermen, for by it they may determine the height of trees up to a required diameter, or to a limb or crook before cutting the tree and in case the height is not what is required, the tree need not be felled. 2. In the above problem, suppose the line of sight on the pole E F had passed the figures or indicated a height of Say 10 ft. 5 in. ; What would have been the height of the tree from the ground to B Ans. 474 feet. STATEMENT. Ft. 12 | 89 10 ft. 5 in. — 3 ft. = 7 ft. 5 in. = 7 a ft. = #ft. 2 12 443 ft. from C to B. 3 ft. = distance from C to the ground. ºsmºm- 473 ft., Ans. 344 SOULE's PHILOSOPHIC PRACTICAL MATHEMATICs. * 3. What is the height of a tree, from the ground to the top, when the pole E F is placed 40 ft. from the tree, the pole H G is placed 14 feet beyond the pole E F, and the line of sight strikes the pole E F at a height of 18 ft. 4% inches, and the eye point being 3 feet above the ground? Ans. 4283 ft. OPERATION. Ft. Ft. 18 ft. 4; in. — 3 ft. = 15 ft. 4; in. = 1844 in. = 334 in. 2 || 369 2 || 369 40 ft. -- 14 ft. = 414 ft. 12 Or 12 || 498 12 in. in 1 ft. 3 || 2 18 º 2 | 83 498 in. *ss= | flººmºsº 4253 ft. 3 ft. 4283 ft., Ans. 4. If in triangles like the above, the pole F E is 24 ft. from the tree, the pole H G is 3 feet beyond pole F E, the eye point or line C I being 3 feet from the ground, and the line of sight on pole FE crosses the figure 16 ft. 3 in., what is the length of the tree from C to Bº Ans. 1194 ft. TO FIND THE LENGTH OF A PART OF A TREE BROKEN OFF AT THE TOP WIEHEN ONE END OF THE BROKEN PART RESTS ON THE STUMP AND THE OTEHER END IS ON TEIE GEROUND. PROBLEM. 715. 1. A tree 100 feet high broke in a storm, and the top touched the ground 40 feet from the foot of the tree. What was the length of the portion broken off 3 Ans. 58 feet. Explanation.—In all problems of this kind, we add the square of the base to the square of the height of the tree, and divide the sum by twice the height of the tree, the quotient will be the length broken off... Or, divide the square of the base by twice the height of the tree, and to the quotient add one-half the height of the tree. OPERATION. \ 402 = 1600 1600 + 10000 = 11600 -- 200 = 58 feet. 1002 = 10000 à Y LINEAR MEASURE. 345 TO FIND OR MEASURE THE DISTANCE BETWEEN TWO POINTS ONE OF WHICH IS INACCESSIBLE. A. C PROBLEMIS. 716. 1. What is the distance between A and B measured from the point B without access to A * Ans. 90 feet. Explanation.—To determine this distance, we resort to the principles of Geometry and proceed thus: First prolong an indef- inite straight line drawn through the points A and B say to C. Then from the point B measure at right angles to the line A B C, any convenient distance, say 30 feet to the point D. Then project an indefiniteline drawn from A through D making line A. D. Then from point D and perpendicular to line A D project a line to the line A C until it intersects the line A B C at the point, say O; then measure the distance from B to O which, say is 10 feet. Then the distance from A to B =#. Thus: 30 x 30 = 900 ft. 900 ft. – 10 = 90 feet. This principle of Geometry may often be applied to advantage by any one, to measure the distance across rivers or other inaccess- ible places. 2. Considering the above diagram, what is the dis- tance from A to B, when the distance from B to D is 50 feet and the distance from B to O is 6 ft. 4 in. : Ans. 394++ ft. 3. What is the width of a river in paces and feet, when the distance from B to D in a diagram like the above, is 60 paces and the distance from B to O is 94 paces, supposing each pace to be 3 feet Ans. 385% paces, 11574 feet. TO FIND THE DISTANCE BETWEEN TWO INACCESSIBLE OBJECTS, As Two TREES ON THE OPPOSITE SIDE OF A RIVER, PROBLEMIS. 717. 1. What is the distance between A and B in the diagram, which are located on the opposite side of a river ? Ans. 700 feet. Explanation.—In all problems of this kind, first find the dis- tances AC and BC by the preceding proposition, which are, say, 500 and 600 feet. 2d. Divide these distances by 10 or 100, by point- ing off one or two places and thus obtain proportional distances; in this problem, we divide by 100 and thus obtain 6 and 5 as the proportionals. 3rd. Lay off on the prolongation of AC and BC their respective proportionals. 4th. Connect and measure ab, which is the proportional for AB, and multiply the same by 100, the product will equal the distance AB. In this problem, we assume that ab is 7 feet, which multiplied by 100 equals 700 feet the distance from A to B. 346 soul E's PHILosophic PRACTICAL MATHEMATICS. * * 2. Standing on a plateau, I can see two mountain peaks to the westward. I ascertain the distance to the one on the left to be 25 miles, and to the one on the right to be 30 miles. Laying off the proportionals 25 feet and 30 feet, I find the distance between them to be 40 feet. What is the distance between the peaks f Ans. 40 miles. MENSURATION OF SURFACES, —-tº- 718. The unit of measure for surfaces is a square whose side is some fixed length, such as an inch, foot, yard, rod, etc. In the solution of problems, in mensuration of surfaces, much time may often be saved by applying the following principle which has been demonstrated in geometry: That all similar surfaces or areas are to each other as the squares of their like dimensions. TO FIND THE AREA OF PARALLELOGRAMS, EITHER RECTANGLE, REHOMBOLD, RHOMBUS, OR SQUARE. PROBLEMS. || || Z 7 719. 1. What is the area of a rectangular garden 240 ft. long and 120 ft. Wide 3 Ans. 28800 sq. ft. OPERATION. 240 ft. long. Explanation.—In all squares or rectangular figures, we 120 ft. wide. multiply the length by the width, in the same units of *=º 28800 sq. ft., Ans. measure, and in the product we have the required area, 2. How many square yards are there in the surface of a wall 212 feet long and 32 feet high 3 Ans. 753% sq. yards. 3. What is the area of a rectangular floor 20 feet long and 14 feet wide Ans. 280 sq. feet. * MENSURATION OF SURFACES. 347 f 4. How many Square feet in a right-angled triangle whose base is 12 feet, and perpendicular height is 8 feet? Ans. 48 sq. ft. OPERATION. Ea:planation.—In all problems of this kind, 12 ft. long. we multiply the length by the height which 8 ft. high gives the sq. ft. of a rectangle of equal • Illgil. length and height. This result is then di- *memºmºsºmamm- vided by 2, since a right-angled triangle is 96-1-2=48 SQ. ft., Ans. equal to but one-half of a rectangle of equal length and height. 5. What is the area of a triangle whose base is 40 feet and altitude or per- pendicular height 18 feet? Ans. 360 sq. feet. TO FIND THE AREA OF A TRAPEZOID. PROBLEMIS. 720. 1. How many square feet in the surface of a trapezoid whose parallel sides are respectively 108 and 130, and the perpendicular distance between them 32 feet 7 Ans. 3808 sq. feet. OPERATION. Explanation.—In all problems of 108 this kind, we multiply the sum of the 130 parallel sides by the perpendicular 238 × 32 = 7616 distance between them, and take # of 7616 -- 2 = 3808 sq. ft., Ans. the product. 2. What is the area of a trapezoid whose parallel sides are 30 and 40 feet and whose perpendicular is 15 feet 3 Ans. 525 sq. feet. TO FIND THE AREA OF A TRAPEZIUM. PROBLEMIS. 721. 1. How many square feet in a trapezium whose diagonal is 42 feet, and the perpendiculars 18 feet and 16 feet? Ans. 714 sq. feet. OPERATION. Explanation.—In all 42 ft. length of diagonal. problems of this kind, ë ; a 111. we multiply the diagonal 34 ft. Sum of the perpendiculars. by the sum of the per- gº- pendiculars which in- 2)1428 product. tersect it from opposite tº-º-º-º-e angles, and take # of the 714 sq. feet., Ans product for the area. o • ? e 2. What is the area of a trapezium whose diagonal is 30 feet, and the perpendiculars are respectively 7 feet 6 inches, and 5 feet 8 inches? Ans. 1973 sq. feet. NOTE.--To find the area of an irregular polygon, divide the figure into triangles and tra- peziums, and find the area of each separately. The sum of these areas will be the area of the whole polygon. 348 soule's PHILOSOPHIC PRACTICAL MATHEMATICS. TO FIND TEIE AREA OF A TRIANGLE. EXAMPLE. 722. What is the area of a triangle whose base is 40 feet and altitude or perpendicular height 18 feet? * Ans. 360 sq. feet. OPERATION. Explanation.—In all problems of this kind, 40 ft. length of base. we multiply the base by the altitude, and take 18 ft, altitude. one-half of the product, and the result will 2 ) 720 be the area. 360 sq. ft., Ans. TO FIND THE AREA OF A SCALENE TRIANGLE, WHEN THE THREE SIDES ARE GIVEN. 723. 1. How many square feet are there in a triangle whose sides are 30, 40 and 50 feet 1 Ans. 600 sq. feet. OPERATION. 30 60 & 60 60 40 30 40 50 50 -> tº- º -ºm 30 1st remainder. 20 26 remainder. 10 3d remainder. 2) 120 60 60 × 30 × 20 × 10 = 360000 V 360000 = 600 feet., Ans. Ea:planation.—In all problems of this kind, we first add the three sides together and take half their sum ; then, from this half sum we deduct each side separately; then multiply together the half sum and each of the three remainders, and extract the square root of the product; the result will be the required area. 2. Suppose the base of a triangle to be 200 feet and each of the other two sides is 50 feet; what would be the area 3 AM.S. Nothing. NOTE.-This problem has puzzled thousands of students. Does it mystify you ? 3. What is the area of an equilateral triangular figure each side of which is 40 feet, 7 Ans. 692.8-H ft. OPERATION INDICATED. Explanation.—In all problems of this-kind, square one side and multiply the product by 40 x 40 = 1600; .433. The .433 being the ratio between the area of a square and the area of a triangle, 1600 × .433 = 692.8 + ft. = Area. the side of which is equal to one side of the square. See Table on page 350. To find the area of the face or base of different kinds of regular polygons, multiply the Square of one side by the tabular area set opposite the polygon of the same number of sides, as shown in the Table on page 350. * MENSURATION OF SURFACES. 349 TO FIND THE AREA OF A CIRCLE, when THE DIAMETER OR THE CIRCUMFERENCE IS GIVEN. PROBLEMIS. 724. 1. What is the number of square yards in a circular piece of ground which is 20 yards in diameter? Ans. 314.16 sq. yds. OPERATION. Explanation.—In all prob- lems º this kind, lº º: e tiply the diameter by itse 20 yds. diameter = length. º§ shown in the operation, 20 yds. diameter = width. and then multiply this pro- e-ºº-e duct by .7854, which has 400 = the sq. yds. in a square which is 20 yds., long º tºº. and 20 yards wide. ratio between the area of 400 × .7854 = 314.1600 sq. yds., Ans. a square and the area of a circle, the diameter of which is equal to one side of the square. 2. What is the area of a circle whose circumference is 157.08 feet? Ans. 1963.5 sq. feet. OPERATION INDICATED. 157.08 -- 3. 1416 = 50 ft. = diameter. Then 50 × 50 = 2500; 2500 × .7854 = 1963.5 square feet area. . Or, if it is preferred, the square of the circumference may be multiplied by .0795S. NOTE.-The decimal .7854 is + of 3.1416, and is the area of a circle whose diameter is 1; and .07958 is + of (1 – 3.1416); or, .07958 is + of the diameter of a circle whose circumference is 1. 3. The diameter of a circular garden is 56.4188 + feet. How many square feet does it contain 3 Ans. 2499.99 —H = practically 2500 sq. ft. TO FIND THE AREA OF A CIRCULAR RING, OR THE AREA INCLUDED BETWEEN THE CIRCUMFERENCES OF TWO CIRCLES HAVING A COMMON CENTER. PROBLEM. 725. What is the area included between two concentric circles, the diam- eters of which are 15 and 8 feet? Ans. 126.4494 sq. feet. OPERATION. 152 = 225 Explanation.—In all problems of 8* = 64 this kind, we multiply the difference * =º between the squares of the diameters by Difference, #, the decimal .7854, and the product will be the area. 126.4494, Ans. 35o SouLE's PHILOSOPHIC PRACTICAL MATHEMATICs. $ºr 726. 15 feet 3 PROBLEMI. OPERATION. 20 15 300 .7854 235.6200 sq. feet, Ans. TO FIND THE AREA OF AN ELLIPSE. What is the area of an ellipse whose diameters or axes are 20 and Ans. 235.62 sq. feet. Explanation.—In all prob- lems of this kind, we multiply the two diameters together, and their product by the deci- mal .7854, and the result will be the area. For the operations to find the entire surface of cylindrical, triangular, octag- onal, and other solids, see Mensuration of Solids. TABLE. 727. The following Table gives the names of the ten regular polygons, and shows their areas when the side of each is equal to 1. It also shows the length of the radius of the inscribed circle. No. of sides. Names. Areas. Radius of inscribed Circle. 3 Triangle, 0.4330127 0.2886751 4 Square, 1.0000000 0.5000000 5 Pentagon, 1.7204774 0.688.1910 6 Hexagon, 2.5980,762 0.8660254 7 Heptagon, 3.6339124 1.03826.17 8 Octagon, 4.8284271 1.2071068 9 Nonagon, 6.1818242 1.3737887 10 Decagon, 7.6942088 1.5388418 11 Undecagon, 9.3656404 1.2028437 12 Duodecagon, 11.1961524 1.8660254 To illustrate the use of this table, we present the following problems: 1. What is the area of an equilateral triangle, whose sides are 40 inches? 40 x 40 = 1600; 1600 × .4330127 = 692.8203200 OPERATION. Ans. 692.82032 sq. in. Explanation.—In all problems of this kind, square one of the sides of the polygon and multiply the product by the tabular area number set opposite the named polygon in the above table. NOTE.-This operation is based upon the principle that the areas of similar polygons are to cach other as the squares of their like sides. * 2. What is the area of an octagonal polygon one side of which is 30 inches f Ans. 4345.58439 sq. in. Operation indicated. 30° x 4.8284271. 3. The side of a decagon is 20+ feet. What is its area 3 Ans. 3233.4912 -- sq. feet. (204)” x 7.6942088. * Operation indicated. X- MENSURATION OF SURFACES. 35 I TO FIND TEIE AREA OF AN IRREGULAR POLYGON OF FOUR OR MORE SIDES. 728. 1. Divide the figure into triangles by diagonals connecting some one angular point with each of the others. 2. Compute the area of each triangle. 3. Add their respective areas and the sum will be the area of the polygon. TO FIND THE CONVEX. SURFACE OF A PRISM OR A CYLINDER. PROBLEMIS. 729. 1. What is the area of the convex surface of a pentagonal prism whose altitude is 8 ft. and each side 4 ft. ? Ans. 160 square feet. sº sº s º - OPERATION. |||{i : º º 4 ft. x 5 = 20 ft. perimeter, 20 ft. x 8 = 160 sq. ft. i i †† º d ſ Explanation.—In all problems of this kind, multiply the perimeter of the base by the altitude. | 2. What is the area of the convex surface of a trian- ſº •r º º *- | #|| gular prism whose altitude is 64 feet, and the sides of its base i are 3, 5, and 7 feet respectively 3 Ans. 974 sq. ft. | | º | | | OPERATION. | . | ##|| 3 ft. -- 5 ft. -- 7 ft. = 15 ft. perimeter. 15 ft. × 64 ft. = 974 sq. ft. Ans. *:: **esses **- -- ºs--~~~~ 3. What is the area of the convex surface of a cylinder, whose altitude is 4 feet 3 inches, and the circumference of its | base is 5 ft. 6 in. OPERATION. 4 ft. 3 in. = 51 in. ; 5 ft 6 in. = 66 in. ; 66 in. x 51 = 3366 sq. in. equals 23 sq. ft. 44 sq. in. convex surface. . . . . # , ; ; ; NOTE.--To find the entire surface, add the area of the bases or ends to the convex surface. 352 SOULE's PHILOSOPHIC PRACTICAL MATHEMATICS. Yºr TO FIND THE AREA OF A SEGMENT OF A CIRCLE WHEN THE DIAMETER AND THE HEIGHT OR VIERSED SINE ARE GIVEN. PROBLEM. 730. What is the area of a segment of a circle whose diameter is 72 inches and the versed sine is 12 inches? Ans. 446,0313 + Sq. in., area. OPERATION BY THE VERSED SINE METHOD. Versed sine 12 in. -- the diameter, 72 in. = .1666; = quotient of versed sine. Find the quotient of .166 in the column of versed sines, in the following table; then take the area number which is in the next column to the right, and multiply it by the square of the diameter, the result will be the area of the segment A D B. The tabular number is .08554 × 72* = 443,4393 + sq. in., area of segment. As the quotient of the versed sine by the diameter was .166; instead of .106, the area would be nearer accurate to use as a multiplier the number opposite .167 in the table. This number is .08629; and this x 72° = 447,3273 + sq. in., area of segment. For greater accuracy we would proceed as follows: .166 = .08554 .167 = .08629 Difference, .00075 × .63 = .00050 + .08554 = .08604, the sum by which the square of the diameter is to be multiplied. Then, .08604 × 72* = 446.0313 + sq. in., area of segment. NOTE 1.--To find the area of a segment greater than a semi-circle, find the area of the lesser segment and subtract the same from the area of the whole circle. NOTE 2.--To find the area of a zone of a circle, find the areas of the two segments and subtract the sum from the area of the whole circle. NOTE 3.−To find the chord of a segment of a circle when the diameter and versed sine are given, see Article 734, page 356, also Article 807, page 398. "SGIOVAIRIſlS HO NOIJLWXIſlSNGIW * £g£ LI36I' 9." j6ffIt fa' 9I960° 8I' 88890° aI' #26TO’ 90° 9&L6 I" 666" 60ff"I' 686" A,8960' 6LI' jL390" 6II' 1.1810" 690° †896I' 863.” jøgſ I’ 883" 9760° 8LI” 60320” 8II* 98IO' 890° 3796 I’ 163" 636; I’ L93" #8860” J.J.I." gif|T90' 1II’ 881. IO" 190° Igj6I’ 963,” fg|If I' 983.” Z0860” 91.I." 8090" 9II' 181, I0° 990° 986 I' 96%" 690; I' Q96" I9360° 91.I." 9IO90° GII' I69I0° QQ0 8936 I" f63' #869 I' #93' QQI60° fºLI' 99.670' fII' 97.910' fg0° LLI6I" £63' 68 I’ 893" 8060' 81. I* 68870." 3II' I09I0° 920° 9806I' 363" QI88I' 333" f0060' 31, I’ 93.870." ZII' 999 IO’ Øg.0" g668 I’ T63" I918 I’ I93' 63680° ILI' 991.70° III" ČIGIO’ IGO' g068 I’ 63" 9799.I.' 93." 39880” LI” Dj'O' II' 89ſLO’ ç0° fºL88I’ 68%" Č998I’ '666' 611,80° 69 I' 89970' 60I' gåf IO" 670° 83/8 I' 886" 81,78.I.' 863.” f0180° 89 I' g1970° 80I' 38g.TO’ 8f0° £898 I’ 186" #693 I’ L33' 63.980° 191° jLgſ'O' MOI' 689 IO' 1.70° 3798.I.' 98%" IIggſ' 92.3" fgg80° 99T." agff.ſ)" 90I' 16&IO’ 9f0° &gſ'8I' 98%" 1338T." 933." 8780' g9I' I6370' QOI’ ggz IO’ g;0° I988.I.' f86" ## Ig|I' #23' 90780° j9I' Igſ'O' f0I’ #IZIO’ ff0° 31,38 I' 98%" 908I' 93.3" 38880” 99.I.' 69370° gOI" £1.TIO" gf0° G8I9 I' 386" 1,166 I’ 333" 89.380° 39L." 80&f()' 30I* £g|II0° &f()" 3608 I' T86" j68&I' I33" Q8I80° I9I' 8ñIf.0° TOI’ 96OIO' If0° 3008 I' 83" II83 I' 63" III.80° 9I' 180f O’ I' fgOIO" f0° &I62, I’ 61.3." 831&I’ 6T3" 88080” 69 I’ 16070" 660° QIOIO’ 690° 3382, I’ 813" 9796 I’ 8I3" Q961.0° 89 I' 896.90° 860” 91.600° 880” £8/2, I’ L13' 9996I’ J.IG" 3681.0° 191" 80690° 160° 89600° 190° $792, I’ 91%." I8pg|I’ 9IZ" 38/0' 99.I.' 6f.880" 960° I0600: 990° Tºggº, I' 913.” 6683 I’ g|I3" 1.7/10" ggſ' 61,90° g60' f0800° g30° g972.I.' f/6" J.I.93 I" #Ig" Q1910° #9 I' 381.90° f60' LZ800° j90° 9/8LI' 813' 983&I." 9I3" 809/10" 99T- #1930° £60° I6100° 990° 1832, I’ 313' 99 IGI’ &IG" I8910° 39 I' 9I990" 360' 99.100° 380° E6L2, I' IZZ” I/03 I’ IIZ" 69'ſ 10° Ig|I' 899.90° I60' Iz100' I90° 60Iſ...I' Jó," 66II* Ig’ 1881.0° g|I' I0990° 60° 98.900" 90° 300, I' 696," 806 II* 603° 9I910' 6;I' jjig0' 680° 9.g.900" 630° I869 I’ 893" 138LI” 803.” 973.10" 8FI’ 18890° 880” 61900' 830° f/89 I’ 196" 97 LII* J.06" fºLI10' 1.f. I’ 8890" 280° 18900' 130° 99/9I' 993.” Q99TI." 903" 80T10' 9f I’ f/.330° 980” QQQ00° 930° 9999 L’ 993." #89 II* Q03' 99.010" gfL' 8I330° ç80° gzg00' 930° 8/99.I.' j93" 909 II." #03" Č9690° ff I’ 29I90' #80° £6700° #20" 679.I.' 893" göf II* 906" £6890° £f I' 10I90° £80° Z9500" 930° 3079 I' Č93' gig II" 303" Č890° &#I' Čg090° 380° zgi ()0° 330° fºL89 I' I93" 39&II* I03" 99.190° If I' 16630" I80° £Of()0° IZO" 9339 I' 93.” 38III" z' 88990' #I' gp630° 80° g/.900" 30° 63T9.I.' 693." 20III" 66].' #I990° 69 I’ 68330° 610" 1j900" 6T0° I909.I.' 893" 930II" 86I" gj990° 89 I' g8830° 81.0° £900' 8IO" #969 I' 193" 9760I’ 16I" 91.f30° I,8I? £81,30° 11.0" #62.00° 1.T0° 9189 I' | 996' j980I' | 961" || 10790' | 98I" || 66130' | 910 || 89200 910' 68/9I’ G93" j810I’ g6I' 68890° Q9I" 91930° 9/10" #200° gTO" 301.9 I' j93" Q010I’ j6I" IL390° fgI’ #3930' #10" £300° jTO’ g|I99 I' 893" 93.90I’ 96I’ 80390° 99T- IL930° 81.0° 16IO0° £IO" 839QI' ØØ" 1790I’ 66L" Q9I90° 38L." 6IQ30" 3.10" QLI00° &IO" Ifºg I' Iç3" 89.70ſ." I6I' 19090° IgE." 89%0° ILO" 99T00° IIO" gºgg|I' 93" 680I’ 6I' 90° gI' D.If 30° 10' £g|IOO" I0° 8939 I' 673' &I80I' 681 ° 99.690° 6&I' 99960' 690° 9II00' 600° £8IGI’ 873' 983OI’ 88I' 998.90° 8&I’ çI930° 890° g6000" 800° g609 I' Ajø." gg|OI’ 18L * 661,90° 1&I." g9360' 190° 81000' 100° 6009 I' 9%’ 1100I ' 98.I.' 88.190° 9&I' GIZZO' 990° £9000' 900° $367 I’ Gjø" I' 98.I." 99990° Q&I' Q9I30° g00° 17000' Q00° 1887 I’ ##2" Č660° #8L 990° jøI' LII60' £90' #9000' #00° Ig/j. I’ gfa' gf860” 98 I* iggg.0° 9&I' 89.030" 990° £3000" 900' Q99; I' " 191.60" 38I." 69f20° 33 I' 3.030" £90° &IO00' 200" 89? I' Ifg" 6960° I8I' fOfg0° IZI* 316I()" I90° #0000" I00" Q "3 III e sº ‘eup º º •ouſ - Cr *9 UII e ‘eory ºes | poss § ea.IV "Eos tº ge.IV floS pº, eary ‘Eas pº “eaty 30S pºº, iº ‘S.LXIV.I. 'IVſlöGI CINVSſ)0HJ, GINO OLNI (ICICIIAICI CINW ‘XLINſ). CIGI OL CIGINſ)SSV GITO:HIO v HO IILGIWWICI GIHL ‘(IIOSHIO V IO SLNGIWł)GIS EIHL HO SVGISIV (HHI, HO GITIVI, Igº. +g? ‘soilvywah.LVW Tvollovia or HaOSOTIHà S.3TſhoS * 1368" | g" iſ 1399." 9ſ' #09.Ig" gy" 98.91%." 83" || 1.j933" fg” LI63" | 667' 71, Igg" 697" 90% Ig" 6Iſ," 6831.3" 619" £gf83” 683" 1,068." 867" Q1098." 89p" J.0II8" 8If" 36T13." 818." 89.98%" 893" 1688" 1,67" Q1679.’ 1.9f" 800 Ig" LIf" g601.3" 1.19% 89.38%" 188” 1888" 967" Q1879." 99p" 60609" 9If" 8669%" 918" 691.93" 99.9° J.188" g6?" 91.1.j9" ggſ' II803.” g|If" T0693." 91.8% f/093° ggg" 1988” #67" 91.978." fgſ." ČI103" jJi' #0893" fºL3’ 8633" fgg" 1988." 96.7" 1.99 fg" ggſ' #I903" g|If" 80/93.” 3/8" 98863.” 893" J.W88” 267" 11.778." Øgiſ' g|IQ09" &If" II993." 31,8" I6/.33" 399." 1988” I6'5" 819.j9." Igy" J.If.08° IIſ." #Iggz" II,8" 1,6933." Igg" 1,388" 67." 81,378" g;" 6I908° If" 8Ij93" 18" 30933” 99" LI88” 6.87° 61, Ifºg." 6ff" 6309" 60ſ." Tø893" 698" 60%." 633" 1088.” 887° 61,078." 877° 3&IO3." 80ſ" g3393' 898.” g|Iſºza," 82.3" 26/8" J.87° 8698" Ljj,” £3009" 107° 8&I93" 198" Tø936." Løg" 1818." 98?” 8888" 9ff" Q3666." 90ſ" 38.09%" 998." 833%" 933." 2.),),8" g87° I8198" giff" J.3863.” Q07” 99.69%" Q93" fg|IZZ” g33" 1918" #87° 38998." #ff' 631,63" f0ſ' 68893" f09" f03%" fºg." 1918" 987' 38.999" | * giff' I8966." 90ſ' Øf 1,93' 998." 1.76T3' 92.3" Ji, ),8" Z87° 98793.” zji" 99966,” 3Of" Lj993." £99" 898T3" &g" 1918" I87° f8899." Iff' g£763" IOf" Igggz" I93" 91.I.Z.' Izg" 131,8" 87° i8388" ##" 18863.” ſ' ggſ ga." 99" 199T3" 29" J.I.19.' 617° g8T99" 63+" 68.363" 668" 69893" 698" 819. I&" 6Ig" II.018" 81,7° 98099." 89?' If I63" 868." 99326’ 898.” 8?I&” 8Ig" TL698" J.Lſ” 18638" 1.97° gif|O63" 168° 19TQ3" 198" 188T3" J.Ig" I1899." 91.7" 18838" 99p" gf686" 969.” I).093° 998." 76&It," 9Ig" TL198" Q1.j 881,38" ggſ' 8#886" Q63" 91.67%." Q98” I0&IG" g|Ig" I1998” fli' 68938" fgſ' Q186" f63" 8873" jgg" 80IIZ" fTg." I?,999' 91.7" 6938" ggſ' Čg.98%" 868." †SLjø" 999' g|IOI&" 9Ig" II.j99' 31.j 6f 69." Zgſ” #9986,” 368" 689;&" £gg" 33603.” 3Ig" IL898." ILſ” I6938" Igip' 19 fº," I68" 969.73% Igg" £80%." IIg" 31398.” 1.7" 96638" gif' 69.88%" 68" 867 fº,” gg" 881.03" Ig" 31,[98' 697" f6Izg" 6Zi" Č938%" 688° gOffa' 6fg" gj903" 609" 31,098.” 89?" Q6038" 8&#" tº I33" 888° 109.jø" 8fg" 99.90%" 808" 31,698° 197" 966 Ig" 137' 19086" 188" ČI" 1.f3" 9f 03° 108" & 1899." 997" 168 Ig" 9&#" 696).3" 988” J.IIf?" 9fg" 899.03" 909." $11,98' goi' 861, Ig" gaſ” 31.81%" Q99" £30%" gig." 91.306." Q09" 9/1999' fgj' 66918" faj' 91.1.1%." j99° 13683" fjg' jSIO3." #09" £1999' g3?' 9Ig" 93.7" 1,1913.” 888" 38883,” gfg" 36006" 903" jºigg" Z9f" 309 Ig" &#" 08913.” £88” 19183" afg" z' 308" iſ 1999' I9F' 3Of Ig" IZip' £8f 13' I88” ϗ. Ifg" 8066 I’ I09" •eely 5es º •eoly 39S pº, “eary 30S tº, “ea.IV -3oS pº “eary 3es º, ‘ponwy, woO-QUIOSHIO V HO SULNGINE)SIS (HHI, HO SVGISIV (HHI, HO GIT8DVJ, Yºr MENSURATION OF SURFACES. 355 TO FIND THE AREA OF A SEGMENT OF A CIRCLE WHEN THE OHORD AND THE HEIGHT OR, VERSED SINE ARE GIVEN. PROBLEMI. 732. What is the area of a segment, the chord of which is 9 ft. 3 in. and the height or versed sine is 4 ft. ? Ans. 4036.9828 + sq. in., area of the segment A. D. B. , OPERATION. C ºt seºwº A -11 | B A é. #. B º : D D First Step to find the diameter of the circle of which the given segment is a part. A B = 111 in. = whole chord. (55%)* -- 48 = 64.17 -- in. difference between A P = 554 in. = # chord. (liameter and versed sine. P D = 48 in. 64.17 – 48 = 112.17 in. diameter of circle. 112.17 - 2 = 56.085 in. radius A. C. Second Step to find the quotient of the versed sine divided by the diameter. 48 in. versed sine -- the diameter, 112.17 = .4279, quotient. The tabular number for .427 = .31996 {{ {{ 46 44 .428 = .32095 Difference is, .00099 Then, .00099 × .9, the difference between .4279 and .427, - .000891. Hence .31996 -- .000891 = .320851, the sum by which the square of the diameter is to be multiplied. .320851 × 112.17% = 4036.9828 + sq. in., area of the segment A. D. B. A SHORT APPROXIMATE METHOD TO FIND THE AREA OF A SEG- MENT OF A CIRCLE, WITHOUT THE VERSED SINE TABLE. PROBLEMI. 733. What is the area of a segment of a circle, the chord of the segment being 24 feet and the height or versed sine of the segment 6 feet? Ans. 1004 sq. feet. . 24 feet. OPERATION. At B Explanation.—In all problems of this kind, in which the seg- 4 x 24 × 6 = 576 ment is less than a semi-circle, 3 × 63 we proceed as follows: To 4 add, -ār- = 27 times the product of the chord by the versed sine, add the quo- tient of 3 times the cube of the versed sine divided by the chord, and then divide the sum by 6. area, 100.5 sq. ft. 6 feet. Or, if preferred, we may perform the operation as follows: To 4 divide by 6) 603 *=mºs, 356 SouLE's PHILOSOPHIC PRACTICAL MATHEMATICS. ºf •e- times the square of the chord, add 3 times the square of the versed sine; then multiply the sum by the versed sine and divide the product by 6 times the chord. Thus: 24” x 4 = 2304 6° x 3 = 108 241.2 × 6 = 14472. sºmº- 14472 -- (6 x 24) = 100% 2412 - ADVANTAGE OF THIS METHOD, The advantage of this method is that it can be used without the table of versed sines. The result produced is slightly different from that produced by the use of the versed sine tabular method as shown above, but for most practical purposes, it may be used when the versed sine table is not at hand. OPERATION OF THE ABOVE PROBLEM BY THE USE OF THE WERSED SINE TABLE. AT 24 feet. —B 24 ft. length of chord -- 2 = 12 ft., 4 of chord. 12* + 6 ft. versed sine = 24 ft., difference be- tween the diameter and versed sine. 24 + 6 = 30 ft., diameter, 6 -- 30 = .2 quotient. Tabular number for .2 is .111823. D ..111823 x 30°= 100.6407 sq. ft., area of segment. 6 feet. TO FIND THE CHORD OF A SEGMENT OF A CIRCLE WHEN THE DLAMETER AND VERSED SINE ARE GIVEN. PROBLEM. 734. What is the chord of a segment, of a circle, the diameter of which is 72 inches and the versed sine 12 inchest Ans. 53.664 in., length of chord. OPERATION. First Step.–Diameter 72 in. — versed sine 12 in. = 60 × versed sine, 12 in. = 720. • Second Step. V720 = 26.832 in. = # of chord. 4 2 4°) 320 = 53.664 in. chord of segment. C 72" 276 528) 4400 4224 5363) 17600 A º D SGL’ 16089 D 5366°) 151100 107324 Explanation.—In all operations of this kind, first subtract the versed sine from the diameter and multiply the remainder by the versed sine; then extract the square root of the product and multiply the quotient by 2; the result will be the length of the chord of the segment. X- MENSURATION OF SURFACES, 357 TO FIND THE RADIUS OF THE INSCRIBED CIRCLE OF A REGULAR POLYGON. PROBLEMIS. 735. 1. What is the radius of an inscribed circle on a hexagon, whose sides are each 50 inclies? Ans. 43.30127 radius of circle. OPERATION. Explanation.—In all problems of this kind, multiply one side of the polygon by the tabular * f – radius number set opposite the named polygon 50 × .8660254 = 43.3012700 in the radius column in the table of areas, on page 350. 2. The respective sides of a nonagon are 3 feet. What is the diameter of the 1nscribed circle 3 Ans. 8.2424 + feet. Diameter of circle, OPERATION. 3 × 1.3737387 = 4.1212161 = radius. 4.1212161 × 2 = 8.2424.322 diameter. PRACTICA, PROBLEMS IN MENSURATION OF SURFACES. PROBLEMS. 736. 1. How many square rods in a plat of ground, the length of which is 200 rods and the width 60% Ans. 12000 sq. rods. 2. How many square feet in a floor 16 feet 8 inches long, and 10 feet 3 inclies wide # Ans. 1703 sq. feet. OPERATION. 3 || 50 12 || 200 4 || 41 Or, 12 123 170% sq. feet., Ans. 1703 sq. feet., Ans. 3. How many square yards in a floor 60 feet long and 40 feet 10 inches wide # Ans. 2723 sq. yards. OPERATION. 60 × 40} = 2450 sq. ft.; 2450 -- 9 = 2723 sq. yds. 4. A water pipe is 50 feet 9 inches long, and its diameter is 30 inches. What is its concave surface? Ans. 57397.032 sq. inches. OPERATION. 30 x 3.1416 = 94.248 circumference, or linear width of pipe, 94.248 x 609 in., length = 57397.032 sq. in., Ans. * 5. How many square yards of plastering in one wall of a house 32 feet 4 inches long and 15 feet 3 inches high 3 -- Ans. 54;s Sq. yds. 358 soul E's PHILOSOPHIC PRACTICAL MATHEMATICS. º: 6. How many square feet in the bottom of a cistern whose diameter is 11 feet 8 inches? Ans. 106.90.163 sq. ft. 7. What is the area of the base of a cylinder whose circumference is 62.832 inchest * Ans. 314.16 sq. in. 8. A lumberman has 25 boards, each 14 feet long and 15 inches wide. How many square feet in all? Ans. 437.4 sq. ft. 9. Find the area in square feet of a plank 22 feet 3 inches long, 24 feet wide at one end and 14 inches wide at the other ? Ans. 38.3 sq. ft. 10. Find the area of a piece of ground which is 406 feet long and is of the following width at different points, equally distant from each other: at the wider end, 210 feet; at the narrower end, 165 feet; near the wider end, 180 feet; near the narrower end, 142 feet; in the middle, 300 feet? Ans. 821644 sq. ft. NOTE.—Draw a diagram of the ground before working the problem. OPERATION INDICATED TO FIND THE AVERAGE WIDTH. 210 + 165 = 1874, + 180 + 142 + 300 = 2023 ft. average width. *mºm, 2 4 11. A steamboat boiler is 45 feet long and 50 inches in diameter; it has 3 fluos, each 10 inches, in its exterior diameter; how many square inches does the boiler contain on its entire interior surface # Ans. 88278.96 sq. in. OPERATION TO FIND THE CONCAVE SURFACE OF THE BOILE R. 50 × 3.1416 = 157.08 in., circumference. 540 in., length. 84823.20 sq. in., in concave surface. 3455.76 area of ends. 88278.96 sq. in., the entire interior surface of the boiler. OPERATION OPERATION TO FIND THE SURFACE OF THE 2 ENDS. TO FIND THE SURFACE OF THE 6 ENDS OF THE 3 FLUES. 50? – 2500 2500 × .7854 – 1963.50 103 = 100 2 100 × .7854 = 78.54 ºmsºmºsºmºsºms 6 Area of 2 ends, - - - - 3927.00 ºsmºmº Area of the 6 ends of 3 flues, 471.24 || Area of the 6 ends, - - - 471.24 Area of 2 ends, - - - - 3455.76 & 12. If a man occupies a space 20 inches by 30 inches, how many men may stand in a semicircular space which is 200 feet in diameter ? sº Ans. 3769 men with 552 sq. in., unoccupied. * * MENSURATION OF SURFACES. 359 13. A piece of board is 16 inches long, 9 inches wide. How can it be cut into two pieces so that by placing them together in another form they will make a square 12 inches on each side 3 y— 4. Answer. 1. Commence * | as shown in the first fig- ure 4 inches from either -4- 4. end, and saw down 3 in. *: 2. Then at a right an- 4. 4 gle from this point, saw 3 3 toward the longer end 4. 4 inches. 3. Then at a right angle from this point, saw down 3 in- ches. 4. At a right angle from this point saw 4 inches in the same direction as the first 4 inches were sawed. 5. Lastly from this point and at right angles with the last line, Saw 3 inches, through the board, and arrange the pieces as shown in the second figure. NOTE.-In order to saw the board as above described, it would be necessary to bore holes at the four angles, where the changes of direction were made. These holes would be plugged when the pieces were placed in the square form. 14. A circular piece of ground contains 10 acres; what is º its diameter in rods, and feet? Ans. 45.135 + diameter in rods. 744.7275 + feet. \ OPERATION INDICATED. 10 × 160 = 1600 = the sq. rods in 10 acres; 1600 -i-.7854 = 2037.1785 = the sq. rods in a square one side of which is equal to the diameter of the 10 acre circle; VT2037.1785 = 45.135 + rods = one side of the square and the diameter of the circle; 45.135 × 10% = 744,7275 + feet. 15. A gardener wishes to lay out a circular garden that ſ will contain one acre of ground; how many feet long must be a © rope that, fastened by one end to a stake in the center of the \ garden, will enable him to describe the one acre ? \ Ans. 117.75 + feet. OPERATION INIDICATED. 1 acre contains 43560 sq. feet; 43560 -- .7854 = 55462.1848-|- ; v55462,1848-1- =235.50 ft., diameter; 235.50+ -- 2 = 117.75 + ft. = radius or length of rope. 16. Cheops, the largest of the Egyptian pyramids, is square at its base and measures 764 feet on each side; how many acres of ground does it cover? Ans. 13 acres 63 sq. rās. 294 sq. yds. 1 sq. ft. 36o SOULE's PHILOSOPHIC PRACTICAL MATHEMATICS. jºr OPERATION INDICATED. 764 x 764 = 583696 sq. ft.; 583696 sq. ft. -- 9 = 64855 sq. yds. and 1 sq. ft.; 64855 sq. yds. -- 30+ = 2143 sq. rods and 294 sq. yds.; 2143 sq. rods -- 160 = 13.A. and 63 sq. rods. 17. A modern stock raiser has a cow that he wishes to feed upon 2500 sq. ft. of pasture per day. If he ties the cow by the tail with a rope, and if the head and body of the cow be 74 ft. long and the tail to the point where it is tied be 34 feet long, what must be the length of the rope to give the cow the desired feet of past- urage, no allowance being made for the curve in the line of measure, on account of the height of the cow % Ans. 17.2094 + feet. OPERATION INDICATED. 2500 + .7854 = 3183,0914-- ; V 3183,0914 = 56.4188 -- 2 = 28.2094 – (74 + 34) 11 = 17.2094. ARTIFICERS OR MECHANICS' WORK, IN SURFACES, -º- PAVING YARDS AND WALKS. 737. 1. How many bricks will be required to pave a sidewalk 64 feet long and 11 feet 8 inches wide, each brick being 8 inches long and 4 inches wide? Ans. 3360 bricks. OPERATION INDICATED. Length of brick = 8 || 768 = length of sidewalk in inches. Width “ “ = 4 || 140 = width of Sidewalk in inches. | 3360 = number of bricks to pave the sidewalk. Ea:planation.—In all problems of this kind, we first make the statement to ascertain the number of square inches in the sidewalk, by multiplying the length by the width in the unit of inches; and then we divide this result by the product of the length and the width of a brick. 2. In the above problem, how many square yards in the sidewalk, and what would be the cost of the paving at 90% per square yard 3 Ans. 82% square yards $74.66 -- cost. OPERATION TO FIND SQUARE YARDS. 64 3 || 35 9 82## square yards at 90.2 = $74.663. Y ARTIFICERs' OR MECHANICs’ work, IN SURFACES. 361 3. How many German flags, each 16 in. by 16 in., will it take to pave a yard 45 feet square ? Ans. 11391's flags. 4. A yard is 24 feet 3 inches long by 11 feet 5 inches wide; how many bricks, 4 by 8 inches, will it take to pave it, no allowance to be made for the openings between the bricks? Ans. 1245}} bricks. 5. A circular court is 75 feet in diameter; in the center there is a fountain 15 feet in diameter; how many tiles, each 6 inches square, will it require to tile the court 7 Ans. 16964.64 tiles. OPERATION. 75% = 5625 sq. of the diameter of court. Ea:planation.—We first find the dif- 152 = 2.25 4. ‘‘ ‘‘ 4% “ fountain. ference between the squares of the diameters of the court and fountain; we then multiply this difference by 5400 difference. .7854, and produce the number of 6 | .7854 square feet in the area of the ring; we 6 | 1.44 then reduce this to inches and divide the same by the number of inches in a tºº, wº. ººm-º. tile. 16964.64, Ans. NotE.—In actual practice, by reason of the waste in making the circle, many more tiles would be required than are produced by this operation. 6. A circular court is 30 feet in diameter. How many tiles, each 6 inches square, will it require to cover the court, making no allowance for waste? Ans. 2827.44 tiles. 7. What will it cost to pave a yard that is 140 ft. long, 34 feet 4 inches wide, with shillinger pavement, at $3.25 per square yard 2 Ans. $1735.74%r. 8. A contract is made with a company to pave a street with rosetta pave- ment and to curb the same with 4 x 18 inch granite. The price is fixed at $14 per square for the pavement, and $1.10 per running foot for the curbing. What will be the cost for paving one square or block which is 320 feet long, the street being 42 feet 6 inches wide 3 Ans. $1904 for paving. $704 for curbing. 9. How many sods, 12 by 16 inches will it require to sod a yard 40 ft. long and 28 ft. 4 inches wide 3 Ans. 850 sods. FRAMING, BUILDING, LAYING FLOORS, WAINSCOTING, ETC. 738. The work of carpenters and joiners is done by the lineal foot, the Square foot, the square yard, or by the square of 100 square feet. Framing the large timbers used in building, such as sills, posts, principal rafters, etc., the framing of hips and valleys, and the work of chamfering posts and girders, raising plates and bond timbers, bridging joists, making mouldings, archi- traves, etc., are counted by the lineal foot. * The charge for dressing posts and girders, making window shutters, doors, wainscoting, shelving, ceilings, lattice-work, cornice, etc., is by the square foot; sometimes some of this work is charged by the lineal foot. 362 SOULE's PHILOSOPHIC PRACTICAL MATHEMATICs. * ; The charge for framing joists, small rafters, studding, putting oil weather- boarding, shingling, sheathing, laying floors, making partitions, board fences, etc., is by the square yard or the square. NOTE.-In measuring weather-boarding, all openings over 4 feet wide are to be deducted. PROBLEMS. 1. A carpenter in repairing a house, framed 3 sills 8x6 inches, each 42 ft. long, and 6 sills 8x8 inches, each 18 feet 6 inches long; what was the number of lineal feet, and what was the cost at 7.2 per lineal foot ? Ans. $16.59. OPERATION. 42 × 3 = 126 184 × 6 = 111 $23, feet at 7 g = $16.59. 2. A room is 22 feet 3 inches long, and 18 feet 6 inches wide; it has two doors each 4 feet wide. What will be the cost to wainscot this room 3 feet 4 inches high at 60% per square yard? Ans. $16.33%. OPERATION. 224 × 2 = 44; 2 | 1.47 = distamee around room. 184 × 2 = 37 3 || 10 = height of wainscoting. ºme smºs 9 = feet in Sq. ya. 81; sº 4 × 2 = 8 27# sq. yds. at 602. = $16.33%. 73% 3. What will be the cost to floor a room 34 feet 6 inches long and 22 ft. 4 inches wide, at $4 per square 3 Ans. $30.82. 4. What will it cost to make twelve 8 panel doors 84 ft. x 34 ft. 13 inches thick at 25% per square foot of 1 inch thick? Ans. $127.50. 5. What will it cost to make 24 shutters each 22 inches wide 14 inches thick and 74 feet long at 20g per square foot of 1 incli thick? Ans. $82.50. 6. How many squares in the floors of a house containing 8 rooms measuring as follows: 2 rooms are 16 by 20 feet, 2 are 154 by 20 feet, 2 are 14 by 18 feet, and 2 are 12 feet 4 inches by 18 feet? Ans. 22#. 7. A sub-contractor puts up the frame work of a building at 90% per square. The building is 85 feet long, 60 feet wide, and from the bottom of the sill to the top of the rafter plate is 31 feet 8 inches. From the end of the rafters on one side, measured over the peak or apex of the roof, to the end of the rafters on the oppo- site side is 69 feet 2 inches. The studding in the partitions on the first story meas- ure 270 feet long and 15 feet 6 inches high; and on the second story it is 310 feet 4 inches long and 14 feet 3 inches high. The sleepers in all the rooms on the lower floor measure 84 feet 6 inches long by 59% feet wide; and the joists in all the rooms on the second floor measure the same as the sleepers on the first floor. The two gables, or the vertical triangular ends of the building from the eaves or cornice to the top are 60 feet at the base, and as the rafters are + pitch, they have an altitude of 15 feet. What is the cost of erecting the frame work 3 Ans. $311.37-H. º: ARTIFICERS' OR MECHANICs' work, 1N surf Aces. 363 OPERATION INDICATED. 85 + 85 + 60 + 60 = 290 ft. the measurement around the building. 290 × 31# = 91834 sq. ft. in the 4 sides or walls. 85 × 69% = 5879% sq. ft. in the roof. 270 × 15% = 4.185 sq. ft. in the partitions on the first floor. 310# × 14+ = 44224 sq. ft. in the partitions on the second floor. 844 × 59% = 5013; sq. ft. in the sleepers on the first floor. 84 × 59% = 5013; sq. ft. in the joists on the second floor. 60 × 15 × 2 = 900 sq. ft. in the two gables. 2 34597-1's sq. ft. in the entire frame work. 34597-14 sq. ft. -- 100 = 345.97+); squares, at 90.2 = $311.373. 8. What is the cost to slate a roof 127 ft. 4 inches long and 56 feet wide, at $11.50 per square, and what will be the weight of slate on the roof, allowing a square of slate to weigh 600 pounds 3 Ans. $820.023. 42784 lbs. or, 21 tons and 784 lbs. PLUMBERS’ WORK. 739. Plumbers’ work is done by the pound, hundred-weight, or by special agreement. The following Table shows the weight of a square foot of sheet lead, accord- ing to its thickness; and also the common weight of a yard of leaden pipe, accord- ing to the diameter of the bore : Thickness of Lead. |Pounds to a square foot. || Bore of Lead Pipes. Pounds per yard. Inch. Inch. +% 5.899 0; 10 # 6.554 1 12 # 7.373 14 16 # 8.427 14 18 # 9.831 1; 21 # 11.797 2 24 1. What is the cost of 261 feet of leaden pipe, of 14 inch bore, at 11 cents per pound, allowing as per the table that each yard weighs 18 pounds? Ans. $172,26. OPERATION INDICATED. 261 -- 3 = 87 yards. w 3 || 261 87 × 18 = 1566 lbs. Or, 18 1566 x 11 g = $172.26 11 | $172,26, Ans. 2. What is the weight of lead, ; of an inch thick, that covers a surface 32 feet long and 8 feet 6 inches wide, estimating the Weight at 73 pounds per square foot ? º Ans. 2006 pounds. 364 SOULE's PHILOSOPHIC PRACTICAL MATHEMATICs. * SLATING AND SEIINGLING ROOFS. 740. 1. The roof of a building is 72 feet 6 inches long and measures 46 feet 9 inches from eave to eave. How many slates and how many squares of slating are there in the roof, allowing a slate to cover a space 4; by 8 inches and not allow- ing for the double course at the eaves? Ans. 13557; slates. 33}#} sq. of slating. OPERATION TO FIND THE NUMBER OF SLATES. Explanation.—In all problems of this Width of slate = 8 || 870 in. = length of roof. kind. We first find the nºber of sºlºe 561 in. = width “ “ inches in the roof by multiplying togeth- OOIL Ill. = W1 er the length and the width of the roof Length of slate 9 || 2 in the unit of inches; and then divide exposed on the – the same by the number of square inches roof. 13557; slates, Ans. that each slate covers. OPERATION TO FIND THE NUMBER OF SQUARES OF SLATING. Explanation.—Here we first make the 12 || 870 statement to find the number of square 1. 561 feet and then divide by 100, which is the number of square feet in a square, as per 33}#} Squares, Ans. Article 531, page 244. 2. How many shingles will it require to shingle a house that is 54 feet long and 35 feet 10 inches from eave to eave, estimating that 5 inches of each shingle will be laid to the weather, and allowing for the double course at the eaves on each side 3 Ans. 14256 shingles. NotE 1.-The unit width of a shingle is 4 inches. NOTE 2.—Add 10 inches to the width for the double courses. 3. A flat roof is 208 feet 2 inches long and 28 feet 5 inches wide. How many square yards of tin will be required to cover it, and what will be the cost at $1.15 per square yard 3 Ans. 657++3 sq. yds. $755.85-H cost. 4. A building is 64 feet long and 40 feet wide; the rafters have one-quarter pitch, allowing a projection of 6 inches on each end of the roof, and 1 foot on each side for the eaves, how many slates and how many squares of slating will it require to slate the building, if the slates are 6 inches wide and 5 inches of their length are exposed to the weather, no allowance to be made for the ridge of the roof or for the double course at the eaves? Ans. 14577; slates. 30% ## Squares. OPERATION TO FIND THE LENGTH OF THE RAFTERS. 100 400 102 10 202 20 500 ARTIFICERS' AND MECHANICs' work, IN SURFACES. 365 V 500 = 22.36 + feet = length of rafter. Practically, the length of rafter is ſº tºº 22 ft. 4; in. Projection of rafter is es * * tº s sº 1 ft. 0 in. Length of rafter, º as tº ſº is º 23 ft. 4; in. 2 Length of two opposite rafters, or width of roof, 46 ft. 83 in. OPERATION OPERATION TO FIND THE NUMBER OF SLATES. TO FIND THE NUMBER OF SQUARES. 64 ft. -- 6 in. -- 6 in. = 65 ft. = 780 in. 46 ft. 83 in. = 5603 inches. 65 * | 3 | 1682 780 12 3 | 1682 & 100 6 * * = * = 5 30+}# squares. 14577; slates, Ans. NotE.—By one-quarter pitch is meant, in mechanical language, that the apex of the roof is to be + the width of the building higher than the plane of the base of the rafters. PLASTERERS’ WORK. 741. The work of plasterers is of two kinds, viz.: ceiling, which is plaster- ing on laths, and rendering, which is plastering on Walls. In measuring plasterers’ work, the square foot, square yard, or square is used. The windows, doors, fire places, and base board are generally deducted. In some states, one-half of all openings over two feet wide are deducted. Plasterers have also adopted the following regulations regarding the measur- ing of their work : 1. To measure all circular walls and ceilings, twice. 2. To charge extra for corners of chimneys and other external angles when made with “gauged ” mortar. 3. To add one foot in length of the cornice for each mitre or “return.” 4. Stucco work, when more than 12 inches wide, is done by the square foot. - PROBLEMIS. 1. How many square yards of plastering in the walls and ceiling of a room 344 feet long, 21 feet 2 inches wide, and 14 feet 10 inches high, deducting 6 windows, 7 by 4 feet, and 2 doors, 9 by 34 feet, and the base board 8 inches wide, and what would it cost at 242 per sq. yard 1 Ans. 231; sq. yards. • $55.50 practically. 366 sou LE's PHILOSOPHIC PRACTICAL MATHEMATICS. A. OPERATION INDICATED. 34; + 34} + 21} + 21% = 111 ft. distance around the room. 14 ft. 10 in. height of room, - 8 in. width of base = 14 ft. 2 in. height of room plastered. 1113 x 14% = 15773 sq. ft. in the walls of room. 34; x 21% = 730+ sq. ft. in the ceiling. 23074% sq. ft. in the walls and ceiling. DEDUCTIONS. 9 ft. – 8 in. = 8 ft. length of doors, allowing for width of base board not measured in the area of the Walls. 84 x 34 × 2 = 584 sq. ft. in the 2 doors. 7 × 4 × 6 = 168 sq. ft. in the 6 windows. 2263 sq. ft. deductions. 23073% – 2264 = 2081; sq. ft. 20813% + 9 = 231#, sq. yds. at 24 g = $55.50, practically. 2. A room 20 feet 6 inches long, 17 feet 4 inches wide, has two chimneys extending 14 inches from the wall; what would be the cost of a cornice 16 inches wide around this room, allowing for-12 miters, 4 at the corners of the room and 8 at the two chimneys, at 18% per square foot ? Ans. $22.16. OPERATION. 20% + 20% + 17* + 17% = 75% ft. around the room, not counting chimneys. 14 × 4 = 56 inches = 43 ft. width of the chimney walls. 1 ft. allowed for each miter 12 ft. * 92% = length of cornice. 924 × 1 = 1233 sq. feet, at 18% = $22.16. 3. What will it cost to plaster the walls and ceiling of a house of 10 rooms, 4 of which are 20x164 feet and 144 feet high; 3 are 18×16 feet and 13 feet high and 3 are 164 × 12# feet and 13 feet high; allowance being made for 20 windows 74x34 feet, and 18 doors 94×34 feet, at 232 per sq. yard Ans. $275.42. NOTE,--The measurements in this problem are exclusive of the base board. 4. What will it cost to plaster the walls and ceiling of 2 rooms at 374 cents per square yard, each room 24 feet 3 inches long, 12 feet’9 inches wide, and 11 feet 6 inches high, deducting 5 windows, 6 by 3 feet 4 inches, and 2 doors, 8 feet 8 inches by 3 feet 6 inches, in each room, and the base board 8 inches wide 3 Ans. $79,57+. PAINTING AND GLAZING. 742. Painting is done either by the square yard, or the square, or by esti- mate. Glazing is done by the light or pane. Painters and glaziers have adopted the following rules for measuring their Work: 1. In weather-boarding, measure not only the width of the board, but also Y ARTIFICERS AND MECHANICs' work, IN SURFACEs. 367 the width of the exposed edge. For partition boards, when beaded, add to the width of the boards ; inch for each bead. *, 2. For paling or picket fences, measure the height, and add thereto the width of each rail, which gives the width or height. 3. To fixid the length, add to the length of the fence twice the width of each post, if rectangular, or one-half the circumference, if cylindrical. 4. In measuring the length and width of panel doors, shutters, etc., press the measuring line into all the quirks and mouldings. 5. In measuring weather-boarding and wall painting, deduct half the open- lingS. ºf 6. In measuring shingle or board roofs, measure the butts or edges; for trellis or lattice work, double the measure, and for venetian shutters add one-half. 7. In measuring the width of pilasters, commence at the wall and measure round to the bead at the end of the jamb casing; pressing the line into all the quirks. 8. For plain rosettes in pilasters, add 1 foot to the length; for carved rosettes, add 2 feet to the length. 9. In measuring window jambs, cornice, etc., press the line or tape into all indentations and concave surfaces. 10. When the measurement is made by the square foot, it is customary to include the whole sash, and circular or oval windows as square. PROBLEMIS. 1. What will the glazing cost for a house containing 46 windows of 8 lights each, and 7 windows of 18 lights, at 25g per light 3 Ans. $123.50 2. What will be the cost of the painting of 43 ft. high wainscoting of 4 inch wide beaded boards, of a room 18 feet 6 inches by 13 feet 4 inches, at 242 per sq. yard 7 Ans. $8,594. OPERATION INDICATED 18} + 184 + 13% + 134 = 633 ft. measurement around the room. 633 = 764 in. -- 4 in., the width of a beaded board, = 191 beads, or half inches, to add to the measurement around the room; 191 half inches = 95% inches = 7# ft. 33 + 7# = 713 ft. length of wainscoting. 1; x 4 = 322# sq. ft. = 35+ sq. yds. at 242 = $8.59%. 3. What will it cost to paint both sides of a paling or picket fence 120 feet long, 5 ft. high, the posts being 8 feet apart from center to center and 4× 6 in. With the narrow side set to the fence; the rails being 2 × 4 inches with the wide Sidº facing the fence, at 25% per sq. yard? Ans. $42.81% #. OPERATION INDICATED. 5 ft. high -- 4 in. + 4 in. = 53 ft. = height of fence including the width of the two rails. 16 posts 6 in. wide, 2 sides each = 1 ft. for each post, to be added to the length of the fence. 120 + 16 = 136 ft. length of fence, one side. 136 × 2 = 272 ft. length of fence, both sides. 272 × 53 = 15414 sq. ft. 1541; -- 9 = 1714 sq. yds. 1713 ºr x 25% = $42,81% #. 368 SOULE's PHILOSOPHIC PRACTICAL MATHEMATICs. ºr 4. How many panes of glass, 12 by 10 inches, in a box of 100 feet? Ans. 120 panes. OPERATION INIDICATED. - Pane. 1 pane 10x12 = 120 inch. 1 sq. ft. = 144 sq in. 1. 12 || 144 144×100 = 14400 sq. in. in the box. 14400 or thus: 120 144 or, 10 || 100 -- 120 = 120 panes. 100 5. How many panes of glass in 44 boxes, each containing 50 sq. ft., the panes being 22 × 24 inches, and what will be the cost at $7 per 100 sq. feet 3 Ans. 600 panes. $154.00 cost. 6. What will it cost to glaze 13 windows, 3 feet by 7 feet 4 in., and 22 win- dows, 3 feet by 63 feet, at 25 cents per square foot ? Ans. $178.75. CARDETING FLOORS. PROBLEMS. 743. 1. How many yards of carpeting that is 27 inches wide, will be required to cover the floor of a parlor that is 32 feet 4 inches long and 25 feet 6 inches wide, making no allowance for waste in matching or turning under ? Ans. 122# yds. FIRST OPERATION. 36 || 388 Explanation.—In this solution, we first find the number 27 || 306 of square inches in the floor, by multiplying together the length and width in the unit of inches. We then divide by the product of the length and width of a yard of the carpet, 122# yds., Ans. which is the number of square inches in one yard of it. SECOND OPERATION INDICATED. 12 || 388 3 || 97 Explanation.—In this solution, 12 || 306 2 || 51 we first find the number of square * gards in the floor, which would be 9 Or, 9 the number of yards required if 27 | 36 27 36 the carpeting was 1 yard, or 36 - - inches wide. But since the carpet- –4– -4- ing is not 1 yard wide, we multiply 122# yds., Ans. 122# yds., Ans. by 36, which gives the number of * yards required if the carpeting was but 1 inch wide, and we then divide by 27, the width of the carpeting, and produce the correct result. We reason as follows: Since it requires this expressed number of yards when it is 36 inches wide, if it were but 1 inch wide, it would require 36 times as imany yards, and if 27 inches wide, the 27th part. NOTE.-In this connection, we will remark that although the result of this problem is math- ematically correct, it is not correct in a practical sense; i. e., in practice it would require more yards of carpeting than the mathematical measurement of the room gives. This is occasioned by the matching of the flowers or figures of the carpet, and by the unavoidable waste by reason of the room being a little wider or longer than an even number of widths of the carpeting, which necessitates the use of another width of carpet. The increased number of yards over the mathematical result, by the foregoing causes, will depend upon the size of the figure in the carpet, and whether the carpet is laid lengthwise or crosswise of the room; and the flower or figure of the carpet, and the entrance to the room, whether from the side or end, often determines which way the carpet shall be laid. . These are important facts for parties who buy carpets on their own measurement of rooms, and if observed, will often save expense and much vexation. The two following problems elucidate these points and make clear the practical manner of working problems of this kind : Yºr ARTIFICERS AND MECHANICs' work, IN SURFACEs. 369 2. How many yards of carpet, 27 inches wide, will be required to cover a floor 21 feet 10 inches long and 18 feet 6 inches wide, laid lengthwise, and allowing 6 inches on each strip for matching? Ans, 67 yards. OPERATION INDICATED. 18 ft. 6 in. = 222 inches. 222 inches -- 27 = 8*, practically, 9 widths of Carpet, #} of one width will be waste, either cut off or folded under. 21 ft. 10 in. + 6 in., allowed for matching, - 22 ft. 4 in. = length of each strip. 22 ft. 4 in. x 9 = 201 ft. -- 3 = 67 yds. 3. If in the above problem, the carpet had been laid crosswise with the same allowance, how many yards would have been required ? Ans. 63.4 yards. OPERATION INDICATED. 21 ft. 10 in. = 262 inches. 262 in. -- 27 = 94%, practically, 10 widths of carpet. • 18 ft. 6 in. + 6 in. allowed for matching, - 19 ft. = length of each strip. 19 ft. x 10 = 190 + 3 = 634 yds. 4. How many yards of carpet 36 inches wide, and what will be the cost at $1.35 per yard laid, to cover a floor 44 ft. 7 inches long, 31 ft. 6 in. wide, laid cross- wise of the room, allowing 4 inches for matching Ans. 1594 yds. $214.87; cost. 5. A parlor is 29 feet long by 22 feet wide. It is carpeted with 36 inch carpet at $1.75 a yard, surrounded by a carpet border 18 inches wide, at $1.25 per linear yard. If the border is laid without being mitered and the carpet is laid crosswise of the room, and an allowance of 4 inches on each strip is made for matching the carpet, what will be the total cost? Ans. $141.50. OPERATION INDICATED. 29 ft. – 3 = 26 ft. -- 3 = 83, practically, 9 widths. 22 ft. – 3 = 19 ft. -- 4 in. (matching) = 194 ft. length of strip. 9 × 19 = 174 ft. -- 3 = 58 yds. carpet. 58 × $1} = $101.50 cost of carpet. - Then, 29 × 2 = 58 ft. length of carpet strips on two sides of room. 22 — 3 = 19 × 2 = 38 ft. length of carpet strips on two ends of room. 58 + 38 = 96 ft. + 3 = 32 yds. length of carpet strips. 32 × $1.25 = $40.00 + $101.50 = $141.50, Ans. NoTE.—When the border is mitered, there will be as many yards of border as there are yards distance in the four sides of the room. 37O SOULE's PHILOSOPHIC PRACTICAL MATHEMATICs. * • MENSURATION OF SOLIDS, 744. The mensuration of solids is divided into two parts, viz.: 1st, the mensuration of their solidities; and, 2d, the mensuration of their surfaces. The Unit of Measure for solids is a cube whose side is some fixed length, such as an inch, foot, yard, rod, etc. The Area of a figure is the measure of its surface. The Solidity or Contents of a solid is the number of cubes that it is equal to, or contains; or its capacity expressed in cubic measure of any fixed unit. In the solution of problems in mensuration of Solids, much time may often be saved by applying the following principles which have been demonstrated in geometry: That all similar solids are to each other as the cubes of their like dimensions. DEFINITIONS. 745. A Rectangular or Quadrilateral Solid is a solid which has length; | width, and thickness, and iſ is bounded by six sides or | | | - Jaces. 747. A Cube is a Solid whose sides or faces are all equal Squares. 749. A Cylinder is a solid having two faces or bases, which are equal parallel circles, and which has an equal diameter in any parallel plane between them. 751. A Frustum of a cone is the part which # remains after the top is cut off by a plane parallel to the base. 746. A Pyramid is a Solid whose base is any kind of a polygon, and its other faces, triangles unit- ed at a common point called the verteac. 748. A Frustum of a Pyramid is the part - which remains after the # top is cut off by a plane parallel to the base. 750. A Cone is a solid having one face or base which is a circle, and a COnvex or curved surface terminating in a point, called the vertea. 752. A Sphere is a solid bounded by a curved surface, all the points of which are equally distant in called the center. The Radius of a sphere is a line drawn from the center to any part of the circumference. The axis, or diameter of a sphere is a line passing through the center and terminated by the cir- cumference. 753. A Prolate Sphe- roid is a solid, elongated revolution of an ellipse about its longer axis. in the direction of a line joining the poles; or, it is : a solid generated by the W 754. An Oblate Sphe- roid is a solid flattened \ or compressed at the A poles; or, it is a Solid gen- 7 erated by the revolution of an ellipse about its shorter axis. X} MENSURATION OF SOLIDS, 37 I TO FIND THE SOLIDITY OF A PRISM, PARALLELOPIPEDON, OR CUBE. PROBLEMIS. 755. 1. A rectangular box is 4 feet long, 3 feet 4 inches wide, and 2 feet high. How many solid, or cubic feet does it contain? Ans. 263 cu. ft. OPERATION. 4 4 º ... #= bical or rectangular solids ſº * * 3 !” Or, 12 gº we multiply together thé | | j length, width, and height, ºff== — — gº tºº-º-º-º: in the same units of meas- | | | ": 26; Cll. ft., 263 Cll, ft., ure, and in the product we Ans. Ans. have the required solidity. 2. What are the solid contents of a prism, the sides of whose base are 3 and 4 feet, and whose altitude is 6 feet? • Ans. 72 cu. ft. TO FIND THE SURFACE OF A PRISM, PARALLELOPIPEDON, OR OUBE. PROBLEMIS. 756. 1. How many square feet of surface does the box in Problem No. 1 above, contain Ans. 56 sq. ft. OPERATION. £ 4 × 3 × 2 = 263 sq. ft. on two sides. 4 × 2 × 2 = 16 sq. ft. on the other two sides. 3% x 2 × 2 = 18% sq. ft. on the ends. 56 sq. ft. in the entire surface. 2. What is the surface of a rectangular prism, the sides of whose base are 3 and 4 feet, and whose altitude is 6 feet? Ans. 108 sq. feet. TO FIND THE SOLIDITY OF A SQUARE OR RECTANGULAR PYRAMID. PROBLEMI. 757. A square or rectangular pyramid is 8 feet high and each side of the base is 4 feet 6 inches. How many cubic feet does it contain? Ans. 54 cu. ft. OPERATION. 8 8 Fº * º: 2 4 of this kind, we multiply the 2 º r # # height by the area of the base, † O * and then divide by 3, because a ſº º º 3 3 8quare pyramid has been demon- º º f g º b ź. |% * | *- gº- sº-º strated in geemetry to be # of a 4 º % 54 cu. ft., 54 cu. ft., rectangular solid of equal height ſº Ans Ans and area of base. NOTE.--To find the solidity or volume of a regular pyramid that is not square or rectangular, multiply the square of one side by the tabular area set opposite the polygon of the same number of sides as shown in the Table on page 350, which will give the area of the base. Then proceed as in the above problem, by multiplying by the height and dividing by 3. 372 SOULE's PHILOSOPHIC PRACTICAL MATHEMATICs. * TO FIND THE SURFACE OF A SQUARE OR RECTANGULAR PYRAMID. PROBLEM. 758. How many square feet of surface in a rectangular pyramid whose slant height is 8 feet and whose base is 44 feet? Ans. 924 sq. ft. OPERATION. 4 × 8 × 4 = 144. 144 - 2 = 72 sq. ft. area of four sides. 4 × 44 = 20+ sq. ft. area of base. 72 + 20+ = 924 sq. ft. area, or surface of pyramid. TO FIND THE SOLIDITY OF HEXAGONAL OR OTHER REGULAR PYRAMIDS NOT SQUARE OR RECTANGULAR. PROBLEMI. 759. What are the solid contents of a hexagonal pyramid whose altitude is 11 feet, and each side of whose base is 2 feet? Ans. 38.105.1176 cu. ft. OPERATION 2* = 4 feet = square of one side. Explanation.—In all problems of this character, we multiply the 4 × 2.5980762 = 10.3923048 sq. ft., area of base. square of one side by the tabular * 11 altitude. area set opposite the polygon of **msºmºmºmº the same number of sides, as shown 3) 114.3153528 in the Table, page 350, and this *=====s**E=== product is multiplied by the alti- 38.105.1176 solid feet, Ans. tude and the result is divided by 3. TO FIND THE ENTIRE SURFACE OF AN ocTAGONAL OR OTHER REGULAR PYRAMID, NOT SQUARE OR RECTANGULAR. PROBLEMIS. 760. 1. What is the entire surface of an octagonal pyramid whose slant height is 11 feet and each side of whose base is 2 feet? Ans. 107.3136 sq. ft. surface. OPERATION. 2 × 8 = 16 ft., perimeter of the base. 16 × 5% (one half of slant height) = 88 sq. ft., convex surface. 2* = 4 ft., square of one side. 4 × 4,8284 = 19.3136 sq. ft., area of base. 88 + 19.3136 = 107.3136 sq. ft., entire surface. Explanation.—In all problems of this character, we multiply the perimeter of the base by half the slant height; the result is the convex surface. Then to find the area of the base, multiply the square of the side by 4.8284, which is the tabular area set opposite the polygon of the same number of sides, as shown in the Table on page 350. 2. What is the surface of a regular octagonal pyramid, whose slant height is 12 feet, and each side of the base 5 feet 3 Ans. 360.7106775 sq. ft. * - MENSURATION OF SOLIDS. 373 TO FIND THE SOLIDITY OF THE FRUSTUM OF A SQUARE PYRAMID. PROBLEM. 761. A frustum of a square or four-sided pyramid is 8 feet high, lower base 7 feet, and upper base 6 feet. How many solid feet does it contain Ans. 338; cu. ft. OPERATION. 7* = 49 = area of the greater base. 6° = 36 = “ “ “ lesser {{ * Azº. 7 × 6 = 42 = geometrical mean proportional between the two bases. º 3) 127 º *mºmºmºs * 42} = average area of the frustum of the pyramid. 8 = height. 3383 cu. feet, Ans. Explanation.—In all problems of this kind, to find the solidity of the frustum of a pyramid, we add together the areas of the two bases, and a geometrical mean proportional between them, then divide the sum by 3 and multiply the result by the altitude. NOTE.—To find the solidity or volume of the frustum of a pyramid that is not square or rectangular, we multiply the square of one side of each base, by the tabular area set opposite the polygon of the same number of sides as shown in the Table on page 350, which will give the area of the two bases. Then, we multiply the two bases together and extract the square root of the product; then add the quotient and the areas of the two bases together and divide the sum by 3; the result is the average area of the frustum of a pyramid which multiplied by the height, gives the solidity of the frustum. i TO FIND THE SURFACE OF THE FRUSTUM OF A SQUARE OR REC– TANGULAR PYRAMID. PROBLEMI. 762. How many square feet in the entire surface of a rectangular pyramid whose slant height is 8 ft., lower base 7 ft., and upper base 6 feet? Ans. 293 sq. ft. surface. OPERATION. 7 x 4 = 28 ft., perimeter of lower base, 6 × 4 = 24 ft., perimeter of upper base. 28 + 24 = 52 ft., sum of perimeters of bases. 52 x 8 = 416 - 2 = 208 ft., convex surface. º 7* = 49 sq. ft. area of lower base. 6* = 36 sq. ft. area of upper base. 208 + 49 + 36 = 293 sq. ft. entire surface. Explanation.—In all problems of this kind, we first find the convex surface of the frustum by multiplying the sum of the perimeters of the two bases by the slant height of the frustum and dividing by 2. Then, to find the entire surface of the frustum, add to the convex surface the areas of the two bases. NOTE.—When the frustum of the pyramid is not four sided, the area of the two bases is found by multiplying the square of each side by the tabular area set opposite the polygon of the same number of sides in the Table on page 350. 374 SOULE's PHILOSOPHIC PRACTICAL MATHEMATICs. X- TO FIND THE SOLIDITY OF PRISMS OR SOLIDS OF DIFFERENT SIDES. PROBLEMIS. 763. 1. How many cubic inches in an octagonal prism or solid, one side of whose base is 20 inches and whose length is 60 inches º Ans. 115882.2504 cu. in. - OPERATION. Explanation.—In all problems of this kind, first square one of the sides of the polygon. 2. Multiply the prod- 202 = 400; 400 x 4.8284271 - uct by the tabular number set oppo- 1931.37084 square inches area of base. site the polygon of the same number 1931.37084 × 60 = 115882,2504 cu. in. of sides in the Table, page 350. 3. Mºly the last product by the ength. 2. The side of a pentagonal prism is 2 feet, its length is 30 feet. How many cubic feet does it contain Ż Ans. 206,457288 cu. ft. ( ). PERATION INDICATED. 23 × 1.7204774 × 30. TO FIND TEIE SOLID CONTENTS OF A CYLINDER. PROBLEMIS. 764. 1. A cylinder is 8 feet high and 4 feet 6 inches in diameter. How many cubic feet does it contain 3 Ans. 127.2348 cu. ft. OPERATION. ===s 2 9 12 || 54 Explanation.—In all fºr. 2 || 9 Or. 12 || 54 problems of this kind, | 8 7 8 we multiply the square | | pº of the diameter, by the | #| || — .7854 .7854 height, which gives the |# #. tºmº. solidity of a rectangular §º 127.2348 cu. ft., 127.2348 cu. ft., solid, whose height and Ans. Ans. width of sides are equal to the height and diame- ter of the cylinder. Then we multiply by .7854, the ratio between the area of a square and that of a circle, whose diameter is equal to one side of the square. NOTE.—The solidity of a cylinder is equal to the solidity of a square prism, whose sides and altitude are the same as the diameter and altitude of the cylinder, multiplied by the decimal .7854. 2. What are the solid contents of a cylinder, whose altitude is 6 feet, and diameter 4 feet? Ans. 75.3984 cu ft. TO FIND TEIE SURFACE OF A CYLINDER. PROBLEMIS. 765. 1. In the cylinder in Probem No. 1 above, how many square feet are contained on its entire surface 3 Ans. 144.9063 sq. ft. OPERATION. - 44 × 3.1416 = 14.1372 ft. circumference. Ea:planation.—In all prob- 14.1372 × 8 = 113.0976 sq. ft. area in convex surface. lems of this kind, we multi- 442 × .7854 = 15.90434 sq. ft. in lower base. ply the circumference by the 4}* x .7854 = 15.90434 sq. ft. in upper base. height, and to the product, -ºº ºmis ºmºmºmº add the area of the two 144.9063 sq. ft. in surface of cylinder. ends or bases. Yºr MENSURATION OF SOLIDS. 375 2. What is the surface of a cylinder 6 feet long and 4 feet in diameter ? Ans. 100.5312 sq. feet. TO FIND THE SOLID CONTENTS OF A CONE. º PROBLEMIS. 766. 1. A cone is 8 feet high, and its base is 4 feet 6 inches in diameter. How many solid feet does it contain 3 Ans. 42.4116 cu. ft. OPERATION. 8 8 Explanation.—In all 2 9 12 || 54 problems of this kind, 2 | 9 OI’ 12 || 54 H. make the i. * 7 * 10n as 1n a cylinder an | .7854 .7854 then divide by 3, because W 3 3 the 8olidity of a come is # -- tº-º tº- of #: whose º: }\ | g t t #º º 42.4116 cu. ft., 42.4116 cu. ft, ºr “” Ans. Ans. 2. What is the solidity of a cone, the diameter of whose base is 5 feet, and altitude 12 feet 3 Ans. 78.54 cubic feet. TO FIND THE SURFACE OF A CONE. PROBLEMIS. 767. 1. How many square feet on the entire surface of a cone whose Slant height is 8 feet, and whose base is 4 ft. 6 inches in diameter ? Ans. 71.4531 sq. ft. Surface. OPERATION. 44 × 3.1416 = 14.1372 circumference of the base. 14.1372 x 8 = 113.0976 - 2 = 56.5488 sq. ft. convex surface. 4;” x .7854 = 14.9043 sq. ft. area of base. 56.5488 –H 14.9043 = 71.4531 entire surface. Ea:planation.—In all problems of this kind, we multiply the circumference by the slant height, and divide the product by 2; then, to the quotient thus obtained, add the area of the base, and the result will be the entire surface. - 2. What is the surface of a cone, the diameter of whose base is 5 feet, and slant height 12 feet? Ans. 113.883 sq. ft. Surface. TO FIND THE SOLID CONTENTS OF A FRUSTUM OF A CONE. PROBLEMI. 768. A frustum of a cone is 8 feet high, lower base 7 feet in diameter, and upper base 6 feet in diameter. How many Solid feet does it contain 3 Ans. 265.9888 cu. ft. 376 soul E's PHILOSOPHIC PRACTICAL MATHEMATICs. FIRST OPERATION. square of greater diameter. * = ** “ lesser & 4 3 || 127 = geometrical mean between the two .7854 diameters. 8 | 265.9888 42} = area of the base of a rectangular cu. ft., Solid, the sides of whose bases are Ans equal to the average diameter of the frustum of the cone. Explanation.—In all problems of this kind, we first find, as is, shown in the operation, the area of a rectangular solid, the sides of whose bases are equal to the average diameter of the frustum of the cone; then we multiply this by .7854, to reduce the area of a square to the area of a circle, and then multiply by the height. NOTE.—In the above solution, we found the geometrical mean between the diameters of the two bases by simply multiplying the two diameters together. This is by far the shortest and easiest method of obtaining the geometrical mean between two numbers, and it is correct because, by demonstration, it is proved that the product of any two numbers is equal to the square root of the product of the 8quares of those two numbers. The usual method of finding the geometrical mean between the diameters is as follows: Square each diameter, multiply their products together, extract the square root of the product, the result is the geometrical mean between the diameters of the two bases. OPERATION. Thus: 72 = 49; 6* = 36; 49 × 36 = 1764. V1764 = 42 the geometrical mean, the same as obtained in the solution above, by simply multiplying the two diameters together. NoTE.—The last method would be used when the areas of the bases and not the diameters are given. SECOND OPERATION BY THE PRISMOIDAL FORMUL.A. 7 × 7 = 49 = area of the lower base. 6 × 6 = 36 = area of the upper base. 2) 13 13 169 = 4 times the area of the middle Section *E*==º between the two bases. 6)254 42} = average area of the frustum of a square pyramid of the same dimensions as those of this frustum of a cone. 42; x .7854 = 33.2486 = average area of the frustum of the cone. 33.2486 × 8 = 265.9888 = solid contents or cubic feet. gº x 5 = 424 x 4 NOTE.-The above solution is in accordance with the prismoidal formula, by which the solid- ity of cubes, rectangular solids, cones, cylinders, pyramids, frustums of cones or pyramids and Several other forms of solids may be determined. * The prismoidal formula is as follows: Add together the areas of the two ends or bases and four times the middle section parallel to them. Then divide this sum by 6, and multiply the quo- tient by the height, or depth. * MENSURATION OF SOLIDS. 377 TO FIND THE ENTIRE SURFACE OF THE FRUSTUM OF A CONE. PROBLEM. 769. In the above problem, what is the entire surface of the frustum? Ans. 230.1222 sq. ft. OPERATION 7 x 3.1416 = 21.9912 = circumference of greater base. Ea:planation.—In all prob- 6 × 3.1416 = 18.8496 = circumference of lesser base. lems of this kind, multiply *mºsºsºme the bases by 3.1416 and the 40.8408 × 8 = 326.7264 sum of their products by the 326.7264 - 2 = 163.3632 sq. ft. convex surface. height; then divide this prod- 7° x .7854 = 38.4846 sq. ft. of larger base. uct by 2 which gives sq. ft. 6° x .7854 = 28.2744 sq. ft. of lesser base. in convex surface; to this $ºmmºn e result, add the areas of the 230.1222 sq. ft. in entire surface. two bases. TO FIND THE SOLIDITY OF A SPHERE. PROBLEMIS. 770. 1. A sphere is 4 feet in diameter. How many cubic feet does it contain 3 Ans. 33.5104 cu. ft. º- OPERATION. 4°, or 4 × 4 × 4 = 64 cu. ft. in a cube which is 4 feet on each side. 64 × .5236 = 33.5104 cu. ft., Ans. Explanation.—In all problems of this kind, we cube the diameter by multiplying it by itself 3 times, as shown in the operation, and then multiply this result by .5236, which is the ratio between the solidity of a cube and that of a sphere, whose diameter is equal to one side of the cube. NOTE.--To find the side of a cube which may be cut from a given sphere, square the diam- eter, divide by 3, and extract the square root of the quotient. 2. What are the solid contents of a sphere whose diameter is 12 feet? Ans. 904.7808 cubic feet. TO FIND THE SURFACE OF A SPHERE. PROBLEMIS. 771. 1. In Problem No. 1 above, what is the surface of the sphere? Ans. 50.2656 sq. ft. OPERATION. 4 × 3.1416 = 12.5664 circumference. Explanation.—In all problems of this kind, wo & ) ºf emºs PK48 & & multiply the circumference by the diameter, 12.5664 x 4 = 50.2656 sq. ft. surface. and the result is the surface. 2. What is the surface of a sphere whose diameter is 12 feet 3 & Ans. 452.3904 Sq. feet. 378 soule's PHILOSOPHIC PRACTICAL MATHEMATICS. A. sº t TO FIND THE SOLIDITY OF A HEMISPHERE. PROBLEM. 772. How many cubic inches in a hemisphere whose diameter is 60 inches and radius 30 inches 2 Ans. 56548.8 cu. in. OPERATION INDICATED. 60 § Explanation.—In all problems of this kind, 5236 multiply the square of the diameter by the &-º-º-º-º-º: radius, and multiply the product by .5236. TO FIND THE SOLIDITY OF A SEGMENT OF A SPHERE. PROBLEMS, 773. 1. How many cubic inches in a spherical Segment which has a diam- eter of 60 inches and a depth of 20 inches? Ans. 32463.2 cu. in. OPERATION. 60 -- 2 = 30 in. radius. 69" 302 × 3 = 2700. 2700 + 20° = 3100 = three times the square of the radius and the square of the depth. 3100 × 20 × .5236 = 32463.2 the cu. in. in the spherical Segment. § Jºrplanation.—In all problems of this kind, to three times the square of the radius, we add the square of the depth or height; then multiply this sum by the depth and the product by .5236. NoTE.—When the segment is greater than a hemisphere, find the solidity of the lesser seg- ment and subtract the same from the solidity of the entire sphere. 2. What is the solidity of a spherical segment, the height of which is 6 feet and the diameter of the base 20 feet 7 Ans. 1055.5776 cu. ft. NOTE 1.--To find the surface of a segment of a sphere, multiply the height by the circumfer- ence of the segment, and add the product to the area of the base. NOTE 2.—To find the solidity of a spherical zone, or the frustum of a sphere, add to the sum of the squares of the radii of the two ends or bases, # of the square of the height of the zone or frus- tum ; then multiply this sum by the height and this product by 1.5708. * MENSURATION OF SOLIDS. 379 TO FIND THE SOLIDITY OF A PROLATE SPEIEROID, PROBLEMI. 774. A prolate spheroid has a transverse, or longer diameter of 8 feet, and a conjugate, or shorter diameter of 5 feet. How many cubic feet does it contain 3 Ans. 104.72 cu. ft. OPERATION. 5 = diameter. 8 5 = diameter. Or, 5 .7854 = ratio of circle, etc. 5 8 = height or transverse diameter. .5236 3 || 2 = ratio between C. & P. S. * | 104.72 cu. ft., Ans. º º | º 104.72 cu. ft., Ans. Ea:planation.—In the first statement, we indicate the solution for a cylinder of equal height and diameter as the prolate spheroid, and then multiply by #, because a prolate spheroid is equal, to # of a cylinder of equal height and diameter. In the second statement, we indicate the solution for a rectangular solid, by multiplying together the three dimensions, and then multiply by .5236, which is the ratio between the solidity of a cube and that of a sphere, the diameter of which is equal to one side of the cube. TO FIND THE SURFACE OF A PEROLATE SPEIEROID. PROBLEMI. 775. In the above problem, what is the surface? Ans. 104.7849 sq. ft. OPERATION. 8* = 64. 5* = 25. 64 + 25 = 89 -- 2 = 444; V44} = 6.6708 × 3.1416 x 5 = 104.7849264 sq. ft. Explanation.—In all problems of this kind, square the diameters, and multiply the square root of half their sum by 3.1416, and this product by the conjugate or shorter diameter. The result will be the surface. TO FINID THE SOLIDITY OF AN OBLATE SPEIEROID. PROBLEMI. 776. An oblate spheroid has a height or shorter diameter of 5 feet, and a width or longer diameter of 8 feet. How many solid feet does it contain 3 Ans. 167.552 cu. ft. OPERATION. 8 = diameter. 8 8 = diameter. Or, 8 .7854 = ratio of circle, etc. .5236. 5 = height or conjugate diameter. 5 3 || 2 = ratio between C. & O. S. 167552 cu. ft., Ans. Explanation.—In the first statement, we indicate the solution for a cylinder of equal height 38o soulE's PHILosophic PRACTICAL MATHEMATICs. X\ and diameter as the oblate spheroid, and then multiply by #, since an oblate spheroid is equal to # of a cylinder of equal height and diameter. -- For an explanation of the second statement, see the explanation in the problem under Article 774. TO FIND THE SURFACE OF AN OBLATE SPEIEROID. PROBLEM. 777. In the above problem, how many square feet on the surface? Ans. 167.6558+ sq. ft. OPERATION. 5* = 25. 8* = 64. 25 + 64 = 89 + 2 = 44%. V 44; = 6.6708 × 3.1416 × 8 = 167.65588224 sq. ft. Ea:planation.—In all problems of this kind, square the diameters and multiply the square root of half the sum by 3.1416, and this product by the transverse or longer diameter. The result will be the surface. TO FIND THE SOLIDITY OF A SEMI-PROLATE SPEIEROID. _- PROBLEM. 778. How many cubic inches in one-half of a prolate spheroid, the conjugate diameter A B, of which is 60 inches, and the semi-transverse diameter C D, being 40 inches f Ans. 75398.4 cu. in. OPERATION INDICATED. 3. Or thus: 60° x 40 × .5236. 4 º NoTE.—See Article 774, page 379. 'D' 401? TO FIND THE SOLIDITY OF A SEMI OBLATE SPHEROID. PROBLEM. 779. How many cubic inches in one-half of an oblate spheroid, whose longer diameter A B is 80 inches, and whose semi-conjugate diameter C D is 20 inéhes? Ans. 67020.8 cu. in. jºr MENSURATION OF SOLIDS. 381 OPERATION INDICATED. Or thus: 80° x 20 × .5236. r |: 20 .5236 NOTE.-See Article 776, page 379. 20', TO FIND THE SOLIDITY OF A SEGMENT OF A PROLATE SPHEROID. PROBLEMI. 780. In a prolate spheroid, having a transverse diameter A B of 96 inches, and a conjugate diameter C D of 60 inches, the height of the segment E B F is 20 inches. What is the solidity of the segment? Ans. 20289.5 cu. in. 96// A OPERATION. 96 x 3 = 288; 20 × 2 = 40 ; 288 — 40 = 248; 248 x 20° × .5236 = 51941.12 inches. Then 962 : 60% : : 51941.12 : 20289.5. Ea:planation.—In all problems of this kind, multiply the transverse diameter by 3; then from this product, subtract two times the height E F of the segment; then multiply the remainder by the square of the height of the segment and this product by .5236; then multiply this result by the square of the conjugate diameter and divide the product E by the square of the transverse diameter. 2011 C D 60” TO FIND THE SOLIDITY OF A SEGMENT OF AN OBLATE SPEIEROID. PROBLEM. 781. In an oblate spheroid the transverse diameter A B is 96 inches, the conjugate diameter C D is 60 inches, and the height of the segment E D F is 10 inches. What is the solidity or capacity of the Segment 7 Ans. 8770.3 cu. in. 60 t OPERATION. 96 × 3 = 288; 10 × 2 = 20; 288 – 20 = 268; 268 × 102 × .5236 = 14032.48. Then 96 : 60 : : 14032.48 : 8770.3, Earplanation.—In all problems of this kind, multiply the transverse diameter by 3, and from the product subtract two times the height of the segment; then multiply the remain- der by the square of the height of the segment and this product by .5236; then multiply this result by the conjugate diameter and divide the product by the transverse diameter. B 961, 382 SOULE's PHILOSOPHIC PRACTICAL MATHEMATICs. jºr REMARKS ON THE MEASUREMENT OF SEGMENTs. The measurement of segments is attended with much difficulty in the prac- tical affairs of life. As shown in Article 773, page 378, the measurement of a seg- ment of a circle is a very simple operation. But, as shown in Article 780, page 381, and Article 781, page 381, the measurements of the segments of prolate and oblate spheroids are very difficult operations, and require as elements, in their solutions, the transverse and the conjugate diameters of the spheroids. Hence the difficulty of the operation in practice, where the diameter and the depth of the segment only are known. The operations to measure the segments of spheres, of prolate, and of oblate spheroids are, as shown in the above problems, very different, and unless we know what kind of a segment we have to measure, we cannot perform the operation correctly. And this knowledge cannot, in all cases, be determined from the diam- eter and the depth of the segment. To illustrate this point, we will suppose that it is required to find the capacity of a segment in the form of a sugar kettle, or a boiler, whose diameter is 70 inches and depth is 25 inches. 2–GTS Before we can proceed with accuracy, we must A g- }* X —-1 70" know whether the segment is that of a sphere, or of a prolate or of an oblate spheroid. T) If we proceed to measure it as a segment of a 25'ſ sphere, the result of our operation will be greater than if we had measured it as a segment of a prolate spheroid, and less than if we had measured it as a segment of an oblate spheroid. To have measured it as a segment of a prolate or of an oblate spheroid, we must have known the transverse and conjugate diameters of the figure of which the segment is a part. And this knowledge We could not have obtained from the given diameter and depth of the segment. From the above, we see the difficulty in practice of measuring segments of spherical solids. The best that can be done in cases where the figure is not known, and where the dimensions given are limited to the diameter and the depth of the segment, is to measure it as a segment of a sphere, and then, if it is possible, to judge from the form and dimensions to what class of segments it belongs; to allow a small deduction in case it is a segment of a prolate spheroid, and to allow a small increase in case it is a segment of an oblate spheroid. NotE.—In many cases where the capacity of segments of an unknown class is required, the most expeditious and accurate method of determining the matter, would be to discard mathema- tics, and to measure the vessel by a gallon measure. sº * MENSURATION OF solids. 383 TO FIND THE SOLIDITY OF THE MIDDLE FRUSTUM OF A PROLATE SPEIEROID. PROBLEMI. 782. The middle frustum of a prolate spheroid-i o, is 70 inches in length, 60 inches e d, conjugate diameter, and 50 inches at its ends, ef and g h. What is its Solidity or capacity? Ans. 177762.2 cu. in. OPERATION. k º all problems of this ind, first square the conjugate diame- 60° × 2 = 7200 + 50° = 9700. Her and multiply the product by 2; Fº then, to this product, add the square of Then 9700 × 70 × .2618 = the diameter of one end; then multi- 177762.2 cubic inches. ply this sum by the length of the frus- tum, and the product by .2618. TO FIND THE SOLIDITY OF THE MIDIDLE FRUSTUM OR ZONE OF AN ELLIPTIC SPINDLE. --- -- PROBLEM. 783. In the frustum of an elliptic spindle, the length e f is 70 inches, the greatest diameter a b is 60 inches, the least diameter 50 inches, and the middle diameter gh, (equally distant from the middle and end) is 56 inches. What is the solidity or capacity ? Ans. 170834.972 cu. in. OPERATION. Explanation.—In all problems of this * kind, add together the squares of the 2 2 — 2 3 c Q. 60° -- 50° = 6100 + (56 × 2) greatest and least diameters, and the = 18644. square of double the medium diameter; Then 18644 × 70 × .1309 = then multiply the sum by the length 170834,972. and the product by .1309. TO GAUGE OR FIND THE SOLIDITY OF CASKS OR BARRELS. PROBLEMI. 784. In a cask 70 inches long, 60 inches conjugate or bung diameter, and 50 inches head diameter, how many cubic inches 3 Ans. 1724.11.008 cu. in. OPERATION. 60 — 50 = 10, difference be- Eacplanation.—In all problems of this tween bung and head diame- kind, add to the lesser diameter .5.6 terS. y e or .7 of the difference between the 10 × .6 = 6. -- 50 = 56 av- erage diameter of cask. Then 562 × .7854 × 70 = the staves may be slight, medium, or 17241.1.008 cubic inches. above medium. NotE.—In the three preceding problems, the cask, the middle frustum of a prolate spheroid, and the middle frustum of an elliptic spindle are very similar prisms, and to show the different results by the different methods of work we gave the same dimensions to each prism, with the additional diameter to the frustum of the elliptical spindle. The results are as follows: 1. By the method of measuring middle frustums of prolate spheroids 177762.2. 2. By the method of measuring middle frustum of elliptical spindle 170834.972. 3. By the method of measuring casks and barrels 17241.1.008 cubic inches. diameters, according as the curve in 384 SOULE's PHILOSOPHIC PRACTICAL MATHEMATICs, TO FIND THE SOLIDITY OF A CYLINDRICAL RING. PROBLEM. 785. What is the solidity of a cylindrical ring that is 3 inches thick and has an inner diameter of 15 inches? Ans. 399.7188 cubic inches. OPERATION. Explanation.—In all problems of this * kind, first add to the thickness of the ring 3 + 15 = 18 × 3° = 162 × the inner diameter, and then multiply 2.4674 = 399.7188 cubic inches. this sum by the square of the thickness, ==== and the product by 2.4674. NoTE.—To find the convex surface of a cylindrical ring : First add to the thickness of the ring the inner diameter, then multiply this sum by the thickness, and the product by 9.8696. TO FIND THE CUBICAL CONTENTS OF A WEDGE. PROBLEMIS. 786. 1. A wedge is 20 inches long, 6 inches wide and 4 inches thick. How many cubic inches does it contain 3 Ans. 240 cubic inches. FIRST OPERATION. 6 + (6 × 2) = 18; 18 × 20 x 4 = 1440; 1440 - 6 = 240 cu. in. Explanation.—In all problems of this kind, add to the length of the edge or point, twice the length of the back or head; then multiply this amount by the length and the thickness of the wedge and divide the product by 6, the quotient will be the cubical contents. NotE.—When a wedge is a true prism as the above, the solidity is equal to the product of its three dimensions, divided by 2. 2. In the above problem, what would have been the number of cubic inches, if the edge of the wedge had been 10 inches long Ans. 293; cubic inches. -- OPERATION. 10 + (6 × 2) = 22 × 20 × 4 = 1760 - 6 = 2933 cubic inches. MEASUREMENT OF LOGS, TIMBER, LUMBER, AND BOARDS. 787. There are various kinds of scales, and rules, and tables, that lumber dealers have and often use in the measurement of lumber, by means of which very little calculation is required. But as these scales and rules are readily understood on inspection, and as the results obtained by their use are in many cases only approximative, we therefore omit a description of them, and present accurate meth- ods by means of figures. In the measurement of round timber, different customs and methods prevail in different localities, which we will point out in the problems presented a few pages further on. A Standard Saw Log is 12 feet long and 24 inches in diameter. ROUND TIMBER. 385 ROUND TIMBER, -º- TO FIND THE SOLIDITY OR OUBIC FEET OF ROUND TIMBER OR LOGS. PROBLEMS. 788. 1. The length of a log is 30 feet, and the average girt or circumfer- ence is 7 feet; how many cubic feet does it contain? Ans. 117.6 cubic feet. FIRST OPERATION. 7 Explanation.r-In all problems of this character, we 7 multiply the square of the average girt or circumfer- 30 ence by the length, and the product thus obtained by 8, and then divide by 100 or point off two figures in 100 || 8 this last product, and we have the solidity very ºs- nearly. This method of work º: and º: 3. º than the usual methods, and while the result is a 1173 cubic feet, Ans. little in excess of the mathematical result based on e º the presumption that the log, or tree, is a regular º it is nearer the exact measure of the frustum of a cone than most of the usual methods Of WOrk. li º following statement gives the correct mathematical result considering the log as a cylinder: (7 x 7 x 7854 x 30) + (3.1416 x 3.1416) = 116.9786 + cu. ft. 2. What are the contents by the two above methods, of a log 62 feet 5 inches long, that girts 10 feet 9 inches at the larger end, and 3 feet 7 inches at the Smaller end? Ans. 256.463 cu. ft. by the first method. 255.108 cu. ft. by the second method. OPERATION BY THE FIRST METHOD, Ft. In, 12 || 86 10 9 girt of larger end. 12 || 86 3 7 girt of smaller end. 12 || 749 *-*- 100 || 8 2) 14 4 - & 256.463-H cu. ft., Ans. 7 2 approximate average girt or circumference. OPERATION BY THE SECOND METHOD, 12 || 86 3.1416 12 || 86 3.1416 | .7854 12 || 749 255.108-- cubic feet, Ans 3. A log is 50 feet long, 40 inches diameter at the larger end and 20 inches diameter at the smaller end. How many cubic feet does it contain, measured by 386 soul E's PHILOSOPHIC PRACTICAL MATHEMATICS Yºr the first above method, and how many when measured accurately as the frustum of a cone # Ans. 246.741 cu. ft. by first method. 254.527 cu. ft. accurate measurement. FIRST METHOD. 40 x 3.1416 = 125.6640 circumference of larger end. 20 × 3.1416 = 62.8320 circumference of smaller end. 125.664 + 62.832 = 188.496 - 2 = 94.248 approximate average cir- cumference or girt. Then (94.248° x 50 x 8) + (144 x 100) = 246.741+ cubic feet. OPERATION FOR, ACCURATE MEASUREMENT AS A FRUSTUM OF A CONE. 402 = 1600 202 = 400 × 2800 + 3 = 933% accurate area of a square one side of 40 × 20 = 800 which is equal to the average diameter of the log. Then (9334 × 50 × .7854) -- 144 = 254,527+ cubic feet. NoTE.—This is the only method that is mathematically correct; but it is not adopted by lumber dealers when buying or selling logs. They make allowance for crooks, splits and other defects, and hence adopt approximate methods. Logs are more generally bought by board meas- ure, and the different methods adopted for board measure will be presented under the heading of Lumber and Board Measure, two pages following. TO REDUCE ROUND TIMBER TO SQUARE TIMBER. PROBLEM. 789. How many solid feet of square timber are there in a stick of round timber 28 feet long, 34 inches in diameter at the larger end, and 21 at the smaller end ? Ans. 65+ cubic feet. OPERATION. * 34 in., diameter of larger end. 3 || 55 21 4 { % “ Smaller “ ” 12 || 55 tº- 3 || 28 2) 55 12 *- 3) 27# in., approximate average or mean diameter. 9% = # deducted. º-ºº- 18% in., average of each side of the square stick. Explanation.—In working problems of this character, different methods are used by different lumber dealers. In this solution we allowed # of the average diameter for the squaring of the stick, and thus obtained 18% inches as the average measure of one side of the square stick. Then having a square stick (18+ inches by 18% inches), we multiply the three dimensions together in the unit of feet and obtain cubic feet. This method of work is generally used, but where the timber is large, and especially where there is but little taper to the stick, the deduction of # is found to be too great, hence some dealers deduct but 4 of the average diameter instead of #. To work the foregoing problem by deducting #, we produce the following figures and result: 65;}} cu. ft. Ans. 4) 27# in., average diameter as above. | 8 | 165 63 in., - # deduction. 8 || 165 4- 12 203 in., average of each side of the square stick. 12 || 28 82}#} cu. ft., Ans. 4): ROUND TIMBER. 387 TO FIND THE CUBICAL OR SOLID CONTENTS OF SQUARED OR FOUR-SIDED TIMEER. PROBLEMI. 790. A piece of timber is 34 feet long, 16 inches wide, and 15 inches thick; how many solid feet does it contain 3 Ans. 563 cubic feet. OPERATION. 34 Explanation.—In all problems of this character, we 12 | 16 multiply the length, width and thickness together in 12 || 15 the same unit of measure, and the result is the cubical contents. 563 cubic feet, Ans. TO FIND THE CUBICAL CONTENTS OF SQUARED OR FOUR-SIDED TIMBER THAT TAPERS REGULARLY. PROBLEM. 791. The larger end of a rectangular stick of timber is 25 inches wide by 20 inches thick; the smaller 16 inches wide by 12 inches thick, and the length is 30 feet; how many cubic feet does it contain? Ans. 694; cubic feet. FIRST OPERATION. width. Thick- * IleSS. Larger end 25 × 20 = 500 sq. in. in larger end. 12 || 334 Smaller “ 16 × 12 = 192 sq. in. in smaller end. 12 *mmº tº 30 41 × 32 = 1312 = 4 times the middle sec- — ºsmºmsºme tion between the ends. 6) 2004 69 is cubic feet, Ans. 334 sq. in., average area of the stick. Ea:planation.—In all problems of this character, where the taper of the adjacent sides is not in the same ratio, we first multiply together the width and thickness of the two ends, and then add together the widths of the two ends and also the thickness, and multiply together their sums. We then add the three products together and divide the sum by 6, which gives the average area of the stick in square inches, which we reduce to the unit of feet, by dividing by 12 × 12, and multiply by the length, and obtain cubic feet. NOTE.—When the timber does not taper regularly, measure separately the parts that do, and take the sum for the entire solidity. SECOND OPERATION BY THE PRISMOIDAL FORMULA, ELUCIDATED ON PAGE 376. 25 × 20 = 500 sq. in. in larger end. 16 × 12 = 192 sq. in. in Smaller end. 2) 4.1 × 32 204 × 16 = 328 x 4 = 1312 = 4 times the middle section between the 2 -*- al’é2,S. 6 ) 2004 tº-º- 334 = Sq. in. average area of the stick. Then 334 x 30 - 144 = 69 is cubic feet, Ans. 388 SOULE's PHILOSOPHIC PRACTICAL MATHEMATICs. * TO FIND THE LENGTH THAT MUST BE CUT OFF OF A SQUARE PIECE OF TIMBER WHICH DOES NOT TAPER, TO OBTAIN \ A GIVEN SOLIDITY. - PROBLEMS. 792. 1. A piece of timber is 8 inches wide and 6 inches thick, how much must be cut off to make 1 cubic foot ? Ans. 36 inches. OPERATION. 1 cubic foot = 1728 cubic in. 8 || 1728 8 × 6 = 48 square inches. Or, 6 1728 -- 48 = 36 in., Ans. gº 36 in., Ans. 2. A piece of timber is 24 inches wide and 16 inches thick; what length is required to make 4 cubic feet? Ans. 18 inches. OPERATION. 24 || 4 16 || 1728 | 18 inches., Ans. 3. A stick of timber is 4 inches thick and 16 inches wide; how much must be cut off to make 6 cubic feet 3 Ans. 134 feet. © º LUMBER AND BOARD MEMSURE, -Ö- 793. Lumber, as the term is here used, includes boards, planks, joists, scantlings, and sawed timber of all kinds. 794. A Standard Board is one that is 12 feet long, 12 inches wide, and 1 inch thick. Hence 1 Board Foot is 12 inches long, 12 inches wide, and 1 inch thick, or 1 foot long, 1 foot wide, and 1 inch thick, and contains 144 square or board inches. NOTE.- Since 1 board foot contains but 144 board inches, there are 12 times as many board feet as cubic feet in lumber, timber, and logs. Hence to change board feet to cubic feet, divide by 12; and to change cubic feet to board feet, multiply by 12. 795. A Standard Saw Log is 12 feet long and 24 inches in diameter. ROUND TIMEER, OR LOGS REDUCED TO BOARD MEASURE. 796. There are various methods of ascertaining approximately the number Yºr LUMBER AND BOARD MEASURE. 389 of feet of square edged inch boards that can be cut from a given log; different methods are adopted by different lumber dealers, which give approximate results sufficiently accurate to answer practical purposes. * The actual number of feet varies according as the logs vary in not being per- fectly straight cylinders, and according to the manner of sawing them. If the log is first sawed into inch boards, and each board is then separately edged, it will give several feet more than if the log is first slabbed, and then sawed into inch boards. PROBLEM. 797. How many feet can be sawed from a log 20 feet long, and 18 inches in diameter at the smaller end ? Ans. 245 feet. OPERATION BY THE DOYLE METHOD, OR, RULE. 18 in., diameter. d †† th. method, 4 inches are e Or educted from the diameter in order to 4 in., allowed to square the log. reduce the log to a square. Then the square º º º of the remainder will be the board feet of a 14 in., each side of the Square stick. log 16 feet long. Then since the yield of 14 logs is in the ratio of their lengths, we 20 therefore divide by 16 to obtain the board feet in a log 1 foot long, and multiply by 20 to obtain the board feet in a log 20 feet long. 16 smºs- 245 sq. feet, Ans. GENERAL DIRECTIONS FOR DOYLE'S RULE. 798. From the diameter of the smaller end deduct 4 inches; then square the remainder and multiply the product by the length of the log given, and divide this product by 16. The above method is the basis of the table given in Scribner's Lumber and Log Book, which is probably used more generally than all other books of this kind. Though this method, from a mathematical point of view, is only approximate, it is practical in its simplicity and is regarded with favor by lumber dealers. This method favors the buyer in the case of small logs, and the seller in the case of large logs. For logs 14 inches in diameter, this method gives the same result as squaring the log and allowing # for saw kerf. For logs 35 inches in diameter, it gives # of the solid contents; and for logs 50 inches in diameter this method is equivalent to allowing nothing for slab and less than % for saw kerf. In the combination of figures, to produce this method, it is estimated that the saw cuts # of an inch each time it goes through the stick, and as the standard board is 1 inch thick, it is clear that 4 of the stick is cut away by the saw, and accordingly an allowance is made therefor. The Doyle method is adopted by the New Orleans lumber dealers, with the following regulations regarding t CLASSIFICATION, INSPECTION AND DEDUCTION. 799. Logs are classified as cypress, pine and oak. 39C) SOULE's PHILOSOPHIC PRACTICAL MATHEMATICS. A. DEDUCTIONS. For the hollow butts of cypress, deduct the length of the hollow. For the red heart of pine, and for all pecky logs, deduct 10 feet from the length of all such logs. For crooks, knots and wind shakes, deductions are made as per agreement. OTHER METEIODS. 800. Some lumber dealers adopt the following method: Subtract from the diameter of the smaller end eacpressed in inches, # or 4 of itself, according as the logs are small or large, and multiply the square of the remainder by the length in feet, and divide the product by 12. PROBLEMI. How many board feet in a log 22 feet long, and 28 inches in diameter at the smaller end, working first by the Doyle method, second by deducting #, and third by deducting # of the diameter? Ans. By the Doyle method, 792 board feet; by deducting #, 638;# board feet; by deducting #, 8084 board feet. OPERATION. OPERATION. OPERATION. By the Doyle method. When # of the diameter is When + of the diameter is deducted. deducted. 28 28 28 4 9% 7 24 183 21 24 - 21 16 || 22 3 || 56 12 22 tº-º-º-e 3 || 56 +--------> 792 ft., Ans. 12 22 808; ft., Ans. | 638% ft., Ans. TO FIND THE BOARD OR SQUARE FEET IN PLANKS, GIRDERS, SCANTLINGS, JOISTS, AND SQUARE TIMBER. PROBLEMS. 801. 1. How many square feet of lumber in a board 16 feet 4 inches long and 15 inches wide 3 Ans. 2013 Sq. ft. OPERATION. * tº 3 49 12 196 Explanation.—In all problems of 4 || 5 or, 12 || 15 this kind, we multiply the length º tº-º-º-º: and width together in the unit of Practical Solution.—Since the board is 15 inches wide, it contains 14 square feet for each foot of length. Hence 14 times 16# = 20+ square feet. Ans. X} LUMBER AND BOARD MEASURE. 391 2. A board is 20 feet 6 inches long, 21 inches wide at one end and 15 inches at the other end. How many square feet does it contain } Ans. 30% sq. ft. OPERATION." 21 in., wider end. 2 || 41 15 in., narrower end. 12 | 18 2) 36 | 30% sq. ft., Ans. 18 in., average or mean width. Practical Solution.—Since the average width of the board is 18 in. wide, it contains 14 sq. ft. for each foot of length. Hence 14 times 20% = 30% sq. ft. Ans. 3. How many square feet in a plank 24 feet long, 22% inches wide, and 3 inches thick? Ans. 135 sq. ft. OPERATION. 24 Explanation.—As the board foot is 1 inch in thickness, 2 || 45 it is clear that when the thickness exceeds 1 inch the 12 || 3 o measurement must be increased accordingly ; hence when- ever the thickness exceeds 1 inch, we multiply by the *º thickness. When the thickness is less than 1 inch, by 135 sq. ft., Ans. custom, no deduction is made, the measurement being in that case the same as if the lumber was 1 inch thick. Practical Solution.—Since the board is 224 in. wide, and 3 in. thick, it is practically 67.4 sq. in. or 53+ sq. ft. for each foot of length. Hence 5}} times 24 = 135 sq. ft. Ans. 4. What is the number of board feet in 16 pieces of scantling each 20 feet long, 4 inches wide, and 3 inches thick? Ans. 320 board feet. Practical Solution.—4 in. wide and 3 in. thick, is equivalent to 12 in. wide or 1 sq. foot for each foot of scantling; and as there are 16 scantlings, hence 20 times 16 is 320 sq. ft. Ans. 5. How many board feet in 200 girders each 30 feet long, 15 inches wide, and 2 inches thick. And what will they cost at $18 per M. ? Ans. 15000 board feet; $270 cost. Practical Solution.—15 in. wide, by 2 in. thick = 24 sq. ft. to the foot. Hence 24 times 30 = 75 × 200 = 15000 sq. ft.; and $18 × 15m = $270. Ans. 6. What will be the cost of 4 black walnut boards, each 10 feet 9 inches long, 28% inches wide, and 1% inches thick, at $55 per M.3 Ans. $9.829##. 7. A piece of timber is 34 feet long, 16 inches wide, and 15 inches thick. How many board feet does it contain 3 Ans. 680 sq. ft. Practical Solution.— 16 in. = 1 ft. wide by 15 in. thick = 20 sq. ft. for each foot of length. Hence 20 times 34 = 680 sq. ft. Ans. 8. A rectangular telegraph pole is 60 feet long, 16 inches square at the larger end and 6 inches square at the Smaller end. How many cubic and how many board feet does it contain } Ans. 53; cubic feet; 6463 board feet. NOTE.-See Article 761, page 373. 392 soul E's PHILOSOPHIC PRACTICAL MATHEMATICS. * 9. A circular telegraph pole is 60 feet long, 16 inches in diameter at the larger end, and 6 inches in diameter at the smaller end. How many cubic and how many board feet does it contain, and what is the cost at 15% per cubic foot ? Ans. 42.32434 cubic feet; 507.892 board feet; $6,34865 cost. NotE.—See Article 768, page 375. 10. How many superficial or board feet in 40 scantlings, each 154 feet long, 4 inches wide, and 2 inches thick? Ans. 4134 feet. SOLUTION STATEMENT. 1. 3. Practical Solution.—4 × 2 = 8 in. = 3 sq. ft. 154 × * | * 40 = 620 × # = 4134 sq. ft. 40 or thus: - 153 x 40 = 620 – 4 of itself = (206§) 4134 sq. ft., Ans. 4135 sq. ft., Ans. 11. How many square feet in 62 joists, each 20 feet long, 14 inches wide, and 3 inches thick; and what will be the cost at $30 per thousand? Ans. 4340 feet. $130.20, cost. 12. How many square feet (board measure) in a square piece of timber, 20 feet 8 inches long, 14 inches wide, and 9 inches thick? Ans. 217 feet. 13 What will be the cost of 846 feet of boards at $22.50 per M., and of 240 feet of flooring at $32.75 per M. 3 Ans. $26,894. 14. The width of a board is 16 inches, what must be its length that it may contain 24 square feet? * Ans. 18 feet. OPERATION. 24 sq. ft., contents of the board. 144 sq. in., in each square foot. 3456 sq. in. -- 16 in., the width, – 216 in. -- 12 *mme I = 18 feet, the length of the board. 3456 sq. in., contents of the board. - 15. A plank is 2 inches thick and 14 feet long, how wide must it be to contain 42 feet ſº Ans. 18 inches. OPERATION. 42 sq. feet in the board. 144 sq. inches in each sq. foot. e º 6048 - 168 = 36 inches, width of plank 1 inch 6048 sq. inches. thick. 14 feet, length of plank. 36 -- 2 = 18 inches, width of plank 2 inches 12 in. in 1 foot. thick. 168 inches, length of plank. 16. A board is 8 inches wide; what length will make 2 square feet? w Ans. 3 feet. OPERATION. 2 sq. feet. 144 Sq. inches in each sq. foot. 288 sq. in. -- 8 in., the width of the board, = ºmºsºme 36 in. or 3 feet. Ans. 288 sq. in. in 2 sq. feet. PRACTICAL PROBLEMS IN THE MENSURATION OF SOLIDS, 393 PRACTICAL PROBLEMS IN THE MENSURATION OF SOLIDS, —-º- TO FIND THE NUMBER OR OUBIC FEET AND INCHES IN BOXES AND SOLIDS OF DIFFERENT FORMS. PROBLEMIS. 802. 1. How many cubic feet in a box 6 feet long, 3 feet wide, and 2 feet deep f Ans. 36 cubic feet. OPERATION. Explanation.—In all problems of this character, we have but to multiply the three dimensions 6 × 3 × 2 = 36 cu. ft., Ans. together in the unit of feet. 2. How many cubic feet in a box 4 feet 3 inches long, 3 feet wide, and 16 inches deep 3 Ans. 17 cubic feet, OPERATION 12 || 51 4 || 17 Explanation.—Whenever any of 3 OT 3 the dimensions are given in inches, 12 | 16 ' '3 || 4 we either divide them by 12, in order to reduce them to feet, or cº-º ºmº gº sº else we use the inches as a fraction 17 cu. ft., Ans. 17 cu. ft., Ans. of a foot, as shown by the second line statement. OPERATION. 3. How many solid feet in 8 12 28 boxes, each 28 inches long, 10 inches }. 10 wide, and 6 inches deep 3 12 || 6 Ans. 7% solid feet. s 7 # solid feet, Ans. 4. How many cubic feet in a cylinder 6 feet long, and 3 feet 4 inches in diameter ? Ans. 52.36 cubic feet. 5. How many cubic feet in a frustum of a cone, whose height is 6 feet, diameter of the greater end is 4 feet and of the smaller end 3 feet? Ans. 58.1196 cubic feet. NOTE.-See Problem on page 376. 6. How many Solid feet in a sphere that is 30 inches in diameter 3 Ans. 8.18125 solid feet. 7. How many cubic inches in a cannon ball which is 13 inches in diameter 7 Ans. 1150.3492 cu. in. 8. Suppose an orange to be exactly spherical and 4 inches in diameter, how 394 SOULE's PHILOSOPHIC PRACTICAL MATHEMATICS. * many Oranges, 2 inches in diameter and spherical in form, will it take to equal the larger orange? Ans. 8. FIRST OPERATION. SECOND OPERATION. 4* = 64 cube of larger orange. | ; : 2* = 8 “ “ smaller 4 * * 64 + 8 = 8 oranges, Ans. 33.5104 -- 4.1888 = 8 oranges, Ans. . . Explanation.—The first solution of this problem is based upon the geometrical fact that all solids are to each other as the cubes of their like dimensions. - In the second solution, we first find the solid contents of each, and then we find how many times the contents of the larger orange is equal to the contents of the smaller. 9. If an apple, spherical in form, and 12 inches in circumference, sells for 10 cents, what is the proportional value of 3 apples, also spherical in form, that are 5 inches in circumference? Ans. 2 #2. 10. How many cubic inches in a semi-oblate spheroid which is 70 inches in diameter, and 30 inches deep? Ans. 76969.2 cu. inches. NOTE. – See Article 779. 11. An orange peddler sells two oranges which are each 3 inches in diameter, or 3 oranges which are each 2 inches in diameter, for 52. Allowing the oranges to be perfect spheres, which is the better purchase, and how many cubic inches of orange would be gained ? Ans. The better purchase would be the 2 Oranges each 3 in. in diameter. The gain would be 15.708 cu. in. TO FIND THE COST OF FEEIGHT PER, CUBIC FOOT ON BOXES. PROBLEMS. 803. 1. What will be the freight on a box 9 feet 3 inches long, 4 feet. 6 inches wide, 2 feet 10 inches deep, at 30 cents a cubic foot ? Ans. $35.38%. 2. What is the freight on 8 boxes, which are each 3 feet 3 inches long, 2 feet 8 inches wide, and 21 inches deep, at 202 per cu. foot ? Ans. $24,263. TO FIND THE NUMBER OF BUSHELS IN VESSELS, BOXES, BINS, ETC. PROBLEMS. 804. 1. How many bushels will a bin hold, that is 10 feet long, 8 feet 6 inches wide, and 5 feet 2 inches deep 3 Ans. 352.89-H bushels. OPERATION. #! Explanation.—In this problem, we multiply the three 215042 | 102 dimensions together, in the unit of inches, which gives 62 cubic inches, and then divide by 2150.42 the number of 100 cubic inches in a bushel. Iły reason of the decimals in 352.89-H. Ans the 2150.42 we also multiply by 100 to cancel the decimal. NotE.—2150.4 cu. in. instead of 2150.42 cu. in. is often used, and gives a result sufficiently accurate for practical purposes. Had we used 2150.4 the result would have been 352.9. X- PRACTICAL PROBLEMS IN THE MENSURATION OF SOLIDS. 395 2. A wagon box is 12 feet long, 44 feet wide, and 15 inches high. How many bushels will it hold when even full? Ans. 54.24+ bushels. . NotE.-A very short way to find the number of bushels any box or vessel will hold is to multiply the cubic feet it contains by 45 and divide the product by 56. This is correct because the ratio between 2150.4 and 1728.0 is as 56 to 45. 3. A planter wishes to build a bin that will hold 100 bushels; it is to be 10 feet long and 44 feet high. What must be the width of the bin 3 Ans. 33.185-H inches. OPERATION. 2150.42 = cu. in. in 1 bu. 120 in. = length. 100 = bushels. 54 in. = height. 215042.00 = cu. in. in 100 bu. 6480 sq. inches. 215042 + 6480 = 33.185+ inches, Ans. 4. How many bushels in a cylinder shaped box, whose height is 10 feet, and diameter 10 feet 3 Ans. 631.125 bu. 1St OPERATION. 2d OPERATION. 120 10 Ea:planation.—In the second operation, we first 120 10 make the statement to find cubic feet in the 120 10 cylinder; then instead of multiplying by 1728 *f PF to reduce the cubic feet to inches, and dividing 2150.4 | .7854 .7854 by 2150.4 to reduce the cubic inches to bushels, *m- I - 56 || 45 we multiply by 45 and divide by 56. These - sºmeº- numbers are obtained by reducing # to its lowest terms. NotE.-In this problem, we use 2150.4 cu. in. as a bushel which is, for ordinary purposes, sufficiently correct. 5. A sugar kettle in the form of a half of a prolate spheroid is 42 inches deep and 50 inches in diameter. How many bushels, and how many gallons will it hold # Ans. 25.566–H bushels. 238 gallons. OPERATIONS INDICATED. Bushels. Bushels. Gallons. Gallons. 42 42 50 50 42 50 42 50 | 50 or, .7854 50 Or, .7854 50 3 2 50 3 2 2150.42 .5236 2150.42 231 .5236 231 | 25.566 bu. 25.566 bu. 238 gals, 238 gals. NoTE.—See Article 778. TO FIND THE NUMBER OF CUBIC YARDS IN LEVEES OR EXCAVA— TIONS. PROBLEMS. 805. 1. How many cubic yards in a levee 80 rods long, 60 feet wide at the base, 124 feet at the top, and 5 feet 4 inches average depth ? Ans. 9451# cubic yards. 396 soule's PHILOSOPHIC PRACTICAL MATHEMATICs. * OPERATION. 60 Width at the base. 80 Explanation.—Here 12; “ “ “ top. 2 || 33 we first multiply the -º-e 4 || 145 three dimensions to- 2) 72} 12 || 64 gether in the unit of 27 feet, * then divide * ~ * by 27, the number of 36+ average width. a- - mºs cubic feet in a cubic 945.13% cu. yas., Ans. yard. 2. A railroad contractor excavated a cut through a hill 840 feet long, 70 feet wide on top, and 30 feet wide on the bottom, average depth 50 feet. What was the cost at 212 per cubic yard 7 Ans. $16333.33%. 3. A tunnel was excavated 1350 feet long with a cross section or area of 660 sq. feet. How many cubic yards, and what was the cost at 30¢ per cu. yard? Ans. 33000 cu. yards. $9900, cost. TO FIND THE NUMBER OF GALLONS IN VESSELS, CISTERNs, ETC. PROBLEMIS. 806. 1. How many gallons will a box hold, that is 5 feet long, 2 feet 4 inches wide, and 3 feet deep 3 Ans. 261.81–H gallons. OPERATION. 60 Explannation,-In this problem, we multiply the three 231 ; dimensions together in the unit of inches, and divide by 231, the number of cubic inches in a gallon. 261.81-H gal., Ans. 2. How many gallons in a cylindrical cistern or tank 12 feet 4 inches high, and 8 feet 4 inches in diameter ? Ans. 5032 gals. 1st Full Operation. 2d Contracted 3d Contracted 4th Contracted 5th Contracted Operation. Operation. Operation. Operation. 148 148 3 || 37 3 || 37 3 ||37 feet. 100 100 3 || 25 3 || 25 3 || 25 “ 100 { 100 25 25 3 || 25 “ 231 | .7854 34 5.8752 5.875 8 || 47 “ 5032.0000 gals. 5032 gals. 5032 gals. 5031.8287 5031,8287 Explanation to 18t Operation.—In all problems of this character, we multiply the height and the square of the diameter together in the unit of inches, then the product thus produced by the decimal .7854, and then divide by 231, the number of cubic inches in a gallon. Explanation of 2d Operation.—Since.7854 is equal to 231, .0034 times, we may shorten the work by simply multiplying by .0034, omitting the division by 231. In the operation we set 34 only, remembering there will be four decimal places in the answer. Baplanation of the 3d Operation.—In this operation we use the three factoral dimensions in the unit of feet, and multiply their product by 5.8752. This 5.8752 is produced thus: In a cubic foot there are 1728 cu. inches; this we multiply by .7854, which is the area of a circle whose diameter is equal to one side of a square, and then divide the product by 231 the number of cubic inches in a gallon. Thus the 5.8752 is the number of gallons in a cubic foot reduced to a cylinder. Explanation of the 4th and 5th Operations.—In the 4th operation we shorten the work by mul- tiplying by 5.875 instead of 5.8752, and produce an answer nearly or practically correct. In the 5th operation we consider that as 5.875 gallons equals 5% gallons, and as 5% equals # gallons, we hence, multiply by 47 and divide by 8. NOTE.-The 4th and 5th methods are so nearly correct that they are used by many builders and engineers. We prefer the 2d Contracted Operation, as it is exact and in the majority of cases it is shorter than the other methods. 3. How many pints in a cylindrical vessel, whose height is 14 inehes and diameter 12; inches 3 Ans. 59.5 pints. #: PRACTICAL PROBLEMS IN THE MENSURATION OF SOLIDs. 397 OPERATION. 14 2 || 25 e 2 || 25 Ea:planation.—Here we first find the number of 34 gallons, the same as in the preceding problem, and : then we multiply by 4, which gives us quarts, and *- then by 2, which gives us pints. 59.5 pints, Ans. 4. How many gallons of water will a steamboat boiler hold, that is 40 feet long, 5 feet 4 inches in diameter, allowing for 3 flues, each 15 inches in diameter Ans. 5583.072 gals. OPERATION. OPERATION. TO FIND THE CONTENTS IN GALLONS OF THE | TO FIND THE DEDUCTIONS IN CONSEQUENCE WHOLE BOILER. OF THE 3 FLUES. 40 40 12 12 64 15 64 15 34 34 -ms - sºmmº- sºmmº- 3 6684.672 sm - mammº mºnºmº 1101.600 1101.600 5583.072 gals., Ans. 5. How many gallons in a cistern which is in the form of a frustram of a cone, whose height is 9 feet 6 inches, lower base 7 feet 2 inches, and upper base 6 feet 8 inches : Ans. 2671.3392 gals. OPERATION. 862 – 7396 3 || 20676 Ea:planation.—In all prob" 80.2 = 6400 114 }..."; th; character, we -> I'S nd the contents or 86 X 80 = 6880 34 - ::::::::::: the cistern i. =sºmmºn wºmm mºm-º ºmme cubic inches, according to the 20676 2671.3392 gals., Ans. principles jº PP- 376 and 396. NotE.—231 cancels .7854, giving a quotient of .0034. 6. How many quarts will a bucket hold, that is 10 inches deep, 12% inches in diameter at the top, and 94 inches at the bottom ? Ans. 16.2038 quarts. OPERATION, 124° = 156 + 16 || 5719 94* = 85% 3 || 10 124 × 94 = 115 § 34 4 357+'s º- | | 16.2038 quarts, Ans. 7. How many pints in a conical cup, 4 inches in diameter at the top, and 6 inches deep 3 Ans. .8704 pint. OPERATION. ; Explanation.—The contents of a cone being # of the 6 contents of a cylinder of equal height and diameter, we * proceed the same as in the measurement of a cylinder, and 3 || 2 then divide by 3. We multiply by 4 and 2 to reduce gallons mº mmmº to pints. 8704 pt., Ans 398 SOULE's PHILOSOPHIC PRACTICAL MATHEMATICS. T. 8. How many gallons will a tub hold which is 34 inches upper diameter, 29 inches lower diameter, and 21 inches deep 3 Ans. 70.9954 gallons. 9. How many gallons in a boiler or kettle, the lower part of which is in the form of a hemis- phere, with a diameter of 60 inches and a depth of 30 inches, and the upper part being in the form of a frustum of a cone, whose lower base is 60 inches, upper base 70 inches and the depth or height being 12 inches? Ans. 417.52 gallons. OPERATION OPERATION TO FIND THE CONTENTS OF THE TO FIND THE CONTENTS OF THE HEMISPHERE PORTION. FRUSTUM PORTION. 60 60 60° = 3600 3 || 12700 3 || 12700 60 60 70? – 4900 12 12 231 | .5236 or thus : .00223 70 × 60 = 4200 231 .7854 or thus: .0034 30 30 $º ºsmºsº tº ºmºmºmºsº 12700 — 172.72 gals. 172.72 gals. 244.8 gals. | 244.8 gals. 3 NOTE.—The decimal .00223 is the quotient of .5236 – 231; and the decimal .0034 is the quotient of .7854 – 231. Explanation.—See Article 772, page 378, and Article 768, page 375, for a full explanation for finding the contents of hemispheres and frustums of cones. The reason for dividung by 231 is that 231 cubic inches constitute a gallon. TO FIND THE NUMBER OF GALLONS IN A HORIZONTAL CYLINDER WEHEN TEIE DIAMETER AND THE LENGTH OF THE CYLINDER AND THE DEPTH OF THE LIQUID ARE GIVEN. PROBLEMS. 807. 1. How many gallons in a cylinder 18 feet 4 inches long, 6 feet diam- eter, the depth of the liquid being 12 inches Ans. 424.7917+ gals. 18° 4". | 2. ! i s | le i e-T-7– : ~7 OPEIRATION. First Step.–To find length of chord: Diameter 72 in. — 12 in. depth of liquid = 60 × 12 in. depth of liquid = 720. V 720 = 26.832 + in. # of chord x 2 = 53.664-i- in. chord, or width of liquid surface. NOTE.-See page 356, for the operation to extract the square root of 720. * PRACTICAL PROBLEMS IN THE MENSURATION OF SOLIDS. 399 Second Step.–Depth of liquid: 12 in. -- the diameter, 72 in. = .16663 = tabular number. Third Step.–See Table, page 353, and find the quotient .166 in the column of Versed sines. Then take the area of segment noted in the next column and multi- ply it by the Square of the diameter, the result will be the area of the segment of the cylinder. Thus: The tabular area for .166 = .08554 The tabular area for .167 = .08629 Difference is .00075 .00075 × .63 or .66+ = .00050. Then .166 = .08554 And .00063 = .00050 .08604 = the sum by which the square of the diameter is to be multiplied. Fourth Step.–.08604 × 72* = 446.0313 + sq. in. in area of segment. Fifth Step.—To find gallons: 446.0313 x 220 in. length of cylinder. 231 cu. in. in...a gallon. = 424.7917+ gallons. NOTE: 1.--To find the area of a segment greater than a semi-circle, find the area of the lesser segment, and subtract the same from the area of the whole circle. NOTE 2.—To find the area of a zone of a circle, find the areas of the two segments and sub- tract the sum from the area of the whole circle. 2. How many gallons in a tank of the following description and dimen- sions: The upper portion is in the form a vertical rectangular prism, 13 feet 5 inches deep, 11 feet 9 inches long, and 9 feet 3 inches wide. The bottom is a segment of a horizontal cylinder, the chord being 9 feet 3 inches, and the height or versed sine 4 feet 3 Ans. 13372.404 gallons. 41 Feet 9. _56,ºrºC - *. A *#. B : : : º OPERATION > -º : D First Step.—To find the gallons in the º ... • *** cubical part of the tank: .# +II" TB A. 161 in. deep. A ſ Y. 141 in. long. 32; f{ 111 in. wide. section between top 6) 760 and bottom. -*- 418 bushels. 1263 sq. ft. average area of crib. 1263 × 11 ft. height -- 3% = 418 bushels. This crib is in the form of a frustum of a pyramid. NotE 1.-See Article 791, page 387, for an explanation of the solution of Frustums of Pyramidal Solids. Also see page 373. NOTE: 2. —It is found by experiment that from 3% to 33 cubic feet of corn in the shuck is required to give one bushel of shelled corn. PRACTICAL PROBLEMS IN THE MENSURATION OF SOLIDS. 4. I 3 3. A corn crib, filled with corn on the ear, is 20 × 8 feet at the top, and 16 × 6 feet at the bottom and 11 feet high. Allowing two even bushels of corn on the ear to make one bushel of shelled corn, how many bushels of shelled corn are there in the Crib 7 Ans. 6963 bushels. NOTE 1.-The form of this crib is a frustum of a pyramid. NOTE 2. —For accurate methods of finding the number of bushels or gallons in boxes, bins, tanks, etc., see page-394. NOTE: 3.--Most corn cribs have parallel and vertical sides and ends. In all such cases, mul- tiply the length, width, and height together in the unit of feet, thus producing cubic feet. Then divide by 2 or 2% as above explained. * MEASUREMENT OF GRAIN WHEN IN REGULAR HEAPS. IPROTRLEMI. sº 825. A farmer has a heap of grain in a conical form, the diameter of which is 14 feet 4 inches and the depth 5 feet 3 inches; how many bushels does it contain 3 Ans. 226.906 bu. OPERATION. 172 Explanation.—The first part of this operation is the 172 same as for a cylinder, and then we divide by 3 for the 2150.4 || 63 reason that the contents of a cone are # of a cylinder of ſº pºw equal height and diameter. Had the grain been heaped .7854 against the side of a wall or building, we would divide 3 the result of this operation by 2, because in such a case the heap would be but # a cone. Had it been placed in a 226.906 b A right angled corner, we would divide the result of this * * **, operation by 4, for the reason that the pile would then be + of a cone. GAUGING BARRELS, CASKS, ETC. 826. Gauging is a method of finding the contents, or cubical capacities of casks, barrels, etc. In practice, the measurement or gauging of casks and barrels is generally performed by means of instruments known as diagonal and ullage or “wanting” rods. THE USE OF THE DIAGONAL AND UILLAGE IRODS. 827. To gauge barrels or casks with these rods, place the diagonal rod into the bung and extend the pointed end to the lower side of each end of the barrel or cask, and thus find the average or mean diagonal distance; then on the side of the rod at the point of the mean diagonal distance, will be found the number repre- senting the full capacity of the vessel in gallons. If the barrel or cask is not full, then the ullage or “wanting” rod is inserted vertically into the bung, and at the highest point where the liquid touches this rod will be found a number 4. I4. soulE's PHILOSOPHIC PRACTICAL MATHEMATICs. * representing the number of gallons “wanting” to make the barrel full. The difference between the full capacity of the barrel, as shown by the first rod, and the gallons “wanting” will be the contents of the barrel or cask. The diagonal rod may be prepared by finding the bung diagonal of a one gallon barrel of medium curve, which is 7.15+ inches, and then to find the diagonal number for each successive number of gallons, multiply the 7.15+ inches by the cube root of the successive numbers, 2, 3, 4, 5, etc., of gallons. Thus, to find the diagonal number of inches for 8 gallons, we proceed as follows : V8–2; then 7.15+ × 2 = 14.3+ inches. There is no way of finding the exact capacity of casks, barrels, etc., for the reason that they are not constructed in exact conformity with any regular mathe- matical figure. The curvature of the staves, for nearly all kinds of casks and barrels, is slightly different, and hence all measurements of their contents are only approximately correct. * The capacity of a cask is equivalent to that of a cylinder having the same length and a diameter equal to the mean diameter of the cask. To find the contents of a cask or barrel without the aid of the diagonal rods, we first measure the length and the head and bung, or center diameters, inside measure. Then, having the two diameters, we approximate the mean diameter according to the quantity of curve, as follows: When the curve is but slight, we add 4 or .5 of the difference between the two diameters to the lesser dia- meter; when the staves have a medium curve, we add .6, and when they are above a medium curve, we add 65 or .7. Having thus produced the average or mean diameter, we proceed the same as in the measurement of a cylinder. A barrel with slightly A barrel with medium A barrel with above curved staves. curved staves. medium curved Staves. FROBLEMS. 1. A barrel is 26 inches long, 17 inches in diameter at the head, and 20 inches in diameter at the bung or center. The staves have a medium curve. How many gallons will it hold 3 Ans. 31.244-H gallons. OPERATION. 20 in., center diameter. 18.8 18.8 17 in., head diameter. * 18.8 18.8 sº .7854 or thus: .0034. 3 in., difference. 231 || 26 26 .6 adjustment for curve in staves. tº-º-º-e 31.244096 gals., Ans. 31.244096 gals. 1.8 = .6 of the difference. 17 head diameter added. 18.8 average or mean diameter. NotE.—By the use of the gauging rods, a barrel of the above dimensions gauged 31-F gallons. * PRACTICAL PROBLEMS IN THE MENSURATION OF SOLIDS. 415 2. The inside measurements of a certain barrel are as follows: Length, from chime to chime, 30 inches; bung diameter, 22.8 inches; head diameter, 19.5 inches. How many gallons will it contain by the mathematical method of gauging? º Ans. 47.0618-- gallons. OPERATION. 22.8' = bung diameter. 21.48 21.48 19.5” = head diameter. 21.48 | 21.48 º-msm-mºs 231 .7854 or thus: .0034 3.3" = difference, 30 30 ° .6 -º-º: º- *== 47.0618-1- gals. 47.0618–H 1.98" = .6 of the difference. gallons. 19.5" = head diameter. 21.48’’ = average or mean diameter. COMPARISON OF MEASUREMENTS. 828. A barrel of the above dimensions was gauged by the gauging rods and gave a capacity of 47+ gallons. The same barrel, when measured by a gallon measure, held 47 gallons and 1 gill. When the same barrel was measured with a head diameter of 19.4 inches, the other dimensions being the same as above, it showed a capacity of 46.8867–H gallons. 1. A cask is 40 inches long, 31% inches head diameter, and 38% inches bung diameter. What is its capacity in wine and beer gallons? Ans. 181.93–H gallons wine. 149.02+ gallons beer. OPERATION. * 38# in., bung diameter. 38.75 31; in., head diameter. 31.50 amºmºmº OI’ 74 in., difference. 7 7.25 .7 .7 5.0% = .7 of the difference. 5.075 31.5 head diameter added. 31.5 36.575 average or mean diameter. 36.575 OPERATION OPERATION TO FIND WINE GALLONS. TO FIND BEER GALLONS. 36.575 36.575 36.575 36.575 .7854 .7854 231 40 282 | 40 181.9313+ gals., Ans. 149.0288-|- gals., Ans. 416 SOULE's PHILOSOPHIC PRACTICAL MATHEMATICs, * TONNAGE OF WESSELS. 829. The Tonnage of a vessel is the number of tons weight it will carry with safety. º The carrying capacity of vessels, that is, the amount of space available for stowing away the cargo is reckoned in tons. . The adopted measurement is one hundred cubic feet of space to a ton. This measurement does not include the deck space, where part of the cargo is sometimes carried. The registered tonnage is always less than the actual tonnage. To determine accurately the tonnage of a vessel or steamboat, is a very difficult problem. The methods in practical use do not give accurate results. The best method probably would be to divide the vessel into several sections, and then, by numerous measurements, find the average length, width and depth of each sec- tion, and having by this means found the contents in cubic feet, the actual tonnage is found by dividing the cubic feet by 40. The following is the United States method of finding the tonnage of vessels: Measure in feet, above the upper deck, the length of the vessel, from the forepart of the main stem to the afterpart of the sternpost. Then measure the breadth taken at the widest part above the main wale, on the outside, and the depth from the under-side of the deck-plank to the ceiling in the hold. Then, from the length take three-fifths of the breadth, and multiply the remainder by the breadth and depth, and the product divided by 95 will give the tonnage of a single-decker. If the vessel is double-decked, substitute half of the breadth for the depth, and proceed as directed for single-decked vessels. * The following is the carpenters method of measuring the tonnage of vessels: Measure the length, breadth, and depth, all in feet. Divide the continued product of the dimensions by 95, and the quotient will be the tonnage. If the vessel is double-decked, half the breadth is taken as the depth. NOTE.-For more extended work on the Tonnage of Vessels and Steamboats, see Haswell’s Book for Engineers and Mechanics, or the Merchants' and Shipmasters' Manual, or the Cyclopedia Britannica or the Century Dictionary. DRILL EXERCISE IN MENSURATION OF SOLIDS. 417 DRILL EXERCISE IN MENSURATION OF SOLIDS, —-4– 880. Statements showing the capacity in gallons of each of the following fig- ures. The dimensions of each are given above, and the statement to obtain gallons is given below each figure: Height 6 feet. IIeight 6 feet. º 6 feet. Height 6 feet. º i ; lij º § | | | § | º Diameter 40 in. Diameter of base Upper Diam. 30 in. Each side of base 40 inches. Lower Diam. 40 in. 40 inches. # Gals. ~ - 3 || 3700 302 = 900 231 |.7854 40? – 1600 72 40 × 30 = 1200 *m. 231 | .7854 3700 72 3 sº*i. - Tranºlam. * All the sides I) ia meter eter 6 ft. Transversel)iam 6ft. - 30 inches. g e d g * : A ſh & Side of lower base 40 inches. 40 inches. Conjugate do 40 in Conjugate 40 in. 40 inches. V- #: Fº iſſiº Gals. 72 302 = 900 72 40? – 1600 * 231 | .5236 40 × 30 = 1200 40 3700 3 Diameter 60 in. Depth 30 in. Diameter 60 in. Depth 40 in. l)iameter 60 in. Depth 20 in. & O " 's ** 60 – 2 = 30 30° X 3 – 2700 Gals. 2700 + 20% = 3100 60 Gals. 231 || 60 60 Gals. .5236 231 || 60 3100 30 .5236 231 20 mºmsºmºmºsºs 40 | .5236 418 SouLE's PHILOSOPHIC PRACTICAL MATHEMATICS. A. See Problem and Solution, page 398. Length 60 in. #. Diam. 40 in. IH Diam. 32 in. A M- : D * See Article 781. 40 – 32 = 8 & is: = #'s See Article 783. See Article 782. See Article 785. 37.62 × 60 × .0034 = . . . . gallons. NotE.—To find the capacity in bushels of the above figures, divide by 2150.4 or 2150.42, instead of 231. To find the number of cubic feet instead of gallons, divide by 12, 12, and 12 or 1728, instead of 231. 4% MECHANICAL POWERS. 4 IQ MECHANICAL POWERS, —º- 831. The Mechanical Powers consist of six simple machines, or the elements of machinery, by the scientific use of which we are enabled with a given force and velocity, to accumulate a greater force with less velocity, or a less force with a greater velocity; or in other words, to change either the direction or the inten- sity of a force, or both direction and intensity. These Powers are called the Lever, the Wheel and Aacle, the Pulley, the Inclined Plane, the Wedge, and the Screw. To fully understand the nature of a machine, four things must be considered, viz.: 1st, The force or power which acts on, or is applied to the machine; 2d, The resistance or weight which is to be overcome or moved by the power; 3d, The center of motion, called the Fulcrum, which is the point over which the power and resist- ance act; and 4th, The respective velocities of the power and the resistance. A machine is in equilibrium when the power and the resistance are equal; in which case it is either at rest or in a state of uniform motion. TEIE LEVER, 832. The Lever is a rod or bar of metal, wood, or other material, which is capable of moving around the fulcrum. Levers may be either straight or bent; simple or compound. They are usually divided into three classes, according to the position of the fulcrum in rela- tion to the weight and power. FIGURE 1, is a lever of the first class, where the fulcrum is between the weight and the power. FIGURE 2, is a lever of the second class, where the weight is between the fulcrum and the power. FIGURE 3, is a lever of the third class, where the power is between the fulcrum and the weight. 42O SOULE's PHILOSOPHIC PRACTICAL MATHEMATICS. X. The Arms of a lever are the lines on each side of the fulcrum, at right angles to the direction of the power and the weight. In the three figures given above, the levers being horizontal and the forces vertical, the arms of the lever are evidently, in each case, the portions into which it is divided. If however, the lever is bent or is inclined to the direction of either, 4 B or both, of the forces, then the arms are the perpen- 2^ diculars between the fulcrum and the directions of the X forces. Thus, in FIGURE 4, the power acting in the \ direction B P, the momentum or force of the power is \ not expressed by P × A F, but by P × B F. The (º) P distance from the fulcrum is called the leverage. When the forces act perpendicularly to the arms of a straight lever, an equilibrium is produced when the weight multiplied by its distance from the ful- crum is equal to the power multiplied by its distance from the fulcrum; or, in other words, the weight is to the power, as the distance from the power to the fulcrum is to the distance from the weight to the fulcrum. # Application of the Lever.—Machines and utensils in daily use furnish us familiar examples of the three classes of levers: Of the first class, we name the crowbar and poker, when used to raise the load or weight on their points; scissors, snuffers and pincers are pairs of levers of this class. The point C, FIG- URE 5, which connects them being the fulcrum. The power is applied at the handles, and the resistance is the object between the blades. Another example of the bent lever is seen in the ordinary truck, FIGURE 6, used for moving heavy goods a short distance. In this machine, the axis of the wheels, F, is the fulcrum, against which the foot is placed, while the weight, at R, is raised off the ground by the hand, applied at P. The Scale Beam, or balance, is one of the most useful applications of the first class of levers. The beam is a lever poised at its centre on a knife-edge of steel, a, FIG. 7. From its ends, A B, are suspended the scale pans, C and E. The Center of gravity, m, is placed below the fulcrum, a, to secure a horizontal position of the beam when in equilibrium. If it coincided with the fulcrum, the balance Would rest equally well in all positions, and if it were above the 5ulcrum the beam Would be upset by a slight disturbance. ¥ MECHANICAL POWERS. 42 I The Steelyard is a lever of the first class, with unequal arms. The mass, Q, to be weighed, is attached to the short arm, A, FIGURE 8, and it is counterpoised by a constant weight, G, shifted upon the longer arm, marked with notches to indicate pounds and ounces, until equilibrium is obtained. It is evident that a pound weight at G will balance as many pounds at Q as the distance G C is greater than A. C. Levers of the second class occur less frequent- ly. A pair of nut-crackers with the fulcrum at the joint C, FIGURE 9, is a double lever of this class. An oar is another example; the water is the fulcrum, the boat is the weight, and the hand the power. A door moving on its hinges, and a wheelbarrow, are other examples of levers of the second class. In levers of the third class, the power, being nearer the fulcrum, is always greater than the weight. On account of this mechanical disadvantage, it is used only when considerable velocity is required, or the resistance is small. The common fire-tongs, sugar-tongs, and 10 sheep-shears are double levers of this class. The most striking illustrations of this class of levers are seen in the animal kingdom. The compact form and beautiful symmetry of ani- mals depend on the fact that their limbs are such levers. The socket of the bone, a, FIG- URE 10, is the fulcrum ; a strong muscle, b c, attached near the socket, is the power, and the weight of the limb and whatever resistance, w, may oppose to motion, is the weight. The fore-arm and hand are raised through a space of one foot, by the contraction of a muscle applied near the elbow, moving through less than 1-12 that space. The muscle, therefore, exerts 12 times the force with which the hand moves. The muscular system is the exact inversion of the system of rigging a ship. The yards are moved through small spaces with great force, by hauling in a great length of rope with small force; but the limbs are moved through great spaces with compar- atively little force, by the contraction of muscles through small spaces with very great force. EXAMPLES. 1. In a lever of the first kind, the fulcrum is placed 10 inches from the weight, the long arm is 8 feet; if a power of 150 pounds be applied at the end of the long arm, what weight will it balance at the end of the short armº Ans. 1440 pounds. OPERATION. Eaplanation.—Since the product of the 150 lbs., power. weight by the distance it is from the fulcrum, 96 in. ión g arm. 150 is equal to the product of the power by the 7 or, 10 | 96 distance it is from the fulcrum, we therefore I fulcrum together, and divide the product by 1440 lbs., . . length of the short arm, and obtain the 1440 lbs., Ans. AnS. required weight. *=ºmºmº 10) 14400 | multiply the power and its distance from the 422 SOULE's PHILOSOPHIC PRACTICAL MATHEMATICs. * 2. In a lever of the second kind, if a weight of 1200 pounds is placed 2 feet from the end resting upon the fulcrum, what power will it require to sustain this weight, the length of the lever being 12 feet 8 inches 3 Ans. 1894%; lbs. OPERATION. 1200 lbs., weight. 1200 24 in., distance of weight from fulcrum. Or, 152 24 28800 product, which -- 152 (inches length of lever) gives | 1894% lbs., the required power. 3. In a lever of the third kind, what power will it require to sustain a weight of 80 pounds, the distance from the fulcrum to the power being 4 feet 4 inches, and from the fulcrum to the weight 6 feet? Ans. 110+} lbs. OPERATION. 80 lbs., weight. 80 72 in., length of lever. Or, 52 | 72 5760 product, which -- 52 gives 110}} lbs., the required power. 110+} Ans. 4. If the orifice of the safety-valve of a steam boiler is 2 inches in diameter and 5 inches from the fulcrum of the lever, how far from the orifice must, a weight of 60 pounds be placed so as to just equal the force of the steam when at a press- ure of 80 pounds to the square inch, making no allowance for the concave surface of the boiler or weight of lever ? Ans. 15.944 in. OPERATION. 2” x .7854 = 3.1416 sq. in. on surface of safety-valve. 80 lbs. pressure to the square inch. 251.3280 pressure on safety-valve. 5 in. distance from fulcrum to the orifice. 69 ) 1256.640ſ, product -- the weight gives 20.94.40 in. distance of weight from the fulcrum. 5 in. distance from fulcrum to orifice deducted. 15.944 in. distance of the weight from the orifice. TEIE WEHEEL AND AXLE. 833. The common lever is chiefly employed to raise weights through small Spaces, by a succession of short intermitting efforts. After the weight has been raised, it must be supported in its new position until the lever can be again adjusted, to repeat the action. The wheel and axle is a modification of the lever, which corrects this defect; and, since it converts the intermitting action of the lever into a con- tinuous motion, it is sometimes called the perpet- wal lever. MECHANICAL POWERS. 423 This machine consists of a cylinder called the axle, turning on a centre, and Connected with a wheel of much greater diameter. The power is applied to the circumference of the wheel, and the weight is attached to a rope wound around the axle in a contrary direc- tion. Instead of the whole wheel, the power may be applied to a handle named a winch, or to one or more spokes inserted in the axle. When the axle is horizontal, the machine is called a Windlass, FIGURE 11, when it is vertical it forms the capstan FIGURE 12, used on shipboard, chiefly to raise the anchor. The head of the capstan is pierced with holes, in each of which a lever can be placed, so that many men can work at the same time, exerting a great force, as is often necessary in raising an anchor, while the recoil of the weight is arrested by a catch at the bottom. The law of equilibrium is the same as in the lever. Draw from the center, or fulcrum c, FIGURE 13, the straight lines c b and c a, or c a', to the points on which the weight and power act; a c b, or aſ c b is evidently a lever of the first class, in which the short arm c b is the radius of the axle, and c a, or c a', the long arm, is the radius of the wheel. Hence, P × a c = W X c b. P: W = c b : a c. Or, That is to say, the wheel and aale are in equilibrium, when the power is to the weight as the radius of the aarle is to the radius of the wheel. In one revolution of the machine, the power moves through a space equal to the circumference of the wheel, and the weight moves through a space equal to the circumference of the axle; hence the power and weight are inversely as their velocities, or the spaces they describe. PROBLEMS, 1. What must be the radius of a wheel, or the length of a crank, by which 500 pounds suspended from an axle of 4 inches radius may be kept in balance by a power of 40 pounds? Ans. 4 feet 2 inches. OPERATION. 4 inches radius. 4 500 pounds weight. 40 || 500 Or, - tº 49 ) 2009 product -- 40 the power. 50 in. = 4 ft. 2 in., Ans. 50 in. = 4 ft. 2 in., radius of wheel or length of crank. Explanation.—The operation of this work is based upon the principle above elucidated, that the power is to the weight as the radius of the axle is to the radius of the wheel or the length of the crank. 424 SOULE's PHILOSOPHIC PRACTICAL MATHEMATICs. * OPERATION. 2. What must be the diameter of 28 in., length of crank. an axle in order that a power of 150 150 pounds power. pounds, applied at the end of a crank 28 inches long, measuring from the center of the axle, will balance 800 pounds? 54 in., radius of axle. Ans. 10% inches. 2 8% ) 429% product -- the weight. 104 in., diameter of axle. 3. If the anchor of a vessel weighs 3000 pounds, what force will be required from each of 5 sailors to raise the same, working at the capstan with bars 54 feet long, the diameter of the capstan being 16 inches, allowing that each bar enters the capstan 4 inches 3 Ans. 684 pounds. OPERATION. 5 ft. 6 in. length of bar. 8 in. radius of capstan. 6 ft. 2 in. 4 in. distance the bar enters the capstan. 5 ft. 10 in. = 70 in., length of bar from the center of capstan. 16 -- 2 = 8 in., radius of capstan. 3000 lbs., weight of anchor. 79 ) 2400% product -- the length of bar. 5) 3424 lbs., power required -- 5, the number of sailors. 684 lbs., force required of each sailor. THE PULLEY. 834. Fixed Pulley.—The usual form of this machine is a 14 small wheel, turning on its axis, and having a groove on its edge, to admit a flexible rope or chain. In the simple fixed pulley, FIGURE 14, there is no mechanical advantage, except that which may arise from changing the direction of the N power. Whatever force is exerted at P, is transmitted, with- b out increase or diminution (except from friction and the º rigidity of the rope), to the resistance at the other end of the cord. From the axis C, draw C a and C b, radii of the wheel, at right angles to the direction of the forces; a C b represents a lever of the first class, with equal arms; hence, in equilibri- wm, the power and weight must be equal, and they describe equal 8pacé8. Wy * MECHANICAL POWERS. 425 Movable Pulley.—When the block or frame is not fixed, TN 15 X the pulley is said to be movable. The weight is suspended D F from the axis of the movable pulley, and the cord is fastened at one end, and passing over a fixed pulley, is acted on by the power at the other. In this arrangement, FIGURE 15, it is plain that the weight is supported equally by the power and the beam at D. For the pulley acts as a lever of the second class, whose arms are to each other as 1 : 2; the fulcrum is at b, b c is the leverage of the weight, and b a the leverage of the power. The diameter b a is twice the radius b c, therefore equilibrium will obtain when the power is equal to one-half of the weight: i. e., b{−Pi—a P: W = b c : b a = 1 : 2, therefore, W P = + To raise the weight one foot, each side of the cord must be shortened one foot, and the power, consequently, passes over two feet. The space traversed by the power is twice the space described by the weight. Compound Pulleys.-Sometimes compound pulleys are used, each consisting of a block which contains two or more single pulleys, generally placed side by side, in separate mortises of the block. Such an arrangement is shown in FIGURE 16. The weight is attached to the movable block, and the fixed one only serves to give the power the required direction. It is easily seen that the power required at P is just the same as would be required at any point between A and B. The weight is divided equally among the pulleys of the movable block, and, of course, among the cords passing around them ; and as the power required to sustain a given weight is diminished one-half by a single movable pulley, it follows that such a system will be in equilibrium when the power is equal to the weight divided by the number of cords, or by twice the number of movable pulleys. P: W = 1 : 2n ; or, P = ". 2n In this system, as in the single movable pulley, the Space through which the weight is raised, is as much less than the space through which the power descends, as the weight is greater than the power. e *— P: W = velocity of weight : velocity of power. If the power is moved through 6 feet, fig. 16, each division of the cord in which the movable block hangs will be shortened one foot, and the weight raised one foot. 426 SouLE's PHILOSOPHIC PRACTICAL MATHEMATICs. X. Another system of pulleys is represented in FIGURE 17. In this arrangement each pulley hangs by a separate cord, one end of which is attached to a fixed support, and the other to the adjacent pulley. The effect of the power is rapidly augmented, being doubled by each movable pulley added to the system. The numbers placed near the cords show what part of the weight is sustained by each, and by each pulley. Such a system, however, is of little practical use, on account of its limited range. In the common block system, fig. 16 (in practice the pulleys or sheaves of each block are placed side by side, to save room, here they are separated for sake of clearness), the motion may continue until the movable block touches the fixed one; but in this only till D and E come together, at which time A will have been raised only g of fhat distance. P; W = 1 + 2*, or, P = } PROBLEMIS. 1. What power will support a weight of 1200 pounds in two movable pulleys? Ans. 300 pounds. OPERATION. 1200 weight + 4, twice the number of movable pulleys, = 300 lbs., Ans. 2. In a compound pulley (see fig. 16), containing three single pulleys in the movable and immovable blocks, what power will support 3000 pounds? Ans. 500 pounds. OPERATION. 3000 - 6 = 500 pounds, Ans. 3. In a block and tackle with 4 sheaves in the fixed block and 3 in the movable block, what power will be required to support 5000 pounds 3 Ans. 714% pounds. OPERATION. 5000 -- 7 = 7.14% pounds, Ans. NotE.—The pulleys in a block and tackle are called sheaves. ‘ſk MECHANICAL POWERS. 427 THE INCLINED PLANE. 835. This mechanical power is commonly used whenever heavy loads, especially such as may be rolled, are to be raised a moderate height. In this way casks are moved in and out of cellars, and loaded upon carts. The common dray is itself an incline plane (as is 18 Clearly seen by inspecting FIG- URE 18). Suppose a cask weigh- ing 500 lbs. is to be raised 4 feet by means of a plank 12 feet long; it is plain, that while the cask ascends only 4 feet, the Mºtſº power must exert itself through Wºº tº:- 12 feet, and hence, 12:4 = 500 : 1663, the force necessary to roll the cask. In mechanics, the inclined plane is a hard, Smooth surface, inclined obliquely to the resistance. The length of the plane is R S, figure 19, S T its height, and R. T its base. The power may be applied, a—In a direction parallel to the length; b—Or parallel to the base. c—Or in any other direction. In each case the conditions of equilibrium may be derived from those of the lever. Application of the power parallel to the length of the inclined plane.— When a body is placed upon an inclined plane, FIGURE 19, its weight, which is the 19 resistance to be overcome, acts in the direc- tion of the force of gravity, namely in the perpendicular b a. Let the power, P, act, by means of the cord, in the direction a c par- allel to the inclined plane R S, then from the point a, draw a d at right angles with the in- clined plane, and complete the parallelogram a c b d. The force of gravity will be resolved into two other forces; one represented by b c causing pressure on the inclined plane; the other, represented by c a, tending to cause motion down the inclined plane. This latter force is to be balanced by the power applied to move the body. The body will therefore be sustained when P: W = a c : a b, P: W = S T : B S; OT and since the triangles a b c and R S T are similar, y P = W x ST R.S. This may be illustrated by an apparatus constructed like that shown in the figure. 428 soul E's PHILOSOPHIC PRACTICAL MATHEMATICs. * If the direction of the power is parallel to the inclined plane, equilibrium will obtain when the power is to the weight as the height of the plane is to its length. While the weight is raised through a space equal to the vertical height of the plane, the power must move through a space equal to its length. If the length of a plane is 10 feet, and its height 2 feet, P must move 10 feet while W is raised 2 feet; hence the power and weight are inversely as their velocities. Application of the power parallel to the base of the inclined plane.—In the second case, let the power act in the direction of a 20 P, FIGURE 20, parallel to B C, the base of the plane; and draw the lines b a and b c perpendicular to the di- rection of the power and weight; then a b c is a bent lever, having its fulcrum at b, and equilibrium will take place when P: W = b c : a b, P: W = A B : B C; OI’ & the triangles a b c and A B O being similar, } P = W X A B tºmº B C. If the direction of the power is parallel to the base of the plane, equilibrium will obtain when the power is to the weight as the height of the plane is to the length of its base. k In this case, the space described by the power is to the space described by the weight as the base of the plane is to its height. PROBLEMIS. 1. The length of an inclined plane is 10 feet and its height 3 feet, what power applied parallel to the plane will be required to balance a weight of 500 pounds? Ans. 150 pounds. OPERATION, 500 pounds, weight. 500 3 feet, height. Or, 10 || 3 1500 product -- the length of the plane, 10 ft., gives 150 | *=. lbs., the power required. - 2. The height of an inclined plane is 12 feet and the length 15 feet, what weight applied parallel to the plane will support a power of 150 pounds? Ans. 1873 pounds. OPERATION. 150 pounds, power. - 150 15 feet, length of plane. or, 19 || 15 2250 product -- the height, 12 feet, gives 1874 lbs., the | 1874 *- -- ** required weight. MECHANICAL POWERS. 4.29 3. The base of an inclined plane is 12 feet and the height 4 feet, what power applied parallel to the base of the inclined plane will support a weight of 500 pounds? Ans. 1663 pounds. OPERATION. 500 pounds, weight. 500 4 feet, height. Or, 12 | 4 or, 500 x # 2000 product -- the length of the base, 12 feet, 166; gives 1663 lbs., the power required. TEIE WEDGE. 836. Instead of lifting a load by moving it along an inclined plane, the same result may be obtained by moving the plane under the load. When used in this manner, the inclined plane is called a wedge. It is custom- 21 ary, however, to join two planes base to base. In FIGURE 21, A B is called the back of the wedge, A C and B C its sides, and d C its length. The power is applied to the back of the wedge, so as to drive it between two bodies, and overcome their resistance. The resistance may act at right angles to the length or to the sides of the wedge. In the first case, it resembles an inclined plane, when the power is parallel to the base; and hence the forces will be in equilibrium when the power is to the resistance as the back of the wedge is to its length. In the second case, it is similar to a plane when the power is parallel to the length; and therefore in equilibrium when the power is to the resistance as the back of the wedge is to its side. - If the wedge is isosceles, the power is to the weight as half the back of the wedge is to one of its sides. The power is supposed to move through a space equal to the length of the wedge, while the resistance yields to the extent of its breadth. Application of the wedge.—As a mechanical power, the wedge is used only where great force is to be exerted in a limited space. In oil-mills, the seeds from which vegetable oils are obtained are sometimes compressed with enormous force by means of a wedge. It is everywhere employed to split masses of stone and timber. The edges of all cutting tools, as saws, knives, chisels, razors, shears, etc., and the points of piercing instruments, as awls, nails, pins, needles, etc., are modified wedges. Chisels intended to cut wood, have their edge at an angle of about 30 deg.; for cutting brass, from 50 deg. to 60 deg.; and for iron, about 80 deg. to 90 deg. The softer or more yielding the substance to be divided the more acute the wedge may be constructed. In general, tools which are urged by press- ure, admit of being sharper than those which are driven by a blow. 43O SOULE's PHILOSOPHIC PRACTICAL MATHEMATICs. jºr The theory of the wedge gives but very little aid in estimating its effects, as it takes no account of friction, which so largely modifies the results, and the pro- portion between pressure and a blow cannot be defined. PROBLEMS. 1. The head of a wedge is 2 inches thick and the length of each inclined side is 8 inches; what will be the measure of its effect if struck by a force equal to 200 pounds? Ans. 1600 pounds. OPERATION. 200 pounds force. 8 inches, length of inclined side. 1600 product + 1, which is ; the thickness of the head of the wedge, gives 1600 pounds, the measure of the effect required. 2. The head of a wedge is 4 inches thick and the length of each inclined side is 15 inches; what pressure, applied to the head of the wedge, is required to drive it between two surfaces, the resisting force of which is equal to 300 pounds? Ans. 40 pounds. OPERATION. 300 pounds, resisting force. 2 inches, # of the thickness of the wedge. 600 product -- 15, the length of the inclined side, gives 40 pounds, the pressure required. TEIE SCREW. 837. The screw is a spiral thread, or groove, winding round a cylinder, so as to cut all the lines drawn on its surface, parallel to its axis at the same angle. The screw has the same relation to the ordinary inclined plane, that a spiral staircase has to a straight one. The thread of a screw projects from the surface of the cylinder, and is de- signed to fit into a spiral groove, cut in the interior of a block called the nut; a lever is also fixed in the head of the cylinder to which the power is applied. The combination of these parts forms the mechanical power technically called the Screw. In working the screw, the resistance acts on the inclined face of the thread, and the power parallel to the base of the screw. This corresponds to the case in which the direction of the power is parallel to the base of the inclined plane. Yºr MECHANICAL POWERS. 43 I Equilibrium will, therefore, take place when the power is to the resistance as the dis- tance between the threads of the screw is to the circumference described by the power. P; W = h : 2 R. and P = W x sº- 2 Idir h being the distance between the threads, R the radius of the screw, or of the length of the lever attached to it, and the ratio of the circumference of a circle to its diameter. During each revolution the power describes a circle, whose circumference depends on the length of the lever, but the end of the screw advances only the distance between two threads; thus in this, as in all cases of the use of machines, What is gained in power is lost in velocity. Mechanical efficiency and applications of the screw.—The mechanical efficiency of the screw is augmented, either by increasing the length of the lever, or by lessening the distance between the threads. If the 22 threads of a screw are 3 of an inch apart, and the power à describes a circle of 5 feet (120 half-inches) circumfer- ence, a power of 1 pound will balance a resistance of 120 pounds; if the threads are # inch apart, 1 pound will balance 240 pounds, the efficiency being doubled. Fine screws are therefore more powerful than coarse ones. The applications of this most useful mechanical power are too numerous to mention, but no more fre- Quent or important example of its use can be named than is seen in its use in presses of nearly all kinds. A good illustration of which is seen in the copying press, FIGURE 22. - The endless screw is a contrivance by which a slow motion is imparted to a wheel, as shown in FIGURE 23. The threads of the screw act upon the cogs of the wheel, and serve to move the weight Q, attached to the axis M. L. If we call the radius of the circle described by the winch D B = r, and let h = the distance between the threads of the screw, we shall have the power of the screw O =#. Let R = MF, and R' = M L, l, and the power of the wheel and axle will = #. Then W: P = 2ar x R : h x R'; 2tr R E P -- tº w 4 × b. R/ Therefore the weight is to the power as the circumference of the circle described by the winch D, multiplied by the radius of the wheel, is to the distance between the threads of the screw multiplied by the radius of the axis. 4.32 SOULE's PHILOSOPHIC PRACTICAL MATHEMATICS. A. PROBLEMIS. e 1. The distance between the threads of a screw is # of an inch, and the circumference described by the handle, or lever, is 4 feet; what weight can be raised by a power of 250 pounds, applied at the end of the lever? Ans. 48000 pounds. OPERATION. 4 feet circumference of handle. 12 inches in 1 foot. As {{ {{ 4 4. 4 + “ “ 1 inch. 192 + “ “ 48 inches. 250 pounds power. 48000 “ Ans. 2. The distance between the pitch threads of a screw is # of an inch, and the length of the crank or lever is 16 inches, what weight can be raised by a power of 400 pounds applied at the extremity of the lever? Ans. 32.1699.84 pounds. * OPERATION. 1. inches length of lever 3 1ā; inches diameter of the circle described by the lever. .141 100.5312 inches circumference of the circle described by the lever, 8 * inches in 1 inch. 804.2496 inches in 100.5312 inches. 400 pounds power. 321699.8400 pounds, Ans. 3. The distance between the threads of a screw is # of an inch, the handle or lever is 10 feet long, what power will be required to sustain a weight of 50000 pounds? Ans. 26.5257+ pounds. OPERATION. 10 feet length of handle. 2 20 3.1416 50000 62.8320 10 12 2 Oſ. 3.1416 753.9840 12 5 – # in. 5 2 2) 3769.9200 = } in. 1884.96 = #in. 50000 -- 1884.96 = 26.5257+ pounds, Ans. MECHANICAL POWERS. 433 In the foregoing work on Mechanical Powers, much of the work, especially the introduction to the subjects treated, was taken from Silliman’s Principles of Physics, a book of great worth, and to which we refer for a more comprehensive discussion of the subject here briefly treated. RRºº as se a S$$$$$...s §§ § Nº. º º Ž % Ş N sº § § w V)2 2,…º : 22 2% º ye ę §§ sº sº *ść 㺠ºğ SRS S$ º ſº º 838. In the above diagram of wheels, A is a pinion with 14 teeth and makes 50 revolutions per minute. B is a spur wheel with 42 teeth and is connected with pinion A. C is a pinion with 28 teeth and is carried by the shaft of wheel B. D is a spur wheel with 104 teeth and is engaged with pinion C. E is a pinion with 24 teeth and is carried by engaging wheel B. F is a spur wheel with 112 teeth engaged with pinion E. A is the engine shaft pinion at 50 revolutions per minute and is therefore the pinion motor. What is the cycle of the whole double train in time, and how many revolutions will each wheel make during the cycle? Ans. 10% min. cycle of the whole train. A makes 546 revolutions. B, C and E each make 182 revolutions. D makes 49 revolutions. F makes 39 revolutions. OPERATION. A makes 50 revolutions per minute. B #4 × 50 = ** revolutions per minute. NOTE.-C and E have the same time of revolution as B. D # x * = ** revolutions per minute. F #4, x * = ** é & {{ & 4 Since A makes 50 revolutions in one minute, it will make 1 revolution in ºr minute; in like manner B, D and F will make one revolution in ºr, ſº and minutes respectively. The L. C. M. of sº, º, ºr and # = ** or 10# min., Ans, to 1st, question. Since A makes 1 revolution in ºf min., in 10# min., it will make as many revolutions as 10# is equal to gº which is 546 times; in like manner B (also C and E), D and F will make respectively 182, 49 and 39 revolutions during the cycle. NotE.—See Problem 5, page 215. ...” 839. Per Cent is derived from the Latin phrase per centum, and means literally on or of the hundred. Or more briefly, the hundredth. 840. The Sign of Per Cent is %. Thus 5% is read 5 per cent. 841. Percentage is a term applied to all calculations in which 100 is used as the unit of measure or the basis of comparison. It may be applied to all tran- sactions of business, to all departments of science, and to every thing in nature. It is in general use in almost every department of practical life. Thus we say 20% gain or loss; 10% of the number or quantity; 25% better , or lower quality; 20 is 80% of 25; 30% increase or decrease of the products of field, mine, or factory; the general won 75% of the battles fought and lost 45% of his army; the illiteracy of the State is 10% of the population; the scholarship of the student is 98%, etc. - 842. In the application or the practical computation of Percentage, the following elements are considered, viz: the Base, the Rate, the Percentage, and the A mount or Difference. 843. The Base is the number on which the per cent is calculated. Thus, in the expression, “What is 5% of 70 ?” the base is 70. 844. The Rate is the number of hundredths to be taken. Thus, in the question, “What is 5% of 70 ?” the rate is 5. 845. The Percentage is the result obtained by taking the given per cent of the base. Thus, in the statement, 5% of 70 is 3.50, the percentage is 3.50. 846. The Amount is the sum of the base and the percentage. Thus, if the base is 70, and the percentage is 3.5, the amount is 70 + 3.5 = 73.5, 847. The Difference is the remainder obtained by subtracting the percen- tage from the base. Thus, if the base is 70 and the percentage is 3.5, the difference is 70 — 3.5 = 66.5. NOTE.-The relationship between the above elements or terms is such, that when any two of them are given, the others can be found. In all problems of per cent, two of these elements must be stated. 848. Since per cent means on the hundred, it is clear that 1% of any number or thing is the hundredth part of it; and if 1% is the hundredth part, then 2%, 3%, 10%, 25%, etc., are 2, 3, 10, 25, etc., times as many or as much. Thus, 1% of $400 is $4, and 2% would be twice as much, which is $8; and (434) º: PERCENTAGE. 435 1% of 200 pounds would be 2 pounds, and 10% would be 10 times as much, which is 20 pounds. Per cent may be expressed as a common or a decimal fraction. Thus, 2% = +$5, or .02; 3% = rig, or .03; 25% = **, or .25; 624% = ### or .62%; 100% = }}}, or 1; 125% = ##, or 1.25; 250% = }}}, or 2.50. 7. 849. PER THOUSAND, PER MILLE OR PER M. In some of the professions and in some lines of business, the unit of measure or the basis of comparison is 1000. -- Thus, the statistics and the tables of mortality are given per M ; the rate of exchange is often quoted per M.; the price of lumber is quoted per M feet; and the price of many kinds of articles of commerce is quoted per M. NOTE.-M, is the Latin numeral meaning 1000. PROBLEMS IN PIER CENTAGE. 850. 1. What is 6% of 150 bales? Ans. 9 bales. OPERATION. 1.50 Explanation.—In all per cent problems of this kind, we first find 1% by 6 pointing off 2 places, and then multiply by the given rate per cent. We +-º-º-º-º: reason as follows: 19% of any thing is the hundredth part of it; hence 1% 9.00 Ans. of 150 bales is 1.50 bales, and 6% is 6 times as much, which is 9 bales. 2. What is 4% of $5103 Ans. $20.40. OPERATION. $5,10 Explanation.—First find 196 by dividing by 100, which is done by 4 pointing off 2 places, and then multiply by 4. We reason thus, 1% of $510 is $5.10, and 4% is 4 times as much, which is $20.40. In all percentage operations, we always find 1% first, and then the 96 $20.40 Ans. required; by this system, we will always have in the answer as many decimal figures as we had in the multiplicand. We should never decimal the rate per cent. 3. What is 40% of 375 oranges } Ans. 150 oranges. OPERATION. 3.75 40 150,00 Ans. 4. A stock raiser had 860 head of cattle and they increased 80%. What was the increase, and how many has he now Ans. 688 increase. 1548 present stock. 5. A man has $4854.20, and gives to various charitable and educational institutions 25%. How much has he left 3 Ans. $364065. 6. A factor sold 48524 pounds of cotton at 1242 per pound, and charged 24% commission. What was his commission ? Ans. $151,6375. 7. The population of a city, ten years ago, was 200000. During the past ten years it has increased 12.4%. What is the present population ? Ans. 225000. 8. A planter owned 550 sheep, and in 1 year they increased 50%; how many did he then have 3 Ans. 825. 436 SouLE's PHILosophic PRACTICAL MATHEMATICS. º: 9. What is 34% of $1720.15? Ans. $55,90-H. NOTE.-In practice, whenever the decimal or fraction of a cent, in the answer, exceeds #, it is counted as a whole cent, and when it is less than 3, it is not counted. 10. What is 124% of a cargo that weighs 1280 tons 16 cwt. 3 qrs. and 20 pounds 3 Ans. 160 T. 2 cwt. 0 qr, 11 lbs. 14 oz. 851. When the rate per cent, is an aliquot part of 100, the per cent may be obtained by a simple division operation. Thus, to obtain 25% of 940, we divide by 4, for the reason that 25 on the hundred, or ºr, is #, and # of 940 is 235, the a DSWeI’. To aid the calculator in this contracted work, we present the following 852. T A P T , FX. 14% = ſº-º; and conversely gº- 14% |50 % = }= }; and conversely # =50 % #%= ºri “ . . .';= 1..? |534%=###=#; “ . . . =534% 13% = ºats; * * *= 13% |55.9% = ºf “ “ #==55.7% 23% =#=#; “ “ ,- 25% |564%=#= %; “ “ , =564% 34% =#=#6; “ “ # = 3;% |584% =#= , ; “ “ , =584% 64% =+% =#; “ {{ * = 64% |624% =###=#; “ {{ # =62.4% 63% =#=#; “ “ #= 63% 65 % =# =#; “ “ #=65 % 84% = #;= #3; “ {{ # = 84% |663% =###= #; “ {{ # =663% 10 % = Pº, -ºo; “ “ Th; 10 % |68#% =###=++; “ “ +}=683% 124% =###= }; “ “ # =124% |73;%=#=}}; “ “ #=73;% 134% =#=#; “ “ #=134% |75 % =# = #; “ “ # =75 % 163% =#= }; “ “ # =163% |814% =###=#; “ “ ##=814% 18;% =#=#; “ {{ #=18#% |834% =###= #; “ {{ # =834% 20 % =#% = }; “ “ # =20 % |85 % =# = }}; “ “ #}=85 % 25 % = ** = }; “ “ # =25 % |874% =###= }; “ “ # =874% 263% =###=#; “ “ #=263% |913% =###=}}; “ “ +}=913% 31}% =###=#; “ “ #=314%|933% =###=#; “ “ #=934% 33%=#- #; ; ; # =#%|3%=#=#; . . #=::3% 35 % =# =#; “ “ ſº-35 % |95 % =#%=#; “ . . “ , ##=95 % 3.3%=#= #: .. # =#% 100% =}}}= the . º or the 40 % =+** = #; # = O Whole OI a Uning. 413% =###- #: {{ 44 *E* 150% =}#}= one and one-half times the 43#% =###– #; “ {4 *=433% whole number. 45 % = ** = }; “ {{ 3% =45 % |200% =#}}= twº ºnes the Whole num- 463% =###- #; “ 44 # =463% €I’. ALIQUOT PARTS OF 1000. 624% = +3 1874% = # 416#% = } 625 % = 3 8334% = } §% = +'s 250 % = + 4373% = # #% = } | 8663% = }} 83; ſº = # 2663% = + 4663% = # 6874% = }} | 875 % = } 125 % = # 312.4% - +3 5334% = # 7334% -> +} 916.3% = +. 1334% = |* | 3334% = } 5624% = # 750 % = } 933}7% = # 1663% = } 375 % = } 5833% = # | 812.4% = # | 937.3% = }; PERCENTAGE. 437 PROBLEMS. 1. What is 20% of $4264.20% Ans. $852.84. 1st OPERATION. 2d OPERATION. $42.64.20 Explanation.—Since 20% is + 20 5 || $4264.20 of the amount, we therefore, $ 852.84 Ans i. the º *divide $ 852.8400 Ans. e g ºnus own ºne co- 2. What is 24% of $500 Ans. $12.50. 3. What is 10% of 450 pounds? Ans. 45 pounds. 4. What is 12.4% of 1600 chickens? Ans. 200 chickens. 5. What is 20% of 444 apples? Ans. 88% apples. CLASSIFIED PROBLEMS. 853. To find the Percentage and the Amount or Difference, when the Base and the Rate Per Cent are given. Or, in Commercial Problems, to find the selling price, when the cost and the gain or the loss per cent are given. 1. What is 20% of 5503 Ans. 110. OPERATION. 5.50 Eacplanation.—In all problems of this kind, 20 or 5 || 550 for reasons above given, we first divide by *=- 100, which is done by pointing off two places 110, Ans. and then multiply by the given rate. Or, 110.00 Ans. when the rate is an aliquot part of 100, we take such a part of the number as the rate per cent is part of 100. 2. What is 25% increase of 500 pounds, and what is the amount? Ans. 125 percentage increase, and 625 amount. OPERATION. 5.00 e 6 º Eacol tion.—In th robl we find the 25 Or 4 || 500 :planation n Unis problem, we IIn percentage as above, and then add the same 125.00 125 = percentage. to the base. 125 + 500 = 625 pounds, amount. 854. From the foregoing elucidations we derive the following general direc- tions for finding the Percentage and the Amount or Difference: 1. To find the Percentage, divide the base by 100, and then multiply by the rate. Or, take such a part of the base as the rate is part of 100. 2. To find the Amount, add the percentage to the base. 3. To find the Difference, subtract the percentage from the base. NOTE.--When the base is a compound number, reduce the whole number to the lowest named denomination; or, reduce the lower denominations to a decimal of the highest. 438 SOULE's PHILOSOPHIC PRACTICAL MATHEMATICS. * 3. What is 163% of 1272? Ans. 212. 4. What is 17% of 3850 hats? Ans. 654.50 hats. 5. What is 40% of $1611.15? Ans. $644.46. 6. Bought goods at 24g, and sold them at a profit of 25%. What was the selling price? Ans. 302. 7. Goods cost $2.40 a yard, and sold at 20% loss. What was the selling price 2 Ans. $1.92. 8. What is 52% of $514.63? Ans. $267.60–1–. 9. What is 624% of 3240 peaches? * Ans. 2025 peaches. 10. What is 163% of 600 books? Ans. 100 books. 11. A steamer running at a speed of 12 miles an hour increased her speed 12.4%. What was her speed after the increase? Ans. 13% miles per hour. 12. Bought 2462 bushels of corn and sold 333% of it. How many bushels were left 3 Ans. 1641; bushels. 13. A capitalist had $85000 and invested 25% of it. How many dollars did he invest ? Ans. $21250. 14. The product of a factory in 1894, was 94.400000 pounds, and in 1895, it was 12.4% less. How many pounds was the deficiency? Ans. 11800000 pounds. 15. Which is greater, and what is the difference between 8% on 1200, and 7% on 1300% Ans. 1st, 8% on 1200; 2nd, 5. 16. A bankrupt merchant who owed $43000, paid 21%. How much did he pay? Ans. $9030. 17. A man had $1400 and spent 10% of it, and then 20% of the remainder. How much did he spend? Ans. $392. 18. Paid $8.50 per barrel for flour and sold it at a gain of 10%. What was the selling price? Ans. $9,35. 19. Paid $15 for a coat and sold it at 20% gain. What was the profit? Ans. $3. 20. Goods cost $22.50 per dozen, and sold at 334% gain. What was the gain per dozen 3 Ans. $7.50, 21. A grocer bought 160 dozen eggs and sold 84% of them. How many did he sell, and how many dozen are left? Ans. 13% doz. sold; 1463 doz. left. 22. A resides 14 miles from school; he walked 37.4% of the distance and rode the remainder. Eſow many feet did he ride 3 AnS. 4950 feet. 855. To find the Rate Per Cent, when the Base and the Amount or Difference are given, or when the Base and the Percentage are given. Or in Commercial problems, to find the Rate Per Cent when the cost and the selling price, or the Cost and the Gain or Loss are given. * PERCENTAGE. 439 1. The base is 40 and the amount is 50. What was the rate per cent 3 OPERATION. 40 = base. 50 = amount, or the base } + the Percentage. 10 = Percentage, gain, increase. 100 40 | 25 = gain or increase on 100. Ans. 25%. Explanation.—In all problems of this kind, we first find the percentage, increase, or decrease the gain or loss, as shown in the operation. Then by inspection and reason, we see that it was the base 40 which produced the 10 percent- age or gain. We then reason as follows: Since 40 gained 10, 1 will gain the 40th part, and 100 will gain 100 times as much, which is 25. It may and should be here asked, where we get the 100 and how do we know that the result represents per cent ? We answer: The 100 comes from the problem itself. The question of the problem, “What was the rate per cent,” if expressed as it is understood, would read: What was the gain or increase on 100 % By this we see where the 100 comes from, and as the reasoning showed that the result was a gain on the 100, it is therefore clear that it is gain per cent. 2. The population of a city in 1885, was 68000; in 1886, it was 72760. What was the rate per cent increase? OPERATION. 68000 = base. 72760 = amount, base + P. or increase. 4760 = Percentage = increase or gain. 68000 || 100 | 7 = increase or gain on every 100, and hence is 7%. Ans. 7%. Explanation.—The reasoning for this problem is the same as in the preceding one, and hence is omitted. 3. A hat cost $3.50 and sold for $4.20. What was the rate per cent gain } OPERATION. $3.50 = cost = base. 4.20 = selling price = amount. 702 = gain = percentage. 350 100 20 = gain on every 1002 and hence is rate % gain. Ans. 20%. Ezplanation.—As in the second problem above, We then Since $3.50 gain 70c., 10. will gain the 350th part and 100c. will gain 100 times as we first find the gain or percentage. reason thus: much, which is 20. 4. Goods cost $3.50 and sold for $2.80. What was the rate per cent loss? OPERATION. $3.50 = cost = base. . 2.80 = selling price = difference. 70% = loss = percentage. 350 || 100 20 = loss on every 100% and hence is rate % loss. Ans. 20%. Explanation.—In this problem, we first find the loss or percentage, and then reason as in the above problem. 44O soul E's PHILOSOPHIC PRACTICAL MATHEMATICs. * 5. Goods cost $22 and sold at a profit of $5.50. What was the per cent gain 3 Ans. 25%. OPERATION. 5.50 = gain = percentage. Explanation.—In this problem, we have the 22.00 || 100 gain or percentage given, and hence have but wº-tº 4. 25 = gain = gain 76. to make the proportional statement, the reason- ing for which is the same as above. 6. Yesterday the thermometer registered 76 degrees; to-day it stands 11 degrees lower. What is the per cent decrease in temperature? Ans. 14 °, 7%. OPERATION. g 11 = percentage = decrease. Explanation.—The reasoning for the propor- 76 || 100 tional statement of this problem is the same in the main as in problem 3, and is therefore 14; , 7% decrease in temperature. omitted. 856. To find what per cent one number is of another, or the Base and Percent. age given to find the 76 or Rate. 1. What per cent of 15 is 12? Or, 12 is what per cent of 153 Ans. 80%. OPERATION. 100% of 15 is 15. Eacplanation.—In this problem, 15 is the base or unit of dº comparison; and to produce the required proportional Ill IIIl-. 15 100% bers to solve the question, we find 100% of it, and reason. 12 thus: Since 100% gives 15, conversely 15 required 100%; hence 1 will require the 15th part and 12 will require 12 times as, 80% Ans. much, which is 80%. OPERATION. 100% of 12 is 12. 2. What per cent of 12 is 15? % or 15 is what per cent of 12? 100 Ans. 125%. 12 || 15 125% Ans. 857. To find what % one number is greater than another, or the Amount and Base given to find the ſ, or Rate. 1. What per cent is 15 more than 12? Or, 3 is what % of 12? Ans. 25%. OPERATION. 100% of 12 is 12. Ea:planation.—Here 12 is the base or unit of comparison; and to produce the required proportional numbers to solve f 0 the problem, we find 100% of 12 and reason thus: Since 100% 12 | 3 gives 12, conversely 12 requires 100%; and since 12 requires | 100%, 1 will require the Pº part and 3 will require 3 times as, 25% Ans. much. * . PERCENTAGE. 44. I 858. To find what % one number is less than another, or the Difference and Base given to find the 7% or Rate. *. * OPERATION. 100% of 15 is 15. 1. What per cent is 12 less than 15 + fº or 3 is what % of 15? Ans. 20%. 15 || 3 20% Ans. 859. From the foregoing elucidations, we derive the following directions for finding the rate per cent: 1. To find the Rate % when the Base and Amount, or the Base and the Differ- ence, or the Cost and the Selling Price are given, first find the percentage—the increase or decrease, the gain or loss—and then make the proportional statement shown in the operations; or thus : The base or cost: the increase or decrease, the gain or the loss : : 100: the required rate per cent. 2. When the base and the percentage of increase or decrease, or the base and the gain or loss, are given, then make the above proportional statement at once. Or, if it is desired to ignore all processes of reasoning, thus : Multiply the increase or decrease, the gain or loss, by 100, and divide by the base or cost. PROBLEMIS. 1. The base is 1750; the percentage is 43.75. What is the rate % tº Ans. 24%. 2. A yard of cloth cost 162 and sold for 182. What was the gain ſº Ans. 12; 9%. 3. Paid $120 for a horse and sold it at a profit of $22. What was the gain Ż, 3 Ans. 18; 7%. 4. In a population of 240000, there were 6214 deaths in 12 months. What was the rate per cent 3 Ans. 2.58++%. 5. What per cent of 54 is 6? Ans. 11% 7%. 6. What per cent of $540 is $67.50% Ans. 12.4%. 7. $2.50 is what % of $60,40? º * Ans. 4 ſº, 7%. 8. 9 is what % of 216? Ans. 4% 7%. 9. An invoice of goods cost in New York $3840; the freight was $57.60. What was the rate % Ans. 14%. 10. On a bill of $421.80, a discount of $21,09 was allowed. What was the 7% discount # * Ans. 5%. 11. What % of a number is ; of it? Ans. 624%. OPERATION. Let 100 = the number. Then 3 of 100 is 62%, and 62% on the hundred is 624%. 12. What % of a number is 5% of 15% of it? Ans. #%, or .75%. 442 SOULE's PHILOSOPHIC PRACTICAL MATHEMATICs. . . . . 13. What % of a number is 10% of 20% of 25% of it? Ans, sº, or 5%. 14, 15% of 60 is what % of 75% Ans. 12%. 15. 40 is what per cent of 83 Ans. 500%. 16. 6 is what % of 24? Ans. 25%. 17. A passenger train runs 25 miles an hour; an express train runs 35 miles an hour. What % slower does the passenger train run, and what % faster does the express train run, when compared with each other ? - Ans. Pass. train runs 284% slower. Ex. train runs 40% faster. 18. Single tickets cost 25%; in packages of 10 they cost 20g each. What is the 7% gain by buying by the package, and what is the ſº loss by buying by the single ticket? Ans. 25% gain by buying by the package. 20% loss by {{ “ single ticket. 19. What % of a number is # of it? Ans. 68%. 20. What is 2% of 124% of 663% of 25% of a number 3 Ans. # 7%. OPERATION. 100 assumed as the number. Then rºw x # × ### × #5 or rāg x * x #o; or 5% × # × 3 × # = gºod. Then ga'on of 100 is gº, and gº on the hundred is gº. 76. 21. In the above problem, what would be the percentage on $4800 3 Ans. $2. 0 Q # X O O OPERATION. gº, 7 of $4800 is $2 or thus: 25% of $4800 is $1200 : 663% of $1200 is $800 : 12.4% of $800 is $100; 2% of $100 is $2, answer. 22. What is 48% of 163% of 37.4% of 624% of a number 3 Ans. 13%. 23. The cotton crop of the Southern States ending the fiscal year, Sept. 1, 1884, was 5713200 bales; and for the fiscal year ending Sept. 1, 1885, was 5655900 bales. 1. What % more was produced in 1884 than in 1885? 2. What % less in 1885 than in 1884 3. What % is 5713200 bales of 5655900 bales # 4. What % is 5655900 bales Of 5713200 % Ans. 1.0131-H ſº more was produced in 1884. 1.00296-1- ?, less “ {{ 1885. 101.01314-?, 5713200 is of 5655900. 98.997.05+%, 5655900 is of 5713200. 24. A man has due him $45, and compounds on receipt of $36. What % did he lose ? Ans. 20%. 25. A broker bought bonds at 90% on the dollar and sold them at 95% on the dollar. What % did he gain } Ans. 53%. 26. In a year of 365 days, 67 days are rainy. What % of the days are not rainy ? Ans. 8144%. 27. According to the Carlisle mortality tables, 43 persons of every 5879 of 25 years of age die annually. What is the rate % of deaths? Ans. .731-H 7%. 860. To find the BASE when the Amount or Difference and the Rate per cent Increase or Decrease are given. Or, in Commercial Problems, to find the cost when the Selling price and the Gain or Loss per cent are given. º PERCENTAGE. 443 1. The manufactured value of goods is $2100, which is 20% more than the Value of the raw material. What was the value of the raw material? Ans. $1750. FIRST OPERATION. $100 = assumed base or value of raw Explanation.—By considering the problem, we material. see that the $2100 is the amount of the lº Of 9ſ) — e the raw material and the 20% cost to manufac- 20 = 20% increase value. ture the goods, we also see"that the 30% was calculated on the value of the raw material, $120 = manufactured value. and not on the $2100, the value of the manufac- tured goods. Hence there are no figures in the problem upon whose face we can calculate the 4) 100 = assumed base. 20% cost to manufacture. We therefore, as 120 | 2100 shown in the operation, assume 100, as the base *- ºmºmºm- or value of the raw material, and on this we $1750 cost of raw material. calculate and add thereto the 20% cost to manufacture. This gives an amount of $120, as the manufactured value of goods from a base or raw material value of $100, Now with these values which contain the same ratio of base and amount, or of raw material and of manufactured goods, that exists between the $2100 of manufactured goods and the required value of the raw material from which it was produced, we make the proportional statement, shown by the operation and obtain the required base or raw material value. In making the solution statement, we place the $100 assumed base or value of raw material on the line and reason thus: Since $120 amount or manufactured value at a gain of 20% require $100 base or value of raw material, $1 will require the 120th part, and $2100 amount or manufac- tured value will require 2100 times as much. In assuming a number to represent cost, it is immaterial so far as correct results are ooncerned, what number we assume; but we always select 100 for the reason that per cent being on the hundred, our operation is facilitated to a greater extent by 100 than by any other number. The foregoing reasoning and solution are applicable to all problems of like character, regardless of the rate of gain or loss per cent. But when the gain or loss per cent is an aliquot part of 100, the operation may be very much shortened by the following process of work and reasoning : SECONTD OPERATION. 6 ) $2100 Explanation.—In this solution, since the rate Tº RTNT_ 1 l f % is an aliquot of 100, we reason thus: Since 350 = # of base or value of r. m. the amount, $2100, contains a gain of 20%, and 5 since 20% is equal to # of a thing, the $2100 is -*msºmº- therefore the base or value of the raw material, $1750 = base or value of r. m. plus + of the same, which makes #, and since $2100, is ; of the base, # of the base is the # part, and ; or the whole base is 5 times as much. 2. Sold goods for $40 and gained 25%. What was the cost of the goods? Ans. $32. FIRST OPERATION, $100 = cost assumed. ^. Explanation.—Here, omitting the explanatory 25 = 25% gain added. remarks given in the preceding problem, We º assume $100 as the base or cost and reason thus: $125 = selling price. To gain 25% we must sell what cost $100 for $ $125; then since $125 selling price at a gain of 100 25%, required $100 cost, $1 selling price will 125 | 40 require the 125th part, and $40 selling price will * : *- require 40 times as much, which is $32 $32 cost, Ans. 444 SOULE's PHILOSOPHIC PRACTICAL MATHEMATICs. * SECOND OPERATION. 5) $40 = selling price. Explanation.—For this operation, We T63, SOIl T8 = + of cost. thus : Since the $40 selling price contains a gain 4 of 25% and since 25% is 4 of the cost, the $40 is hence # of the cost; and since $40 is # of the cost, sº- # of the cost is the # part, which is $8; and # or $32 cost, Ans. the whole cost is 4 times as much, which is $32. 3. Sold goods for $30 and lost 25%. What did the goods cost? Ans. $40. IFIRST OPERATION. $100 = cost assumed. Ea:planation.—Thereasoning for the proportion- 25 = 25% loss deducted. al statement is as follows: Since $75 selling -msm price at a loss of 25% required $100 cost, $1 $75 = selling price. selling price will require the 75th part, and $30 $ selling price will require 30 times as much. 100 75 30 NOTE.—In practice the line statement is all that needs to be made. The assumed base or cost -s | º and the amount or selling price resulting there- $40 cost, Ans. from, should be mentally performed. SECOND OPERATION. 3) $30 = selling price. Explanation.—In this solution, we reason as 10 = # of cost follows: Since the $30 selling price is the cost º º less 25%, it is therefore # of the cost; and since # of the cost is $30, 4 is the # part which is $10, tºº-ºº- and # or the whole cost is 4 times as much which $40 cost, Ans. is $40. 861. From the foregoing elucidations, we derive the following general direc- tions for finding the base or cost when the amount, or difference, or selling price, and the rate per cent are given : 1. To find the Base or Cost, first assume 100 to represent base or cost; on it calculate the given rate % to find the percentage; then either add it to or subtract it from the base or cost according as the rate % is a gain or a loss, an increase or a decrease, and thus produce an amount or difference, which has the same ratio of value to the 100 assumed base or cost as the given. Amount or Difference has to the required base or cost. Now, with this ratio of values and the given amount or difference, make the proportional statement shown in the operation ; or thus : The produced amount or difference : the assumed base or cost: : the given amount or difference : the required base or cost. 2. If it is desired to ignore all processes of reasoning in the solution, the following brief and arbitrary directions will solve this class of problems : Multiply the given amount or difference or selling price by 100, and divide by 100 plus the gain ſº, or minus the loss %. * PROBLEMS. 4. Sold goods for $20,624 and gained 25%. What did the goods cost? | Ans. $16.50. PERCENTAGE. 445 5. Sold a watch for $70 and lost 20%. What did it cost? Ans. $87.50. 6. The rent of a house is $67.50 per month, which is 124% advance on last year's rent. What was the rent last year? Ans. $60. 7. The amount is 2663, the rate % is 334. What is the base? Ans. 200. 8. The difference is 400, the rate % is 20. What is the base? Ans. 500. 9. What number increased by 10% of itself is 1815? Ans. 1650. 10. $18.17 is 15% more than what sum ? Ans. $15,80. 11. $4.50 is 50% less than what sum ? Ans. $9. 12. A produce merchant sold corn at 6042 per bushel, which is 10% more than he paid for it. What did he pay for it? & Ans. 55%. 13. A carpenter, after using 80% of his lumber had 500 feet on hand. How many feet had he at first 3 Ans. 2500 feet. 14. A shoe factory manufactured 2000 pairs of shoes, which weighed 5500 pounds, not considering the thread and nails. There was a waste of 84% in manufacturing. How many pounds of leather were used in making the shoes? \ Ans. 6000 pounds. 15. Sold sugar at 84% per pound and gained 64 per cent. What did it cost? Ans. 82. 16. My merchandise account is debited with $78500 and credited with $72225. The gain / on sales has been 124. What was the cost of the sales, and how much merchandise have I on hand 3 Ans. $64200, cost of sales. $14300, mase. on hand. 17. A planter lost by a storm 20% of his grain and has 29120 bushels left. How many bushels had he before the storm, and how many bushels did he lose ? Ans. 36400 bu. before the storm. 7280 bu, lost. 18. Sold goods for $100 and gained 40%. What did they cost 3 Ans. $71.42%. 19. Sold goods for $175 and gained 150%. What did they cost? Ans. $70. 20. What number increased by 25% of itself is $535? Ans. $428. 21. $72 is 20% more than what sum ? Ans. $60. 22. What number diminished by 25% of itself is 5103 Ans, 680. 23. A firm lost 14% of its capital by the failure of another house. The remaining capital was $278864,46. What was the whole capital? Ans. $324261. STATEMENT. $ 100 = capital. 86 || 278864.46. 24. A firm pays $9600 rent for its store, which is 20% more than it paid last year. What was last year's rent? - Ans. $8000. 25. If 40 boxes of peaches be sold for $113.75, at a gain of 624%, what was the cost and selling price per box : Ans. $1.75 cost. $2.84; selling price. 446 soulE's PHILOSOPHIC PRACTICAL MATHEMATICS. * 862. To find the Base when the Rate per cent and percentage are given. Or, in Commercial Problems, to find the cost when the Rate per cent and the Gain or Loss are given. 1. The rate per cent is 8 and the percentage is 36. What is the base? Ans. 450. OPERATION. 100 = assumed base. fi Explanation.—In all º”. ‘ā, this kind, we rst assume 100 base and find the percentage 8% calculated on assumed base. thereon at the given rate %. By this work we ** produce a base and a percentage containing the 8.00 = percentage on assumed base. same ratio that exists between the given per- centage and the required base. Having these 100 = assumed base ratio numbers, we then make a proportional 8 | 36 † statement as shown in the operation. In this problem we find the percentage to be wºm-º. 8. Then placing the assumed base on the state- 450 base, Ans. ment line, we reason thus: Since 100 base at 8% produced 8 percentage, conversely 8 percentage required 100 base; and since 8 percentage requires 100 base, 1 percentage will require the 8th part, and 36 percentage will require 36 times as much, which is 450. NotE.—In practice, only that part of the operation shown by the statement line should be made; the other work should be mentally performed. 2. 40 is 5% of what number 7 Ans. 800. OPERATION INDICATED. 100 5 | 40 3. On a sale of a lot of goods, a loss of $17 was sustained. The % loss was 20. What was the cost 3 Ans. $85. OPERATION. 100 = assumed cost. 20% $20.00 = loss on assumed cost. Explanation.—In this problem, the reasoning is the same as in the first problem, and hence is 100 omitted. 20 || 17 & | $85 cost, Ans. 863. From the foregoing elucidations, we derive the following general direc- tions for finding the base or cost when the rate per cent and the percentage or gain or loss are given : 1. To find the Base or Cost, first assume 100 to represent base or cost, and then Jind the percentage or gain or loss thereon at the given Rate %. Then, for reasons given in the eaplanation of the first problem, make the proportional statement as shown in the operation. * PERCENTAGE. 447 - 2. If it is desired to ignore all reasoning in the solution, then multiply the percentage, gain or loss, by 100, and divide by the given rate %. PROBLEMIS. 4. The rate per cent is 10 and the percentage is 108. What is the base ? Ans. 1080. 5. 3021 is 12.4% of what number 3 Ans. 24168. 6. Goods were sold at a profit of 25% and a gain of $5.60 was realized. What was the cost 3 Ans. $22.40. 7. Sold a watch for $26.50 more than it cost and gained 50%. What did it cost, and what was the selling price? Ans. $53 cost. $79.50 selling price, 8. City taxes are 2%, and a man paid $886.42. What was the assessed value of his property ? - Ans. $44321. 9. Ełła is #76 of what number ? Ans. #. 10. 84 is 1% of what number ? Ans. 850. 11. State taxes are .6%, and a man paid $67.50. What is the assessed value of his property ? Ans. $11250. 12. 5% of $180 is 12.4% of what sum ? Ans. $72. 13. A man lost $35.88, which was 8% of what he had at first. How much did he have left 3 Ans. $412.62. 14. A planter sold 2430 barrels of molasses and had 25% of his yearly product left. What was his yearly product? Ans. 3240 barrels. 864. To find the Gain or Loss per cent, when the Cost and the Selling price are given. - 1. Goods cost $3 and sold for $4. What was the gain ſ, 3 Ans. 33.3%. NOTE.-For the operation and reasoning for this class of problems, see Article 855. 2. Bought a horse for $180 and sold it for $160. What per cent was lost 3 Ans, 114%. 3. Sold flour which cost $7.50 at $8 per barrel. What was the gain per cent 3 Ans. 63%. 4. Bought shirts at $16.50 per dozen and sold them at $1.75 a piece. What % did I gain 3 Ans. 27# 7. 5. Bought 3 apples for 102 and sold them at 5% each. What % did I gain? Ans. 50%. 6. Bought tea at $1.25 per Ib, and sold it at 74% per oz. What % did I lose ? Ans. 4%. 865. To find the Cost, when the Gain per cent and the Selling price or the Gain or Loss are given * 1. Sold goods for $15 and gained 25%. What did they cost? Ans. $12. NOTE.—For the operation and reason for the solution of this class of problems, see Article 859. 448 SOULE's PHILOSOPHIC PRACTICAL MATHEMATICS. * 2. Sold a vest for $2 more that it cost and gained 333%. What did it * Ans. $6. OPERATION INDICATED. 100 100 cost assumed, 33% ſº of 100 = 334; then 334 2 3. If 32 per pound is lost by selling coffee at a loss of 20%, what was the cost 3 Ans. 15% per Ib. 4. Sold goods for $19.80 per dozen and lost 10%. What was the cost per single article 3 Ans. $1.83%. 5. A loss of 12 per pound was sustained by selling at 7 & 76 loss. What was the cost 3 Ans. 13%. 6. Sold corn at 564% per bushel and gained 124%. What did it cost 3 Ans. 502. 7. Sold gloves at a profit of 252 per pair, and gained 30%. What was the cost per dozen : Ans. $10. 8. Sold butter at 352 a pound and lost 63%. What was the cost 3 & * Ans. 3742. 9. I gained $15 by selling a horse at 163% gain. What did the horse cost me? Ans. $90. 10. A clothier sold a coat for $18 and thereby lost 11:#7% What did the coat cost 3 Ans. $20.25. 11. A dealer asked $25 for a suit of clothes, but took off 10% to effect a sale, and thereby gained 12.4%. What did the suit cost him 3 Ans. $20. 12. 144% was lost by selling silk after deducting 25% from $4 per yard asking price. What was the cost per yard 3 Ans. $3.50. ONE SELLING PRICE AND THE GAIN OR LOSS PER CENT OF THE SAME BEING GIVEN, TO FIND WHAT IT SHOULD BE SOLD FOR IN ORDER TO GAIN OR LOSE A DIFFERENT PER CENT. 866. 1. Sold merchandise for $196 and lost 30 per cent; what should it have been sold for to gain 20 per cent 3 Ans. $336. FIRST OPERATION. $ C 109 79 196 $280 cost. 56 = 20% gain. $336 Ans. SECOND OPERATION. S. P. $100 cost assumed. $100 cost assumed. $120 20 = 20% gain added. 30 = 30% loss. 70 | 196 $120 selling price. $70 selling price. $336 Ans. NotE.—In practice, the assuming of figures and the adding to and deducting from them should always be mental work. 2. Sold goods for 18% and lost 10 per cent; what should they have been sold for, to gain 10 per cent? Ans. 222. PERCENTAGE. 44.9 3. Sold hats for $4.50 and gained 50 per cent; at what price should they have been sold to gain 25 per cent? Ans. $3.75. 4. Sold boots at $50 per dozen and lost 163 per cent; at what price per pair should they have been sold to gain 20 per cent? Ans. $6. 5. Sold goods for 21+2 and lost 15 per cent? for what should they have been sold to lose 10 per cent ? Ans. 224%. TWO SELLING PRICES, AND THE GAIN OR LOSS PER CENT OF THE FIRST BEING GIVEN, TO FIND THE GAIN OR LOSS PER CENT OF THE SECOND. 867. 1. Sold goods at $12 and lost 25 per cent; what per cent would have been gained or lost had they been sold for $18? Ans. 124% gain. FIRST OPERATION. SECOND OPERATION. $ C G. $ 100 $2 13 3 75 12 16 || 100 2 12 75 $16 cost. 12#% gain. Ans. 225 18 selling price. um==ºe 1123, $2 gain. 100 124% Ans. 2. If 20 per cent is gained by selling goods at 242, what per cent would be gained if the goods were sold for 30¢ 3 Ans. 50%. 3. Sold flour at $4.80 per barrel and lost 40 per cent, what per cent would have been gained or lost if the flour had been sold at $7.80 per barrel? Ans. 24%. 4. If sugar is sold at 1232 per pound, and a gain of 124 per cent realized, what would be the gain per cent if the sugar was sold at 13+2% Ans. 20.4% gain. TO COMPUTE PER CENT ON ENGLISH MONEY. 868. 1. What is 3% of £ 647 18s. 5d.” Ans. £19. 8s. 9d. FIRST OPERATION, SECOND OPERATION. 36 6.47.18.05 fº 6.47.18.05 Ea:planation.—In the first operation, 3 3 we divide the £, s. and d. each by 100 as shown by the decimal points sº ===ºmºsºmºmºmºmºsºms and thus obtain 196. Then we mul- £19.41.54.15 = 3%. fºl9.43.15.03 tiply by 3 and obtain 396. We then 20 20 reduce the hundredths of £. s. and d. to shillings and pence, according to 8s.74 8S.75 the principles of denominate numbers, S.'ſ S. as shown on page 260. 12 12 In the second operation, we found 1% ºsmº º gºmºmºmºe as in the first operation, and then, when 9d.03 9d,03 multiplying by the required rate per cent, we reduced mentally the .156 to hundredths of shillings,” by dividing by 12, and the .55s, to hundredths of pounds, by dividing by 20. Then we reduced the hundredths of £. s. and d. to shillings and pence as in the first operation. 45O soul E's PHILOSOPHIC PRACTICAL MATHEMATICS. * TO REDUCE SHILLINGS AND PENCE TO THE DECIMAL OF A POUND, AND TO REDUCE THE DECIMAL OF A POUND TO SEIILLINGS AND PENCE. 869. 1. Reduce 12s. 6d. to the decimal of a pound. SECOND OPERATION. Multiply the shillings by .05 and the pence by .004% and 12s. 6d. = £0.625 add the results together. Thus: 12 12s. 6d. 12s. 6d. em-º- .05 .004; or thus, 5 (hund’ths) 4% (thous'ths) 240) 150 (.625 º- tº-º-º: -b .60--.025=3.625. 60 (hund'ſ hs)+25 (thous'ths)=36.625 NoTE.—The multipliers in the second operation are produced thus: Since 20s. = £1., 1s. = £ºſ, or .05 of a £. Since 240d. = £1, 1d. = £3+n, or .004% of a £. In like manner the decimal of a farthing would be £540, or .001 ºr of a £. 2. Reduce .625 of a pound to shillings and pence. Explanation.—In the 2d operation of the first prob- lem, we showed how the multipliers, .05 and .004%, OPERATION. were obtained and used to reduce shillings and pence to the decimal of a pound. Now since the decimal FIRST OPERATION. Divide .62 N 12S was produced by these multiplers it is clear that by by 5 - *** or thus, using them as divisors, we will reproduce the shillings º 625 and pence. We first divide the first two figures of Divide .025 o the decimal, .625 by .05 which gives 12s. and .02 over. IV1C16) . } = 6d. We then divide the .025 by .004; and obtain 6 pence. by 4% 12s. 6d. In practice, the operation is performed by dividing the first two figures of the decimal by 5 and the remainder, if any, and the third decimal figure by 4%. PROBLEMIS. 3. Reduce 15s. 11d. to the decimal of a pound. Ans. £.796: 4. Reduce .679 of a pound to shillings and pence. Ans. 13s. 7d. 5. What is 3% of £647 18s. 5d. } Ans. £19 8s. 9d. SOLUTION OF THE FIFTH PROBLEM. FIRST OPERATION. SECOND OPERATION. * THIRD OPERATION. £647. 18s. 5d. = £ 647.921 :6647 18s. 5d. f:647 18s. 5d. r 3% 3% % £19.43.763 #19.41.54.15 :E19.43.15 3 20 20 20 8S, .75260 8s. 74 8S.75 12 12 12 9d. .0312 9d. 03 9d. 03 6. What is 5% of £2614 1s. 2d. Ans. £130 14s. 1d. 7. What is 1% of £100 1s. 1d. Ans. £1 0s. 0.136. 8. What is 4 per cent. of 12s. 7d. } Ans. 3f. 9. What is 60 per cent. of £500 1s. 8d.: Ans. £300 1s. 0d. NOTE—See pages 484, 569, 661, 744 to 758 for further work in English money. PROFIT AND LOSS. 870. Profit and Loss are terms which express the gain or loss in business transacted, and which involve the principles of percentage in all its various com- binations, as applied under different subjects, such as Commission, Brokerage, etc. The subject is of great practical value and should receive the thoughtful attention of the business aspirant. * PERCENTAGE. 451 Profit or Gain is the sum resulting from business or from business trans- actions in excess of the Cost and Expense of the Business. Loss is the sum spent or incurred to carry on business or to effect business transactions, in excess of the receipts or income of the business. MARKING GOODS. 871. To Mark Goods at a Given Gaim or Loss Per Cent. 1. Bought butter at 25¢ per pound. At what price must it be sold to gain 25 per cent ? Ans. 31.4%. 25 OPERATION. 64 = 25% gain. 25% = ** = *-i-º-º 31#2, Ans. 2. Bought cheese at 15% per pound. At what price must it be sold to lose 10 per cent 3 Ans. 1342. 15 OPERATION. 1.5 = 10% loss. 1. T 13.5%, or 1342, Ans. NOTE.—When the per cent. gain or loss is an aliquot part of 100, all we have to do, when marking goods, is to add to or subtract from the cost, such a part of itself as the rate per cent, is part of 100. 10% = }} O PRACTICAL OPERATIONS IN MARKING GOODS. 872. In marking goods, it is the custom of most merchants to use a private mark, either of arbitrary characters or letters, instead of figures, to show the cost and selling price, or, as some mark goods, to show the cost, the wholesale and the retail price. If letters are used, any word or phrase that contains 10 different letters may be selected. Any of the following words or phrases may be adopted for a Marking word or Key word : Cash Profit. Regulation. Importance. |Black Horse. Hard Moneys. Now be sharp. Washington. My Wife's Hat. Dont be lazy. Brick House. Buy for Cash. Corn Basket. Purchasing. God help us. X. Lord Save it. To illustrate the work, we will select the words “Cash Profit,” and with the ten letters represent the ten figures as follows: 1 2 3 4. 5 6 7 8 9 0 C. A. S. H. P. R. O. F. I T To avoid the repetition of any letter or character, an extra letter or character called a “Repeater,” is used. By this means, the uninitiated are less likely to discover the key word or mark. - When the repeater is used, it should follow the letter or character which it repeats. 452 soul E's PHILOSOPHIC PRACTICAL MATHEMATICS. * . With the above key word, and L as a repeater, what is the mark for the cost and selling price of goods that cost $18.60, on which 20% gain is required? Ans. C F R T cost. A L S A selling price. Instead of using one mark, as above, for cost and selling price, it is the custom of many merchants to use two different marks, one for cost and one for selling price. To illustrate this work, we will use the above mark, Cash Profit, for the selling price mark, and select the word Regulation for the cost price mark. Our marks then stand as follows: Oost Price Mark. Selling Price Mark. 1 2 3 4 5 6 7 8 9 0 1 2 3 4 5 6 7 8 9 0 R. E. G. U L A T I O N C A S H P R O F I T With these key words, and B as a repeater, what is the mark for the cost and selling price of goods that cost 50g per yard and that are held for sale at a gain of 10 per cent 3 Ans. L N cost. P B selling price. NOTE.--To distinguish dollars from cents, some use the dollar or cent mark, some the decimal point, and some make no designation of either in the mark, but consider the mark as all cents. Instead of using words for marks, many merchants use characters. Any ten characters may be selected to represent the ten figures. The following double cross is frequently used, thus: The double cross may be 1 || 2 | 3 iº used in many ways, thus: With 0 as the alsº tenth charac- 1 2 3 4 5 6 7 8 9 0 3|4|8 7 || 8 || 9 ter and x as L. L T, [] D T [..] [T O 5 || 2 || 7 6 || 1 || 2 7 || 8 || 9 a repeater. 8|9||1 5|4|3 With Buy for Cash, as a cost mark, using D as a repeater, and with the characters shown in the first double cross above as the mark for the selling price, mark the cost and selling price of the following articles, at the gain or loss per cent. given : F D H Cost $ 4.40 (a) 10% loss, mark, tº Cost $ 2.50 (a) 20% gain, mark, U_O H.: ; {{ .18 (a) 25% gain, mark, “ 65.00 (a) 40% loss, mark, “ 24.00 (a) 12:4% gain, mark, 46 .15 (a) 334% loss, mark, Mark in letters from the Key of Buy for Cash, with D for a repeater, Cost and Selling Price of the following: f h Cost $ 2.40 a 25% gain, Ans. #. Cost $ 3.00 (a) 10% loss. {{ .16 (a) 12:4% gain, {{ 4.50 (a) 40% loss. “ 15.00 (a) 33.4% gain, * {{ .10 @ 20% loss. “ 16.50 (a) 30% gain, “ 75.00 (a) 5% loss. Find the selling price of the following: 1. Of goods which cost 182 and sold at 334% gain. Ans. 242. 2. Of goods which cost $240 and sold at 40% gain. Ans. $336. 3. Of goods which cost $4.25 and sold at 163% gain. Ans. $4.95%. PERCENTAGE. 453 4. Of goods which cost $14.50 and sold at 30% loss. Ans. $10.15. 5. A merchant paid $108 per dozen for coats. At what price per coat must he sell them to gain 50% º Ans. $13.50. 6. Bought silk for $1.60 per yard. At what price must it be sold to gain 140% Ans. $3.84. A PUZZLING PEROBLEM. 873. A nest of five tubs cost $3.00. What should be the selling price of each tub, to gain 334% on cost 3 OPERATION BY AssumED RATIOS OF VALUE, TO OBTAIN ONE OF MANY ANSWERS. $3.00 cost + 334% = $4.00, selling price for the nest of 5 tubs. 22 + 32 + 5% + 82 + 12% = 30%, which is the sum of the assumed ratio of values. Then the following proportional statements give the respective value of each tub: First or smallest Tub. Second Tub. Third Tub. Fourth Tub. Fifth Tub. 2g 3 g 5 g 8 g 122 30 || 4.00 30 || 4.00 30 || 4.00 30 || 4.00 30 || 4.00 | 26%, Ans. 402, Ans. | 66%, Ans. | $1.063, Ans. $1.60, Ans. Practically, where the nickel is the smallest coin used in trade, the prices would be as follows: 1st tub, 25g ; 2d tub, 402; 3d. tub, 65%; 4th tub, $1.10; 5th tub, $1.60. SECOND OPERATION BY PROPORTION, ACCORDING TO THE SERIAL NUMBERS OF THE TUBS. The following statements give the value of the respective tubs: No. of smallest tub, 1 4 & ** 2d “ 2 First Or g * 4t tº 3d 44 “ 3 Smallest Tub. Second Tub. Third Tub. Fourth Tub. Fifth;Tub. • * : * 4th “ “ 4 $ $ $ $ $ “ “ largest “ 5 4.00 4.00 4.00 | 4.00 4.00 15 || 1 15 || 2 15 || 3 15 || 4 15 || 5 Sum of serial Nos. 15 *===ºmes mºs 26}c. 53;c. 80c. 1.06;c. 1.33:#c NOTE.-Other ratios of value may be assumed according to the judgment of the calculator. Or, if it is deemed proper, the prices may be made proportional to the capacity or volume of the respective tubs. 2. What must be the selling price per box of a nest of 7 boxes which cost $15, the gain ſ, being 25? IN THE FOLLOWING ARTICLES THE LETTERS INDICATE THE COST PRICE. 874. Write the cost in figures, and write the selling price in letters, at a gain of 20%, using Dont be lazy as the key, and any other letter as a repeater. 1. Hats, $dayc per doz. 4. Oranges, beg per doz. 7. Flour, $t.lb per bbl. 2. Gloves, $o.cb per pair. 5. Books, $n.yc each. 8. Coffee, odg per lb. 3. Cuffs, $o.by per doz. 6. Shirts, $d.ob each. 9. Sheeting, dm3% per yd. Ans. to the last, 1342 cost, 16#2 selling price. Ans, to the first, $18.00 cost. $od.ey selling price. '454 SOULE's PHILOSOPHIC PRACTICAL MATHEMATICS. Y A RAPID PRACTICAL PROCESS OF FINDING THE SELLING PRICE PER SINGLE ARTICLE, AT A CERTAIN PROFIT WHEN THE COST PRICE IS PER DOZEN. 875. 1. Bought shoes at $7.50 per dozen; at what price per pair must I sell them to gain 20% º Ans. 752. In all questions of this kind, when the goods are bought by the dozen and the gain per cent is 20, we simply divide by ten or remove the decimal point one place to the left, and the result is the selling price per article. This is a very important contraction for merchants who buy at auction, as by it they can instantly determine at each bid what the selling price must be per article, in order to give a fair profit. The philosophy of this work is simple and will be easily comprehended by the elucidation of the following problem: If shirts cost $22,50 per dozen, what must they be sold for, a piece, to gain 20% * Ans. $2.25. OPERATION. y $ Explanation.—For reasons often repeated we assume $100 cost, and reason as follows: In order to gain 20% we must sell 10 22.50 what cost $100 for $120. This selling price we place on the Ø º statement line, and continue the reason thus: If $100 cost I? require $120 selling price, $1 cost will require the 100th part, and 22% cost will require 224%; times as much. . Then as the result of this statement will give the selling price of a dozen, and as we wish to obtain the selling price for a single article, we therefore divide by 12. This completes the reasoning and statement, and by the same we see why we remove the decimal point. First cancel the 0 on the 120 and one 0 on the 100; then cancel the 12 on each side of the line, and we have the cost price remaining on the right and 10 on the left. . It is obvious that this will always be the case when the per cent gain is 20, and hence the work of dividing the cost by 10, which is done by simply removing the decimal point, to find the Selling price per article, is made clear and comprehensible. 876. T A TE T, TE FOR DIFFERENT RATES PER CENT TO BE USED ACCORDING TO THE ABOVE PRINCIPLES : & To gain 20 % remove the decimal point one place to the left { { {{ 25 % 6 & 4 { {{ and add # itself. {{ 26 % { % & 4 {{ {{ {{ 3% 44 46 28 % {{ & 4 {{ {{ {{ i’s {{ éé 30 % { % & 4 {{ {{ {{ * “ {{ 32 % {{ {{ {{ {{ {{ +'s {{ {{ 334% { % {{ {{ {{ {{ . # “ & 4 35 % {{ {{ {{ {{ {{ # “ & 4 *37.4% {{ 4% {{ {{ {{ + “ {{ 40 % {{ {{ {{ {{ {{ # “ {{ 44 % {{ {{ 64 {{ {{ # “ { % 50 % {{ {{ {{ {{ 46 # {{ § { 60 % 4% 6% {{ {{ (4 # “ { % 80 % {{ {{ 4% {{ {{ # “ { % 12.4% 4% {{ 44 {{ and subtract Tº “ {{ 4% 4% & 44 1 ... }, . . . . . . . Žt 70 Tº 6 &é 10 % 46 {{ 6% {{ {{ * 44 *NOTE.-This is practically, not accurately correct. According to the above what would be the price per article to gain 20% of the following goods bought by the dozen: Hats, $32, Gloves, $18.50. Shoes, $44.50. Coats, $150. Pants, $37.50. Knives, $3.75. Chairs, $21,25. * PERCENTAGE. 455 TO FIND THE PRICE AT WHICH GOODS MUST BE MARKED SO THAT A GIVEN PER CENT OF GAIN OR LOSS MAY BE REALIZED AFTER ALLOWING FOR STATED DISCOUNTS, BAD DEBTS, WASTE, COMMISSION, ETC. 877. 1. A merchant marks his goods at 40% gain, but supplies his whole- Sale buyers at a discount of 20% from list price. What is his gain ſ, 3 Ans. 12% gain. OPERATION. $100 = COSt. 40 = gain Ż. $112 = selling price. *mºmºmºsºmeº 100 = cost. $140 = list price. sºmºsºms 28 = 20% discount. $ 12 = gain, and as this gain is *=mºm on the hundred it is $112 = selling price. therefore gain Ż. 2. If a merchant marks his goods at 25 per cent profit and effects sales at 20 per cent discount on retail price, does he gain or lose? Ans. Neither. 3. If a merchant marks his goods at 50 per cent profit and effects sales at 40 per cent discount on retail price, does he gain or lose, and if so how much 3 Ans. Loses 10%. 4. A merchant wishes to increase the price of an article which he now Sells for $9, so that he can deduct 10% and still receive the present price; what must be the advanced price? Ans. $10. OPERATION. $100 advanced price assumed. $ 10% deducted. 100 90 9 $90 selling price. **** $10 Ans. 5. A merchant wishes to mark his goods in such a manner that when he sells at wholesale he may discount 20% from his retail price and still gain 10% on cost. Accordingly, what must be the retail price of goods that cost $1.60 per yard 3 Ans. $2.20. OPERATION. $1.60 = COSt. $ 16 = 10% gain. 100 = assumed retail price. 80 | 1.76 $1.76 = wholesale price. *E== $2.20 = retail price, Ans. 6. What must be the asking price of goods that cost 602 per pound, So that 10% may be deducted and a gain of 20% still be realized ? Ans. 802. 7. What must be the asking price of goods that cost $15, so that 40% may be deducted and the goods sold at cost 3 Ans. $25. 8. Goods cost 45g per pound; what must they be marked so that by deduc- ting 25% sales may be effected at a loss of 10% Ans. 54%. 456 SOULE's PHILOSOPHIC PRACTICAL MATHEMATICs. ºr 9. Paid 182 per dozen for eggs. Allowing 5% for breakage and 10% for uncollectable sales, how much per dozen must they be sold for to gain 14% on first cost 3 Ans. 242. OPERATION OPERATION OPERATION to find selling price to allow 10% for un- to allow for 5% Or thus: to gain 14% collectable sales. breakage. 18g. 14% 100 100 114 90 |, 2052 95 | .2280 90 || 100 .0252 fººm. , — 95 | 18 .18 .2280g. $.2400 Ans. ºsº 24g .2052 Ans. 10. Goods cost $10. What must be the asking price so that 15% may be deducted and a gain of 25% realized on first cost? Ans. $14.70+}. 11. Goods cost $12. What must be asked for them so that 20% may be deducted from the asking price, 64% be allowed for waste, 10% allowed for “bad debts,” and 25% gain made on first cost 3 Ans. $22#. OPERATION. $12 100 100 | 100 125 3 = 25% gain. 93% 15 90 16 80 17; Or, 93; 100 *ss — — — tº-ºº-ºº ºmº 90 100 $15 16 17; 22} 80 12 $ 223 12. Bought a cargo of coffee at 10g a pound. Paid 10% duty. What must I ask for the coffee per pound, so that I may fall 10% on my asking price, allow 10% for loss by shrinkage of coffee, lose 10% of sales in bad debts, and still gain 10% on my investment (full cost)? Ans. 16###2. TO MARK DIFFERENT LOTS OF GOODS IN AN INVOICE. 878. Bought in New York an invoice of goods amounting to $2400. Paid freight and other charges to deliver them in New Orleans, $102. In the invoice are 8 dozen hats which cost in New York $27 per dozen. 1. What was the ſº of the charges # 2. What was the cost of 1 hat in New Orleans? 3. What is the retail price per hat, from which 20% may be deducted and the hats sold at whole- sale at a gain of 25% on New Orleans cost? Ans. 44% charges. $2.34+cost of 1 hat in New Orleans. $3.66—retail price of 1 hat. MARKING GOODS, BASED ON FACTORY COST AND ON SELLING PRICE, ETC. 879. Goods cost at the factory, $1400. Boxing, $4. Drayage to dock, $9.50. Freight to destination, $30. Drayage from wharf to store, $9. (1). What is the PERCENTAGE. 457 floor cost of the goods? (2). What is the percentage of the charges? (3). What price must the goods be marked to gain 20% on the floor cost. (4). Based on floor cost, what must the goods be sold for, so that the gain will be 20% of the selling price? (5). When the goods are marked to gain 20% on the sales, What per cent is such gain on the factory cost? Ans. (1). $1452.50 (2). 33% (3). $1743.00 (4). $1815,625 (5). 29.6875% OPERATIONS, (1). To find floor cost. (2). To find percentage of (3). To find gain of 20% on charges. floor cost. 1400. 4. 52.50 $1452.50=floor cost. 9.50 * 1400 || 100. 290.50=20% gain. 30. * 9. 3.75 or 33% $1743.00=selling price to gain wºmeºs 20% on floor cost. $1452.50 (4). To find gain of 20% on (5) To find what the 20% gain on the sales over floor cost. sales is on factory CoSt. $ 100. $1815.625 = selling price. 80 || 1452.50 1400.000 = factory cost. | sists,825= selling price to $ 415.625=gain—$1400–29.6875%. gain 20% on sales. Goods cost at the factory, $130. (1). Estimating that the expenses to place the goods in the store of the purchaser and the discount allowed to customers equal 12% of Factory cost, what must the goods be marked to gain, 10% on the Store cost? (2). Based on floor cost, what must the goods be marked and sold for so that the gain will be 10% of the selling price? Ans. (1). $160,16 (2). $161.77; OPERATIONS. FIRST. SECOND. | $130. $100. 15.60=12%. $90 145.60 $145.60=floor cost. $161.77% =Selling price to 14.56=10% on floor cost. gain 10% on sales. $160.16=price marked. TO IMPORT AND MARK DIFFERENT LOTS OF ENGLISH GOODS. 879%. Imported from England an Invoice of goods amounting to £1450. 8s. 4d. Paid freight $190.50. Insurance $132.15. In the invoice were 40 dozen pocket knives which cost £160, 2s. 8d. The duty on the knives was 25% ad valorem. What was the invoice cost in United States Money, and what must be the retail selling price per knife to allow 20% discount and to sell at wholesale at a gain of 25% on the full importing cost? Ans. $3.28%. OPERATION, £1450-8-4=#1450.416--x4.8665=$7058.45 inv. cost of goods in United States Money. £160-2–8–42160.133—H·x 4.8665=$779.29 { { ‘‘ knives § { é & $190.50+$132.15=$322.65, Insurance and freight. (322.65×100)—$7058.45=4.57+% charges on invoice. $779.29 ×4.57% =$35.61 charges on knives. $779.29x25% =$194.82 duties on knives. $779.29–H35.61-I-S194.82=$1009.72 full cost of knives in New Orleans. $1009.72–40–$25.24 cost of one dozen knives in New Orleans. $25.24–12=$2.10 cost of one knife in New Orleans. $2.10–1–25% =$2.62% wholesale price per knife in New Orleans. ($2.62}x100)—80=$3.28; retail price of one knife in New Orleans. REMARK. —To find importing cost, each lot of goods in a foreign invoice must be treated in manner as above shown. due care being given to the different rates of duty on the different lines and the grade or qualities of the goods. 458 SOULE's PHILOSOPHIC PRACTICAL MATHEMATICs. jºr TRADE DISCOUNTS AND LIST PRICES. 880. Trade Discounts are allowances or rebates made by merchants, manufacturers and other classes of business men from their regular marked or list prices. It is customary for many classes of business men to have fixed price lists of their goods based on the highest market rate, and when the market declines, instead of changing the fixed price list, they allow a discount or a series of discounts from the list prices. These discounts will be shown in the problems following. Merchants and Manufacturers also frequently allow certain discounts or rebates from the retail price, proportioned to the amount of the bill sold. Some lines of goods are sold at “TIME PRICEs,” conditioned on certain discounts if paid prior to the time specified. These terms and conditions are often printed on the “Bill Heads,” and read as follows: “Terms 4 months, or in 30 days less 5 per cent.”; or “Terms 90 days or 2 per cent. discount in 30 days, or 3 per cent. discount in 10 days,” etc. In many cases, where goods are sold on Time prices, the rate per cent. allowed is a matter to The agreed upon by the parties, and is generally 5 per cent., 10 per cent., or 25 per cent., according to the length of time that the bill is paid before due, the hazard of the credit, the profit realized on the sale of the goods, the necessities of the parties for money, and the rate of interest at which money can be obtained at the bank. The discount is allowed on the face of the bill or invoice without regard to time, notwith- standing that time and the rate of interest on money were considered in determining the rate of discount. With a view to give a better understanding of this system of discounting, we will state that 196 discount on bills maturing in 30 days is equal to 12 per cent. interest. 2% discount on bills maturing in 30 days is equal to 24 per cent. interest. 3% discount on bills maturing in 60 days is equal to 18 per cent. interest. 5% discount on bills maturing in 3 months is equal to 20 per cent. interest. 5% discount on bills maturing in 4 months is equal to 15 per cent. interest. 5% discount on bills maturing in 6 months is equal to 10 per cent. interest 5% discount on bills maturing in 12 months is equal to 5 per cent. interest. When no rate of discount has been agreed upon, and the bills are paid before maturity, business men usually allow and deduct from the face of the bill the current rate of interest for the time the bill was paid before due. PROBLEMS. g 1. A bill of goods at list price amounts to $462.50. A discount of 15 per cent. is allowed. what is the net amount 3 Ans. $393.12%. OPERATION, $46,250 = 10% $462.50 23.125 = 5% 69.37% $69.375 = 15% discount. $393.12% Answer. *- NOTE.—To find 15 per cent. multiply by 10 and add # of the product. 2 An invoice of merchandise at regular list prices amounts to $3480. What is the net amount, the series of trade discounts being 25 per cent., 10 per cent and 5 per cent f * Ans. $2231.55. OPERATION. $3480 = list price. 870 = 25% discount. Explanation.—From the list price, we first deduct the 25 per cent, discount, and from the successive $2610 * h t deducted 261 = 10% discount, remainders the other per cents are “ e The order in which the series of discounts is $2349 deducted does not change the result. 117.45 = 5% discount. $2231.55 = net amount. In this manner, the discount may be found in any series, * PERCENTAGE. 459 TO FIND THE SINGLE OR EQUIVALENT DISCOUNT, FROM A SERIES OF DISCOUNTS. 881. 1. In the above problem, what single discount on the list price is equivalent to the series of 25 per cent, 10 per cent. and 5 per cent. Ans, 35%%. FIRST OPERATION. $100 = list price. º Discounts. Explanation.- We first assume 25 = 25 per cent. discount 25 $100 as the list price, and then -** *º- deduct therefrom the 25 per 75 cent. discount; then from the remainder † º, 10 per .50 = 1 ... di .50 cent. discount: then from the 7.5 0 per cent. discount 7.5 second remainder $67.50, deduct re 5 per cent. discount, which 67.50 . 3, * F.” § 64%. . º subtracted from the assume 3.3750 = 5 per cent. discount 3.375 $100 list price, gives $35% dis- -*-*-* * e -*=s*- count, which being on the $ 64.1250 = net price. 35.875% =354% hundred, is 35% per cent, dis- - º count. Or, by adding the sever- 100. = list price, as above. al discounts as shown in the - operation,...we also obtain 35g 35.875 per cent. = equivalent single discount. Per cent discount. A series of any number of discounts, may be Worked in the same manner. SECOND OPERATION. 100 — 25% = .75 Then 75 x 90 × 95 = .641250 of 100, or 100 – 10% = .90 & 64.125 per cent. and 100 – 64.125 = 35.875 100 — 5% = .95 ( per cent, the single equivalent discount. A series of any number of discounts may be worked in the same manner. PROBLEMIS. 2. The list price of a bill is $1781.40; the rates of discount are 33% per cent., 20 per cent., 10 per cent. and 5 per cent. What is the net amount? Ans. $812.3184. 3. A merchant sold a bill of goods amounting to $530 on which he allowed 40 per cent. and # discount. What was the net value of the bill? Ans. $265. 4. Which is the greater discount, and what is the difference on a sale of goods amounting to $1420, 40 per cent., or 25 per cent. and 15 per cent.” Ans. 40 pr. ct, is $53.25 greater. 5. What is the equivalent per cent. of #, 33% per cent, #, and 10 per cent. discount ºf Ans. 76 per cent. 6. A manufacturer after allowing # and 20% discount, received for a lot of goods $3360 What was the catalogue or list price? Ans. $6300. 460 SouLE's PHILOSOPHIC PRACTICAL MATHEMATICs. & [7. I FRANK DUMAs, NEW ORLEANs, August 12, 1895. Bot. of C. McGuigin. TERMS-Cash; 10% off. 462 lbs. Prime Coffee, 375 ‘‘ Good é & 180 * * Fair é & 216 “ Ordinary “ 74 “ Common. “ Less 10% discount. I 8. I JONES & WEISS, TERMS-Cash; 10 per cent. and 5 per cent discount. dº (2) 21 c. 97.02 º & 4 22 c. 82|50 º & & 21 c. 37|80 gºe & 4 20; c. 44|28 tº { { 193C. 14|43 $248 43 NEw ORLEANs, Jan'y 21, 1895. Bot. Of Stewart dº Henderson. 34 bbls. La. Oranges, lar., & 27 boxes Messina Lemons, 63 cases Malaga Grapes, * * 45 boxes California Pears, 5 mats Dates, 593 lbs., gº tº Less 10% and 5% off. [9. I J. M. BUTCHEE, TERMS-Cash; 12% per cent. and 10 per cent, discount. # tº * gº (2) $5.75 & gº * , & 4 6.00 * > gº tº-e é & 1.75 gº gº * & & 4.50 sº sº gº ( & 73c. 714|73 NEW ORLEANs, Jan'y 31, 1895. Bot. of J. B. Cundiff. 875 bbls. Nes. Potatoes, 440 “ P. B. Potatoes, 325 “ Perfection Flour, 1324 “ St. L. XX. Flour, 112 “ Family Clear Pork, 650 “ Prime Pork, wº 220 kegs Pig Feet, tº º 124 half bbls. F. M. Beef, 1872 lbs. Choice Ham, * * 289 “ B. Bacon, * e- 106 Pig Tongues, tº gº I)is. 124% and 10%. * * sº tº- (2) $ 4.25 tº e * & 4 3.87% * sº *- & & 8.50 g- º sº & & 6.62% tº # * sº & 4 17.50 sº sº tº & 4 13.75 gº sº tº & 4 7.50 * - sº wº- & 4 11. * cº gº & & 14 c. sº sº gº tº & & 94 c. tº tº cº-e & 4 8 c. 3.116727 * PERCENTAGE. 461 10. [ 1. NEW ORLEANS, Nov. 17, 1895. GEO: B. BRACKETT & CO., Bot. of R. Spencer Soulé. TERMS-Cash; 163 per cent. off. 1427 bu. No. 1 Winter Wheat, - - - - - - a $1.5 856 “ No. 2 Winter Wheat, * * tº - * * {{ 1.47 420 “ Ill. No. 1 White do. -> •º º * tº- - {{ 1.41 3145 “ W. Corn, - - - - - - - - - “ .70 1040 ** B. Oats. * - º º - º - - --> * { .55 6835|87 Less 163%. s = º s - * * * -> -º- [11. I NEw ORLEANs, August 12, 1895. E. L. CHAPPUIS, Bot. Of W. Reed Carradine. TERMS-4 months or 5 per cent. and 2% per cent. discount in 10 days. Aug. 12 |42 bbls. Flour, “Winter Patents” - - - - (a) $5.25 220 |50 16 ‘‘ “ Extra Fancy º º - * - { { 5.00 80|00 27 “ Fancy - tº- - * * - & 4 4.75 128|25 60 * * ‘‘ Choice - º - - º - { { 4.50 270|00 36 “ “ Family - - -> - º - { { 4.75 171|00 70 “ Minnesota Patents - - º - { { 5.75 402|50 89 • { “ Minn. Bakers’ Grade - - Q- - & 4 5.14 457|46 79 “ Corn Meal, - - - * - - - sº - & 4 3.20 252|80 12 “ Grits, - sº - * ~ * - -- - & 4 3.10 37|20 10 “ Hominy, º - º - - es - 4 & 2.65 26|50 5 “ Rye Flour, - - º º - º - { { 3.90 19||50 9 “ Oat Meal, º-s, - - º - tº - & 4 5.75 51|75 211746 Discount 5% and 24%. - - - - - NotE 1.-The above bill was paid 6 days after the purchase was made. See page 457, for an explanation of Trade Discounts. When Trade Discounts are not allowed and bills are paid before maturity, interest for the unexpired time, usually allowed the payor. NOTE: 2. —For other bills covering many lines of business, see pages 283 to 295. Also, see pages 278 to 283, for Rapid Practical Methods of Bill Computations. 12. A dealer bought goods at 25% and 10% discount on list prices and sold at the list price. What was his gain per cent? Ans. 48#. OPERATION. Then the following Let $100 = the ſist price. Statement : 25 $100 = sales at list price. $ -ms 67# = cost of list price. 2 | 65 $ 75 ºsmºsºm- 135 | 2 7; $ 32} = gain on cost. 100 $ 67; cost. 48%. Ans. 13. A merchant bought an invoice of goods Sept. 15, 1890, amounting to $1200, on the following terms: “4 months or 5% discount in 30 days.” The bill was paid Oct. 13, 1890. How much was paid f Ans. $1140. 462 soul.E's PHILOSOPHIC PRACTICAL MATHEMATICS. * 14. Goods are bought at 33.4% and 20% discount from list prices, and sold at 30% and ſº off. What is the gain per cent? Ans. 184%. OPERATION. $100 = assumed as list price. 100 = assumed list price. 20 = 20% discount on purchase. 30 = 30% discount on sales. 80 70 26% = 334% discount. 7 = 10% discount on sales. $ 53% = cost. $ 63 = selling price of $53; cost. 53% = cost. $ 93 = gain on $53% cost. Then if $534 gain $93, what will 100 gain : Ans. 184%. 15. What % is gained by selling hats at $3 each, that cost $42 per dozen with 163% and 20% off? Ans. 284%. 16. What was the list price of goods that sold for $1.26 per gallon, with 50%, 10% and 12.4% discount Ans. $3.20. 17. A business man insured property to the amount of $65000 at 13%. He was allowed a rebate of 20% for cash. What was the net premium paid? Ans. $910. 18. A wholesale dealer sold a bill of goods amounting to $2000, on 6 months' credit. A few days after the sale had been completed, the purchaser proposed to pay the bill if the wholesale dealer would allow him 5 per cent discount. His prop- osition was accepted and the bill paid. What amount of money was required ? AnS. $1900. * OPERATION. $2000 face of bill. 100 = 5% discount. $1900 amount required to pay the bill. 19. At the same time that the above transaction was made, the same whole- Sale dealer sold to another merchant, on 6 months' credit, goods amounting to $3000. The purchaser of this bill proposed to pay $1900, provided the wholesale dealer would allow him 5 per cent discount on the amount of invoice paid, which the wholesale dealer agreed to do, and the $1900 was paid. What amount does the purchaser of the goods still owe ? Ans. $1000. PERCENTAGE. 463 OPERATION. $100 invoice assumed. Explanation.—The solution of this problem is 5 = 5% discount deducted. entirely different from the solution of the pre- ceding one. The discount being allowed on the * amount of the invoice that the money will pay, $95 cash. and as the whole of the invoice is not paid, we cannot therefore calculate on the $1900 cash or on the face of the $3000 invoice. We are hence 100 obliged to assume some number to represent pºse invoice, and upon which we may calculate and 95 | 1900 deduct the 5 per cent discount, and thus obtain * ====== the necessary equivalent values with which to 2000 make a solution. As shown in the operation, 3000 We assume $100 to represent the invoice, and then by deducting the 5 per cent discount we Rºssºmº have in the remainder $95 as the cash equivalent $1000 Ans. Aft of the $100 invoice; or as the amount of cash required to pay $100 invoice. Having these equivalent values, we place the $100 assumed invoice on our statement line, and reason thus: If $95 cash will pay $100 invoice, $1 cash will pay the 95th part, and $1900 cash will pay 1900 times as much. The result of which is $2000. Then having the amount of invoice that $1900 will pay or is equal to, we deduct the same from the whole invoice, $3000, and have in the remainder $1000, the balance that the purchaser still owes. 20. A country merchant purchased $4860 worth of goods, on 4 months' time. He then proposed to pay cash if the city merchant would discount 5 per cent on the face of the bill, which he agreed to do. What amount will pay the bill? Ans. $4617. 21. Another country merchant purchased $84.50 worth of merchandise, on 4 months' time. He then proposed to pay $5000 cash on condition that he be allowed 5 per cent discount on the amount of bill that he pays, which proposition is accepted. What amount does he still owe ? Ans. $3186.84. 22. A retailer bought from a wine merchant, on 3 months' credit, goods amounting to $1500, and from a grocery merchant, on 3 months' credit, goods amounting to $1800. But, without waiting for the maturity of the bills, he pays the wine merchant cash in full less 5 per cent discount allowed, and to the grocery merchant he pays $1000 cash, for which he is to be allowed 5 per cent discount on the amount of invoice that it will pay. How much did he pay the Wine merchant? And how much does he still owe the grocery merchant } * º Ans. $1425, he paid the wine merchant. $747.37, he owes the grocery merchant. 23. A debtor pays his merchant $500 in nickles at 74% discount. How much credit does he receive 3 Ans. $462.50. 24. A debtor owes $555 and pays the same in nickles at 73% discount. What amount of nickles will it take to pay the debt 3 Ans. $600.00. 464 SOULE's PHILOSOPHIC PRACTICAL MATHEMATICS. ºr 882. TABLES OF TRADE DISCOUNTS. TABLE No. 1. tº e tº te 4. g tº -- ſº gº ºt and : pret # prºt of or § prºct. º: 5 & & é & 3 & 4 7; & & & 4 92; & 4 & 4 10 & & é & 5 & 4 sº- 14% & & & 4 85} {{ * & & 10 “ “ 5 “ and 23 pr. ct. 16; “ “ 833; “ “ 15 & & 6 & 5 & t *-ºs * 19+ & & & 4 80% & 4 & 4 15 & 4 “ 10 & 4 * sº ; !, 10 and 5 pr. Ct. 20 “ 4 & 1. & 4 . - 20 “ “ 10 “ and 5 pr. ct 25 “ “ 5 “ c. v. $ 25 “ “ 10 “ <- *- 25 “ “ 10 “ and 5 pr. ct 25 “ “ 20 “ e. --. 25 º : 20 and 10 pr. ct. ; : : * : * 10 " and 30 “ ** 10 “ . : 23# & 4 & & 76# & 4 & 4 27# “ & 4 723; “ “ 24 & & & 4 76 4 & & 4 28 { { ( & 72 & ( & & 31} & 4 { { 68% ( & & 4 28% & 4 & & 71+ 4 & & 4 32+ & 4 & 4 67; ( & & 4 35} & & & & 64% & ( & & 40 & 4 & 4 60 4 & & 4 46 & 4 é & 54 ... & 4 & 4 481.5 & 4 s & 51.1% & & &&. 33} { { { { 66} & 4 & & 37 & 4 ( & 63 & ( & & 44 & 4 & & 56 4 & 4 & 47% & 4 & 4 52} 4 & 4 & 58 & 4 & 4 42 & 4 & 4 62} & 4 & 4 37} 4 & 64 64TÉd “ & 4 35 ºr “ “ 38+ & 4 ( & 61% & 4 & & 41% & 4 ( & 58% & ( & & 48 & 4 { { 52 { { & 4 53} & 4 & 4 46# ( & & 4 55% “ & 4 44%; “ “ 43 & 4 & 4 57° 4 & 6& 46 & 4 & 4 54 4 * 4 & 49 & 4 & 4 51 & 4 & 4 52 {{ é & 48 & 6 & 4 58 & & & 4 42 & & & & 62} & 4 & 4 37% & 4 & 4 66% & 4 & 4 33} 4 * * * 69}} “ & 4 30; “ “ 52} & 4 & 4 47% 4 & & 6 pr ct. 30 & 4 “ 20 4 & tº- 30 6 & “ 25 & 4 rºs 30 “ “ 25 “ and 20 pr. ct. tº 30 “ “ 25 “ “ 20 “ and 10 pr. ct, * § 25 º ** 20 tº “ 10 “ and 5 pr. ct. ; & 4 & 4 1. & 4 . . gº tºp 35 “ ** 20 tº º tº 35 “ “ 20 “ and 10 pr. ct. ; º º 20 . ** 10 “ and : & & * & 1; « : 40 & 4 “ 15 & 4 º 40 $4 “ 20 é & t- 40 6 & ** 30 & 4 * 40 “ “ 30 “ and 10 pr. ct 40 & 4 << 30 “ ** 20 ...” º § : : sº . “ 20 “ and 10 pr. ct. p I’, C t : 50 “ << 20 “ tºº sºs tº 60 4 & & 4 40 4 & 6 & 50 “ “ 30 “ 4-ºxº * -> tº 65 & 4 & 4 35 4 4 & 8 50 “ “ 30 “ and 20 pr. ct. gº 72 “ “ 28 tº “ 74% { { 4 & 25} é & & & 67." & 4 & 4 36 & 4 & 4 68 & 4 & 4 32 & 4 & & 71} & 4 & 4 28} & 4 & 4 76 & 4 { { 24 4 & & 4 82 & 4 & 4 18 4 & & 4 85% & & & 4 14% & 4 é & 86 & £ & 4 14 & 4 & 4 90 & 4 { { 10 & 4 & & 93 & 4 & 4 7 & 4 & 4 § “ “ gº tº * 20".” and 10 pr. ci. 60 “ << 10 & & sº sº º 60 “ << 20 “ sº sº 60 “ “ 20 “ and 10 pr. ct. 70 “ “ 20 “ * “º 70 & 4 § { 40 é & sº s 70 “ “ 40 “ and 20 pr. ct. 80 ** << 30 “ sº gº 80 4 & “ 50 ( & g-º tº 80 ** “ 50 “ and 30 pr. ct. 90 ** “ 5 “ ſº sº 90% { { é & 9} ( & & & 90 é & é & 10 4 { * - gºs 91 { { & 4 9 & & 4 & APPLICATION OF THE ABOVE TABLE. © A merchant sold goods amounting to $321.80, and allowed 20%, 10% and 5% off. What was the net amount of the bill? FIRST OPERATION. SECOND OPERATION. $321.80 313% the equivalent of 20%, — 10% and 5%. $321.80 $99.7580 = 31% of $321.80. 68%% = % on. 1.9308 = } of $321.80. ººmsºmºsºms *==ºssº $218.8240 = 68% of $321.80. $101.6888 = discount. i:373 - 6° 3321.80. $220.11 = net amount. $220.11 = net amount. ºr PERCENTAGE. 465 TABLE No. 2. * Mark your And your net Mark your And your net goods And take off gain goods And take off gain at a gain of will be at a gain of Will be 5 % 2 % 21% 96 50% 30 96 gain 5 % 10 “ 2} & 4 7+ 4 & 50 “ 25 & 4 12; “ 12} & 4 3 { { 9} & 4 50 “ 20 & 4 20 & 4 16; “ 5 & 4 10; “ 50 “ 15 & 4 274 “ 20 “ 10 & 4 8 “ 50 “ 10 & 4 35 “ 20 * * 7} & 4 11 & 4 50 “ 5 & 4 42% & 4 20 * * 5 & 4 14 (4 50 “ 20 & 10 * * 8 * * 25 15 & 4 6+ “ 60 “ 33; & 4 6; “ 25 10 & 4 124 “ 60 “ 30 & 4 12 * * 25 5 & 4 18#. { { 60 * * 25 & 4 20 é & 25 “ 10 & 5 “ 6; “ 60 “ 20 & 4 28 “ 25 tº 20 & 4 0 & 4 60 * * 15 & 4 36 ( & 30 “ 20 & t 4 * * 60 “ 10 & 4 44 * * 30 & 4 15 & 4 10} {{ 60 & & 5 & 4 52 & 4 30 “ 10 & 4 17 t < 60 * * 25 & 10 * * 8 * * 30 “ 5 4 & 23# & & 70 “ 40 & 4 2 & 4 30 * * 15 & 5 “ 4# “ 70 “ 33% & 4 13+ “ 33; “ 25 & 4 O & & 70 “ 30 & 4 19 & 4 33} & 4 20 & 4 6# & 4 70 4 & 25 & 4 27} & 4 334 “ 15 ( ; 134 “ 70 20 & 4 36 4 & 33; “ 10 & & 20 “ • 70 30 & 5 “ 13; “ 334 “ 5 4 & 26; “ 80 ‘‘ 40 & 4 8 & ! 33; “ 10 & 5 & 23 “ 11%; “ 80 * * 33; & 4 20 << 40 * * 25 & 4 5 * * 80 “ 30 & & 26 tº 40 * * 20 & 4 12 & 4 80 * * 25 & 4 35 tº 40 “ 15 & 4 19 < 80 “ 30 & 10 * * 13.4 “ 40 “ 10 { { 26 & 4 90 “ 40 é & 14 “ 40 * * 5 {{ 33 & & 90 “ 33% ( & 26; “ 40 * * 15 & 5 & 24 “ 10}#} “ 90 * * 30 & 4 33 44 50 * * 40 ** | lose 10 “ 90 * * 20 & 10 & 5 “ 29## “ 50 “. 33# & 4 0 & 4 APPLICATION OF THE TABLE. Goods cost $12.50 and were marked at 20% gain, from which a discount of 10% was allowed. What was the net gain per cent? Ans. 8% net gain. FIRST OPERATION. SECOND OPERATION Cost $12.50 $13.50 selling price. $ $100 cost assumed. 20% gain 2.50 12.50 cost price. 20% gain. •=s* =mmºns 1. -º- Marked price $15.00 $ 1.00 gain. 124 || 100 120 marked price. 10% discount 1.50 -*=- I ºmsm-me 12 = 10% discount. - =m- 8% -*-*= Selling price $13.50 $108 selling price. 100 COSt. $ 8 gain on a hundred and hence 8%. 466 soul E's PHILOSOPHIC PRACTICAL MATHEMATICS. TABLE NO. 3. If you are allowed And sell at the list If you are allowed And sell at the list a discount from rice a discount from price the list price of your gain will be the list price of your gain will be 2} % 23# 9% 33} % 50 % 5 & 4 5+ 4 & 40 { { 66# & 4 5 & 24 & 4 774; “ 40 & 20 & 10 “ 131}} “ 8% & 4 9 ºr & 4 50 { { 100 “ 10 & 4 # & 4 60 { { 150 “ 10 & 5 & 4 16+?? “ 66# { { 200 “ 12+ & 4 14% “ 70 & & 233; “ 15 & 4 17}} “ 75 & 4 300 “ 16# & 4 20 & & 80 { { 400 & 4 20 & 4 25 “ 20 & 10 & 5 “ 46 ºr “ 25 & 4 33; “ 30 & 4 42% & 4 APPLICATION OF THE TABLE. The list price of goods is $500, from which 5% and 24% discount is allowed. What is the net gain per cent 3 Ans. 7###%. FIRST OPERATION. SECOND OPERATION. $500 list price. $100 list price assumed. 25 = 5%. 363 x 100 5 = 5%. 8 || 59 — = 7###%. 741 || 8 $475 463; 95 100 11% = 24%. 23 = 24%. *- 7###%. $463; cost. 923 = cost. 500 $73 - gain. $ 36% gain. 883. TO FIND TEIE VALUE PER, TON OF SUGAR CANE. NOTE.-For the leading facts regarding this new subject, we are indebted to Prof. W. C. STUBBS, Director of the La. Sugar and State Experimental Station. The method to determine the price to pay for Sugar Cane, varies somewhat with different manufacturers of Sugar in the different sections of the sugar district of Louisiana. The following Work and statements will make clear the various methods used: Cane is usually sold upon three factors, as follows: 1. The price of refined sugar in the New Orleans Market. 2. The percentage of juice extracted from the cane. 3. The percentage of Sucrose contained in the cane. The value of the cane being a certain price for each of these elements or factors. The element of glucose is not considered as a material factor in determining the price of cane. The average chemical analysis of Louisiana sugar cane is about 90 per cent juice and 10% fibre. The juice contains on an average about 12% of sucrose, 2% of glucose, and 1 to 14% of solids not sugar. NOTE.-Very few of the sugar mills in Louisiana extract more than an average of 72 to 80% of juice from the cane. A} PERCENTAGE. 467 QUANTITATIVE CHEMICAL ANALYSIS. 884. Quantitative Chemical Analysis consists in the determination and Separation of the relative proportions of the component parts of which a substance is composed. The Several parts are expressed in per cents of the whole substance. In the analysis of cane juice, molasses, etc., the component parts are, as above stated, Sucrose, glucose and solids, that are not sugar. The Co-efficient of Purity is the per cent that the sucrose is of the whole Solids extracted from the cane. Thus, as in the above average in which the juice contains 12% sucrose, 2% glucose and 14% of foreign solids, not sugar, the co-efficient of purity is found as follows: 12 + 2 + 1} = 15%; then, 12 × 100 + 15% = 77.4%%. The Glucose Ratio is the per cent that the glucose is of the sucrose, and is found as follows, using the above figures: (2 × 100) -- 12 = 163%. The per cent of the foreign element, not sugar, is found as follows: (14 × 100) —H· 15; F 9:4%. Fertilizers. The component parts generally considered in fertilizing sub- stances are potash, nitrogen and phosphoric acid. THE SUCROSE MULTIPLIER AND BIOW TO FIND IT. 885. A Sucrose Multiplier is a number used as a factor in determining the price of sugar cane per ton, and is found as follows: 1. By estimating an average extraction of 80% of juice from the cane. 2. By allowing the seller 50% of the juice. 3. By estimating an average of 10 or 10.4% sucrose in the juice. FIRST OPERATION TO FIND A SUCROSE MULTIPLIER. 886. 1 ton of cane = 2000 lbs. 80% of which is 1600 lbs. of juice. } of -1600 lbs. = 800 lbs. juice allowed the seller. (800 or 8 is the sucrose multiplier). Hence, 800 x 104% x 42, the price of Prime Yellow Clarified Sugar in New Orleans, gives $3.36, the price per ton of cane. These figures practicalized become 8 × 104 × 4 = $3.36, the price per ton of cane. (The two 0's in the 800 and the % in the 10.4% being omitted). SECOND OPERATION TO FIND A MULTIPLIER. 887. 1 ton = 2000 lbs., 80% = 1600 lbs. -- 2 = 800. 10% of 800 = 80 pounds sucrose or sugar. (80 is the sucrose multiplier). 468 soul E's PHILOSOPHIC PRACTICAL MATHEMATICS. º Hence, 80 × 42, the price of Prime Yellow Clarified Sugar in New Orleans, is $3.20, price per ton of cane. Or thus, 10.4% of 800 = 84. Hence 84 x 42. = $3.36, the price per ton of CaLle. NotE.-According to the above method and principles, when the 9% of juice extracted is 70, T2, 75,833, 85 or 90, the multipliers would be respectively, 7 or 70, 7.2 or 72, 7.5 or 75, 8.34 or 83; 8.5 or 85 and 9 or 90 according to the method used. In like manner, the sucrose multiplier would be found for other per cents of juice extracted. PROBLEMS. 888. 1. If the price of refined sugar in New Orleans is 4 cents per pound and the percentage of sucrose in cane juice is 12 per cent, what would be the value of cane per ton, at 8 cents per unit of sucrose and of the price of refined sugar 3 Ans. $3.84. OPERATION. 82 x 12 x 4 = $3.84 per ton. 2. A purchaser of sugar cane says to a planter “I will give you 84 cents per unit for each 1% of sucrose that your cane contains, multiplied by the price per pound of refined sugar in New Orleans.” If the price of refined sugar in New Orleans is 442 per pound, and the juice contains 13.4 per cent sucrose, what is the value of the cane per ton ? Ans. $5.1255. OPERATION. 2 9 w 13.4 84 × 13.4 × 44 = $5.125-1- or thus, 2 17 *- NOTE.—The 84 cents per unit in this problem, equals 85% (8.5) of juice extraction as shown by the above operations to find multipliers. 889. Another method of determining the price of sugar cane per ton. In some of the sugar districts of Louisiana, the price or value of cane per ton is determined by allowing 834 pounds of sugar per ton of cane, and to estimate its value at the New Orleans market price for Prime Yellow Clarified Sugar. PROBLEMIS. 890. 1. Allowing 834 pounds of sugar per ton of cane, what is cane worth when Prime Yellow Clarified Sugar is quoted at 3:42 in New Orleans? Ans. $2.653. OPERATION. 16 51 3 250 3 * x 83% = $2,653 or, thus: * | $2.65; NOTE:-The 83% multiplier here used, is found as shown above in the “second operation to find a multiplier,” substituting 834% for juice extracted in place of the 80 there used. 2. Using the figures of the above problem, and allowing that sugar cane in Louisiana will produce 152 pounds per ton, what would be the gross profit to a mill that purchased and manufactured 4162 tons of cane. No allowance to be made for molasses or syrup that may be made, or the expenses to manufacture, or to market? Ans. $9109.573. NoTE.—150 to 160 pounds of sugar per ton of cane is the average for the cane of Louisiana. * PERCENTAGE. 469 OPERATION TO FIND COST. OPERATION TO FIND SALES AND GAIN. 16 || 51 16 || 51 3 || 250 152 $20164.89 4.162 © 4.162 11055.31% $11055.31#. $20164.89 $ 9109.57; gain. f NoTE.-Some purchasers of sugar cane pay per ton, 80 cents, more or less, for each cent o the New Orleans price of Prime Yellow Clarified Sugar. This 80 which is used as a multiplier of the New Orleans price of Prime Yellow Clarified Sugar, is found as above shown in the second operation, Article 887, to find a multiplier allowing 80% of extraction of juice from the cane. 3. How many pounds of sucrose are there in 1840 pounds of cane juice which contains 13.6% of sucrose? Ans. 250.24 pounds. 4. If the solids in the juice of cane are 154%, and the co-efficient of purity is 77%, how many pounds of sucrose are there in 660 gallons of juice, allowing 84 pounds to the gallon ? Ans. 660 pounds of sucrose. FIRST OPERATION. SECOND OPERATION. 774; x 15.4% = 12% sucrose. 660 × 8% = 5500 pounds. 660 × 8% = 5500 pounds of juice. 5500 x 15% = 852.50 lbs. solids. 5500 × 12% = 660 pounds of sucrose. 852.50 × 774% = 660 lbs. sucrose. 5. In the above problem, if the glucose ratio was 163%, how many pounds of glucose would there be? Ans. 110 pounds. OPERATION. 163 × 12 = 2% glucose. 660 × 8% = 5500 pounds of juice. 5500 × 2% = 110 pounds of glucose. 6. How many pounds of sucrose are there in 64 tons of sugar cane, of which 864% is juice, and 14% of the juice is sucrose? Ans. 1513.75 pounds. OPERATION. 2000 x 64 = 12500 lbs. × 864% = 10812.5 lbs. × 14% = 1513.75 lbs. 7. From 8 tons and 500 pounds of cane which was passed through a mill, the bagasse weighed 3630 pounds. What was the per cent of juice extracted? Ans. 78%. OPERATIONS INDICATEI). % % 100 16500 100 16500 || 3630 3630 16500 | 12870 tº- Or thus: * == 12870 78% iſ: } 78% extracted. 8. 12870 pounds of juice were extracted from a lot of cane; if the rate per cent of extraction was 78%, how many tons of cane were there? Ans. 84 tons. OPERATION INIDICATET). 100 78 || 12870 2000 or (12870 × 100) -- (78 x 2000). 47O SOULE's PHILOSOPHIC PRACTICAL MATHEMATICs. A. 9. In 53 tons of cotton seed fertilizing meal, there is 64% of nitrogen, 34% of phosphoric acid and 1.9% of potash. How many pounds of each fertilizing ingredient? Ans. 715 pounds of nitrogen. 343.75 pounds of phosphoric acid. 209 pounds of potash. 10. In 3400 pounds of bone meal, there are 1604 pounds of nitrogen and 790.8 pounds of phosphoric acid; what is the per cent of each 3 Ans. 4.72+ 7% nitrogen. 23.26+ 7% phosphoric acid. MISCELLANEOUS PRACTICAL QUESTIONS IN PERCENTAGE. 891. 1. Bought 6 dozen shirts for $129.60; at what price per shirt must I sell them to gain 50 per cent? Ans. $2.70. 2. Bought butter at 48% per pound; at what price per ounce must I sell it to gain 663 per cent 2 Ans. 52. 3. Bought flour at $7.50 per barrel; at what price must it be sold to lose 10 per cent 7 Ans. $6.75. 4. Sold goods to the amount of $250 and gained 25 per cent; what did they cost 3 Ans. $200. 5. Sold flour at $8.75 and lost 124 per cent; what did it cost? Ans. $10. 6. Bought gloves at $4 per dozen and sold them at 50¢ per pair; what per cent did I gain } Ans. 50%. 7. My merchandise account is debited $145000 and credited $174000. There is no merchandise on hand. What per cent did I gain on sales? Ans. 20%. 8. My merchandise account is debited $280000 and credited $314212.50. I have on hand $44797.50. What is my gain, and what my gain per cent? Ans. $79010 gain. 28%, 7% gain. 9. A manufacturer received a certain quantity of cotton to be manufactured into cloth. He delivered 182400 yards, averaging 44 ounces to the yard. Allowing that there was a waste of 10 per cent in manufacturing the cloth, how many pounds of cotton did he receive 3 Ans. 57000 pounds. 10. I sell a certain quality and style of goods at $1.10 per yard. My rival in trade sells the same quality and style at $1.25 per yard. What per cent more does he charge for his goods than I? Ans. 13+%. 11. I sell flour at $8.50 per barrel. My rival sells the same kind at $9. What per cent less do I sell than he 3 Ans. 53%. 12. The assessment of real and personal property of a city amounts to $150,000,000. The various expenses of the municipal government are approximated and estimated at $2,800,000. What must be the per cent tax assessment to raise the amount 3 & Ans. 1.863%. 13. Sold coffee at 18% per pound and lost 10 per cent; what should I have sold it for, to have gained 124 per eent? gº Ans. 22#2. 14. Sold butter at 35% per pound and gained 25 per cent; what per cent would I have gained, had I sold it at 30% Ans. 74%. º: PERCENTAGE. 47 I 15. Sold potatoes at $3.15 per barrel and lost 10 per cent; what per cent would I have gained or lost had I sold them at $4.3 Ans. 143% gain. 16. Sold rice at 842 and lost 84 per cent; what per cent would I have gained or lost, had I sold it at 92% Ans. 1% loss. 17. Whiskey which cost 90% per gallon is compounded with water in the proportion of 1 gallon of water to 2 gallons of whiskey, and the mixture is sold for 85% per gallon. What % is gained? Ans. 413%. 18. A faithful accountant, who was receiving a compeusation of $720 per year, had his salary increased 33.4%. How much does he now receive per month ? Ans, $80. 19. A firm paid $300 per month rent and was raised 25%. What is their Current rent 3 Ans. $375. 20. A clerk who received $125 per month had his salary reduced 20%. What did he then receive? Ans. $100. 21. A man who was receiving $2.50 per day demanded an increase of 40%. What would then be his daily pay? Ans. $3.50. 22. A merchant compounded, in equal quantities, sugar that cost 62 per pound and sugar that cost 92 per pound. He then sold the mixture at 9% per pound. What % did he gain? Ans. 20%. OPERATION: * g 1 pound at 62 = 62. 92 sales. 2 || 3 gain. 1 “ “ 92 = 92. 7.4% cost. 15 2 -ºmsºmºms -ms-smºs 100 2) 15g. 14% gain. smºs | 1 lb. of the com- 20% gain, Ans. pound cost 7#2. 23. A merchant bought 5 bbls. flour for $40 and sold the same at a gain of 25%. He then bought 5 bbls. more for $40, and sold the same at a loss of 25%. Did he gain or lose, and if so, how much? Ans. He neither gained nor lost. OPERATION. $40 cost 2 25% gain gives $50 selling price = $10 gain. $40 sold Ø 25% loss gives $30 selling price = $10 loss. 24. A merchant sold 5 bbls. of inferior flour for $40 and gained 25%. He then sold 5 bbls. of superior flour for $40 and lost 25%. Did he gain or lose in the two transactions, and if so how much Ans. He lost $5}. OPERATION. $40 selling price @ 25% gain gives $32 cost = $ 8 gain. $40 selling price @ 25% loss gives $53; cost = $134 loss. Net loss $ 5; 25. A grocer bought 10 barrels of molasses each containing 40 gallons, at 50g per gallon. He then put the molasses in kegs holding 8 gallons each, and sold the same as 10 gallon kegs, at 55% per gallon. What was his profit, and what % did he gain? Ans. $75 profit; 37; % gain. 472 soul E's PHILOSOPHIC PRACTICAL MATHEMATICS. jºr * 26. A fruit dealer sold 4 peaches for 52 and gained 564%. What % would he have gained by selling 5 for 6 cents? Ans. 50%. OPERATION. 5 -- 4 = 1.4% 100 cost. 6 -- 5 =1}g 5 2 156+ sales. 156} | 1.4 4 || 5 5 4 **- 1#2—#2 =#2 gain. | 100 Or 5 || 6 #2 cost. - * : -º-º-º: 50% gain. 150 sales, 50% gain. 27. A. and B. are two merchants; they desire to barter rice and sugar. A. has rice, market value 6% per pound; and B. has sugar, market value 82 per pound. At the time of the exchange or barter, A. suggests to B. that in order to influence the market reports, he will place his rice at 72 per pound and that B. shall advance his sugar accordingly. B. accepts the proposition. What should be the exchange price of B's sugar 3 Ans. 9% per pound. FIRST OPERATION. 2 72 = exchange value of 1 = gain. 82 = market value of A’s rice. 6 || 100 B’s sugar. 6% = market value of - º- 1$2 = 163% gain. ººmsºmº Sal ſle. 163% gain, &= - 12 = the increase on 94% = exchange value. Sal (162. SECOND OPERATION Ž ** 6% + 12 = 7 g = exchange 1 = gain on 62. 82 + 142 = 9.4% = exchange value of A's rice. 6 || 8 value of B's sugar. 13% gain on 82. 28. Suppose in the above problem, that B. had proposed to reduce his sugar 12 per pound, and that A. should reduce his rice accordingly. What would be the exchange price of A's rice? Ans. 542. 29. Suppose in the above problem that A. and B. had each raised 1g on the market value of their rice and sugar, how much 7, would B. have lost, and what % would A. have gained ? Ans. B. would have lost 34%. A. “ “ gained 3}#}. OPERATION TO FIND B's 'ſ, Loss. g 94% = correct exchange value, as above. 3 || 1 = loss. 9 g = incorrect exchange value, as supposed. 28 3 100 #2 = loss by incorrect exchange value. - 34% loss. Ans. PERCENTAGE. 473 OPERATION TO FIND A's 9, GAIN. g 92 = B's selling price. 72 = A's incorrect selling 4 || 1 = gain. 8 || 6 price. 27 | 4 gººmsº 6#2 = A's correct selling 100 6% = price A, should price. emº have sold for when #2 = A's gain by selling 3}}% gain, Ans. B. sold for 99. at 72. EXPLAN ATION FOIR, SECOND OPERATION. 82, B's market value -- 12 = 92, B's selling price; then since 8% sell for 92, 12 will sell for the # part, and 6¢, A's market value, will sell for 6 times as many, which is 6% ; then since A. really sold for 7%, when he should have sold for 632, he gained 72 — 632, - #2; and if 632 gain #2, 12 will gain the 63 or *# part, and 100% will gain 100 times as much, which is 34% or %. 30. Bought coffee at 152 per pound and sold it at 182; what would I gain by buying $75 worth of coffee? Ans. $15. 31. Bought coffee at 15% per pound and sold it at 182; what did I gain by selling $75 worth of coffee? Ans. $12.50. 32. A real estate speculator sold a house and lot for $7200 and gained 20 per cent. He then invested the $7200 in merchandise which was sold at a loss of 20 per cent. Was the result of the two transactions a gain or a loss, and if so how much 3 Ans. $240 loss. 33. A grocer sold a barrel of flour for $8.75 and gained 25 per cent. Soon after he sold to the amount of $637 and gained 40 per cent. What was the last sold at per barrel, and how many barrels were there ? Ans. 65 bbls. at $9.80. 34. A general fought 32 battles and lost 4; what per cent did he lose? Ans. 12.4%. 35. A merchant has due him $32 and compounds the claim on receipt of $4; what per cent did he lose ? Ans. 874%. 36. The net assets of a bankrupt are $72000, which is 15 per cent of his liabilities. What are his liabilities? Ans. $480000. 37. A person pays $30 per month for board, which is 20 per cent of his monthly salary. What is his salary per month # Ans. $150. 38. Paid $6 for sawing 4 cords of wood, which was 163 per cent of its cost; what did it cost per cord 3 Ans. $9. $ cost. STATEMENT. 100 50 || 3 4 || 6 39. A mother having a basket of oranges, took out 50 per cent for her children; of these she gave 40 per cent to her son who gave 4 of his to his sister, who thus received 3 oranges. How many were there in the basket at first 3 Ans, 30. OPERATION. Let 100% = the whole number of oranges. 50% = the number taken out. 474 SouLE's PHILOSOPHIC PRACTICAL MATHEMATICs. & 40% of 50% = 20% = given to son. # of 20% = 10% given to sister. 10% = 3 oranges. Then 10% = 3, 100% = 30 oranges, answer. Or thus: 100% = the whole number of oranges. 50% = } taken out. 40% = # of 3 = } given to son. # x + = Pºs = given to sister. Pö = 3, then }} = 30, the whole number in the basket. 40. A merchant marks his goods 50 per cent above cost, but supplies his Wholesale patrons at 25 per cent discount on retail price; what per cent gain does he make 3 Ans 12#7%. 41. A merchant asked for a lot of goods 20 per cent more than they cost, but they being a little soiled he closed the sale at 10 per cent less than his asking price, and realized a profit of $112. What did his goods cost? What was his asking price? And for how much did he sell them 3 Ans. $1400 cost. $1680 asking price. $1512 was the selling price. $100 - $ 20 | 100 * 8 112 $120 OPERATION. 12 $1400 cost. *=== 280 = 20% gain. $108 100 $1680 asking price. t-ºm-º-º-º: 168 = 10% discount. $8 gain. $1512 selling price. 42. If the cost of purchasing and shipping goods from New York to New Orleans is 8 per cent on New York cost, what per cent gain on the first cost must the New Orleans merchant mark his goods in order to clear a profit of 334 per cent On full cost $ What amount of goods must he purchase to make a net profit of $35200 on New York cost? What would be his New Orleans profit? Ans. He must mark his goods at 44% gain. He must purchase $80000 worth. His N. O. profit would be $28800. OPERATION. $100 cost. 8% freight. $ C $80000 | 199 25 8% $108 total cost. 11 A4 lºgg 2200 *=s* 36 == 334%. — $ 6400 | $S{}900 Ans. 35200 $144 N. O. S. P. * = º 100 $28800 Ans. $44 gain. 43. A merchant bought flour at $7.50 per barrel; what must he ask for it in order that he may fall 10 per cent on his asking price and still gain 20 per cent on the cost 3 s Ans. $10. PERCENTAGE. 475 44. Sold a barrel of oranges for $6 and gained 20%. I then invested the $6 in merchandise which I sold at a loss of 20%. What was the gain or loss by the two transactions? Ans. 20% loss. 45. How many apples must I buy so that after allowing 25% of them to be eaten and 20% of the remainder to be given away, I may sell just 1 dozen : Ans. 20 appleS. OPERATION. 100 100 100 75 | 12 80 | 16 Or, 75 100 e- *E-º: 80 | 12 16 20 Ans. 46. A fruit dealer has pears worth 52 a piece, but will sell 6 for 25%. What per cent would be gained by buying 6 for 25¢, and what per cent would be lost by buying them separately at 52 each 3 Ans. 19, 20% gain; 29, 163% loss. OPERATION, 6) 25 6 | 5 6 || 5 T4, 4}=25 | 6 5 | 100 5 100 &= assºmsºmºsº-ºº: tºº *=se smºº 163% loss. # 20 %gain. 47. The New Orleans Cotton Factory receives from a merchant 200 bales or 90000 lbs. of Muddling Cotton to manufacture into sheetings. The charge for man- ufacturing is fixed at 3% per yard, and payment is to be made in Middling Cotton at 12g per pound. After the factory had delivered to the merchant 128800 yards, which weighs 8 ounces per yard, a settlement was made on the following conditions: It was agreed to allow 123 per cent waste in the manufacture; the factory to retain the cotton remaining on hand at 12%., and for the balance due to receive Middling fair instead of Middling at 13%. How many pounds of cotton does the merchant owe the factory? Ans. 14584* lbs. OPERATION. 100 128800 yds. 874 64400 16 || 8 73600 lbs. Cotton. | 64400 lbs. 90000 {{ 4 * g 128800 yards. 16400 lbs. on hand. 3 gº 12g *=º $3864.00 $1968.00 1968.00 $1896.00 - 13 g = 14584; lbs. 48. Bought coffee at 192 per pound; allowing that it will fall short 5 per cent in weighing and that 8 per cent of the sales will be uncollectible, for how much per pound must it be sold to clear 15 per cent profit on the cost 3 Ans. 252. 476 SOULE's PHILOSOPHIC PRACTICAL MATHEMATICs. º OPERATION. 192 cost. 100 100 115 100 | 115 285 = 15% gain. 95 | .2185 92 || 23 or, 95 || 100 or, 95 || 100 *º-º-º: *mºmº || º gº ºmºmºmºmºmº 92 || 19 92 || 100 .21852. 23%. | 252 Ans. | 19 49. A grocer bought at auction 200 baskets of champagne for $2800; what must he ask for it per basket so that he may discount 20 per cent on his asking price, allow 10 per cent of sales to be bad pay, 24 per cent commission for selling, and still make a profit of 100 per cent on cost 3 Ans. $40. FIRST OPERATION. SECOND OPERATION. $ 1998 100 $ 7 2009 80 | 200 100 374 || 250 109 || 23%. 2 874 100 8% 200 2% 100 2800 ** 2000 *m-emsmº, 200 |250 $40 Ans. *= 285} $40 Ans. 50. A merchant sold a pair of shoes for $3.20 and lost 20 per cent; what should he have asked for his shoes in order that he might have fallen 20 per cent on his asking price and still have gained 20 per cent on cost 3 Ans. $6. 51. If it costs a merchant 5% over factory price, to put his goods in store, and he sells at a profit of 10% on full or store cost, and pays a tax of 35% on sales, how much goods must he sell per month, to equal a salary of $300 per month And what % did he actually make on the goods, including tax as part of cost, in addition to the 5%; and what amount must be invested to produce $300 per month? Ans. 10.44225% net gain on store cost. $3316.59-i- monthly sales. 9,938+ 9, gain including tax as part of cost. $2872.94-H amt. to be invested to gain $300 per mo. OPERATION. $100 = assumed factory cost. Cost at factory, $100. 5 = 5% to place goods in store. Charges to put in Store, 5. -º-º-º-º: Tax on sales, .05775 $105 = Store COSt. gºmºsºme 10.50 = 10% gain on store cost. Total cost, $105,05775 $115.50 = selling price. .05775 = }o'ſ, tax on sales. 10.44225 tºº-ºº: 105.05775 100 $115.44225 = net receipts after paying tax. 105. = Store cost. 9.939-1-7% gain including &== tax as part $ 10.44225% net gain on store cost. of cost. $ 115,44225 100 COSt. 10.44225 || 300 115,44225 || 33.16.591.25 $33.16.59125 monthly sales. | $2872,94+ amount to be in- vested to gain $300 per mo, PERCENTAGE. 477 52. An importer sold # of a cargo at 224% gain; # of remainder at 163% gain; # of remainder at 10% loss, and the balance at 40% loss. There was a profit of $335.40 on all. What was the cost of the cargo? Let $100 Then # Then # of remainder Then + of remainder Then the remainder # # = } # = OPERATION. = the cost of cargo. $334 at 224% gain 33% at 163% gain 8; at 10 % loss 25 at 40 % loss –4– 12. : $100 Then $13;g — $10# = $2% net gain. Then $2; : $100:: $335.40 : $15093 cost of cargo, : or thus, $ 73 gain. 5; gain. # loss. 10 loss. $13+g gain $10; loss 100 9 • 335.40 20 $15093,00 53. A live stock dealer sold two horses, each at the same price. On one horse he gained 25 per cent, and on the other he lost 25 per cent. His net loss on the two sales was $12. OPERATION.—FIRST PART. What was the cost of each horse ? • Ans. $72 cost of 1st horse. $120 cost of 2nd horse. 1. Assume that one horse was sold for $100 at a gain of 25 per cent. 2 { % {{ Then find the cost of each horse, and {{ {{ { % then proceed as shown in the operation. Statement to find cost of first horse sold a 25% gain. 100 100 $80, cost 1st horse. 125 Statement to find cost of the two horses actually sold. $ 213; 13; 12 lsº OPERATION.—SECOND PART. Statement to find cost of second horse sold Ø 25% loss. 100 75 100 | $1334, cost 2nd horse. OPERATION —THIRD PAR.T. Statement to find cost of first horse actually sold $ 0 2134 || 192 S $ 2 | $7 $100 at a loss of 25 per cent. the loss on the sale of both horses; $ 80 cost first horse. 133; cost second horse $213; cost of both horses 200 assumed selling price of both horses, $13; loss on sale of both horses. Statement to find cost of second horse actually sold $ 1334 2134 || 192 | $120 478 SOULE's PHILOSOPHIC PRACTICAL MATHEMATICs. º - - SECOND OPERATION. Let $1 = sale. # or 25% gain on cost = } or 20% on sales. # or 25% loss on cost = } or 334% on sales. OPERATION FOR FIRST HORSE. OPERATION FOR SECOND HORSE. $1 = selling price. $1 = selling price. less # = 25% gain on cost. plus # = 25% loss on cost. $# = cost of 1st horse. $14 = cost of 2nd horse. # loss — # gain = # = $12. $ then, $90 = sale of 1st horse. 12 — # = 18 = gain on 1st horse. 2 || 15 *=º dº I am-ems $72 = cost of 1st horse. | $90 = sale of each horse. and, $90 = Sale of 2nd horse. + # = 30 = loss on 2nd horse. $120 = cost of 2nd horse. 54. A. and B. are both merchants and conducting the same character of business. A. desires to exchange or barter $9384 worth of sugar for $9384 worth of rice. The market price of the sugar is 11g, and the market price of the rice is 82. B. agrees to exchange but suggests to A. that each increase #2 per pound with a View to ingrease the market price for future sales; to this proposition A. cheerfully assents, and they exchange at the increased price, A's sugar at 114% and B's rice at 842, to the amount of $9384 worth. By reason of the #2 increase of price, what was their respective gain and loss in dollars; their gain and loss in pounds; their relative gain and loss per cent; their respective real gain and loss per cent, and their respective real investment; and at what price should A. have valued his sugar in order not to have lost by the #2 increase on B's rice # * Ans. $144, A’s loss and B's gain. 1309 fºr lbs. sugar, or 1800 lbs. rice, A. loses and B. gains. 1}}} %, A's relative loss per cent and B's relative gain per cent. 1}}#%, A's real loss per cent. 1#%, B's real gain per cent. $8976, A’s real investment. $8832, B's real investment. 11++g, A. should have valued his sugar, in order not to have lost by reason of the #2 increase on B's rice. * , PERCENTAGE. 4.79 CONTRACTED SOLUTION. $9384.00 - 11c. = 85309 ºr lbs. sugar, should exchange, A. sells and B. receives. $9384.00 - 11}c. = 81600 lbs. sugar, did exchange, A. sells and B. receives. ~ 3709 ºr lbs. sugar, A. gains and B. loses. $9384.00 -- 8c. = 117300 lbs. rice, should exchange, A. receives and B. sells. $9384.00 + 84c. = 110400 lbs. rice, did exchange, A. receives and B. sells. 6900 lbs. rice, A. loses and B. gains. Then, 6900 lbs. rice at 8c. = $552 = A's loss and B's gain. And, 3709 ºr lbs. sugar at 11c. = $408 = A's gain and B’s loss. $144 = A's net loss and B's net gain. Also, #c. —– 11}c. = ºrc. = A's gain and B’s loss on 1 cent of Sugar. And, #c. - 84c. = Tºrc. = A's loss and B's gain on 1 cent of rice. Then, *fc. — ºc. = 33rc. = A's net loss and B's net gain on 1C. of barter. Which equals 1}}}% = A's relative loss per cent and B's relative gain per cent. The real gain and loss per cent is found by increasing the relative per cent in the same ratio as the original prices, thus: #TC. × * = +37c. = 1+}}% = A's real loss per cent. #I c. X * = rºzc. = 13.3% = B's real gain per cent. Also, $9384 × #= $8976 = A's real investment. And, & $9384 x + = $8832 = B's real investment. Then, $144.00 – 11c. = 1309 ºr lbs. sugar, A's loss and B's gain. And, $144.00 – 8c. = 1800 lbs. rice, B's gain and A’s loss. Finally, B's ic. increase on 8c. = 64%; and 64 per cent increase on A's 11c. = +&c. which added to the 11c. gives 11+}c. as the price that A. should have valued his sugar in making the exchange. Tºsº (g --- sº e º e º ºs e º 'º e º a s e s = • * * * * ºr e º sº e º 'º - e º 'º e º e s tº s is e º e º a s e s is e º s = e º 'º e s a sº e º ºs e º 'º a s m e º e º e s ∈ e s is a se –4% Çommission and Brokerage. 892. Commission is the sum charged by an agent, factor, correspondent, broker or commission merchant, for buying or selling goods or other property, col- lecting debts, making advances, or transacting other business of a similar nature. The sum thus charged is generally calculated at a certain rate per cent on the amount of purchase, sale, collection, advance or other business transacted. The party who transacts business for others is called the Agent, Broker, Collector, Solicitor, Factor or Commission Merchant, according to the nature Of the business. The party for whom the business is transacted is called the Principal. NOTE 1.-Commission merchants or factors are usually placed in possession of the goods bought or sold, and transact the business in their own names. NOTE 2.—Brokers do not have possession of the goods or securities bought or sold and transact the business in the name of those who employ them 893. A Consignment is a quantity of goods shipped or sent by one person or firm to another, to be sold on commission for account of the shipper. 894. A Consignor is a person who ships goods to be sold on commission for his account. 895. A Consignee is a person to whom goods are shipped to be sold for account of another. 896. An Account Sales, is an itemized statement rendered by the Agent or Consignee to the Shipper or Consignor, showing in detail the sales, the charges of all kinds, and the net proceeds. 897. The Net Proceeds of a consignment is the amount of money due the consignor, after deducting from the total sales the commission and all other charges. 898. An Invoice or Account of Purchase is an itemized statement rendered by an agent to his principal showing in detail the cost of goods bought, per his order, and all expenses or charges attending the purchase and shipment. 899. Guarantee is a promise or obligation by which the Agent becomes surety for the payment of goods sold on credit, or for the performance of some duty devolving upon others, or for the grade or quality of goods bought ; and for this responsibility a percentum charge is made, which is called Guarantee. THE RATE OF COMMISSION OR BROKERAGE TO CELARGE. 900. The rate per cent commission or brokerage charged for transacting business for others is usually fixed by the Chamber of Commerce or Board of Trade (480) * - COMMISSION AND BROKERAGE. - 481 of the city where the person transacting the business resides, and it varies according to the nature and extent of the business. A less per cent is generally charged by a broker than by a commission merchant, because the work he has to perform requires less labor and time, and his expenses for office rent, services, employees, etc., are less. The charges made by brokers, and the customs regulating the business of brokers, are generally fixed by the Association of Brokers and will be fully presented under the subject of Stocks and Bonds. We will here remark that it is the custom of the New Orleans brokers to charge on the par value of Gold, Stocks or Bonds, and not on the purchase or selling price, except as follows: on stocks selling at and over $50 per share, 50g per share is charged; on stocks selling at $25 and under $50 per share, 25g per share is charged; and for stocks selling below $25 per share, a charge is made as per agreement. 901. The principles of percentage apply to Commission and Brokerage and the questions are classed as follows: TO FIND THE COMMISSION WEHTEN THE COST AND SELLING PERICE AND TEIE PER CENT OF COMMISSION OF BROKERAGE ARE GIVEN. PIROELEMS. 1. An agent sold $2845 worth of goods and charged 2 per cent commission. What was his commission ? Ans. $56.90. OPERATION. $28.45 Explanation.—As was explained in the first problem of 2 percentage, page 435, we first get 1 per cent by dividing by 100, whichi. done by placing the º point º º tens and hundreds figures, then 2 per cent by multi- $56.90 Ans. plying by 2. gures, 2. Sold 5 hlids., 5418 lbs. Sugar at 1242, and charged 24 per cent commission. What is my commission and what is the net proceeds? Ans. $16.93 commission. $660.32 net proceeds. OPERATION. # 541890 Explanation.—In this problem, we first find the value of the sugar, and then the 2% per cent of the same. The 677.250 Val f multiplication of the 5418 lbs. by 12} is performed, for $677. value or Sugar. reasons given on pages 94 and 436, by multiplying by 100, as amsºmºsºmºsºmºmºs indicated by the two small naughts, and then dividing by $ 16.93 = 24% commission. 8; or to express it differently, divide by 8 and carry, the : º P. 1. º : º º 2% i. performed by multiplying by 10, as indicated by the Sma $660.32 net proceeds. :*.*,gººd then *ś by 4, as explained on pages 4 and 436. 3. A real estate agent sold a piece of real estate for $28465, and charged 1 per cent commission. What was his commission ? Ans. $284,65. 482 SOULE's PHILOSOPHIC PRACTICAL MATHEMATICs. * 4. Collected $2842.80, and charged 14 per cent brokerage. What is my brokerage f Ans. $35,534. 5. Bought 20 shares of $100 each of Germania Bank stock, at 15 per cent premium, and paid brokerage 3 per cent. What was the brokerage and the cost of the stock # Ans. $10 brokerage. $2310 cost of stock. . NotE.—Brokerage is charged on par value. 6. Exchanged through a broker $1280 fractional currency for bills of a larger denomination. The broker charged + per cent; what was the brokerage 3 Ans. $3.20. 7. Sold 208 shares of $100 each of National Bank stock at $924. Paid brokerage 4 per cent. What are my net proceeds, and what is the brokerage? Ans. $19084 net proceeds. $104 brokerage. 8. A commission merchant has 10 hlids., 12000 lbs. of sugar on commission and sells it through a broker at 82 per pound. His commission is 24 per cent, and the brokerage is 14 per cent. All other charges amount to $27. What are the net proceeds, the commission and the brokerage? Ans. $36 commission and brokerage. $897 net proceeds. TO FIND THE AMOUNT TO BE INVESTED AND THE COMMISSION, - WHEN BOTH ARE INCLUDED IN A GIVEN SUM. PROBLEMIS. 902. 1. Received $10000 from a correspondent with instructions to invest, the proceeds, after taking my commission out, in merchandise for his account. I charge 2 per cent commission. What is my commission and what amount do I invest ? Ans. $196,08 commission. $9803.92 invested. OPERATION. $100 assumed investment. Explanation.—We first observe that, as com- 2 = 2% commission mission is allowed on the amount invested only, ſº-ſº e we cannot calculate on the sº for º: © º would be getting commission on the amoun $102 cash required to invest $100. invested jià. º due us. According- ly, for reasons previously given in percentage, $ Invested. pages 443 and 463, we assume $100 to represent 100 the amount invested, * OIl tº: * ... calculate and add to the same the 2 per Çen 102 10000 ($9803.92 commission, which gives $102 as the cash equiva- gº-me ºmº lent of $100 investment and the commission, for making the same. With these figures, it is clear that we can invest as many hundred & dollars as $10000 is equal to $102; or philosophiº; ally, placing the $100 assumed investment on the statement line, we reason thus: If $102 cash will invest $100, $1 will invest the 102d part, and $10000 will invest 10000 times as much, which is $9803.92. On this amount we are allowed commission which at 2 percent is, to the nearest unit ºf à, º,08. The commission may also be found by deducting the investment from the whole alloull º * COMMISSION AND BROKERAGE. 483 2. A country merchant purchased $84.50 worth of merchandise, on 4 months’ time. He then proposes to pay $5000 cash on condition that he be allowed 5 per cent discount on the amount of bill that he pays, which proposition is accepted. What amount does he still owe ? Ans. $3186.84. 3. The net proceeds of a consignment due a consignor is $2511.25, which I am instructed to invest, after deducting commission, in sugar at 10 cents per pound. I charge 24 per cent commission. How many pounds of sugar can be purchased ? Ans. 24500 lbs. 4. A wine dealer remitted to a merchandise broker in New York $1965.60 to be invested in claret. The broker charges 24 per cent commission for investing and 24 per cent for guaranteeing the quality, both of which are to be deducted from the $1965,60. How much money was invested in wine, and how many boxes were there, the price per box being $6.50? Ans. $1872 investment. 288 boxes. TO FIND THE AMOUNT INVESTED, OR TOTAL SALES, WHEN THE COMMISSION AND THE RATE PER CENT COMMISSION ARE GIVEN. FIROBLEMIS. 903. 1. An agent's commission for selling goods was $56,90, the rate per cent was 2. What was the amount or gross sales Ans. $2845. OPERATION. sº * Sales. Bæplanation.—In all problems of this kind, O assume $100 and find the commission or broker- $2.00 commission on $100. age thereon. Then with the relationship num- bers thus obtained, make a proportional state- $ ment as shown in the operation, reasoning thus: 2 * Since $2 commission requires $100 sales, $1 sºmeºmºsºmsºmºmºmº commission will require the 3 part, and $56,90 | $2845.00 Ans. will require 56.90 times as much. 2. A commission merchant received $27.88 commission for selling merchan- dise, and guaranteeing the payment thereof. The rate of commission was 5 per cent and the rate of the guarantee was 3 per cent. What was the amount sold 3 Ans. $348.50. 3. A broker received $71.25 for selling gold at # per cent brokerage. What amount did he sell? Ans. $28500. 484 SOULE's PHILOSOPHIC PRACTICAL MATHEMATICS. † TO FIND THE RATE PER CENT COMMISSION OR BROKERAGE WEHEN THE COMMISSION AND THE AMOUNT OF SALES, COLLECTIONS OR INVESTMENTS ARE GIVEN. PROBLEMIS. An agent collected $3400 and his commission was $85; what was Ans, 24%. 904. 1. the rate of commission ? FIRST OPERATION. 100% rate of commission assumed. 100% of $3400 is $3400. Explanation.—In this solution, we assume 100 per cent and find 100 per cent of the amount. Then with the relationship numbers thus obtain- "100 ed, we make a proportional statement as shown 3400 | 85 – in the operation, reasoning thus: Since $3400 require 100 per cent, $1 will require the 3400th 24% Ans. SECOND OPERATION. part, and $85 will require 85 times as much. Ea:planation.—In this second solution, we 85 reason as follows: Since the commission on 3400 | 100 $3400 is $85, the commission on $1 would be the -*- : -m-m- 3400th part, and on $100, it would be 100 times $24 = 24%. as much. 2. A broker received $180.25 for collecting $7210. What per cent broker- age did he charge 3 Ans. 23%. 3. A commission merchant bought per order, 150 barrels flour at $8 a barrel. He paid drayage and freight $42. The whole invoice cost $1279.26. What was his commission and rate per cent of commission? Ans. $37.26 commission. 3% rate of commission. 4. An attorney collected $2210 and sent me $2099.50. What rate per cent did he charge? Ans. 5%. 5. Paid $602.16 taxes on property assessed at $23160. What was the rate per cent? Ans. 2.6%. TO FIND THE COMMISSION OR BROKERAGE AT ANY RATE PER CENT ON ENGLISH CURRENCY. 905. FIRST OPERATION. 1. What is 4 per cent of £5860 16s. 7d. 3 SECOND OPERATION. Ans. £234 8s. 8d. THIRD OPERATION. - £58,60 16s. 7d. 3658.60 16s. 7d. #5860 16s. 7d. 4% 4% 5 4% #234.40 .64 .28 fº234.43 6 4 .80+ .029% 20 20 See page 450 for reducing shillings and pence to the deci- 8.64S. 8.66s. mal of a pound. 12 12 #5860.829 4% 7.966. 7,966. #234.43316 20 8.66320S 12 7.9584d Explanation.—In the first operation, we reasoned as follows: 1 per cent of £5860 16s. 7d. is * COMMISSION AND BROKERAGE. 485 £58,60.16s. .07d., and 4 per cent is 4 times as much, which is in the first operation #234.40.64s. and .28d. These hundredths of pounds, shillings and pence we then reduce to shillings and pence according to the principles of denominate numbers, as shown in the operation. g In the second operation, we first found 1 per cent in the same manner as in the first Opera- tion, but when we multiplied by 4, we reduced the .28d. to hundredths of shillings by dividing by 12, which gave us .02s. and .04d. We then multiplied the .16s. by 4, and produced .64s., to which We added the .02s, and obtained 66s, which we reduced to hundredths of pounds by dividing by 20, which gave us .03 pounds and .06s. We then reduced the .43 pounds .06 shillings and .04 pence to shillings and pence, as explained in the first operation. In the third operation, we first reduce the shillings and pence to the decimal of a pound and . 4 per cent on the whole amount. We then reduce the decimal of the pound to shillings and pence. 2. What is 34 per cent of £78 4s. 10d. 3 Ans. £2 14s. 9d. 3. What is # per cent of £200 19s. 3d. } Ans. 10s. 0d. 2f. 4. What is 24 per cent on £426. 11s. 7d. ? Ans. £1013S. 3d. NOTE. 1.--To compute 23 per cent, first compute it at 5 per cent and then divide by 2. Or º : º: º per cent and divide by 40, as 2% per cent is #5. Or find 10 per cent and divide by , aS 2# is + of 10. NOTE 2.—See Interest on English Money. MISOELLANEOUS PROBLEMS. 906. 1. What is the commission for selling $1620.80 worth of goods at 24 per cent” Ans. $40.52. 2. A commission merchant invested $5050 for a planter, and charged 1 per cent commission. What was his commission ? Ans. $50.50. 3. A merchant for whom I do business remits me $5050 with instructions to invest the proceeds, after deducting 1 per cent commission, in certain goods. What is my commission ? Ans. $50. 4. Bought $5000 gold at $1124, and sold the same at 154 per cent premium. Paid + per cent brokerage for purchasing and # per gent for selling. How much did I gain 3 Ans. $112.50 currency. 5. Bought $5000 of bank stock at $1124, and sold the same at 10+ per cent premium. Paid # per cent brokerage for purchasing and # per cent for selling. How much did I lose ? Ans. $137.50 currency. 6. Invested through a broker $10825 in U. S. Bonds at 108 per cent. Paid + per cent brokerage. What was the brokerage, and what the face of the bonds? Ans. $25 brokerage. $10000 face of bonds. OPERATION. $100 par value of bonds assumed. $ bonds. 8 = 8% premium. 433 100 + = brokerage. 4 # = +% | 10825 $108} = cost of $100 bonds. $10000 bonds. 25 brokerage. 7. A broker collected $14812.75 and re-invested the proceeds in Securities at par. He charged according to custom # per cent for collecting on the amount collected, and 3 per cent for investing on the amount invested. What was his brokerage 3 Ans, $147,39-H. 486 soul.E's PHILOSOPHIC PRACTICAL MATHEMATICS. * 8. The commission for selling books is 25 per cent and an agent received $235. What was the amount of sales Ans. $940. 9. An auctioneer paid me $2954.80 net proceeds for goods sold. The charges for selling were $28.20 and his commission 5 per cent. What was the amount of sales? Ans. $3140. 10. A salesman sold $6837 of clothing upon which there is a profit of 33; per cent, of which he received 10 per cent commission. What was his commission? Ans. $227.90. 11. A salesman sold $4800 worth of goods and received $240, commission. What was the rate of commission ? Ans. 5%. 12. A commission merchant received $2000 to be invested in supplies. He charges 24 per cent commission for investing, 1 per cent is allowed for charges on the supplies, and 2 per cent for insurance on cost plus 10 per cent of cost. What Sum Was invested in supplies, and how much was his commission ? Ans. $1891.30 invested in supplies. $47.76 — commission. OPERATION. $100 = 1st cost assumed. $101 = cost of supplies. 1 = 1% charges. 10 = % of cost of supplies. 101 = cost of supplies. $10.10 = 10% of cost. $ 2} = % ..º. 101 = cost of supplies added. * * - ºf - 11.10 = t -- 10%. $2.525 = 23% commission. $111 § - *iº. Aggregate cost of $100 supplies. $2.2220 = 2%. Insurance. $100,000 = cost assumed. 100 = cost assumed. 1,000 = charges. 105.747 | 2000.000 2.525 = commission. 2.222 = insurance. | $1891.30 = invested in supplies. — 1 0 $105.747 = cost to buy and 18.91 = 1% charges. ship $100 supplies. $1910.21 = cost and charges of supplies. 2} = % commission. RECAPITULATION. $1891.30 = 1st cost of supplies. $47.755+ commission. 18.91 = 1% charges. 47.76 = 24% commission. oPERATION TO FIND INSURANCE. 42.03 = 2% insurance. $1910.21 = cost and charges of supplies. 191.02 = 10%. $2101.23 = sum insured. 2 = 96 insurance, $42.0246 = insurance. 13. A retailer bought from a wine merchant, on 3 months' credit, goods amounting to $1500, and from a grocery merchant, on 3 months' credit, goods amounting to $1800. But, without waiting for the maturity of the bills, he pays the wine merchant cash in full less 5 per cent discount allowed, and to the grocery merchant he pays $1000 cash, for which he is to be allowed 5 per cent discount on the amount of invoice that it will pay. How much did he pay the wine merchantº And how much does he still owe the grocery merchant? Ans. $1425, he paid the wine merchant. $747.37, he owes the grocery merchant. 14. A merchant received 180 bales of cotton which average 450 pounds to the bale. The cotton was sold at 16#g per pound, and 24 per cent commission and # per cent brokerage were charged for selling. Allowing 4 per cent loss on the cotton received for sampling, classing and loose, and approximating the other charges at $756.20, what amount of goods could be purchased with the net pro- ceeds, after deducting 24 per cent commission on the face of the money invested? Ans. $12006.83. $2000.00 = sum received. º ankruptcy. & --- =ss- 907. Bankruptcy is a failure in business and the condition of being insol- Vent or unable to pay indebtedness. e NOTE.-The word bankrupt is of Italian origin and is derived from the custom in many towns, in the middle ages, of breaking the counter or bench (bancus), in the public exchange or market place, that had been occupied by merchants who had subsequently failed. 908. A Bankrupt or Insolvent is a person or business man who breaks or fails, or becomes unable to pay his debts in the ordinary or regular course of trade. 909. An Assignee is a person that is elected by the creditors of the bank- rupt, or appointed by the court, to receive the assets of the bankrupt, pay the expenses of the assignment, and use the proceeds of the property in paying the Creditors of the bankrupt, as far as the money will allow, according to law and equity. 910. An Assignment is the transfer of the property of a bankrupt to the aSS1gnee. 911. A Schedule is a list of all the assets and liabilities of the bankrupt containing the names and the place of business or residence of all his DEBTORS and CREDITORS, and the amount due from or to each. 912. A Debtor is a person or firm who owes the bankrupt. 913. A Creditor is a person or firm whom the bankrupt OWES. 914. The Assets or Resources of a bankrupt are the entire property except Such as may be exempt by law. 915. The Liabilities are what the bankrupt owes at the time of his failure. 916. Assets that can be converted into cash at their stated valuation are called AVAILABLE AssETs; and those that cannot be immediately converted into cash without a discount or rebate are called NOMINAL ASSETS. 917. Preferred Creditors are persons whom the law allows to collect their claims in full, such as operatives, and employes to whom limited sums are DUE for services rendered within a few months of the assignment. & 918. A Dividend is the sum or per cent paid to the creditors by the assignee out of the assets of the bankrupt. Dividends of a part of the funds of a bankrupt are often made before the final settlement of the estate. 919. As Discharge is an order or decree by a court of bankruptcy, evidenced by a certificate therefrom, that the bankrupt has complied with all the requirements of law governing cases of bankruptcy, and is therefore released or discharged from all his debts. (487) 488 SOULE's PHILOSOPHIC PRACTICAL MATHEMATIcs. & PROBLEMIS. 1. The assets of a bankrupt amount to $203413.39. The liabilities amount to $872546. Estimating that the expenses for settling the estate will amount to $7385, and allowing the assignee 2 per cent on the full amount of assets, what per cent will the bankrupt estate pay ? Ans. 22%. OPERATION. $2034.13.39 assets. Explanation.—From the total assets, we deduct the expenses and assignee's charges, and thus § the net proceeds. We then observe that 72546, the liabilities, are to receive $191960.12, $191960.12 net proceeds. and to find the rate per cent that $191960.12 is of $872546 we place the net proceeds on the statement line, and reason thus: If $872546 00 are to receive $191960.12, $1 will receive the 11453.27 expenses and assignee's com. tº 191960.12 872546th part, and $100 will receive 100 times 872546.00 || 100 as much, which is 22 per cent. Or, we may -* first assume and place on our statement line 100 22% Ans. per cent, and reason as in Articles 856 and 857. 2. A. and B. failed. Their assets were $6720, liabilities $30750, expenses $250. The assignee charged 5 per cent on the assets. What was his commission and what per cent did the creditors receive? Ans. $336 commission. 19.947+%. 3. In the above problem, what would have been the assignee's commission, and the per cent the creditors would have received if the assignee had charged 5 per cent on the amount he paid to creditors and for expenses? Ans. $320 commission. 20% received by creditors. OPERATION. 100 $6400 6150 105 || 6720 250 expenses. 30750 || 100 $6400 paid out. $6150 paid to creditors. | 20% paid to 5% ,-- creditors. $320.00 commission. 4. X. and Y. failed. The net proceeds, after deducting $440 expenses and 24 per cent commission of the assignee, were $7555. The liabilities were $47970. What per cent will the creditors receive and what was the assignee's commission On the total amount of resources? Ans. 15.74%. $205 commission. OPERATION, 7.555 100 8200 100 440 97% 7995 - 24% commission. 47970 || 7555 $7995 | $8200 total assets. $205.00 commission. | 15.74% 5. A firm becomes bankrupt; its liabilities amount to $340180, and its assets amount to $73216.58. Allowing the assignee 5 per cent commission on the first $1000, 24 per cent on the next $5000, and 1 per cent on the remainder of the assets, and estimating the other expenses at $2810, what rate per cent will the estate pay, and how much will a creditor receive to whom $41319.65 are due 3 Ans. 20.4478 + 9%. $8448.95 +, the creditor receives. 6. A bankrupt estate paid a dividend of 184 per cent. The amount received by a creditor, to whom 4 of the liabilities were due, was $7400. What were the liabilities of the estate? Ans. $320000. axes and Licenses. ----------------------------iss aº Aºk--4--- -º-war -wº- 920. Taxes are sums of money assessed according to law on persons, property, incomes, or products, for the use of the nation, State, county or parish, incorporate city, or society. 921. A Poll Tax or Capitation Tax is a certain sum assessed on persons Without regard to property. It is generally assessed by the State on the legal Voters. & 922. A Property Tax or Direct Tax is a certain sum assessed on the estimated value of real and personal property. This tax is usually a certain per cent or a specified number of cents or mills on $100. In some cities and States the rate per cent tax is limited. * 923. An Indirect or Excise Tax is a tax on articles of consumption in their transfer from one person to another. It is also levied on licenses to pursue certain trades and to deal in certain commodities. 924. Real Property or Real Estate consists of houses, lands and all immovable property. 925. Personal Property consists of money, merchandise, stocks, mortgages, notes receivable, furniture, live stock, etc., etc. 926. An Assessor is a person appointed to appraise the value of property subject to taxation. & 927. A Collector or Receiver of Taxes is a public or government officer, who has been elected or appointed, to collect or receive taxes. 928. An ASSessment Roll is a list or schedule containing the names of all persons liable to taxation in the State, parish, or city for which the tax is assessed, and the valuation of each person's taxable property. The assessment roll must be prepared before making an assessment of taxes. 929. A License is a special tax or charge imposed by the national, State, or municipal authorities, upon merchants, bankers, manufacturers, and nearly all classes of business men, for the privilege of prosecuting their various occupations and businesses within the limits of the authority imposing the tax and granting the license. A proper and equitable assessment of this tax requires much wisdom and experience, a faithful regard to duty, and unalloyed honesty. 930. U. S. Income Tax is at present, 1895, an assessment made by the general government on all incomes exceeding $4000, with special exceptions. NOTE.-The Income Tax has been declared unconstitutional by the U. S. Supreme Court. 931. The Budget is a financial statement of items showing the amount of taxes to be raised. It is also a financial statement with business men, showing the items of expenditure of the business for the current or succeeding fiscal year. (489) 490 SOULE's PHILOSOPHIC PRACTICAL MATHEMATICS. * 932. The method of assessing taxes, though in some respects slightly different in some States and cities, is nevertheless virtually the same. -- In making an assessment of taxes the first thing in order, after the assess- ment roll has been prepared, is to ascertain the amount of tax required to be raised, to which amount must be added the expenses to collect the same, and the sum or per centum allowed as being uncollectible. From the sum total of these three amounts are to be deducted any special tax, the amount of poll tax, or the amount of license tax, or all according as the assessment is one for a State, parish, or city. Instead of increasing the amount of tax to be raised by the per centum allowed for non-collectible tax, we may deduct the same per cent from the taxable property or assessment roll, and use the remainder as the amount to calculate the rate per cent upon. Having these facts and figures, we then find the per cent of tax. PROBLEMIS. 933. 1. The budget for the estimated expenditures and liabilities of a State, for the coming year, amounts to $4500000. The inventory or assessment roll amounts to $325000000. Allowing $600000 to be received for licenses and $50000 from polls, and estimating that 10 per cent of the assessment will be uncollectible, and that 5 per cent will be required to pay the expenses for collection, what will be the rate per cent of taxes on the assessment roll ? Ans. 1.3855 + 76. OPERATION. $4500000 amount of budget. Explanation.—From the amount of the budget, 650000 licenses and polls deducted. we first deduct the amount of the license and &ºi=-º-º-º-º-mº-ºm-º. poll taxes, which leaves a balance of $3850000. $3850000 amount of net tax to be raised. We then find the amount to be assessed in order to allow 10 per cent for non-collectible tax. To do this we first assume $100 to represent the amount to be assessed, from which we deduct the 10 per cent allowed for non-collections, and 100 have in the remainder $90, the net collections as 90 || 3850000 the equivalent of $100 tax to be assessed. We Tºº------ then reason thus: If $90 cash collections require $4277777.77 total amount to be assessed. $100 tax to be assessed, $1 will require the 90th Statement to find the amount to be assessed and part, and $3850000 will require 3850000 times as allow 5 per cent commission for collecting. much. This gives us $4277777.77 as the amount of tax to be assessed in order to allow 10 per cent 100 for non-collections and still realize $3850000 95 || 4277777.77 cash. We next find the amount to be assessed tº in order to allow 5 per cent commission for collection charges. To do this we again assume $100 to represent the amount to be assessed, and Statement to find the amount to be assessed to allow 10 per cent for non-collections. $4502923.97 total amount to be assessed. Statement to find the rate per cent tax. from this we deduct the 5 per cent commission, $ and obtain $95 cash as the equivalent of $100 4502923.97 assessment ; then placing the $100 assumed 325000000 || 100 assessment on the statement line, we reason thus: If $95 cash in order to pay 5 per cent 1.3855 -- % Ans. commission require $100 assessment, $1 cash will require the 95th part, and $4277777.77 will - require 4277777.77 times as much. This gives us the amount of tax to be assessed, which is $4502923.97. Having now the total amount of tax to be assessed, and the total amount of property as shown by the assessment roll on which the assess- ment is to be made, we can find the rate per cent of tax. To do this we first observe that $325000000 are to pay or produce $4502923.97, and to ascertain the rate per cent we place the $4502923.97 on the statement line and reason thus: If $325000000 are to pay or produce $4502923.97, $1 will pay or produce the 325000000th part, and $100, 100 times as much. The result of this state- ament is the correct answer of the problem, carried to four decimals. In practice 18 per cent or 11% per cent would probably be used. º: TAXES AND LICENSES. 491 2. The taxable property of a village amounts to $924600. The tax to be raised including collection charges amounts to $23492. Allowing that $5000 will be received for licenses and special taxes, and making no allowance for non-collections, what is the rate per cent, and how much will be the tax of Mr. A., whose real and personal property amounts to $15380? Ans. 2%, rate of tax. $307.60 Mr. A’s tax. 3. The budget for the estimated expenditures and liabilities of a city are as follows: EXPENIDITURES. Salaries of city officers, clerks, collectors, etc., * º e $454,580 Wages of street, wharfage, drainage, etc., departments, º wº 424,000 Office expenses, º - -> tº º º º - * 29,900 Printing, º - º º º º - tº gº - gº 56,000 General service, - º * º - * - tºs 1,282,700 Contingent, - - º - -> º º e sº * - 50,000 Deficit of previous year, º -> tº º sº º * * 475,000 Total estimate of expenditures, - º tº º & $2,772,180 LIABILITIES, Bonds maturing and interest on Same, tº -- º $1,808,508 7-#% certificates to redeem, - - º - - º 860,600 Special bonds, to be paid, <- - sº tº gº sº 70,000 Total amount of liabilities, - - *- - tº º $2,739,108 Total estimate of expenditures and liabilities, - - - $5,511,288 The assessment roll for real and personal property of the city, on which the tax is to be assessed, amounts to $139,848,204. Allowing 10 per cent for non- collections, and estimating that the receipts from new issues of bonds, from the markets, wharves, licenses, and all other revenues of the city will equal the amount of liabilities, what will be the rate per cent tax? Ans. 2.2025 + 7. 4. I am assessed as follows: Real estate $18600. Personal property $4000 Money at interest $2500. Poll tax $2. The rate per cent for city tax is 2 per cent and for the State, 6 per mille; there is also a special city tax of 45 mills on every hundred dollars. How much is my tax, for each, the city, State and special taxes, and what was the total amount 3 Ans. $502.00 city. 11.29% special. 150.60 State. 2.00 poll. $665.894 total amount. 492 SOULE's PHILOSOPHIC PRACTICAL MATHEMATICS. * 5. A citizen paid $502.00 for City taxes, including $2 poll tax and the rate per cent was 2. What was his property assessed for 3 Ans. $25000. 6. If a property holder pays $663 for city, State and special taxes, and the rate for City is 24 per cent, for State, # per cent, and for special 65 mills on the hundred dollars, what is the assessed value of his property? Ans. $20000. 7. A town desires to raise a tax of 24 mills on the dollar. There are 700 polls at $1, the personal and real property is assessed at $1,400,000. What is the rate of tax and the amount of tax of a citizen whose property, personal and real, is assessed at $11550 % Ans. .2% rate of tax. $24.10 tax of citizen, including poll tax. 934. To facilitate the operation of apportioning taxes among a large number of persons, the tax on $1, $2, $3, etc., at the ascertained rate per cent may be found first and arranged as in the following table: TABLE AT THREE AND ONE-HALF MILLS TO THE DOLLAR. WALUE AMOUNT VALUE AMOUNT VALUE AMOUNT VALUE AMOUNT OIF OF OF OF PROPERTY. | OF TAX. PROPERTY. OF TAX. PROPERTY. OF TAX. | PROPERTY, OF TAX. $ 1 .0035 $ 10 .035 $ 100 .35 $ 1000 $ 3.50 2 .007 20. .07 200 .70 2000 7.00 3 .01.05 30 .105 300 $1.05 3000 10.50 4 .014 40 .14 400 1.40 4000 14.00 5 .0175 50 .175 b00 1.75 5000 17.50 6 .021 60 .21 600 2.10 6000 21.00 7 .0245 70 .245 700 2.45 7000 24.50 8 .028 80 .28 800 2.80 8000 28.00 9 .0315 90 .315 900 3.15 9000 31.50 10 | .035 100 .35 1000 3.50 10000 35.00 gºld. (IRI). Silver and UICITeſt MOſley. - - C - C - C --> --> C_-_C - - - - - - - - - - - - C - C - C - Cº - c s - e s - CDC C Dºº-Dºº Cºcº C. CTGº-Tºc Cº-Cºcº C. C. Cºcºcº =N 935. Premium is the per cent that an amount is increased. 936. Discount is the per cent that an amount is decreased, 937. Currency is some circulating medium representing money, used as a Substitute for gold or the fixed measure of value. 938. Uncurrent Money is some circulating medium that is valued less than gold, or less than the fixed measures of value. The coin of some foreign countries is also classed as uncurrent money. When one currency is at a certain per cent premium over another, the per cent discount on the currency of lesser value is not the same as the per cent premium on that of greater value; thus, when gold is 25 per cent premium, the corresponding discount on currency is but 20 per cent; and when gold is 200 per cent premium, $100 in currency is worth $33; in gold, or 663 per cent discount. TO FIND THE CORRESPONDING DISCOUNT ON PAPER CURRENCY OR ON SILVER WHEN WE HAVE THE PREMIUM ON GOLD. 1. If gold is 25 per cent premium, what is the corresponding discount Ans. 20%. 939. on currency? FIRST SOLUTION. OPERATION. $100 gold assumed. 25 = 25% premium. sºmsºmºmºs $125 currency value of $100 gold. Explanation.—Here we are constrained to assume some number to Work from. Accordingly, for reasons often given in similar questions, we assume $100 gold and to the same add the 25 per cent premium, and by this work we obtain $125 as the equivalent currency value of $ Gold. • $100 gold, and having these equivalent 100 values we place the $100 gold on the 125 statement line, and reason thus: If $125 100 currency is worth $100 gold, $1 currency &===s** $80 gold value of $100 currency. 100 *== $20 = 20% discount, Ans. is worth the 125th part, and $100 currency instead of $1 is worth 100 times as much. The result of this reasoning and state- ment gives us $80, as the gold value of $100 currency, which deducted from $100 currency gives 20 per cent discount on currency. (493) 494 soul E's PHILOSOPHIC PRACTICAL MATHEMATICS. º SECOND SOLUTION. OPERATION. $100 gold assumed. ſº. ñº º: of º: R – Q K te SOIUltil On 18 the SãIn 6 a.S that given ln the 25 = 2 5% premium. first solution, so far as º: the sºº-º-º-º: equivalent value in currency of $100 gold $125 currency value of $100 gold. at 25 per cent premium. Then instead of finding the value of $100 currency, we % find the per cent at once by placing the 3% 25 per cent on the statement line, and reasoning thus: If $125 currency give 25 per cent discount, $1 currency will give 100 the 125th part, and $100 will give 100 times as much. It will º tº: tº 25 per cent premium on old is the | 20% discount, Ans. tº: discount on *::::::::: $125 currency, which the result shows to be 20 per cent. 125 2. Gold is 12# per cent premium; what is the corresponding discount on Silver ? Ans. 114%. 3. Gold is 150 per cent premium; what is the corresponding discount on paper currency Ans. 60%. 4. Gold is 500 per cent premium; what is the corresponding discount on paper currency 3 * Ans. 834%. TO FIND TEIE CORRESPONDING PEREMIUM ON GOLD WHEN WE HAVE TEIE DISCOUNT ON CURRENOY OR ON SILVER. 940. 1. If currency is 20 per cent discount, what is the corresponding premium on gold 3 Ans. 25%. FIRST SOLUTION. OPERATION. $100 currency assumed. 20 = 20% discount deducted. Ea:planation.—In the solution of this problem, we first assume $100 currency and then find the equivalent value of the $80 value of $100 currency in gold. same at 20 per cent discount in gold. Having these two values, we reason $ Cur. º thus: If $80 gold is equal to $100 cur- 100 rency, $1 gold is equal to the 80th part, 80 and $100 gold is equal to 100 times as much. The result of this reasoning gives 100 $125 currency as the value of $100 gold. sº-º-º: º: e IS, Vallle à, D. , we nav * *i; value º gold in currency. cent premium on gold, which corresponds 0 deducted from same gives. to or is the equivalent of 20 per cent dis- tº count on currency. ! $25 = 25% premium, Ans. º: GOLD, CURRENCY, SILVER AND UNCURRENT MONEY. 495 SECOND SOLUTION. OPERATION. ~ $100 currency assumed. Eacplanation.—In this solution, we first 20 = 20% discount deducted. find the equivalent value in gold of $100 currency at 20 per cent discount, in the º same manner as in the first solution. We 80 Value of $100 currency in gold. then place the 20 per cent discount on the $100 currency on our statement line, and % reason thus: If $80 gold give 20 per cent 20 premium, $1 gold will give the 80th part, and $100 gold will give 100 times as 80 much. It will be observed that 20 per 100 cent discount on $100 currency is the corresponding premium on (its equiva- lent) $80 gold, which the result shows to tºmº sº. 25% premium, Ans. be 25 per cent. 2. The discount on silver is 40 per cent; what is the corresponding premium on gold 3 Ans. 663%. 3. The discount on currency is 90 per cent; what is the corresponding premium on gold 3 Ans. 900%. 4. If the discount on currency is 99.1%+ per cent, what is the corresponding premium on gold? Ans. 10000%. 5. If the discount on currency is 100 per cent; what is the corresponding premium on gold 3 Ans. Infinite. TO FIND THE CORRESPONDING PREMIUM ON CURRENCY OR ON SILVER, WHEN WE HAVE THE DISCOUNT ON UNCURRENT MONEY. 941. In comparing gold and currency, or currency and uncurrent money, currency is by the custom of bankers and money dealers used as the standard unit of value. 1. When the old paper money of the city of New Orleans was at 10 per cent discount, what was the corresponding premium on currency 3 Ans. 114% premium. OPERATION. $100 currency or silver assumed. 10 = 10% discount. $90 value of $100 uncurrent money. Ea:planation.—The reasoning for this problem is the same as the second solution # of the preceding question. Hence it is 90 omitted. 100 | 11}} premium, Ans. 2. When uncurrent money is 50 per cent discount, what is the corresponding premium on currency or on silver? Ans. 100% premium. 3. The discount on uncurrent paper money is 5 per cent; what is the cor- responding premium on current money? Ans. 5; 7%. 496 soul E's PHILOSOPHIC PRACTICAL MATHEMATICS. tº TO FIND THE WALUE OF GOLD IN CURRENCY OR IN SILVER OR TO FIND WHAT AMOUNT OF CURRENCY OR OF SILVER IT WILL REQUIRE TO PAY A CERTAIN DEBT IN GOLD, WHEN THE GOLD IS AT A CER— TAIN PER CENT PREMIUM. 942. 1. I owe a gold debt of $2000. Gold is 15 per cent premium. What amount of currency will it require to pay the debt 3 Ans. $2300. OPERATION. $2000 Explanation.—It is clear that as the premium is on 1.15 the gold, $100 gold is equal to $115 currency, and hence it will require as many times $115 as $2000 is - equal to $100, which is 20; or as is shown in the $2300.00 Ans. operation, we multiply by the 2000 and divide by 100; or we may reason thus: When gold is 15 per cent premium, $1 is equal to $1.15 currency, and $2000 is equal to 2000 times as much. 2. Gold is 124 per cent premium; what is $6840 worth in currency or in Silver ? Ans. $7695. 3. Gold is 185 per cent premium; what amount of currency will it require to pay a gold debt of $18500? Ans. $52725. TO FIND THE WALUE OF GOLD IN CURRENCY OR IN SILVER WHEN TEIE DISCOUNT ON CURRENCY OR ON SLLVER IS GIVEN. 943. 1. Currency is 25 per cent discount. What is $2000 gold worth in currency? Ans. $26663. OPERATION. $ $100 currency assumed. 100 currency. 25 = 25% discount. 75 2000 -me $75 = gold value of $100 currency. | $26668 Ans. 2. Currency is 124 per cent discount; what is $3672 gold worth in currency? Ans. $4196.57%. TO EIND THE WALUE OF CIUFRIENCY OR OF SILVER IN GOLD WEHEN TEIE DISCOUNT ON CURRENCY OR ON SILVER IS GIVEN. 944. 1. Currency is 25 per cent discount below gold; what is $2000 worth in goldf Ans. $1500. OPERATION. 25% of $2000 is $500, $2000 – $500 = $1500. 2. Silver is 10 per cent discount below gold; what is $1755 worth in gold 3 Ans. $1579.50. * GOLD, CURRENCY, SILVER AND UNCURRENT MONEY. 497 TO FIND WHAT AMOUNT OF GOLD IT WILL REQUIRE TO PAY A CERTAIN DEBT IN CURRENCY, OR IN SILVER WHEN THE GOLD IS AT A CERTAIN PER CENT PREMIUM. 945. 1. I owe a currency debt of $2000. Gold is 15 per cent premium. What amount of gold will it require to pay the debt? Ans. $1739.13%. OPERATION. $100 gold assumed. Ea:planation.—In the solution of this problem, we first 15 = 15% premium. assume $100 gold, and then add thereto 15 per cent of the tº-me same and thus obtain $115 as the value in currency of $100 *** * old; or, in other words, we find by these equivalent values $115 currency. that sióðgoid wiil pay $115 currency debt. with these ratio figures we make the solution statement, and placing $ Gold. the $100 gold on the line, we reason thus: If $115 currency 100 is equal to $100 gold, $1 currency is equal to the 115th part, 115 and $2000 currency is equal to 2000 times as much ; or thus: If $100 gold will pay a currency debt of $115, then by 2000 transposition $1 currency debt will require the 115th part of *=- $100 gold, and $2000 currency debt will require 2000 times as $1739.13% Ans. much. The result is $1739.13; gold. 2. Gold is 124 per cent premium ; what amount will it require to pay a silver debt of $584.15% * Ans. $519.244. 3. A merchant owes in currency a debt of $21294.10, which he pays in gold at 1083. What amount of gold will pay the debt? Ans. $19580.7844. 4. A currency debt of $5200 is paid in gold at 160 per cent premium. How much gold is required ? Ans. $2000. MISCELLANEOUS PROBLEMS IN GOLD, CURRENCY, SILVER AND DNCURRENT MONEY. 946. 1. Gold is 334 per cent premium; what is the corresponding discount on currency ? Ans. 25%. 2. Currency is 334 per cent discount; what is the corresponding premium on gold 3 Ans. 50%. 3. The discount on uncurrent paper money is 124 per cent; what is the corresponding premium on currency 3 Ans. 144%. 4. Gold is 143 per cent premium; what is $16000 worth in currency? Ans. $18340. 5. Gold is 143 per cent premium; what amount will it require to pay a currency debt of $18340? Ans. $16000. 6. Gold is 50 per cent premium; how much silver can I buy with $1000 gold 3 Ans. $1500. 7. Gold is 50 per cent premium; how much gold can I buy with $1000 Silver? Ans. $666,663. 498 SOULE's PHILOSOPHIC PRACTICAL MATHEMATICs. * 8. Uncurrent money is 10 per cent discount, and gold is 10 per cent premium. What almount of gold can I buy for $12000 uncurrent money? Ans. $9818.18; 1. OPERATION. $12000 uncurrent money. $ 90 1200 = 10% discount. 100 100 100 110 || 10800 or thus: 110 | 12000 $10800 currency. | $98.18.18 ºr $98.18.18%r 9. How much uncurrent money at 373 per cent discount can I buy with $50 currency * Ans. $80. 10. How much uncurrent money at 124 per cent discount can be bought for $4500 gold at 94 per cent premium ? Ans. $5618.57%. 11. Uncurrent money is 5 per cent discount, and a young dealer supposing that currency was the same rate per cent premium, sold $10000 currency at 5 per cent premium. What was his loss, and his loss per cent? Ans. $26.31##, uncurrent money loss. #3%, loss on current money. OPERATION. 100 96 $10000 95 || 100 5 *% *=º. 95 || 100 esmºmºmºsºmsºmºsºme e $1054; value $100 current money. or, – * $26.314 loss in un- 100 5 §96 premium. Current, 5 % discount. money. 51*;96 premium on current money. 5 *% discount. e *5% --º *96 loss on clu'rent money. Or thus, $10000 currency. 100 5 95 || 10000 $ 500.00 premium. $10526.31+} amount of uncurrent money he should have received. 10000.00 10500.00 $10500.00 uncurrent money 26.31+; loss in uncurrent money. i received. 12. A broker purchased from one class of customers uncurrent money at 5% discount, and paid current money; and from another class of customers he purchased current money at 5% premium, and paid uncurrent money. During the day he purchased $100000 of each, current and uncurrent money. What was his gain, and gain 'ſ', on the purchase of uncurrent money? What was his loss, and loss % on the purchase of current money 2 and how much did the parties lose, who sold the current money, by reason of their ignorance of the correct principles of percentage? Ans. 1. The broker gained $5000 in uncurrent money purchased, and gained 5; 96 on the current money paid out. NOTE.--When uncurrent money is 59% discount, current money is 54% 96 premium. Ans. 2. The broker lost $5000 in uncurrent money paid for currency, and lost 4}{96 on the uncurrent money paid out. NotE.—When uncurrent money is 4}{96 discount, current money is 5% premium. Ans. 3. The parties who sold the current money lost $263.15}} in uncurrent money, or $250 in current money by reason of their ignorance of percentage calculations. NotE.—These parties should have received $105000 uncurrent money for $997.50 current money, cr $105263.15}} for $100000 current money. * GOLD, CURRENCY, SILVER AND UNCURRENT MONEY. 499. 13. A broker purchased $100000 of uncurrent money at 10 per cent discount, for which he paid current money. Then with the $100000 uncurrent money he bought Current money at 10 per cent premium. What were his net profits and his gain per cent Ans. $909.09 ºr gain in current money. 185% gain on currency invested. OPERATION. $100000 uncurrent money CUR e t . MONEY. 10000 = 10% discount deducted. ić º UNCUR. MONEY. UNCUR. MONEY. $90000 cost in current money. 110 || 100000 $ GAIN. $90909.09: current mone e W. 11 || 10000.00 º 90000.00 || 100 . 90000.00 current investment. $909,09 ºr gain on current investment. 193% gain, Ans. 14. The selling price of coffee is 20g when gold is 25 per cent premium; What should it sell for when the premium on gold is 15 per cent? Ans. 18%. OPERATION. $ Ea:planation.—In this solu- 100 tion, we first find the gold 125 value of the 20c. currency, 20 which is 16c. The reasoning º for this work was fully explain- ed in Article 945, page 497 ; then having the value when 16 old pric º © #, 3. # tº: en gold is at par gold is at par, to find the 5 o D º value in currency when gold is — e e tº 15 per cent premium we simply 18% selling price when gold is 15% premium. add the 15 per cent, as shown in the operation, and have in the result the selling price desired. 15. When gold is 10 per cent premium, goods sell for $2.75; what should they sell for when gold is worth 132? Ans. $3.30. 16. If goods are worth $4.50 when gold is 150, what are they worth when gold is at par 7 Ans. $3. 17. If goods are worth $4 in currency, what are they worth in uncurrent money which is 10 per cent discount? Ans. $4.44%. 18. The gold price of goods is $2; what are they worth in uncurrent money, 7 the premium on gold being 124 per cent, and the discount on uncurrent money being 124 per cent ? Ans. $2.57%. FIRST OPERATION. SECOND OPERATION. $ $ 2.00 $2.00 100 874 Or, 25 87; 112} mºmºmº 2.25 $2.25 $2.57% Ans. $2,574, Ans. gºustomhouse Business. --------------------------------se 947. Custom Houses are buildings or offices erected or established by the Imational government, where the collection of duties or customs on goods imported is made, and where vessels are entered and cleared, etc. 948. Duties or Customs are taxes levied on certain imported goods for the benefit of the general government and the protection of certain kinds of home industry. NOTE.-In some countries, customs or taxes are charged both on imported and exported goods. The government of the United States charges customs only on goods imported. There are two kinds of duties, specific and ad valorem. 949. Specific Duty is a tax levied on the weight or measure per ton, bale, pound, gallon, or yard, without regard to its cost or value. 950. Ad Valorem Duty is a certain per cent levied or assessed on the actual cost of the goods in the country from which they were imported, as shown by the invoice or the appraised value in the absence of an invoice. NOTE.—On certain kinds of goods both a specific and an ad valorem duty is assessed. 951. An Invoice or Manifest is an itemized account of the goods imported or shipped to a purchaser, agent, factor, etc., with the actual cost or value. Regard- ing foreign invoices, the law requires: sº 1. That all invoices of imported merchandise shall be made out in the cur- rency of the place or country from whence the importations shall be made, or if pur- chased in the currency actually paid therefor, shall contain a correct description of such merchandise, etc. 2. That all such invoices, shall at or before the shipment of the merchandise be produced to the consul, vice-consul, or commercial agent of the United States of the consular district in which the merchandise was manufactured or purchased as the case may be, for export to the United States, and shall have indorsed thereon, when so produced, a declaration signed by the purchaser, manufacturer, owner, or agent, setting forth that the invoice is in all respects correct and true, and was made at the place from which the merchandise is to be exported to the United States; that it contains, if the merchandise is to be obtained by purchase, a true and full statement of the time when, the place where, the person from whom the same was purchased, and the actual cost thereof, and of all charges thereon, as provided by this act; and that no discounts, bounties, or drawbacks are contained in the invoice, but such as have been actually allowed thereon, etc. (500) º: CUSTOMHOUSE BUSINESS. 5ot 3. That no different invoice of the merchandise mentioned in the invoice so produced has been or will be furnished to any one. The most of invoices are made out in the weights and measures of the country from which the goods are imported. 952. Tariff is a list of goods alphabetically arranged, with the rates of duties and drawbacks, established by the laws of the United States. 953. The Free List comprises such goods as are imported free of duty. 954. Tonnage Duty is a tax levied per ton on vessels for the privilege Of entering ports. 955. Ports of Entry are places where customhouses are established, and it is lawful to receive goods into a country only at ports of entry. NOTE: 1.—There are several kinds of entries, such as, entry of a vessel from a foreign port With passengers, entry of merchandise for immediate use, bonded warehouse entry, re-warehouse entry, withdrawal entry for consumption, or exportation, etc., etc. NOTE_2.—The forms for the various entries are generally furnished by the customhouse or the custom-house broker. 956. Bonded Warehouses are buildings in which imported goods on which duties have not been paid are placed under bond to the general government by the importer, until he may wish to put them on the market or export them. f d NOTE 1.-Goods may be withdrawn from bonded warehouses for export, without the payment of duties. NOTE: 2.—Goods remaining in a bonded warehouse more than one year, are liable to an addi- i. duty. And goods remaining over three years are considered as abandoned to the govern- Iſle Ilú. 957. Smuggling is the act of bringing foreign goods into this country without paying duties on them. This is done 1, By not entering them at the Customhouse, and 2, By showing less than their real value in the invoice. It is a crime for the prevention and punishment of which stringent laws have been enacted and a strict supervision is exercised by the general government. 958. A Clearance is a certificate given by the collector of a port after the requirements of law have been complied with, that the vessel has been properly entered. 959. Duties are collected at the port of entry by a Customhouse officer appointed by the United States government, under the title of collector of the port. NOTE.—Duties are not computed on the fractions of a dollar; when less than 50c. they are rejected; if 50c. or more, they are counted as a dollar. The collector of the port appoints deputy collectors, appraisers, weighers, gaugers, inspectors, etc., and supervises all entries and papers, estimates, duties, and receives all moneys and securities. 960. A Naval Officer is appointed at the more important posts, whose duty it is to receive copies of all manifests, to countersign all documents issued by the collector, to certify his estimates and accounts, etc., etc. 961. The Surveyor superintends the employees of the collector, inspects vessels and cargoes, and revises all entries and permits. He is personally respon- sible to the collector of the port. 5O2 SOULE's PHILOSOPHIC PRACTICAL MATHEMATICS. * 962. The Appraiser is an officer whose duty it is to examine all imported goods and to determine their dutiable value so that ad valorem duty may be charged. 963. The Store-keeper is an officer who has charge of the warehouse. 964. A Customhouse Broker is a person who is thoroughly acquainted with Customhouse laws, and acting under power of attorney, makes entries, secures permits and transacts business for importers, for which service he makes a charge. 965. Internal Revenue is the revenue derived from the sale of public lands and postage stamps, and from taxes on certain home manufactured articles, such as distilled and malt liquors. The laws prescribing the Internal Revenue are called ExCISE LAWs, in contradistinction to the Tariff Laws, which prescribe Import Duties. 966. Excise Duties are taxes or licenses for the manufacture or sale of certain articles consumed at home or in their transfer from one person to another. 967. A Drawback is money refunded for import duties which has been previously paid. When goods on which duty has been paid are exported, the amount of duty thus paid is refunded. 968. Allowances are deductions made in accordance with law before estimating the duties. 969. Gross Weight is the whole weight of the goods, including the weight of the hogshead, barrel, box, bag, etc., that contains them. 970. Net Weight is what remains after all allowances have been deducted. 971. Leakage and Breakage. By Act of Congress of 1894, all allowance for leakage and breakage was repealed. 972. Draft is the allowance made for waste in weighing goods. 973. Tare is the allowance made for the weight of the box, crate, barrel, or bags containing the goods. The long ton of 2240 pounds and 112 lbs. to the cwt. is invariably employed in the Customhouses of the United States. The United States Revenue Laws have been often changed during the past few years and are still agitating Congress and the people of the country more than almost any other question of government. Tariff or Free Trade, High or Low Tariff, are questions demanding the serious attention of our best law-makers, statesmen, and financiers. PROBLEMI. 974. 1. What is the specific duty on 150 casks of alchohol, each cask 60 gallons capacity, but which when gauged, are found to be 240 gallons short of full capacity ? What is the duty at 15% per gallon ? Ans. $1314. OPERATION. 150 × 60 = 9000 gallons, gross quantity. Explanation.—In , , all cases * - * - in which the specific duty is 9000 gals. – 240 gals. Short measure = required, find the net quantity 8760 gals. net quantity. and compute the duty there- 8760 gals. × 152 duty = $1314 specific duty. * comp y NotE 1.--To find the net quantity of liquids in casks, the law requires that they shall be gauged by a Customhouse gauger. NoTE 2.—To find the net quantity of liquids in bottles, the bottles must be counted and -certified by a Customhouse appraiser. * CUSTOMHOUSE BUSINESS. 503 TO FIND AD VALOREM DUTY. PROBLEM. 975. 1. What is the ad valorem duty on an invoice of 12 dozen Geneva watches, amounting to 21800 francs, if the duty is 25 per cent? Ans. $1051.75. OPERATION. 21800 × 19.3 g = $4207.40 Ea:planation.—In all cases in which ad valorem duty is $4207 × 25% = $1051.75 required, find the net value and multiply the same by the rate of duty, NotE 1.-The 40c., in the net value, being less than 50c., are not used in the Customhouse, when computing duty. NOTE 2.--The intrinsic and Customhouse value of a franc is 19.3 cents. See Talble of Foreign Money. TO FIND THE RATE OF DUTY WEIEN TEIE NET VALUE AND AMOUNT OF DUTY ARE GIVEN. PROBLEMIS. 976. 1. The net value of an invoice is $1260 and the duty is $441. What was the rate 3 OPERATIONS INDICATED. 100% rate. 1260 || 441 Or, 1260 Ans. 35%. 2. A merchant imported from London, England, a case of woolen goods. The cost of the goods was £310 5s. 6d; commission 24 per cent. Freight £3 4s. Consul fees 16s. 8d. The weight of the case was 640 pounds gross; allowing 10 per cent for tare, what is the amount of duty, the specific being 50% per pound and the ad valorem 35 per cent 3 OPERATION. # S. d. 310 5 6 cost of goods. 7 15 2 = 24% commission on cost of goods. 3.18 0 8 total dutiable cost. fº318,0333 x $4.8665 = $1547.71 = dutiable cost in dollars. $1548 x 35% = $541.80 ad valorem duty. 640 lbs. – 64 lbs. tare = 576 lbs. weight of goods. 576 x 50g = $288 specific duty. $541.80 + $288 = $829.80 total duty. Ans. $829.80. Explanation.—Commis- sion is by law a part of the dutiable cost, but freight and Consul fees are not. The intrinsic and the Customhouse value of the £ is $4.8665. See Table of Foreign Money The 71 cents in the $1547.71 being more than 50, $1 is added to the dollars, making $1548 dutiable cost. To find the commission on £. s. d. and to reduce £. s. and d. to the decimal of a £, see page 484. NotE.—The Tariff Law of 1894, reduces the duty on woolen goods to 40 per cent on goods valued at not over 50c. per pound, and to 50 per cent on goods valued at more than 50c. per pound. 504 soulE's PHILOSOPHIC PRACTICAL MATHEMATICs. * 3. Imported from Antwerp 3600 meters of Brussels carpet, 27 inches wide at 4.50 francs per meter. The specific duty is 442 per sq. yd, and the ad valorem 35 an ºrº º P-y per cent. Allowing 39.37 in. to a meter, what was the duty ? Ans. $2393.66. PARTIAL OPERATION. 3600m x 4.50 francs = 16200 francs. 16200 fr. x 19.3 = $3126.60 dutiable value. $3127 × 35% = $1094.45 ad valorem duty. 3600m = 3937 yds. 3937 yds. 27 inches wide = 2952; sq. yds. 2952; sq. yds. × 442 = $1299.21 specific duty. 42. $1094.45 + 1299.21 = $2393.66 total duty. NOTE.—The specific duty on Brussels carpets was repealed by the Tariff Bill of 1894, and the ad valorem duty increased to 40 per cent. 4. Imported from Geneva, Italy, 20 blocks of marble each block measuring 8x4x2 ft. The duty is 50g per cubic foot. The total invoice is 33700 lire. What is the duty ? Ans. $640. NOTE. –A lire is the same in value as a franc, 19.3 cents. 5. Imported from Hamburg, Germany, an invoice of toys amounting in dutiable value to 12200 marks. What is the duty at 45 per cent on 6000 marks and 30 per cent on 6200 marks? Ans. $1085.28. NotE.—The intrinsic and customhouse value of a mark is 23.8 cents. See Table of Foreign Money. 6. The dutiable value of an invoice of leather from Vienna, Austria, is 5850 crowns. What is the duty at 20 per cent ad valorem Ans. $237.51. NOTE.-The value of a crown is 20.3 cents. 7. What is the duty on 449650 pounds of R. R. iron at #g per pound? Ans. $1573.77%. 8. Imported from Havana 60m cigars, weighing 900 lbs., total Invoice in American money is $1500. Under the Revenue Law of 1894, the specific duty is $4 per lb. gross, and the ad valorem duty is 25 per cent. What is the duty, what was the cost of the cigars per m, in Havana, and what was the importing cost, not counting freight and insurance? Ans. $3975.00 duty. $25 per m, Havana cost. $91.25 per m, importing cost. CUSTOMHOUSE BUSINESS. 505 977. AN INVOICE OF COFFEE IMPORTED FROM RIO JANEIRO, BRAZIL. NOTE —This involce is made in the monetary unit and in the unit of weight of the United States, which has been the custom sunce March, 1890. Invoice of 1997 bags of coffee shipped by HERNANDEZ BROTHERs & Co., on board the British Str. Hassell, for New Orleans. For account and risk of whom 1t may concern and consigned to order. L 'if; 200 bags coffee wg. 26,200 lbs. (a) 1741c. p. lb., cost and freight, - - $ 4561|42 N 10/21 |295 4 & & b 38,645 ** 1686C, & $ 4 & 6 & & 8 tº tº 6515|55 O 21/27 | 153 { { & 6 20,043 ‘‘ 1631C. $ $ 4 ( & & { * * *s 3269|01 P 28/43 || 702 * { 4 4 91,962 “ 1594C, & 6 & 6 & 5 & 4 as sº 14658|74 R 44/49 || 192 * * 4 & 25,152 “ 1558C, 4 & 6 & 4 & { { tº gº 39.1868 S 50/54 214 4. & 4 28,034 ** 1533c. * { { { { { 4 4 tº º 4297|61 T 55/60 |241 & 6 & 4 31,571 ** 1496c. ( ; ; 4 t 4 4 & tº º 4723|02 Less freight to New Orleans, on 1997 bags (a 60c. and 5 per cent per bag. *::::: º 4,8388.170 Exchange, 4.85 - - - - - - - - slºoessly, By our draft (a) 90 d/st on London, £8388 17.0 *R* ==me ammºm, E, & O. E RIO DE JANEIRO, 28 June, 1894. HERNANDEZ BROTHERS & Co. Verify all the computations in the above bill. See Index, English Exchange, for the method of reducing the $40,685.92 to the English money of account. For the entries resulting from the importation of goods from Foreign countries, and from the remittance of exchange in payment therefor, See Soulé's new Science and Practice of Accounts. g 506 Yºr SOULE's PHILOSOPHIC PRACTICAL MATHEMATICS. 978. INVOICE OF COFFEE IMPORTED FROM RIO JANEIRO, BRAZIL. NOTE.-This Invoice is made in the monetary unit, and in the unit of weights, of Brazil, which were used in Brazil, up to March, 1890. * Invoice of 4000 bags Coffee shipped by C. SINNOTT & CO., on board the British Steamer “Plato, " for NEw ORLEANs, Messrs. J. M. BUTCHEE & CO., * for account and risk of and consigned to the order of Messrs. R. SOULE & CO. B F 400 B G 600 B EI 483 B J 450 B FC 522 B IN 545 B G 500 B IP 500 4000 bags of 60 kilos = 16337.64 arrs. © 3 $840 p. arroba. e gº º º tº Charges: Duty on Customhouse valuation of 290 reis p. kilo - - - - - - - on 240,000 kilos = Rs. 69: 600,000 a 11 per cent. - & * . gº s º & Difference of 79 rs. per Kilo = Rs. 18,960,000 & 4 per cent tº dº tº & Capatazias 60 rs. p. bag s sº tº dº sº º º ſº tº ſº tº * Brokerage 50 rs. “ • * * * * * * * * * * * * 4000 bags a 700 rs p. bag - - - - - - - - - - - - Mending bags, &e. - - - - - - - - - - - - - Fire Insurance as gº tº gº e º º ºs e s = * * * Consular Certificate sº ºn m = ± is ºn as as as ºn s = Negro hire º m & tº e º ºs º º is * as tº º & Commission for purchasing a 24 per cent. Shipped against cable order dated 2d Feb., 1890, from New York. = E. dº O. E. Rio DE JANEIRO, 27 February, 1890. O. SINNNOTT dº CO. NotE 1.-A Kilo is the French unit of weight and equals 2.2046 Avoirdupois pounds. NOTE: 2. —Tho Arroba is the Brazilian unit of weight and is equal to 32.38 Avoirdu NOTE 3.- 14.69 Kilos are equal to ONE Arroba. Hence to reduce Kilos to Arrobas * | 7656.000 º wº wº 2800 º 000 840|710 62,736 w 12,558,000 Rs. is pounds. ivide the Kilos b 75,294|540 1,882|360 77,176,000 14.69. Nº. 4.—See Index, Brazilian Exchange, for the Brazilian Money of Account and the manner of reducing Rs. to 3. S. and d. 979. STATEMENT OF FULL COST OF INVOICE OF COFFEE PER “PLATO.” J. M. BUTCHEE & CO., In account with C. SINNOTT & CO. 1890 Feby. ſ ! | 27 || To Invoice Cost of 4000 bags of Coffee, per “PLATO,” to New Orleans, ‘‘ Propo. of cost of Cablegrams - * * g gº ºn * º s ‘‘ Stamps on drafts & “Dill Brokerage. 3 16 per cent on Rs 77 : 426 $ 100 sº me & º gº * e- Rs. By our draft a 90 d/st, on Sunnott & Soule, London. exchange 21;d. = £6976.8.3 Rs. Cost on board, including freight of 40c. and 5 per cent per bag = 6.78c. per it. E. dº O. E. Rio DE JANEIRO, Feby. 27, 1890. O. SINNOTT dº CO. 77 : 176|900 18||000 86|030 170 100 Rs. 77 : 426 26|100 smº-mº 77 : 426 Verify the calculations in the Invoice and in the Account Current and Interest Account which follows. INOTE: 1.- It is the custom to charge ºft on the amount invested plus the brokerage. INoTE 2.-Sometimes the statement of the fi cost is made at the end of the Invoice. f NotE.-This Account Current and Interest Account, is presented in advance of interest in order to show the whole work of import- ing and the payment of the invoice in a connected form. See Index, Interest on English Money and Accounts Current and Interest Account on English Money. 980. ACCOUNT CURRENT AND INTEREST ACCOUNT. J. M. Butchee dº Co.'s Coffee per “Plato” to New Orleans, in Acct. Current and Interest Acct, a 5% with Sinnott & Soulé. Dr. sº Cr. 1890 1890 Mar. 5|To Draft at 90 dB. - . 4.6976|| 8 3|June. 6| 3 || 2 |17| 4 Apr. ; 2. By Remittance (a 60 ds. ||3:2000 0|0|| June || 4 5||1| 7| 5 Apr. 16. “ ** & 4 4|| “. do. & & & & 2000, O O 6|| 3 || |16| 5 f"350 (a) 19s. 96 - 64|| 2 || 5 || Moh. 2774|| |13| 0 * 7 do. 4 & 6 & 3000 0|| 0 9| 0 “Bank Com. on £7040 : ** 13 ** Balance of acct. (a) #96. - - - - 8|16| 0 Coffee per “Cheviot” 66314|10|| May 29|11|| |19|10 “Stamps - - - - 3||10| 0 Balance of Interest 6|| 8 “Interest, (balance) 6|| 8 ** Postages - - - - 12| 6 ** Balance Cash 9 June 609||19|| 0 1884 10 3|10| 4 7663/14 10 3|10| 4 - || – |-- - || – || - E. E. F. Flirt LONDON, April 16, 1890. SINNOTT (; SOULE). REMARKS ON THE DEBIT SIDE. 1. The above is an Account Current and Interest Account by the interest method worked to the latest date in the account, which is June 9th. See the subject of Accounts Current. The first money column on the Debit side contains the items that J. M. Butchee & Co. owe Sinnott & Soulé. 3. The next column shows when they are due. . 4. The next column shows the number of intervening days between the date that the items are due, and the date to which the account is made up. (June 9). The last column shows the interest on each item for the time it was due, counting 365 days in a year. 6. The Marine Insurance is effected on £7350 which is an estimated value with a certain per cent added. 7. The 19 shillings per cent, insurance on the value is subject to a varying per cent. discount. The rate of insurance also varies. 8. The Bank commission of ¥ per cent is computed on the first cost £6976–8–3 plus the insurance £64–2–5. 9. The items of debit below the item of insurance were not due till the day the account was made up; and hence no interest is computed thereon. & 10. The 19 shillings per cent means 19s. on every £100. REMARKS ON THE CREDIT SIDE. 1. On this side the first money column shows the payments. 2. The next column shows the dates when the payments were due. 3. The next column shows the number of days intervening between the date that the items of payment are due, and the date to which the account is made up. (June 9). deb ta The last column shows the interest on each item of payment for the time it was due, and also the balance of interest, from the ebit side. NOTE.—This Acceount Current and Interest Account, and the two preceding Invoices are exact copies except, the names of the original instruments in an importing house in New Orleans. 3. ITSLI1"a ITC E. =ss- 981. Insurance is a contract by which one party, in consideration of a eertain sum called a premium, and which is proportioned to the risk involved, under takes to compensate the other party for loss on a specified thing, from specified causes, within a specified time. 982. Insurance is divided into two general classes: 1. Property Insurance. 2. Personal Insurance. 983. Property Insurance is security against loss by fire or damage by transportation, or the injury or death of cattle, etc., and includes Fire Insurance, River Insurance, Marine Insurance, Transit Insurance and Live Stock Insurance. 984. Fire Insurance is the insurance against loss or damage by fire of all kinds of property on land, such as houses, merchandise, factories, etc. 985. River Insurance is the insurance of river steamers, barges, and their cargoes. 986. Marine Insurance is the insurance of ocean vessels and their cargoes. NOTE.—In river or marine insurance, when the steamer or vessel only is insured, nu is called Bull Insurance; and when the cargo only is insured, it is called Cargo Insurance. 987. Transit Insurance is the insurance of goods or property during transportation by land, or by both land and water. 988. Live Stock Insurance is the insurance of live stock, such as horses, mules, etc., against accident or death within a specified time. 989. Personal Insurance includes Life Insurance, Health Insurance and Accident Insurance. 990. Life Insurance is the insurance on the lives of persons whereby a certain sum of money is to be paid to them when they attain a certain age, or at their death, the amount is to be paid to their heirs, or to some one named in the policy. 991. Health Insurance is the insurance by which the insured parties receive a weekly allowance in case of sickness. 992. Accident Insurance is the insurance of persons against death or certain injuries received under certain circumstances while at home, or when traveling. NOTE.-The following conditions are embraced in the policies of most Accident Insurance Companies, giving such companies an unjust advantage over their patrons. “The insurance under this Certificate shall not extend to or cover disappearances, or injuries whether fatal, or disabling, of which there is no visible mark on the body of the insured; nor (508) { INSURANCE. 5og extend to, or cover ACCIDENTAL INJURIES or death, resulting from, or caused directly or indirectly, wholly or in part, by hernia, fits, vertigo, somnambulism or disease in any form, gas or poison in any form or manner, contact with Poisonous substancEs, surgical operations or medical treat- ment, dueling, fighting or wrestling, war or riot, lifting or over exertion, suicide, felonious or otherwise, sane or insane, sunstroke or freezing, riding or driving races, voluntary exposure to unnecessary danger, nor extend to or cover intentional injuries inflicted by the insured or any other person, or where the accident, injury or death happens while the insured is under the influence of intoxicating drinks or narcotics, or in consequence thereof; or while or in consequence of, violating the law or the rules of any company or corporation; or while employed in mining, blasting or wrecking, or in the manufacture, sale or transportation of gun-powder, or any other explosive compound, (unless insured to cover such occupation). Standing or riding upon the platforms of any conveyance using steam or electricity as a motive power, or entering or attempting to leave such conveyance while the same is in motion, or walking on the road-bed or bridge of any railway, are hazards not counted or covered by this insurance, and no sum will be paid for injuries or death in consequence of such exposure, or while thus exposed, happening to any person other than railroad employes, who shall have been accepted under such occupation.” 993. There are also other forms or kinds of insurance under different names according to the nature of the subject of insurance, or the objects or interests insured: as Guarantee or Fidelity Insurance, Co-operative Insurance, or Benevolent Insurance, Boiler Insurance, and Plate Glass Insurance. 994. Guarantee or Fidelity Insurance is the insurance against loss that may occur through fraud or unworthy agents or employés who occupy positions of trust. Guarantee or Fidelity Companies issue bonds of suretyship for a specified amount to employers, guaranteeing the faithful conduct and the honesty of the party employed for a given time, for which they charge the employé a given per cent on the face of the bond. Employés must possess good characters and good business capacity before they can be thus guaranteed. - Some guarantee companies also guarantee titles to lands. 995. Co-operative Insurance or Benevolent Insurance is that in which the members of some Company, Association or Society, contribute weekly or monthly, or at stated times, to a fund from which a specified sum is paid to the heirs of deceased members, or to some person named in the Benefit Certificate or Policy of the deceased member. The sum contributed depends upon the age of the person at the time he becomes a member, and the amount of his benefit certificate or policy. Of this class the Knights of Honor, the Legion of Honor, the Catholic Knights of America and the Royal Arcanum, are leading examples. 996. The Insurer, generally a stock or mutual company doing business under the title of an Insurance Company, is the party that contracts to indemnify the other. The Insurer, after having signed the policy, is called the underwriter, so called from the fact of having signed under the conditions of the policy. 997. The Insured or ASSured is the party that is protected by the under- taking or contract of the insurer. 998. The Policy is the written or printed instrument containing the numer- ous conditions of the contract between the insurer and the insured. In marine and river insurance, there are two kinds of policies, valued and open. 999. A Walued Policy is one in which a value has been set upon the property or interest insured, and inserted in the policy. A valued policy is often called a special policy. 5 Io SOULE's PHILOSOPHIC PRACTICAL MATHEMATICS. Yºr On country risks, most Insurance Companies limit the amount of insurance to three-fourths or two-thirds of the appraised value of the property insured. In many policies, especially on buildings, the Insurance Companies reserve the right to repair, rebuild or replace the property destroyed or damaged with other of like kind and quality, within a reasonable time. 1000. An Open Policy is one in which the amount of interest or property is not fixed by the policy, but is left to be ascertained in case of loss or damage. In OPEN POLICIES for River and Marine Insurance, additional insurance may be entered, from time to time, as goods are shipped, either on the policy or in a pass book, which is regarded as having the force of the policy. Premiums on OPEN POLICIES are paid at the end of a voyage, or monthly, or quarterly, or otherwise, as may be agreed upon. 1000%. Open Endorsement Policy is a policy issued to the assured in blank, to cover on property held by the assured, as described in the several entries to book attached, while in such places and insured for such periods of time, and at such rates of premium, and for such amounts as shall be written. Hence, no liability on the part of the Company exists, until application is accepted. These entries are hence called endorsements. 1001. Perpetual Policies are sometimes issued, the rate being usually equal to that of TEN annual premiums. In PERPETUAL POLICIES, the premium is considered merely a deposit with the Insurance Company; for at any time, at the instance of either party, the policy may be cancelled, and 90 per cent of the premium or deposit must be returned to the policy holder. Policies are not transferable without the consent of the insurers. 1002. The Subject of the Insurance is the property itself, and the title or interest that the assured has, is called his Insurable Interest. 1003. The premium is the price paid for the insurance. 1004. The Rate of Premium is either a certain per cent on the amount insured, as # per cent or 13 per cent, or it is a specified number of cents on one hundred dollars, as a rate of 65%, or $1.25, which means respectively, 652 and $1.25 on each $100 insured. 1005. The Rate of Premium depends in a great measure upon the time for which the policy is issued and the degree of risk, which is so varied, that the calculation of the rate becomes one of probabilities. 1006. Short Rates. Rates for less than a year are called SHORT RATEs, and are proportionally higher than yearly rates. 1007. Return Premium. When a policy is taken for a year and is can- celled at the request of the insured prior to the close of the year, a Return Premium is paid to the party insured. In such cases, the company has the right to charge Short Rates for the time the policy has been in force. 1008. A Risk is the probability that the subject of the insurance may be lost or damaged, or it is the danger to which the thing insured is exposed. Risks are denominated “hazardous,” “extra hazardous,” and “Specially hazardous,” according to the probability of loss, which depends very much upon the combustibility or perishable nature of the subject of insurance and its sur- roundings. For example, a hardwarc store would be less liable to burn than a dry goods store, and a store in which inflammable substances or explosive materials * INSURANCE, 5 II were kept would be more liable than either; and a hotel is more liable than a private residence. 1009. Reinsurance is a contract which the first insurer enters into, in order to relieve himself from some of the risks that he has undertaken, by dividing them with other insurers. 1010. Cancellation of Policies. A policy is cancelled when the agreement between the party insured and the insurers is annulled. 1011. Transferring Policies. A policy is transferred when the insured, With the consent and acceptance of the insurers, conveys or transfers his rights under the terms and conditions of the policy to some third person. 1012. An Insurance Agent is a person who is duly authorized to act for an insurance company in soliciting business, contracting risks, collecting pre- miums, and adjusting losses. 1013. An Insurance Broker is a person who effects insurance and negoti- ates policies, for which service he is paid a commission or brokerage by the com- pany taking the risk. Brokers are regarded as agents of the parties insured, and not of the insur- ance company. 1014. Insurance Companies are of several kinds according to the principle or form of their organization: 1. Stock Companies. 2. Mutual Companies. 3. Mixed or Stock and Mutual Companies. 1015. A Stock Insurance Company is one in which the Capital is divided into shares, which are subscribed, paid for and owned by persons or firms called stockholders. The stockholders share all the profits and are liable for the losses, to the extent of the value of the shares owned. 1016. A Mutual Insurance Company is one in which the profits and losses are shared by all who are insured, who are the policy holders. In this form of company, there are no stockholders or capital stock. The capital consists of unused premiums of the reserved earnings and of the invest- ments of the company. 1017. A Mixed or Stock and Mutual Insurance Company is one which combines the principles of both the Stock and Mutual Companies, i. e. they have a Capital stock and shareholders, and divide a portion of the net profits among share- holders and policy holders. Generally in mixed companies a limited dividend of 3 to 10 per cent is first paid to the stockholders, a portion of the profits is carried to 8wrplus or reserve fund, and the surplus profits, if any, are divided among the participating policy holders. 1018. Non-participating policies are sometimes issued at reduced rates, by mutual and by mixed companies, and such policies do not share in the losses or gains of the company. In Some mixed companies, the capital is unlimited, provided it is not less than a certain sum; and in the organization of the company, it is represented by deposit notes; and subsequently by the installments collected, premiums paid on 5I2 SouLE's PHILOSOPHIC PRACTICAL MATHEMATICS. * insurances, and by the sums earned in the business. Iu these companies, the insured becomes, by the mere fact of insurance, a member of the company, and is obliged, therefore, to observe its rules and recognize the authority of its agents. 1019. Insurance Scrip consists of the certificates given by mutual and mixed companies to the insured, in lieu of cash dividends. They are equivalent to receipts or certificates for money loaned. They generally bear interest, are nego- tiable by delivery and transfer on the books of the company, and are payable at Such time as the company may subsequently specify. 1020. The Surplus of an INSURANCE COMPANY is the excess of the assets OVer the liabilities. Capital stock and unearned premiums are considered a part of the liabilities when estimating the surplus or when determining gains and losses. 1021. Misrepresentation. Any material misrepresentation prejudicial to the insurers, whether intentional or not, and whether made by the principal or his agent, Vitiates the policy. , 1022. Bottomry and Respondentia. Bottomry is a contract by which the Ship owner, or the master on his behalf, pledges the keel, tackle, and apparel of his ship as security for money which he borrows for the use of the ship, in contempla- tion of a particular voyage or for a particular and fixed period of time. It is a Condition of a bottomry contract that if the ship should be lost during the Voyage, Or by any of the perils enumerated in the contract, the lender shall lose his money; but if the ship arrives in safety, then he shall receive his principal and also the interest agreed upon, however much it may exceed the usual or legal interest for the use of money. When the money is loaned on other property exposed to maritime risks, the contract is called Respondentia. º 1023. Salvage is a compensation allowed parties for their voluntary services in Saving a ship, steamboat, cargo, or other property from the perils of fire or water. ADJUSTMENT OF LOSSES. 1024. In Marine Insurance the policies contain an “Average Clause" by Which, “if the vessel is valued in the statement thereof, beyond the amount insured, the insurers are to pay such part only of the sum insured (or of the partial loss) as the amount insured bears to such valuation.” And in case of the cargo, if that “is valued beyond the amount insured, the insurers are to pay such part only of the sum insured (or of the partial loss) as the amount insured bears to the full valuation.” The effect of this principle is that the insured is himself the insurer for that part or portion of the property insured. For example: If a vessel valued at $40000 were insured for # value, or $30000, and were totally destroyed, the company or the insurers would pay the $30000; but if the vessel were damaged to the amount of $10000, the company Would pay only three-fourths of the partial loss, or $7500. Again, if a cargo valued at $40000 were insured for $10000, and sustained a loss of $10000 or more, the insurers would pay only $2500. 1925. In Fire Insurance, under the ordinary Fire Insurance Policy, which does not contain the average or the 75 per cent clause, the company will pay the full amount of the loss or damage, provided it does not exceed the sum covered by insurance. Thus, if a house or stock of goods were insured for $10000 and a partial loss of say $7000 be sustained, the company would pay the full loss or $7000. But if the loss should reach $11000 or more, the company would pay only $10000, the amount specified in the policy. Sometimes FIRE INSURANCE POLICIEs contain the “Average Clause,” and then the company issuing such policies would pay only such portion of the loss as the amount insured bears to the total value of the property. NotE.—The usual form of the “Average Clause,” to be used when policy covers any mer- chandise in two or more locations, is as follows: * “It is a part of the consideration of this policy and the basis upon which the rate of premium #: INSURANCE. 5I3 is fixed, that the assured shall maintain insurance on each item of property insured by this policy 9qual to the actual cash value thereof, and that failing so to do, the assured shall be an insurer to the extent of such deficit, and in that event shall bear his, her or their proportion of any loss.” 1026. Adjustments of Losses, under policies containing the “Average Clause,” would be made as follows: Suppose property valued at $20000 was stored in two warehouses, and insured for $15000, and that the goods in one warehouse were destroyed by fire making a partial loss of say $8000, the company would pay $6000, i.e. such proportion of the loss as the amount insured bears to the full value of the property. 1027. To Adjust the loss when several different insurance companies have risks on the Same property, the loss is divided among all the companies in proportion to the amount of their respective risks, whether all the companies be solvent or not. Thus, if goods valued at $25000 be insured in Co. A, for $3000, in Co. B, for $5000, and in Co. C, for $10000, making $18000, and a loss of $9000 should be sustained, Co.'s A, B, and C, would pay respectively, º, ø, and +3 of the loss. 1028. The 75 per cent Co-insurance Clause, which is now generally inserted in fire policies, reads as follows: “It is a part of the consideration of this policy and the basis uponl Which the rate of premium is fixed, that the assured shall at all times maintain a total insurance on the property, insured by this policy, of not less than 75 per cent of the total cash value thereof (as covered under the several items of this policy,) and that, failing so to do, the assured shal become co-insurers to the extent of the deficiency, and in that event, shall bear their proportion of any loss occurring under this policy.” * 1029. The Lightning Clause reads as follows: “This policy shall cover any direct loss or damage caused by lightning, (meaning thereby the commonly accepted use of the term Lightning; and in no case to include loss or damage by cyclone, tornado, or windstorm,) not exceeding the sum insured, nor the interest of the insured in the property, and subject in all other respects to the terms and conditions of this policy. Provided, however, if there shall be any other insurance on" said property, this company shall be liable only pro rata with such other insurance, for any direct loss by Lightning, whether such other insurance be against direct loss by lightning or not.” 1029a. Iron Safe Clause.—1st. The assured will take a complete itemized inventory of stock on hand at least once in each calendar year, and unless such inventory has been taken within twelve calendar months prior to the date of this policy, one shall be taken in detail within thirty days of issuance of this policy, or this policy shall be null and void from such date. 2nd. The assured will keep a set of books, which shall clearly and plainly present a com- plete record of business transacted, including all purchases, sales and shipments, both for cash and credit, from date of inventory as provided for in first section of this clause and also from date of last preceding inventory, if such has been taken, and during the continuance of this policy. 3rd. The assured will keep such books and inventory, and also the last preceding inventory, if such has been taken, Securely locked in a fire-proof safe at night, and at all times when the build- ing mentioned in this policy is not actually open for business; or failing in this, the assured will keep such books and inventories in some secure place not exposed to a fire which would destroy the aforesaid building; and unless such books and inventories are produced and delivered to this company for examination after loss or damage by fire to the personal property insured hereunder, this policy shall be null and void and no suit or action shall be maintained hereon. It is further agreed that the receipt of such books and inventories and the examination of the same shall not be an admission of any liability under the policy, nor a waiver of any defense to same. 1029b. The Foundation Clause. It is understood that the foundations and piling on which the within described building rests, are not included in this Insurance, and in no event shall they be taken as part of the valuation of the building for the purpose of arriving at the assured’s contributory proportion in the application of the Co-Insurance Clause. The foundations being exempted from the operation of the co-insurance clause of this policy and not being covered by this insurance, it is hereby understood and agreed that in the event of loss, if the insuring company should elect to rebuild, and it be found necessary on account of statutory or other causes to replace the foundations in whole or in part, the cost of replacing or repairing same will be at the expense of the assured. 5 I4. SOULE's PHILOSOPHIC PRACTICAL MATHEMATICs. º 1030. The Three-quarter Loss Clause or Country Clause inserted in the country policies issued by many Insurance Companies, reads as follows: “It is agreed and understood to be a condition of this insurance, that in case of any loss or damage under this policy, this company shall not be liable for an amount greater than THREE- FourTHS of said loss, not exceeding the sum herein insured, the other ONE-FourTH to be borne by the assured, and in the event of other insurance on property covered under this policy, this company shall not be liable for more than its proportion of three-fourths of such loss or damage.” 1030a. The Three-fourths Walue Clause is as follows: “It is understood and agreed to be a condition of this insurance, that in the event of loss or damage by fire to the property insured under this policy, this company shall not be liable for an amount greater than three-fourths of the actual cash value of each item of property insured by this policy (not exceeding the amount insured on each such item) at the time immediately preceding such loss or damage; and in the event of additional insurance—if any is permitted hereon—then this company shall be liable for its proportion only of three-fourths such cash value of each item insured at the time of the fire, not exceeding the amount insured on each such item.” 1031. A Floating Policy sometimes termed a “ BLANKET POLICY,” covers goods in a number of different warehouses, docks, etc., with the “Average Clause” added. IMPORTANCE OR VALUE OF INSURANCE. 1032. Insurance, considered in all of its various relations and different applications, is one of the most important and valuable institutions of the age, without which trade and commerce would languish, and the poor and medium classes of society would be deprived of many blessings and opportunities of success, which through it they now enjoy. Considering the hazards which threaten property and the uncertainty of human life, prudence, wisdom, regard for financial obligations, and affections for family, demand that every man should insure both his property and his life. For a business man not to insure his property is evidence of his incapacity, and unless his goods or other property is fully paid for, it is proof of his Want of fidelity to his creditors. The only exception to this is where the party has sufficient capital, outside of his business, to enable him to continue business, pay his financial obligations and to support those dependent upon him, in case of loss. In regard to Life Insurance Companies, it must be said that while they are doing a grandly humane work and merit the favorable consideration and patronage of all men, some of the companies are characterized for extravagance beyond prudence or reason; and some have violated the principles of ethics in their prom- ises and statements of facilities, advantages and resources. These facts make it necessary for the applicant for Life Insurance to investi- gate and exercise judgment and discretion in selecting a company. The various Social, Co-operative or Benevolent Insurance Associations with their low rate of assessments, and their economical system of management, offer to the poor man facilities for insuring, which as he loves his mother, his wife, his children, or his fellow-man, he should accept. These Co-operative Insurance Associations are the children of this progressive age; they are supplementing the work of the regular Life Companies, and are proving a blessing to mankind. 1033. Read the Policy of Insurance. To all who insure, whatever may be the subject of insurance, or the character of the company in which you insure, we say, read your policies and fully understand your rights and duties. In case of loss of furniture, books, pictures, wearing apparel, jewelry, etc., the insured is required to give a full itemized statement of the same before the Yºr INSURANCE. 5 I 5 insurance will be paid. The importance therefore of having a list or an inventory of them safely preserved is easily seen. The PARTY INSURED should also fully understand that if he insures his house and then sustains a loss on his kitchen, or his furniture, he has no claims on the insurers for loss or damage. His policy protects only such property as is specified therein. He should also understand that where his house, when insured, is damaged, and said damage paid for or repaired by the insurers, that his policy is reduced to the extent of the damage sustained. A want of this knowledge frequently results in unpleasant controversies between the insured and insurers. The problems in insurance involve some of the principles of per cent, and sometimes, especially in life insurance, the calculation of the average length of human life, and the worth of money invested at various rates of interest, and in many different Ways. TO FIND THE RATE OF PREMIUM. 1034. The first thing to be determined by an insurance company after its organization, is the rate per cent premium to charge. To determine this for river insurance, independent of the charges and statistics of other companies, we present the following figures: From the statistics of river commerce, we will suppose that during the past season there were 40 steamers plying between the ports of New Orleans and St. Louis, the aggregate value of the same being $2,400,000, and which, in accordance with the regulations of insurance companies in New Orleans, was insured for two- thirds of its value, $1,600,000; that during the season 1 steamer was lost, the value of which was $45,000, 3 of which is - $30,000 And 1 was damaged to the amount of $18,000, § of which is - - - 12,000 Making a total loss of - ſº sº * gº $42,000 STATEMENT TO OBTAIN TEIE PER CENT LOSS. $ 42000 wo 100 23% loss, Ans. By these figures, we see that to insure a steamer in this trade, the probabili- ties of loss are 23 per cent, and hence the rate per cent of premium must be 23 per cent, plus such increase as may be required to remunerate the insurer for his trouble, cover expenses, and leave a fair margin for profit. To show approximately the per cent to charge for fire risks, we present the 5 I6 SouLE's PHILOSOPHIC PRACTICAL MATHEMATICs. * following figures, which are very nearly the actual figures of one of the New Orleans Insurance Companies for one year's unusually disastrous business: Amount of Fire Risks, $8,500,000 Losses on Fire Risks, $75,540 {{ “ Marine { { 2,800,000 {{ {{ Marine {{ 51,000 {{ {{ River 4 1,750,000 {{ {{ River 4 20,210 $13,050,000 $146,750 The expenses of the Company during the year were - $36,000 “ taxes {{ {{ * { {{ {{ 46 - 12,000 By these figures, we see that $8,500,000 have lost - tº º $75,540 To which we add # of the expense gº tº {º º sº 12,000 And #####, of the taxes - tº tº tº i tº 7,816 Total loss and expense on fire risks e © ę $95,356 Having the total losses of the fire risks, we then find the per cent loss by the following statement 100% $ 95.356 8500000 || 95.356 8500000 100 1.1218 + 7% (nearly 13%). 1.1218 + 7% Ans. The result of this work shows that the probabilities of loss, including expenses and taxes are 1.1218 + 7%, and hence the rate per cent of premium must be 1.1218 + 7%, plus such increase as may be necessary to produce such profit as equity would warrant and the wisdom of the Directors of the Company may deter- mine. To fix the premium at 14 per cent, the Company would realize on the fire risks, as shown by the above figures, a profit of $10,894. In actual practice. Imany other small items of expenditure and allowance might be made, and thus bring the result still nearer the exact premium to charge. The supply of water and the efficiency of the fire department would also be con- sidered. Our subject in presenting this work is more to show the general manner of arriving at a very important result than the minute details of the operation, which must necessarily differ in every case, and can only be fully understood as they occur in the actual business of a company; in actual operations, the figures showing the amount insured and the amount of loss sustained for several years would be takeºustead ºf: the result of one year's business. ºp & gº •e." gº & % INSURANCE. 517 TO FIND THE AMOUNT OF PREMIUM, OR COST OF INSURANCE, WEHEN THE AMOUNT INSURED AND THE RATE OF PREMIUM ARE GIVEN. PROBLEMs. 1035. 1. A real estate owner insures for one year two houses, valued at $7000 each, at 13 per cent. What is the premium ? Ans. $157.50. 2. What is the premium for an insurance of $120000 on a hotel, at $2.25 per $100% Ans. $2700. 3. A ship was insured for $60000 at 14 per cent, and her cargo for $25000 at $1.30 on $100. What was the cost of the insurance? Ans. $1225. 4. What is the premium pro-rata on a $3500 policy, dated Sept. 12, 1894, and expiring Jan. 30, 1895, at the annual rate of 75% per $100% Ans. $10,064. NOTE.-30 days to a month is allowed in this problem. 5. What is the premium on a $3500 policy dated as in the above problem at the short rate of 95g per $100? Ans. $12.74;. 6. Insured my house for $8000, furniture for $4500, library for $1500, paint- ings for $1000, piano for $500 and stable for $200. The policy costs $1.50; the rate of premium is 1+ per cent, with 20 per cent discount or rebate; what is the cost of the insurance and what is the actual rate of premium charged ? Ans. $158.50 cost of insurance. 1% actual premium. NOTE.-The law of Louisiana does not allow insurance companies to give a rebate. The net rate of insurance must be stated. OPERATION TO OBTAIN COST OPERATION TO OBTAIN THE ACTUAL OF INSURANCE. RATE PER CENT. $15700 sum insured. |14% premium = #% or thus: 14% rate of prem. |20% discount = # of # = # per cent, $ PREMIUM GE º which deducted from # leaves 1% 157 o $196.25 amt. of prem. | actual premium. 15799 39.25 = 20% dis. 109 $ 57 00 t or thus: 157. net prem. 1.50 policy. 14% = $1.25 1% Ans. 20% of which is .25, and which $158.50 cost of ins. deducted leaves $1, which = 1%. 7. A merchant insured for one year his house for $12000, his furniture for $5000, library for $3000, paintings for $2000, statuary for $2500, silver-plate for $1500, piano for $500, mirrors for $2000, cisterns for $400, stables for $1000, fences for $500, and wearing apparel for $3000. The policy cost $1.50; the rate of pre- mium was 14 per cent, with 15 per cent rebate. What was the cost of the insur- ance, and what was the actual per cent paid 3 Ans. $427.35, cost of the insurance. 14% 76, actually paid. 8. What will be the cost to insure a stock of extra extra hazardous goods valued at $28500, at $3.25 on $100? Ans. $926,25. 5 I 8 soul.E's PHILOSOPHIC PRACTICAL MATHEMATICS. * 9. A real estate owner insures 2 dwellings valued respectively at $8500 and $7000; and also the rent of same, which is $1800 for the $8500 dwelling and $1500 for the $7000 dwelling. The rate was 13 per cent. What did the insurance cost him # Ans. $282.00. 10. The following entries are made on an “open policy” or in the pass book used with an open policy: Make the extensions and find the total amount due. DATES NAMES OF STEAMER IFROM TO SUBJECT AMT. INs. RATE PREM. 1895 Nov. 3 || Garland, New Orleans. Shreveport, Mdse. 4100|00 24 “ | 5 || Pargoud, 4 t é & Wicksburg, Mdse, 650000| # “ | 8 || T. P. Leathers, { { & 4 St. Louis, Mdse. 980000. 1; “ 14| Queen City, * { & 4 Cincinnati, Moise. 2300|00| 2 “ 27| City of New Orleans. & 4 & 4 Memphis, Mdse. 1685|00| 1 $ 37335 11. What will it cost to renew a policy of insurance on property valued at $24200 at the rate g per cent premium ? Ans. $211.75. 12. An Insurance company unsured a block of buildings valued at $400000 at 2 per cent, allowing a rebate of 15 per cent. The company subsequently re-insured $100000 at 13 per cent, $100000 at 14 per cent, and $100000 at 13 per cent. How much premium did the company receive? How much did it pay for re-insurance, and at what rate per cent were its receipts on the amount that it protects? Ans. $6800, premium received. $4125, premium paid. 2% 7%. PARTIAL OPERATION. $400000 at 2% = $8000 – 15% = $6800. $100000 at 14% = $1125. $100000 at 14% = $1250. $100000 at 13% = $1750. $1125 + $1250 + $1750 = $4125. $6800 – $4125 = $2675. $400000 – $300000 = $100000. ($2675 × 100) + $100000 = 2.675% = 2%%. 13. You insure your house for $9000; furniture for $4500; jewelry for $6000; books for $1800; paintings for $2500; statuary for $1400; silver-plate for $1200; piano for $400; cisterns and outhouses for $1500; wearing apparel for $2500. The rate is 14 per cent with 15 per cent rebate. What is the cost of the Insurance, and what, in case of a fire and a total loss, would be your various duties in order to col- lect the insurance? Ans. To the first, $327.25. Answer to the second : Your first duty would be to read your policy and act in accordance with its conditions. 2. To make an itemized statement or bill of the furniture, jewelry, books, paintings, statues, silver-plate and wearing apparel, and present the same with your account to the insurance company. 14. If you keep an insurance on an extra hazardous stock of goods for 20 years at 5 per cent, and they then burn and you collect the full insurance, have you gained or lost 3 Ans. Making no allowance for interest, you have lost premium paid to the amount of the value of the goods. X}. INSURANCE, 519 TO FIND WHAT AMOUNT TO INSURE SO AS TO COVER BOTH THE PROPERTY INSURED AND PREMIUM PAID, WHEN WE ELAVE THE AMOUNT OF PROPERTY AND THE RATE PER CENT PREMIUM GIVEN. PROBLEMIS. 1036. 1. I wish to insure a stock of goods valued at $44100, for such a sum that in case of loss, I will receive the value of the merchandise and the premium paid for insurance, or in other words I wish to insure the merchandise and the premium paid for the insurance. The rate is 2 per cent. For what amount must I insure, and what is the premium ? Ans. $45000, amount to insure. $900, premium. FIRST OPERATION. $100 assumed. $ ſº first = 2 * e 3.SSlllll C o represent 2 % premium. 98 100 the value of the goods, plus the 2% premium for $98 value of goods. 44.100 insuring the goods. We then deduct therefrom the $45000 amount to insure. 2 per cent premium and 2 º obtain $98, as the value % premium. of the goods, which, with e the premium paid for in- $900.00 premium. Suring the goods required an insurance of $100. We then reason thus: Since $98 value of goods require an insurance of $100, $1 will require the 98th part, and $44100 will require 44100 times as much, the result of which is $45000, We then calculate the 2 per cent on the $45000, and find the premium to be $900. SECOND OPERATION. $44100 value of goods. $ 2% premium. 100 98 $882.00 premium on goods. 882 *-*s $900 premium on goods and premium. 44100 value of goods added. $45000 amount to insure. 2. My house, situated in a country village, is valued at $12000, and the furniture at $5000. I wish to insure # of the value of the house and furniture, so that in case of loss I may receive the # value of both and the premium paid for the insurance. The rate is 1% per cent; policy cost $1.50; the rebate or discount is 10 per cent. What amount of insurance must I effect and what will be the cost 3 Ans. $12954.03, insurance. $205.53, cost. PARTIAL OPERATION. $12000 + $5000 = $17000. # of $17000 = $12750. 13% = $1.75, 10% of $1.75 = .175g. $1.75— .175 = $1.575, $100–$1,575 = $98.425. ($12750 × 100) + $98.425 = $12954.03 to be insured. 13% on $12954.03 = $226.70 - 10% rebate = $204,03 + $1.50 = $205,53. 52O SOULE's PHILOSOPHIC PRACTICAL MATHEMATICS. * 3. A cargo of cotton is valued at $260000; for what sum must a policy be taken to cover both cotton and premium, the rate being $1.624 on $100 Ans. $264294.79-H. 4. For what amount must I insure my dwelling, valued at $9600, so that in case of loss I may receive both the value of the house and the premium paid The rate of insurance is 1% per cent, rebate 20 per cent 7 Ans. $9736.30-H. PARTIAL OPERATION. 7 1#% = 4. 20% = }, + x # = ſ. 3 — ſº, = ## = 13% net premium. $100–13% = $983. ($9600 x 100) + 98% = $9736.30-H. TO FIND THE AMOUNT INSURED, WHEN WE HAVE THE AMOUNT OF PREMIUM PAID AND THE RATE PER CENT PREMIUM GIVEN. PROBLEMS. 1037. 1. The cost of insuring a house was $114.00; the cost of policy was $1.50; the rate of insurance was 14 per cent. What was the amount insured ? Ans. $9000. & Statement to find the sum insured. Explanation.—We first OPERATIO N deduct the i. º : * olicy from the total COS $114.00 cost of insurance. 100 | jº, and have in 1.50 cost of policy. 5 || 4 the remainder $112.50 the 112.50 amount of premium which is 1+ per cent of the sum insured. The reasoning $9000.00 Ans. for the statement to find this sum is the same aS given on page 446. 2. . A merchant paid $121.50 for insuring his store. The rate of premium was $1.50 per $100; the rebate was 20 per cent; the policy cost $1.50. For what $112.50 amount of premium. amount did he insure his store? * Ans. $10000. OPERATION. $121.50 premium and policy. $1.50 rate of premium. 100 1.50 policy. .30 20% rebate. 1.20 | 120.00 $120.00 premium. $1.20 net rate of premium. $10000. 3. The premium for insuring # of a stock of goods at # per cent was $137.814. What was the value of the goods? Ans. $24500. 4. The cost of insuring an invoice of merchandise was $180,104. The rate of insurance was 1% per cent; the rebate was 10 per cent; the cost of the policy $1.50. What was the amount of the invoice 3 Ans. $17640. OPERATION INDICATED. 14% = 1.125 – 10% rebate. (.1125) = 1.0125%. $180.105 — $1.50 = $178.605. (178.605 × 100) -- $1.0125 = $17640. * INSURANCE. 52 I * TO FIND THE RATE PER CENT PREMIUM, when WB HAVE THE AMOUNT INSURED AND THE AMOUNT OF PREMIUM GIVEN. PROBLEMs. 1038. 1. The cost of insuring a stock of extra hazardous goods, valued at $38400, was $1153.50 including the cost of policy, $1.50. What was the rate per cent premium ? Ans. 3%. OPERATION. Statement to find the rate per cent Explanation.—We first de- premium. duct from the total cost, the cost of the policy, and thus º 0 obtain $1152, which is the $1153.50 cost of ins. 100 amount of premium at a certain 1.50 cost of policy. 38400 || 1152 per cent on the value of the goods. We then make the g statement to find the rate per $1152.00 amt. of prem. •º 3% Ans. cent. The reasoning for this statement is the same as that given on pages, 439 and 440. ºm-sam-mm- 2. A merchant insures his goods for $12000. The insurers allow a rebate of 25 per cent on the premium, and charge $1.50 for the policy. The cost of insurance and policy was $163.50. What was the rate of premium on $100? Ans. $1.80, or 1.8%. Statement to find the amount of Statement to find the rate per - before the 25 OPERATION. Pººr cent of premium. $163.50 cost of insurance. 100 216 1.50 cost of policy. 75 162 12000 || 100 $162.00 net prem. paid. | $216 amt. of prem. chg'd | slº, = 1.8%. 3. The premium paid for insuring a house valued at $9400, was $70.50. What was the rate per cent ? Ans. #%. 4. I paid $193.50 for insuring my house for $12000. The policy cost $1.50. The rebate, in lieu of participation in the profits of the company, was 20 per cent. What was the rate per cent premium charged, and what was the actual rate paid? Ans. 2% premium charged. 14% actual premium paid. 522 SOULE's PHILOSOPHIC PRACTICAL MATHEMATICS. fºr PRACTICAL LABOR SAVING SYSTEM OF CASTING UNEARNED PREMIUMS. 1089. The following useful table is the result of the experience and genius of Mr. H. CARPENTER, an expert insurance agent of New Orleans, through whose kindness we are permitted to use it. TABLE FOR CASTING THE UNEARNED PREMIUMS, THE YEAR COMMENCING ON THE k FIRST OF JANUARY. KTo find the unearned premium, multiply the premium charged by the amount opposite the date of the policy). DATE. JANUARY. DATE. FEBRUARY. DATE. MARCH. DATE. A PRIL. 1. .0027 1 .0876 1 .1643 1 .2492 2 .0055 2 .0904 2 .1671 2 .2519 3 .0082 3 .0931 - 3 .1698 3 .2547 4 .0109 4 .0958 4 .1725 4 .2574 5 .0137 5 .0986 5 .1753 5 .2601 6 .0164 6 .1013 6 .1780 6 .2629 7 .0192 7 . 1041 7 .1807 7 .2656 8 .02.19 8 ..1068 8 .1835 8 .2683 9 .0246 9 .1095 9 .1862 9 .2711 10 .0274 10 .1123 10 .1889 10 .2739 11 .0301 11 .1150 11 .1918 11 .2766 12 .0328 12 .1177 12 .1944 12 .2793 13 0356 13 .1205 13 .1972 13 .2820 14 .0383 14 .1232 14 .1999 14 .2848 15 .04.11 15 .1260 15 .2027 15 .2876 16 .0438 16 .1287 16 .2054 16 .2903 17 .0466 17 .1314 17 .2081 17 .2930 18 .0493 18 , 1342 18 .2109 18 .2958 19 .0520 19 .1369 19 .2136 19 .2985 20 .0548 20 .1397 20 .2164 20 .3013 21 .0575 21 .1424 21 .2191 21 .3040 22 .0602 22 .1451 22 .2218 22 .3067 23 .0630 23 . 1479 23 .2246 23 .3095 24 .0657 24 .1506 24 .2273 24 .3122 25 .0685 25 .1534 25 ,2300 25 .3150 26 .0712 26 . 1561 26 .2328 26 .3177 27 .0739 27 . 1588 27 .2355 27 3204 28 .0767 28 . 1616 28 .2383 28 .3232 29 ,0794. 29 . 1643 29 .2410 29 .3259 30 .0822 30 .2437 30 .3287 31 .0849 31 .2465 The unit of this table is based on the premium of $1. for 1 year, which divided by 365, the number of days in a year, gives the premium on $1 for 1 day, which, carried to 4 places of decimals, is .0027; this being multiplied by the number of the days of the year, gives the tabular multi- pliers, which in the table presented is only extended to 4 months, or 119 days of the year. Having these multipliers, by simply multiplying the amount of the premium by the multiplier in the table corresponding with the date of the policy, we produce the unexpired premium. For example, the premium on a policy issued for 1 year from March 17th, amounts to $80. To find the unearned premium on the 31st day of December, we look for the multiplier opposite March the 17th, in the 20. * INSURANCE. 523 table, and with it multiply the $80 thus: $80 × .2081 = $16,6480, or practically $17; or in practice the premium may be multiplied by the cents only increased by 1, when the decimal is in excess of one-half, thus: $80 × .21 = $16.80; or practically $17. The above table was prepared for a company, whose fiscal year begins on the 1st of January; but it can easily be adapted to the work of other offices whose fiscal year begins at a different date, by substituting the first day of their business year for January 1st. Want of space prevents the insertion of the table for the entire year, but the 4 months presented is sufficient to demonstrate the simplicity, convenience and value of It. ADJUSTMENT OF LOSSES. 1040. 1. Jones & Smith hold three policies on a stock of goods; one policy in Co. A, for $5000, one in Co. B, for $8000, and one in Co. C, for $3000. A fire occurs and damages the goods to the amount of $9860. What is due from each Company ? Ans. $3081.25 from Co. A. $4930.00 from Co. B. $1848,75 from Co. C. OPERATIONS INDICATED. $5000 Co. A. CO. B. CO. C. 8000 9860 9860 9860 3000 16000 || 5000 16000 || 8000 16000 || 3000 $16000 2. A dwelling is insured in Company X, for $5500, and in company Y, for $3500. The house is destroyed by fire, and the companies find that they can re-build it for $7000; this amount, in accordance with the conditions of the policies, they offer to the owner, who accepts it. What sum will each company pay? Ans. Co. X, $4277.78. Co. Y, $2722.22. 3. J. M. Butchee & Co., carry policies on goods in 16 companies for $150000, Company A carries a risk of $10000. The whole stock of goods is destroyed by fire. Merchandise Account in the books of J. M. Butchee & Co. is debited with $875000 and credited with $862500. By comparing the cost and sales of 100 arti- cles of goods, it is determined and agreed between the insurers and insured that 15 per cent gain has been realized on sales, and a settlement is made on this basis. Make the bill for the amount due from Company A. OPERATION. $ $875000 total cost. 115 100 750000 cost of Sales. 86.2500 $125000 cost of goods lost. $750000 cost of sales. (BILL.) NEW ORLEANs, July 18, 1895. To J. M. Butchee & Co., Dr. For Yºº, of $125000, Mdse. destroyed by fire at No. 108 Magazine Street, New Orleans, June 2, 1895. - - - - - - - - - - INSURANCE CO. A., - - - $8333,33 Received payment, 524 soul E's PHILOSOPHIC PRACTICAL MATHEMATICS. Adjustment under the Average Clause. 1. A merchant has goods valued at $20000 in two warehouses, on which he has a policy, with the “Average Clause,” for $8000. The goods in the warehouse in which he had $12000 was damaged by fire to the amount of $6000. What is due by the Insurance Company ? Ans. $2400. OPERATION. $12000 insured by owner. $6000 8000 insured by company. 20000 || 8000 or, 20000: 8000 :: 6000 : 2400 s:0000 total insurance. $2400 Adjustments under the Three-fourth Loss or Country Clause. 1. A country merchant who had a $10000 policy on his goods, with the three- quarter loss clause, sustained a loss by fire amounting to $8000. How much do the insurers owe the merchant 3 Ans. $6000. NotE.—In policies containing the three-quarter loss clause, the insurers, in case of a total loss pay only three-fourths of the value of the property destroyed, and if the loss is less than the sum insured, the insurers pay only three-fourths of the loss. 2. If in Problem 1, with the three-quarter loss clause, how much would the merchant have received: 1. If the loss had been $10000; 2. If the loss had been $12000; 3. If the loss had been $16000 Ans. 1. $ 7500 on a $10000 loss. 2. $ 9000 on a $12000 loss. 3. $10000 on a $16000 loss. REMARKs.—In case of a $10000 or a $12000 loss, as previously stated in note to Problem 1, the underwriters are responsible for # of the loss, because it does not exceed the face of the policy and because of the # condition clause of the policy. In case of the $16000 loss, # of the same being $12000, the underwriters or insurers carrying the $10000, are responsible for the full face of the policy, $10000. NotE.-$10000 is # of $13333; ; hence, for a loss of $13333} or more, the insurers are respon- sible for the face of the $10000 policy. * Marine Adjustments. 1. The owner of § of a vessel insured # of his interest at 24 per cent; the sum paid for premium was $562.50. The vessel sustained a loss by fire of $5000. What sum do the insurers owe him # Ans. $1406.25. OPERATION INDICATED. $100 22500 5000 2 3 || 4 80000 22500 562.50 *- | a-º. -msm--assº- $30000 = # of value of vessel. $1406.25 $22500.00 = # of his # 3 || 8 $80000 = value of vessel. NOTE.-In Marine Insurance, according to the “Average Clause,” the insurers pay for a PARTIAL LOSS of property PARTIALLY insured, only such a proportion of the loss, as the sum insured bears to the whole value of the property. * 2. A cargo of fruit valued at $8500, and insured for $8000 was damaged at sea and sold at auction on arrival in port for $7500. What was paid by the insurers on the adjustment of the loss? Ans. $941.17-H. Adjustment under the Seventy-five per cent Co-insurance Clause. 1. A merchant effected $40000 insurance on property worth $60000. His policies contain the 75 per cent co-insurance clause. A loss of $30000 was ¥ INSURANCE. 525 Sustained ; what sum will the merchant receive from the underwriters in a correct adjustment of the loss, and what will be his loss as a co-insurer? Ans. $26666; due from underwriters. 3333; merchant's loss as a co-insurer. NOTE.—See Article 1028, page 513, for the 75 per cent Insurance Clause. FIRST OPERATION. $60000 × 75% = $45000 amount to be, or which is really insured. $45000 – 40000 = $5000 the sum that the merchant is co-insurer. Thus making, under the 75% clause of the policy, $45000 insurance on the property 45000 $30000 Ea:planation.—The reasoning for the line statement is as 40000 follows: Since $45000 insurance lose $30000, $1 will lose the | 45000dth part and 40000 (the sum insured by the underwriters) $26666; Ans. will lose 40000 times as much. $30000 — 26666} = $3333; merchant’s loss as co-insurer. SECOND OPERATION. $30000 $40000 × 663 = $26666}, Explanation.—Since $45000 lose $30000, $1 45000 || 100 which is the sum lost and will lose the 45000dth part, and $100 will g--- to be paid by the insur- lose 100 times as much. The loss thus pro- 663% loss. ance companies. duced, being on $100, dollars, is hence the loss per cent. $5000 × 663% = $3333; which is the sum lost and to be borne by the merchant under the 75% co-insurance clause. 2. Suppose in Problem 1, that the loss had been $45000, how much would the Insurance Companies owe ? Ans. $40000. REMARKS.—When the loss is equal to or greater than the full amount insured by the underwriters and owner as co-insurer, then the underwriters will owe the face of the policies. se 3. Suppose in Problem 1, that a total loss of $50000 or $60000 had been sustained, how much would the Insurance Companies owe ? Ans. $40000. REMARKS.—See remark of Problem 2. In no case on fire insurance can the underwriters owe to the insured more than the face of the policies. Adjustment under the Three-fourths Value Clause. Under this clause, which is inserted in some policies, the insured will receive the full amount of his loss, provided it does not exceed three-fourths of the value of the property. Thus: 1st. Suppose property valued at $1000, is insured for $750, and a loss of $600 is sustained, the insured will receive $600 from the underwriters. 2d. Suppose property valued at $1000 is insured for $750, and a loss of $850 or more is sustained, the insured will receive $750 from the underwriters. REMARKS.—This three-fourths value clause differs from the three-fourths loss or country clause explained in Articles 1030 and 1040, pages 514 and 524, inasmuch as under this three-fourths value clause the insured will, in case of a partial loss not exceeding three-fourths of the sound value of the property insured, receive the full amount of loss. But under the three-fourths loss claw8e the insured will receive only three-fourths of the amount of his loss whether it be more or less, not exceeding the face of the policy. See Article 1030a. & FINDING THE LOSS OF MERCHANDISE DESTROYED BY FIRE AND AIDJUSTING TH LOSS WITH THE INSURANCE COMPANIES CARRYING THE RISKS. PROBLEMS. 1041. To find the loss of Mdse. by fire is a far more difficult question than is supposed by one man in a thousand. To elucidate this matter, let us assume the following conditions: 1. That Jones & Smith carry $90,000 insurance on Mdse. in 12 Companies. 2. That their entire stock is destroyed by fire. 3. That their Double Entry Ledger, with all purchases and sales posted to date of fire, shows Mdse. Account debited $486,500 and credited $467,450. From these assumed facts, what has been the loss and what does Insurance Co. A OWe, which carried a policy of $8000 on the stock? º The first step in a case of this kind is to read your policies and notify the underwriters of all the known facts of the fire. Then consult with them regarding your loss, exhibiting all books and papers pertinent thereto. When the underwriters are satisfied that all is fair, then the question of GAIN or Loss per cent on the sales of goods must be determined. And the solution to 526 SouLE's PHILOSOPHIC PRACTICAL MATHEMATICS. Yºr this question is the golden key that unlocks the treasury of the underwriters carrying the risks. No correct adjustment of loss can be made before this question is solved. It is to the interest of Jones & Smith to show a large gain per cent on sales; and it is to the interest of the Insurance Companies to show a small gain per cent on sales. If an agreement cannot be reached by the two parties in interest, from the general accounts, then an average gain per cent must be determined by listing the cost and sales of a number of the leading lines of goods, thus: Statement to find average gain Statement to find cost of sales of Mdse. based on Cost. Sales. per cent. an average gain of 16.19 per cent. $ 4. 4.80 4.31 100 .22 .30 116.19 || 467450.00 1.50 1.65 26.62 | 100 2.75 ( 3.00 * | $402315.17 cost of sales. .15 .18 | 16.19 96 gain. 18. 21.00 Total cost of Mdse., $486500.00 $26,62 || $30.93 Cost of sales “ 402315.17 Net Gain, $4.31 Loss by fire, $ 84.184.83 In the above statement of cost and sales, to abridge the work, we limited the statement to six items. In practice, FIFTY TO ONE HUNDRED should be used. In the selection of the items lies the great difficulty. The underwriters insisting on items upon which a small gain was realized, and Jones & Smith insisting on items upon which a large gain was realized. A nice discrimina- tion is required in making this statement to render justice to both parties. The author hereof, in behalf of his client, had nine sittings of two hours each, with Insurance Adjusters, in a vain : make a statement of costs and sales, from which the average gain per cent should be accepted. Where the parties in interest fail to agree upon this point, arbitration of the question seems to our mind to be the most ethical and reasonable. f $8 The following statement shows the amount of loss due from the Insurance Co. having a risk Of $8000. Total amount insured, $90000. Total amount lost, $84.184.83. Hence the Co. owes sº of $8000 = $7483.10. INSURING GOODS. 1042. Business prudence demands that merchants and manufacturers should give careful attention to their stock of goods and to the amount of insurance covering the same, adding or cancelling policies so as to keep fully protected and losing nothing by over-insurance. A careful record should be kept of the date the policies expire, and daily inspection should be made thereof. The insured should also note with care all the special clauses and conditions imposed by his policy. The Lightning, the Average, the Foundation and the 75% or more Co-Insurance clauses should be thoroughly understood when the policy is written. Points to be Specially Observed in the Mdse. Insurance. When considering merchandise account and the statement of costs and sales, due regard must be given to the following elements on purchases and sales: e Sal (1). Trade Discounts. (2), Cash Discounts. (3). Rebates. (4). Time Purchases and Time ableS. The merchandise returned to sellers, and the merchandise re-purchased from purchasers, if entered in merchandise account, must be eliminated therefrom. After considering all these ele- ments, and finding the loss, then the question of depreciation and appreciation of the goods lost is to be considered, in order to find the cash value of the goods, which the policy of insurance COVOTS. It is the custom of Insurance Companies, when adjusting fire losses, to allow #9% discount for each 30 days, on time purchases and on time sales. This is equal to 6% interest. VALUED POLICIES ON IMMOVABLE PROPERTY. - The Valued Policy Insurance Law of Louisiana, enacted in 1900, requires that the insurance on immovable property shall be the value assessed or appraised “by the insurer, or as by him per- mitted to be assessed at the time of the issuance of the policy,” and that this value “shall be conclusively taken to be the true value of the property at the time of the issuance of the policy, and the true value of the property at the time of damage or destruction.” sº g On this value, Insurers are obliged to adjust the loss. The law, however, gives the insurers the right to restore the damage or to replace the property destroyed if they prefer to do so, instead of making payment therefor. * By this law the adjustments of loss are easily and expeditiously made, and the vexatious contentions, so often made by the insurers under the former law, are obviated. GENERAL AND PARTICULAR AVERAGE. 527 TAKING STOCK OR INVENTORYING GOODS. When “taking stock,” or inventorying goods and other property preparatory to closing the books and declaring the gains and losses at the close of the fiscal year, or in case of a corpora- tion, at a dividend period, the current market value should be placed upon the goods and all other property, stocks, bonds, etc. NOTE.-In the case of stocks and bonds which have an unstable value and a wide margin of fluctuation, the average Weekly or monthly value is sometimes placed upon them. Circumstances, judgment and ethics are the proper factors to govern such cases. In the case of merchandise, instead of listing every article at the current market value With allowances for depreciation on account of damage, out of fashion or style, the inventory is often taken on different lines of goods, at the orignal invoice price, and then a specified per cent. reduction or addition is made thereto, according to the present market price of such lines of goods; allowances being made also for damage, old stock, etc. In some cases of unimproved real property, it is often difficult to determine its market value. Whether it is worth the original cost, more or less, plus taxes, is a question to be decided by the facts attending the case, and the judgment of the appraisers. General and Particular Average. 1043. General Average is the proportional contribution by all parties con- cerned in a vessel, cargo or freight, towards a loss or damage sustained or incurred for the common safety or benefit. 1044. Particular Average is the contribution for loss or damage incurred or sustained by or for one or more of the interests, vessel, cargo or freight; such loss much be borne by the interest or interests which sustained the loss. 1045. . An Average Adjuster is one whose profession it is to average and adjust losses in which general and particular average cases occur. He is required to be familiar with insurance laws governing the subject, and to determine which loss should be charged to General Average and which to Particular Average, and then to apportion the loss and expense among the several contributory interests. To determine, in all cases, what kind and character of losses and damages are subject to general average and what to particular average is a very difficult matter and often gives rise to prolonged litigation. Arnould's Maritime Law and Abbott, on Shipping, give valuable information on this topic. The following are some of the losses and damages subject to general average: 1st. The damage sustained by a ship purposely run ashore to prevent her foundering at sea or driving on the rocks. 2d. All damage purposely done to the vessel or cargo, to preserve the whole from impending danger, such as the cutting away of masts, riggings, etc., when the ship is in distress. 3d. All charges incurred for the general good, by entering a foreign port in distress. 4th. The expense for dis- charging the cargo for the purpose of repairing the vessel, or for floating her when she accidentally gets aground. 5th. The expense of pilotage when putting into a port in distress. 6th. The expense of labor to pump the ship after having sprung a leak. In some cases, seamen's Wages and the provisions of the ship's company are subjects of general average. The following are a few of the losses and damages subject to particular average : 1st. Damage by fire from accidental causes, unattended by a sacrifice to extinguish the fire; also the damage to goods by water thrown upon the burning goods to extinguish the fire, which is regarded not a sacrifice of goods, but a real advantage to them. 2d. The damage done to a vessel and cargo by being run foul of accidentally and without fault on either side. 3d. The springing of masts, breaking of the upper works or timbers, parting of cables, and all other loss and expense upon the ship occasioned by the perils, insured against not purposely incurred, and not amounting to a total loss. 528 SOULE's PHILOSOPHIC PRACTICAL MATHEMATICS. * 1046. Jettison. The goods thrown overboard, the cutting away of masts or cables, and whatever the master of a vessel does in distress for the safety or preservation of the whole, is called jettison. In making an adjustment of general average, it is necessary to first take an account of the various losses which are to be paid by contribution, and secondly, to take another account of the value of all the articles that are to contribute. In the list of contributory articles—vessel, cargo and freight—must be included the goods, etc., thrown overboard; for otherwise the owners of these goods would receive full Value for them and pay nothing towards the loss. In estimating the amount of loss to the vessel for masts, rigging, etc., it is the custom to deduct of the cost of the new articles; for being new they will be of greater value than the articles lost. In estimating the value of the contributory interests, the general custom is as follows: To value the ship at what it is worth at the end of the voyage; to value the cargo at what it would have brought at its port of destination, but it is sometimes valued at its invoice price at the port of lading; and to value the freight at # the amount due at the time of the jettison or other sacrifice. But in some ports the value of the freight is determined by deducting from the gross amount the wages for seamen, pilotage, etc. In New York, # of the freight is deducted. As remarked in the introduction of this subject, it is so vast and complicated that it cannot be satisfactorily treated in a work of this kind, and it is questionable Whether or not it is proper to present it in works of this kind. It is a subject that merits and has received the attention of the most profound jurists of both hemi- spheres. We have briefly introduced it, in conformity to custom, and present the following, to show the operation for a general and particular average computation : PROBLEMIS. 1. The steamer City of Baltimore left New Orleans for Philadelphia with a general cargo amounting to $150000. During the voyage, the vessel encountered very severe weather, and goods to the amount of $4500 were thrown overboard. Masts and rigging were cut away which cost $810 to replace; one anchor cost $250. The expenses of the vessel in port while being repaired were $540. The charges for protest and adjustment of average were $250. The gross amount of freight amounted to $12420. The value of the vessel is $90000. There were 300 hbds. of sugar on board, 158 of which were damaged by water, according to the report of the surveyor, to the extent of 34 hbds., which, at $100 amounts to $3400. The goods and vessel were fully insured. What is the general and particular average loss per cent., and the amount of loss sustained by each contributing interest, estimating the cargo at the port of lading invoice? Ans, 2.44884 + 7% general average loss. 114% particular average loss. $2203.97 to be contributed by vessel. $3673.26 “ {{ “ cargo. $ 202.77 4. 4% “ freight. $3400.00 ($ {{ “ sugar. * GENERAL AND PARTICULAR AVERAGE, 529 OPERATION. General Average Loss. Masts, rigging, etc., - - - -- * - &n $810 Less # deducted because new * - sº - * 270 $ 540 Goods thrown overboard - *-> - º - s 4500 Anchor, cost of new - T - º - sº • Ge 250 Expenses in port º º º - tº- - es 540 Protest and average adjustmen º - º - ºs 250 Total amount of general average loss - wº $6080 Particular Average Loss. 34 hlids. sugar (a) $100 tº- - i- º sº tº- - tº - $3400 Contributory Interests to General Average Loss. Value of steamer (when repaired) - sº as $ 90000 Cargo (invoice value) º tº - - sº - 150000 Freight - - º º º º - s - $12420 Less #, allowed for expenses - - s - 4140 8280 Total amount of contributory interests $248280 Contributory Interests to Particular Average Loss. 300 hbds. sugar (a) $100 - - ºn - gº *- sº * ºn $30000 Statement to find the General Average per Statement to find the Particular Average cent Loss. per cent Loss. $ LOSS. $ LOSS. 6080 3400 248.280 100 º 30000 || 100 2####% = 2.44884 + 7. 11; % Statement showing the amount of loss that each contributing interest must pay. Wessel, $ 90000 (a) 2.44884% tºº º $2203.97 Cargo, 150000 (a) 2.44884% - “º - 3673.26 Freight, 8280 (a) 2.44884% - tº- - 202.77 Sugar, 30000 (a) 118% - º wº- 3400.00 REMARKS.–In actual practice, average adjusters individualize every con- signee's invoice, and specify the loss on each article of goods. They also itemize every expense incurred in the settlement of the various losses. When the rate per cent loss contains an inconvenient fraction, as is the case in the above example, it is extended to three, four or more places of decimals, and a table prepared therefrom to facilitate the operation of multiplying the amounts of the different invoices composing the cargo. If the contributing interests were few in number, time might be saved by making proportional statements to find the loss that each interest must Sustain. 53o soul E's PHILOSOPHIC PRACTICAL MATHEMATICS. jºr * 2. In the above problem, suppose that Jones & Co. had an invoice of general merchandise valued at $2000, fully insured, which suffered no damage, what would be the sum due by Jones & Co. or by their insurers? Ans. $48.97+. OPERATION. $2000 x 2.44884+% = $48.97+. NoTE.—Since the cargo is one of the contributory interests to the general average, each . of goods in the cargo (or his insurer) must pay on his invoice the general average per cent Of 108S. 3. In the same problem, suppose that Smith & Co. had an invoice of sugar valued at $1000, which suffered no loss, what would be the sum due by Smith & Co. or by his insurers? * Ans. $113.33%. OPERATION. $1000 x 113% = $113.334. NOTE:-The loss on sugar is a Particular Average Loss inasmuch as it was not incurred for the common safety of the vessel, cargo and freight, therefore each owner of sugar (or his insurer) must pay on his involce the particular average per cent loss. - 4. The ship Queen of the Ocean, on her voyage from Bombay to New Orleans, was struck by a severe gale in the Gulf of Mexico, and her sails, rigging and upper Works were torn and damaged to the amount of $1500. To relieve her sufferings, goods were thrown overboard to the amount of $9400. And one mast was cut away, the cost of which, to replace it, was $360. The cost of the surveys, protest and adjustment of average amounted to $850. The vessel, when repaired, was worth $50000. The value of the cargo in New Orleans was $110000. The freight amounted to $9840. The cargo and vessel were fully insured. What was the general and particular average loss per cent sustained by each of the contributory interests 7 Ans. 6+++4% or 6.2980+% general average loss. 2% particular average loss. OPERATION. General Average Loss. Value of goods thrown overboard º ge º- º $ 9.400 Cost of mast - º - º - tº - $360 Less # for new - - -> > * º -> - 120 240 Cost of protest, surveys and adjustment of average 850 Total amount of general average loss - $10490 º: GENERAL AND PARTICULAR AWERAGE. 531 Particular Average Loss. Cost to repair damage to sails, rigging and upper works - - $1500 Less # deducted for being new tº gº * = g- tº º 500 Net amount of particular average loss - tº s * > ſº $1000 Contributory Interests to General Average Loss. . Value of vessel (after being repaired) tº ſº tº $ 50000 Value of cargo in New Orleans gº ºn tº gº 110000 Freight sº tº tº sº º gº gº e tºº $9840 Less #, allowed for expenses s ſº ſº º ſº 3280 6560 º Total amount of contributory interests - $166560 Contributory Interests to the Particular Average Loss. Value of vessel tº is tº ſº º gºe * > E = $50000 Statement to find the general average loss per cent that each interest must sustain, $ LOSS 10490 166560 100 | 6}#}% Ans. or 6.2980-- 7%. Statement to find the particular average per cent loss that the insurers of the vessel must sustain. $ 1000 50000 || 100 2% Ans. ife Insurance. A-/-/-/-/-/-/******A*/~~~~~~ES. *RS- 1047. Life Insurance is a contract by which a life insurance company, in con- sideration of certain payments made by the insured, agrees to pay a specified sum of money to his heirs at his death, or to himself if living, at a specified age. 1048. Life Insurance Companies are organized on the plan of Stock Com- panies, Mutual Companies, Mixed Companies, and on the Co-operative and Benevo- lent Association plan. NOTE.—See Fire Insurance for definition of each plan, page 511. IMPORTANCE OF LIFE INSURANCE. 1049. The subject of Life Insurance is of vast importance to all classes of people, the rich as well as the poor. For in the ever-evolving drama of life, fortunes often disappear, and in the contests for wealth and fame small gains to the many are the rule, while to some, cold faced poverty is the life and death companion. Hence the rich should insure to guard against misfortune; those who are not rich should insure so that in case of early death, their families and those dependent upon them for support, would be protected from want and suffering; or in case of old age, they would possess the means through an Endowment, or an Annuity Policy, or a Trust Certificate, of self-support. From the policy treasuries of the various forms of Life and Co-operative Insurance, come millions of dollars annually to relieve the necessities of distressed widows, of orphan children, and of decrepit people. And for this, thanks and gratitude are due to the wisdom, to the humanity, and to the practical sense of those who provided the insurance. The world is most liberal and gracious in bestowing upon needy, afflicted, and suffering humanity, words of sympathy, tears of pity, and prayers of supplication. But these gifts, however bounteous, do not pay rent bills; they do not supply covering for naked bodies; they do not provide sustenance for hungry stomachs, or medicine for fevered brains. Therefore, by the light of these facts, and since by law of nature all must live before they die, we conclude that it is the first and high- est duty of man to provide for himself and those dependent upon him, and to assist in the financial charities of the world. And all this we submit can be best accom- plished through Life or Co-operative Insurance. (532) LIFE INSURANCE. 533 The Policies of Life Insurance vary in their conditions and are named as follows: 1050. ORDINARY LIFE, LIMITED PAYMENT LIFE, TERM. ENDow MENT, RESERVE ENDOWMENT, ANNUITY, TonTINE SAVINGs or INVESTMENT, Joint LIFE POLICY, AND TRUST CERTIFICATE POLICY. There are also some other special policies issued by some companies. 1051. An Ordinary Life Policy is one on which the premium is paid annually during the life of the insured, and at his death, the amount of the policy is paid to the party named therein. 1052. A Limited Payment Life Policy is one on which the premium is paid annually for a certain number of years, specified at the time the policy is issued or until the death of the insured, should that occur before the elapse of the time speci- fied. This class of policies is also issued on single payments, or on 5, 10 or more annual payments. When issued on one payment, the insured receives annual cash dividends from the company. 1053. A Term Policy is one payable at the death of the insured if it occur during a given number of years, the payment of the annual premiums to continue till the policy expires. gº 1054. An Endowment Policy is one that is paid to the insured at the end of a specified number of years, or to his heirs in case he should die within the time. The premium is payable annually, or in cash, in 5 or 10 annual payments. 1055. A Reserve Endowment Policy is one combining the Life and Endow- ment Plans, the premium being the same as in a Life Policy, but the insurance terminates at such a time as the insurer names, when an endowment will be paid equal to the legal reserve of the policy. 1056. An Annuity Policy is one which secures to the insured the payment of a specified sum annually, as long as he lives. The premium is paid in one cash payment. 1057. A Tontine Savings or Investment Policy is one payable at the death of the insured, but in which the excess of premiums received over the claims by death and expenses, is divided at the end of a specified period of time, among the survivors of those insured for that specified time, and at this time these survivors may either accept the money as in an Endowment Policy, or change the form of their policy as may be agreed. 1058. A Joint Life Policy is one payable at the death of the first, of two or more persons, to the survivor. The premium is payable in the same manner as in Life Policies. - 1059. A Trust Certificate Policy is a policy issued at a rate of premium much lower than the ordinary policies, and by which the beneficiary named therein receives a specified number, more or less, as may be desired, of annual payments after the death of the insured. 534 SOULE's PHILOSOPHIC PRACTICAL MATHEMATICs. * For the heads of families, this is one of the best methods of insurance, as by it the money can neither be lost nor squandered, and the family receives a certain annual income to protect them from care, to educate the children, etc. 1060. A Non-forfeiting Policy is one which does not become void because of the non-payment of premiums; provided that two or three annual payments have been made thereon. NOTE:-The limited forfeit law of life insurance of Massachusetts, makes the policy non- forfeiting when two full annual payments have been made; the laws of the State of New York, make the policy non-forfeiting when three full annual payments have been made. 1061. A Dividend in Life Insurance Companies is a share of the profits or surplus paid to the stockholders in stock companies, to policy holders in mutual Companies, and to stockholders and policy holders in mixed companies, after the Iiabilities of the company are reserved. 1062. The Reserve Fund of a Life Insurance Company is a sum which, invested at a given rate of interest, together with the maturing premiums on exist- ing policies will produce a sufficient revenue to pay all obligations as they mature Or arise. 1063. Surplus is the sum left, after providing for the liabilities, claims and expenses of the company. 1064. Script is a Certificate entitling the policy holder to certain profits or Surplus, when payable. 1065. Loading or Margin, is a percentage added to the net premium to defray expenses and provide for an excess of mortality. 1066. The Reserve of Life Insurance Companies is that part of the premiums of all policies, with the interest thereon, which is reserved or set aside as a fund for the payment of policies when they become due. The law of New York State requires Life Insurance Companies to reserve such a part of the premiums as will, at 4 per cent interest, amount to a sufficient sum to pay all policies. The State of Massachusetts specifies 4% per cent interest on the reserve. 1067. Walue of a Policy. “The net value of a policy is the difference between the net single premium for the sum insured at the age of the policy holder when the policy is valued, and the present value of all future net premiums calcula- ted to be received on the life of the party insured. The gross value of a policy is the difference between the net single premium, as given above, and the present Value of all future gross premiums to be received on the policy.” 1068. The Surrendered Walue or Market Walue of a policy is commonly determined by deducting from the reserve 25 to 50 per cent, as a surrender charge, and to pay the remainder as an equitable surrender value. Much discussion has followed this method of finding the surrendered value of policies, to which we refer those interested to the “Principles and Practice of Life Insurance.” 1069. The Rate of Premium is based mainly upon the probability of life as shown in the following tables; but, specifically, it involves six elements: 1. The reserve fund, as above defined. 2. The probable rate of interest on the reserve * LIFE INSURANCE. 535 fund. 3. The cost of mortality, which is the estimated amount of each insurer's share of the losses by death each year. 4. The loading, which is the part of the premium required for expenses. 5. The age of the insured. 6. The period of insur- ance and the kind of policy. CO-OPERATIVE OR BENEVOLENT INSURANCE ASSOCIATIONS. 1070. Associations of this kind, of which there are several, as described on page 509, differ materially in their general features and plan of work, from the regular Life Insurance Companies. They also differ from one another in Some respects, while on many points there is a close similarity. The Knights of Honor, which is one of the leading Co-operative Associations or Orders of this class, works on the following plan: Members are admitted by vote into social societies called Lodges, and on the payment of an assessment of from one to two dollars, according to age, receive a Benefit Certificate for $2000, These certificates are the equivalent of the policies in Life Insurance Companies and are payable at the death of the member to the beneficiary named therein. The continuance of the Benefit Certificate is conditioned on the prompt payment of all subsequent assessments upon the holder or member. To replenish the fund for the payment of the Benefit Certificates, assessments are made at different times on a graded plan according to the age of the members, which vary in amount from $1 to $1.50. The sum thus raised is paid into the General Fund called the Widows' and Orphans' Benefit Fund, from which is paid the Benefit Certificates of all deceased members until the fund is reduced below $2000. Then another assessment is made and paid, and thus the fund is maintained and the certificates of deceased members are paid. gº Very similar principles and methods govern nearly all the Co-operative Insur- ance Associations, and thus they offer economical advantages to all who desire to insure, but more especially do they commend themselves to persons of small or limi- ted means. By the co-operative plan of insurance, the assessments are collected and paid over to the general fund by the officers of the societies or lodges, without charge or commission, and the reserve and surplus, it is claimed, are in the pockets of the members. The Knights of Honor issue half rate Benefit Certificates for $1000. Some of the other Co-operative Associations issue Benefit Certificates ranging from $500 to $5000. f - The amount of the assessments is determined upon the expectancy of life, the age of the member when he joins the order, and some other elements. 1071. Debenture. Insurance or Investment, is an investment evidenced by a written instrument specifying the amount invested, the revenue to be derived therefrom, the conditions upon which it is received, the manner of payment, etc. While some Debenture Companies are conducted ethically, and are based upon correct financial and mathematical principles to ensure their solvency, many are founded upon a chimerical and delusive basis, and are conducted through fraud and 536 SOULE's PHILOSOPHIC PRACTICAL MATHEMATICs. ºr false pretense. Hence all who would patronize Debenture Companies should critically investigate their Charter, the financial principles upon which they work, the management, promises, etc. of the company, before they invest in Debenture Bonds. 1072. From the Manual of the Life Association of America, and the Principles and Practice of Life Insurance, we collate the following Statistics and the Tables of Comparative Expectancy of Life and of Mortality. BASIS OF LIFE INSURANCE. 1073. The duration of human life is governed by a law as regular in its opera- tion as any other law. While the actual continuance of any individual life is wholly uncertain, the average duration of a large number of lives can be accurately fore- told. Ten thousand persons of ordinary health, at the age of 35 years, will survive 31 years longer, on an average. Consequently, the expectation of life for a person 35 years of age is 31 years. sº The basis on which the whole structure of life insurance rests is the law of mortality among the human race. Strange as it may seem, war, pestilence, and famine cause but slight variations in the operation of this law. In spite of these destructive agencies, happening as it were by chance, in spite of all the unforeseen accidents and catastrophies to which human life is exposed; in spite of the absolute uncertainty of prolonged life in any individual case, yet the average duration of existence among a large number of persons, say 10,000 or 100,000 is a mathematical verity—a certainty as fixed and invariable as any other law of nature. HOW HAS THIS LAW BEEN DISCOVERED AND WHAT ARE THE PROOFS OF IT?— 1074. Within the last 200 years, close observation and actual measurement of the duration of life have been made among different classes of persons in different countries, and under entirely different sets of circumstances. These tests, made independently of one another, extended to embrace large numbers of persons—in some instances, entire populations—so fully corroborate and verify one another, that there is left no reason to doubt that the law of mortality is accurately determined, and that it is in fact an absolute law of nature. The separate and distinct observations commonly adduced as proof or authority for this law are thirteen in number. The following are those in general use in America and England: The American Experience, the Carlisle, the Farr or English Life No. 3, the Actuaries or Combined Experience, and the Northampton. 1075. The American Eaſperience is made from actual experience of several leading American Companies, extending over a period of 25 years. 1076. The Carlisle table is a record of the mortality of the town of Carlisle, England, from 1778 to 1787. 1077. The Farr or English Life No. 3, is a record of mortality of all ages throughout Great Britain, from 1838 to 1844. 1078. The Actuaries or Combined Experience is a record of mortality based on the record of 62,537 assured lives in 17 Life Companies in England. The Northampton Table was prepared by Dr. Price of Northampton, England. 2} LIEE INSURANCE. 537 TABLE NO. 1. 1079. C4OMPARATIVE EXPECTANCY OF LIFE. A Table showing the expectancy of life in years and hundredths of a year, deduced from four English and the American Experience Tables. Farr or |Actuaries |American Farr or |Actuaries|American Age. North- | Carlisle. English , or Com- || Experi- || Age. North- Carlisle, English , or Qom. Experi- ampton. Life, İbined Ex-| ence. ampton. Life, [bined Ex- ence. No. 3. pelience. No. 3. perience. 1. 32.74 44, 67 || 46.65 º e º e º e º º 53 | 16.54 18.97 17.67 | 18.16 | 18.79 2 37.79 || 47.55 || 48.83 tº e º e a e e Q 54 | 16,06 18.28 17.06 17.50 18.09 3 39.55 49.81 || 49.61 tº e º e tº e o º 55 15.58 17.58 16.45 16.86 17.40 4 | 40.58 || 50.76 || 49.81 & © Cº. e. tº e º 'º 56 | 15.10 16.89 15.86 16.22 16.72 5 40.84 || 51.24 || 49.71 tº e º ſo 57 || 14.63 | 16.21 15.26 15.59 || 16.05 6 || 41.07 || 51.16 || 49.39 G e º O 58 || 14.15 15.55 14.68 || 14.97 15.39 7 41.03 || 50.79 || 48.92 tº e º e © e º tº 59 || 13.68 14.92 14.10 14.37 14.74 8 || 40.79 || 50.24 || 48.37 tº e º e - e º & 60 | 13.21 14.34 13.53 13.77 || 14.09 9 40.39 49.57 || 47.74 e - - - tº e º º 61 | 12.74 13,82 12.96 13, 18 13.47 10 39,78 || 48.82 || 47.05 || 48.36 || 48.72 62 12.28 13.31 12.41 12.61 | 12.86 11 || 39.14 || 48.04 || 46.31 || 47.68 48.08 63 || 11.81 | 12.81 11.87 12.05 || 12.26 12 38.49 || 47.27 || 45.54 47.01 47.45 64 11.35 12.30 11.34 11.51 11.68 13 || 37.83 || 46.50 || 44.76 || 46.33 46.82 65 10.88 || 11.79 10.82 10.97 || 11.10 14 37.17 | 45.74 || 43.97 45.64 46. 16 66 || 10.42 11.27 10.32 10.46 10.54 15 || 36.51 || 44.99 || 43.18 || 44.96 || 45.50 67 9.95 || 10.75 9,83 9.96 || 10.00 16 || 35.85 || 44.27 || 42.40 || 44.27 44.85 68 9.50 | 10.23 9.36 9.47 9.48 17 | 35.20 || 43 57 || 41.64 || 43,58 44.19 69 9,05 9.70 8.90 9.00 8.98 18 || 34.58 || 42.87 || 40 90 || 42.88 43.53 70 8.60 9.15 8.45 8,54 8.48 19 33.99 || 42.16 || 40.17 || 42.19 42.87 71 8.17 8.65 8.03 8.10 8.00 20 33.43 || 41.46 || 39 48 || 41.49 42.20 72 7,74 8.16 T.62 7.67 7.54 21 32.90 | 40.75 || 38.80 40.79 41.53 73 7.32 7.72 7.22 7.26 7,10 22 32.39 || 40.03 || 38.13 40.09 40.85 74 6,92 7.33 6.85 6.86 6.68 23 31.87 || 39.31 || 37.46 || 39.39 40. 17 75 6.54 7.00 6.49 6 48 6.28 24 31.36 || 38.58 || 36.79 |- 38.68 39.49 76 6,18 6.99 6.15 6.11 5.88 25 30.85 || 37.86 || 36.12 37.98 38.81 77 5.83 6.40 5.82 5.76 5.48 26 30.33 || 37.13 ig5.44 || 37.27 38.11 78 5.48 6.11 5,51 5.42 5.10 27 | 29.82 36.40 || 34.77 || 36.56 37.43 79 5.11 5.80 5.2.1 5.09 4,74 28 29.30 || 35.68 || 34.10 35.86 36.73 80 4.75 5.51 4.93 4.78 4.38 29 28.79 34.99 || 33.43 35.15 36.03 1. 4.41 5.20 4.66 4.48 4.04 30 28.27 | 34.34 || 32.76 || 34.43 35.33 82 4.09 4.93 4.41 4.18 3.71 31 27.75 33.68 || 32.09 33.72 34.62 83 3.80 4.65 4.17 3.90 3.39 32 27.24 || 33.02 || 31.42 33.01 33.92 84 3.58 4.39 3.95 3.63 3.08 33 26.72 32 36 || 30.74 32.30 33.21 85 3.37 4.13 3.73 3.36 2.77 34 26.20 31.68 30.07 || 31.58 32.50 86 3.18 3.90 3.53 3.10 2.47 35 25.68 31 ()0 || 29.40 30.87 31.78 87 3.01 3.71 3.34 2.84 2.19 36 25, 16 || 30.32 28.73 30.15 31.07 88 2.86 3.60 3.16 2.59 1.91 37 24.64 || 29.63 || 28.06 || 29.44 30.35 89 2.66 3.47 3.00 2.35 1.66 38 24. 12 28 65 27 39 28.72 29.63 90 2.41 3.28 2.04 2.11 1.42 39 23.60 28.27 | 26.72 28.00 28.90 91 2.08 3.26 2.69 1.89 1.19 40 23.07 27.61 || 26.06 || 27.28 28.18 92 1.75 3.37 2.55 1.67 .98 41 22.56 26.97 25.39 26.56 27.45 93 1.37 3.48 2.41 1.47 .80 42 22.04 26.34 24.73 25.84 26.72 94. 1.05 3.53 2.29 1.28 .64 43 21 54 25 71 || 24.07 25.12 25.99 95 .75 3.53 2.17 1.12 .50 44 21.03 || 25.09 || 23.41 24.40 25.27 96 .50 3.46 2.06 .99 tº º º 45 20.52 24.45 22.76 || 23.69 24.54 97 tº e º 3.28 1.95 .89 46 20.02 23.81 22.11 22.97 23.81 98 • Q ſº 3.07 1.85 .75 o 47 19.51 23.17 | 21.46 22.27 23.08 99 2.77 1.76 .50 48 19.00 22.50 20.82 21.56 22.36 100 2.28 1,68 tº dº º o 49 18.49 || 21,81 20.17 | 20.87 21.63 101 1.79 - e º º e 50 17.99 21, 11 | 19.54 20.18 20.91 102 1.30 tº gº 51 17.50 | 20,39 || 18.90 19.50 | 20.20 103 | . . . . .83 & e º º tº $ tº e © e º ºs 52 17.02 || 19.68 | 18.28 | 18.82 | 19.49 104 | . . . . .50 © c e e e - © º O Gº e 9 3. 538 SOULES PHILOSOPHIC PRACTICAL MATHEMATICS. ºf 1080 [TABLE NO. 2]. • COM PA RATIVE MoRTALITY TABLE,-showing the number expected to die out of 1000 a-- persons entering each year, according to three English and the American Experience Tables. Age. |American Experience. Carlisle. | Actuaries. English Life No. 3. Age. 10 7.49 f • * * * . e e º & tº e º e C - 15 7.63 ū e s tº © e º ºf © e º 'º tº & 20 7.80 tº º tº e tº G & e tº e º e & e 21 7.85 Q & © e. tº - e. e. tº e º 'º tº e 22 7. 90 tº e º e e e & e tº a º e e 23 7.95 tº a tº e 24 8.01 tº º º º tº e º e tº º 4 ºn © º 25 8.06 7, 31 7.77 9.20 25 26 8.13 7, 36 7.88 9.38 26 27 8.19 7.76 8.00 9.55 27 28 8.26 8.69 8. 13 9.74 28 29 8.34 9.82 8.27 9.93 29 30 8.42 10.10 8.42 10. 13 30 31 8.51 10.20 8.57 10.34 31 32 8.60 10. 13 8.74 10.56 32 33 8.71 10.05 8.91 10.80 33 34 8.83 10.15 9.09 11.05 34 35 8,94 10.25 9.28 11.33 35 36 9.08 10.55 9.48 11.62 36 37 9.23 10.85 9.68 11.94 37 38 9.40 11. 16 9.90 12.29 38 39 9.58 11.87 10. 13 12.65 39 4 9.79 13.05 10.36 13,06 40 41 10.00 13.77 10.61 13.48 41 42 10.25 14.37 10.89 13.94 42 43 10.51 14.58 11.25 14.44 43 44 10.81 14.79 11.69 14,97 44 45 11. 16 14.80 12. 21 15.54 45 46 11.56 14.81 12.83 16. 15 46 47 12.00 14, 60 13, 51 16.18 47 48 12.50 13.93 14.25 17.49 48 49 13. 10 13.68 15.06 18.23 49 50 13.78 13.41 15.93 19,02 50 51 14.54 14.29 16, 89 20.42 51 52 15 38 15. 20 17.94 21.45 52 53 16,33 16 14 19.09 22,51 53 54 17.39 16.89 20.31 23.64 54 55 18,57 17.92 21.66 24.85 55 56 19.88 19.00 23.12 26.17 56 57 21.33 20.89 24.67 27.63 57 58 23.10 24 20 26.38 29.25 58 59 24.72 28. 27 28. 24 31.05 59 60 26.69 33.48 30 .33 33.05 60 61 28 88 35, 78 32.61 35.29 61 62 31, 29 37.40 35. 12 37.77 62 63 33.94 38.25 37.83 40.53 63 64 36,87 39.77 40.82 43.60 64 65 40, 12 41 08 44.08 46.98 65 66 43.76 42.50 47.61 50.71 66 67 47.64 44.38 51.47 54.83 67 68 52.00 46.45 55.63 59. 33 68 69 56.76 49. 10 60.08 64.25 69 70 61.99 51.64 64.93 69.62 70 71 67.66 58.85 70.01 72.70 71 72 73.73 68. 12 75.80 78.54 72 75 94.36 | . . . . . . . . . . . . . . . . . . . . 80 144. 46 | . . . . . . . . . . . . . . . . . . . . 85 235. 55 . . . . . . . . . . . . . . . . . . . " 90 454. 55 | . . . . . . . . . . . . . . . . . . . . 94 857.14 ' . . . . . . . . . . . . . " ' ". . . . . . The above table shows how many of 1000 persons will die in the year, for each age The foregoing tables and the intºrest and annui tions of Life Insurance Companies. determined, but that of a multitude of lives is easily ascertained. iven. e principal data for the calcula: one life can be of the calcula- tables, given farther on, furnish º y life insurers or others, that the duration of an It is not claimed - By reason of this, the ultimate resul tions of life insurers are always correct, for if one person lives longer than those insured with him, he helps those who die early; and if he dies early, they help him. . º LIFE INSURANCE. 539 TABLE NO. 3. 1081. SEHOWING TELE ANNUAL PREMIUM RATES FOR AN INSURANCE OF $1000. LIFE POLICIES.—PAYABLE AT DEATH ONLY. EnſlºwmºntPOliſles—Payallbasińlltalèſ, OI at DBālliſ VIII, ll Payments to continue for - * Twent Age. Fift T t Age. ge Life Ten Years º º Tºy Ten Years. º ºy *ś. ge 21 $18.60 $40.50 $30.80 $26.10 $105,40 $66,90 $48.20 $76.30 21 22 19 10 41, 20 31,30 26.60 105.50 67.00 48.30 76.50 22 23 19.50 41.90 31 90 27.00 105,70 67.10 48.40 76.60 23 24 20.00 42.70 32.50 27.60 105.80 67.20 48.60 76.70 24 25 20.50 43.50 33.10 28, 10 105,90 67,40 48.70 76.90 25 26 21 00 44.30 33 80 28 60 106.00 67.50 48.90 77.00 26 27 21.50 45.20 34 40 29.20 106.10 67,60 49.00 77.20 27 28 22.10 46, 10 35 10 29.80 106.30 67.80 49.20 77.40 28 29 22.70 47.00 35 90 30 50 106,40 68.00 49.40 77.60 29 30 23 30 48 00 36.60 31.10 106,60 68.20 49.60 77.80 30 31 24.00 49. 10 37 40 31.80 106.80 68,30 49.80 78.10 31 32 24,70 50.10 38,30 32.60 107,00 68 60 50.10 78.30 32 33 25.50 51.20 39.10 33.30 107.20 68 80 50.30 78.60 33 34 26.30 52.40 40.00 34.10 107.40 69,00 50.60 78.90 34 35 27, 10 53.60 41 00 35.00 107,60 69.30 50.90 79.20 35 36 28 00 54,80 42 ()0 35,80 107.80 69.60 51.30 79.60 36 36 29 00 56 20 43,00 36 80 108 10 69 90 51.70 80.00 37 38 30,00 57 50 44 10 37.70 108.4() 70.20 52.10 80.40 38 39 31, 10 59 00 - 45 30 38 80 108.70 70,60 52.50 80.90 39 40 32,20 60 40 46.50 39.80 109,10 71,00 53.00 81.40 40 41 33 40 62.00 47 70 41.00 109,40 71,50 53.60 81.90 41 42 34,70 63.60 49 00 42.20 109,80 72.00 54.20 82.60 42 43 36. 10 65.30 50.40 43.50 110 30 72,50 54.80 83.30 43 44 37.50 67.10 51.90 44.80 110,80 73 10 55.60 84.00 44 45 39.10 69.00 53 40 46.20 111.30 73.80 56.40 84.90 45 46 40,70 70.90 55 10 47.80 112.00 74.60 57.30 85.80 46 47 42.50 72.90 56 80 49.4() 112,60 75.40 58.30 86.80 47 48 44.40 75.10 58.60 51.10 113.40 76 30 59.40 88.00 48 49 46.40 77.30 60 50 52,90 114.20 77.30 60.70 89.20 49 50 48.50 || 79.60 62.50 54.80 115.10 78.40 62.00 90.60 50 51 50.80 82.10 64.60 56.90 || 116.10 79.70 63.50 92.10 51 52 53 30 84,60 66 90 59,10 117.20 81.00 65.20 93.70 52 53 55 90 87.30 69 20 61.40 118.40 82.50 67.00 95.50 53 54 58 70 90.10 71.80 63.90 119,80 84, 10 68.90 97.50 54 55 61, 60 93,00 74.40 66,60 121,20 85.90 71.10 99.60 55 56 64.80 96.10 77.30 69.50 | 122,80 87.90 73.50 101.90 56 57 68.20 99 30 80 30 72,60 124.60 90.10 76.10 104.40 57 58 71.80 102.70 83 50 75.80 126.50 92.50 78.90 107.10 58 59 75.70 106 30 86.90 79.40 | 128.70 95.10 82.10 110.10 59 60 79.90 110.10 90.60 83.20 131.00 98.00 85.50 113.20 60 61 84.30 114.10 94 50 87.30 133.60 101.20 ! . . . . . . . . . . . . . 61 62 89.10 118 30 98 70 91.70 136,40 104.70 | . . . . . . g 62 63 94.20 122.70 103 20 96.40 139,60 108.50 | . . . . . . . . . . . . . 63 64 99 60 127 50 108 00 101.50 143,00 112.70 e e º e º e e e º e º 'º 64 65 105.50 132.50 113.20 107.10 146,80 117.30 . . . . . . . . . . . . . 65 Not E 1.-The above table is taken by permission from THE MUTUAL LIFE INSURANCE COM- PANY of New York. NotE 2.-Policies which do not participate in the dividends of a company are issued at a lower rate of premium, than given in the above table. NOTE 3.-The above rates of premium are for annual payments. In some cases the payments are made semi-annually or quarterly, when a small per cent is added to the annual payment before dividing by 2 or 4. 54o SouLE's PHILOSOPHIC PRACTICAL MATHEMATICS. * PROBLEMIS. 1. What is the annual premium for a man, aged 21 years, on a Life Policy for $5000 % Ans. $93,00. OPERATION. sisº, Explanation.—The premium on $1000 at the age of 21 is, as +- shown in the table, $18.60; hence, for $5000, it is 5 times $93.00 Ans. as much. 2. What is the annual premium on a 10-year Endowment Policy for $5000, the age being 21 years? Ans, $527.00. OPERATION. $105,40 5 Explanation.—Since the premium on $1000 for the con- -º-mºmºmºsº- ditions named is $105.40, for $5000 it is 5 times as much. $527.00 3. A man is 32 years old and takes out a 20-year Endowment Policy for $25000. What is his annual premium ? Ans, $1252.50. 4. A person, 28 years old, took out a 15-year Endowment Policy for $2500. What is the annual premium ? Ans. $169.50. 5. What is the annual premium on a Life Policy for $8500, payable in ten annual payments, the insured being 44 years of age? Ans. $570.35. 6. A man, 21 years old. took out a 10-year Endowment Policy, and at its maturity, the premiums paid amounted to $5270. What was the amount of the policy 3 Ans. $5000. OPERATION. Explanation.—Since the premium for 10 years was $5270, for one year it is the ºr part, $527. Then since the annual $5270 - 10 = $527 payment on a $1000 ten-year Endowment Policy is, per table, $105.40 for a person aged 21 years, there were as many 527 -- 105.40 = 5 thousands thousands insured as $537 is equal to 'siojºſo, which is 5 5 × $1000 = $5000 times. Hence there was $5000 insurance. 7. Mr. X, aged 22 years, took out a Life Policy payable in 15 years. After making eight annual payments amounting to $876.40, he died. What was the face of his policy 3 Ans. $3500. 8. A man, 21 years old, paid $93.00 premium annually, on a policy of $5000. What was the rate per cent? Ans. 1.86%. OPERATION INDICATED. 93 100% 5000 || 100 or, 5000 || 93 9. The annual premium on a $25000 Endowment Policy, payable in twenty years, for a person aged 32, is $1252.50. What is the rate per cent? Ans. 5.01%. 10. A man, 28 years old, paid an annual premium of 6.78 per cent on a 15- year Endowment Policy of $2500. What was the annual premium, and how much would he pay in all ? Ans. $169.50 annual premium. $2542.50 total premium. $2500 x 6.78% = $169.50. OPERATION INDICATED. }; × 15 = $2542.50. 11. A man, 25 years old, took out a Life Policy for $100000 and paid the regular annual premiums until his death, at the age of 69 years. How much did he pay in premiums? AnS. $90200. LIFE INSURANCE. 541 TABLE NO. 4. d 1082. AMERICAN EXPERIENCE TABLE OF MORTALITY. Wumber Expecation mber Number IExpectation Age. Nº. * º Age. *: Dying. . Life. 10 100000 749 48.72 53 66797 1091 18.79 11 99251 746 48,08 54 65706 1143 18.09 12 98505 743 47.45 55 64563 1199 17.40 13 97.762 740 46.80 56 63364 1260 16.72 14 97022 737 46.16 57 62104 1325 16,05 15 96.285 735 45.50 58 60779 1394 15.39 16 95550 732 44.85 59 59385 1468 14.74 17 94818 729 44.19 60 57917 1546 14.10 18 94089 727 43.53 61 56371 1628 13,47 19 93362 725 42.87 62 54743 1713 12,86 20 92637 723 42.20 63 53030 1800 12.26 21 91914 722 41.53 64 51230 1889 11.67 22 91.192 721 40.85 65 49341 1980 11.10 23 , 90471 720 40.17 66 47361 2070 10.54 24 89751 719 39.49 67 45291 2158 10,00 25 89.032 718 38.81 68 43.133 2243 9.47 26 88314 718 38.12 69 40890 2321 8.97 27 87596 718 37.43 70 38569 2391 8.48 28 86878 718 36.73 71 36178 2448 8.00 29 86160 719 36.03 72 33730 2487 7.55 30 85441 720 35.33 73 31243 2505 7.11 31 84721 721 34.63 74 28738 2501 6.68 32 84000 723 33.92 75 26237 2476 6.27 33 83277 726 33.21 76 23761 2431 5.88 34 82551 729 32.50 77 21330 2369 5,49 35 81822 732 31.78 78 18961 2291 5,11 36 81090 737 31.07 79 16670 2196 4.74 37 80353 742 30.35 80 14474 2091 4,39 38 796.11 749 29.62 81 12383 1964 4,05 39 78862 756 28.90 82 104.19 1816 3.71 40 78106 765 28.18 83 8603 1648 3.39 41 7734.1 774 27.45 84 6955 1470 3.08 42 76567 785 26.72 85 5485 1292 2.77 43 75782 797 26.00 86 4193 1114 2.47 44 74985 812 25.27 87 3079 933 2.18 45 74.173 828 24.54 88 2146 744 1.91 46 73345 848 23.81 89 1402 555 1.66 47 72497 870 23.08 90 847 385 1.42 48 71627 896 22.36 91 462 246 1.19 49 70731 927 21.63 92 216 137 .98 50 69804. 962 20.91 93 79 58 .80 51 68842 1001 20.20 94 21 18 ,64 52 67841 1044 19.49 95 3 3 .50 NoTE 1.-This table is adopted by the State of New York and several States, in estimating life endowments. NOTE: 2.—Massachusetts has adopted the TABLEs prepared by DR. DIGGLEsworth, for estimating life estates. NOTE 3.-The CARLISLE TABLES prepared by MILNE are generally used in England. 542 soul E's PHILOSOPHIC PRACTICAL MATHEMATICS. * THE PROBABILITY OF HUMAN LIFE. 1083. The probability of life is the likelihood which a person has, according to the table of mortality, of living a certain number of years. The probability of death, is the likelihood a person has of dying within a given period. As it is absolutely certain that a person will always be dead or alive, the probability of death is found by subtracting the probability of life from unity, and vice versa. PROBLEM. 1. What is the probability that a person, aged 30 years, will live to be 50 years old, according to the figures of the American Experience Table 3 Ans. .81698. OPERATION. According to the table, the number living, of the 100000, at the age of 30, is 85441; and the number living, at the age of 50, is 69804; consequently, 15637 have died. The probability of any member surviving for 20 years, will-be expressed by the fraction of which 69804 is the numerator, and 85441 is the denominator; and that the probability of dying will be expressed by the fraction in which 15637 is the numerator and 85441 the denominator. Thus ##### = .81698, the probability of living. And ##### = .18302, the probability of dying. It will be observed that these amounts stand to each other as complement and supplement. Hence, when we have one of the amounts we can find the other by subtracting it from the unit. Thus 1 — .81698 = .18302; and 1 — .18302 = .81698. NOTE.—As there are 85441 chances in all of the persons aged 30 attaining to 50 years, and as the probability is that only 69804 will live, the contingency that each individual will live, is .81698, and hence the contingency of his dying is .18302 2. A man 25 years old pays $140.50 annual premium on a 20 year policy of $5000. What would this annual payment be worth to the company if continued for 20 years, and the company put the payments at 4% compound interest ? Ans. $4183.82. Operation by the Annuity Table, page 925. $140.50 × 29.778079 = $4183,82. 3. Suppose in the above problem that the payment of $140.50 was continued for 20 years and was annually invested at 4% compound interest, what would be the value of the investments at the expectancy of the policy holder's life, based upon the American Experience Table, (38.81 years)? OPERATION. First step by the Annuity Table, page 925. $140.50 × 29,778079 = $4183.82 = value of the annual payments or annuity for 20 years. 38.81 years — 20 years = 18.81 years, which is the time the value of the Annuity $4183.82, was at compound interest at 4%. Second step by C. I. Table, page 611. $4183.82 × 2.02581652 = $8475.65 = compound amount for 18 years. Int, on $8475.65 at 4%, .81 years 274.61 $8750.26 Ans. For further information regarding Life Insurance, see “The Principles and Practice of Life Insurance,” or consult the officers of some Life Insurance Company. For Co-operative Insurance, consult some member of a Co-operative Association. nterest. -----------------N ƺ º Aº. * ur-er 1084. Interest is defined by many political economists to be the sum the borrower pays for the use of money. But this definition is opposed by the more acute and critical political economists, who define interest to be the share of the product obtained by the owner of capital. 1085. The Rate of Interest is the per cent paid for the capital or money borrowed, for a specified time. It was formerly believed that the rate depended upon the abundance or scar- city of capital in the money market at any given time. But this opinion has been proven to be erroneous. The rate is now shown, by our best economists, to be governed by the productiveness or profits of capital employed in the various lines of business. MILL says, after thoroughly arguing the case, “it is an error to imagine that the rate of interest bears any necessary relation to the quantity or value of money in circulation.” RICARDO says, that “the rate of interest is not regulated by the rate at which the bank will lend, but by the rate of profit which can be made by the employment of capital, and which is totally independent of the quantity and of the value of money.” - NOTE.-The value of money is its purchasing power. But independent of this general law based upon profits, the rate is sometimes affected by the greater hazard that is incurred by loaning to less responsible parties or on less certain security, and also by reason of the difficulties that sometimes attend the transactions of loan, because of the legal limit placed upon the rate, called Usury Laws. Again, the rate is influenced by the duration of the loan. Money loaned for a short period, commands a higher rate than money loaned for a long period. And call loans bear a lower rate than loans made for a definite period of time. HISTORY OF INTEREST. 1086. For upward of thirty centuries, the mind of man has been engrossed with this subject, and yet no unanimity of opinion has been reached, as regards the individual rights of men who loan and borrow money, to fix the rate of compensa- tion to be paid to the owner or lender thereof. In the early ages, the charging and receiving of compensation for the use of money was called usury, and was denounced (543) 544 soul E's PHILOSOPHIC PRACTICAL MATHEMATICS. º: as unjust. This mistaken and prejudicial view undoubtedly arose from a misunder- standing or misinterpretation of some of the enactments of the Mosaic laws, which read as follows: “Thou shalt not lend upon usury to thy brother; usury of money, usury of victuals, usury of anything that is lent upon usury; unto a stranger thou mayest lend upon usury; but unto thy brother thou shalt not lend upon usury.” (Deuteronomy, XXIII, 19, 20.) And, “If thou lend money to any of my people that is poor by thee, thou shalt not be to him a usurer, neither shalt thou lay upon him usury.” (Exodus, XXII, 25.) And again, in speaking of a poor brother, “Thou shalt not give him thy money upon usury, nor lend him thy victuals for increase.” (Leviticus, XXV, 37.) These laws are specific and direct in their application, and do not forbid the charging of usury except to poor brothers. For ages, the followers of Moses and the disciples of his teachings were the money lenders of Europe for usury; and by reason of this, and the fact that they were as a class shrewd, trafficking, and a money dealing people, is to be attributed the unjust prejudice, that in a measure to this day exists against them and others who follow the profession of money lending. ARISTOTLE, the Grecian philosopher, gravely contended that, as money could not beget money it was barren, and usury should not be charged for its use. In this regard, as in many others, the old philosopher did not philosophize aright. He did not see that with money the borrower could add to his flocks, his fields and his merchandise, and thus profit by the increase of all. The Greeks were not, however, governed by the opinion of Aristotle, for in his age, they paid an average rate of about 30 per cent on the capital employed in the shipping trade, and about 12 per cent for capital employed in business on land. The Roman Law allowed interest or usury, and during the days of the repub- lic, the rate of interest or usury was very high. This resulted mainly from the fact that the debtors, the plebeians, repeatedly threatened and sometimes made efforts to deprive their creditors, generally the patricians, not only of the interest on their capital, but also of the capital or principal itself. But from the time of the 4th century, the writings of the Romish Church Fathers and the Canon Law prohibit interest. This prohibition was based upon the Mosaic law as above quoted and upon the Sophistry of Aristotle. The council of Vienna, Austria, in 1311, declared any defense of interest to be heresy. In England, from the earliest period of history to the reign of Henry VIII, interest was forbidden to all persons except to Jews and foreigners, and they were often plundered to enrich the crown, on the miserable pretext of punishment, for what the government called their “hellish extortions.” But the opposition of the people to this ruinous interdiction against interest, by the government, resulted in a statute passed in 1546, allowing interest to the extent of 10 per cent per annum. Yet, in the reign of Edward VI, in 1552, the government again prohibited the taking or charging of any interest, and passed an INTEREST. 545 act declaring that, taking interest was “a vice most odious and detestable” and “ contrary to the word of God.” On the passage of this act, interest rose to 14 per cent and continued at this rate until 1571, when an act was passed repealing the act of Edward VI, and re-en- acting that of Henry VIII, allowing 10 per cent interest. The word interest was now substituted for the word usury, and the meaning of usury was changed to mean the rate per cent charged in excess of the legal rate. By the middle of the 17th century, nearly all the governments of Europe followed England in abolishing the Usury Laws, and establishing fixed rates. But as the fixed rates were evaded just as the prohibition had been evaded before, and as the legally fixed rates resulted in making the rate of interest higher than it would have been without such laws, all European States have since abolished Usury Laws. In 1839, England abolished Usury Laws on commercial paper running less than twelve months; in 1850, on all capital except loans on real estate; and in 1854, they were abolished altogether. The United States stands almost alone in retaining Usury Laws. The legal rate of interest in the National Banking Law is 7 per cent, in States where no rate is fixed. In States where the State Laws fix the rate per cent, the National Banks are to be governed accordingly. See Table of Interest Laws of the different States and territories, page 548, for the various rates per cent allowed, penalties for usury, and days of grace. EVIL EFFECTS OF USURY LAWS. 1087. Usury Laws, like other laws fixing prices, are not only ineffectual, but they prevent all persons, whose credit is not first class, from obtaining loans from capitalists of the highest character, and force borrowers to secure loans from those who are less scrupulous and who are also compelled to charge above the current rate a premium sufficient to compensate the lender for the extra risk and odium incurred in violating the USury Law. Usury Laws undertake to prevent what is not an evil, and hence what should not be interfered with ; consequently, they are productive of injury instead of bene- fit. Who would pretend that it is just for the legislature to compel underwriters to insure extra hazardous or specially hazardous properties, on the same terms or rate per cent as they write simply hazardous risks. And yet such a law would be no more absurd and unjust than the law of usury which requires that the rate per cent of interest shall be the same on capital loaned to those whose Security or character are widely different. Again, it is impossible to make the legal rate a fair rate; for the market rate is continually changing, and independent of the hazard of the risk, a rate that would have been fair at one time would be too low at another time, and perhaps extortionate at another time. A loan for a short period commands a higher rate 546 soul.E's PHILOSOPHIC PRACTICAL MATHEMATICS. Yºr than for a long period; and a rate that is fair for a short period would be unfair for a long period. The only way a fair rate can be determined is by competition in the money market, when all the elements of supply and demand, of risk, of time, and of profit, are duly considered by the borrower and the lender. BORROWERS AND LENDERS. 1088. When the Usury Laws were passed, the poorer classes were the bor- rowers and the rich were the lenders, and hence the Usury Laws, were enacted mainly with a view to protect the poor, against high rates of interest. But at the present time, the borrowers are largely the rich men, and the rich corporations, and a large part of the lenders, are the poorer or medium classes. Hence, considering the reason for the passage of the Usury Laws, it follows logic- ally that they should be now abolished. One evidence of this change of borrowers and lenders is the fact that in 1894, three million depositors, mostly poor people, had nearly 1,300,000,000 dollars on deposit in the savings banks. Another evidence is the fact that hundreds of mill- ions of dollars of bonds of various kinds of corporations, of States, and of nations, are held by the poorer classes. Considering the Usury Laws in all their bearings, the conclusions are as follows: 1. They are unwise and unjust. 2. They are practically inoperative. 3. They are evidence of financial inexperience on the part of the law makers. 4. They are violative of the rights of borrowers and lenders, and of the ethical principles of business. 5. They should be repealed. - That a law regulating interest in cases of judgments, successions, and trans- actions in which no contracts of interest are made, is necessary, there is no question of doubt. But further than this, the question of interest should be left to the judg- ment of the contracting parties. DEFINITIONS. 1089. Interest is the sum paid for the productiveness of capital. INTEREST is calculated on a scale of a certain number of units on every $100 for 1 year, and hence it combines per cent and per annum, from the Latin per centum, on a hundred, and per annum, for a year. 1090. The Principal is the sum of money or capital on which interest is charged or paid. 1091. The Rate of Interest is the per cent paid on the principal, for its use, for a specified time. 1092. Time is the period for which the principal is loaned or bears interest. 2} INTEREST. 547 1093. The Amount is the sum of the principal and the interest. 1094. The Proceeds is the difference between the principal and the interest. The subject of interest is classified into several divisions, viz: Simple Interest, Annual, Semi-Annual, and Quarterly Interest, and Compound Interest, which we will present in the order here given. 1095. Simple Interest is the interest on the principal unincreased by inter- est, however long overdue. Annual, Semi-Annual and Quarterly Interest, is Simple Interest, and the interest On the Simple Interest from the time it becomes due until paid. . e NOTE.-Annual, Semi-Annual and Quarterly Interest is allowed by some States on notes and contracts where the interest is Payable Annually, Semi-Annually or Quarterly. Compound Interest is interest on both the principal and the interest, i. e. it is interest where the principal is increased at the expiration of each period of interest payment by the interest on the principal for such period, whether for 12, 6, 3, or 1 month; or for one day. NOTE.—Compound Interest is allowed in some States, under certain conditions. • 1096. Legal Interest is the rate per cent fixed by the law of each state, to apply when no agreement is made. In Louisiana, it is 5 per cent. 1097. Conventional Interest is the rate per cent agreed upon by the parties concerned. The law of many of the States places a limit to this interest. In Louisiana, the limit is 8 per cent. 1098. Usury is a higher rate per cent than the law allows. The law of dif. ferent States prescribes different penalties for usury. In Louisiana, the penalty is the forfeiture of all interest. The Principal, the Interest, the Rate, the Time, and the Amount or Proceeds, constitute the five quantities involved in interest questions; and when any three of these are given, the others may be found. Hence there are five classes of interest questions. 548 SOULE's PHILOSOPHIC PRACTICAL MATHEMATICS. 1099. TABLE OF INTEREST LAWS. The following table shows the Interest Laws of the different States and Territories compiled from the State and Territorial Statutes of 1891. Rate allowed States and Territories. *...* º §* Penalties for Usury Gº; Iſºſ) writing. | PEIR CFNT: PER CENT | ALABAMA, . . . . . . . . . . . . . . . . 8 8 Forfeiture of all the Interest. . . . . . . . . . Grace ARIZONA, . . . . . . . . . . . . . . . 7 Any rate |None . . . . . . . . . . . . . . . . . . . . . . . . . . . . . . . . . . Sº, ARKANSAS, . . . . . . . . . . . . . . . 6 10 Forfeiture of principal and interest. ... No statute. *CALIFORNIA, . . . . . . . . . . . . 7 * | Any rate |None . . . . . . . . . . . . . . . . . . . . . . . . . . . . . . . . . No grace. COLORADO, . . . . . . . . . . . . . . e 8 Any rate |None . . . . . . . . . . . . . . . . . . . . . . . . . . . . . . . . . Grace. CONNECTICUT, . . . . . . . . . . . . 6 6 None . . . . . . . . . . . . . . . . . . . . . . . . . . . . . . . . . Grace. DAKOTA, NORTH... . . . . . . . 7 12 Forfeiture of excess. . . . . . . . . . . . . . . . . . . Grace. & 4 SOUTH . . . . . . . . . . 7 12 Forfeiture of all the interest. . . . . . . . . . Grace. DELAWARE, . . . . . . . . . . . . . . 6 6 Forfeiture of principal and interest. ...|Grace, DIST. OF COLUMBIA, . . . . . . 6 10 Forfeiture of all the interest. . . . . . . . . Grace. FLORIDA, . . . . . . . . . . . . . . . . . 8 Any rate |None. . . . . . 6 tº e º is e a e º a tº e º 'º e º & e s tº e º e ..|No statute. GEORGIA, . . . . . . . . . . . . . . . . 7 8 Forfeiture of excess of interest. . . . . . . . Grace. flipAHO, . . . . . . . . . . . . . . . . . . 10 18 Forfeiture of thrice the excess of int.. No grace. ILLINOIS,. . . . . . . . . . . . . . . . . 6 8 Forfeiture of all the interest. . . . . . . . . . Grace. INDIANA, . . . . . . . . . . . . . . . . e 6 8 Forfeiture of excess of interest........ Grace. INDIAN TERRITORY, ...... 6 6 Forfeiture of principal and interest....|Grace. IOWA, . . . . . . . . . . . . . . . . . . . e 6 8 Forfeiture of 10 per cent on amount....|Grace. KANSAS, . . . . . . . . . . . . . . . . . . 6 10 Forfeiture of excess of interest. . . . . . . . Grace. KENTUCKY, . . . . . . . . . . . . . . 6 6 Forfeiture of excess. . . . . . . . . . . . . . . . . . . [Grace. fLOUISLANA, . . . . . . . . . . . . . . 5 8 Forfeiture of all the interest. . . . . . . . . . Grace. MAINE, . . . . . . . . . . . . . . . . . . . 6 Any rate |None . . . . . . . . . . . . . . . e - a e e e s e º e = * * * * * * * * Grace. MARYLAND, . . . . . . . . . . . . . . 6 Forfeiture of excess of interest. . . . . . . . Grace. MASSACHUSETTS, ... . . . . . . . 6 Any rate |None . . . . . . . . . . . ... . . . . . . . . . . . . . . . . . . . Grace. MICHIGAN, . . . . . . . . . . . . . . . 6 10 Forfeiture of excess of interest. . . . . . . . Grace. MINNESOTA, . . . . . . . . . . . . . . 7 10 Forfeiture of excess over 10 per cent...|Grace. MISSISSIPPI, . . . . . . . . . . . . 6 10 Forfeiture of all the interest. . . . . . . . . . Grace. MISSOURI, . . . . . . . . . . . . . º 6 10 Forfeiture of all the interest. . . . . . . . . . Grace. MONTANA, . . . . . . . . . . . . . . . . 10 Any rate |None . . . . . . . . . . . . . . . . . . . . . . . . . . . . . . ... No grace. NEBRASKA, . . . . . . . . . . . . . . . 7 10 Forfeiture of interest and cost.........|Grace, NEVADA, . . . . . . . . . . . . © e º O e 7 Any rate |None . . . . . . . . . . . . . . . . . . . . . . . . . . . . . . . . . Grace. NEW HAMPSHIRE, . . . . . . . . 6 6 Forfeiture of thrice the excess. . . . . . . . . Grace. NEW JERSEY, . . . . . . . . . . . . 6 6 Forfeiture of all the interest. . . . . . . . . . Grace. NEW MEXICO, . . . . . . . . . . º 6 12 $100 fine. . . . . . . . . . . . . . . . . . . . . . . . . . . ... Grace, |NEW YORK, . . . . . . . . . . . . . 6 6 Forfeiture of principal and int. and fine not exceeding $1000, or imprison-X |No grace. ment not exceeding 6 months or both NORTH CAROLINA, . . . . . . . . 6 8 Forfeiture of all the interest, etc. . . . . . Grace. OHIO, . . . . . . . . . . . . . . . . . . 6 8 Forfeiture of excess above 6 per cent. . Grace. OKLAHOMA TERRITORY,... 7 10 Forfeiture of all the interest. . . . . . . . [Grace. OREGON, . . . . . . & © tº º e º 'º e º º º 8 10 Forfeiture of principal and interest....|Grace. §PENNSYLVANIA, . . . . . . . . . 6 6 Forfeiture of excess of interest. . . . . ... Grace. RHODE ISLAND, . . . . . . . . . . . 6 Any rate |None . . . . . . . . . . a e s m = e º e º e º e º e © e º e º e - © tº e Grace. SOUTH CAROLINA, . . . . . . . . 7 8 Forfeiture of all the interest. . . . . . . . ... Grace. TENNESSEE, . . . . . . . . . . . . . . 6 6 Forfeiture of excess of interest. . . . . . . . Grace. TTEXAS, . . . . . . . . . . . . . . . . . 8 12 Forfeiture of all the interest. . . . . tº e º 'º º º Grace. UTAH, . . . . . . . . . . . . . . . . . . . tº 8 Any rate |None . . . . . . . . . . . . . . . . . . . tº 6 & e º ſº tº * * . . . . . Grace. VERMONT, ..... tº e s tº e a t e e º a 6 6 Forfeiture of excess of interest. . . . . ...|Grace. VIRGINIA, . . . . . . . . . . . . . . . . 6 6 Forfeiture of excess over 6 per cent....|Grace. WASHINGTON, . . . . . . . . . . . . 10 Any rate |None . . . . . . . . . . . . . . . . . . . . . . . . . . . . . . . . [Grace. WEST VIRGINIA, . . . . . . . . . . 6 6 Forfeiture of excess of interest. . . . . . . . Grace. WISCONSIN, . . . . . . . . . . . . . . 7 10 Forfeiture of all the interest. . . . . . . . ... Grace. WYOMING, ................ 12 Any rate |None . . . . . . . . . . . . . . . . . . . . e e e s e s a e º 'º º e º 'º Grace. *In California, compound interest is allowed by written contract. Written contract. e collected provided it be added to the principal and by a new contract made iIn Idaho, compound interestls allowed b †In Louisiana interest upon interest may a new debt. On Bottomry and Respondentia Loans, a hi § lected provided it be embodied in the instrument as discount. |In New York, on demand loans of $5000 and upward, which are secured by collaterals, any rate of Interest may be agreed upon in Writing, In ennsylvania, Commission Merchants may contract for 7 per cent. In Texas, open accounts draw 8 per cent interest from the 1st day of January after the same are made. gher rate than 8 may be collected. Also a higher rate may be col- * PROMISSORY NOTES AND NEGOTIABLE PAPER. 549 PROMISSORY NOTES AND NEGOTIABLE PAPER * Ah, a ~ wºr-w 1100. A Negotiable Promissory Note is an unconditional promise in writing, to pay to the order of a certain party, or to bearer, at a certain time, a specified sum of money. The following is the usual form of a negotiable promissory note: NEW ORLEANs, March 21, 1895, Thirty days after date, for value received, I promise to pay to the order of P. W. Sherwood, Five Hundred Dollars. H. JONES. Due April 20/23, 1895. 1101. Parties to a Note. The original parties to a note are those whose Inames appear in the paper, when it is made. Subsequent parties are those who become interested in the paper after it is made. In the above note, H. Jones is the promissor, maker and payor. P. W. Sher- wood is the promissee, payee, holder, and first indorser. The holder of a note is the party who owns it. The first indorser of a note, is he to whom it is made payable, whether his name appears first on the back of the note or not. Parties whose names are written above that of the first indorser are sureties, and like the maker, are responsible without protest. If, in the above note, the words “the order of,” before the name of the payee, had been omitted, it would have been unnegotiable. No particular form of words is necessary to form a valid promissory note. The omission of the words “value received" does not render the note invalid, but in the hands of the original holder it would be necessary, if required by the maker, to prove that value had been received, A promissory note bears no interest until maturity unless specified, and in either case unless the per cent is specified, it will bear only the legal rate. 1102. Negotiable Paper is that which may be transferred from one owner to another by assignment or indorsement. There are several kinds of negotiable paper, namely: Promissory Notes, Bills of Exchange, Due Bills, Bank Notes, Checks on Banks or Bankers, Coupon Bonds, Certificates of Deposit, Let- ters of Credit, and Bills of Lading. -- e e - Negotiable paper that does not specify the time of payment, is payable immediately, and if the place of payment is not specified, it is payable wherever 1t is held at maturity. When the plače of payment is specified, it must then be presented at such place the day that it legally matures. The customs and laws of the place in which negotiable instruments are made payable, always apply. 1103. The Face of a note, draft, etc., is the sum specified or named therein and promised to be paid. 1104. The Maturity of a note is the day that the note becomes due. 1105. Days of Grace are days allowed for the payment of a note, draft, etc., after the expiration of the time specified in the instrument. By custom in most of 55o SOULE's PHILOSOPHIC PRACTICAL MATHEMATICS. * . the States 3 days of grace are allowed on all notes, drafts, etc., that are not drawn without grace, at sight or on demand. NOTE.-In some States sight drafts are accepted, and bear three days of grace. 1106. Dishonoring a note is the failure to pay it when due. 1107. The Protesting of a note is a notice to the indorsers that it is due, that payment has been demanded and refused, and that the holder looks to them for the payment. The object of protesting a note is to Secure the responsibility of the indorsers. Protests are generally made by a notary, 1108. Discount Day is the day that a note, draft, etc., is discounted. Many bankers and business men when discounting notes, etc., charge interest for this day. 1109. Discounting Notes, etc., consists, according to the custom of bankers, in calculating the interest on their face for the unexpired time including 3 days of grace, and in some communities discount day, at some specified rate per cent, and deducting the same from the face. 1110. The Proceeds or Cash Walue of Notes, etc., is what remains after the interest or discount is deducted. INDORSEMENTS ON PROMISSORY NOTES AND BILLS, 1111. An Indorsement is anything written on the back of a note or bill, which pertains to the payment thereof. 1112. An Indorser is the party who writes his name on the back of a note or bill. The first indorser on a note, is he to whom the note is made payable. The first indorser on an accepted bill, is the Drawer. - A note or bill is not negotiable unless it is indorsed by the payee, except in the case of a note which is made payable to the bearer. Indorsements vary in form as follows: 1. Blank. 2. In full. 3. Qualified. 4. Restrictive. 5. Conditional. 6. Protest and Notice of Protest Waived.- The following elucidates the above indorsements, J. B. Anderson being the indorser: 1. (Indorsement in Blank). (Second form of Qualified 5. (Conditional Indorse- J. B. Anderson, Indorsement). ment). Pay to the order of A. B. 2. (Indorsement in Full). J. B. Anderson, Ray to A. B., when he shall Pay to the order of A. B. Agent. have completed the house J. B. Anderson, he is now building for me. 4. (Restrictive Indorsement). J. B. Anderson. 3. (Qualified Indorsement). Pay to A. B. only. Pay to the order of A. B., J. B. Anderson. 6. (Protest Waived). without recourse to me. or thus: J. B. Anderson. Pay to the Fourth National Protest and Notice of Pro- or thus: Bank of St. Louis, for my test waived. Without recourse. account. - J. B. Anderson. J. B. Anderson. J. B. Anderson. NoTE.—When indorsing notes, bills, or checks, write your name on the back in the same way as it is written on the face. If your name is wrongly spelled on the face, then you should also mis-spell it in the indorsement. Or you may both mis-spell it as it is on the face of the paper, and then write it correctly. #: MISCELLANEOUS COMMERCIAL INSTRUMENTS OF WEITING. 55I DISCOUNTING NOTES AND ACCEPTANCES. 1113. It is the custom of the banks and the business men of New Orleans, When discounting notes, to count the actual unexpired days, including one DISCOUNT DAY; i. e., the day the note is discounted. REMARKS. In 1904, the Legislature of Louisiana adopted the New York Commercial Instrument Law, thus making nearly half the States of the Union which have adopted this Law, up to December 1904. This Law does not allow days of grace on Commercial paper. Hence when maturing notes and bills, and when discounting or computing interest on them in this book, and in states which have adopted this Law, consider the notes to be drawn for 3 days more than the time specified therein, except such notes as may be drawn without grace. NOTE. —For a full discussion and elucidation of the foregoing indorsements of accommoda- tion paper, acceptance or payment for honor, treatment of accommodation paper, renewals or extension of notes, and an extended presentation of commercial instruments of writing, see Soulé's New Science and Practice of Accounts, pages 25 to 39 and 199 to 201. MISCELLANEOUS COMMERCIAL INSTRUMENTS OF WRITING, -º-eº. Pe-º- FIECEIPTS. 1114. 1. Receipt in full where payments have been previously made. NEw ORLEANs, November 6, 1895. Received from H. J. Calvert, Three Thousand Dollars, Two Thousand having been previously paid, making in all, Five Thousand Dollars, R. W. ABBOTT. 2. Receipt for Rent. NEW ORLEANs, December 4, 1895. Received from W. A. Beer, Fifty Dollars, for rent of house No. 457 Baronne Street, for November, 1895. E. R. GURLEY. 3. Receipt through a Third Party. NEW ORLEANs, January 3, 1895. Received from R. B. Montgomery, through F. A. Golden, Twenty-three Hundred, Forty-seven and 3% Dollars, in full of all demands to date. GEO. B. MUSE. NoTE.—The words in italics have no money value whatever. A receipt is evidence for the amount named therein, and no words can make it worth any more. Hence, the words in italics are useless, save as general information. 552 SOULE's PHILOSOPHIC PRACTICAL MATHEMATICs. 4. Receipt to apply as an Indorsement on a Note. NEW ORLEANs, November 16, 1895. Received from Geo. B. Brackett, Three Hundred Dollars, the same to apply on his note of One Thousand Dollars. T. C. HENRY. NOTE.-Money paid on account of notes should be, when practicable, indorsed on the back of the note. The above payment thus indorsed would read as follows: Received on the within note Three Hundred Dollars. NEW ORLEANs, November 16, 1895. 4. T. C. HENRY. N O T E S . —-º- 5. An Individual Note. NEW ORLEANs, July 16, 1895. Sixty days after date, for value received, I promise to pay to the order of H. A. Spencer, One Thousand Six Hundred Dollars. Due September 14/17, 1895. R. P. HENRY. 6. A Joint Note. NEW ORLEANs, July 16, 1895. Two months after date, for value received, we promise to pay to the order of S. S. Packard, Sixteen Thousand Dollars. Due September 16/19, 1895. A. G. MAYLIE, C. S. CLARKE. N. B.-In this note, the joint makers are each liable for one-half of the sum named in the Inote. 7. A Joint and Separate Note. JNEw ORLEANs, February 4, 1895. Six months after date, for value received, we or either of us promise to pay to the order of H. B. Stevens & Co., Nine Thousand Two Hundred Dol- lars, payable at the Canal Bank in New Orleans. Due August 4f7, 1895. O. H. KARSTENDIEK, A. V. DEGRUY, H. C., YOUNG. N. B.-In this note, in place of the words, we or either of us, the words, we jointly and severally, may be used. {E NOTES. 553 8. Collateral Note. $5000. NEW ORLEANs, May 14, 1895. Ninety days after date, I promise to pay to the order of J. G. Anderson, Five Thousand Dollars, for value received, with interest at the rate of six per cent per annum from date hereof until paid. This note is secured by pledge of the securities mentioned on the reverse hereof, and in Qase of its non-payment at maturity, or should the drawer hereof, when called on, refuse or fail to keep the margin hereon good, the holder is hereby authorized to sell the said securities at public or private i. Without recourse to legal proceedings, and to make any transfers that may be required, applying proceeds of sale towards the payment of within note. Due Aug. 12f15, 1895. L. W. WELLS. NOTE.-On the back of this note must be itemized the securities pledged for its protection. 9. A Note with a certain Party as First Indorser. If Henry Levy wishes to give his note to Louis Winn, and to give J. D. Purcell, as first indorser, the note would be drawn as follows: $1621. NEW ORLEANs, February 4, 1895. Thirty days after date, for value received, I promise to pay to the order of J. D. Purcell, One Thousand Six Hundred Twenty-one Dollars. Due March 6/9, 1895. HENRY LEVY. This note must now be indorsed by J. D. PURCELL, and delivered to Louis WINN. 10. A Note with Surety. NEW ORLEANs, March 21, 1895. Four months after date, for value received, I promise to pay to the order of N. E. Ball, Eight Thousand Dollars. B. TURNER, JR. I hereby, for value received, agree to become security on the above note. C. W. BUTLER, NOTE.-The contract of surety may be indorsed on the back of the note. 11. A Note with a Guarantor. NEW ORLEANs, March 21, 1895. Sixty days after date, for value received, I promise to pay to the order of L. Powell, Twenty-four Hundred Five and ſº Dollars. J. W. LUCE, JR. For value received, I guarantee the payment of the above note at maturity. F. P. BOLTON. NotE.—The contract of guarantee may be indorsed on the back of the note with the word “within” in place of the word “above.” 554 SOULE's PHILOSOPHIC PRACTICAL MATHEMATIcs. 13. 4 Note of a Married Woman in Louisiana. NEW ORLEANs, March 21, 1895. Ninety days after date, for value received, I promise to pay to the order of J. H. McNeely, Forty-six Hundred Seventy-three and ºf Dollars. - MRS, KATIE HOPKINS, I hereby authorize my wife to sign the above note. A. B. HOPKINS. 13. A Note signed with a cross by a party who cannot write. NEw Orleans, May 23, 1895. $ One year after date, for value received, I promise to pay to the order of S. S. Levy, Twenty-five Hundred Dollars. his HENRY X SCHNEIDER, mark Witnesses: E. P. DELANNE. H. C. AYCOCK. BILLS OF EXCHANGE, DRAFTS AND CHECKS, —- sº.--º-º- -sºr ºv-wº- 1115. A draft, check or bill of exchange is a written order for the payment of money unconditionally. The bill may be printed, except the signature of the drawer—that must be written. t 14. A Time Bill or Draft, Time reckoned from Acceptance. $56231.3% NEW ORLEANs, January 1, 1895. Thirty days after sight, pay to the order of W. Wintz, Five Thousand |Six Hundred Twenty-three and Yºo Dollars. To A. Greenwood, : W. C. KEEVER. New Orleans. PARTIES TO BILLS AND DRAFTS. There are three original parties to a bill of exchange, viz: the drawer, drawee and payee. The subsequent parties to a bill are those to whom it may be successively transferred. In the above bill, W. C. Keever is the drawer, A. Greenwood is the drawee, and when he accepts the bill, he will become the acceptor, maker, and payor, and the drawer then becomes first indorser. W. Wintz is the º: BILLS OF EXCHANGE, DRAFTS AND CHECKS. 555 payee. Sight bills are paid at sight in Louisiana; but in some States, they are accepted and bear three days of grace. Drafts and bills should be presented for acceptance Without delay, and if acceptance is refused, they should be protested for non-acceptance in order to hold the drawer. Accepted bills bear grace, are protested for non-payment the same as notes, and are subject to all the laws governing promissory negotiable notes. 15. A Sight Bill. $1500, NEW ORLEANs, February 3, 1895. At sight, pay to the order of C. B. Mödinger, Fifteen Hundred Dollars, and charge the same to account of To W. H. Reynolds, A. L. SOULE. Alexandria, La. 16. Bank Check. NEW ORLEANs, May 1, 1895. SOULE COLLEGE BANK. Pay to the order of H. C. Brown, Two Thousand Seventeen and 54% Dol- lars. $2017 fºr. J. J. BELL & CO. 17. Order, Payable in Merchandise. NEW ORLEANs, April 11, 1895. MR. J. VEGA. Pay to Mr. J. N. Harrison or order, Forty-four and #6 Dollars in merchandise, and charge the same to my account. C. W. BUTLER, 18. Certificate of Deposit. No. 400. SouLÉ CollFGE BANK, NEw ORLEANs, July 5, 1895. N. L. Barfield has deposited in this bank One Thousand Dollars, payable to his order on return of this certificate properly indorsed. $1000. M. GREEN, Cashier. 19. Due Bill. NEW ORLEANs, April 1, 1895. Due to Wm. Hardie or order, on demand, One Thousand Seven Hundred Dollars, with interest from date at 8 per cent. GEO. C. FAHEY. 556 soul E's PHILOSOPHIC PRACTICAL MATHEMATICs. 20. Due Bill Payable in Merchandise. NEw ORLEANs, March 16, 1895. Due R. C. Spencer or order, Seventy-five Dollars, payable in merchan- dise from my store, May 1, 1895. C. W. GLYNN. 21. A Set of Domestic Bills. (1). SouLí, CoLLEGE BANK, NEW ORLEANs, January 3, 1895. At sight of this First of Exchange, (second unpaid) pay to the order of G. Loomis, Two Thousand Dollars, value received. To the City National Bank, EDWARD E. SOULE. of New York. Cashier. 1116. (2). SOULF COLLEGE BANK, NEW ORLEANs, January 3, 1895. At sight of this Second of Exchange, (first unpaid) pay to the order of G. Loomis, Two Thousand Dollars, value received. To the City National Bank, EDWARD E. SOULE. of New York. * Cashier. -º- SPECIAL LAWS AND BUSINESS CUSTOMS TO BE OBSERVED IN INTEREST AND DISCOUNT CALCULATIONS. A__* —a w-uy-w enactmentS. The following are some of these laws and customs governing the most of the mercantile communities of America: 2. To consider the year as consisting of 12 months of 30 days each, or 360 days, for a divisor in all interest or discount calculations. NOTE.—The American, English, and French governments, and the State of New York con- sider the year as consisting of 365 days, hence in calculating interest on United States, English or French bonds, or on notes maturing and payable in New York, the divisor would be 365 instead of 360. See Interest Divisor, pages 560 and 561. 1. In performing the operations of interest calculations, we must observe certain special laws, and also certain business customs which, by reason of being in general use or of long standing, have become law without any legislative Yºr SPECIAL LAWS AND BUSINESS CUSTOMS. 557 3. To allow 3 days of grace on all commercial paper, not payable on demand, at sight, or without grace, except in States in which days of grace are not allowed. NOTE.-Also to allow three days of grace on sight drafts or bills of exchange in States in which sight bills are accepted. 4. In maturing notes, drafts, etc., that are drawn in years or months, it is the custom to count from the day of the month that the instrument is dated to the same day of the month in which it matures. Thus a note dated July 15, 1895, and made payable 3 months after date, matures October 15/18, 1895. 5. In maturing notes, drafts, etc., that are drawn in days, it is the custom to mature them in days. Thus, if a note was dated July 15, 1895, the same as above, but made payable in 90 days after date, it would mature October 13/16, 1895. 6. Notes, acceptances, etc., maturing on the 29th, 30th or 31st days of months having less than 29, 30 or 31 days, are considered due on the last day of the month, and allowing for grace are payable on the 3d of the following month. Thus, a note dated December 28, 29, 30 or 31, 1895, and payable 2 months after date, matures February 28th and March 3d, 1896. 7. Notes, drafts, etc., maturing on Sunday or a legal holiday, are payable in Louisiana, the first business day following. In most of the States, commercial paper maturing on Sunday or a legal holiday, is payable on the preceding business day. 8. When discounting notes or drafts drawn in years or months, it is the custom to count the actual days of unexpired time, including 3 days grace and also discount day, in cities or places where interest is computed for the day the note is discounted. Thus, a note dated July 15, 1895, and made payable 3 months after date, would mature October 15/18, 1895, and if discounted the day it was drawn, the discount time would be 96 days, including grace and discount day. If this same note had been made payable 90 days after date, it would have matured October 13/16, 1895, and if discounted the day it was drawn, the discount time would have been 94 days, allowing grace and discount day. And in like manner a note made payable in one year would be discounted for 369 days, if discounted the day it was drawn, including grace and discount day. In discount computations, the time that has elapsed is deducted from the actual time the note has to run. NOTE 1.-In cities or places where no interest is charged for the discount day, but where 3 days of grace are allowed, the discount time on the above notes would be 1 day less on each, making them respectively 95, 93 and 368 days. NoTE 2.—In States where days of grace are not allowed, and the discount day is not included, the discount time would be 4 days less on each of the above notes, making respectively 92, 90 and 365 days. TO COLLECT INTEREST-BEARING NOTES. 1117. In collecting a note that is drawn in months or years and which bears interest, it is customary to charge interest for the months at 30, or years at 360 days each, instead of the actual days. 558 SouLE's PHILOSOPHIC PRACTICAL MATHEMATICs. X. If a note is dated March 10, 1895, and made payable 1 year after date with interest, it would mature March 10/13, 1896, and if held by the payee till maturity, he would collect the face of the note and the interest for one year or 360 or 363 days. In case the note was not paid until the expiration of the 3 days of grace, then the holder would be entitled, in equity at least, to interest for 3 days of grace, making 363 days. REMARKS.—The question of charging interest for the 3 days of grace, as here stated, when the payor avails himself of the 3 days of grace, is not as far as we can learn, covered by any statute or legal decision; and our financial men are not agreed upon this point. Some argue that in States where the law allows the maker of notes 3 days of grace, he should not be charged interest for those days. Others argue that while the law allows 3 days of grace in which to pay the note or bill, it does not stop the interest on those days any more than it does on any of the other days the instrument has to run. The equity of the question is clearly in favor of charging interest for the days of grace, and we shall so present the matter in this work. It is an ethical principle in business that value must be given for value received. That no person shall use the property of another without rendering compensation therefor. Hence, if the payor of a note retains the use of the payee's money for the 3 days of grace, he, the payor, should pay for such use. And moreover, the 3 days of grace in States allowing grace, is as much a part of the specified time a note has to run as the time written in the body of the note. ELUCIDATION OF THIS PRINCIPLE. 1118. A note is given March 26, 1895, for $5000, payable one year after date with interest at 8 per cent. What is the interest due in States allowing grace, if collected March 26 or 27 or 28 or 29, 1896? Ans. See statements following. Statement to find inter- | Statement to find inter- I Statement to find inter- | Statement to find inter- est due March 26. est due March 27. est due March 28. est due March 29. $50.00 $50.00 $50.00 $50.00 360 8 8 360 | 8 8 | 360 360 361 362 360 || 363 $400.00 Ans. $401.11; Ans. $402.22; Ans. $403.33% Ans. TO DISCOUNT NOTES BEARING INTEREST. 1119. In discounting a note drawn in months or years and which bears inter- est, it is the custom to add to the face of the note the interest for the months at 30, or years at 360 days each, plus three days of grace, which gives the maturity value of the note, and then to discount the sum thus produced, also for the actual number of days including 3 days grace and also 1 discount day, in places where the discount day is included. See page 580, for the operations of discounting notes that bear interest. jºr SPECIAL LAWS AND BUSINESS CUSTOMS. 559 POINTS IN DRAWING NOTES. 1120. The following may be useful to those who have frequent occasion to draw notes: 1. 33 days = 4 weeks and 5 days; 63 days = 9 weeks; 93 days = 13 weeks and 2 days. Therefore a note at 30 days will fall due five days later than the day of the week on which it was given. A note at 60 days will fall due on the same day of the week. A note at 90 days will fall due two days later. 2. In drawing notes or accepting drafts, it should be borne in mind that . often bills of different dates, having the same time to run, will mature on the same day and thus occasion unexpected inconvenience. Thus a note drawn January 31st for 3 months, and another drawn January 30th for 3 months, both mature April 30th and May 3d. Sometimes one day’s difference in the date or acceptance will make 2, 3 or 4 days' difference in the maturing of bills having the same time to run, thus: Notes drawn or drafts accepted April the 30th at 3 months, mature July 30th and August 2d, but if dated or accepted May 1st, they will mature August 1/4, 2 days later. If a note was drawn February the 28th at 6 months, it would mature August 28th and 31st; but if dated March 1st, it would mature September 1st and 4th, 4 days later. An observance of these principles by accountants and business men will frequently save much inconvenience and sometimes result in preserving the right of action against indorsers. 3. In discounting notes, the most of bankers reject the cents if less than 50, and if in excess of 50, they increase the sum by $1 and reject the cents. In the operations of interest in this book, we will use the exact figures. For Time Tables, see page 298 of this book. Table of Interest Laws of the States and territories, as existing, in 1891, See page 548. INTEREST DIVISORS, 1121. An Interest Divisor is a number which used as a divisor of any prin- cipal will give Interest in the quotient. The Interest Divisor differs according to the rate per cent, and also according to the number of days, 360 or 365, used in a year as a basis for interest computations, 1122. Interest, as shown in the preceding remarks and definitions, is a com- pound of per cent, on a 100, and per annum, for a year, 360 days. Hence to obtain 1 per cent interest for 1 year, on any principal, we simply divide by 100. And to obtain 1 per cent interest for 1 day, on any principal, we first divide by 100 to get 1 per cent interest for 1 year and then divide that quotient by 360 to get the interest for one day. Or we may divide the principal at once by 36000 which is the product of 100, the basis of per cent, and 360 days, the basis of a year. * W. 56O SOULE's PHILOSOPHIC PRACTICAL MATHEMATICS. - The quotient arising by this division, being interest, we therefore name the 36000 the Interest Divisor for 1 per cent. NotE.—when 365 days are used as the basis of interest computations: then the 365 would be used where 360 is used herein, and the 1 per cent Interest Divisor would be 36500. Having thus produced the interest at 1 per cent for one year or one day, to find the interest at any desired rate per cent and for any desired number of years or days, we have but to multiply this interest by the desired rate per cent and the desired number of years or days. TABLE OF INTEREST DIVISORS. 1123. To aid in understanding the interest divisor and the use of the same, we present the following table which gives the interest divisors at 1, 2, 3, 4, 44, 5, 6, 8, 9, 10, 12, 15, 18, 20, 24, 25, 30 and 36 per cent: Interest Interest Interest % D1v1sors. % Divisors. % Divisors. 36000 - 1 = 36000 36000 - 6 = 6000 36000 - 18 = 2000 36000 - 2 = 18000 36000 - 8 = 4500 36000 - 20 = 1800 36000 - 3 = 12000 36000 - 9 = 4000 36000 -- 24 = 1500 36000 - 4 = 9000 36000 - 10 = 3600 36000 -- 25 = 1440 36000 - 4} = 8000 36000 - 12 = 3000 36000 - 30 = 1200 36000 - 5 = 7200 36000 - 15 = 2400 36000 - 36 = 1000 When the rate of interest will not cancel the 36000 without a remainder, then we proceed as shown in the statement for the second operation following: NoTE.—In many computations, Equation of Accounts, Accounts Current and Interest Accounts by the product or equation method, Interest on Daily Cash Balances, Cash Notes, True Discount, etc., the Interest Divisor is of great service, and should be well understood by all accountants. BASIS OF ALL INTEREST COMPUTATIONS. 1124. The foregoing is the basis of all interest computations, and by working in accordance there with we avoid all the arbitrary rules which confuse and confound the millions. To perform the operations of interest in detail, as above indicated, would require considerable time and labor. Hence, with a view to economize both, and still work from the foundation principles of interest, we combine reason and can- cellation with the foregoing principles and evolve a brief, a simple, and a universal formula, applicable to all interest computations. Ǻ INTEREST EVOLUTION. 561 A UNIVERSAL FORMULA FOR COMPUTING INTEREST. 1125. The solution of the following problem shows the application of the foregoing principles of interest, and the evolution in the operations by which the brief, simple, and universal formula is obtained: What is the interest on $72000 at 8 per cent for 11 days? Ans. $176. First Operation, in detail. 36000) $72000 ($2 = interest at 1% for 1 day. = 8%. $16 = interest at 8% for 1 day. 11 = 11 days. $176 = interest at 8% for 11 days. Fourth Operation in Interest Evolution. Third Operation in Interest Evolution. Second Operation in Interest Evolution. $ $ 72000 or, 72000 72000 720.00 100 || 8 36000 || 8 4500 | 11 45 | 11 360 | 11 11 -* -wºme sºm-- *-* : *- | $176, Ans. $176.00 Ans. $176, Ans. $176, Ans. Explanation.—In the FIRST operation, we divided by 36000, the 1 per cent interest divisor and obtained $2, the interest at 1 per cent for 1 day; this we multiplied by the rate, 8 per cent, and obtained $16, the interest at 8 per cent for one day; this we multiplied by the days, 11, and obtained $176, the interest at 8 per cent for 11 days. In the SECOND operation, we indicated, on the statement line, the work of the first operation, and then cancelled. In the THIRD operation, we mentally divided the 36000 by the rate per cent, 8, and produced 4500, the 8 per cent interest divisor. By this mental cancellation, we very much shortened the operation. In the FOURTH operation, we first produce the 4500, the 8 per cent interest divisor, then cancel the two 0's and in compensation therefor point off two places in the principal. By this mental cancellation, we shorten the operation to the greatest practical limit, and present a universal formula for interest computations, far superior to the arbitrary rules given in the arithmetics and calculators now before the public. TABLE OF CONTRACTED INTEREST DIVISORS. 1126. The following table shows the Contracted Interest Divisors, for the most usual rates per cent : TABLE OF CONTRACTED INTEREST DIVISORS, ID % Interest % Interest % Interest S. Divisors. D8. 0 Divisors. DS. Divisors. 360 - 1 = 360 360 - 5 = 72 360 - 15 = 24 360 - 2 = 180 360 - 6 = 60 360 - 18 = 20 360 + 2} = 144 360 - 8 = 45 360 - 20 = 18 360 - 3 = 120 360 - 9 = 40 360 - 24 = 15 360 - 4 = 90 360 - 10 = 36 360 - 30 = 12 360 + 4 = 80 360 - 12 = 30 360 - 36 = 10 NoTE.—When the rate per cent is not a factor of 360, such as 7 per cent, 11 per cent, etc., then 360 will be the Interest Divisor, and the rate per cent will be used as a multiplier as shown in the first operation of the first problem following this table. 562 SOULE's PHILOSOPHIC PRACTICAL MATHEMATICS. ºr A PHILOSOPHIO METHOD OF USING THE FACTORS OF THE INTER- EST DIVISOR, 1127. In the practical computation of interest, we prefer to use the factors of the Interest Divisor (100 and 360, or 365) in a slightly different manner from that shown by the four operations above, yet strictly in accordance with reason and logic. bl In order to fully elucidate the work, we will re-state and re-work the above problem. What is the interest on $72000 at 8 per cent for 11 days? Ans. $176. FIRST OPERATION. SECOND OPERATION. Explanation.—In the first opera- tion, our statement i.to the statement made in the Second Oper- 720.00 720.00 ation preceding, except that we 8 45 || 11 first divide the principal, $72000, 360 | 11 ſº-ſº by 100, by pointing off two places, •º-ºmmºn $176.00, Ans and then by 360, instead of using $176.00. A • * ~ * o the 36000 as a single divisor. .UU, Ans. The reasoning for the work based upon the foregoing elucidations, is as follows: 1 per cent interest on any principal for 1 year is the Tºo part of it, which we produce by pointing off two places. Then, at 8 per cent it is 8 times as much, which is indicated by writing the 8 on the increasing side of the statement line. Then, since the interest, as indicated by the statement is for 1 year, for one day it is the 360th part, which is indicated by writing the 360 on the decreasing side of the statement line; and for 11 days, it is 11 times as much as for 1 day, which is indicated by writing the 11 on the increasing side of the statement line. NOTE.-The reasoning for the second operation, is as follows: The interest on $72000 for 1 year at 1 per cent is the 100th part, which is $720; and for 1 day at 8 per cent it is the 45th part, and for 11 days it is 11 times as much as it is for 1 day. This is a philosophical and logical method of work, step by step, and it is universal in its application. In the second operation, our statement is the same as that in the fourth oper- ation preceding and constitutes the most valuable method possible, of computing interest. The reasoning is the same as in the first operation, except, instead of multi- plying by 8 and dividing by 360, we mentally divide the 360 by 8, and then use the quotient, 45, as a contracted interest divisor. In this manner, we contract the opera- tion to the greatest practical limit and use reason and logic throughout the Solution. PROBLEMS IN INTEREST WORKED EY THE PHILOSOPHIC SYSTEM. 1128. The Principal, Rate Per Cent, and Time given to find the Interest, the Amount, or the Proceeds : FOR YEARS. 1. What is the interest on $560 at 8 per cent for 3 years? Ans. $134,40. OPERATION. $5.60 = 1 % of principal. Explanation.—Considering that interest invol- 8 = 8 % ves per cent and per annum, as elucidated in O e the foregoing work, and in consonance with the e foregoing logical method of operation, we here $44.80 = int. for 1 year. reason as follows: The interest on $560 at 1 per 3 = years. cent for 1 year is the hundredth part, $5,60, * . we express by pointing off two *...*; — i and at 8 per cent it is 8 times as much, which is $134.40 = int. for 3 yrs., Ans. $44.80; and for 3 years it is 3 times as much as for 1 year, which is $134.40. INTEREST. 563 FOR YEARS AND MONTHS. 2. What sum must be paid for the interest on $820 at 8 per cent for 1 year and 9 months. OPERATION. $ :* = 1% of principal. 0 e 12 131 = months. $114.80 = int. for 1 yr. 9 mos., Ans. Ans. $114,80. Ea:planation.—In this and in all problems where there are months in the time, we reason as follows: The interest on $820 at 1 per cent for 1 year, or 12 months, is the hundredth part, $8.20; and at 8 per cent it is 8 times as much; and for i month instead of 13, it is the i3: part; and for 21 months (1 year and 9 months reduced to months) it is 21 times as much as it is for 1 month. FOR YEARS, MONTHS AND DAYS. 3. What is the interest on $1230.40 at 9 per cent for 2 years, 5 months, and 24 days? Ans. $274.99-H. FIRST OPERATION. SECOND OPERATION. $ f 12.3040 = 1% of principal. 12.3040 9 = 9%. 40 | 894 360 | 894 = days. – 1 —— $274,9944 $274.9944 = int. for 2 yrs. 5 mos. and 24 ds. THIRD OPERATION. $12.30.40 $ t 9 12.3040 *E=E--msmºs 360 9 $110.73.60 = interest for 1 year. 174: 2 •º-ºººººº- *mºmº -mºsºs 53.5224 $221.47.20 = interest for 2 years. 53.52.24 = interest for 174 days. $274.99.44 Explanation.—In this and in all problems where there are days in the time, in accordance with the foregoing elucidated principles, we reason as follows in the first operation : The interest on $1230.40 at 1 per cent for 1 year is the hundredth part, $12.3040§ and at 9 per cent it is 9 times as much; and for 1 day, instead of 1 year, it is the 360th part; an and 24 days reduced to days) it is 894 times as much. for 894 days (2 years, 5 months In the second operation, the same reasoning governed the statement, but instead of writing the 9 per cent and the 360, we used the Contracted Interest Divisor, as elucidated in the foregoing work. GENERAL DIRECTIONS FOR, CALCULATING INTEREST. 1129. From the foregoing elucidations, we derive the following general direc- tions for calculating interest: 1. For years, first find 1 per cent of the principal, by dividing by 100 (pointing 564 SouLE's PHILOSOPHIC PRACTICAL MATHEMATICs. 2% off two places); then multiply by the rate per cent, and this product by the number of 3years. See problem 1, page 562. 2. For months, write the principal on the statement line and find 1 per cent, by dividing by 100, or pointing off two places; then indicate on the statement line the division by 12, and the multiplication by the rate per cent and the number of months, and work out the statement. See problem 2, page 563. 3. For days, write the principal on the statement line and find 1 per cent by pointing off two places, or by indicating the division by 100; then indicate on the state- ment line, the divison by 360, (or 365 when that is used) and the multiplication by the ºrate per cent and the number of days, and work out the statement, cancelling as much as possible. See problem 3, page 563. NOTE.-Instead of indicating the division by 360 and the multiplication by the rate per cent, We may simply divide by the quotient (the contracted Interest Divisor) of 360 divided by the rate per cent, as explained on page 562, 4. 4. To find the amount, add the interest to the principal. 5. To find the proceeds, subtract the interest from the principal. NoTE.—There are always as many decimals in the answer as there are on the statement line, and no more. PROBLEMIS. What is the interest on $1350 at 8 per cent for 64 days? Ans. $19.20. What is the interest on $550 at 7 per cent for 72 days? Ans, $7.70. What is the interest on $727.20 at 5 per cent for 11 days? Ans. $1.111. What is the interest on $7200 at 10 per cent for 121 days? Ans. $242. 10. What is the interest on $3155.16 at 6 per cent for 5 years? Ans. $946,548. 11. What is the interest on $2344.80 at 8 per cent for 3 months? Ans. $46.896. 12. What are the interest and the amount of $5000 at 8 per cent for 2 years, 5 months, and 15 days 3 * Ans. $983.33% interest; $5983.33% amount. NOTE.--To obtain the amount, add the interest to the principal. 13. What are the interest and the proceeds of $45000 at 4 per cent for 8 months and 20 days 3 Ans. $1300 interest; $43700 proceeds. NotE.—To obtain the proceeds, subtract the interest from the principal. 14. What is the interest on $1440 at 7 per cent for 123 days? Ans. $34,44. 15. What is the interest on $1711.90 for 34 days at 15 per cent 7 Ans. $24.25-H. 16. What is the interest on $3240 for 94 days at 9 per cent 3 Ans. $76.14. 17. What is the interest on $21636.72 for 63 days at 10 per cent? Ans. $378.64-H. 18. What is the interest on $6840 for 5 years, 8 months, and 14 days at 43 per cent? Ans. $1756.17. 19. What is the amount of $2700 for 1 year, 2 months, and 5 days at 5 per cent 3 Ans. $2859,374. : * INTEREST. 565 20. What are the proceeds of $5284.50 for 4 months, and 12 days at 7 per cent # Ans. $5148.86+. 21. What is the interest on $1500000 for 3 days at # per cent per day? Ans. $11250.00. OPERATION INDICATED. $ $15000.00 15000.00 #% 90 = 4% per day = 90% per year. 360 || 3 or thus: $ 3750.00 = interest for 1 day. 3 = days. $11250.00, Ans. -ºm- $11250.00 = interest for 3 days. 22. What is the interest on $30000 at 4 per cent per month for 28 days? Ans. $1120.00. , OPERATION INDICATED. $ $ 300.00 A 300.00 48 = 4% per month = 48% per year. OI’ 4 360 | 28 % 30 28 $1120.00, Ans. | $1120.00, Ans. 23. What is the interest on $260000 for 11 days at # per cent per day? Ans. $10725.00. 24. What are the interest and the amount of $1864.500 from July 12 to Sep- tember 13, at 44 per cent per annum, last day inclusive? Ans. $14682.94 interest. $1879182.94 amount. 25. What is the interest on $2700 for 50 days at 2 per cent per month? Ans. $90. 26. A note for $4100, having 80 days to run, is discounted at 14 per cent per month. What are the proceeds? Ans. $3963.33. 27. What are the interest and proceeds of $10000 for 3600 days at 10 per cent” Ans. $10000, interest. No proceeds. 28. The cash price of a lot of goods is $300. A buyer purchases the goods on a credit of 90 days when money is worth 8 per cent. What ought to be the credit price # Ans. $306.00, OPERATION INDICATED. * $ 3.00 45 || 90 | $6.00 int. 29. The terms for an invoice of goods amounting to $2350 are “4 months, or 5 per cent discount for cash in 10 days.” The buyer paid cash in 10 days. What did he gain or lose, money being worth 6 per cent? Ans. $76.57 gain. OPERATION INDICATED. $ 2232.50 $2350 – 5% ($117.50) = $2232.50. 60 | 110 $117.50 – $40.93 = $76,57. º- | $40,9291+. 566 soul E's PHILOSOPHIC PRACTICAL MATHEMATICs. * THE PEIILOSOPHIC SYSTEM OF CONTRACTING INTEREST QUESTIONS. 1130. There are a great many methods of contracting interest calculations, but the greater number of them are applicable only to special combinations of numbers, while others require the memorizing of arbitrary rules and exceptions instead of the exercise of the reasoning faculties. The philosophic system which we here present is general in its application, and is based upon one beautiful system of work by which we solve every interest problem, and give a reason for every figure of the work without the aid of rules, no matter what may be the sum, the rate per cent, or the time. The following problems will elucidate this beautiful system of contraction: PROBLEMS TO BE SOLVED MENTALLY WITHOUT THE AID OF PEN OR PENCIL. . What is the interest on $1200 for 64 days at 5 per cent? What is the interest on $1400 for 78 days at 6 per cent? What is the interest on $1500 for 34 days at 8 per cent? What is the interest on $2000 for 124 days at 9 per cent? What is the interest on $1528.60 for 51 days at 12 per cent? : OPERATION OF FIRST QUESTION. 5% $ 12.00 72 | 64. $10.663, Ans. FOURTH QUESTION. 9% $ 20.00 40 | 124 $62.00, Ans. SECOND QUESTION. 6% $ 14.00 60 78 $18.20, Ans. OPERATION OF THIRD QUESTION. 8% $ 15.00 45 || 34 $11.33%, Ans. FIFTH QUESTION. 12% $ 15.28.60 30 51 $25,9862, Ans. In working out these statements, we make no figures but the result, or answer; thus in working the first question we see that the 1200 and the 72 may be canceled by 12, and that 1200 is equal to 12, 100 times, and that 72 is equal to 12, 6 times; we have therefore but to multiply the 64 by 100 and divide by 6; all of which is mentally performed. In the second example, we cancel mentally, one 0 on the 1400 and the 0 on 60, then the 78 by 6, which gives us a quotient of 13, with which we multiply 140, and produce the answer. In the third example, we cancel, mentally, the 45 and the 1500 by 15; we then annex the two 0s to the 34 and divide by 3; the result is the answer. In the fourth example, we cancel a 0 on each side of our statement line, then the 200 or the 124 by 4, and multiply the two factors together, which gives us the correct result or answer. In the fifth example, we cancel a 0 on each side of our statement line, then the 51 by 3, which gives us a quotient of 17, with which we multiply the 15286, and produce the correct result. º: INTEREST. 567 In all interest calculations, we must be careful to point off 1 per cent in commencing the work, and in the final result point off the same number of figures. We commend this system of interest calculations and this method of con-, traction over all others, because of their simplicity and their easy application to all problems in interest without referring to rules, and because, taking the problems as they naturally occur in buisness, they are the shortest and by far the easiest. ... NOTE.-In the following problems, place the figures on the statement line and solvementally, Writing no figures but the answer. 6. Find the interest on $1440 for 67 days at 24% Ans. $6.70. 7. 44 {{ “ 8400 for 28 days at 3% Ans. 19.60. 8. {{ {{ “ 1600 for 54 days at 34% Ans. 8.40. 9. { % { % “ 2817 for 48 days at 44% Ans. 16.90 10. { % {{ “ 2160 for 64 days at 5% Ans. 19.20. 11. { % { % “ 3000 for 134 months at 6% Ans. 202.50 12. € $ {{ “ 450 for 40 days at 7% Ans. 3.50. 13. 44 {{ “ 15000 for 33 days at 8% 3 Ans. 110.00. 14. é & 44 “ 2408 for 10 days at 9% Ans. 6.02. 15. { % {{ “ 1137.60 for 72 days at 10% Ans. 22.752. 16. {{ 46 “ 25000 for 18 days at 12% Ans. 150.00. 17. 4% 4% “ 728.27 for 96 days at 15% Ans. 29.1308. PECULIAR INTEREST QUESTIONS. 1131. 1. What is the interest on 102 for 10 days at 10 per cent? Ans. a's g º STATEMENT. & Explanation.—In problems in which there are not two 100 | 10 figures to represent dollars, instead of indicating the 10 divison by prefixing and pointing off two 0's we divide 360 | 10 the sum by 100 and place the result or quotient on the *== statement line, and then proceed as usual. In this prob- #2 Ans. lem, the interest on 10c. for 1 year at 1 per cent is Tºc. 2. What is the interest on $1.15 for 15 days at 6 per cent? Ans. ##2. 3. What is the interest on 12#2 at 44 per cent for 104 days? Ans. Hºoſe. OPERATION INDICATED. g g g 2 || 25 * 2 || 25 2 || 25 100 OT 36000 OI’ 8000 2 9 2 || 9 . . 2 21 360 º 2 || 21 2 || 21 4. What is the interest on 1632 at 5% per cent for 8 days, 6 hours, and 24 minutes? Ans. Hääog. 5. What is the interest on 1242 for 54 days at 64 per cent f Ans. ###2. 6. What is the interest on 642 for 10 days, 4 hours, and 25 minutes at 4; per cent 3 Ans, a ####52. 568 soule's PHILOSOPHIC PRACTICAL MATHEMATICS. † \)IFFERENCE OF INTEREST BY USDNG THREE HUNDEED AND SIXTY AND THREE HUNDRED AND SIXTY-FIVE DAYS TO THE YEAR. 1132. Since there are 365 days in a common year, it follows that when 360 days are used as an interest divisor on the basis of a year, the interest is ## = # greater than would be produced by using 365 as the interest divisor, and when 365 days are used as the basis of a year or the interest divisor, the result is sås = }; less than would be produced by dividing by 360. The only exception to the above is when the time is in even years; then there is no difference. The following problem will more clearly elucidate this difference: 1. What is the interest on $7300 for 81 days at 8 per cent when using 360 days for a year, and also when using 365 days for a year? Ans. See operations following. Operation for 360 days to the year. Operation for 365 days to the year. \ 7.300 7300 360 | 8 365 || 8 81 81 $131.40 int. $129.60 int. Operations to show that the interest for 360 days is y's more than the interest for 365 days, and that the interest for 365 days is ºg less than for 360 days. $131.40 - 73 = $1.80. $129.60 - 72 = $1.80. $131.40 — $1.80 = $129.60. $129.60 + $1.80 = $131.40 INTEREST AT THEEE EIUNDRED AND SIXTY_FIVE DAYS TO TEIE YEAR. 1133. 1. What is the interest on $5000 for 93 days at 7 per cent, using 365 days for a divisor 3 Ans. $89.18. OPERATION. * 00 Explanation.—The reasoning in this problem is as follows: The interest on $5000 for 1 year at 1 per cent is 365 % $50, and at 7 per cent 7 times as much; and for a day, t_j instead of a year, it is the 365th part, and for 93 days it $89.18 Ans is 93 times as much as for 1 day. 2. What is the interest on $2500 at 6 per cent for 146 days, counting 365 days as the interest year? Ans. $60.00. 3. What is the interest on $1111.11 at 11 per cent for 11 times 11 days, counting 365 days to the year? Ans. $40.51-H. INTEREST. 569 INTEREST ON UNITED STATES BONDS. 1134. registed bonds from July 1, 1885, to August 1, 1890? 6 OI’ $3049.32 Ans. 1. What interest has accrued on $10000 United States 6 per cent Ans. $3049,32. OPERATION. $10000 $ 6 % 10000 smºsºmºsºmºmºsº 6 $600.00 365 30 5 years. =-y- $49,32 $3000.00 int. for 5 years. 49.32 int. for 30 days. *=mºmºs $3049.32 Ans. NOTE.-The United States Government computes interest on the basis of 365 days to a year and for any number of days less than a year a proportional part is taken: Thus, for 19 days, º: would be taken. 2. What interest has accrued on $250000 United States 44 per cent funded loan from September 1st, 1882, to December 1st, 1890? $ 250000 2 || 9 OT 365 3010 smºmºmºse ºmºmºmº tº $92773.97 Ans. Ans. $92773.97 gold. OPERATION. $250000 $ 4.4% 250000 gººmsºn s = 2 || 9 $11250.00 365 | 90 8 years. ammº ºmºmº-3 tº-º-º-º-º: sº $2773.97 $90000.00 int. for 8 yrs. 2773.97 int. for 90 dS. $92773.97 Ans. INTEREST ON ENGLISEI MONEY. 1135. 1. uary 1, 1894, to July 1, 1895? STATEMENT. £ 1000.00 3 365 || 546 tºº fºº!487.67 Ans. What interest has accrued on £100000 English consols from Jan- Ans. £4487.67. Ea:planation.—Consols are English bonds that bear 3 per cent interest. See Stocks and Bonds for full explan- ation. NotE.—365 days are used in Interest Computations in England. 57o soul E's PHILOSOPHIC PRACTICAL MATHEMATICs. Af 2. What is the interest on £1420 12s. 9d. for 63 days at 5 per cent? Ans. £12.5s. 2d. + OPERATION. £ S. d. fº £12.26–H Explanation. — We 1420 12 9 1420.638 20 here first reduce the 5 4% 365 || 5 *mma--mº shillings and pence to 63 5.20S. the decimal of a pound, .60 +.037} = £.638. 12 and then proceed in £12.26030+ -: the usual manner of 2.40d. computing interest. NotE.—For an explanation of the method of reducing shillings and pence to the decimal of a pound, see Article 869, page 450. 3. What is the interest on £31994s. 43d., at 6 per cent for 63 days allowing 360 days to the year? Ans. £33 11s. 10d. º OPERATION. 4s. 4d. £ £33.59179 5 4% 31.99.219 20 60 | 63 -º-º-º-º- .20 + .018; = £.219. -º- 2 tº- 11. 83580S. | £33.59179 12 10. 02960. 4. What is the interest on 14s. 7d., for 50 days at 8 per cent, allowing 365 days to the year? Ans. 1.9176d. practically 26. 5. What is the interest on £8 3s. 10d., for 34 days at 5 per cent, allowing 360 days to the year? Ans. 9.284d.; practically 9d. 6. What is the interest on £15940 12s. 6d. for 125 days at 8 per cent, allow. ing, as is the English custom, 365 days to the year? Ans. £436 14s. 7d. NotE.—See Index, English Exchange, for operations in English Exchange. INTEREST ON FRENCEI CURRENCY. 1136. 1. What is the interest on fr. 55620.25, for 73 days at 4 per cent? Ans. Fr. 444.96. STATEMENT. Fr. 556.20.25 4 365 | 73 | -*Rººmsºmº- | Fr. 444,9620 Ans. Nore...—365 days are used in interest computations in France. 2. What is the interest on fr. 4190.80 for 165 days at 3 per cent? Ans. Fr. 56.83+. INTEREST. 57.1 INTEREST ON GERMAN CURRENCY. 1137. 1. What is the interest on 6870.60 marks for 64 days at 44 per cent? Ans. 54.21+ marks. OPERATION INDICATED. 68.70.60 2 || 9 365 | 64 NOTE.—365 days are used in interest computations in Germany. 2. What is the interest on 128400 marks for 235 days at 5 per cent? # Ans. 4133.42+ marks. DIFFERENT METHODS OF COMPUTING INTEREST. 1138. We present the following different methods, not because they possess any great merit, but that they may be contrasted with our philosophic system, and also that they may be used for reference by business men who have previously learned one of these methods, and who may not have the time or desire to learn a new, though better method, { BANKERS METHOD, OR THE SIXTY-DAY OR SIX PER CENT METHOD. 1139. The following problems will show the method of calculating interest as used by many bankers. NotE.—It is the uniform custom of bankers in their daily operations of banking, to calculate interest from interest tables. 1. What is the interest on $2485 for 75 days at 6 per cent? Ans. $31,06. OPERATION. # of $24 85 = interest for 60 days. Explanation.—The basis of this work is that...? = 6 21 = 46 {{ 15 ($ per cent for 60 days is equal to 1 per cent for 360 days, and º obtain ºp. cent for 60 #. We &__ simply divide the principal, or sum on which we $31 || 06 Ans. “ 44 75 4 .# io find the it; §1. this we do in this case by drawing a vertical line through the sum, so as to cut off the two right hand figures of the dollars. Then, as 75 days is 3 more than 60, we add + of the interest for 60 days to itself, and thus produce $31.06, the correct interest. In case we had but one figure representing dollars, then we would prefix a 0 before drawing the line, and if no dollar figures, then two 0s would be prefixed. If we wish the interest for a different per cent than 6, we would add to or subtract from 6 per cent interest such a part of itself as the rate per cent was more or less than 6. NotE.—The interest for years and months is found by multiplying the interest for 60 days, or two months, by one half the number of months. tº gº * 572 soul.E's PHILOSOPHIC PRACTICAL MATHEMATICS. * 2. What is the interest on $543 for 63 days at 5 per cent? Ans. $4.75. OPERATION BY THE BANKERS' METHOD, OPERATION BY OUR PHILOSOPHIC METEIOD, gº of $5 || 43 = interest for 60 days at 6% - 2715 = 4% “ 3 “ “ 6% $ — 5.43 # of $5 7015 44 “ 63 “ “ 6% 8 72 63 7 E 9502 {{ 4 63 44 4. 1% sºme ºsmºs e-º-º- ºms | 3801 $4 || 7513 Ans. “ “ 63 “ “ 5% tº-ºº ºsmº $4.75; Ans. Ezplanation.—Here we deduct # of the interest at 6 per cent, and thus obtain in the remain- der 5 per cent. 3. What is the interest on $1428 for 35 days at 8 per cent? Ans. $11.103. OPERATION BY THE BANKERS’ METHOD. OPERATION BY OUR PHILOSOPHIC SYSTEM OR METHOD. # ) $14 || 28 interest for 60 days at 6% # ) 7 || 14 46 “ 30 “ “ 6% t $ 1 || 19 { % “ 5 “ “ 6% 14.28 *mº emº 9 4; 3# 7 # ) $8 || 33 {{ “ 35 “ “ 6% ammº 2 || 773 is # of 6% interest added. | 9996 $11 | 103 Ans. $11.10; Ans. 4. What is the interest of $1260 for 106 days at 6 per cent? Ans. $22.26. Operation by the Bankers’ or 60-day or 6 per cent Method. Operation by our Philo- } sophic Method. $12.60 = interest for 60 d. Cut off two places to the right. 6.30 = interest for 30 d. 3 of $12.60 = $6.30 $ 21 2.10 = interest for 10 d. 3 of 6.30 = 2.10 #" 1.26 = interest for 6 d. } of 6.30 = 1.26 39 $22.26 = interest for 106 days. $22.26 5. What is the interest of $2340 for 3 months, 27 days at 7 per cent? Ans. $53,234. Operation by the Bankers’ or 60-day Method. Operation by our Philo- gº sophic System. $23.40 = interest for 60 days, or 2 months. e º 9 11.70 = interest for 30 days, or 1 month. * 3 7.80 = interest for 20 days, # of 23.40 = $7.80 369 || 7 2.34 = interest for 6 days, # of 11.70 = 2.34 6 || 117 39 = interest for 1 day, # of 2.34 = 39 $45.63 = interest for given time at 6% * 819 7.61 = interest for given time at 1% # of $45.63 = $7.61 31941 $53.24 = interest for given time at 7% NOTE.—Add one cent when the fraction is # or more. $53,23# Ans. º: INTEREST. 573 6. What is the interest of $3600 for 5 months, 19 days, at 4 per cent? Ans. $67.60. Operation by the Bankers’ or 60 day Method. Operation by our Philo- sophic System. $36.00 = interest for 60 days, or 2 months. 18.00 = interest for 1 month. $ | 36.9% 40 90.00 = interest for 5 months, $18.00 x 5 = $90.00 gſ | 169 9.00 = int. for 15 d. 15 = 3 of 60. 4 of $36.00 = $9.00 - | 1.80 = int. for 3 d. 3 = } of 15. # of 9.00 = 1.80 60 = 1nt. for 1 d. = # of 3. § of 1.80 = .60 $101.40 = interest for given time at 6% 33.80 = interest for given time at 2% $67,60 Ans. 67.60 = interest for given time at 4% Take # of $101.40 from itself. 7. Which of the above methods has the greater merit? Ans. The Philosophic. Taking all principals, all rates per cent and all periods of time, the Philo- sophic System is far superior to all others. TEIE THIRTY-SIX PER CENT METHOD OF COMPUTING INTEREST. 1140. This method is based upon the Interest Divisor for 36 per cent which is, as shown on page 560, 1000. Hence, as elucidated on page 561, to find the interest on any amount for 1 day at 36 per cent, divide by 1000; then compute it for the required time and rate per cent. PROBLEMI. 1. What is the interest on $2758.40 for 75 days at 8 per cent? Ans. $45.97+. OPERATION INDICATED. t Explanation.-Since the 36 per cent Interest 2,75840 = interest for 1 day at 36%. Divisor is 1000, we divide the amount by 1000 36 || 8 and thus obtain $2.75840 interest for 1 day at 36 per cent. Then, as indicated in the statement, 75 we divide the interest for 1 day by 36, to obtain -* | *-ºº-mºmº- - the interest at 1 per cent; this * We *: - e c ply by 8 to obtain the interest at 8 per cent, an $45,97334 Ans the result thus produced, by 75 to obtain the Interest for 75 days. Various methods are used by different parties to conclude the operation after dividing by 1000. But as none of the methods are equal to the above for ease and simplicity, we will not occupy space with them. The 100 day method, of computing interest is also based upon the 36 per cent Interest Divisor, and like the above method, is far inferior to the unrivaled Philo- 574 soul E's PHILOSOPHIC PRACTICAL MATHEMATICS. ºr sophic System, as shown in previous problems, pages 572 and 573 and by the foi- Iowing operation of the above problem. OPERATION INDICATED BY THE PEIILOSOPHIC SYSTEMI. $ $ 27.5840 27.5840 45 || 75 t Or, 3 || 5 $45,9733; Ans. $45,9733; Ans. THE S. L. BLACK METHOD OF CALCULATING INTEREST. 1141. The following method is generally known as the S. L. Black Method, it having been taught by him some 40 years ago. It is one of the best of the rule methods and yet it surrenders without a condition to our Philosophic System. NoTE.—See the operations of the same problems by the two systems. 1. What is the interest on $2400 for 1 year, 4 months and 15 days, at 6 per cent 3 Ans. $198. OPERATION. $24.00 16.5 Explanation.—In this problem, we first reduce 1200 the 1 year to months and add to it the 4 months, thus making 16; we then divide the days by 3 gºººº-ºº and annex the quotient 5 to the 16 months. We $198,000 Ans. then multiply the number thus obtained by such * a part of the sum $2400 as the 6 per cent is part of 12, which is #, and produce the answer in mills. By this method of computing interest, when the rate per cent is less than 12, reduce the time to months and annea, thereto # of the days, if any, and then multiply the sum or principal by such a part of the time, or the time by such a part of the sum or principal as the rate per cent is part of 12, and point off three, as the answer is in mills, when there are days. When there are cents in the principal, point off five places. When the rate is 12, multiply all of the time by all of the principal. When the rate is more than 12, first find the interest at 12 per cent and then add such a part of the interest to itself as the excess of rate is part of 12. The basis of this system is that 1 per cent for one year is 12 per cent for one month, and hence to find the interest for months at 12 per cent, we have but to multiply the sum by the number of months, or the number of months by the sum, and the answer is in cents. INTEREST. e 575 2. What is the interest on $1230 for 48 days at 8 per cent? Asts, $13.12. OPERATION. OPERATION #) $1230 BY OUR PHILOSOPHIC SYSTEM. TIO x 2 = 820 $ #) 48 = 16 # 4; | º ; $13.120 Ans. $13.12 Ans. Earplanation.—In this example we see that 8 per cent is # of 12, and that § of $1230 is $820, which we multiply by 16, which is # of the days, and we have the answer in mills. 3. What is the interest on $6400 for 94 days at 9 per cent # Ans. $150,40. OPERATION OPERATION BY THE S. L. BLACK METHOD, IBY OUR PEIILOSOPHIC SYSTEM. # of $6400 $ * * = $4800 4} |94 31#. smºm- -*mºsºmºmºsºmº $150.40 Ans. 148800 1600 *mm mºme $150,400 Ans. 4. What is the interest on $1512 for 35 days at 10 per cent? Ans. $14.70. OPERATION BY THE S. L. BLACK SYSTEMI, OPERATION BY OUR PHILOSOPHIC SYSTEM. 252 50 4 # $1512 $1512 $ 11 | 420 _* 36 gº 42 1260 $16,632 113 or, 1.008 $14.70 Ans. 13860 1 ºr gao 840 # ) sº interest at 12%. $14,700 Ans. $14.70 interest at 10%. By the contrasts here made, it is clear that our philosophic system possesses advantages over the very best rule work known. 576 SOULE's PHILOSOPHIC PRACTICAL MATHEMATICs. * A NEW TABLE. 1142. The following table is used by many accountants, with advantage, in interest computations. 6% per annum = }% of the face of a note or debt for 30 days, 44 8% “ “ = 3% “ {{ {{ {{ 4 4 30 9% “ “ = #% “ 4% {{ {{ 44 44 30 44 10% “ “ = #% “ {{ {{ {{ 44 4 30 4 12% “ “ = 1 % “ {{ {{ {{ £4 4 30 44 15% “ “ = 14% “ {{ {{ {{ 44 & 30 (£ 18% “ “ = 14% “ 4% {{ {{ {{ {{ 30 4 20% “ “ = 13% “ {{ {{ {{ {{ {{ 30 4 24% “ “ = 2 % “ 4% {{ {4 4 4 30 4 The following problem will illustrate the method of work by the use of this table: PROBLEMS. 1. What is the interest on $843.60 for 15 days at 8 per cent? Ans. $2.81. OPERATION. Explanation.—As 8 per cent per annum is equal to 3 per cent of the face for 30 days, and as 15 days is 3 of 30, it is plain that the interest is # of 3 per cent (= #) of the face $8.43.60 of the $843.60, which is $2.81. Had the time been 18 e- days, we would have first found it for 15 days, then added $2.81.20 Ans. # of the interest to itself, as 18 is # or 20 per cent more than 15. Had it been for 20 days, we could have found it for 15, and then added # of the interest to itself, or we could have found it for 30 days and then deducted # of the interest, or we could have reasoned thus: 20 days being # of 30, and 8 per cent per annum being 3 per cent of the face for 30 days, then # of # (= 3 of the face) is the interest. By a little reflection, it is readily seen that all of these contracted methods are but the canceled results of our philosophic system, and to the ready calculator they can often be used mentally to great advantage. 2. What is the interest on $1624.50 for 22 days at 6 per cent? 3. What is the interest on $420.80 for 34 days at 9 per cent? BANKERS’ - AND MERCHANTS’ DISCOUNT. 1143. In the preceding work, we have defined simple interest without making mention of Bank Discount ; and as there has been and still is much discussion on the subject of bank discount, we will therefore appropriate a small space to a con- sideration of the subject before giving examples. Bank Discount, or the more gen- eral term, Discount, is interest paid in advance. Hence, when bankers and business men discount notes or debts, where the element of time is considered as well as the rate per cent, they get the interest on the face of the note or debt for the actual time, and at the specified rate per cent. The sum thus obtained, and called interest or discount, is then deducted from the face of the note or debt discounted, and the remainder or difference is the cash value or proceeds. ºf BANKERs' DISCOUNT. 577 Many persons uninformed on the subject of business customs pertaining to the Operations of interest, earnestly contend that bank discount is the same as another division of interest called True Discount. True discount differs materially from bank discount as we shall elucidate a few pages further on, and consists in find- ing what sum of money loaned for a certain time at a certain rate per cent will amount to a given sum ; or, to define it differently, it is the finding and deducting of such a sum called true discount, from the face of a note or debt, as will leave a remainder which, if the simple interest be added thereto for the same time and rate, will reproduce the original sum. The real difference between bank and true discount arises from the fact that in bank discount we operate on the face of the note or debt, which, of course, includes the interest charged; and in true discount we operate on the true present value of the note or debt, and hence the difference amounts to the simple interest on the true discount. The term true discount does not imply that bank discount is false or untrue. It is thus named to distinguish it from bank discount, and because the true discount of a note or debt is just equal to the interest on the proceeds of the sum from which the true discount was deducted. 1144. PROBLEMS IN BANKERS" AND MERCHANTS’ DISCOUNT. NotE,--The following examples are all of a practical character, and each involves some point of law or of business custom necessary to be known in dealing in notes and securities, or in per- forming the daily discount transactions of the merchant and the banker. TO MATURE AND DISCOUNT NOTES, WHEN DRAWN IN MONTHS AND WHEN DRAWN IN DAYS. NoTE.—Observe carefully the difference in the maturity and discount of the two following notes: $2540.80 * NEW ORLEANs, December 18, 1895. 1. Two months after date, for value received, I promise to pay to the order of Frank Draxler, Two Thousand Five Hundred Forty and #6 Dollars. A. D. HOFELINE. When does this note mature? What are the proceeds, if discounted the day it was drawn at 9 per cent ? * Ans. It matures February 18/21, 1896. The net proceeds are $2498.88. OPERATION. Explanation.—In maturing this note in accordance with 40 #10s law, we count the months, but in discounting in accord- * ºsmºº-ºº: ance with business custom in New Orleans, we count the | $41.9232 discount. actual number of days in the two months and add thereto $2498.88 proceeds. 3 days of grace and discount day. 578 soule's PHILOSOPHIC PRACTICAL MATHEMATICS. * $2540.80 NEW ORLEANs, December 18, 1895. 2. Sixty days after date, for value received, I promise to pay to the order of C. Reynolds, Two Thousand Five Hundred Forty and #, Dollars. J. B. ANDERSON. When does this note mature? What are the proceeds, if discounted the day it was drawn, at 9 per cent? Ans. February 16/19, 1896, it matures, $2500.15, proceeds. OPERATION. 25.40.80 Explanation.—In thus problem, according to law, we mature in days, and according to custom we discount in 40 || 64 | $40,6528 discount. days, counting grace and discount day. *-*= $2500.15 proceeds. GENERAL DIRECTIONS FOR BANKERS’ AND MERCEHANTS’ DISCOUNT, 1145. From the foregoing elucidations, we derive the following general direc- tions for Bankers' and Merchants' Discount: 1. Calculate the interest on the note at the given rate for the actual number of days that the note has to run, plus three days of grace and discount day. 2. Subtract the interest thus found from the face of the note ; the remainder will be the proceeds. NOTE: 1. – When notes bear interest, find the amount or value of the same at maturity, and calculate the discount on such maturity value. NotE 2 — In many cities and States, interest is not charged for discount day. Not E 3.-In all problems following, discount day is included unless otherwise stated, PROBLEMS. 3. January 3, 1895, a note was drawn for $1200 and made payable one year after date, without interest. When does this note mature? If discounted on the day that it was drawn at 8 per cent, what was the discount and proceeds? Ans. January 3/6, 1896, it matures, $98.40, discount. $1101.60, proceeds. OPERATION. Explanation.—This note being drawn in years, we therefore, in 12.00 10 conformity to custom, mature it in years, and when we discount 8 it, in conformity to a different business custom, we count the 3 36% 369 123 actual days of unexpired time, including 3 days grace and dis- | —— count day. In making the statement, we use the same reasoning | $98,40 as in the third problem of Interest. $6231.50. NEW ORLEANs, November 1, 1895. 4. Ninety days after date, for value received, I promise to pay to the order of R. Cole, Six Thousand Two Hundred Thirty-one and º, Dollars; payable at the Germania National Bank, New Orleans. ' J. P. WILKINSON. Yºr BANKERs' DISCOUNT. 579 When does this note become due What are the proceeds, if discounted December 23, 1895, at 6 per cent? Ans. January 30/2 1896, it is due. $6187.88, proceeds. OPERATION. $ Explanation.—Had this note been discounted 62.31.5% the day that it was drawn, it would have been discounted for 94 days; but as it was not dis- counted until December 23d, we see that 52 days § 360 || 42 7 have already expired, and as discount is only *m-meems charged for the time that the note or debt has $43,6205 discount. to run, we therefore deduct the 52 days from the $6TS7.33 T d 94, and obtain 42 days unexpired time for which ºn 7. net proceeds we discount the note. 5. A note for $7281.27, bearing date September 14, 1895, and payable 90 days after date, was discounted at the Germania National Bank, September 28, 1895, at 8 per cent. What were the interest and net proceeds, and when does the note mature ? Ans. $129.44, interest. t; $7151.83, net proceeds. December 13/16, 1895, it matures. 6. A draft, bearing date July 18, 1895, and accepted July the 21st, 1895, for $10000, payable 30 days after sight, was discounted August 7th, at 8 per cent. What were the net proceeds, and when does the draft mature? Ans. $9962.22, net proceeds. August 20/23, 1895, it matures. 7. A note for $5800, bearing date February 12, 1895, and payable 6 months after date, was discounted by a note broker March 1, 1895, at 12 per cent. What was the maturity of the note? What was the discount, and what the cash value or proceeds? Ans. August 12/15, 1895, it matured. $324.80, interest. $5475.20, cash value. 8. How much is the interest on $20000, for 64 days at 8 per cent? Ans. $284.44. 9. A note for $75000 dated August 19, 1895, and payable 60 days after date, was discounted August 20, 1895, at 5% per cent. What were the proceeds counting days of grace and discount day? Ans. $74245.31. 10. In the above problem, what would have been the proceeds, allowing 365 days to the year, and not counting discount day ? Ans. $74.267.47. OPERATION INDICATED. $ 750.00 365 || 62 4 23 732.53 discount. 11. A note for $385000, bearing date May 7, 1895, payable 15 days after date, was discounted May 8, 1895, at 64 per cent. What was the discount, not counting discount day ? º Ans. $1181.74. 58O - SOULE's PHILOSOPHIC PRACTICAL MATHEMATICS. ºr TO COLLECT NOTES THAT BEAR INTEREST AND EVENTUAL INTEREST. 1146. 1. January 3, 1895, a note is drawn for $1200 and made payable one year after date, with interest at 8 per cent. What is the interest and amount due the holder at the maturity of the note. When does the note fall due # Ans. $96, interest. $1296, amount. Due, January 3/6, 1896. OPERATION IF PAID JANUARY 3, 1896. OPERATION IF PAID JANUARY 6, 1896. $1200 face of note. $ 1200 8% 1200 45 || 363 $96.80 int. for 1 yr. and 3 ds. or, 45 || 360 – | $ 96 $1296.80 amount. $96.00 interest for 1 year. $1296 amount. & NOTE.-Had the note been paid January 4, interest would have been charged for 361 days; if paid January 5, interest would have been computed for 362 days, See pages 557 and 558, for an explanation for thus counting the days. 2. I receive for the sale of real estate a note for $9000, bearing date January 5, 1895, payable two years after date with 6 per cent current, and 8 per cent event- ual interest. The note was paid August 10, 1897; what was the amount received? Ans. $10512.50. NOTE.-Eventual Interest is the interest the note bears after maturity, in case it is not paid at maturity. OPERATION. Face of note - tº * * gºs * : ge º * tº a t- $9000 Interest for 2 years and 3 days, at 6 per cent - & $1084.50 Interest on faee of note from Jan. 8, '97, to Aug. 10, '97, at 8 per cent (214 days) wº * tº- 428.00 1512.50 Amount due August 10, 1897 sº m 'm as ºn tº $10512.50 3. Received a note dated October 2, 1895, for $6666.67, payable one year after date, without grace with 6 per cent interest. When does it mature, and if I hold the note until it matures what will be the interest and what the amount due 3 Ans. October 2d, 1896, it matures. $400, interest. $7066.67, amount due. TO DISCOUNT NOTES THAT BEAR INTEREST. $4500. *. NEW ORLEANs, June 4, 1895. 1147. 1. Four months after date, for value received, I promise to pay to the order of O'Neil, Sullivan & Co., Four Thousand Five Hundred Dollars, with 6 per cent interest. G. PHARR. º: BANKERs' DISCOUNT. 58 I - When does this note mature? If discounted the day it was drawn by a note broker at 8 per cent, what proceeds would the holder receive # Ans. It matures October 4/7, 1895. The proceeds are $4463.67. OPERATION OPERATION To find the amount or value of the note To discount the maturity value of the note at maturity. $ and find the proceeds. * 4500 4592.25 60 | 123 45 126 $92.25 = int. for 123 ds, at 6%. $ 128,5830 = interest or discount. 4500. = face of note added. 4592.25 = value of note at matu- wºmºsºms mºs 3. tº-º-º-º: rity. $4592.25 = amount or value of note $4463.67 = proceeds of note. at maturity. Explanation.—In this problem, we first find the amount or value of the note at maturity. This we do by calculating and adding to the face of the note, the 6 per cent interest that it bears. for 4 months and 3 days. This work gives us $4592.25 as the value of the note when it matures; hence it is clear that this is the amount to be discounted. We then discount the $4592.25 according to business custom for the actual unexplped time, including 3 days of grace and discount day, at the specified 8 per cent. $9730. NEW ORLEANs, November 24, 1895. 2. One year after date, for value received, I promise to pay to the order of E. B. Cope, Nine Thousand Seven Hundred and Thirty Dollars, with interest at 8 per cent. LOUIS BUSH. What is the above note worth on the 1st June, 1896, if discounted at 12 per cent # - Ans, $9884. OPERATION OPERATION To find the worth of the note at maturity. To discount or find the cash value of the note June 1, 1896. $ 9730 $10514.89 45 363 30 180 $ 784.89 = int. for 1 yr. and 3 ds. $ 630.8934 = interest or discount. 9730. = face of note. 10514.89 = principal or value of *E=º ºsmºsºms ºr note at maturity. $10514.89 = value of note at matu. $9884.00 = proceeds or value of rity. note June 1, 1896. - Ea.planation.—In this as in the preceding example, we first find the value of the note at maturity by adding to the face of the same the 8 per cent interest that it bears for one year and 3 days; and then we discount the maturity value at 12 per cent for the unexpired time, counting actual days, with grace and discount day. In finding the time we count thus: June 29, July 31, August 31, September 30, October 31 and November 24 days, 3 days grace and discount day (June the 1st) = 180 days. 582 SOULE's PHILOSOPHIC PRACTICAL MATHEMATICS. º: 3. A mortgage note for $20000, dated December 16, 1895, payable 18 months after date, with 8 per cent interest, was discounted January 6, 1896, at 7 per cent. What was the maturity value and the proceeds? Ans. $224.13.33 = maturity value. $20103.51 = proceeds. NOTE.-The mortgage matured June 16119, 1897. Interest added at 8 per cent for 18 months and 3 days (543 days) equals $2413.33. Maturity value discounted at 7 per cent, for the time § January 6, 1896, to June 16/19, 1897, including three days of grace and discount day, equals ayS. 4. A note drawn April 2, 1895, at 3 months, for $1635.15, bearing 8 per cent interest, was discounted June 1, 1895, at 8 per cent. When did the note mature? W. at is it worth at maturity, and what was the cash value when discounted Ans. July 2/5, 1895, it matures. $1668.94, worth of note July 8, 1895. $1655.96, cash value June 1, 1895. NOTE.-Interest is added for 93 days. The maturity value is discounted for 35 days, which includes discount day. 4. CASEI NOTES 77 OR DRAFTS WHICH, WIHEN DISCOUNTED, WILL PRODUCE A SPECIFIED SUM. 1148. 1. For what sum must a 60 day note be drawn, so that when discounted at 8 per cent, the proceeds will be $8872? Ans. $9000. FIRST OPERATION. Explanation.—As it is the custom of bankers to calcu- º º *: on the . º notes, 1. ºf'; * plain that if we were to a the simple interest of the $4500 note assumed. $8872 to itself for the time and rate º, and draw the 64 interest for 64 days at 8%. note for the amount thus produced, it would not, when discounted, produce the . sum for the reason that the interest on this increased amount would be more than $4436 p roceeds, Cash. the interest on the first sum. The deficit would be the interest on the interest first obtained, plus the interest on each succeeding sum of 1nterest, interminably. Conse- 4500 quently, to produce exact results, we cannot work on the 4436 | 8872 face of the sum that we desire to obtain for the note * when discounted. We are therefore constrained to assume some number to represent the face of the note to be drawn, $9000, Ans. and to facilitate the operation we assume $4500 the 8 per cent Interest Divisor, to represent the face of the note. We assume the Interest Divisor for the reason that the interest thereon is always as many dollars as there are days in the time; and knowing this we are saved the labor of computing the interest on the sum assumed. Accordingly, in this problem, the time being 64 days, the interest or discount on $4500 for 64 days at 8 per cent is $64, which subtracted from $4500, the assumed note, gives $4436 proceeds. We now observe that as we discounted the $4500 for 64 days at 8 per cent, the same ratio exists between the $4436 proceeds and the $4500 assumed note as exists between the $8872 proceeds and the face of the note required to produce the same when discounted for the given time and rate pr. ct, Hence we have but to find the proportional result of these two ratios. To do this we place the $4500 assumed note on the statement line and reason thus: Since $4436 cash require $4500 note, $1 cash will require the 4436th part and $8872 will require 8872 times as much. SECOND OPERATION By assuming $100 as the face of the note. $100 note assumed. $100 $ 45 | 64 1.42% 100 - ammºms *- 887.20 | 9 $1.42% interest. $98.57% proceeds. 8872.00 | $9000 = face of note. * CASH NOTES. 583 GENERAL DIRECTIONS. 1149. From the foregoing elucidations, we derive the following general direc. tions for finding the face of “Cash Notes:” 1. Assume as the face of the note the Interest Divisor for the rate per cent given or $100, and find the proceeds of the same for the given time and rate per cent. 2. Then divide the assumed note multiplied by the required proceeds, by the proceeds of the assumed note. PROBLEMIS. 2. What must be the face of a note, so that, when discounted for 94 days at 12 per cent, it will produce $10000 proceeds? Ans. $10323.47. OPERATION By the use of the Interest Divisor, $3000 note assumed. 3000 note assumed. 94 interest for 94 days at 12 per cent. 2906 || 10000 $2906 cash proceeds. $10323.47, Ans. 3. A creditor owed me a balance of $3212.65 and settled the same with his “Cash Note,” payable 4 months after date. The note was dated June 17, 1895. Allowing 8 per cent interest, what was the face of the note, and when does it mature ? Ans. $3305.19-H face of note. October 17/20, 1895, it matures. 4. What must be the face of a note to net or produce $1777.95, when dis- counted at 7 per cent for 63 days? AnS. $1800. OPERATION. $ $36000 = note assumed. 36000 441 = interest for 63 days at 7%. 35559 || 1777.95 $1800.00, Ans. $35559 = proceeds. Explanation.—There being no 7 per cent Interest Divisor, we assume as the face of the note the 1 per cent Interest Divisor and then multiply the interest at 1 per cent, which is $63 (as many dollars as there are days) by 7, the rate per cent and thus obtain $441 interest. The statement is then made as in the preceding examples. NotE.—Whenever there is no Interest Divisor for the rate per cent given, the Interest Divisor for 1 per cent should be assumed and the interest found thereon as above, or, if preferred, by assuming and discounting a $100 note, as shown in the second operation, page 582. 5. A creditor owed me a balance of $6812.45 and settled the same with his “Cash Note,” payable 4 months after date. The note was dated June 17, 1895, allowing 8 per cent interest. What was the face of the note, and when does it mature? Ans. $7008.69, face of note. October 17/20, 1895, it matures. 584 SOULE's PHILOSOPHIC PRACTICAL MATHEMATICs. * 6. A merchant wishes to obtain $7500 from bank, on a 30 day note, The rate of discount is 8 per cent. What size note must he offer so that, when dis- counted, he will receive the exact sum ? Ans. $7557.10. 7. A creditor owes $1250 and wishes to pay it with a draft at 45 days sight, without grace or discount day. What must be the face of the draft, the rate of dis- count being 8 per cent? Ans. $1262.63. 8. For what sum must a note be drawn for 45 days, without grace or dis. count day, to settle a cash balance of $10862.50, interest at 10 per cent - Ans. $11000. 4. CASEH NOTES.” WITH INTEREST, COMMISSION, AND BROKERAGE COMBINED. 1150. 1. A customer desires to obtain from a bank $8000 on his 90 day note. In conformity with bank custom, which prudence and safety demand, he is required to have one or more indorsers on the note, and as his correspondent I indorse and negotiate the note for him. The rate of bank discount is 8 per cent; I charge 24 per cent commission for indorsing, and # per cent brokerage for negotiating. What must be the face of the note % - Ans. $8406.80. OPERATION, Face of note assumed, - gº - E- - tº- -> tº- $4500. Interest on same for 94 days at 8 per cent - - $ 94. Commission on same at 24 per cent tº - wº 112.50 Brokerage on same at # per cent - - - - 11.25 — 217.75 Cash value, or proceeds of the assumed note. - - - - $4282.25 $ Explanation.—In this solution, for reasons given in the solution of the first problem of 4500 = note assumed. * Cash Notes,” we assume the 8 per cent Interest 4282.25 | 8000.00 Divisor as the face of the required note, and *-*sºn &mmº-º from it we deduct the interest, commission and $8406.80, Ans. brokerage, and thus produce the necessary relationship numbers, as explained in the first solution with which we make the proportional statement, the result of which gives the correct 3D SWOI’. SECOND OPERATION. By assuming $100 as the face of the note. - * $ 100 face of note assumed. 100 45 94 856.45 || 9 8000.00 $2.08; interest. smm.sºmem-mº 2.50 = 24% commission. $8406.80 Ans. 25 = #% brokerage. $4.83; amount of deduction. 100. note assumed. $95.16% proceeds. * CASH NOTES., 585 To discount the above note, the following figures would be produced: Face of note, * * * * * * * * - - $8406.80 IXEDUCTIONS. Interest on face of note for 94 days at 8 per cent, - $175.61 Commission on facé of note at 24 per cent, - tº 210.17 Brokerage on face of note at # per cent, tº- tºs 21.02 —-- 406.80 Net proceeds, - sº * tº- - - º - - $8000.00 2. A merchant owes a cash balance of $2540.37, which he wishes to settle by note at 120 days for such a sum as, when discounted at 8 per cent, and allowing 24 per cent commission for indorsing, will net the exact cash balance. What must be the face of the note? Ans. $2681.29. 3. What must be the face of a note for 60 days to net $1720, discount at 5 per cent and 2% per cent commission for indorsing ? Ans. $1780.33-H. 4. A merchant owes a cash balance of $4000, which he wishes to settle by note at 120 days for such a sum as, when discounted at 8 per cent, allowing 24 per cent commission for indorsing, will net the exact cash balance. What must be the face of the note % AnS, $4221,88. CASH NOTE WITH COMMISSION ON FACE AND BROKERAGE ON THE BALANCE, 1151. 1. A merchant owes his correspondent $3250 cash, for which it is agreed that he shall give his note for such a sum as will net the amount due, after allowing for the following deductions: Interest for 63 days at 10 per cent; commis’ sion on the face of the note at 24 per cent; and 3 per cent brokerage on the balance, viz., on the remainder of the note, after the interest and commission shall have been deducted. For what face must the note be drawn 3 Ans. $3411,31. OPERATION, $3600 face of note assumed. $63 = interest. $ 90 = commission. 3600 6859.53 || 2 $153 = interest and commission. 3250.00 3600 = note assumed. *ms $3411.31 Ans. $3447 = proceeds. 17.23% = }% brokerage on $3447 $3429.76% NotE.—Observe that the interest and commission are required on the face of the note, and brokerage on the balance, after deducting interest and commission. - 586 SOULE's PHILOSOPHIC PRACTICAL MATHEMATICs. * SECOND OPERATION. $ 100 note assumed. $ sº 199 25 4.25 int. and com. 100 A 36 £3 7 190.5425 | 2 º $95.75 proceeds. 3250.0000 $1.75 interest. .47873 = }% brok. on $95.75. *m-mºsºmeºm, 2.50 = 24% com. — $3411.31 $95.27124 net proceeds. AnS. $4.25 = int. &com. To discount the above note, the following figures would be produced: Face of note, - - - - - - - - - - $3411.31 DEDUCTIONS. Interest on face of note for 63 days at 10 per cent, - $59.70 Commission on face of note at 24 per cent, tº - 85.28 — 144.98 Balance, . . . . . . . . . . $3266.33 Brokerage on balance at 4 per cent, - ſº tº- º º ‘º 16.33 Net proceeds of note, - tº - tº- - gn $3250.00 CASH NOTE WITH BROKERAGE ON THE FACE AND COMMISSION ON THE BALANCE. 1152. 1. For what sum must a note be drawn to produce $742.50, after the following deductions are made: 184 days interest at 12 per cent, 4 per cent broker. age on the face of the note, and 24 per cent commission on the balance # Ans. $813.46+. OPERATION $3000 face of note assumed. $ 184 = interest. $2808.50 7.50 = brokerage. 70.2125 = com. On $2808.50 $3000 2738.2875 || 742.5000 *-m-mºs mºmsm- $ 191.50 = int. and brok. $2738,2875 3000.00 = note assumed. | $813.46 $2808.50 = proceeds. NotE.—Observe that the interest and brokerage are required on the face of the note, while the commission is required only on the remainder, after deducting Interest and brokerage. SECOND OPERATION. $ 100. = note assumed. $ 100 face of note assumed. 6.38% = int. and brok. 100 30 | 184 - * 182.5525 2 $93.61663 = proceeds. 742.5000 | $6.13% interest. 2.3404% = .24% com. on * * * * .25 = #% brokerage. *-* * $93.61663. $813.46+ $91.27624 net proceeds. Ans. $6.38% amt. of deductions. *. CASH NOTES. 587 CASEI NOTE WITH INTEREST ON THE FACE AND COMMISSION ON THE REMAINDER. 1153. 1. A merchant wishes to draw a note for such a sum that, when discounted for 64 days at 9 per cent, and 24 per cent commission on the remainder is deducted, it will net $25000. What must be the face of the note 3 Ans. $26057.95. OPERATION $4000 = note assumed. $ 4000 64 = interest. 3837.60 25000.00 $26057.95 Ans. $3936 = proceeds. 98.40 = commission. $3837.60 = net proceeds. NOTE. –Observe that the commission is required on the remainder only. SECOND OPERATION. $ $ N. 10% note assumed. 100 49 || 64 95.94 || 25000.00 $1.60 interest on $100 note assumed. * | $26057.95 Ans. 100.00 note assumed. $98.40 proceeds of $100 note assumed. 2.46 = 24% commission on remainder or proceeds. $95.94 net proceeds of the $100 note assumed. In discounting the above note, the following figures would be produced: Face of note, es wº º & * a {-, } * * tº wº $26057.95 Less interest for 64 days at 9 per cent, a t- * > º º 416.93 Remainder, - * * > • * {- «º tº gº * . ge $25641.02 Less commission at 24 per cent, tº tº º ſº gº º 641.02 Net proceeds, † º º º * , º gº tº º 4- $25000.00 CASEH NOTE WITH INTEREST ON TEIE FACE AND COMMISSION ON TEIE NET SUM. 1154. 1. A merchant wishes to draw a note for such a sum that, when dis- counted for 64 days at 9 per cent, and 23 per cent commission on the net sum is deducted, it will produce $25000. What must be the face of the note? Ans. $26041.67. OPERATION. $4000 = note assumed. Net sum to be produced, $25000 $ 4000 64 = interest. Com. on same at 24%, - 625 || 3936 || 25625 $3936 = proceeds. Amt. to be obtained from | $26041.67 Ans. bank, - - - $25625 NotE.-Observe that the commission is required on the net sum only. 588 soul E's PHILOSOPHIC PRACTICAL MATHEMATICS. * SECOND OPERATION. $ Net sum to be pro- 10% note assumed. duced, - - - $25000 4|| || 64 Commission on same 100 *º- at 24 per cent - gº 625 | 98.40 || 25625.00 | $1.60 interest. - 100. face of assumed Amount to be obtain- $26041.67 Ans. note. ed from bank, - - $25625 $98.40 proceeds. In discounting the above note, the following figures would be produced: Face of note, 3- - - - tº * = gº * - º tº $26041.67 CHARGES OR, DEDUCTIONS. Interest on face of note for 64 days at 9 per cent, . º º 416.67 $25625.00 1024 || $100 25025 $25000 net proceeds required. For further work of a kindred nature to the foregoing, see Domestic Exchange. COMPLEX AND EXPERT INTEREST PROBLEMS. 1155. 1. A planter borrows $3040, and agrees to pay at the close of each year $380 for ten years. What rate of interest does he pay? Ans. 24%. OPERATION. | $760 100 25% for 10 years. $380 for 10 years = $3800 paid. 3040 $3800 – $3040 = $760 interest paid in 10 years. -º-º-º- 25 + 10 = 24% for one year. EQUAL ANNUAL PAYMENTS OF PRINCIPAL AND INTEREST. 1156. 2. Bought a piece of land for $5000.00 agreeing to pay 8 per cent interest, and to pay principal and interest in five equal annual payments, how much is the annual payment 3 *. Ans. $1252.28. OPERATION. $5000 - $3.9927 = $1252.28. Explanation.—By the conditions of the problem, we observe that the $5000 is the present worth of an annuity, the time being 5 years and the rate per cent 8. Hence, as the present worth of $1, multiplied by the annuity, would give the full present worth, it is clear that if we divide jºr CASH NOTES. 589 the given present worth ($5000) by the present worth of $1, for the given time and rate per cent, the quotient will be the required annuity. By referring to the annuity table, we find the present value of $1 annuity per annum at compound interest for 5 years at 8 per cent to be $3.9927. Then by dividing $5000 by $3.9927, we obtain $1252.28 as the equal annual payment. The reasoning is, since $3.9927 (present value of $1 annuity, for 5 years at 8 per cent) require $1 investment, then $1 present value, etc. will require the 3,9927th part of $1, and $5000 present value, will require 5000 times as much. To obtain the present value of $1 annuity, per annum, at compound interest for 5 years at 8 per cent, without access to annuity tables, first find the compound amount of $1 for 5 years at 8 per cent. Thus: FIRST3 SECONIO 3 $100 Then add 1.08 -***m- $100 1st, $108.00 108 1.08 116.64 -**- 125.9712 2d, $116,6400 136.048896 1.08 = *-* **- - $586.660096 = final value of $100, annuity for 5 3d, $125.971200 years at 8 per cent compound 1.08 interest. Divide by 100 = -mºmsm--sºms $5.8666 -i-, final value of $1. 4th, $136.04889600 1.08 5th, $146,9328076800 = compound amount of $100, which divided by $100 = $1.4693280768. The $5.8666 -- 1.4693280768 = $3.9927 present value of $1 annuity per annum at compound interest for 5 years at 8 per cent. A DIFFICULT PRACTICAL QUESTION IN RENEWING NOTES. 1157. 1. Jones has a note for $4000 in bank which falls due August 19, 1895. It is agreed that he shall pay the bank $2000 cash and a new 90 day note for what may be due. The bank is to deduct the interest or discount on the new note at 8 per cent for 94 days, from the $2000 cash paid, apply the remainder on the $4000 due, and take the new note for the balance. What is the interest on the new note, how much will the cash pay on the old Inote, and what is the face of the new note % Ans. $ 42.67 is the interest on the new note. 1957.33 is the amount paid on the old note. 2042.67 is the face of the new note. OPERATION TO FIND THE FACE OF THE NEW NOTE, For the explanation, see “Cash Notes,” page 582. 4500 4406 || 2000 $2042.67 face of new note. By the conditions of the transaction and in accordance with bank custom, the interest or discount on the renewal note must be paid in advance, and the Sum required to pay it is to be taken out of the $2000 cash paid. By the above opera- 590 soul E's PHILOSOPHIC PRACTICAL MATHEMATICS. * tion, which involves the principles of cash notes, we see that the face of the renewal note is $2042.67; and consequently, the discount is $42.67. This $42.67 discount subtracted from $2000 cash paid, leaves $1957.33 to be credited on the old note. 2. A man owes $700 now due, and it is agreed that he shall pay $400 cash and give his note for 1 year, without grace or discount day, for the balance that may be due; the discount on the note at 6 per cent to be paid in advance out of the $400 cash paid. How much of the $400 cash is to be credited on the $700 and what is the face of the renewal note % Ans. $380.57 cash to credit on $700 debt. $319.43 face of note. NOTE.—This note is discounted for 365 days. TO FIND THE VALUE OF NOTES BEARING INTEREST AND MATUR- ING IN SUCCESSIVE AND CONSECUTIVE MONTEIS. 1158. April 8, 1894, a party purchased from a real estate holder a house and lot for $7500, on the following terms and conditions: $300 was paid in cash. 36 notes of $200 each bearing 7 per cent interest were given; three of the notes were made payable in 3 months, July 8, 1894. The other 33 notes were made payable monthly after July 8, 1894. After the three notes payable July 8, and 19 notes payable monthly after July 8 had been paid, the holder sold the 14 remaining notes at 7 per cent per annum discount. Counting 30 days to the month without grace or discount day, what were the proceeds of the 14 notes, Bankers' Piº * AnS. $3136.74. SOLUTION. $7500 cost of house and lot, less $300 paid in cash = $7200 which was paid in 36 notes of $200 each. The 3 notes paid July 8, and the 19 notes paid monthly in successive payments after July 8, make 22 notes which deducted from the 36 notes leave 14 notes on hand, 19 months after July 8, which is February 8, 1896. Each of the 14 notes of $200 has accrued interest at 7 per cent from April 8, 1894, and each will continue to bear interest until paid. And as they fall due respectively and successively monthly after February 8, 1896, the following state- ment and operation shows the maturity value of the 14 notes, 23 to 36 inclusive: QC # F É Maturi # § = | Face #| *ºrneº || Maturi Date *| | Dis. 's Nº. Date. Matures. 2 *: º Discounted. Matures. # É Count. Proceeds. 3. #| cent. 3| 3 # =|3 23| $200 April 8, 1894 Mar. 8, 1896 s $26.83; $226.83%|Feb. 8, 1896 Mar. 8, 1896, 1||7| $1.32 $225.51 24 “ 8, 1894 Apr. 8, 189624. 28.00 228.00 || “ 8, 1896 Apr. 8, 1896 2.7 2.66 225.34 25 “ 8, 1894 May 8, 1896.25, 29.16# 229.16; “ 8, 1896 May 8, 1896 3|7 4.01] 225.16 2 “ 8, 1894 June 8, 189626. 30.33: 230.33}| “ 8, 1896June 8, 1896 4|7 5.37| 224.96 27 “ 8, 1894 July 8, 189627, 31.50 231.50 “ 8, 1896July 8, 1896 5||7 6.75 224.75 28 “ 8, 1894 Aug. 8, 1896.28. 32.66; 232.66; “ 8, 1896 Aug. 8, 1896 6||7 || 8.14, 224.53 29 “ 8, 1894 Sep. 8, 1896.29, 33.83%. 233.83}| “ 8, 1896 Sep. 8, 1896 7|7 9.53 224.30 30 “ 8, 1894 Oct. 8, 1896.30 35.00 235.00 || “ 8, 1896 Oct. 8, 1896 8|7| 10.97 224.03 31 “ 8, 1894 Nov. 8, 1896.31] 36.16; 236.16; “ 8, 1896 Nov. 8, 1896 9| 7 | 12.40 223.77 32 “ 8, 1894 Dec. 8, 1896.32. 37.33; 237.33}| “ 8, 1896 Dec. 8, 1896,10|7| 13.84 223.49 33 “ 8, 1894 Jan. 8, 1897|33 38.50 || 238.50 “ 8, 1896Jan. 8, 1897|11||7| 15.30 223.20 34 “ 8, 1894 Feb. 8, 189734, 39.663] 239.663 “ 8, 1896 Feb. 8, 1897|12| 7 | 16.78 222.89 35 “ 8, 1894 Mar. 8, 1897|35 40.83; 240.83; “ 8, 1896 Mar. 8, 1897|13|7| 18.26 222.57 36 7| 19.76) 222.24 “ 8, 1894 Apr. 8, 1897|36 42.00 242.00 “ 8, * 8, 1897|14 2800 = face of the 14 notes. sists೑.8% $145.09 $3136.74 VALUE OF STOCK. 591 This statement.and operation show the following results: 1. The face of the 14 notes $2800. 2. The maturity accrued interest on each and all the notes. 3. The maturity value of each and all the notes. 4. The discount on each and all !. : 5. The net proceeds or present value, February 8, 1896, of each and all iſle Il O'DeS. TO FIND THE WALUE OF STOCK WHICH PAYS A SPECIFIED MONTHLY DIVIDEND OR REVENUE, AND HAS A SPECIFIED TIME TO RUN. 1159. 1. A party holds $2700 of stock, which pays $80 per month revenue, (353% interest per annum). The stock has 27 years to run. What is the present value of the stock the current rate of interest being 6%, allowing 6% interest on all the monthly payments of the revenue or income 3 Ans. $12614,02 A SOLUTION. OPERATION Face of Stock º - - - - - $2700.00 Receipts by monthly payments of $80 or To find the sum of the terms of the monthly interest for 27 years at 353% 25920.00 the monthly payments. Interest on $80 monthly payments on 1 + 323 = 324 $2700, for 52326 terms of 1 mºnº, (1 20930.40 324 - 2 = 162 to 323 inclusive) = 43603 yrs, at 6% - 30. 162 × 323 = 52326 Value of stock at maturity - - $49550.40 VALUE OF $100 STOCK FOR 27 YEARS AT 6 PER CENT. Face of Stock assumed. sº -> - tº-e - mº - - $100.00 100 Interest on $100 for 27 years at 6 per cent - s - - 162.00 392.82 49550.40 Interest on 50c., the monthly interest on $100 at 6 per cent, for 52326 terms of 1 month = 4360+ years at 6 per cent, - 130.82 $12614.02 Ans. $392.82 2. Suppose in the above problem, interest is allowed only on the annual pay- ments of the monthly revenue instead of the monthly payments, what would be the present value of the stock # Ans. $12575,34. SOLUTION. OPERATION Face of Stock for 27 vears at 3 § - - - $ 2700.00 find th f the t f Monthly interest for 27 years at 355 per cent To ". sº º O or at $80 per month º - ºn tº - 25920.00 Interest on $960 yearly payments made by 1 to 26 monthly installments on $2700 for 351 terms 1 or, 27 of 1 year, (1 to 26) at 6 per cent, value of 2 ) 27 13 stock at maturity, - - - - - - 20217.60 13} 351 $48837.60 26 351 VALUE OF $100 FOR 27 YEARS AT 6 PER CENT. Face of stock assumed, - - - - - - - - $100.00 100 Interest on $100 for 27 years at 6 per cent, - º - - 162.00 388.36 || 48837.60 Interest on $6, the yearly interest on $100 for 351 terms of 1 year at 6 per cent, am me an as ºn tº me - 126.36 $12575.34 Ans. Value of $100, - - - - - - - - - - $388.36 NoTE.—See Stocks and Bonds in this work for extended operations of a similar character. 592 soule's PHILOSOPHIC PRACTICAL MATHEMATICS. fºr TO FIND THE RATE OF INTEREST RECEIVED AND PAID IN THE SALE OF PROPERTY ON THE INSTALLMENT PLAN. 1160. A Homestead Association sold to Levin Cooper Soulé, a house and lot for $5000, on the following conditions of payment: Cash $1000; the remainder in 80 notes of $50 each, bearing 7 per cent interest and payable monthly, as the months expire, in 80 consecutive months. In this transaction, what is the rate per cent interest that the homestead makes and what per cent interest does Levin Cooper Soulé pay on the $4000? Ans. The Homestead makes and Levin Cooper Soulé loses, 3.543% + 3.453% = 7% + .5512+ 9% use of inter- est on interest payments, = 7.5512+ 7. SOLUTION.—First Step. Since the 80 notes bear 7 per cent interest and are payable in consecutive monthly periods as the months successively expire, the interest thereon is obtained by finding the interest on $50 at 7 per cent for the whole number of months that all the notes bear interest. This we find as follows: The notes mature successively in 1 to 80 months, thus forming an arithmetical series of 80 terms; hence the sum of the first and last terms, multiplied by half the number of terms, gives the total number of months that all the notes drew interest. Thus, 1 + 80 = 81 × 40 = 3240 months. . FIRST OPERATION SECOND OPERATION To find the interest. To find the interest. $ Int, on first note for 1 mo. is - $ .29# 50 Int, on last note for 80 mos. is - 23.33% 12 7 3240 Int, on first and last notes is - $23.62% | $ 945 interest on all the notes. $23,624 — 2 = $11.84% average interest per month. 11.844 × 80 months = $945 interest on all the notes. Not E. –This $945 interest was received on the $4000 during the 80 months. Second Step. STATEMENT REMARK.—The Homestead Association To find the rate per cent interest on the $4000. receives only 3.543% interest . On the w ſº $4000 but it had the use of the monthly 12 | 80 266; | 945 payments of the monthly maturing notes tºms — & ſº sº | $266; 3.543% Ans. with the accrued interest, thus making Lº wº an arithmetical series of 79 terms of note See page 599, for an explanation of º this kind of work. and interest payments. YY RATE OF INTEREST RECEIVED AND PAID, 593 Third Step. To determine the worth to the Homestead Association of the 79 note pay- ments at 7 per cent interest, we proceed as follows: The 79 note payments form an arithmetical series of 79 terms; therefore, as above explained, the sum of the first and last terms, multiplied by half the number of terms, gives the total number of months that the Homestead received interest on the $50 monthly note payments. Thus, 79 + 1 = 80, the sum of the number of terms. 80 × 39% = 3160 months that the Homestead had the use of $50. NOTE.-39% is # of the number of terms. The interest on $50 for 3160 months at 7 per cent is (******) $9213. This sum being the interest worth of the 79 note payments, during the 80 months, it is as justly to be credited to the receipts of the $4000 as is the $945 interest on the monthly maturing notes. The rate per cent interest that the $921; is of $4000 for the 80 months, worked as in the above problem to find the rate per cent $945 was of $4000, is 3.453 per cent, which added to 3.543 per cent obtained above by the interest received, = 7 per cent the Homestead gains on the $4000, exclusive of the Interest on the interest of the 79 monthly payments of interest received. Fourth Step. To include the interest on the interest of the 79 successive monthly payments of accrued interest as gain for the Homestead, we proceed as follows: The payment of interest on the first $50 note for one month at 7 per cent was 29% cents which was used by the Homestead for 79 months; the interest on the second note was 584 cents which was used by the Homestead for 78 months; thus the interest increases in an arithmetical ratio of 29% cents, and the time decreases by an arithmetical ratio of 1 month. The following statement shows in detail the interest on 79 accrued interest monthly payments: .29% × 79 = 23.04% 6,413 × 58 = 372.16; 12.54 × 37 = 464.04% | 18.66% x 16 = 298.66% .584 × 78 = 45.50 6.703 × 57 = 382.37} | 12.83; x 36 = 462.00 | 18.95% x 15 = 284.374 .874 × 77 = 67.37+ 7.00 × 56 = 392, 13.124 × 35 = 459.37} | 19.25 × 14 = 269.50 1.163 x 76 = 88.66; 7.29% x 55 == 401.04% | 13.413 × 34 = 456.16# | 19.54% x 13 = 254.04% 1.45; x 75 = 109.37% 7.584 × 54 = 409.50 13.703 × 33 = 452.37} | 19.83% x 12 = 238.00 1.75 × 74 = 129.50 7.873 × 53 = 417.374 14.00 × 32 = 448.00 20.12% x 11 = 221.37% 2.04% x 73 = 149.04% 8.163 x 52 = 424,66; 14.294 × 31 = 443.04% 20,413 × 10 = 204.16% 2.33} x 72 = 168. 8,453 x 51 = 431.374 || 14.58; x 30 = 437.50 | 20.703 × 9 = 186.37% 2.624 × 71 = 186.37% 8.75 × 50 = 437.50 | 14.874 × 29 = 431.37} | 21.00 × 8 = 168.00 2.91; x 70 = 204.16# 9.04% x 49 = 443.04% 15.163 x 28 = 424.66% 21.29% X 7 = 149,04% 3 203 × 69 = 221.374 9.33% x 48 = 448. 15.455 x 27 = 417.37} | 21.58+ x 6 = 129.50 3.50 × 68 = 238. 9.623 x 47 = 452,37} | 15.75 × 26 = 409.50 21.87% x 5 = 109.37} 3.79% × 67 = 254.04% 9.913 × 46 = 456.16# | 16.04% × 25 = 401.04% 22.163 x 4 = 88.66% 4.08% x 66 = 269.50 10.203 × 45 = 459,37} | 16.33+ x 24 = 392.00 22.45% x 3 = 67.37% 4.374 × 65 = 284.374 || 10.50 × 44 = 462.00 | 16.624 × 23 = 382.37% 22.75 × 2 = 45.50 4.663 × 64 = 298.66; 10.79% x 43 = 464.04% | 16.91; x 22 = 372.16# 23.04 × 1 = 23.04% 4.955 x 63 = 312.373 || 11.08; x 42 = 465.50 || 17.20; x 21 = 361.37% 23.33.4 × 0 = 00.00 5.25 × 62 = 325.50 11.374 × 41 = 466.374 17.50 × 20 = 350.00 —— 5.543 x 61 = 338.04% 11,663 × 40 = 466.66; 17.79; × 19 = 338.04% |Tot. products, $24885.00 5.83% x 60 = 350. 11.953 × 39 = 466.37} | 18.08% x 18 = 325.50 6.124 × 59 = 361.374 / 12.25 × 38 = 465.50 | 18.374 x 17 = 312.37% By multiplying each interest payment by the time it has to run, we have in the product the sum upon which to compute the interest for one month to equal the interest on the 79 interest payments for the 79 different periods of time. 594 soule's PHILOSOPHIC PRACTICAL MATHEMATICs. * REMARK.—See Article 1161, following this solution for an abbreviated method of finding the interest that would accrue on the interest payments made on a Specified number of equal installments payable at successive and consecutive periods. Fifth Step. By the above statement we find there was $945 interest paid, thus: 29, 4- 23.33% = 23.62% × (80 + 2). We also find that the interest on the interest of the 79 accrued interest payments amounts to $145,164, which is equal to .5512+ per cent on $4000 for 79 months, as per the following operations: OPERATION OPERATION To find the interest on $24885 To find the per cent that or thus: for 1 month at 7 per cent. $145.16+ is of $4000 for $ 79 months, $145.164 - 263; 1% 248.85 40.00 = .5512+ ſº. 263% 145.16# 12 || 7 12 || 79 100 *= | s= summ-ass- – $145.164 interest. $263; This .5512+ 76 added to the 7 per cent that the Homestead gains, as shown in the first part of the Solution, gives 7.5512+ 7, that the Homestead realizes on the $4000 during the 80 months, the use of money being worth 7 per cent. Siarth Step. The buyer pays 7 per cent interest on the notes as they mature. Thus he pays 29% cents on the first note and $23.33% on last note, the sum of which is $23,623. This $23.62% -- 2 = $11,814 average monthly interest. $11.814 × 80 mos. = $945 interest paid by the buyer on the $4000. This is, as shown in the first part of the work, second step, 3.543 per cent. By the monthly payments the buyer loses the use of the 79 successive monthly payments of $50 and the value of such use as shown above in the third step, is $921; which = 3.453 per cent on the $4000. Then 3.543 + 3.453 = 7 per cent interest that he pays. .* The buyer also loses the use (not the payment) of the interest on the interest of the 79 successive monthly payments, which, as shown in the fifth step, amounts to $145.16%. This amount, as shown in the fifth step, is equal to .5512+ 7% on the $4000 for 79 months. Hence the buyer loses, altogether, 3.543% + 3.455% + .5512+ 7% = 7.5512+ 76 on the $4000 for 79 months, the use of money being worth 7 per cent. & TO FIND THE AMOUNT OF INTEREST THAT WOULD ACCRUE ON THE INTERESTS RECEIVED ON THE SERIAL OR PERIODICAL PAYMENTS OF A GIVEN AMOUNT DIVIDED UP INTO A CERTAIN NUMBER OF EQUAL INSTALLMENTS, AND PAYABLE AT THE END OF SUCCESSIVE CONSECUTIVE PERIODS, 1161. The method of finding the sum of the payments and their regular interests having been given in full in the preceding problem, we present the fol. Iowing shorter method of finding the amount of the interest on the Interest pay 4× INTEREST ON INTEREST PAYMENTS. 595. ments, supposing each original interest payment to be invested at simple interest from the time paid until the maturity of the last payment. The first interest payment will bear interest for one month less than the whole number of months; hence, if we let A represent the interest on the first pay- ment for one month, and let M equal the remaining number of months, we will have the following series and their summation to represent the successive interests, and their total aggregate or final sum. Diagram for the summation of the compounding of the successive increasing monthly inter. ests, with the successive decreasing monthly periods, INTEREST.S. 1st month = a, X m = am — 0 2d “ = 2a × m — 1 = 2am — 2a. 3d ‘‘ = 3a × m — 2 = 3am — 6a 4th “ = 4a × m — 3 = 4am — 12a 5th ** = 5a × m — 4 = 5am — 20a 6th ‘‘ = 6a × m — 5 = 6am — 30a 21am — 70a Sum of interests = a (21m – 70) The quantity (21m) is the sum of the arithmetical series from 1 to 6 = (21) multiplied by the number of months m (6) = (21 × 6). And the minus quantity to be subtracted (70), the sum of the last column, is the sum of a compound series or rectangular pyramidal series, and is found for any term, (n) by cubing it, and sub- tracting (n), and dividing the remainder by 3, hence formula (**) or as in above diagram (6°– 6) = (70). 3 Or, it may be found by multiplying the number of terms (n) by one less, and one more than itself as 5 × 6 × 7 and divide their product by 3 = 70, which sub- tracted from (21 m) the remainder is the equivalent number of months to be multi- plied by one month’s interest (a), which will give the amount on which to calculate the interest for one month, at the given rate. When a number of payments have been made to find the final Maturity Value of the interest on the remaining payments, proceed as follows: First find the value of the interests for the whole period by the method just given; then find the value of that part of the series that has been paid, and subtract the last from the first. To find the Present Value of the unmatured part of a series, find the final value as above, and discount the amount by the customary methods. The usual way would be by the principles of Bank Discount. A more ethical way would be to add the accrued simple interest on the unmatured notes to their face value. CONDENSED METHODS FOR FINDING THE MATURITY VALUE OF TEIE INTEREST ON THE INTERESTS OF A SERIES OF PERIODICAL NOTES OR PAYMENTS. 1162. First.—Find the simple interest on the first payment for one month, or period, at the given rate per cent, and call it (a) or 1st result. Second.—Find the sum of the arithmetical series from one to the number of periods (s) and multiply it 596 soul E's PHILOSOPHIC PRACTICAL MATHEMATICS. † by the number of months m = (sm) or 2nd result. Third.—Find the sum of a com- pound series by multiplying the number of terms (n) by one less and by one more than itself, and divide the product by three, or by this formula (n°– n) = 3rd result c. 3 Fourth-Subtract the third result from the second result, and multiply the remainder by (a) or first result, and the product is the sum on which to compute the interest for one month to equal the interest on the interest for the 79 different periods of time. These four operations may be expressed by the following formula : A ( SIOl _nº-n 3 This formula gives the number of dollars, on which to calculate the interest for one month at the given rate; (m) represents the number of months the first months' interest would run, and is therefore one less than the whole number of terms (n) of the original series. To apply these methods to the foregoing problem we proceed as follows: FIRST STEP SECOND STEP THIRD STEP To find the interest on $50 for To find the sum of the arith- || To find the sum of a compound 1 month at 7 per cent. metical series and the prod- Ser JéS $ uct thereof by the months. |80 × 79 × 81 = 511920 – 50 1 + 80 = 81 × 40 = 3240 || 3 = 170640 = c or Third 12 || 7 E SUIT], result. tºmº ºmºmºs 3240 × 79 = 255960 = S, | $.29% int. = a or First m. or Second result. result. FOURTH STEP. Result S. m. 255960 – result c 170640 = 85320 = d or Fourth result. FIFTH STEP. Result d 85320 × result a 29%g = $24885.00, which is the amount upon which to compute the interest for one month, to equal the interest on the 79 interest pay- ments for 79 different periods of time, as shown in the above detailed solution. TRUE DISCOUNT, 1163. True Discount is such a deduction from the face of notes or debts, as is equal to the simple interest on the remainder for the same time and rate for which the deduction was made. For a statement of the difference between True and Bank Discount, see our remarks on Interest and Bank Discount, page 576. 1164. The Present Worth of a note or debt is the sum that remains after the true discount is deducted from the face; or it is the principal which, at the specified rate of interest and time, would amount to the face of the note or debt when it becomes due, For example, if we loan $500 for 1 year at 8 per cent, it will amount to $540. Hence we see that the present worth of $540, due one year hence at 8 per cent, is $500. Yºr TRUE DISCOUNT. 597 The proceeds or cash value of this $540, by the bankers' method of discount is but $496.80, which is $3.20 less than by the true discount system. True discount is used but very little in business. Business men generally use the Merchants' and Bankers' System of Discount. The function or purpose of true discount is properly to determine what sums are to be invested to produce a specified amount at a given rate of interest and for a Stated period of time. And in this respect it is the same as interest where we have the time, rate per cent, and interest or amount given to find the principal. The following problems will fully illustrate the operations of True Discount: PROBLEMS, 1. What is the present worth or cash value, and the true discount of a note of $9000 for 94 days at 8 per cent? Or thus: What sum loaned or placed at interest for 94 days at 8 per cent, will produce $184.15 interest, and amount to $9000. Ans. $8815.85 present worth. $184.15 true discount. OPERATION. $4500 present worth assumed. 4500 $9000 face of note. 94 interest. 4594 || 9000 8815.85 present worth. $4594 amount 94 days hence. | $8815.85 present worth. $ 184.15 true discount. Explanation.—By inspection and reason, we see that as true discount is the interest on the PRESENT worth, we cannot therefore operate on the $9000, the face of the note, and hence we are obliged to assume some number to represent present worth; and to facilitate the operation. We assume $4500, the 8 per cent Interest Divisor to represent the present worth. As explained in Cash Notes, page 582, we assume the Interest Divisor for the reason that the interest thereon is always. as many dollars as there are days in the TIME. Knowing this we are saved the labor of computing the interest on the sum assumed. Accordingly the interest on the $4500 for 94 days at 8 per cent 23. $94; which added to the $4500 gives $4594, as the amount of $4500 assumed present worth due 94 days hence at 8 per cent. º We now observe that as we have used the time and rate given in the problem, there is the same relationship between the $4594 amount and the $4500 of assumed present worth, as there is between the $9000 amount of note and the unknown present worth of the same ; and hence to find the present worth of the $9000 amount, we have but to perform a proportional operatiºn as shown by the second line statement in the solution. The reasoning for this statement is as follows: Since $4594 amount require $4500 present worth, $1 will require the 4594th part, and $9000 Will require. 9000 times as much. SECOND OPERATION By assuming $100 as the present worth. $100 $ 45 || 94 100 cº- 918.80 || 9 | $2.08; = interest. 9000.00 100. = assumed present Worth. smm.sºme == --> $8815.85 present worth. $102.08% = amount 94 days hence. 9000.00 amount of note. smm mºm- $184.15 true discount. 598 SOULE's PHILOSOPHIC PRACTICAL MATHEMATICs. GENERAL DIRECTIONS. 1165. From the foregoing elucidations, we derive the following general direc- tions for calculating True Discount: 1. Assume as the present worth, the Interest Divisor for the given rate per cent and find the amount of the same for the given time and rate per cent. 2. Then divide the assumed present worth multiplied by the given amount, by the amount of the assumed present worth. NOTE.-$100 may be assumed as the present worth, instead of the interest divisor, if preferred. 2. What is the true discount on $3450 for 1 year and 8 months at 9 per cent, Counting 30 days to the month, and not allowing grace? Ans. $450. NOTE.-This problem may be stated as follows: * 3. What sum loaned for 1 year and 8 months, at 9 per cent will produce $450 interest and amount to $3450% - OPERATION INDICATED. $4000 $3450 $4000 4600 || 3450 3000 Or thus: 600 || 450 Cºmº. | -E-ºsmº *- smºº-º- 0. $3000 = present worth. $ 450 = true discount. $3000 Not E. –1 year and 8 months = 20 months = 600 days, and hence the interest on the assumed 9 per cent interest divisor is $600, 4. What sum must be loaned for 2 years at 7 per cent to amount to $8550? o Ans. $7500. 5. What is the present worth of a claim of $2468.45 due in 123 days, money worth 13 per cent per month ? Ans. $2325.44. 6. What principal must be loaned for 183 days to amount to $3915.90, inter- est at 6 per cent? Ans. $3800. 7. A merchant sold $5000 worth of goods on 4 months credit, and then offered the purchaser 3 per cent discount for cash, which was accepted. What was the discount, and how much did it exceed the true discount at 8 per cent? Ans. $150 discount. $129.87 true discount. $20.13 excess. NotE.—In transactions of this kind count 30 days to the month and allow no grace or dus- &ount day. 8. Goods were bought to the amount of $4200 on 3 months' credit when the current rate of interest was 44 per cent. How much cash could be paid for the goods without loss to either party ? - Ans. $4153.28. OPERATION INDICATED, $8000 8090 || 4200 9. A owes B $8000 due in 6 months. B proposes to deduct 5 per cent from the face of the bill for cash. A accepts the proposition and borrows the money at 6 per cent. How much does A gain or lose? Ans. $172.00 A gains. OPERATION INDICATED. $8000 400 = 5 per cent discount. Interest on $7600 for 6 months at 6 per cent is $228 º-º-mºmºmº- $400 — $228 = $172 gain. $7600 TO FIND RATE PER CENT. 599 10. What is the present worth of $4391.68 due in 1 year and 4 months, money Worth 5 per cent, and no allowance for days of grace or discount day? Ans. $4117.20. 11. What is the true discount on $8765.25 at 9 per cent for 495 days? Ans. $965.25. PROBLEMS INVOLVING INTEREST OPERATIONS. 1166. To find the Rate per cent when the Principal, Time, and Interest or Proceeds are given : 1. The interest on $9000 for 94 days is $188; what was the rate per cent? Ans. 8%. OPERATION OPERATION Explanation.—In all problems To find the 1nterest on $9000 To find the rate per cent. ** d *: * º: P. for the given time at 1 per assume some rate to work from, - 1 cent assumed. 23.50 | ić 00 and as the ratio will be the same, o º whatever rate per cent we assume, -- therefore, to facilitate the work, 9000 8% Ans. we assume 1 per cent, then, in 360 94 order to obtain a sum of interest *: the same º - to the 1 per cent assumed, that $23.50 interest. the $188 bears to the required rate per cent, we calculate the lnterest on the principal for the given time at 1 per cent. Accordingly we find, in this problem, $23.50 interest, We then reason as follows: Since 1 per cent gives $23.50 interest, by transposition $23.50 required 1 per cent; and since $23.50 interest required 1 per cent, 1 cent interest will require the 2350th part and 18800c interest will require 18800 times as much. – º-m-m-m- GENERAL DIRECTIONS. 1167. From the foregoing elucidations, we derive the following general direc- tions for finding the rate per cent: 1. Assume 1 per cent and find the interest on the principal for the given time at the 1 per cent. 2. Then divide the given interest by the 1 per cent interest on the principal for the given time. NOTE.- When the proceeds are given, first subtract the same from the principal or face of note, and thus find the interest or discount PROBLEMS. 2. The interest on $1000 for 288 days was $48. What was the rate per cent? Ans. 6%. 3. A merchant loaned $1580 for 120 days and received for its use $52,663; at what rate per cent did he loan it? Ans. 10%. 4. A note for $524.80 was discounted for 47 days and $518.6336 proceeds received. At what rate per cent was it discounted ? Ans. 9%. 5. A received $3679.20 for his note of $4200 which had 186 days to run. At what rate per cent was it discounted ? Ans. 24%. 6OO SOULE's PHILOSOPHIC PRACTICAL MATHEMATICs. ºr 6. A note for $7543.80 was discounted for 34 days and $7401.306 proceeds received. At what rate per cent was it discounted ? Ans. 20%. 7. A merchant invested in goods $14000 and commenced the merchandising business. He continued 8 months, and then from his books he found that his gains on sales were $5215, and his store expenses of all kinds, $2705. What was the rate per cent per annum gain; what was his gain per cent on investment; and how much more did he gain by his business than if he had loaned his money at 10 per cent interest and received $125 per month for his services? Ans. 17}} % gain on investment. 26%; % gain per annum. $576.66% gain on his business in exeess of what he would have gained by loaning his money at 10 per cent and serving others at $125 per month. OPERATION INDICATED. Gain on sales, * → * * $5215 $2510 Expense, sº tº * * * 2705 14000 || 100 Net gain, tº tº e º $2510 | 17++% gain on investment. (1743% x 12) - 8 = 26% 7% gain per annum. Interest on $14000, 8 mos. at 10% = $933,-- salary 8 mos. at $125 = $1000, = $1933}. Net gain of business $2510 – $1933% = $5763. TO FIND THE PRINCIPAL WHEN THE RATE PER CENT, TIME, AND INTEREST OR AMOUNT OR PROCEEDS ARE GIVEN. 1168. 1. What principal loaned for 94 days at 8 per cent will produce $188 interest ? Ans. $9000. FIRST OPERATION. $4500 assumed principal. 94 | 188 | $9000 principal, Ans. • SECOND OPERATION. OPERATION OPERATION Explanation.-In all prob- lems of this kind, we as- To find the interest on the To find the principal. ..". º Interest º incipal. or $100 as principal, an assumed principal $ then calculate the interest 100 thereon for 1. #". rate tº e and time. This is done in 100 assumed principal. 18.80 || 9 order to produce an interest 45 | 94 188.00 which bears the same re- * &ºm=s=mº lationship to the assumed 2.083 i I’OSU, O principal that the given in- $ 08; interest $9000 Ans terest bears to the required principal that produced it. In the first operation we assumed the 8 per cent Interest Divisor, $4500, as the principal; and Seeing that the interest on $4500 for the given time and rate per cent is $94, we reason thus: Since $4500 principal gives $94 interest, conversely $94 interest required $4500 principal. And since $94 Yºr INTEREST To FIND THE PRINCIPAL. 6OI interest required $4500 principal $1 interest will require the 94th part, and $188interest will require 188 times as much. In the second solution we found the interest on the assumed principal to be $2,083, Having this interest, we make the proportional statement, reasoning as follows: Since $100 principal gives $2,085 interest, conversely $2.08; interest required $100 principal. And sincel c. ($2.08; reduced) interest require $100 principal, &c. interest will require the 1880th part, and # or a whole cent will require 9 times as much; and since 1c. interest requires the result of the statement now made, 18800c. ($188 reduced to cents) will require 18800 times as much. - NoTE.—We much prefer the first operation, for the reason that the interest on the Interest Divisor, for the time and at the rate percent, will always be the same as the number of days. And hence, this being known, we save making one interest calculation in the operation. 2. What principal loaned for 121 days at 10 per cent will amount to $7442? Ans. $7.200. OPERATION. $ Explanation.—When the amount is 3600 = assumed principal. given, instead of the interest, then Amount = 3721 7442 = amount. add the interest on the assumed princi- ! — pal to the assumed principal and then | $7200, Ans. make the proportional statement. 3. What principal discounted for 369 days at 8 per cent will give $1836 proceeds? Ans. $2000. OPERATION. $ ſºlº º º == & © are given, instead of the interest, then Proceeds = 4131 § - tº: principal. subtract the interest on the assumed I’OCCCCHS = = proceeds. principal from the assumed principal -º-, - and then make the proportional state- $2000, Ans. ment. GENERAL DIRECTIONS. 1169. From the foregoing elucidations, we derive the following general direc- tions for finding the principal : 1. Assume $100 or the Interest Divisor as principal, and calculate thereon the interest for the given time and rate per cent. 2. Then divide the assumed principal multiplied by the given interest, by the interest on the assumed principal. NOTE.—If the amount is given instead of the interest, then add the interest on the assumed principal to the assumed principal, and divide the sum into the product of the assumed principal and the given amount. If the proceeds are given instead of the interest, then subtract the interest on the assumed principal from the assumed principal, and divide the difference 1nto the product of the assumed principal and the given proceeds. PROBLEMS. 4. A banker loaned a sum of money for 369 days at 8 per cent and received $164 interest. What was the sum loaned ? Ans. $2000. 5. A merchant loaned a certain sum for 1 year and 3 months at 5 per cent, and received $750 interest. What was the principal loaned ? Ans. $12000. 6. What principal loaned for 90 days at 6 per cent will amount to $2030% Ans. $2000. 6O2 soul.E's PHILOSOPHIC PRACTICAL MATHEMATICs. º: 7. Discounted a note for 183 days at 5 per cent and received $1637.30 pro- ceeds. What was the face of the note? Ans. $1680. 8. What principal loaned for 64 days at 8 per cent will amount to $1369.20% Ans. $1350. 9. A merchant discounted a note for 64 days at 5 per cent and received $178.40 proceeds. What was the face of the note * Ans. $180. 10. A father desires to place his son in college for a four years' course of study. The yearly expense for board, tuition, etc., is $400, and for his clothes, etc., he allows him $150 per year. He now wishes to know what sum of money he must place at interest at 5 per cent, so that the interest will just pay the yearly expense of his son 3 Ans. $11000. 11. What principal loaned for 124 days at 7 per cent will produce $130.20 interest ? Ans. $5400. OPERATION y 36000 = 1% int. divisor as assumed prin. Explanation.—Since there is no 868.00 130.20 7 per cent Interest Divisor, we assume as principal 36000, the 1 per cent Interest Divisor, and then $5400, Ans. multiply the interest at 1 per cent, which is equal in dollars to the number of days, by 7, the rate per cent, and thus produce $868.00 interest. The solution statement is then made as in the preceding problems. 12. What principal loaned for 72 days at 11 per cent will produce $33 interest ? Ans. $1500. TO FIND THE TIME, WHEN THE PRINCIPAL, RATE PER CENT, AND INTEREST, OR WHEN THE PRINCIPAL AND THE AMOUNT OR PROCEEDS AND THE RATE PER CENT ARE GIVEN. PROBLEMIS. 1170. 1. A note for $9000 was discounted at 8 per cent, and the interest amounted to $188. For how many days was it discounted ? Ans. 94 days. 1 year or 360 days assumed. OPERATION OPERATION To find the interest on the $9000 at 8 per cent To find the number of days. for 1 year assumed. ds. $90.00 principal. 360 8% interest. 720 | 188 $720.00 interest for 360 days. 94 days, Ans. Explanation.—In all problems of this kind, we first assume 360 days, or 1 year. Then, in order to obtain a sum of interest that bears the same relationship to the 360 days of assumed time that the $188 given interest bears to the required time, we calculate the interest, on the principal loaned for the assumed time at the given per cent. Accordingly, we produce, in this problem, $720 interest. We then reason as follows: Since 1 year's time, with the given principal and rate, giye $720 interest, by transposition $720 interest required 1 year's or 360 days’ time; and since $720 interest required 360 days' time, $1 interest will require the 720th part, and $188 interest will require 188 times as many, which is 94 days. * Nºrz-In case the interest is computed at 365 days to the year, then 365 would be the days tº SSUlſſle Ol. ºr INTEREST TO FIND THE TIME, 603 2. A merchant borrowed $1850 at 8 per cent, and paid $197,334 for its use. How long did he have the money? Ans. 1 year and 4 months. OPERATION INDICATED. 1 year or 360 days assumed. $18.50 Y. DS. 8% | 1. 360 tº Eºmº 148 148 || 592 $148.00 interest. 3 592 Or, 3 480 ds. = 1 yr. 4 mos. 14 yrs. = 1 yr. 4 mos. GENERAL DIRECTIONS. 1171. From the foregoing elucidations, we derive the following general direc- tions for finding the time: 1. Assume 360 days, 1 year, and find the interest on the given principal at #3 given rate per cent and for the assumed time. J.-, ºr 2. Then divide the given interest multiplied by the assumed time, by the iº on the principal for the rate per cent and assumed time. NOTE: 1.--When the amount is given, first subtract the interest from the same to find the principal. NOTE 2.--When the proceeds are given, first add the interest to the proceeds to find the principal. NOTE 3.—In case the interest or dišcount has been computed at 365 days to the year, then assume 365 days. PROBLEMIS. 3. A note for $6000 was discounted at 5 per cent and the interest or discount was $78.33%. For how many days was it discounted ? Ans. 94 days. 4. A merchant borrowed $2500 at 44 per cent, and paid $38.75 for its use. How long did he have the money? Ans. 124 days. 5. Loaned a sum of money at 8 per cent until it amounted to $461.37. The interest was $6.37. How long was it loaned ? Ans. 63 days. 6. A note was discounted at 10 per cent and $800 proceeds received. The discount was $200. For what time was the note discounted ? $ Ans. 720 days, or 2 years. 7. A broker loaned $50000 at 12 per cent, and received $550 interest for it. How long did he lend it? Ans. 33 days. 8. How long will it take for $2000 to amount to $3000 at 10 per cent interest? Ans. 5 years. 9. A note dated November 1, 1895, was discounted December 23, 1895, at 6 per cent. The face of the note was $6231.50 and the proceeds $6187.8795. Grace and discount day were counted. For how many days was it discounted and how long did it run after being discounted ? Ans. 42 days discounted. 41 days to run after discounted not including discount day. OPERATION INDICATED. $6231.50 DS. 6 360 373.89 || 43.6205 $ 373,8900 42 days. 6O4 SOULE's PHILOSOPHIC PRACTICAL MATHEMATICs. * 10. A note for $9730 bearing 8 per cent interest dated November 24, 1895, payable in one year, was discounted at 12 per cent, the proceeds being $9883.9934. What was the time and the date of discount, allowing grace and discount day? Ans. 180 days time. June 1, 1896 date discounted. OPERATION INDICATED. $ 1 year, or 360 days assumed. 9730. $10514,8866-1- 45 || 363 12% $ 784.8866–1– interest. $ 1261.786392 interest for 1 year. 9730.00 $10514.8866–1– maturity value. 9883.9934 proceeds. $ 630,8932 discount. 180 = days the note was discounted NOTE. —365 days —H 3 days of grace and 1 discount day = 369 days had the note been dis. counted November 24th, 369 – 180 = 189 that the note was discounted after November 24th, which is June 1, 1896. See Time Table, page 298 1172. T A. B. L TE SHOWING IN HOW MANY YEARS A GIVEN PRINCIPAL WILL DOUBLE ITSELF. ds. 1261.7863 | 360 = 1 year. 630.8932 e AT COMPOUND INTEREST. | COMPOUND INTEREST ON 1 DOLLAR FOR 100 YEARS. IRATE At Simple – | & Compounded Interest. cººl yº, º Amount. || Years. | Per Cent. Accumulation. 96. YEARS. YEARS. YEARS. YEARS. $1 100 1 $ 2.75 1 100. 69.666 69,487 69.400 1 100 2 7.25 1} 66.66 46.556 46.382 46.298 1. 100 2} 11.75 2 50.00 35.004 34.830 34.743 1 100 3 19.25 2} 40.00 28.071 27.899 27.812 1 100 3# 31.25 3 33.33 23.450 23.278 23.191 1 100 4. 50.50 3# 28.57 20.150 19.977 | 19.890 1. 100 4% 81.25 4 25.00 17.673 17.502 17,415 1 100 5 131.50 4} 22.22 15,748 15.576 15.490 1 100 6 340.00 5 20.00 14.207 14.036 13.946 1 100 7 868.00 5% 18.18 12.946 12.775 12,686 1 100 8 2,203.00 6 16.67 11.896 11.725 11.639 1 100 9 5,543.00 6# 15.38 11.007 10.836 10,750 1 100 10 13,809.00 7 14.29 10.245 10.075 9,989 1. 100 12 84,675.00 73 13.33 9.585 9.914 9.328 1 100 15 1,174,405.00 8 12.50 9.006 8.837 8.751 1. 100 18 15,145,007.00 8} 11,76 8 497 8.346 8.241 1 100 24 2,551,799,404.09 9 11.11 8.043 7.874 7,788 1 100 9} 10.50 7.638 7.468 7.383 10 10.00 7.273 7.121 7,026 12 8.34 6.110 DIFFERENCE BETWEEN BANK DISCOUNT AND TRUE DISCOUNT, 1173. 1. What are the proceeds or cash value of $8000 for 64 days at 10 per cent, by the bankers' system of discount, and what is the present worth or cash value by the true discount system of work? Ans. $7857.78, proceeds by bankers' system. $7860.26, present work by true discount. º: BANK AND TRUE D1scount CoMPARED. 605 OPERATIONS. Banking System. True Discount System. $8000 $3600 36 || 64 3664 || 8000 8000. $ 142.22 bank discount. $7857.78 proceeds, $7860.26 present worth. 8000.00 $ 139.74 true discount. 2. What is the bankers' discount and the true discount of $15000 for 75 days at 1 per cent per month, and what is the difference 3 Ans. $375.00, bank discount. $365.85, true discount. $9.15, difference. º 3. What is the difference between the interest on $5000 for 3 years 4 months and 20 days (1220 days) at 6 per cent, and the true discount for the same time and rate # Ans. $171.79. 4. I hold a note of $10000, due in 10 years or 3600 days without grace. How much will I receive for it if I discount it at 10 per cent bank discount, and how much if I discount it at the same rate per cent by the true discount system? Ans. Nothing by bank discount. $5000 by true discount. TO FIND THE RATE PER CENT OF INTEREST PAID, OR THE RATE PER CENT TRUE DISCOUNT, WHEN NOTES, ETC., ARE DISCOUNTED BY BANK CUSTOM. 1174. 1. A banker discounted two notes of $500 each, one for 34 days at 6 per cent, and the other for 240 days at 6 per cent. What was the rate per cent true discount charged on each note 3 500 face of note. 60 || 34 $2.834 interest. $497.163 proceeds. $ 5.00 face of note. 69 || 249 4 $20.00 interest. $480. proceeds. Ans. 64% % on the 34 days' note. 64% on the 240 days' note. OPERATION FOR 34 DAYS. $ 3 || 14.91.50 360 || 34 | .4695% int. on pro- ceeds at 1%. OPERATION FOR 240 DAYS. 4šſ 4 3 369 || 24% 80 $3.20 int. at 1% on proceeds. % 1 8 3.2% | 20.9% 50 64% Ans. 6O6 SouLE's PHILOSOPHIC PRACTICAL MATHEMATICs. º: łºplanation.-In answering these questions we first find, as shown by the first statement, the Proceeds of each note according to the bankers' system of discounting. Then, considering the proceeds as principal, we have the question contained in Problem No. 1, on page 599, which is the principal, time and interest given, to find the rate per cent. To answer this question we first find, as shown in the second statement, the interest on the proceeds of the notes now used as principals, for the respective time at 1 per cent, and then make the proportional statement to find the true discount with this interest, the 1 per cent, and the interest obtained by discounting the notes according to bank custom. The reasoning for the last two statements of each question was fully explained on page 599, and hence we now omit it. NOTE 1–By the result of this work we see that the rate per cent of bank discount is larger than the same rate per cent of true discount, and what is still more important, we see that this increased difference of rate increases in proportion to the time for which the note has to run. This being the case on all notes of the same face and at the same rate, it seems natural to conclude that bankers would prefer long time paper. But the advantages of short credits are generally con- sidered to be more than equivalent for any excess that might be gained by extending the time. The reason for this increase of rate per cent on long time notes or transactions is that, on them the discount becomes larger, and hence the proceeds or principal to which it is referred becomes smaller. NotE 2. —Instead of working on the face of the notes in the above questions, we could as well have assumed $100 or $1000, or better still the Interest Divisor. By working from assumed figures the operation is often simplified, and in questions where no principal or face of note is given we are obliged to work from assumed numbers 2. What is the rate of interest, or rate per cent true discount on a 63 day note at 8 per cent and 10 per cent ? Ans. 8.*.*, 7% true discount, at 8%. 10...'", 9% true discount, at 10%. 3. What is the rate of interest, or rate per cent true discount on a 360 day note at 5, 6, 7, 8, 9, 10 and 12, per cent 3 Ans. 5*, 6}}, 7#3, 843, 934, 11%, 13 F1, per cent. TO FIND THE RATE PER CENT OF TRUE DISCOUNT CORRESPONDING TO A GIVEN RATE OF INTEREST. 1175. 1. If I wish to realize 10 per cent interest on 94 day notes, at what rate per cent is the true discount 3 Ans. 9;###3%. FIRST OPERATION. $3600 assumed present worth or proceeds. $36.94 94 interest on same for 94 days. 360 | 94 0 Int. 1” 9.643; 94.00 interest. 93$$$$ 96 Ans. Explanation.— In this solution we first assume $3600 the Interest Divisor at 10 per cent as the present worth or proceeds, and then add thereto the interest for 94 days at 10 per cent. This operation gives $3694 which is the principal at interest for 94 days at 10 per cent which produced $94 interest. The next step is to compute the interest on the $3694 for 94 days at 1 per cent, which is $9.6443. Having this interest, and the interest on $3600 for 94 days at 10 per cent, we then make the third statement, reasoning thus: Since $9.643; interest require 1 per cent, $1 will require the . $9,6438th part, and $94.00 interest will require 94.00 times as much. SECOND OPERATION By assuming $100 present worth or proceeds. $3694 principal or sum loaned. | $ 9.64% interest at 1%. 0 % 1 162 23.5000 93.3%; 96 Ans. $ $ 100 present worth assumed. 9 | 92350 36 94 360 | 94 43,404 s | Tlºw. 9 $2.61% interest. 26.79%2c. interest. $100. present worth. $102.61 principal. * INTEREST. 6O7 2. What is the corresponding rate of true discount for a 63 day note at 8 per cent ? Ans. 74}} %. IFIRST OPERATION. h $4500 = assumed present worth or proceeds. $ % 63 = interest on same at 8% for 63 days. 45.63 1 ºs-sms me 360 | 63 7.98% | 63,00 $4563 = principal. * | $7.98% interest. 733} % Ans. SECOND OPERATION By assuming $100 as the present worth or proceeds. $ $ % 100 P. W. 1.01,40 1 45 || 63 360 | 63 .3549 || 2 * - 1.4000 $1.40 interest. ,1774; interest. 100. P. W. t ºs-e-mas $101.40 principal. 3. What is the true discount to correspond with 93 day notes at 12 per cent, and with 360 day notes at 15 per cent and at 20 per cent 3 Ans. 11 º', 'ſ, on 93 day note at 12%. 13% 7% on 360 day note at 15%. 163% on 360 day note at 20%. 7#3} % true discount. ANNUAL, SEMI-ANNUAL AND QUARTERLY INTEREST. 1176. Annual, Semi-Annual and Quarterly Interest is the regular simple interest on notes or debts, and the interest on the unpaid simple interest from the time it becomes due until paid. Annual, Semi-Annual and Quarterly Interest is due after it has been accruing for the time specified for the interest payments, except the interest on the interest which is not due till final payment. NOTE. –In some States, a neglect to collect the annual interest on notes drawn, with interest payable annually, is considered as a waiver of the contract to pay annual Interest. It is a principle of law that, in the absence of any agreement, all money or claims bearing interest, or that is due and not paid, shall draw simple interest from the time that they become due until paid. Hence, to entitle a party to annual, semi- annual or quarterly interest, a special agreement to that effect is necessary. In case of notes, this agreement is inserted and becomes one of the obligations of the instrument. Thus, the following note contains the annual interest contract: $6000. ST. LOUIs, September 1st, 1895. Four years after date, for value received, we promise to pay to the order of D. G. HESSEE, Six Thousand Dollars with interest at 8 per cent, payable annually. MONTGOMERY & POTTS. In this note the words in italics make it an annual interest note, by specifying when the interest shall be paid ; and hence, if the interest is not paid at the end of each year, it becomes by virtue of the terms of the instrument, and in accordance with law and equity, a new debt, and will draw simple interest until paid. 6O8 soule's PHILOSOPHIC PRACTICAL MATHEMATICS. The following work will show the operation of annual interest: 1. In the foregoing note nothing being paid until maturity, what will be the amount then due 3 Ans. $8150.40. OPERATION - Explanation.—By the conditions of the note we see that the inter- est was payable at the end of Face of note, $6000.00 (Interest on note for 1 year at 8%, $480.) Interest on note for 4 years at 8%, 1920.00 ...h year, but as it was not paid Interest on $480 for 3 years at 8%, $115.20 until the maturity of the note, Interest on $480 for 2 years at 8%, 76.80 }. º,º: find pººl. 0 º Interest ODI tºle Ia C6. O €. In Ote Interest on $480 for 1 year at 8%, 38,40 230.40 for the 4 years at 8 per cent. We g then observe that the first year's Total amount due at maturity, $8150.40 interest, $480, was due and not paid for 3 years; that the second year's interest, $480, was due and not paid for 2 years; and that the third year's interest, $480, was due and not paid for 1 year; and hence, by the conditions of the contract, and in conformity to law and equity, we find the interest on the three sums of annual interest for the respective time that they were due and not paid. The last year's interest being paid when due, there is no interest to charge thereon. We now add the total annual interest and the simple interest on the annual interest for the time that it was due and not paid, to the face of the note, and thus obtain the amount due at the maturity of the note. A Quarterly Interest Note, CHICAGO, August 10, 1895. Two years after date, for value received, I promise to pay to the order of QUIN & HILL Four Thousand Three Hundred Seventy-two Dollars, with 10 per cent interest payable quarterly. $4372. J. L. CAILLOUET. 2. If nothing is paid on the above note until maturity, what amount will then be due # Ans. $5322.91. OPERATION Face of note, tº- *- - L- es º * - gº tº gº * * gº gº ºn $4372.00 Interest on same for 2 years at 10 per cent, tº • * & gº sº sº tºº º ſº 874.40 (The interest on face of the note for 1 quarter is $109.30.) Interest on 1st quarter's interest, $109.30, for 21 months, E. : gº tºº wº $19.13 Interest on 2d quarter's interest, 109.30, for 18 months, tº ſº tº tº-> 16.40 Interest on 3d quarter's interest, 109.30, for 15 months, tº gº º ºs 13.66 gº Interest on 4th quarter's interest, 109.30, for 12 months, º tº º ſº tº 10.93 Interest on 5th quarter's interest, 109.30, for 9 months, tºº * . * > tº e 8.20 Interest on 6th quarter's interest, 109.30, for 6 months, tº- g- tº tº 5.46 Interest on 7th quarter’s interest, 109.30, for 3 months, tº º tº a gº sº tº 2.73 Interest on 8th quarter's interest, 109.30, for 0 months, º gº ºne º ſº- Amount of interest on quarterly interest, * º sº tº * . º {- gº 76.51 Total amount due at maturity, tº- * . & gº {_º tº wº * * gº * , $5322.91 NotE.—Instead of finding the interest on each quarter's interest separately, we could have added the months of time together, making 84, and then found the interest on 1 quarter's interest for the whole time, 84 months or 7 years. A Semi-Annual Interest Due Bill. $660. NATCHEz, October 15, 1895. Due A. & M. Moses, for value received, Six Hundred and Sixty Dollars, with interest at 6 per cent payable semi-annually. FOWLER & DAVIS. Yºr INTEREST. 609 3. Nothing being paid on the above due bill until the day of settlement, two years 4 months and 20 days after date, what was then due Ans. $760,01. OPERATION. Face of due bill, * * * * * * * * * * * * * * * Interest on same for 2 years, 4 months and 20 days at 6 per cent, * * * * * (The interest on the face of due bill for 6 months is $19.80.) Interest on 1st semi-annual interest, $19.80, for 22 months 20 days. Interest on 2d semi-annual interest, 19.80, for 16 months 20 days. Interest on 3d semi-annual interest, 19.80, for 10 months 20 days. Interest on 4th semi-annual interest, 19.80, for 4 months 20 days. $660.00 94.60 5,41 $760.01 Interest on $19.80 semi-annual interest for 54 months 20 days at 6 per cent = - Total amount due on settlement, * $º - - gº * -> sº -> - COMPOUND INTEREST. 1177. Compound interest is interest in which the principal is increased at the expiration of each period of interest payment by the interest on the principal for such period. The amount thus produced forming a new principal on which the interest is calculated till the next period of interest payment, when the new princi. pal is increased by the addition of the last interest, and so on from one period of interest payment to another, until final settlement is made. The periods of time for the interest payments may be annually, semi-annually, quarterly, monthly, weekly, or daily. There is no law forbidding the charging and receiving of compound interest, provided the person to whom the money was loaned is willing to pay it, but it cannot be legally collected unless there had been a previous agreement to that effect. 1. What is the compound interest of $9000 for 4 years at 8 per cent? Ans. $3244.40. FIRST OPERATION. SECOND OPERATION. Principal, * * * * * $9000.00 | Principal for the 1st year, - $9000.00 Interest on principal for 1 year at 8%, 720.00 1.08 New principal for the 2d year, - $9720.00 | Principal for the 2d year, - $9720.00 Interest on same for 1 year at 8%, 777.60 1.08 New principal for the 3d year, º $10497,60 | Principal for the 3d year, - $10497.60 Interest on same for 1 year at 8%, 839.80 1.08 New principal for the 4th year, - $11337.40 | Principal for the 4th year, - $11337.4080 Interest on same for 1 year at 8%, 907.00 1.08 The amount for 4 years is - - $12244.40 || Amount at the end of the 4th year, $12244.400640 First principal deducted, - - 9000.00 | First principal deducted, - 9000.00 Compound interest for 4 years, - $3244.40 | Compound interest, - - - $3244.40 Explanation.—The operation of this problem is so clear that an explanation is almost unnec- essary. We first find the interest on the principal for 1 year, and add it to the same, which pro- duces a new principal for the second year's interest; and in like manner we continue till the end of the 4 years; then we deduct the first principal from the amount and in the remainder have the compound interest for 4 years at 8 per cent. In the 2d operation, we first assume $1 and then find the amount of the same for 1 year at 8 6 IO soule's PHILOSOPHIC PRACTICAL MATHEMATICS. * per cent, which is $1.08. Then, having the amount of $1, we find the amount of $9000 by multi- plying the same by the $9000, and thus we continue to multiply the $1.08 by the successive amounts produced until the close of the time. Then to find the interest we deduct the first principal from the last amount. Had we raised the ratio $1.08 to the fourth power and then multiplied by the $9000, we would have produced the same result. Thus, $1.08×31.08×31.08×31.08×39000–$12244.40064000. 2. What is the compound interest of $500 for 1 year and 6 months at 6 per cent payable quarterly? Ans. $46.72. OPERATION. Principal, - * - - - - $500.00 | Principal for 5th quarter brought for’d, $530.68 Interest on same for 3 months at 6%, 7.50 | Interest on same for 5th quarter, - 7.96 New principal for 2d quarter, - - $507.50 | New principal for 6th quarter, - tº- $538.64 Interest on same for 2d quarter, - 7.61 || Interest on same for 6th quarter, º 8.08 New principal for 3d quarter, - - $515.11 || The amount for 1 year and 6 months is $546.72 Interest on same for 3d quarter, - 7.73 First principal deducted, - - º 500.00 New principal for 4th quarter, - $522.84 || Compound int. for 1 year and 6 months, $46.72 Interest on same for 4th quarter, * - 7.84 New principal for 5th quarter, - - $530.68 | 3. What is the compound interest of $1000 for 2 years 3 months and 5 days at 10 per cent, payable semi-annually 3 Ans. $247.58. OPERATION. Principal, -> * $1000.00 | Prin. for 4th 6 months brought for’d. $1157.62% Int. on the same for 6 months at 10%, 50.00 || Interest on same for 6 months at 10%, 57.88% New principal for the 2d 6 months, $1050.00 | New principal for 3 mos. and 5 days, $1215.50; Interest on same for 6 months at 10%, 52.50 | Int. on same for 3 mos. and 5 ds. at 10%, 32.08 New principal for the 3d 6 months, $1102.50 || Amount for 2 years 3 months and 5 ds. $1247.58 Interest on same for 6 months at 10%, 55.12% | First principal deducted, - - 1000.00 New principal for the 4th 6 months, $1157.62} | Compound int. for 2 yrs. 3 mos. and 5 ds. $247.58 4. What will $1450 amount to at compound interest for 1 year 2 months and 12 days at 6 per cent compounded monthly? Ans. $1557.97. In practice, the finding of compound interest is rendered more expeditious by the use of the following Compound Interest Table: NoTE.—To find the sum of the amounts of compound interest in the tables from 1 year to any other year inclusive, without making the additions, take the compound interest for the next year greater than the highest given, at the same per cent, and subtract from this the amount of 1 year at the head of the table in the same column; then remove the decimal point two places to the right, and divide by the rate per cent; considered as an integer, the quotient will equal the sum of all the amounts in the column from 1 year to the given year inclusive. INTEREST, 6 II 1178. A M O U N T OF $1, £1, 1 FR., AT COMPOUND INTEREST IN ANY NUMBER OF YEARs, NOT EXCEEDING FIFTY-FIVE. Yrs. 1 per cent. 13 per cent. || 2 per cent. 24 per cent. || 3 per cent. |3} per cent. || 4 per cent. 1. 1.0100 1.0150 1.0200 0000 | 1.0250 0000 | 1,0300 0000 | 1,0350 0000 || 1,0400 0000 2 1,0201 1,0302 1,0404 OO00 | 1,0506 2500 | 1.0609 OOOO | 1,0712 2500 | 1,0816 0000 3 1.0303 1.0457 1,0612 0800 | 1.0768 9062 | 1,0927 2700 | 1,1087 1787 | 1.1248 6400 4 1,0406 1.0614 1,0824 3216 | 1.1038 1289 | 1.1255 0881 | 1.1475 2300 | 1.1698 5856 5 1.0510 1.0773 1.1040 8080 | 1.1314 0821 | 1.1592 74.07 | 1.1876 86.31 | 1,2166 5290 6 1.0615 1,0934 1.1261 6.242 | 1.1596 93.42 | 1.1940 5230 1.2292 5533 1,2653 1902 7 1,0721 1.1098 1.1486 8567 | 1.1886 8575 | 1.2298 7387 | 1.2722 7.926 | 1.3159 3178. 8 1.0829 1.1265 1.1716 5938 1,2184 0290 | 1.2667 7008 || 1.3168 09()4 || 1,3685 6905 9 1.0937 1.1434 1.1950 92.57 | 1,2488 6297 | 1.3047 7318 || 1.3628 9735 | 1.4233 1181 10 1.1046 1.1605 1.2189 94.42 | 1.2800 8454 | 1.3439 1638 | 1.4105 9876 | 1.4802 4428. 11 1.1157 1.1779 1.2433 7431 | 1.3120 8666 | 1.3842 3387 | 1.4599 6972 | 1.5394 5406 12 1.1268 1.1956 1.2682 4179 | 1.3448 8882 | 1.4257 6089 | 1.5110 6866 | 1.6010 3222 13 1.1381 1.2136 1.2936 O663 | 1.3785 1104 || 1.4685 3371 || 1 5639 5606 | 1.6650 7351 14 1.1495 1,2318 1.3194 7876 | 1.4129 7382 | 1.5135 8972 | 1.6.186 9452 | 1.7316 7645 15 1.1610 1.2502 1.3458 6834 | 1.4482 98.17 | 1.5579 6742 | 1.6753 4883 || 1,8009 4351 16 1.1726 1.2690 1.3727 8570 | 1,4845 0.562 | 1,6047 O644 | 1.7339 8601 | 1.8729 8125. 17 1.1843 1.2880 1,4002 4142 | 1.5216 1826 || 1 6528 4763 | 1.7946 7555 | 1.9479 00:50 18 1.1961 1.3073 1.4282 4625 | 1.5596 5872 | 1.7024 3306 | 1.8574 8920 | 2.0258 1652 19 1.2081 1.3270 1.4568 1117 | 1.5986 5019 | 1.7535 0605 || 1 9225 0132 2.1068 4918. 20 1.2202 1.3469 1.4859 4740 | 1.6386 1644 | 1.8061 1123 | 1.9897 8886 2.1911 2314 21 1.2324 1,3671 1.5156 6634 | 1.6795 8185 | 1.8602 94.57 2,0594 3147 2.2787 6807 22 1,2447 1.3876 1.5459 7967 | 1.7215 7140 | 1.9161 0341 2.1315 1158 2.3699 1879 23 1.2572 1.4084 1.5768 9926 | 1.7646 1068 || 1.9735 8651 2,2061 1448 2,4647 1555 24 1.2697 1.4295 1.6084 3725 | 1.8087 2595 || 2.0327 9411 2.2833 2849 2,5633 0417 25 1.2824 1.4509 1.6406 0599 || 1.8539 4410 || 2.0937 7793 2,8632 4498 || 2,6658 3633 26 1.2953 1.4727 1.6734 1811 | 1.9002 9270 2.1565 9127 | 2.4459 5856 2.7724 6979 27 .1.3082 1.4948 1.7068 8648 || 1.9478 0002 || 2,2212 8901 || 2.5315 6711 2,8833 6858 28 1.3213 1.5172 1.7410 2421 | 1.9964. 9502 || 2.2879 2768 2,6201 7.196 || 2,9987 0332. 29 1.3345 1.5400 1.7758 4469 || 2.0464 0739 || 2 3565 6551 2.71.18 7798 || 3.1186 5145. 30 1.3478 1,5631 1.81.13 6158 || 2.0975 6758 2.4272 6247 2,8067 93.70 || 3,2433 9751 31 1.3613 1.5865 1.8475 8882 2.1500 0677 || 2.5000 8035 | 2,9050 3148 3.3731 3341 32 1.3749 1.6103 1.8845 4059 || 2.2037 5694 2.5750 8276 3.0067 O759 || 3.5080 5875 33 1.3887 1.6345 1.9222 3140 2.2588 5086 2.6523 3524 || 3.1119 4235 | 3,6483 8110 34 1.4026 1.6590 1.9606 7603 || 2.3153 2213 2.7319 O530 3.2208 6033 || 3.7943 1634 35 1.4166 1.6839 1.9998 8955 2.3732 0519 || 2.8138 6245 || 3.3335 9045 3.9460 8899 36 1.4308 1.7091 2.0398 8734 2.4325 3532 2.8982 7833 || 3,4502 6611 || 4,1039 3255 37 1.4451 1.7348 2.0806 8509 || 2.4933 4870 || 2.9852 2668 || 3.5710 2543 4.2680 8986 38 1.4595 1.7608 2.1222 98.79 2.5556 8242 || 3.0747 8348 || 3 6960 1132 || 4,4388 1345 39 1.4741 1.7872 2.1647 4477 2.6195 7448 || 3.1670 2698 || 3.8253 7171 || 4,6163 6599 40 1.4889 1.8140 2.2080 3966 2.6850 6384 || 3.2620 3779 || 3.9592 5972 || 4,8010 2063. 41 1.5038 1.8412 2.2522 0046 || 2.7521 9043 3.3598 98.93 || 4,0978 3381 || 4,9930 6145. 42 1.5188 1,8688 2.2972 4447 2.8209 9520 | 3.4606 9589 || 4.2412 5799 || 5,1927 8391 43 1.5340 1.8969 2.3431 8936 2.8915 2008 || 3.5645 1677 4.3897 0202 || 5,4004: 9527 44 1.5493 1.9253 2.3900 5314 || 2.9638 0808 || 3.6714 5227 || 4.5433 4160 || 5.6165. 1508 45 1.5648 1,9542 2.4378 5421 || 3.0379 0328 || 3.7815 9584 || 4.7023 5855 || 5,8411 7568 46 1.5805 1.9835 2.4866 1129 || 3.1138 5086 || 3.8950 4372 || 4.8669 4110 || 6.0748 2271 47 1.5963 2.0133 2.5363 4351 || 3. 1916 9713 || 4.0118 9503 || 5.0372 8404 || 6.3178 1562 48 1.6122 2.0435 2.5870 7039 || 3.2714 8956 || 4.1322 5188 5.2135 8898 || 6,5705 2824 49 1.6283 2.0741 2.6388 1179 || 3 3532 7680 || 4.2562 1934 || 5.3960 6459 || 6.8333 4937 50 1.6446 2.1052 2.6915 8803 || 3.4371 0872 || 4.3839 0602 || 5.5849 2686 || 7.1066 8335 51 1. . . . . . . . . . . . . . . . . . . . . . . . . . 2.7454 1979 || 3.5230 3644 || 4.5154 2320 5.7803 9930 || 7.3909 5068 52 |. . . . . . . . . . . . . . . . . . . . tº e ºs 2.8003 2819 || 3.6111 1235 || 4.6508 8590 5.9827 1327 7.6865 8871 53 l. . . . . . . . . . . . . . e tº e º e e 2.8563 3475 || 3.7013 9016 || 4.7904 1247 6.1921 0824 || 7.9940 5226 54 |. . . . . . . . . . . . . . . . . . . tº º º G 2.9134 6144 || 3.7939 2491 || 4.9341 2485 6.4088 3202 || 8.3138 1435 55 . . . . . . . . . . . . . . . . . . . . . . . . . . 2.9717 3067 || 3.8887 7303 || 5.0821 4859 6.6331 4114 || 8.6463 6692 Subtract $1 from the amount in this table to find the interest. z I9 ‘soilvywahlvyn Tvollovºid of HāosOTIHà S.3Tnos ‘aso.1aquy out pug oº oldeq SITQ uſ quinouſe out, urory Tº qoergqns 937 I690'68I 919 &SOF'ſ II I99 88I6'89 |g|I0 0918"I? | 9I& 8099.73 || 608 99.39°WI | LI30 gogg’II gg LLW 6L18'ILL | 610 LI96'70I 093 I608'89 609 I&I9'88 || 703 0993'83 || I96 8966'8I LL98 g[LL'OI fg 393 &L76'99 I | 67? Igé6'96 || IW6 ga.80'69 || 733 1980'98 || 986 9886"I& | 18; 6716'81 |gg88 1.108'OI £g 0&g 6&70°37L | 969 Iſºg'88 807 0901.’īg | 08W, 89&L'88 £g8 8969'03 | 680 8379 aſ goj9 8893.6 gg 388 66&I'6&I 696 9670'I8 || GIW 1899'09 | 89L 0619"I8 || 989 8939'6I 869 1070'3I | 0670 I68; 6 || Ig 63g 8068'LII | TO3 glgg'71, g3I 9106'97 Igg 0.297'66 | 879 TO&W'8I 866 £197'II 1393 9290:6 || 0g 319 68IL'90I 880 6LI&’89 || 06I 7137'87 || 008 6639'13 || 070 g/18'LI I88 8136'OI | LOIL. 9379-8 || 6p. 888 &LIO'L6 || 018 3989'39 18l. 90I6'07 || 990 68&L'93 || LIL. 8868.'9L | 169 &LOW'OI | 1.99g WLZ’8 || 8p. £g8 716I'88 |987 91.IV'19 || 6&I 0383'18 || 010 1970'73 19T 6997'g|I IIL 6906'6 || 6789 gºl6'1, 1+ I68 g6LI'08 || 6Iſ, 1919'39 898 0717'78 783 96.7'33 g18 7069'WI 38g 37876 1961 WPLG'L | 9F 188 7068%, I98 &Lög'87 || 767 7036"Ig | 8Ig 7600'I& 80I 979.’8I | 810 0986'8 |gf8? &#z'), g; I91, Oſ36'99 || 1.69 6938"77 || LIL 69gg'63 |969 7839'6I 6L8 7986%I 80g Ilgg’8 || 06Zz I986:9 | ## 269 0076'09 || 860 I919'07 || 707 9998'13 | 8pg £778'8I 97g 7093°3L | 699 96FI’8 |8188 p1999 gh a66 999."j9 || 0&g glT8'18 618 7688°gó | 899 &WI'LL | 138 0199 II 918 g[91'1, 8pg|I 91.gg’9 | af, II8 Ig31'67 619 3983 V8 | 688 7697'83 | 669 9330.9L | 019 8306'0I | 688 6L68'1, 7600 ISLO’9 If. 999 zögg'gſ 003 f607'Ig | g I3 grâL'I& 819 7716"VI 6LI 1983'0I | 1886680'ſ, fgp9 goI8'g | OF 811, LWWI’Iſ | LI8 691883 | 1.16 &g II’03 || 706 8766'8I g10 g801.6 &Ig 1701.9 |8066 8999'g | 68 fgſ £707'18 g08 998793 || 991 3939'8I WIL 36108I | 739 &pg|I’6 £11, 7988.9 I&6I &Qzg"g 33 987 6600'78 |ggg 8893'76 993 9973 LI | 18L 9833%I I18 0989'8 690 WI30’9 |6709 8960'g | 1.3 g08 9&I6'08 || 093 &Igg'66 | 8Ll I896'gſ | 667 6837'II | 039 &LVI’8 I9L 816L'g 9781, 81.18'ſ 9g 698 yaoſ'86 619 6817:03 gry 898. VI g[8 g319.0L | 898 0989-1 || 791 09Ig’g | 181; glg9% gg. 669 91.7gg3 |60ſ 7832.8I 988 IQ69.8L | 98T I816.6 | 893 0Iggºl | 08p 38gg’g fg|I9 go97'ſ fg fºg Igó3'36, p83 038L'/...I 962 0919 &I | 868 89386 | 668 goß’9 g88 Ig()0°g | 8108 OF1a'ſ g; 89A, 1.8LI'I& 883, 8891, '91 088 0181 II 801 &g|Il'8 || 198889?’9 gly 6791'ſ foſs 6630'ſ zg 937 876T'6I | 969 LI977 I 769 9198'0I 6&I IgVI'8 900 IS809 g68 0869'W gºlg 8816'g ig 930 F677"LI g8, 91.93'8I 699 969001 || 0gg 3&I9'l &I6 ſigºlºg | V&W 6L23'ſ g181 gºl'g | Og 086 0898'g| | Tö8 I&LI'GI | 671 &llg’6 Ilg gy II'l 618 88IV'g 998 I9LI'ſ 6F93 OW39'g 6: 936 603?"#L g69 T191’II 790 Ilg.98 || 788 88799 || 1989 III'g | I63 IO36'g | 6666 962;"g | 3: 376 660T £I | Tö8 0973"OI gT9 0886.1 919 88Ig'9 || 69ſ. 8338'ſ 69g 7881-8 |9960 038g'g 12 990, 1816"II | 61.9 I668"6 689 8968'1 63g 8208'g 088 867g'ſ L&L 9999'g | IO6), 90FI’g | 92 690 ºf 88"OT || 108 08:39'8 ggl fºr 3.9 |9&g ſlºg | 101, 8163'ſ Oſg g08g’g | 978 #900'g gz D38 L678"6 388 OII6'ſ, 108 IIſ gº || 019 £310'g 978 68.707 | 666 0933°g | 888.I 09.18% | #: jø0 £796'8 gjº, 81.93"/, 189 #118 g | 663 g071'ſ 16; 1618.8 883 g[10'g g899 Izgl’g gz 670, 307ſ'8 #00 98.99'9 70? g38?'g | LIO 7067'ſ 713 g6098 || 109 aggóg IOgg 9999% ºz 667 &007' 1, 110 880I '9 188 88.80'g | 739 goſ I’ſ 989 g668"g | 939 698.3 9TIP goag'z Iz 000 g/.31°9 80I ji'09"g Ilg 6099'ſ gp8 9698.8 ggg Il,038 116 agggz ZOFI LII; a Oz 068 69 II'9 3I9 917 I’9 II0 1918.7 gló 99198 g66 9930’8 || 609 6939% Igo9 81.08% 6I 9/. I 6699'9 f03, ID.I.P."f 96I 0966.8 £38 66188 || 368 gp983 || 36L 99073 || 1181, #80g'z 31 90), 7790°g jºgg 9/.33°7 I8I 0001'8 || 39L 889 I-8 831, 1369'3 | 88T 0363"g | IS9. g.g.II'z II 08), 6769'? 690 801.6'8 937 6937’8 889 IEgó’6 || LIg 8073'3 || 971, 838L'3 |g|IO, gazorg | 9T £877 31.1. I*; g38 fºg'g I69 I&LI’8 g[g 0691% | 689 g368% 38& 681.0°g | WW28 gºgg'I gT £86 f/6/,'8 016, 1,[78'8 986 T186% 378 g81.9% 070 6093'3' | 918 6616"I | agſ'ſ 6198"I | FI &I), 339ſ'8 970 8990'g 183 96I1"g | 09W 860.7% £83 638.1% I6'ſ 9988'I OI96 Igll'I 81 j93 fºg I'g 879 9&I8'3 I01, I8Ig'3 9I6 Igg3% 996 ICI0% | 899 8961"I g718 8969"I | aſ ISI Igg.8% j96, 7089'3 068 91.88% 039 870I’& 986 3868"I | 763 80IL'I g08g 8:29"I II 9&f 1869 & 189 80.98% 093 689 I’8 WIg IL96'I Lly 806L'I 976 8839"I af69 63gg'I OI 1/f 61.98% 886 8LLI'3 970 0666"I 369 7888"I | 061, 7689"I | agg gigg'I fig6 098?'I | 6 888 98.71% 939 Ø66"I 308 6098"I 398 ISIL'I I87 8869"I fgg fºllſ"I I900 Izzy'I | 8 ILI 1876"I I63 0838"I £7& 88[L'I 918 1909"I 808 980g"I | #00 T107"I g3I9 8099"I | 1. 0I9 9 ILL"I I00 IM1.9"I £71, 8989'I 708 100g"I I6L 98IV'I 996 0079'I | a LO9 &208"I 9 00I gOT3"I Ojº, 9883"I I86 8697"I | LIg 9307"I 993 3888"I 918 aglz"I #6T8 T9;z'I g 000 Ij9?"I 9I8 g|II?"I 9688 7098'I 096 LOI8"I 01.1, 739&"I £90 gCIz"I 093I gačI*I | # 000 OI88"I O63 09:63"I 0&I 1693'I 087 0933'I | 09I 0I6I’I Ogg 91.9L'I | a 199 IIñI*I g 000 00I3"I O00 IS8I’I 000 799 I’I 000 677 I’I | 000 98&I"I | 000 gaOI"I 0096 0360'I | a 000 000I 'I O00 0060"I 000 0080"I | 000 0010"I | 000 0090"I 000 0090"I | 0000 OgñO'I I "Juoo Iod OI ‘Auoo Iod 6 || Tuao Ied 8 || "Juao Iod 1 || “luoo Iod 9 || Aueo Ied g | Tuao Iod #7 |'s IX ‘āAII-XLIII psIqqāox'ſ LoN “savºix IO IqawnN XNy NI LSGI’IGILNI (INoodwoo Lv “gº I to ‘I3: “I; ho * *62 II * JL N ſl O TWIV X: COMPOUND INTEREST. 613 TO COMPUTE COMPOUND INTEREST BY THE USE OF THE COMIPOUND INTEREST TABLE. 1180. To find the amount or interest when the principal, time and rate are given : 1. What is the compound interest on $9000 for 4 years at 8 per cent? Ans. $3244.40. OPERATION. Ea:planation.—To find the compound interest § 36(). sº the use of the compoun $1.360489. = compound amount of $1 for 4 years. interest tables, first find 9000 = principal. in the table the compound amount of $1 for the time. $12244.40.1000 = compound amount of $9000 for 4 years. ººl rate, then aultiply this amount by the prin- 9000. = principal. cipal, and from the prod- e uct subtract the principal; $ 3244.40 = compound interest. the remainder will be the compound interest. NOTE 1.-If the interest is compounded semi-annually, then find the compound amount on $1 for twise the number of years and for # the rate per cent. * Thus, the compound interest on $1 for 8 years at 6 per cent payable semi-annually, is the same as the compºund interest on $1 for 16 years at 3 per cent, payable annually. NOTE: 2. —Iſ the interest is compounded quarterly, then find the compound amount on $1 for 4 times the number of years and for + the rate per cent. Thus, the compound interest on $1 for 6 years at 8 per cent, payable quarterly, is the same as the compound interest on $1 for 24 years at 2 per cent, payable annually. º 3.—When compounded monthly, consider each month a year and divide the rate per cent by 12. NOTE 4.—When the amount is required for a number of years not contained in the table, compute the same by multiplying together the amounts of any two number of years, whose sum equals the number of years required. Thus, if the amount of 86 years at 4 per cent is required, multiply together the compound amount of 46 and 40 years, or 36 and 50 years: For example, the amount for 46 years is 6.07482–H and for 40 years it is 4.80102-H and the product is 29.1663323+ which is the amount of $1 for 86 years at 4 per cent. NOTE: 5.-If partial payments are made on notes or debts bearing compound interest, the compound amount of the principal must be first found, and the sum of the compound amounts of the payments subtracted from it. 2. What is the compound interest on $800 for 6 years, 4 months at 44 per cent 3 Ans. $257.43. OPERATION. $1.30226012 = compound amount of $1 for 6 years at 44 per cent. 800 = principal. $1041.80809600 = compound amount of $800 for 6 years. 15,6275 = interest on $1041.81 for 4 months, at 44 per cent. $1057.43 = compound amount on $800 for 6 years, 4 months at 44 per cent. 800. = principal. Ǻmºsºmºsºmsºmºsº $257.43 = compound interest on $800 for 6 years, 4 months at 44 per cent. NoTE.—When the time includes months or days, the interest for such number of months or days is computed on the compound amount for the even years, and added thereto, as shown above. 614 SOULE's PHILOSOPHIC PRACTICAL MATHEMATICs. * 3. What is the compound interest on $5000 for 12 years, 1 month, 12 days at 10 per cent compounded quarterly 3 Ans. $16548.29 compound amount. $11548.29 compound interest. OPERATION. $3.27148956 = compound amount of $1 for 48 years at 24 per cent. 5000 = principal. $16357,44780000 = compound amount of $5000 for 12 years at 10 per cent. 190.84 = interest on compound amount of $16357.45 at 10% for 42 ds., **smºm or for 168 ds, at 24%. $16548.29 = compound amount. 5000.00 = principal deducted. & sºmºmºmºsºs º ºsmº $11548.29 = compound interest. 4. What is the compound interest on $3000 for 20 years, 3 months, 16 days at 3 per cent, payable semi-annually? Ans. $2490.07. TO FIND THE PRINCIPAL WHEN THE COMPOUND AMOUNT OR INTER— EST, THE TIME, AND THE RATE ARE GIVEN. 1181. 1. What principal at 8 per cent compound interest will amount to, $12244.401, in 4 years? Ans. $9000. OPERATION. $12244.401 - $1.360489 = $9000. Explanation.—The compound amount of $1 for 4 years at 8 per cent is $1.360489 and hence, $12244.401 is the compound amount of as many dollars as it is equal to $1.360489, which 1s $9000. Therefore, in all problems of this kind to find the principal, we divide the amount or interest by the amount or interest of $1 for the given time and rate. 2. What principal at 4 per cent compound interest will produce $10000 interest in 9 years? ... Ans. $23623.25. OPERATION INDICATED. $10000 - .42331181. NotE.—The $.42331181 is the compound interest on $1 for 9 years at 4 per cent. See table and subtract $1 from compound amount. TO FIND THE TIME, WHEN THE PRINCIPAL, THE RATE AND THE COMPOUND AMOUNT OR INTEREST IS GIVEN. 1182. 1. In what time will $9000 amount to $12244.401 at 8 per cent? * Ans. 4 years. OPERATION. : $12244.401-- $9000 = $1.360489 = compound amount of $1 which, in the table cor- responds to 4 years. Explanation.—Since $12244.401 is the compound amount of $9000 at 8 per cent in a certain * COMPOUND INTEREST. 6 I5 time, the amount of $1 will be the 9000th part, which is $1.360489 and which we find in the table under 8 per cent to correspond to 4 years. Therefore, in all problems of this kind to find the time, we divide the amount by the given principal and in the quotient we have the amount of $1 for the time. Then we find this amount in the table under the given rate, and opposite thereto is the required time. 2. In what time will $2000 amount to $4584,0366 at 5 per cent compound interest? Ans. 17 years. TO FIND THE RATE WHEN THE PRINCIPAL, THE COMPOUND AMOUNT OR INTEREST, AND THE TIME ARE GIVEN. 1183. 1. At what rate will $9000 amount to $12244.401 in 4 years? Ans. 8%. OPERATION. $12244.401 - $9000 = $1.360489, which in the table corresponds to 8 per cent. Explanation.—Since $9000 amounts to $12244.401 in 4 years at the required rate, $1 for the same time and rate will amount to the 9000th part, which is, $1.360489, and which, in the table opposite 4 years corresponds to 8 per cent. Therefore, in all problems of this kind to find the rate, we divide the amount by the given principal and the quotient will be the amount of $1 for the given tiime at the required rate. Then we refer to the table and find this amount opposite the given time, and the rate at the top of the column is the required rate. 2. At what rate will $8000 amount to $22072.252 in 15 years? Ans. 7%. NotE.—See the subject of Annuities for further work involving the principles of simple and compound interest investments. SIMPLE, ANNUAL, AND COMPOUND INTEREST COMPARED. 1184. What is the simple, the annual, and the compound interest on $5000, for 4 years at 8 per cent? Ans. $1600, simple interest. $1792, annual interest. $1802.45, compound interest. OPERATION FOR ANNUAL INTEREST. OPERATION FOR SIMPLE INTEREST. Interest on the $5000 principal for 4 years at 8%, - - $1600 (The annual interest is $400 per year). $50.00 Interest on $400 for 3 years, 8% Interest on 400 for 2 years, Interest on 400 for 1 year, = $400.00 - 4 yrs. the 1nt. on $400 for 6 years at 8 per cent, - º º -> 192 $1600.00 Ans. $1792 OPERATION FOR COMPOUND INTEREST BY THE COMPOUND INTEREST TABLE. $1.36048896 = compound amount of $1 for 4 years at 8 per cent. 5000 = principal. $6802.4480000 = compound amount of $5000 for 4 years at 8 per cent. 5000. = principal deducted. $1802.45 = compound interest on $5000 for 4 years at 8 per cent. 6 I 6 SOULE's PHILOSOPHIC PRACTICAL MATHEMATICs. * By the foregoing work we see that the difference between simple, annual, and compound interest in their effect, depends upon the time when interest money, if due and not paid, begins to draw interest. As remarked in annual interest, a debt begins to draw interest when due; but the time when a debt of interest becomes due, is a question in some cases governed by business custom, and in other cases it is a question to be decided by the parties interested. In bank discount, the interest is payable in advance. In simple interest, it is not considered due until the final payment of the principal. In annual interest, it is due after it has been accruing for the time specified for interest payments, except the interest on interest which is not due till final pay- ment. Compound interest supposes all interest, whether upon principal or interest, to be due at the end of such equal successive intervals of time as may be agreed upon. r When the interest is considered due the instant it has accrued and all inter- est is made to draw interest, it is termed instantaneous compound interest. PARTIAL PAYMENTS, OR PAYMENTS BY INSTALLMENTS. 1185. Partial payments, or payments by installments as the terms imply, are the payments made at different times, of promissory notes, acceptances, bonds, personal accounts, or other obligations. At the time these partial payments are made, the creditor should and usually does specify in writing, on the back of the note or other instrument, the sum paid and the date of payment, and then signs his name. It is a principle of law that a note does not draw interest until it is due, unless the words “with interest” are contained therein. *. There are several systems of computing interest upon notes, bonds, personal accounts, and other obligations upon which partial payments have been made; and as the different systems generally give different results, according as the time intervening between the payments is more or less than a year, or the interest more or less than the payment, it is therefore impossible in advance of knowing the exact facts of each case, to say which system would be strictly just to all parties. Among these various systems, the five following are mostly used: The United States System ; the Merchants' System; the Vermont System ; the New Hampshire System ; and the Connecticut System. A These five systems we will define, and then illustrate by examples the United States and the Merchants' Systems, which are in general use outside of the bound- aries of the States of Vermont, Connecticut and New Hampshire. THE UNITED STATES SYSTEM. 1186. The following is the decision of Chancellor Kent for computing interest where partial payments have been made, and having been adopted by the Supreme 4× PARTIAL PAYMENTS, 617 Court of the United States, as also by several other States, is hence termed the United States System. I.-The rule for casting interest when partial payments have been made, is to apply the pay- ment, in the first place, to the discharge of the interest due. II.-If the payment exceeds the interest, the surplus goes towards discharging the principal, and the subsequent interest is to be computed on the balance of principal remaining due. III.—If the payment be less than the interest, the surplus of interest must not be taken to augment the principal; but interest continues on the former principal until the period when the payments taken together exceed the interest due, and then the surplus is to be applied towards discharging the principal, and interest is to be computed on the balance as aforesaid. [See Chancellor Kent, Johnson's Chancery Rep., Vol. 1, p. 17]. The practical result of this system is the loss of the use of money paid by the person owing the debt when the same is less than the interest, from the time paid until the sum of the payments exceeds the interest, and also the compounding of the money in favor of the creditor as often as the amount paid exceeds the interest then due, and hence two reasons for the debtor to defer payment as long as possible. Thus, presuming a debtor owes a note for $10000 bearing 6 per cent interest, and that he pays thereon $50 per month, at the close of the year he still owes the $10000. Had he loaned the $50 monthly payments at 6 per cent he would have had at the end of the year $616.50 to pay on his note and the accrued interest, $10600, leaving a debt of $9983.50 instead of $10000. TELE MERCEIANTS’ SYSTEM. 1187. The Merchants' System consists in computing the interest on the prin- cipal, or original debt, from the time it became due until the close of the civil, or of the fiscal year, and adding it to the principal; and then computing the interest also upon the payments made, if any, during the year, from the time they were made to the end of the civil, or fiscal year, and adding the same to the payments made; then deducting the sum of the payments and interest from the amount of principal and interest. The remainder will be a new principal. In this manner the operation is continued from year to year, or from settlement to settlement, if made oftener than once a year, until the final settlement. The periods of time for which the interest is computed and the balance brought down are termed rests. Merchants frequently close their accounts every six months, and bankers every quarter, in which case the directions given above for yearly settlements would be made semi-annually and quarterly. The effect of this system is the same as the account current and interest account with the creditors, made as often as the accounts are balanced, and the practical result is the compounding of money for both parties as often as settlements are made. The Merchants' System is the only system that is applicable to book accounts as well as notes, bonds, etc., and it is, we think, practically nearer true justice to both parties, debtor and creditor, than the United States System. 1188. VERMONT SYSTEM IS I. “When payments are made on notes, bills, or similar obligations, whether payable on demand or 6 I 8 soul E's PHILOSOPHIC PRACTICAL MATHEMATICS. * at a specified time, with interest, such payments shall be applied: FIRST, TO LIQUIDATE THE INTEREST that has accrued at the time of such payments, and SECONDLY, TO THE EXTINGUISHMENT OF THE PRINCIPAL. II. “ The annual interests that shall remain unpaid on notes, bills, or similar obligations, whether payable on demand or at a specified time, “with interest annually,” shall be SUBJECT to simple INTEREST from the time they become due to the time of final settlement. III. “If payments have not been made in any year, reckoning from the time such annual interest began to accrue, the amount of 8 uch payments at the end of the year, with interest thereon from the time of payment, shall be applied: FIRST, to LIQUIDATE the SIMPLE INTEREST that has accrued from the UNPAID ANNUAL INTERESTS; SECONDLY, to LIQUIDATE the ANNUAL INTERESTs that have become due ; THIRDLY, to the ExTINGUISHMENT of the PRINCIPAL.” ^ TEIE NEW HAMPSHIRE SYSTEM. 1189. The New Hampshire System is essentially the same as the preceding when partial payments are made on notes “with interest annually.” But “where payments are made expressly on account of interest accruing but not then due, they are applied when the interest falls due, without interest on such payments.” THE CONNECTICUT SYSTEM. 1190. The Connecticut System, according to the Supreme Court of Connecti- cut, is as follows: Compute the interest on the principal to the time of the first payment; if that be one year or more from the time the interest commenced, add it to the principal, and deduct the payment from the sum total. If they be after payments made, compute the interest on the balance due to the next payment, and then deduct the payment as above; and in like manner from one payment to another, till all the payments are absorbed, provided the time between one payment and another be one year or more. If any payments be made before one year's interest has accrued then compute the interest on the principal sum due on the obligation for 1 year, add it to the principal, and compute the interest on the sum paid from the time it was paid up to the end of the year; add it to the sum paid, and deduct that sum from the principal and interest added as above. s If any payments be made of a less sum than the interest arising at the time of such payments, no interest is to be computed, but only on the principal sum for any period. NoTE—If a year extends beyond the time of settlement, find the amount of the remaining principal to the time of settlement; find also the amount of the payment or payments, if any, from the time they were paid to the time of settlement, and subtract their sum from the amount of the principal. Yºr PARTIAL PAYMENTS, 619 1191. PROBLEMS WORKED BY THE UNITED STATES SYSTEM.” 1. $9000. NEW ORLEANs, February 3d, 1894. On demand, for value received, we promise to pay to the order of CUMMINGS & CENAs, Nine Thousand Dollars with interest at six (6) per cent. g MOYSE & WILSON. The following payments were made and indorsed on the foregoing note: September 10th, 1894, - $1500.00 March 5th, 1895, - $3953.75 December 18th, 1894, - 100.00 July 20th, 1895, - 2000.00 Counting 30 days to the month, what was the amount due April 1, 1896? Ans. $2177.43. OPERATION. 1894 * $ | c. Feb. 3| To face of note tº tº * º tº gº &_º tº s * sº * * * * > 9000:00 Sept. 10 “interest on same from Feb. 3 to Sept. 10, 217 days at 6 per cent - º 325|50 é & ** | ** amount due sº gº gº ſº sº * * * sº º &º * * * & 9325 50 é & & 4 By cash, (1st payment), * * * * * * * * * 1500|00 & 4 é & ‘‘ balance due º tº- gº & # * . * = gº ge gº dº tº gº 1825 50 Dec. 18| “ Interest on balance due from Sep. 10 to Dec. 18, 98 days at 6% - $127.82 & 6 & 4 By cash, (2d payment), gº * * * tº sº tº gº & 100.00 is “| “ balance of interest - - - - - - - - - $27.82 95 March 5 “ int. on bal. due ($7825.50) from Dec. 18 to March 5, 77 days at 6% 100.43 which added to balance of Interest due makes *- sº — | 12825 & 6 & & & 4 amount due &E tº sº E: * - gº gº «-» º tº gº - sº 7953 75 & 4 & ‘ By cash, (3d payment), gº ºn tº as tº dº tº º tº 3953|75 &: $ 6 & 4 balance due sº tº sº * * * tººk gº gº tºº * * & º 4000 00 July 20 “interest on balance due flom March 5 to July 20, 135 days at 6% - cº 9000 & 4 & 4 & 4 amount due * {-º * ſº †º tº & ºn * * - gº Cº. ſº ºn º sº 4090 00 é & & 4 By cash, (4th payment), gº tº º tº ſº tº gº gº * & 200000 4 & & 4 ‘‘ balance due - *º-º tº tº ºf tº e * * gº † gº ‘. * > ( : 2000 00 1896 April 1| “interest on balance due from July 20 to April 1, 251 days at 6% - - 8743 & 4 “| “...amount due April 1, 1896 - tº tº ſº gº * tº tº tº 2177.43 Explanation.—In the operation of this problem we found that the interest due on the 18th of December was greater than the payment then made; hence, according to the principles of this system of work, we did not add the same to the principal. Had we done so the result would have been compound interest on the excess of interest, $27.82, to the disadvantage of the debtor, and in a sense that the law of this system does not allow. We accordingly deducted the payment from the interest, and reserved the excess until the next payment, to which excess we then added the interest on balance due from December 18 to March 5, 1895, and thus obtain $128.25 interest, which being less than the payment then made, we added to the balance due September 10, 1894. Instead of deducting the payment made from the interest, we could have set the whole interest aside until the sum of the payments exceeded the sum of the interests, and then proceeded in the regular manner by adding the sum of the interest to the amount due, and then deducting the sum of the payments therefrom. But we think that the method shown in the operation is the clearest, and hence prefer it. * 62O soul E's PHILOSOPHIC PRACTICAL MATHEMATICS. 2. “ $5200. A MARSHALL, TEXAS, July 28, 1894. PATTERSON & TURMAN. t October 1st, 1894, - $1200 August December 8th, 1894, - 1000 June 25th, 1895, - 100 what was then due # 1st, 1895, November 11th, 1895, The following payments were made and indorsed on the above due bill: $1500 900 Ans. $938.30. Due, from date hereof, to WOMACK & WILLIAMS, or order, Five Thousand Two Hundred Dollars, for value received, with interest at 10 per cent. A final settlement was made December 15th, 1895. Counting actual time, OPERATION. T1394 |_$.1% July 28. To amount due - --> * se tº- gº * --> sº ſº sº {-º sº- $º 5200.00 Oct. 1| “ interest on same from July 28 to Oct. 1st, 65 days at 10 per cent - sº 93|89. & & & & {6 amount due ge tº ſº º * * º * - tº tº gº &º gº * 5293 89. & t $ 6 By cash, (1st payment), gºs º gº ºne ºs tº sº, sº sº 1200|00 & 4 & 4 { { balance due ſº sº tº tº- gº * * wº e- # = gº sº * -º & 4093 89. Dec. 8| “ interest on same from Oct. 1 to Dec. 8, 68 days at 10 per cent - sº 77|33. & 6 & & ‘‘ amount due - tº * > 4 º' * - sº *E* $º tº sº sº sº º 4171 22 & 4 & 4 By cash, (2d payment), - - - - - - - - - - 100000 & 4 & 4 & 4 balance due - tº * > * - tº- gº & º sº & gºe tº wº 3171:22. 1895 June 25 “ int. on same from Dec. 8, '94, to June 25, '95, 199 days at 10% - $175.30 & & & 4 By cash, (3d payment), a. * * * * * * * * 100.00 & 4 “| “ balance of interest due - - - - - - - - - $75.30 Aug. 1| “ int. on bal. due ($3171.22) from June 25 to Aug. 1, 37 days at 10% 32.59 $ which added to balance of interest makes - tº gº tº gº * - 10789 & 4 ** | ** amount due * sº sº * gº tº 4 º' & ºt gº gº gº sº 3279|11 4 & & 4 By cash, (4th payment), º, sº as tº tº º sº gº º 1500|00. “ ** | ** balance due - - - - - - - - - - - - - 1779|11 rov. 11| “ interest on same from Aug. 1 to Nov. 11, 102 days at 10 per cent - tº 50|41 & & & 4 & 4 amount due gº tº sº tº sº gº º sº * * tº º &º 4.- : 1829 52. & 4 4 & By cash, (5th payment), tº º ºs º we dº º ºs ºne 90000 & 6 & & & 4 balance due sº tº- * sº gº tºº {º tº ſº &= ſº tº 929.52 Dec. 15| ‘‘ interest on same from Nov. 11 to Dec. 15, 34 days at 10 per cent - - 8|78. & 4 “| “ amount due on final settlement December 15, 1895, gº ºf tº gº 938|30. Explanation.—In the operation of this problem, we have counted actual time, which we, regard as the better way at all times. { PARTIAL PAYMENTS, 62 I 1192. PROBLEMS WORKED BY THE MERCEIANTS’ SYSTEM. $9000. NEW ORLEANs, February 3, 1894. On demand, for value received, we promise to pay to the order of CUMMINGS & CENAS, Nine Thousand Dollars with interest at 6 per cent. MOYSE & WILSON. The following payments were made and indorsed on the foregoing note: September 10th, 1894, $1500.00 March 5th, 1895, - $3953.75 December 18th, 1894, 100.00 July 20th, 1895, - 2000.00 Counting 30 days to the month, and making an annual settlement on the first of January, 1895, what was the amount due April 1st, 1896? Ans. $2.165.40. OPERATION. 1894 $ | c. Feb. 3| To face of note - tº- - - tº ſº - - sº - - s º 9000|00 1895 | - Jan. 1) “interest on same from Feb. 3 to Jan 1, 328 days at 6 per cent & # - 492|00 é & “| “ amount due to date * - * tº- - - ºr - - tº º 9492|00 1894 sº 10 By cash, (1st payment), - - - - - - - $1500.00 1895 Jan. 1 “ interest on same from September 10 to January 1, 111 days at 6 per cent - - - - * - º 27.75 1894 º 18 By cash, (2d payment), tº E- - - - - = 100.00 5 Jan. 1 “ interest on same from December 18 to January 1, 13 days at 6 per cent - & º - tº - - .22 & 4 & & “ total credits to date - - - - - - - - - 1627|97 { { ‘‘I To balance due this date - - s º º ºs e - • * º 7864|03 1896 Jan. 1| “ interest on same from Jan. 1 to Jan. 1, 360 days at 6 per cent - - 471184 & 4 ‘‘ ‘‘ amount due to date º cº-º º tº tº tº- & 4- tº º *- 8335|87 1895 *** March 5 By cash, (3d payment), ge ºn tº sº tº - - $3953.75 1896 Jan. 1 “ interest on same from March 5 to January 1, 296 days at 6 per cent - - - - - º - - 195.05 1895 - July 20 “ cash, (4th payment), º tº - * = * - 2000,00 1896 Jan. 1 “ interest on same from July 20 to January 1, 161 days º , at 6 per cent - - - - tº ºs º – 53.67 & 4 ( & ‘‘ total credits to date - - - - sº - - º 620247 & 4 ‘‘l To balance due this date - - º - - º º- - º 213340 April 1| “ interest on same from Jan. 1 to April 1, 90 days at 6% - *- sº 3200 § { “| “ amount due on final settlement * tº- - tº tº- - - gº 2165|40 Explanation.—In the solution of this problem, as shown by the operation, we made the first partial settlement at the close of the civil year, notwithstanding a year had not elapsed from the date of the obligation. We did this because it is strictly in conformity with the merchant's system of closing his accounts at the end of each civil or his fiscal year, and because we believe that it is the only correct way of applying the merchants’ system of partial payments. Merchants enerally not only desire, but by the municipal, State and general government laws are required to #. their gains and losses, resources and liabilities on the first of January of each year. With- out this information, correct returns for the assessment of various taxes could not be rendered, nor 622 soul E's PHILOSOPHIC PRACTICAL MATHEMATICs. º would their books show a correct condition of their business. Had we, as some authors direct, computed the interest from February 3, 1894, to February 3, 1895, and then from February 3, 1895, to February 3, 1896, and then from February 3, 1896, to April 1, 1896, the final result would have been slightly different, and when the books were closed on the first of January of each year, the true gains and resources of the business would not have been shown by the books. In counting the time we used 30 days to the month in order to compare the result of the #.'; with the result of the first and same problem worked by the United States System, on page 619. By the United States System the amount due on final settlement is - $2177.43 And by the Merchants' System as above, * * º º ºs º 2165.40 Excess produced by the United States System, - - - $12,03 2. * $5200. MARSHALL, TEXAS, July 28, 1894. Due from date hereof to Wom ACK & WILLIAMs, or order, Five Thousand Two Hundred Dollars for value received, with interest at 10 per cent. PATTERSON & TURMAN. The following payments were made and indorsed on the above duº bill: October 1st, 1894, - $1200 August 1st, 1895, - $1500 December 8th, 1894, - 1000 November 11th, 1895, - 900 June 25th, 1895, - 100 f A final settlement was made December 15, 1895. Counting actual time, and making a partial settlement at the close of the civil year 1894, what amount was due on final settlement” Ans. $927.77. OPERATION. 1894 - $ | c. July 28. To face of due bill - - - º tº- <- ſº - tº 5200|00 1895 Jan. 1| “interest on same from July 28 to Jan. 1, 157 days at 10% * * 22678 4 & “| ‘‘ amount due this date tº- tº º º wº- - º tº- sº 5426||78 1894 Oct. 1 By cash, (1st payment), - - - - - - - $1200.00 & 4 é & “ interest on same from October 1 to January 1, 92 days at 10 per cent - - - gº tº º tº- tº 30.67 Dec. 8 “ cash, (2d payment), tº ºs - is sº tº tº 1000.00 & 4 & 4 “ interest on same from December 8 to January 1, 24 days at 10 per cent - - - & cº- Q- i.e. 6.67 1895 -*º- Jan. 1 “ total credits to date - - - - - - - - - 2237|34 & 4 “| To balance due to date - - tº ſº tº . gº tº- º tº gº 3189 4. Dec 15| “ interest on same from January 1 to December 15, 348 days at 10% tº º wº & 4 “| “ amount due this date - - - tº ºs - - * * º 349775 June 25 By cash, (3d payment), sº º º ºs º º ºs $100.00 €C. 15 “ interest on same from June 25 to December 15, 173 f days at 10 per cent - - - gº - - º 4.81 Aug. 1 “ cash, (4th payment), - - - - - - - 1500.00 Dec. 15 “ interest on same from August 1 to December 15, 136 days at 10 per cent - - - - - tº- tº 56.67 NOV 11 “ cash, (5th payment), - º - º - e- tº 900.00 Dec 15 “ interest on same from November 11 to December 15, 34 days at 10 per cent - . - - - - - 8,50 6& & 4 “ total credits to date - - - - - * - º ºs 256998 &é “| To amount due on final settlement • m as mº' s m º ºs ºn Explanation.—By comparing the result of this problem with the result of the second and same problem worked by the United States System, on page 620, we have the following figures: * PARTIAL PAYMENTS. 623 Amount due on final settlement by the United States System, - gº $938.30 Amount due as above by the Merchants' System, is e º º 927.77 Excess produced by the United States System, - - - $10.53 Where the payments are small, or the interval of time between them is more than a year, the Merchants' System would produce an excess over the United States System. * 1193. MISCELLANEOUS PROBLEMS INVOLVING THE UNITED STATES AND MERCHANTS’ SYSTEMS OF WORK. 1. $3740. VICKSBURG, May 1, 1894. Two years after date, we promise to pay to the order of STEWART & BRADSTON, Three Thousand Seven Hundred Forty dollars, with interest at eight (8) per cent, value received. BOWLES & STEWAR.T. The following payments were made and indorsed on the foregoing note: September 16th, 1894, - $500 October 28th, 1895, - - $1200 February 1st, 1895, - 75 April 1st, 1896, - - 300 May 4th, 1895, - 1500 Counting actual number of days, and working by the United States System, what was the amount due at the maturity of the note, May 4th, 1896? Ans. $560.68. 2. $2000. o LITTLE ROCK, ARK., January 12, 1894. Due SCULL & MCREY, Two Thousand Dollars, with interest at 8 per cent. FOSTER & SULLIVAN. The following payments were made and indorsed on the foregoing note: November 1st, 1894, - $600 May 9th, 1895, . . $400 January 18th, 1895, - 750 December 1st, 1895, - - 200 Counting actual time, and working by the Merchants' System, what was due on final settlement, April 10, 1896? Ans. $249.84. 3. $6840.50. On the 18th of October, 1894, a judgment was rendered by one of the District Courts of New Orleans for Six Thousand Eight Hundred Forty and #, Dollars. (The legal rate of interest in Louisiana is 5 per cent). The following payments were made thereon: January 1st, 1895, - - $1000 November 25th, 1895, - $1000 May 14th, 1895, - - 1000 March 10th, 1896, - 3000 A final settlement was made April 21, 1896. Counting actual time and working by the United States System, what was then due Ans. $1225.24. 624 SOULE's PHILOSOPHIC PRACTICAL MATHEMATICs. * * 4. $20000. WACO, TEXAS, January 1, 1894. On demand, for value received, we promise to pay to the order of HIGHTOWER & ROSE, Twenty Thousand Dollars, with 10 per cent interest. POLLY, VINYARD & Co. - Semi-annual payments of $3000 each were made on the above note from date to January 1, 1897, at which time a final settlement was made. Counting actual time, what was then due by the United States System, and also by the Merchants' System of partial payments? Ans. $9466.19, by the United States System. $9329.23, by the Merchants' System. 1194. PARTIAL PAYMENTS ON NOTES DRAWN FOR A SPECIFIED TIME AND BEARING INTEREST AFTER IMATURITY. * $7000. SUMMIT, MISS., June 1, 1894. One year after date, for value received, we promise to pay to the order of CURTIs & NEWMAN, Seven Thousand Dollars, with 8 per cent interest after maturity until paid. TYLER & MILLS. The following payments were made and indorsed on this note: December 1st, 1894, º - º - s $2000 April 10th, 1895, - - s - - 2500 October 17th, 1895, º - º - - 1500 A final settlement was made December 20, 1895. Counting actual time, what was then due 3 Ans. $972.48. OPERATION. 1894 $...]9. June 1| Face of note due June 1/4, 1895 ºs - º - - - - - º 700000 Dec. 1 By cash, (1st payment), - - - - - - - - - - - $2000.00 “ interest on same from December 1 to June 4, 1895, 185 1895 days at 8 per cent - tº- sº - tº ºs - 82.22 | April 10 ‘‘ cash, (2d payment), - tº º tº tº- - 2500.00 | ‘‘ interest on same from April 10 to June 4, 1895, 55 days at 8 per cent - - - - - - - - - 30.56 “ total amount of credits before the maturity of the – note June 4, 1895 - - - - - - - - - 461278 June 4| To balance due at the maturity of the note - - - - - , , - - 2387|22 “ interest on same from June 4, 1895, to Oct. 17, 1895, 135 days at 8% sº 71.62 Oct. 17| ‘‘ amount due this date es e º sº º sº me - º tº e 2458|84 & 4 & 4 By cash, (3d payment), - - - - - - - - - 150000 ‘‘ balance due - - & tº- - º º - º * - º 958.84 Dec. 20 “interest on same from Oct. 17, to Dec. 20, 64 days at 8% gº tº ºs 13|64 & & “| “ amount due on final settlement - - - - - - * * * 97248 Explanation.—Where partial payments are made on notes of this character before they mature, we must first find the balance due at maturity. To do this we deduct from the face of the note the payments made before due, and the interest on such payments from the date of payment till the maturity of the note. Having thus produced the balance due on the note or obligation at the time of maturity, we then proceed with it, and the remaining payments if any, the same as in the preceding work of partial payments, either by the United States or Merchants' System. . In the operation of the above problem, we worked by the United States System, which is the system in general use for transactions of this kind. * PARTIAL PAYMENTS. 625 1195. PARTIAL PAYMENTS ON NOTES, ETC., DRAWN FoR A SPECIFIED TIME, AND which BEAR ANNUAL, SEMI-ANNUAL OR QUARTERLY INTEREST. $9000. NEW ORLEANs, March 17, 1889. Three years after date, for value received, I promise to pay to the order of FRANCIS & BAKER, Nine Thousand Dollars with eight (8) per cent interest, pay- able annually. E. W. JONES. The following payments were made and indorsed on this note: November 1st, 1889, - $1000 May 15th, 1891, - - $2000 July 10th, 1890, - 1500 January 4th, 1892, - - 1800 Counting 30 days to the month, what was due June 17, 1892, at which time a final settlement was made 1 Ans. $4552.69, by 1st operation. $4559,63, by 2d operation. IFIRST OPERATION. 1889 $ | c. March [17] To face of note tº sº dº sº sº º ºs º ºr ºt tº mº 900000 1890 “ interest on same from March 17, 1889, to March 17, 1890, 1 year at 8% - 72000 Mº 17| ‘‘ amount due this date sº- tº-e º- sº $º- tº- gºs s * - tº 97.2000 1889 Nov. 1. By cash, (1st payment), tº G- - tº- - * - $1000.00 “ interest on same from November 1, 1889, to March 17, 1890 1890, 4 months 16 days at 8 per cent - - - 30,22 March 17 “ total credits to date - - - - - - - - – 1030/22 i s 91 “| To balance due this date am s m sº em me as * * * 8689|78 March 17 “ interest on same from March 17, 1890, to March 17, 1891, 1 year at 8% - 695.18 & 4 “| “ amount due this date - - - - - - - - - - - 9384|96 1890 July 10 By cash, (2d payment), - - - - - - - $1500.00 “ interest on same from July 10, 1890, to March 17, 1891, 1891 8 months 7 days at 8 per cent - - - - 82.33 March 17 “ total credits to date - - - - - - - –— 1582/33 is 9 “| “ balance due this date sºme º sm em as sº * * as tº 7802|63 2 March 17 “ interest on same from March 17, 1891, to March 17, 1892, 1 year at 8% - 624.21 “ I”, “amount due this date - - - - - - - - - - - 8,2884 1891 May 15 By cash, (3d payment), - - - - - - - $2000.00 “ interest on same from May 15, 1891, to March 17, 1892, 1892 10 months 2 days at 8 per cent gº tº tº * 134.22 Jan. 4 “ cash, (4th payment), - - - - - - - - 1800.00 ‘‘ interest on same from January 4, 1892, to March 17, 1892, 2 months 13 days at 8% mº as as ºn 29.20 March 17 ‘‘ total credits to date cº- ºr º º * - - —— 1396342 & 4 “| To balance due this date - - - - - - - - - - - 4463|42 June 17| “ interest on same from March 17, 1892, to June 17, 1892, 3 months at 8% - 89|27 & 4 “| “ balance due on final settlement * = e º sº a ºn * 4552|69 Explanation.—Transactions of this character have not occurred with sufficient frequency to establish any real custom pertaining to the method of their solution; and hence, in the absence of both custom and law, and in consequence of there being so many different systems of partial 626 SOULE's PHILOSOPHIC PRACTICAL MATHEMATICs. * payments, accountants and authors are divided in their opinions as to which of several methods should be used in cases of this kind. For this reason we present two solutions, to the first of which we give preference. The contract of annual interest specifies the period of time that the same shall be paid, and therefore it seems reasonable that the settlements for the partial payments should be made at the same time; and in accordance with this view, we have made them at the close of each year from the date of the note. Had the interest been payable semi-annually or quar- terly, we would have made the partial payments accordingly. As shown in the operation, we first find the interest on the face of the note for one year and add it to the same, and then from this amount we deduct the pay- ment made during the year, increased by the interest on the payment from the date that it was made until the end of the year; and thus we continue until the maturity of the note, and then for the remainder of the time up to the final settlement, we proceed in the same manner as in the Merchants' System of partial payments. In transactions of this nature, where there are payments made subsequent to the maturity of the note, they would be treated the same as in the Merchants' System. In fact the only difference between the operations of this work and the Merchants' System is the different time or rests at which the partial payment Settlements are made. By this system of work both parties receive yearly compound interest. SECOND OPERATION. 1889 $ |º: March 17| To face of note - º - - - º - - - º - - 900000 1892 • June 17 ‘‘ int. on same from March 17, '89, to June 17, '92, 3 yrs. and 3 mos, at 8% 234000 & 4 “l ‘‘ amount of note at time of settlement mºs - - - º - - 11340 oo { % “| “ interest on $720 (the 1st annual interest) from March 17, 1890, to June 17, 1892, 2 years and 3 months at 8% - sº - - $129.60 & 4 “| “ interest on $720 (the 2d annual interest) from March 17, 1891, to June 17, 1892, 1 year and 3 months at 8% º as tº - 72.00 é & ‘‘ ‘‘ amount of interest on annual interest º - -> - º - - 201|60 & 4 “| “ total amount due this date -> º dº Q- - º º --> &- 11541.60 1889 Nov. 1 By cash, (1st payment), * - - - we ºw - $1000.00 ‘‘ interest on same from Nov. 1, 1889, to June 17, 1892, 2 years 7 months and 16 days at 8% e- - tº- 210.22 1890 July 10 “ cash, (2d payment), <- - º- º g- -> º 1500.00 ‘‘ interest on same from July 10, 1890, to June 17, 1892, 1 year 11 months and 7 days at 8% tº dº º 232,33 1891 May 15 “ cash, (3d payment), gº sº - tº- º tº- º 2000.00 “ interest on same from May 15, 1891, to June 17, 1892, 1 year 1 month and 2 days at 8% - - - - 174.22 1892 Jan. 4 “ cash, (4th payment), gº tº - * º * tº- * 1800.00 ‘‘ interest on same from January 4, 1892, to June 17, 1892, 5 months and 13 days at 8% tº º tº º 65.20 June 17 “ total credits to date - - - - - - - - - 6981|97 & 4 “| To balance due on final settlement as as sº gº ºn tº º º 4559|63 Explanation.—In this solution we first find the interest on the face of the note from the day of date to the day of final settlement, and then the interest on the annual Interest from the time it became due until the final settlement. Then, from the total of these three amounts We deduct the sum of the payments, increased by the interest on each from the time paid until the day of settle- ment. The remainder is what is then due, NotE.—In the foregoing operations of interest and partial payments, where the actual time was counted, 29 days were reckoned for February in all the leap years that occurred. † EQUATION OF ACCOUNTS. 627 EQUATION OR AVERAGE OF PAYMENTS AND ACCOUNTS. 1196. Equation, or Average of Payments and Accounts, is the process of finding the mean or average time for the payment, in one amount, of several sums of money, notes or debts due at different dates; or of the balance of accounts that contain both debit and credit items. The Equated Time is the date at which the various sums due at different times may be paid in one amount without injustice to either party. The Adjustment of Interest is the real object of equating or averaging payments and accounts. The principles upon which the process of the work is based are that the interest of any sum paid any length of time before it is due is equitably balanced by the interest of an equal sum paid an equal time after it is due. Hence in all equa- tions or averages, the interest on the sums paid, before due by equation, must equal the interest on the sums paid after they are due by equation. The Assumed Due Date, or Focal Date is the date at which we assume all items of debit or credit or both to be due, and from which we count the debit or credit of time in the equation. The Debit or Credit of Time is the time an item has to run from the assumed due date until it is due. The Earliest Date in an account or of the items to be equated, is used in this work as the assumed due date or focal date. NOTE.—Authors and accountants differ in regard to the date to assume as the due or focal date. Some assume the earliest date, some the latest date, some the first of the month in which the earliest date occurs and some assume a date prior or subsequent to the earliest date. The result is the same whatever the date assumed. To work from the earliest date each item of debit, except the first, is subject to bank discount ; and to work from the latest date, each item, except the latest, is entitled to interest. º The subject is divided into simple and compound equation or average. Simple Equation, or Average, is the process of finding the average time of payment for debts or accounts, which contain debit or credit items only. Compound Equation, or Average, is the process of finding the equated average time of payment for the balance of accounts, which contain both debit and credit items. Equations with Special Conditions. For equations involving special con- ditions, see pages 643 to 652 and 1009 to 1013 of this book. There are numerous methods given by various authors for the equation and averaging of accounts; but we believe that the introduction of all of them would confuse and bewilder the student rather than elucidate and make clear the operations and principles of the Work. The methods we present are universal in their application, have received the approval of practical and scientific accountants, and are short, concise and require in their use the exercise of reason instead of the memorizing of rules. 628 SouLE's PHILOSOPHIC PRACTICAL MATHEMATICs. * PROBLEMS TO FIND THE EQUIVALENT OR AVERAGE TIME FOR THE PAYMENT OF SEVERAL SUMS OF MONEY DUE AT DIFFERENT TIMES, OR WITH DIFFERENT TERMS OF CREDIT. 1197. 1. January 1, 1895, Z purchased from A goods amounting to $3000, which he agrees to pay as follows: $800 in three months; $1200 in eight months, and the remainder, $1000, in twelve months. What is the equated or average time for the payment of the whole amount, so that neither party will gain or lose by the use of money? Ans. 8 months. OPERATION BY THE PRODUCT SYSTEM. Z's Z’s Z’s 1895 Debit of Dolls. Credit of Months. Credit of Months on $1. January 1, $800 X 3 = 2400 46 1, 1200 X 8 = 9600 {{ 1, 1000 X 12 = 12000 mº-º-º-º: * Total debit of dollars $3000 Total credit of months on $1, 24000 24000 -- 3000 = 8 months. Ans. 8 months equated time. Explanation.-By the terms of the contract we see that Z is debited with $800, $1200 and $1000 in money, and credited on the respective amounts with three, eight and twelve months in time, from January 1, which is the due date of all the items; and since he is debited with various sums of money and credited with various periods of time, it is clear, by the exercise of our reason, that, in order to solve the question, we must first find Z's total credit of time on $1, from the due date. To find his total credit of time on $1, we reason thus: Three months’ credit on $800 is equal to 800 times three months' credit on $1, which is 2400 months; then eight months’ credit on $1200 is equal to 1200 times eight months' credit on $1; and then twelve months’ credit on $1000 is equal to 1000 times 12 months' credit on $1; which added gives 24000 months’ credit on $1. Having this total credit of time on $1, we then find the total amount of his debit in dollars by adding the various sums together. We then reason thus: Since Z has a credit of 24000 months’ time on $1, on $3000 he will have the 3000th part, which is eight months. Here it may be asked, why Z should Inot have 3000 times as much credit on $3000 as on $1, instead of the 3000th part 3 To this question we reply that Z's credit on $1, for 24000 months, is but an equivalent for the various terms of credit due him by the conditions of the problem; and hence, if he is to receive a credit on the whole amount, $3000, that he owes, instead of on $1. the credit allowed to produce an equivalent, to the 24000 months on $1, or the various terms of credit, according to the first conditions, must necessarily be in reciprocal proportion to the amount to which it is to apply. To illustrate the correctness of the foregoing operation, by the principles of interest, we present the following figures: By the first conditions of the problem, Z was to pay $800 in three months, $1200 in eight months and $1000 in twelve months; but as Z did not pay the $800 until the equated time, eight months, he therefore gained and A lost the interest of the same for five months, which at, say, 8 per cent, is $26.67. The $1200 Z was to have paid in eight months, and as the equated time is eight months, there is neither gain nor loss to either Z or A on the same, by reason of the equation. * EQUATION OF ACCOUNTS. 629 The $1000 Z was to have paid in twelve months; but by reason of the equa- tion, he paid it in eight months; and hence he lost and Agained the interest of the same for four months, which, at the same rate as above, 8 per cent, is $26.67. By this we see that A's gains and Z's losses, or Z's gains and A's losses are equal, and that the interest on the sums paid before they are due by equation, is equal to the interest on the sums paid after they are due by equation. This will always be the case, no matter how many sums are paid before or after the equated time. To elucidate the work of equation further, we present the following operation: 1895. Months Cr. January 1, $800 3 at 12% $24 interest. 1200 8 at 12% E '96" , 1000 12 at 12% = 120 “ $3000 $39 ) $249 interest. 8 months. Ea:planation.-By this method we see that if Z had paid the various debits of dollars, at the time of purchase, without waiting for the expiration of the various terms of credit, he would have been entitled, as shown in the operation, at 12 per cent interest, to $240 interest deduction on the $3000. But as he does not wish to pay the whole amount in advance of the whole credit, but at a period of time, so that the interest on the sums paid before due will equal the interest on the sums paid after they are due ; it is, therefore, by the exercise of our reason, plain that the time it will require the $3000, at 12 per cent, to produce $240, will be the equated time, or the time to pay the whole amount, so that Z's gains and A’s losses, or Z's losses and A’s gains, by interest, shall be equal. To find this time, we compute the interest on the $3000, for one month at 12 per cent, which gives us $30. We then reason thus: If $30 interest requires one month's time, $1 will require the thirtieth part, and $240 will require 240 times as many months, which, worked, gives eight months, the equated time. Had the terms of credit been days instead of months, we would then have found the interest on the whole sum for one day. The equated time will be the same regardless of the rate of interest used. We used 12 per cent because, in the most of cases, it is easier, and requires less time to calculate interest at 12 per cent than at any other rate. By this method of work, interest tables can be used to advantage by those who desire. The interest method, however, we do not regard with much favor. The product method, or system, is better and more easily comprehended by learners. NotE.-The accuracy of the result of the foregoing methods of work has been questioned by several good authors, and much warm discussion has been indulged in on the subject; but as the arguments advanced against the correctness of these methods are alike subtle and fallacious, we shall, therefore, be very brief in considering them. The following example and argument is generally used to elucidate the asserted incorrectness of the foregoing methods of finding the equated time: “If a man owes me $200, $100 of which is payable now, and $100 payable in two years, the equated time is not one year. For in deferring the payment of the first $100 one year, he ought to pay the $100, plus the interest, which, at 6 per cent, is $106; , but for the $100 which he pays one year before it is due, he ought to pay the present worth of $100, which, at 6 per cent, is $94.33%; and $106 -- 94.33}} = $200.33}}; whereas, by methods in general use, he only pays $200.” By this argument it is contended that I lose $6, the interest on $100, for 2 years only, and gain only $5.66, the true discount on $100 for 1 year, making a difference of nearly 34 cents. But the argument is fallacious, and the error is in supposing that I lose $6, when really I only lose the present worth of $6, for the $6 interest is not due until the close of two years, and hence I should receive $100, plus the present worth of $6, due in one year, which is $5.66–H and which added to the $100 due the first year = 105.66+ that I should receive; but as I only receive $100, I therefore actually lose $5,66–H which is just equal to the gain on the $100 received for the present worth ($94,33}}) of the $100 for one year before it was due. 63o SOULE's PHILOSOPHIC PRACTICAL MATHEMATICS. jºr 2. A owes Z $200, payable now; $500 payable in 4 months; $1000 payable in 6 months; and $1400 payable in 9 months. What is the equated time for paying all ? Ans. 6 months and 19 days. OPERATION. Debit of Dollars, Credit of Months. Credit of Months on $1. 1000 X 6 -: 60 1499 X 9 2– 126 Total debit of dollars $31 Total credit of months on $1 206 206 -- 31 = 63% months = 6 months 1944 days. Explanation.—We here see that A owes various sums of money, and has due him various terms of credit of time, and as shown in the first problem, we first find a total credit of time on $1. The reasoning for the work is the same as given in the first problem, hence omitted. Before making the multiplications, we cancel each sum of dollars by 100, and thus save figures. When the fraction of a day in the answer, or equated time, is more than }, it is customary to count 1 Whole day; and, when less than 3 to omit it. 3. A merchant owes 4 notes, dated January 1, 1895. The first is for $500, payable in 30 days; the second is for $800, payable in 40 days; the third is for $1200, payable in 90 days; and the fourth is for $2000, payable in 120 days. What is the equated time for the payment of all? Ans. 88 days, which, added to Jan. 1, 1895, the date of the notes, gives March 30, 1895, as the date to pay all at one time. Dr. of Dolls. Cr. of Days. Cr. of days on $1. $500 X 30 = 15000 800 , X 40 E 32000 1200 X 90 E 108000 2000 X 120 3– 240000 $4500 total debit of dollars. 395000 total credit of days on $1. 395000 - 4500 = 873 days. 4. A loaned Z $600 for 9 months, how long ought Z to loan. A $800 to equal the favor? Ans. 6 months, 22 days. OPERATION. Dr. of Dolls. Cr. of Months, Months Cr, on $1. 8% 54% $600 X 9 E 5400 63 months = 6 mos. 224 ds. 5. A bought of Z the following invoices: January 2, - tº 9 tº gº tº. $400 66 9, - tº tº gº gº 210 {{ 15, - tº sº tº tº 350 {{ 21, - tº sº º tº t 800 46 30, - tº tº * > tº 1100 ~ What is the equated time for paying all? Ans. January 20. º: EQUATION OF ACCOUNTS. 631 FIRST OPERATION WORKING FROM THE EARLIEST DATE. Dr. Of Dolls, Cr. of days. Cr. of days on $1. January 2, $400 X 0 F. { % 9, 210 X 7 = 1470 4 - 15, 350 X 13 – 4550 § { 21, 800 X 19 F 15200 & 4 30, 1100 X 28 F 30800 $2.860 ) 52020 (18 days. Earplanation.—In this problem the various debits of dollars were credited at different dates, 3. as there was no credit of time allowed, each item of debut is therefore due at the time of pur- CIla.S6), In this operation we assume the earliest date January 2, as the due date or focal date and then proceed to find the time that each purchase or debut item is due after the assumed due date. We reason thus: The $400 being due the day of purchase, it has 0 days credit of time. The $210 purchase on the 9th is really due that day; but presuming 1 t due on the earliest date, January 2, Z therefore owes A time from January 2 to January 9, which is 7 days’ credit on the same. The $350 bought on the 15th is really due that day; but presuming it due on the earliest date, January 2, Z owes A time from January 2 to January 15, - 13 days' credit on the same. The same reasoning applied to the $800 and $1100, purchases of the 21st and 30th, gives respectively 19 and 28 days’ credit due to A. Having thus made or presumed all the purchases due January 2, we proceed to find the credit that Z owes on $1, and then on the whole amount in the same manner as explained in the first problem of equations. The result of the equation is 18 days’ credit due A on the whole amount, $2860, which, for the reason that we make all the debit items due January 2, must be added to that date in order to give A the credit due to him; 18 days added to January 2 gives January 20 as the equated time for paying the whole amount, $2860. To more fully elucidate the work of equation, we present the following: SECOND OPERATION WORKING FROM THE LATEST DATE, IDr. Of Dolls. Dr. of Days. Dr. of days on $1. January 2, $400 X 28 = 11200 {{ 9, 210 X 21 E 4.410 “ 15, 350 X 15 - 5250 “ 21, 800 X 9 = 7200 “ 30, 1100 X 0 $28.6% ) 2806% (93.g3 days, practically 10 days. Explanation.—By working from the latest date, January 30, we see that as each debit item was due the day that it was purchased, that the $400 bought January 2 was therefore due 28 days prior to January 30; and hence A owes the time as well as the money. The $210 bought on the prior to January 30; and hence A owes the time as well as the money. The $210 bought on the 9th was due 21 days; and again A owes Z the time and money; the $350 and $800, purchased respec- tively January 15 and 21, are due respectively 15 and 9 days, for which A owes Z the time; the $1100 bought on the 30th is due that day, and hence A owes no time thereon. Having now the days’ time that A owes Z on the various items of dollars to find his debit of days on $1, and then on the whole amount that he owes, we proceed to multiply and divide, as shown in the operation, and for the reasons given in the first problem on equations. The result of the work is 10 days' debut of time that A owes Z on the whole amount of $2860; and to give Z the credit or advantage of this time, it is clear that the 10 days must be counted backward from January 30, the day that all of the debit items of dollars were due ; and by deducting the 10 days, from. January 30, we obtain January the 20th as the equated time for the payment of the whole amount. 632 SOULE's PHILOSOPHIC PRACTICAL MATHEMATICS. 6. What is the equated time for the payment of the following account 3 Ans. October 5, 1895. S. J. & M. M. KOHLMAN. To LEVY & HILLER, Dr. 1895. g 541|85 May 1 To merchandise at 2 months - - - - -> - - - 3) June 28 ‘‘ cash º º * - * - º tº- cº- - - - 210|30 August 16 “ merchandise at 3 months - - -> e- - - - sº § § September 20 “ merchandise at 3 months - - e- º -> - - - 525 - $207805 SOLUTIONS. FIRST operaTION, Assuming The EARLIEST DATE, MAY 1, To WORK FROM, Dr. of Dolls. Contract Cr. DS. Cr. Days assuming each Cr. of purchase due May 1. Ds, on $1. May 1 $541.85 61 + 00 = 61 33062 June 28, 210.30 00 + 58 = 58 12180 August 16, 800.00 92 + 107 - 199 15.20% September 20, 525.90 91 + 142 - 233 122558 $2078.05 327000 327000 days credit on $1 - 2078 = 157 days equated time for the payment of the whole amount, after May 1, 1895, which is October 5, 1895. Ea:planation.—In this problem, like the preceding, the various debits of dollars were created at different periods of time; but unlike the preceding problem, all the items, except the cash, have a credit of time allowed the purchasers. The principles involved in the preceding problems are alike applicable to the solution of this and all other questions in equations. In order to find the total credit of time due the purchasers on $1, we first find the credit that each debit of dollars is entitled to by the terms of the contract from the assumed due date, May 1, in the operation. The debit of $541.85, of May 1, has, by the conditions of the purchase, 2 months (61 days) credit. The debit of $210.30, June 28, has no contract credit of time; but because we presume it due May 1, it has from May 1 to June 28, 58 days credit from that presumed time. The debit of $800, August 16, has 3 months, 92 days contract credit; and because we assumed it due May 1, it has, from May 1 to August 16, 107 days additional credit, making 199 days credit. The debit of $525.90, of September 20, has 3 months, 91 days contract credit; and because we presume or assume it due May 1, it has, from May 1 to September 20, 142 days additional credit, making 233 days credit from May 1. Having thus the credit of time for each item of debit of dollars, we proceed to find the equated time, the same as in the preceding problems, with this exception: In multiplying the first and last terms of days and dollars, we used, respectively, $542 and $526, instead of the exact amounts, $541.85 and $525.90; and in the second term we multiplied by $210, instead of $210.30; and in the division, we omitted the cents in the diviso. . In practice the gents need not and by accountants, are not used in equation operations of multiplying and dividing. Whenever they exceed fifty the amount is increased by $1; and when- ever they are less than fifty, they are omitted. By operating thus with the cents, time and labor are saved without effecting any perceptible difference in the result. The result of the operation is 157 days credit due the purchasers on the whole amount, $2078.05, after May 1, the assumed date that each debut of dollars was due. This added to May 1, gives October 5, 1895, as the equated time. * EQUATION OF Accounts. 633 SECOND OPERATION, Assum ING THE LATEST DATE, SEPTEMBER 20, 1895, As THE & DUE OR FOCAL DATE. Dr. of Contract Dr. Cr, Dr. of Ds. Cr. of Ds. Dolls. Cr. of Ds. of Ds. of Ds, on $1. on $1. May 1, $541.85 61 *- 142 +E 81 43902 June 28, 210.30 00 *- 84 2– 84 17640 Aug. 16, 800.00 92 *- 35 57 45600 Sept. 20, 525.90 91 * 0 91 47.866 $2078.05 6151, 93466 Debit of days on $1 . º - - ºn tº º - - 61542 Excess of credit of days on $1 is º * = º - - - 31924. 31924 - $2078 = 15.4, days, practically 15 days. . Explanation.—In this operation we assume the latest date in the account to work from and reason thus: The $541.85 purchased May 1, not being settled for until September 20, the purchasers, therefore, owe time on the same from May 1 to September 20, which is 142 days; this we set in the column of debit days, and deduct therefrom the 61 days credit allowed by the terms of the contract, which leaves an excess of 81 debit days. Then the $210.30 of June 28, not being settled for until the assumed date, September 20, the purchasers owe time on the same from June 28 to September 20, which is 84 days; this we set in the column of debit days, and having no contract credit to deduct therefrom, it remains a net debit of days. Then the $800 debit of August 16 not being settled for until September 20, the purchasers owe time on the same from the date of purchase to the assumed date of settlement, which is 35 days; this we also set in the column of debit days, and deducting the same from the 92 days credit allowed by the contract of purchase, we have an excess of 57 credit days, which we extend to the column of credit days. Then the $525.90 debit of September 20 being by assumption settled on that day, there is no debit of time, and hence the whole credit of days, 91, allowed by the terms of the agreement, is extended to the column of credit days. Having now found the various debits and credits of days on the different debits of dollars, we next find, by multiplication, the debit and credit of days on $1, and then the difference, which is 31924 days credit on $1; this we divide by the whole amount of debit dollars, and thus obtain 15 days, the equated time which, for #. pºly given, is counted forward from the assumed date, and gives October 5, 1895, as the final TeSult. 7. A owes Z $2000 due January 1, 1895; May 1, 1894, he pays Z $800; when is the balance due # Ans, June 12, 1895, OPERATION. $2000 800 245 days credit on $800 (from May 1, 1894, to January 1, 1895,) = 196000 tº-º-º-º- days credit on $1. $1200 1200 ) 196000 1634 days credit on $1200 after January 1, 1895. Ea:planation.—When the debtor makes partial payments on the amount that he owes before it is due, it is clear that he is entitled to interest on the money paid, or to a credit on the balance after it becomes due, which is equal to the credit on the amount paid before it is due. 8. A is owing Z $2000, which is due by contract July 4, 1894. On the 14th of February, 1894, A wishes to pay Z such a sum as will justly extend the payment of the balance until October 7, 1894. What amount must he pay, and what amount will remain unpaid” Ans. $808.51, to be paid. $1191.49, to remain unpaid. From Feb. 14, 1894, to July 4, 1894, are 140 days. From July 4, 1894, to Oct. 7, 1894, are 95 “ 235 ($ 634 SouLE's PHILOSOPHIC PRACTICAL MATHEMATICS. * Here we see that the time from the date of payment (Feb. 14) to maturity of the whole sum (July 4) is greater than the time from the maturity to the date of the extension, Oct. 7; and as the credit of days on $1 is in direct proportion to the amount of dollars multiplied by the credit of time thereon, it is clear that we here require a less sum to be paid before due than to remain unpaid after it is due, to produce equivalent credits; hence, the amount to be paid before maturity and also the amount to remain unpaid after maturity will be in reciprocal proportion to the two periods of time, 140 and 95 days. The following statements produce the desired reciprocal results: OPERATION OPERATION To find the amount to be paid before maturity. To find the amount to be paid after maturity. $ $ 2000 2000 235 | 95 235 | 140 $808.51 Ans. $1191.49 Ans. To show the whole work of reciprocal proportion involved in the foregoing statement, we present the following: — —l— — — 23 – + + 1.9 = Hº = Tº 1 - 95 = GF = Tºššū —235– 1 3 3 00 1. By this we see that the reciprocal of 140 is ++o, and of 95 it is ºs, which added give rāšāo. With these reciprocal proportional numbers, we make the fol- lowing solution statements: OPERATION OPERATION To find the amount to be paid before maturity, To find the amount to be paid after maturity. $ $ 2000 2000 235 | 13300 235 | 13300 13300 | 95 13300 || 140 $808.51 Ans. $1191.49 Ans. To show that the credit of time on $808.51 for 140 days, is equivalent to the credit of time on $1191.49 for 95 days, we present the following: Cr. of Ds. Cr. of Ds: on $1. $808.51 X 140 = 113191+ 1191.49 X 95 F. 113191+ $2000.00 NoTE.—To those accustomed to work on the interest system, we could say that, in the main, the same process of reasoning as given above, would be used. Where we use the term eredit of day8, the interest method of reasoning would require the term credit of interest. 9. Sold merchandise to LACY & RogERs, as follows: 1895. 1895. Feb. 1, $500.00 at 60 days. May 21, $200.00 at 00 days. March 18, 1019.68 “ 90 “ July 28, 400.00 “ 60 “ May 6, 704.25 4 30 % ſº When does the account mature by equation? Ans. June 13, 1895. 2}. EQUATION OF ACCOUNTS. 635 OPERATION BY FIRST METHOD, 1895. Dr. of Dolls. Cr. of Ds. Cr. of Ds. on $1. 60 Feb. 1, $500.00 + 0 = 60 = 30000 March 18, 1019.68 90 + 45 = 135 = 137700 May 6, 704.25 30 + 94 - 124 = 87296 6é 21, 200.00 00 + 109 F 109 S- 21800 July 28, 400.00 60 + 177 F. 237 F. 94.800 $2823.93 f 371596 371596 days credit on $1 – $2824 = 132 days credit on $2823.93, which added to February 1, the assumed maturity of each item of debit, gives June 13 as the equated maturity of the account. 10. Suppose in the above problem that the purchasers, LACY & RogFRs, had desired on August 1, to settle the account by a “cash” note, payable November 1/4, 1895, allowing interest at 8 per cent, for what face must the note be drawn? Ans. $2916.91. OPERATION INDICATED. Interest on $2823.93 from June 13 to August 1, = 49 days at 8 per cent = $30,75. $2823.93 + $30.75 = $2854.68, due August 1, and for which a cash note at 8 per cent is given payable November 1/4, -95 days, + discount day, = 96 days. 4500 × $2854.68 + (4500 – 96) = $2916.91, face of cash note. C. B. LOTT & CO. To CAHILL & LEIDENHEIMER Dr. 1896. Jan. 20|To merchandise at 4 months gº cº sº {- º sº t- - wº- gº 1500|00 Feb. 15|| “. { { • 3 “ & tº- s £º sº sº &- s {- sº 840|75 March 9| “ $ 4 • * 4 44 * º tº * sº sº sº &= tº- s 200|28 April 1| “ acceptance “ 2 “ (with grace), º •- sº e & - 500|50 3041|53 11. May 1, LOTT & Co. settle by note drawn for such a time as will mature at the equated date of the account. For how many days will the note be drawn Ans. 23 days without grace, or 20 days with grace. NotE 1.-In the operation, count actual days and allow for leap years. NoTE 2.—The equated date is 125 days after January 20 = May 25, less 1 day for leap year = May 24th. 12. Suppose, in the above problem, that LOTT & Co. had settled the account May 1, by paying cash, what amount would they have paid, money worth 8 per cent? Ans. $3025.98. OPERATION INDICATED. Since, as shown in the above problem, the equated date for the payment of the account was May 24, therefore if the same is paid on May 1, the amount is to be discounted for 23 days. $3041.53 $3041.53 amount. Thus: 45 || 23 15.55 discount. $15.5455–H discount. $3025.98 net proceeds. 636 soul E's PHILOSOPHIC PRACTICAL MATHEMATICS. * REMARKS.–It will be observed that in all of our work, where dates were given, we have counted the actual days intervening between two dates. This we believe to be the only correct system. Many accountants however, to save a little time, count 30 days to the month, by which the final result is often changed. To find the time between dates and also in extending the equated time forward or counting it backward from the date assumed, whether the earliest or latest, a Time Table is often used to advantage. See Time Table on page 298. COMPOUND EQUATIONS, OR AVERAGE OF ACCOUNTS. 1198. When the equated time is counted forward and when it is counted backward. - In all compound equations of payments or average of accounts, when the balance of dollars and the balance of days are on the same side of the account (the debit or the credit) the equated time is counted forward. When the balance of dollars and the balance of days are on the opposite sides of the account, the equated time is counted backward. An account with the earliest date on the debit side and the final balance of dollars and balance of days on the debit side, equated time is counted forward when the earliest date is assumed, and backward when the latest date is assumed as the maturity of the dollar items. 1. What is the equated time for the payment of the following account? Ans. July 17, 1895. I) r. PATTON, SWIFT & Co. Or. 1895. 1895. Mar. 5. To mdse., at 60 days - $80000|April |10| By cash, - A- sº- $50000 April 22 “ “ 90 “ * 200000|May 28 “ “ - - - 1000|00 June | 1 | * * * * * * 30 ** tº- 1200,00|Aug. 9| “ “ - - - 800|00 “ [20] ‘ ‘ & 4 - 400|00 “ balance, * * * 2100.00 \ $4400|00 sºoooo FIRST OPERATION Assuming the earliest date in the account to work from. 1895. ; º; %" º | 1895. ºf "5" pººl. Mar. 5, $800 60 + 0 = 60 48000 April 10, $500 36 18000 April 22, 2000 90 + 48 = 138 276000 || May 28, 1000 84 84000 June 1, 1200 30 + 88 = 118 141600 || Aug. 9, 800 157 125600 “ 20, 400 00 107 42800 $4400 508400 $2300 227600 2300 227600 Bal. of $ 2100 Bal. of days 280800 280800 + 2100 = 134 days credit due PATTON, Sw1FT & Co. on the balance Yºr EQUATION OF ACCOUNTS. 637 $2100, and which, for reasons given in the first problem, is counted forward from the assumed maturity of all the debit and also credit items of dollars, and gives July 17, 1895, as the equated time for settling the balance of the account. Explanation.—In this problem, PATTON Swift & Co. have both a debit and credit of dollars, and also a debut and credit of days, but by our method of work, the solution is not in the least complicated by reason of the credit of dollars and debit of days. The operation is necessarily longer, but the two, and only two principles that we use the (debit of dollars and credit of days) are as applicable to questions of this character as to those containing a debit of dollars and credit of days only. In the main, the operation of this problem is the same as in the preceding examples. We first assume the earliest date, March 5, as the due date of all the items of debit and credit of dollars on both sides of the account, and then found, as is fully explained in the preceding work, and adjusted the proper credit and debit of days on each item of debit and credit of dollars, accord- ing to the conditions of the contract and the assumption of March 5, as the maturity of all the dollar items of the account. We thus produce a net credit of 280800 days on $1, due to PATTON, SWIFT & Co., which, as shown in the operation, is equal to 134 days on the balance, $2100 that they OW €, SECOND OPERATION Assuming the latest date in the account to work from. gº . Of Cr. of Cr. of Cr. of 1895. §§ Gººf H.” f pº ºl. 1895. ići. B. DS. ºil. Mar. 5, $ 800 60 – 157 – 97 77600 Apr. 10, $ 500 121 60500 Apr. 22, 2000 90 – 109 = 19 38000 || May 28, 1000 73 73000 June 1, 1200 30 — 69 = 39 * 46800 || Aug. 9, 800 0 “ 20, 400 00 — 50 = 50 20000 $4400 182400 $2300 133500 2300 133500 Bal. of $ 2100 Bal. of days 48900 48900 - 2100 = 23 days that PATTON, SWIFT & Co. owe, August 9, on the balance $2100, and to give the parties to whom this 23 days credit is due, the advantage of it in the settlement of this account, it is clear by the exercise of our reason that we must count backward from August 9, which gives us July 17, 1895, as the equated time. Having in the last problem and in preceding work on equations illustrated the principles and operation of work by assuming either the earliest or latest date as the maturity of all the debit and credit items of dollars, and preferring the method of assuming the earliest date, to save space, we shall therefore give but one solution of subsequent problems, and that by the earliest date method. An account with the earliest date on the credit side, the balance of dollars and days on the debit side, equated time counted forward from the earliest date, 2. What is the equated date of the balance of the following account 3 Ans. Jan. 6, 1896. Dr. ... WEIL, HONOR & Fox. Cr, 1895. 1895. Oct. 4|To Mdse., at 2 months || $250000|Sept. 8|By Cash, $ 80000 Nov. 1| “ Cash, s ge 300000||OCt. 20 % {{ †º 1500|00 Dec. 18 “ Acpt. at 1 month 100000|| 4 ||25 | {{ {{ sº 20000 NOV. 14 “ “ tº 5 70000 638 SOULE's PHILOSOPHIC PRACTICAL MATHEMATICs. ºr OPERATION. Dr. of Cr. of * Cr. of Cr. of Dr. . of 1895. #ji. #." D.I. 1895. §. *.” p.º. Oct. 4, $2500 61 26 = 87 217500 I Sept. 8, $ 800 0 Nov. 1, 3000 0 54 162000 || Oct. 20, 1500 42 63000 Dec. 18, 1000 34 101 = 135 135000 “ 25, 200 47 9400 Nov. 14, 700 67 46900 $6500 514500 $3200 119300 3200 119300 $3300 bal. of dolls. 395200 bal. of days. 395200 + 3300 = 11933, practically 120 days. Ea.planation.—This 120 days credit of time on the balance of dollars, is due to WEIL, HoNor & FOX; hence the time of payment is extended 120 days after the due date. An account with the earliest date on the debit side, the balance of dollars and days on the credit side, equated time counted forward from the earliest date. 3. When is the balance of the following account due 3 Ans. Feb. 21, 1895. I}r. WOODS & WINN. Or. 1895. 1895. Jan. 21|To Cash, - - sº $1420,00||Feb. 1|By Cash, - $500000 Feb. 1| “ Mdse., tºº - 180000|Mar. [10] {{ {{ - 700000 April 7| “ “ at 2 months, 950|40 OPERATION. Assuming the earliest date as the maturity of the debit and credit items. Dr. of Cr. of Cr. of ſº Cr. of Dr. of IDr. Of 1895. łºń. 3. Dººl. 1895. ji. is.” pººl. Jan. 21, $1420.00 0 Feb. 1, $5000.00 11 = 55000 Feb. 1, 1800.00 0 11 19800 || Mar. 10, 7000.00 48 = 336000 Apr. 7, 950.40 61 76 -: 137 130150 $4170.40 149950 $12000.00 39.1000 4170.40 149950 Bal of dollars, sº,60 Bal. del, 21030 241050 days -- 7830 ($7829.60) gives 31 days. Explanation.—This 31 days time WooDs & WINN owe to the parties with whom they have this account, on the balance $7829.60, and as this balance of money is due to them, it is clear that, in order to settle the 31 days time, the payment of the balance must be extended 31 days after January 21, the assumed maturity of all the dollar items of the account, which gives February 21. An account with the earliest date on the debit side, the balance of dollars on the debit side and balance of days on the credit side, equated time counted backward from the earliest date. 4. What is the equated time for the payment of the balance of the following account Ans. Aug. 5, 1894. * EQUATION OF ACCOUNTS, 639 Dr. SPENCER & FOLSOM. Or. 1895. 1895. July |1|To Mdse., at 60 ds. $1200,00|July 22|By Cash, $ 50000 Aug. 10 “ {{ {{ 60 4 130000|Aug. 10 “ { % 60000 Sept. 25. “ {{ {{ 30 % 110000|| “ |10| “ Acpt. at 30 ds. 50000 Oct. 6| 44 {{ {{ 10 % 2000|00||OCt. 6| “ Cash, 1500|00 Dec. 1| “ Note at 90 ds. 200000 OPERATION. Working from the earliest date. Dr. Of Cr. Of Cr. of Cr. Of Dr. Of Dr. of 1895. #. Sº p. 1. 1895. ji. *.” Dººl. July 1, $1200 60 0 = 60 72000 || July 22, $ 500 21 10500 Aug. 10, 1300 60 40 = 100 130000 || Aug. 10, 600 40 24000 - Sept. 25, 1100 30 86 = 116 127600 “ 10, 500 33 40 = 73 36500 Oct. 6, 2000 10 97 = 107 214000 || Oct. 6, 1500 97 145500 Dec. 1, 2000 93 153 = 246 492000 $5600 543600 $5100 708500 5100 543600 $500 balance of dollars. Balance of days, 164900 164900 days divided by 500 = 330 days. Explanation.—By the operation of this problem, we find that SPENCER & FOLSOM owe both a balance of dollars and a balance of days, and hence, by the exercise of our reason, we see that in order to settle or pay in kind the 330 days, the balance of dollars must be made due 330 days prior to July 1, the assumed due or focal date of all the debit and credit items of dollars. 330 days º backward from July 1 gives August 5, 1894, as the equated time for paying the balance of Olla,I’S. By the foregoing problems and illustrations, we see, for reasons given in the solutions, that, when the balance of dollars and balance of days occur on the same side of the account, whether the debit or the credit, the equated time is counted forward from the earliest date; and that, when the balance of dollars and balance of days occur on opposite sides of the account, the equated time is counted backward from the earliest date. An account with the earliest date on credit side and balance of dollars on credit si:3, and balance of days on debit side, equated time counted backward from the earliest date. 5. What is the average date of the following account 3 Ans. April 10, 1895. Dr. LACY & ROBINSON. Or. 1895. 1895. July | 6|To Mdse., 90 days, $210000|June 14|By Cash, $5441|00 Aug. 30. “ {{ 90 ($ 50000|July 28 “ “ 300000 Sept. 18. “ (4 60 (£ 20000|Aug. 1| “. “ 300|00 Oct. 1} {{ (< 30 % 90000 64O soule's PHILosophic PRACTICAL MATHEMATICS. & OPERATION. Dr. of Cr. Cr. Of * Cr. of Dr. of Dr. of 1895, #. “Bº” pººl. 1895, Éli. “5,” Dººl. July 6, $2100 90 22 == 112 235200 June 14, $5441 00 Aug. 30, 500 90 77 = 167 83500 || July 28, 3000 44 132000 Sept. 18, 200 60 96 = 156 31200 || Aug. 1, 300 48 14400 Oct. 1, 900 30 109 = 139 125100 $3700 475000 $8741 146400 146400 3700 Bal. of days, 328600 $5041 bal. of dolls. 328600 days -- 5041 = 65 days. Explanation.—By the operation of this problem, we find that LACY & ROBINSON, have due to them both a balance of days and a balance of dollars, and hence, in order to give them credit for the 65 days due to them, we must pay the balance of money 65 days before the assumed date, June 14, and consequently the time is here counted backward from June 14. 6. A owes B $1600, due in 6 months. A pays $800 in 2 months. What is the equated date for paying the balance? Ans. 10 months. FIRST OPERATION. $1600 × 6 months = 9600 = months credit due A on $1. 800 × 2 months = 1600 = months debit due by A on $1. 800 8000 = months credit balance due A on $1. 8000 -- $800 = 10 = months credit due A on $800. SECOND OPERATION. Dr. of Dolls. Cr. of Mos. Cr. of Mos. on $1. Cr. of Dolls, Dr. of Mos. I)r. of Mos. on $1, $1600 X 6 ~ 9600 $800 X 2 2- 1600 800 1600 $ 800 8000 Explanation.—In the operations of the above problem, we first observe that while A owes B $1600 in money, B owes A 6 months time to pay the money. And 6 months credit on $1600 is equal to 9600 months credit on $1. ... Then, as A paid $800 in 2 months B owes A for the $800, and A owes B for the 2 months credit on $800, which equals 1600 months credit on $1. The balance of account i. º A debit $800 and credit 8000 months on $1. 8000 - 800 = 10 months credit on the ala,D C6 g 7. A owes B a note dated March 10, 1895, due in 183 days for $1200. June 14, 1895, A paid on account $300. What is the equated time for paying the balance, counting the actual days in each month ? Ans. Oct. 8, 1895. FIRST OPERATION. $1200 x 183 = 219600 = days credit due A on $1. 300 × 96 = 28800 = days debit that A owes B on $1. $ 900 190800 = balance of days credit due A on $1. 190800 - 900 = 212 days credit due A on $900. And 212 days after March 10, 1895, – Oct. 8, 1895. See Time Table. ti Explanation.—From March 10 to June 14, is 96 days. See above problem for further explana- 1OLl, º: EQUATION OF ACCOUNTS. 641 SECOND OPERATION. Dr. of Dolls. Cr. of Ds. Cr. of DS. on $1. Cr. of Dolls, Dr. of Ds. Dr. of Ds. on $1. Mar. 10, $1200 183 219600 June 14, $300 96 28800 300 28800 $ 900 190800 190800 - 900 = 212 days credit on $900 after March 10, 1895, as Oct. 8, 1895. 8. May 14, 1895, A owes B $800 due in 4 months and $500 due in 3 months. A paid on account $300 in 1 month and $100 in 3 months. When is the balance due by equation ? Ans. 4 months 17 days. OPERATION INDICATED. Dr. of Dolls. Cr. of Mos, on $1. Cr. of Dolls. Dr. of Mos, on $1, $ 800 X 4 E. 3200 $300 X 1 F 300 500 X 3 - 1500 100 X 3 – 300 $1300 4700 $400 600 400 600 $ 900 4100 43 mos. = 4 mos. 16; d.s. 9. May 14, 1895, A owes B $800 due in 4 months, and $500 due in 3 months. A paid on account $700 in 4 months and $500 in 6 months. When is the balance due # Ans. June 14, 1894. OPERATION INDICATED. $ 800 X 4 - 3200 $700 X 4 - 2800 500 X 3 - 1500 500 X 6 ~ 3000 $1300 4700 $1200 5800 1200 4700 $ 100 1100 1100 + $100 = 11 months, i. e. the $100 balance was due 11 months before the date of May 14, 1895, which is June 14, 1894. In other words A owes B interest on $100 for 11 months. Hence the time is counted backward. 10. May 14, 1895, A owes B $800 due in 4 months and $500 due in 3 months. A paid $900 in 4 months and $600 in 5 months. What is the equated date for the settlement of the balance due A { Ans. Feb. 29, 1896. OPERATION INDICATED. $800 X 4 = 3200 $900 X 4 = 3600 500 X 3 - 1500 600 X 5 -: 3000 $1300 4700 $1500 6600 1300 4700 $ 200 1900 1900 + $200 = 94 months, after May 14, 1895, - Feb. 29, 1896. Explanation.—The result of the operation shows that B owes A $200 and that A owes B 94 months credit on the same, from May 14, 1895. Hence while Bowes A the $200 he need not pay it until 94 months after May 14, 1895. It must not be forgotten that all items of debit and credit are assumed to be due on the earliest date. 642 soul E's PHILOSOPHIC PRACTICAL MATHEMATICS. *e AN ACCOUNT EQUATED AND SETTLED BY NOTE. 1199. 1. The following bills were sold to TYLER & MILLs on a credit of 90 days each : January 4, 1895, - $824.15 May 21, 1895, - - - $718.15 February 19, 1895, - 371.68 June 7, 1895, - - - 2120.80 April 1, 1895, - 1410.00 The following payments were made on the above purchases: March 1, 1895, - - $600 cash. May 10, 1895, - $800 cash. April 1, 1895, - - 350 “ June 8, 1895, - 1000 “ What is the equated time for paying the balance of the account? Ans. Oct. 4, 1895. OPERATION. Dr. Of Cr of Cr of Cr. of Dr. of Dr. of 1895. #ji. “Bº” D.'...'. 1895. ji. "Bº" pººl. Jan. 4, $824.15 90 O - 90 74160 || Mar. 1, $600 56 33600 Feb. 19, 371 68 90 46 + 136 50592 || April 1, 350 87 30450 April 1, 1410.00 90 87 + 177 249570 || May 10, 800 126 100800 May 21, 718.15 90 137 = 227 162986 || June 8, 1000 155 155000 June 7, 2120.80 90 154 = 244 517524 $5444.78 * 1054832 $2750 3.19850 2750.00 3.19850 $2694.78 bal. of dolls. 734982 bal. of days. 734982 days -- 2695 = 273 days, which counted forward from J anuary 4, 1895, gives October 4, 1895, as the equated time. 2. Suppose in the above problem that TYLER & MILLs had, on the 8th of June, given their note due at the equated date for the balance due, for how many days would the note have been drawn 3 Ans. 118 days without grace, or 115 days with grace. NOTE:-The equated date being October 4, 1895, we simply find the intervening time between June 8 and October 4, which is 118 days. 3. Suppose again, in problem No. 1, that TYLER & MILLs had settled June 8, the balance due in cash, allowing 8 per cent for the use of money and not count- ing discount day, what sum would be required to pay the balance? Ans. $2624.12; which is called the cash balance. OPERATION. $ Fº Eaplanation.—The equated maturity of the balance being Oct. 4, 2694.78 1895, it is clear that if TYLER & MILLs pay it June 8, they should 45 || 118 pay be allowed the interest on the same for the intervening time, 118 | $70,66 days, which is, as shown by the opposite statement, $70.66. EQUATION OF ACCOUNTS. 643 4. May 10, 1895, A owes Z $3000, due in 3 months. June 4, 1895, A paid Z $2000 on account, when is the balance due # Ans. Dec. 22, 1895. OPERATION. 1895. Ds. DS. on $1. 1895. DS. Ds. on $1. May 10, $3000 92 = 276000 June 4, $2000 25 = 50000 2000 50000 $1000 ) 226000 226 ds. EQUATION OF ACCOUNT SALES. 1200. An Account of Sales is a statement of the articles and price of merchandise or other property sold, the charges incurred in effecting the sales, and the net proceeds due the party or parties for whose account the goods were sold, and to Whom the account sales is rendered. The net proceeds are due as cash at the equated date of the different sales and charges. When equating account sales, it is usual to consider the freight and drayage due on the receipt of the goods. The commission is often considered not due until the last day of sale, but some accountants equate the time for the charge of commission. In large sales, or where there is a long interval of time between sales, the commission should be equated. The other charges are sometimes considered due on the last day of sale and sometimes on the day that each item was paid, or the average date. In case of account sales for merchandise sold on joint account, the interest of the consignee is sometimes considered due at the date of shipment and sometimes at the date the goods were received. In justice to both parties, and in order to preserve a uniformity in the accounts of each party interested in the goods, this matter should be agreed upon at the time the parties contract for the joint account operation. The consignee's gain or loss in joint account transactions is generally, and should always be made due, the same as the commission, at the equated date of the sales. NOTE,--It is the custom of cotton factors in New Orleans, to consider the proceeds of cotton sales due 7 days after the sale was made. This 7 days use of the proceeds is considered a just compensation for the interest on the money advanced for charges, and saves the time and labor of equating the account sales. Flour and produce dealers consider the proceeds of goods sold on commission due ten days after the sale was made. This allowance is made in place of equating the account sales, and is considered just to both parties. 644 SOULE's PHILOSOPHIC PRACTICAL MATHEMATICS. PROBLEMIS. 1. When are the net proceeds of the following consignment due? Ans. Nov. 4, 1895. No. 1820. Sales by W. H. MoRGAN & Co. of 1200 bbls. flour, received Oct. 11, 1895, per steamer Susie Silver, for account of SCHREIBER & EICKE, Belleville, Ill. Oct. |12|Sold 100 bbls. Sucker City, cash, a $8 * * * * * * * 80000 & 4 15| “ 200 “ & 4 & 4 4 & Ø $74 sº sº &- wº tº- <- * 1575.00 “ “| “. 284 “ & 4 “ 20 ds., a $7; tº s tº m s tº º 2130|00 & 4 21| 44 8 * * & 4 ‘‘ cash, (a) $8+ tº ess º º º & &- 66100 “ |26 “ 200 “ & 4 “ 10 ds., a $7+ * dº •-º * tº º <-º 1500|00 ** 29 “ 408 “ & & “ 15 ds., a $7; sº *- º wº sº wº *- 3060|00 1200 $9131|00 CHARGES. ‘Oct. 11|Freight, 40 cts. per bbl., $480; Charges per B. L., $240, - tº sº 72000 ‘‘ 29|Labor, $12; Tarpaulins, $6; Inspection, $24, º g- * : tº tºº 4200 “ “|Drayage, $49; Storage, $54; Insurance, Fire $21.84—River $67.50, - 192|34 “ “[Skidding, $1.60; Cooperage — tº gº tº gº tº ſº sº 1|60 Nov. 2 Commission on sales at 24 per cent, - - - * † tº º sº 228/28|| 1184|22 Net proceeds to your credit due by equation Nov. 4, 1895, - - - $7946|78 E. & O. E. NEW ORLEANs, November 1, 1895, WM. H. MORGAN & CO., per F, HORN. OPERATION. 1895 Ds Ds. 1895. Ds Ds. Ds. ‘Oct. 11, $720.00 0 Oct. 12, $800.00 0 1 800 “ 29, 235.94 18 4248 “ 15, 1575.00 0 4 6300 Nov. 2, 228.28 22 5016 “ 15, 2130.00 20 4 = 24 51120 “ 21, 66.00 0 10 660 “ 26, 1500.00 10 15 = 25 37500 “ 29, 3060.00 15 18 = 33 100.980 $1184.22 9264 $9131.00 197360 1184,22 9264 $7946.78 188096 188096 - 7947 = 24 days, which counted forward from October 11, gives November 4, 1895, as the equated date. Explanation.—In this equation we considered the freight and drayage due on the receipt of of sale. & 4 & 4 ( & & 6 9 15, 21, 26, 29, OPERATION Ds. Ds. $ 800 0 1575 3 = 2130 20 + 3 = 23 = 66 9 = 1500 10 —- 14 = 24 = 3060 15 + 17 = 32 = $9131 188229 -- $9131 = 21 days. This equation gives us 21 days, which counted forward from October 12, gives November 2, as the equated date of the sales. 4725 48990 594 36000 97920 188229 To find the equated date of the sales only, at which date the commission is due. Oct. 12, & 4 1 the goods, the commission on the average date of the sales, and the other charges on the last day EQUATION OF ACCOUNTS. 645 EQUATE THE FOLLOWING ACCOUNT SALES ON JOINT ACCOUNT. 2. Sales of 1000 bbls. potatoes, received per steamer R. E. Lee, Nov. 2, and sold on the joint account of the shippers, SPENCER & FREDERICK, and ourselves, each 3, by MOSES, LEHMAN & Co. 1895 - Nov. 3|Sold 300 bbls. Peach Blows, cash, (a) $3 º - - - solo & 4 9| “ 450 “ Neshannock, 20 ds., a $3.20 - - * - 1440|00 “ [15] “ 60 “ 4 & cash, @ $3} º - * - 195|00 “ |28 “ 50 “ Pink Eyes, 10 ds.; (a) $3} * - sº - 175100 Dec. 4 “ 140 “ Peach Blows, 15 ds., (a) $3} - - º 490|00 1000 $3200.00. CHARGES, Nov. 2 Freight, 40 cts. per bbl., - - - - - - - - - 400|00 Drayage, * - - - tº- - º - wº º º - 50|00 Insurance, - - º - - -> tº - sº - º - | 21|50 Storage, $51.25; labor, $8.50, - * †- - wº - - - 59.75 Commission (a) 2% per cent, dº - - - gº tº ºn - 8000 611|25. Net proceeds, - * - º - s - º - tº $2588.75 Cost of in voice, * - &= - º - sº - º 2000|00. Net gain, - - - - - - - - - - - - $588.75 Our # net gain, - - - º - º - * * * - $294,37 Spencer & Frederick's # net gain, - tº - * - sº º $294.38 4 & $ & # invoice, tº tºp tº- tº - gº - 1000 00 & 4 ( & net proceeds, * - sº º ºs º- 1294.38 (exclusive of $ invoice previously credited,) due by equation Dec. 19, 1895, * * * ºs & & - *> - sº -> E. & O. E. NEW ORLEANs, December 5, 1895. MOSES, LEHMAN & CO., Per LOUIS LEVY. OPERATION. 1895. Ds. Ds. 1895. Ds. DS Ds. Nov. 2, Fr’t & Dray. $450.00 0 Nov. 3, $900.00 1 900 Dec. 1, Ins., Sto., &c. 81.25 29 2349 ‘‘ 9, 1440.00 20 7 – 27 *38880. Nov. 24, Commission '80.00 22 1760 “ 15, 195.00 0 13 = 13 2535 “ 24, Our # gain 2.94.37 22 6468 “ 28, 175.00 10 26 = 36 6300 “ 2, “ , inv. 1000.00 0 Dec. 4, 490.00 15 32 = 47 23030. $1905.62 10577 $3200.00 71645 1905.62 10577 $1294.38 61068. according to agreement. account sales does not give the date of purchase. 61068 - $1294 = 47 days. Explanation.—In this operation we have considered the freight and drayage due the day that the produce was received. The other charges, except commission, we considered due December 1, as the greater portion of them must by custom have been paid then, if not paid when they were respectively incurred. Strict justice would require the equated date of charges in all account sales, but in practice it is not always done. The commission and gain are due on the average date of the sales, and the 3 invoice the day that the produce was received, or the date of purchase We have here considered the # invoice due the date received, as the 646 soul E's PHILOSOPHIC PRACTICAL MATHEMATICS. Yºr OPERATION To equate the sales only, at which date the commission and gain are due. 1895. Ds. Ds. Ds: Nov. 3, $900 0 “ 9, 1440 20 + 6 = 26 = 37.440 “ 15, 195 0 -- 12 = 12 = 2340 “ 28 175 10 + 25 = 35 = 6125 Dec. 4, 490 15 + 31 = 46 = 22540 $ 68445 68445 -- $3200 = 21 days. NoTE.-In the equation of account sales for joint account transactions, in which losses are sustained, they are entered on the credit side of the account in the operation of equation. ACCOUNT SALES. 1895 Jan. 3|Sold for cash, 500 bbls., Ex. Flour at $9. tº- - - - - $450000 & 4 8|Sold at 10 days, 100 bbls., Flour at $8.50, - •- - - - 850|00 $535000 CHARGES. Jan. 2|Sundry charges, - - - - - - - - - - $290|50 & 4 8|Commission on sales at 2% per cent, as sº-, * > • * 133|75|| 424/25 Net proceeds, - - - * º wº sº - -- º * $4925||75 What is the equated date of the proceeds in the above account sales? Ans. Jan. 6, 1895. OPERATION. Jan. 2, Charges, - - $291 0 = 0 || Jan. 3, Sales, * - $4500 1 = 4,500 ‘‘ 8, Commission, - - 134 6 = 804 “ 8, “ sº - 850 16 = 13,600 $425 804| $5350 18,100 425 804 17296 -- $4925 = 4 days, after Jan. 2, = Jan. 6/95. $4925 17,296 SALES OF MIDSE. ON JOINT ACCOUNT. Folio. 1895 COST AND CHARGES. Jan. 4|Received, per Steamer Golden Rule, from W. B. Henry, to be sold for his and our joint account, each #: 400 bbls. Mess Pork at $20, * - * *- - - $8000 4|Com. Sales Dr. to W. B. Henry, our # º * - - - 11 400000|| 4000|00 To Charges: & 4|Cash for Freight, - - - - - - - - $120 | C. 1 4| “ for Drayage, - - - - - - - -> 24 o Insurance, Fire, g- - º - - - - --> 20 ( & River, * - - - - - º tº- 180 Inspection, $12; Cooperage, $3, - * * * º 15 7 359|00|| 35900 Com. S. Dr.—Our 3 and charges, - s º º - - 11 || 4359|00 11 To Commission on Sales at 2% per cent, - tº - - - 11 226 25 226,25 “ W. B. Henry's # Inv. me * * * *-> * $4000 & 4 do # Net Gain, -> - - - - 232.38 11|W. B. Henry's Net Proceeds, - º -> º- - - - 11 4232|38|| 4232.38 Due by Equation February 7/95: 11|To P. and Loss, our # Net Gain, - º º º- - - 12 232|37 232;37 11|Com. Sales Dr. to close account, º º tº e dº tº- - 11 4691|00 905000 * SALES ON Joint Account. 647 SALES OF THE ABOVE MDSE. Folio. 1895 Jan. 6W. J. Vizard, Cash.|C. 1 100 bbls. Mess Pork (damaged) a $15, - - - - - 150000 9|S. J. Blessing, Note 15 ds. 100 bbls. Mess Pork a $26, E* - * . gº sº º Q- 2/11 2600|00 11|Geo. W. Weingart, Jr., 10 ds. 200 bbls. Pork a $24.75, - tº º †- gº tº sº gº 8/11 495.000 905000 When are W. B. Henry's net proceeds due # Ans. Feb. 7, 1895. OPERATION. Jan. 4, § Inv., & Chgs., $4359 0 = 0 1 Jan. 6, Sales, $1500 2 F 3,000 “ 11, Com & Gain, 459 7 = 32.13 “ 9, “ 2600 23 —r 59,800 *=====s== “ 11, “ 4950 17 - 84,150 $4818 3213 sºme sº-mºme *= º ºsmºs $9050 146,950 4818 3,213 143737–$4232=34 days, after Jan. 4, which is Feb. 7/95. $4232 143,737 SALES ON JOINT ACCOUNT. Folio. 1895 COST AND CHARGES. Jan. 20 Received, per Steamer J. M. White, from D. Hughes, to be sold on the joint account of himself, J. W. Moses and ourselves each # : 500 bbls. XX Flour (a) $7.50 = $3750. To which we have added from store: 100 bbls. Molasses (a) $18 = $1800 Cr. Mdse., - tº- •ºm t- 1 D. Hughes, Cr, for our # of his Inv., & e tº sº gº & 12|| 1250,00|| 1250.00 Com. Sales, Dr. for our # of our Inv., * * * * * V 60000|| 60000 D. Hughes, Dr., for # of our Inv., $600. tº º * * * *- 12 J. W. Moses, Dr., for # of our Inv., $600. º, tº sº º 12|| 2 h Charges : 20|Cash for Freight, $80; Drayage, $30, * tº ºn age º C, 1|| 11000 Labor, $2; Inspection, $10, * * * * * * * 1200 Insurance, Fire, * tº- 3- º * > Gºe • * ge *E* 1800 do River, &= * $º sº * - sº wº sº º 7 7500|| 21500 Com. Sales, Dr. for our # cost and charges, - - - - 11|| 2065!00 29|Commission on sales a 2, per cent, sº sº sº sº we 11|T10555|| 10555 29|Cash refunded to W. J. Vizard— Amount allowed on damaged { C. 2 20000 Molasses, gº º wº- 4- <-> gºe º * Ge. * * Charges : 29|Storage, $10; Cooperage, $4, - - - - - - - 7 14|00 14|00 D. Hughes, # of his Inv., i- se <--> & * * * $1250.00 £ 6 # of our Inv., - *- * - ſº &º sº 600.00 $1850.00 Less his # Net Loss, 4-6 iº ** º ſº * * tº 620.95 29|D. Hughes' Net Proceeds, Cr. ——|| 12|| 122905] 1229,05 Due by Equation Feb. 6/95: 29|J. W. Moses, of D. Hughes' Inv., - º tº sº $1250.00 do # of our Inv., sº tº º * * sº 600.00 $1850.00 Less his # Net Loss, - gº tº e tº tºº Gº 620.95 J. W. Moses, Net Proceeds, Cr. sº º ºs º º sº 12|| 1229.05|| 122905 Due by equation Feb. 6/95. * -º 29|Com. Sales, Dr., to close account, - - - tº dº ſº 11|| 257765 4842.65- 648 SOULE's PHILOSOPHIC PRACTICAL MATHEMATICs. * SALES OF THE ABOVE MDSE. Follo. 1895 * Jan. 29|W. J. Wizard, Cash. 500 bbls. XX Flour (a) $5, - s ‘Es & gº * - * -; 250000 100 bbls. Molasses, 3826 gallons, a 45c., *E. * * g- C. 2 || 1721||70|| 4221||70. 29|By Profit and Loss for our # Net Loss, - dº $º sº tº- 12/11 620.95. 4842|65, When are the net proceeds of D. Hughes and J. W. Moses, due by equation in the above account 3 Ans. Feb. 6/95. OPERATION. Jan. 20, # Inv. & Chgs, $2065 0 = 0 || Jan. 29, Sales, $4222 9 – 37,998 “ 29, Com. & Chgs., 320 9 – 2880 “ 29, § Loss, 621 9 = 5,589 $2385 2880 $4843 43,587 2385 2,880 $2458 )40,707(17 ds- 40707 - 2458 = 17 days, after Jan. 20/95, - Feb. 6/95. EQUATION OF RENT NOTES. 1201. 1. January 1, 1895, A. leases of Z. a store for 1 year, at $500 per month, amounting to $6000, on the following conditions: The rent to be paid monthly in advance, as follows: The first month's rent is to be paid in cash, and monthly maturity rent notes given for the remaining 11 months rent. After the lease is signed, A. proposes to give one note for the whole rent payable at such a time that neither party will gain nor lose by the change of terms; to this proposition Z. agrees. What is the time for the payment of the new note? Ans. June 16, 1895. FIRST OPERATION.—BY Months WITHOUT AL- SECOND OPERATION.—BY ACTUAL DAYS AND AL-. LOWING FOR GRACE. LOWING FOR 3 DAYS OF GRACE ON EACH NOTE. Mos. Mos Cr. on $1. Ds. Ds. Cr. on $1. $500 0 0 C $500 0 O 500 1 500 1 500 33 16500 500 2 1000 2 500 61 30500 500 3 1500 3 500 92 46000 500 4 2000 4 500 122 61000 500 5 2500 5 500 153 76500 500 6 3000 6 500 183 91500 500 7 3500 7 500 214 107000 500 8 4000 8 500 245 122:500 500 9 4500 9 500 275 137500 500 10 5000 10 500 306 153000 500 11 5500 11 500 336 168000 $6000 ) 33000 $6000 } 1010000 5% months. June 16, ’95. º 1683 days. June 18, '95. Yºr EQUATION OF RENT NOTES. 649 2. Suppose in the above problem the lessee had desired to give, and the lessor had consented to receive, two notes of equal amount, payable in periods of time in the ratio of 1 and 4, what would have been the time of each note, allowing three days of grace on each of the 11 notes? OPERATION. As shown in the second operation above, A., the lessee, has a credit of 1010000 days on $1, on the presumption that the $6000 are due January 1, 1895. Now, by the terms of the problem, we must find the respective credits on the two new notes so that the total credit on $1 shall be equivalent to º days on the 11 notes which he first proposed to give. This we do by the following Statements: 1st note, $3000 × 1 = 3000 I day. | 4 ds. 2d note, 3000 x 4 = 12000 15000 || 1010000 15000 || 1010000 15000 | 67% ds. = 2 mos. 7# ds, 269; d.s. = 8 mos. 29% ds. PROOF. 1st note $3000 × 67; d.s. = 202000 ds. credit on $1. 2d note $3000 × 269; d.s. = 808000 ds. credit on $1. 1010000 total ds. credit on $1. The aggregate days credit on $1 is the same as shown above on the eleven notes. See pages 651 and 652 for full discussion of thus kind of work. EQUATION OF ACCOUNTS AND NOTES BEARING INTEREST. 1202. 1. A. owes Z. $500 due March 8, 1895, and bearing interest at 8 per cent; $1700, due May 17, 1895, and bearing interest at 6 per cent; and $350, due June 1, 1895, and bearing interest at 8 per cent. What is the equated time for the payment of the whole debt? Ans. May 3, 1895. OPERATION. 1895 Ds. Mar. 8, $ 500 8% F $ 4000 0 May 17, 1700 696 = 10200 70 714000 June 1, 350 8% = 2800 85 238000 $17000 ) 952000 952000 days -- 17000 = 56 days. 2. A. holds Z's note, due August 10, 1895, for $5000, bearing 8 per cent interest from maturity, and Z. holds A's acceptance, due October 24, 1895, for $14000, bearing interest at 5 per cent from maturity. What is the equated time for paying the balance? Ans. Feb. 1, 1896. OPERATION. 1895, DS, Ds. 1895. Ds. Ds. Aug. 10, $5000 8% = $40000 0 0000 || Oct. 24, $14000 5% = sº 75 = 5250000 4000 $30000 ) 5250000 ******mº 175 ds. 65o SOULE's PHILOSOPHIC PRACTICAL MATHEMATICs. † 5250000 days -- 30000 = 175 days, which counted forward from August 10, 1895, gives February 1, 1896, as the equated date for the payment of the balance. By this equation, we see that the $5000 due August 10, 1895, was paid 175 days after it was due; and that the $14000, due October 24, 1895, was paid 100 days after it was due. Interest on $ 5000 at 8% for 175 days = $194,44 “ “ 14000 “ 5% tº 100 “ = 194,44 3. A. holds notes and acceptances against Z. as follows: His note due Jan. 28, '95, for $2000, bearing 8% interest. {{ 4% 4 Feb. 4, '95, & 4 4200, {{ 6% 46 “ acpt. “ Mar. 10, '95, “ 7000, “ 6% “ 44 note “ “ 31, ’95, “ 2500, {{ 7% {{ Z. holds the following claims against A. : His note due Feb. 14, '95, for $2400, bearing 5% interest. “ acpt. “ Mar. 21, ’95, “ 3000, “ 8% “ “ note “ April 9, '95, “ 5000, without interest. What is the equated time for paying the balance of the above account, allow- ing 8 per cent interest on the balance? Ans. March 5, 1895. OPERATION. 1895. 1895. Jan. 28, $2000 8% = $16000 0 = Feb. 14, $2400 5% = $12000 17 = 204000 Feb. 4, 4200 6% = 25200 7 = 176400 || Mar. 21, 3000 8% = 24000 52 = 1248000 Mar. 10, 7000 6% = 42000 41 = 1722000 || April 9, 5000 0% = 00000 71 = 0000000 “ 31, 2500 796 = 17500 62 = 1085000 $15700 $100700 2983400 $10400 $36000 1452000 10400 1452000 $5300 a 8% = $42400 ) 1531400 1531400 + 42400 = 36 ºr days. Explanation.—By the operation we obtain practically 36 days, which counted forward from January 28, gives March 5, 1895, as the equated date for the payment of the balance, allowing 8 per cent interest on same. NotE.—The operation also shows a debit of 1531400 days on $1, at 1 per cent, and as the §. of the balance of $5300 multiplied by the 8 per cent and the 363& days just equals 1531400 ays, it is clear that the equated time is March 5, 1895. EQUATION OF NOTES, 65 I EQUATION OF NOTES MATURING AT DIFFERENT DATES, AND THE SUBSTITUTION OF OTHERS OF DIFFERENT AMOUNTS AND MATURING AT EQUAL INTERVALS OF TIME. 1203. 1. A. owes Z. three notes bearing date, and drawn for the amount and time respectively, as follows: August 10, 1895, $2000 payable 90 days after date; September 12, 1895, $1200 payable 6 months after date; and November 25, 1895, $4800 payable 1 year after date. A. wishes to take up or redeem these three notes with four others of equal amounts and maturing at equal intervals of time, so that there will be no loss or gain to either party. What is the time of each new note * OPERATION. 1895 Dr. Of Contract Total Cr. Cr, of Ds. <- Dolls. Cr. of DS. of Ds. on $1. Aug. 10, $2000 90 -- 0 = 90 180000 Sept. 12, 1200 182 + 33 = 215 258000 Nov. 25, 4800 366 -- 107 = 473 2270.400 4 ) $8000 2708400 $2000 each new note. - Having now found the number of days credit due A. on the three old notes, from August 10, 1895, the day that we assume them all due, we have next to find the respective number of days credit on each of the 4 new notes in proportion to 1, 2, 3, and 4, as required by the conditions of the problem, so that the total credit of the same shall be equivalent to the 2708400 days on the 3 old notes. We therefore, in order to produce the necessary proportional numbers to make a solution state- ment, proceed as follows: Ds. Cr. assumed. Ds. Cr. on $1. 1st New Note $2000 1 2000 2d “ & 4 2000 2 4000 3d 44 & 4 2000 3 6000 4th ** & 4 2000 4 8000 20000 By this work we obtain 20000 days credit on the 4 new notes, and as the assumed credit of 1, 2, 3, and 4 days gave this 20000 days, by transposition, we see that the 20000 days credit required the 1, 2, 3, and 4 days time, and as we wish to distribute 2708400 days credit on $1, on the time of the 4 new notes, the following proportional statements will give the correct time of each note. Statement for the 1st Note. Statement for the 2d Note. Statement for the 3d Note. Statement for the 4th Note. E). DS. DS. DS. 1 2 3 4 2708400 20000 | 2708400 20000 || 2708400 20000 || 2708.400 135%; | 270}}}}} *406 fºr 541}#}}} 652 - soule's PHILOSOPHIC PRACTICAL MATHEMATICS. A. The fractions of a day in the result cannot be avoided, and to use them as equitably as the conditions of the problem will admit of, we omit the fractions in each result and increase the whole numbers of the 2d and 4th results, 1 day each. The reasoning for the first statement is as follows: If 20000 days credit on $1 require 1 day's time, or a note of $2000 payable 1 day after sight, 1 day's credit will require the 20000th part, and 2708400 days credit will require 2708400 times as much. For each of the other statements the reasoning is the same, except that we have respectively, 2, 3, and 4 days assumed time on the 2d, 3d, and 4th notes. Instead of making these 4 statements, we could, in this problem, have multi- plied the result of the first by the assumed numbers, 2, 3, and 4, and thus have obtained the result. This method of work is applicable to all questions of this character, regardless of the proportion of time that the new notes may be desired to run. To elucidate still further that the credit on the new notes is equivalent to the credit on the old notes, we present the following: --- DS. Cr. on $1. 1st New Note $2000 135 days 270000 2d 44 & 4 2000 271 “ 542000 3d 44 6 & 2000 406 “ 81.2000 4th 84 & 4 2000 542 “ 1084000 Total credit on $1, 2708000 4 & & 4 “ $1, of old notes, 2708400 Excess of credit on old notes, 400 NOTE.-The small excess of credit on the old notes results from the fact that we did not and cannot use the exact time as shown in the four statements above. In the first and third notes the small fraction of time was omitted. While in the second and fourth notes the fractions of time being each more than one-half day, they were each counted as one day. The small excess thus Fººd is only equal to one-fifth of a day on one of the new notes, which is too small a credit to © U186 ~##–Z ^2(5¢)\ \ º Aunt Current and Interest AC(OInts, aid Cash BālāICES. =N Aº Aº-º. w = w wr 1204. When it is desired to settle an account by cash, or to find the balance that is due at any specified date, for the purpose of closing the books or determining the full resources and liabilities of the firm, one of two methods, the interest or the product, is generally used. THE INTEREST METEHOD. 1205. The first and most usual method is to find the interest on each item of debit and credit of dollars from the time it became due up to the date of Settle- ment, or balancing of the accounts, and then charging or crediting the debtor with the balance of interest, according as it is against him or in his favor. TEIE PRODUCT METEIOD. 1206. The second, and next most usual method, is to find the balance of interest due the debtor or creditor, by first finding the balance of the product of the debit and credit of days on $1, the same as in equation of accounts when work- ing from the latest date, and then dividing the same by the interest divisor, and thus obtaining the interest, which is charged or credited to the debtor as in the first method. º In our problems and operations, we will illustrate both of these methods and also a new method by which the operation is shortened and the accountant facili- tated in the discharge of his duties. THE EQUATION METEOD. 1207. This is a new method which we name the Equation Method, for the reason that we assume all of the dollar items due on the earliest date in the account, and proceed with all the debit and credit items of dollars, the same as we worked equations when we assumed the earliest date to work from. PROBLEMIS. 1. What is the cash balance of the following account, December 31, 1895, interest at 8 per cent ? Ans. $1531.42. IDr. RENCH & WURZLOW. Or. 1895 1895 Feb. 4To Mdse. (a) 90 days, - - - $ 50000||Mar. [10|By Cash, - - - - - - - $ 30000 Mar. 28 “Cash, - - - - - - - 120000|April 25 “ Acpt. 2 60 days, - - - 500|00 May | 3| “ Mdse. a 30 days, - - - 70000||June 7| “ Cash, - - - - - - - 40000 Sept. 14 “ “ (a) 30 days, - - - 150000|||Oct. |19| “ Proceeds of flour due by equation October 22, 1250|00 (653) 654 SOULE's PHILOSOPHIC PRACTICAL MATHEMATICs. Yºr FIRST OPERATION BY THE INTEREST METHOD, RENCH & WURzLow in Account Current and Interest Account at 8% to December 31, 1895, with MITCHELL & BRANDNER. Dr. Cr. Yºº ps. at Amt Yº..." |Ds.| Int. || Amt. ji 1895. | 1895. Feb. 4 To Mdse. a 90 ds. May 5240|| 2667|$50000|Mar. 10By Cash, Mar. 10296, 1973|$30000 Mar. 28 “Cash, Mar. 28278|| 74|13||120000|Apr. 25 “ Acpt.(a) 60.ds.|June 27|187| 2078||50000 May |3| “Mdse. a 30ds.|June 2212|| 3298|| 70000|June 7 “ Cash, ** | 7207 1840|| 40000 Sept]14 “ “ “30 “ Oct. 14 78| 2600||150000|Oct. 19 “ Proceeds of flour, due by 15978 equation Oct. 22 Oct. 22 70|| 1945||1250.00 Dec. 31. “Bal. of int. 78.36|| 8142 78.36 Dec. 31|By bal. due to date, 1531|42 398142 398142 Explanation.—In this solution we first find, after maturing the debit and credit items of dol- lars, the number of days that each debit and credit item of dollars has been due prior to December 31, 1895, and then compute the interest on the same for the time at 8 per cent. We then take the difference between the debit and credit of interest, and as the excess or balance is against Rench & Wurzlow, we charge it to them, and then to find the cash balance we take the difference between the debit and credit amounts of the account. SECOND OPERATION BY THE PRODUCT METHOD, RENCH & WURzlow in Account Current and Interest Account at 8% to December 31, 1895, with MITCHELL & BRANDNER. I}r. Or. DR. DR. CR. CR. | Wºn DS. Prod'ct || Amt. Yººn LS Prod’ct || Amt. 1895. 1895. Feb. 4 To Mdse. a 90ds.|May 5240||120000|50000|Mar. |10|By Cash, Mar. 10.296 88800|| 30000 Mar, 28 “Cash, Mar. 28278||333600||120000|Apr. 25 “ Acpt.(a 600s. June 27|187| 93500|50000 May 3 “Mdse. (a) 30 ds. June 2212||148400|| 70000|June 7 “ Cash, ** | 7207|| 82800|| 40000 Sept14 “ “ (a 30 “|Oct. 14 78||117000||150000|Oct. 19 “ Proceeds of flour, due by Dec. 31.No. ds. int. on $1, 719000 equation Oct. 22 Oct. 22 70|87500||125000 352600 tºº Dec. 31|No. ds. int. on $1, 352600 “ 31|To Bal. of days |366400 “ 31|By Balance due interest on $1 to date, 1531|42 which divided by the 8 per cent int. div- isor 4500 gives $81.42 int. 8142 } 398142 398142 Explanation.—In this solution, we first mature all of the debit, and credit items of dollars, and then find the number of days that each item of dollars on the debit and credit sides of the account has been due prior to the day of settlement, December 31. Having these number of days, we proceed thus: Taking the first transaction on the debit side, we see that the $500 was due May 5, 240 days before December 31, and hence Rench & Wurzlow owe interest on the same for 240 days, or on $1 for 500 times 240 days, which is 120000 days. In this manner we continue with all of the * ACCOUNTS CURRENT AND INTEREST ACCOUNTS. 655 debit items, and produce a debit of 719000 days interest on $1 against Rench & Wurzlow. We then pass to the credit side, and in the first transaction we see that Rench & Wurzlow paid $300, 296 days before December 31, and hence they have interest due them thereon for 296 days, or on $1 for 300 times 296 days, which is 88800 days. In this manner we continue with all of the credit items and produce a credit of 352600 days on $1 in favor of Rench & Wurzlow. We then take the differ- ence between the debit and credit of days interest on $1, and find the balance to be 366400 days interest that Rench and Wurzlow owe on $1. This 366400 we divide by the 8 per cent interest divisor, and obtain $81.42 interest, which is charged against Rench & Wurzlow, and the account balanced as in the first solution. THIRD OPERATION BY THE EQUATION METHOD. RENCH & WURZLOW in Account Current and Interest Account at 8% to December 31, 1895, with MITCHELL & BRANDNER. Dr. Or, CR. DR. - I).R. - - CR, DS. |Prod'ct || Amt. Ds. |Prod'ct Amt. 1895. * 1895. - Feb. |4|To Mdse. a 90 ds. -- 0 90 45000|50000; Mar. |10|By Cash, 34 10200|| 30000 Mar. 28 “Cash, 52 62400||1200.00 Apr. 25 “ Acpt. (a) 60 ds. --80|143| 71500|50000 May 3| “ Mdse. a 30 ds. -- 88||118, 82600|| 70000|June || 7 “ Cash, 123| 49200|| 400|00 Sept 14 “ “ a 30 ds. --222,252.378000||150000; Oct. |19| “ Cash proceeds of flour due by equa- No. of ds. Cr. on $1, 568000||390000 tion Oct. 22, 260|325000||1250.00 No. of ds. Dr. on $1, 934400 $3900 - 2450 tºº-º-º: - Bal. of Dr. ds. on $1, 366400 ** 455900||245000 which – the 8 per Dec. 31|By $1450 × 330 478500 cent interest divisor - 4500 gives balance No. of ds. Dr. on $1. 934400 of interest 8142||Dec. 31|By Balance due to date, 1531|42 - 3981|42 3981|42 | Explanation.—In this solution, we first assume the earliest date as the maturity of all the debit and credit items of dollars, and then allow an equivalent credit of days on the purchases, and charge the corresponding debit of days on the payments, in the same manner as we worked equation of accounts, when we assumed the earliest date as the maturity of all the dollar debit and credit items. By this work we obtain a credit of 568000 days on $1, in favor of Rench & Wurzlow, and a debit of 455900 days on $1 against Rench & Wurzlow. It should be borne in mind that the debit and credit of days are shown on the sides of the account, reverse to the debit and credit of dollars; i. e., the debit of days on the credit side of dollars, and vice versa. These debits and credits of days on $1, represent the contract credit of days on the various transactions, including the debit and credit of days obtained by assuming all the dollar items due on the earliest date, February 4, and hence, in effect, justly make both the debit and credit of all the dollar items due on the earliest date, February 4; and as we settle or close the account Decem- Ber 31, it is clear that Rench & Wurzlow owe interest on $3900, from February 4, to December 31, 330 days, which is equal to 1287000 days on $1; and that they have due to them, by Mitchell & Brandner, interest on the money paid, $2450, from February 4, to December 31, 330 days, which is equal to 808500 days on $1, and when deducted from the 1287000 days that they owe, leaves a bal- ance of 478500 days that they still owe on $1. º It is obvious that instead of taking the difference between the debit and credit of days, we could have added the respective numbers to the debit and credit of days first found. By this elu- cidation, we see clearly how the balance of days is obtained. * © But in practice, and in the operation of this account, to save time and labor, instead of mul- tiplying both the total debit and credit of dollars by the days intervening between the earliest date and the date of settlement, we first find the difference or balance between the debit and credit of dollars, and then multiply the same by the intervening time, and thus produce the correct balance of debit or credit of days. In this account the difference is, $3900–$2450 = $1450, which multi- plied by 330 days, the intervening time, gives the 478500 days debit on $1, which we add to the debit of days obtained by the first work, and thus obtain a total debit of 934400 days interest on $1; Having now the total days interest on $1 that Rench & Wurzlow owe to Mitchell & Brandner, and that Mitchell & Brandner owe to Rench & Wurzlow, we take the difference and obtain a balance of 366400 days interest on $1, that Rench & Wurzlow owe, which, we divide by the 8% interest divisor, 4500, and obtain $81.42. This interest we charge to Rench & Wurzlow, and balance the account as in the preceding solutions. 656 SOULE's PHILOSOPHIC PRACTICAL MATHEMATICs. * THE EQUATION METHOD presents advantages which no other method can. It is shorter than most other methods, and identical with equation of payments when the earliest date is assumed as the maturity date of the dollar items. By the Interest and Product Methods, the interest or product on any item of dollars cannot be found until the day that the account is settled or closed, or the date for closing determined, and by reason of this, accountants are pressed with Work at stated periods of rendering accounts or of closing books, even when “mak- ing out” individual accounts. But by the Equation Method the products may be Computed on all of the debit and credit items of dollars at the pleasure of the book- keeper, and when the account is settled or closed, one subtraction, one multiplica- tion, and one division operation is performed, and the balance of interest is obtained. By this method, should it be desired to equate the account after the interest products are found, one subtraction and one division operation accomplishes the object as shown by the following operation: No. of days Cr. on $1 is 568000 Debit, $3900 No. of days Dr. on $1 is 455900 Credit, 2450 Credit balance, 112100 Debit balance, $1450 112100 - 1450 = 77; days, which added to Feb. 4, gives April 22, the equated date. Inter- est on $1450 from April 22, to Dec. 31, = 253 days, is $81.52. This gives 10 cents excess of interest over the former methods and results, from the fact that we did not use the ºth of a day in the equated date, and hence the 253 days used in computing the interest were too many by 38th of a day. 2. What is the cash balance of the following account, September 1, 1895, interest at 8 per cent * Ans. $14075.37. I)r. ROBERSON & BRICE. -- Cr. 1895. 1895. Mar. [21]To Mdse., - - - - 150000 Jan. 24|By Cash, - & tº º 500000 May 16 “ Cash, - - - - 210000 || Mar. 8 “‘ ‘‘ * - - - 750000 June || 4 || “. “ - se - - 50000 || Apr. 28 “ “ - - - - 1200|00 July 3| “ Mdse. (2 4 months - 3200|00 June | 1 | * * * * - º ºs º- 1400|00 “ [11] “ Cash, - gº - sº 400|00 July |10| “ Acpt. (a) 90 days, - - 8000|00 Aug. 20 “ Mdse. (a) 2 months, º 1800.00 FIRST OPERATION BY THE INTEREST METHOD. ROBERSON & BRICE in Account Current and Interest Account at 8% to September 1, 1895, with SIMION & EHREN. IDr. Cr. | Yº..." |Ds.|| Int. || Amt. Y." |Ds. Int. || Amt. | 1895. | |, |1895. Mar. 21 To Mdse., Mar. 21164|| 54.67|150000|Jan. 24. By Cash, Jan. 24220,24444|| 500000 May 16 “ Cash, May 16108||5040 210000|Mar. || 8 || “. Mar. 8|17729500|| 750000 .June 4. ‘‘ ‘‘ June 4, 89| 989|| 50000|Apr. 28 “ “ Apr. 28.126 3360|| 1200.00 July 3, “ Mdse., a 4 mos. Nov. 3 63| 4480|| 320000|June 1| “ . “ June 1| 92. 28.62|| 140000 ‘‘ [11] “ Cash, July 11, 52|| 462|| 40000|July 10 “ Acpt. (a) 90 Oct. 11 40 7111| 800000 Aug.20 “ Mdse., a 2 mos. Oct. 20 49| 1960|| 180000 days, Dr. of red interest $64.40 | Total credit Cr. of red of interest, 601|66 interest 71.11 ! Total debit of ** Bal. Of | ! interest, 126.29 int. in red ——" 671 By bal. of cre- | | dit of int. 4.7537 |12629 Sep. 1|To Balance due this date, | 140753 | 235753 e 2357537 4% ACCOUNTS CURRENT AND INTEREST ACCOUNTS. 657 Explanation.—In this account, we have what is called “red interest,” by which is meant interest which appears by reason of the method on either the debit or credit side of the account when injustice it belongs and will be carried to the opposite side. , The transactions of July 3 and August 20, on the debit side, and of July 10, on the credit side of the account, did not mature until after the date of settlement, and hence it is clear that the interest on each, from the day of settlement until the maturity of these transactions, belongs to the reverse side of the account, and to facilitate the addition of the column without including the same, and also to facilitate transfer- ring it to the side to which it belongs, it is generally entered in redink. In this account, not being able to print with red ink, we have indicated the numbers that the accountant would have made red, by printing them in italic figures. Instead of thus indicating by red, the interest that appears on one side when it belongs to the other, some accountants have a debit and credit column for interest on both the debit and credit sides of the account. * SECOND OPERATION BY THE PRODUCT METHOD. ROBERSON & BRICE in Account Current and Interest Account at 8% to September 1, 1895, with SIMON & EHREN. I)r. Or. DR. DR. CR. CR. Yºn DS. Prod'ct || Amt. When Ds. |Prod'ct. || Amt. 1895. 1895. * Mar. 21. To Midse., Mar. 21164|246000|| 150000|Jan. 24. By Cash, Jan. 242201.100000|| 500000 May 16 ‘‘ Cash, May 16108||226800|| 210000|Mar. 8 “ “ Mar. 81.77||1327500|| 750000 Junel 4 “ M. Mºunt 4| 89|| 44500|| 50000 #. * #. *:: ; #}} July 3. “ Se.. (2) U1.D.6) UIIlê y months, Nov. 3| 63|201600|| 320000|July 10 “ Acpt. (a 90 “ 11| “ Cash, July 11, 52| 20800|| 40000 days, Oct. 11 40|| 330000|| 800000 Aug.20 ** M dse. (a) 2 g months, Oct. 20 49|| 88200 1800.00 No. of ds. Cr. ‘‘ Balance of on $1, 2707500 prod. in red, 30200 No. of ds. Dr. on $1, 568300 568300 Bal, of ds. Cr. * on $1, 2139200 which divid- ed by the 8% Sep. 1|To Balance due int. ii. 456% this date, 1407538 gives 475.38 2357538 |2357538 Explanation.—By the Product Method of work, we have “red products” instead of “red inter- est,” which are treated in every respect the same as “red interest,” and for the same reasons as given in the solution of this problem By the Interest method. By reason of the fractions of a cent interest in the operation by the Interest Method, we obtain 1 cent more interest by this Method, than by the Interest Method. 658 SOULE's PHILOSOPHIC PRACTICAL MATHEMATICS. THIRD OFERATION BY THE EQUATION METHOD, Roberson & BRICE in Account Current and Interest Account at 8% to September 1, 1895, with SIMON & EHREN. I)r. Cr. CR. IDR. DR. CR. Ds. |Product. || Amt. DS. |Product. || Amt. 1895. 1895. *=-º-º-º- Mar. 21|To Mdse., 56 84000|| 150000|Jan. 24|By Cash, 0 500000 May 16 “ Cash, 112. 235200 || 2100|00 || Mar. 8 ‘‘ ‘‘ 43| 322500|| 750000 June || 4 | ** ** 131|| 65500|| 50000|Apr. 28 “ “ 94| 112800|| 1200.00 July 3| “ Mºise. a 4 months June | 1. ‘‘ ‘‘ 128, 179200|| 140000 (123 days), 283 905600|| 320000|July 10 “ Acpt. a 90 ds. (93 “ [11] “ Cash, 168| 67200|| 400|00 plus 167), 2602080000|| 800000 Aug. 20 “ Mdse. (a) 2 months (61 days), 269| 484200|| 1800|00 26.94500||2310000 Sept. 1|By Bal. of days Cr. 1841700|| 9500|00 on $1, 2139200 Sept. 1|By $13600 mult. by 2992000 which divided by 220, the 8% int. div. “ 1|No. of ds. Cr. on $1, 4833700 4500 gives 4.7538 4833700 “ | 1|To Bal. due this date, 1407538 - 2357538 2357538 Explanation.—By the Equation Method, we have no red interest or red products. The reason ing for the work is the same as given in the first problem by the Equation Method, and hence is omitted. 3. What is the cash balance of the following account, October 24, 1895, interest at 6 per cent 3 Ans. $1704.35 I)r. A. DOERR. Or. 1895. º - Jan. 1|To Mdse. (3) 60 days, - - $50000 Mar. 10 “ “ (no credit), - - 80000 Apr. 1| “ Cash, * º º- - 15000 Sept. 20 “ Mdse. (a) 60 days, - - 200|00 OPERATION BY THE EQUATION METHOD, Dr. A. DoERR in Account Current and Int. Acct. at 6% to Oct. 24, 1895, with A. Rivier. Or. CR. IDR. T)R. CR. Ds. Prod'ct || Amt. DS. Prod'ct || Amt. 1895. º- 1895, smm- Jan. | 1 |To Mdse. (a) 60 ds, 60 30000|50000|Oct. 24|By $1650 × 296, No. of Mar. 10 “ “ (no credit), 68 54400|| 80000 ds. Cr. on $1, 488400 Apr. 1| “ Cash, 90 13500|| 15000 “ 24|By Bal. due this date, 1704|35 Sept. 20 “ Mdse. (a) 60 ds. 322 64400|| 20000 Oct. 24|No. of ds. Cr. on $1, 162300||1650|00 “ 24|To bal. (326100) of ds. Dr. on $1, which—the 6% int. divisor gives 54|35 1704|35 1704|35 * ACCOUNTS CURRENT AND INTEREST ACCOUNTS. 659 4. What is the cash balance of the following account, December 31, 1895, interest at 10 per cent? Ans. $5549.72. I)r. C. ENDERLIN. Or. - T1895. Oct. 8|By Cash, - - - - $300000 Nov. 6| “ Dft, a 30 days, - - 50000 Dec. |10| “ Acpt. (3) 60 days, - 2000|00 OPERATION BY THE PRODUCT METHOD, C. ENDERLIN in Account Current and Int. Acct. at 10% to Dec. 31, 1895, with HAGAN & HENDERSON. Dr. Or. DR. DR. CR. CR. Yº..." |Ds. Prodetſ Amt. Y." |Ds. * Amt. 1895. 1895. Dec. 31|To Bal. due this Oct. 8By Cash, Oct. 8 84|252000|300000 date, 554972 Nov. 6 ‘‘ Dft. (a) 30 ds.|Dec. 9 11000 º 1896. pe 10 “ Acpt.a 60ds.|Feb. 11 42|| 84000|200000 No, of ds. Cr. on $1. 263000 No. of ds. Dr. on $1 (red), 84000 Bal. of ds. Cr. on $1, 179000 which div, by the 10% int. div. 3600 gives 49.72 5549 H 5549.72 AN ACCOUNT WITH SPECIAL CONDITIONS OF DISCOUNT FOR CASH PAYMENTS FROM WEHICH AN ACCOUNT CURRENT AND INTER- EST ACCOUNT AND AN EQUATION OF ACCTS. ARE MADE. 1208. J. McDonald bought of John Gunn, on the following dates and con- ditions: Jan. 3, 1895. $237.65 on 90 days, or 3% discount for cash in 5 days. “ 17 { { 122.50 On 60 :* 3% { { ( & ( & {{ “ 24, “ 324.00 on 66 “ 3% “ “ “ . “ 31, “ isoo) on 30 “ 3% “ “ “ . Feb. 10, “ 200.00 on 130 “ 1% “ “ “ “ & 4 15, & 4 100.00 On 120 & 4 3% & 4 ( & & 4 & 4 “ 24, “ 75.85 met { { J. McDonald paid cash on account of said purchases as follows: January 10, 1895, $150.00. January 21, 1895, $100.00 April 5, 1895, $300.00. REQUIRED: 1. What is the balance due July 1, 1895, at 6 per cent interest? 66O soul E's PHILOSOPHIC PRACTICAL MATHEMATICS. º: 2. If equated, what would be the equated date for paying the balance of the account 3 REMARKS. It will be observed that there was a conditional discount privi- lege on each bill purchased. But since the purchaser did not avail himself of said privilege by paying cash within the specified time, the privilege became inoperative and cannot be considered in the solution of the problem. Special or adjunct conditions and terms of contracts, are not binding upon either party thereto, unless strictly conformed to by both parties. OPERATION BY THE PRODUCT SYSTEM TO FIND BALANCE DUE JULY 1, 1895. J. McDONALD in Account Current and Interest Account with JoHN GUNN, interest at 6% to July 1, 1895. I}r. Or. DR. DR. CR. CR. Y.* Ds.|Prod'etl Amt. Whº Ds. |Prod'ot| Amt. 1895, º 1895. * Jan. 390 ds, or 3% for Jan. 10|Cash, - - 172|| 25800|| 15000 cash in 5 ds. ||Apr. 3 89| 21182||23765 “ 21 “ * > tº 161|| 16100|| 10000 “ 1760 ds. or 3% for Apr. 5 “ tº use 87| 26100|| 30000 cash in 5 ds. |Mar. 18105||12915|| 12250 “ 2460 ds. or 2% for No. ds. Cr. on $1, 68000 cash in 5 ds. “ 25 98|| 31752|| 32400|July |1|By Balance due, 66524 “ .3130 ds, or 2% for cash in 5 ds. ** 2121|| 18150|| 15000 Feb. 10120 ds. or 4% for cash in 5 ds. ||June 10 21|| 4200|| 20000 “ 15120 ds. or 3% for cash in 5 ds. ** 15, 16|| 1600|| 10000 “ 24|Net, Feb. 24|127| 9652|| 7585 No. ds. Dr. on $1, 99.451 68000 To Bal. ds. Dr. on $1, 31451 which div. by 6% int. div. 6000 gives 524 121524 121524 OPERATION TO EQUATE THE ACCOUNT. 1895. Ds. Cr. Ds. Cr. on $1. 1895. Ds. Dr. Ds. Dr. on $1 Jan. 3, $237.65 90 - - 21420 Jan. 10, $150.00 7 - - - - - - 1050 17, 122.50 60 + 14 = 74. - - 9102 21, 100.00 18 - - - - - - 1800 24, 324.00 60 + 21 = 81 . - 26244 Apr. 5, 300.00 92 - - - - - - 27600 * 31, 150.00 30 + 28 = 58 - - 8700 &=ºmºmºssº Feb. 10, 200.00 120 + 38 = 158 - - 31600 $550.00 30450 15, 100.00 120 + 43 = 163 - - 16300 24, 75.85 net 52 - 3952 $1210.00 117318 *..., 550.00 30450 $660.00 86868 – 660 = 131# ds. practically, 132 ds., after Jan, 3, 1895, - May 15, 1895, equated date. NOTE.-Interest on $660, May 15, to July 1, 47 days, = $5.17. Interest shown in the Account ‘. i. Interest Account is $5.24. The difference results from the fraction of a day lost in the equated date. ACCOUNTS CURRENT AND INTEREST ACCOUNTS 66 I of a convenient width to copy in the account current book. In practice, instead of arranging the credit of the account to the right of the debit side, it is frequently placed below it, in order that it may be made on paper The following account current illustrates this form : f E. RICKER in Account Current and Int. Acct. at 8% to October 1, 1895, with T. H. BRODE & Co. I) EBITS. 1895. April June Oct. 1895. July Sept. Oct. 25 10 17 1 To Mdse. (a) 30 days, º “ Cash, tº- 4- “ Mdse. (3) 60 days, s “ Red interest from credit, Total debit of interest, “ credit of interest, To Balance of debit interest, 1 CREDITS. By Cash, - - - “ Acpt. 2' 30 days, tº By Balance due this date, When due. Ds. Int. Amt. April 24, 160|| 6400||180000 ** |10| 174|| 46|40||1200.00 Aug. 16 46|| 2044||200000 467 13551 30|67 10484 5104|84 When due. Ds. Int. Amt. July 1| 92|| 30|67||150000 Oct. 8 7. 4|67|300000 30|67 60484 5104|84 ACCOUNTS CURRENT AND INTEREST ACCOUNTS IN ENGLISH MONEY. *209. 5. What is the cash balance of the following account, July 1, 1895, intelest at 6 per eent? Ans. £462 5s.14d. I)r. R. G. MUSGROVE, JR. Cr. 3' |8. d. 36 s.ld. 1895. T |T|T | 1895. * I - I -m- Jan. 15|To Cash, - tº- * 500 8 6 Feb. 10|By Cash, <- ſº º 1100|00|| 8 April 8 “ “ º tº sº 1000|14|| 3 || Mar. 5 ‘‘ ‘‘ sº em - 800|10| 4. June 18 “Ex. 2 60 days, tº 100000| 0 |May 21 “ “ tº gº - 1050" 6||11 662 soule's PHILOSOPHIC PRACTICAL MATHEMATICS. † OPERATION BY THE INTEREST METHOD. R. G. MUSGROve, J.R., in Account Current and Int. Acct. at 6% to July 1, 1895, with A. CARON. JDr. Cº. * DS. Int. Amount. Yººn * Int. Amount. 1895. £ s. d. 11895. f º d. Jan. 15To Cash, Jan. 15167||131864|| 500 8 6 ||Feb. 10|By Cash, Feb. 10,141||251700 ||110000| 8 Apr. 8 “ “ Apr. 8 84|14 02:100014| 3 |Mar. || 5 || “. Mar. 5118||15|1410 || 800|10| 4 June 18 “ Fx. at May 21 “ “ May 21 41||7|3| 64|1050, 611 60 ds. Aug. 20 50|| 8 6 8||100000 0 July |1|Am't of Cr. 27|18, 9 int. 4815 4+ & 4 Red Int. 8| 6 || 8 Am’t of Int. 57 2 # 27|18, 9 July 1|To Bal. due “ 1|Bal. of Cr. this date, 47818, 5} int. 29, 33} 2980 1'24 2980. 12; NOTE.—See page 507 for a similar account, Dec. 1, 1895, A. and B. agreed to enter into a joint speculation, each # in the purchase of 200 shares of factory stock. A. is to advance the margins and to settle with the broker for the purchase of the stock, and to render an account to B. Accordingly, Dec. 5, A. advanced to his broker, H. Lee, $2000, and Dec. 20 he advanced $1000 more. Dec. 5, the broker purchased 200 shares of factory stock at 933, $18750. Com- mission for purchasing at $ per cent, $25. Dec. 22, the broker collected a 14 per cent dividend $300, and March 23, 1894, he collected a second 14 per cent dividend $300. May 6, the broker rendered his account to A. which A. settled on the 8th of May, and the stock was delivered to him. June 26, A. received a $300 dividend on the stock. June 30, 1896, A. rendered his account to B. Required: 1. The broker's account showing the balance due him May 6th and 8th. 2. The account of A. showing balance due him by B. June 30, 1894. SALES ON JOINT ACCOUNT. 663 OPERATION. Two Hundred Shares Stock at 6 per cent per annum to May 6th, 1895. Date. Amt. |Ds 1894. - Dec. 5|To Cash, 200 shares @ 93 18775|00||152 “ 5 “ Com. @ 4% Less Cr. Int. Bal. Dr. Int. 393|18 - 1916818 1895. [T *me I sº- May | 6|To Balance due, 15568|18 Int, on same for 2 ds. (a) 6%, 5||19 May | 8 Amount due, 15573|37 Int. Date. Amt. |Ds. 1894. - Dec. 5|Cash, 200000||152 475|63 || “. [20] ‘ ‘ 100000||137 “ 22|Dividend 13%, 30000||135 82|45 1895. Mar. 123 & & 13%, 30000| 44 Total Cr. Int. 360000 May | 6|By Balance due, 15568|Z8 1916818 Int. 5067 2283 6|75 82|45 ACCOUNT OF A. WITH A. AND B. A. and B. each # in account with A. from Dec. 5, 1894, to June 30, 1895, at 6 per cent per annum. Date. Amt. Ds. Int. Date. Amt. Ds. Int. 1894. 1895, Dec. 5|To Cash—margin to June 26|By Cash, Div. 13%, 300|00| 4 20 broker, 200000|207|| 6900 “ |30| “ Balance, 18511||73 ** 20 To Cash—margin to broker, 100000||192|| 3200 1895. May | 8|To Cash—paid broker • as per his account||15573|37 || 53||137|56 Total Dr. Int. 238|56 Less Cr. Int. 20 Bal. Dr. Int. || 238|36 - - 1881173 1881173 June 30 To Balance, 185I1||73 *- REMARKS. - A's # - - $9255.86 B's # (B. owes A. June 30) º 9255.87 $13511.73 When B. pays A. $9.255.87 then A. & B. will each own # of 200 shares of stock as per account with broker. NotE.—In this account we change the locations of the columns for amount, days and interest, and omit the “when due” column. This is in accordance with the form in general use by Stock Brokers. See “Stocks and Bonds” in this work for further accounts with brokers. Geo. Soulé & Sons, of New Orleans La., have paid the following drafts, drawn on them by J. L. Pvorman & Co., of Bremen, Germany, a correspondent house: February 15, draft dated January 15th, for one month without grace for 1000 marks, exchange 954. February 28,-draft dated January 30th, for one month without grace for 1500 marks, exchange 93%. March 14, draft dated February 14th, for one month without grace, for 2000 marks, exchange 96%. Geo. Soulé & Sons have drawn on their Bremen correspondent, the following drafts: marks, one month without grace, exchange 95. What is the balance of marks and dollars due to Geo. Soulé & Sons, June 30, 1895, interest at 6% 7 Ans. 2233.24 marks. $534,38. February 1, draft for 1052.63 marks, one month without grace, exchange 95. March 2, draft for 1263.16 OPERATION J. L. Poorman & Co. in Account Current and Interest Account at 6 per cent to June 30, 1895, with Geo. Soulé dº Sons. Dr. Or. 1895. Yººn DS Interest. Amount. 1895. Wºm DS Interest. Amount, M. $. Marks. || Dollars. M. $. Marks. ||Dollars. Feb. 15To Cash, dft. Jan. Feb. 1|By Cash, dft. 1 mo. 15, ex 95+, Feb. 15||135|| 2250|| 536||100000|| 238||13 (no grace) ex. 95|Mar. 1||121| 2123| 504|105263|| 25000 “ 28 To Cash, dft. Jan. Mar. 2|By Cash, dft. 1 mo. 30, ex. 93%, “ 28||122|| 30|50|| 7.16||150000|| 35203 (no grace) ex. 95||Apr. 2|| 89|| 1874|| 4|45||1263.16|| 30000 Mar. 14|To Cash, dft. Feb. 14, ex. 96%, Mar. 14||108|| 3600|| 869|200000|| 48250 3997|| 949 June 30|By Balance, 2333,24|| 53.433 89.00|| 21.21 -* | *- 3997 949 4903|| 11|72 4549.03||1084.38 --> 4549,03||1084|38 1895. sm- I* * | * | * -* -ºs- F June 30|To Balance, * 53438 NotE.—See Index, “German Exchange” for the reduction of marks to dollars. REMARKs.—See Stocks and Bonds in this work for Accounts Current and Interest Accounts with Brokers. 3. 33alkers' Interest I. Daily Balances. gº arriagra=== *N- — — ——se 1210. In the foregoing Interest Accounts Current, we have illustrated the different methods of finding the cash balance of accounts at stated periods, or at Such irregular times as a settlement may be desired. It is a general custom with most bankers to balance accounts with their correspondents every month, or every 3, 6, or 12 months, allowing interest on all credits, and charging interest on all debits, and bringing the balance of interest to the debit or credit side of the account, as the case may be. The balance of interest thus entered and brought down to the new account, will subsequently draw interest the same as any other item in the account. The effect of thus closing and bringing down the interest balances is the compounding of money as often as the accounts are thus closed, as we fully explained in the Merchants' System of Partial Pay- ments. But in addition to the accounts current and interest accounts with their correspondents, some bankers allow interest to their depositors on the daily bal- ances of their respective accounts, and to elucidate the method of computing interest on such balances, as well as the interest on the daily balances of accounts with correspondents, we present the following IPROBLEMS : 1. What is the balance due February 1, 1895, on the following account, at 5 per cent interest on daily balances, counting 365 days to the year? ALBERT LEE & EDWARD E. SOULE. LMr. Interest at 5%. Or. 1895. 1895. Jan. 1|To Checks, sº es 30000|Jan. 1|By Cash, tº gº 1200.00 é & 2 & 4 & 4 º tº º 900 00 & 8 2 & & & 4 º sº 80000 & & 3| “. 4 & tº ºt 400|00 || “. 4| < * * tº ſº 102000 & 4 6|| “. { { tº tº 1100|00 || “. 5| “ tº = 75000 & 4 7 & 4 é & tºº s 800 00 & 4 6 { % 6 & tº tº- 50000 € $ 9 & 4 4 & g- sº 500 00 & 4 8 & 4 ( & sº ſº 90000 ** 14 “ & 4 gº tº 60000 || 44 9| 4: “ tº Ee 20000 ** 16| 44 & 4 gº tº 35000 || 4 |11| < * * - - 100000 ** [21] & 4 & 4 ſº tº 200000 || “ 15| < * * * ſº º 40000 “ 25 tº & 4 tº & 10000 || 4 |17| < * * tº sº. 65000 << |28 “ ( & tº dº 21000 || 4 |18 | < ** * = 15000 “ (30) “ & 4 a & 75|00 || 4 |22 4° 44 tº sº 34000 Feb. 1| “ Balance, - - 290%|73 || “ 24, 44 44 tº sº 22000 “ 27| “ “ tº ſº. 210000 Feb. 1|Interest to date, - 773 10237.73 1023773 (665) 666 SouLE's PHILOSOPHIC PRACTICAL MATHEMATICS. FIRST OPERATION BY THE USUAL METHOD, 1211. The above account presents the figures as they stand on the Individual Ledger of the bank, and from this ledger or the pass book the daily balances are taken and arranged on a separate sheet of paper, in a statement form as fol- lows: EQUITY NATIONAL BANK, N. O., in account with A. L. & E. E. SOULF, interest at I)R. l 90000 2 80000 3 40000 4. 142000 5 217000 6 157000 7 77000 8 167000 9 137000 10 137000 11 237000 12 237000 13 2370:00 14 177000 15 217000 16 182000 17 2470 ſº 18 26.2000 19 262000 20 2.20% 21 62000 22 96000 23 96000 24 118090 25 108000 26 108000 27 3180.00 28 2970.00 29 2970:00 30 2895|00 31 2895.00 Total daily bal. || 5643000 5 per cent to February 1, 1895. Explanation.—In finding the daily balance, we observe that, on the 1st of January, A. L. & E. E. Soulé deposited - - * $1200 and drew out - gº tº {º tº 4- ſº º 300 leaving a balance of - , ,- ū-º º * =e $ 900 The second day they deposited - *º- *- tº 800 which added to the balance of the first day gives $1700 of this amount they drew out {E_* ſº 900 leaving a balance of - - tº º º º $800 On the 3d, they made no deposit but drew out - 400 leaving on deposit s & * -> tº * = $400 On the 4th, they deposited º * ºn sº * = 1020 this producing a balance of Eº gº - - $1420 In this manner we continue through the month and find a total daily balance of $56430.00 for 1 day against the bank and in their favor, which divided by 7300, the 5% interest divisor for 365 days to the year, gives $7.73 interest due to the depositors. Had the rate of interest been 4 per cent, we would have first found the interest at 5 per cent, and then reduced it # of itself. Had it been 3 per cent, we would have found the interest at 5 per cent, and then reduced it # of itself. Had it been 6 per cent, or 7 per cent, we would have found it at 5 per cent, and then increased it # for 6 per cent and # for 7 per cent. Or to find the interest at any rate per cent, we multiply the balance of the daily balances by the rate and divide by 36500 when 365 days are counted a year, and by 36000 when 360 days are counted a year. LEDGER ACCOUNTS. 667 SECOND OPERATION.—(LEDGER ACCOUNT). ALBERT LEE & EDWARD E. SOULE. Daily Balances. — — |Ds.|Tot. Daily Bal. || Monthly Interest. I)?'. Cr. Dr. Cr. | 30000|By Cash Dep. 1200.00 90000|| 1 90000 90000|| “ “ “ 80000 80000: 1 80000|| 40000 400|00|| 1 40000 4 & 1 & 4 & 1020.00 142000|| 1 142000 & ( & & 44 75000 2170ſ)0|| 1 217000 110000|| “ “ “ 50000 1570|00|| 1 157000 80000 770|00|| 1 77000 4t tº 4 & 6 90000 1670|00|| 1 167000 50000|| “ “ “ 200|00 1370|00|| 2 27.4000 { q & 4 & 4 100000 2370|00|| 3 71 1000 60000 1770|00|| 1 177000 * 4& 64 q6 40000 2170|00|| 1 217000 35000 1820 |00|| 1 182000 64 && 64 65000 247000]. 1 247000 4 & 5 & 66 15000 2620 00|| 3 7860.00 2000.00 620 00|| 1 62000 4 & 84 & 6 34000 960|00|| 2 1920.00 4 & 4 & 64 22000 118000|| 1 lº 10000 108000|2 2100.00 64 & 6 4 & 2100.00 318000|| 1 3.18.9% 21000 297.000|2 594000 7500 2895'00|| 2 579000 290273|| “ Interest, 773 |. div. b I * -º-º- sº. 5643000 73 1023773 10237.73 | Explanation.—We here present a new form of Individual Ledger in which we keep the depositors’ accounts, and perform the operation to find the interest on the daily balances. Where there are a large number of accounts on which interest is allowed on daily balances, this form of Ledger may be used to advantage. 2. What is the balance of the following account on March 1, 1896, making monthly settlements and allowing interest on the debit of the daily balances at 8 per cent, and on the credit of the daily balances at 6 per cent, and counting 365 days to the year? Ans. $17372,18. NEW YORK CITY NATIONAL BANK.—Interest 8% on debits and 6% on credits. 1896. Dr. Cr. || Daily Balances. Tot. Daily Bal. | Monthly Int. Jan. wº By Cash, wº sºlo solo 4 tº º 5 || 2250000 04 100000|| “. “ : 350000|| 6 2100000 6& | || || 300000 650000|7 4;º ** 28 750000|| “. “ 1140000 1040000|| 4 41600 à IFeb. 104.1333| “ Interest, 13:33 *- * I - 00.11110000| 1820 2541338||____ 2541333 '-1} = −. Feb |By Balance, 104.1333 104.1333|| 4 4165332 * * 400'00|| “ Cash, 160000 1161333|| 3 34839.99 • * 180000|| ". . 2100.00 1191333| 5 59566.65 * , 600000||... . Žº 61.1333| 4 24453.32 { * 100000|| “. “ 800000 13113|33 || 6 7867998 4 i t & 4 & 380000 16913:33|| 6 10147998 * { 1700'00|| “. “ 210000 17313|33 || 1 1731333 Mar. 1 17372,18|| “ Interest. 58.85 I lºº- I sºme 357.85: 588s 2827218 2827213 | || |- | | ! Explanation.—In this account we have interest on the debit of daily balances at 8 per cent, and on the credit of daily balances at 6 per cent, and hence to find the interest on the monthly footings of the daily balances, we multiply the same by the respective rates per cent, 8 and 6, and divide by the one per cent, interest divisor 36500. 668 soule's PHILOSOPHIC PRACTICAL MATHEMATICS. º 3. What is the balance of the following account June 1, 1895, allowing interest on daily balances at 6 per cent, and counting 360 days to the year? & Ans. $7083.75. THIRD NATIONAL BANK, BALTIMORE. Daily Balances. D Tot. Daily Bal. S. 1895. I}r. o Dr. Cr. D?'. Cr. May $ $ 4 & 5000 aw & 4 10000 4 & 6 & 4 & 7500 4 4 2000 1500 & 4 200000|| 5500 57.100 Ea.planation.—In this account we present another style of ruling for the Gene: al Ledger. Where each debit and credit item is posted separately, this form of ledger may be used to advan- tage. The interest being the same on both sides of the account, the difference of the total daily balances is here divided by 6000, the 6 per cent. interest divisor for 360 days to the year. sºils his misſing Bill Aºus ---------------------------------------------N *—aº—º- --sºr—w-ur- 1212. Savings Banks are Financial Institutions for the receipt on deposit of Small sums of money for which a moderate rate of interest is allowed on stated Conditions. * A savings bank furnishes each depositor with a pass book, in which are recorded all sums deposited and all sums withdrawn. This book contains the Conditions on which deposits are received and interest is allowed and credited. It also contains the interest credited to the depositor at the end of each interest term. Such interest is entered in red ink. ... NOTE.-It is not the custom of savings banks to calculate interest on a fraction of a dollar, but in finding the amount at the end of each interest term, fractions of dollars are included. 1213. The Deposits in Savings Banks are usually paid on demand, though the charter of the bank and the conditions of deposit give the institution the right to require 60 or 90 days notice. 1214. The Interest Term is the period of time at the close of which interest is computed and credited as a deposit. It is thus compounded, if not withdrawn. The Interest Term varies with different savings banks; with some it is 1 month, with others 3 and with others 6. Interest is usually declared and credited June 30 and December 31. In some savings banks, deposits commence to draw interest on the first of each quarter, January 1, April 1, July 1, and Oct. 1. In others the money must be on deposit at least three months before it will be entitled to receive any interest. Some of the Savings Banks of New Orleans give interest on net deposits that have been in the bank 3 months; others require deposits to have been in bank 4 months before giving interest thereon. PROBLEMIS. 1. Allowing interest at 3 per cent per annum, and declaring interest semi- annually, what was due on the following account, January 1, 1896? NOTE.-Deposits to be in bank 3 months, before they bear interest. Dr. Soulé College Savings Bank in account with C. McGuigin. Cr. 1895. .* 1895, Jan, 1|To Balance, - - tº 421165 Feb. 10 By Check, wº tº º 90|30 “ 22 “ Cash, tº e * sº 5000 June 16. “ & 4 {-º tº º 651:5 Mar. [16] ‘‘ ‘‘ tº ſº tº 115|50 Nov. 1| ** &&. ſº tº 3880 May 10| “ “ tº º ºs 80.25 Aug. 14 “ “ tº tº tº 10000 Oct. I 11 tº “ & Eº º 7500 (669) 67o soul E's PHILOSOPHIC PRACTICAL MATHEMATICs. º: FIRST OPERATION. NOTE.-In this operation, we have counted actual time and given the depositor credit of 1c. for every fraction of #c. or over every alternate time. Date. Deposits. Checks. | Int. on. || From *. º Interest. | Tot. Int. | 1895. Jan. 1|To Balance, - - 421165 “ 22 “ Dep. gº - 5000 90.30 381,00|Jan. 1||June 30|| 181 575 Mar. [16] ‘‘ ‘‘ sº- - 11550 11500|Mar. 16|| “ 30|| 107 1100 6.75 May 10 “ “ º - 80.25 65.15 1500|May 10|| “ 30|| 52 O7 June 30 “ Interest, - - 6||75|| 51870 “ Balance, - - 67415|| 674|15 July | 1.To Balance, - -) 51870 518,00|July | 1||Dec. 31|| 184 794 Aug. 14 “ Dep. º - 10000 10000|Aug. 14|| “ 31|| 140 116|| 17 Oct. 1| 4 || “. * > - 7500 3880 Dec. 31 “ Interest, - tº- 9.17 “ Balance, - - 66.407 70287| 70287 1896. Jan. 1|To Balance, - - 66407 NOTE.-The depositor is credited 1 cent for each 3 cent interest every alternate time. SECOND OPERATION. Operation reckoning in months of 30 days and counting odd days to the nearest convenient fraction of a month. Date. Deposits. Checks. || Int. on. || From TO Mos. || Interest. || Tot. Int. | 1895. Jan º 2. Tº Hºnºe, - -> º, 65 90.30|| 381|00|J 1|J 30|| 6 572 ep. - - à Ll. LIIlê Mar. 16 ‘‘ ‘‘ º - 11550 11500|Mar. 16 “ 30|| 34 100 672 May 10 “ “ †- - 80.25 65.15 1500|May 10 “ 30|| 13 06 June 30 “ Interest, - -> 6.7% “ Balance, - 31867 67412 67412 July |1|To Balance, - - 51867 T| 518.00% uly |1|Dec. 31|| 6 777 Aug. 14 “ Dep. º - 10000 10000|Aug. 14 “ 31|| 44 1||13 896 Oct. 1} < * * * tº - 7500 3880 sº-º-º-º: Dec. 31 “ Interest, - - 896 “ Balance, - 665& 70263| 702 63 1896. IT | Jan. 1|To Balance, - - 66383 Ea:planation.—Since the balance $421.65 was not in for 3 months before a check was drawn which would decrease it, it is clear that interest will not be calculated on that amount. The check of February 10th is to be deducted first from the last deposit preceding February 10th, (deposit of January 22d, $50), and then the excess of check over this deposit from the balance of January 1st, 1895; $471.65 sum of deposits of January 1st and 22d, less $90 leaves $381, on which we calculate interest for 6 months at 3% per annum, which is $5,72. Next calculate interest on $115 for 34 months at 3% per annum, which is $1.00. Deposit of May 10th not being in 3 months, no interest is aredited on same at June period. Total interest to be credited June 30th, 1895, is $6,72. NOTE.-Balance the book and bring balance down. ºr SAVINGS BANKS AND SAVINGS BANK ACCOUNTS, 671 To calculate interest to be credited December 31st, 1895, Calculate interest on balance $518 for 6 months at 3% per annum = $7.77 and add to this sum the interest on $15 (the excess of deposit of May 10th over check of June 16th), for 13 months = 6 “ents. The same result would be obtained by calculating interest on the balance $518.67 minus the $15 excess of deposit of May 10th over check of June 16th, for 6 months = $7.55, and adding to this Sum the interest on the $15 for a period of 73 months = 28 cents. Worked by either method the interest amounts to $7.83. Then calculate interest on $100 for 44 months at 3% per annum, = $1.13. Total amount interest December 31st, 1895, - $8.96. Balance of account J anuary 1st, 1896, = $663,83. To find interest when book is balanced but once a year, say on January 1st. First find interest on $381 (difference between the sum of the deposits of January 1st and 22d, and the check of February 10th) for 6 months = $5.72. Next get interest on $115 for 34 months *=$1.00. Total amount of interest to be credited June 30th, 1895, = $6.72. To find interest due on December 31st, 1895. First calculate interest on $381 for 6 months = $5.72; next calculate interest on $115 for 6 months = $1.72; next calculate interest on $15 (difference between deposit of May 10th and check of June 16), for 7# months = 29 cents; next calculate interest on $6 for 6 months = 9 cents; next ealculate interest on $100 for 43 months = $1.12. Total interest to be credited on December 31st, 1895, = $8.94. Balance of account January 1st, 1896, = $663.87. NOTE 1.-On general principles, interest would not be allowed on balance of deposit of October 1st, ($75–38.80). According to circumstances, interest would or would not be allowed. NoTE 2.-The second operation above is the method generally adopted by Savings Banks. It is labor saving and on the general run of savings accounts, there is very little difference in the final result. NOTE 3.-Fractional parts of a dollar are not considered in calculating interest. NotE 4. In subtracting checks from deposits there is no settled custom. Some banks sub- tract checks from 1st deposit, other banks subtract checks from the deposit immediately preceding date of check and again, other banks count back 3 or 4 months (according to the time the deposits Imust be on hand before drawing interest) and subtract the check from the deposit nearest to that date. 2. How much was due on the following account January 1, 1895, interest at 3 per cent per annum, declared semi-annually, and allowed after 3 months? Ans, $707.37. Dr. Soulé College Savings Bank in account with Katie Childress. Cr. 1894. 1895. Dec. 3|To Cash, tº- tº * 20000 Jan. 18|By Check, - - * 5000 1895. May 4 “ “ * - * 7000 Jan. [15] ** ** - * - º 11080 Feb. 20) ** ** 4- tº- º 50|40 Apr. 28 “ “ tº- i- e 150|00 June |10| ** ** * * * 300|00 672 SouLE's PHILOSOPHIC PRACTICAL MATHEMATICS. OPERATION. IXate. Deposits. Checks. || Int. on. || From TO Mos. || Interest. || Tot. Int. 1894. - | Dec. 3To Deposit, - - $20000 $20000||Dec. 3|June 30|| 61%; 345 1895. Jan. [15] ** & 4 $º tºº 11080 5000 6000|Jan. 15 “ 30|| 5% 83 Feb. 20. “ & & tº tº 5040 5000||Feb. 20 “ 30|| 44 54 482 Apr. 28. “ & & * --> 15000 7000 June 10 ** & & as ſº 30000 “ 30 “ Interest, - - 4.8% “ Balance, - - 696 o: s31602 s1802 July 1|To Balance, - - $696.02 31600|July | 1 Dec. 31|| 6 474 Apr. 28 “ Deposit, - - 8000|Apr. 28 “ 31|| 8ºr 161 tº June 10 ** é & dºe - 30000|June 10 “ 31|| 6; 500 1135 Dec. 31 “ Interest, - º 1135 ‘‘ Balance, - - 70737 $707|37 || $70737 1896. Jºl Jan. 1|To Balance, - sº NotE.—This operation for calculating interest due on Dec. 31, 1895, applies to 2d part of following explanation for finding interest due on December 31, 1895. In this account no interest would be credited January 1, 1895. To find interest due on June 30, 1895. Find interest on $200 for 61%, months at 3 per cent per annum = $3.45. Then calculate inter- est on excess of deposit of Jauuary 15, over check of January 18, - $60 for 54 months at 3 per cent per annum = 83 cents. Then calculate interest on $50 deposit of Feb. 20, for 4 months at 3 per cent per annum = 54 cents. Total interest to be credited on June 30, is $4.82. Balance of account on July 1, 1895, is $696.02. To find interest due Dec. 31, 1895. 1. Calculate interest on balance due for 6 months = $10.44 and add to it the interest on excess of deposit of April 28, over check of May 4, $80 for 2+ months = 41 cents, also add interest on $300 for 3 months = 50 cents = total interest $11,35. 2. The same result would be obtained by getting interest on $696 — $380 = $316 for 6 months = $4.74 and adding to this interest the interest on $80 for 8 ºr months, = $1.61 plus the interest on $300 for 63 months = 5.00 = total interest credited Dec. 31, 1895, – $11.35. The reason that we subtract the $380 and not $450 from $696, is because the check of May 4, for $70 cancels that much of the $150 deposit of April 28. Balance due Jan. 1, 1896, $707.37. SECOND OPERATION. Where the book is balanced only on Jan. 1, of each year. Since the deposit of Dec. 3, was not in for three months previous to the crediting up of inter- est, none will be credited on the account on Dec. 31, 1894. To find interest due June 30, 1895, proceed as follows: First calculate interest on $200 for 61%; months = $3.45. Next calculate interest on $60 (the difference between the deposit of the 15th January and the check of the 18th), for 5% months = 83 cents. Next calculate interest on $50 for 4+ months = 54 cents. Total interest to be credited June 30, 1895, − $482. To find interest to be credited Dec. 31, 1895, first calculate interest on $200 for 6 months = $3.00; next calculate interest on $60, difference between check of Jan. 18, and deposit of Jan. 15, for 6 months = 90 cents; next calculate interest on $50 for 6 months = 75 cents; next calculate interest on $80, (difference between deposit of April 28, and check of May 4), for 81's months = $1.61; next calculate interest on $300 for 63 months = $5.00; next calculate interest on $4 for 6 months = 6 cents. Total amount of interest to be credited Dec. 31, 1895, = $11.32. The difference, 3 cents between the two statements, arises from the fact that the odd cents which were considered when the account is balanced on July 1, 1895, amounts to $2 and the inter- est on the same for 6 months is 3 cents. * SAVINGS BANKS AND SAVINGS BANK Accounts, 673 THIRD OPERATION. Operation when the checks drawn are cancelled into the first deposit regardless of the fact that other deposits have been made between the time of the first one and the first check drawn. To calculate the interest due June 30, 1895, first calculate the interest due on $150 (difference between first deposit and first check) from Dec. 3, to May 4, − 5 months = $1.88; next calculate interest on $80, (difference between balance of $150 and check of May 4), to July 1, = 14% months = 37 cents; next calculate interest on $110 for 54 months = $1.51; next calculate interest on $50 for 4% months = 54 cents. Total amount of interest to be credited on June 30, 1895, - $4,30. To find interest to be credited Dec. 31, 1895, without book being balanced on July 1, 1895. First calculate interest on $4 for 6 months = 6 cents; next calculate interest on $80 (differ- ence between deposit of Dec. 3, and sum of checks of Jan. 18 and May 4), for 6 months = $1.20 next calculate interest on $110 for 6 months = $1.65; next calculate interest on $50 for 6 months = 75 cents; next calculate interest on $150 for 8 ºr months = $3.02; next calculate interest on $300 for 63 months = $5.00. Total amount of interest to be credited Dec. 31, 1895, - $11.68. Balance due Jan. 1, 1896, = $707.18. FOURTH OPERATION. To find interest due Dec. 31, 1895, if book had been balanced on July 1, 1895. Balance at that date by above third method would have been $695.50. Interest on balance for 6 months = $10.43. Interest on $150 for 2 ºr months = 77c. Interest on $300 for # months = 50 cents. Interest to be credited Dec. 31, 1895, - $11,70. Balance due Jan. 1, 1896, = $707.20. The difference between the two amounts due is the result of the odd cents adding $1.50 and drawing interest for 6 months = 2 cents. - 3. Allowing interest at 3 per cent per annum and declaring interest semi- annually what was due on the following account, Jan. 1, 1896. NotE.—Deposits to be in bank 3 months before they bear interest. Dr. Soulé College Savings Bank in account with F. E. Gillespie, Cr. 1895. Mar. 16|To Cash, - - - - 500|00 Apr. 10 “ “ tº º cº- - 20000 June || 3 ** ** sº ſº G- º 150|00 Sept. [15] “ “ - º tº- Q- 240|00 OPERATION. Date. Deposits. Checks. || Int, on. From TO Mos Interest. || Tot. Int. 1895. | —” Mar. 16 To Deposit, - - $50000 $50000|Mar. 16 June 30|| 34 4|39 439 Apr. 10 “ & 4 - º 20000 June 3. “ 4 & tº tº 15000 “ 30 “ Interest, - tº 439 $85439 July | 1ſo Amt. brought down, $85439 §4%|July ||Dºº. #| || 7|56 Apr. 10 “ Deposit, - - 200% Apr. 10 “ 31|| 84 433 June || 3: “ 4 & tº gºe 15000|June 3 ** 31|| 6 ºr 259 Sep. 15. “ £6 tº tº 24000 24000|Sep. 15 “ 31 210 Dec. 31 “ Interest, - - 1658 - 1658 $1110 97 1896. *- Jan. 1íTo Amt. brought down, sinº 674 SOULE's PHILOSOPHIC PRACTICAL MATHEMATICs. * EXPLANATION. To find the interest to be credited June 30, 1895. First calculate interest on $500 for 33 months at 3 per cent per annum = $4.39. The deposits of April 10 and June 3, do not draw interest since they were not on deposit for 3 months. Interest to be credited on June 30, = $4 39. To find interest to be credited Dec. 31, 1895. First calculate interest on total amount deposited and interest = $854.38 for 6 months at 3 per cent per annum = $12.81. To this amount add interest on $200 for 2% months = $1.33 and also add interest on $150 for 95 months = 34 cents. Then add interest on $240 for 34 months = 2.10. Total interest to be credited on Dec. 31, = 16.58. The same result can be obtained by getting the interest on $854–$350 = $504 for 6 months = $7.56. Then add to this amount interest on $200 for 83 months = $4.33, and interest on $150 for 646 months = $2.59. Then add interest on $240 for 33 months = $2.10. Total interest to be credited Dec. 31, = $16.58. Amount due Jan. 1, 1896, = $1110.97. ##### 1215. To find the interest and the rate per cent of interest realized by a Loan and Investment Co. on certain conditions. 1. A Loan and Investment Co. loans $3000 for 5 years, at 8 per cent per annum. The interest ($1200) is added in advance and the whole amount $4200 is made payable in 60 notes of $70 each payable monthly, without interest. Allowing that the company may re-invest the monthly collections of the notes at 8 per cent simple interest, what would be the total interest received by the Company, and what would be the rate per cent per annum ? Ans. $2026 total interest; 13% rate per cent per annum. OPERATION. Interest on $3000 for 5 years at 8 per cent is, -- - * * * - º - - $1200.00 & 6 “$70 each monthly payment at 8 per cent for 1770 months which is the sum of the arithmetical series from 1 to 59 months, for the 60 notes, the last note's interest not being invested is, - = - - * * - - tº ame - 826.00 Total interest received by the Company for 5 years, * * - - tº º - $2026.00 $2026 – 5 = $405.20 = average interest for 1 year. ($405.20 × 100) -- $3000.00 = 13% per cent rate per annum. m NotE.—The sum of the arithmetical series is found by adding the first and last terms of pay; ment and multiplying the sum by one-half the number by terms or notes. Thus, 0 + 59 = 59 × 30 - #70 § ºlºng half the sum of the extremes by the number of terms, thus: (0 + 59) - 2 = X -: e 2. Suppose in problem 1, above, that the monthly note and interest collec- tions were re-invested at simple interest as therein stated, what would have been the respective interest gain for each year? OPERATION. Interest on $3000 for 1 year at 8%, $240.00 & 4 “ $70 at 8%, 66 mos. {sum of series 0 to 11), ($30.80) 30.80 $270.80 1st year. “ “ $3000 for 1 year at 8%, 240.00 “ “ $70 at 8%, 210 mos. (sum of series 12 to 23) (98.00) 98.00 338.00 2d year. “ “ $3000 for 1 year at 8%, 240.00 & 4 “ $70 at 8%, 354 mos. (sum of series 24 to 35) (165.20) 165.20 405.20 3d year. “ “ $3000 for 1 year at 8%, º 240.00 “ “ $70 at 8%, 498 mos. (sum of series 36 to 47) (232.40) 232.40 472.40 4th year. “ “ $3000 for 1 year at 8%, 240.00 “ “ $70 at 8%, 642 mos. (sum of series 48 to 59) (299.60) 299.60 539.60 5th year. Total interest for 5 years, as in the first problem, ($826.00) $2026.00 3. Suppose in problem 1, above, that the monthly collection of interest on the notes and the interest on such interest were used, at the close of each year, as a part of the dividend fund, what would be the amount of interest earned yearly 3 OPERATION FOR THE FIRST YEAR. OPERATION FOR SECOND YEAR. Interest on $3000, 1 year at 8 per cent, $240.00 Interest on $70 for 66 months at 8 per cent, which is the sum of the arith- metical series from 11 to 0, is 30.80 Total interest revenue, first year, $270.80 NotE.—The first year there were but 11 invest- ments made during the 12 months, as the first note was not collected till the close of the first month. Interest on $3000, 1 year at 8 per cent, $240.00 Interest on $70 for 78 months at 8 per cent, which is the sum of the arith- metical series from 12 to 1, is 36.40 Total interest revenue, second year, $276.40 NoTE.—The $70 collected on the first of the first month of the second year was invested, thus making 12 investments. The operation of the three subsequent years will be the same as for the second year. REMARKs.—See pages 590 to 596, for the Gain and Gain Per Cent per Annum, for certain kinds of Loans and Investments. (675) NOTE.-See Soulé's New Science and Practice of Accounts for 200 pages of Corporation Accounting. 1216. Stocks are the transferable shares of CORPORATIONS-STOCK COMPANIES- Specifying the amount of capital owned therein by the stock or shareholders. 1217. The Stock or the Capital Stock of a Stock Company is the capital Contributed by stockholders and invested in the business. 1218. A Corporation or Stock Company is an Association of men created and authorized by law to act as a single person, under a common name. The mem- bers of a corporation succeed each other, so that the Association continues always the same, notwithstanding the change of the individuals which compose it. Cor- porations are in law artificial persons. The capital of corporations is divided into shares of equal value, which are transferable and may be bought and sold like any other property. The management of corporations is generally vested in a Board of Directors who are named in the Charter, or elected by the stockholders, and who select the officers to conduct the business of the Company or Corporation. 1219. Certificates of Stock are instruments issued by a CORPORATION, certi- fying under the signatures of the proper officers that the holder is the owner of a specified number of shares of its CAPITAL STOCK. Thus, the following is a general form of a CERTIFICATE OF STOCK : Shares. # ŽVo. # exº OF Louisiawa SOULE COLLEGE BANK. his i; to {ertifu, That es e e s is a e º sº s the Proprietor of...........Shares of One Hundred Dollars each, in the Capital Stock of S0ULE COLLEGE BANK, which is # transferable on the books of the Bank by the said Stockholder, i. # or.… Attorney or Agent, on the surrender of this # Certificate. & Witness, the Seal of the Bank attested by the Signatures § of the President and Cashier thereof. ; New Orleans, 189...... # Cashier. President. (676) vſ. STOCKS AND BONDS. 677 1220. The Par or Nominal Walue of Stock is the sum named in the instru. ment or certificate of stock. When stock sells for more than the original or par value, it is said to be above par or at a premium. When for less, it is below par or at a discount. 1221. The Market Walue of stocks or bonds is what they will sell for in open market. In the case of stocks, this value depends upon the dividends they pay compared with the current rate of interest in the money market; and in the case of bonds, the market value depends upon the degree of confidence entertained by capitalists of their being paid at maturity, and the rate of interest they bear compared with the current rate of interest in the money market. The success of incorporated companies has much to do with the market value, and the sobriety, industry, fidelity, and capacity of the officers and directors of companies have almost all to do with the success of the company. 1222. The Intrinsic or Liquidation Walue of Stock is the sum that each share would be worth in case the CORPORATION went into liquidation, or it is the quotient of the net resources after all liabilities are paid, by the number of shares. 1223. Preference or Preferred Stock is stock issued by some CORPORATIONS which is entitled to an extra amount of interest or to a specified per cent dividend out of the profits, before the common stock dividend is declared. The circumstances which give rise to PREFERRED STOCK are varied and are generally based upon equity. A corporation may be in great distress for money, and to relieve the company, certain stockholders or other parties may advance the required money for which preferred stock is issued to them. Again, in the re-organ- ization of railroads or banks, preferred stocks may be issued to save the sacrifice of the company's property, or to obtain the means to carry on the company’s busi- IłęSS, 1224. Guaranteed Stock consists of shares on which a certain amount of money or interest is due, payment of which is guaranteed, thus rendering the shares especially valuable. 1225. Forfeited Stock is subscribed stock on which installments may or may not have been paid, and on which the subscriber fails to pay installments when called for. Such stock is forfeited to the company according to the terms of the Charter. 1226. Debenture Stock is a writing acknowledging a debt. 2. A deed of mortgage given by railway companies for borrowed money. 3. The bonds and securities issued for borrowed money, by towns and corporations. NOTE.—This term is in general use with English Financiers, but it is rarely used by American Bankers or Security dealers. 1227. An Installment is a certain per centum of the capital stock which subscribers are required to pay at a specified time. -- 1228. An Assessment is a per centum tax levied and collected from the stockholders to make up losses or deficiencies. 1229. Gross Earnings are the total receipts of the company. 678 SOULE's PHILOSOPHIC PRACTICAL MATHEMATICs. Yºr 1230. Net Earnings or Net Profits are what remains after all expenses are deducted. 1231. A Reserve Fund, or Contingent Fund, or Redemption Fund is a certain portion of the yearly profits of the company, reserved or set aside for the purpose of creating a fund to meet unforeseen emergencies or accidents. DIVIDENDS. 1232. A Dividend is a pro rata division of the profits or a portion thereof, among the stockholders of a corporation. 1233. A Cash Dividend is a certain amount of the profit of a company dis- tributed in cash among the stockholders annually, semi-annually, or quarterly. NOTE.-Dividends, when declared, are generally a certain per cent on the par value of the shares, but sometimes they are a certain per cent on the paid-up stock. 1234. An Installment Dividend is a certain amount of the profits of a, com- pany apportioned to the stockholders and applied to the payment of their unpaid Subscriptions. An installment diyidend is in effect the paying of a cash dividend and the collection of an installment of equal amount. 1235. A Stock Dividend is a certain amount of the profits of a company apportioned to the stockholders and retained by the company, for which new paid-up stock is issued to the stockholders. Or, in the absence of a profit, it may be addi- tional stock, to be paid out of the capital or the future profits of the company, in which latter case, it is equivalent to the operation of “watering” stock. 1236. A Forced Cash Dividend, or a Capital Stock Dividend, or a Fictitious Dividend is, in the absence of a profit or reserve fund, a certain per cent of the capital of the company distributed among the stockholders. The method usually adopted is to place an inflated value on the real estate, plant, bonds, personal accounts, or other resources of the corporation, or by charg- ing expense and repairs to the property accounts, by which means false profits are shown, and then Forced or Fictitious dividends are declared. The object of a forced or fictitious dividend is deception, and the result dishonest. For by it, the public are led to believe that the company is prosperous when it is not, and hence pay more for the stock than it is worth. NOTE.-In this connection it must not be forgotten that sometimes, from unusual causes, companies do not make a yearly or semi-annual profit, and then they declare a dividend from a portion of the previous gains of the company, which have been set aside under the head of RESERVE FUND for that purpose. A dividend of this character is not to be classed as a forced or fictitious dividend. In the early history of railroads and some other companies, forced or fictitious dividends were of frequent occurrence. 1237. Watered Stock is that which has been increased above the authorized capital by the issuance and distribution among the stockholders of new stock, for which no payment is or will be made. When these waſtERED SHAREs are made transferable, the act of watering is violative of every principle of ethics, and should secure for the parties thereto the judgment of a Criminal Court. Yºr BONDS, 679 When the WATERED SHAREs are not made transferable, then the transaction is one of deception, the motive for which will appear plain when we consider that the charters of some companies require that their dividends shall not exceed a cer- tain per cent on the Capital Stock, and that whenever the net profits of the company are sufficient to produce a dividend in excess of the specified per cent, that the company shall reduce its rates of tariff. Hence, to avoid this reduction of its rates, new stock is issued and distributed pro rata to the stockholders, by which means the capital stock is increased and the rate per cent dividend decreased and kept Within the prescribed limits, and the company is thus enabled to continue its high charges and heavy profits. 1238. Stock is often watered in the re-organization of a railroad or the con- Solidation of different railroads, or the consolidation of other corporations, when the property or plants are estimated at an increased valuation. If the property has increased in value from any cause, then such watering might be honest, though it is not the legal way to increase the capital stock of a corporation. NOTE.-The watering of stock has been indulged in by some of the leading railway companies of the country and by other corporations, to an alarming extent. Watered stock amount does not represent any real investment, but only increased earnings, and hence, with the reduction of freight and passenger transportation charges, which the competition of new roads now building and to be built will certainly effect all watered stock, will eventually prove a loss to the holders. For, when the incomes of the companies are reduced to their normal level by the means indicated, the entire issue of stock will in like manner be reduced to the original value of the first issue. We can, in a measure, forgive, if we cannot justify, the vendor of watered whiskey or rum, on the moral plea of his having performed a philanthropic act by selling a less poisonous article when watered than when pure, but no justification whatever can be offered in extenuation of the sin and crime committed by Watering Stock. If the capital of the company is not commensurate with the workings and demands of the company, the capital stock can be increased in a legitimate and honest manner 1239. Clandestine Stock is first, a new edition of stock issued without public notice and placed on the market to raise money to cover losses or expenses of which the public have no knowledge. NOTE.—Clandestine Stock has also been issued under the following circumstances: The market value of stock is very high, and the company issues and distributes pro rata, at par, to the stockholders of the company, a certain amount of new stock. This new stock is placed on the market, and the full market value of the original stock is realized before the fact is known that it is Clandestine. BONDS. 1240. Bonds are written or printed obligations of Individuals, Corporations, Cities, States or nations, to pay a specified sum of money at a certain time. BONDS generally bear a fixed rate of interest payable, annually, semi-annually or quarterly, and when issued by individuals or corporations for loans are Secured by mortgages upon their property. The security of the Bonds of States or nations, usually have no other security than the confidence of the people. BONDS are issued in denominations varying from ten dollars to fifty thousand dollars. 1241. Coupon Bonds are those having interest tickets or coupons attached, 68O SOULE's PHILOSOPHIC PRACTICAL, MATHEMATICS. Y Which specify the interest due each year, half year, or quarter, and which are cut off as they are paid, and held as receipts. 1242. Registered Bonds are bonds that are payable to the order of the holder or owner, and consequently cannot be negotiated without assignment duly acknow. ledged. BONDS are known as First Mortgage, Second Mortgage, Debenture, etc., Consols, Income, Sinking Fund, Adjustment, etc., according to their priority of lien, the class of property upon which they are secured or other characteristics. BONDS are also named from the rate of interest they bear, or from the dates at Which they are payable, or from both. Thus: Louisiana 4's, United States 4's of 1907, New Orleans City Premium Bonds, United States 3's. 1243. A Mortgage is a conditional conveyance or sale of real or personal property as security for the payment of a debt or for the performance of a duty. The written instrument is termed the MORTGAGE and is property or a resource to the holder. 1244. Consols (from consolidated), are bonds issued in redemption of two or more classes of bonds which are consolidated into the new bond, and generally at a reduced rate of interest. * ENGLISH OONSOLS. 1245. English Consols are 3 per cent English bonds. The term is abbrevia- ted from the words consolidated annuities. NOTE: 1.—These bonds were issued by virtue of an Act of Parliament to redeem several separate issues of government bonds bearing various rates of interest, and with the condition that while the interest was regularly paid the government could not be required to pay the principal. They are hence practically perpetual annuities. In the act creating them, they are termed consoli- dated annuities. The quotations of these 3 per cent bonds or “Congols” indicate ordinarily the state of the money market, as they form a large part of the English public debt. NotE 2.-In England the term STOCK is limited to the Government Stocks and Annuities; and the term SHARE is used for the Capital Stock of Corporations. 1246. French Rentes are the Government Stocks, which bear various rates of interest. NOTE.-In France the term RENTES has the same limitation as STOCKS in England. DNITED STATES BONDS. The following are the interest bearing bonds of the United States, outstanding April 1, 1895, as per the statement of the United States Secretary of the Treasury: 1247. 1. Funded Loan of 1891. These are bonds which were authorized by an Act of Congress of July 14, 1870, and Jan. 20, 1871, to refund the national debt. They bear interest at the rate of 44 per cent, payable quarterly in gold. They are redeemable at the option of the United States 15 years after Act of Authorization. Amount outstanding April 1, 1895, was $25,364,500. Yºr STOCK EXCHANGE. 68t 1248. 2. Funded Loan of 1907. These bonds were authorized by an Act of Congress of July 11, 1870, and Jan. 20, 1871. They bear 4 per cent gold interest, payable quarterly, and are redeemable July 1, 1907. Amount outstanding April 1, 1895, was $555,624,850. 1249. 3. Refunding Certificates. These certificates were authorized by an Act of Congress of February 26, 1879. They bear 4 per cent interest, payable quarterly. $54,710 are outstanding April 1, 1895. * 1250. 4. Loan of 1904. These bonds were authorized by Act of Congress of Jan. 14, 1875, they bear 5 per cent interest, payable quarterly, and are redeemable February 1, 1904. Amount outstanding April 1, 1895, $100,000,000. They were placed on the market in 1894. 1251. 5. Loan of 1925. These bonds were also authorized by the Act of Congress of Jan. 14, 1875. They bear 4 per cent interest, payable quarterly, and are redeemable Feb. 1, 1925. Amount outstanding April 1, 1895, was $28,807,900. 1252. 6. U. S. Pacific R. R. Currency Sixes. These are 30 year 6 per cent currency registered bonds, which the government issued to the various com- panies chartered by Congress to build railways to and from the Pacific coast. They were issued on the completion of 20 miles of road at the rate of $16000, $32000, and $48000 per mile, according to the expense and difficulty of construction. The interest is payable Jan. 1, and July 1. Amount outstanding April 1, 1895, was $64,623,512. In addition to the foregoing interest-bearing debt the United States had, April 1, 1895, the following indebtedness: 1. Bonded debt on which interest had ceased since maturity, $1,770,250.26. 2. United States Notes, National Bank Notes and Fractional Currency, $381,025,096.92. 3. Gold and Silver Certificates, and Certificates of Deposit, all of which is offset by an equal amount in the Treasury, $567,944,442.00. STOCK EXCHANGE. 1253. Stock Exchanges are Associations of Brokers, generally incorporated bodies, whose business it is to buy and sell stocks, bonds, and other securities. The regular commission for buying and selling securities at the New York Exchange is # per cent on par value, except for mining stocks, on which contract rates are usually charged. The NEW ORLEANS STOCK EXCHANGE is an Association organized for the purpose of buying and selling stocks, bonds, and other securities. The regulations of the association are very stringent and specific, in order to prevent unworthy persons from being admitted, and to insure fidelity and capacity in the performance of contracts. To become a full member of the New Orleans Stock Exchange, the applicant must be of good character, must be regularly engaged as exchange, stock, coin, or bill broker, and own at least one share of the stock of the exchange. The 682 SouLE's PHILOSOPHIC PRACTICAL MATHEMATICs. y number of shares of the New Orleans Stock Exchange is 70, and the par of each is $10. The present (1895,) market value per share is about $800. The NEW ORLEANS STOCK EXCHANGE has an INSURANCE FUND which was Created by an assessment of $15 per member. Upon the death of any member of the Exchange, each surviving member is assessed $15, which is paid into the INSURANCE FUND, and within 60 days after the proof of the death of any member the amount of money thus realized is paid to the legal heirs of the deceased member. The commission for buying or selling in the New Orleans Stock Exchange is as follows: # per cent on the par of City.and State Bonds. On all other bonds selling at and above par, 3 per cent on par value. H On stock selling at and over $60 per share, 50 cents per share is charged. On stocks selling at $25 and under $60 per share, 25 cents per share is charged. On-stocks selling below $25 per share, a contract charge is made. On Scripts, Warrants, etc., 3 per cent is charged on par value. ~. .* In the sale of stocks, the following regulations are also observed by the New Orleans Stock Exchange: º 1. On all contracts of time exceeding 3 days, one day's written notice must be given before the stocks or securities can be delivered or demanded. 2. All sales not “cash’” are considered time. No purchase on sale at the option of the buyer or seller for 3 days, shall bear interest. 3. All purchases and sales beyond 3 days shall be “flat” unless otherwise expressed. * 4. On all time bargains, the rate of interest shall be 8 per cent, to be cal- culated by days according to bank usage. 1254. DEFINITIONS OF WORDS AND PHRASES USED BY FINAN- CIERS, BROKERS AND BANKERS. A Trust is a corporation formed by large dealers in a certain article, whose object is to control the market, thereby forcing prices above the level established by free competition, or reducing them below the cost to produce the article in order to suppress opposition. Trusts sometimes fairly and honorably reduce prices, by reason of an extensive trade, of buying material and manufacturing articles in large quantities at a few centers of trade, etc. A Syndicate is a combination of Brokers, Bankers or Capitalists, who undertake to place large loans and transact other financial business. Collaterals, are Securities, Bills Receivable, Stocks, Bonds, etc., which are pledged and placed in the possession of an obligee, in addition to the principal security, to secure the payment of money of the performance of an obligation entered into by the obligor. Hypothecating Stocks and Bonds is depositing them as collateral security for money borrowed or debts contracted. 4 Margin is a deposit made with a broker by a person who wishes to speculate in buying or selling stocks or bonds, to enable the broker to carry the stocks or bonds and protect himself against any loss that may occur by an unfavorable change in the market price. The margin varies from 5 to 20 per cent of the value of the stocks or bonds. NOTE.-In case the market moves so as to lessen the per cent margin and threaten loss to the broker who is “carrying” the stock or bonds, the margin must be made good, on the notice of the broker, by the deposit of an additional sum. If this is not done the broker is justified in ; the securities to protect himself against the loss of the money he has advanced to or for is principal. * Bºrge interest on all sums advanced for carrying stocks or bonds, allow interest on margins, and charge the regular per cent brokerage for buying and for selling. jºr DEFINITIONS OF WORDS AND PHRASES. 683 Difference is the variation of the price for which stock is contracted and the market rate the day of delivery. In cases like this the broker against whom the variation exists frequently pays the difference in money instead of supplying or receiving the stock. Banker, the proprietor of a bank; one who deals in money, and negotiates bills of exchange. Broker, one who receives and executes orders for other parties. Factor or Commission Merchant, one who sells goods on commission. Jobber or Merchant, one who deals in Stocks or Merchandise on his own account. Stag or Outsider, one who does business outside of the exchange. Bear is an operator who is “Short of Stocks,” or who wishes to purchase stocks; and hence one who labors to decrease or depress the price that he may buy or “fill” at lower rates. The term is also applied to all parties who wish to purchase goods. The name is derived from the nature of the bear to pull things down with his paws. Bull is an operator who is “long of stocks,” or who wishes to sell stocks; hence one who labors to advance or inflate the prices. The term is also applied to all parties who have goods to sell. The name is derived from the nature or character of the bull to throw things up with his horns. Future Sales or Seller's or Buyer's Option, the privilege given to either party of delivering the Stock at any time within the blank days specified. Cash Sales, to be delivered and paid for the same day before 2 P. M., if sold before 1 P. M., and before 3 P. M., if made after 1 P. M. A Regular Sale, payable the next day before 2 P. M. Selling Short, selling what you have not in the hope of buying for less before called upon for a delivery. - Long of Stocks, buying and holding the same in the hope of a rise. Short of Stocks means that you have sold that which you do not possess in the hope of buying at lower prices and thus realizing a profit. * A Wash Sale, a sham transaction between two brokers for the purpose of causing an advance or decline in prices. Wash sales are not allowed in well managed exchanges. To make a Turn, sell for Cash and then buy it back on regular sale. Milking or Lowering, lowering prices to induce holders to sell, and then the clique step in and buy. Salting Down, saving what you have made. Busy Bees, a clique or several parties working together with some special object in view to influence prices. To Fill or to Cover, buying Stock or Merchandise to fill contract of short sales. To Unload or to Get Out, to sell Stock or Merchandise that is burdensome, either at a gain or a loss. Lame Duck, one who has lost heavily or has been badly crippled, but who has not failed. Dead Duck, one who is unable to fulfill his contracts and fails. Opening Day, the first day that the books are opened after a dividend has been declared. Settling Day, a day that the things bought or sold are delivered or paid for, or a day that the con- tract matured. Contango, the postponement of settling day. Point, one of any unit or measure of price, either increase or decrease. Black List, a record kept of members of the exchange or customers of banks who violate their contracts. A Put Contract, a contract by which you have the right to put or sell to your broker, One Hundred Shares of specified stock at a fixed price, during thirty days. For this contract you pay your Broker $106.25. A Call Contract, a contract by which you have the right to call or buy from your Broker, One Hundred Shares of specified stock at a fixed price, during thirty days. For this contract you pay your Broker $106.25. A Spread is a contract by which you have the right to put to, or call from your Broker, One Hun- dred Shares of specified stock at a fixed price, in thirty days. For this contract you will pay your Broker $212.50. NOTE.-A Straddle is where a dealer is “long’’ of one option and “short” of another. He has thus straddled the market. Also where a dealer buys for future delivery is one market and sells in another. 684 soule's PHILOSOPHIC PRACTICAL MATHEMATICs. * * S. 3, 10, 30, etc., mean that the Seller has three, ten, thirty, etc. days in which to deliver Stocks or Merchandise bought. - B. 3, 10, 30, 30, etc., mean that the Buyer has three, ten, twenty, thirty, etc. days in which to receive Stocks or Merchandise bought. - Eac. Dividend means that the sale is made with the dividend off. C. means a cash transaction. R. means a regular sale. P. F. means preferred stock. G. T. D. means guaranteed. Ex. Coup. means without coupons or interest. B. & Int, means that the buyer is to receive the interest on the stocks bought. B. Flat means that the buyer is not to receive the interest. b. c., between calls, means that the sale was made between the first and second call of the stock, there being two or more calls each day. Corner is an operation by several jobbers or operator.—“longs”—who form a clique to buy and hold stocks or merchandise, thus producing a scarcity in the market which results in high prices, and a loss to the “shorts” or others who are obliged to buy, and are hence cornered. Corners, frequently by result disastrously to the cliques or “longs” who organize them as well as to the “shorts” as they are left with a load that they cannot discharge except at a great loss. Spread Eagle is the operation of a broker who sells a given quantity of stocks on time, say, 60, and buys the same quantity at a lower price, say, 60. If both contracts run 60 days he will make .# difference; but if he is compelled to deliver by buyer or seller before the full time he may suffer loss. Bucket or Curbstone Brokers are brokers who are not members of the stock exchange and though some of them are men of honor, many are lame ducks and some are only speculators without capital; they are not governed by as strict rules and do not abide by the strict letter of the law as do members of the exchange. - Rite Flying is a term applied to a certain class of transactions of an extra hazardous character, and engaged in only by persons who are non-observing of the principles of ethics. The follow- ing will serve to illustrate the transaction: A, is without capital; he solicits of B. the privilege of drawing on him at 10, 30, or 60 days, with the assurance that before the maturity of the draft he will place funds in his hands to meet the same. B. consents; A, draws, negotiates the draft, and uses the proceeds. When this draft is about to mature, he draws and negotiates another sufficiently larger than the first to net the face of the first. This process of drawing and selling, he keeps up until by prosperous business he clears enough to cover the discounts on his drafts, and to meet the last, one ; or, until his repeated losses and clandestine dealings involve him in inextricable difficulties, and he becomes a disgraced and ruined man. PROBLEMS IN STOCKS AND BONDS. 1255. To buy and sell Stocks and Bonds on our own account. 1. What will 50 shares of Germania National Bank stock cost at 15 per cent premium ? Ans. $5750. OPERATION. $115 × 50 = $5750. Ans. 2. Bought 20 shares of National Bank stock at 95 per cent. What did it cost 7 Ans. $1900. 3. Sold 40 shares of City Railroad stock at 1254. What did Ireceive for it? Ans. $5020. 4. Bought 75 shares of Chattanooga Railroad stock at 90 per cent, and sold the same at 5 per cent premium. How much did I gain 3 Ans. $1125. 4x PROBLEMS IN STOCKS AND BONDS. 685 1256. To invest in stocks and Bonds on our own account. 1. How many shares of $100 each of the City National Bank stock can I buy for $2280, the stock being 14 per cent premium ? Ans. 20 shares. FIRST OPERATION. 1 share = $100 par value. Ea:planation.—In this solution, we first find the 14 = 14% premium. par amount of stock that we can buy with the * * money, and then the number of shares. In the $114 cash value. operation we first added to the $100, par value of 1 share, the 14 per cent premium, which gave 100 us $114 cost value; we then placed the $100 par 114 || 2280 value on our statement line, and reasoned thus: | Since $114 cash will buy $100 of stock, $1 cash $2000 = par of stock. will buy the 114th part, and $2280 will buy 2280 times as much. The result of which is $2000. which divided by $100 = 20 shares. Ans. It is then clear that as 1 share is $100, there are as #ºny shares as $2000 are equal to $100, which IS 2#.U). SECOND OPERATION. SHARE. Explanation.—Here we place 1 share on our | 1 statement line, and reason thus: Since $114 114 || 2:280 cash will buy 1 share, $1 will buy the 114th part, *=-º-º-e and $2280 will buy 2280 times as much. The 20 shares. Ans. result of which is 20 shares. 2. Invested $15375 in City bonds, at 384 per cent discount. What amount did I buy? Ans. $25000. OPERATION INDICATED. $100 – $384 = $614; $15375 – $61} = $25000. Ans. ' 3. A merchant invested $3675 in Louisiana Frear Stone Manufacturing Company stock, at 5 per cent premium, and sold the stock at $1144. What was his gain } Ans. $319.374, gain. 4. Invested $10450 in Louisiana 4's at 1044. What is my annual income from the same 3 Ans. $400. 1257. To buy and sell Stocks through a Broker. 1. Bought 25 shares of New Orleans Wrecking Company stock, at 95 per cent. Paid New Orleans Board of Brokers rates of brokerage. What did it cost? Ans. $2387.50. OPERATION INDICATED. 25 shares at $100 = $2500 par; $2500 × 95% = $2375. $2375 + 4% brok. $12.50 = $2387.50. Ans. 2. Sold 100 shares of New Orleans 5 per cent Water Works bonds at 40 per cent discount. Paid 3 per cent brokerage. What were the proceeds? Ans. $5950. 3. Bought 80 shares of $50 each of the Mechanics' & Traders' Bank stock, at 75 per cent, and sold the same at 10 per cent discount. Paid brokerage # per cent for buying and 4 per cent for selling. What was my gain? Ans. $560. 686 soule's PHILOSOPHIC PRACTICAL MATHEMATICs. .* 1258. To invest in Stocks and Bonds through a Broker. 1. Invested $9056.25 in Louisiana Savings Bank stock, at a premium of 20+ per cent. Brokerage 3 per cent. How many shares did I buy 3 Ans. 75. FIRST OPERATION. 1 share = $100 par value. Explanation.—We here first find the cost of 20+ = 20.4% premium. $100 par value at the rate of premium and bro- # = }% brokerage. kerage given in the problem. This we do by ** == adding to the $100 par value the 20+ per cent $120% cost of $100 par. premium and 4 per cent brokerage, which gives $120%; with these ratio figures we make the $ PAR VAL. proportional solution statement, as shown in 100 the operation, to find the par amount of Stock 483 || 4 bought. The reasoning for the statement is as 9056.25 follows: Since #4 (120% reduced) dollars cash Eº-º-º-º: will buy $100 par value of stock, 4 of a dollar 100 ) $7500.00 amount of stock. cash will buy the 483d part, and # or a whole -- - dollar will buy 4 times as much, and $9056.25 75 shares. Ans. cash will buy 9056.25 times as much. The result of which is $7500. Then as 1 share is worth $100, it is plain that we have as many shares as the $7500 is equal to $100, which is 75. SECOND OPERATION, SHARE, Ea:planation.—In this solution we work the 1 same as in the first to obtain the cost of $100 3 || 4 par value of stock; then, instead of finding theſ | 9056.25 N amount of stock that we can buy, we find at once the number of shares. This we do by 75 shares. Ans. placing 1 share, instead of the par value of the same, on our statement line, and reasoning thus: Since $##3 ($120% reduced) cash will buy 1 share, ($4 will buy the 483d part, and $3 will buy 4i. as many, and, $9056%; will buy 9056%; times as many as $1. The result of this gives us 75 Bºla,TOSs .* NOTE.-We present the two operations beeause in some problems we wish to know the number of shares, and in others we wish to know the par value or amount of stock. 2. Paid $1162.50 for City of New Orleans 4's at 33 per cent discount. Bro- kerage 3 per cent. What is my semi-annual interest on them? Ans. $24. OPERATION INDICATED. 33% discount. $ PAR. $1200 * #96 brok, deducted. | 100 4% 96% | 1162.50 3#96 net discount. * * == $48.00 int. for 1 year. $100 par val. of stock. $1200 bonds bot. $96; cost of $100 par. $48 – 2 = $24 int. for 6 mos. 3. Invested $15912 in City of New Orleans 5 per cent bonds at 1034, bro- kerage # per cent. What is the annual income 3 Ans. $765.00. 4. What amount must I invest in 34 per cent bonds which are selling at 104, brokerage 4 per cent, so that I may have an income of $525 per quarter f Ans. $62700.00. OPERATION INDICATED. $525 x 4 = $2100. 3.} | * } = $60000 x 4% = $2700 + $60000 = $62700. Ans. * PROBLEMS IN STOCKS AND BONJOS, 687 1259. The capital and rate per cent given, to find the dividend. 1. A merchant owns 200 shares of bank stock, the par value of which is $50. How much will he receive when a dividend of 6 per cent is declared ? Ans. $600. OPERATION. $50 × 200 = $10000 × 6% = $600 dividend. 2. What is the amount of a 6 per cent dividend on 120 shares of New Orleans Gas Light Company stock? Ans. $720. 3. Invested $5747.50 in State National Bank stock, at 104. Paid brokerage for buying 3 per cent. How many shares did I buy, and what is my annual income therefrom when the dividends are 4 per cent semi-annually? Ans. 55 shares. $440 annual income. 1260. The capital or investment and net gain given, to find the rate per cent dividend or per cent gain. 1. The capital of a Bank is $1500000. The net profits for six months' busi- ness are $192814.58. The Board of Directors resolve to pass $12814.58 to the credit of “reserve fund,” and to distribute the remainder to the stockholders. What is the rate per cent of the semi-annual dividend ? Ans. 12%. OPERATION. $192814.58 met profits. $ 12814.58 passed to reserve fund. 180000 smºmºmºmºsºmº-ºº 1500000 || 100 $180000.00 profit for a dividend. 12% Ans. 2. I subscribed for 100 shares of bank stock. After paying three install- ments of 25 per cent each, a dividend is declared. The capital stock of the Bank is $300000; the net profits are $38460.50. Passing $8460.50 to reserve fund, what is the rate per cent dividend and the rate per cent profit on my investment? Ans. 10% dividend. 134% gain on investment. 3. An investor has $14700 in manufacturing stock, and receives $367.50 per quarter, dividend. What is his annual per cent gain? Ans. 10%. 4. Invested $20000 and gained $1500. What was the gain per cent? & Ans. 73%. OPERATION INDICATED. $ 1500 20000 100 5. Bought 50 shares of stock at $90 per share, and sold the same at 12% per cent discount. What per cent did I lose on the investment? Ans, 24% loss. OPERATION INDICATED. $ 50 × $90 = $4500 cost. * 125 = loss. 50 × $874 = 4375 selling price. 4500 | 100 688 SOULE's PHILOSOPHIC PRACTICAL MATHEMATICS. jºr 1261. The rate per cent dividend or interest, and the price of Stock given, to find the rate per cent gain on investment. 1. Bought New Orleans City 6's at 25 per cent discount. What is my per cent gain on investment? Ans. 8%. OPERATION. $100 par value. $ GAIN. Ea:planation.—As shown in the operation, $100 par value 25% discount. | 6 cost but $75, and hence the 6 per cent interest is really sºmsºmºsºm- 75 | 100 gained on the $75 cash investment. In making the pro- $75 cost. portional statement to find the gain per cent, we reason 8% Ans thus: Since $75 gain $6, $1 will gain the 75th part, and $100, 100 times as much. 2. Bought 6 per cent bonds at 5 per cent premium. What is my per cent gain on investment, making no allowance for the use of the semi-annual interest? Ans. 5}%. 3. If I buy 50 shares of bank stock at 190, and pay 3 per cent brokerage, and the bank declares a semi-annual dividend of 15 per cent, what is my per cent gain? Ans. 154*7%. OPERATION INDICATED. $190 cost. 15% semi-annual dividend. $ #96 brok. on par value: 2 30 _*% p 381 || 2 $1904 total cost of stock. 30% yearly dividend. | 100 -* 4. Which is the better investment, 8's bought at a premium of 20 per cent, or 6's at a discount of 25 per cent 3 Ans. 6's are 14% better. 5. A capitalist buys Louisiana 6's, having 10 years to run, at a discount of 15 per cent. What will be his rate per cent interest gain if he holds them until maturity and then collects the face and interest ? Ans. 844%. OPERATION. $100 par of bond. $100 par of bond. $100 face of bond. 15 = 15% discount. 6% interest. 60 int. for 10 years. $85 cost of bond. $6.00 interest for 1 year. $160 value at maturity. 10 years. dº. 85 cost of bond. $60.00 interest for 10 years. $75 gain in 10 years. We now have to find at what rate per cent interest $85, for 10 years, will produce $75 interest. OPERATION. $85 9% 75 10 years. 1° 0 - 85 || 100 -ºs- 8.50 || 75.0 OF 10 $8.50 int. for 10 years at 1%. | j x- 8+} % Ans. 8}# 96. Ea:planation.—For the reasoning of this last work, see Interest Problem, on page 599. 6. Bought 5 per cent interest bearing bonds at 98 per cent. These bonds 30. PROBLEMS IN Stocks AND BONDs. 689 have 7 years and 6 months to run. What will be the gain per cent, the interest being paid semi-annually, making no allowance for the use of the semi-annual interest ? Ans. 5; % or 5.102+%. 7. In the above problem allowing 6 per cent simple interest on the annual interest of the bonds, what would be the annual gain per cent made on the invest- ment 3 Ans, 5}}%. OPERATION INDICATED. Interest on $5 for 12 mos. at 6% = 30c. $5 + 30c. = $5.30. $ 5.30, 98.00 100 53.3% 1262. The per cent that Stocks bear, and the per cent that is desired to be gained, given, to find the price at which Stock must be bought in order to wealize the desired gain per cent. 1. At what price must stock be bought which pays 6 per cent, in order that the investment will pay 8 per cent? * Ans. $75 per share of $100. FIRST OPERATION. $100 assumed price. Eacplanation.—Here, having no value to work 8% gain. from, we assume $100, and then find the 8 per cent gain thereon to be $8. We then see that $8.00 amount of gain. $100 value of stock gives $8 gain, and by trans- position that $8 gain requires $100 value of stock. The question is now simply this: Since $8 gain 100 require $100 value, what value will $6 gain 8 || 6 require? The exact reasoning for the statement *º is, since $8 gain require $100 value, $1 will $75 Ans. ...he 8th part, and $6 will require 6 times 8.8 IIlllCIls 3 2. If I loan my money, I can get 10 per cent interest. At what discount must I buy Louisiana Levee bonds, that bear 8 per cent, in order to obtain the same per cent on the investment? Ans. 20% discount. OPERATION INDICATED. 10 | *}=sso. 100–80 = 20. 3. Interest on money is 44 per cent. What price can I pay for stock that pays 6 per cent dividends? Ans. $133}. OPERATION INDICATED. $ 100 100 9 || 2 or, 4% | 6 | 6 4. Which is the best investment to make with $52000: To loan it at 12 per cent interest on good collaterals; to invest it in stocks at 65 per cent that pay 690 SouLE's PHILOSOPHIC PRACTICAL MATHEMATICs. &A semi-annual dividends of 4 per cent ; or to buy merchandise which we can sell during the year for $67380, with an attendant expense of $9400? Ans. Stocks by a per cent over the 12 per cent loan, besides the use of one semi-annual dividend for 6 months; and by ## per cent over the merchandise investment, not making any allowance for the use of the money collected prior to the close of the year. OPERATION INDICATED. 65 | io } = 12#%. $67380 – $52000 = $15380 gain on sales; less $9400 expense. = ($5980 net gain × 100) -- 52000 = 11339%. 12.4% — 12% = *%. 12#96 — 11}}% = #96. 1263. The rate per cent that Stocks bear, the cost of them, and the gain or income desired to be realized from them given, to find the sum to be invested. 1. What sum must be invested in Louisiana 6's, at 75 per cent, to realize an income of $3000 per year 2 Ans. $37500. OPERATION. $100 par of investment assumed. 6% rate of interest. * 00 $6.00 amount of income on $100 stock, $50000 par amount of investment. 75% cost of $100 stock. $37500.00 cost of $50000 stock or amt. of investment. Explanation.-Here we assume $100 as the par of the investment, and find the 6 per cent income on the same to be $6. Then, with these ratio figures and the $3000 income, we make the proportional solution statement by placing the $100 par of investment on the statement line, and reason thus: If $6 income require $100 par of investment, $1 will require the 6th part, and $3000 income will require 3000 times as much. The result of this is $50000, par of the investment, which multiplied by the price, 75 per cent, gives a cash investment of $37500. 2. How many shares of bank stock, of $50 each, must be bought at 10 per cent premium in order that an income of $1500 may be realized, on the presumption that the semi-annual dividends will be 6 per cent, and what will be the amount invested ? Ans. 250 shares. $13750, amount invested, OPERATION. $ $12500 par of stock. 100 1250 10 per cent premium. 12 || 1500 $12500 par of stock -- $50 = 250 shares. $13750 amount invested. 3. If 6 per cent bonds are selling at 5 per cent discount, what amount must I invest in them in order to receive a yearly income of $1200? Ans. $19000. * PROBLEMS IN STOCKS AND BONDS. 691 } 1264. The cost of Stocks, and the premium or discount at which they were bought, given, to find the par value. 1. Bought railroad stock at 70 per cent discount, and paid $7.50 per share. What is the par value # OPERATION. $100 par value assumed. 70 = 70% discount. $30 cost of $100 par. Ans. $25. Explanation.—Having no face of stock to work on, we hence assume $100 as the par value on which we calculate, and then deduct therefrom the 70 per cent discount, and thus obtain $80 as the cost of $100 stock at 70 per cent discount. This gives us the necessary ratio figures to solve the question; and in the line statement of the $ Solution we place the $100 par value on the line, | 100 and reason thus: Since $30 cost at 70 per cent 30 || 7.50 discount requires $100 par value, $1 will require | *== the 30th part, and $7,50 will require 7,50times $25.00 Ans. as much. 2. Sold bank stock for $55.50 and gained 11 per cent. What is the par of the Stock # Ans. $50. OPERATION. $100 par assumed. $ 11 = 11% premium. 100 E-º-º-º- 111 || 55.50 $111 cost. mºmsºme | $50 Ans. 1265. The time that Bonds have to run and the interest they draw, given, to find the price at which they must be purchased so that, when paid at maturity without estimating the use of the annual interest, the amount received will equal a certain rate per cent on the investment. 1. What must be paid for State bonds bearing 4 per cent interest and having 8 years to run, in order to make the investment equal to 5 per cent ? No allowance to be made for interest on the interest of the bonds paid annually, semi-annually or quarterly 3 Ans. $94,285-H. OPERATION. $100 face of bond. $100 investment. $ 4% interest. 5% interest. 100 140 || 132 $4.00 interest for 1 year. $5.00 interest for 1 year. 8 years. 8 years. $ 40.00 interest for 8 years. $94,285-H investing price of the bonds. $32.00 interest for 8 years. 100.00 face of bond. 100.00 face of bond. $132.00 = value of bond at $140.00 = value of investment maturity. at 5% at the end of 8 years. Ea:planation.—As shown in the operation, we first find the value of a $100 bond and the value of $100 investment each for the time at the respective rates per cent. Having these respective values, we see that $100 at 4 per cent produces $132, and at 5 per cent, $140; and as we wish to realize 5 per cent on the investment, we place the $100 assumed value on the statement line, and with the ratio numbers produced reason thus: Since $140 value in 8 years at 5 per cent requires $100 investment, $1 will require the 140th part, and $132 value will require 132 times as much, which gives $94,285-i- investing price of the bonds. NOTE.—This method does not take into consideration the interest on the interest received annually on the bonds. To make such an allowance the price of the bonds would be a little higher according as the payments of interest were made annually, semi-annually or quarterly. See the problems following, for such allowance. Also see problem on page 591. 692 SouLE's PHILOSOPHIC PRACTICAL MATHEMATICs. * 1266. The face of Bonds or Notes, the time to run, and the rate per cent of yearly, half-yearly or quarterly interest that they bear, given, to find the value at maturity. 1. What is the value at maturity of $1000 Louisiana 8's due in 5 years, estimating simple interest on the annual interest at 8 per cent? Ans. $1464. OPERATION. Face of bond, - - sº - agº - tº - tº - º - - - - - $1000 Annual interest for 5 years at 8 per cent, • * * * * * * * * * 400 Interest on $80 (the yearly interest on the $1000) for, 4 + 3 + 2 + 1 = 10 years at 8%, - 64 Value at maturity, - - - - - - - - - - - - - $1464 2. What is the value at maturity of a $10000 bond due in 10 years with 5 per cent semi-annual interest, allowing simple interest on the semi-annual interest at 5 per cent? : Ans. $16,187.50. OPERATION. OPERATION. To find the terms and years. | Face of bond, - - - - - - - - $10000.00 1. Semi-annual interest for 10 years at 5 per cent, º 5000.00 19 Interest on $250 (the semi-annual interest on $10000) - for the 190 terms of 3 years (the series of 1 to 20 × 9} = 190 – 2 = 95 years. 19 inclusive) = 95 years at 5 per cent, tº cº- 1187.50 Value of bond at maturity, º - - $16,187.50 1267. The face of Bonds or Notes, the time to run, and the rate per cent of yearly, half-yearly or quarterly interest that they bear, given, to find the present value or worth of the same in order to realize from the interest or dividends and final payment an equivalent to a given rate per cent per annum on the investment. 1. What is the present value or worth of a $10000 bond due in 10 years, with 5 per cent semi-annual interest, so that the investment may be equivalent to 10 per cent per annum payable semi-annually, allowing simple interest on all the semi- annual interests on the bond at 5 per cent? Ans. $6540.41. OPERATION To find the value of Bond at maturity. Face of bond, -- * tº - ſº tº tº º ºs º- º *i; }; Semi-annual interest for 10 years at 5 per cent, - ſº - tº - - " - tº- Interest on $250 (the semi-annual interest on $10000) for the 190 terms (the series of 1 to 19 inclusive) of 3 years = 95 years at 5 per cent, - - - , - º 1187.50 Value of bond at maturity, - * * s - sº * º - - $16187.50 OPERATION To find the value of a $100 Bond for 10 years, at 10% semi-annual interest, the investment rate. Present value assumed, - - - - - - - - - - - - - - $100.00 Semi-annual interest for 10 years at 10 per cent, • * * * * * * * 100.00 Interest on $5 (the semi-annual interest on the $100) for the 190 terms (the series of 1 to 19 inclusive) of 3 years = 95 years at 10 per cent, - - - - - - - 47.50 Value of $100 in 10 years at 10 per cent, semi-annual interest, - - - $247.50 4. PROBLEMS IN STOCKS AND BONDS. 693 OPERATION Explanation.—As shown in the operation, we To find the present value of the maturity value first find the maturity value of the bond. Then, of the Bond in order to produce the necessary relationship o numbers, representing present and maturity $ ASSUMED VAL. values according to the conditions of the prob- 100 lem, we assume $100 present value, to which we 247.50 | 16187.50 add the semi-annual interest and the simple | *- interest on the semi-annual interest for the 10 $6540.41 years at 10 per cent, and thus obtain $247.50, the maturity value of the $100 assumed value on the same conditions as we did for the $10000 bond. We now see that as $100 present value require $247.50 maturity value, by transposition $247.50 maturity value require $100 present value. The reasoning for the line statement with these relationship figures is so simple and has been given so often that we here omit it. THE OPERATION REVERSED To elucidate the problem still further, we present the figures produced by reversing the work. Present cash value of bond, * * * * * * * * * * * * $6540.41 Semi-annual interest on same for 10 years at 10 per cent, - º - - - tº- 6540.40 Interest on $327.02 (the semi-annual interest on $6540.41) for 190 terms of 3 years = 95 years at 10 per cent, - - - - - - - - - * - ºn are 3106.69 -** Value of bond at'maturity, - - - - - - - - - - $16187.50 2. What is the present value of a $10000 bond due in 10 years, with 5 per cent semi-annual interest, in order that the investment may be equivalent to 10 per cent per annum payable annually, allowing simple interest on the semi-annual interest or dividend of the bond accruing yearly at 10 per cent per annum ? Ans. $7040.82. OPERATION OPERATION To find the terms and years. To find the value of Bond at maturity. 9 Face of bond, Eº - - º sº. * - sº tº $10000. 1 Semi-annual interest on same for 10 years at 5 per cent, 5000 - Interest on $500 (the sum of two semi-annual payments 10 × 4} = 45. on the $10000) for 45 terms of years (the decreasing series from 9 to 1 inclusive) at 10 per cent, - -> 2250, Value of bond at maturity. - - - - $17250. OPERATION To find the value of a $100 Bond for 10 years, annual interest at 10 per cent. Present value assumed, - º tº - - º - -> º tºº - - - - $100 Annual interest for 10 years at 10 per cent, * * * * * * * * * * 100 Interest on $10 (the sum of two semi-annual payments on the $100) for 45 terms of years (the decreasing series from 9 to 1 inclusive) at 10 per cent, - - - - - - 45 Value of $100 in 10 years at 10 per cent annual interest, tº- º -> - - $245, OPERATION To find the present value of the maturity value of the Bond. Ea:planation.—The operation of this and the preceding problem is 100 245 | 17250 so explicit that a special explana- $7040.82 Ans. tion is considered unnecessary. THE OPERATION REVERSED. Present value of bond, m as * * * * * * * * * * = $7040.82 Annual interest for 10 years at 10 per cent * > * 7040.82 Interest on $704.08 (the annual interest on $7040.82) for 45 terms of years as above, at 10 per cent, º * * * * * * , s = º * * 3168.36, Value of bond at maturity, - - - - - - - - - - $17250.00 694 SOULE's PHILOSOPHIC PRACTICAL MATHEMATICS. * 3. , What is the present value of a $10000 bond due in 10 years, with 5 per cent Semi-annual interest, in order that the investment may be equivalent to 10 per cent per annum payable semi-annually, allowing simple interest on all the semi- annual interests of the bond at 10 per cent 3 Ans. $7020.20. OPERATION To find the value of the Bond at maturity. Face of bond, - tº a G gº tº gº tº gº º sº. tº sº tº - - - $10000 Semi-annual interest for 10 years at 5 per cent, - - s s *- tº E- s ſº-º: 5000 Interest on $250 (the semi-annual interest on $10000) for the 190 terms of 3 years (the decreasing series from 19 to 1 inclusive) = 95 years at 10 per cent, - tº {-} sº 2375 Value of bond at maturity, - - - - - - - - - - $17375 OPERATION To find the value of a $100 Bond for 10 years, semi-annual interest at 10 per cent. Present value assumed, - se tº º * -º sº ºs Exe sº * s wº- * * - $100.00 Semi-annual interest for 10 years at 10 per cent, eme s ∈ º ºs ºs º gº 100.00 Interest on $5 (the semi-annual interest on the $100) for the 190 terms of 3 years (the decreasing series from 19 to 1 inclusive) = 95 years at 10 per cent, * > tº gº 47.50 Value of $100 in 10 years at 10 per cent semi-annual interest, - tº tº - $247.50 OPERATION To find the present value of the maturity value of the Bond. $ 100 247.50 || 17375 $7020.20 + Ans. THE OFIERATION REVERSED. Present value of bond, - wº º gº- º * wº &= º * - tº tº tº º ſº $7020.20 Semi-annual interest on same for 10 years at 10 per cent, * > * > ºs im tº º gºe 7020.20 Interest on $351.01 (the semi-annual interest on $7020.20) for 190 terms of 4 years = 95 years at 10 per cent, - " - tº e º 4- * * * . * * se wº ſº, & ſº 3334.60 Value of bond at maturity, sº - - - - - - - - - $17375.00 4. What is the present value of a $10000 bond due in 10 years, with 5 per cent semi-annual interest, in order that the investment may be equivalent to 10 per cent per annum, allowing compound interest on the semi-annual payments of inter- est at 10 per cent 3 Ans. $7004.52. OPERATION To find the worth of the Bond at maturity. Face of Bond, * tºp wº tºº {-ºr sº es tºº & &º * gº º ge gº $10000.00 Semi-annual interest on same for 10 years at 5 per cent, tº- gº º sº gº tº 5000.00 Compound interest on $250 (the semi-annual interest on $10000) compounded annually at 10 per cent for the sum of the 3 yearly decreasing series of terms from 9% to # years inclusive, - * sº gº tº sº tº gº * gº tº E tº º 3167.93 Value of bond at maturity, - - - - - - - - - - $18167.93 OPERATION To find the present value of the maturity value of the Bond. $2.5937425 ) $18167.9300000 ($7004.52 Ans. 117325000 135753000 60658750 We here divide the maturity valne by the compound amount of $1 for 10 years at 10 per cent, 3} PROBLEMS IN STOCKS AND BONDs. 695 Whigh, for convenience we obtain from the compound interest table on page 612. The result or quotient, $7004.52, is the present value of the bond. Pºplanation.-In the first part of the operation, to find the compound interest on the $250 for the sum of the decreasing series of terms of # years, from 9% to 3, at 10 per cent annual compound interest, to economize time we used the compound interest tables; but as our tables are calculated 9nly for the integral or round years, we find it necessary to separate the 4 yearly decreasing series into two yearly series, viz.: A yearly integral series from 9 to 1 inclusive, and a yearly fractional . from 9% to 1% inclusive, and also the final 3 year. We then find in the table, in the 10 per cent COIll Iſln. The compound amount of the integral series from 1 to 9 years to be, tºº tºº tº º $14,9374246 From which we deduct the sum of 9 additions of $1 principal, sº gº tºº gº 9. And obtain, compound interest, - - - - - - - - - - - $5.9374246 Then to find the amount of the fractional series from 9 to 13: We multiply the amount of the full series $14.9374246 by $1.05; The product of this multiplication is $15.68429583, from which we deduct $9, the sum of 9 additions of $1 principal, and obtain, compound interest, sºme 6.684295.83 To these sums we add the compound interest on $1 for 3 a year at 10 per cent, which is * * ºn tº- sº tº gº & e º dº *E º ºs º .05 And thus obtain the total compound interest, - - - - - - - - $12.67172043 This we multiply by $250, the semi-annual interest on the bond, - gº * - * = 250 And produce, as the total annual compound interest on all the semi-annual inter- ests of the $10000 bond, gº sº tº º tº- gº tº s tº mº gº $3167.9301075 TO FIND THE VALUE OF BONDs UNDER WARYING CONDITIONs, STATED IN THE PROBLEMS. 1268. 1. What price must be paid for a $1000 bond bearing 4 per cent inter- est payable annually, having 8 years to run, so that the investment may be equal to 5 per cent 3 Ans. $935.367. REMARKS 1.-In questions of this kind, in which there are no conditions regarding, simple, annual or compound interest, compound interest is computed upon the annual payments of interest on the bond. (But not on the interest of the annual interest that may be realized on the investment). When such rate is not specified, the rate given in the bond is taken in the solution of the question. 2. The value of a bond in the market as an investment at a given rate, depends, other things being equal, upon the rate per cent interest at which the re-investments may be made of the various accruing interest payments. 3. When a bond is bought at a premium, some part of each interest install- ment received on the bond must be reinvested at compound interest, and so con- tinued until the maturity of the bond. The accumulations of this interest investment must amount exactly to the premium paid on the bond. For when the bond is paid, the face only will be received. 4. When a bond is bought at a discount, by the same reasoning as above, the interest installments received will be in excess compared with the actual amount paid for the bond, and hence there will be at each interest paying period a certain amount that may be expended and yet leave at the maturity of the bond the exact amount invested therein. NOTE.-For the solution of problems for bond investments, tables have been prepared by 696 soul E's PHILOSOPHIC PRACTICAL MATHEMATICs. * different authors, and are in general use by bankers, brokers, and investors, These tables are prepared on different plains, according to certain conditions of investing and re-investing, and by their use various investing results are shown without the tedious computations involving the prin- ciples of compound interest. In the first solution of the above problem, we will make use of the Compound Interest Table, page 611, and in the second solution we will use the table showing the Present Worth of an annuity, and the table showing Present Value of $1, when time and rate per cent are given. Index, Present Worth). EIRST SOLUTION. In this solution, we consider the problem as an annuity of $40 per annum for 8 years, with a payment of $1000 at the expiration of that time. And since the present worth of an annuity may be found by dividing the payment by the com- pound amount of $1 for the specified time and at the given rate per cent, hence we will so divide each of the 8 payments on the bond. Annuity Compound Yalue of Payments. Amount of $1. Annuity. 1st $40 — $1.05 = $38.09 2d 40 –– 1.1025 = 36.28 3d 40 — 1.1576 = 34.55 4th 40 — 1.2155 = 32.90 5th 40 — 1.2763 = 31.34–H 6th 40 — 1.34 - 29.85–H 7th 40 — 1.4071 = 28.42 8th 1040 – 1.4775 = 703.93–H Market value of bond on condi- $935.36+ tions named in the problem. IPROOF. Cost of bond, - - - , , , - - $935.36 Interest on same, 1 year at 5% tº 46.77 $982.13 Less interest on bond, 1st year, gº 40.00 Balance, - - - - - - $942.13 Interest on same, 2d year at 5% sº 47.11 $989.24 Less interest on bond, 2d year, {--> 40.00 Balance, - - - - - gº tº gº $949.24 Interest on same, 3d year at 5% * = 47.46 $996.70 Less interest on bond, 3d year, gº 40.00 Balance, - - - - - - - - $956.70 Interest on same, 4 years at 5% wº 47.84 $1004.54 Less interest on bond, 4th year, gº 40.00 Balance forward, - - - $ 964.54 (See SECOND SOLUTION. In this solution, we also resolve the question into an annuity as in the first solution. Then since the present worth of an annuity may be found by multiplying the amount of an annuity by the present worth of an annuity of $1 for the time specified and the rate per cent given, we therefore multiply the $40 annuity by $6.463213, which is the present worth of an annuity of $1 for 8 years at 5 per cent, as shown in the annuity tables, which see. Thus we obtain $258.52 as the value of the $40 annuity. Then to find the present worth of the $1000 bond, we multiply the same by $.676839, which is the present value of $1 for 8 years at 5 per cent as shown in the table of present Worth, which see. By this we obtain as the present worth of the bond, - " - - $676.84 Value of the annuity as above gº- 258.52-- Value of bond and annuities, - $935.36+ Balance brought forward, - - $ 964.54 Interest on same, 5th year 5% sº 48.23. $1012.77 Less interest on bond, 5th year, - 40.00. Balance, - - - - - - $972.77 Interest on same, 6th year at 5% - 48.64 $1021.41 Less interest on bond, 6th year, * } 40.00. Balance, - - - - - - - $ 981.41 Interest on same, 7th year at 5% - 49.07 $1030.48. Less interest on bond, 7th year, tº 40.00 $ 990.48 Interest on same, 8th year at 5% - 49.52 $1040.00 Less interest on bond, 8th year, tº a 40.00 Balance, - - - - - - $1000.00 Less bond redeemed, º gº tº 1000.00, 2. What price should be paid for a $500 bond bearing 6 per cent interest, due in 9 years, so that the investment will produce 8 per cent income, no allowance. to be made for the interest on the annual interest payments? Ans. $447.67+. 4× PROBLEMS IN STOCKS AND BONDS. 697 2’ OPERATION. Face of bond, $500.00 | Investment of - - - $500.00 500 Int, for 9 years at 6% 270.00 | Int. on same for 9 yrs. at 8% "360.00 860 | 770 Maturity value, $770.00 | Value of investment, tº $860.00 | — — $447.67+3 cash val. of bond. FIRST PROOF. Cash value of bond, $447,67+3 447.67+3 || $322.323# Maturity value of bond, 770.00 9 || 100 Gain on cost of bond in 9 years, $322.32%; | 8% gain on investment. SECOND PROOF. Interest on $447.67 for 9 years at 8 per cent, - - - - - - - - - $322.33 Cash value of bond, sº - sº gº - sº tº º * * tº-3 * = * . 447.67 Maturity value of bond, ſº tº- *º * - tº sº tº ge s tº 6 º' sº $7000 º 3. If in the above problem, interest at 6 per cent is allowed on the annual interest payments, what would be the market price of the bond to make the invest- ment equal to 8 per cent simple interest ? Ans. $485.34%. oPERATION. Face of bond, - - - - - $500.00 | Investment of, $500.00 500 Int. for 9 yrs. at 6%, - - - 270.00 | Int. 9 yrs. at 8%, 360.00 860 834.80 Int, on annual int. ($30) for 36 yrs. 64.80 mºmºs º ºsmºs , -— Value of investment, $860,00 $485.34%; val. Value of bond at maturity, - - $834.80 of bond. FIRST PROOF. Market value of bond as above, - - - - - - - - - - - - $485.35 Interest on same for 9 years at 8 per cent, - - - - - - - - - - 349.45 Maturity value of bond, - - - - - - - - - - - - - $834.80 SECOND PROOF, Cash value of bond, $485.34%; 349.45%; Maturity value of investment, 834.80 ** 100 Gain on cost of bond in 9 years, $349.45%; 8% gain. 4. If in problem 2 above, annual interest at 6 per cent is allowed on the $30 annual interest payments on the bond, and annual interest at 8 per cent is also allowed on the $40 annual interest payments on the investment, what would be the market price of the bond to make the investment equal to 8 per cent? Ans. $438.27—. OPERATION Face of bond, - - - - - $500.00 | Investment of, - ºp as sº - $500.00 Interest for 9 years at 6 per cent, - 270.00 || Int. for 9 years at 8 per cent, - - 360.00 Int. on annual int. ($30) for 36 yrs. - 64.80 | Int. on annual int. ($40) for 36 yrs, at 8%. 115.20 Value of bond at maturity, - - $834.80 | Val. of investment at the close of 9 yrs. $975.20 $500 investment. 975.20 | 834.80 $428.02 value, or market price of bond on conditions named. 698 SOULE's PHILOSOPHIC PRACTICAL MATHEMATICs. * PROOF, Market value of bond, - gº * s º º tº * tº a sº tºº tº º tº $428.02 Interest on same, for 9 years at 8 per cent, - tº gº º tº sº gº º ºs & 308.17 Interest on $34.24 (which is the interest on $428.02 for 1 year at 8 per cent) for 36 years (the series of 1 to 8 inclusive) at 8 per cent, iº tº ſ ºf tº º e. tº & e tºº 98.61 Maturity value of bond, - - - - - - - - - - - - - $834.80 |PRACTICAL MISCELLANEOUS EXAMPLES IN STOCKS AND BONDS. 1269. 1. When the annual dividend of stock is 12 per cent, and the interest of money is 10 per cent, at what premium ought the stock to sell? Ans. 20%. OPERATION INDICATED, 00 2 lo | $120 — 100 = 20% Ans. 2. What would be my income by investing $4100 in 6 per cent stock at 82 per cent 3 Ans. $300. 3. I bought stock at 6 per cent below par, and sold the same at 24 per cent above par. Paid brokerage 4 per cent for buying and 4 per cent for selling. My gain in the transaction was $750. What was the amount of my investment, includ- ing brokerage for buying, and what was the brokerage for buying and selling? Ans. $9450 investment. $100 brokerage. OPERATION. 6 % gain on purchase. $ 23% gain on sale. 100 15 2 84% total gain. 750 1 % = 3% brok. for buying and 3% for selling. $10000 par of stock. 7+% net gain. 69% discount. $600.00 discount. $10000 — $600 = $9400 + $50 brokerage for buying = $9450 investment. Brokerage on $10000 at #96 for buying and 3% for selling = $100. 4. A merchant sells 6 per cent bonds at 96, and invests the proceeds in 8's at par. How much per cent does he increase his income? Ans. 28%. OPERATION. $100 696 bonds. $96 8% bonds. $ 6 8 1.68 * =º tºº (6.00 | 100 $6.00 income on 6% bonds. $7.68 income on 8% bonds. 6.00 28% Ans. $1.68 gain. * PROBLEMS IN STOCKS AND BONDS. 699 5. A merchant invested $13725 in Louisiana 6's at 914; he held them until they rose to 94, then sold out and bought New Orleans City 7 per cent bonds at 70%. How much did he increase his annual income? Ans. $500. OPERATION, $ $ | 100 | 100 183 || 2 $15000 141 || 2 13725 94 14100 | $15000 $14100.00 $20000 % 7 O $900.00 $1400.00 – $900 = $500 Ans. The free Banking Law of Louisiana requires that the stocks deposited with the Auditor of Public Accounts as security for bank note circulation “shall always be, or be made equal to stock of the State bearing not less that 6 per cent interest.” The Auditor is prohibited from receiving “stock or bonds at a rate above their par value, or above their market value.” 6. What amount of circulating notes could a bank receive on stock that produces but 5 per cent? Ans. 833% of the par value of the stock deposited. OPERATION. $ 100 6 || 5 | $835 – 83% Ans. 7. Suppose in the above example that the stock or bonds deposited had been 7 per cent, and the market value 90 per cent, what amount of circulating notes Would the bank have received ? Ans. 105% of par value of stock. OPERATION. $100 par value. $ 90% 100 -sº 6 || 6.30 $90.00 market value. -* 7% | $105 = 105% Ans. $6.30 interest. 8. Suppose again that the stock or bonds deposited had been 8 per cent, and the market value 110, what amount of circulating notes would the bank have received ? Ans. 1334% of par value of stock. OPERATION. $ 100 6 || 8 | $133} = 1334% Ans. 9. In the last question, had the Auditor been allowed to receive bonds at their market value, which would have been a very impolitic transaction, what amount of circulating notes would the bank have received ? • Ans. 1463% of par value of stock. OPERATION. $100 alſ. 10 = 10% premium. $ -* 100 $110 market value. 6 || 8.80 8% — $8.80 interest $1463 = 1463% 7OO SOULE's PHILOSOPHIC PRACTICAL MATHEMATICs. X. 10. I invested $2598.75 in 4 per cent bonds at 74%, and after receiving a year's interest sold the bonds. My whole gain, including the year's interest, was $315; at what price did I sell the bonds? Ans. 794%. OPERATION. $ Whole gain, $315. | 100 Interest, 140. 297 || 4 - -- 2598.75 Gain on sale, $175. Cost of bonds, 2598.75 $3500 par value of bonds bought. 4% Selling price of bonds, $2773.75 $140.00 interest. $2773.75 –– $3500 = $793 = 7.94% selling price of $100 par of bonds. 11. A merchant owns $40000 of 6 per cent railroad first mortgage bonds, now selling at 85 per cent. He sells and invests the proceeds in 6 per cent bonds, at 20 per cent discount. What will be his increase of income per year? Ans. $150. TO FIND DIVIDENDS AND THE VALUE OF STOCK AFTER DECLAR-. ING CASH INSTALLMENT, STOCK, AND FORCED DIVIDENDS. 1270. 1. A capitalist subscribed $25000 stock in a bank, the capital stock of which is $1,000,000, but only 50 per cent paid in. A cash dividend of 10 per cent on the par value is declared and paid. What amount of money does the capitalist receive? What would be the market value of the 50 per cent paid stock to make- the investment equal to 12 per cent interest? Ans. $2500 dividend received. $834 market value of stock. FIRST OPERATION. SECOND OPERATION. $ $ 100 50 1 year's int. 12 || 20 one year's div. 6 || 10 $1663 - 2 = $83} 83% market value. 2. Suppose in the above example, that an installment dividend of 10 per cent. had been declared and credited to the stockholders on their unpaid stock, and that the market value of the stock before the declaring of the dividend was $60, what. would be the value of the stock after the dividend had been declared? Ans. $72. OPERATION. |; 50 || 60 lsº am. PROBLEMS IN STOCKS AND BONDS. 7or 3. Suppose again that the stock with 50 per cent paid, was worth in the market 60 per cent of its par, and that a stock dividend of 10 per cent be declared and delivered to the stockholders, what would then be the value of the stock # Ans. 54.4%. OPERATION. $ $ 60 120 55 50 110 || 100 OF $54, Ans. y $109 ºr value of par - 2 = $54+r Ans. 4. Suppose again that the stock with 50 per cent paid, was worth in the market 60 per cent of its par, and that a forced dividend of 10 per cent be declared and paid to the stockholders, what would then be the value of the stock? Ans. 54%. OPERATION. $ 120 90 $108 value of $100 par - 2 = $54 = 54%. $ 60 50 || 45 Or, 100 | $54 = 54% Ans. 5. Sections 15 and 16 of the United States National Banking law require that every banking association, after having complied with the provisions of the Banking Act, preliminary to the commencement of banking business under its provisions, shall transfer and deliver to the Treasurer of the United States any United States bonds bearing interest, to an amount not less than one-third of the capital stock paid in, and that the banking association making such deposit shall be entitled to receive from the Controller of Currency circulating notes equal in amount to ninety per cent of the current market value of the United States bonds so transferred and delivered, but not exceeding the par value thereof, if bearing interest at the rate of six per cent, or of equivalent United States bonds bearing a less rate of interest. In accordance with this law a bank deposited with the United States Treasurer $800000 United States 6 per cent bonds, the current market value of which was par. What amount of circulating notes can the bank receive from the United States Controller of Currency Ans. $720000. OPERATION. $800000 0% $720000.00 Ans. 6. Suppose in problem 5, that the current market value of the United States 6 per cent bonds deposited with the United States Treasurer, had been 110, what amount of circulating notes would have been received? Ans. $792000. OPERATION, $110 market value. $ $800000 6% interest 90 $110 99% 6 || 6.60 or, 90 ºmmº- $6.60 $792000.00 Ans. $99.00 = 99% $99.00 = 99% 7oz soul E's PHILOSOPHIC PRACTICAL MATHEMATICS. * 7. Suppose in problem 5 that the United States bonds deposited had been United States 44 per cent, at a current market value of 114, what would have been the amount of circulating notes received? Ans. $615600. OPERATION. $114 market value. $ $114 $800000 43% interest. 90 90 76.95 *==ºms -sºmimsº 6 || 5.13 Or, gºmºmºmºs ºmºmºmºms gººmsºmºmºmºmºmºmºmº $5.13 interest. $102.60 $615600.0000 $76.95 4% 6) $461.70 $76.95 8. Suppose again in problem 5, that the United States bonds deposited had been United States 5 per cent bonds, the current market value of which was 84 per cent, what would have been the amount of circulating notes received ? Ans. $504000. OPERATION. 84 market value. 5% rate of interest. $4.20 interest. 84 90 $ $800000 *== ! 90 63% 90 or, 7560 Or, | 84 gºmmºnsººmmismº 6 || 4.20 5 6 || 5 $504000 Ans. $63.00 = 63% 6) 37800 $63.00 = 63% $63.00 = 63% 9. Suppose again in problem 5, that the $800000 United States bonds depos- ited had been United States 6 per cent bonds, the current market value of which was $1134, what would have been the amount of circulating notes received ? Ans. $800000. OPERATION. $1.13% market value. $ Ea:planation.—In this problem we find that 6 % 90 the 6 per cent on the market value of the 6 | 6.81 bonds is equal to 61% ºf per cent on the par, and $6.81 interest. * hence, if 6 per cent par bonds give 90 per $102.15 = 102*f;96 cent issue of circulating notes, 6 ºf per cent will give 102.1% per cent issue; but in accor- dance with the law governing the case, as shown in the above extract, the Controller is allowed to issue only 100 per cent of circulating notes on the par of the bonds deposited. 10. A merchant subscribed $10000 in a banking incorporation, the paid up capital of which was $400000. A cash dividend of 4 per cent on the par value of the stock was declared. The paid up capital being 40 per cent of the capital stock, what per cent gain dots he receive on his investment, and what is the capital of the bank? Ans. 10% gain. $1,000,000 capital. 4 ºf PROBLEMS IN STOCKS AND BONDS. 7O3 TO CONSOLIDATE CORPORATIONS, WATER THE STOCK AND FIND TEIE MARKET VALUE OF THE WATERED STOCK. 1271. 1. Two railroad companies, A. with a capital of $25,000,000, and X. with a capital of $15,000,000 consolidated under the corporate name of A. & X. R. R. Co. New stock was issued to redeem the old in the following manner: (1). For every four shares of the stock of the A. road, five shares were issued. (2). The stock of the X. road was watered 20 per cent. What was the capital stock of the Consolidated Co., and how many shares of watered stock were added? Ans. $49,250,000 capital stock of Consolidated Co, 92,500 shares watered stock. OPERATION, 250000 = Shares of CO. A. 150000 = shares of Co. X. 62500 = shares of watered stock. 30000 = 20% watered stock. 312500 = shares issued. 180000 = shares issued. 180000 492500 = shares at $100 = $49250000 capital of Consolidated Co. 62500 + 30000 = 92500 shares watered stock. 2. In the above problem, suppose the market value of the stock of the A. road to have been before the consolidation, 120, and that of the X. road to have been 85, what would a share of the new consolidated company be worth, in the market? FIRST OPERATION. 1 share $100 of pure stock of Co. A. = 120 in the market. 1+ shares $125 of watered stock of Co. A. = 120 in the market. Hence º = $96 the value of 1 share watered stock of Co. A. 1 share $100 of pure stock of Co. X. = 85 in the market. 1 share $120 of watered stock of Co. X. = 85 in the market. Hence *** = $70% the value of 1 share watered stock of Co. X. $96 -– $703 = $166; -- 2 = $831; the value of 1 share of consolidated stock, SECOND OPERATION. $25,000,000 = capital of Co. A. 120 6,250,000 = watered stock of Co. A. 31250000 || 25000000 $31,250,000 = watered capital of Co. A. $96 value of one share. $15000000 = capital of Co. X. 85 6. 3000000 = watered stock of Co. X. 18000000 || 15000000 $18000000 = watered capital of Co. X. $70; value of one share. $96 -- 703 = 1663 – 2 = $83F2 value of one share of consolidated stock. 3. The paid up capital stock of a newly incorporated company is $300000. The business of the company gives evidence of very large profits, and as a conse- quence the market value of the stock is high, say $220 per share, and the demand greater than the supply. The stockholders therefore, in order to supply this demand 7O4 soule's PHILOSOPHIC PRACTICAL MATHEMATICS. * and enrich themselves, clandestinely resolve to “water the stock” to the extent of $200000. Accordingly they issue and divide pro rata among the stockholders $200000 new stock, thus increasing the capital to $500000. What would be the market value of the stock after it was thus “watered”? Ans. $132. OPERATION. $100 original stock worth $220, , $300000 original stock. 66; watered stock. 200000 watered stock. $1663 present stock. $500000 present stock. $ $ 220 220 1663 100 or, 500000 || 300000 | $132 Ans. - $132 Ans. 4. The capital stock of a corporation consists of $600000 regular, $200000 preferred, and $200000 guaranteed stock. A semi-annual dividend of 3 per cent is declared on the regular stock, 5 per cent on the preferred stock, and 6 per cent on the guaranteed stock. $7562.10 were carried to reserve fund. What was the net gain of the company during the six months business? Ans. $47562.10. To FIND THE MARGIN TO CARRY STOCK. 1272. 1. A speculator deposited $7000 margin with a broker and instructed him to purchase 500 shares of X. insurance stock and 150 shares of X. bank Stock. The margin is to be kept good at 10 per cent. The broker purchased at once the 500 shares of insurance stock at 90, and the 150 shares of bank stock at 110. At the close of six days, the insurance stock had depreciated 4 points and the bank stock had advanced 2 points. What additional sum must be deposited with the broker to maintain the margin at 10 per cent, allowing # per cent commission for buying, # per cent for selling, and 6 per cent interest? AnS. $1579.66. OPERATION. Dr. S P E C U L A T O R. Cr. To 500 shares at 90 - - - $45000.00 | By Int. on $7000 margin, 6 ds, at 6% 7.00 ** 150 shares at 110 tº * - 16500.00 | “ Value of Stock this date: ** Commission on 650 500 shares at 86 º - - - $43000.00 shares (a)"# per cent, - - 162.50 | 150 shares at 112 - - - - 16800.00 “ Int, on $61662.50, 6 ds. at 6%, 61.66 *s-º “ Commission for selling, - - 162.50 $59807.00 Total indebtedness to date, $61886.66 Credit as per opposite, - 59807.00 Loss to date, - - - $2079.66 650 shares at $100 par = $65000 at 10% = $6500, margin required. Loss to date, 2079.66 to be made good. Margin required, $8579.66 Margin deposited, 7000.00 Additional margin required, $1579.66 NOTE. 1.-In making statements of this kind, the commission for selling is considered due to the broker on the date the statement is made. For in case the required additional margin was not furnished to the broker, he would have the right to sell the stock at once and thus the commission would be earned. NOTE_2.—Margin is estimated on the par value, unless otherwise agreed. The margin is to protect the broker in case the stocks should decline in value. * PROBLEMS IN STOCKS AND BONDS. 7 of ACCOUNTS CURRENT AND INTEREST ACCOUNTS WITH BROKERS. 1273. June 19, 1895, Frank Soulé placed $5000 as margin with H. Lee, his broker, and instructed him to purchase the following stocks: 100 shares of Crescent City IR. R. stock; 200 shares of Metropolitan Bank stock, and 50 shares of Sun Mutual Insurance stock. June 21st the broker bought on account of the above order, 70 shares Crescent City R. R. stock at 105; 115 shares Metropolitan Bank Stock at 165 and 30 shares Sun Mutual Insurance stock at 1124. June 24, he bought 30 shares of Crescent City R. R. stock at 104; ; 85 shares of Metropolitan Bank stock at 1684 and 20 shares of Sun Mutual Insurance stock at 111. June 25, pursuant to instructions, the broker sold 175 shares of Metropolitan Bank stock at 1703; and June 28, he sold 50 shares Crescent City R. R. stock at 1043. July 3, a settlement was made with the broker. Make the bill of the broker showing balance of account, allowing interest on margin, on commission and on purchases and sales at 8 per cent, and # per cent broker's commission on both purchase and sale of stocks. OPERATION. Dr. Frank Soulé, in account with H. Lee, to July 3/95 at 8 per cent. Cr. Amount. || Ds. || Int, Amount. || Ds. Int. 1895 1895 June 21|70 sh. C. C. R. R. June |19|Cash deposited as stk. (a) 105, 7350|00 margin, 5000|00 21|115 sh. Metropol- Int. on same, 14 itan Bk. stk. (a) ds. (a) 8%, 14 || 15|56 165, 1897500 25|175 sh. Metropol- 21|30 sh. Sun Insur- itan Bk. stk. (a) ancestk. (a) 112}|| 337500 * 1704, 2.9881|25 21|Com. on 215sh. Ø Int. on same, 8 3. 96, 53|75 ds. (a) 8%, 8 || 53|12 21|Int, on $29753.75 28|50 Sh. C. C. R. R. 12 ds. (a) 8%, 12 79|34 stk. (a) 104%, 522500 24|30 sh. C. C. R. R. Int. On same, 5 Stk. (a) 104%, 3135|00 ds. (a) 8%, 5 5|81 24|85, sh. Metropol- itan Bk. stk. (a) Tot. credit int. 74|49 74|49 1684, 14322.50 July 11|By Balance, 9459|36 24|20 sh. Sun Insur- ance stk, a 111, 222000 24|Com on 135 sh. (a) #%, 3375 24|Int. on $19711.25 9 ds. (a) 8%, * 9 39|42 25|Com. on 175 sh, (a) +%, 43|75|| 8 08 28||Com. 50 sh. (a) #%, 12|50|| 5 01 Tot. debit int. 11885 118|85 * 49640.10|| 49640|10 NOTE: 1.—Commission is due as soon as a purchase or sale is made, and will draw interest from said time. NoTE 2.—See Accounts Current and Interest Accounts, in this book, pages 653 to 661. 7O6 SOULE's PHILOSOPHIC PRACTICAL MATHEMATICs. \ jºr ACCOUNT CURRENT WITH BROKERS, WITH “SHORT SALES.” 1274. 1. May 13, L. C. Soulé, deposited with his broker, S. Simon, $10000 and instructed him to sell short S. 60, on his account, 100 shares Citizen's Bank stock and 300 shares Carrollton R. R. stock. May 15th, said stock was sold at the follow- ing prices: Citizen's Bank stock at 85; Carrollton R. R. stock at 1094. July 10, notice was given to the buyer and stock was bought to “cover” short sales at the following prices: Citizen's Bank stock at 81; 200 shares Carrollton R. R. stock at 107, and 100 shares of same at 1054. Make the account showing balance due July 11, allowing 8 per cent interest on margin and on commission, but not on short sales, and compute the commission on sales and purchases each at # per cent? Ans. $1245.92 bal. due L. C. Soulé. Account which the student will verify: Dr. L. C. Soulé, in account with S. Simon, to July 11/95, at 8 per cent. Cr. Amount. || Ds. Int. Amount. || Ds. Int. 1895 1895 May 15|Com. on 400 sh. May 13|Cash deposit as Ø 4%, 10000 margin, 1000000 Int. on $100. 57 15|100 sh. Citizen’s ds. (a) 8%, 57 1|27 Bk. stk. (a) 85, 850000 July |10|100 sh. Citizen's 15|300 sh. Carrollton Bk. stk. (a) 81, 810000 R. R. stk. (a) 10|200 sh. Carrollton 1094, 32850|00 R. R. Stºk. (a) 107|| 21400|00 July 11|Int, on $10000 10|100 sh. Carrollton margin 59 d.s. R. R. stk. (a) 8%, 59 || 131|11 105+, 1052500 10|Com. on 400 sh. (a) 131|11 131|11 #%, 10000 Int. on $40125, 1 day @ 8%, 1 8|92 10|19 10|19 To Balance, 1245||92 41481|11 41481|11 NOTE 1.-Brokers do not allow interest on “short sales,” as there is no payment or receipt of money. NOTE. 2.-It is the custom of brokers to allow interest on cash sales, and to charge interest on purchases and also on the commission for buying and selling regardless of the condition of sale. REMARK.—See Accounts Current and Interest Accounts, pages 653 to 661. 2. August the 1st, 1895, a capitalist placed in the hands of Z., a broker, $25000 for investment and speculation in buying and selling gold, stocks, and bonds, for 60 days. Z. immediately purchased 200 shares of Texas Railroad stock at 60, “b. 26,” no margin required, and sold “short” 400 shares of New Orleans, Mobile and Chattanooga Railroad stock at 1024, “b. 10.” Five days after Z. called in the Texas Railroad stock and sold it to A. at 61%; August 5, he bought $20000 of five X- PROBLEMS IN STOCKS AND BONDS. 707 twenties at 1094; at the same time the party to whom the New Orleans, Mobile and Chattanooga Railroad stock was sold, called it in; Z. paid the difference, the stock being valued at 1023; August 24, Z. sold the five twenties at 110+ cash, and at the Same time made a further sale of $15000, in the same kind of bonds, at 1104, “s. 20," which he purchases and delivers in eight days, at 109%; September 17, Z. sold $20000 in gold at 1374, “s. 10, " which was not delivered at the expiration of the ten days, but settled at 137; Z, then bought “flat” $25000 City 8's at 91, and sold the same “flat” at 954. The capitalist and broker now settled. By the terms of the contract, Z. is to receive # per cent brokerage on the par of actual purchases and sales, no interest is to be allowed. What amount of money is due the capitalist? Ans. $26375. OPERATION. Capitalist in account with Z. Dr. Cr. . By cash deposited, gº º gº º $º gº se wº #Et sº 4 * * $25000.00 To 200 shares Texas R. R. stock at 60, t- * --> tº- sº cº - $12000.00 “. .9% brokerage on same, tº tº- º gº tºº ſº sº º 25.00 By 400 *:::. O. M. & Chattanooga R. R. stock at 1024, short b. 10, $4 tº To #96 brokerage on same, tº e º sº. tº º sº $º º 50.00 By 200 shares Texas R. R. stock at 61%, * * gº tº gº tº º 12375.00 To #% brokerage on same, - &= gº gºe tº º tº-> *-*. * - 25.00 “ $20000 U. S. 5–20's at 1094, & gº tº tº sº eme sº mº 21825.00 “ #96 brokerage on same, - - tº as tº º gº º ºs 25.00 “ loss on 400 shares N. O. M. & Chattanooga R. R. stock, bought at 102} and settled at 1024, $41150, se tº e º º ºs 150.00 By $20000 U. S. 5–20's at 110+, tº º sº º º & * > gº tº 22050.00 To #96 brokerage on same, tº sº tº º tº- tº º 25.00 By $15000 U. S. 5–20's at 110}, sº em º gº ºn a tº gº º 16518.75 To #96 brokerage on same, tº º tº ºne ſº & E º 18.75 “ $15000 U. S. 5–20's at 1094, - - - tº ſº tº gº tº 16481.25 “ #96 brokerage on same, - - - - - - - - - 18.75 By $20000 gold, s. 10, at 1374, $27500. To #96 brokerage on same, * tº º ºs sº ºne º º 25.00 By gain on $20000 gold, bought at 137} and settled at 137 ($27.400), * 100.00 To $25000 City Warrants at 91, - tºº gº * cº gº gº tº 22750.00 “ #96 brokerage on same, - - - - - - - - - 31.25 By $25000 City Warrants at 954, - - - - is tº & ºne tº 238.12.50 To #96 brokerage on same, * * * > tº º sº me tº 31.25 Capitalist's total debits, tº tº dº dº- tºº cº * * dº $73481.25 Capitalist's total credits, sº º sº ºr me tº dº sº $99856.25 73481.25 Amount due the capitalist, - tºº tº Gº tº º & tº º $26375.00 UNITED STATES STOCKS AND BONDS IN EUROPE. 1275. The principal markets in Europe, for United States stocks and bonds, are London, Paris, Hamburg, and Frankfort. The European quotations are the prices of $100 bonds or stocks in U. S. gold, but not at the U. S. valuation, as will be seen in the two following articles. The accrued interest on the stock or bond at the date of purchase generally constitutes \ wº 708 SOULE's PHILOSOPHIC PRACTICAL MATHEMATICs. º a part of its value. In some places however they are sold “flat” and then the accrued interest must be added. The London quotations of United States stocks and bonds are based upon an assumed value of the pound sterling. This assumed value is $5, modified by the rate of exchange. Hence to find the actual value of English quotations of Ameri- can Securities, we must multiply the quoted price by the rate of exchange. Thus, if Illinois Central R. R. stock is quoted in London at 103; and the rate of exchange is 4.87; we find the actual value of the quotation by the following operation: 103.75 × 4.875 łº 3) In Paris at the Bourse, the quotations are based upon the conventional value of 5 francs for $1, while the actual value of a dollar in exchange transactions is more than 5 francs. Hence to find the true American value of the Paris Bourse quotations, we must deduct therefrom the exchange rate. The German price of American Securities is the marks given for $100 par of American stocks or bonds. The rate of German exchange is based, by American bankers, upon the equivalent value of 4 marks expressed in dollars and cents. The exchange par of 4 marks is 9542; the rate is 95% more or less according as premium is eharged or discount allowed. See German Exchange. To ascertain the value of bonds for remittance and sale in Europe, bankers base their computations upon the buying rate of exchange. To ascertain the net proceeds of American stocks or bonds sold in Europe, allowances must be made for cost of remittance, insurance and commission; and if bills of exchange are drawn against the net proceeds, allowance must also be made for the selling rate of exchange. TO FIND THE PRICE OF STOCKS IN THE UNITED STATES, WHEN THE ENGLISH QUOTATION AND THE RATE OF EXCELANGE ARE GIVEN. PROBLEMIS. * 1276. 1. Chicago, Milwaukee and St. Paul Railway stock is quoted in London at 63% and the rate of exchange is 4,864. What is the true value of the stock? Ans. $62,03. OPERATION INDICATED. 63.75 4 || 255 63.75 × 4.865 5 || 4.865 OT, 2 || 973 or, – te 5 5 2. Louisiana 4's are quoted in London at 96%, exchange is 4,854. What is the equivalent American quotation? Ans. $94,02. 4× PROBLEMS IN STOCKS AND BONDS. 7O9 TO FIND THE PRICE OF UNITED STATES SECURITIES IN LONDON, WHEN THE AMERICAN QUOTATION AND THE RATE OF EXCEIANGE ARE GIVEN. PROBLEMIS. 1277. 1. New York Central and Hudson River Railway stock is quoted in New York at 112}, exchange is 4.863. What is the equivalent price in London? Ans. $115.20+. OPERATION INI) ICATED, 112.125 112.125 × 5.00 4.86625 || 5.00 or, 4.86625 2. When the Southern Pacific 1st mortgage 6 per cent bonds are quoted in New Orleans at 1214, exchange being 4.87%, what should be the equivalent London quotation? Ans. $124,294-. OPERATION INDICATED. - || 121.25 121.25 × 5.00 4,8775 || 5.00 Or, --ms- 4,8775 OPERATIONS IN FRENCEL AND AMERICAN QUOTATIONS. PROBLEMIS. 1278. 1. If the Paris Bourse quotations of United States 4 per cent bonds are 984, exchange 5.15, what would be the equivalent value in the United States market? Ans. $95.63+. OPERATION. The $984 Paris Bourse quotation or price being based upon the conventional value of 5 francs for $1, which is too high a value, it must therefore be reduced in the ratio of 5.15 francs (the exchange value of $1) for 5 francs. Statement to thus reduce it. 5 × 98.50 1 5.15 5.00 or, –— 98.50 5.15 $95.631 U. S. market value of Bourse quotation. 2. If the Paris Bourse quotation of United States bonds is 1063, exchange 5.644, what would be the equivalent value in the United States market? Ans. $94.552--. OPERATION INDICATED. $ 1 106.75 × 5.00 11.29 2 Or, 5.00 5.645 4 || 427 $94,552+ U. S. market value. See Foreign Exchange in this work for Extended Operations in Exchange. ſº – ; (Exchange. •e--------------------RN —-º'-- * * w-wºr-w 1279. The term Exchange has various significations or meanings, according to the Sense or manner in which it is used. The general acceptation of the term is the act of giving or receiving one thing for another. It is also used to designate the place where merchants, brokers, or other classes of business men assemble to transact business at certain hours as the Cotton Exchange, the Produce Exchange, etc. When used in this sense, the term is often contracted into “change”; as at change or on change, meaning at the exchange. EXCHANGE, when the term is used by commercial men and bankers in con- nection with transactions of finance, signifies the method or system of paying debts in distant cities by means of BILLS OF EXCHANGE, without the shipment or trans- mission of money. In this sense we here use the term, and now treat the subject. To elucidate the workings and to show the practical utility of exchange, let us suppose, 1. That the sugar and cotton merchants of New Orleans ship their sugar and cotton to New York for sale, and receive in payment currency or coin, which is forwarded to them by express. 2. Suppose that the dry goods and grocery merchants of New York ship their goods to New Orleans for sale, and receive in payment currency or coin, which is forwarded to them by express. Now it is clear that if the sugar and cotton purchasers in New York had paid the grocery and dry goods merchants there, and the grocery and dry goods purchasers in New Orleans had paid the sugar and cotton merchants there, the whole business might have been settled without the expense and risk that attend the shipping of currency or coin from one place to the other. This would be accomplished by the sugar and cotton merchants drawing drafts or bills of exchange on the sugar and cotton buyers in New York in favor of the grocery and dry goods buyers of New Orleans, who having loought and paid for these drafts, could remit them to the grocery and dry goods sellers in New York, in payment of their purchases. On presentation to the sugar and cotton buyers, these drafts would be paid, and in this manner the debts due from New York to New Orleans and the debts due from New Orleans to New York are both paid without the double shipment or transmission of eurrency or coin from each place to the other. It is evident that to make this system general, and to include all kinds of trade in various places, it would frequently be very difficult for those having bills of exchange to sell, to find purchasers, and viee versa. To obviate this difficulty, exchange brokers and exchange banks are established. \ They buy bills with their own capital from those who have money to receive, and send them forward for credit, and then sell their own bills drawn against this credit in such amounts as purchasers may require. * The exchange broker and exchange bank continue the purchase and sale of bills of exchange in this manner, charging a commission only for their services, use of capital, and risk incurred in buying bills that might not be honored when presented, until the demand to purchase bills becomes greater than the supply of bills offered for sale or vice versa. Then they either sell their bills at a premium or par, and buy at par or discount. When the demand is in excess of the supply, they will charge not only a commission, but a. premium sufficient to cover the expense of freight and insurance to ship the funds to meet their (710) * EXCHANGE. 7 II bills at maturity, or to pay the interest on the amount of their over-drafts until the commerce of the two places shall have adjusted the difference. They would also, in this case, buy at a premium. When the supply is in excess of the demand, they will sell at a discount sufficient to cover the expenses of shipping their funds from the place where the drafts are payable to their own place of business. The principal cause that produces the excess of supply or demand of bills of exchange, to which we have here made mention, is the BALANCE OF TRADE. 1280. Over-trading, or the balance of trade, is in itself a very important subject, and is worthy of the profoundest consideration by business men. In this connection we do not propose to discuss the subject, but for the infor- mation of students we will briefly define it. By the “BALANCE OF TRADE'' is meant the monetary excess of business transactions that one country or city has over another with which she deals. For example, if the United States sells more to England than England sells to the United States, then the balance of trade is in favor of the United States. If the reverse is the case, then the balance of trade is against the United States. Or, if New Orleans sells more to St. Louis than she buys from St. Louis, then the balance of trade is in her favor; but if the reverse is the case, then the balance of trade is in favor of St. Louis. 1281. ORIGIN OF EXCELANGE, The origin of negotiable bills of exchange has been a subject of discussion on the part of legal and commercial writers for ages, and it is yet undecided in what age and among what nation they were first used. Some claim that they were known among the Romans and Grecians, but there is no proof to substantiate the claim. It is granted however, on good authority, that such a transaction as a request by A., in Rome, to B., in Alexandria, to pay to C. on his, A’s account, the money that B. owes A., was of common occurrence; but a request of this character, and a com- pliance there with, do not constitute a bill of exchange. , In 1272 bills of exchange were in use in Venice, but there is no evidence to show that they were then negotiable. There are, however, records of them at an earlier date, and various theories have been presented, on questionable evidence, to account for their origin. The French writers ascribe their origin to the Jews in 1181, when oppressively and wickedly expelled from their homes, in the kingdom of France, by Philip Augustus. But this date and circumstance, or cause of origin, is questioned by other writers. Mr. Reddie, in his Historical View of the Law of Maritime Commerce, says, “that if we inquire what cause has led to the invention, the true answer is, the necessities of commerce; but that if we inquire who were the inventors, in what position and by whom these necessities were most strongly felt, and what persons, experiencing the urgency of these necessities in the most lively manner, produced the thing invented, it would be absurd to call the extension of commerce the inventor, for this would be to confound the mover with the agent. It is also highly probable that the Jews, being in these ages, as we have seen, the chief money lenders, persecuted from mistaken religious views and on account of their alleged pecuniary extortions, scattered over the European kingdoms, yet in a manner forced to keep up a pretty con- stant communication with each other, clever and acute naturally, and comparatively skillful in such business from having been trained to it for generations, were really the first inventors of bills of exchange in a rude state.” But he concludes that there is no evidence that they made the invention at the precise time and by reason of their expulsion from France. 712 SOULE's PHILOSOPHIC PRACTICAL MATHEMATICs. º: 1282. THE IMPORTANCE OF EXCHANGE. Money was the first great Invention by or through which commercial transactions were extended and facilitated. It answered, in the language of Mr. Parson, in his treatise on Notes and Bills, “all the purposes for which it was wanted, for many ages. It satisfied all the requirements of social life, and of commerce, through the early Eastern empires, and those of Greece and Rome. It is said, but upon somewhat doubtful authority, that some kind of paper money was used in Tartary, or China, or Japan, a thousand years ago; but nothing is known certainly about this. In Europe, gold and silver money were the only circulating medium, and were sufficient; but five or six hun- dred years ago, the discovery was made of a new circulating medium, bills of exchange, of which it is the characteristic quality that it represents that which represents everything else. “The use of money enlarged human intercourse, or so much of it as may be included in the widest sense of the word commerce. It made interchanges possible and easy, which would other- wise have been very difficult, if not impossible. We cannot imagine, for example, the whole commerce of Greece and Rome, or a hundredth part of it, carried on by actual barter of commodi- ties. Precisely in the same way, the invention and use of paper to represent money gave a new enlargement to commercial intercourse, and greatly increased its facilities and its possibilities. For we could not now suppose the commercial intercourse between America and Europe, for example, to be carried on wholly by actual exchange of the precious metals, as must be the case if bills and notes were abandoned, without a cost and hindrance which would be fatal to a very large part of it. The invention and use of money conferred upon mankind the vast benefits which have ever flowed therefrom, because money represented all other commodities, and for no other reason whatever. He who had any superfluities on hand was no longer obliged to take the trouble of storing them and the risk of their destruction, or to save them by exchanging them for the superfluities of some accessible neighbor, whether these were precisely what he wanted or not. For now he might sell his superfluities, and their value was then invested in something easily preserved, and which could always be exchanged for the very article he wanted, as soon as he found it within his reach. But after a time, this exchange was to be made in such quantities, and at such distances, that it cost too much in time and trouble to be profitable; and here is a natural limit to commerce by mere money, which seems to have been reached by the nations of Europe some few centuries ago. “Beyond this, therefore, it is plain that commerce could not have grown, unless new facili- ties, by means of new instruments, had been provided for it; and this was done by the invention and use of bills of exchange. Just as, by the help of money, a hundred oxen could be exchanged for a hundred pieces of cloth, at distances which would have made the actual transfer of the oxen and the cloth too onerous to be advantageous, so now commercial transactions which would have required large bags or boxes of money to be sent back and forth at great cost, both of time and money, and with much trouble and some hazard, can be carried into full effect, with equal prompti- tude, safety, and facility, by exchanging small pieces of paper. And these two inventions, one made at the beginning of human society and the other but a few centuries since, are useful for precisely the same reason: Money represents all commodities, and 80 prevents the necessity of an actual exchange of commodities; and bills and notes represent money, and so prevent the necessity of an actual transfer of money. “Moreover, as gold and silver were first used by weight, and it was a distinct though very speedy improvement to coin them into money, so bills of exchange were first used only for the benefit of a specified payee, but were soon perfected into the indispensable instrument of commerce which they now are by being made negotiable. For this adds an entirely new element to their utility. By means of indorsements, which may be extended indefinitely, negotiable paper not merely makes money in one place or at one time, but becomes money in another place or at another time, without actual transfer; and not merely makes credits the equivalent of money, but it represents and carries with it the accumulated credit of all who become parties to the paper.” * EXCHANGE. 7I 3 BILLS OF EXCHANGE. 1283. A Bill of Exchange is a written order for the payment of a specified sum of money unconditionally. (The bill may be printed with the exception of the signature of the drawer, which must be written). Bills of exchange are divided into two classes, Domestic or Inland, and Foreign. A domestic or inland bill is one drawn and made payable in the same country. A foreign bill is one drawn in one country and made payable in another. NOTE.-It is generally understood that a bill drawn in any city and State of the United States and made payable in any other city and State of the United States, is a domestic or inland bill; but it has been decided by our courts, that a bill drawn in one State and made payable in another, is a foreign bill, for the reason that each State is a sovereign government in many particulars. But nothwithstandin g this legal decision, in commercial parlance, domestic exchange means exchange on any of the States and territories of the United States. ...For the form of Bills of Exchange, Drafts, Notes, etc., and for an explanation of the parties #.# ºrge, and for other forms of commercial instruments of writing, see pages 549 to OI UIllS OOOK. 1284. SET OF FOREIGN BILLS OF EXCELANGE. 361800.9.7. NEW ORLEANs, April 22, 1895. Sixty days after sight of this First of Exchange (second and third unpaid), pay to the order of J. M. Butchee & Co., Eighteen Hundred Pounds, Nine Shillings and Seven Pence, Sterling. TO MESSRs. QUEALY & HARVEY, Liverpool, Eng. MEAD & WELSH. 3.1800.9.7. NEw ORLEANs, April 22, 1895. Sixty days after sight of this Second of Exchange (first and third unpaid), pay to the order of J. M. Butchee & Co., Eighteen Hundred Pounds, Nine Shillings and Seven Pence, Sterling. To MEssRs. QUEALY & HARVEY, Iliverpool, Eng. MEAID & WELSH. 39.1800.9.7. NEw ORLEANs, April 22, 1895. Sixty days after sight of this Third of Exchange (first and second unpaid), pay to the order of J. M. Butchee & Co., Eighteen Hundred Pounds, Nine Shillings and Seven Pence, Sterling. TO MESSRs. QUEALY & HARVEY, Liverpool, Eng. MEAD & WELSH. NOTE.-Exchange is drawn in sets in order to secure greater safety in transmission through the mails. All the bills of a set are of the same tenor and date, and are sent by different mails and when either is received and paid the other or others become void. In domestic exchange, it is a recent custom of many banks to fill but one exchange, and if that should be lost, then to fill the duplicate. In foreign exchange, many merchants mail by different steamers two of the set and hold the third. 7 I4 SOULE's PHILOSOPHIC PRACTICAL MATHEMATICs. . A. LETTERS OF CREDIT. . . 1285. A Letter of Credit is a written instrument drawn by a banker, instructing any of several correspondent banks in different cities and countries, to pay to the holder such sums of money as he may wish, not exceeding the amount specified in the Letter of Credit, and promising to re-pay the sums so advaneed to the person or persons advancin g them, or to honor his or their bills drawn for such amounts as they may have individually furnished. ... . N9 tº 1.-Letters of Credit are drawn in different forms according to the ideas of the parties issuing them, and the requirements of persons applying for them. NOTE.2-A Letter of Credit differs from a bill of exchange in that it is payable at different Places, at different times and in different sums. While a bill of exchange is payable at a certain Place, at a certain time and for a specified amount. LETTERS OF CREDIT are of great convenience to travelers by enabling them to draw such sums at such times and at such places as they may wish. To procure a Letter of Credit, the party is required to make a deposit of cash or Securities with a banker who is prepared to issue them, for the amount of the Letter of Credit. An Interest Account is kept by the bank or banker issuing the Letter of Cre- dit with the holder thereof, and at the close of the journey, an account is rendered and settlement made, of the interest and of the unexpended balance, if any. The interest allowed on the deposit varies from 13 to 3 per cent, and the interest charged on the payments is the current rate. When the holder of a Letter of Credit wishes to draw any money, he presents the same to any banker named therein or on the list of the drawer's correspondents, who will pay him the amount he wishes, in the currency of the country in which he may be, at the regular rate of exchange then existing, and either take his receipt or a draft on the drawing bank for the amount. This receipt or draft is remitted to the drawing bank and is charged to the account of the payee or holder of the Letter of Credit. Foreign bankers who make payment on Letters of Credit, usually charge the payees or holders a small commission, varying according to amount paid, from # to 1 per cent. Form of a Letter of Credit. No. 504. Office of HARNEY, CADY & Co., Bankers, For £350. NEw York, July 1, 1895. To our Correspondents : GENTLEMEN.—We have opened a credit for Three Hundred Fifty Pounds Sterling, in favor of Mr. A. L. Green, whose name appears below in his own hand-writing, and we respectfully request that you honor his drafts on us, at sight, for such funds as he may require, not exceeding the available extent of this letter, indorsing all such drafts paid by you on this credit, which will continue in force until January 1, 1896. Very Respectfully, 2%ary / &4. & Gó. (Signature of the holder). CZ 2% 22%ae. NOTE.—The purchaser of this Letter of Credit is supplied with a list of Harney, Cady & Co's correspondents and their addresses, to whom he is authorized to present it. This list is often printed on the Letter of Credit. º: EXCHANGE. 715 AMERICAN EXPRESS COMPANY TRAVELERS’ CEHECKS. 1286. As an equivalent of the Bankers’ Letters of Credit, the American Express Co. now issue travelers' checks, payable in various parts of nearly all the countries of the world. Like Letters of Credit, these checks are a great service to the traveling public. They may be used by different members of a family or party, are of convenient form, are easily utilized as money, offer a safe and a cheap way of carrying money when traveling in our own or in foreign countries, and they may be purchased in hundreds of cities where Letters of Credit cannot be obtained. Form of an Express Company Travelers’ Check. WHEN COUNTERSIGNED BELOW 2-ºricanºpressºs WITH THIS SIGNATURE. * º Chêque *E OOOOOO 789. ſimtriran ºxpress (jumpani, Will Daytſ the Orlºſ Of #20.00; —s º In United States and Canadal; ; ; ; GERMANY §. §ºhol and º £TSTD Fr.) (;t," Mks. Pf. Imreſt. Kr.TOre|FIF. Tº G 7%pen?y Øollars. #|f|}|{2|}|º]};}} ; 39 49 || O2 COUNTERSIGNED : (sEE signatur E ABovE). TREAS. TEIE FACE AND CEIARACTER OF FOREIGN BILLS. 1287. 1. Foreign Bills of Exchange are, with rare exceptions, drawn in the monetary unit or money of account of the country where they are made payable or in which the drawee resides. See Monetary Units of different nations. 2. Foreign Bills of Exchange are accepted and bear grace whether drawn at sight or on time. Bills are sometimes drawn with “acceptance waived, ” in which case they are not subject to protest for non-acceptance, but if not paid at maturity, would then be protested for non-payment. The most of our foreign exchange is effected through a few of the most prominent mercantile and financial centers of Europe, such as London, Paris, Hamburg, Amsterdam, Frankfort, Bremen, Antwerp, Vienna, Berlin, and Leipsic. In mercantile parlance, we have Short Sight Bills, Long Bills, Bankers’ Bills, Commercial Bills, Clear Bills, Bill of Lading Bills, or Documentary Exchange, A 1 Bills, and Prime Bills. 7 16 SOULE's PHILOSOPHIC PRACTICAL MATHEMATICs. * A Short Sight Bill is a bill drawn at 3 days sight. The most of English bills, and also other foreign bills, are drawn at 60 days sight, and are called Long JBills. Bankers’ Bills are those sold by bankers, Commercial Bills are those sold by merchants. Clear Bills are those that have no collateral security, but are negotiated Solely upon the responsibility of their makers and indorsers. Bill of Lading Bills or Documentary Exchange are those secured or protected by the assignment and transfer of the bill of lading, and policy of insur- ance, of Some merchandise or commodity consigned for sale under order at the maturity and for account of the bill. Bills are sometimes secured by other collaterals than bills of lading and policies of insurance. A 1 Bills are either commercial or bankers' bills that are considered by reason of the wealth, capacity and integrity of the makers and drawees, perfectly good. Prime Bills are generally regarded as synonymous with A 1 bills, but by some they are considered a shade better. We, however, cannot conceive how they can be better than A 1, when A 1’s are perfectly good. * PAR OF EXCHANGE. 1288. The Par of Exchange is the established value of the monetary unit of one country in that of another, and it is either intrinsic or commercial. The Intrinsic Par of Exchange is the value of the monetary unit of one country expressed in that of some other, and is determined by comparing the weight and purity of the coins of different countries. Thus the intrinsic par with England is now, 1895, $4.8665; with France it is $.193, and with Germany it is $.238. The Commercial Par of Exchange is the market value of the coin of one country when bought or sold in another; and this value varies somewhat according to the law of supply and demand. See English, French, German, and other Foreign Exchange for the quotations of Foreign Exchange. THE COMMERCIAL RATE OR COURSE OF EXCELANGE. 1289. The Rate of Exchange is the variable price that is paid for bills. There are three conditions on which bills of exchange can be bought and sold by first parties; they are, par, premium, or discount. To understand fully the reasons that combine to produce these three different rates, requires careful study of the whole subject of exchange. For domestic exchange, the following reasons are the principal ones governing the rate: The “balance of trade” and the probability of its remaining for some time in favor of * & EXCHANGE. g 717 or against the city where the bills are drawn; the opportunity or advantages pre- sented to the seller of the exchange to use his money in his own city or where the exchange is made payable; the rate of interest that the seller has to pay on his over-drafts; and the expense of shipping money to meet his over-drafts, or of shipping his surplus funds in another city to his own. The rate of exchange was formerly based exclusively upon per cent. But during the past few years many bankers throughout the United States base the rate for domestic exchange upon the M. (one thousand.) We shall therefore use both methods in this work. BUSINESS CUSTOMS TO BE OBSERVED IN BUYING AND SELLING EXCELANGE. 1290. When exchange is bought or sold through agents, it is the custom, in the absence of a special contract, to allow the agent a commission upon the face of the bill, plus the banker's premium, or minus the banker's discount. And when bought through an agent and a broker, to allow both the commission and brokerage on the face of the bill, plus the banker's premium, or minus his discount. When money is invested at a premium or discount, it is the custom of the banker, founded on equitable principles, to charge a premium or allow a discount on the face of the bill that the money will buy, instead of calculating the same on the amount invested. PROBLEMS IN DOMESTIC OR INLAND EXCHANGE. Problems in buying on our own account. 1291. 1. What cost a bill for $15000, bought at par 3 Ans. $15000. 2. What cost a bill for $15000, bought at 1 per cent premium ? Ans. $15150. 3. What cost a bill for $15000, bought at 1 per cent discount Ans. $14850. 4. What cost a bill for $15000, bought at $3.25 per M. premium ? Ans. $15048.75. OPERATION FOR THE 1 ST PROBLEM. $15000 face and cost of bill. OPERATION FOR THE 2D PROBLEM. OPERATION FOR THE 3D PROBLEM. $15000 face of bill. $15000 face of bill. 150 = 1% premium. 150 = 1 % discount. $15150 cost. $14850 cost. OPERATION FOR THE 4TH PROBLEM. $15000 $15000 3+ M. 3.25 = <=º or, – or, $15000 × 1003.25 = $15048.75000 cost of bill. $45.000 $48,75000 3.75 $48,750 = premium. 718 SOULE's PHILOSOPHIC PRACTICAL MATHEMATICS. * Problems in buying through an Agent. 5. What cost a bill for $15000, bought at par, paying the agent 1 per cent commission ? Ans. $15150. 6. What cost a bill for $15000, bought at 1 per cent premium, paying the agent 1 per cent commission ? Ans. $15301.50. 7. What cost a bill for $15000, bought at 1 per cent discount, paying the agent 1 per cent commission ? Ans. $14998.50. 8. What cost a bill for $15000, bought at $3.25 per M. discount, paying the agent 1 per cent commission ? Ans. $15100.76 OPERATION FOR THE 5TH PROBLEM. $15000 face of bill. 150 = 1% commission. $15150 cost. OPERATION FOR THE 6TH PROBLEM. OPERATION FOR THE 7TH PROBLEM. $15000 face of bill. $15000 face of bill. 150 = 1 % premium. 150 = 1% discount. $15150 face + premium. $14850 face — discount. 151.50 = 1 % commission. 148.50 = 1 % commission. $15301.50 cost. $14998.50 cost. OPERATION FOR THE 8TH PROBLEM. $15000 face of bill $14951.25 48.75 = $3.25 per M. discount. 149.51 = 1 % commission. $14951.25 face — discount. $15100.76.cost. Explanation.—In solving questions of this character, in conformity to business custom, we allow the agent a commission on the face of the bill, plus the banker's premium or minus the banker's discount. *- Problems in buying through an Agent and a Broker. 9. What cost a bill for $15000, bought at par, paying the agent 1 per cent eommission and the broker # per cent brokerage? Ans. $15225. 10. What cost a bill for $15000, bought at 1 per cent premium, paying the agent 1 per cent commission and the broker # per cent brokerage? Ans. $15377.25. 11. What cost a bill for $15000, bought at 1 per cent discount, paying the agent 1 per cent commission and the broker # per cent brokerage? Ans. $15072.75. 12. What cost a bill for $15000, bought at $3.25 per M. premium, paying the agent 1 per cent commission and the broker # per cent brokerage 3 Ans. $15274.49. º: EXCHANGE. 719 OPERATION FOR THE 9TH PROBLEM. $15000 face of bill. 150 = 1 % commission. 75 = } % brokerage. $15225 cost. OPERATION FOR THE 10TH PROBLEM. OPERATION FOR THI, 11TH PROBLEM. $15000 face of bill. $15000 face of bill. 150 = 1 % premium. 150 = 1 9% discount. $15150 face -- premium. $14850 face — discount. 151.50 = 1 % commission. 75.75 = } % brokerage. $15377.25 cost. 148 50 = 1 96 commission. 74.25 = # 96 brokerage $15072.75 cost. OPERATION FOR THE 12TH PROCLEM. $15000 face of bill. $15048.75 48.75 = $3.25 per M. premium. 150.49 = 1 % commission. sºmºsºsºm-ºsmº? 75.25 = } % brokerage. $15048.75 = face -- premium. *E-º $15274.49 = cost. Explanation.—Where the purchase of exchange is made through both an agent and a broker, or where a charge is made for commission and brokerage, in conformity to custom, in the absence of a special agreement to the contrary, we allow both the commission and brokerage on the face of the bill, plus the premium charged or minus the discount allowed. Problems in selling on our own account. 13. What will be realized for $15000 sold at par’? Ans. $15000. 14. What will be realized for $15000 sold at 1 per cent premium ? Ans. $15150. 15. What will be realized for $15000 sold at 1 per cent discount? Ans. $14850. 16. What will be realized for $15000 sold at $3.25 per M. premium ? Ans, $15048.75. NOTE.—For the solution of the above four problems, see the solution of problems 1, 2, 3 and 4, page 717. Problems in selling through an Agent 17. What will be realized for $15000 sold at par, paying 1 per cent commis- sion ? Ans. $14850. 18. What will be realized for $15000 sold at 1 per cent premium, paying 1 per cent commission ? Ans. $14998.50. 19. What will be realized for $15000 sold at 1 per cent discount, paying 1 per cent commission ? Ans. $14701.50. 20. What will be realized for $15000 sold at $3.25 per M. discount, paying 1 per cent commission? Ans. $14801,74, 72O soul E's PHILOSOPHIC PRACTICAL MATHEMATICS. jºr OPERATION FOR THE 17TH PROBLEM. $15000 150 = 1 % commission. $14850 amount realized. OPERATION FOR THE 18th PROBLEM. ſ OPERATION FOR THE 19TH PROBLEM. $15000 $15000 150 = 1 % premium. 150 = 1% discount. $15150 face + premium. $14850 face — discount. 151.50 = 1 % commission. 148.50 = 1 % commission. $14998.50 amount realized. $14701.50 amount realized. OPERATION FOR THE 201EI PROBLEM. $15000 $14951.25 48.75 = $3.25 per M. discount. 149.51 = 1 % commission. $14951.25 face — discount. $14801.74 = amount realized. Problems in selling through an Agent and a Broker. 21. What will be realized for $15000 sold at par, paying 1 per cent commis- sion and # per cent brokerage 3 Ans. $14775. 22. What will be realized for $15000 sold at 1 per cent premium, paying 1 per cent commission and 3 per cent brokerage 3 Ans. $14922.75. 23. What will be realized for $15000 sold at 1 per cent discount, paying 1 per cent commission and 4 per cent brokerage 3 Ans. $14627.25. 24. What will be realized for $15000 sold at $3.25 per M. premium, paying 1 per cent commission and # per cent brokerage 3 Ans, $14823.01. OPERATION FOR THE 21ST PROBLEM. $15000 1 % oommission $150 } 225 # 96 brokerage 75 240 Amount realized $14775 OPERATION FOR THE 22D PROBLEM. OPERATION FOR THE 23D PROBLEM. $15000.00 $15000.00 1 % premium, º ge =-> * 150.00 || 1 % discount, g; s ſº gº 150.00 1 9 $ $15150.00 $148 $14850.00 9% commission, $151.50 = | 1 % commission, $148.50 # % brokerage, '**ś, } gº º 227.25 # % brokerage, ? 74.25 } wº º- 222.75 Amount realized, tº cº e- $14922.75 Amount realized, gº * tº $14627.25 OPERATION FOR THE 24TH PROBLEM. $15000 1 % commission = - - - - $150.49 48.75 = $3.25 per M. premium. # 96 brokerage = tº sº tº as 75.25 $15048.75 $225.74 225.74 = commission and brokerage. $14823.01 = amount realized. NOTE.-In the foregoing 24 problems we have omitted extended explanations, believing that the operations alone will render them easily understood. The student must be careful to observe X- EXCHANGE. 72 I that the agent and broker are allowed, when buying or selling exchange, a commission and brokerage on the face of the bill plus the premium charged or minus the discount allowed. We now approach exchange work that is generally considered very difficult, and although by our philosophic system it is rendered comparatively easy and simple, yet it will require the exercise of unimpaired brains and a remembrance of the business customs pertaining to exchange operations, to fully comprehend it. We, therefore, here solicit the student’s earnest attention and closest reasoning. Problems in investing money in Earchange. 25. What amount of bill can be bought for $15000 at parº Ans. $15000. 26. What amount of bill can be bought for $15000 at 1 per cent premium? Ans. $14851.49. 27. What amount of bill can be bought for $15000 at 1 per cent discount? Ans. $15151.52. 28. What amount of bill can be bought for $15000 at $3.25 per M. premium ? Ans, $14951.16. OPERATION FOR THE 26TH PROBLEM. Eacplanation.—Since by business custom, and in this case strict justice, the banker is allowed $100 amount of bill assumed. a premium only on the face of the bill that the 1 = 1 % premium. money will buy, after paying the premium, it is *mº clear that we cannot compute the premium on $101 cash cost of $100 bill. the $15000, for were we to do so, we would be Computing premium upon premium, which is not allowed. To illustrate this we will make 100 the figures, thus: 1 per cent of $15000 is $150, 101 || 15000.00 ($14851.49 which deducted from the $15000 leaves $14850, ºmºsºmeºmºsº as the face of the bill, and as 1 per cent of $14850 is but $148.50, it is plain that it is incor- rect to work on the face of the sum to be invest- ed. Having, hence, no figures upon the face of which we can work, we are obliged to assume some number to represent the amount or face of the bill, and upon which we may work according to the conditions of the problem and produce relationship numbers of face and cost of bill, with which and the $15000 we may make a proportional solution statement. We then assume $100 as the face of the bill, and compute and add thereto the premium charged, and thus produce $101 as the cash cost of the $100 bill. This gives us the required relationship numbers with which and the $15000 we make the solution statement. In making this statement we first observe that as a $100 bill cost $101 in cash, by transposition $101 in cash will buy a $100 bill, and hence we can buy as many $100 bills as $15000 cost is equal to $101 Cash; but to make the operation clear and philosophic we place the $100 assumed face on the statement line, and reason thus: If $101, cash will buy a $100 bill, $1 in cash will buy the 101st part, and $15000 in cash will buy 15000 times as much, which is $14851.49, answer. NOTE.-In all work of this character, in actual practice, the assuming of the $100, and the finding and adding thereto the premium, or the deducting therefrom the discount, should be per- formed mentally. The foregoing work gives the only exact method of solving problems of this character, but some accountants solve them approximately by first deducting the premium from the amount to be invested, then add to the remainder, the premium on the first premium, and thus continue the alternation of subtracting and adding until the desired degree of accuracy is obtained. To illustrate this method, we will solve the foregoing problem by it. OPERATION. $15000.00 sum to be invested. 150.00 = 1% premium deducted. $14850.00 1.50 = 1 % on $150 premium added. sºmmemºs as sºmeºmºs $14851.50 .015 = 1 96 on $1.50 premium subtracted. $14851.485 Ans. In some cases, in which the sum to be invested and the rate per cent premium or discount is even numbers, this method can be used to advantage, but generally the exact method is preferable. 722 soul.E's PHILOSOPHIC PRACTICAL MATHEMATICS. * OPERATION FOR THE 27TH PROBLEM. Explanation.—Again having no number repre- senting the face of the bill, on which we can $100 face of bill assumed. work, we therefore assume $100 in order to work 1 = 1 % discount. upon and produce the necessary relationship * Inumbers, as explained in the preceding problem, $99 cash cost of $100 bill. to solve the question. On this assumed $100 we compute and deduct therefrom the 1 per cent $ discount, and thus obtain $99 as the cash equi- 100 valent, or as the cash cost of the $100 bill. We 99 || 15000 then observe by the exercise of our reason that * as a $100 bill cost $99, by transposition $99 cash $15151.52 will buy $100 bill, and hence we can buy as many $100 bills of exchange as $15000 are equal to 99. But working more philosophically, we place the $100, assumed face of bill, on our state- ment line, and reason thus: If $99 cash will buy a $100 bill of exchange, $1 will buy the 99th part, and $15000 will buy 15000 times as much, which is $15151.52, answer. OPERATION FOR THE 28TH PROBLEM. $1000 face of bill assumed. $1000 = face of bill assumed. 3.25 = $3.25 per M. premium. 3+ = $3.25 per M. premium. $1003.25 = cost of $1000 bill. $1003} $ Or, 1000 1000 4013 || 4 1003.25 | 15000.00 15000 |$14951.16 Ans. $14951.16 Ans. Problems in investing through an Agent. 29. What amount of bill can be bought for $15000 at par, paying the agent 1 per cent commission? Ans. $14851.49. 30. What amount of bill can be bought for $15000 at 1 per cent premium, paying the agent 1 per cent commission ? Ans. $14704.45. 31. What amount of bill can be bought for $15000 at 1 per cent discount, paying the agent 1 per cent commission? Ans. $15001.50. 32. What amount of bill can be bought for $15000 at $3.25 per M. premium, paying the agent 1 per cent commission? Ans. $14803.38. OPERATION FOR THE 29TH PROBLEM. Explanation.—In this problem the agent is entitled to a commission on the face of the bill; and as there is no premium or discount to con- 100 sider, the operation and reasoning is the same 101 || 15000 ($14851.49) as in the 21st problem of investing, where the & banker was entitled to a premium on the face of the bill. FIRST SOLUTION OF THE 30TH PROBLEM. OPERATION OPERATION To take out the Agent's commission. To take out the Banker's premium and find face of bill. $100 amount invested assumed. $100 face of bill assumed. 1 = 1 % commission. 1 = 1 % premium. $101 cash required to invest $100. $101 cash cost of $100 bill. * * * 100 101 || 15000 ($14851.49 invested). 101 || 14851.49 ( $14704.45) Ans. $14851.49 $14,704.45 1% 1% $148.51 commission. $147.04 premium. Explanation.—As it is the business custom, in exchange transactions, to allow the agent a 4× EXCHANGE. 723 commission on the face of the bill or drafts, plus the premium charged or minus the discount allowed by the banker or seller of the exchange, we ºtherefore first take the agent's commission out of the amount to be invested. To do this we work and reason the same as we did in finding the face of the bill in problem No. 26, under the head of Investing. The result of this statement is $14851.49 to be invested, and $148.51 is the agent's commission. Then, having the amount to be invested, we invest the same, as shown by the second state ment, with the banker at 1 per cent premium. The reasoning for this second statement is the same as that given in problem No. 26, under the head of Investing. NOTE.—By finding 1 per cent on the amount invested, or by finding the difference between the $15000 and the amount invested, we have the agent's commission; and by finding 1 per cent on the face of the bill, or by finding the difference between the $14851.49 and the face of the bill, $14704.45, we have the banker's premium. To add the agent's commission and the banker's premium together, and make but one pro- portional statement thus, 100 102 || 15000 would give an incorrect result, for the reason that the agent is entitled to a commission on the face of the bill plus the banker’s premium. SECOND SOLUTION OF THE 30TH PROBLEM. Instead of making two separate line statements, we can state it thus: $ 100 101 || 15000 101 || 100 10201 | 150000000 ($14704.45) Ans. FIRST SOLUTION OF THE 31ST PROBLEM. OPERATION OPERATION To take out the Agent's commission. To find face of bill. $100 amount invested assumed. $100 face of bill assumed. 1 = 1 % commission. 1 = 1 % discount. $101 cash required to invest $100. $99 cash cost of $100 bill. $ 100 100 101 || 15000 ($14851.49) Ans. - 99 || 14851.49 ( $15001.50) Ans. Explanation.—The operation of this problem is so nearly the same as that of the preceding one, and the reasoning for the statements being precisely the same, we therefore omit repeating Whenn. SECOND SOLUTION OF THE 31ST PROBLEM. OPERATION. Explanation.—To save time and figures we $ prefer the one line statement in all cases where 100 it can be made. But in case the agent takes 101 || 15000 his commission out before going to the bank to 99 || 100 purchase exchange, as he would naturally do, then separate Statements would necessarily have 99.99 150000000 ($15001.50) Ans. to be made. OPERATION FOR THE 32D PROBLEM. i 1000 100 00 1003.25 | 1.4851.49 Or, 101 || 15000.00 101 || 15000 *-*mm- 1003.25 | 1000 *- $14803.38 -**m-. | $14851.49 724 * soul E's PHILOSOPHIC PRACTICAL MATHEMATICs. w Problems in investing through an Agent and a Broker. 33. What amount of bill can be bought for $15000 at par, paying the agent 1 per cent commission and the broker per cent brokerage? Ans. $14778.33. 34. What amount of bill can be bought for $15000 at 1 per cent premium, paying the agent 1 per cent commission and the broker # per cent brokerage? Ans. $14632.01. 35. What amount of bill can be bought for $15000 at 1 per cent discount, paying the agent 1 per cent commission and the broker # per cent brokerage 3 Ans. $14927.61. 36. What amount of bill can be bought for $15000 at $3.25 per M. discount, paying the agent 1 per cent commission and the broker # per cent brokerage? Ans. $14826.52. SOLUTION OF THE 33D PROBLEM. OPERATION. Ea:planation.—In this, as in the above problems in Investing, and for reasons therein given, we $100.00 assumed as amount invested. assume and work upon $100. And as it is the 0 = 1 % commission. business custom to allow both the commission and brokerage on the face of the bill plus the t-mº banker's premium or minus his discount, we $101.50 cash cost of $100 bill. compute as shown in the operation, and add to - the $100 the commission and brokerage, and thus obtain $101.50 as the cash cost, or cash 100 equivalent, of a $100 bill bought at 1 per cent 15000.00 ($14778.33) Ans. Commission and 3 per cent brokerage. .50 = } % brokerage. 101.50 The reasoning for the line statement is the same as given in preceding problems where commission only was charged. FIRST SOLUTION OF THE 34TH PROBLEM. & OPERATION OPERATION To take out commission and brokerage. To take out premium and find face of bill. $100.00 assumed as amount invested. * $100 assumed face of bill. 1.00 = 1 % commission. 1 = 1 % premium. .50 = } % brokerage. *- $101 cash cost of a $100 bill. $101.50 cash required to invest $100. 100 100 101.50 | 15000.00 ($14778.33 Ans. 101 || 14778.33 ($14632.01 Ans. The reasoning for these statements is the same as in the preceding problems and hence is omitted. PROOF BY REVERSING THE OPERATION. - - - - - - - - - - - - $14632.01 Face of bill, * - - Banker's premium at 1 % added, - º º tº - #º - - tº gº º 146.32 Face of bill plus premium, º - - - - - - - - - - $14778.33 Commission at 1 % (on $14778.33); - - * - - tº * -> dº ſº tº 147.78 * * * * * * * * * * * 73.89 Brokerage at # 96 (on $14778.33), Total amount, - - - - - - - - - - - - - $15000.00 EXCHANGE. 725 SECOND SOLUTION OF THE 34TH PROBLEM. $ 100 15000.00 100 $14632.01 Ans. 101.50 101 $ 100 2 15000 100 $14632.01 Ans. 203 101 FIRST SOLUTION OF THE 35TH PROBLEM. OPERATION To take out commission and brokerage. $100.00 assumed as amount invested. 1.00 = 1 % commission. .50 = } % brokerage. $101.50 cash required to invest $100. 100 101.50 | 15000.00 ($14778.33 Ans. OPERATION To find face of bill. $100 assumed face of bill. 1 = 1 % discount. $99 cash cost of $100 bill. 100 99 || 14778.33 ( $14927.61 Ans. SECOND SOLUTION OF THE 35TH PROBLEM. 100 15000.00 100 $14927.61 Ans. 101.50 99 $ 100 2 15000 100 $14927.61 Ans. 203 99 PROOF BY REVERSING THE OPERATION. Face of bill, tº º ſº - Banker's discount at 1 % deducted, Face of bill minus discount, * * tº- Commission at 1 % (on $14778.33), Brokerage at # 96 (on $14778.33), Total amount, - - - - OPERATION FOR THE 36TH PROBLEM. $100 assumed. 1 = 1 % commission. .50 = } % brokerage. $101.50 $ 100 101.50 | 15000.00 $14778.33 $1000 assumed face of bill. $14927.61 149.28 $14778.33 147.78 73.89 $15000.00. , 3.25 = $3.25 per M. discount. $996.75 cost of $1000 bill. $ 1000 996.75 || 14778.33 | $14826.52 Ans. 726 SOULE's PHILOSOPHIC PRACTICAL MATHEMATICS. MISCELLANEOUS PROBLEMS, INVOLVING THE FOREGOING PRIN- CIPLES. 1292. 1. Find the cost of $2850 New York exchange at # per cent premium. Ans. $2860.69. 2. Find the cost of $6540 exchange on Boston at $2.75 per M. premium. Ans. $6557.99. 3. Buy $16250 exchange on San Francisco at $4.50 per M. premium and find the cost. Ans. $163.23.13. 4. Buy $30400 exchange on New York at $1.12% per M. discount and find the cost. Ans. $30365.80. . 5. What cost a bill for $22368.40 bought at 3 per cent discount ? Ans. $22228.60. 6. Find the cost of $5862.70 exchange bought through an agent at $4.50 per M. premium paying the agent # per cent commission. Ans. $5918.53. 7. Find the cost of $7250.80 exchange at # per cent premium paying an agent 1 per cent commission and a broker # per cent brokerage. Ans. $7396.36. 8. Bought $24910.60 exchange at $3.70 per M. discount, and paid 4 per cent commission, and # per cent brokerage. How much did I pay ? Ans. $25004.57. 9. What cost a bill for $38215 bought at par paying an agent # per cent commission and a broker 4 per cent brokerage 3 Ans. $38501.61. 10. Invested $8500 in exchange at $5 per M. premium. What was the face -Of the bill ? Ans. $8457.71. 11. Invested $9000 in exchange at $4.50 per M. discount. What was the face of the bill ? Ans. $9040.68. 12. Exchange is quoted at 24 per cent premium, how much can be bought for $12500 % Ans. $12.195.12. 13. Exchange is quoted at 13 per cent discount, how much can be bought for $46490% Ans. $47257.94. 14. If you pay 1 per cent commission and 4 per cent brokerage and buy exchange at $1.25 per M. discount, how much exchange can you buy for $15000! Ans. $14796.83. 15. Bought the following: Exchange $4700 at # per cent premium; $6250 at # per cent premium; $5600 at # per cent discount; $800 at $1.10 per M. premium; $16420.65 at $1 per M. discount, and through a broker $12286.45 at par; paid the broker $5 per M. on the last exchange. How much exchange did I buy ald what -did it cost 3 Ans. $46057.10, bot. $46.141.75, cost. 16. You owe a New York merchant, for goods bought of him $18400, which you wish to remit in exchange. The rate is $2.50 per M. discount. What is the cost Of the bill ? Ans. $18354.00 NOTE.—The student of book-keeping may make the Cash Book entry for both firms for this transaction. * EXCHANGE. 727 17. You owe a New York merchant, for goods sold for him, in your capacity as a commission merchant, $18400, with which you wish to buy and remit exchange. The rate is $2.50 per M. discount. What is the face of the bill? Ans. $18446.12. NOTE.-The student of book-keeping may make the Cash Book entry for both firms for this transaction. ſº 18. Jones of New Orleans owes Smith of Chicago, a balance of $1018 for Which he buys exchange at $1 per M. premium, and remits to Smith. What did the bill cost 3 Ans. $1019.02. ... NOTE:-The student of book-keeping may make the Cash Book entries in both Jones and Smith’s books, for this transaction. 19. A. of Galveston owes B. of Dallas, for net proceeds of cotton, $8232.38 which he invests in New York Exchange at $1 per M. premium. What is the face of the bill? Ans, $8224.16. NOTE.-The student of book-keeping may make the Cash Book entries in the books of A. and B. for this transaction. 20. A commission merchant sold a consignment of goods for $28900. The charges were $2140, commission 24 per cent. With the net proceeds he purchased ©xchange at 14 per cent premium. What was the face of the bill? Ans. $25652.71. NOTE 1.- The student of book-keeping may make the Cash Book entry for the purchase of the bill by the consignee and for its collection by the consignor. NoTE 2.-See Soulé's New Science and Practice of Accounts, pages 178, 179, 272 and 273, edition of 1895 or 1896, for the proper entries for such transactions. TO FIND THE RATE OF EXCELANGE. 1293. 1. Paid $2436 for a bill of $2400. What was the rate per cent Ans. 13%. OPERATION, $2436 $ 2400 36 tº ºsmºmes sº 2400 100 $36 = premium. tº-mº ºsmº 13% Ans. 2. The cost of exchange was $3484.60; the face of the bill was $3500. What was the rate per M. and the rate per cent discount 3 Ans. $4.40 per M. or, .44% discount. OPERATION. $3500 15.40 15.40 2484.60 3500 || 1000 3500 || 100 * = $15.40 = discount. | $4.40 per M. discount. .44% or Iº96. 728 SOULE's PHILOSOPHIC PRACTICAL MATHEMATICS. * 3. Exchange cost $4050.15, face of bill $4000; paid brokerage for buying # per cent. What was the rate per M. premium and the brokerage? Ans. $5 per M. premium. $30.15 brokerage. OPERATION. $4020 $ 100 4000 20 100.75 | 4050.15 * &=ºmº-ºº-ºº. 4000 || 1000 *==== &ºm==& $20 premium. mºsº $4020.00 = face of bill -- premium. $5 per M. premium. 4050.15 = cost of bill. $30.15 = brokerage. 4. A bill of exchange cost $6564.514, the face was $6500; paid commission 1 per cent and brokerage 3 per cent. What was the rate per cent discount, the commission and the brokerage 3 * Ans. See operation. OPERATION. $100 $ $6500 1} 100 6467.50 1014 || 6564.51+ $32.50 = discount on $6500. $1013 | $6467.50 = face of bill — ; 96 discount. $6467.50 at 1 % = $64.67+ commission. 32.50 $6467.50 at # 9% = $32.33% brokerage. 6500.00 || 100 #96 discount. TO FIND THE FACE OF BILLS WHEN THE COST AND RATE ARE GIVEN. 1294. 1. The cost of a bill was $9024.75; the rate was $2.75 per M. What was the face of the bill ? Ans. $9000. OPERATION. $ 1000 assumed face of bill. Cost of assumed face, 1002.75 |9024.75 cost of bill. $9000 Ans. Face of bin. 2. Paid $3244.14 for a bill of exchange bought at $ per cent premium, bro. kerage 4 per cent. What was the face? Ans. $3200. OPERATION. $ $ 100 assumed face. 100 100 100.50 || 3244.14 807 || 8 100.50 | 3244.14 3228 or, 100 | $3228. = face + premium. msºmºsº sº, 100% $3200 Ans. $3200 Ans. EXCHANGE. 729 3. The cost of a bill was $15377.25. Paid 1 per cent premium, 1 per cent commission and 3 per cent brokerage. What was the face? Ans. $15000, 4. Invested through a broker $15000. Paid brokerage 1 per cent; face of bill was $14803.38. What was the premium per M 7 Ans. $3.25. OPERATION. 100 48.11 101 || 15000 14803.38 1000 | $14851.49 = face of bill -- banker's premium. | $3.25 per M. premium. 14803.38 = face of bill. $48.11 = banker's premium. NOTE.—In working the statement to find the $3.25 per M. premium, the last dividend which gives the final figure 5 is a very little too small to produce even 5. This results mainly from the fact that the divisor $14803.38 is slightly in excess of the true amount, which is $14803.379-H. 5. The face of a bill for a sum invested at 1 per cent discount, paying an agent 1 per cent commission and a broker ; per cent brokerage, is $14927.61. What was the amount invested? An S. $15000. NotE.—See problem 35, page 725 for aid in the solution of this problem. 6. Invested in exchange at 14 per cent discount and received a bill for $4100. What was the sum invested? Ans. $4038.50. OPERATION. $4100 61.50 = 1; 96 discount. $4038.50 = sum invested. TO FIND THE RATE OF EXCHANGE ON TIME BILLS. 1295. 1. What would be the equivalent rate per cent premium or discount on a 30 day bill, money being worth 6 per cent and sight exchange on the same place being 2 per cent discount? Ans. 2%% discount. OPERATION. $100 assumed to work on. $100 $ 2% 100 60 || 33 $2.00 discount due the purchaser. —— .55 interest ** ** 55c. interest. —— . | $2.55 sum of dis. and int. due the purchaser, and which being on the $100, is hence equal to 2% 9% dis. 73o SOULE's PHILOSOPHIC PRACTICAL MATHEMATICS. * \, 2. What would be the equivalent rate per cent premium or discount on 60 day bills, money worth 8 per cent and sight exchange 1 per cent premium ? w Ans. #% discount. OPERATION. $100 assumed to work on. $100 foo 1% premium due the banker. 1 45 || 63 $1.00 premium. | $1.40 int. due the purchaser. 1.00 premium. .40 excess of interest, which being on the $100 and in favor of the purchaser is equal to # 96 discount. 3. What is the equivalent per cent premium or discount on a 15 day bill, money being worth 4 per cent and sight exchange $ per cent premium ? Ans. ##% premium. OPERATION. $100 assumed to work on. * $100 $ #% º 100 90 18 $# = 873 c. prem. in favor of the banker. -msg. *- 200. interest. | 200. interest in favor of the purchaser. 67#c. excess of premium. Eacplanation.—By these statements, we see that there is an excess of the premium of 67% c., and as it is on the $100, and in favor of the banker, it is therefore the per cent premium on 15 day bills. By reduction we find it equal to 3% per cent. 4. Sight exchange is $4 per M. discount, and money is worth 5 per cent. What is the rate of exchange per M. for 90 day bills allowing grace, but not discount day & Ans, $16,913 per M. discount. OPERATION. $1000 assumed to work on. $ 4 = $4 per M. discount. 1000 72 93 $ 4.000 = discount in favor of buyer. 12.91} = interest in favor of buyer. $12.913 interest in favor of buyer. - *~. $16.913 interest and discount in favor of buyer, and which being on the $1000 is, therefore, the rate of discount per M. TIME EXCELANGE. 1296. 1. What is the cost of a $15000 bill drawn at 60 days sight, money being worth 8 per cent and sight exchange being 1 per cent premium, allowing for 3 days of grace? Ans. $14940. FIRST OPERATION. w $15000 face of bill. $ Eacplanation.—As the check or bill has 150 = 1% premium for sight. 15000 63 days to run after presentation, it is 45 || 63 clear that we pay it 63 days before due, $15150 cost of a sight bill. and hence we find and deduct from the 210 int. on $15000 for 63 days at 8%. $210 sight cost the banker's discount on the face of the bill, $15000, for the 63 days $14940 cash cost of bill. at i.per cent, and thus obtain the present Cash cost. & NOTE.-In practice, 10, 30, 60, and 90 day exchange is generally drawn at a certain per cent premium or discount, which renders work of this character of but little use. x- EXCHANGE. 73 I SECOND OPERATION. $100.00 assumed cost. $ 1.00 = 1% premium on sight bill. 100 --- 45 || 63 $101.00 cost of sight bill. -º- 1.40 int. on $100 ass. for 63 ds. at 8%. | $1.40 $99.60 cost of $100, 60 day bill. 15000. $14940.0000 cost of $15000, 60 day bill. THIRD OPERATION. Premium as above on $100 = $1 Interest as above on 100 = 1.40 .40c. discount on the $100 or # 96 discount. $15000 $15000 .40 60 $60.0000 = discount. $14940 = cost of bill. 2. What is the cost of $8000 exchange on Baltimore, drawn at 30 days sight, money being worth 6 per cent, sight exchange being 2 per cent discount? Ans. $7796. OPERATION. $8000 face of bill. $ 160 = 2 % discount. 8000 *-- 60 || 33 $7840 cash cost of a sight bill. -e 44 interest. $44.00 interest. $7796 cash cost of $8000, 30 day bill. 3. A merchant owes a balance of $2511.25, which he wishes to pay with a draft on St. Louis at 30 days sight. Allowing money to be worth 6 per cent, and sight exchange or drafts to sell at 1 per cent premium, for what amount must he draw his draft so that, when sold, it will net the amount that he owes? Ans. $2500. OPERATION. $100.00 face of draft assumed. $ $ 1.00 = 1 % premium. 100 100 60 33 100.45 || 2511.25 $101.00 cash value of $100 sight draft. --- .55 int. on $100 for 33 days at 69%. 55c. interest. $2500 Ans. $100.45 cash value of $100, 30 day draft. To sell this draft the following figures are produced: OPERATION. $2500.00 face of draft. $ 25.00 = 1 % wremium for sight drafts. 2500 ---, 60 || 33 $2525.00 value of $2500 sight draft. * 13.75 interest. $13.75 interest. $2511.25 cash value of 30 day draft. 732 SOULE's PHILOSOPHIC PRACTICAL MATHEMATICs. 2. f EXCELANGE AND CASEI NOTES COMBINED. 1297. 1. I have a balance due S. S. Packard & Co., New York, of $6500. He instructs me to purchase sight exehange and remit to him. Sight checks are 4 per cent discount; I charge 4 per cent commission for investing, and give my note, with collaterals, for such an amount as, when discounted at 8 per cent for 34 days, will net the amount due the banker for the exchange. What is the commission, the face of the note, and the face of the bill? Ans. $32.34 commission. $6516.90 face of note. $6500.16 face of bill. S O L U TI O N . OPERATION To take out # 96 commission. $ | 100 201 || 2 6500 $6467.66 OPERATION To invest $6467.66 at # 96 disc’t. 199 | 2 | 6467.66 | $6500.16 OPERATION To find the face of the note. $ assumed note. 4500 4466 | 6467.66 $6516.90 Ans. 2. Bought of the Merchants' Bank the following exchange: A check on New York for $2500, at # per cent discount; a check on St. Louis for $2000, at # per cent premium; and a gold check on Galveston for $5000, at # per cent discount, gold being 10 per cent premium. For the cost of these three checks I gave $4000 currency and my cash note at 30 days, at 8 per cent, secured by collaterals. What is the face of the note, allowing grace and discount day? Ans. $6016.71. OPERATION INDICATED. $2500 at # 96 discount = $18.75 = $2481.25 cost. 2000 at # 9% premium = 17.50 = 2017.50 cost. 4500 4466 || 5971.25 |sºn Ans. Gold } º at # 9% discount = 27.50 = 5472.50 cost. $99.71.25 = total cost. 4000.00 = cash paid. $5971.25 DISCOUNT ON TIME DRAFTS. 1298. When time drafts or bills of exchange are negotiated before maturity, they are subject to discount on the same conditions as promissory notes. In trans- actions of this character, the premium or discount, and the interest are computed on the face of the draft or bill. y IEXAMPLES. 1. $8000. NEW ORLEANs, July 10, 1895. Sixty days after sight, for value received, pay to the order of Geo. Fernandez, Eight Thousand Dollars, and charge the same to our account. To “BAAB & MASPERO, SCRIBNER & WAGNER. ST. LOUIs, Mo. EXCHANGE. 733 What are the net proceeds of this draft if discounted the same day that it Was drawn at 8 per cent, the rate of exchange being 1 per cent premium ? Ans. $7966.22. OPERATION. $8000.00 face of draft. 80.00 = 1 % premium. $8080.00 value of draft. 113.78 interest. $7966.22 net proceeds. Ans. $ 80.00 45 || 64 | $113.78 interest. 2. What is the value of a draft for $10000, payable 15 days after sight, exchange # per cent discount, interest 8 per cent? Ans. $9882.78. º OPERATION. $10000.00 face of draft. 75.00 = # 96 discount. - *- $9925.00 value of draft. 42.22 interest. $9882.78 Ans. $ 100.00 45 19 $42.22 interest. To buy, sell, and invest in eacchange and pay commission and brokerage on the face of the bill only. 1299. 1. What will a bill for $15000 cost, bought at 1 per cent premium, paying 1 per cent commission and # per cent brokerage on the face of the bill? OPERATION. Face of bill, ** - - - - $15000 1 % premium, - - - $150 196 commission, - - - 150 # 96 brokerage, wº- me - 75 37 ºm- 5 $15375 Cost of bill, - - * - Ans. $15375. Explanation.—As the commission and bro- kerage are here, as well as the premium, charged on the face of the bill, they could have been added, making 24 per cent, and the correct result obtained by adding to the $15000 the amount of 24 per cent of itself. 2. What will be realized for a $15000 bill sold at 1 per cent premium, paying 1 per cent commission and 3 per cent brokerage on the face of the bill? Ans. $14925. OPERATION. Face of bill, - - * - 1 % premium, - tº Value of bill, º gº 1 % commission, tº º # 96 brokerage, e - - - - - $15000 - - - - 150 - - - - - $15150 - - $150 - sº 75 -mº 225 $14925 734 soule's PHILOSOPHIC PRACTICAL MATHEMATICS. º 3. What amount of bill can be bought for $15000 at 1 per cent premium, paying 1 per cent commission and 3 per cent brokerage on the face of the "bill? Ans. $14634.15. OPERATION. $100.00 assumed face of bill. $ Explanation.—The reasoning 1.00 = 1 % premium. 100 e º 1.00 = 1 $ P. mission. 102.50 | 15000.00 for the line statement is the .50 = } % brokerage. --- same as given in the 26th prob- --- 14634.15 Ans. º e $102.50 cash cost of $100 bill. $ lem, and hence is omitted. NOTE.-By the foregoing work, we have illustrated 39 different transactions in purchasing, selling, and investing in domestic exchange according to the custom governing exchange transac- tions, and also several different problems where, by special agreement, the commission and brokerage are charged on the face of the bill. This classification is entirely new, much more extended than ever before presented, and fully illustrates work that has always confused students and the great majority of business men. MISCELLANTEOUS PROBLEMS IN DOMESTIC EXCEIANGE. 1299. 1. What is the amount of the banker's premium on $25000. New Orleans exchange at 3 per cent premium; what is the agent's commission at # per cent, and what is the cost of the bill? Ans. $156.25, banker's premium. $125.78, agent's commission. $25282.03, cost of bill. 2. A merchant invested through his correspondent $7840.50 in exchange on Philadelphia at # per cent discount. The correspondent charged for investing 1 per cent commission and paid a broker + per cent for transacting the business. What was the face of the bill, what the banker's discount, what the correspondent's commission, and what the broker's brokerage 7 Ans. $7802.22, face of bill. $58.52, discount. $77.44, commission. $19.36, brokerage. 3. Bought exchange at 1 per cent discount and paid $2385.90; what was the face of the bill ? Ans. $2410. OPERATION INDICATED. $100–$1 = 99. ($2385.90 × 100) – 99 = $2410. 4. A merchant in New Orleans owes a merchant in St. Louis $15000. Exchange on St. Louis is 1 per cent premium, and exchange in St. Louis on New Orleans is 1 per cent discount; making no allowance for the use of money or telegraph expenses, which is it better for the New Orleans merchant to do, buy in New Orleans at 1 per cent premium and remit, or to instruct the St. Louis merchant to draw on him for such a sum as when sold at 1 per cent discount will net the $15000 % Ans. $1.52 better that the New Orleans merchant buy and remit. NotE.—See solution for the following problem. EXCHANGE. 735 5. In the above problem, estimating that money is worth to the New Orleans merchant 8 per cent, and that he instructs the St. Louis merchant by telegram costing $2.40, to draw on him at sight for such an amount as when sold at 1 per cent discount will net the $15000, and that it will require 4 days for the St. Louis bill to reach the New Orleans merchant, what will be the gain to each merchant by this mode of settling, over that of buying a bill in New Orleans at 1 per cent premium, money being worth 6 per cent per annum in St. Louis, and 4 days time required to transmit the exchange if bought in New Orleans? Ans. $9.55 gain to the New Orleans merchant. $10 gain to the St. Louis merchant. SOLUTION. OPERATION OPERATION Showing the cost of $15000 exchange bought in Showing face of bill drawn in St. Louis and New Orleans at 1 per cent premium. sold at 1 per cent discount to net $15000. $15000 face of bill. $ 150 = 1 % premium. 100 99 || 15000 $15150 cost. ----- $15151.52 face of bill. Face of St. Louis bill, * - * º º - -> - º cº use º º $15151.52 Cost of New Orleans bill, - - - º - - º - -> º sº sº * 15150.00 Loss to the New Orleans merchant by paying the St. Louis bill, - - - - - $1.52 To which we add telegraph charges, se ºn sº me m sº s º º º 2.40 And produce a loss of - sº me as as amº m * ºn e º 'º º $3.92 Thus far we see that the New Orleans merchant has lost $3.92; but by reason of the St. Louis merchant drawing on him, he has had the use of $15150, the cost of a $15000 bill in New Orleans, for 4 days at 8 per cent; and the St. Louis merchant has had the use of $15000, the proceeds of the St. Louis bill, for 4 days at 6 per cent. OPERATION O PERATION To find the worth of $15150 for 4 days at 8 %. To find the worth of $15000 for 4 days at 69%. $ $ 15150 15000 45 || 4 60 || 4. $13.47 worth or int. - $10 worth or int. By these iast two operations we see that the St. Louis merchant gains by the use of $15000, for 4 days at 6 per cent, $10; and that the New Orleans merchant gained by the use of $15150, for 4 days at 8 per cent, - sº tº es tº as tº m - - - - - - $13.47 From which we deduct the losses, as shown in the first part of the operation, - - * * 3,92 And thus obtain a net gain of - - - - • * * * * * * * $9,55 Arbitration of Exchange. ===N —w 1300. Arbitration of Exchange is the process of finding the proportional cost of exchange between two cities or countries, by means of one or more interme- diate exchanges. The object of this system of exchange is to ascertain, by comparison of the costs or rates of exchange, between several places, the most advantageous channel through which to remit bills. The subject is divided into simple and compound arbitration. Simple Arbitration is the process of finding the cost or rate of exchange between two cities through a third. Compound Arbitration is the process of finding the cost or rate of exchange between two cities through several others. Exchange of different articles of merchandise, and different weights and measures of different nations, may be arbitrated in the same manner as bills of exchange and currencies. PROTELEMIS. TO FIND THE COST OF A BILL THAT WILL PAY A. CERTAIN SUM IN ANOTEHER PLACE. 1301. 1. A merchant of New Orleans owes a balance of $5000 in New York, which he wishes to pay. Direct exchange on New York is 1 per cent premium; exchange at the time on St. Louis is 4 per cent discount, and St. Louis exchange on New York is # per cent premium. Allowing # per cent brokerage for reinvesting on the net investment or banker's cost of the, bill in St. Louis, which is the better way for the New Orleans merchant to remit, and how much will he gain by remitting the better way? Ans. Through St. Louis is the better way. $37.63, gain. OPERATION For direct Exchange on New York. $5000 face of bill. 50 = 1 % premium. $5050 cost of bill. (736) +): ARBITRATION OF EXCHANGE. 737 FIRST OPERATION For finding the cost of Exchange by remitting through St. Louis. $ wº Bill on St. Louis, 100 | 99.50 cost of bill on St. Louis at # % discount. Bill on New York, 100 | 100.50 cost of bill on New York at # 96 premium. Face of bill on N. Y. 100 | 100.25 cost of bill on New York at # 96 brokerage. 5000 dollars invested. $5012.37 Ans. SECOND OPERATION. Statement of costs. $5000.00 face of bill. 25.00 = } % premium in St. Louis. Cost by direct exchange, - tº- $5050.00 Eºmºsºm-º-mºm. Cost through St. Louis, - - 5012.37 $5025.00 St. Louis cost. *===- 12.56 = + 96 brokerage in St. Louis. $37.63 gain. $5037.56 cost including brokerage. 25.19 = } % discount in New Orleans on St. Louis bill. $5012.37 cost of $5037.56 in New Orleans. 2. Suppose in the above problem, that the # per cent brokerage had been charged on the face of the bill, instead of the net investment or banker's cost, What would have been the cost of the exchange through St. Louis? Ans. $5012.31. FIRST OPERATION. $ Bill on St. Louis, 100 99.50 cost of bill on St. Louis at # 96 discount. Bill on N. Y. 100 || 100.75 cost of bill on N. Y. at # 96 premium and # 96 brokerage. 5000 dollars invested. $5012.31 Ans. SECOND OPERATION. $5000.00 face of bill. 25.00 = } % premium. 12.50 = + 96 brokerage. $5037.50 cost of bill in St. Louis including premium and brokerage. 25.19 = } % discount, N. O. exchange on St. Louis. $5012.31 net cost in New Orleans. 3. Suppose again, that in the above problem brokerage had been charged on the whole amount for investing, by the St. Louis agent, what would have been the cost of the exchange through St. Louis? Ans. $5012.40. FIRST OPERATION. Bill on St. Louis, - sº - º -º 100 99.50 cost of bill in St. Louis at # 9% dis. Bill on New York, - - º 100 || 100.50 cost of bill on N. Y. at # 96 premium. Proceeds of am’t invested, brok. deducted, 99.75 | 100 amount invested in N. Y. exchange. 5000 dollars invested. | $5012.40 Ans. SECOND OPERATION. $5000 face of bill. 25 = } 96 premium on New York. *==-- $5025 St. Louis cost. 99.75 || 100.00 $5037.59 total cost including brokerage on same for investing. 9 = } % discount on St. Louis exchange in New Orleans. $5012.40 net cost in New Orleans. 738 SOULE's PHILOSOPHIC PRACTICAL MATHEMATICS. A. TO FIND THE PROCEEDS OR FACE OF A BILL FOR A CERTAIN SUM INVESTED. 1302. 1. A merchant of New Orleans wishes to invest and remit $5000 to a New York correspondent. Exchange on New York is 1 per cent premium; exchange on St. Louis, at the same time, is 4 per cent discount, and St. Louis exchange on New York is # per cent premium, allowing # per cent brokerage on the met amount invested by the agent, which is the better way to remit and how much will he gain by remitting the better way? Ans. Through St. Louis is the better way. $37.16 gain. SOLUTIONS. OPERATION, For direct exchange on New York: $ 100 face of bill on New York. Cost of bill 101 || 5000 - me summing $4950.50 face of bill. Ans. OPERATION. For remitting through St. Louis: $ Cost of bill - - 99.50 | 100.00 face of bill on St. Louis. Cost of inv. - 100.25 | 100.00 amount invested by agent in N. Y. exchange. Cost of bill on N. Y. 100.50 | 100.00 face of bill on New York. 5000 dollars invested. | $4987.66 proceeds or face of bill. 4950.50 proceeds by direct exchange. --sº $37.16 gain as above. 2. Suppose that in the same problem the agent had charged # per cent brokerage on the whole amount invested, what would have been the proceeds on the face of the bill? Ans. $4987.62. OPERATION, $ Cost of bill on St. Louis, 99.50 | 100 face of bill on St. Louis. Cost of investment, ,100. 99.75 amount invested by agent in N. Y. exchange. Cost of bill on New York, 100.50 | 100.00 face of bill on New York. 5000 dollars invested. | $4987.62 proceeds or face of bill. 3. Suppose again, in the above problem that the agent had charged + per cent brokerage on the face of the bill, what would have been the proceeds or face Of bill? Ans. $4987.72. OPERATION. $ Cost of bill on St. Louis, 99.50 100.00 face of bill on St. Louis. Cost of N. Y. exchange, 100.75 | 100.00 amt. invested in N. Y. ex. Invested at # 96 prem, and # 96 brok. | 5000 dollars invested. | $4987.72 proceeds or face of bill. * ARBITRATION OF EXCHANGE. 739 In the foregoing problems, we have elucidated the three different methods of agents of charging commission or brokerage for reinvesting, when the cost of a bill for a certain sum is required, and also when the proceeds or face of a bill for a certain sum invested is required. There is no statute law regulating the method of making this charge; and as the business custom relating thereto is not uniform, it is, therefore, a matter of Special agreement. In the arbitration of exchange, brokerage or commission is charged at each intermediate place through which it is remitted. In the arbitration of foreign exchange, the commission or brokerage for reinvesting is sometimes included in the rate of exchange; but when not thus included, a separate charge is made. FORMULA OF STATEMENTS. 1303. To elucidate the operation of charging commission or brokerage on the net investment or face of bill, or whole amount for investment, when buying or investing, we present the following formula of statements: When buying a certain amount of exchange. When investing a certain sum in exchange. To charge commission or brokerage on the To charge commission or brokerage on the met cost of bill, the statement is made thus: net cost of bill, the statement is made thus: 100 || 100.50 100.50 | 100. To charge commission or brokerage on the To charge commission or brokerage on the jace of bill, the statement is made thus: 100 100.50 + banker's premium or — banker's dis. To charge commission or brokerage on the whole amount, the statement is made thus: 99.50 | 100 face of bill, the statement is made thus: 100.50 –– 100. the banker’s premium or — banker’s dis. To charge commission or brokerage on the whole amount, the statement is made thus: | 100 99.50. Explanation.—In these statements, we have assumed 3 per cent as the commission or brokerage eharged. By Net Investment is meant the sum invested after the agent's brokerage or commission has been deducted. By Whole Amount of Investment is meant the sum invested, or to be invested, before deducting the agent's brokerage or commission. By Face of Bill is meant the amount of the bill that is obtained from the banker, after paying the agent's commission or brokerage, and the banker's premium, or allowing for the banker's discount. 74O SOULE's PHILOSOPHIC PRACTICAL MATHEMATICs. & To find the equivalent rate of direct exchange to correspond with the arbi- trated rate, we would omit from the statements the last amount on the increasing side; or, we would divide the final result of the statement, or answer, by the last amount; or, if the last amount is of a different unit, as it often is in foreign exchange, we would divide by its equivalent, reduced to the same unit. To elucidate this, we present the third example above; thus: 99.50 | 100.00 100.75 || 100.00 .9975-1-96 dis. Ans. ENGLISH EXCHANGE. 1304. In the introduction of exchange we defined it to be, in a commercial sense, a system of paying debts in distant places by means of bills of exchange, without the transmission or shipment of money. By the term English Exchange, we mean the system of paying debts in England and her dependent Provinces without the transmission of money. ** 1305. HISTORY OF THE ENGLISH MONETARY POUND. The English monetary pound is of French origin. In the year A. D. 768, Charlemagne, or Charles the great, ascended the throne of France, and during his reign he wielded the scepter with such consummate skill and wisdom as to win the plaudits of the world and weave around his name a brighter luster than was radiated from the jewels in his crown. In the year 800 he was crowned by the Pope of Rome as “Charles Augustus, Emperor of the West,” and during the reign of this great king and emperor, which continued till 816, he accomplished what had been undertaken by his imperial predecessor, Augustus the First, the unification of the money of his empire. He ordained that the French livre or peºind weight of silver should constitute the money unit of value of the French kingdom; and this monetary pound of Charlemagne, reduced in weight, lived in daily use through the vicissitudes of ten centuries in France, until 1792 when the French Nation adopted the “metric system” of weights and measures, and the Franc, decimally divided, as the monetary unit. It then disappeared from the money drawers of France. In A. D. 1066, this Livre or pound was carried across the channel into England by William of Normandy, and as conqueror he imposed it on the English people. The “Tower Pound” of A. D. 1066, the first coined in England, actually contained one pound weight of silver. But the word has long since ceased to possess any truthful significance, for the English monetary pound has been for five centuries, since the year 1200, dwindling in weight, until now the 20 shillings into which it is divided weigh but 3%; ounces troy. In 1620, when America was colonized, the people brought with them, or soon adopted, the English pound as their unit of money. Some of the colonies still further reduced the weight of º: ENGLISH EXCHANGE. 74. I the English twenty shillings, and some coined smaller denominations, with sub-divisions in copper. This they continued until 1660, when Charles the second reascended the throne of England, and issued his royal order prohibiting any further coinage by the colonies. The pound then remained the monetary unit until 1783 when England recognized the political independence of the United States, and further, until the adoption of the present unit of money, the dollar. On the 6th of July, 1785, the Continental Congress passed the memorable monetary ordinance reported by the “grand Committee of thirteen,” by which they rejected the denominations of pounds, shillings, pence and farthings, and established the dollar as the permanent monetary unit of the United States, with the proviso, that it should be decimally divided. The precise weight, however, was not fixed until August 8th, 1786. The weight and value of the American silver dollar was then made to conform in value, or very nearly so, to the value of the old Spanish Carolus pillar dollar which, by the usage of bankers for ages had been the standard of value for the monetary pound during its many changes in weight, As a matter of history, we would here remark that this Spanish dollar was not original with Spain, she having borrowed it from Austria, during her union with Austria under the empire of Charles the Fifth. The true ancestor of the American dollar is the “Joachim’s thaler," first coined in the mines of the Bohemian valley of Saint Joachim (or James). From 1786 to 1834, the commercial value of the English monetary pound, compared with the United States dollar, was $4.44%, or £9 sterling was equal to $40, and vice versa. With this equivalent of value between the monetary units of the two nations, we have but to multiply pounds by 40 and divide the product by 9, to find the equivalent of pounds in dollars, and vice wersa to find the equivalent of dollars in pounds. Thus, to reduce £270 to dollars. And to reduce $1200 to pounds. 40 9 9 ) 10800 40 ) 10800 $1200 38.270 But, by Act of Congress, taking effect August 1st, 1834, the weight of the American coin was reduced. The ten dollar gold eagle, which then weighed 270 grains troy, +} pure, was reduced in weight 12 grains, and all other denominations of gold coin were reduced in like proportion. Con- gress, also, in 1837, further reduced the coin by increasing the alloy from 's to hi. By reason of this decrease of weight and increase of alloy, the relative value of pure gold in the United States was enhanced, and the intrinsic value of the pound sterling, the English gold sovereign, was thus enhanced or increased in like proportion. This increased value of the sovereign or pound sterling is ascertained by calculating the actual number of grains of pure gold contained in the standard coin of Great Britain, and estimating the same according to the United States valuation of gold. The following work gives this increased value of the pound sterling, elucidates the manner of obtaining it, and furnishes the data for the basis of the bankers' rate of English exchange, from 1834 to 1874, which was nearly 94 per cent, more or less, (according as premium was charged or discount allowed in selling the exchange) premium on the old $4.44% value of £1: 742 SOULE's PHILOSOPHIC PRACTICAL MATHEMATICS. ºr From the original weight of the U. S. eagle ($10 gold piece) which was - £ a 270.0 grs. We first deduct - - - - - - - - - * * * * * 12.0 grs. And have in the remainder * * * * * * * * Eº as ºs 258.0 grs. its present weight, ºr of which is pure gold. From this we deduct To alloy, tº- tº º * sº º dº tºº sº * * tº- º 25.8 grs. And have in the remainder *º- tº sº tº gº gº * ºn ge iº º 232.2 grs. of pure gold. By Act of Parliament, one pound weight of standard (+} pure) gold is coined into £46 14s. 6d., which gives the sovereign, the monetary pound, a weight of - 123.2744 grs. From which we deduct ºf alloy - - - - - - tº sº me tº as 10.2728 grs. And have in the remainder, of pure gold, gº º ºs ems tº tº me tº 113.0016 grs. Having now the weight of pure gold in the monetary unit of England and of One of the denominations of the American monetary unit, to ascertain the value of the English pound in the American unit of value, we have but to measure or value its contents in pure gold by the United States valuation of gold. OPERATION Explanation.—According to the foregoing work, we To find the value of the English see that $10 American gold coin contains 232.2 grains monetary pound <> of pure gold, and hence, by transposition, 232.2 grains y p * . are worth $10; with these facts and premises we place $ the $10 on the statement line, and reason as follows: 10 ' Since 232.2 grains of pure gold are legally worth $10, 232.2 | 113.0016 1 grain is worth the 232.2d part, and 113.0016 grains -- “” (the weight of the sovereign in pure gold) are worth 113.0016 times as much, which worked, gives $4.8665. | $4.8665 By deducting from this $4.8665 the old par value of the pound, $4,4444, we have a gain of 42.21 cents, which gives nearly 94 per cent gain on the old par value, or the value before the changes in the American coin in 1834 and 1837. Thus, (.4221 × 100) -- 4.4444 = 9}}}}#. This value, $4.8665, is, by Act of Congress of March 3d, 1873, which went into effect January 1st, 1874, the present value of the pound sterling and the basis of English exchange. By the same act, bankers and exchange dealers were required to base their exchange transactions with England upon this value, and allowing for premium, or discount, quote the rate of exchange in the U. S. dollar and cent equivalent of the English monetary pound. In pursuance of this law, bankers and exchange dealers, have adopted $4.86; as the par of exchange and quote the rate $4.86% more or less according as a premium or discount is declared. º # $ g ãº- t * CŞ COUNTRY. STANDARD, Monetary Unit. HääE; COINS. • s * -T Argentine Republic......|Gold and silver.......|Peso ... . . . . . . . . . . . . . . . . . $0.96,5 Gº argentine ($4.82,4) and # argentine. Silver; peso and SO 1VISIODS. 3 > * Gold: former system-4 florins ($1.92,9), 8 florins ($3.85,8), Š- Austria-Hungary. . . . . . . . Gold. . . . . . . . . . . . . . . . . Crown . . . . . . . . . . . . . . . .20,3 ducat ($2.28,7) and 4 ducats ($9.15,8). Silver: 1 and 2 florins. § : g Gold: present system-200rowns ($4.05,2); 10 crowns ($2.02,6). Š- P Belgium. . . . . . . . . . . . . . . . . . Gold and silver... . . . . Franc . . . . . . . . . . . . . . .19,3 |Gold: 10 and 20 francs. Silver: 5 francs. Bolivia. . . . . . . . . . . . . . . . . . . Silver . . . . . . . . . . ... ...|Boliviano . . . . . . tº e º s .45,5 |Silver: boliviano and divisions. § * Brazil. . . . . . . . . . . . . . . . . Gold . . . . . . . . . . . . . . . . Milreis. . . . . . . . . . . . . . . . . .54,6 |Gold: 5, 10, and 20 milreis, Silver: }, 1, and 2 milreis. § es Ho British Possessions N. Gold ...... * e º e g º ſº gº © tº º Dollar . . . . . . . . . . . . . . 2 | 1.00 3. .S. A. * Newfound- § - | land). $ ź Central Amer. States— Š S (- Costa Rica.......... Q > Guatemala... . . ..... RS SD Honduras.......... } |Silver . . . . . ... . . . . . . . . [Peso . . . . . . tº e º ſº e s = e º e .45,5 |Silver: peso and divisions. "S * > * }old: 1, 2, 5, and 20 yen. S $ P Japan. ... . . . . . . . . . . s & ſº gº Gold and Silver*...... Yen. . . . . . §::::::: .49,1 Silver: yen. y y s S ºf Liberia. . . . .........'Gold... . . . . . . . . . dº e g º e e Dollar............ ..... 1.00 • S S ºr: Mexico...................Silver. . . . . . . . . . . ..... [Dollar . . . . . . . . . . . . . tº e º º .49,5 |Gold: dollar ($0.98,3), 2%, 5, 10, and 20 dollars. Silver: dollar 5 S E-i * (or peso) and divisions. 3 Netherlands..............'Gold and silver. . . . . . . Florin ... ...... s & .40,2 |Gold: 10 florins. Silver: #, 1, and 23 florins. S P. Newfoundland...... .....'Gold..... © e º 'º e º e & ... ... [Dollar. . . . . . . . tº º e º s 1.01,4 |Gold: 2 dollars ($2.02,7). 'S P4 ||Norway.................. Gold... . . . . ...........ICrown . . . . . . . . . . . . . . tº e .26,8 |Gold: 10 and 20 crowns. S p Peru . . . . . . . . . . . . . . . . . . . . Silver. . . . . . . . . . . . . . $9. , . . . . . . . . . . . . . . . . .45,5 |Silver: sol and divisions. SY ºi Portugal . . . . . . . . . . tº e º e o 'º ..]Gold...... ... . . . . . . . . . [Milreis.... . Gold ‘’’’ 1%, §: 1, 2, 5, º: # l f($ es e º + O .77,2 |Gold: imperial ($7.71.8), and # imperial f($3.86). 's : Russia. . . . . . . . . . . . . . . . . . . . Silver f........ ....... [Ruble.. } Silver . . . . . .36,4 |Silver: }, +, and 1 ruble. p ( S p. Spain e & tº tº e º º tº G tº Gold and silver....... Peseta, g is is tº a • * * * 19,3 |Gold: 25 pesetas. Silver: 5 pesetas. g s fº Sweden,...;. • * * * * * * * e a e s a Gold...... .... • . . . . ... ... Crown . . . . . . . . . . . . .26,8 |Gold: 10 and 20 crowns, se º |Switzerland......... . . . . [Gold and silver. . . . . . . [Franc.. . . . . . . . * * .19,3 |Gold: 5, 10, 20, 50, and 100 francs. Silver: 5 francs. Š PH Tripoli . . . . . . ..... . . . . . . . . [Silver. . . . . . . . ........|Mahbub of 20 piasters . .41,1 *S. Turkey ..................|Gold.......... . . . . . . . . .[iºster . . . . . . . . . . . . . . .04.4 |Gold: 25, 50, 100,250, and 500 piasters. * CO |vºeia ... ............'Gold and silver. . . . . . . [Bolivar . . . . . . . . . . . . . .19,3 |Gold: 5, 10, 20, 50, and 100 bolivars. Silver ; 5 bolivars. : ãºi t i # i | * * 744 SOULE's PHILOSOPHIC PRACTICAL MATHEMATICS. * THE RATE OF EXCELANGE. NotE.—For the Face and Character of Foreign Bills, see Article 1287, page 715. 1307. The basis of English Exchange, as above stated, is $4.86% more or less, according as premium or discount is declared. By custom of exchange dealers, this rate is generally increased or decreased by a varying scale of 3 cent, or 3 point, which is very nearly, and for practical convenience, considered and quoted tº per cent. The exact per cent figures for the #2 increase or decrease on the intrinsic value of the pound sterling are #### per cent or decimally expressed.102743-- per cent. In some cases, the rate varies by # and 3 cent in the quotations. TO FIND THE RATE OF EXCELANGE WHEN A PREMIUM IS CELARGED. 1308. 1. Considering the balance of trade, the use of money, etc., the seller of English Exchange wishes to charge a premium of 1 per cent. What would be the rate of bills 3 Ans. $4.914. OPERATION OPERATION Based on the intrinsic value of the pound. $4.8665 = value of pound sterling, .0486–H = 1 % premium charged. $4.9151 = (practically $4.914). Based on the exchange value of the pound. $4.865 = banker's basis of exchange. .05 = prem. charged, allowing tº 96 to equal -- #c. increase. $4.915 = (practically $4.91%). 2. What is the rate of exchange when per cent discount is allowed ? FIRST OPERATION. By subtracting # per cent to the dollar and cent value of £1. $4.8665 = value of £1. .0243–H = } % discount. $4.8422 = (practically $4.84) Ans. Ans. $4.84. SECOND OPERATION. By allowing To per cent to equal #c. decrease on the banker's basis of exchange. $4,865 = banker's basis of exchange. .025 = discount. $4,840 – Ans. Ea:planation.—In the second operation since, I'd per cent equals 3 c. 4 per cent, which is ºr per cent, equals Éc. which is 2}c. or 2.5 cents discount. TO FIND THE PREMIUM CHARGED OR THE DISCOUNT ALLOWED WHEN THE RATE OF EXCEIANGE IS GIVEN. 1309. FIRST OPERATION. $4.915 $4.8665 .0485 .0485 4.8665 100 . . ##### 9% practically 1 % Ans. rate of exchange. value of pound sterling. = the premium. 1. The rate of exchange is $4.91%, what was the premium charged 3 Ans. 1%. SECOND OPERATION. º $4.915 $4.865 .050 .050 4.865 | 100 #}}} % practically 1%. Ans. *. ENGLISH EXCHANGE. 745 2. The rate of exchange is $4.84, what was the discount allowed? Ans. #%. FIRST OPERATION. SECOND OPERATION. $4.84 = rate of exchange. $4.84 = rate of exchange. 4.8665 = value Of £1. 4.865 = basis or par of exchange. .0265 = discount. .025 = discount. .0265 .025 4.8665 | 100 4.865 || 100 ###33 (practically # 96). | #}} (practically # 9%). THE RATE AND THE PAR OF EXCELANGE GIVEN TO FIND THE PER CENT PREMIUM OR DISCOUNT DECLARED. 1310. 1. The rate of exchange is 4,873, the par of exchange is 4.86%. What is the per cent premium charged? Ans. .2569% premium. OPERATION. C. C. 4.87; 4 || 5 4 || 5 4.86% 4.865 || 1000 or, 973 || 2 -*- -* 100 1}c. | .2569 % | —— 2. The rate of exchange is 4.80; the par is 4.864. What is the per cent discount allowed ? Ans. 1.33607% discount. PROBLEMS IN ENGLISEI EXCELANGE TO REDUCE THE ENGLISH MONETARY UNIT TO THAT OF THE UNITED STATES. 1311. 1. What is the cost of a bill of exchange on London for £5000; exchange 4.87; # Ans. $24375. OPERATION, *º, * Ea:planation.—Since the rate of exchange is the exchange value of £1, it is clear that by multiplying 2435000 2500 324375,00 Ans. the rate by the number of pounds we will obtain the cost. 2. What will flá86 11s. 8d. cost in currency, exchange at 4.85% º Ans. $7217,36. 746 SOULE's PHILOSOPHIC PRACTICAL MATHEMATICS. SOLUTION. OPERATION To reduce the 11s. 8d. to the decimal of a pound. 11 8 5 4; 5- 0.5 = 2.5881. OPERATION To reduce pounds to dollars. 361486.5833 4.85% rate of exchange. 7209929005 7432916 $7217.361921 Ans. NOTE.-See Article 869, page 450, for an explanation for reducing shillings and pence to the decimal of a pound. 3. What will £21468.3s. London exchange cost at 5.32? OPERATION To reduce the 3s. to the decimal of a pound. 3s. × .05 = .15 pounds. or, 3s. – 20 = .15 pounds. Ans. $114210.56. OPERATION To reduce the pounds to dollars. 4.21468.15" 5.32 $114210.5580 Ans. NOTE:-In this and many of the problems in exchange operations, we use the multiplicand as the multiplier. This we do in conformity with custom and for practical convenience. 4. A merchant imports an invoice of goods from England, amounting to £1723. 14s. 11d. What is it worth in United States money at the legal value of the sovereign 3 FIRST SOLUTION. OPERATION To reduce 14s. 11d. to the decimal of a pound. 14 11 5 4; .70 .0453 .70 # .745% SECOND OPERATION. 3.1723.746 $4 Ans. $8388.61. OPERATION To reduce pounds to dollars. 4. 1723.746 $4.8665 legal value of £1. s388.809090 Ans. Explanation.—This opera- tion is based upon the pre- sumption that £1. = $4.86; # of $6894.984 = amount considering 1 pound worth $4. $4.00 instead of $4.8665; hence the *g of 1378.996 = } of $4 valuation of 1 pound, .80 slight excess, of 28-1-c. in the 114.916 = g of + of $4 valuation of 1 pound, 6# answer. It is a shorter (Jeth- od than the first and is used $8388.896 = amount considering £1 to be = to $4.86% by many accountants. As shown in the operation we first assume the pound sterling to be worth $4, then add to that value # of the $4 valuation, and also Yºg of the # of the $4 value. 5. A merchant imported an invoice of English goods, amounting to £12400 2s. 8d. The importation duty is 40 per cent on the invoice price. What is the cost 2. ENGLISH EXCHANGE, 747 of the invoice in United States money, including the duty but no other importing charges? Ans. $84483.35. OPERATION INDICATED. :E12400.1333 4.8665 $60345.25 cost of invoice. 24138.10 = 40 % duty on cost. *m. $84483.35 importing cost. 6. By telegram, we learn that medium cotton is selling in Liverpool at 7%d. Sterling exchange is 5.29. The freight on cotton to Liverpool in sailing bottoms is *d. per pound. From these figures, making no allowance for the insurance, interest on money, and the charges to effect sales, what is the equivalent value of cotton of like classification in U. S. money? Ans. Practically 1642. OPERATION 74d. Liverpool price. 1%d. freight charges deducted. 7#d. = 7.3125d., which -- 240 = .030469 pounds. .030469 × 5.29 the rate of exchange gives $.16118101, practically 164c. Ans. ** d. d. Ea:planation.—As many producers 7.3125 or thus: 16 117 of cotton unacquainted with the 240 || 5.29 240 5.29 customs and workings of commerce, * -º *m- -º look upon and regard these figures $.1611797 Ans. $.1611797 Ans. as the price of cotton in our own market, we will remark that tº closely approximate even, from data of this character, the price of cotton in our market, we must allow for the insurance to ship cotton to Liverpool, the various charges to effect sales in Liverpool, the interest on money, and in addition to this we must take into consideration the probability of the increase or decrease in the Liverpool price of cotton, the rate of exchange, and lastly the margin for profit to the buyer must be considered. 7. Good middling cotton is quoted in Liverpool at 54d. What is the equiva lent price in United States money, the rate of exchange being 4.90 and no allow ances made? Ans. $.10718--, practically 10+$2. A NEW METHOD, 1312. The following new method may be often used with advantage when the amount is in pence and the rate of exchange is near 4.86. 1. What is the equivalent in United States money of 8d., exchange 4.88% Ans. $.16263, practically 1642. OPERATION. 8d X 26. = 16c. Ea:planation.—By this method of work we first multiply 8d x *iſc. = 00.2 the pence by 24, c. which is the value of 1d. when .2c. × 43 = 00063 exchange is 4.86; then to that part of the product obtain- ed by multiplying by Zºo, we add to or subtract Tº part $.1626; of the same for every #c. increase or decrease of the rate of exchange from 4.86. In this example the rate of exchange being 4.88, the difference between the same and 4.86 in half-cents is 4, hence we multiply the .2c. by ſº. 748 SouLE's PHILOSOPHIC PRACTICAL MATHEMATICs. * 2. What is the equivalent in United States gold of 93d., exchange 4.87; # Ans. Practically 1942. OPERATION OPERATION By the new method. By the first method. 93d X 24"; c. = 19.2375c. d. .2375c. × hºc. = 00.0593–1– 2 | 19 *E* sº 240 || 4,875 19.2968–H Ans. .19296––c. Ans. 3. What is the equivalent in cents of 2s. 6d., exchange at 4.90? Ans. 61#2. OPERATION OPERATION By the new method. By the first method. 2s. 6d. = 30d. × 245c. = 60.75c. d. *. .75c. × Tº = .5 30 * * ===mme 240 || 4.90 61.25c. Ans. * - sºmºsºms 61}c. Ans. PROBLEMIS. 1. What is the cost of a bill for £65241.3. 11, exchange $4.88%? Ans. $318703.24. 2. What is the importing cost of an English invoice of goods amounting to #365241 3. 11 ? Ans. $317496.28. 3. Exchange on Liverpool is quoted at 490, and wheat in Liverpool is quoted at 42s. 8d. per quarter, what is the equivalent value per bushel in the United States monetary unit? Ans. $1.303. OPERATION INIDICATED. 512 240 || 4.90 480 || 60 NotE.—480 pounds is one quarter. See Table of Weights and Measures. 4. A merchant imported from J. Wood, London, goods amounting to £460 14. 10, on which there is a duty of 50 per cent. What is the cost in United States money? Ans. $3363.30. NOTE.—Commercial students should make the Journal and Cash Book entries for the above transactions. 5. A merchant bought and remitted to J. Wood, London, a bill for £460 14s. 10d. when exchange was 4.88. What was the cost? Ans. $2248.42. NoTE.—Commercial students should make the Cash Book entries for the above transactions. TO REDUCE THE UNITED STATES MONETARY UNIT TO ENGLISH. 1313. 1. What amount of English exchange can I buy for $24375; the rate of exchange being 4.87; # Ans. £5000. OPERATION. 4.875) $24375.000 (£5000. Ans. Ea:planation.—Since the rate of exchange is the dollar and cent 24375 exchange equivalent of £1 Sterling, it is evident that we ma gº sº --- buy as many pounds as $24375 is times equal to $4.87#. $ º: ENGLISH EXCHANGE. 749 { 2. What amount of English exchange can be bought for $40250.65, the rate of exchange being 5.32? Ans. £7565 18S. 3d. OPERATION, $40250.65 – 5.32 = 67565.9116. 20 S. 18. 2320 12 d. 2,784 3. What will be the face of a 60 day bill of exchange on Liverpool, that can be bought for $7217.36, exchange at $4.853; ; Ans. £1486 11s. 8d. 4. A merchant owes a London correspondent $8119.53 currency, which he desires to invest and remit. The rate of exchange is 4.853, and gold is 12# per cent premium. What is the face of the bill? Ans. £148611s. 8d. FIRST SOLUTION. SECOND SOLUTION. $4.855 gold rate of exchange. 100 .606875 = 12} % premium gold. 112} 8119.530 *-* 4.855 5.461875 currency rate of exchange. $8119,53 – 5,461875 = 361486.5829 361486,5829 20 | 20 * = -se--º *-*s, *-*- S. 11.6580 S. 11.6580 12 12 *º-º-º-º: *-*-* d. 7.896 d. 7.896 * THIRD SOLUTION. Operation to buy gold. 4.855) $7217.36 ( cuses, O 100 S. 11.6580 112} | 8119,53 12 $7217.36 gold. d. 7.896 5. A Manchester merchant consigned to us an invoice of goods which, pursu- ant to his order, were sold for his account. The net proceeds are $11240.20 which we wish to remit; exchange is 5.41. What is the face of the bill? Ans. £2077 13s.5d. 6. I have a balance of $4210.12 belonging to a Liverpool merchant, which I am instructed to invest in 60 day English exchange and remit. The rate of exchange is $4.84%; I charge 4 per cent commission for investing; what is my commission and what is the face of the bill? Ans. $20.95 commission. £865 1s. 8d.-- face of bill. PER CENT AND INTEREST ON ENGLISEI MONEY. 1314. For the method of computing per cent on English money, see Article 868, page 449, and for the method of computing interest on English money see Article 1135, page 569. For accounts current and interest accounts in English money, see Article 1209, page 661. 75o soul E's PHILOSOPHIC PRACTICAL MATHEMATICS. Yºr TO FIND THE RATE OF EXCHANGE WHEN THE FACE OF THE BILL AND THE COST ARE GIVEN. 1315. 1. The face of a bill on London was £5000, the cost was $24375. What was the rate of exchange 3 Ans. $4.874. OPERATION INDICATED. 5000) 24375 ($4.875 Ans. 2. The cost of £46014s. 10d. was $2248.42. What was the rate of exchange? Ans. $4.88. PRACTICAL MISOELLANEOUS PROBLEMS. 1316. 1. A Liverpool cotton factor sold 200 bales of cotton, weighing 90000 pounds, at 8:#d. per pound. He charges 24 per cent commission for selling. What is his commission, and what are the net proceeds? Ans. £820s. 73d. commission. 3:3199 4s. 4d. net proceeds. FIRST OPERATION. 90000 weight. Commission reduced to £. s. and d. 8#d. price. 12 ) 19687.50d. commission as above. 810000 - 22500 20 ) 1640.7; d. = 1640s. 73d. 787500d. value in d. £82 23% rate of commission. ººm---sºmºmºs f'82 0s. 7#d. commission. 19687.50d. commission in d. 787500.00 value as above. 12 ) 767812.50d. Y proceeds in d. tºmsºmº. reduced to £. 20 ) 63984.44d. S. and d. :63199.4s. SECOND OPERATION. 90000 lbs. weight of cotton. #23281 5s. 8#d. price per pound. 24% rate of commission. 810000 3682. 0s. 7#d. commission. 22500 3281.5 0 sales of cotton. 12 ) 787500d. ) value in d. #3199. 4s. 4d. net proceeds. dººm-ºn-me l reduced to 20 ) 65625s. } {... and s. º--me #3281.5s. | NotE.—To compute 24 per cent commission, consider the whole amount 100 per cent and º by 40, as 2% per cent is zºo of 100. Or first compute the commission at 5 per cent and divide 2. A merchant in New Orleans has a balance of $5000 currency due to his Liverpool correspondent, and is instructed to invest the same in 60 day sterling exchange and remit. The rate of exchange is $4.86; the New Orleans merchant eharges 1 per cent commission for investing. What is the face of the bill? Ans. £1018 12s. 5d. ENGLISH EXCHANGE. 751 FIRST OPERATION To take out 1 per cent commission. $100 investment assumed. 1 = 1 % commission. $101 cash required to invest $100. $ 100 101 || 5000 $4950.50 SECOND OPERATION To convert dollars to pounds. 4.86 || 4950.50 #21018.6214 20 12,4280 12 5,136 3. What is the amount of duty on the following invoice, the United States tariff rates being 132 per pound on the hoop iron, and 10¢ per pound on the files not over 10 inches long, and 30 per cent on the cost of same; and 62 on the files over 10 inches long, and 30 per cent on the cost of same? Messrs. RAND & BERCEGEAY. Ans. $276.98. BIRMINGHAM, 17 September, 1895. Bot. of JNo. LUDLOW & SONs. 235 Bundles. £ | 8. d. || 3 || 8. d. N T. C. Q. L. R. B. 75 bundles Elephant Hoop Iron 14 by 18 1 19 2 0 - 40 4 * * * * * { * 1% by 17 1 0 0 0 - - 30 “ § { $ 4 e is 13 by 16 13 0 0 - - 40 ** & 4 $ 4 t 4 24 by 14 1 0 0 0 - - N. O. 50 “ { % & 4 t is 3 by 13 1 5 0 0 - - 5 17 2 0 - - At £715s. per ton, - - - - - - • * * 45 || 10 || 7 || Delivered free in Liverpool. * * * * * * * * 45 || 10 || 7 Cask 348. º 48 doz. W. & S. Butchers hand saw Taper Files 3% in 3/4 - - 8 Weight. 60 “ 4 t * ! * { 4 in 3/9 - - 11 || 5 75 “ & 4 * { tº e 4% in 4/3 - - 15 18 || 9 C. Q. L. 6 * * { { mill & 10 in 11/9 - - 3 || 10 || 6 1 2 8 -º- 38 || 14 || 3 Discount, 35 per cent. * tº e º as * * * 13 || 10 || 11 Weight. 25 || 3 || 4 9 doz. W. & S. Butchers mill saw Taper Files 12 in 16/6 - - 7 || 8 || 6 C. Q. L. 6 4 * { % * { 14 in 23/ - 6 || 18 | 2 0 *== *- 14 || 6 || 6 Discount, 32% per cent, - - as º º * G - 4 || 13 || 1 9 || 13 || 5 Cask, 7/9; carriage to Liverpool, 5/3, º tº º º º Q_º - 13 35 | 9 || 9 RECAPITULATION. Amount of 235 bundles, - - - - - º es tº - ºn tº 45 10 || 7 Amount of cask 348, - - - *- - - º - º º - 35 | 9 31|| 0 || 4 CHARGES. I 2 9 1 1 4 Commission, 2% per cent on £45 10s. 7d. ; 3 per cent on £359s. 9d. - - 2 || 4 || 1 - 83 || 4 || 5’ Shipping, port charges and bill lading, - - - - - - - 1 || 0 || 4 Consul's certificate, - - - - - - - - - - - 11 Due in cash this day, * gº tº tº º 'º - 84 || 15 || 9 E. E. JNO. LUDLow & SONS. 752 SOULE's PHILOSOPHIC PRACTICAL MATHEMATICS. ºr OPERATION. Weight of iron, 5 t. 17 c. 2 qr. 0 lb, = 13160 lbs. at 14c. * sº º tº ge ę tº $197.40 ‘‘ ‘‘ files not over 10 in. long, 1 c. 2 qr. 8 lbs. = 176 lbs. at 10c. &= gº $17.60 “ “ files over 10 in. long, 1 c. 2 qr. 0 lb. = 168 lbs. at 6c. gº a * = 10.08 Cost of files, £359s. 9d. at $4.8665 = $172,70; 30% on $173. is - sº •º º 51.90 *=s 79.58 Amount of duty * = tº me º 'º me - - - - - $276.98 TO FIND COST OF DIFFERENT ARTICLES OF IMPORTED GOODS. 1817. 1. To ascertain the cost of the different articles of imported goods, the amount of all the charges is first found, and then the invoice price of each article is increased in the proportion that the charges bear to the invoice price of the goods. When the cost is thus found the profits are then added. In foreign invoices it is frequently convenient to first find the per cent that the charges are of the invoice, and then increase the cost of each article according to that per cent. 2. A merchant imported from Birmingham, England, an invoice of goods amounting to £1723 14s. 11d. In the invoice were 12 dozen knives which cost £28 8s. 6d; allowing the duty to be 40 per cent and the freight to be $234.90, what was the per cent of freight on invoice and duty, and what was the retail selling price per knife to gain 25 per cent on the importation cost? Ans. 2%. $1.71 retail selling price. OPERATION. 3.1723.14. 11 = £1723,746 × $4.8665 $8388.61 OPERATION 40% duty on $8388.61 3355.44 tºmºmºmºsºmºmºmº To find the rate per cent of freight. Cost of whole invoice, $11744.05 :628 8s. 6d. £28.425 × $4.8665 $138.33 234.90 40% duty on $138.33, 55.33+ 11744.05 || 100 Cost of 12 dozen knives, $193.66–– |2.00016% practically 2%. 2% freight on knives, 3.87–1– N. O. cost of 12 doz. including frt. $197.53–H 25% gain on N. O. cost, 49.38–1– Price of 12 doz. knives, 12 ) 246.92– Price of 1 doz. knives, 12 ) 20.57+ Retail price of 1 knife.' $1.71–H Ans. * NoTE 1.—See pages 456 and 457, for Practical Work, Marking Goods and Computing the Rate Per Cent of Freight. NOTE 2.-See Customhouse Business for Importing Goods, pages 503 to 508 for other problems in importing goods. XP' & a ENGLISH EXCHANGE. 753 COMPUTATIONS OF FEEIGHT IN ENGLISH MONTEY. 1818. 1. What is the freight on 540 bales cotton, weighing 243084 pounds, at #d. per pound from New Orleans to Liverpool? Ans. £633 0s. 8d. FIRST OPERATION. SECOND OPERATION. 243084 D. #d. 8 5 12 || 243084 8 ) 1215420 20 12 ) 151927.5d. £633.0318 tºº 20 20 ) 12660 = 8d. tº ºs. OS. 6260 :6633 0s. 12 7d. 512 2. What is the freight and primage on 68 bales cotton, weighing 31416 pounds, at ##d. per pound; primage 5 per cent Ans. £122 14s. 4d., freight. £62s. 9d., primage. FIRST OPERATION. SECOND OPERATION 3.1416 #122 14s. 4d. amount of freight. To find freight. +3d. 5 % rate of primage. tº-º-º-º-ºººººmsº ** = me D. 16 ) 471240 £6.13.11.10% amount of primage. 16 || 15 tº sº, 20 12 || 31416 12 ) 29452.5d. 20 * -º 2S.71 wºmmºnsººmsºmº 20 ) 2454 = 43d. 12 £iºns, E122 = 14s. 8d.62+ ººmsºmºsºme 14.S. 376 NOTE.-Primage is a charge in addition to the freight; 12 originally it was a gratuity to the captain of the vessel for his * -º-º-º-º-º: particular care of the goods. It now belongs to the owners or 4d. 512 freighters of the vessel, unless by agreement, it is allowed in whole or in part, to the captain. 3. What will be realized in United States money from the freight and primage of 21000 bushels of wheat from New Orleans to Liverpool; freight 123d. per bushel, primage 5 per cent; exchange being 5.40? Ans. $6.201.56. FIRST OPERATION. 12; d. freight. Explanation.—We here reduce and place the #d. = 5 % primage on 12#d. rate of freight and primage 13; d. on our state- - *-*E* ment line, and multiply the same by 21000, the 13; d. freight and primage per bushel. number of bushels on which freight and primage are charged. . This gives us the amount of D. freight and primage in pence, which we reduce 8 || 105 to pounds by dividing by 12 and 20, or 240. 12 || 21000 Then, as the result thus obtained is pounds, or 20 || 5.40 pounds and decimals of pounds, we reduce the | * = same to United States money by multiplying by $6201.56 Ans. the rate of exchange 5,40. 754 soulE's PHILOSOPHIC PRACTICAL MATHEMATICs. * SECOND OPERATION. 21000 bushels. £11488s. 9d. = £1148.4375+. 12}d. rate of freight. 5.40 262500d. freight. $6.201.56250 Ans. 13125d. = 5 % primage. 12 ) 275625d. freight and primage. 20 ) 22968 9d. .61148 8s. 4. What is the freight and primage in United States money on 25000 bushels of corn from New Orleans to Liverpool, at 24s, per imperial quarter of 480 pounds; primage 5 per cent, and exchange 4.86? .* Ans. $17860.50. FIRST OPERATION. SECOND OPERATION. S. 25000 × 56 = 1400000 pounds. 24 480 || 56 1400000 25000 480 || 24 100 105 {-º-º-º: 20 || 4.86 20 || 70000S. $17860.50 Ans. 423500 amount of freight. 175 = 5% primage on £3500. £3675 freight & prim. in Liverpool. 4.86 rate of exchange. $1786.50 Ans. TO FIND THE PURCHASE PRICE OF ARTICLES TO FILL THE ORDERS OF ENGLISH MERCHANTS, AT A SPECIFIED PRICE, FREE OF ALL CHARGES WHEN DISCHARGED IN BRITISH PORTS. 1319. 1. A Liverpool correspondent instructs me to purchase and ship to him 7200 centals of white wheat, on condition that the total cost, including all expenses for purchasing, shipping, etc., shall not exceed 11s. 8d. per cental when placed on the dock at Liverpool. The freight is 25s. per imperial quarter, shipping expenses and commission for purchasing estimated at 9 per cent; exchange $4.90. What is the maximum price to be paid per bushel? Ans. 87;ºg. SOLUTION. Statement to find the freight on 1 Cental. S. 25 480 || 100 | 5s. 24d. freight on 1 cental. 11s. 8d. Liverpool price per cental. 5 2+ freight expenses. 6s. 53d. = 774d. = ***.d. to pay for wheat and remaining charges. * ENGLISH EXCHANGE. 755. • STATEMENT Explanation.—We first find, as shown in the first statement, the freight on 100 pounds, and To find the purchasing price of wheat. then deduct the same from the Liverpool limits of price, and thus obtain 6s. 5%d. = 77.3d., with which we are to buy the wheat and pay purchas- 2 | 155 ing and shipping charges. Then to find the 100 || 60 rice that this sum will pay per bushel, after 240 eing reduced to American money at the rate 4.90 of exchange given, and paying all charges, We 109 || 100 place the +3*.d. (774) on our statement line, and -*- reason thus: If 100 pounds (1 cental) cost fººd, 87: §c. Ans. 1 pound will cost the 100th part, and 60 pounds. or 1 bushel will cost 60 times as much. Then to convert the pence to pounds, we divide by 240; to convert the English monetary unit to American, we multiply by the rate of exchange 4.90, then to deduct the shipping expenses and our commission, both being on the same value, we multiply by 100 and divide by 109. The final result of the entire statement is the sum that we can pay for one bushel of wheat and comply with the conditions of the problem. 2. A London correspondent advises the purchase and shipment of 5,000 bbls. of Extra Flour at 32s., 6d. per bbl. C. I. F. (That is with Cost, Insurance, Freight and all other charges to deliver the same on the wharf at London, paid.) The freight is 2 s. 8 d. per bbl.; primage, 5%; all other shipping expenses, including Insurance, Freight, Drayage, Labor, etc., are estimated at 4% on first cost; Commission for purchasing, 5%; Exchange, $4.85. What price can be paid for the flour in the American market. Ans. $6.5954-H. OPERATION. 2s. 8d. freight. 2s. 8d. freight. D. 32s. 6d. Liverpool price. 5% primage. 1} primage. sº }: 2 9% deducted. am-mºº- -º 0 || 4.85 *-ºsmºsºm- 1.60 = 1; d. primage. 2s. 9%d. 104 || 100 29s. 8%d. = 356%d. = +**d. 105 || 100 $6.595-H Ans. To reverse the question and operation, and find the London cost of flour bought here at $6.5954+, with all the specified charges added, we produce the following figures: FIRST OPERATION. 6.5954 cost of 1 barrel in New Orleans. 100 || 104 100 || 105 4.85 £1,485 = £1 9s. 8.4d. 2 9.6 freight and primage. £1 12s. 6d. Ilondon cost. Ans. SECOND OPERATION. $6.5954 purchasing price of flour in New Orleans. .2638 = 4% shipping expenses on $6.5954. $6.8592 cost of flour with 4% shipping charges added. .3429 = 5% commission. $7.2021 cost of flour in London in U. S. money, freight and primage unpaid. $ 4.85 | 7.2021 |ELºs = <1 ºr 84 20 2 9.6 freight and primage added. 9,700 £1 12s. 6d. London cost. Ans. 12 ºmº-º- 8d. 4 756 SOULE's PHILOSOPHIC PRACTICAL MATHEMATICs. * 3. American tallow is quoted in London at 44s. 6d. per cwt. Allowing $124 per American short ton for freight, 5 per cent for primage, 3 per cent of prime cost for shipping expenses, and 5 per cent of sales for London charges, what is the maximum price to be paid in New Orleans when exchange is 4.80, in order to be sold at the London quotation, and a gain of 10 per cent on prime cost realized? Ans. $,0743599. OPERATION. 44s. 6d. London selling price. 44s. 6d. London selling price. 5% rate of London charges. 2 216 London charges. 2s. 2,64, commission. 42s. 3 ºd. = 507 ºd. London net proceeds, per cwt. Statement to find the net proceeds of 1 pound in U. S. money. D. 5073 240 || 4.80 112 $.0905892–H net proceeds of 1 pound. ; Operation to find the freight and primage on 1 pound. Freight on 1 ton, - - - - - - - - - - - - - - - $12.50 Primage at 5% on $124, * sº tº * * * * tº º ºs º º ºs .625 Freight and primage on 2000 pounds, tº gº ºs - - - - - 2000 ) $13.125 Freight and primage on 1 pound, tº ſº * * * * ~ * = - $.0065625 Which we deduct from the London proceeds of 1 pound, tº º sº º tº sº .0905892 N. O. cost, including 3% shipping charges and 10% gain on prime cost, dº gº º $.0840267 Operation to deduct the 3 per cent shipping charges and 10 per cent gain. | 100 113 | .0840267 $.0743599 Ans. N. O. prime cost, or the maximum N. O. purchasing price. EXCHANGE BETWEEN LONDON AND WARIOUS OTHER COUNTRIES. 1320. London, England, has been called the great clearing house of the world. The following table shows the rates of exchange of recent dates between London and the cities and countries named in the list. It should be remembered that these rates do not show the intrinsic value of the different monetary units, and that they are subject to a limited increase or decrease through the same causes that govern the rate of exchange in our own country. They are presented as average rates, for general information concerning exchange. 4. #: PROBLEMS INVOLVING ENGLISH EXCHANGE. 756% A New Orleans house received a cable message offering 8d. for 100 bales of cotton, good middling, good color and staple, f. i c. and 6% tare in Manchester- What would be the total New Orleans selling price and the price per pound, allow- ing freight at 282 per 100 lbs., and insurance at $74, exchange at 4.86% Ans. $7400.72 selling price. Price per lb. $.1480144. FIRST OPERATION. 100 B/C = 50000 lbs. Less 6% tare, 3000 “ mºm-ammº 47000 lbs. net (a) 8d.:-376000d. at 4.86 = $7614.00 Less freight on 50000 lbs. (a) 282 per C. lbs. 140.00 Operation to Reduce pence to dollars, exchange 486 $7474.00 d. $7614.4-10% ($761.40) = $8375.40 the 376000 amount to insure. 240 || 4.86 #% insurance on $8375.40 = 73.28 | $7614.00 Selling price, - - º - - $7400,72 $7400.72 - 50000 lbs. = $.14801-1- PROOF. 8d, a 486 = $162 = price of 1 lb. 8ól. .00972 = 6% tare. 240 || 486 * - -Eºmº- || Pº-smº- .15228 = net price. $.162 .0014657 = g- insurance. Operation to find the amount .1508143 = price less insurance. to insure. .0028 freight at 282 per C. pounds. $ .15228 = net price. tº ºn e .015228 = 10% increase. $.1480.143 net price of 1 pound. E-smº -m- 50000 = lbs. $ .167508 = amt. to insure (a) sº-º-º: • - #9% = $,0014657. $7400,715 = purchasing price. IPROOF. SECOND OPERATION, * 6 The following inaccurate but approximate method is used .48 = 6% by some cotton house accountants. 7.52 .0724 = ins. 100 B/C = 50000 lbs. Less 6% = 3000 “ * 7.4476 *º- .14 = freight, Ex. 480. 47000 (a) 8d. = fºlò66–13–4 || – Less freight @ 282 per 100 lbs., $140 = 29– 3–4 || 7.3076d. (at Exchange, 480) (7.3076d. x 4.86) £1537–10–0 || -- 240 =$.1479789 per lb. Less gºſ, ins. on £1566 plus 10% 15— 1–5 × 50000 lbs... = $7398.95 Difference caused by Selling price, - *- - tº- - £1522–8–7 the decimals, -> - 6 Exchange (a) 486 = - tº - - $7399.01 *m tºm-mm- $7399.01 NOTE: 1.-The inaccuracy of this method consists in using 480 as the value of the pound sterling, in calculating the freight and in omitting the shillings and pence on amount insured. NOTE: 2. —The first operation is strictly accurate and much shorter and easier, as we work in dollars and cents, instead of pounds, shillings and pence, after the first item. -- 2 O 's PHILO A A Q º 61 SOULES P SOPHIC PRACTICAL MATHEMATICS jºr A firm in Manchester, Eng., instructs its correspondent in New Orleans to buy 100 bales of good middling, good color and staple cotton on the following conditions: 8d. per pound f. i. C. and 6% tare. What price per pound may the New Orleans house pay, allowing $1.10 per bale expenses in New Orleans, 50 cents per bale commission, 30 cents per 100 lbs. freight, 1% insurance, estimating each bale to weigh 500 Ans. for 1st operation, $.1427. { % {{ 20. as tº .1448. IFIRST OPERATION. SECOND OPERATION. 86. (a) Ex. 480 = .16 cents 8d. Ex. 48665 = .1622–1– 1% insurance = .0016 1% insurance = .0016 = .1584 = .1606 6% tare 9504 6% tare 96+ 30g frt. per 100 lbs. .1489 30g frt, per 100 lbs. .1510 = .32 on 1 lb. 30 = .32 on 1 lb. 30 50g com. on 500 .1459 50¢ com. on 500 lbs. .1480 lbs. = .1% on 1 lb. 10 = .1% on 1 lb. 10 1102 exp. on 500 .1449 110g exp. on 500 lbs. .1470 lbs. =.222 on 1 lb. 22 = .22g on 1 lb. 22 N. O. buying price .1427 N. O. buying price .1448 NOTE: 1.-It is the custom of cotton buyers in problems of this kind to base their calcula- tions on a 480 valuation of the pound sterliug, thus facilitating the operation to some extent. NOTE: 2.—The difference between Exchange 480 and 48665 is .0665c. = 1.3854% or, practi. cally 1396. Hence, by adding 1; 96 to the 480 Exchange price we produce the 48665 Exchange price nearly. NoTE 3.-By multiplying the .480 Exchange price by the current rate of exchange and dividing the product by 480 we would produce the price at the current rate of exchange. #: ENGLISH EXCHANGE. - 757 EXCELANGE BETWEEN LONDON AND WARIOUS OTHER COUNTRIES. 1820. London, England, has been called the great clearing house of the world. The following table shows the rates of exchange of recent dates between London and the cities and countries named in the list. It should be remembered that these rates do not show the intrinsic value of the different monetary units, and that they are subject to a limited increase or decrease through the same causes that govern the rate of exchange in our own Country. They are presented as average rates, for general information concerning exchange. Exchange on Paris and other cities in France 25.25, fr. and cts. for £1. & 4 “ Amsterdam 4 & * { ‘‘ ‘‘ Holland 12.3, fl. and stiv. for £1. 4 & “ Brussels & ( & & “ “ Belgium 25.30, fr. and cts. for £1. 4 & ** Rome & 4 { { “ “ Italy (gold) 25.40, lire and cent per £1. 4 & ‘‘ Rome & & ( & “ “ Italy (paper) 28.10, lire and cent per £1. & 4 ** Berlin 4 & { { “ “ Germany 20.55, mks. and pf. for £1. & & ** Athens & 4 & 4 “ “ Greece 28.25, dr. and cts. for £1. 6 & ** Vienna, 4 & 2 & “ “ Austria 23.90, crowns. & & ** Madrid & 4 é & “ “ Spain. 25.25, peseta and cts. for £1. & 4 ** Lisbon “ “ “ “ Portugal 52d. for 1 Milreis. 4 & “ St. Petersburg “ “ “ “ Russia 30d. for 1 Rouble. é & “ Hong Kong 4 * & 6 ** ** China. 46d. for 1 Dollar. & & “Shanghai ** ** ** “ China. 62d. for 1 Tael. 4 & “ Calcutta, “ “ “ “ India 22d. for 1 Rupee. 4 & “ Rio de Janeiro ** ** “ “ S. America 26d for 1 Milreis of Brazil. & 4 “Copenhagen ** ** ** ** Denmark 13d. for 1 Crown. & 4 ** New York 4 & 4 & ** ** United States 49d. for 1 Dollar. NOTE. 1.-The milries of Portugal is equal to 52}d. or $1.08 which is a much larger unit than the milries of Brazil, which is equal to 54.6 cents. The table for Portugal money is as follows: 1 milreis = 2; crusados = 25 reales = 1000 reis. I = 10 ** = 400 * * 1 é & = 40 4 & NotE 2. —By the decree of October 19, 1868, the money of Spain was assimilated to that of France, and the Peseta of reales was considered as equivalent to a franc and was taken as the monetary unit. - NotE 3.—The new monetary unit of Denmark is the Krona or Crown which equals 100 Ore. 1 Crown = 26.8 cents or 13+d. NotE 4.—The money of account of the Netherlands (Holland) is the florin of 100 cents, worth 40.2 cents United States gold. The florin was formerly divided into 20 stivers. London. Eachange on New York. London. Eacchange on Paris. 1. What is the cost of a bill on New 2. What is the cost of a bill on Paris York for $5000, exchange at 494d. 3 for fr. 10540.50. exchange 25.25% Ans. £1026 0s. 10d. Ans. £417 8s. 11d. OPERATION. OPERATION, $500 face of bill. 25.25) 10540.50 (£417.4455 494d. exchange. 20 12) 23250d. * * 20) 20520 10d. 10d. 92 381026 08. 758 SOULE's PHILOSOPHIC PRACTICAL MATHEMATICS. & London. Eacchange on Calcutta. 3. What is the cost of a bill on Calcutta for 2000 company rupees 4 annas; exchange 23d. } Ans. £19113s. 10d. OPERATION. * 2000 Company rupees 4 annas = 2000.25 Company rupees. 23d. 4 Explanation.—The money of ae- 3, Illſlå.S = *====ºs = .25 12 ) 46005.75d. count in Calcutta is the rupee, which is divided into 16 annas, and each 20 ) 3833 9}d. ) *E=-º-º-º-º-º: # anna into 12 pice. fº 191 13s. London. Eachange on Shanghai. 4. What is the cost of a bill for 1250 taels 5 maces; exchange 584d. Ans. £304 16s. 24d. FIRST OPERATION, Ea:planation.-Accounts are kept in China in tael, maces, candarines and cash. The table is 1250 taels 5 maces = 12505 maces = 125050 10 cash, 1 candarine, 10 candarines, 1 mace and candarines = 1250500 cash. 10 maces, 1 tael. The tael of Shanghai is equiv- alent to $1.14 while the tael of Haikwan or 1250500 customs tael, is equivalent to $1.27. 1000 || 584 C * º Il Oº gold and silver coins in gamsºs ºssº ina. e only money made is the cash, and 12 ) 73154.250d. it is * coined %. ºft It is composed of 6 **E= parts of copper and 4 of lead. It is circular in 20 ) 6096. 2.250d. = 2+d. form, marked on one side only and has a square amºsºmsºmº hole in the center, by which means they are 3304 16s. carried like beads on a string or wire. Foreign coins are in general circulation in the ports SECOND OPERATION. open to the commerce of the world and they are considered as bullion rather than coin, and 1250 taels 5 maces = 1250.5 are usually taken by weight. , Silver ingots are 1250.5 × 58+ = 73154.25d. also used as money, being weighed by the unit 73154.25 reduced to £. s. and d. = £304 16s. tael. See Exchange on China. t 2#d. as in the first operation. UNITED STATES VALUE OF UNITED STATES STOCKS AND BONDs, WHEN QUOTED IN LONDON, AND THE LONDON VALUE OF SAME WEEN QUOTED IN THE UNETED STATES. PROBLEMIS. 1321. 1. United States 4 per cent bonds are quoted in London at 1154; what is the equivalent quotation in the United States, sterling exchange being 4.84? *~ Ans. 112.26;. OPERATION. 4.86 × 1153 = 56133 – 5 = 112.26%. NOTE.-All American stocks and bonds are quoted in London on an assumed value of the pound sterling. This assumed value is $5, modified by the rate of exchange. See Stocks and Bonds in this book, pages 708 and 709. 2. The quotation of American stocks in New York is 934, exchange is 4.864. What is the equivalent London quotation ? Ans. 95.8-1-. OPERATION. 93.25 × $5 = 46625 - 4.86} = 95.8+. 3. Louisiana 4 per cent consols are quoted in New Orleans at 101, sterling exchange is 4.85. What is the equivalent London quotation ? Ans. 104.12+. FRENCH EXCHANGE. 7.59 FRENCEI EXCELANGE. NOTE.-See French Monetary Table, page 235. 1322. The intrinsic value of the franc and the United States legal value for Customhouse computations, is $.193 or 1932. The rate or course of eachange on France is always the variable number of francs, or francs and centimes, allowed for a dollar. The basis for the computations of the rate of exchange is 5.20 francs for $1 gold, and it is quoted 5.20 more or less according as premium is charged or discount allowed. NOTE.—In English exchange the rate is the dollar and cent value of £1, according as premium is charged or discount allowed. But in French exchange the rate is the franc and centime value of $1. Hence in English exchange the higher the rate the dearer is exchange. But in French exchange the higher the rate the cheaper is exchange, and the lower the rate the higher is exchange. TO FIND THE RATE OF EXCHANGE WEHEN A PEREMIUM IS OHARGED, 1323. 1. Taking into consideration the balance of trade, the worth of money, etc., the banker or seller of French bills determines to charge say, 2 per cent pre- mium. What would be the gold rate, and if gold is 15 per cent premium, what would be the currency rate for French bills? Ans. Fr. 5.09%, gold rate. Fr. 4.433+, currency rate. OPERATION. Fr. 5.20 = $1.00 par of exchange. Ea:planation.—By the operation, we see that 02 = 2 % premium added. the cost of fr, 5.20 exchange, at 2 per cent $1.02 cost of 5.20 exchange. premium, is $1.02. The reasoning for the line Fr. statement is as follows: If $1.02 buy fr. 5.20, 5.20 ic. will buy the 102d part, and $1, or 100c. will 1.02 1.00 Thuy 100 times as much, which is practically fr. | Fr. 5.09%+ practically fr. 5.09%. 5.09%. * To find the currency rate of exchange at 2 per cent premium, gold at 15 per cent premium, we make the following statements: FIRST STATEMENT. SECOND STATEMENT. Fr. 5.20 = $1.00 par of exchange, 5.20 .02 = 2% premium. 1.02 | 1.00 --> 115 100 $1.02 gold cost of fr. 5.20. wº-ºº-º- .153 = 15% premium on gold. Fr. 4,433 Ans. ----- $1.173 currency cost of fr. 5.20 exchange. Practically fr. 4.433. Fr. 5.20 1.173 | 1.000 f Fr. 4,433 Ans. 760 soul E's PHILOSOPHIC PRACTICAL MATHEMATICs. . º TO FIND THE RATE OF EXCHANGE WHEN A DISCOUNT IS ALLOWED. 1324. 1. What would be the gold rate if the banker or seller allowed 1 per cent discount, and if gold was 124 per cent premium, what would be the currency rate? Ans. Fr. 5.252, gold rate. Fr. 4.6689, currency rate. OPERATION. Fr. 5.20 = $1.00 par of exchange. Fr. .01 = 1% discount. | 5.20 .99 il.00 Fr. 5.252+ Ans. Practically fr. 5.25%. Operation to find the currency rate. $.99 cost of fr. 5.20 exchange. Fr. Fr. 5.20 = $1.00 Fr. 5.20 .01 5.20 .99 || 1.00 * *- 1.11375 | 1.00000 112} | 100 or, .99 *m-º-º-º-º: .12375 = 124% prem. Fr. 4.6689 Ans. Fr. 4.6689 Ans. $1.11375 cost of fr. 5.20. TO FIND THE PREMIUM CHARGED OR THE DISCOUNT ALLOWED, WHEN THE RATE OF EXCELANGE AND THE PAR ARE GIVEN. 1325. 1. The rate of exchange is 5.09%+, the par is 5.20, what was the rate per cent premium charged? Ans. 2%. OPERATION. Fr. 5.20 par of $1. G. Or, with the G. 5.09%+ = rate of exchange deducted. 10}} fractions re- 51 || 5.20 •ºsmºs - 5.093+ | 100 duced thus: 26000 || 51 .10}} gain on fr. 5.09%+. | - 100 | 2% Ans. 2% Ans. NotE.—It will be observed that the seller gains on his investment, of 5.09%+ francs and not on the par of 5.20. 2. The currency rate of exchange is 4.433, the premium on gold is 15 per cent, what was the rate per cent premium charged, or discount allowed by the seller of the exchange? Ans. 2% premium. OPERATION. Fr. 4.433 currency rate. G. s .665 = 15% premium on gold. ,102 -º-º: 5.098 || 100 Fr. 5.098 gold rate. -ºmsºme - 5.20 par of exchange. 2+ practically 2% premium. .102 gain on fr. 5.098. 4k FRENCH EXCHANGE. 761 3. The rate of exchange is 5.25+, and the par 5.20, what was the rate of dis- count allowed by the seller of the exchange? Ans. 1% discount. OPERATION. Fr. 5.20 par of $1. L. 5.25+ rate of exchange. 4 || 2 dºme 2101 || 4 100 | $38% praetically 1% discount. NOTE.--It will be observed that the seller loses on his investment of 5.25+ francs and not on the par of 5.20. We have omitted the reasoning in the line statements of the foregoing prob- lems, because so often given in previous operations. Pages 437 to 448 give full explanations of similar percentage line statements. .05% loss by the exchange. TO CONVERT UNITED STATES MONEY INTO FRANCS, 1326. 1. What is the equivalent value in francs of $5000, exchange 5.15% Ans. Fr. 25775. OPERATION. Fr. § exchange value of $1. Ea:planation.—As the rate of exchange is the number of francs and centimes given for $1, 2575000 and as the rate is here fr. 5.15%, it is clear that 2500 for $5000 we can buy 5000 times the fr. 5,154; &= - hence we have but to multiply the rate of tºm-º. exchange by dollars to convert dollars to francs. Fr. 25775.00 Ans. 2. What amount of exchange can I buy for $1410.60, exchange at 4.45}} Ans. Fr. 6280.6965. OPERATION. $1410.60 × 4.45+ = fr. 6280.6965 Ans. TO CONVERT FRENCEI MONEY INTO UNITED STATES DOLLARS. 1327. 1. What is the equivalent value in dollars of fr. 25775, exchange 5.15% Ans. $5000. OPRRATION. * Explanation.—As the fr. 5.15} is the exchange equivalent of $1, it is clear that in fr. 25775 we 5.15} = 5.155 ) 25775,000 ($5000 Ans. have as many dollars as it is times equal to fr. 5.15}. Hence we divide the francs by the rate of exchange. 2. What is the cost of fr. 50500.404, exchange fr. 4.05% Ans. $12446.18. OPERATION. N T. 2 || 101000.81 or, 4.0575 ) 50500.405 ( 12446.18 Ans. 1623 || 4 $12446.18 Ans. 762 soul E's PHILOSOPHIC PRACTICAL MATHEMATICS. * TO INVEST IN FRANCS FOR A CORRESPONDENT AND CHARGE COMMISSION. 1328. 1. Invested $10000 in French exchange at 5.20. My commission on the amount invested is 3 per cent; what is the face of the bill bought? Ans. Fr. 51741.24, face of bill. OPERATION. $ 100 201 || 2 10000 5.20 Fr. 51741.24 Ans. * TO PUFCHASE FRENCEI EXCELANGE ON ORDERS FROM DISTANT CITIES. 1329. 1. A banker in New York instructs his correspondent in New Orleans to purchase fr. 200000. He limits the price, including all expenses, at 5.20. New Orleans sight exchange on New York at the time is # per cent discount. The cor- respondent who makes the purchase charges 4 per cent commission on prime cost. What is the cost of the bill, what is the commission and discount, and what is the maximum purchasing rate of exchange? Ans. $38461.54, cost. $189.93, commission. $284.90, discount. $5.26; maximum purchasing rate of exchange. solution. Operation to find the cost of the 200000 francs at 5.20, the limited price. Fr. 200000.00 -- 5.20 = $38461.54 cost, including 4% commission and #96 discount. Operation to deduct #96 commission and #96 discount, and thus produce the prime cost. $100.00 prime cost assumed. .5 0 = }% commission. 100 .75 = #96 discount. 101.25 || 38461.54 $101.25 = cost including the commission and discount. | $37986.71 prime cost. 189.93 = }% commission. 284.90 = #96 discount. --- $38461.54 Ans. Total cost. Operation to find the maximum purchasing rate of exchange. 37986.71 ) 200000 (5.2649, practically 5.26; maximum purchasing rate of exchange. 2. Suppose in the above problem, the price had been 5.25 and the rate of New Orleans exchange on New York had been # per cent premium, instead of # per cent discount, what would have been the full cost of the 200000 francs? Ans. $38381.67. 4. FRENCH EXCHANGE. 763 SOLUTION. Operation to find the cost of the 200000 francs at 5.25, the limited price, without considering the # per cent premium or the # per cent commission, Fr. 200000.00 - 5.25 = $38095.24 cost. This cost includes the # per cent commission but is exclusive of the allowance to be made for the # per cent premium. The following operation will allow for the # per cent premium, and exclude or deduct the 4 per cent commission. $100.00 prime cost assumed. .50 = }% commission. 100 * 99.75 | 38095.24 $100.50 cost with #% commission included. --sºmº 5 = #% premium on $100 deducted. $38190.72 prime cost. gºmº- 190.95 = }% commission added. $99.75 cost of $100 including 4% commission --se * and allowing for the #96 premium. $38381.67 Ans. Full cost. 286.43 = #% premium deducted. $38095.24 net cost after paying #96 com- mission and receiving #% premium on exchange. MISOELLANEOUS PRACTICAL PROBLEMS. 1330. 1. What cost fr. 2500.50, exchange 5.32% Ans. $470.02. 2. Find the value of 6400 francs at 5.214. Ans. $1227.23. 3. Find the value of 5940.75 francs at 5.18%. Ans. $1145.20. 4. How many francs can be bought for $10000 at 5.22; ? Ans. Fr. 52250. 5. Invest $4000 in francs at 5.254. Ans. Fr. 21010, 6. What are the proceeds of a draft on Paris for 32500 francs, exchange 5.30, and brokerage on exchange # per cent Ans. $6124.40. OPERATION. 32500 - 5.30 = $6132.07. 96 on $6132.07 = $7.67. $6132.07 – $7.67 = $6124.40 Ans. 7. What is the cost of a draft on Paris for 32500 francs, exchange 5.30, brokerage on exchange # per cent 3 Ans. $6139.74. 8. What will be the face of a bill on Havre that is bought for $5000, exchange being 5.353% Ans. Fr. 26768.75. 9. Sold exchange for 12000 francs on Paris through a broker; exchange 5.23, brokerage 4 per cent. What were the proceeds of the bill? Ans. $2288.72. 10. A French invoice amounts to 5840.60 francs; the duty on same is 50 per cent, the freight is $480. What is the importing cost? Ans, $2170.85. OPERATION. Fr. 5840.60 × $.193 = $1127.24 invoice -- 50% duty = $1690.85, -ī- $480 freight = $2170.85 import- ing cost. 11. What cost a bill for the above fr. 5840.60, exchange at 5.23? Ans. $1116.75. 764 SOULE's PHILOSOPHIC PRACTICAL MATHEMATICs. * 12. Imported from France, merchandise amounting to fr. 8542.50. What is the value in United States money? Ans. $1648.70, 13. Bought through a broker a bill for fr. 25800. Exchange 5.16, brokerage + per cent. What was the cost of the bill ? Ans. $5012.50. NOTE.-25800.00 - 5.16 = $5000. 4% on $5000 = $12.50. $5000 + $12.50 = $5012.50. 14. Invested through a broker $7400 in French exchange at 5.24, paid per cent brokerage. What was the face of the bill? Ans. Fr. 38727.58+. NOTE.-$7400.00 – 100% = $7390.76 × 5.24 = fr. 38727.5824. 15. Received $1425 for a French bill that was sold through a broker at 5.25; the brokerage was # per cent. What was the face of the bill? * Ans. Fr. 7500. OPERATION INDICATED. $1425 -- 99% = $1428.57+ x 5.25 = fr. 7499.99-H, practically fr. 7500. 1331. PARIS Exchange oN- Fr. C. 25.20 per £1 Sterling. London, short and 3 months, 3 & 4 99.50 ** 100 Francs. e & 4 & 4 Belgium, Italy, “ (: 3 é & tºs tº gº 89.50 “ 100 Lire (paper). Netherlands, 3 months, º * * º 211.50 ** 100 Florins. Germany, 3 * * tº tº º tº e tº 122.50 ** 100 Marks. Austria, 3 * * * = g- gº º tº- 105 ** 100 Crowns. Spain, 3 * * - - - - 505 ** 100 Pesos. Portugal, 3 “ tº- †- tº gº 550 ** 100 Milreis. Russia, 3 * * tºº. wº - - 312 ** 100 Roubles. NotE.—These are average rates and may be more or less according to circumstances govern- ing exchange. EXCHANGE ON BELGIUM, SWITZERLAND, ITALY, SPAIN, GREECE, AND FINLAND. 1332. The monetary unit of all these nations is the same, in value, as that of France, and hence the operations of exchange would be the same as in French exchange. NOTE 1.-In Italy the monetary unit is called the lire, of 100 centesimi, in Spain the peseta, of 100 centesimos; in Greece, the drachme of 100 lepta; and in Finland, the mark of 100 pfennigs NOTE: 2.—The old monetary unit of Spain was the hard dollar or duro, valued at 99c. con- taining 20 reales, each reale containing 34 maravedis, GERMAN EXCELANGE. NOTE.—See German Monetary Table, page 236. 1333. The present money of account throughout the German Empire is the mark (Reichsmarks). NOTE:-Following the German and French war, came the formation of the German Empire and the unification of her Monetary Units, the Thaler, Florin and Marc Banco. By acts of the German Parliament taking effect, one Dec. 4, 1871, and one July 9, 1873, the “MARK" was adopted as the monetary unit of the German Empire. The German Monetary Law of Dec. 4, 1871, ordains 1st, That “there shall be coined an º: GERMAN EXCHANGE. 765: imperial gold coin, 1394 pieces of which shall contain one pound of pure gold,” 2d, That the “tenth of this gold coin shall be called MARK, and shall be divided into 100 pfennigs.” The same law also provides that “besides the imperial gold coin of 10 marks, there shall be coined imperial gold coins of 20 marks, of which 69% pieces shall contain one pound of pure gold.” By act taking effect July 9th, 1873, 5 mark gold coin are also authorized, 279 of which must contain one pound of pure gold. By this same act the following coins are also authorized: 1st. Silver coins, 5 mark, 2 mark, 1 mark, 50 penny and 20 penny pieces. 2d. Nickel coins, 10 penny and 5 penny pieces. 8d. . Copper coins, 2 penny and 1 penny pieces. A pound of fine silver is coined into 20 five mark, 50 two mark, 100 one mark, 200 fifty penny and 500 twenty penny pieces. The gold and silver coins are all ºf pure and To alloy. The intrinsic and the United States Customhouse value of the mark in U. S. gold is 23.8 cents; which is a smaller value than any of the previous German monetary units; and because of its small value, exchange dealers and bankers find it more convenient to base the rate of German exchange upon the equivalent value of 4 gold marks expressed in United States money. Thus 23.82 x 4 = 95.2 cents, practically for exchange operations, 95% gold, which is quoted more or less according as a premium is charged or a discount is allowed. To find the currency or silver rate, the premium on gold is added to the gold rate. TO CONVERT GERMAN MONEY INTO AN EXCELANGE EQUIVALENT OF UNITED STATES DOLLAPS. 1334. 1. What is the gold cost of 15000 marks, exchange at 94% Ans. $3525. OPERATION. C. Ea:planation.—The rate of exchange being the 4 # 000 price of 4 marks, we reason thus: Since 4 marks Ö equal 94 cents, 1 mark is equal to the 4th part, $3525.00 Ans. and 15000 marks, 15000 times as much. 2. What will a bill of exchange for 24832 marks cost; exchange at 95%, gold 11 per cent premium ? Ans. $6580.79. OPERATION. C. Ea.planation.—Here we reason thus: Since 4 * #sº marks cost 13+ cents, 1 mark will cost the # *sº-º-º-º: part, and 24832 will cost 24832 times as much, $5928.64 gold. te which is $5928.64 gold, to which we add the 652.15 = 11% premium on gold. premium on gold and produce the correct result $6580.79 Ans. $6580.79. 3. Received instructions from a correspondent to purchase German exchange to the amount of 4189. marks and 45 pfennigs. Exchange is 1073. What is the cost? Ams. $1125.91. OPERATION INDICATED. C. 2 || 215 4 || 4189.45 766 soul E's PHILOSOPHIC PRACTICAL MATHEMATICS * ge TO CONVERT UNITED STATES DOLLARS INTO AN EXCHANGE EQUIV- ALENT OF MARKS. 1335. 1. What amount of German marks can be bought for $10000 gold, exchange being 96? Ans. 41666.67 marks, or 41666. marks and 67 pfennigs. OPERATION. M. Ea:planation.—In this problem, we reason thus: | 4 Since 96 cents buy 4 marks, 1 cent will buy the 96 |10000.00 96th part, and 1000000 cents will buy 1000000 41666.66% marks. times as many. 2. Invested $1260.55 silver in German exchange at 1054. What is the face Of the bill ? Ans. 4790.69 ms. NOTE.-The rate of exchange in this and the two following problems is based upon the supposition that gold is at a premium over silver. * FIRST OPERATION. SECOND OF ERATION. M. M. 4 4 4.21 | 4 & 105.25 | 1260.65 1260.55 -- |ſº marks. Ans. 4790.69 marks. Ans. 3. A merchant invested for a correspondent $5000 silver, in German exchange. The rate of exchange was 104; he charged 1 per cent commission on the silver invested. What amount of marks did he buy ? Ans. 19040.37 ms. T * FIRST OPERATION, SECOND OPERATION. $ $ M, | 100 100 | 4 º; 5000.00 101 || 5000 104 || 4950.495 1.04 || 4 mºmºmºsºm-º. ===- - * = &ºm=º lsº invested. 19040.37 marks. 19040.37 marks. + 4. Invested for a correspondent $20000 silver, in German exchange at 1083. How many marks did I buy, and how much does my correspondent owe me, allowing 1 per cent commission on the $20000 for investing 3 Ans. 73563.22 marks. $20200. he owes. 1336. HAMBURG EXCHANGE ON.— Portugal, 3 “ 3 & 4 450 ** 100 Milreis. Spain, 80.25 ‘‘ 100 Pesetas. Ms. Ps. London, short and 2 months, tº • * = 20.42 per £1 Sterling. & 4 & 4 gº sº tº & 4 #, . . . . . . . ;as #: Nº. é & º § * * * * * * sº 169,50 ** 100 Florins. New Yor & 4 & º tº º * , 413 ** 100 Dollars. Austria, 3 months, sº * > 4-> 175 “* 100 Florins. Italy, & 4 sº ºp tºº 75 ‘‘ 100 Lire. • NOTE.-These are average rates subject to an increase or decrease according to circumstances governing exchange. - & AUSTRIAN EXCHANGE. 767 MISCELLANEOUS PRACTICAL PROBLEMS. 1337. 1. Imported from E. Gretzner of Dresden, goods amounting to 85420.75 marks. If the duty is 60 per cent and the freight $340,30, what is the importing cost 3 Ans. $32868.52. OPERATION. Marks 85420.75 × $.238 = $20330.14, Dresden cost of invoice; 60% duty = $12198.08 $20330.14 + $12198.08 -- $340.30 = $32868.52. NOTE.—The commercial student should make the proper entries for the above transactions. 2. Remitted to E. Gretzner, a bill of exchange for the above invoice of 85420.75 marks, exchange 964. What was the cost of the bill? Ans. $20607.76. OPERATION INDICATED. 85420.75 4 | .965 NotE.—The commercial student should make the proper entries for the above transactions. 3. Imported from A. T. Kohn, Hamburg, Germany, merchandise amounting to 4212.15 marks. What is the importing value in American money? Ans. $1002.49. 4. Remitted to A. T. Kohn, a bill for 4212.15 marks bought at 94. What did it cost 3 Ans. $989.86. NOTE.-The commercial student should make the proper entries for the remittance of this bill. See pages 248, 249, 270, 271, 272 and 273, of Soulé's New Science and Practice of Accounts, for the proper Invoice and Cash Book entries for such transactions as the above. 5. Invested for a correspondent $20000 in German exchange, at 95%; charged A per cent brokerage. What was the face of the bill and what was my brokerage? Ans. 83571.65 marks face of bill. $99.50 brokerage. 6. Bought for a correspondent 20000 marks at 954, and charged # per cent brokerage. What was the cost of the marks, what was my brokerage, and what was the total cost 3 Ans. $4762.50 cost of marks. $11.91 brokerage. $4774.41 total cost. AUSTRIAN EXCHANGE. 1338. The kingdom of Austria-Hungary, by law of August 2, 1892, changed its monetary standard from silver to gold, and by the same law it changed its monetary unit from the florin to the krone (Crown) which is divided into one hundred “hellers.” Under this new law the coin of Austria-Hungary, is as follows: (1). 20 and 10 gold crown pieces. (2). 1 crown silver pieces. (3). 20 and 10 nickle and heller pieces. (4). 2 and 1 bronze heller pieces. 768 SOULE's PHILOSOPHIC PRACTICAL MATHEMATICs. * The value of the gold crown is 20.3 cents American gold. At this valuation the United States Customhouse estimates merchandise imported from Austria. See Table of Foreign Coins for the value of the former monetary units. Notwithstanding the new monetary unit and the new monetary standard adopted by Austria in 1892, American and European bankers continue (August, 1895,) to base Austrian exchange on the value of the florin as it existed prior to 1892. This gold value of the florin is 48.2 cents; the silver value is 38 cents more or less, according to the United States mint value of silver. The ratio value between the new unit, the crown, and the old unit the florin, is practically 100 crowns equals 42 florins gold; or, 100 fiorins equals 237.10 crowns. The eacchange value of the florin varies according to the premium or discount declared, and according to the value of silver in the United States. NotE.—The Metric System of Weights and Measures is compulsory in Austria-Hungary. PROBLEMIS. TO CONVERT AUSTRIAN MONEY TO THE EXCELANGE EQUIVALENT OF UNITED STATES DOLLARS. 1339. 1. What is the cost of 1480.45 florins, exchange at 414 cents? Ans. $614.39. OPERATION. Fl. 1480.45 41% $614,3867; Ans. 2. Bought 5862.70 florins, exchange 45, and paid 3 per cent commission. What was the cost $ Ans. $2651.41. TO CONVERT UNITED STATES DOLLARS TO THE EXCHANGE EQUIV- ALENT OF AUSTRIAN MONEY. 1340. 1. How many florins can be purchased for $9703.45, exchange at 48% cents 3 Ans. Fl. 20007.11 +. OPERATION. 97.03.45 .485)#9703.450(Fl. 20007.11+ Ans. or thus: 97 || 2 2. Invested, through a broker, $20000 in florins, exchange 40 cents, broker- age 4 per cent. What was the face of the bill? AnS. Fl. 49875.30. B R A Z II, I A N E X C H A N G. E. EXCHANGE ON RIO JANEIRO. 1341. The money of account of Brazil is the reis, 1000 of which make a milreis. BRAZILIAN EXCHANGE. 769 The INTRINSIC value of the milreis in United States money is 54.6 cents. The exchange value is 54.6 cents for 1000 reis, more or less, according as premium or discount is declared. In the notation of accounts, the milreis are separated from the reis by a sign thus (B, or thus $, called cifrao; and the milreis from the million by a colon thus: Rs. 4: 500 $ 200, is read four thousand five hundred milreis, two hundred reis. NOTE.-The French Metric System of Weights and Measures is the standard, and has been compulsory in Brazil, since 1872. But the ancient weights and measures are still partly employed by business men. The old Libra = 1.012 lbs. avoirdupois, and the arroba = 32.38 lbs. avoirdupois. PROBLEMIS. TO CONVERT BRAZILIAN MONEY TO THE EXCHANGE EQUIVALENT OF UNITED STATES DOLLARS. 1342. 1. What is Rs. 2: 430 $ 360 worth, exchange at 55; 3 Ans. $1348.85. OPERATION. Rs. 2430.360 × 55} = $1348.8498. 2. What will Rs. 467 $ 000, Brazilian exchange cost, exchange 53; ? Ans. $249.85. TO CONVERT UNITED STATES DOLLARS TO THE EXCHANGE EQUIVA- LENT OF BRAZILIAN MONEY. 1343. 1. How many milreis can be purchased for $2400, exchange at 51 ? Ans. Rs. 4: 705 SS 882 +. OPERATION. Rs. 1000 .51 || 2400.00 Rs. 4: 705 S 882 2. Invested $20000 in exchange on Rio Janeiro, exchange at 52%. What is the face of the bill ? Ans. Rs. 38 : 095 SS 238. EXCELANGE ON BRAZIL IN ENGLISH MONEY. NotE.—Most of the exchange on Brazil by United States Bankers and Merchants is drawn in Pounds, SHILLINGs, and PENCE. 77o soul.E's PHILOSOPHIC PRACTICAL MATHEMATICS. TO REDUCE BRAZILIAN MONEY TO ENGLISH. PROBLEMS. 1344. 1. An invoice of coffee amounted to Rs. 77 : 426 $ 100, exchange at What was the face of the bill in sterling? 21;d. per milreis, or per 1000 reis. Ans. £6976 8S. 3d. OPERATION. Rs. 77 : 426 $ 100 x 21:#d. = 1674339,4124d. 12 | 1674339.4125d. 20 139528S. 3d. 36976. 8s. 2. What is the face of a bill for Rs. 62 : 418 $ 610, exchange at 24; d? Ans. £6274 7s. 5d. TO REDUCE UNITED STATES MONEY TO ENGLISH. 1345. 1. What is the face of a bill for $27646.98, exchange at $4.85? Ans. £5700 8s. 2d. OPERATION. ...sºssmoº 0 8.1640 12 1.968 2. An invoice of coffee amounts to $8490.16. What is the face of a sterling bill for the amount, exchange at 4.86? Ans. £1746 18s. 11d. NOTE.—For invoices of coffee as they are made in Rio de Janeiro, and for the exchange, and the accounts current and interest accounts resulting from the importation of coffee from Brazil, see Customhouse Business, pages 505 to 508. EXCELANGE ON PORTUGAL. 1346. The monetary unit of Portugal is the MILREIS, the intrinsic and Customhouse value of which is $1.08. (534d). The exchange value is $1.08 more or less, according as premium or discount is declared. The operations of exchange are the same in form as those in Brazilian exchange. The monetary table is as follows: 1 Milreis = 24 Crusados = 25 reales = 1000 Reis. 1 & & - 10 & 4 - 400 & 4 1 * * = 40 ** The almude of Lisbon = 3.7 gallons. The almude of Oporto = 5.6 gallons. Yºk EXCHANGE ON VARIOUS COUNTRIES. 77 I EXCHANGE ON HONDURAS, COSTA RICA, GUATEMALA, NICARAGUA, SALVADOR AND COLOMBIA. 1347. The monetary unit of all these countries is the silver PEso, or dollar of 100 centavos, the value of which is, January 1, 1895, 45.5 cents United States gold. This value of the silver PESO, is determined by the value of silver at the United States Mints. The rate of exchange is the CENTS charged for a peso, and varies according to the general principles of exchange, the political and civil conditions of these countries and the facilities of trade between them and the United States. EXCHANGE ON PERU, ECUADOR AND CHILI. 1348. The monetary unit of Peru is the silver sol, valued at, January 1, 1895, 45.5 cents gold. The monetary unit of Ecuador is the silver SUCRſ, the value of which is, January 1, 1895, 45.5 cents gold. The monetary unit of Chili is the gold and silver PESO of 100 centavos, valued at 91.2 cents gold. The rate of the exchange is the cents charged for a sol, or SUCRE, or PESO. NotE.—See remarks above on Honduras Exchange. EXCELANGE ON MEXICO AND THE ARGENTINE REPUBLIC. 1349. The monetary unit of Mexico is the silver PESO or DOLLAR of 100 centavos, the value of which is, January 1, 1895, 49.5 cents gold. The gold dollar or peso of Mexico, is worth 98.3 cents. The rate of exchange is the cents charged for a PESO. The monetary unit of Argentine Republic is the gold and silver PESO of 100 centesimos, the intrinsic value of which is 96.5 cents gold. The rate of exchange is the cents charged for a PESO. CUBAN EXCELANGE. 1350. The monetary unit of Cuba is the gold and silver PESO of 100 centesi- mos, the intrinsic value of which is 92.6 cents. The rate of exchange is the cents charged for a PES. NoTE.—See Customhouse Business, page 504, for problems in importing goods from Cuba. R U S S I A N E X C H A N G. E. EXCELANGE ON ST. PETERSBURG. 1351. The money of account of Russia is the ruble of 100 kopecks. 772 SOULE's PHILOSOPHIC PRACTICAL MATHEMATICs. Yºr The silver ruble is the legal unit and it contains 17.9961 grams, and is worth, January 1, 1895, in American gold 36.4 cents. The gold ruble is worth January 1, 1895, in American gold 77.2 cents. The exchange value is 36 or 77 cents, more or less, according to the premium or discount declared, and according as the exchange is for silver or gold rubles. Russia has an IMPERIAL gold coin worth $7.718, and a half IMPERIAL gold coin worth $3.859. Paper rubles are in general circulation in Russia, and their value is much below that of the silver ruble. Since the reign of Peter the Great, ten changes have been made in the weights and fineness of Russian silver coin. NoTE.—In Russia the Julian calendar, or old style, is still retained. This calendar is now, and till the year 1900 will remain, 12 days later than the new style, and in leap years 13 days later. Thus, January 15 in the United States, is January 3d in Russia. From the year 1900 to 2000, the difference will be 13 days, and in leap years 14 days. This deviation commences March 1, each year. PROBLEMIS. TO CONVERT RUSSIAN MONEY TO THE EXCELANGE EQUIVALENT OF UNITED STATES DOLLARS. 1352. 1. What is the exchange equivalent or cost of 940 rubles, 28 kopecks, exchange at 69.4% Ans. $651.14. OPERATION. 940 rubles, 28 kopecks = 940.28 rubles. - 69+ $651.1439 2. What cost 31428.65 rubles, exchange at 73%, brokerage for buying # per cent 3 º: Ans. $23157.81. TO CONVERT UNITED STATES DOLLARS INTO AN EXCHANGE EQUIV- ALENT OF RUSSIAN RUBLES. 1353. 1. What amount of exchange on St. Petersburg can I buy for $846.10, exchange at 70 ? Ans. 1208.71 rubles. OPERATION. 846.10 – 70 = 1208.71%. 2. Invested $5000 in exchange on Moscow at 68. What was the face of the bill Ans. 7352.94 rubles. OPERATION. $5000 – 68 = 7352.94-H. JAPANESE EXCHANGE. 1354. The monetary unit of Japan is the gold YEN or DOLLAR which contains 100 sens. It is worth intrinsically 99.7 cents gold. The value of the Silver YEN X- EXCHANGE ON VARIOUS COUNTRIES. 773 is, January 1, 1895, 49.1 cents American gold. The gold coins consist of 20, 10, 5, 2, and 1 YEN pieces. The silver coins are 1 YEN and 50, 20, 10 and 5 SEN pieces. The copper coins are 2 SENS, 1, § and #6 SEN pieces. Paper currency is in general circulation, of which there are various denominations, corresponding to those in Coins. The rate of exchange is the CENTS charged for a YEN which varies according to the general principles governing exchange, and according as the exchange is for gold or silver YENS. NOTE.—The metallic value of silver in foreign coins depends upon the mint market value of bullion silver in the United States. PROBLEMIS. 1. What cost a bill on Tokio, for Y. 2530.40, exchange at 984 g. * Ans. $2492.44. OPERATION. 2530.40 × 984 = 2491.4440. 2. How many YENs can be bought for $5000, exchange at $1.023% Ans. 4878.04+ yens. OPERATION. $5000 — 1.02} = 4878.04 + yens. 3. An invoice of silk goods imported from Yokohama, amounts to 4210.60 YENS. If the Customhouse value is 99.7 cents per YEN and the rate of exchange is $1, what is the importing cost of the goods, and what is the cost of the bill of exchange, to remit in payment of the invoice? Ans. $4197.97 cost of goods. 4210.60 cost of bill. OPERATION. 4210.60 × 99.7c. = $4197.96820 4210.60 × $1. = 4210.60 4. The United States duty on the above invoice is 50 per cent ad valorem. Supposing the freight and insurance to be 10 per cent on the invoice, what would be the total cost to the importer ? Ans. $6716.75. NOTE.-Commercial students should make the Journal and Cash Book entries for all the transactions in the above 3d and 4th problems. See Soule's New Science and Practice of Accounts, pages 249 and 270 to 271, for the proper entries. EXCELANGE ON SWEDEN, NORWAY AND DENMARK. 1355. The monetary unit of Sweden, Norway and Denmark, is the gold KRONE or GRowN of 100 ore, the intrinsic value of which is 26.8 cents gold. The rate of exchange is the cents charged for a crown, or, the value of 100 crowns in dollars. 774 SOULE's PHILOSOPHIC PRACTICAL MATHEMATICs. * CHINESE EXCHANGE. 1356. The money of account in China consists of the TAEL, MACE, CANDARINE and CASH. The only official coinage of China is the copper CASH, of which about 1600 to 1700 equals 1 Haikwan or customs tael. Large payments are made by weight of silver bullion, the standard or unit of weight being the LIANG or TAEL. Hence the TAEL is a unit of Weight rather than a monetary unit. By treaty, it is equal in weight to 13 ounces avoirdupois, and an Haikwan tael is this weight of pure silver. The following is the table of weight: 10 Sze = 1 Hu. 10 Hu = 1 Hao. f 10 Hao = 1 Li (nominal cash). 10 Li = 1 Fun (Candaren). 10 Fun = 1 Tsien (Mace). 10 Tsien = 1 Liang (Tael) 1% oz. avoirdupois by treaty. 16 Liang = 1 Kin (Catty) 14 lbs. { { { { “. 100 Kin = 1 Tan (Picul) 1334 lbs. { { & 4 & 4 NOTE.-The Chinese standards of weight and length vary throughout the Empire. The intrinsic value of the silver tael of Shanghai is, January 1, 1895, 67.3% American gold; and that of Haikwan or customs, is 74.9 cents gold. NOTE.—By an imperial decree, issued in 1890, the silver dollar coined at the new Canton Mint is made current all over the Empire. This dollar is equal in value to the United States and Mexican silver dollar and to the silver Yen of Japan, taken at the United States Mint, market value. The rate of exchange is the variable dollar and cents charged for a tael. PROBLEMS. 1. What is the cost of a bill for 2400 taels, exchange at 674 cents? Ans. $1620. OPERATION. 2400 × .67% = $1620. 2. What cost 1650 taels 840 cash, exchange at 75 cents? Ans. $1238.13. OPERATION. 1650.840 at .75 = $1238.13. 3. Invested $3000 in taels, exchange at 72 cents. What was the amount bought 3 Ans. 4166.6663 taels, or 4166 taels and 6663 cash, or 4166 taels, 6 mace, 6 eandarine, and 63 cash. OPERATION. $3000+ 72c. = 4166.666;. ARBITRATION OF FOREIGN EXCELANGE. 1357. 1. A merchant in New Orleans wishes to remit 10000 marks to Hamburg, direct exchange is 962. Exchange on London is 4.87; London +% ARBITRATION OF FOREIGN EXCHANGE, 775 exchange on Hamburg is 20.50 m. for £1; New Orleans exchange on Paris is fr. 5.10; Paris exchange on London is fr. 25.10, and London exchange on Hamburg is 20.50 m. for £1. Which is the most advantageous way to remit? Ans. Through London. SOLUTION. First operation to find the cost of direct exchange. C. 96 4 || 10000 | $2400.00 cost of direct exchange. Second operation to find the cost of exchange through London. $ 4.87 = 1f. 1:9 – m. 20.50 | 10000 marks. | $2375.61 cost through London. Third operation to find the cost of exchange through Paris and London. Francs tº- 5.10 | 1.00 dollar. Pound sterling 1 25.10 francs. Marks - 20.50 | 1 pound sterling. 10000 marks. $2400.77 cost through Paris and London. 2. I owe a correspondent in Paris fr. 25000, which I wish to settle by remitting exchange. Direct exchange on Paris is 5.10. tº But before making a purchase I find on inquiry that exchange on London is 4.87, and London exchange on Paris is £1 for fr. 25.20. I also learn that exchange on New York is # per cent discount, and that New York exchange on London is 4.86%, and London exchange on Paris as above, fr. 25.20. Allowing # per cent commission on the whole amount invested at each intermediate place for re-investing, which is the most advantageous way to remit 3 º Ans. Through New York and London. SOLUTION. First operation to find the cost by direct exchange. Francs 5.10 | 1 25000.00 francs. *= assºsºme $4901.96 cost, Ans. Second operation to find the cost through London. Proceeds of amount invested 99.50 100.00 assumed investment. Francs given for £1 25.20 | 1 pound sterling. Pound sterling 1 || 4.87 exchange value of 1:8. 25000 francs. $4855.64 cost. Ans. Or thus : 99.50 || 100 25.20 | ? = francs. ? = £. 25000.00 4.87 | ? = francs. ? = £. ? = $4855.63 Ans. Third Operation to find the Cost through New York and London. Assumed face of New York bill 100 | 99.25 cost of New York bill. Amount invested in New York 99.50 | 100 cost of investment in New York. Amount invested in London 99.50 | 100.00 cost of investment in London. Francs given for £1 25.20 | 1 pound sterling. Pound 1 || 4,865 exchange value of 1:6. 25000 francs. $4838.46 cost. Ans. 776 SOULE's PHILOSOPHIC PRACTICAL MATHEMATICs. jºr 3. Suppose in the operation through New York and London that commission for reinvesting had been allowed only on the net amount invested, what would have been the cost of exchange 3 Ans. $4838.21. OPERATION. Assumed face of New York bill 100 99.25 cost of New York bill. Assumed investment in New York 100 || 100.50 cost of New York investment at , % commission. Assumed investment in London 100 || 100.50 cost of investment in London at 3% commission. Francs given for £1 25.20 | 1 pound sterling. - Pound sterling 1 || 4.865 exchange value of 1.6. 25000 francs. $4838.21 Ans. THE UNIFICATION OF TEIE MONEY OF THE WORLD. 1358. The importance of a uniform monetary measure for the world is so great that We cannot close our work on exchange without referring thereto. y The great variety of monetary units of the different nations of the world, and the consequent complications and intricacies of exchange, as well as the loss of time tand increase of labor that are thereby entailed upon commercial men, proves most clearly the need for the unification of the money of the world. Since the downfall of the Roman empire, the various countries of Europe have been afflicted with a heterogeneous coinage, and for ages past the loss of time and money, resulting from this cause, has been seriously felt by travelers and business men of all classes and climes. Napoleon the First, looking down on the world from the rock of St. Helena, declared that what Europe most needed was a common law, a common measure, and a common money. This solemn and wise utterance was a legacy not only to Europe but to the whole family of nations. In 1821, the American Secretary of State, John Quincy Adams, in his celebrated report to the Congress of the United States, pointed out the incalculable advantages of a common measure and a common money, which should, in his own comprehensive language, “overspread the globe from the equator to the poles.” With clear political sagacity, he saw, and said, that the object could only be accomplished “by a general convention of nations to which the world shall be parties,” and “in which the energies of opinion must precede those of legislation.” In 1867, the delegates of nineteen nations, the United States being one, representing 320 millions of people, met at Paris to consider the important question of unifying the money of the different nations, and in 1892 an international monetary conference was held in Brussels, at which the question of a single monetary unit for the leading nations of the earth was, with other financial matters, discussed; and although much good has resulted from these conferences, the great object remains unaccomplished. - We, therefore, urge the commercial student and general reader to labor in behalf of this cause until the dollar of the United States, the Pound of England, the Franc of France, the Mark of Germany, the Crown of Austria, the Ruble of Russia, the Milries of Brazil, the Tael of China, the Yen of Japan, and the other monetary units of the world are all of the same metal, weight, and purity. The names need not be the same, but the value and subdivisions should. This, the age demands, commerce requires, and common sense approves; and but for the preju- dice, the stupidity, bigotry, and the false national pride and fear of a few crowned heads, it would have been accomplished years ago. s TORAGE. G). ---------------------R A-4--a vºy ºur 1359. Storage is the keeping of goods in a store, warehouse, or other place of deposit, until required for use or shipment. 1360. Storage is also the price or charge made for the keeping of goods in a . Storehouse. 1861. Cold Storage is storage in refrigerating chambers or other places artificially cooled, to preserve articles liable to be damaged by heat. 1362. Bonded Warehouses are Government storehouses in which imported goods on which duties have not been paid, are stored. 1363. A Grain Warehouse, in which grain is stored, is called an elevator. 1364. Cash Storage is storage that is computed and paid on each withdrawaſ or delivery of goods. 1365. Storage Receipts or Warehouse Receipts are receipts showing the amount of goods in the storehouse, and are frequently bought and sold, especially those for grain and other produce, and are used as collaterals in discount and exchange transactions. 1366. Credit or Time Storage is storage on one or several lots of goods that may be received and delivered in different quantities at different times, and the charges made periodically, or at the final withdrawal. 1367. The Rates of storage charges are generally fixed by the Boards of Trade or Chambers of Commerce of the different cities, and are most usually made by the month of 30 days, per box, barrel, bale, sack, etc. No deductions are allowed when merchandise or property is withdrawn before the close of the month. A fractional part of a month is counted as a full month. In some cities and towns, if the property is taken out within fifteen days after the expiration of the first month, a half month is charged, but if after fifteen days a whole month. The owners or agents of property pay all expenses incurred at the store or warehouse for receiving, storing and delivering the property. (777) 778 soul E's PHILOSOPHIC PRACTICAL MATHEMATICS. º 1368. Average Storage. Independent of the regular storage charges goods are sometimes received, and, by special agreement, charges are made for the actual time they were stored. In cases of this kind it is customary, in computing the storage, to average the time, and charge a certain rate per month of thirty days. PROBLEMS. TO FIND THE AMOUNT OF STORAGE PER ARTICLE PER MONTH. 1369. 1. What is the cost of storage on 500 barrels molasses from November 3, to December 12, 1895, at 62 per barrel per month ? - Ans. $60. OPERATION. 500 bbls. from Nov. 3, to Dec. 3, − 1 month - - - - - at 6c. = $30.00 500 bbls. from Dec. 3, to Dec. 9, - fraction of month - - - - at 6c. = 30.00 Total amount, - - - - - - - - - - - $60.00 Ea:planation.—The general custom, as above stated, is to charge for a whole month whenever the goods are on storage a fraction of a month. 2. What would be the cost of storage on 300 bbls. flour received Oct. 10, and delivered Oct. 28; 400 bbls. received Oct. 16, and delivered Nov. 15, and 600 bbls. received Nov. 18, and delivered Nov. 30, at 5% per bbl. per month 3 Ans. $65. OPERATION. 300 bbls. Oct. 10, to Oct. 28, fraction of a month º - º at 5c. = $15.00 400 bbls. Oct. 16, to Nov. 15, 1 month - - - - - - at 5c. = 20.00 600 bbls. Nov. 10, to Nov. 30, fraction of a month cº- tº- º at 5C. = 30.00 $65.00 TO FIND THE AMOUNT OF STORAGE FOR AVERAGE PERIOD OF THIRTY DAYS PER, AFTICLE. | 1370. 1. What is the cost of storage on 500 bbls, molasses received Nov. 3, and delivered Dec. 12, 1895, at 6 cents per barrel; Ans. $39. OPERATION. - 1895. Nov. 3. 500 bbls. for 39 days = 19500 bbls. for 1 day. 6c. × 650 = $39. 19500 - 30 = 650 bbls. for 30 days, or 1 month. TO FIND THE AMOUNT OF STORAGE PER AVERAGE PERIOD OF THIRTY DAYS PER ARTICLE WEHEN DELIVERIES HAVE BEEN MADE. 1371. 1. From the warehouse books we find the following receipts and deliv- eries of flour, for the account of F. W. Ahsen & Co. By contract, storage is charged for the actual time the flour was on storage, at the rate of 5% per barrel per month: STORAGE DAILY BALANCE METHOD, 779 Receipts : Nov. 4, 1895, 200 bbls.; Nov. 5, 700 bbls.; Nov. 15, 1300 bbls.; Nov. 29, 3800 bbls.; Nov. 30, 1200 bbls. Deliveries: Nov. 7, 1895, 1000 bbls.; Nov. 11, 500 bbls.; Nov. 18, 600 bbls.; Nov. 20, 100 bbls.; Nov. 27, 1000 bbls.; Dec. 5, 1400 bbls.; Dec. 7, 1200 bbls.; Dec. 13, 800 bbls.; Dec. 16, 150 bbls.; Dec. 21, 300 bbls.; Dec. 24, 500 bbls.; Dec. 27, 300 bbls. What is the amount of storage due January 1, 1896, and how many barrels remain in the warehouse? Ans. $234.15. 1150 barrels in warehouse. FIRST OPERATION BY THE DAILY BALANCE METHOD. F. W. AHSEN & CO. * §3 1895. No. bbls. received. No. bbls. delivered. No. bbls. in warehouse. : g No. bºº for 2. aº Nov. 4 2000 2000 1 2000 { { 5 700 2700 2 5400 & 4 7 1000 1700 4 6800 ** 11 500 1200 4 4800 ** 15 1300 2500 3 7500 ** 18 600 1900 2 3800 4 |20 100 1800 7 12600 tº 27 1000 800 2 1600 44 |29 3800 4600 1 4600 44 |30 1200 5800 5 29000 I)ec. 5 1400 4400 2 8800 & 4 7 1200 3200 6 19200 44 |13 800 2400 3 7200 {{ {16 150 2250 5 11250 4 & 21 300 1950 3 5850 4 & 24 500 1450 3 4350 « |27 300 1150 5 5750 9000 7850 30 ) 140500 4683; 9000 bbls. Teceived. Practically 4683 barrels. 7850 “ delivered. 5c. 1150 “ in warehouse. $234.15. Ans. Ea:planation.—By the operation of the work, we see that there were 140500 barrels on storage for 1 day, and allowing 30 days for the month, there were 4683% (practically 4683) barrels on storage for one month, which, at 5c. per barrel per month, amounts to $234.15. This method of equating the time and making up the account for storage is applicable also to various other departments of business. Butchers and live stock dealers frequently hire their stock pastured or fed, with the privilege of entering and withdrawing stock as they may require, and pay for the actual time that stock was pastured. 78o SOULE's PHILOSOPHIC PRACTICAL MATHEMATICs. ºr SECOND OPERATION BY THE PRODUCT METHOD, I)r. Or. Date. Feceived. Days. Products. I)ate. Deliveries. Days. Products. 1895 ſ 1895 Nov. 4 2000) 58 116000||NOV. 7 1000 55 55000 & 4 5 700) 57 39900 ‘‘ 11 500 51 25500 ** 15 1300|| 47 61100 ** 18 600| 44 26400 44 (29 3800 33 125400 “ 20 100. 42 4200 ** 30 1200| 32 38400 “ 27 1000 35 35000 tº- Dec. 5 1400 27 37800 9000 & & 7 1200 25 30000 7850 - ** 13 800. 19 15200 * ‘‘ 16 150 16 2400 1150 bbls. on hand. “ (21 300 11 3300 ** 24 500 8 4000 “ 27 300 5 1500 . Total Dr. products, 380800 7850 Tot. Cr. prod’t 240300 ‘‘ CT. { % 240300 Bal. of Dr. products, 140500 140500 + 30 = 4683% practically, 4683 barrels, 5c. x 4683 = $234.15 storage. Explanation.—By this method, we assume that all items of receipt were in storage from the date received till January 1, and find the equivalent storage on 1 barrel for such time for each item of receipt, which gives 380800 days storage on 1 barrel. Then, since there were deliveries made, we find the number of days that each item of delivery was made before January 1, and the equivalent storage on 1 barrel for such time, for each item of delivery, which gives 240300 days storage on 1 barrel. This is deducted from the total debit product and gives a net debit of 140500 days storage on 1 barrel, which we divide by 30, the storage period, and obtain the number of barrels on which storage is due for one month at 5 cents. 2. The proprietor of a pasture received and delivered cattle for account of John Gunn, as follows: Received : Jan. 1, 1895, 900 head; Jan. 5, 350 head; Jan. 21, 1100 head. Lelivered : Jan. 2, 1895, 120 head; Jan. 3, 120 head; Jan. 4, 150 head; Jan. 14, 360 head; Jan. 17, 200 head; Jan. 23, 250 head; Jan. 27, 400 head; Jan. 28, 120 head; Jan. 31, 100 head. What will be the cost for pasturing these cattle from Jan. 1, to Feb. 1, 1895, counting the actual time that the stock was pastured, at 75 cents per head per week, (7 days)? Ans. $2556.42%. OPERATION BY THE DAILY BALANCE METHOD. Account of Pasturage of Cattle, at seventy-five cents per head per week (seven days), for JOHN GUNN, from January 1, to February 1, 1895. - | Number head Number head Number head in No. days. Number head pastured 1895. received. delivered. pasturage. pastured. for one day. Jan. | 1 900 900 1 900 2 * 120 780 1. 780 3 120 660 1. 660 4 150 510 1 510 5 350 860 9 7740 14 360 500 3 1500 17 200 300 4. 1200 21 1100 1400 2 2800 23 250 1150 4 4600 27 400 750 1 750 28 120 630 3 1890 31 100 530 1 530 | 2350 1820 23860 ** = . Number received 2350 7 ) 23860 number pastured for one day. Number delivered 1820 tº-ºm-º: 3408% average number pastured for one week. Number in pasture 530 75e. × 3408} = $2556.42%. A. STORAGE. 781 TO FIND THE STORAGE ON GOODS RECEIVED AND DELIVERED AT DIFFERENT DATES, THE STORAGE TERM AND RATE BEING GIVEN. 1872. 1. The following are the receipts and deliveries of coffee by a general Warehouse: º * Received. Delivered. 1895. July 7, 800 bags. 1895. July 24, 700 bags. “ 28, 250 “ Aug. 4, 100 “ Aug. 13, 300 “ & 4 8, 200 “ “ 26, 350 “ How much is due for storage August 26, at the rate of 3 cents per bag for a period of 10 days, or any part of 10 days? Ans. $85.50. OPERATION. Received. Delivered. Storage Charges. July 7, 800 July 24, 700 700 at 6c. - $42.00 “ 28, 250 Aug. 4, 100 100 at 90. - 9.00 Aug. 13, 300 “ 8, 200 200 at 6c. - 12.00 “ 26, 350 50 at 90. - 4.50 300 at 6c. - 18.00 1350 1350 $85.50 total storage. NOTE.—All goods delivered are deducted from the oldest receipts in the warehouse. Ea:planation.—By inspection, we see that the first delivery of 700 bags, July 24, was made 17 days after the first receipt of 800 bags, which is one term and a fraction of a term = 2 terms. Hence a charge is made on the 700 bags for 2 terms at 6 cents per bag = $42. The remaining 100 bags received July 7, were delivered Aug. 4, which is 28 days, = 2*, terms = 3 terms. Hence a charge is made on the 100 bags for 3 terms at 9 cents per bag = $9. Aug. 8, 200 bags were delivered which are applied to the 250 received July 28, thus making 11 days storage = 1%; term = 2 terms = 6 cents storage on the 200 bags = $12. Aug. 26, 350 bags were delivered. 50 of these bags were applied on the receipt of July 28, and 300 on the receipt of Aug. 13. The 50 bags owe storage from July 28, to Aug. 26, = 29 days = 2*, terms = 3 terms or 9 cents per bag = $4.50. The 300 #" owe storage from Aug. 13, to Aug. 26, = 13 days = 1%; terms = 2 terms at 6 cents per bag = 2. The following are the receipts and deliveries of corn by a grain elevator: Received. Delivered. 1895. May 4, 2000 bushels. 1895. May 31, 1200 bushels. “ 28, 1500 & 4 June 20, 2000 & 4 June 4, 1000 & 4 July 15, 4300 & 4 July 10, 3000 “ The storage on the above corn was 3 cents per bushel for the first 15 days or fraction thereof, and 2 cents per bushel for each subsequent 15 days or fractional part thereof. What is due for storage July 15, 1895? Ans. $379. OPERATION. Storage due on 1200 bu. May 4, to May 31, = 27 days = 2 terms at 5 cents = t- e- tº $60 Storage due on (2000-1200 =) 800 bu. May 4, to June 20, - 47 days = 4 terms at 9 cents = - 72 Storage due on 1200 bu. (amount taken from receipt of May 28, and added to balance of May 4, to make delivery of June 20,) from May 28, to June 20, - 23 days = 2 terms at 5 cents = º {- º - º - g- º º sº - tº sº g- º 60 Storage due on 300 bu. (balance of 1500-1200) from May 28, to July 15, - 48 days = 4 terms at 9 cents = * = as * * * * * * * * * * cº º 27 Storage due on 1000 bu. from June 4, to July 15, – 41 days = 3 terms at 7 cents = - - 70 Storage due on 3000 bu. from July 10 to July 15, = 5 days = 1 term at 3 cents = -> * 90 782 soul E's PHILOSOPHIC PRACTICAL MATHEMATICS. Yºr TO FIND THE STORAGE ON GRAIN SOLD IN THE EIDEVATOR OR WAREHOUSE. 1373. 1. A grain warehouse received, stored and delivered wheat as follows: Received. Delivered. 1895. May 3, 7000 bushels. 1895. May 13, 6000 bushels. “ 20, 6000 { { June 9, 7000 4 & “ 26, 9000 & 4 “ 13, 8000 ( & June 8, 8000 & 4 “ 26, 9000 & & 30000 30000 Compute the total and the extra storage on this account based on the follow- ing conditions: The seller to pay 2 cents per bushel for storage for the first 20 days, or part of 20 days; the buyer to pay 4 cent per bushel for each subsequent 10 days or part of 10 days; 5 days are to be allowed the purchaser without extra storage during which time the wheat is to be delivered. OPERATION. Storage on Date. Receipts || With ||Withdraw. Withdraw. | From To Days. 9. Rate. Amt. Total. Extra. bushels. || drawals. ||als from als from receipts. balances. 1895 May 3| 7000 1000 May 3|June 14 42 3# 3500 15|00 ** 13 6000 6000 “ 3|May 18, 15 2 12000 ** 20 6000 June 9 7000 6000 ** 20 June 14|| 25 2} | 15000 30|00 May 26 9000 1000 “ 26 July | 1| 36 3 30|00 10|00 June 13 8000 8000 ** 26 June 18 23 2} 20000 4000 ** | 8 8000 ** 26 9000 8000 June | 8|July | 1 23 2% 20000|$73500 4000 $135|00 Explanation.—First find storage on 6000 bu. delivered May 13, from May 3, to May 18, (May 13, +- 5 days) = 15 days or 1 term at 2 cents a bu. = $120.00. Then since the balance of the 7000 – 6000 = 1000 bu. remained in storage from May 3, to the next sale of 7000 bu. on June 9, we must find storage on same 1000 bu. from May 3, to June 14, (June 9 + 5 days) = 42 days or 4 terms at 3% cents = $35. Balance of sale of June 9, must have been taken out of the receipt of 6000 bu, on May 20, hence we find storage on 6000 bu. from May 20, to June 14, (June 9 + 5 days) = 25 days or 2 terms at 24 cents = $150. Now find storage due on the withdrawal of June 13, 8000 bu. from May 26, to June 18, (June 13 + 5 days) = 23 days or 2 terms at 24 cents = $200. Since the withdrawal of 8000 bu. on the 13th of June, left a balance of 1000 bu. of the receipt of May 26, we find storage on that 1000 bu. from May 26, to July 1, (5 days credit added to with- drawal of June 26,) = 36 days or 3 terms at 3 cents = $30. The final work is to find the storage on the last receipt of 8000 bu. on June 8, from June 8, to July 1, (June 26 + 5 days) = 23 days or 2 terms at 24 cents = $200. Thus producing $735 total storage and $135 extra storage, $735 — $135 = $600 storage due by the seller and $135 due by the buyer. * * f STORAGE. 783 2. What is the storage on the following receipts and deliveries of flour at 1 cent per barrel for the first 10 days, or fraction thereof, and 3 cent for each subsequent 5 days or fraction thereof; Ans. $14.25. Receipts. Deliveries, 1895. Feb. 3, 500 barrels. 1895. Feb. 11, 100 barrels. 21, 250 & 4 “ 18, 200 6 & 24, 300 & 4 “ 22, 225 & 4 28, 150 & 4 “ 24, 300 & 4 *mmº Mar. 5, 375 & 4 1200 *º-smº 1200 Storage on Date. Receipts || Withdraw- || Withdrawals From TO Days. | Rate. Amt. Total. barrels. als gross. from Balances. receipts. 1895 Feb. | 3 500 400 ** 11 100 100 Feb. 3|Feb. 11| 8 || 1 c. 1|00 ** 18 200 200 200 “ 3| “.. [18] 15 1}c. 3|00 ** 21 250 200 ** | 3| “.. [22] 19 2c. 4|00 ** 22 225 25 ** 21| “ 22| 1 || 10, 25 225 44 (24 300 225 ** 21| ** |24|| 3 || 10. 2|25 ** 124 300 75 ** [24] ** 24| 0 225 ** 24|Mar. || 5 || 9 || 1C. 2|25 ** 28 150 Mar. 5 375 150 << |28 ‘‘ 5; 5 1c. 1|50 | | 1.425 3. Elevator “A” of the Mississippi Elevator Co., received and delivered for B. Milmo & Co., wheat as follows: 1894. Nov. 18, received 545040 bushels, which were delivered Jan. 31, 1895. Dec. 6, & 4 474020 & 4 & 4 & 4 & 4 & 4 31, & 4 { { 11, & 4 326500 & 4 & 4 & 4 & & 4 31, & & 1895. Jan. 12, ** 692045. & & & 4 & & & 4 “ 31, “ & & 23, & 4 75000 6 & 6 & é & & 4 & 4 31, & & { { 26, & 4 52050 ( & “ & & é & 4 & & & (31, & 6 Total bushels, 2164725. According to the Rules and Tariff of the Elevator Co., 4 cent for each 10 days or part of 10 days is charged for storage, and 1 cent per bushel is charged for delivering. No charge is made on the fractions of a bushel. due for storage and delivery 3 What is the amount Ans. $483.30. The student will make the bill for the above, as per the following form: 784 . SouLE's PHILOSOPHIC PRACTICAL MATHEMATICs. * FOFM OF BILL. E L E W A T O R. “A.” NEW ORLEANs, January 31, 1895. Messrs B. MILMo & Co., To Mississippi Elevator Co. Dr., For storage charges on wheat. Wheat. Corn. In Out Shipped on (a) || Amount. 1894 1895 --- 5450|30 Ex. El. A Bins. Nov. 18||Jan. 31||S. S. Pendarves, 3 4740/20 Dec. | 6 ( & ( & $11850 3265100 ** 11 ( & & 4 2} 1895 6920.45 Jan. [12 ( & & 4 103.80 750 QQ & 4 23 { { & & 1+ 52015.0 ** 126 ( & & 4 650 21647|25 bushels. | $483|30 Received payment. ... NOTE.-In this bill the student will find the extensions of the first, third and fifth items of delivery, and the rate of cents to charge for the second, fourth and sixth items. º ſ Nº || yº º Allllfacturing (Il Shārēs and Tūllage. =N —º-º-º- 1374. Manufacturing on shares means that the manufacturer receives a Specified per cent or part of the articles or things manufactured in compensation for services rendered in manufacturing the material supplied to him. This method of paying for services is quite common in certain lines of business. By tollage is meant the taking of a specified portion of goods or articles in compensation for services bestowed upon the goods or articles, or upon the owner's part thereof, after taking the toll out; as the grain taken by the miller in payment for grinding. PROBLEMS, 1. A planter sends to a miller to have ground 80 bushels of corn. The miller charges } of the whole number of bushels for grinding. How much is his toll ? Ans. 10 bushels. OPERATION. Explanation.—Since the miller charges # of the whole number of bushels for grinding, he is 8 ) 80 bushels. entitled to as many bushels as the 80 is times sº-º-º: equal to 8, which is 10 times. Hence we see 10 bushels, Ans. that he charges 10 bushels for grinding 70 for the planter or 1 bushel toll for every 7 bushels ground for the planter. 2. Suppose, in the preceding example, that the miller had, by request, ground the whole 80 bushels for the planter, and that the planter had agreed to send the miller his toll the next day. How much corn should he send ? Ans. 11% bushels, = 11 bush. 1 p.k. 5 qts. 13 pts. FIRST OPERATION, 8 ) 80 Explanation.—By the exercise of our reason, *== we see that, as the miller's toll, 10 bushels, is # 10. Toll for grinding 4. of the whole quantity, he actually charges the toll for grinding # of the corn for the planter, TOLL. and hence it is clear, as shown by the line | 10 statement, that if the grinding of # of the corn 7 || 8 for the planter is worth 10 bushels toll, the | *mºmº grinding of # is worth the seventh part, and # 11; bushels. Ans. or the whole, is worth 8 times as much, which is 11} bushels. (785) 786 soul E's PHILOSOPHIC PRACTICAL MATHEMATICs. * SECOND OPERATION. 8 ) 80 bushels corn. 10 bushels toll. 70 bushels ground corn. Ezplanation.—In this solution we use numbers representing the actual number of bushels of corn brought to the mill and ground, instead of the fractional numbers as in the first solution, and in order to show clearly the relationship TOLL. . numbers, and the manner of obtaining them, 10 we present in the first part of the operation the 70 80 figures showing the bushels of toll and ground | *=- - corn or meal that the miller and planter would 11% bushels corn. Ans. have respectively received, had the usual custom of taking toll from the amount of corn brought to the mill been observed. By these figures we see that 70 bushels of ground corn require 10 bushels of toll, and as by the agreement the planter was to receive 80 bushels of ground corn, it is clear, as shown by the line statement, that if 70 bushels ground corn require 10 bushels toll, 1 bushel will require the seventieth part, and 80 bushels 80 times as much, which is 11} bushels. THIRD opFRATION. 100 bushels assumed as the quantity ground. B. 12} is # of same for grinding. 100 *-* • 874 80 87% is left after deducting toll. 913 bushels required to grind and deliver 80, and pay toll, and 80 deducted from 91% gives 11}. Or, To find the toll on 91% bushels, we operate thus: 8 ) 91% 11}. Ans. 3. A planter agrees to gin and bale the cotton of his neighbor for 'g under this agreement he ginned, baled and delivered 20 bales, weighing How many pounds of cotton is due him for his labor? pounds. FIRST OPERATION. 1 pound = #3 — I'o - † = 9200 pounds the owner's share of an unknown quantity to be ginned. Hence the following statement: 9200 9 10 102223 pounds = +3 the total quantity to gin as both the owner's and the ginner's shares. Then 102223 – 9200 = 10223 pounds the ginner's share, or the quantity yet to be ginned. ; and 9200 Ans. 10223 pounds. Explanation.—By the conditions of the prob- lem, the planter who gins and bales the cotton is entitled to fºr of the whole quantity ginned, and had he taken his pay out of the 9200 pounds he would have received 920 pounds. But as the whole 9200 pounds were delivered to the owner, the planter must now gin such an additional quantity as will pay him according to the con- tract. By the terms of the contract he is to receive ºf of all that is ginned, and hence as shown by the operation, for each pound ginned, Tº is for himself and Yº for the owner. There- fore the 9200 pounds is ºr of the total quantity that must be ginned, on which I'ſ may be com- puted as the share or compensation of the planter. Hence, since ºr = 9200 pounds +8 = 102223 pounds; "6 of which is 10223 pounds yet to beginned by and for the planter. SECOND OPERATION. 1 pound ginned, – I'd pound ginner's share, - ſº, pound owner's share. 9200 pounds ginned for owner. º's of 9200 = 920 pounds ginner's share, to be paid in cotton to be ginned. 9200 pounds – 920 pounds = 8280 pounds. Then the following proportional statement: 8280 : 9200 :: 920 : 10223 pounds yet to be ginned. NoTE.—The explanation for this operation is similar to that of the second solution, problem 2. STORAGE. * 787 THIRD OPERATION. 9200 – 10 = 920 pounds due for ginning 9200 pounds. 1 pound = #8 - To = 1°, pounds quantity ginned for the owner to earn ºr pound. Hence for each pound ginned the planter receives To pound and the owner #. Since it is worth 920 pounds to gin nº of the cotton for the owner, to gin fºr it is worth the 4th part, and to gin +3 it is worth ten times as much, which is 10223 pounds 4. A planter contracts with his laborers to give them # of all the corn they can raise on his plantation. The division to be made on the stalk. When the corn becomes ripe, he contracts to pay them gº of his share for gathering the same. The entire crop amounted to 14800 bushels. What was the planter's share # - Ans. 7104 bushels. OPERATION. # ) 14800 total number of bushels. º; ) 7400 bushels = planter's #. 296 bushels = ºr paid for gathering. 7104 bushels = planter’s share. 5. A rice planter contracts to have his rice harvested, thrashed and sacked for # or 25 per cent of the yield. The contractors harvested, thrashed, sacked and delivered to the planter 16500 sacks of 162 lbs. each of rough rice. It is then agreed that they shall continue to harvest, thrash and sack enough more rice to pay them according to the contract, and that the planter shall purchase the same at $1.80 per sack. What sum will the planter owe the contractors? Ans. $9900. FIRST OPERATION. 1 sack — 4 = + = 16500 sacks the owner's proportion or share of an unknown number to be harvested, thrashed and sacked. Hence the following statement: 22000 sacks to be harvested, thrashed and sacked. # of 22000 = 5500 sacks, the share of the contractors or harvestors. 5500 × $1.80 = $9900 Ans. SECOND OPERATION. 16500 – 4 = 4125 sacks due contractors. 1 — # = } 4125 3 || 4 | 5500 sacks to be harvested, thrashed and sacked for the contractors. 5500 × $1.80 = $9900 Ans. NoTE.—See second and third operations of problem 3. THIRD OPERATION. # = 25%; 100% — 25% = 75%; then 16500 sacks = 75% of a number; hence 1% will be the 75th part, and 100%, or the number will be 100 times as much. Thus, as 75: 100 : ; 16500 : 22000. 25% of 22000 is 5500 × $1.80 = $9900, NoTE.—Rice that has been harvested is thrashed and sacked for 8 to 10 cents per sack or barrel of 162 pounds. 788 SOULE's PHILOSOPHIC PRACTICAL MATHEMATICS. * 6. The owner of timbered land engaged laborers to make and bunch shingles; and for their services they were to receive # of what they manufactured. They manufactured 240000 shingles. How many of the number are they entitled to according to the terms of the agreement 3 Ans. 80000. 7. Suppose, in the preceding example, that the laborers had been required to manufacture, in excess of the 240000, the shingles to which they were entitled. How many would they have manufactured for themselves in order to make a just Settlement 3 Ans. 120000. 8. A planter agrees to gin his neighbor's cotton, and receive in payment for his labor and use of machinery ſº of the cotton ginned. He received 70000 pounds of seed cotton, which made 20000 pounds of ginned cotton. How many pounds of ginned cotton is he entitled to ? Ans. 2000 pounds. 9. The same planter referred to in the preceding problem agreed with another neighbor to gin his cotton, and receive in payment for his services and use of machinery seed cotton to the amount of Tº of the seed cotton ginned. He received 70000 pounds of seed cotton. How many pounds must he deduct before ginning to pay him according to contract, and how many pounds of ginned cotton did he deliver to his neighbor, allowing 34 pounds of seed cotton to make one pound of ginned? Ans. 6363 ºr pounds to be deducted. * 18181;ºr pounds he delivered. OPERATION. To + 1 or +} = }} 3}) 63636 ºr pounds to gin. 11) 70000 total amount received. smass 6363 ºr pounds of seed cotton due the ginner. 18181*r pounds ginned cotton delivered to the planter. Explanation.—In this operation we see, by the exercise of our judgment, that, as the planter gins the cotton for ºf of the amount ginned, we cannot compute the T'ſ on the 70000 pounds received, for that would be charging "I'm on the amount ginned, and on the cotton charged for ginning. By careful thought we see that if ºr of what is ginned is charged for ginning, then the amount received to gin is equal to Thy plus 1, or 43, which is +}, and hence if the amount received is +% of the amount ginned, ºr is the amount to be deducted for ginning. SECOND OPERATION. 100 pounds assumed to gin. P. 10 pounds due for ginning. 100 Cºsmºs 110 || 70000 110 pounds of seed cotton required to gin 100. tºº-º-º-º-º- 63636 ºr pounds to gin. 70000 6363 ºr pounds of S. C. due the ginner. 34 ) 63636 ºr 1818.13+ pounds of ginned cotton. Eacplanation.—The principles of this second solution have been so repeatedly explained in Per Centum, Commission and Brokerage operations, that we deem it unnecessary to occupy space with them. 10. A housekeeper wishes to supply his family with 30 bushels of flour. His miller charges ſº toll for grinding. How many bushels of wheat must he send to mill in order to obtain the exact 30 bushels of flour? Ans. 32.Ér bush. = 32 bu. 2 p. 7 qts, ºr pts. 3 LLIGATION. Øo).957-EEEEEEE EcºSEcº-EEEEEEEEEEEEEEEEEàNS 1375. Alligation, which signifies a tie or binding, is a process for solving Questions relating to the mixing or compounding of different kinds of ingredients, or of combining different qualities or values. It is generally treated under two heads or divisions, viz: Alligation Medial and Alligation. Alternate. 1376. Alligation Medial, or Medial Proportion, is the process of finding the mean or average price or rate of a mixture of different kinds or qualities of ingredients, when the quantity and value of each kind are given. PROBLEMS. 1. A merchant mixes 400 lbs. of sugar worth 9 cents per lb., and 600 lbs. worth 14 cents, with 1000 lbs. worth 15 cents; what is the mixture worth per pound? - Ans. 13; cents. OPERATION. 400 lbs. at 9e. = $36.00 Explanation.—In this example, we have only ‘‘ ‘‘ 14c. – 84.00 to multiply each given quantity by its price to 1000 ‘‘ ‘‘ 15C. - 150.00 , obtain its full value; then it is evident that the cº- sum of these values, $270.00, divided by the 2000 lbs. = $270.00 whole number of pounds, 2000, will give the mean value, or average price per pound of the Hence 1 lb. = Ans. 13}c. mixture. 2. If I mix 50 lbs. of tea at 60 cents per lb., with 40 lbs. at 75 cents and 100 lbs. at 80 cents, what is the average value of the mixture per pound 3 Ans. 73}; cents. 3. What per cent of Alcohol in a mixture of 40 gallons, 85 per cent strong; 60 gallons, 90 per cent strong; 50 gallons, 98 per cent strong; and 30 gallons, 95 per cent strong? Ans. 9144%. OPERATION INDICATED. 40 gals. (a) 85% = 3400 60 ( & “ 90% = 5400 50 “ “ 98% = 4900 30 “ “ 95% = 2850 180 ) 16550 ( 4. What is the purity of a mixture of 20 pwt. of gold, 14 carats fine; 30 pwt. 16 carats fine; 50 pvt. 12 carats fine; 40 pvt. 18 carats fine; and 10 pvt. pure gold 3 Ans. 15+ carats fine. ALLIGATION AITERNATE. 1377. Alligation Alternate is a kind of Indeterminate Reciprocal Proportion, and is the process of finding the ratios of the quantities of the articles, whose values or qualities are given, that must be taken to form a compound or mixture of a given mean rate of value or quality. (789) 790 . soule's PHILOSOPHIC PRACTICAL MATHEMATICs. * It is the reverse of Alligation Medial, and may be proved by it. The solution of questions in Alligation Alternate is based on the principle of an equality of gains and losses; that is, in the composition of a mixture, the gains on the articles whose values are less than the mean value, must be equal to the losses on the articles whose values are greater than the mean value. Thus, if we mix 1 pound of sugar worth 10 cents with 1 pound worth 12 cents, we will have 2 pounds worth 22 cents, or an average of 11 cents per pound. We here see that the gain of 1 cent on the first pound is just equal to the loss on the second pound; the amounts used being equal, the gain and loss are relatively equal. But suppose we had 1 pound worth 10 cents that we wished to mix with sugar worth 13 cents, so as to make a compound worth 12 cents per pound, we see that there will be a gain of 2 cents on 1 pound of the first kind, and a loss of 1 cent on 1 pound of the second kind, and hence to produce an equilibrium of gains and losses, we must take enough of the kind at 13 cents to give a loss of 2 cents, that is 2 pounds. Here we see that the quantity required of each kind is inversely proportional to the gain or loss on 1 pound of that kind, as compared with the other, and if there were several other kinds used, the same reasoning would apply, as shown by the following illustrations: The subject is treated under four cases or conditions. CASE I.—Analytic Method. TO FIND THE PROPORTIONAL PARTS TO BE USED WHEN THE MEAN OR AVERAGE PRICE OF A MIXTURE AND TEIE PRICES OF TEIE DIFFERENT ARTICLES ARE GIVEN. 1378. 1. What relative quantities of sugar, at 5 cents per lb. and 8 cents per lb., must be used to make a compound worth 6 cents per pound? Ans. 2 lbs. at 5 cents. 1 lb. at 8 cents. OPERATION. Eacplanation.—If a compound of two grades of sugar, one worth 5 cents and the other worth 1 || 2 8 cents, be made in certain proportions, and - - -sms - - the mixture be priced or valued at 6 cents, there 5|| 1 |2 will be a gain on the 5 cent grade and a loss on Ans the 8 cent grade. This gain and loss in the 8| 3 |1 whole mixture must be exactly equal, and hence we must take such quantities of each as will PROOF. give a gain and a loss of equal amount in the unit of value, here 1 cent; the result will be 2 lbs. at 5c. = 10c. the correct quantities to be compounded. 1 lb. at 8c. = 8c. By selling 1 pound of sugar worth 5 cents for tº- s=- 6 cents, there will be a gain of 6 — 5 = 1 cent 3 lbs. at 6c. = 18C. hence we place 1 pound opposite the 5. By selling 1 pound of sugar worth 8 cents for 6 cents, there will be a loss of 8 — 6 = 2 cents, and to lose 1 cent will require # of a pound, hence we place 3 pound opposite the 8. Therefore, 1 pound at 5 cents and # pound at 8 cents are the proportional quantities for the compound or mixture. Now, since the gain and loss are equal in these proportional quantities, it is clear that they will be equal in any number of times the same. Therefore, we may multiply the proportional quantities by any number and produce other proportional numbers. In this problem, we multiply by 2, the least common multiple of their denominators, and thus obtain 2 pounds at 5 cents and 1 pound at 8 cents, for the mixture. NOTE.—See page 159, for the Least Common Multiple. * ALLIGATION. 791 2. What proportions of tea at 48 cents, 60 cents, 66 cents, and 72 cents a pound, must be mixed together so that the mixture shall be worth 64 cents per pound? Ans. 1 lb. at 48 cents, 2 lbs. at 66 cents, 1 lb. at 60 cents, 2 lbs. at 72 cents. FIRST OPERATION. PROOF. 1 || 2 || 3 || 4 || 5 1 lb. at 48c. = 48c. * | *-* | *-* mm-- *-* | * *== 1 ‘‘ ‘‘ 60C. — 60C. ſ 48] T's 1 1 2 lbs. “ 66c. = $1.32 l ( & & 4 - 64: ; #| || Ans. 2 72c. = 1.44 (#| || 2 2 6 “ “ 64c. = $3.84 Ea:planation.—To maintain the equality of gains and losses, we must always compare the prices of two articles or ingredients, offe that is greater and one that is less than the average or mean price, and treat each pair or couplet thus compared as a separate problem. In the first operation, we formed two couplets and compared 48 and 72, and 60 and 66. In the second operation, we compared 48 and 66, and 60 and 72. In the first operation, we find 1, By selling 1 pound of tea worth 48 cents, for 64 cents, there will be a gain of 16 cents; and to gain 1 cent, would require I's of a pound, which we write in the first column opposite the 48 cents. 2. By selling 1 pound of tea worth 72 cents, for 64 cents, there is a loss of 8 cents; and to lose 1 cent, would require # of a pound, which is written in the first column opposite the 72 cents. 3. By selling 1 pound of tea worth 60 cents, for 64 cents, there is a gain of 4 cents; and to gain 1 cent, would require + of a pound, which we write in the second column, opposite the 60 cents. 4. By selling 1 pound of tea worth 66 cents, for 64 cents, there is a loss of 2 cents; and to lose 1 cent would require # of a pound, which we write in the second column, opposite the 66 cents. Now having finished with the couplets and produced the correct proportional quantities, we reduce the fractional quantities to integers or whole numbers, which is done by multiplying the quantities in the respective columns of gain and loss proportionals, by the least common multiple of their respective denominators. Thus, Yºg and * were multiplied by 16, giving 1 and 2 pounds, which we write in the third column. The + and # were multiplied by 4, giving 1 and 2 pounds which we place in the fourth column. The proportional quantities are then arranged in column five. NotE 1.--When the prices of the ingredients and the mean rate are whole numbers, the intermediate steps, as shown in columns 1 to 4, may be omitted, and the same results obtained by taking the difference between each article used as a couplet and the mean rate, and placing it opposite the other number of the couplet. See the linking method a few pages beyond. NotE 2.—There will be as many columns of proportional quantities as there are couplets. SECONID OPERATION, 1 2 3 || 4 5 - *- I -mº i t "-sºme NOTE.-This operation shows that by com- § Th; 1 2 : paring different prices, different proportional 643 à # 8 8 Ans. quantities are obtained. 72 # 1 1 NOTE. 1.--When there is an odd quantity, compare the same with one that has been already compared and take the sum of the two numbers opposite the one used twice. NotE 2.—A Common Factor may be used in any couplet or omitted from it without changing the proportional parts. Hence it is obvious that there may be any number of answers in the same proportion. 79.2 SOULE's PHILOSOPHIC PRACTICAL MATHEMATICs. * 1379. Loss AND GAIN METHOD, THIRD OPERATION AND PROOF. Proportional Ratio. quantities. 1 lb. at 48c. = + 16c. & *** — 1 lb. at 48c. = 48c. 64. 1 lb. at 69g. = + 49. }= 20c. gain = i. 1 lb, at 60c. = 60c. 1 lb. at 66c. = — 20. . 10c, loss = 1 2 lbs. at 66c. = 132c. 1 lb. at 72c. = — 8c. T º - 2 lbs. at 72c. = 144c. 6 lbs. at 64c. = 384c. Explanation.—In this operation, we first write the prices of the given commodities in a column in regular order, beginning with the smallest, and the mean price, 64 cents, on the left, as a basis of comparison. We then see that when we use 1 pound of tea at 48 cents, in a mixture to be worth 64 cents, we have a gain of 16 cents; we place this to the right of the price that produced it, and mark it plus, to indicate a gain. ... We then compare the next price, 60 cents, with the mean price, 64 cents, as in the first case, and find a gain of 4 cents, which we place as before to the right of its producing term, 60, and mark it plus also. Then proceeding with the next term, 66, we find by comparing it with the mean 64, that in using 1 pound there will be a loss of 2 cents; we place this to the right of 66, and mark it minus (—) to indicate a loss. Comparing the fourth term, 72, in the same manner, we find a loss of 8 cents, which we place and mark as in the last. Having now found the gains and losses on all the terms taken singly, as compared with the mean term on a basis of 1 lb. of each kind, we next add all the gains, marked plus, together, giving a total gain of 20 cents, and set to the right. We then add all the losses, marked minus, giving a total loss of 10 cents, and place to the right. Then, as these two amounts have a common measure, 10, we divide them both by 10, and set the quotients still to the right. These results have the same ratio to each other as the whole gain and loss, and hence we have the ratio of the gain to the loss as 2 is to 1, or a relative gain of 2 cents to a loss of 1 cent in the admixture of 1 pound of each kind of tea. But as the gains and losses in the whole mixture must be the same, we must equalize them by changing the relative proportions of the ingredients used; in order to do this, we must multiply the quanti- ties used, 1 pound of each, by the reciprocal ratio of its gain or loss. Now, the ratio of gain to loss is as 2 to 1, and their reciprocal ratio is as 1 to 2. Therefore we multiply the given quantities, 1 pound of each ingredient, that gave a gain, or that stands opposite the sign plus, by the relative Inumber or ratio of loss, 1, and place the product to the right in a column headed “Proportional Quantities;” then we multiply the units, 1 pound, opposite the minus sign, by the relative number or ratio of gain, 2, and set the results in the column of proportionals. These proportionals are the quantities required to be taken of each kind of the ingredients at the prices opposite to which they stand. To prove the correctness of the work, multiply these proportional quantities by their respective prices, and set the products to the right. The sum of the products divided by the sum of the proportionals will give for a quotient the mean price, as in the example. By this method the proportional quantities of all the commodities that give a gain are always equal, and all those that give losses are equal. These proportionals may be multiplied by any number, and hence, an indefinite number of answers having the same ratios will be found. LINKING METHOD, 1380. We will now present another method by which the reciprocation of ratios is more readily effected, and the results may be different from those of the first two methods, though both are correct. The reason however is not so apparent, though this method is often used, and has given the name to this branch of Arith- metic Alligation, as the process is indicated by alligating, or linking, one rate that is less than the mean rate with one that is greater, as shown below: OPERATION AND PROOF. 48T 8 lbs. at 48c. = $3.84 Explanation.—We here write the simples as in 64 60 2 lbs. at 60c. = 1.20 the first method, and link one that is less with 66 4 lbs. at 66c. = 2.64 one that is greater than the mean rate; that is, 72_| 16 lbs. at 72c, - 11.52 we link 48 with 72, and 60 with 66. We then -*. *- take the first couplet, 48 and 72, and comparing 30 lbs. at 64c. = $19.20 each term with the mean rate 64, we find there will be a gain on 1 pound costing 48 cents, when * ALLIGATION, 793 used in a compound valued at 64 cents, of 16 cents, and a loss of 8 cents on 1 pound costing 72 cents. Now, as the gain is to the loss as 16 is to 8, in this couplet where 1 pound of each is used, and as the mixture must give an equal combination of gains and losses from each couplet, we know from reason and previous explanations, that the quantities used must be taken in an inverse or reciprocal ratio to the gain and loss, that is, as 8 is to 16. We therefore place the difference between 64 and 48 = 16, not opposite the 48, to represent the gain as in the second method, but opposite the 72, to represent the relative quantity to be taken at that price; and for the same reason we place the difference between 64 and 72 = 8, opposite the 48. The terms of the other couplet, 60 and 66 linked together, are treated in the same manner, by placing the difference between the mean rate, 64, and each term, opposite to the one with which it is linked. These differences thus arranged constitute the proportional quantities that are to be taken of the several ingredients that form the compound, at the prices opposite to which they stand. The proof is the same as in the first method. When there are more than three terms, in which two or more are less, and two or more are greater than the mean, they can be linked in various ways, each variation giving a new set of ratios, and as each set of ratios admit of indefinite multiples, the answers to questions of this character, with those conditions, may be infinite. When any rate has several others linked with it, each alligated term will produce separate differences, and their sum will be the proportional required for that rate. PROBLEMS WORKED BY THE ANALYTIC METEIOD, 1381. 1. A wine merchant has wine worth $1.10, $1.80, $2.50, per gallon, which he wishes to mix with water so that the compound will be worth $1.50 per gallon. What will be the proportional quantities of wine and water ? Ans. 10 gallons water. 3 “ wine at $1.10. 4 { { { % {{ $1.80. 15 & 4 {4 {{ $2.50. OPERATION. | 1 || 2 || 3 || 4 || 5 0.0% ºf 10 10 \ $1.5% # i ; ; Ans. 2.5%l #6 15 15 2. A grocer has three grades of coffee worth respectively 10, 11, and 152 per pound. In what proportions must they be mixed so that the compound would be worth 122 per pound? Ans. In equal quantities. OPERATION. PROOF. | 1 || 2 || 3 || 4 || 5 3 lbs. at 10c. = $.30 3 lbs. at 11c. = .33 10| + 3 3 3 Ibs. at 15c. = .45 12 & 11 1. 3 || 3 × Ans. * ºsmºmº 15| | | + || 2––| 1 || 3 9 lbs. at 12c. = $1.08 Explanation.—Here we have two articles less than the mean rate, and one, greater. First compare 10 and 15 as the first couplet with 12, thus: 10 from 12, 2, 33 12 from 15, 3, #. Second com- pare 11 and 15 as the second couplet with 12, thus: 11 from 12, 1; 12 from 15, 3, #. Then, multiply the proportional numbers in column 1 by 6, and those in column, 2 by 3. The 6, and 3 being respectively the least common multiple of the denominators of the numbers in the respective columns. Then arrange these results in the 5th column. 794 SouLE's PHILOSOPHIC PRACTICAL MATHEMATICS. º 3. A planter has oats worth 40 cents, corn worth 60 cents, and barley worth 70 cents per bushel. He wishes to mix them so as to make a compound worth b0 cents per bushel. What proportions must he use? Ans. 3 bushels of oats. t 1 bushel of corn. 1 bushel of barley. OPERATION. | 1 | 2 | 3 | 4 | 5 Explanation.—Here we have two articles greater than the mean rate and one less. First compare 40 40. To | I'ſ 2+| 1 || 3 and 70, as the first couplet, with 50, the mean price. 50 & 60 To 1 || 1 Ans. Second compare 40 and 60, as the Second couplet, 70 ºf 1 1 with 50. Then proceed as in the previous problems. 4. What relative qualities of flour worth $43, $5, and $7+ per barrel, must be sold to realize an average price of $6+? Ans. 4 bbls. at $44; 4 bbls. at $5; and 12 bbls. at $74. OPERATION. Ea:planation.—First, use 43 and 74 as the first couplet, | 1 || 2 || 8 || 4 || 5 which gives 1}=4=# and 1. Second, use 5 and 7+ as the second, couplet, which 4}| # 4 4 gives 1+=#=# and 1. Then multiply the proportional 64 & 5 # 4 || 4 Ans. numbers in column 1 by 7 and those in column 2 by 5. 7#| 1 || 1 || 7-i-| 5 || 12 See problem 1, page 790, and problem 2, page 793. 5. If I mix wines worth respectively 60 cents, 80 cents, $1.00, $1.30, and $1.50 per gallon, how many gallons of each kind will be required to form a mixture worth $1.10 per gallon ? Ans. 4 gals. at 60%, 2 gals. at 80g, 2 gals. at $1, 4 gals. at $1.30, 5 gals. at $1.50. And other answers by different coupling. | | | * | * | * | * | * 60 go 4 4 80– * 2 2 1103 1.00] Y's 2 || 2 X Ans. 1.30— #5 go 3-H 1 || 4 1.50 || zºo 5 5 6. Mixed oats at 40 cents, corn at 50 cents, rye at 75 cents, barley at 80 cents, and wheat at $1 per bushel. How many bushels will it take to make a mixture worth 70 cents per bushel? *s Oats. Corn. Rye. Barley. Wheat. Ans. By the 1st, or reciprocal method, 1 bu. 2 bu. 4 bu. 2 bu. 1 bu. Ans. By the loss and gain method, 9 bu. 9 bu. 10 bu. 10 bu. 10 bu. Ans. By the linking method. 6 bu. 3 bu. 4 bu. 4 bu. 6 bu. And other ratios by different methods of linking. 7. A goldsmith would mix gold 12 carats fine, with some of 13, 14, 16, 18, and 20 carats, and alloy valued at nothing. How much of each kind will be required to make a compound of 15 carats fineness? Ans. By the 1st or reciprocal method, 5 parts of 12 carats; 3 parts of 13 carats; 1 part each 14 and 16 carats; 2 parts of 18 carats; and 3 parts of 20 carats. And other ratios by different method of work. 2} ALLIGATION. 795 CASE—II. 1382. When the quantity of one of the commodities composing a mixture of a given mean value is given, to find the quantity of each of the others. 1. A merchant has molasses worth 35 cents, 52 cents, and 70 cents a gallon. He wishes to form a mixture worth 60 cents per gallon, and to have the same contain 120 gallons of the 35 cents per gallon molasses. How many gallons of each grade of molasses must he take 3 Ans. See operation. OPERATION. 1 || 2 || 3 4 || 5 Explanation.—We first find, as in case 1, *-º- ºmº-E i = | *-m-tº-º sºm-ºmº || * article 1378, the proportional quantities to be 2 35 ºf 2 2 × 60 = 120 gallons of the 35 cents, 5 gallons of the 52 cents, 60 & 52 + 5 5 × 60 = 300), Ans. and 9 gallons of the 70 cents molasses. But 70 Y'ſ I'd 5-Hl 4 || 9 × 60 = 540 since 120 gallons of the 35 cents molasses, instead of 2, are to be used, we must take 1%0. = 60 times each of the other grades or priced molasses, in order that the gain and loss may be equal. Hence we have 2 × 60 = 120 gallons at 35 cents, 5 × 60 = 300 gallons at 52 cents, and 9 × 60 = 540 gallons at 70 cents. 2. A grocer has 100 pounds of coffee, worth 10 cents per pound, which he wishes to mix with other grades of coffee worth respectively 12 cents, 14 cents, 15 cents, 16 cents, and 18 cents per pound, so that the mixture will be worth 15 cents per pound. How much of each kind will be required? Ans. See operation. OPERATION. 1 || 2 3 | 4 | 5 || 6 || 7 10- || || 3 3 × 33} = 100 pounds. #- # 1. 1 × 33} = ; º 1 1 || 1 × 334 = 33% ‘ 15 i; any quantity at 15c. Ans. 16:- 1 || 1 3-H 1 || 4 × 33} = 133 pounds. 18—|| # 5 5 × 33} = 1663. “ Explanation.—We first find the relative or proportional quantities, as in case 1, these quantities are shown in column 7. Then, since 100 pounds of coffee worth 10 cents per pound are to be used, we divide 100 by 3, the relative number for the 10 cents coffee, and in the quotient 33% we have the ratio with which to multiply each of the proportional quantities, as shown in column 7, to obtain the quantity required of each. X. NOTE:-The 15 cent grade of coffee being of the same value as the mean rate, it is not considered in the operation. It may be omitted from the list of prices, if desired. 3. A merchant has 280 pounds of coffee, worth 16 cents per pound, which he wishes to mix with other qualities worth 18 cents, 21 cents, 22 cents, and 24 cents respectively. How much of each additional kind will be required to compose a mixture that will be worth 20 cents per pound? Ans. By 1st method, 280 lbs. each at 16, 22, and 24 cents, and 560 lbs. each at 18, and 21 cents. And other ratios by different methods of work and different linkings. Operation by the 1st method. 1 || 2 || 3 || 4 || 5 || 6 | 16| # 1 1 × 280 = 280 18 # | } 1+| 1 || 2 × 280 = 560 20 & 21 1 2 || 2 × 280 = 560 22 # 1 1 × 280 = 280 24 + 1 1 × 280 = 280 280 + 1 = 280 ratio number to multiply each of the proportional quantities. 796 soul E's PHILOSOPHIC PRACTICAL MATHEMATICs. * 4. A stock broker has 140 shares of R. R. stock that cost him $60 per share. How many more shares must he buy at $80 per share, so that he can sell the whole at $75 per share and gain 20 per cent on his investment? Ans. 20 shares at $80. OPERATIONS. Statement to find average cost By the 1st method. of 1 share sold at 20% gain. 1 2 3 100 * * : *= I tºms 120 75 62.É. # | 14 || 7 *º- 80 * 2 1 × 20 = 20 Ans. $62+ cost. 140 - 7 = 20 ratio number by which to multiply proportional quantity of railroad stock. By the linking method. By the loss and gain method. 60, 174 = ** = 7 60 24 gain = 1 = 7 62.É. 2} = } = 1 62}{3}}}#. = 7 = 1 1 × 20 = 20 Ans. 1 × 20 = 20 Ans. 1383. , When the quantity of two or more of the ingredients or articles is given or limited, to find the quantity of each of the others. º 1. How many pounds of Naples walnuts at 30 cents, and of almonds at 27 cents, must be mixed with 300 pounds Tenn. peanuts at 5 cents, 200 pounds of Italian chestnuts at 8 cents, 150 pounds of Brazil nuts at 9 cents, and 250 pounds of Louisiana pecans at 16 cents, so that the average price of the mixture may be worth 15 cents OPERATION, 300 lbs. Tenn. peanuts at 5c. = $15.00 200 lbs. Italian chestnuts at 8c. = 16.00 150 lbs. Brazil nuts at 90. = 13.50 250 lbs. Louisiana pecans at 16c. = 40.00 900 lbs. $84.50 ( 9, sc. = average or mean price of the given articles. 1 || 3 || 3 || 4 || 5 91s] ºr | Pº 270 |-|-216 || 486 900 pounds. 15 & 27 101 || 101 X x 13% = { 1872, “ 30 101 101 1874, “ 900 – 486 = 1% == ratio of increase for each proportional quantity. Explanation.—In all problems of this kind, we * find the average or mean price of the articles or ingredients given, according to the principles of Alligation Medial. See Article 1376. Second, we find the proportional quantities of the average price of the articles given and of the articles required, according to the principles of case 1. . See Article 1378. Third, we find the ratio number with which to multiply the proportional quantities so that 900 pounds of the given articles may be used. As shown in the operation, the proportional quantities are 486 pounds of the given articles and 101 pounds each of the ungiven quantities, the walnuts and the almonds. And since 900 pounds of the given articles are to be used, we divide 900 by 486 and obtain 1% as the ratio number with which to multiply the proportional numbers. ALLIGATION. 7974 2. A goldsmith has 11 oz. of gold, 14 carats fine, 12 oz. of 15 carats, and 17 oz. of 18 carats. He wishes to mix with these, gold of 17 carats, 20 carats, and 22 Carats, so as to make the compound 19 carats fine. How much of each of the three. last will be required? 11 oz 12 oz 17 Oz 40 oz. 640 – 40 = 1 oz., average 16 carats. Ans. See operations. OPERATIONS. FIRST STEP. . at 14 carats = 154 carats. – 180 ** = 306 “ ſ =640 6 & STEP. e= i <===s = ºmº || *====s* | *s-s-s-s Oz. Oz. Carats. Carats. 16| # l 1 40 at 16 = 640 19 17 # 1 Proportional 1 40 = 40 at 17 = 680 20 1 2 quantities. 27 * * * * 80 at 20 = 1600 22| || 1. 40 at 22 = 880 3860 40 – 1 = 40 = ratio multiplier. Or thus, for the second step : Oz. Carats. OE. 16–1 1 × 40 = 40 16- 3 × 13+ = 40 1944- | 3 × 40 = 120 Or 19| || | | < j} = } 20-l-l 3 × 40 = 120 thus: 20 2 x 13+ = 26} 22– 2 × 40 = 80 22—l 3 × 13% = 40 , 40 - 1 = 40 ratio number. 40 -i- 3 = 13% ratio number. 3. A broker has 60 shares of stock that cost $40 per share, and 40 shares that cost $75 per share. How many more shares must he buy at $90 per share, so that he can sell the whole at $50 per share and lose 33% per cent Ans. 140 shares at $90. OPERATIONS. Statement to find average By the 1st method. cost to lose 33}%. 40 || 3's 3 100 75 & 75 66; 50 90 || || || 7 × 20 = 140 Ans. | $75 cost. 60 – 3 = 20 ratio multiplier. By loss and gain method By the linking method. £i 15 = 3 75 40 35 gain = 7 = 3 75 § 15 loss = 3 = 7 × 20 = 140 Ans. 60 - 3 = 20 ratio multiplier. * 75 | 901 35 = 7 × 20 = 140 Ans. 60 – 3 = 20 ratio multiplier. 798 SouLE's PHILOSOPHIC PRACTICAL MATHEMATICs. CASE—IV. 1884. When the quantity and rate of the mixture to be made is given, to find the several quantities composing it, whose rates are given. * 1. A manufacturer has cotton worth 5 cents, jute worth 9 cents, and wool worth 20 cents per pound. He desires to make a mixture of 533 pounds worth 12 cents per pound. How many pounds of each must he use ? Ans. 164 each of cotton and jute, and 205 of wool. OPERATION. | | Explanation.—We first find the proportional quantities according to Case 1, Article 1378. 5| + 8 8 × 20% = 164 Then we add the proportional quantities 12 & 9 + 8 8 × 20% = 164 together which gives 26. Then, since the re- 20 # 7–H 3 || 10 × 20+ = 205 quired mixture is to contain 533 pounds, we *sº * *s divide 533 by 26 and obtain 20% as the ratio 6 533 multiplier for all the proportional quantities, thus producing 164 pounds each of cotton and jute, and 205 pounds of wool. 2. A wine merchant has an order for a cask of wine containing 126 gallons, to cost just $1.20 per gallon. Not having any wine of this price, he makes a mixture of other wines worth respectively 80 cents, $1.00, $1.30, $1.50, $1.80, and Water worth nothing. How many gallons of each kind will be required? * Ans. See operation. FIRST OPERATION. Proportional numbers. gals. ſ 0| T40 º 1. ...] 91% water at 0 = $000. | 1: £o 3. 3 3 23# wine ºs 80c. = 23.26% 1. * 1 j} e-º-º: 9% “ “$1.00 = 9.69% 1203 i. . 5|3} < 91% = 195, “ “ 1.30 = 25.20 | 150 3% 4 4 38}} “ “ 1.50 = 58.15A, U 180 go 2 3] 19é, “ “ 1.80 = 34.89% 13 126 gals. at $1.20 = $151.20 126 — 13 = 91% the ratio multiplier. NOTE:- Different answers may be produced by different methods forming couplets, or by linking, or by the method shown in Article 1379. Operation and proof by the loss and gain method as elucidated in Article 1379. Gal. Cts. Ratio. Gals. Cts. $ ſ 1, water at 0 = + 120 5 × 3 = 15 at 0 = 0.00 1, wine at 80 = + 40 X = 180 G. = 9 5 × 3 = 15 at 80 = 12.00 $ 1, “ at 100 = + 20 5 × 3 = 15 at 100 = 15.00 * 1.20) 1, “ at 130 = — 10 9 × 3 = 27 at 130 = 35.10 1, “ at 150 = — 30 X = 100 L. = 5 9 × 3 = 27 at 150 = 40.50 (i. * { at 180 = — 60 9 × 3 = 27 at 180 = 48.60 126 at 120 = 15.20 126 - 42 = 3 proportional multiplier. Operation and proof by the linking method as elucidated in Article 1380. 0—I 60 6 × 4} = 27 gals. at 0c. = $ . .00 80–, 30 3 × 4} = 134 “ at 80c. = 10.80 *..., | 103 10 1 × 4} = 4; “ at 100c. = 4.50 1.20 | OT – 4 & – © 130 20 2 × 44 = 9 at 130c. = 11.70 150–1 || 40 4 × 4} = 18 “ at 150c. = 27.00 18 12 × 4 + = 54 “ at 180c. = 97.20 28)126(4; 126 “ at 120c. = $151.20 +): * ALLIGATION. - 799 3. A liquor dealer has whiskey of 1000 proof, and another grade of 65° proof. How many gallons of each must he take to maké a barrel of 46 gallons of 90° proof? Ans. 32% gallons of 1000 proof. 13% gallons of 65° proof. OPERATION, Gal. Gal. - * * * *m i wºme 46 46 90% 65] gº || 2 = % of 65° 7 2 7 5 100 ſo | 5 = } of 100° -mºmºmº | — | 13} 32% 4. A farmer carried 120 fowls to market, consisting of turkeys, geese, ducks, and chickens, and sold the turkeys for $2.25 a piece, the geese for $1.50 a piece, the ducks for 50 cents a piece, and the chickens for 25 cents a piece. He received $120 for the lot, averaging $1 a piece for all. How many of each kind were there? Ans. See operation. Operation by the analytic method. 1 || 2 || 3 || 4 || 5 - 25. As 5 5 60 chickens. 100) .5% 3% 1 || 1 × 12 = 12 ducks. 150 8% 1 || 1 T ) 12 geese. 225. His 3 3 36 turkeys. 10 120 120 – 10 = 12 = ratio multiplier. Answer by loss and gain method. Answer by the linking method. Chickens, - - se 35 Chickens, 50 20 Ducks iº- - º 35 e Ducks, 20 Or 50 Geese, tº- - - 25 Geese, 20 30 Turkeys, - º - 25 Turkeys, 30 20 5. A contractor hired 5 men to work. He paid per day to each as follows, viz: to A. $5, to B. $4, to C. $3, to D. $2, to E. $1. Altogether they worked 120 days, for which he paid them $420. How many days did each work, and what amount did each receive 3 Operation and answer, by the analytic method. | 1. 2 | 3 4 || 5 || 6 || 7 E ſ 1 | } 6 6 ) 25% × 1 = $ 25%. E. D 2 # 2 3 | 83 × 2 = 17+ D. C 3} { 3 2 2 || 2 X x 4} = { 84 × 3 = 25% C. B 4 2 2 6+| 3 || 8 | 34% x 4 = 13.7% B. A 5 || 3 10 | 10 J 42% x 5 = 21.4% A. 2 Days, 120 $420 3% $420 120 – 28 = 4% multiplier. $420 - 120 = $34 per day. Answer by the loss and gain method. A worked 36 days at $5 per day = $180. & 4 36 { { $4 ( & 144. B > C & 4 16 4 & $3 & 4 3– 48. D & 4 16 ( & $2 { { ~ 32. E & 4 16 “ $1 ** = 16. 120 $420. 8OO SOULE's PHILOSOPHIC PRACTICAL MATHEMATICS. * 6. A stock raiser sold 100 animals for $100; he sold pigs at $4 each, calves at $1}, and sheep at $34. How many of each kind did he sell? Ans. 42 pigs, 54 calves, 4 sheep. OPERATION. 1 | 2 3 | 4 Ea:planation.—By the analytic method of Case 1, we find that the first couplet, as shown in Pigs #| 3 | # ||15–H (3) = 1 column 3, requires 15 pigs to 4 sheep; and that Calves | 1% 3 (6) = 2 second couplet, as shown in column 4, requires $1 1 pig to 2 calves. Then by inspection, we see that the sum of these proportional columns, 3 Sheep. U33| # 4 and 4, which is (19 -i- 3) 22, will not divide ===º sºmeºmºsºmsºmºms 100, the number of animals sold, without a 19 3 remainder, and as any remainder would give a fraction of an animal, we cannot, therefore, in 100 — 19 = 81 a problem where animals are the articles or ingredients use a multiplier containing a frac- 81 - 3 = 27 the ratio multiplier of the 4th tidn. Hence we must determine some number column of proportionals. without a fraction with which to multiply the proportional numbers in one of the proportional columns to produce the 100 animals. This we do by trial as follows: First subtract 3, the sum of the 4th column of proportionals, from 100 = 97 remainder. Now as this remainder cannot be divided by 19, the sum of the 3d column of proportionals, without a remainder, our first trial fails and we must try another way. We now subtract 19, the sum of the 3d column of proportionals, from 100 = 81, which can be divided by 3, the sum of the 4th column of proportionals, giving a quotient of 27. This 27 is the ratio multiplier of the 4th column of proportionals. Thus: 1 (Pig) × 27 = 27 pigs; to which we add 15 pigs shown in the 3d column of proportionals, = 42 pigs; then, 2 (calves) × 27 = 54 calves. Then, as shown in the 3d column of proportionals, there are 4 sheep. NoTE. 1.--Whenever the division of the remainder by the sum of the unused proportional column, does not give an integer as a multiplier, we must, by multiplication of one of the other of the proportional columns and a trial at division, try to find some integral multiplier. NOTE: 2. —It should be remembered, as stated in note on page 791, that a common factor may The used or omitted in any couplet or column of proportional quantities without changing the pro- portional parts. 7. A man purchased 100 killed and dressed animals consisting of cows, goats and pigs, for $100. The price of each was as follows: Cows $10 a piece, goats $3 a piece, and pigs 50 cents a piece. How many of each kind did he purchase ? Ans. 91% pigs, 4% goats, 4% cows. Operation giving fractional numbers. | | | * | * | * ||_ Average # * | * ||18-H 4 ||22) 913 pigs at $# = $45; Cost 3 # 1 || 1 ſ $1 1 : x 4} = { 44 goats at $3 = $123 10| | 1 | | 4; cows at $10 = $41; 2. 100 animals $100 100 - 24 = 4; ratio multiplier. 8. Suppose in the above problem that the animals were alive, how many of each kind did he purchase ? Ans. 94 pigs, 1 goat, and 5 cows. OPERATION. te | 1 2 3 4 100 — 5 = 95 * 95 – 19 = 5 ratio multiplier. #| 2 || 2 |18-H 4 $1 & 3 # 1 18 × 5 = 90 + 4 = 94 pigs at $4 = $47 10| || 1 1 × 5 = 5 cows at $10 = $50 *====º mºsºmºs 1 goat at $3 = $ 3 19 5 *s-sº 100 animals $100 * ALLIGATION. *g 8o I 9. Suppose in the above problem the price of the cows had been $10, the price of the goats had been $1, and the pigs 124 cents. How many of each would he have purchased ? Ans. 7 cows, 21 goats, and 72 pigs. 10. A person bought 100 head of cattle for $100 to-wit: Cows $10; yearlings $5; calves $2; and goats at 50 cents each. How many were there of each kind? Answers 1 and 2 as per operations below. Answers 3, 4, 5, 6, 7, 8, 9, and 10 may be produced according to the manner of forming couplets and variations in finding ratio multipliers. Cows, 1 4 1. 1 1 1 1 1 1 4 Yearlings, 1 1 2 3 4 5 6 7 8 2 Calves, 24 5 21 18 15 12 9 6 3 2 Goats, 74. 90 76 78 80 82 84 86 88 92 100 100 100 100 100 100 100 100 100 100 OPERATION. 1 || 2 || 3 4 || 5 || 6 Cows ( 10|| $ 1. × 1 = 1 COW at $10 = $10 Yearlings || 5 + 1. × 1 = 1 yearling at 5 = 5 Calves $1 & 2 1. , 1 × 24 = 24 calves at 2 = 48 ; × 1 = 18 Goats #|| } # # || 18 || 8 || 2 × 1 = 8) = 74 goats at # = 37 —|—|—X 24 = 48 *E=mº Eºmºmºmº 19 || 9 || 3 100 animals $100 100 — (19 + 9) = 72 72 – 3 == 24 ratio number for proportional column 6. The ratio number for columns 4 and 5 is 1. Or, we may vary the work as follows: 5 times 3, the sum of column 6, = 15 To which add sum of column 5, 9 Gives, 24 Then, 100 — 24 = 76. 76 – 19 = 4 the integer ratio multiplier for column 4. 1 is the multiplier for column 5, and 5 is the multiplier for column 6. See the following partial operation: 4 || 5 || 6 1. × 4 = 4 cows. 1 × 1 1 yearling. 1 × 5 = 5 calves. × 4 = 72 R 90 goats. 18 8 2 × 1 = 8 *E* × 5 = 10) 100 animals. 11. A drover bought 100 head of sheep, for which he paid $100, paying $3 a piece for the wethers; $1.50 a piece for the ewes; and 50 cents a piece for the lambs; how many of each did he buy? 8O2 SouLE's PHILOSOPHIC PRACTICAL MATHEMATICS. jºr OPERATION. | 1 || 2 2 || 2 si'i 2 { # 4 -º- 4 x 5 = 20. 2 2 100 — 20 = 80. 80 – 5 = 16. 4 | Proportional columns. 3 || 4 4 2 × 16 = 64 * 2}: 5 = 10 X 5 = 1. × 16 = Ea:planation.—Having found the proportional numbers shown in columns 3 and 4, according to Case 1, and Method, 1, then, in trial for integer multipliers, multiply 4, the sum of column 4, by 5 = 20, and subtract this product from 100. Then divide the remainder 80 by 5, the sum of column 3, - 16 which is the ratio multiplier for column 3. The ratio multiplier for column 4 is 5, the sum of column 3. 74 lambs at $ # = $37 10 ewes at 13 = 15 16 wethers at 3 = 48 100 animals $100 Several other answers may be produced by different methods and by variations in finding ratio multipliers. Partnership. § --→ =N 1385. Partnership is a contract whereby two or more persons agree to combine their property, labor, knowledge, credit, good will, or skill, (one or more of these) for the purpose of a common undertaking and the acquisition of a common profit. The partnership or business is called Firm, House, Concern, or Company. The money, merchandise, or other kind of property that is contributed or brought into the partnership by the members thereof, is called Capital, or Capital Stock, or Capital in Trade. Sharing Gains and Losses. The gains and losses of a partnership are shared most generally in certain fractional or percentage parts agreed upon at the time the partnership is formed. In determining these parts, the money or property investment, the business experience, the skill, the commercial standing or credit of each partner, and the service to be rendered by each, are all considered as elements in fixing the respective shares of gains and losses. Sometimes one partner furnishes the money capital and the other supplies the business knowledge and credit, and each shares the gains and losses equally. In other cases in which the business knowledge and experience are about equal and the monetary capital invested is unequal, an Account Current and Interest Account is kept with each partner, and the gains and losses are divided equally. In a Commercial or a General Partnership in which no agreement has been made regarding the division of gains and losses, they will be divided equally, by law. NOTE.—In the practice of accounts, it is quite common to keep two Ledger Accounts with each partner; one is called Capital Account or Stock. Account or Investment Account. To this account, all investments are credited. The other account is called Private Account or Current Account. To this account all withdrawals are charged and salary, if any, is credited. When the books are closed, the loss or gain of each partner is carried to his Private or Current Account, and then this account is closed into his Capital or Stock Account. 1386. Partnership is divided into different classes, as shown in the following synopsis: 1387. Commercial or General Partners are such as conduct any regular and lawful business for the purpose of gain. PARTNERSHIPS IN LOUISIANA. In Louisiana, the only Civil Law State in the Union, there are the following classes of Partnerships: 1. Commercial Partners. 2. Ordinary Partners. 3. Part- ners In Commendam. 4. Limited Partners. 1388. Commercial Partnership embraces the following lines of business: 1. For the purchase of any personal property and the sale thereof, either in the same state or changed by manufacture. 2. For buying or selling any personal property, as factors or brokers. 3. For carrying personal property for hire in ships or other vessels. Commercial Partnerships are divided into General and Special. (803). 8O4 SOULE's PHILOSOPHIC PRACTICAL MATHEMATICs. Yºk 1389. Every commercial partner is the accredited agent of his firm, and each member of a firm is liable to the creditors thereof, without limit. 1390. Community of Profit is the critical test to prove the existence of a part- nership. 1391. Ordinary Partners are all that are not commercial, as mechanics, farmers, lawyers, hotel-keepers, etc. Ordinary Partners are divided into Univer. Sal and Particular. 1892. Universal Partnership is one in which the parties make common stock of all the property they respectively possess, real and personal, or personal only. 1393. Particular Partnership is one formed for a special business not of a Commercial nature. 1894. Ordinary Partners are liable to the creditors of the firm pro rata, i. e. in proportion to the number of partners, without reference to the proportion of the Stock or profits to which each is entitled. 1395. Partners In Commendam. In Louisiana, a Partner In Commendam is one who contracts with some house or firm, either Commercial or Ordinary, which is already established, to invest a specified amount and to be exempt from any liability exceeding the sum or amount invested or agreed to be invested. A Partner In Commendam can take no part in the business; and the business must not be carried on in the name of such partner, though it may be used after the firm name, with the Words, Partner In Commendam. The violation of either of these conditions will make the Partner In Commendam a Commercial Partner, liable without limit. 1396, Limited Partners in Louisiana. By an Act of the Legislature of Louisiana, passed June 29, 1888, it is made lawful for any number of persons, not less than three, who shall contribute not less than $5000, to organize a corporation for the purpose of carrying on any lawful business or enterprise, not otherwise spe- cially provided for, excepting the stock jobbing business of any kind. All Limited Partners of this kind shall place the word “Limited,” as the last word of the corporate name whereveritis written or used. Failure to so use the word “Limited,” renders every member participant in such omission, liable for any indebtedness, damage or liability arising therefrom. PARTNERSEIIPS IN TELE COMMON LAW STATES. 1397. Partnerships, under the Common Law, are as follows: 1. Trading. 2. Non-Trading. 1398. Trading or Commercial Partnerships are such as are formed for buy- ing, selling, or manufacturing, or otherwise procuring for sale and selling articles for a profit. All such partners are responsible to creditors, without limit. 1399. Non-Trading Partners are those engaged in the prosecution of Some occupation or calling, not of a trading or commercial character, as lawyers, mechan- ics, physicians, etc. In Non-Trading Partnerships, a partner does not possess general power to bind the firm, and, as a general rule, has no power to contract partnership debts and issue commercial paper. * PARTNERSHIP. 8O5 1400. Trading Partnerships are sub-divided into the following classes: 1. Universal. 2. General. 3. Particular. 4. Limited. 1401. A Universal Partnership is one in which all property of every nature owned by the parties is contributed, and all profits, however made, are for joint benefit. 1402. A General Partnership is one in which the parties carry on all their trade and business for the joint benefit and profit of all, whether the investments are equal or unequal. 1403. A Particular Partnership is one formed for a special branch of trade, as distinguished from the general business or employ of the partners, or one of them. 1404. A Limited Partnership is one wherein one or more of the partners are, by the terms of the contract, liable to creditors for such an amount only as he or they shall have invested, or shall have agreed to invest in the partnership. The fol- lowing points must be observed in all partnerships of this class: 1. There must be at least one Commercial, General, or Unlimited Partner. 2. The contract of partner- ship must be duly recorded with the Clerk of the County Court, or with some officer specified by law. NOTE.—In some States, Limited Partners are not permitted to take an active part in the busi- ness, or to have their names appear in the firm name. SPECIAL NAMES GIVEN TO PARTNERS. 1405, The following special appellations are given to partners in their differ- ent relations: 1403. A Nominal or 0stensible Partner is one who holds himself out to the world as such, but who has no interest in the business; he merely lends his name and credit to the firm. All such partners are liable for the debts of the firm, although they receive none of the profits. 1407. A Silent, a Secret, a Sleeping, or a Dormant Partner is one who is not announced or known to the public as a partner, but who participates in the profits. All such partners are liable, without limit, to the creditors of the firm. 1408. A Sub-Partner is one who agrees with one of the partners to partici- pate in that partner's share of the gain, and to bear a part of his losses. 1409, Partners are responsible to one another for any violation of the articles of agreement among themselves. FORMATION OF PARTNERSHIPS. 1410. Partnerships may be formed by three different methods: 1. By Articles of Agreement formally executed and delivered. 2. By a Verbal Agreement. 3. IBy the Acts of the Parties. When a contract of partnership is evidenced by written articles, which, under all circumstances, should be done, great care should be taken to express clearly all of the provisions, stipulations and terms that the parties desire to have enforced against their co-partners. 8O6 SOULE's PHILOSOPHIC PRACTICAL MATHEMATICS. * NoTE:-The Law of Partnership is a subject of general interest. With the duties, obligations, and responsibilities of this peculiar connection, every mercantile man should be acquainted.’ It is a species of connection which pervades the mercantile world—which has existed and will exist in all ages and nations. It is a word of most extensive usage; it comprehends equally the union of a day, and those venerable guilds and ancient associations, which, in the country whence our laws havé been derived, have existed for centuries; it applies at once to the ephemeral operations of advent- urers whose projects commence and terminate in the compass of a few hours, and to the mighty transactions of that company of merchants, which, commencing with a factory on the shores of India, has overthrown kingdoms, and from their ruins cemented a splendid empire. The inducement to its formation results from the combination of qualities and the concurrence of circumstances neces- Sary to successful commercial adventure. It is a connection than which none more close and intimate can exist among men. It places in the power of him with whom you form it, your property and your reputation. His virtues and his skill, in unison with your own, may raise you to the pinnacle of prosperity. His folly, or his crimes, may strip you of every flourishing branch and leaf, and leave you a naked, withered, and dishonored trunk. It should be formed, therefore, with little less care than the marriage tie, to which it has been, by a late chancellor, not inaptly compared.— Law Journal, Vol. 1. *. DISSOLUTION OF PARTNERSEIIPS. 1411. Partnerships may be dissolved for any one of the following reasons: 1. By the expiration of the time for which the partnership was formed. 2. By the mutual consent of all the partners. 3. By death, or the physical or mental incapac ity of a partner to perform his duties in the partnership. 4. By the personal bank- ruptcy of a partner. 5. By a decree of the court for intemperance, immorality, violation of the articles of agreement, fraudulent conduct, etc. 6. By the comple- tion of the work or objects for which it was formed. 1412. The following is one of the most common forms for written instruments of Partnership Agreement, or Articles of Partnership: ARTICLES OF PARTNERSHIP, OR ARTICLES OF AGREEMENT. Articles of Partnership made and entered into this first day of January, A. D. 1893, between BATTLE BELL, N. L. BARFIELD and R. M. CLEMENT, all of the City of New Orleans and State of ‘Louisiana, Witnesseth : That the said parties hereto, having mutual confidence in each other, do hereby form with each other a Partnership Agreement on the terms and conditions following, that is to say: 1st. The partnership shall be for carrying on the Grocery, Western Produce and General Commission Business, from the first day of January, A. D. 1893, for the term of three years, thence ensuing with the condition that any party may be at liberty to dissolve the partnership at the expiration of each and every six months, from the period of its commencement, by serving upon the others, at least three months previous to such expiration, a written notice specifying that dissolu- tion will take place at such time. 2d. The said partnership shall be conducted and carried on under the partnership name, style and firm of BELL, BARFIELD & Co., and the place of business shall be in the City of New Orleans, State of Louisiana. 3d. That the said BATTLE BELL shall contribute to the capital of said partnership twenty thousand dollars legal currency. That N. L. BARFIELD shall contribute to the capital of the partnership fifteen thousand dollars legal currency. That R. M. CLEMENT shall contribute to the capital of said partnership ten thousand dollars legal currency. These investments by the respective parties to the said partnership, shall constitute the capi- tal of said partnership, for the prosecution of the said business. But each partner is entitled to draw out of the profits or capital of said business, for his own separate use, a sum not exceeding one hundred and fifty dollars per month, while the said partnership continues. 4th. It is agreed that the gains and losses shall be shared equally, i.e. one-third each by each partner; and that an interest account shall be kept with each partner. X- PARTNERSHIP. 807 5th. That each of the parties hereto shall diligently employ his time and capacity in the business of said partnership, and be faithful to the others in all transactions relating to the same; and that none will, without the written consent of the others, employ either the capital or credit of the partnership in any other than partnership business. 6th. That in all matters respecting the transactions of the partnership and the management of the business, the expressed opinion of the majority of the parties to this agreement shall govern and be binding on all of said parties; and in cases of difficulty they shall have power to settle up or sell the concern. 7th. That the books of account shall be kept according to the principles of Double Entry by the said firm, and correct entries made therein of all moneys, goods, effects, debts, sales, pur- chases, receipts, payments, and all other transactions of the said partnership. 8th. That none of the partners, during the continuance of this partnership, shall assume any liability for another or others, by means of indorsing or of becoming guarantor or surety, without first obtaining the written consent of the other parties thereto, nor shall any partner lend any of the funds of the partnership without such consent of each of the other partners. 9th. That at the expiration of each and every year, from the commencement of this partner- ship, an account of stock, effects, credits, debits, and all partnership transactions shall be taken, the books balanced, and the net gain or loss carried to the respective partners' accounts, according to article four of this agreement. 10th. That at the close of the partnership from whatever cause, the parties hereto agree to make a true, just and final account of all things, truly adjust the same, and after all the partnership business is adjusted, that they will equitably settle with each other, according to the conditions of this agreement. IN TESTIMONY WHEREOF, the parties to these presents have hereunto set their hands and seal, the day and year first above written. º - BATTLE BELL, {l. s. ; WITNESS: a-ANº. J. N. BRADFORD. N. L. BARFIELD, {L. s.3 JAMES HANCOCK. *-*N* W. B. CUNYUS. R. M. CLEMENT. { L. s.; 1413. By article four of the above Articles of Partnership Agreement, we See that although the investments by the different partners are not the Same, the gains and losses are to be shared equally, and that an interest account is to be kept. The keeping of an interest account between the partners, at the same rate per cent, is in effect equalizing the capital of the different partners; i. e., it remu- nerates the partners having the excess of capital out of the private funds, or the individual gain of the partners who have the lesser capital. 1414. The money invested by each partner is in effect a loan to the firm, and which the firm holds in trust for the protection of the creditor, and at the same time uses to carry on the business. Any other articles or clauses that the parties may desire, and which are not inconsistent with established law, may be inserted in Articles of Partnership. In all partnership contracts, each partner should have, and should carefully preserve, a copy of the Written evidence of contract. Partnership Settlements. - 2f v-v wº, ºr Tw 1415. What Quadratics is to Algebra, what Differential Calculus is to Mathe- matics, what the Supreme Court is to our System of Jurisprudence, so is Partner- ship Settlements to the Science of Accounts. By Partnership Settlements we mean: 1. The finding and the adjustment of the financial standing of partners at the close of the business, i.e., finding what is the monetary interest of net capital, or the net insolvency of each partner in the business at the time the statement is made and the adjustment thereof on the books Or among the partners. 2. The finding and the adjustment of the unsettled finan- cial affairs of business men who are not partners, but who have had business trans- actions with each other, which involve monetary debits and credits. e The adjustment of these relationships under all the multitudinous conditions which arise among business men, requires on the part of the accountant a clear mind, skill in accounts, thoroughness in practical mathematics, and an intimate acquaintance with the principles of commercial law, business customs, and com- mercial ethics. His attention should be given first, to the account current, or the monetary debit and credit standing of each partner, or of each party in interest; and secondly, to the law governing the business of the parties modified by the spe- cial contracts entered into by them. Where no specific contracts were made, and where law or business custom does not cover the case, equity must be applied, and, strict justice between man and man must be rendered. The imperative necessity for partnership settlements is of frequent occur- rence. In the regular course of business, they should be made as often as once a year. Whenever a new partner is admitted, or an old partner retires, and whenever a change of interest or share in the gains and losses of the business takes place among the partners, and at the final dissolution of the firm, partnership settlements must in justice to all parties be made. GAINS AND LOSSES. 1416. The most important question, and the first to be ascertained, in settle- ments between partners, is the determination of the gain or loss of the business. TERMS AND DEFINITIONS. 1417. The following terms and definitions are necessary to be known in Work- ing Partnership Settlements: 1418. Resources, Assets or Effects are the property of all kinds possessing value, and belonging to a firm, corporation, or individual. NotE.—The amount of withdrawals of cash or property by the proprietor or partners, is to the business or firm, a resource. 1419, Liabilities are the debts or obligations of all kinds owed by a firm, corporation, or individual. NoTE.—The amount of investment by the proprietor or partners, is to the business or firm, a liability. (808). jºr PARTNERSHIP SETTLEMENTS, 809 1420. Investment is the amount of money and property contributed to the capital of a firm, or appropriated for the transaction of business on private account. 1421. Net Investment is the excess of investments over the withdrawals; or in case of no withdrawals, it is the total investment. 1422. Average Investment is a sum which, invested for the whole term of the business, will produce an amount of interest equal to the interest on the different actual investments for the time that they were respectively in the business. 1423. Capital is the total value of the resources of a firm or individual. 1424. Net Capital, or Present Worth is the excess of the resources over the liabilities at the close of business, or at any time during business. NOTE.-The net investment of the proprietor or partners, is not to be included in the liabilities when the net capital is to be found. 1425. Insolvency is the indebtedness of a firm or individual in excess of the I’éSOULI'CeS. NOTE.—The withdrawals by the proprietor or partners are not to be included in the resources when the insolvency is to be found. 1426. Net Gain is the excess of the resources over the liabilities, including the withdrawals and investments by the proprietor or partners. 1427. Net LOSS is the excess of the liabilities over the resources including the withdrawals and investments by the proprietor or partners. 1428. The Net Gain or Net LOSS is also the difference between the total gain and total loss. TWO METEHODS OF DETERMINING THE GAIN OR LOSS OF A BUSINESS. 1429. There are two methods of determining the gain or loss at the close of a business, or at any time during the business. They are as follows: 1st. To collect and classify all the various items of gain and loss, and then find the difference between their respective sums. This method can only be used when the books have been correctly kept by Double Entry. 2d. To collect and classify all the various items of resources and liabilities of the firm, and then find the difference between their réspective sums or aggregates. This difference is a gain, when the resources exceed the liabilities, and a loss, when the liabilities exceed the resources. This fact is clear, for the reason that when the business was commenced, no matter what was the net capital or net insolvency, the resources and liabilities were in balance, and it is obvious to the accountant that nothing but a gain or a loss could destroy that balance. N. B. When classifying the items of the resources and liabilities, to find the gain or loss of a business as above directed, include as resources the withdrawals of the proprietor or partners, and include as liabilities, the investments of the proprie- tor or partners. NOTE.-Instead of including both the investments and withdrawals of the partners in the list of resources and liabilities, the net investment or net withdrawal may be used, thus abridging the work. 1430. The principles of Accounts upon which this is based, are as follows : 1st. That the firm owes each partner for what he invests, the same as it would OWe any other person for money or property deposited with the firm 8 IO soul E's PHILOSOPHIC PRACTICAL MATHEMATICs. Yºr 2d. That each partner owes the firm of which he is a member, for all with- drawals, the same as any other person would owe the firm who received money or goods from it. NOTE: 1.--To the critical accountant, there is a clear distinction between the firm itself and the members composing it. NOTE: 2.—In the case of a single proprietorship business, it is considered that the business owes the proprietor for his investments, and that he owes the business for his withdrawals. Thus a single proprietor of a business possesses two individualities, one as an individual, and one as a business man. The second method of determining the gain or loss, as above described, can be used under all circumstances where it is possible to find the resources and lia- bilities, and it is the only method that can be used when the books have been kept by Single Entry, or have been Incorrectly kept by Double Entry, or when no books have been kept—only memorandums have been made. 1431. ITEMS OF GAIN AND LOSS NOT TO BE INCLUDED IN THE STATE- MENT OF RESOURCES AND LIABILITIES. When making the statement of resources and liabilities, care must be taken not to include any items of gain or loss. All of the various items of gain and loss are represented in the list of resources and liabilities. This is evident from the fact 1st, that a loss is the result of a resource disposed of, without receiving property value therefor, and hence is absent from the list of resources, and thus the amount of resources is decreased and the loss is correspondingly increased. 2d, that a gain is the result of a resource received, without giving property value therefor, and hence is present to increase the list of resources, and thereby augments the gain. 1432. ORAI, JEXERCISES. A. commenced business with a capital of $8000, and closed with an insolvency of $1500. What was his loss? Ans. $9500. B. commenced business with an insolvency of $3000, and closed with a capital of $2000. What was his gain? Ans. $5000. C. commenced business with a capital of $2000, and closed with a capital of $400. What was his loss? Ans. $1600. D. commenced business with a capital of $4000, and gained $1400 during the business. What was his capital at closing? Ans. $5400. E. commenced business with an insolvency of $3500, and gained $1500 during the year. What was his indebtedness at the close of the year? Ans. $2000. F. commenced business with a capital of $6200. He lost during business $2100. What was his capital at closing? Ans. $4100. G. closed business with an insolvency of $400. He gained $1800 during busi- ness. What was his insolvency at commencing % Ans. $2200. H. commenced business with an insolvency of $1700. Gained during the year $2500. What was his capital at closing? Ans. $800. 1433. WEITTEN PROBLEMIS. 1. A. and B. are partners equal in gains and losses. At the close of business A. has a net credit of $6500, and B. a net credit of $3500. There is no property on hand, and no other liabilities. What settlement should they make? Ans. B. must pay A. $1500 out of his private funds. * PARTNERSHIP SETTLEMENTS. 8II OPERATION. Resources. I Liabilities. i IXEBIT. CREDIT. $ 6500 * & 3500 A’s credit $6500–$5000 loss= - - - - - - $1500 #) $10000 loss. B's loss $5000–$3500 credit= - - $1500 $ 5000–A’s 3 loss. 5000–B's # loss. * Explanation.—Considering that all investments of partners are liabilities of the firm, as shown above, and that these investments amount to $10000, with no resources to offset them, it is there- fore clear that there has been a loss of this amount, which must be shared by the members of the firm as agreed in their articles of partnership. See Articles 45 and 46. 2. A. and B. are partners, equal in gains and losses. At the close of the busi- ness, A. has a net debit of $6500, and B. a net debit of $3500. There are no other resources and no liabilities. What settlement should they make? Ans. A. must pay to B. $1500 out of his private funds. OPERATION. W Resources. Liabilities. DEBIT. CREDIT, $6500 A's debit $6500–$5000 gain = - $1500 3500 B's gain $5000–$3500 debit = - - - - - $1500 2)$10000 $5000 = A's net gain. $5000 = B's net gain. Explanation.—Since, as above shown, the withdrawals of the partners are to the firm a resource, and since the resources of the firm are $10000, with no liabilities, it is therefore clear that there has 'been a gain to the firm of $10000, which is to be shared by the partners according to the agreement in the articles of partnership. 3. A. and B. are partners, equal in gains and losses. A. has a net credit of $6500 and B. a net debit of $3500. There are no other resources or liabilities. What settlement should they make? Ans. B. must pay to A. $5000 out of his private funds. OPERATION. Resources. | Liabilities. DEBIT. CREDIT. $3500 $6500 a' 3500 * A’s credit $6500–$1500 loss = - - - - - $5000 $)$3000 loss. B's debit $3500 + $1500 loss = - $5000 $1500 = A’s loss. $1500 = B’s loss. Explanation.—In this problem, one partner owes his firm, and the firm owes one of its partners, The excess of the liabilities over the resources is $3000, which is the loss of the firm, and which must be shared according to agreement, equally. The operation shows clearly, that B. must pay to his partner $5000. 4. C. A. Hines and J. L. Hall are partners. Hines ; and Hall 4 in gains and losses. During the business year C. A. Hines invested $24000 and drew out $1700. J. L. Hall invested $6000 and drew out $900. They have Mdse. $32000, Cash $4000, Bills Receivable $5000, Personal Accounts $11000, Bank Stock $2000. They owe on Bills Payable $3000, and on Personal Accounts $6000. They gained on Mdse. $23308, on Interest $162, and on Bank Stock $320. Expense shows a loss of $5800, Insurance a loss of $360, and Exchange a loss of $30. What is the net gain or loss, and what is the net capital of each partner? Ans. $17600 net gain. $35500 C. A. Hines' net capital. $9500 J. L. Hall's net capital. 8 I 2 SOULE's PHILOSOPHIC PRACTICAL MATHEMATICs. * & Operation to find gain Operation to find gain by the Resource by the Loss and and Liability Method. Gain Method. *º. Liabilities. se 2000 C. A. Hines $22300 ‘ºols; 4000 || J. L. Hall 5100 360 | 162 #. § 30 320 2000 $6190 $23790 Resources $54000 $36400 Losses | 6190 Liabilities 36400 Net gain $17600 $17600 = net gain. PARTNERS’ NET CAPITAL. C. A. Hines' # net gain ($17600) is - - $13200 J. L. Hall's 3 net gain ($17600) is - - - $4400 do net investment - - - - 22300 do net investment - - - - 5100 do net capital at closing - - $35500 do net capital at closing - - $9500 IExplanation.—The operations of this problem and the general statements and directions above given, regarding the two methods of finding the gain or loss of a business, make it clear without repeating general principles. 5. A., B. and C. are partners, A. #, and B. and C. each #, in gains and losses. At the close of business, their accounts stand as follows: A. is debited $3000 and credited with $15000; B. is debited with $6000 and C. is credited with $10000. They have cash in bank $28000. There are no other resources or liabilities. What is the financial standing of each partner, and what settlement should be made? Ans. B. will pay $3000 from his private funds. A. will receive $18000 and C. will receive $13000. OPERATION. Resources. Liabilities. B’s net withdrawal - gº - $ 6000 A’s net investment - gº se sº $12000 Cash - * - tº e Gº * - 28000 C’s net investment - wº- s tºº 10000 Total resources gº e tº – $34000 $22000 Total liabilities - gº º sº - 22000 Net gain me tº-º * * º - $12000 A’s 3 net gain is - - - $ 6000 || B's 4 net gain is - - - - $3000 C's # net gain is - - - $ 3000 A’s net investment is - - 12000 || B's net withdrawal is - - 6000 | C's net investment is - 10000 A's net capital is - - - $18000 | B's net insolvency is - - $3000 | C's net capital is - - $13000 Eacplanation.—From these figures, it is clear that B. must pay $3000 out of his private funds, which, with the $28000 cash in bank, will make $31000; of which A. will receive $18000 and C. $13000. 6. Suppose, in the preceding problem, that B. has no private property and proposes to give A. and C. each, his note. What should be the face of the note given to each 3 Ans. A. $1741.94. C. $1258.06. OPERATION. First divide the $28000 cash between A. and C. in proportion to their respective net capitals, thus: A’s Share. C’s Share. A’s capital - - - - - $18000 $28000 00 * $28000 00 C’s capital - - - - - 13000 $31000 | 18000 $31000 || 13000 A's and C's Capital - - $31000 $16258 06 $11741 94 A’s net capital - sº º sº tºº $18000 00 C's net capital º $º º tº $13000 00 A’s proportion of cash - - - 16258 06 || C's proportion of cash - - - 11741 94 A's face of note - alº dº jº tº 3 $ 1741 94 | C’s face of note | tº ſº tºº $ 1258 06 X- PARTNERSHIP SETTLEMENTS. 813 7. A. and B. were commercial partners, equal in gains and losses; no interest to be allowed on investments. October, 1892, A. invested $8000, and B. $5000; May 1st, 1893, the firm was dissolved, and from an incorrectly kept set of books, the fol- lowing facts and figures were taken by the partners and submitted for settlement: A. has a net debit of $2100; B. a net credit of $3500; cash on hand, #349; 10 shares of Bank Stock, market value $1100; Expense Account is debited $5100; Profit and Loss is debited $3000 and credited $500; the firm owe on a note $2000, and interest on same $60. Approved, #. Make, in practical accounting form, the statement showing the correct settle- ment of the partnership affairs of this firm. STATEMENT SETTLEMENT. STATEMENT SHOWING THE RESOURCES AND LIABILITIES, THE NET GAIN, THE CON- DITION OF THE PARTNERS’ ACCOUNTS AND THE FINAL SETTLEMENT,9. THE COMMERCIAL FIRM OF A. & B., BASED UPON A STATEMENT OF FACTS APPROVED BY THEM MAY 1, 1893. ſ RESOURCES : A's net withdrawal tº- tº- - º - - - - sº #% º Cash on hand - - - º - - - - -- - tº- 1100|00 Bank Stock, market value - - wº- - - -*. - * | - Total amount of resources - wº- e º - º 6600|00 LIABILITIES : 500|00 B's net investment - * - *- - * - - gº 3500 Bills Payable - - - - - - - - - face $2000 } 206000 Int, due - * - 60 Total amount of liabilities - - - - - - - 5560|00 Net gain of the firm - - - - * - - * 1040|00 DIVISION OF GAIN : 0 A's # net gain - e- - *- - gº - - *- tº- - 5200 B’s net § - - - * - - - * * * º 520,00|| 1040 00 A A’S NET INSOLVENCY : A's net withdrawal - - - s - * - - &º 2100.00 A's # net gain, deducted - - - - - eºs tº - º 52000 A's present net insolvency - - tº- - - - - - 1580|00 B’s NET CAPITAL : B’s net investment - - - º - - - - - 350000 B's # net gain added - * - tº * * * * * 52000 B's present net capital - - - - - sº -> - - 402000 Net capital of firm am me me - ºne as º º _|_2440 00 FINAL SETTLEMENT : - In final settlement, A. must pay to B., out of his private funds, or give his individual note for - - - tº ºs º ºs I58000 B. must receive the cash - - - " - -. * tº- 340000 and the bank stock - - - - - - - - t- 110000 making a total of - - - * - tº m ºn as 6080|00 B. must then pay the note - - - - - - - - 200000 and interest on same * - * - sº - - - 6000|| 206000 B. will then have, as above, a net capital of * * * * 4020|00 NEw ÖRLEANs, May 1, 1893. GEO, SOULÉ, Accountant. 814 SOULE's PHILOSOPHIC PRACTICAL MATHEMATICs. * 8. The following is a Trial Balance of a Double Entry Ledger, and the inventory of stock at the close of the fiscal year, July 1, 1893: TRIAL BALANCE. E. P. Heald, (partner) - - - - - - - - - 2050,00|| 15500|00 A. Harris, (partner) gº - º - - - - tº- 2550|00|| 480000 Cash e tº- tº- - - as ºn tº tº- - fº º- 18567,00|| 16892|00 Mdse. tº - - - - sm as tº *E - - -> 6972|00|| 585250 J. M. Swoop - - - - - - - - - - - 40000|| 900|00 W. Voges * * * * * * * * * * = 967|50 Expense - - - * - ºn tº -> º - - - 52000 Profit and Loss º sº - * - º * , - º 4- 6743|00 City Railroad Stock 4- - ºs ºs º- tº º -> - 5500|00|| 12:500 Commission - - - - - - - - - - - 20000 | 44269||50|| 44269|50 Inventory of property unsold: Merchandise $680.40, Railroad Stock $5450. NoTE.—The gains and losses are divided as follows: E. P. Heald two-thirds, and A. Harris one-third. 9. Required, the financial standing of the partners by the two methods of determining gains and losses. - FIRST OPERATION BY THE RESOURCE AND LIABILITY METHOD. STATEMENT OR BALANCE SHEET SHOWING THE RESOURCES AND LIABILITIES, THE NET LOSS OF THE FIRM, THE NET CAPITAL OF E. P. HEALD, AND THE NET INSOLVENCY OF A. HARRIS, JULY 1, 1893. W RESOURCES : Mdse. as per inventory - - - - - º - º sº 680|40 Railroad Stock, as per inventory - - - e - - sº 545000 Cash on hand - - * - * tº- - * - º ſº 1675|00 Personal Accounts, Dr. - - º- - - * - - º 967|50 Total amount of resources – - - º - º *- 87.7290 LIABILITIES : E. P. Heald's net investment sº tº- - tº - sº &= 13450|00 Archie Harris’ net investmentſ tº- -, - * : *- tº gº 2250|00 Personal Accounts, Cr. - - - - - * * - ſº - 50000||. Total amount of liabilities - - - g - - sº *- 16200|00 Net loss of business - * - - is sº - º - - 7427|10 PARTNERS’ PRESENT NET CAPITAL: E. P. Heald's net Investment º - - tº- - º º 13450|00 E. P. Heald’s two-thirds net loss - - - s - - º 4951|40 E. P. Heald's net capital at closing - - - - - - 8498.60 Archie Harris’ net investment •ºme º- ſºy dºg º tºp * -º 225000 Archie Harris’ one-third loss - º * > - º º º ſº 2475||70 Archie Harris' net insolvency at closing * - - - - 22570 ºf PARTNERSHIP. SETTLEMENTS. - 815 SECOND OPERATION BY THE Loss AND GAIN METHOD. MERCHANDISE. RAILROAD STOCK. PROFIT AND LOSS. Cost $6972|Sales $5852.50|Cost $5500||Dividend $ 125 Net Debut $6743.00|$ 75 gain R. R. Stock Invent. 680.40|P. & L. 75|Invent. 5450 $5575 $5575 Loss on Mdse. 439.10 P. & L. 439.10 i Loss by Exp. 520.00, 200 gain on Com. $6972 $6972.00 Total loss - - $7702.10|$275 total gain. 275.00 $7427.10 net loss. E. P. Heald’s two-thirds net loss is $ 4951 40 A. Harris' one-third net loss is - - $2475 70 do net investment is - – 13450 00 do net investment is - - - 2250 00 do net capital is - - - $8498 60 do net insolvency is - - - $ 225 70 10. Two physicians, Jones and Smith, formed a partnership for one year, ‘equal in gains and losses. They opened two offices, one in the upper and one in the lower part of their city. Jones took charge of the upper office and had a practice amounting to $4510; of this amount he collected $3600, leaving $910 outstanding. The expenses of his office were $1100, which he paid out of the money collected. He also paid $300 from the money collected on account of the expenses contracted by his partner for the lower office. - Smith took charge of the lower office and had a practice to the amount of $3812; of this sum he collected and used personally, $3050. He also collected and paid on account of expenses $400, leaving $362 outstanding, of which $200 is worth- less. He contracted expenses amounting to $1641, and gave the firm's note on account of expenses for $350 which is still unpaid. At the close of the year Smith retires and Jones continues the practice. They engaged an accountant to make a correct statement showing how they stood financially with their business, and with each other. How should that statement be made 3 Answer, as follows: FIRST STATEMENT. :STATEMENT OF SETTLEMENT OF THE PARTNERSHIP AFFAIRS OF JONES & SMITH. *– Amount earned by the practice of Jones - - - - - 4510|00 Amount earned by the practice of Smith - - - $3812 00 Less worthless accounts º - - - - 200 00 3612|00 Total amount earned - - - - - - - - 81.2200 Expenses of Jones' office - - - - - - - - 1100|00 * “ Smith's office - - - - - - - - 164100 Total expenses - - - - - - - - - 2741|00 Net gain or revenue - - • * * *- - - 5381|00 Jones' one-half net gain - -: º * - --> * - wº- 263050 Smith's one-half net gain * - - - - - - * 2690|50 5381|00|| 5381|00 JONES' ACCOUNT. T * By one-half net gain * -* - - - *- *- g- sº 2690/50 To collections -> º - - * - - - - - 3600|00 Less amount paid for expenses $1100+300 = - - - tº- 140000|| 2200.00 Balance due Jones - - - - - - - - - - || _|_49050 SMITH's ACCOUNT. - *- By one-half het gain * * * * * * * * * 2690|50 To collections used personally * * * * * tº º 3050|00 35950 l i_Balance due by Smith to Jones - - - - - - - (continued.) • ' - SOULE's PHILOSOPHIC PRACTICAL MATHEMATICS. (Continued from the preceding page.) RESOURCES : Personal accounts due on Jones' practice Personal accounts due on Smith's practice Less worthless accounts Smith's indebtedness as above Total resources of the firm LLABILITIES OF FIRM : As shown above the total expenses were Of this amount there was paid by Jones By Smith $400 cash, $350 note = Total expenses paid Balance due for expenses Face of note due Total amount due by the firm Net capital of firm (due to Jones) FINAL SETTLEMENT. In settlement, Smith must pay to Jones from his indebtedness as above Jones will collect the resources due, whi thus placing in his hands From this sum we will pay th and the note outstanding Total amount to be paid by Jones Balance in his possession is which is his met capital as above. Personal accounts due Jones' office Personal accounts due Smith $362 00 Less worthless accounts Jones' withdrawals - Total resources Total liabilities Net gain - - $1100+300= ch are e balance due on expenses 910|00 162|00 107200 359|50 1431|50, 2741|00 2150 591 350 00 00 00 941 490 s private funds, his º 359150 1072100 1431/50, gº - - 591 350 º sº 941|00, 490 SECOND OPERATION TO FIND THE GAIN. RESOURCES : 910 00 $ 200 00 162 00 2200 00 3050 00 $6322 00 941 00 $5381 00 é & - º g- º LIABILITIES : $ 591 00. 350 00' Due for expenses Due for note - Total liabilities $ 941 00: Having the gain, the statement settlement would be made as shown above- ¥ - * PARTNERSHIP SETTLEMENTS. 8 I 7 11. The following facts and figures are submitted by the members of a com- mercial firm for adjustment: Jones and Smith are commercial partners. January 1, 1892, Jones Invested $20000, and Smith $5000. The business was continued until January 1, 1893. During the business, each partner devoted his services to the business, and neither drew out, nor added to, his capital. There was no agreement regarding their respective shares of gain or loss. January 1, 1893, a dissolution took place, and after the goods were sold, the liabilities to third parties paid, and the resources col- lected, there was cash in bank $35000. According to law, custom, and equity, what is the amount due each partner? STATEMENT OF SETTLEMENT. STATEMENT OF THE RESOURCES AND LIABILITIES, NET GAIN, INTEREST ON INVEST- MENTS, AND THE PARTNERS’ NET CAPITAL OF THE COMMERCIAL FIRM OF JONES & SMITH, JANUARY 1, 1893: RESOURCES : Cash on hand - &=} * * tº Eº tº ſº * ge gº & 3500000 LIABILITIES : Jones' net investment - * * gº * * ſº º tº º º º 2000000 { Smith’s net investment - - - - - - - - - 5000|00 Total liabilities † : gº {-> is ſº tº º gºe tº 2500000 Net gain of the firm - - - - - - - - - 1000000 Jones’ one-half net gain - - - - - - - - - 500000 Smith's one-half net gain * * * * * me sº * 500000 10000|00|| 10000 00. PARTNERS’ INTEREST ACCOUNT : Jones' investment $20000. 1 yr. (a) 8% tº gº tºe tº sº 1600|00 Smith's investment $5000. 1 yr. (a) 8% tº sº tº dº * > 400|00 Balance of interest favor of Jones & tº º tº tº a gºs 1200.00, Smith's one-half debit of interest is gº º tº tº † - * : 600|00 Jones' one-half credit of interest is tº e * > & ºt tºº ; : 600|00i 1200 00 1200.00 JONES' NET CAPITAL AT CLOSING: Jones’ net investment as above tºº gº wº gº * * tº º 20000|00 “ one-half net gain as above - tº tº gº tº ſº tº 500000 ‘‘ one-half credit of interest - gº tº sº e gº ſº 60000 Jones’ present net capital gº º ºs is sº º sº tº 2560000 * SMITH's NET CAPITAL AT CLOSING: Smith’s net investment as above - s gº º * gº * 500000 “ one-half gain as above * º sº tº tº dº tº 500000 “ total investment and gain tº-3 tº sº tº ſº. * - 10000|00 & 4 one-half debit of interest deducted tº a * > sº * 600|00 “ present net capital - - - - - - - - 940000 Total net capital of firm at closing as ºn tº º ºs 3500000 NEW ORLEANs, January 1, 1893. GEO. SOULF, Accountant. NOTE.—According to law and custom, commercial partners, in the absence of any agreement pertaining to the division of the gains and losses, share the same equally. And in accordance with equity, if no agreement is made to the contrary, interest is allowed on the investments and with- drawals of each partner. * s 3.18 soul E's PHILosophic PRACTICAL MATHEMATICs. ¥ sECOND FORMULA of statement. RESOURCES : Cash on hand - sº - * sº- º - º - º - 3500000 LIABILITIES : Jones’ net investment - A- g- tº --> º - $20000 Interest on same, one year at 8% - - - tº ºr 1600 Jones' total investment - - º ºn tº e de - ºn 21600|00 Smith's net investment - - - - - - - $5000 “ interest on same, one year at 8% * * * 400 “ total investment tº º cº- ºn º tº gº 540000|| 2700000 Net gain of the firm, allowing interest on investments - tº 8000|00 Jones’ one-half net gåin - * - - tº tº - º 4000|00 Smith's one-half net gain - sº ºn - sº tº - * 400000 8000|00|| 8000|00 PARTNERS’ NET CAPITAL AT CLOSING: Jones’ net investment as above º gº - - - $21600 “ one-half net gain - º º - - - 4000 “ present net capital º e- º- - tº gº - E-3 25600|00 Smith's net investment as above - - - as sº $5400 “ one-half net gain * tº ºs - tº g- 4000 “ present net capital - * - sº- - - ºn 900 00 Total net capital of firm at closing - * =y - º $3500000 12. The following statement was received from a book-keeper: “Adams and Lee were equal partners in business, the amount of capital was $2500 each. By bad management the business was unsuccessful, and on closing their books, they found that the money and goods on hand would just pay cer- tain debts which they owed in New Orleans. This being done, the firm had no debts due them nor did they owe any. But each partner had a special cash account on the books, as each had had, from time to time, small amounts placed to his credit, which amounts were used for the benefit of the firm, and on closing the books, Adams had a cash balance due him of $800, and Lee a balance of $200. They now desire to settle this between themselves by a note, and asked my advice, and I proposed this settlement: that Adams is to give his note to Lee for $200, and Lee to give his note to Adams for $800, or to make one note from Lee to Adams for $600. I am not positive that I am right, and so refer the matter to higher authority and request your decision whether or not I am correct.” Is the solution of the book-keeper correct? - Ans. No. Lee should give Adams his note for $300. See problem 1, 13. The following statement was received from a book-keeper: “Will you be kind enough to explain to me how to close my books, the Trial Balance of which I hand you herewith ? “The merchandise on hand as per Stock Book is $38116.23. Please also make statement showing gain of business.” (Continued.) Y. PARTNERSHIP SETTLEMENTS. 819, The following is the TRIAL BALANCE: Cash on hand - - - - - - $ 443 49 Stock - - - - - - - - - $24012 52" Mdse. as per Ledger - - - - 14232 01 Bills Payable - - - - - - - 4728 24 Profit and Loss - - - - - 22187.87 Sundry accounts - - - - - - 18642 64 Sundry accounts - - - - - 10520 03 $47383 40 $47383 40 SOLUTION. a Statement of the resources and liabilities and the net gain of the commercial firm of , Galveston, Tex., August 1, 1893: RESOURCES : Cash on hand - - - - - - - - was tº gº 443/49 Mdse. as per account stock - - * & º g- s gº 38116123 Sundry personal accounts gº *E= †-8 * £ºs gº º *-. 1052003 Total resources of firm - gº ame tº ſº tº sº tº- 4907975 LIABILITIES : Stock—net investment - gº * sº &_i ſº º -> wº 24012|52 Bills payable - # = *- * > tºº º tº i- * > -> sº 472824 Sundry personal accounts - - - - - tº gº tº 18642|64 Total liabilities of firm - - - - - * , sº gº 4.738340 Net gain of firm i º s gº tº º ºs is tº tº º 1696.35. To close the books, first credit Mdse. account with the stock on hand, and then close the account to profit and loss; then, there being, as per statement submitted, no other loss and gain account, close profit and loss to stock account. Then close stock account and all other open accounts to or by balance; then rule and add the accounts, bring the balances down and take a trial balance of the balances. 14. The following statement and question were presented by a book-keeper: “A., B., C. and D. are partners, equal in gains and losses. During the time business was continued, A. drew $260; B. drew $155; C. drew $180; and D. drew $10. There is due A. for rent $150; to B. for services $150; to C. for services $240, and to D. for services $120. º, “The net gain of the business independent of salaries due was $1860. What is the amount due each partner?” Ans. A. $190, O. $360. B. 295, D. 410. OPERATION. Net gain, per statement, is - tº- tº-8 gº sº tº gº sº $1860 00 Less amount due for rent and salaries º tº asp - sº 660 00 Amount to be equally divided - --> &= tºº tºº tº sº 1200 00 Each partner's share of gain tº e ºs º º tº ºs 300 00 GAINS. WITHDRAWALS. BALANCE, RENTS & SALARIES. AMT. D.U.E. A. $300.00 — $260.00 == $ 40.00 + $150.00 F. $190.00 A. B. 300.00 — 155.00 - 145.00 + 150.00 F: 295.00 B. C. 300.00 — 180.00 F. 120.00 + 240.00 – 360.00 C. D. 300.00 — 10.00 F. 290.00 + 120.00 = 410.00 D. 15. The following statement of facts occurred with two joint owners of a. steamboat and was submitted for settlement as per conditions named below. “J. Jones owns five-eighths of the steamboat Star Light, and S. Smith owns three-eighths of said steamer. The boat is valued at $16000. The boat. 82O soul E's PHILOSOPHIC PRACTICAL MATHEMATICs. Yº owes liabilities amounting to $6324.28; $3490.13 of the liabilities are owing to J. Jones, one of the joint owners. The remainder of the liabilities are due to various parties and amount to $2834.15. “J. Jones holds a note of S. Smith for $1000, now due. “It is agreed that one of the joint owners shall buy the other's interest, and they therefore desire a statement showing how much Jones should pay Smith should he purchase his interest, and how much Smith should pay Jones should he purchase his interest in the boat.” What is the statement to be made 3 NOTE.—Joint owners of steamboats are responsible as to themselves for the liabilities of the boat in proportion to their respective interests in the boat. f STATEMENT BASED UPON THE AIBOVE FACTS SHOWING AMOUNT DUE TO EITHER OWNER OF THE STEAMBOAT STAR LIGHT BY THE OTHER IN CASE EITHER SHOULD PURCHASE OF THE OTHER HIS INTEREST IN THE SAID BOAT AND ASSUME ALL LIABILITIES TO THIRD PATRTIES. 1. Steamboat Star Light, valued at - *- :- º gº * - * 16000|00 J. Jones' five-eighths interest is - gº tº -- «º sº * -º 10000|00 S. Smith's three-eighths interest is gº º •º - ºg wº 6000|00 & 1600000|| 1600000 - 2. - Liabilities of the boat are - " - * - ſº tº a *g, * sº 6324|28 J. Jones' five-eighths of liabilities are sº &- tº- tº e- 3952|68 S. Smith's three-eighths of liabilities are - - - - - 2371|60 632428|| 632428 3. Should J. Jones buy S. Smith's interest in the boat he would pay as follows: For S. Smith's three-eighths interest - - tº º qº -> 600000 For his, (J. Jones') five-eighths of liabilities *E* sº tº gº 3952|68 Total amount to be paid & tº º º sº tºº tº a 9952|68 Less the amount advanced by him and included in the liabilities 3490|13 Less S. Smith's note held by him as above - sº tº- <= * 100000 Total amount of deductions - tº º gº tº s s 4490|13 Net amount to be paid by J. Jones {-> tº tº ºs tº-ºº: gºs 5462 55 * Liabilities to third parties assumed by him - sº tº tº gº 2834|15 Net amount to be paid to S. Smith & sº - º sº g-r 2628|40 4. Should S. Smith buy J. Jones’ interest in the boat he would pay as follows: * For J. Jones' five-eighths interest - - - - - - 10000|00 For his, (S. Smith's) three-eighths of liabilities * = s gº 2371|60 For his, (S. Smith's) note held by J. Jones * * tº sº 1000|00 Total amount to be paid by S. Smith - sº º sº gº 13371/60 Less Liabilities due third parties gº * = * - sº * * º 2834.15 Net amount to be paid to J. Jones tº º tº tº º * = º 10537 45 16. The following question was received from a book-keeper: “Messrs. A. and B. entered into partnership, share and share alike in the gains and losses. They agreed to adjust their current accounts at the end of each year, so that each shall draw equally. Now, at the close of the first year, the debit of x- PARTNERSHIP SETTLEMENTS. 82 I A's current account is $627.24 in excess of B's. What entry should be made to close their current accounts?” Answer as follows: B. must draw from the firm $627.24, or A. must pay to B., out of his private funds, one-half of the $627.24 which he has drawn in excess. In case B. draws the $627.24, the entry would be, B. to Cash for the amount. In case A. paid B. one-half of his excess of withdrawals, the entry would be made thus: B. - - - - - - - - $313 62 To A. - - - - - - - - $313 62 In case it is desired that A. shall pay B., and in case A. has no private funds and is obliged to draw the necessary amount from the firm to equalize the current accounts, the entries would be as follows: FIRST. A. - - - - - - - - $627 24 To Cash -> -> º cº- sº º sº e- $627 24 SECOND. - - - - - - - - $627 24 To A. - - - - - - - - - $627 24 In place of the two entries above, one resultant entry may be made thus: - - - - - - - - $627 24 To Cash e * -º º * * sº - $627 24 With proper explanations. 17. Smith and Brown have 42000 logs to drive down a certain river. Of this number, 15000 have been taken for other parties at the agreed rate of $6 per 100. Whatever the balance costs to drive, they are to pay in proportion to the amount of logs each of them owns. Smith has 12000 logs and Brown 15000. Smith pays in wages and supplies $900; Brown pays in wages and supplies $800; Smith collected $600 for 10000 logs at $6 per 100; Brown collects $300 for 5000 logs at $6 per 100; Smith's books show the account of Smith and Brown charged with $900 and credited with $600. The question is: What further Journal Entry or entries has Smith to make to close this account and show his cost or expense of driving his 12000 logs? What has he to pay Brown, or what to receive from Brown, as the case may be 3 NOTE.-The above problem was submitted by a correspondent to the readers of “BUSINESS.” The following is our solution. OPERATION : 1. The expense of driving the 42000 logs was $1700, of which Smith paid $900 and Brown $800. We therefore credit Smith and Brown respectively, with the amount paid. Dr. Smith. Cr. Dr. Brown. Cr. $600 00 $900 00 $300 00 || $800 00 355 56 444 44 2. Of the $900 collected for driving the logs of third parties, Smith collected $600 and Brown $300. We debit each for the amount he collected. 3. By deducting the $900 received for driving the logs belonging to third parties from the $1700, the cost of shipping 42000 logs, there is a balance of $800, which, by the terms of the problem, is to be borne by Smith and Brown, in proportion to the logs owned by them respectively. OPERATION TO FIND THE AMOUNT EACH OWIES OF THE EXPENSE. Smith 12000 logs. Smith. Brown. Brown 15000 “ $ 800 00 $ 800 00 27000 || 12000 00 27000 || 15000 00 27000 $ 355 56 Tº IIIAI 822 SOULE's PHILOSOPHIC PRACTICAL MATHEMATICs. * 4. We now charge each in the above accounts with the amount he owes for expense. The accounts then show that Smith owes Brown $55.56. When this is paid, Brown will be debited and Smith credited. Smith had debited in his own books, the account of Smith & Brown with the $900 paid by him, and credited the same account with the $600 collected. The entries for these transactions were as follows: 1. Smith & Brown *-*. tº- * $900 00 To Smith (representing the cash or property paid out) - $900 00 2. Smith (representing the cash received) - T - - $600 To Smith & Brown tº- ©º tº- 4- º sºs tº- tº- - $600 00 These entries produce the following accounts: Dr. Smith & Brown. Cr. Dr. Smith. Cr. $900 00 || $600 00 $600 00 $900 00 NOTE.—To close these accounts, Smith is to credit Smith & Brown with $355.56, which is the i. it cost to ship his logs, and to debit Smith & Brown with $55.56, the amount he paid to. I’OWIl. The following journal entries should be made to effect the object: 1. Log Expense (or Smith) - - - - - - $355 56 To Smith & Brown *- sº gº sº *- tº-º sº - $355 56 2. Smith & Brown *s º * * * «-» º $ 55 56 To Smith (representing cash) 4.- º gº - - - $ 55 56 When these entries are posted, the accounts will stand as follows: Dr. Smith & Brown. Cr. Dr. Smith. Cr. Dr. Log Expense. Cr. $900 00 || $ 600 00 $600 00 $900 00 $355 56 55 56 355 56 55 56 $955 56|s 95556 By closing Log Expense to Smith, will produce a balance of both accounts. NOTE 1.-It will be observed that the account of Brown in the first part of the statement. does not belong to the books of Smith. NOTE 2.-If it is preferred, the debit item of Log Expense may be charged directly to Smith. 18. Jones, a planter, agreed with his neighbor, Smith, to manufacture shin- gles from timber furnished by Smith, and to receive for his services and the use of machinery, one-fourth of the number manufactured. He manufactured and delivered to Smith 300,000 shingles. How many must be manufactured for himself to pay him according to contract 3 Ans. 100,000 shingles. FIRST OPERATION. 1,–4 =#. Then 300,000 is three-fourths of the number to be manufactured; and since three- fourths of the number is 300,000, one-fourth of the number would be the third part, which is 100,000. SECOND OPERATION. According to the terms of the contract 1 = the whole number manufactured. and + = “ number paid for manufacturing. and the difference, #- “ number manufactured for Smith. Hence the 300,000 = # of the number to be manufactured and conversely # = 300,000; then since # = 300,000 + = the third part which is 100,000 and 4 or the whole number = 4 times as much §." 400,000, # of which is 100,000, the number to be manufactured to pay for manufacturing 5 e * 19. A merchant received from a planter 500,000 pounds of seed cotton to gin and bale. He 1s to receive for the service is of the seed cotton ginned and baled. Yºr PARTNERSHIP SETTLEMENTS. 823 How many of the 500,000 pounds of seed cotton received must the merchant gin for the planter? Ans. 454545+ pounds seed cotton to gin for the planter. OPERATION. 1 or 100 = assumed pounds to gin. To or 10 = pounds due for ginning. 175 or 110 = pounds of seed cotton required to gin 1 or 100 pounds of seed cotton. Then: 11%; : 1 :: 500,000 : x or, 110 : 100:: 500,000: × ; or thus: | 1 100 11 || 10 110 || 500,000 500,000 — 454545 ºr fås. 454545 ºr fös. Ans. 20. A father wishes to divide $14 between his son and daughter, so that his daughter will receive one-third more than his son. How much will he give to each? Ans. $6 to son. $8 to daughter. OPERATION. $1 = son's share. º Son Daughter. 1} = daughter's share. $ sº- 14 14 2} = } = the sum of the proportions 7 3 7 3 according to the condition. 3 || 4 $6 $8 21. A father willed his estate valued at $40000 to his three children, in pro- portion, as follows: John one-third, Henry one-fourth, and Katie one-fifth. Before the settlement was made, Henry died. What sum should John and Katie each receive 3 Ans. John $25000. Katie $15000. OPERATION. # -- # = # = the sum of the proportional John. Katie. interests of John and Katie. 40000 40000 8 || 15 8 || 15 3 5 $25000 $15000 NOTE.-Henry having died, his one-fourth proportional interest is therefore to be divided between John and Katie in proportion to their proportional interests, as is also the remainder of the estate. EXAMPLES. 22. B. A. Root and J. H. Knox have been doing a partnership business under the firm and style of Root & Knox. They now wish to settle their business and dissolve co-partnership. The following is a list of their resources and liabilities, obtained by taking an account of stock, and from their memorandum books: cash on hand and in bank, $2600; merchandise in store, $7400; bank stock, market value, $1400; bills receivable, $2100; personal accounts due the firm, $3000. They owe on notes $800, and on open accounts $500. B. A. Root invested $5000 and withdrew $700. J. EI. Knox invested $4500 and withdrew $500. What was the gain or loss of the business? Ans. $6900 gain. OPERATION. RESOURCES : LIABILITIES : Cash - * - - $2600 B. A. Root's net investment - $4300 Merchandise - s -> - - 7400 J. H. Knox’s “ & 4 • ems 4000 Bank stock - s - - tº- 1400 Bills payable mºs - * tº- 800 Bills receivable - - - - 2100 Personal accounts - sº 500 Personal accounts * - - 3000 Total resources - - - $16500 ** liabilities - - - 9600 Total liabilities - - - $9600. Net gain - - - - $6900 Ans. Explanation.—In the operation of this question, we simply classify and add the resources and liabilities of the firm, and then find the difference between their sums, which, because the resources are the greater, is a gain. 824 SOULE's PHILOSOPHIC PRACTICAL MATHEMATICs. * 23. Suppose, in the foregoing example, that the partners had shared the gains equally, what would have been the net capital of each partner on closing? Ans. $7750 Root's capital. $7450 Knox's capital. OPERATION. Net gain as shown above, $6900. B. A. Root's one-half net gain - - - - - - - - - - - - $3450 & 4 investment - - - - - - tº - - - $5000 Less his withdrawals - - - - sº gº - º 700 B. A. Root's net investment - - s - - - - - - 4300 & & net capital - - - - - - - - - - - - , $7750 J. H. Knox's one-half net gain - - - - - - - - - - - $3450 4 & investment - - - dº - - 3-8 - - * $4500 Less his withdrawals - * - - - - - - 500 J. H. Knox's net investment - - * - - º * - *s {- - 4000 é & net capital - - - - - E - - - ºn - $7450 -ms- 24. M. W. Conoly, Chas. Block, and C. Riggs have been doing business under the commercial firm and name of Conoly, Block & Riggs. The gains and losses are to be shared as follows: Conoly one-half, Block one-fourth, and Riggs one-fourth. During the business, Conoly invested $12000 and drew out $1200; Block invested $5000 and drew out $1600; and Riggs invested $6000 and drew out $700. The following is a list of the resources and liabilities of the firm at the time of closing: cash on hand and in bank, $3600; merchandisein store, per inventory, $10850; house and lot, No. 1428 St. Charles street, valued at $4800; bank stock of various banks, market value, $2250; office and store fixtures, $1300; horse and wagon, $540; bills receivable, $3200; personal accounts due the firm, $2500, on which it is agreed to allow 10% for doubtful debts. The liabilities are: bills payable, $5000; mort- gage on the real estate for $2000; personal accounts, $4500. What is each partner's net capital and the present worth of the firm on the closing of the business? Ans. Conoly’s net capital, $9695.00. Block’s {{ 2847.50 Biggs' {{ 4747.50 Present worth of firm, $17290.00. OPERATION TO FIND THE GAIN. RESOURCES : LIABILITIES : Cash tº º - -> sº - $ 3600 M. W. Conoly invested - $12000 Merchandise - - - tº *- 10850 & 4 withdrew - 1200 Real estate - - sº * * - 4800 Bank stock - - - - - 22:50 M. W. Conoly’s net invest. $10800 Office and store fixtures & - 1300 Chas. Block invested – $5000 Horse and wagon º- tº - 540 & 4 Withdrew - 1600 Bills receivable - * º - 3200 Personal accounts º - $2500 Chas. Block's net invest. 3400 Less 10% cº- ºr º 250 C. Riggs invested - - $6000 & 4 withdrew - tº- 700 2250 , - C. Riggs' net investment 5300 Bills payable tº- tº 5000 Mortgage - m am 2000 Personal accounts - tº- 4500 Total resources º º - $28790 Total liabilities - $31000 “ resources tº- 28790 Net loss - - - $2210 PARTNERSHIP SETTLEMENTS. 825 EACH PARTNER'S SHARE OF LOSS. M. W. CONOLY'S NET CAPITAL. M. W. Conoly's # of which is $1105 00 M. W. Conoly’s net investment $10800 00 Chas. Block's # of which is 552 50 4 & # net loss - - 1105 00 C. Riggs' + of which is 552 50 *-ºs é & net cap’l at closing $9695 00 C. BLOCK's NET CAPITAL. C. RIGGS' NET CAPITAL. Chas. Block's net investment $3400 00 C. Riggs’ net investment - - $5300 00 & 4 + net loss - - - 552 50 & 4 + net loss - - - - 552 50 é & net capital at closing $2847 50 “ net capital at closing $4747 50 The net capital of the firm at closing is $9695.00 + $2847.50 + $4747.50 = $17290.00. NOTE.-In the following questions, we have the resources, liabilities (except the partners’ investments) and the net gain or loss given to find the net capital at commencing. 25. J. A. V. Barton has just closed his business with a gain of $5280. His present resources and liabilities are as follows: cash, $9000; merchandise unsold, $1500; bills receivable, face value, $6000; personal accounts, $4500. He owes on notes $5000, and to sundry persons $3500. What was his net capital at com- mencing 3 AnS. $7220. | OPERATION. RESOURCES : LIABILITIES . Cash -> tº- º - - sº - $9000 Bills payable * - - * – $5000 Merchandise - -> - - sº º 1500 Personal accounts - - º -*. 3500 Bills receivable - - - sº - 6000 Personal accounts - - - sº - 4500 Total liabilities - - me - $8500 Total resources - - º - $21000 Resources - - me tº me - * tº º * * * * - * - $21000 Liabilities - º º - - - - " - - º tº 4- * - - º 8500 Present net capital - - - º sº - -- - --> * - tº ºs - $12500 From which we deduct the net gain tº - º - º º º ſº- º - 5280 And obtain - º - º - - &- º º sº t- gº º $7220 which is the amount of net capital at commencing. Explanation.—In the solution of this example, we first find the total amount of resources and liabilities; second, we deduct the liabilities from the resources and obtain $12500 as the present net capital. Then, since the present net capital includes the gain, it is clear that by deducting the gain, $5280, from the present net capital, we have in the remainder the net capital at commencing, which is as above $7220. 26. A. and Z. lost, during business, $8400. Their resources at closing are: cash, $10000; merchandise, $8000; bills receivable, $6000; personal accounts, $3500. They owe on notes $12000, and personal accounts, $7500. A. invested three- fourths and Z. one-fourth of the capital. What was the amount invested by each at the commencement of the business? Ans. $12300 by A. $4100 by Z. º OPERATION. RESOURCES : LIABILITIES : Cash - - - - - - - $10000 || Bills payable - - - - - $12000 Merchandise - - tº º - - 8000 Personal accounts *- -> º - 7500 Bills receivable - - gº --> - 6000 Personal accounts 3500 Total liabilities - - - - $19500 Total resources - º - - $27500 826 SOULE's PHILOSOPHIC. PRACTICAL MATHEMATICs. * Resources wº †º gº gº s tºº tº gº ge gº tºº tº * . gº tº sº $27500 Liabilities tº º * * * * * * * * = * * * * * 19500. Present net capital or worth - - - - - - - - - - - - $8000 To which we add the loss - sº tº dº &= * * wº tº tº º sº tºº 8400 And obtain * * * tº-º * wº # = * > sº tº $16400 which is the net capital of the firm at commencement. Then, as A. invested three-fourths of the capital and Z. one-fourth, it is clear that three-fourths. of $16400 is A's capital, and one-fourth of $16400 is Z's capital. & # of $16400 = $12300, A's capital. # of $16400 = $4100, Z's capital. 27. A. and Z. were partners. They were both insolvent when they com- menced business, and the firm assumed the liabilities of each. The proportion of insolvency of the partners was, A. § and Z. §. They were equal in gains and losses. During the business their profits were $7100, and at the close of the business their resources were: cash, $2800; merchandise, $11400; and personal accounts, $1800. Their liabilities at closing were: bills payable, $12800; and personal accounts, $5400. What was the net insolvency of the firm and of each partner at commencing? What was the capital or insolvency of each partner at closing? Ans. $9300, insolvency of firm at commencing. $6200, {{ Of A. “ 4% $3100, {{ Of Z. “ {{ $2650, {{ of A. “ closing. $450, net capital of Z. “ 4% OPERATION. RESOURCES : LIABILITIES : Cash -, - - - - - - $ 2800 Bills payable - - - - - $12800 Merchandise Gºe sº ſº- * º 11400 Personal accounts *- tº tº Eº 5400 Personal accounts gº $º gº tº- 1800 Total liabilities - * - º-s, ſº $18200 Total resources gº tº gº - $16000 Af Resources º tº * sº - - - dº i. e. sº gº * > s tº ſº tºº $16000 Liabilities º agº, tº * * - g- gº gº sº sº * sº tº gº tº sº 18200 Net insolvency at closing tº º º ºs º ºs º ºs º ºs º º $2200 Gain during business added - - E tº ſº sº gº - - - - - 7100 Net insolvency.of firm at commencing - 4-º - - $9300 # of $9300=$6200, A’s insolvency at commencing. # of $9300=$3100, Z's insolvency at commencing. Total gain, $7100. A's one-half of which is #º $3550 Z's one-half of which is sº $3550. A's insolvency at commencing - - $6200 | Z's insolvency at commencing - - $3100 “ # net gain tº gº tº sº gº 3550 “ # net gain * * * * * * 3550. “ present net insolvency - - $2650 “ present net capital sº s = $450 Explanation.—The difference between the resources and liabilities at closing, not including the partners' accounts, gives us the insolvency of the firm at closing, $2200. To this amount, we add the gain and obtain the insolvency at commencing, $9300. Had the difference between the resources and liabilities at closing been a net capital, not exceeding the gain, which by the terms of the ques- tion it could not have been, we would, for obvious reasons, have deducted the same from the gain in order to have found the insolvency of the firm at commencing. 28. W., X. and Y. have dissolved partnership, and from their irregularly kept books and memoranda, they submit the following statement of facts and figures for a final settlement. They were equal in gains and losses. They have cash on * PARTNERSHIP SETTLEMENTS. 827 hand, $14500; merchandise in store, per inventory, $24200; bank stock, market value, $2100. They own the store that they occupy, which is valued at $31000. The store and office furniture is valued at $3800. The personal accounts due them amount to $9500, on which it is agreed to allow 20% for doubtful accounts. They hold bills receivable amounting, face value, to $8000, on which there is $540 interest due. They owe on notes, $5000, on which there is $150 interest due, and on open accounts they owe $4500. During the business W. invested $25000 and withdrew $3250. 4% 4% X. 4. 22500 {{ 2810. 4% 44 Y. & 4 21000 &é 4100. They have kept a private book which shows the expense to have been $5768. W. desires to withdraw from the business, and hence wishes to know his present net capital. How much is it? * Ans. $29666,67. OPERATION. RESOURCES : LIABILITIES : Cash - - - - - - - $14500 || W’s net investment sº tº gº - $21750 Merchandise º tº sº * - wº 24200 | X’s “ & 6 &=> cº- iº $º 19690 Bank stock - tº sº sº sº dº 2100 Y’s “ é & wº tº- {-} gº 16900 Real estate - - tº sº * > - 31000 || Bills payable * * *—- ºg tº: 5000 Store and office furnitur wº- * = cº 3800 | Interest on same - * = tºº gº sº 150 Personal accounts - - $9500 Personal accounts - - - - - 4500 Less 20% - - - - 1900 — 7600 Total liabilities - - - - $67990 Bills receivable - sº s sº <--> 8000 Interest on same - sº tº tº * 540 Total resources tº a ſº tº - $91740 Resources * = gº gº wº tºº * * : sº tº tº e º cº sº tºº (º tº º $91740 00 Liabilities - † = sº tº ſº * - sº dº º ſº * e ºs tº ºs 67990 00 Total net gain tº *º tº tº tº tº gº © tº 3 gº ºs º * - + )#23750 00 W’s one-third net gain is - - - - - - - - - - - - $ 7916 67 “ net investment wº tº tº º gº tº º tº tº * … º tºo tº-e tºº 21750 00 “ present net capital - - - - - - - - - - - - $29666 67 29. A., B. and C. enter into co-partnership and agree to share the gains and losses equally. It is also agreed that A. shall keep the books and receive there- for a salary of $1800 per year; that B. shall act as chief salesman and receive there- for a salary of $1500; and that C. shall act as salesman and receive a salary therefor of $1200. At the close of the year, there was a net gain, exclusive of the $4500 sala- ries, of $3600. How much of the gain is due to each partner? Ans. A. $1500. B. $1200. C. $900, OPERATION. CR. DR. CR. A’s salary - - - - $1800 – $300 = $1500 B’s “ tº E - & º 1500 — 300 = 1200 C’s “ tº gº gº sº 1200 — 300 = 900 Total liabilities - - dº $4500 Less gain - - - Gº 3600 Net loss sº * * * * - + ) $900 A •º Each partner's one-third loss $300 828 SOULE's PHILOSOPHIC PRACTICAL MATHEMATICs. y 30. W. S. Meek, doing business in Lewisville, Ark., had his store and goods burned. His books were also destroyed, and from a trial balance, taken the day that the fire occurred, and preserved by being at his residence, the face value of the ledger is shown to be as follows: DR. CR. W. S. Meek me sº ſº tº e tºº $ 1200 $14400 Merchandise ſº e- tº *† 32600 24800 Cash - tº age gº º tº º tº tº 50524 47215 Bills receivable - sº * = tº tº º 1600 600 Bills payable tº tº tº sº gº 1100 3100 Interest º sº * > gº tº tº 120 50 M. & T. R. R. stock - * - tº -> * > 1000 Expense * > º º - - gº 780 Personal accounts due gº & # sº 4500 Personal accounts owing - - - 3259 $934.24 $934.24 His goods were fully insured, and in settling with the insurance company, it was agreed, after thorough investigation and close figuring, to allow 35% gain on the sales of merchandise. The R. R. stock is valued at cost, making an allowance of 10% on the personal accounts due for worthless accounts. What was the net gain of W. S. Meek during business, and what was his present net capital? Ans. $5129.63 gain. $18329.63 capital. OPERATION. RESOURCES : LIABILITIES : Insurance Co. for Mdse. burned, al- W. S. Meek's net investment - $13200 00 lowing 35% gain on sales - " - $14229 63 || Bills payable tº gº tº my 2000 00 Cash on hand $º tºº &s sº 3309 00 | Personal accounts ſº tº ſº 3259 00 Bills receivable º tº & gº 1000 00 sºmºmºmº- M. & T. R. R. stock &º tºº gº 1000 00 Total liabilities $º tº tº $18459 00 Personal accounts - $4500 Less 10% - - - 450 4050 00 Total resources sº * gº $23588 63 Resources sº tº tº- tº tº dº sº tºº te tº tº tº > º gº – $23588 63 Liabilities iº sº gº dºe * : gº & ge º * ſº º sº tº 4. It * > 18459 OO Net gain during business - - - - - - - - - - - - $ 5129 63 Net investment added gº gº tº ºs º iº wº gº iº ſº * - º tºº 13200 00 Net capital at closing - - - - - - - - - - - - - $18329 63 Ea:planation.—It will be observed that in the statement of resources and liabilities, we did not include the expense or interest accounts. They were omitted for the reason that they do not here show any available resource or real liability, and hence could not be embraced in a statement of this character to obtain gain or loss. It is true that the gains and losses of the business were affected by these accounts, but it was done through some property account at the time the debits and credits were created. This is the case with all accounts of this character, such as: Premium, Discount, Commission, Profit and Loss, etc., which appear on the face of a double entry Ledger, and which must not therefore be classed as resources or liabilities. In problem No. 28, we included the interest due on the matured bills receivable and bills payable because the amounts there represented actual resources and liabilities. To find the amount of merchandise on hand, we first find the cost of the amount sold, and then by deducting the cost of the sales from the total cost, we obtain the amount lost by fire. XF PARTNERSHIP SETTLEMENTS. 829 31. The following is a trial balance of W. B. Taylor's Ledger: DR. CR. W. B. Taylor - - - - - $ 1600 $15000 Cash gº tº sº sº tº ſº tº 60000 55000 Real estate - - - iºg sº tº 4000 5000 Merchandise - * * * $º ſº tº 21000 27500 Expense tº º ºs tº tº tº 4500 Bank stock - - - - - - 800 1050 R. R. stock - - - - tº gº 1000 Interest * > tº º sº tºº. tº 350 100 Commission - - - - - - 1000 Personal accounts, Dr. tº tº tº 13900 Personal accounts, Cr. tº tº º º- 2500 $107150 $107150 The net gain of the business has been $9214.60. The only property on hand, unsold, is merchandise and R. R. stock. What is the value of the merchandise? Ans. $5214,60. OPERATION. RESOURCES : LIABILITIES : Cash - gº gºs º s * > - $ 5000 W. B. Taylor’s net investment - $13400 00 R. R. stock - - - - - - 1000 Personal accounts tº º º 2500 00 Personal accounts - - - - 13900 *===ºse *==mº Liabilities - - - - $15900 00 Resources, not including merchan- Net gain added - - - 9214 60 dise - - - - - $19900 *=== Total Hiabilities and gain - - - - $25,114 60 Less known resources e- tºº ſº tº 19900 00 Value of merchandise - - - - $ 52.14 60 Or, Known resources, as above - - - - - - - - - - $19900 00 Liabilities, as above sº gº º -, - dº gº º tº - - 15900 00 Gain - - - - - - - - - - - - - - $4000 00 Which deducted from the whole gain tº º tº tº tº tº ſº 9214 60 Gives a gain balance of - - - - - - - - - $5214 60 which must be the value of the merchandise on hand in order to produce the whole gain. 32. A. and Z. are partners in business with equal investments and equal in gains and losses. During the business their total net gain is $4410. A. proposes to draw out 25% of his share of the gain, allowing the balance to remain as an addi. tional investment. Z. proposes to draw out 20% of his gain, leaving the balance as an additional investment. The total net gain is equal to 12.4% of the net invest- ment. What is the net investment of each after the gains have been adjusted according to the above basis # Ans. A's, $19293.75. Z’s, $19404,00. OPERATION. | 100 = assumed net investment of A. and Z. 12# 4410 $35280 total original investment by A. and Z. $17640 A's one-half of original investment. 17640 Z’s 6 & & 4 6 & 836 soule's PHILosophic PRACTICAL MATHEMATICS. * A's one-half of net gain is - - $2205 00, Z's one-half of net gain is - - $2205 60 ** 25% withdrawals - - - 551 25 “20% withdrawals - - - 441 00 “ additional investment tº-e - $ 1653 75 “ additional investment - - $ 1764 00 “ original net investment - - 17640 00 “ original net investment - - 17640 00 “ present net investment - - $19293 75 “ present net investment - - $19404 00 33. A dishonest book-keeper closed a merchant's books and found the net gain to be $13412,45. After he had closed the books, he destroyed the Cash Book and the cash account in the Ledger, and ran away with the cash on hand. The resources of the house, not including the cash, amount to $85418.20, and the liabili- ties amount to $91300.15. What amount of cash did the book-keeper steal? Ans. $19294.40. OPERATION. Resources - tº º - º º º - tº- $85418 20 Liabilities * º - g- º sº - - 91300 15 Loss, by reason of not including cash - - - - $ 5881 95 Net gain - sº- - - . - º - - - 13412 45 Amount of cash stolen - - *- sº º - - $19294 40 34. A. J. Gibson, D. Kinsella, and J. W. Sandidge associate themselves in business with equal investments and equal interests in the gains and losses. It is agreed that each partner shall be charged $3 per day for absence from duty. During the year A. J. Gibson was absent 41 days; D. Kinsella, 65 days; and J. W. Sandidge, 34 days. How shall the partners adjust the matter between themselves without making any entries in their books? Ans. D. Kinsella must pay to A. J. Gibson $17, and to J. W. Sandidge $38. OPERATION. DR. CR. DR. CR. A. J. Gibson owes for 41 days at $3 = $123 $140 – $17 D. Kinsella ** ** 65 “ “ 3 = 195 140 = $55 J. W. Sandidge “ “ 34 “ “ 3 = 102 140 = 38 Total amount due the partners - $420 $55 $55 One-third of which due to each is $140 35. X., Y. and Z. are partners. X. one-half, Y. one-fourth, and Z. one-fourth interest in gains and losses. On closing business, after all the property has been divided, it is found that X's account has $500 excess of debit, and that Y’s account has $150 excess of debit. What settlement is necessary to properly adjust the matter between the partners? Ans. X. must pay Y. $12.50, and Z. $162.50. OPERATION. 2 RESOURCES : X? h tº $325 00 y g - - * º - - 500 s one-half of resources is - º 5 #. º: - - * ºr - - sº Y’s one-fourth “ & 4 - E- 162 50 * * Z's one-fourth “ & 4 * * 162 50 Total resources * * - - $650 DR. CR. DR. CR. X’s - - $500 $325.00 = $175 Y’s tº- e. 150 162.50 = $ 12.5 Z's tº cº- 162.50 = 162.50 * PARTNERSHIP SETTLEMENTS. 831 36. Geo. J. Siegel and Chas. Herzog contracted with B. W. Brown & Co. to erect a cotton factory for the sum of $18000. Not wishing to incur the expense of a book-keeper, they agreed that each should keep a correct account of all receipts and disbursements on account of the contract, and when the work was completed they would have a general settlement. & On the completion of the factory, their accounts and business affairs pertain- ing thereto stand thus: G. J. Siegel has paid for material and wages $7124, and received from B. W. Brown & Co., at various times, on account of the contract, $5250, Chas. Herzog has paid for material and wages $5418, and received from B. W. Brown & Co., on account of the contract, $4870. They owe to various parties for material furnished and labor performed, $1985. What has been the profit? How much is due from B. W. Brown & Co., and how much of it is due to Siegel and how much to Herzog, after paying the amount due for the material and labor? Ans. $3473 profit; $3610.50 due to Siegel; $2284.50 due to Herzog; $7880 due by B. W. Brown & Co. OPERATION. Contract price to build the factory * - º - - tº- - tºg - * $18000 Contractors’ costs to build the same are: Amounts paid by G. J. Siegel - - - - - - - - - $7124 & 4 & 4 Chas. Herzog, - - s - - º * G- - 5418 & 4 unpaid - -- - - sº - º e- - - ‘- 1985 Total contractors’ cost - º - †-> t- º *- ſº- sºme tº- 14527 Net profit º - - me - º - - - - dº - $3473 G. J. Siegel’s one-half net gain is . $1736.50 Chas. Herzog's one-half net gain is - $1736.50 & 4 credit (am’t paid) is - 7124.00 credit (am’t paid) is - 5418.00 4 & total credit is - - - $8860.50 $ & total credit is - - - $7154.50 & 4 debit (am’t received) is 5250.00 & 4 debit (am’t received) is 4870.00 & & loalance of credit is - $3610.50 & 8 balance of credit is - $2284 50 B. W. Brown & Co. contracted to pay - - - nº- * - - * - $18000 & 4 & 4 paid to Siegel - - - - - tº - - - $5250 & 4 & 4 paid to Herzog tº- - - gº ºn s - * - 4870 10120 * { & 4 owe a balance of - - - - - - - - - $7880 - - RECAPITULATION OF BALANCES DUE, Balance due to G. J. Siegel - - - - - - $3610.50 & 4 “ Chas. Herzog - - - - - - 2284.50 & 4 “ for material and labor - - - - 1985.00 Which added gives - - - - - $7880.00 which is the balanº due tº Bºw Brown & Co. 37. T. R. Winn, of New Orleans, and C. P. Johnson, of Texas, enter into a contract to buy and sell cattle, and share the gains and losses equally. To com- mence the business, T. R. Winn remitted to C. P. Johnson a sight check for $20000. Johnson purchased at different times to the amount of $35421.50, and shipped to 832 soulE's PHILOSOPHIC PRACTICAL MATHEMATICs. # Winn at different times, from which he has sold to the amount of $28740. Johnson has sold to the amount of $7196. It is now proposed and agreed to dissolve the partnership. In making up their memorandum accounts for final settlement, it is found that Winn has paid for expenses of the business $785.30, and Johnson $1014.65. That Winn has cattle on hand, valued at New Orleans market price, amounting to $2116, and that Johnson has on hand cattle that cost $1871. It is agreed in making the settlement that each shall retain the cattle in his possession at the valuation above given and pay cash for the balance due the other. What was the gain or loss? What is the cash balance due by one partner to the other, and by which partner is it due # Ans. $2701,55, gain. $8719.93, bal. due by Winn to Johnson. OPERATION. T. R. WINN’S ACCOUNT: C. P. JOHNSON'S ACCOUNT: DR. CR. DR. CR. By cash, original investment $20000.00 | By cash investment - $15421.50 “ paid for expenses 785.30 “ paid for expenses 1014.65 To cash, rec’d from sale of cattle $28740 To cash rec’d from sale of cattle $7196 ‘‘ cattle, value on hand º 2116 “ cattle, value on hand - 1871 $30856, $20785.30 $9067 $16436.15 RECAPITULATION. RESOURCES : LIABILITIES : T. R. Winn Withdrew - - º $30856.00 T. R. Winn’s investment - - º $20785.30 C. P. Johnson “ tº- - º 9067.00 || C. P. Johnson’s “ - - º 16436.15 Total resources - - - $39923.00 Total liabilities - - - $37221.45 37221.45 Net gain - - - - * $2701.55 T. R. Winn’s investment - †- $20785.30 C. P. Johnson's investment - - $16436.15 & 4 one-half net gain e- 1350.77 é & one-half net gain - 1350.78 & 4 total Interest - - $22136.07 & 4 total interest - $17786.93 & 4 withdrawals - - 30856.00 & 4 withdrawals - 9067.00 & 4 balance of indebtedness $8719.93 & 6 balance of capital $8719.93 which he owes to his partner. which is due to him by his partner. 38. “A co-partnership existed between C., D., E. & F., of New York; they agreed to put into the business $25000 each. C. was to have one-eighth, D. one-sixth, E. one-fourth, and F. one-third; and at the expiration of the year, the balance of gain in excess of such proportions, was to be placed to profit and loss account for the next year, as well as to cover any loss that might arise after balancing their books. On adjusting their accounts, they found a gain of $95676.15, but that, instead of the partners fulfilling their contract, C. and D. had put in on account of their capital only $15000 each. In this case, what sum or proportion of the gain must be placed to their respective credits?” The foregoing problem we take from a recent treatise on Commercial Calcu- lation, wherein the author made, by an approximation solution only, 1296 figures. Yºr PARTNERSHIP SETTLEMENTs. 833 In our solution, working all our statements and counting all the figures, we make but 269. The following is OUR SOLUTION. By the first condition' of the co-partnership, each partner, C., D., E. and F., was required to invest $25000; and by the second condition, they were to share the gains, respectively $, $, 4, #, (which added, equals g,) leaving 4 undivided, which was to be placed to the credit of their profit and loss account. Now, since the first condition of the co-partnership was not complied with by C. and D., they having invested less than they were required to invest by the first condition, their respective shares of the gain, ; and #, must be reduced in the same proportion as the amounts invested by them were reduced from the amounts required to be invested by the first condition of the co-partner- ship. And as C. invested but $15000; instead of $25000 as required by the first condition, and since $15000 is # of $25000, C. is therefore to receive # of 4 of the gain, according to the second condition, Which is ºs for his proportional share. D. having invested only $15000, which is but # of the amount required by the first condition, he is therefore to receive but 3 of the # of the gain specified in the second condition, which is ºr for his proportional share. E. and F. having complied with the first condition of the co-partnership, their respective shares, and also the # of the gain to be credited to profit and loss, according to the second condition, remain unchanged. From the foregoing deductions, we have the following proportional interests of the gain: C. ſº, D. Tº E. #, F. , and P. and L. §. Having ascertained the respective proportional interests of the gain at the close of the year, we have but to divide the whole gain in accordance there with. By adding C's #, D's Tºi, E's #, Fs #, and P. and L’s $, we find they sum up but +43. The deficit of Tºfti of unity is occasioned by reason of the decrease of C's and D's shares of gain and will result in augmenting the share of gain of each partner and of profit and loss, in proportion to their respective shares. These increased proportions are respectively: C’s #5 of 120 - 9 - T#5 D’s To of 120 -: 12 - Tºg NOTE.--The 106 is the sum E’s + of 120 - 30 - *}; of the respective shares as F's # of 120 - 40 - Yº; shown above. P. and L's # of 120 F. 15 - Yºs The whole gain of the business being $95676.15, Tºs of the gain is : 106) 95676.15 (902.6052 C’s Share of the gain is 9 times = $ 8123.45 ID's & 4 & 4 & 4 12 “ ſ F. 10831.26 E’s & 4 4 & & 4 30 “ $902.6052 = 2.7078.15 F's “ . . . . 40 “ | •= 36104.21 P. and L’s “ & 4 & 4 15 “ = 13539.08 $95676.15 39. A. contracted with Z. to serve him in the capacity of agent in the pur- chase and sale of goods. July 1, Z. delivered to A. merchandise amounting to $820.70, and cash $600. A. bought at various times merchandise valued at $4039.20, and sold at various times merchandise amounting to $3615.55. At the end of the year, Z. Wishes to close the agency and Settle with A. A. is allowed $300 for services. There is $1508.40 worth of merchandise on hand, which A. returns to Z. What 834 soulE's PHILosophic PRACTICAL MATHEMATICs. 4× has been the gain or loss on the sales of merchandise, the gain or loss of the busi. ness, and how much does Z. owe A. or A. owe Z. ? Ans. $264,05, gain on sales. $35.95, loss of the business. Z. owes A. $123.65. OPERATION OPERATION To find how A. & Z. stand in a Dr. and Cr. sense. To find the Gain on the Sakes of Merchandise. A. IN ACCOUNT WITH Z. MERCHANDISE ACCOUNT. Dr. Cr. Dr. Cr. To value of Mdse. invested in To cash paid to him - - $ 600.00 the business - - $ 820.70 ‘‘ rec'd from sales of Mdlse. 3615.55 “ value of Mdse. purchased “Mdse on hand the day of set- during business - - 4039.20 tlement gº tº tº- 1508.40 By amount received from By cash paid for Mdse. Mdse. sold gº $3615.55 purchased - e- $4039.20 * value of Mdse. On “ Mdse. returned - 1508.40 hand * sº 1508.40 “ salary for services 300.00 || To Profit and Loss (for gain on To Balance (amount due to A.) 123.65 sales) Eº gº gº dº 264.05 $5847.60 $5847.60 $5123.95 §5123.95 OPERATION To find the Loss of the Business. Expense incurred to conduct the business (A’s salary) {- * £º tº r Q = iº sº $300.00 Gain as above :- º º tº º sº * º º tº g-º tº- tº- sº 264.05 Net loss of the business - * s sº s $-º tº- sº &= sº sº º $ºn $35.95 SECOND OPERATION TO FIND IRALANCE DUE TO A. A. in Account with Z. e Dr. Cr. To cash, paid to him - - - - $600.00 | By Mdse. returned - - - - $1508.40 “ Mdse. delivered to him - * sº 820.70 || “ salary - gºe º sº g- & ſº 300.00 ** Gain on sales s sº * => sº 264.05 To Balance due to A. - sº *-* sº 123.65 $1808.40 $1808.40 40. J. Jones, a merchant in New Orleans, has a branch store in the country. He places in charge of said store, H. Smith, at a salary of $600. At the time Smith took charge, the books showed the following accounts: Goods on hand, $2100; cash, $50; bills receivable, $130; personal accounts receivable, $340; bills payable, $620. The rent of the store was $20 per month. At the end of the year, Jones examines the business and finds goods in stock, $1800; goods bought, $4500; goods sold, $6300; expenses paid, $72; rent paid, $50; salary paid, $250; bills receivable in safe, $300; bills payable, outstanding, $750; personal accounts receivable, $840; personal accounts payable, $120. Received for interest, $36; for running a post office, $180; for other services, $24, Paid the joroprietor, cash, $300. * PARTNERSHIP SETTLEMENTS. 835 Required, 1. The net gain, allowing 10% of the bills receivable and personal accounts receivable for bad debts. 2. The present worth of the country house. 3. The correct cash balance. Ans. $714 net gain. $2414 present worth. $998 cash on hand. The above problem was presented to us with a request to supply the required 2.IlSW62TS. The following are our solutions: FIRST SOLUTION. 1. OPERATION TO FIND NET INVESTMENT OF J. JONES. J. JONIES. To bills payable - - - - - $ 620 | By Mdse. - - - - - $2100 ** cash drawn tº- º - º tº- 300 | ** cash - tº- - * wº- - tº- 50 To balance, met investment º Gº tº 1700 ‘‘ bills receivable - - E - ſº- 130 “ personal accounts receivable - dº 340 $2620 $2620 2. OPERATION TO FIND CASH BALANCE. J. Jones, Branch Store. To Mdse. at opening - - - - $2100 | By net investment - - - - $1700 “ “ purchased me - = - 4500 || “ Mdse. Sales - - * - was 6300 “ cash, for expenses - - sº 4- 72 ‘‘ bills payable at closin g- - -> 750 * * * * * * rent - nº º g-º º 50 || “ personal accounts at closing - - 120 “ “ “ salary - - - - - 250 | ** interest received - se - - 36 “ personal accounts receivable at closing 840 “ post Öffice salary -> * - - 180. “bills receivable at closing - - tº 300 “ service, cash - - - - - 24 - Total credits - - * > - - $9110 Total debits - - *- - º tº $8112 “ debits - - * dº ſº. 8112 Cash balance - - - - - - $ 998 3. OPERATION TO FIND TEIE NET GAIN AND THE NET WORTH OF THE BUSINESS. Resources. Liabilities. Cash on hand - - - - - - $998 || Bills payable - - - - - - $ 750 Mdse. ** ** - i- *- - º ſº 1800 | Personal accounts - - * º - 120 Bills receivable - º - $300 270 Due for rent - - - - *- - 190 Less 10% - tº me * 30 “ salary - m tº tº º 350 Personal accounts - º - 840 756 J. Jones' net investment tº tº- º- 1700 Less 10% º me sm - 84 Total resources * * tº- $3824 Total liabilities - gº - $3110 ** liabilities sº- - - tº 31.10 Net gain - - - - * tº $ 714 J. Jones' net investment - - sº º tº- • * ſº- - * tº- - ſº- - $1700 J. Jones’ net gain * , ~ * * * tº gº tº tº * * me tº mº - 714. J. Jones' net worth - - * gº - sº gº - - sº - - sº g- - $2414 Explanation.—In problems where the conditions of the business are given at the opening and at the closing, the correct cash balance may be determined as follows: 1. Open an account with the proprietor and find his net investment by crediting his account with the cash and all other pro- 836 SOULE's PHILOSOPHIC PRACTICAL MATHEMATICs. X- perty advanced on commencing or during the business; and then debiting the same with all liabili- . at commencing, and for all withdrawals during the business, as shown in the first operation a UOV6. 2. Open an account with the Branch Store and debit the same with the balance of all real accounts except cash which is not known, and also with the total of the representative or loss and gain accounts, such as merchandise, real estate, expense, rent, salary, etc. Then credit the Branch Store account as follows: 1st.—With the proprietor's net investment as shown in the account first opened with him. 2d.—Credit it with the credit balances of all real accounts, and with the totals of all representative accounts, such as merchandise, interest, etc. When this has been done, take the difference between the aggregate debits and the aggregate credits, and the same will be the correct cash balance, as shown in the operation. Now, having the cash balance, to find the net gain and the net capital make the usual statement of resources and liabilities as shown in the third opera- tion. NOTE.-In case of partnerships, first open accounts with each partner and find the net invest- ment, as above. SECOND SOLUTION. J. J. ONES. To bills payable - - - º - $ 620 | By Mdse. - - ºn - - - $2100 ** cash drawn * - - g- - 300 | ** Cash - - º - - - 50 To balance met capital - - gº ºn £414 || “ bills receivable * * * * 130 “ personal accounts receivable - º 340 “ net gain from P. and L. tº º 714. $3334 $3334 Merchandise. To inventory at commencing sº - $2100 | By sales - - - º * - º $6300 “ purchases - - - - - - 4500 || “ inventory at closing ºn tº º 1800 To profit and loss - - - - - 1500 $8100 $8100 C. A. S. H. . To amount on hand at commencing - $ 50 | By expenses - - tº- -: - fºr $ 72 ‘‘ interest - - - * * * 36 || “ rent, paid on account - - - 50 “ P. O. receipts - - - - - 180 || “ salary “ “ & 4 º - º 250 “ sundries - - - - 24 || “. J. Jones ‘‘ ‘‘ & 4 sº - - 300 “ Mdse. sales * * - - - $6300 “Mdse. purchases - - º $4500 *** * * * *|†º º º $4922 By balance on hand as sº - * 998 $5920 $5920 Bills Receivable. Dr. at commencing * * $130 “ at closing - - - - - - 300 • * increase during business - - - $170 X} PARTNERSHIP SETTLEMENTS. 837 Personal Accounts EReceivable. —A-i- Dr. at commencing - - - - - $340 “ at closing - * = 4- 4-> sº 840 “ increase during business - - - $500 IBills Payable. Cr. at commencing - - - - - $620 * “ at closing - - - - - 750 “ increase during business - - - $130 OPERATION TO FIND THE NET GAIN BY THE RESOURCE AND LIABILITY METHOD. Besources and Liabilities. Mdse. inventory at closing tºº – $1800 | Due for rent - - - - - - $ 190 Cash, at closing - - - - - 998 “ salary ess tº e º º ſº 350 Bills receivable at closing - - $300 270 | Bills payable sº º tº tº tº 750 Less 10% for bad debts - - 30 Personal accounts - - - - - 120 Personal accounts at closing - 840 756 J. Jones' net investment - - - - 1700 Less 10% for bad debts - $º 84 * -ºssº tº-ºº- Total liabilities - - - - $31.10 Total resources - - * wº $3824 “ liabilities - - * wº 31.10 Net gain tº º ºs º ºs $ 714 * J. Jones' net investment - - 1700 & 4 “ capital - sº -> $2414 OPERATION TO FIND THE NET GAIN BY THE LOSS AND GAIN METHOD, LProfit and LOss. To rent - - - - - - - $ 240 | By Mdse. - - - - - - - $1500 “ salary of clerk - wº- s gº sº- 600 | ** interest sº * sº s e- sº 36 “expenses - tº- *º * gº * - 72 “ P. O. salary s º gº tº-> * 180 “10% loss on personal accounts -g 84 “sundry services wº as ºs & 24 “10% loss on bills receivable - tº- 30 Total loss {º º tº sº, sº $1026 Total gain <--> - - - - $1740 “ loss &- gº sº sº º 1026 Net gain - - - - - - $ 714 NOTE.-This form of statement can be made only in cases where all the items of gain and loss are given, or where they can be found from the facts presented. Ea:planation.—In this solution we have opened accounts with the business man, and with the resources and liabilities, and thus show the details of the business more fully than were shown by the first solution. In the solution, we have shown the two methods of determining the gains of a business, and two methods of showing the proprietor's net capital or networth. THIRD METHOD OF SOLUTION. The problem may be solved by first finding the net gain of the business as shown in the second solution by the loss and gain method. Then, having the net gain, make the resource and liability statement as shown. In the second solution, except the item of cash, which is yet unknown. This statement will give resources $2826 and liabilities $31.10. Then since there was a gain of $714, it is evident, from the principles given in article 1429 that the resources must exceed the liabili- ties by that sum, $714, which would make the º of the resources $3110 –– $714 = $3824, Then, since the resources with the item of cash omitted, amount only to $2826, therefore the differ- ence between $3824 and $2826, which is $998, must be the correct cash balance. Having the cash balance and the net gain, it remains only to add the net gain to the proprietor's net investment and thus produce his net worth at closing. 838 SOULE's PHILOSOPHIC PRACTICAL MATHEMATICs. x- 41. S. Hicks established a branch store in a country town and placed W. Wood in charge, at a salary of $40 per month, and a commission of 10% of the profits. Rent of store was $360 per year and salary of assistant clerk, $25 per month. S. Hicks placed in the store merchandise amounting to $2210, and handed W. Wood $100 for account of the business. At the expiration of sixteen months, Mr. Wood having become dissipated and untrustworthy, a settlement was made. From the irregularly kept books and the inventory of goods, the following facts were learned: Merchandise on hand, $3440; merchandise bought during the business, $15200; merchandise sold during the business, $17760; bills receivable on hand, $3100; personal accounts due the house, $1050; three shares of leather factory stock, $270; bills payable outstanding, $4820; personal accounts payable, $2300, not including the clerks’ salaries. Cash on hand, $1280; paid on account of rent, $400; paid assistant clerk on account of salary, $350; paid S. Hicks, $1200; W. Wood drew $900. What has been the gain of the business? What is the net capital of S. Hicks? What is the condition of W. Wood's account? Ans. 1. Net gain, $1040. 2. S. Hicks' net capital, $2046. 3. W. Wood owes S. Hicks, $156. SOLUTION. OPERATION TO FIND S. HICKS' NET INVESTMENT. S. H I C E S - To cash, drawn out {- gº gº ſº- $1200 | By Mdse. invested sº gº gº * * $2210. To balance met investment gº º =} 1110 || “ cash & 4 e- &= * tº sº 100 $2310 $2310. OPERATION TO FIND THE NET GAIN OF THE BUSINESS. Besources. Liabilities. Mdse. on hand º gºe gº; e- t- S. Hicks’ net investment tº- gº gº $1110. Bills receivable as º gº me sº 3100 | Bills payable - - * * * 4820 Personal aceounts receivable tº- * 1050 | Personal accounts - sº * tº s &= 2300 Leather factory stock - gº tº-º tº 270 | Due for rent tº e * s - $480 } 80. Cash on hand tºº {-º * sº- tº 1280 | Less amount paid - cº- cº- - 400 W. Wood Eº i__ * gº – 900 260 Assistant's salary - sº sº - $400 } 50. Less salary, 16 months - - - 640 Less amount paid - - - - 350 Total resources 4- gº gº tºº $9400 Total liabilities * = gº gº tº- 8360 Net gain - - - - - - $1040 Total liabilities - - - - $8360, OPERATION TO FIND THE BALANCE OF OPERATION TO FIND S. HICKS' NET W. WOOD'S ACCOUNT, CAPITAL. W. Wood, salary 16 months @ $40 $640 S. Hicks’ net investment - - - $1110. & 4 10% of $1040 profit - 104 & 4 90% of $1040 profit - gº •º 936. Total amount due sº º sº $744 W. Wood has drawn - a- * -º sº 900 “ owes S. Hicks - - - $156 | S. Hicks' net capital - - - - $2046. 42. Jones of New Orleans, employs A. to conduct his branch house at New Iberia, which had been in operation for a year, and the books showed at the last t * PARTNERSHIP SETTLEMENTS. 839 closing the following results, viz: Cash on hand, $500; Mdse., $2000; bills receiv- able, $1600; personal accounts receivable, $900, and bills payable, $700; and per- Sonal accounts payable, $1300. The agent manages the business for another year, and presents the following statement of the Ledger accounts at the closing of the current year, except the cash account not given. Mdse. Dr., $15000; Cr., $14000; balance on hand, $2400; real estate on hand, $1500; bills receivable, $1200; personal accounts receivable, $1000; interest account Dr., $80; bills payable account Cr., $2000; personal accounts payable, $1600; commission account Cr., $50. What was the cash balance, net gain, and present worth ? Ans. cash, $1870. Net gain, $1370. Proprietor's net worth, $4370. OPERATION TO FIND THE NET INVESTMENT. Jones’ Account. Bills payable - - - - - - - $700 | Cash - - - - - - - $ 500 Personal accounts payable - sº º 1300 || Mölse. wº- tº- - e- - º sº 2000 Bills receivable - - - º sº 1600 Personal accounts receivable - º - 900 Total CT. - - s - º - $5000 ‘‘ Dr. - * sº º &- 4- 2000 Total debit - - - - - - $2000 | Net investment - - - - - $3000 OPERATION TO FIND THE CASH BALANCE. Branch. Account. Mdse. Dr. - - - - - - $15000 || Mdse. Cr. - - as ºs º- - $14000 Real estate - - º - - º 1500 | Bills payable - - tº- - º 2000 Bills receivable - - - - * 1200 | Personal accounts payable - - º 1600 Personal accounts receivable * sº 1000 l Commissions - - •- - - {- 50 Interest and discount - - -> tº- 80 | Proprietor's net investment * ºn 3000 Total - - s - - - $18780 To cash balance - º -> - º 1870 $20650 $20650. OPERATION TO FIND THE NET GAIN. Resources. Liabilities. Cash as above * * * * * $1870 | Proprietor's net investment - tº $3000 Real estate - - * - - º 1500 | Bills payable - - - sº- gº 2000 Hills receivable - sº - - g- 1200 | Personal accounts payable - - - 1600 Personal accounts receivable - - 1000 Mdse. on hand - e - - sº 2400 Total liabilities t- * - sº $6600 “ reSources º - - tº 7970 Total resources - - - - 7970 Net gain * * * ºn 4 º $1370. OPERATION TO FIND THE NET WORTH. Jones’ net investment - tº- - - º - - $3000 Jones’ net gain - e- - º tº- - sº - º 1370 Jones’ net worth * - sº º - tº ſº º $4370 Explanation.—This problem may be solved in the same manner as shown in the several solutions of problem 40, and of problem 41. But to abridge and simplify the work, we have solved it by 84o soul E's PHILOSOPHIC PRACTICAL MATHEMATICs. ºf another method based on the ultimate results of the accounts, which method is applicable to all similar problems. The following directions render this method clear: 1st. Open an account for the proprietor, and credit the same with the cash, merchandise, per- sonal accounts receivable, bills receivable and all other property, furnished at commencing, and with any other advances made during the business. Then debit the account with the bills payable, personal accounts payable, and any other obligations due on commencing, and for any withdrawals during the business. The balance of his account will be his net investment. 2d. Open an account with the branch house and debit the same, first, with the debit balances of all the real accounts, except cash, which is not known; second, with the totals of all the property and representative accounts. Then credit the branch house account first, with the proprietor's net investment; second, with the credit balances of all real accounts; and third, for the totals of all property and representative accounts. The sum of all the debits, subtracted from the sum of all the credits, should be the balance of cash on hand. 4. 3d. Make a statement of all the given net resources including cash, and, of all the given net liabilities, including the proprietor's net investment. Their difference will be the net gain of the business. - NOTE.-If allowances are to be made for doubtful debts, unpaid expenses, rents, salaries, etc., for which no entries have been made, and for licenses, taxes, insurance, etc., that have been paid, but whose time has not expired, make the necessary adjusting entries in the list of resources and liabilities 43. A. M. Perdue, a merchant in Pine Bluff, Arkansas, engages M. J. Mul- ler to take charge of his branch store at Linnwood. The following assets and liabil- ities are shown by a statement made the day M. J. Muller assumed charge : Cash, $275; merchandise in stock, $2990; bills receivable, $180; personal accounts Cr., $340: bills payable, $210. For conducting the business, M. J. Muller is to receive $40 per month; rent of store is to be $20 per month. At the end of fifteen months, Perdue wishes to sell out his business, and on investigating his affairs, finds the following state of things: Insurance paid, $162.50; sundry expenses paid, $135; one Jefferson Co. bond ($100), worth $90; merchandise bought, $3450; merchandise sold, $5905; merchandise on hand, $3245; rent paid, $300; salary paid, $500; per- sonal accounts Dr., $475; personal accounts Cr., $375; bills receivable, $325; bills payable, $290; paid interest and discount, $12.50; received interest and discount, $10. Perdue has drawn out $200. He cancels his policy of insurance and is allowed $55, return premiums. Find gain of business, present net capital of A. M. Perdue, and cash on hand 3 Ans. Cash on hand, $890. Net gain, $1565. Net capital, $4260. First Operation by the Method shown in Problem 42. Ferdue’s Account. Personal accounts payable - - - $ 340 | Cash - - * - - * - $ 275 Bills payable - sº - -- - 210 || Merchandise - - - - - - 2990 "Withdrawal - - 4- +- - tº- 200 || Bills receivable sº - -> - * 180 Total credit 4- - - & - $3445 ‘‘ debit - - - tº- - 750 Net investment - - sº - $2695 ! Net gain as below. - *m tº - 1565 Total debit sº º tº º - $750 Net capital at closing * - $4260 * PARTNERSHIP SETTLEMENTS. 841 Branch. Account. Insurance paid - - - - $ 162.50 | Insurance returned - - - - $ 55.00 Sundry expenses paid - - - - 135.00 || Merchandise *- - - - - 5905.00 Jefferson Co. bond - º - - 90.00 | Personal accounts Cr. - - - e- 375.00 Mdse. bought - - - - - 3450.00 | Bills payable - - - - - 290.00 Rent paid º - - * * - 300.00 || Interest – * - - - *- ºn 10.00 Salary paid - * * tº º - 500.00 | Perdue's net investment - - - 2695.00 Mdse. at commencing - - - - 2990.00 Personal accounts receivable gº - 475.00 Bills receivable - - - º - 325.00 Interest and discount - º- ºr cº- 12.50 Total º - - - - $8440.00 To cash balance - - - wº 890.00 $9330.00 $9330.00 Resources. Liabilities. Cash as above - - - - - $ 890 | Perdue's net investment º - - $2695 Mdse. on hand -> - - E- - 3245 | Bills payable º - - - - 290 Personal accounts receivable - -> 475 | Personal accounts payable - - - 375 Jefferson Co. bond - - cº- sº º 90 | Salary due gº ſº sº & sºme 100 IBills receivable º - - sºs - 325 Total resources - t- gºe tº- $5025 Less liabilities - - <-> - 3460 Net gain - - * cº- cº- * - $1565 Total liabilities tº º Cº- - g- $3460 Second Operation by the Detailed Account Method. Perdue's Capital Account. OPERATION TO FIND CASH RECEIVED AND PAID ON ACCOUNT OF MERCHANDISE. Withdrawal - - $ 200|Investment - - $2895 Merchandise. Net capital - - 4260 |Gain - - - - - 1565 - Mdse. bought $3450|Mdse. sold - - $5905 $4460 $4460|Fă by bińs pay. $80 Rec’d bills rec. $145 TDue on personal ‘‘ bond - 90 accounts - 35 Personal acct's — 115 due - - - 475 tº — 710 . Merchandise. *- Paid in cash - $3345|Received cash $5195 At commencing - $2990|Sold - - - - - $5905 º Bought - - - - 3450|Inventory - - - 3245 Bills Payable. Profit and loss - - 2710 On commencing $210 $9150 $9150 “ closing - " - 290 Increase * - s so Int, and Disc't. Ičent. Cash. - Paid $12.50 | Rec’d $10 $300 On hand - - - $ 275|Expenses paid $135.00 d Insurance returned 55|Rent paid - - 300.00 Interest received - 10|Perdue withdrew 200.00 Insurance. Salary. Rec’d from Mdse. Insurance paid ; s sales - - - - *|Rºy Pº ºPd. $162.50 | Retd $55|Paid $500 Mdse. paid - 3345.00 Due 100 Interest paid - 12.50 Balance on hand 890.00 Bond. Lapense. $5545 $5545.00 —I- - *-*-* so siast 842 SOULE's PHILOSOPHIC PRACTICAL MATHEMATICs. 3. Bills Receivable. | Personal Accounts Or. | Personal Accounts Dr. On commenc'g $180 On commenc'g $340||On closing - $475 At closing - 325 “ closing - 375 Increase - $145 Increase - $ 45 Profit and Loss. Resources and Liabilities. Rent sº - $300.00 || Mdse. gain - $2710.00 || Cash s - $ 890 | Perdue's net inv. $2695 Salary - - 600.00 | Interest º 10.00 || Mölse. sº gº 3245 | Bills payable 296). Interest - - 12.50 * Bills receivable 325 | Personal accts. Cr. 375 $162.50 } Bond - - 90 | Salary due 100 Insurance 55.50 (107.50 Personal accounts 475 $3460 Expenses - 135.00 Net aain tºº 1565 Perdue's gain 1565.00 et gav 44. A Temperance town placed in the hands of a druggist, who is to act as agent, liquor amounting to $415 and cash $50. The agency continued fifteen months. On settlement, it was found that the agent had purchased goods amount- ing to $4600; sold goods $5810, and had on hand, which he turned over to the town authorities, goods amounting to $1005.50. The agent was to receive a salary of $20 per month and 10% of the gross profits. What does the agent owe the town 3 What was the net gain of the town 3 Ans. 1. Agent owes the town $779.05. 2. Net gain of the town is $1320.45. First Operation to find the Balance Due the Town. DR. Druggist. CR. To merchandise - - - - - $ 415.00 | By merchandise returned - - - $1005.50 ‘‘ cash º sº sº gº º ºs 50.00 || “ 15 months’ salary - - - - 300.00. “ gain on sales - - - - - 1800.50 || “.10% of profits - - - - - 180.05 $1485.55. By balance due town - * gº tº 779.95. $2265.50 $2265.50, Second Operation to find the Balance Due the Town. Druggist. To cash, at commencing - - - 50.00 | By cash, merchandise bought - - $4600.00 ** ** merchandise sales º-º º 5810.00 “ 15 months’ salary - - sº º 300.00 “ 10% of profits * * * * 180.05 By balance due the town es e- * 779.95 $5860.00 $5860.00 OPERATION TO FIND THE GAIN ON MERCHANDISE. Merchandise. Merchandise on commencing - a 3. - $ 415 || Merchandise sales gº tº gº is * $5810.00 & 4 purchased - gº gº tº 4600 44 inventory sº * iſ a 1005.50 Sales and inventory - sº *-* - $6815.50 Total cost - * = gº *…* gº 5015.00 e **— Total cost - - - - - $5015 Gain on merchandise - - - $1800.50, * PARTNERSHIP SETTLEMENTS. 843 OPERATION TO SHOW TOWN’S NET GAIN. Profit and Loss. Salary - - - - - - - $300.00 || Gain on merchandise - - - - $1800.50 10% commission on gross gain - - 180.05 Town’8 met gain - - - - - 1330.45 $1800.50 $1800.50 Explanation.—The above problem, like problem 39, merits the critical attention of the accountant. The operations are made so explicit that we deem an extended explanation unnecessary, for those familiar with accounts. Parties who are not familiar with accounts are not expected to comprehend the work. 45. A., B. and C. bought a lot of merchandise on speculation for $3000. A. paid $1500, B. paid $900 and C. paid $600. It was agreed that each one's interest in the goods should be in proportion to the amount of money invested. They then sold to D. one-fourth interest in the goods for $1800, and A., B. and C. agreed to so adjust the matter that each of them should be one-third owner in the remainder of the goods. What was the financial adjustment between A., B. and C.? Ans. C. paid $360. A. received $1800. |B. received $360. SOLUTION. OPERATION TO FIND THE INTEREST SOLD TO D., | OPERATION TO FIND WHAT C. MUST PAY FOR Tg5 THE INTEREST THAT A. AND B. SOLD RE- AT THE RATE OF $1800 FOR + INTEREST IN SPECTIVELY AND THE ADDITIONAL INTER- $3000, IN ORDER. To own 1% or # INT. EST ROUGHT BY C. IN THE REMAINING GOODS. Amount Interest Interest Interest Interest paid. owned. retained. sold % bought "A. & B. by C. 1800 A. $1500 +3 — ºr = 1°, $ 4. B. 900 ºr — ºr = Tâu 120 || 6 C. 600 #; Jºy = q Tân g -º-º: sºme - $360 $3000 #} #3 + 4 = 1% = the interest sold to D. or thus: #3 — ºr = ** = int. retained by A., B. & Cº. # : Hºo :: $1800 : $360. fºr – 3 = % = intofA.,B. & C. respectively OPERATION TO FIND THE AMOUNT OF MONEY DUE A. AND B. RESPECTIVELY FOR THE INTERESTS SOLD TO D. AND C. Amount received from D. -> - - $1800 A. sold as above - s - º - # Amount received from C. sº - - 360 IB. sold as above - - sº - tº º T#7 Total interest sold - - - - Tº Total amount received is º tº- $2160 A - B which is to be divided between A. and B. in pro- $2160 | $2160 portion to the interest each sold. 36 | 120 36 | 120 120 || 30 120 | 6 $1800 $360 844 +k soule's PHILOSOPHIC PRACTICAL MATHEMATICS. 46. A manufacturing corporation has an agency in a distant city, for the sale of its goods. At the close of the year, December 31st, 1891, the agency showed the following inventory or statement: Merchandise in stock - 4- - e- Accounts receivable - - - - - Cash on hand - gº - tº- - º Store fixtures - -> - tº- - tº- $256,897.00 102,105.00 2,220.40 4,500.00 *- $365,722.40 December 31st, 1892, the agency presented the following resource statement: Merchandise in store - *- - - Accounts receivable - tº- - Cash - - == - - - - Store fixtures - - - º - - $238,305.00 110,300.00 3,515.15 4,200.00 $356,320.15 During the fiscal year, the agency received from the manufacturing company for expenses $30000. and remitted to the company $285,427. The sales of goods for the year were $452,512. Allowing the expense of the business and the loss on sales for bad debts to be equal to 18%% of the sales, what has been the net gain of the agency 3 Ans. $161,178.75. OPERATION Agency at — To Mdse. Dec. 31, 1891 sº - $256,897.00 | By Mdse. Dec. 31, 1892 tº- - $238,305.00 “ accounts receivable, Dec. 31, 1891 102,105.00 || “ accounts receivable, Dec. 31, 1892 110,300.00 ** cash wº- - - º - 2,220.40 “ cash a- -> - º - 3,515.15 “ store fixtures - - me - 4,500.00 || “ store fixtures - - sº - 4,200.00 $365,722.40 $356,320.15 “ cash received from Mfg. Co. - 30,000.00 “ cash remitted to Mfg. Co. - 285,427.00 $395,722.40 $641,747.15 395,722.40 Gain - - - * = - $246,024.75 Expenses and loss, 184% on $452,512 = - tº- tº - &= - 84,846.00 Net gain of Agency $161,178.75 Manufacturing Company Account at the Agency. To net inventory, Dec. 31, 1892 - $356,320.15 • ‘‘ cash remitted - - -: - 285,427.00 Total debit - - - - $641,747.15 “ credit - - mº - 395,722.40 Net debit, - - - - - $246,024.75 Less expense as above - º - 84,846.00 Net debit - - - - - $161,178.75 By net inventory, Dec. 31, 1891 ‘‘ cash received - - º Total credit - - s $365,722.40 30,000.00 $395,722.40 PARTNERSHIP SETTLEMENTS. 845 47. A firm sold goods at 70% discount on list price, the net sales amounting to $200,000. The net gain on the sales was $60000. They now wish to allow 75% discount on list price and increase the sales so as to produce the same amount of gain. What will be the amount of net sales, the amount of list price, and the cost? Ans. $375,000 net sales. $1,500,000 list price. $315,000 cost. OPERATION. 1. Find the list price of $200,000 net sales at 70% discount. 30 : 100 :: 200,000 : $666,666; = the, list price. 2. $666,666; list price–70% = $200,000 net sales, giving a gain of $60000, and a cost of $140,000. 3. $666,666; list price—75% = $166,666; net sales, – $140,000 cost = $26666; gain. Having now produced the gain, $266663, that would have been realized on the net sales of $166,666; at a discount of 75% on list price, we divide the gain that is required, whatever it may be, in this case $60000, by the amount of gain realized, in this case $266663, and in the quotient we have the ratio between the gain realized and the gain required. Thus, to further elucidate, $60000 -- $266664 = 24; which shows that the gain realized at 75% discount ($266663) is but 24 part of the gain required ($60000). It also shows that the net sales which produced the $266663 gain, are but 24 part of the net sales required to give 24 times as much gain. HENCE we MULTIPLY THE $166,666; NET SALES BY 24 AND PRODUCE $375,000, THE NET SALES REQUIRED. Now, since the net sales are only 25% of the list price, therefore 4 times the net sales will be the list price, which is $375,000 x 4 = $1,500,000 or, thus: 100 — 75% = 25, and then 25 : 100 : : $375,000; $1,500,000. PROOF. $1,500,000 list price — 75% = $375,000 net sales — $60000 gain = $315,000 cost. SECOND PROOF. $140,000 cost gained $60000 = 42%. $315,000 cost gained $60000 = 1991%. 42% -- 1941% = 2+ which is the ratio between the gain when 70% discount was allowed and the gain when 75% discount was allowed, as shown above. ADJUSTMENT OF INTEREST ON PARTNERS’ ACCOUNTS. Many of the following questions involve in their solution the principles of the science of Double Entry Book-keeping as well as the principles of the science of numbers, and are therefore of special importance to young accountants and mer- chants who aspire to eminence and success in the line of their professions. - 48. J. M. Bracey, T. A. Hammons, and T. G. Mackie are partners. They shar gains and losses equally, and by the conditions of the partnership, an account cur- rent and interest account is to be kept with each partner, at 8%; i.e., interest at 8% is to be allowed on the investments, and charged on the withdrawals of each part- ner, by which means the investments of each partner are equalized. At the end of the year, when they wish to close their books, it is found that there is interest on the debits and credits of the partners' accounts, as follows: Dr. Cr. Net Cr. J. M. Bracey - - $750.00 $2350.00 => $1600.00 T. A. Hammons - tº- 515.50 1740.00 = 1224.50 T. G. Mackie wº gº 160.75 641.20 = 480.45 Total credit of interest - tº * * * > sº $3304.95 846 SOULE's PHILOSOPHIC PRACTICAL MATHEMATICs. X- What is the correct Journal Entry to be made in the books of the firm, in order to properly adjust the interest between the partners? OPERATION. Amount of credit interest - tº * tºº * $3304.95 Each partners' one-third of which is - - $1101.65 IDr. Cr. Dr. Cr. J. M. Bracey e- * tº- $1101.65 $1600.00 = $498.35 T. A. Hammons - tº- &- 1101.65 1224.50 = 122.85 T. G. Mackie sº- sº- * 1101.65 480.45 = $621.20 $621.20 $621.20 From these figures, the following entry is made: T. G. Mackie To Sundries $621.20 To J. M. Bracey - - - - - - - - $498.35 “T. A. Hammons gº *g tº-º tº me sº * tºº 122.85 The above entry is the only correct and proper one that can be made to adjust the interest that accrues on the partners' investments. But the general Way Of adjusting partnership interest of this kind is by passing it through profit and loss account. This is done by the following entry: Interest (or Profit and Loss) To Sundries - - $3304.95 To J. M. Bracey - - sº me tº tº sº sº ºn tº º $1600.00 “ T. A. Hammons &º tº sº sº sº sº sº * * - s 1224.50 “ T. G. Mackie - * * * -> tºº, sº * * tºº sº ſº * - tº- 480.45 This entry produces the same result, so far as the partners are concerned, as the correct one, but by it a false loss is represented, and hence it is not a proper entry to make. It is clear to any accountant that no loss can be sustained by a firm unless the capital of that firm is thereby decreased, and it is also clear that while partner- ship interest effects changes in the investments of the different members of the firm, it does not decrease the capital of the firm, and therefore no loss has occurred by reason of partnership interest, and consequently the books should not represent a loss. It is a Sorrowful fact, not generally known, that many merchants of reputed respectability, for the purpose of showing small profits in their business, charge their interest accounts with 12, 15, or 20% interest on capital, and their expense account With partners' salaries sufficiently large to absorb nearly all of the gain. This practice has been indulged in to such an extent by the non-observers of the principles of ethics, as to bring disgrace upon honorable merchants in all Sections of the country. The practice should be discontinued. 49. The following problem was presented by a teacher for solution: A., B. and C. form a co-partnership under the following conditions: A. is to manage the business, and to receive therefor $2400 per annum, which amount is to be credited on July 1st. He is to receive interest on his salary and to pay interest on sums withdrawn at the rate of 6% per annum. B. and C. are to furnish the cap- ital, and to receive interest therefor at the rate of 6% per annum; the net gain or loss to be divided equally. B. invests, January 1st, $10000; April 1st, $5000. C. Invests, January 1st, $10000; July 1st, $5000, and draws out, September 16th, $500. A draws out, February 1st, $200; March 1st, $400; July 11th, $500; October 1st, X- 847 PARTNERSHIP SETTLEMENTS. $200; November 21st, $100. At the end of the year the gain—without taking into account either the salary to be paid to A. or the interest on the partners' accounts— is $8437.16. What will be the balance of each partner's account when all the items have been properly entered ? NOTE.—Interest to be computed in months of 30 days. Ans. A's credit balance, $2505.75. B's credit balance, $17307.58. C’s credit balance, $16723.83. OPERATION. ACCOUNT CURRENT AND INTEREST ACCOUNT WITH PARTNERS. DR. A. CR. February 1, 11 months $11.00 interest $200 July 1, 6 months, - $72.00 int. $2400.00 March 1, 10 “ 20.00 & 4 400 By balance of interest 48.84 23.16 July 11, 5; “ 14.17 “ 500 October 1, 3, “ 3.00 é & 200 ‘‘ one-third net gain - - - 1482.59 Nov. 21, 1: “ 67 & 100. - *-m-mº mºst tº-ººººº. $3905.75 $48.84 $1400 1400.00 $2505.75 DR. B. CR. January 1, 12 months $600 interest $10000.00 April 1, 9 ** 225 « 5000.00 By interest - gº * «s sº 825.00 “ net gain - - - - - 1482.58 $17307.58 DR. C. CR. September 16, 33 months $8.75 interest $500.00 | January 1, 12 months, $600 interest $10000.00 * July 1, 6 & 4 150 & 4 5000.00 $750 By balance of int. 8.75 741.25 “one-third net gain - - 1482.58 Y $17223.83 500.00 $16723.83 OPERATION TO FIND THE NET GAIN. Total gain of the business is sº º sº gº tº tº * sº - - - - $8437.16 From this deduct as follows: Balance of interest due A. $ 23.16 & 4 & 4 && B. – 825.00 & & & 4 4 C. - 741.25 $1589.41 Salary paid to A. - - - - - - - 2400.00 3989.41 Wet gain of business - - - - - - - - - - $4447.75 A’s one-third net gain is * s =º sº tº - - - - $1482.59 B’s one-third of net gain is - & sº sº *- * sº sº gºe 1482.58 C’s one-third of net gain is - - - - - - * * * 1482.58 -848 SOULE's PHILOSOPHIC PRACTICAL MATHEMATICs. º Second Operation to adjust the Interest and Cain. Int. Dr. Int. Cr. Net, Int. Cr. Dr. Cr. A. $48.84 $ 72.00 $ 23.16 529.80 $506.64 B. 825.00 825.00 529.80 $295.20 C. 8.75 750.00 741.25 529.81 211.44 Total interest Cr. # $1589.41 $506.64 $506.64 Each partner's one-third int. $529.80% Total gain - - - - * - - tº- * $8437.16 From which deduct salary of A. - - - - - 2400.00 Net gain, exclusive of interest on partners' accounts # $6037.16 Each partner's one-third of net gain - - - - 2012.38% By this method of adjusting the matter, the partners' accounts will stand as follows: Dr. A. Cr. Dr. B. Cr. Dr. C. Cr. $200 $2400 $10000 $500 $10000 400 2012.39 5000 5000 -*-* - 295.20 211.44 500 2012.38 2012.39 200 44.12.39 - *- I ememº-mºs 100 1906.64 $17307.58 Cr. $17223.83 506.64 500.00 $1906.64 $2505.75 Cr. $16723.83 Cr. 50. A. and B. form a partnership, each investing $15,000. A. is to manage the business, and B., who is to be absent, agrees to pay a book-keeper to take his place in the house at $1200 per annum. The gains and losses are to be divided equally. The business was conducted for fifteen months, during which time the book- keeper drew his salary and charged the same to expense. The books were closed at the expiration of the fifteen months. What entry should be made to adjust the salary of the book-keeper? Ans. FIRST FORM OF ENTRY. SECOND FORM OF ENTRY. B. - - - - - - $750 B. - - - - - - $1500 To A. - - - - - - $750 To Profit and Loss, (or Expense) - $1500 For one-half of $1500 salary of book-keeper, For salary of book-keeper, the same having charged to expense instead of to B. been charged to expense when it should have been charged to B. f 51. C. and D. are partners, equal in gains and losses. , C. is to keep the books and manage the financial affairs, for which he is to receive $200 per month. D. is to superintend the purchasing and selling of goods, for which he is to receive $150 per month. At the close of two years, there was a cash gain of $9000, exclusive of the amount paid on account of salaries. How much is due to each partner, C. having drawn $2000 salary, and D. having drawn $2500 salary? Ans. Due to C. $5350. Due to D. $3650. OPERATION. Gain as above † - - * *- º - -> * - - - sº tº- - ‘º $9000 C’s salary 24 months @ $200 = * - - - sº - $4800 Less amount received on account - - gºs - 2000 C’s balance due on salary - tº- tº - - E-l - $2800 D’s salary 24 months @ $150 == - - - * - $3600 Less amount received on account - - º - 2590 D's balance due on salary - - - - Total liability of the firm for salaries - - - - - - - - - $3900 Net gain to be equally divided between C. and D. - - - - - - - $5100 Yºr PARTNERSHIP SETTLEMENTS. 849 C’s one-half net gain is - - - $2550 | D's one-half net gain is - - - $2550 “ balance of salary is - - - - 2800 || “ balance of salary is sº tº m 1100 ‘‘ interest in the $9000 is - º - $5350 | “ interest in the $9000 is - - - $3650 C’s interest $5350 —H D's interest $3650 = $9000. 52. W., X, Y, and Z. are partners. An interest account is to be kept with each partner, for his investments and withdrawals. W. is to share one-half of the gains and losses, and X, Y, and Z. are to share equally the remaining one-half. At the close of the fiscal year, the account current and interest accounts of the partners show the following interest balances: W. $1000 credit; X. $500 debit; Y. $1700 credit; and Z. $400 debit. What is the proper Journal entry on the books of the firm to adjust the matter? Ans. FIRST FORM OF ENTRY, SECOND FORM OF ENTRY. Sundries to Sundries Interest to Sundries - sº $2700 X. gº - º sº - $800 TO W. - * - t- - $1000 Z. Eº - s tº- º 700 ** Z. - *- - sº tº- 1700 To W. se - - º $ 100 “ Y. º - &- º 1400 | Sundries to Interest - tº- tº- 900 X. - - tº- - º 500 Z. - Gºs e- º - 400 OPERATION. RESOURCES : i.IABILITIES : PARTNERS’ INT, ACCOUNTS: Due by X. - - - $500 | Due to W. - - $1000 |ws int. credit - - $1000 ** Z. - - - 400 ** Y. – Gºs - 1700 “ “ liability = 900 Total interest resources $900 Total int. liabilities $2700 || “ net credit of int. $100 ‘‘ ‘‘ resources 900 s º — |X’s inf. debit. - * $500 Net “ liabilities $1800 “ “ liability - 300 W’s int. liabilities $900 “ total debit of int. $800 & 4 & 4 §: & 4 & 4 § ys int. credit º $1700 Z's # “ & 4 300 liability - 30 “ net credit of int. $1400 Z's int. debit - tº- $400 “ “ liability º 300 “ debit of int. º $700 NOTE.-We regard the first form of entry as the better method of adjusting the interest on partners' accounts. See problem 48. 53. Jones and Smith are partners, Jones two-thirds interest and Smith one- third interest in the gains and losses. An account current and interest account is to be kept with each partner. At the close of the fiscal year, Jones has $800 net debit of interest, and Smith has $100 net debit of interest. 1. What entry is required on the books to adjust the matter between the partners? 2. In case the firm had dissolved and settled all matters except the interest, what settlement should they make between themselves? Ans. 1. Jones to Smith $200. $200. 2. Jones should pay Smith, out of his private funds, $200, OPERATION. | Dr. Cr. * Dr. Cr. Jones owes - - - - $800 — $600 = $200 Smith owes * - º - 100 - 300 - $200 Total resources º cº- $900 Jones' two-thirds interest is $600 Smith's one-third interest is 300 - 85O SOULE'S PHILOSOPHIC PRACTICAL MATHEMATICS 54. Four mechanics, W., X, Y. and Z., form a partnership, for carrying on their trade. They are to share gains and losses as follows: W. 3, X. #, Y. 3, Z. 4. They have agreed that for all lost time they shall be charged respectively as follows: W. $4 per day, X. and Y. $3.50 each per day, and Z. $3 per day. At the end of the first year, it is found that they have lost time respectively as follows: W. 42 days, X. 31 days, Y. 53 days, Z. 27 days. What is the correct entry on the books of the firm to equitably adjust the matter among the partners? Ans. Sundries To Sundries, Y. - - - - - - - - $49.75 Z. ſº sº egº, * $º sº sº e 13.12 To W. & º sº tº gº ºne sº $35.62 To X. tº as * * * * * 27.25 OPERATION. W. owes for 42 days at $4 == $168.00 | W’s three-eighths of amount due is - $203.62 X. ‘‘ ‘‘ 31 “ 34 - 108.50 | X's two-eighths 4 & & 4 - 135.75 Y. 4 & & 4 53 & 4 3# F 185.50 Y’s { { & 4 & & * 135.75 Z. “ “ 27 << 3 == 81.00 Z's one-eighth & 4 & & tºº, 67.88 Total amount due the partners, $543,00 $543.00 Dr. Cr. Dr. CT. W. 8-, tº- #º Eº $168.00 $203.62 - $35.62 X. tº- sº º - 108.50 135.75 F 27.25 Y. tº- sº E-º #º- 185.50 135.75 - $49.75 Z. me *-> gº sº- 81.00 67.88 == 13.12 From these figures, we make the foregoing correct adjusting entry as shown in the answer. 55. A. and Z. are partners, equal in gains and losses. After business had been conducted for four months, it was agreed to admit W., and not wishing to close the books at that time it was agreed, after careful investigation and calculation, to allow A. and Z. $3000, as the net gain of the business up to that time; and that the gains and losses of the new partnership shall be shared equally (#) by each partner. What entry is necessary to adjust the $3000 gain between A. and Z. ? Ans. Profit and Loss To Sundries tº- - sº $3000 To A. tº gº- 3- sº $1500 To Z. tº- -> - sº 1500 (With explanation.) 56. A., B. and C. are partners. At the close of the year, A. retires from the business. Preparatory to closing the books and making a final settlement with A., it is agreed to allow 10% discount on all personal accounts due the firm, and 5% discount on all the bills receivable, as doubtful or bad debts. The personal accounts due amount to $21281. The bills receivable amount to $14500. What entry is necessary to adjust this anticipated loss? Ans. Profit and Loss, old account es; sº * tº tº tº sº. - $2853.10 * To Profit and Loss, new account - - ſº gº $2853.10 For 10% allowed as worthless, on $21281 personal accounts $2128.10 “ 5% allowed as worthless on $14500 bills receivable 725.00 Total amount sº tº- * e tº - $2853.10 From these figures, we make the adjusting entry as above. * PARTNERSHIP SETTLEMENTS. 85 I 57. X., Y. and Z. are partners in the wholesale grocery and commission business. At the end of their fiscal year, when they desire to close their books and ascertain their actual gain or loss, they find that “Com. Sales” account has a credit of $48500, resulting from the partial sales of consignments which are unclosed and for which no account sales have been rendered. They also find that “Charges Account” has been debited with $4050 for various charges paid on open consign- ments unsold and partially Sold. They also find that there is $1812 accrued interest due on matured bills receivable, and $280 due on bills payable. Allowing 24% commission on the $48500, what are the necessary entries to properly adjust the commission, charges and interest ? Ans. FNTRY FOR ADJUSTING THE COMMISSION. Commission new account tº ſº- * tºº e- gº g- $1212.50 To Commission old account *- * & * * * - $1212.50 For 24% on $48500 sales of unclosed consignments, per Com. Sales Ledger. ENTRY FOR ADJUSTING THE CHARGES. Charges new account - - - - - - - - - $4050 To Charges old account - - * =: Eºs º ºs º- $4050 For amount paid on unclosed consignments, per Com. Sales Ledger. ENTRY FOR ADJUSTING THE INTEREST. Interest new account - tº- * - sº gº sº es sº - ºr $1532 To Interest old account - - . - es $º º tº $º $1532 For Interest on bills receivable $1812 Less “ “ “ payable 280 Interest in favor of firm $1532 NOTE.-In all entries of this kind, whether new account is debtor to old account, or old account to new account, the new account part of the entry is not to be posted until the old account side is posted and the old account closed and ruled up. See Higher Work of Closing Ledgers in Soulé's New Science and Practice of Accounts. 58. The following question was sent to us by the cashier of a banking insti- tution in Texas, where accounts are kept in both coin and currency: “Our Bank receives deposits in currency and gold. In the regular course of business we have sold $15000 or $20000 gold belonging to depositors, and credited currency exchange account with the premium realized. We now wish to ascertain our net gain for the purpose of declaring a dividend, and as we will have to buy gold to pay our gold depositors when they demand their funds, it is clear that to close our books as the accounts now stand, we would show a false gain equal to the premium paid for gold. Under these circumstances, what entry must we make to adjust the matter and keep our books in correct balance? Gold is now ruling at 114% premium.” Ans. Bxchange old account To Exchange new account. The amount of debit and credit of this entry would be the premium on the gold sold, belonging to depositors, at the current rate per cent. The credit side of this entry would not be posted until the old account was closed and ruled up. 59. The following questions were sent to us by a Texas merchant: “I am doing a general merchandising business and keep accounts in both coin and currency. It often happens that a customer who owes a balance in coin wishes to pay it in currency, and that a 852 SOULE's PHILOSOPHIC PRACTICAL MATHEMATICS. ºt Customer who owes a balance in currency wishes to pay it in coin. Now, suppose the first customer to be Smith and the second Jones, and the balance in each case to be $550, and gold 10% premium, what would be the correct entries to reduce the coin balance to currency and the currency balance to coin, and to settle both accounts?” º Ans. JourNAL ENTRY TO REDUCE THE Coin BALANCE TO Currency. Smith, currency account To Sundries, - - - - - $605 To Smith, coin account - - - tº - $550 To Exchange (10% premium on gold) 55 CASH ENTRY TO SETTLE WITH SMITH. To Smith, (enter in currency cash column,) - - * - * * 605 JOURNAL ENTRY TO REDUCE THE Currency BALANCE TO Coin. Sundries To Jones, currency account - - - - 550 Jones, coin account - - - - - - - - - - - - $500 Exchange (10% premium on gold), - º- - - - - * *- 50 CASH ENTRY TO SETTLE WITH JONES. To Jones, (enter in gold cash column), - - - - - - - 500 A DIFFICULT SETTLEMENT AND JOURNAL ENTRY. 60. Smith, Jones and Brown are commercial partners, Smith one-half, and Jones and Brown one-fourth each in gains and losses. In the regular course of busi- ness, Smith, without consulting Jones and Brown, sells $1800 worth of merchandise, and receives a note for the same at four months, which the book-keeper entered in the books in the regular manner. Jones and Brown, on learning of the transaction, disapproved of it on the ground that the maker of the note is an irresponsible person, and proposed to Smith to sell to him their respective interests at 25% discount, to which proposition Smith agreed. Jones and Brown then indorsed the note over to Smith, who leaves it in the business, and the book-keeper is instructed to make the necessary entry. What is the correct resultant entry 3 Ans. Sundries To Smith - - - - - - º - $225 Jones - - - - - - - - - - - - - $112.50 Brown - - - - - - - - - - - - - 112.50 Statement of the several entries which, when the debits and credits are can- celled to the lowest amounts, give the resultant entry. First entry on receiving the note: Bills receivable gº +- - tº- - - e. - tº- a- $1800 To Merchandise - - - - sº - - &= $1800 Second entry, when Jones and Brown each take from the business $450 worth of property to sell to Smith at 25% discount: Sundries to Bills receivable - - - - - - tº º $900 Jones g- - º - - º - - tº - tº º $450 Brown ºp - •º - & º e- sº Acº --> º º 450 PARTNERSHIP SETTLEMENTS. 853 Third entry, when Jones and Brown sell to Smith at a discount of 25% the property, bills receivable, taken out of the business: { - - - - - $675 - $337.50 º º sº - 337.50 Fourth entry, when Smith invests and guarantees the $900 worth of property bought for $675. Smith to Sundries - tº {- º - “ Jones -> º - -> - º º - º- ** Brown - º - º - ae Bills receivable - 4- - - - -> *- - º - $900 To Smith tº-> - - - - - - º - $900 NOTE.-If it is preferred, the whole note may be taken out of the firm b º y the partners, before making the sale to Smith, and the other entries made to correspond. º Without making the above entries, the 4 proper debits and credits may be determined by the following process of reasoning: Jones and Brown, fearing they may lose $450 each at the maturity of the note, now agree with their partner, Smith, to allow him $112.50 each, ($25% of $450) if he will secure them against this anticipated loss of $450 each. In consideration of the $112.50 from each, Smith agrees or guarantees to protect them against the antic, ipated loss, and hence each owes him $112.50. NOTE.-In case the note proves worthless, Smith will be debited and Bills Receivable credited. 61. W., X, Y. and Z. were partners, each one-fourth in gains and losses. They dissolved partnership, paid all their outstanding liabilities, and divided all their resources. They then examine their books and find the following balances to the credit of each partner: W. $1100; X. $240; Y. $810; Z. $680. What settle- ment must be made between the partners to adjust the matter, and what entry must be made on the books when the final settlement is made 3 Ans. Sundries To Sundries, W. - - - - - - - - - - - - - - $392.50 Y. - - - - - - - - - - - - - 102.50 To X. - - * - s º º - -- $467.50 To Z. - - - - - - - - - 27.50 OPERATION. Cr. Dr. Dr. Cr. Amount due W. * - * - $1100.00 $707.50 -: $392.50 & 4 “ X. º - º 240.00 707.50 > $467.50 & 4 “ Y. º - - 810.00 707.50 – 102.50 << * * Z. * - º 680.00 707.50 – 27.50 Total liabilities is - +) $2830.00 $495.00 $495.00 Each partner's 4 of liabilities $ 707.50 By this operation we see that there is due to W. $392.50, and to Y. $102.50, of which X. owes $467.50 and Z. $27.50. When x. and Z. pay to W. and Y. the respective amounts due by them, it is clear that W. and Y. will then owe them, and hence the above adjusting entry. 854 SOULE's PHILOSOPHIC PRACTICAL MATHEMATICS. X- OPENING ENTRY TO CHANGE SINGLE ENTRY BOOKS TO DOUBLE ENTRY, AND USE THE SINGLE ENTRY LEDGER. 62. A. S. Blaffer and P. Birba are commercial partners, equal in gains and losses. They are keeping their books by the Single Entry System, but desire to change them to the Double Entry System, and use the present Single Entry Ledger. The following formula of work accomplishes the desired object : NEW ORLEANs, May 1, 1893. For the purpose of changing our books from single entry to double entry, we make the following abstract of our commercial affairs this date: IT E M S. RESOURCES, LIABILITIES. Merchandise per stock book tº- * = tºº g- - 17000|00 Cash on hand and in bank - s - e - -º- gº * - 7800|00 Bills receivable, Nos. 1, 2, 3 and 4 sms º º me * 2500|00 Canal Bank stock - sº tº sº tº º m s * 100000 Bills payable, Nos. 1 and 2 – * 185000 Our resources and liabilities shown by our Single Entry Ledger, are as follows: A. S. Blaffer's account - -3 º 1150.00 850000 P. Birba's account - gº -: º 180000 1200000 tºº Dr. 910000 231 personal accounts - tº e ſº sº 67 personal accounts - ºn tº - - - Cr. 4400|00 Total amount of resources - s tº gº tºº & 403:50.00 Total amount of liabilities tº e º ºr tº º tº 267:50.00 26750|00 Net gain to date - sº tºº sº * * e tº e tº 13600|00 A. S. Blaffer's one-half net gain is gº ºs º gº me 680000 P. Birba's one-half net gain is * tº &= * * *- 680000 Approved : A. S. BLAFFER, P. BIRBA. From the foregoing statement, the following Journal entry is made, which converts the Single Entry Ledger into a Double Entry Ledger: FIRST FORM OF ENTRY. SECOND FORM OF ENTRY. Sundries to Sundries Sundries to Sundries v/A. S. Blaffer, (amount posted) $ 1150 v.A. S. Blaffer, (amount posted) $ 1150 1/P. Birba, (amount posted) - 1800 VP. Birba, (amount posted) - 1800 1/231 personal accounts, (posted) 9100 1/231 personal accounts, (posted) 9100 Mdse. per inventory - - - 17000 Mdse. per inventory - - - 17000 Cash on hand tº dº º me tº 7800 Cash on hand - - - - - - 7800 Bills receivable - - - - - 2500 Bills receivable - - - - - 2500 Canal Bank stock - - - - 1000 Canal Bank stock - - - - 1000 * 1/ To A. S. Blaffer, (am’t posted) $ 8500 | V To A. S. Blaffer, (am’t posted) $8500 V “ P. Birba, (amount posted) 12000 4 & 6 & “ # net gain 6800 w/ “ 67 personal acc’ts, (posted) 4400 | V “ P. Birba, (am’t posted) - 12000 “ P. and L., net gain to date 13600 & & 4 & # net gain 6800 “ Bills payable - - - - - 1850 | V “ 67 personal acc’ts, (posted) 4400 “Bills payable - - - - - 1850 NoTE.—The debits and credits, in either of the forms of entry, that are checked and marked “posted,” must not be reposted. *: SETTLEMENTS. 855 PARTNERSHIP TO OPEN NEW BOOKS FROM AN OLD SET WHICH HAS BEEN INCORRECTLY REPT. 63. A. and Z. are partners, doing a mercantile and manufacturing business, and their books are, in their own language, “in an awfully mixed condition.” They therefore desire a new set opened, and accordingly employ an accountant to perform the duty. How should he proceed? Ans. He should first, on loose paper, from the old books, memorandum records of all kinds, and from the partners, ascertain the present resources and liabilities of the firm. them as follows: He should then arrange List of the Resources and Liabilities of A. and Z., January 1st, 1893, as shown by their old irregularly kept books, the inventory of stock on hand, and memoran- dum accounts; the same being agreed to by the members of the firm, and arranged as per subjoined statement to facilitate the opening of a new set of books according to the principles of double entry. RESOURCES : Mdse. per account of stock - - * Cash on hand - - - * - - 10 shares Germania Bank stock, valued $14218 9210 at $130 º - - - - tºs 1300 5 shares N. O and T. R. R. stock, valued at $90 * e- * sº - *- 450 Mill, machinery, tools, etc. - - – 11280 Bills receivable per B. B. º * º 4210 Personal accounts due the firm, as per the following Journal entry, the names being omitted to economize space - 7460 A’s withdrawals in excess of his invest- ments - - - e- - †- 3100 Total amount of resources - - $51228 From the foregoing statement, the following Journal entry is made, opens the new set of books: Sundries To Sundries Molse. - -> - - - $14218 Cash - - - - - - 9210 Germania Bank stock º *- 1300 N. O. & T. R. R. stock - tº- 450 Mill, machinery, tools, etc. - 11280 Bills receivable - * - 4210 81 personal accounts, 7460 (Insert the 81 names) To A. * * * $ 7115 ** Z. º -> º 37058 “ bills payable - 1280 “ personal accounts 2675 (Insert the names of the 33 persons owed.) LIABILITIES : Z's net investment - * - º - $26843 9 personal accounts due by the firm as per old Ledger - - - tº- - 1750 24 personal accounts due by the firm as per memorandums - - †- - 925 Bills payable as per B. B. - tº - 1280. Total liabilities - - - - - $30798 Which deducted from the resources as above - sº - º - 51228 Gives the net gain to date - - - $20430 which A's one-half of net gain is - * - - $10215 LErom which deduct his net withdrawals 3100 Gives his present net capital - {-} - $ 7115 Z's one-half net gain is - * * - 10215 To which add his net investment - - 26843 Gives his present net capital - - - $37058 NOTE.—After this entry is made, the amount of cash should be entered in the Cash Book as amount on hand. 64. A. and B. are partners, equal in gains and losses. A. Owns one-fourth and B. owns three-fourths of the net capital. The resources and liabilities at commencing, not including the partners' investments, were as follows: Cash -> - $ 5000 Mdse. -> sº 15000 RESOURCES: { Bills receivable 2000 Bank stock - 3000 Personal accounts 8000 Total resources - E- $33000 ſ Bills payable – $15000 LIABILITIES: t Personal accounts 6000 Total liabilities $21000 Total resources 33000 Net capital of the firm - $12000 (Continued.) 856 SOULE's PHILOSOPHIC PRACTICAL MATHEMATICs. jºr The resources and liabilities at the close of the fiscal year, not including the Partners' investments, were as follows: * Cash - - - $ 3000 Mdse. - - * 12000 #. . º 9000 ſ Bills payable - $12000 e SOD1 RESOURCES: Bank it. *ounts º º LIABILITIES: | Personal accounts 7000 Real estate - G- 4000 Total liabilities $19000 Total resources $40000 What has been the net gain, and what is each partner's net capital * Ans. Net gain $9000. A’s net capital $7500. B’s net capital $13500. OPERATION. Resources at commencing as above - $33000 |A's one-fourth capital at commencing $3000 Liabilities at commencing as above - 21000 || B's three-fourths capital at commencing 9000 Net capital at commencing as above $12000 Firm’s capital - - - - $12000 Resources at closing as above - - $40000 || A's one-fourth net capital at commencing $3000 Liabilities at closing as above - - . 19000 || A's one-half met gain - - - - 4500 Net capital at closing - - - 21000 || A's net capital at closing - - - $7500 Less net capital at commencing ' - 12000 B’s three-fourths net capital at commencing $9000 Net gain during business - - $9000 || B's one-half net gain - - - - 4500 B’s net capital at closing - º- - - $13500 A's net capital $7500 + B's net capital $13500 = $21000 = the net capital of the firm at closing. 65. J. Jones, J. E. Soulé and B. B. Euston are partners, doing business under the name and firm of Jones, Soulé & Euston. They commenced business January 1, 1893. The gains and losses are to be shared equally, each one-third. July 1, 1893, Jones retires from the concern, and Soulé and Euston continue the busi- ness and liquidate the affairs of the old firm. In order to settle with J. Jones, July 1, the books are to be adjusted so that his account may exhibit the cash balance due to him on that date. With a view to effect this, the following facts have been ascer- tained and agreed to by all of the partners: The merchandise on hand per account of stock is valued at $49528. The merchandise on hand belonging to Mdse. in joint account with H. Davis, each one-half, is valued at $4236. Charges incurred, but not posted to said account, amount to $210. The commission on sales is 24%. The charges incurred, but not posted, on “Commission Sales Account,” amount to $687.65, and the commission on the sales effected, but not posted, amounts to $915.18. It is agreed to allow the new firm 10% on all personal accounts receivable and bills receivable, to cover doubtful debts. Of the cash on hand, $500 is gold, which is worth 10% premium. The balance of S. Brown's account is payable in San Francisco, and exchange is 2% discount. Interest is to be charged on all accounts due the firm, and allowed jºk 857 PARTNERSHIP SETTLEMENTS. on all accounts that the firm owe, at 6%; and also on the investments and with- drawals of the partners, at the same rate. The following shows the face of the Ledger June 30: Due by Due b A. Dr. Cr. A. J. Jones’ capital account - - - - - - $3000000|| Jan. 1 May 4|| $1850,00|| “... “ current account - - - - - - J. E. Soulé's capital account - º - º- - 30000|00|| Jan. | 1 April 20 2408:00|| “. current account - tº- º tº- - B. B. Euston's capital account - * - tº tº- 30000|00|| Jan., | 1 June | 3 1320|00 & 4 current account - 4- - - - Bills payable - * *- vº - º - 10400;00|| Nov. 4 Aug. 20) 5084000|| Bills receivable - - - - * - tº sº 14250|60|| Cash sº tº- tº º tº tº cº- 7141085|| Merchandise - tº- - tº - tº - º - 31581|50 9810|00|| Commission sales, - ºt - º - - - 42138||00 (The balance $32328, due by average August 19.) 8621|00|| Merchandise in joint account with H. Davis - - 14140,00|| July 10 (The debit is our one-half investment.) 7412,40|| Expense º - º - º - º - 810|30|| Interest tº- - - - vº - º - 142070 2130,00| Exchange - - º - tº - º - 3816|40 641815|| Charges * *- sº º sº - º º 451.1|80 230|00|| Commission º º- tº º tº tº * - 7913|10 5450|00|| Profit and LOSS - - ſº a- - - - tº- 800|00 H. Davis sºs &- º - tº tº- - 1200000|| Aug. 17 May 28|| 1340000|| S. Brown º - se - - - * - July 16|| 17815,00|| J. Lewis e- ſº- * - * - E - | W. Wood ºs - sº - tº - * - 445000|| April | 1 Jan. [15 5000|00|| G. B. Bracket e - tº - tº - º - Mar. 3 9405|00|| A. Williams sº- - º - * - ºs - M. Moses º - º ºs * - sº - 5409|80|| Sept. 11 |$228581130 |$228581|30 What are the Journal entries to be made, in accordance with the foregoing agreements and exhibit of facts, to effect the desired settlement with the retiring partner, J. Jones? Mdse. in Joint Account with H. Davis, new account, To Mdse. in Joint Account with H. Davis, old account, For one-half value of Mdse. unsold. Mdse. in Joint Account with H. Davis To Charges “Commission at 24% on sales “ H. Davis, his one-half net proceeds “Profit and Loss, our one-half net gain tentry to credit Charges with the amount due, but not posted, on “Com. Sales”). Charges, new account, $687.65 Ans. (Entry to credit Merchandise in Joint Account with H. Davis). $2118 $2118 (Entry to close the account of Merchandise in Joint Account with H. Davis), To Charges, old account, To Sundries, Commission, new account, To Commission, old account, $7637 $210.00 353.50 6788.25 285.25 (Entry to credit Com. with the amount due, but not posted, on “Com. Sales”). - $915.18 $915.18 858 Yºr SOULE's PHILOSOPHIC PRACTICAL MATHEMATICS. (Entry to allow 10% on all Personal Accounts Receivable and Bills Receivable due the Firm). (Entry to adjust the Premium on Gold and the Discount on S. Brown's account). Profit and Loss, old account, - $9646 Exchange, old account, i-e * $218 To Exchange, new account, $218 To Profit and Loss, mew account, $9646 For 2% disc. on $13400 = $268 Less 10% prem. on 500 gold = 50 (Entry to adjust the Interest on the Partners' | Or, as many accountants would make the entry. Investments and Withdrawals). thus: Sundries To B. B. Euston - su. to Sundries To Interest - - $52.00 J. Jones * gº º sº. sº .24 J. Jones, 57 ds. (a) 6% on $1850 $17,57 J. E. Soulé - - - - 11.16 J. E. Soulé, 71 “ “6% “ 2408 28.49 B. B. Euston, 27 “ “6% “ 1320 5.94 # Or, considering the interests on the investments for 181 days at 6%, and on the withdrawals as in the preceding entry, thus: Interest To Sundries - tº-3 4- tº º tº ºt sº * * $2663 ** J. Jones tº- :- as tºº s wº †-º $887.43 ** J. E. Soulé - sº iº 4- gº gº- sº 876.51 ** B. B. Euston & ſº tºº gº aº gº tº 899,06 (Entry to adjust the Interest or Discount due the Firm on Personal Accounts matured and unmatured, and on Bills Payable unmatured). Interest, new account, - - - - - - - - - $1061.22 To Interest, old account, - &= - dº e- $1061.22 For accrued interest on the following credit balances: S. Brown, $13400.00 for 33 ds. (a) 6% $ 73.70 | H. Davis, 6788.25 for 10 ds. at 6%, 11.31 G. B. Brackett, 5000.00 “ 166 “ “6% 138.33 || M. Moses, A. Williams, 9405.00 ** 119 “ “6% 186.53 Com. Sales, 5409.80 “ 73 “ “ 6%. 65.82 32328.00 “ 50 “ “6%. 269.40 H. Davis, 12000.00 “ 48 “ “6% 96.00 | Bills Payable 10400.00 “ 127 “ “ 6%. 220.13 (Entry to adjust the Interest or Discount due by the Firm on Personal Accounts and Bills Receivable). Interest, old account, - - - - - - - - - - $546.40 To Interest, new account, - - - - - - $546.40 For accrued interest on the following debit balances: J. Lews, $17815 for 16 days a 6% - - $ 47.51 Bills receivable, 50840 “ 51 “ “6% - - 432.14 W. Wood, 4450 “ 90 “ “ 6% - - 66.75 NOTE.-When the foregoing entries shall have been posted, except the new account debits and credits, the gains and losses will be ascertained in the usual way, and the books closed. Profit and Loss should be closed to the partners' current account, and their current account should be closed to their capital account. TEIE ADMISSION OF A NEW PARTNER INTO A COMMERCIAL FIRM. 66. There are many reasons for admitting new partners into commercial houses. i. Some partners are admitted for their money. 2. Some for their reputation. 3. Others for their knowledge or skill. There are also many ways of admitting partners. 1. Some are admitted to an agreed upon interest in the gains and losses, by making a specified cash or note investment. 2. Some, by paying to the old members of the firm, in cash or notes, jk PARTNERSHIP SETTLEMENTS. 859 a stated price for 4, 3, #, or § interest, of the gains and losses. 3. By paying to the members of the firm, a specified sum for a part or share of their respective interests. 4. By paying to the firm a specified sum for 3, #, +, or § interest or ownership in the entire stock or capital of the firm, and a certain share of the gains and losses of the new firm. There are also other ways of admitting new partners, but these are the most general, and we shall briefly consider the entries to be made under the four cases named. In the first case, where a specified cash investment was made for a certain share in the gains and losses, cash would be debited and the party credited for the amount invested. In the second case, where a specified bonus of cash or note payment was made to members of the old firm for a certain interest in the gains and losses, no entry would be made in the books for the cash or notes received by the old members, unless they should make an investment of the same in the firm. They sold only an interest of their respective interests in the gain and losses of the business, and the only thing necessary to be done is to divide the amount received between the old partners, or invest it to their respective credits in proportion to the interest sold, and then to draw new articles of agreement Specifying each partner's interest. To illustrate, let us suppose that Jones and Smith are partners, Jones 3 and Smith # in gains and losses, and that Brown is admitted as a partner with # interest in the gains and losses of the business on the payment of a bonus of $10000 cash, and that Jones and Smith Sell to Brown the # interest in proportion to their respective shares, § and #, and invest the same in the business. . What is the entry, and what are the respective interests of the three partners in the new firm 3 The cash entry in Journal form, is as follows: Cash to Sundries º - tº- a- tº- gº * - º e $10000 To Jones, his two-thirds of $10000 - - - *g $6666.67 “Smith, his one-third of $10000 - - * ge 3333.33 The partners' respective interests are now as follows: Jones four-ninths, Smith two-ninths, and Brown three-ninths of the gains and losses of the new business. In the third case, the old partners who sold, should be debited each for the proportion sold and the new partner credited for the amount bought. To illustrate this, we will again suppose that Jones and Smith are partners, Jones 3 and Smith # in gains and losses, and that Brown pays to Jones and Smith $12000 for a partnership, and # interest in the gains and losses, and that they sell the # interest in proportion to their present shares. What is the entry, and what is the respective interest of each partner in the gains and losses 3 The entry is as follows: Sundries to Brown º -> - º - - « » - - - - $12000 Jones, his two-thirds of $12000 - - - - - - - - - $8000 Smith, his one-third of $12000 - º - - sº - * tº- - 4000 The partners' interests are, Jones four-ninths, Smith two-ninths, and Brown three-ninths. Should Jones and Smith invest this money in the firm, Cash would be debited and they credited. • 86O SOULE's PHILOSOPHIC PRACTICAL MATHEMATICs. H º In the fourth case, where a specified amount was paid for an interest or owner- ship of a certain part of the whole capital of a firm, the amount paid for the interest should be divided between the old partners in proportion to the net capital of each; and then the amount of the one-fourth interest sold should be credited to the pur. chaser, and the old partners debited for the same in proportion to their respective net capital. To illustrate this, we will suppose Soulé and Weiss to compose a firm that sells to Harris # of the net capital of their firm for $30000 cash, and that, while they are now, Soulé # and Weiss + in gains and losses, in the new firm the interests will be Soulé #, Weiss +, and Harris 4. We will further suppose that the books have been closed and exhibit the following statement of accounts: SouL.E.' ' WEISS. CASH. MDSE. | $100000 $10000 $9000 $90000 BILLS RECV. BILLS PAY. H. KENT. $5000 $6000 $10000 This exhibit shows the net resources to be $110000, 4 of which is $27500. For this $27500 and a partnership with one-fourth interest in gains and losses, Harris pays $30000. This $30000 divided between Soulé and Weiss, in proportion to their net capital, gives Soulé $27272.73, and Weiss $2727.27. * 3. The $27500 being the one-fourth of the net capital purchased by Harris, must be credited to him, and Soulé and Weiss debited for the same in proportion to their respective net capital. The following is the adjusting entry: w Sundries to Harris - gº tº- * * * ſº- $27500 Soulé, his proportion - - - $25000 Weiss, his proportion - - - 2500 Articles of Agreement should be drawn by the new firm specifying the invest- ments, interests in gains and losses, whether interest is to be allowed on the part- ners' investments and charged on their withdrawals, etc., etc. See Articles of Agree- ment a few pages further On. Should Soulé and Weiss invest the money received for the sale of their one- fourth net capital, cash would be debited and they credited in the usual manner. A party desiring to purchase an interest in any business should carefully examine the accounts and the goods in stock. He should give close attention to the loss and gain accounts and to the personal accounts due the firm. He should have an account of stocktaken in the presence of some person who is thoroughly informed on the value of the line of goods or material in which the house deals. *IIIsi, and Parlieri Atmº. =N * A Ama A_t v--~~~ *--w 1426. For a definition of Partnership, and a statement of the different classes of Partnership, the Formation of Partnerships, the Dissolution of Partnerships, Articles of Partnerships, Partnership Settlements, etc., see pages 1 to 5. 1427. Partnership Average is the process of finding what proportional amount of the gain and loss is to be given to each member of the firm on final set- tlement, or at any specified time. Sharing Gains and Losses. The amount of gain or loss to be apportioned to each partner, depends upon the following conditions: 1. The gain or loss may be divided in proportion to the net investment or capital of each partner, or some other agreed upon proportional basis. 2. By allowing each partner a salary, and dividing the remainder of the profits according to a fixed proportion. 3. By allowing each partner interest on his net investment, and dividing the remainder of the profits according to a fixed pro- portion. 4. By allowing each partner interest on his net investment, and a salary for services rendered, for skill possessed, or for credit or good-will, and then divid- ing the remainder of the profits according to some fixed proportion. See Partner- ship, page 1. CLASSIFICATION. The subject is classified into simple and compound partnership average. 1428. Simple Partnership Average is the process of finding the proportional amount of gain or loss to be apportioned to each member of the firm when the net capital of each has been invested for the same length of time; or, in brief, it is the proportional division of the gain or loss between the partners, according to the capi- tal invested by each. 1429. Compound Partnership Average is the process of finding the propor. tional amount of gain or loss to be apportioned to each member of the firm when the net capital of each has not been invested for the same length of time; or, in brief, it is the proportional division of the gain or loss between the partners, according to their average met investments and the periods of time that they were in use by the firm. SIMPLE PARTNERSEIIP AVERAGE PROBLEMS. 67. Jones, Smith and Brown contracted to do business as partners for one year. Jones invested $5000, Smith $4000, and Brown $1500. The gain was $3150. What was the proportional share of each partner ? Ans. Jones $1500. Smith $1200. Brown $450. (Continued.) (861). 862 soul E's PHILOSOPHIC PRACTICAL MATHEMATICS. 4× FIRST OPERATION. Statement to find Statement to find Statement to find Jones' share. Smith's share. Brown's share. Jones’ capital - $5000 Smith’s “ - 4000 $ GAIN. $ GAIN. $ GAIN. Brown’s “ - 1500 3150 3150 3150 - 10500 || 5000 10500 | 4000 10500 | 1500 Total capital $10500 - $1500 $1200 $ 450 Explanation.—By adding the individual capital of each partner, we obtain a total capital of $10500, which is the amount that gained and is to receive the $31.50; and as the gain is to be divided in proportion to the respective capital of each partner, we place the gain on our statement line and reason thus: If $10500 capital gains $3150, $1 will gain the 10500th part, and $5000 (Jones’ capital) will gain 5000 times as much. The result of this statement gives us $1500 as Jones' share of the gain. The reasoning for the statements to obtain Smith's and Brown's respective shares is the same as that in the statement to obtain Jones' share, and hence is omitted. SECOND OPERATION. º Statement to find Statement to find | Statement to find Jones' capital - $5% $10 Jones' share. Smith’s share. Brown’s share. Smith’s “ +- 49% 8 $ *... $... Brown’s “ - I 5% 3 21 #0 t 1 | #so 21 gº Total capital - $21 $1500 $1200 $150 Explanation.—In this solution, we cancel the investments of the respective partners by divid- ing each amount by 500, which lessens the figures in the operation without changing the ratios, The same reasoning is applied as given in the first Solution, and hence is omitted. THIRD OPERATION. Statement to find the rate per cent. gain. Jones' capital - - - - $5000 30% of which is $1500 $ GAIN. Smith’s “ - - - - 4000 30% “ “ 1200 3150 Brown's “ - - - - 1500 30% “ “ 450 10500 | 100 3) Total capital - - - $10500 30% gain. 68. A. and Z. formed a partnership for the purpose of speculating in various kinds of business. A. invested $8400, and Z. $6000. At the close of the business, the net loss amounted to $1728. What was each partners' proportional share" Ans. A’s $1008. Z's $720. OPERATION Statement to find A’s loss. Statement to find Z's loss. A’s investment - $84% Z’s 4% º 6000 1728 1728 º 144 | 84 144 60 $144 $1008 $720 69. W., X, Y, and Z. entered into partnership for the purpose of conduct- ing a general merchandising business. W. invested $3500, X. $3000, Y. $2500, and Z. $1000. At the expiration of the partnership, the net gain was $15000. What was each partner's proportional share ? Ans. W. $5250. X. $4500. Y. $3750. Z. $1500. 70. Three young gentlemen, A., O. and P., contracted for the use of a carriage for thirty days for $100, which they agreed to pay in proportion to the number of * # .* PARTNERSHIP SETTLEMENTS. 863 days that each had the use of it. A. used it 9 days; O. 14 days; and P. 6 days, and 1 day it was not in use. What is the amount due by each 3 Ans. A. $31,03. O. $48.28. P. $20.69. 71. A father has $2000 which he wishes to divide among his three children, in proportion to their ages, which are 6, 8 and 11 years. What amount will each receive? Ans. 6 years, $480. 8 years, $640. 11 years, $880. OPERATION. 6 + 8 + 11 = 25 years. Statement to find the amount | Statement to find the amount | Statement to find the amount for the child of 6 years. for the child of 8 years. for the child of 11 years. $ $ $ 2000 2000 2000 25 | 6 25 || 8 25 || 11 $480 $640 $880 COMPOUND PARTNERSEIIP AVERAGE PROBLEMIS. 72. Jones, Smith and Brown associated themselves together in business, and agreed to share the gains and losses in proportion to their respective investments and the time the same were employed by the firm. Jones invested $4000 for 12 months; Smith invested $3500 for 9 months; Brown invested $2000 for 8 months. The gain during the business was $9550. What was each partner's share ? Ans. Jones $4800; Smith $3150. |Brown $1600. OPERATION. Mos. Investment for i mo. Jones invested - tº- tº- - $4000 × 12 = $48000 Smith. “ <- tº- gº tº 3500 × 9 = 31500 BrOWn. “ * > gº g- • * 2000 × 8 = 16000 $95500 Statement to find Jones' share. [Statement to find Smith's share. Statement to find Brown’s share. $ $ $ 9550 9550 9550 95500 || 48000 95500 || 31500 95500 | 16000 -$ 4800 $ 3150 | $ 1600 Ea:planation.—In this problem, the gain being shared in proportion to the money invested and the time for which it was invested, and the amounts and periods of time of each partner being different, we must, therefore, find the respective equivalent investment of each partner for 1 month. This is done, as shown in the operation, by multiplying the amount invested by the time that it was invested. The reasoning for the work is as follows: $4000 invested for 12 months is equivalent to 12 times $4000 for 1 month, which is $48000; then $3500 invested for 9 months is equivalent to 9 times as many dollars for 1 month, which is $31500; and then $2000 invested for 8 months is equivalent to 8 times as many dollars for 1 month, which is $16000. By this work, we obtain the respective investments of each partner for 1 month. We then add the same, to obtain the total capitalinvested for 1 month, and then proceed to find each partner's share of the gain by our line statements, in the same manner and with the same reasoning that is presented in the first problem of Simple Part- nership Average. 73. A. commenced business January 1, 1893, with a capital of $2500; on the 1st of May, he entered into partnership with B. who invested $3000; and on the * 3. 864 SOULE's PHILOSOPHIC PRACTICAL MATHEMATICs. § { 1st of October, C. was admitted as a third partner and invested $4000. At the close - of the year, there was a gain of $5500. What was each partner's apportionment of the gain? Ans. A’s $2500. B's $2000. C's $1000. FIRST OPERATION. 1893. Mos. Am’t for 1 mo. January 1, A. invested - - - $2500 × 12 = $30000 May 1, B. & 4 +- g- s 3000 × 8 = 24000 October 1, C. & & - tº- ºr 4000 × 3 = 12000 , Total capital for 1 month, - - - - - $66000 Statement to find A’s share. Statement to find B's share. Statement to find C’s share GAIN. $ GAIN. $ G. 5500 e 5500 5500 66000 || 30000 66000 || 24000 66000 || 12000 $2500 $2000 $1000 SECOND OPERATION. To find the respective shares of each partner by first finding the rate per cent. gain, we proceed thus: #500 A’s capital for 1 month is $30000, 84% of which is - $2500 66000 || 100 B’s “ “ 1 “ “ 24000, 84% of which is * - 2000 C’s “ “ 1 “ “ 12000, 84% of which is tº- 1000 84% gain. Explanation.—In the first part of the operation of this problem, we could have shortened the work by cancellation, but in order to more fully elucidate this subject, we did not cancel. 74. W., X, Y, and Z. engaged the services of a teacher for 10 months for $600, and agreed to pay the same in proportion to the number of children sent and the time that they attend School. - W. sent 3 for 200 days; X. sent 2, 1 for 200 days and 1 for 120 days; Y. sent 5, 3 for 180 days, 1 for 150 days, and 1 for 90 days; Z. sent 4, 1 for 200 days, and 3 for 160 days. What amount was due from each? Ans. W. $151.26. X. $80,67. Y. $196.64. Z. $171.43. OPERATION. No. sent. aiº. No. º W. e - - - 3 200 600 1 200 = 200 X. - - - - 1 #} = {}} 300 3 180 = 540 Y. e - - * > 1 150 - 150 780 1 90 = 90 1. 200 = 200 Z. - - - - 3 160 = 480 } 680 Total number for 1 day -> - - - 2380 Statement to find the Statement to find the Statement to find the Statement to find the amount due by W. amount due by X. amount due by Y. amount due by Z. $ $ $ 600 600 600 600 2380 600 2380 || 320 2380 || 780 2380 | 680 $151.26 $80.67s $196.64 $171.43 Explanation.—In problems of this character, where there are several parties to receive the #. or contribute to the loss or expense, it would be better first to find the rate per cent. as shown preceding problems. PARTNERSHIP SETTLEMENTS. 865 75. A. and B. formed a partnership in which A. invested $8000 for 10 months, B. $5000 for 12 months. They lost, during the partnership, $700. What was the loss of each partner? Ans. A. $400. B. $300. 76. X., Y. and Z. executed a piece of work for which they received $500. X. worked 15 days of 8 hours each; Y. worked 15 days of 10 hours each; and Z. worked 12 days of 6 hours each. How should the $500 be divided among them ? Ans. X. $175.44. Y. $219.30. Z. $105.26. 77. Jones, Smith and Brown, on the 1st of January, 1893, associated them- selves together as partners for the term of 1 year. January 1, Jones invested $10000; May 1, he drew out $4000; October 1, he drew out $2000; and December 1, he invested $1000. * January 1, Smith invested $6000; July 1, he invested $2000; November 1, he drew out $1000. March 1, Brown invested $4000; April 1, he invested $1000; August 1, he drew out $3000; November 1, he drew out $2500. The gain at the close of the year amounted to $9700. What was each part- ner's share of the gain, and what is Brown's indebtedness to the firm 3 -- Ans. Jones' gain, $4150. Smith's gain, $4100; Brown's gain, $1450, and indebtedness, $500. . FIRST OPERATION. Am’t Invested Mos. for 1 month. January 1, Jones invested - - - $10000 4 $40000 May 1, “ withdrew - ſº º 4000 º $6000 5 30000 October 1, “ withdrew - a º 2000 $4000 2 8000 December 1, “ invested & ſº º sº 1000 $5000 1 5000 Jones’ capital for 1 month tº gº º sm º ºr ºl $83000 Am't Invested s Mos. #. 1 . January 1, Smith invested - - - $6000 6 $36000 July 1, 4 & é & gº tº ſº 2000 $8000 4 32000 November 1, “ withdrew - - tºº 1000 $7000 2 14000 Smith's capital for 1 month - - - - - - - 82000 Am't Invested Mos. for 1 . March 1, Brown invested - - tºº $4000 1 $ 4000 April 1, & 4 4 & * > tº 1000 $5000 4 20000 August 1, ** withdrew - - tºs 3000 $2000 3 6000 November 1, ** withdrew - sº g- 2500 Excess of withdrawals, or amount due the firm by Brown {-º º sº * º $500 2 $30000 Excess of withdrawals for 2 months deducted - - 1000 Brown's capital for 1 month - - - - - 29000 Total capital for 1 month gº ºn tº sº º $194000 366 soulE's PHILOSOPHIC PRACTICAL MATHEMATICS. * Having the total capital for 1 month, to find each partner's share of the gain, We have but to proceed as shown in preceding problems, either by the line state- ment or per centum. SECOND OPERATION. In problems of this character, where there are several investments and with- drawals by the different members of the firm, it is best first to average the account of each partner. The following operation will elucidate this method: Dr. JONIES. - CR; Mos. Mos. Jollars 1893. OS *...] 1893. OS zºº. May 1, $4000 8 = - - $32000 January 1, $10000 12 = - - $120000 October 1, 2000 3 = - 6000 | December 1, 1000 1 = tº 1000 $38000 $121000 38000 Balance of dollars for 1 month gº º * = tº $83000 DR. SIMITEI. CR. Mos. I)oll & ID 1893. OS fº. 1893. Mos tºº. November 1, $1000 2 - - - $2000 || January 1, $6000 12 = - - $72000 July 1, 2000 6 = - - 12000 $84000 2000 Balance of dollars for 1 month - - - - - - $82000 DR. BFOWN. CR. Mos. I)olla Mos. g Dollars 1893. OS fº. 1893. OS f. August 1, $3000 5 = - - $15000 |March 1, $4000 10 =s - - $40000 November 1, 2500 2 = - - 5000 |April 1, 1000 9 = - - 9000 $5500 $20000| $5000 * $49000 5000 20000 Due the firm, $500 Balance of dollars for 1 month - - - - - - $29000. RECAPITULATION. Jones’ capital for 1 month - - - - - - $83000 Smith’s “ for 1 month - - - - • - 82000 Brown’s “ for 1 month, - - - - - - 29000 Total capital for 1 month - - - $194000 Having thus obtained the total capital of the firm for one month, we would proceed as in previous examples to find each partner's share. 78. A. and Z. bought a house for $10000. A. paid $6000 and Z. $4000. The house rents for $150 per month. To how much of the monthly rent is each entitled? Ans. A. $90. Z. $60. * PARTNERSHIP SETTLEMENTs. 867 79. X, Y, and Z. hired a pasture for the season for $90. X. pastured 9 head of mules for 150 days; Y. pastured 11 head of mules for 110 days, and Z. pastured 24 head of mules for 160 days. How much is each to pay? Ans. X. $18.98. Y. $17.02. Z. $54. 80. Four persons, A., B., C. and D., residing contiguous to each other, agree to build a schoolhouse, and contribute to the payment of the same in the reciprocal ratio of their respective distances from the schoolhouse. The cost was $1800. The schoolhouse was kocated # of a mile from A's residence; 1 mile from B's residence; 14 miles from C's residence, and 2 miles from D's residence. How much did each contribute % Ans. A. $837.2043. B. $418.60%. C. 334.8843. D. 209.30}}. NoTE.—The reciprocal of a number is the quotient of 1 divided by the number. OPERATION. A. # mile distant. The reciprocal of # = 2 B. 1. & 4 & & 4 & & 4 4 & 1 - 1 & & & & & 4 & 4 4 & *- $. # & & & 4 & & ( & & 4 # = : Sum of reciprocals - ºp e. -> 41% = #3. Statement to find A’s Statement to find B’s Statement to find C’s Statement to find D's. contribution. contribution. contribution. contribution. $ $ $ $ 1800 1800 1800 1800 43 || 10 43 || 10 43 || 10 43 || 10 2 1 5 || 4 2 $837.2043 $418.60% $334.884; $209.30}} AVERAGING SALES IN COMMISSION. 81. A commission merchant received from three different correspondents, X, Y. and Z., consignments as follows: from X., of Jefferson, Texas, he received 200 bbls. beef; from Y., of Shreveport, La., he received 120 bbls. beef; and from Z., of Galveston, Tex., he received 210 bbls. beef. A sale of the three lots was effected at $11. But on inspection, it was found that Y’s beef was 25% better than Z’s, and X's was 10% better than Y’s. What amount of money is to be credited to the sales account of each correspondent? Ans. X. $2524.80. Y. $1377.17. Z. $1928.03. OPERATION. IBbls. Bbls, of the quality of Z's. X. 200 + 25% = 250 + 10% = 275 Y. 120 + 25% = 150 Z. 210 210 530 635 of truality of Z's. 11 $5830 Operation to find X's credit. Operation to find Y’s credit. Operation to find Z's credit. $ $ $ 635 | 5830 635 5830 - 5830 275 150 635 | 210 $2524.80 $1377.17 $1928.03 Explanation.—The difference in the quality of the beef being given in per cent, we, there- fore, to find the equivalent number of barrels of X’s and Y’s beef according to the standard quality 868 SOULE's PHILOSOPHIC PRACTICAL MATHEMATICs. *. of Z’s, add to Y's 25% of the number of barrels, and to X's, as his is worth 10% more than Y’s, we first add 25% of the number of barrels, which gives 250 of the quality of Y’s, and then to this we add 10% of the number of barrels, and obtain 275 barrels of the quality of Z's. The balance of the operation is too simple to require explanation, and hence we omit it. 82. A commission merchant received on consignment from A., $300 barrels X flour; from B., 240 barrels XX flour; from C., 50 barrels XXX flour; and from D., 100 barrels choice flour. The current market price for Xis $6.00; for XX, $6.50; for XXX, $7.00; and for choice grade $8, with a market tending upward. The four lots were sold at the round figure, $7.00. What was the correct amount to credit to the Sales account of each consignor ? Ans. A. $1927.72. B. $1670.69. C. 374,83. D. 856.76. OPERATION. Bbls. Market Market rate. value. A. 300 (2) 6 == $1800 B. 240 (2) 6} = 1560 C. 50 (2) 7 F 350 D. 100 (2) 8 == 800 690 $4510 Total market value. 7 $4830 Amount of sales. STATEMENT TO FIND A's, B's, C's AND D's RESPECTIVE CREDITs. A’s. B’s. C’s. D’s. $ $ $ 4830 4830 4830 4830 4510 | 1800 4510 | 1560 4510 || 350 4510 || 800 $1927.72 $1670.69 $374,83 $856.76 Ea:planation.—In the operation of this problem, we must find the amount of sales, then the amount of the market value Öf each grade, and then, having these two amounts, we divide the sales between the different consignors in proportion to their respective amounts at the market rate. . #. the price per barrel, we wonld divide the total amount due each by the number of barrels sold for each. COTTON AVERAGE PEROBLEMS. 83. A cotton factor has four bales of cotton, weighing as follows: 450, 460, 398 and 486 pounds. What is the average weight per bale? Ans. 4484 pounds. OPERATION. 450 460 398 486 4) 1794 448# pounds. Ans. 84. A cotton factor has four lots of cotton belonging to four different parties. The first lot consists of 3 bales, weighing 1400 pounds, and is classed as Ordinary, the quoted market price of which is 16 g. The second lot consists of 8 bales, weigh- ing 3500 pounds, and is classed as Low Middling, the quoted market price of which is 1842. The third lot consists of 4 bales, weighing 1800 pounds, and is classed as Strict Middling, the quoted market price of which is 1942. The fourth lot consists Yºr PARTNERSHIP SETTLEMENTS. 869 of 5 bales, weighing 2500 pounds, and is classed as Good Middling, the quoted market price of which is 202. What is the average price per pound? OPERATION Ea:planation.—This method, although sufficiently correct for By the practical method. practical purposes, is not quite accurate, for the reason that the Bales. Cts. weight of different bales of cotton is not the same. But in prac- : % ;# = 1. tice the weight is not often known at the time the average price is 4 @ 19+ = 77% required, and hence the necessity in those cases of using the num- 5 (a) 20 = 100% ber of bales which really represent pounds. To have added the 20 ) $3.73 four prices, 16%, 1849, 1949, and 20% together, and divided by 4, 1843% Ans. would have been a woeful error. OPERATION BY THE EXACT METEIOD. Bales. Pounds. 3 - 1400 (2) 16 gº := $224.00 8 = 3500 (a) 18% - 647.50 4 = 1800 (a) 194% - = 346.50 5 = 2500 (a) 20 g := 500.00 9200 ) $1718.00 (1834%, Ans. 79800 tº 73600 6200 85. Suppose, in the foregoing problem, that a sale of the four lots of cotton had been effected at 192, what would have been the correct increased price for each lot and grade of cotton ? OPERATION. Pounds. Market Price. "Amount. 1400 (2) 16 g. = $224.00 3500 (2) 1849. = 647.50 1800 (2) 194%. - 346.50 2500 (2) 20 g. F 500.00 9200 $1718.00 199. $1748.00 Statement to find price Statement to find price | Statement to find price | Statement to find price of the Ordinary. of the Low Middling. of the Strict Middling. of the Good Middling. g .* 16 2 || 37 4 || 77 20 1718 || 1748 1718 1748 1718 1748 1718 1748 16.279;#3%. Ans. 18,823#9. Ans. 19.586;#39. Ans. 20.349%$3%. Ans. Ea:planation.—In the operation of this problem, we first find the value of each lot of cotton at the quoted market price, and then by adding the same we obtain the total market value, $1718. Then, by the use of our reason, we see that 16, 183, 194, and 20% prices gave $1718 value, and by trans- position that $1718 value required 16, 184, 19+, and 20% prices, and as the whole cotton was sold for $1748, we have this simple proportional question: if $1718 value requires 16, 184, 19+, and 20% prices, what will $1748 value require? The line statements give the exact answers to this question, and complete the solution. * Had the cotton been sold at a price less than the average, the same method of work would be used to find the decreased prices. 87o soulE's PHILOSOPHIC PRACTICAL MATHEMATICS. 3C It has been said by many that questions like the preceding one could not be solved, and it is claimed by others, in whose business similar questions are of daily occurrence, that because the correct results often contain such inconvenient frac- tions, the correct solution is of no advantage. The method in general use is the guessing method; a method of solving arithmetical questions that we cannot indorse. By the guessing method of finding the increased prices, injustice, unintentional of course, is done to some of the owners of the different lots of cotton, for the reason that the guessing cannot be made exactly in the correct proportion; and hence, even though the guessing prices absorb the exact amount received, one owner will have more and another less than his correct proportional increase. By our correct solution of this character of questions, we admit that the fractions in the results are often impracticable according to custom, but by these correct results, extending the figures to two or three places of decimals, we can readily see what the nearest practical fractions are, and hence do the nearest possi- ble justice to all the owners. By our correct method, we avoid the repeated guessing operations that are often necessary to arrive at a satisfactory approximate result, and never have any considerable excess or deficit of money by reason of the increase or decrease in price. There is another objection that is sometimes raised in opposition to the cor- rect method of work. It is, that because there is more cotton of a high grade than there is of a low grade, or vice versa, the increase or decrease of price should not. be in exact proportion to the quoted market rates. If this circumstance is taken into consideration in fixing the selling price of the different lots, then we would increase or decrease the quoted market rate of such cotton as influenced the selling price, and proceed by the exact method of work. To show the practical prices of the above problem, we present the following: 1400 (a) 16+ = $227.50 Or, 1400 (a) 16+ = $227.50 Or, 1400 (a) 164 = $227.50 3500 (a) 18+3 = 658.43 3500 a 18; = 660.62 3500 (a) 18:# = 658.43. 1800 (a) 19% = 352.12 1800 (a) 19% = 351.00 1800 (a) 193 = 353.25 2500 (a) 20:# = 509.37 2500 (a) 20:# = 509.37 2500 (a) 20:# = 509.37. $1747.42 $1748.49 $1748.55. The following table of 16ths and 32nds will facilitate the changing of the decimals of the cents in the price answers to practical equivalent common fractions, in all problems of this character: TABLE OF 16THS AND 32NDS AND THEIR EQUIVALENTS IN DECIMALS. # = .03125 | # = .28125 | }} = .53125 # = .78125. # = Tº = .0625 | }} = # = .3125 | }} = |* = .5625 | #3 = +3 = .8125 # = .09375 | }} = .34375 | }} = .59375 | #} = .84375. # = # = .125 # = # = .375 #} = # = .625 # = g = .875 # = .15625 | }} = .40625 | }} = .65625 | # = .90625. # = * = .1875 ## = ** = .4375 | ## = +} = .6875 #9 = H = .9375 ** = .21875 | }# = .46875 | }} = .71875 # = .96875. # = + = .25 # = } = .5 # = # = .75 # = 1 = 1. PARTNERSHIP SETTLEMENTS. 871 DIVISION AND PROPORTIONAL DIVISION. 86. A benevolent man has $600, of which he wishes to give to A. $, to B. #, to C. § and to D. #. What amount will each receive? Ans. A. $200. B. $150. C. $120. D. $100. OPERATION. A's one-third of $600 = $200 B’s one-fourth of 600 = 150 C’s one-fifth of 600 = 120 D’s one-sixth of 600 = 100 $570 total amount donated. 30 remaining on hand. $600 87. Another benevolent man has $600 which he wishes to give to W., X, Y, and Z., in proportion to #, +, + and #. What amount would each receive 3 Ans. W. $210.53. X. $157.89. Y. $126.32. Z. $105.26. OPERATION. # = W’s proportion. Statement to find W’s propor- || Statement to find X’s propor- + = X’s & 4 tional share. tional share. + — Y’s & 4 & # = Z's & 4 $ gºmers 600 600 ## = the sum of the propor- 19 20 19 || 20 tions which are to re- 3 4 ceive the amount of tº mºs *- $600. $210.53 Ans. $157.89 Statement to find Y’s | Statement to find Z's Explanation.—The operation of this problem proportional share. proportional share. is so simple that special explanation is deemed unnecessary. The preceding example, No. 86, is often given, when the answer to this example is 600 600 required. To show clearly the difference between 19 20 19 || 20 giving certain parts of a sum of money to differ- 5 6 ent parties, and giving the whole sum of money sº- in proportion to certain parts, we have presented $126.32 $105.26 the two preceding questions. 88. A planter has seventeen horses which he wishes to give to his three sons, Henry, William and Robert, in proportion to 3, § and #. How many horses will each receive 3 . Ans. Henry, 9. William, 6, Robert, 2. OPERATION. # = Henry’s proportion. HENRY. WILLIAMs } ROBERT. # = William’s { { - # = Robert's & & Horses. Horses. Horses. tº-ººp 17 17 17 +} = the sum of the pro- 17 | 18 17 | 18 17 | 18 portional s h are s 2 3 9 which are to re- *-º-º-º- * *E= ceive the 17 horses. 9 Ans. 6. Ans. | 2. Ans. 89. What are the respective amounts resulting from the division of $77 pro. portionally to #, #, 4, # 3 Ans. $30, $20, $15, $12. 87.2 soul E's PHILOSOPHIC PRACTICAL MATHEMATICS. * SETTLEMENTS WITH SPECIAL CONDITIONS. 90. A. and Z., residents of New Orleans, hired a team to go to the lake, a distance of five miles and return. The charge for the use of the horses and car. riage was $10. On arriving at the lake, Z. proposed to return by some other convey- ance, to which proposition A. agreed. They also agreed to settle the charges for the use of the horses and carriage in proportion to the distance that each rode. What must each pay ? Ans. A. $6.663. Z. $3.33}. OPERATION. * To find the amount A. | To find the amount Z. A. rode to the lake and returned = 10 miles. must pay. must pay. Z. ‘‘ ‘‘ lake = 5 miles. $ $ fººms 10 10 Total number of miles ridden 15 || 10 15 || 5 by both {- ſº-> sº - = 15 miles. $6 66# Ans $3 33} Ans. 91. Suppose in the above problem, that they had agreed to settle the charge for the use of the horses and carriage justly, what would each have paid? Ans. A. $7.50, Z. $2.50. SOLUTION. The distance to the lake and return being ten miles, and the charge for the use of the horses and carriage being $10, it is clear that ten miles' ride costs $10, and if ten miles’ ride cost $10, one mile ride will cost the 10th part, which is ($10 – 10) $1. Then A. and Z. having ridden together to the lake, a distance of five miles, in justice, they are required to settle equally the expense of the ride that they have equally enjoyed. And the expense for the five miles at $1 per mile is $5, which, settled equally, is ($5 - 2) $2.50 that A., and $2.50 that Z. must pay. This settles the expense to the lake, where Z. retires and A. becomes solely responsible. The expense for the five miles’ ride by A. from the lake to the city, at $1 per mile, is $5, which justice requires A. to settle, as he enjoyed the ride alone. We thus obtain $2.50 + $5 = $7.50 that A. would have paid, and $2.50 that Z. would have paid, had they settled justly. Or, thus: A. and Z. having ridden together to the lake, which is ºr of the whole distance, it is clear that they should, equally, pay ºr of the whole cost. The whole cost being $10, ºr of it is $5, and # of $5 is $2.50, the amount each should pay for riding to the lake. Then, as A. rode the remaining Tº of the whole distance alone, it is clear that he should alone pay the remaining ºr of the whole expense; ºr of $10 is $5, which added to the $2.50 first obtained makes $7.50 that A. is to pay. This completes the solution, and gives $7.50 as A's share of the expense, and $2.50 as Z's share of the expense. 92. A. and Z. bought merchandise to the amount of $800, of which sum A. paid $500 and Z. paid $300. Soon after they sold to G. § of the whole stock for $400. How much of the $400 must A. and Z. receive respectively, in order to con- stitute each, A., Z. and G. § owner of the goods? Ans. A. $350. Z. 50. FIRST OPERATION. Capital Am't of invoice Am't of in- Statement to find Statement to find contrib- each retained. voice each amount due A amount due Z. uted. sells. tº A. $500 – $266; = $233} $ $ Z. 300 — 266# = 33# | 400 400 } of 800 $266; amount of invoice sold. sº % sº io $266; $350. Ans. $50. Ans. * - PARTNERSHIP SETTLEMENTS. 873 SECOND OPERATION. Statement to find Statement to find Amount paid. Part owned. Part retained. Part sold amount due A. amount due Z. Z. - 300 = } – # = ºr 400 400 *- sº- 8 || 24 8 || 24 # of )}(# # 24 7 24 $350 | $5 93. Two newsboys, James and Henry, on taking their seats to eat some cakes that they had purchased, were requested by another member of their frater- nity named “Dick,” to allow him to dine with them. To this request, they cheer. fully assented, and the three ate the cakes. When they had finished, Dick laid on the table 402 for his share of the dinner. How much of this 40g must James receive, who contributed 5 cakes to the dinner, and how much must Henry receive, who contributed 3 cakes 3 Ans. James 352. Henry 52." OPERATION. JAMES, HENRY. James 5 — 23 = 2* = } g ſº Henry 3 — 23 = } = } 40 40 º - 8 || 3 8 || 3 # of 8 § 3 7 3 2} 35% 52 94. A cotton factor insured : 200 bales of cotton for 3 months at 14%; 150 - 4 {{ 44. 4, 2 4% at 13% ; 100 és {{ {{ << 1 { { at #% ; What was the average rate per cent. of insurance, supposing that each bale was of the same value # Ans. 1,29#%. FIRST OPERATION. Bales. Per cent. on 1 bale. 200 a 14% = 300.00 150 a 13% = 206.25 100 (a) #% = 75.00 450 ) 581.25 (1.29#96 Ans. SECOND OPERATION. Bales. Assumed value. Insurance. Operation to find the rate per cent. 200 a $100 = $20000 (a) 14% = $300.00 $ 150 (a) 100 = 15000 (a) 13% = 206.25 581.25 100 (a) 100 = 10000 (a) #96 = 75.00 45000 || 100 $45000 $581.25 1.295% Ans. 95. A. contracted with Z. to deliver to him : 78 bales, 38531 pounds, Good Middling cotton, at 154%, and 72 “ 36500 “ Middling “ at 142; (Continued.) 874 soule's PHILOSOPHIC PRACTICAL MATHEMATICs. Yºr but A. having disposed of the foregoing cotton to other parties, by subsequent agreement A. delivered and Z. received: 2475 pounds of Inferior at 92; 6681 “ “ Ordinary at 102 ; 12219 “ “ Good Ordinary at 122; 7218 “ “ Low Middling at 1342; Z. then demanded the balance due him in Middling and Good Middling cotton. How many pounds of each grade, Middling and Good Middling, were due him 7 Ans. 25527# pounds Middling. 26947# “ Good Middling. OPERATION. Bales. 78 - 38531 pounds Good Middling at 15.4% - $5972.30 72 - 36500 “ Middling at 140 - 5110.00 Total value - * - -> • - $11082.30 RECEIPTS. 2475 pounds Inferior at 9% $222 75 6681 “ Ordinary at 10% 668.10 12219 4 & Good Ordinary at 12% 1466.28 7218 “ Low Middling at 133% 974 43 Total receipts - * - 3331.56 Balance due - * - $7750.74 which is to be paid in Good Middling and Middling at contract price, in proportion to $5972.30 and $5110. Statement to find the amount to be paid in Statement to find the amount to be paid in Good Middling. Muddling. $ $ 7750.74 \ 7750.74 11082.30 || 5972.30 11082.30 || 5110 $4176.91 $3573.83 $4176.91 - 154% = 26947# lbs. of Good $3573.83 – 14% = 25527 ºr lbs. of Middling cottom. Middling cotton. SETTLING ESTATES. 96. A man had two sons and three daughters; to the youngest son, he gave # of 14 times $3600, which was # of the elder son's share, and what the elder son received was # of g of the whole estate; the balance was divided among his three daughters in reciprocal proportion to their ages, 8, 10, 12 years. What was each daughter's share 3 Ans. $7114.87 for the one 8 years old. 5691.89 % {{ 10 $4 4743.24 “ “ 12 {{ SOLUTION. # of 14 = #, and # of $3600 = 2700 = the younger son's share; then if $2700 is # of the elder son's share, # is the 4 of it, which is $1350, and # or the whole share is 5 times as much, which is $6750; then if $6750 is # of g = } of the estate, 3 or the whole estate is 4 times as much, which is Yºr PARTNERSHIP SETTLEMENTS. 875 $27000. Then the two sons' shares, $2700 + $6750 = $9450, deducted from the whole estate, $27000, leaves $17550, belonging to the three daughters in reciprocal proportion to their ages, 8, 10 and 12 years. The reciprocal of 8 is # & & & 4 & 4 10 is To & & ( & # -- I'm -H T's = ºr “ 12 is fºr Having now the reciprocals, the following statements give the respective shares of each daughter: 17550 17550 17550 37 || 120 37 120 37 120 8 10 12 $7114.87 $5691.89 $4743.24 To perform the first part of the solution without the reasoning, we would make the following statements: Statement to find the younger Statement to find the elder Statement to find the whole son's share. son's share. estate. $ $ $ 3600 2700 6750 2 2 5 2 || 7 2 || 3 e-mº-º-º-º-º: 7 || 8 | $6750 $2700 $27000 97. A., B. and C. were to receive $6000 in proportion to #, 4 and #; but B. having died, it is required to divide the money between A. and C. What sum should each receive? Ans. A. $4000. C. §2000. OPERATION. # -H # = } $ Explanation.—B. having died, his # pro- 6000 6000 portional interest is therefore divided be- 3 tween A. and C. in proportion to their pro- 2 2 portional interests, # and #; we therefore $4000 A's share. $2000 C's share. have, as shown in the operation, but to di- vide the whole sum in proportion to # and #. 98. A father willed his estate valued at $40000 to his three children in pro- portion as follows: John #, Henry # and Katie #. Defore the settlement was made, Henry died. What sum should John and Katie each receive 3 Ans. John $25000. Katie $15000. .* OPERATION. KATIE. $ # -- + = fºr the sum of the proportional 40000 40000 interests of John and Katie. Then 8 15 : 15 3 $25000 $15000 NoTE.—Henry having died, his 4 proportional interest is therefore to be divided between John and Katie in proportion to their proportional interests, as shown in the operation. 876 soule's PHILOSOPHIC PRACTICAL MATHEMATICs. Yºr 99. A man at his death left an estate amounting to $35000, to his wife and two children, a son, and daughter. His children being absent in Europe, he directed by will, that if his son returned, his wife should have # of the estate, and the son the remainder; but if the daughter returned, his wife should have 3 and the daugh- ter the remainder. Now, it so happened that they both returned. What sum should each receive 3 Ans. $20000 son. $10000 wife. $5000 daughter. SOLUTION. By the first condition of the will, the son was to receive twice as much as the wife; and by the second condition the wife was to receive twice as much as the daughter. Hence to comply with both conditions, we assume 1 to represent the daughter's share; then twice 1, or 2 will be the wife's share, and twice 2, or 4 will be the son's share; and 1 + 2 + 4 = 7 = the sum of all the shares. Then } of $35000 = $5000, the daughter's share; # of $35000 = $10000, the wife's share; and # of $35000 = $20000, the son's share. 100. A father leaves a number of children and a certain sum of money to be divided among them as follows: The first is to receive $100 and ſº of the remain- der; the second is to receive $200 and I's part of what then remains; again, the third is to receive $300 and ºr of the residue, and so on; each succeeding child is to receive $100 more than the one immediately preceding, and then I's part of what still remains. At last it is found that all the children have received equal sums. What was the amount left, and how many children were there? Ans. The sum divided was $8100 and the number of children 9. SOLUTION. After giving the first $100, there remained the sum minus $100, and ſo of this is ºn of the sum minus $10; hence the first received $100 + ºr of the sum minus $10 or $90+ tº of the sum, and there remained of the whole # of the sum minus $90; giving the second $200, there remained nº of the sum minus $290 and ſo of this is r&d of the sum minus $29; hence the second received $200 + rán of the sum minus $29 or $171 + rào of the sum. Now according to the conditions of the question, the first received the same as the second, hence $90 + ºn of the sum = $171 + +}o of the sum or ſº of the sum = 130 of the sum + $81, or again, rºw of the sum = $81; hence the sum = $8100. Now the first received $100 + To of what remained or $100 + $800 = $900. But since each child is to receive an equal sum, there must be as many children as $8100 is equal to $900 which is 9. PROPORTIONAL DIVISION. 101. Divide 18 oranges between Katie and Sallie, so that Katie will have + more than Sallie. What number will each receive 3 Ans. Katie 10. Sallie 8. FIRST OPERATION. 1 = the assumed number given to Sallie. Statements to find the proportional 1+ = & & & & é & Katie. share of each. 2+ = the sum of the ratio numbers by which SALLIE. KATIE. the oranges are to be divided. 18 | 18 9 || 4 9 || 4 sº 4 || 5 8 *mºsºs PARTNERSHIP SETTLEMENTS. 877 SECOND OPERATION. 4 = the assumed number given to Sallie. SALLIE, KATTE. 5 = é & 4 & & & Katie. 18 18 -sº 9 || 4 9 || 5 9 = the sum of the ratio numbers. - mº- 8 10 102. A young husband has one and a fourth dozen apples which he wishes to divide between his wife and his mother-in-law, so that his mother-in-law will have # less than his wife. How many will he give to each? Ans. Wife 11+. Mother-in-law 33. OPERATION. 1 = the assumed number given to his wife. Statements to find the share of each. # = & 4 (£ & 4 mother-in-law. - WIFE. MOTHER-IN-LAW. 1} = the sum of the ratio numbers, by which the ap- 15 15 ples are to be divided. 4 || 3 4 || 3 * 3 l — 11+ 3# NOTE.-If preferred, 3 and 1 may be assumed as the ratio numbers, in place of 1 and #. 103. Divide $2000 between A., B. and C., so that A's part will be to B's as 2 to 3, and that C. will have as much as A. and B. together, less $200. What amount will each receive? Ans. A. $440. B. $660. C. $900. N SOLUTION. As C. is to have as much as A. and B. together, less $200, A. and B, will therefore have # of $2000 + 4 of $200, which is $1100. Then, as A. and B. are to receive this in the proportion of 2 to 3, A. will therefore receive # and B. # of $1100. # of $1100 = $440 A's share; # of $1100 = $660 B's share; and lastly, as C is to receive as much as A. and B. together, less $200, he will therefore receive $1100 — 200 = 900. 104. A. and B. made an investment in real estate, A. invested $26217.50 and B. invested $1707.40. From the proceeds of a sale of a portion of said real estate, A. drew out $15289.25 and B. drew out $2908.95. B. now desires to be an equal owner with A. in the remainder. How much must B. pay A. to be an equal owner? OPERATION, A. invested, - * > - º - * = º - º - ſº º se - as $26217.50 A. drew out, - * * is ºn tº * * * * * - * * = 15289.25 A's net investment of which B. owes A. one-half, - - - - - - - - $10928.25 B, invested, - --> - * - º - * - º - - $1707.40 B. drew out, - * - sº - sº - gº - :- & -> 2908.95 B’s net withdrawals of which he owes one-half to A. * * * * gºs º- tº 1201.55 Total amount of which B. owes A. one-half, º º º -> º - º * # ) $12129.80 Amount B. must pay to A. sº º - *- gº tº ſº º º - º $6064.90 PROOF. A’s total investment, - - - $26217.50 | B. invested, sº º ºs - - $1707.40 A’s withdrawals, - $15289.25 B. paid to A. - - - - - 6064.90 A. received from B. - 6064.90 *messammº -º-º- B’s total investment, - - º $7772.30 21354.15 B. drew out, * - - - º 2908.95 —º- A's # net investment, tº º ºs $4863.35 | B's net investment, - - - - $4863.35 878 soul E's PHILOSOPHIC PRACTICAL MATHEMATICS. 3. THE LAND AND THE DITCH PROBLEM. 105. A. and B. paid $600 ($300 each) for 300 acres of land. In dividing the land according to quality, it was agreed that A. should have the best quality, and pay therefor 75 cents per acre more than B. How many acres did each receive, and what price per acre did each pay ? Ans. A. received 122.8 acres and paid $2.443 + per acre. B. 64 177.2 4 {{ 1.693 + éé OPERATION. 300 2 75 g = $225; and $600 – 225 = $375 reserved remainder. Then $225 × 4 × 300 = $270000 to which is added 375* = 140625; 270000 + 140625 = v4.10625 = 640.8 + Then 640.8 + 375 = 1015.8, which -- (300 × 2) = 600 = $1.693 + the price per acre paid by B. Then $1.693 + 75 cents = $2.443 + the price per acre paid by A. Then $300 + 1.693 = 177.2 acres that B. receives; and $300 + $2.443 = 122.8 acres that A receives. Explanation.—This and the following problem belong strictly to algebra, and although they may be solved by arithmetical operations deduced from algebraic equations, they cannot be as analytically explained as regular arithmetical problems. The solution of all problems of this character may be performed by observing the following directions: First find the cost of the whole number of acres at the difference between the prices per acre; then subtract this from the price paid for the whole land and reserve the remainder. Then multiply the value produced by the difference in price by 4 times the whole amount paid by him who paid the least per acre, and to the product, add the square of the reserved remainder; then extract the square root of this sum, and to it add the reserved remainder. Then divide this sum by twice the whole number of acres, and the quotient will be the price per acre paid by him who paid the least Then to this price add the difference between the prices paid and the sum will be the price paid by him who paid the greater price. Then divide the amount paid by each by the respective price paid by each, and the quotients will be the number of acres that each is to receive. It will be observed that there is in the problem an interminable decimal which, because it arises in extracting a root, cannot be expressed as a vulgar fraction. It may, however, be approximated to any extent. Problems of this character are often misstated by giving either the price that each is to pay or the number of acres that each is to receive. To give either, ren- ders the problem absurd and insolvable. 106. A. and B., together, agree to dig 100 rods of ditch for $100. The part of the ditch on which A. was employed was more difficult of excavation than the part on which B. was employed; and it was therefore agreed that A. should receive for each rod 25 cents more than B. received for each rod that he dug. How many rods must each dig, and at what price so that each may receive just $50? Ans. B. receives $ .8903 + per rod, and digs 56.16 rods. A. “ $1.1403 per rod, and digs 43.84 rods. OPERATICN, 100 x 25g = $25; and $100 – $25 = $75, reserved remainder. Then $25 × 4 × 50 = 5000 to which we add 75* = 5625; 5000 + 5625 = 10625, the square root of which is 103.07 -H Then 103.07 -- 75 = 178.07, which -- (100 × 2) = 200 = $.8903 + the price per rod received by B. Then $.8903+ 25 g = $1.1403 the price per rod received by A. Then $50 + $.8903 = 56.16 rods that B. digs; and $50 + $1.1403 = 43.84 rods that A. digs. * PARTNERSHIP SETTLEMENTS. 879 107. A., B., C. and D. agree to do a piece of work for $5500. A., B. and C. can do it in 20 days; B., C. and D. in 24 days; C., D. and A. in 30 days; and D., A. and B. in 36 days. In how many days can all do it, working together; in how many days can each do it, working alone; and what part of the pay ought each to receive # Ans. See the solution. SOLUTION. As A., B. and C. can do the work in 20 days, they can do gº of it in 1 day; as B., C., and D. can do it in 24 days, they can do ºr of it in 1 day; as Q, D. and A. can do it ; 30 days, they Call do gº of it in 1 day; and as D.; A. and B. can do it in 36 days, they can do #5 of it in 1 day; there- fore, A., B., C. and D., by working 3 days each, will do gº + g + 3 + 35 = } of the work, and in 1 day they will do # of #} = #6; and as they all do gº of the work in 1 day, it will require as many days to do the whole work as 1 is times equal to #6, which is (1 -— gºs =) 19%r days. Now, by deducting successively the work that three can do in 1 day, when they work tººther from what the four can do in 1 day, we obtain the work that each one will do in 1 day; U18 #5 – 35 = Tºro the work that D. can do in 1 day. #5 — ºr = Tºg the work that A. can do in 1 day. # — ºr = + the work that B. can do in 1 day. #6 — ºf = gº the work that C. can do in 1 day. Having now the work that each can do in 1 day, it is clear that it will require each as many days to do the work alone as 1 is equal to the part of the work performed by each; thus 1 — Tºso gives 1080 days that it would require D. 1 - Tºg gives 108 days that it would require A. 1 - Tºšo gives 56+3 days that it would require B. 1 -— gº gives 43% days that it would require C. In dividing the money, it is evident that each should receive such amount paid as he did of the whole work. uch a part of the whole This is shown by multiplying the amount of work performed b h dail of days that all worked; thus p y each daily by the number A. Tºg × 19 ºr = ſºr A's proportion. B. Tºšo X 1911 = }; B's & & C. ºf X 1911 = ºr C's & f D. Tºso X 191+ = ºr D's & 4 According to these proportions the money is divided thus: *r of $5500 is $1000, A’s share. # of 5500 is 1900, B’s “ fºr of 5500 is 2500, C's “ ºr of 5500 is 100, D's “ Total amount, - - - - - $5500 111. A young hoodlum, a modern evolvement of the human race, stole a bas- ket of peaches and divided them among three brother hoodlums and himself as follows: To the first, he gave 4 of the whole number and + of a peach more; to the second, he gave # of What remained and # of a peach more ; to the third, he gave # of what remained and # of a peach more. The stealer retained what was then left, for himself, which was # the number he gave to the first hoodlum. What Was the number of peaches stolen, aud how many did each hoodlum receive? Ans. 7 peaches were stolen. 1st hoodlum received 2. 2d 46 44 2. 3d {{ 4% 2. The stealer had 1. SOLUTION. In all problems of this kind, the following principle governs: That when the fractional parts of the successive portions are consecutively increasing, each of the parts or shares of the several 88O SOULE's PHILOSOPHIC PRACTICAL MATHEMATICS. jºr persons are equal, except the last one, which, from the nature of things, must be one less than the average. Hence in the given problem, the fact stated that the last one's share is half of the first (and each of the others also) and from the above principle that it must be one less, it must neces- sarily be ONE, and since each of the others is one greater, they must be two each, and since there are three persons having equal portions, there will be 6 + the 1 for the last, making 7 in all. Hence, to solve problems of this kind: add 1 to the last number, (ONE), and multiply by the num- ber of persons, and then subtract 1 from the product. 112. Three persons, say A., B. and C., buy a piece of land for $4569, and the parts for which they pay bear the following proportions to each other, viz: the sum of the first and second, the sum of the first and third, and the sum of the second and third, are to each other as #, #, and #3. How much did each pay, and what part did each own 7 Ans. A. paid $1015; and owns #. B. 46 1523 “ #. C. “ 20303 “ 4. OPERATION. A. and B. = } = To = 5 shares. A. and C. = } = 1% = 6 “ B. and C. = Pºſ = To = 7 “ ** 18 shares; but as each party’s share has been counted twice, we therefore divide the same by 2 and obtain 9 shares. Then, to find the respective shares of A., B. and C., we deduct from the sum of the shares. the number of shares owned by each two of the parties, thus: A., B. and C. own 9 shares. A., B. and C. own 9 shares. A., B. and C. own 9 shares. A. and B. own 5 “ A. and C. own 6 * * B. and C. own 7 : « C. owns 4 * * B.’ owns 3 * * A. owns 2 * * Having the respective shares of each party, we make the following state- ments and produce the amount due to each: A’s 3. B's 3. C's #. A. 2 shares $ $ $ B. 3 * * 4569 4569 4569 C. 4 * * 9 || 2 9 || 3 9 || 4 9 * * $1015; $1523 $2030; 113. Three men take an interest in a mining company. A. put in $480 for 6 months; B. a sum not named for 12 months; and C. $320 for a time not named: when the accounts were settled, A. received $600 for his stock and profits; B. $1200 for his; and C. $520 for his. What was B's stock, and C's time? Ans. B's Stock $800. C's time 15 mos. Investment. Time invested. Am’t rec'd in settlement. Gain. A. $480. 6 mos. $ 600 $120 = }#} = + of stock. B. : 12 mos. 1200 º C. 320. º 520 200 OPERATION. Since A's investment of $480 gained $120 in 6 months, in 12 months, the time B's was invested, it would gain twice as much or $240, which is #8 = } of his investment. Then, since the gain of each is in like proportion, the $1200 received in settlement by B. for 12 months is # of his investment of stock; and since $1200 is 3 of B's investment, 3 is the third part or $400, and two halves or the whole investment is $800, and his gain is $1200 — $800 = $400. Now having B's investment $800, and his gain for 12 months $400, and also C's investment * PARTNERSHIP SETTLEMENTS. 88.1 $320, and his gain $200, to find C's time, we make the following classification and proportional state- ment : CLASSIFICATION. STATEMENT. Months. º 12 Explanation.—In making the Investment. Gain. Months. 320 800 statement, We .*. as º: lows: Since an II]. We St- B’s $800 $400 12 400 | 200 º º *- nt requires 12 months time C’s 320 200 ! ment req 15 mos. to produce $400 gain, $1 would require 800 times as many months, and $320 investment the 320th part. Then, since $400 gain requires 12 months time to pro- duce it, $1 gain will require the 400th part, and $200 gain will require 200 times as many months. 114. Jones, Smith, Brown and Wood formed a partnership. It was agreed that Jones should receive # of the gains, Smith 4, Brown #, and Wood #. The remainder of the gains was to be credited to Reserve Fund, to be used for the Con- tingencies of the business. It was agreed that each partner should invest $15000, on condition of receiving the above proportional share of profits. Jones however invested only $12000 and Smith invested only $10000. The net gain amounted to $16500. Since Jones and Smith failed to invest the required amount, how should the gain be divided ? Ans. Jones $5225; Smith 3265.62%; Brown $3918.75; Wood $3265.624. Beserve Fund $825. SOLUTION. 1. By the terms of the partnership, each partner was to invest $15000 which sum was to entitle each to the following shares of the profits: Jones + + Smith 4 + Brown 4 + Wood # = #3; thus leaving go to be credited to Reserve Fund. 2. Since Jones and Smith failed to invest the full amount of $15000 each, the respective shares of their gain must be reduced in the same proportion as their respective investments were reduced from the $15000 required by the terms of the agreement. The following statements give their reduced shares: $15000 : 12000:: * : *r = Jones' share of gain for his $12000 investment. $15000 : 10000:: + : # = Smith’s share of gain for his $10000 investment. 3. The respective shares of the partners in the #} of the distributable gain are now as fol- lows: Jones 1%, Smith #, Brown ; and Wood # = # which is the sum of the interests or shares that are to receive ## of the gain. 4. The total gain is $16500 *if deducted for Reserve Fund is 825 Amount to be divided is $15675 5. The following statements give the share of each partner: JONES. SMITH. BROWN. WOOD. $15675 $15675 $15675 $15675 4 5 5 4 5 4 5 15 || 4 6 5 6 $5225 | $3265.62% | $3918.75 $3265.62% NOTE.-By the terms of the agreement, the partners were to receive in different proportions ## of the gain, and the remainder, ºo, was to be credited to Reserve Fund. The respective interest of each partner was conditioned upon an investment of $15000; hence those partners who invested less than $15000 are to have their original shares reduced, as shown above, and then the # of the gain is to be divided among the partners in proportion to the revised shares, as shown in the operation. Reserve Fund thus receives ºf of the gain, as per agreement. By this method, the part- ners received the #3 of gain, in equitable proportion to their actual investments, based upon the terms of agreement. This we believe to be the proper method of adjusting the matter. In Problem 38, pg. 832, which involves the same conditions as this problem, the # interest, which was to be credited to Profit and Loss, was allowed to participate in the gain resulting from the decrease in the respective shares of C. and D. because they failed to make full investments as they had agreed. By thus giving Profit and Loss a part of this gain, more than # of the gain of the 882 PARTNERSHIP SETTLEMENTS, * business is credited to this account, and consequently less than g of the gain is distributed among the partners. In the solution of Problem 38, which problem was selected from another work, we adopted the method of the author of the problem, by which he credited PROFIT and Loss with more than # of the gain. We now present the above problem and give a different solution and submit the two methods for the consideration of accountants. * 115. Suppose, in the above problem, that the partners had made the following investments: Jones $12000, Smith $25000, Brown $15000 and Wood $15000. How should the gain have been divided ? Ans. Jones, $3980.95 +. Smith, $6220.24 –. Brown, 2985.71 +. Wood, 2488.10 —. Reserve Fund, $825. OPERATION. $15000 : 12000 : : * : *r = Jones' share of the gain for his $12000 investment. 15000 : 25000 : : + : # = Smith’s “ “ “ “ 25000 “ Then Jones’ ºr + Smith's ºr + Brown's # -- Wood's # = # which is the sum of the shares or interests that are to receive #8 of the gain. The total gain is - - $16500 25 deducted for reserve fund 825 Gain to be divided is - $15675 The following statements give the proportional share of each partner: Jones’ ſº. Smith's #. Brown’s #. Wood's #. $15675 $15675 $15675 $15675 63 || 60 63 60 63 60 63 || 60 15 4 12 || 5 5 6 $3980.95+ $6220.24— $2985.71+ $2488.10— 116 A., B., and C. are general partners in business, equal in gains and losses. They conclude to dissolve their commercial or trading partnership and form a stock company. Accordingly they close their books and find the net capital of each to be as follows: A. $32238, B. $28712, and C. $28644. Total $89594. The stock company is organized with a capital of 1000 shares at $100, amounting to $100000, of which each of the partners of the former business is to subscribe for 333% shares. The excess of the capital of the stock company over the total capital of the old firm is ($100000 — $89594) $10406. It is agreed that to offset or balance this excess, they will value some patents which the old firm possess at $10406, and place the same in the list of assets. What amount of money must B. and C. pay to A. respectively, to adjust the matter equitably among the three ? Ans. See operation. OPERATION.—First Step. The valuation of the patents creates a new asset, which belongs to the members of the old firm in proportion as they shared the gains and losses, # each. Hence $10406 equals $3468.66# due respectively to A., B., C. Second Step. A’s credit $32238 + $3468.67 = $35706.67 — $33333.34 = A's Cr. $2373.33 B’s credit 28712 -- 3468.67 = 32180.67 – 33333.33 = B's Dr. 1152.66 C’s credit 28644 + 3468,66 = 32112.66 – 33333.33 = C's Dr. 1220.67 $89594 $100000.00 $2373.33 10406 valuation placed on patents. *=ºº $100000 -- 3 = $33333.33} = each partner's interest in the stock company. $ From these statements it is clear that B. and C. must pay to A. respectively, $1152.66 and 1220.67. nvolution. eSºº - - - - - - SEC EE C - C - EC Cº - Cº-ºººººº-ºº: ===N 1426. Involution is the process of multiplying a number by itself a certain number of times. The several products are called the powers of the number. The number used as a factor is called the root or first power. The second power or square is the root used twice as a factor. The third power or cube is the root taken three times as a factor. The fourth power or biquadrate is the root taken four times as a factor; and all higher powers indicate the number of times the given number or root enters as a factor. The number of times the root is used as a factor is called a degree, as the 1st, 2d, 3d, and 4th degrees; and the number denoting that degree is called the index, or exponent of the power. Thus, the cube of 5 is represented by 5°. The exponent should be a small figure, and written to the right of the upper part of the root or number. A fraction should be inclosed in a parenthesis before writing the exponent, to indicate that it belongs to the whole expression, otherwise it might be considered as belonging to the numerator alone. Thus the square of # is denoted (#)”. The following arrangement will illustrate the formation of degrees and powers for the number 5; thus: 59 – 5 – 5 = 1 = zero power of 5. 5* = 1 × 5 = 5 = 1st power of 5. 5* = 5 × 5 = 25 = 2d power or square of 5. 5* = 5 × 5 × 5 = 125 = 3d power or cube of 5. 5* = 5 × 5 × 5 × 5 = 625 = 4th power or biquadrate of 5. We here see that there are three things to be noted in Involution, viz.: 1st. The root or number; 2d. The exponent; and 3d. The power. Considering the 1st power as a multiplicand, we see the number of multiplications will be one less than expressed by the exponent. Any power of 1 is 1; any power of a number greater than 1, is greater than the given number; and any power of a number less than 1, is less than the number itself. When the required power is of a high degree, instead of multiplying by the root continually, we can involve any of the subordinate powers, the sum of whose degrees amounts to the degree of the required power; always observing that the product of the powers corresponds to the sum of the exponents. Thus, to find the 7th power of the number 3, we may multiply its 4th power (81) by its 3d power (27), or 37 = 34 × 33. f When the exponent of the required power is a composite number, we may raise the root to a power indicated by one of the factors of the given exponent, and then raise this result to a power indicated by the other factor. Thus, to raise 5 to the 12th power, or 5*, we first find the 4th power of 5 to be 625; we then raise the 625 to the 3d power, because 5** = (5*)*. * * ~ * (883) 884 SOULE's PHILOSOPHIC PRACTICAL MATHEMATICs. * The difference between the last and the preceding proposition should be clearly understood to prevent confusion, their principles being in nowise analogous. To raise a vulgar fraction to any power, raise both numerator and denomina- tor to that power. Mixed numbers should be changed to improper fractions. To find any power of a decimal, or mixed decimal, multiply as in whole numbers, and point off in the result as many places for decimals as will equal the number of decimal places in the root, multiplied by the exponent of the power. EXAMPLES. 1. What is the 2d power of 3465? Ans. 12006225. 2. What is the 5th power of 256% Ans. 1099511627776. 3. What is the 3d power of #3 Ans. ###. 4. What is the 6th power of 33? Ans. 2430%r. 5. What is the 4th power of .125? Ans. .000244140625. 6. What is the cube of 3.024% - Ans. 27.653197824. 7. What is the 7th power of .0043 Ans. .000000000000000016384. 8. What is the 10th power of 7 ? Ans. 282475249. The 2d power of numbers ending in 5, when not too large, may be very readily found by taking all the figures except the 5, and then multiply the number thus taken by itself increased by 1, and to the product annex 25, the square of the 5. Thus, to find the square of 25, we omit the 5, and multiply the 2 by 3 = 6, then annexing 25, we obtain 625. * Find the square of 15, of 35, of 55, of 75, of 105, of 225, of 575. To find any power of a number that is an aliquot part, or aliquot multiple of a hundred or thousand, we form a fraction by taking the given number for a numerator, and 100, or 1000, for a denominator, and reduce to its lowest terms; then raise the numerator of this fraction to the required power, and annex as many naughts as would equal the number of naughts in the 100 or 1000 used, multiplied by the exponent of the power; then divide by the denominator of the reduced fraction, as many times as there are units in the exponent. Thus, to find the cube of 375, we have º # = 4; then, 15° = 3375000000 + 4 cubed, = Ans. 52734375. Or, †† = }; then, 3% = 27000000000 -- 8 cubed, = Ans. 52734375. EXAMPLES. 2. Find the square of 875. Ans. 765625. 3. Find the cube of 625. Ans. 244140625. 4. Find the 4th power of 375. AnS. 1977539.0625. 5. Find the 5th power of 225. Ans. 576650390625. 6. Find the 6th power of 125. AnS. 381469.7265625. 7. Find the 7th power of 75. Ans. 13348388671875. 8. Find the 8th power of 25. Ans. 1525878.90625. 9. Find the 6th power of 163. Ans. 214334.70%. 10. Find the 5th power of 33}. Ans. 41152263;ºr. 11. Find the 4th power of 6663. Ans. 197530864.1974+. 12. Find the product of the cube of 663 by the square of 33333. Ans. 3292.181069958###. volution. A-a-/-/*********A*/-/-/-/=NO =N 1427. Evolution, or the extraction of roots, is the reverse of Involution, and is the process of finding the equal factors, or roots, of numbers or quantities. A root of a number, as illustrated in involution, is a factor which, multiplied by itself a certain number of times, will produce that number. The name or degree of the root is correlative with that of the power. Thus, the Square root is the correlative term of the square or second power, and the cube root that of the third power or cube. The root to be extracted is indicated by placing the radical sign (V) before the number or power, with the index of the root over the sign. Thus, VI25 denotes the cube root of 125. The index of the square root is generally omitted, the radical sign itself always indicating the square root. The root is also sometimes indicated by means of a fractional exponent; thus, 36% denotes the square root of 36; and 64* indicates the cube root of the square of 64, or the square of the cube root of 64. A number that is a perfect power is called a rational number, and its root a rational root ; thus 27, being a perfect cube, is rational. A number that is not a perfect power is called an irrational number or surd, and its root an irrational or surd root; thus 24, being an imperfect power, is a surd, and also its root. A num- ber, however, may be rational to one degree and a surd to another. Thus 27 is a rational cube, but an irrational square. All the rational roots of whole numbers are whole numbers. All the powers of fractional numbers are fractional numbers. All prime numbers, except, 1, have no rational roots. All composite numbers that have rational roots, must have all the exponents of their prime factors divisible by the exponent of the required root. Thus, ‘VºI6 = V35 x 23 = VG = 6. The number of rational roots is comparatively small. The extraction of roots higher than the square and cube is not usually given in arithmetics, though roots of any degree may be easily found by an extension of the method given in this work; but all higher roots are more readily obtained by the use of logarithms. EXTRACTION OF THE SQUARE ROOT. 1428. The extraction of the square root of a number is the process of finding one of its two equal factors. The following numbers in the first line are the square roots of those in the second. Roots: 1 1 3, 4, 5, 6, 7, 8, 9, 10, 100. Squares: 9 16, 25, 36, 49, 64, 8i, 100, 10,000. 7 2, 4 7 7 7 (885) 886 soul E's PHILOSOPHIC PRACTICAL MATHEMATICS. #. We here see that the square root of a number less than 100, is less than 10; the square root of a number less than 10000, is less than 100, etc. Therefore the square of any single figure will always be found in the first two right hand places of the power, and the square of a number of two figures, in the first four places of the power, every two figures in the square giving one figure in the root. For this reason, in the extraction of the square root, we separate the numbers into periods of two figures each, from the right, though the left hand period will sometimes contain but one figure, when its root is less than 4. To investigate the principles for the extraction of the square root, let us take a number of two places of figures as 54, and represent the digit in the tens' place by a (5), and the digit in the units' place by u (4); then the number 54 or (50 + 4) will be represented by 10a; + u. We assume this binomial for the basis of our operations, instead of the number itself, because first, principles can be more clearly demonstrated by generalizing their formula than by using ordinary figures, as these special figures would serve to obscure the exhibition of general principles; and we prefer the above form of the binomial (10a; + w) to the one generally used, viz., (a, + w), because the relative position of these factors, aſ and u, in the expansion, is more easily perceived, and the reason for certain operations, and the location of their results in the extraction of roots, can be more readily explained. EXPANSION, 10a; -- u Explanation.—We here square 10a: + w, i. e., 10a; -- w multiply it by itself. The process is so plain *=== that no explanation is necessary. We find the 10xu w” result to be 1002.” -- 20xu + w”, or in common 100x2 + 10 cu. language would be read thus: One hundred * times the square of the tens, plus twenty times 100.c” -- 20aw —- wº the tens multiplied by the units, plus the square of the units. Let us now arrange the formula in a vertical column on the left, then substitute the digits of the number 54 for their representatives aſ and 100ac2 = 5 × 5 × 100 = 2500 or 25 w, and perform the operations indicated, and we 20acw = 5 × 4 × 20 = 400 or 40 have for the several terms these partial results, w” = 4 × 4 = 16 or 16 viz.: For 100x2 = 2500; for 20xu = 400; for wº = 16; and their sum = 2916 for the square of 100.cº. + 20xu + w” = 2916 = 2916 54. Also observe that the coefficient of ac” is 100, and contains two ciphers; the coefficient of acu is 20, and contains one cipher less than the first, and w? has no cipher, or one less than the last. By virtue of this constant arrangement, we can cancel or discard the ciphers altogether, and arrange the results as in the last column, where we see that after setting down the value of 2* = 25, we place the next term 2a:w = 40, one place to the right, and wº = 16, one place further to the right. This arrangement gives the ordinary binomial square of a -- w = z*-H 2xu + w”, and in this form we will use it for convenience after the first illustrations. Let us now find the square root of 2916, from the above formula, by resolving it into its factors. EVOLUTION, 887 THEORETICAL OPERATION. 100a:* -i- 20xu + w” = 29.16 ( 5 4 Q. -- ?!, 1003:2 25 00 + 20a = 100 ) 4 16 Q!, E 4 X -*- w = 4 × 104 20aw -- w? = 4 16 0 PRACTICAL OPERATION. 29.16 ( 54 25 104 ) 4 16 4 16 *-*. 0 Explanation.—Since evolution is, the reverse of involution, we will commence with the term on the left, 100a:”; we first divide by the 100, by, pointing off from the right, two places; now we know that a, the square root of acº, must be found wholly in the figures to the left of the point, viz., 29, the greatest perfect square of which is 25, whose square root is 5; we place this to the right like a quotient in division, and designate it by a ; we then subtract its square 25 from 29, leaving a remainder 4, to which we annex the second period 16. This new dividend 416, we see is what remains after subtracting the value of 100.cº from 2916, and hence the 416 is equal to the (20ru + w”); now, as we do not yet know the value of u, we will eliminate wº from both terms of 20xu + w”, which gives us (202 -- wy w = 416, which we arrange vertically as in the operation; and as we do know...the value of 20x = 20 x 5 = 100, and that it is many times greater than w, we can consider it as a trial divisor, and say, 20a or 100 divided into 416 gives approximately a quotient 4 = w, which we place in the root. Having now found w = 4, we add it to 100, making 104 = 202 + u, for the true divisor; we now multiply this by w = 4, and obtain 20a;u + w” = 416; subtracting this from the dividend, and finding no remainder, completes the work, and gives the exact root, 54 = x: –H u, or rather 10a; -- w. In the practical operation we omit the two naughts of the 100, and find the first figure of the root as before, subtract its square, and bring down the next period for a new dividend. We then omit the naught of the 20, and use only the 2 to multiply into the tens’ figure of the root 5, thus, 5 × 2 = 10, for a trial divisor; to compensate for the elision of the divisor, we cut off the right hand figure of the dividend before dividing to find the units’ figure of the root, or w, that is, we say 10 into 41 = 4; we then annea, this 4 to the 10, making 104 for the true divisor, which being multiplied by 4, and the product subtracted from the dividend, leaves no remainder; we then place the 4 in the root, which completes the work. The philosophy of the theoretical process should be thoroughly understood, and then the reason for the contractions in the practical method will be apparent. Since all numbers above units may be considered as composed of tens and units, thus 456 may be called 45 tens and 6 units, therefore when there are several figures in the required root we may, after having found the first two left hand figures of the root, consider them as tens, and the next figure to be found as the units' figure, and use the same formula and process for finding the third figure as we did for the second; and the same principle and application may be extended indefinitely to any number of places, so that, by the same kind of operation, the square root of any number may be found, or approximated, as illustrated in the following example: 2. Find the square root of 56475225. OPERATION, Root. 56'47.5225 ( 7515 49 7 × 2 = 145 ) 747 5 725 75 × 2 = 1501 ) 2252 1 1501 751 × 2 = 15025 ) 75.125 75.125 0 Ea:planation.—We first point off into periods of two figures each from the right, them find the greatest perfect square in the first left hand period, 56, to be 49, the square root of which is 7, which we place to the right for the first figure of the root, and its square, 49, we subtract from 56; to the remainder 7 we bring down or annex the second period, 47, giving us 747 for the first dividend; we then multiply the figure in the root 7, by 2 = 14 for the first trial divisor, and ascertain how often it is contained in the dividend exclusive of the right hand figure, i. e., in 74, (for the reason explained in the previous example, having multiplied the root 'by 2 instead of 20); we find the nearest quotient 888 SOULE's PHILOSOPHIC PRACTICAL MATHEMATICs. 3. to be 5, which we place in the root, and also at the right of the trial divisor 14, giving 145 for the true divisor, which we multiply by the 5, and subtract the product 725 from the dividend 747, leaving a remainder of 22, to which we annex the third period, 52, giving 2252 for the second ºlividend; we then multiply the figures in the root, 75 by 2, or add the last figure of the root, 5, to the last divisor, 145, giving 150 for the second trial divisor, which we find is contained in the new dividend exclusive of the right hand figure (225), 1 time; we place this figure 1 in the root, and also to the right of the trial divisor 150, giving 1501 for a true divisor, which we multiply by the l, and subtract the product from the dividend; to the remainder, 751, we annex the fourth period, 25, giving 75.125 for a new dividend, then double the figures of the root, 751, or add the last root figure, 1, to the last divisor, 1501, giving us 1502 for a new trial divisor, and operating as before explained, we find the next root figure to be 5, which we place in the root and at the right of the *: then after multiplying and subtracting, as there is no remainder, we have the exact root o, If there was a remainder, we could annexperiods of ciphers, and continue the operations in the same manner until sufficiently exact, each period of ciphers annexed giving one decimal place in the root. Whenever the significant figures of the given number do not give an exact root, the number is a surd, and the root Indeterminate, and cannot be exactly expressed by any combination of fractions, either vulgar or decimal, but admits of an infinitesimal approximation by decimals. When the product of any divisor by its root figure is greater than its corresponding dividend, We must diminish the last figure of the root and divisor. }. When the dividend, exclusive of its right hand figure, is less than the trial divisor, a cipher must be placed in the root, and also at the right of the divisor, then bring down the next period to the right of the dividend, and proceed as before. When the given number is a decimal, or a whole number and decimal, we point off the decimal part into periods, towards the right from the decimal point, annexing a cipher if necessary to make the number of places even. When the given number is a fraction, reduce it to its lowest terms; if it is a mixed number, change it to an improper fraction; then if both terms of the fraction are perfect powers, extract the root of each; if one or both terms are surds, multiply them together and extract the root of the product to a sufficient approximation, then divide the result by the denominator; or reduce the fraction to a decimal and extract the root. Thus, the square root of #4, or Vă = vº. = 3, Ans, And the square root of 124 = Vºſ = } = 34, Ans. And the square root of g = V5 × 8 = V40 = 6.3245 +, divided by 8 = .7905 +, Ans. Or, Vă = v.6250 = .7905 +, Ans. EXAMPLES. 1. What is the square root of 13778944? Ans. 3712. 2. What is the square root of 2106.81? Ans. 45.9. 3. What is the square root of .00007225? Ans. .0085. 4. What is the square root of 7 ºr ? Ans. 23. 5. What is the square root of 4257+?, ? Ans. 654. 6. What is the value of 59049? º Ans. 243. 7. What is the value of V813 - Ans. 729. 8. What is the square root of 781260581179521? Ans. 27951039. APPLICATIONS OF SQUARE ROOT. 1429. 1. What is the length of the side of a square field, in yards, that contains 40 acres? Ans. 440 yards. 2. What is the length and width of a rectangular field, in yards, that is 24 times as long as wide, and contains 25 acres? Ans. 550 yards long. 220 yards wide. º: EVOLUTION. 889 SOLUTION. 25 acres = 121000 square yards; then 121000 – 24 = 48400 = square of shortest side; then v.484.00 = 220 yards, shortest side. It is demonstrated in geometry that the square of the hypotenuse of a right-angled triangle is equal to the sum of the squares of the other C two sides. In the annexed diagram we see that the square of the hypotenuse ac, is equal to the sum of the squares of the base ab and perpen- dicular be, and hence when any two of the sides are given, the third can be found; thus, when the base and perpendicular are given, we find the square root of the sum of their squares for the hypotenuse; and when the hypotenuse and | Base. B one side are given, we find the square root of the difference of their squares for the other ac” = ab? -- bcº side. i. e., 5* = 4” + 32 Or, 25 = 16 -– 9 We also observe that the areas of all similar triangles, with the same base, have the same ratio to each other as their altitudes, and All similar triangles have their areas proportional to the squares of their corresponding sides. 3. A wall 40 feet high stands on the side of a moat 30 feet wide; find the length of a ladder that will reach from the opposite side of the moat to the top of the Wall ? Ans. 50 feet. 4. In the center of a garden 400 feet square, there is a pole 100 feet high. What is the distance from each corner of the garden to the foot of the pole, and also to the top of the pole? Ans. Distance from each corner to foot of pole, 282.84+ feet. Distance from each corner to top of pole, 300 feet. SOLUTION. # of 400 = 200; then, V2002 × 2 = V/80000 = 282.84+ And V10O2. T80000 = 300. 5. A square plot of ground contains 24 acres; what will be the area, the length of sides, and the diagonal, in yards, of another square field whose sides are ten times as great 3 Ans. Area, 250 acres. Length of side, 1100 yards. Length of diagonal, 1555.63+, yards. SOLUTION. 2} acres × 10% = 250 acres. Then, V250×4840 = 1100 yards on each side. And V2 × 1100° = 1555.63+, yards diagonal. 6. A room is 18 feet wide, 24 feet long, and 124 feet high ; what is the length of the diagonal drawn from one of the lower corners to the upper corner farthest from it # , Ans. 32% feet. SOLUTION. V18” -- 24?–E (12})* = 324. 890 soul E's PHILOSOPHIC PRACTICAL MATHEMATICs. * 7. Two ships sail from the same port; one due east at the rate of 300 miles per day, the other due south at the rate of 125 miles per day. After a certain time they were 1625 miles apart; allowing the ocean to be a plane, how many days were they out 3 Ans. 5 days. OPERATION INDICATED, 300? – 90000 1252 – 15625 V105625 = 325 miles apart the 1st day. Then, 1625 – 325 = 5 days. 8. A rectangular field is 23 times longer than it is wide, and the length of its diagonal is 520 rods. How many acres does it contain } AnS. 600 acres. SOLUTION. V1°-F (2})* = 23, proportional diagonal. And 520 – 23 = 200, ratio of diagonals. Then, 1 rod x 200 = 200 rods wide. And 2% rods X 200 = 480 rods long. 480 × 200 Then ** = 600 acres. 9. A ladder 15 feet long, with its foot a certain distance from a wall, just reaches the top of the wall; another ladder 20 feet long, will reach the top of the wall when its foot is 7 feet farther from the wall than the first. What is the height of the wall, and the distance of the foot of each ladder from the wall? Ans. Shortest base, 9 feet. Longest base, 16 feet. Height of wall, 12 feet. solutios. 20* — (15* + 7°) = 126 -- (2 × 7) = 9 feet shortest base. The other conditions are now easily found. The above solution depends on the following principle, viz, : The difference between the squares of the hypotenuses of two right-angled triangles, whose altitudes are the same, diminished by the square of the difference of their bases, is equal to-twice the rectangle (or product) of the difference of the bases by the shorter side. 10. A pole 10 feet from a wall just reaches the top; when removed 10.8 feet farther from the wall, the upper end descends 8.4 feet. What is the length of the pole and height of the wall? Ans. Pole, 26 feet; Wall, 24 feet. SOLUTION. (10 + 10.8)* — (10)” -- (8.4)” -- (2 × 8.4) = 15.6 Then, 15.6 –H-8.4 = 24 feet, height of wall. Or by principle of next example, thus: (20.8 + 10) × 10.8 = 332.64 - 8.4 = 39.6 39.6 –- 8.4 Then, **** = 24 feet, height of wall. ºr'.' . CUBE ROOT. 891. 11. The foot of a ladder placed 7 feet from the middle of a street, the upper end will reach a point on a wall on the nearer side, 60 feet from the ground, and if turned to the other side, will reach a point 52 feet from the ground. What is the length of the ladder, the width of the street, and the distance of the foot of the ladder from each side of the street 2 Ans. Length of ladder, 65 feet. Width of street, 64 feet. Distance of ladder from nearer wall, 25 feet. Distance of ladder from farther wall, 39 feet. SOLUTION. 60 + 52 = 112 × 8 = 896 -- (2 × 7) or 14 = 64 feet, width of street; then, 64 -- 2 = 32 feet, half the width of the street; and 32 — 7 = 25 feet, distance to nearest wall; and 32 + 7 = 39 feet, distance to the farther waii. - Then, W60?-E25% or V52* T393 = 65 feet, length of ladder. The above solution depends on the following principle, viz.: In two right- angled triangles with the same hypotenuse, the difference of the squares of two similar sides, is equal to the difference of the squares of the other two similar sides, hence 60° — 52* = 896 is also equal to the difference of the squares of the two portions of the width of the street; but the difference of the squares of two num- bers is equal to the sum of the two numbers multiplied by the difference, hence 60 + 52 = 112 × 8 = 896; and since the difference of the two portions of the street is equal to twice the distance of the ladder from the centre, or 2 × 7 = 14, and as the sum of the two portions of the street multiplied by their difference is, by the above principle, equal to 896, if we divide 896 by 14, the quotient, 64 feet, will be the width of the street; the remaining part of the operation is self-evident. CUBE ROOT. 1434. Since the cube of a number is the continued product of the number taken three times as a factor, or raised to the third power, as explained in involu- tion, the cube root of a number is the converse of the cube, or one of the three equal factors of the given number. The following comparison of the digits, etc., and their cubes, will illustrate the relation between numbers in general and their third powers, or cubes. The cubes of the nine digits should be committed to memory. Numbers, 1, 2, 3, 4, 5, 6, 7, 8, 9, 10, 99. Cubes, 1, 8, 27, 64, 125, 216, 343, 512, 729, 1000, 970299. By comparing the above, we observe that the cube of the smallest number of a single figure (1), contains one figure (1), and the cube of the largest single figure (9) contains three figures (729), or two more than the smallest; also the cube of 10 = 1000, and the cube of 99 = 970299, or two places more than the 10. Therefore we see that the cube of a single figure will contain from One to three places; the cube of a number of two figures will contain from four to six places; and in 892 soul.E's PHILOSOPHIC PRACTICAL MATHEMATICs. * . general, the number of places in the cube will be at most three times the given number of places, and at least three times the given number, less two; and con- versely, we may deduce the general principle that a number, or cube, consisting of one to three figures, will give one figure in its root; when of four to six figures, its root will contain two figures; from seven to nine figures, gives three figures in the root, etc. This is the reason why, in extracting the cube root, we point off the number into periods of three figures each, from the right, each period giving one place in the root. The left hand period may contain one, two, or three figures, and its root is always found by inspection, or from the tabular cubes of the nine digits given above. To demonstrate the formation of a cube, let us take a number of two places of figures, as 54, and represent them, as in the elucidation of the square root, by 10a: + u, respectively. Then, raising this to the third power by simple multiplica- tion, we obtain the following expansion, i. e., (10a: + u) * = 1000a:* + 300a:*u + 30aw” + w”. From this quantity we see that the third power of a is taken 1000 times, and hence it is necessary, before finding the value of a, to divide by 1000, because the third power of 10a; has no representation in the first three right hand figures of the entire cube of 10a: + u, but lies wholly to the left. From the above formula we see that the cube of any number consisting of tens and units, is equal to 1000 times the cube of the tens, plus 300 times the square of the tens multiplied by the units, plus 30 times the tens multiplied by the square of the units, plus the cube of the units. We will now expand the number 54 to the third power, by the formula, and for convenience will arrange the terms in columns, to correspond with the numerical denominations, Thus, (10x + w)* = (50 – 4)* Or, 1000a:8 = 5 × 5 × 5 × 1000 = 125000 300acºw – 5 × 5 × 4 × 300 = 30000 30acu” = 5 × 4 × 4 × 30 = 2400 w° = 4 × 4 × 4 == 64 1000x3 + 300ac”w – 30aw” -- w8 = 157464 We here make a = 5 in ten’s place, and u = 4 in unit's place; then perform- ing the operations indicated in the formula, we obtain the several results corres- ponding to the equivalent terms of the formula. We see from these partial results that the third power of the tens (5) has been multiplied by 1000, giving 125000; therefore in extracting the cube root, we must first divide this number by 1000 before proceeding to resolve the 125 into its three equal factors; the first root figure or factor must always be found by inspection or trial, and in this case we find it to be 5 tens, the 5 being the cube root of 125, and the tens the cube of the 1000, which was rejected by division, or pointing off. If our number was just 125000, then 5 tens or 50 would be the required root, but as the whole number is 157464, we know there must be a unit figure to produce the other partial results, and with the 125000 make up the total 157464, whose cube root is required. * CUBE ROOT. - 893 To find the unit's figure, let us resume our analysis by taking the entire number to operate on in regular order, instead of the partial results, thus: 157464 ( 54 1000a:3 – 53 125 300ac2 = 52 × 300 – 7:500 ) 32464 } 30a:w = 5 × 4 × 30 = 600 w” = 4 × 4 = 16 X *sº w or 4 × 8116 = 32464 We first place the formula on the left, observing to eliminate u from the three last terms, as Indicated by the brace, and place the w at the bottom, as a final multiplier when found; we then point off the number into periods of three figures, as before explained, and find by trial the largest cube in the left hand period to be 125, the root of which is 5 tens; we place this root figure to the right, as a quotient in division, and its cube 125 under the first period, and after subtracting, annex the second period. This constitutes the first dividend, and from the formula we see that this amount 32464, contains 300a:*u + 30aw” + w8, but as we do not yet know the unit figure (w), we will eliminate, or take out, (w) from each term, and obtain (300a:* + 30aw -- wº) u = 32464. Now if we knew the value of the quantities con- tained in the parenthesis, we could find u by dividing 32464 by that value. We do know, however, the value of the first term 300a,”, which is 7500, and from a com- parison of the several terms, we see that the first term 300a,” is many times greater than all the other terms; we will therefore use this value, 7500, for a trial divisor, and find that it is contained in the dividend, 4 times, with a remainder which we ignore; we place the 4 in the root, and then perform the operations indicated by the remaining terms in the parenthesis or brace (30aw -- u%), that is, we add to the trial divisor, 7500, 30 times the ten's figure 5 multiplied by the unit's figure 4, or 5 x 4 x 30 = 600, and also the square of the unit's figure 4, or 4 x 4 = 16; the sum of these, 8116, is the complete divisor, and this multiplied by the last root figure, 4, and the product extended and subtracted from the dividend, leaves no remainder; therefore 54 is the exact cube root. 2. Find the cube root of 2797.26264. 279,726'264 ( 654 216 1000.03 – 63 - 300ac2 = 62 × 300 = 1 ( 10800 ) 63726 1st dividend. 30a:w - 6 × 5 × 30 = 2 900 X *=s* * Ans. 14687.459 Kg. 9. Add the above problem in the unit of Tonneaus. Ans. 14.687459 T. 10. What is the sum of 124 sq. M., 6 sq. dm., and 37 sq. cm. ? Ans. 124.0637 sq. M. OPERATION INDICATED. 124. ,06 .0037 - *- 11. Add 42.8 sq. M., 21.65 sq. M., 28 sq. dm., and 4 sq. cm. Ans. 64,7304 sq. M. 12. Add in the unit of Ares, 39.5 A., 25 Ha., and 84 ca. Ans. 2540.34 A. 13. What is the sum of 14.5 c.u. M., 23 cu. dm., 123 cu. cm., and 24 cu. mm. 3 Ans. 14.523123024 cu. M. OPERATION INDICATED. 14.5 .023 .000123 .000000024 14. Add in the unit of Steres, 22 S., 12 Ds., and 15 ds. Ans. 143.5 S. TO SUBTRACT METRIC NUMBERS. 1508. 1. What is the difference between 75.6 M. and 85 mm. 3 Ans. 75.515 M. OPERATION. \ 75.6 — M. Explanation.—In subtraction as in addition, .085 = M. we write the numbers to be subtracted in the base unit of the measure table to which the 75,515 M. Ans. numbers belong, and subtract as in decimals. GENERAL DIRECTIONS. 1509. From the foregoing elucidation, we derive the following general direc- tions for Subtracting Metric Numbers: Write the numbers to be subtracted in the base unit and decimals thereof, of the table to which the numbers belong, and then subtract as in decimals. Art, 447, page 224. PROBLEMIS. 2. From 4324.08 Km. take 123.5 M. in the unit of M. and also of Km. Ans. 4323956.5 M. 4323.9565 Km. 938 -- SOULE's PHILOSOPHIC PRACTICAL MATHEMATICs. * 3. Find the difference between 274.25 L. and 44.5 cl. Ans. 273.805 L. 4. A barrel contained 151.44 L., and 6 L. and 7 dl. leaked out. How many liters still remain # Ans. 144.74 L. 5. What is the difference in L. between 1 Ml. and 1 ml. 3 AnS. 9999.999 L. 6. From 264.5 G. take 28.4 cg. Ans. 264.216 G. 7. From 1428 Kg. take 16.5 Hg. Ans. 1426.35 Kg. 8. What is the difference in T. and Kg. between 2 tonneaus and 2 Kg. ? * Ans. 1.998 T.; 1998 Kg. 9. What is the difference between 88.21 sq. M. and 38 sq. dm.” Ans. 87.83 sq. M. 10. A plantation contains 2471.14 Ha. If 2471.14 A. are sold, how many hectares will remain } Ans. 2446.4286 Ha. 11. From 1528 cu. M. take 1 cu. M. 1 cu. dm. 1 cu. cm. and 1 cu. mm. * AnS. 1526.998998999 Cu. M. 12. A wood dealer bought 150 steres of wood; he sold 70.25 cu. M. How much has he remaining 3 * Ans. 79.75 S., or cu. M. TO MULTIPLY METRIC NUMBERS. 1510. 1. How many meters are there in 11 pieces of cloth, each containing 36.25 meters ? “ Ans. 398.75 M. OPERATION. 36.25 Explanation.—In multiplication of Metric 11 - Numbers, we proceed in the same manner as in simple and decimal numbers. Hence no extend- 398.75 M. Ans. ed educidation and general directions are deemed necessary. 2. What will 204.5 meters cost at $2 per meter? Ans. $409. 3. If a man walks 20000 meters per day, how many kilometers will he walk in 60 days? Ans. 1200 Km. 4. What will 484 dm. cost at $1.50 per meter? Ans. $72.60. 5. What will 484 dm. cost at $1.50 per dm * Ans. $726. 6. What cost 75.2 liters of milk at 52 per L. Ans. $3.76. 7. What cost 200.52 Hl. of corn at $1.60 per Hl.? Ans. $320.83. 8. A fruit dealer bought 6 Hl. of pecans, at $9 per Hl., and sold them at 152 per liter. How much did he gain? Ans. $36. 9. Sold 26.4 Kg. of grapes at 452 per kilo. How much was received for them ż * Ans. $11.88. 10. What cost 27.25 tonneaus of hay at $20.50 per T. * Ans. $558.625. 11. What cost 416.3 Kg. of sugar at 20g Ans. $83.26. 12. Bought 362,3122 G. of silver at 34% per gram. What did it cost? Ans. $12.68–H. 13. How many kilograms in 49000 pills, the weight of each being .5 dg. 8 Ans. 2.45 Kg. 20. METRIC SYSTEM OF WEIGHTS AND MEASURES. 939 14. How many sq. meters in a yard 40.5 M. long and 15.24 M. wide? *. Ans. 617.22 sq. M. 15. A plantation is 2.42 Km. long and 1500 M. wide. How many hectares does it contain 3 Ans. 363 Ha. 16. How many cubic meters in a box 2.8 M. long, 2.1 M. wide, and 8.5 dm. deep 3 Ans. 4.998 cu. M. 17. What will be the freight on 4 boxes, each measuring 3 M. by 26 M. by .9 M. at $5.25 per cu. M. ? Ans. $147.42. 18. How many cubic meters of earth in a levee 140 M. long, 2.3M. deep, and 20 M. wide at the base and 15.2 M. wide at the top 3 Ans. 5667.2 cu. M. 19. How many steres in a pile of wood 28.5 M. long, 3.2 M. high, and 4.3 M. Wide 3 Ans. 392.16 S., or cu. M. 20. How many cubic meters of earth will it take to fill a lot .5 meter deep, the lot being 60.2 meters long, and 25 meters wide? Ans. 752.5 cu. M. TO DIVIDE METRIC NUMBERS. 1511. 1. A man walked 1600 meters in 20 minutes. How many meters did he walk in 1 minute 3 Ans. 80 M. OPERATION. 1600 — 20 = 80 M. Ans. Explanation.—In division of Metric Numbers, we proceed with the operations in the same manner as in simple and decimal numbers. Hence no extended elucidation and general directions are deemed necessary. 2. The steamer Katie ran 500 Km. in 22 hours. How many kilometers did she run per hour? Ans. 22.727–H Km. 3. If 6.5 meters make a suit, how many suits can be made from 195 meters? Ans. 30 suits. 4. In 425 liters, how many hectoliters? g Ans. 4.25 Hl. 5. Bought 45 liters of strawberries for $3.375. What was the price per L. " Ans. 732. 6. If you divide 180 hectoliters of potatoes equally among 24 persons, what will each receive 3 Ans. 7.5 EIl. 7. 21 kilograms of sugar cost $4.62. What was the cost of 1 Kg. ? Ans. 222. 8. How many days will 42.5 T. of coal last a family, if they burn 150 Kg. per day? Ans. 283.33+ days. 9. A druggist has 2.45 Kg. of medicine which he wishes to make into pills, each to contain .5 dg. How many pills will there be? Ans. 49000 pills. 10. A garden contains 900 sq. M., and is 22.5 M. wide. How long is it 3 Ans. 40 M. 11. A side-walk is 80.4 M. long by 4.2 M. wide. How many tiles, each 2.4 dm. long and 1.2 dm. wide, will be required to pave the side-walk 3 Ans. 11725 tiles. 94O soul E's PHILOSOPHIC PRACTICAL MATHEMATICs. ºr 12. How many ares in a piece of land 60 meters long and 42.2 meters wide? * Ans. 25.32 A. 13. A box is 2.5 dm. long, 2 dm. wide, and 1.5 dm. deep. How many of such boxes may be put in a larger box which is 2.5 M. long, 2 M. wide, and 1.5 M. deep Ans. 1000 boxes. 14. If you buy 5.6 dekasteres of wood and use 1.1 cu. meters per day, how long will it last : Ans. 50+} days. TO REDUCE METRIC TO AMERICAN WEIGHTS AND MEASURES. 1512. 1. Reduce 2.6 meters to feet. Ans. 8.53–H feet. FIRST OPERATION. 1 M. = 39.37+ inches, practically. Explanation.—Referring to the table of equiv- tºmº 2.6 alents, we find that 1 M. = (practically) 39.37 12 ) 102.362 inches. in. We then reason as follows: Since 1 M. = 8.530+ feet. Ans. 39.37 in., 2.6 M. are equal to 2.6 times as many, or thus: which is 102.362 in. ; then since 12 in. = 1 ft., 12 ;" & there are ſº as many feet as inches, which is dºmºrºss-sºrs -- *- 8.53–H feet. SECOND OPERATION. 3.2808–1– = feet, the equivalent of a meter. 2.6 8.53008-H = feet, Ans. GENERAL IDIRECTIONS. 1513. From the foregoing elucidations, we derive the following general direc- tions for Reducing Metric to American Weights and Measures: Multiply the equivalent value of the metric unit given, by the metric number to be ºreduced, and them, if necessary, reduce the product to the denomination required. NOTE.-In working the following problems, four decimal places have been taken as a stand- ard for all unit equivalents of more than that many decimals. The calculations are made directly upon the equivalency of the denominations to be reduced. 2. Reduce 408.2 kilometers to yards. Ans. 446407.52 yds. 3. In 24.5 cm. how many inches : Ans. 9.6456-H in. 4. Reduce 70 M. to yards. Ans. 76.552-- yds. 5. Reduce 25.5 liters to dry quarts. Ans. 23.1565-H d. Qts. 6. In 40 kiloliters how many gallons? Ans. 10567.44+ gals. 7. In 200 hectoliters how many bushels? Ans. 567.58+ bus. 8. Reduce 25 grams to Troy grains. Ans. 385.81+ grs. 9. Reduce 75.2 kilograms to pounds Avoirdupois. Ans. 165.7859-H lbs. 10. In 444 tonneaus how many tons? Ans. 489.4256+ tons. 11. Reduce 5.5 mg. to Troy grains. Ans. .0847–H grs. 4. METRIC SYSTEM OF WEIGHTS AND MEASURES. 94. I 12. Reduce 24 ares to square rods. Ans. 94.8912–H Sq. rds. 13. Reduce 85.5 hectares to acres. Ans. 211.279-H acres. 14. In 500 hectares, how many sq. miles? Ans, 1.9305+ Sq. mi. 15. Reduce 150 cu. meters to cu. feet. Ans. 5297.475-H Cu. m. 16. Reduce 7.9 cu. mm. to cu. inches. Ans. .0004819-H cu. in. TO REDUCE AMERICAN TO METRIC WEIGHTS AND MEASURES. 1514. 1. Reduce 40 feet to meters. Ans. 12.192–H M. ()PERATION. § ft. Ea:planation.—Remembering that a meter = 39.37) 480 in. (12.192–H. M. 39.37 inches, practically, we first reduce the or thus: *-*m-mºsºs feet to inches, and then reason as follows: Since 40 in 39.37 in, there is 1 meter, in 480 in, there are 39.37 || 12 as many meters as 480 in, are times = to 39.37 *-* | *=º in., which is 12.192—H. SECOND OPERATION. 3.048 decimeters = the equivalent of 1 foot. 40 121.920 decimeters, which divided by 10 = 12.192 meters, Ans. 2. Reduce 1 mile, 8 rods, 10 feet, and 6 inches to kilometers. Ans. 1.652781–H PGm. OPERATION. 1 mi., 8 rôls., 10 ft., 6 in. 39.37)65070 inches. 320 *** *-*-g &=ºmº 1652.781–H. M. 328 rods. 1.652781–H Km. 16# ** == Or, 5422 feet. 65070 in. × 2.54 cm. = 165278.1+ cm. = 1.652781–H. Km. 12 65070 inches. Explanation.—In this problem, we first reduce the given number to inches, then to meters, as above explained; and then to kilometers by dividing by 1000, the number of meters in a kilometer. GENERAL DIRECTIONS. 1515. From the foregoing elucidations, we derive the following general direc- tions for reducing American to Metric Weights and Measures: 1O. Reduce the given number to the lowest unit named, or to a convenient denomination, of which the equivalent value of the metric unit is known ; then divide the same by the equivalent value of the metric unit, and when necessary reduce the quotient to the required denomination. Or 20. Multiply the given number, reduced to its lowest denomination, by the equivalent value of such denomination in the metric unit. 94.2 SOULE's PHILOSOPHIC PRACTICAL MATHEMATICS. 3. In 70 yards, how many meters? Ans. 64.008-H M. 4. Reduce 35 gallons, 3 quarts, and 1 pint of molasses to liters. #. Ans. 135.8001-H liters. 5. Reduce 5 bushels, 2 pecks, 5 quarts, and 1 pint to liters. Ans. 199.8678-H liters. 6. In 20 gallons of milk, how many liters? Ans. 84,539-H L. 7. In 400 bushels, how many hectoliters? . Ans. 140.9493— Bil. 8. In 3 gills, how many centiliters? Ans. 41.4364-H cl. 9. Reduce 200 pounds Avoirdupois to kilograms. Ans. 90.7194+ Kg. 10. Reduce 15 ounces Avoirdupois to hectograms. Ans. 4.2525+ Hg. 11. Reduce 10 ounces Troy to grams. Ans. 311.0339-H G. 12. Reduce 50 tons to tonneaus. Ans. 45.3597–H T. 13. Reduce 20 grains Troy to centigrams. Ans. 129.6176+ cg. 14. Reduce 240 sq. yards to sq. meters. Ans. 200.6689— sq. M. 15. In 500 sq. feet, how many sq. decimeters? Ans. 4646.8401-H sq. dm. 16. In 200 acres, how many hectares? Ans. 80.9356 Hectares. 17. Reduce 1610 cu. feet to cu. meters. Ans. 45.5877–H cu. M. 18. Reduce 2500 cu. yards to cu. meters. Ans. 1911.3149–H cu. M. 19. In 190 cords, how many steres? Ans. 688.6553–H Steres. MISOELLANEOUS PROBLEMS. 1516. 1. A yard is 21.4 M. long and 15.12 M. wide. How many bricks will it require to pave the yard, each brick being 10.32 cm. long, and 9.16 cm. wide? 2. How many cubic meters in a piece of marble 3.5 M. long, 3.21 M. wide, and 1.85 M. thick? * 3. How many liters in a cylindrical vessel 2.3 M. deep and 8.2 de.. in diame- ter ? 4. What is the weight of 22 bags of wheat averaging 68.42 Kg. ? 5. If 4 men eat 9.5 Kg. a week, how much would 25 men eat at the same rate 3 6. If 4 men can dig 245 M. of a ditch in 24 days, how long will it take 10 men to dig 3200 M. at the same rate 3 7. A room is 6.5 M. long and 4.25 M. wide. How many square meters of carpet will be required to carpet the floor, the carpet being 75 cm. wide, no allow- ance being made for matching patterns or in turning under ? 8. A room is 8.4 M. long, 6.25 M. wide, and 4.5 M. high. How many liters of air does it contain } 9. If the breathing of one person pollutes the air of a room at the rate of .1855 com. per minute, how long will it take, at the same rate, for 50 persons to pollute the air of a room 30 M. long, 20 M. wide, and 8.25 M. high 3 10. If 42 Kg. of cotton are required to make 150 M. of cloth .75 M. wide, how many kilometers of cotton will it take to make 5000 M. of cloth 1.2 M. wide? 11. How many cubic meters in a cylindrical piece of marble 2.1 M. in diameter and 6.42 M. long? - 12. How many square meters, board measure, in 24 pieces of timber 9.32 M. long, 48 cm. wide, and 32 cm. thick? º 1517. Combinations are the different groups which can be made of n things taken m in a group, m being equal to or less than n. Or Combinations are the various ways that different things, the number of which is known, can be chosen or selected taking them one by one, two by two, three by three, etc., without regard to their order. PROPOSITION. 1518. Any number of things being given, to determine in how many ways they may be combined two and two, three and three, etc., without regard to the order. PROBLEMS. 1. If there are four routes 1, 2, 3, 4 leading to New York, in how many different ways can a person go and return ? Ans. 16 ways. Explanation.—Since a person has the choice of going by any route and returning by the same or another route, four choices are offered to him, and since he has the same number of choices with each route (there being four of them) he must therefore have the choice of 16 ways, viz.: Go. Return. Go. Return. Go. Return. Go. Return. 1 1. 2 1 3 1 4 1 1 2 2 2 3 2 4 2 1 3 2 3 3 3 4 3 1 4 2 4. 3 4 4 4 sm-m. g-ºº-ºº- *-mºs ºmºm-º: TAT + TaT + TT + TT– 16 { 2. Suppose in the above problem, the tourist does not desire to return by the same route, how many combinations or choices of route has he? Ans. 12. Explanation.—If he goes by any route, then he has the choice of returning by three others or three ways of making the round trip for each different route leading to New York. Since there are four routes leading there, he has 3 × 4 = 12 choices in the matter, viz.: Go. Return. Go. Return. Go. Return. Go. Return. \ } § 1 1 . 1 & 3 2 & 3 s: 2 *} 2 & 4 & 4 3 3 + 3 + 3 + 3 = 12 From the above the following principle is evolved: If one thing can be done in a different ways, and (when it has been done in any one of these ways) another thing can be done in b different ways, then both can be done in a × b different WayS. (943) 944 SOULE's PHILOSOPHIC PRACTICAL MATHEMATICs. * 3. In how many ways can the letters from the word marble be taken with the letters from the word home 3 Ans. 24 Ways. Explanation.—Each letter of the word marble (there are 6) can be combined with each letter of the word home (there are 4). Therefore we have 6 x 4 = 24 ways they may be taken. Extend- ing the preceding principle, we derive the following: If one thing can be done in a ways, and then a. Second thing can be done in b ways, and a third in c ways, and a fourth in d ways, etc., the number of ways of doing all the things will be a × b X c X d, etc. 4. In how many ways can 5 books be given to five persons? Ans. 120. Explanation.—Since the first book may be given to any one of the five persons, there are five ways of bestowing it. The second book may be given to any one of four persons; the third book to any one of three persons; the fourth book to any one of two persons, and the last book to the fifth person. Hence the books may be bestowed in one of 5 × 4 × 3 × 2 × 1 = 120 ways. 5. In how many ways can a vowel and a consonant be chosen out of the Word logarithms? Work according to first principle. Ans. 21 ways. 6. In how many ways can 4 dolls be given to 3 girls? Ans. 81 Ways. Explanation.—Each doll can be given to any one of the three girls. Therefore the number of ways of disposing of them is 3 × 3 × 3 × 3 = 81 ways. PERMIUTATIONS. —eſpe— 1519. In the definition of combinations, we found that the order of the elements in combination was disregarded. A Permutation of any number of elements or things means a group of that number of elements or things put together with reference to their order or sequence. PROBLEMIS. 1. In how many ways can the letters in the word hound be arranged, taken all at a time Ans. 120 ways. Ea:planation.—The reason given in problem 4 in combinations applies here, and hence is omitted. º Formula 1. The number of permutations of n diifferent elements taken all at a time is * n (n − 1) (n − 2) . . . . . . × 2 × 1. The continued product of this formula is the whole number of permutations of n elements taken all at a time. For convenience this formula may be written | n and is read factorial n. 2. In how many ways can 10 different colored lights be arranged, taken 3 at a time? Ans. 720 ways. Formula 2. n (n – 1) (n — 2) ...... to r factors or n (n − 1) (n − 2) . . . . . . (n – r + 1). Substituting the figures for the letters we have 10 (10 – 1) (10 – 8 + 1) = 10 × 9 x 8 = 720, Ans. ºt PROBABILITIES OR CHANCE. 945 In the formula, n = number of things given to be arranged and r = number of things taken at a time. Formula 3. The number of arrangements of n elements, of which p are alike, q others are alike, and r others are alike etc., is | p X | g × r 3. How many permutations can be made out of the letters of...the Word tintinnabulation? Ans. 12,108,096,000. OPERATION. | 16 | 4 || 3 || 3 4. How many arrangements or permutations can be made out of 8 books, 3 being Arithmetics, 2 being Histories, 2 being Grammars, and 1 being a Dictionary? |2 Ans. 1680. OPERATION. | 8 | 3 || 2 || 2 - -smº - For other and higher cases of permutations, the learner is referred to Wentworth's Complete Algebra. PROBABILITIES OR CHANCE. —eſpe— 1520. This subject has engaged the attention of writers on mathematics since Galileo first wrote on it in 1642. Many mathematicians however, were prejudiced against it on account of its ready applicability to games of chance with cards, dice, etc. When an event can happen in a certain number of ways, and it fails in a certain other number of ways, the chance of its happening can be expressed by a fraction whose numerator expresses the number of chances favorable to the event happening and whose denominator expresses the whole number of cases. Simpson defines the probability of an event happening to be the ratio of the chances by which the event in question can happen to all the chances by which it cannot happen or fail. If m + n be the whole number of cases that can happen, and m represents the number of cases favorable to the happening of the event, the probability of the event happening would be expressed by the fraction H and 946 SOULE's PHILOSOPHIC PRACTICAL MATHEMATICs. Yºr the chances against its happening would be 770, + n" Therefore we see that the sum of the chances of an event happening or not happening is unity or certainty. In tossing a coin once, what chance is there that it will turn up a head or a tail? Since there are only two things possible to turn up, either a head or a tail, it is evident that the probability of its being a head is H = } and the probability that it will be a tail is 1 # i = }. The sum of the chances for and against is 3 + 4 = 1 = certainty. That is, it is certain that one or the other will turn up. What chance is there in two throws of turning a head up twice? According to the subject of combinations or choice, we find that with 2 throws there will be 4 combinations possible. 1st explanation for Problem 1, in Combinations, is applicable, viz: 1 head and head. 2 head and tail. 3 tail and head. 4 tail and tail. Of the four ways in which it is possible for the coins to fall, there is only 1 way in which the event will happen. The probability of the event happening is hence #. It might be shown that the chance of throwing 3 heads or 3 tails in succession, will be $, or 4 × 4 × 4. The chance of two or more single events being known, the chance of their all taking place together, may be found by multiplying the probabilities of the events, considered singly. To find the chance of throwing in one throw all heads or all tails with 2, 3, or 4 pieces, may be found as above. When two pieces are tossed up there are 4 com- binations possible, hence the chance is 4. When three pieces are thrown up there are 8 combinations, for each coin may fall in one of two ways, and there being 3 coins there are 2 × 2 × 2 = 8 combinations possible. The chance is therefore #. Tossing up three pieces at once, is just the same as tossing up the same piece 3 times. e 1. What odds should be offered to make an even bet that in 2 throws a head will fall at least once? Ans. 3 to 1. As we have seen before, in two throws there is only 1 throw in which we find no head, while we have a head in three of them.' 2. What chance is there in one throw of throwing an ace with one die? Ans... } 3. What chance is there in one throw of throwing double aces with two dice? Ans. #'s. PROBABILITIES OR CHANCE, 947 1521. TABLE TO SHow How MANy ways A GIVEN SUM MAY BE THROWN WITH ONE, TWO, THREE, OR MoRE DICE. Number of dice. 1 2 3 4 5 6 | 1 l 2 || 1 || 1 Ea.planation of Table.—If it be required to find how many ways 8 could be thrown with 4 dice, we first look © 3 || 1 || 2 | 1 in the 1st column on the left for 8, and at the top of tº # 4 || 1 || 3 || 3 1 the table for 4, and in the square below opposite to the 3. ETTT. To Tal ITT 8 we find 35, the number of ways in which 8 may be 3 |—|— thrown with 4 dice. The probability of throwing 8 § 6 || 1 5. 10. 10| 5 || 1 points with 4 dice is 1355. The probability of throwing É 7 6 || 15 20| 15| 6 any number of points with a given number of dice is 2. 8 5 21 || 35| 35| 21 found by dividing the number of ways the points can 9 4 || 25 6| 70| 56 be thrown with the given number of dice by that 5 56 7 power of 6 indicated by the number of dice. 10 3 27 | 80|| 126|| 126 11 2 27 | 104| 205 252 12 1 || 25 | 125, 305| 456 IPROBLEMS 1. Two persons sit down to play for a certain sum of money; and agree that he who first gets three games shall be the winner. One of them won 2 games and the other, won 1; but being unwilling to continue their play, they resolve to divide the stake; how much ought each person to receive? If the play were to be continued, and the second should win, then each one having only 1 game to win out, would be entitled to half of the money. But if the first one should win the game, the whole money (#) deposited would be his; and if he lost it, he would have a right to the half. But since one case is as probable as the other, the first player has a right to one-half of the sum of the two parts (3 + 3 = }) = }, and the second player only #. 2. Suppose the first player needs one game to win, and the second three, how should the money be divided ? If 'the first player should win one game, he would be entitled to all (#) the money; and if he lost the game he would be entitled to #, according to the case above. But as both are equally probable, the first player should have # of the two sums (4 + # = }) = }, and the Second player #. 3. Suppose the first player needs two games to be out, and the second three, how should the money be divided ? - If the first player won the game, he would be entitled to , as shown in the 948 soul E's PHILOSOPHIC PRACTICAL MATHEMATICS. Yºr “... problem above. If the second player won, each would be entitled to #. But as both are equally probable, the first player is entitled to # of the sum of the two parts (; + 4 = ##) = +}, and the second to #. 4. If 4 cards are drawn from a pack of 52 cards, what is the chance that four kings will be drawn f FIRST OPERATION. * * * * * * * = 270725 = number of ways four cards can be drawn from the pack. 1 × 2 × 3 × 4 * SECOND OPERATION. Four, cards can be selected so as to be kings in 4 × 3 × 2 × 1 = 24 ways. Therefore the chance of drawing four kings from a pack is grážg or 1 chance in 11280. The first operation depends upon the principle, that “Out of n different elements, the number of selections of r elements is equal to the number of arrange- ments of n elements divided by | rº' The formula for which is n (n − 1) (n — 2) . . . . . . (n – r + 1) | r 2 S = The second operation depends upon the principle, that “The number of arrangements or permutations of n different elements taken all at a time is n (n — 1) (n − 2). . . . . . × 2 × 1.” This formula is represented by Ln. 5. If 2 letters are selected at random, out of the alphabet, what is the chance that both will be vowels? Ans. 1 in 32. OPERATION, Two letters can be selected from 26 in *::: = 325 ways; and 2 vowels can be selected from 5 vowels in ; X : = 10 ways. Therefore the chance of drawing 2 vowels is ºr = ºr or 1 chance in 32. 6. In a box of chess are 16 pawns, 4 knights, 4 bishops and 2 queens. What is the chance of drawing one of each in 4 draws? Ans. 1 in 700. OPERATION, 26 × 25 X 24 × 23 1 × 2 × 3 × 4 1 pawn, ºan be selected from 16 pawns in 16 ways; 1 knight from 4 knights in 4 ways; 1 bishop from 4 bishops in 4 ways, and 1 queen from 2 queens in 2 ways. Hence 1 pawn, 1 knight, 1 bishop, and 1 queen can be drawn in 16 × 4 × 4 × 2 = 512 ways, Hence the chance is gºt or 1 in 700, 4 pieces of chess can be selected from 26 pieces in = 358800 ways. Also Çontinued Fractions. ----------------------------Rs —sº a dº- 1522. A continued fraction is a fraction with one for a numerator, and whose denominator is an integer plus a fraction with one for a numerator, and whose denominator is an integer plus a fraction, etc. NoTE.—For a full elucidation of fractions, see pages 164 to 167, and 216 to 219. Continued fractions were first used by Cataldi, in 1613, and by Lord Brounker, in 1670. Continued fractions are used to express approximately, ratios that by reason of the large numbers used, do not carry with them appreciated values. They are also used to determine approximations to roots of surds, and are also applied to the Solution of indeterminate problems. Reduce # to a continued fraction. 1. 2.7 # 5 -as - 3 + 1 6T1 1 + + DIRECTIONS.—Divide both numerator and denominator by 27 and we have #x ; divide the numerator and the denominator of the second fraction by 4, and 1 we have 3 + I_; divide the numerator and the denominator of the third fraction 6 - # by 3, and we have 1 3-E I 6 —H 1 I-Ei Ans. This completes the work, as the numerator of the last fraction is unity. Reduce the continued fraction 1 3 + 1 6 1 I-E to a common fraction. First Method. 1. 1 + # º by multiplying the numerator and the denominator by 3, adding to the denominator * (949) Reduce the last two terms of the continued fraction to a simple fraction 95o SOULE's PHILOSOPHIC PRACTICAL MATHEMATICs. º: f. 3.& the numerator of the last fraction, and we obtain #. Take this result and the preceding partial fraction, and we have *—. multiply as before, and we have the simple 6 + #7 fraction #1. Take this result and the preceding partial fraction, and we have ; Tº multiply as before, and we have #, the simple fraction. Second Method. 1st step = } = 1st approximation. 2d step = *H = +% = 2d approximation. 3d step = :g-H = # = 3d approximation. 4th step = #H = # = original fraction. Explanation.—Write the first term (#) of the continued fraction as the first approximate fraction. Write this approximate fraction with the next term of the continued fraction and reduce to a simple fraction, and we have : = }; second approximate fraction. Write this ; multiply this fraction l by the denominator of the second term as above, and add to both terms the corresponding terms of 1 × 6 + 1 1 × 19 + 3 Write the third approximate fraction with the last term of the continued fraction, and we have fraction with the next term of the continued fraction and we have : the first approximate fraction ($), and we have = y; the third approximate fraction. # # ; multiply both terms by 3, and add to the product the corresponding terms of the second + approximate fraction (#), and we have ####, = ##, the answer. To illustrate the application of continued fractions, let us find the approxi- mate ratios of a circle to a circumscribed square. The ratio is ###. Reducing this to a continued fraction, we have 1 1 + 1 3 } *— Now obtain the approximate values according to the 1st or 2d methods, and 4 7 1 7.2 18:3, 3 5.5 - 89.3 3 927 7.8 & 4 we have +, #, Tây 32 ++, 2 197 ###, 4 52.7 TI37; 5 JO (, ) iſ Ó O Ú (). NoTE.—Fractions should be reduced to their simplest form before beginning the operation. 1523. 1. The product of three numbers is 71757; one of the numbers is 51, and another 21. What is the third number? Ans. 67. 2. What is that number which being divided by 25, the quotient increased by 6* + 2, the sum diminished by the difference between 41 and 25, the remainder multiplied by 9, and the product divided by 27, the quotient will be 10? - Ans. 200. 3. Henry owes James $28.50 which he wishes to pay with an equal number of picayunes, dimes, quarters, halves and dollars. How many of each will it require ? Ans. 15. 4. What is the fraction, which being divided by gº, will produce #3 Ans. Zºo. 5. What number, multiplied by 3, will give a product of 22? Ans. 33. 6. What fraction, to which if you add +, the sum will be #3 Ans. #. 7. What quantity, to which if you add 3 of itself, the sum will be 75? Ans. 45, OPERATION. # -- 1 = 1% = }, then if # = 75, # will = 15 and # = 45. Ans, 8. What quantity, from which if you subtract 3 of itself, the remainder will be 15% Ans. 24. OPERATION. 1 — # = }, then 15 — # = 24. Ans 9. Divide 18 oranges between A. and B. so that A, will have + more than B. What number will each have? - Ans. A. 10. B. 8. OPERATION. - B. A. 18 18 9 || 4 9 || 4 1 assumed for B. &=ºmº | * 4 5 1+ ( & & 4 A. 8 - -* e- 10 2# = # the sum of the proportions according to the conditions of the question 10. Divide 18 oranges between A. and B. so that A. will have + less than B. What number will each have? Ans. A. 7# B. 10%. OPERATION. B. A. 18 ; 18 7 || 4 7 || 4 1 assumed for B. *-mº | -- 4 || 3 # & 4 ** A. º 10% - --> 7} 1} = 4 the sum of the proportions according to the conditions of the problem. (951) 952 SOULE's PHILOSOPHIC PRACTICAL MATHEMATICs. * - 11. Divide $2000 among A., B. and C. so that A’s part will be to B's as 2 to 3, and that C. will have as much as A. and B. together, less $200. What amount will each receive? SOLUTION. As C. is to have as much as A, and B. together, less $200, A. and B. will therefore have 3 of $2000 + 3 of $200, which is $1100. Then, as A. and B. are to receive this in the proportion to 2 and 3.A. will therefore receive # and B. 3 of $1100. # of $1100 = $440 A's share; # of $1100 = $660 B’s share; and lastly, as C. is to receive as much as A. and B. together, less $200, he will therefore receive $1100 — 200 = $900. 12. Divide 1254 acres among A., B. and C. giving C. 74 acres more than y B. and B. 12# acres more than A. OPERATION INDICATED. B & 4 & & { { { % & 7+ + 12# = ; = number acres C. had more than A. 1 # - & 4 & 4 32# = number acres over an equal division of the balance. . 1253 — 31# = 92% balance. 92.75 –– 3 = 30.91% acres, A’s share. 12.75 -- 30.913 = 43.66: “ B’s “ 20 -i- 30.91} = 50.913 “ C's “ 125.50 acres, total. 13. A. and B. have $9000. 5 times A's money is to 6 times B’s money as 5 to 9. How much has each. OPERATION INDICATED. 5 × A's share = # × 6 times B's share. 1 × A's share = # of B's share. 1 = B's share of the whole. } = A's share of the whole. 1 + 3 = 13 = sum of A. and B's shares. Then, ; : 1: ; $9000: $5400 B's share. : : #: ; $9000: $3600 A's share. 14. A merchant sold 384 bbls. flour, part at $7.25 and part at $5.50 per bbl. He received for the whole $42 more than if he had sold all at $63 a bbl. How many bbls. were sold at each price? OPERATION INDICATED. $7.25 = value 1st quality = - $7.25 5.50 = ** 2d & & - * 5.50 $1.75 = difference in price of 1st 2 | 12.75 quality over 26. $6,37} = average price if half of each quality were sold. 2 | 384 192 = half of amount sold. 1.75)42.00 24 blols. more of 1st quality sold than 192. 192 + 24 = 216 bbls. 1st quality. 192 – 24 == 168 “ 2d & 4 MISCELLANEOUS PROBLEMS. 953 15. A man engaged to work a year for $540, and a suit of clothes. He Worked 9 months and received $396 and the suit of clothes. What was the Suit OPERATION INDICATED. $540 + 1 suit. 45 -- Tº “ 9 months = $396 -- 1 “ 44 + “ $45 + "g, monthly average for twelve months. 44 + # 4 & & 4 { Worth 3 12 months = 1 month = 1 month = $45 – 44 = $1. * nine & 4 § – I's = #. Hence $1 = 3; value of the suit and # = $36 value of suit. 16. A man rides a certain distance at the rate of 6 miles an hour, and Walks back the same distance at the rate of 34 miles an hour. If it takes him 4; hours to go both ways, what is the distance between the two places? OPERATION INDICATED. 1 hour's ride = 6 miles. 1 hour. 7 || 2 6 or 33 : 6 : : 1 : 1% hours. 1} = hours to walk back. 1 = hour to ride there. 2} = hours to ride and walk. 19 || 7 4 6 miles. 9 or 2% : 6: : 43 : 10} 1 10# = number miles places are apart. 17. Two trains, one 210 feet long and the other 230 feet long, were going in the same direction on parallel tracks. Having started together with engines Opposite, one of them passed the other in 15 seconds. When going in opposite directions at the same rate of speed they passed each other in 33 seconds. How fast were they going 3 OPERATION When the shorter train is the faster. 210 feet = length of faster train. 230 feet = length of other train. 440 = number of feet heads of engines were apart at end of 3% seconds. - Seconds. Feet. Feet. 3# 440 | 440 15 QC 15 4 15 1760 = feet travelled by trains in 15 Sec. 1760 210 = feet faster train went over the other. 2 | 1550 775 = feet travelled by slower train in 15 210 seconds. 985 = feet travelled by faster train in 15 seconds. Mi. Mi. 775 985 15 || 60 15 60 5280 || 60 5280 || 60 443 mi, per hour faster train runs. 35; mi. per hour slower train runs. INDICATED. When the longer train is the faster. 1760 230 = feet faster train went over the other. 2 | 1530 765 = feet travelled by slower train in 15 230 seconds 995 = feet travelled by faster train in 15 seconds. Mi. 765 15 || 60 5280 | 60 34}% miles per hour slower train runs. Mi. 995 15 || 60 5280 || 60 45; miles per hour faster train runs. 954 SOULE's PHILOSOPHIC PRACTICAL MATHEMATICS. * * 18. A column of troops, 25 miles long is ordered to a point 25 miles distant. A courier starts simultaneously with the rear of the column, and reaches the head thereof. Returning, he meets the rear of the column at the point where the head originally Was. . Both the troops and the courier are to travel at a uniform rate of speed. How many miles does the courier travel? It is evident from the conditions of the problem that the courier had to travel the whole length of the column (25 miles) plus the distance which the head of the column moved while the courier was overtaking it (a miles). It is also evident that when the courier reached the head of the column, the distance that the rear of the column travelled must have been the extra distance the courier had to travel to overtake the head of the column, viz.: a miles. Then, since the courier; in returning arrives at the end of the column when it had reached the point where the head first lay he must have travelled a miles in returning from the head of the column. C K_ 25 NAILEs . > X Mill-Es K / X MILEs 'N N Z & D C A%– × Mil-Es X 25OMILEs X B rºz 25 N/IIL.E.S. X 25 N/IILES > A = rear of column. O = head of column. B = point head of column must reach. C = point courier reached when he overtook the head of the column. C’ = point where head of column is when overtaken by the courier. D = point rear of column reached when courier had reached the head of the column. From the above explanations and diagram, we obtain the following proportion which, when simplified, gives the desired result: 25-H ac : æ : : a 25 – 3: ac2 = 625 — ac” 2 ac2 = 625 ac2 = 312.5 a = 17.6776–1– miles. Since a = the extra distance the courier travelled to overtake the head, and as he had to i. the same distance to meet the rear, he must have travelled (2 × 17.6776–H) + 25 = 60.3552 19. If a man counts $1 every second and counts 12 hours every day, without stopping, how many days will it take him to count $1000000, and how many to count a billion dollars? Ans. 23# days to count a million. 23148# days = 63 years, 1374; days to count a billion. NotE.—365+ days were considered a year, in reducing the days to years. 20. How many years, of 365+ days each, will it require to count a trillion, by counting 12 hours every day, and counting $2 every second 3 Ans. 31688 years, 32% days. 2} MISCELLANEOUS PROBLEMS. * 955 º 21. § of A's money and # of B's money is $14, and A’s money is $4 more than B's money. How much has each 3 Ans. A. $40; B. $36. OPERATION. $13 w! of $4 = 80c. part of A's $4 excess in the $14; $14.00 – 80c. = $13.20; then } + k = }} = a $ 13.20 - 13.20 11 || 30 11 || 30 5 1 = A's part. 6 || 1 = B's part. $7.20 + .80 A's excess = $8 = } of A's money, $6 = # of B's money, which is, which is, therefore, $40. therefore, $36. 22. What is the interest on £50 12s. 6d. for 93 days at 6 per cent, allowing 365 days to the year 3 º Ans. 15s. 6d. NOTE.-See interest on English Money. 23. A benevolent man gave $5 to each of several beggars and had $15 remaining on hand. Had he given each $10, it would have taken all the money he had. How many beggars were there? Ans. 3. OPERATION, $10 — 5 = $5; $15 - 5 = 3. Ans. 24. A., B. and C. are to receive $26 in proportion to 4, §, #. What will each receive? Ans. A. $12; B. $8; C. $6. OPERATION. A. B. C. $26 $26 $26 13 | 12 13 | 12 13 | 12 # -- # -- + = }} 2 3 4 $12 $8 | $6 25. A. and B. bought at auction an invoice of merchandise for $800, of which sum A. paid $500 and B. $300. They then sold to C. § interest in the merchandise for $400. How much of the $400 must A. and B. receive respectively in order to constitute each, A., B., C. § owner of the goods? Ans. A. $350; B. $50. NOTE.-For the solution of this and similar questions, see pages 872 and 873. 26. A California miner has a spherical ball of gold 2 inches in diameter which he wishes to exchange for spherical balls 1 inch in diameter. How many of the smaller spheres should he receive? Ans. 8. NoTE.—For the solution of all similar questions, see pages 377 and 394. 27. A boy bought a certain number of peaches at the rate of 4 for 5 cents and paid for them with apples at the rate of 2 for 3 cents. How many peaches did he buy, providing it required 300 apples to pay for the peaches? Ans. 360. OPERATION. P. 300 300 5 || 4 2 || 3 2 | 3 | $4.50 Or, 5 || 4 - | $450 value of apples. 360 Ans. 360 956 SOULE's PHILOSOPHIC PRACTICAL MATHEMATICs. Yºr 28. Two little girls, Susie and Katie, were employed by a fruit dealer to peddle oranges; the first day they received 30 oranges each, Susie's being of superior Quality, which they sold as follows: Susie sold 2 for 5 cents, and received 75 cents. Katie { { 3 {{ 5 € $ { % 50 { % * Total receipts, $1.25 The next day Susie was sick, and Katie took the 60 oranges and sold them at the rate of 5 for 10 cents. At night, when she made her returns to the fruit dealer, she had but $1.20; and as 60 oranges brought $1.25 the day before, when sold 2 for 5 cents and 3 for 5 cents, or seemingly 5 for 10 cents, the fruit dealer charged little Katie with stealing the 5 cents. Did little Katie steal the 5 cents, and if not, how do you account for its loss? Ans. Little Katie did not steal it. It was lost because the ratio of 5 for 10 cents ceased when 10 sales had been effected. SOLUTION. As Susie sold 2 oranges at a sale, it is evident that to sell 30 she made 15 sales, which, at 5 cents a sale, amounted to 75 cents; and as Katie, the first day, sold 3 oranges at a sale, it is evident that to sell 30 she made 10 sales, which, at 5 cents a sale, amounted to 50 cents, making, as above, $1.25 for 60. The second day Katie mixed the 60 oranges, and by selling 5 at a sale made 12 sales, which, at 10 cents a sale, amounted to $1.20. By this we see that the 5 cents was lost by the manner of effecting sales; and hence, little Katie is innocent of the crime with which she is charged. The error in making the sales was in mixing and selling all of the oranges at the rate of 5 for 10 cents; for, as shown above, the inferior oranges, 3 for 5 cents, were all sold in 10 sales, and hence the ratio of 3 inferior and 2 superior making 5 for 10 cents, then ceased, and the remain- ing 10 oranges should have been sold 2 for 5 cents. By selling the 60 oranges at the rate of 5 for 10 cents, there must have been 2 sales which consisted wholly of superior oranges. 29. If 5% times 54 yards of cloth, which is 14 times 13 yards wide, cost 23 times 2% dollars, what will 103 times 103 yards cost that are 14 times 14 yards Wide # Ans. $25. 30. A planter hired a laborer for a term of 120 days, on condition that for every day he worked he should receive $1.00 and his board, and that for every day he did not work he should pay 50 cents for his board. At the expiration of the 120 days a settlement was made and he received $76.50. How many days did he work? Ans. 91. OPERATION. Had he worked the 120 days he would have received (at $1 per day) $120. But as he only received * = tº tº gº -*. tº- tº- &= se 76.50 He lost by reason of not working * as * * sº gº tº $43.50 And as he lost each day that he did not work ($1.00, amount of services, + 50 cents, amount paid for board) = $1.50, it is clear that he lost as many days as $43.50 are times equal to $1.50, which is 29; and if he lost 29 days, he therefore worked 120 – 29 = 91 days. 31. A. can do a piece of work in 6 days, and B. can do the same work in 8 days. How many days will it take for both to do the work? Ans. 3%. OPERATION. If A, can do the work in 6 days, in 1 day he can do of it; and if B. can do the work in 8 days, in 1 day he can do # of it; therefore # -H # = }ſ of the work is done in 1 day ; and if ºr of the work is done in 1 day, by transposition #4 of the work require one day, and if ºr of the work require 1 day, ºr will require # of 1 day, and ##, or the whole work, will require 24 times as many days, which is 3}. 4× MISCELLANEOUS PROBLEMs. 957 32. A. and B. can do a piece of work in 10 days; A. alone can do it in 15 days. How many days will it take B. to do it? Ans. 30 days. OPERATION. y’s – ºr = sºr of the work done in 1 day by B. 1 -- a's = 30 days. Ans. 33. A. and B. can do a piece of work in 14 days. A. can do # as much as B. How many days will it take each to do it, working alone? g Ans. 24; days for B. 323 days for A. OPERATION. (B.) # -- (A.) # = 4 work done by A. and B. in one day. Hence in 1 day B. does # of the work. & 4 & 4 And “ 1 “ A. “ # 14 – # = 244 days for B. 14 — 3 = 323 ** A. 34. A cistern has 3 pipes by which it may be filled, and 2 by which it may be emptied. The first of the three filling pipes can fill it in 12 hours, the second in 20 hours and the third in 2 days. The first of the emptying pipes can empty it in 24 hours and the second in 30 hours. If all of the pipes are left open, how many hours will it require to fill the cistern ? Ans. 1243 hours. OPERATION. * + 38 + # = % h. * + k, == +$o 19 ło #6 — T30 = % * 12+3 hours. Ans. 35. If it takes 5 boys 20 days to do a piece of work, and 6 girls 30 days to do the same work, how many days will it take a boy and a girl to do it? Ans. 64% days. OPERATION. d. 5 × 20 = 100. Tºo + Tºo = +&G 1 7 || 450 6 × 30 = 180. wº- =&m-º-º-º-º: 64% days. 36. A barrel of flour will last a man, wife and servant 50 days; but when the man is absent, it will last his wife and servant 90 days. How long would it last the man 3 Ans. 1124 days. OPERATION. |; * — ſo = x3; 2 225 112% 37. A merchant bought 5 bbls. flour for $40 and sold the same at a gain of 25%. He then bought 5 bbls. more for $40, and sold the same at a loss of 25%. Did he gain or lose, and if so, how much 3 Ans. He neither gained nor lost. 38. A merchant sold 5 bbls. of inferior flour for $40 and gained 25%. He then sold 5 bbls. of superior flour for $40 and lost 25%. Did he gain or lose in the two transactions, and if so how much 3 Ans. He lost $54. 958 soul E's PHILOSOPHIC PRACTICAL MATHEMATICS. º: A NOTE WITH COLLATERAL SECURITY AND A MARGIN. 39. A merchant discounted his note for $1200 with a banker and gave him insurance stock as collateral Security. The market value of the stock was $112. A margin of 20% was to be kept good on the stock. How many shares of stock did the merchant deliver and what instrument of writing did he execute and deliver to the banker besides the note? Ans. 1. 14 shares of stock. 2. A special power of attorney to sell the stock. FIRST OPERATION. SECONI) OPERATION. $100 $112 market value of shares. 80 | 1200 22.40 = 20% margin. $1500 = amount of collaterals. $ 89.60 net security per share. 1500 – 112 = 13% shares, practically 14 shares. $1200--89.60 = 13% shares, practically 14 shares. NoTE.—It is the custom, when giving stock collaterals to give a special power of attorney to sell the stock, so that in case the stock should decline and the margin should not be made good, a sale of the stock can be made to protect the payment of the note. 40. A. can do # of a piece of work in 10 days; B. can do # of it in 6 days, and C. can do # of it in 3 days. How many days will it take them all to do the Work # Ans. 34°1's days. OPERATION. § X To = T's the part of the work done by A. in 1 day. d. # X # = & 4 & 4 & 4 B. : : 1 * * | 1 # X # = ºr & 4 { { “ . C. “ 1 “ 113 360 #3 the part of the work done by A., B. and C. in 1 day. 3 ºf Ans. 41. Three persons, A., B. and C., do a piece of work; A. and B. together do 3 of it, and B. and C. do ºr of it. What part of the work is done by B. ? Ans. #. SOLUTION. As A. and B. do à of it, it is clear that C. does the remaining 3 ; and as B. and C. do ºr of it, it is clear that A. does the remaining 1°F. Then, as A. does ºr and C. §, they, together, do ºr + 3 = $3; and if A. and C. do 35 of a piece of work done by A., B. and C., it is clear that B. does the difference between $3 and ##, which is #. 42. A., B., C. and D. agree to do a piece of work for $5500. A., B. and C. can do it in 20 days; B., C. and D. in 24 days; C., D. and A. in 30 days; and D., A. and B. in 36 days. In how many days can all do it, working together; in how many days can each do it, working alone; and what part of the pay ought each to receive # Ans. For the solution see page 879. 43. A planter being asked how many cattle he had, replied that he had # of them in one pasture, 4 in another, and 25 in a third pasture. How many had he? Ans. 60. SOLUTION. * + 4 = "g the number in the first and second pastures. Hence 's from all of his cattle, # = #, the number in the third pasture; 25 is therefore ſº of his cattle; and if & are 25, ſº is the # past, which is 5, and 43, or the whole number, are 12 times 5, which is 60. - jºr dr, MISCELLANEOUS PROBLEMS. 959 44. If A, 4 and # of a man's money is $780, what amount has he? Ans. $800. SOLUTION. # -- + -i- # = }}. Then if #3 equal $780, #5 is the 39th part, which is $20, and #3, or the whole amount, is equal to 40 times ºf, or $20, which is $800. 45. A grocer bought a barrel of molasses, and after # of it had leaked away he drew out 6 gallons, when it was found to be # full. How many gallons did the barrel hold 3 Ans. 40 gals. SOLUTION, 3 — 4 = }; then # — # = #5, then ºr = 6 gallons, and 35 = # of 6 gallons, which is 2, and #3 = 20 times as much, which is 40. 46. A speculator after losing # of his money, and # of the remainder, ha& $1750. How many dollars had he at first. Ans. $7000. SOLUTION. # = his first loss. # X + = + = “ second loss. + = # “ total loss, which, deducted from his whole amount, leaves +, hence $1750 is + of his money, and the whole amount, 4 times as much, which is $7000. 47. A patron of the “wheel of fortune” (misfortune) in the lottery, lost by his first investment # of his money; by his second investment, # of the remainder, and by his third investment, à of the second remainder; he then donated $10 to aid in the destruction of such disreputable institutions, and had $75 left. What Sum had he at first 3 Ans. $680. SOI,UTION. Having lost by the 1st investment 4 of his money, he had # remaining; then having lost by the 2d investment, # of the remainder, he therefore lost # of # = 4; and 4 déducted from # = } as the second remainder; then having lost by his 3d investment # of the 2d remainder, he therefore lost # of # = #; and # deducted from # = # remaining on hand after his 3d loss. Hence, the $10 ºt tº on hand = $85, is # of his first sum, and consequently the whole amount was 8 timeS $85 = g - 48. A merchant deposited his money in 4 banks; in the first, he deposited #; in the second, #; in the third, #; and in the fourth, $2400 more than g of the whole. How much money had he? Ans. $72000. OPERATION. # -- # -- # = nºr = proportion deposited in three banks, hence the difference between ſº, and +3 = ſo is the proportion deposited in the fourth bank. And as the amount deposited in the fourth bank was $2400 more than 1% of the whole, the difference between Ho and ſº, is therefore the pro- portion that $2400 is to the whole sum. ſo — "... = 5%, hence $2400 is 3% of the whole sum, and #3 or the whole sum is 30 times $2400, which is $72000. 49. If 3 men and 5 women can do a piece of work in 14 days, 6 women and 8 boys can do the same work in 104 days, and 4 men and 7 boys can do it in 12; days. How long will it take 2 men, 10 boys and 3 women to do it? Ans. 944 days. NOTE.-For the solution of this problem, see page 331. 96o SOULE's PHILOSOPHIC PRACTICAL MATHEMATICS: * 50. A farmer sold 60 fowls, a part were turkeys and a part chickens; for the turkeys he received $1.10 a piece, and for the chickens, 50 cents a piece, and for the whole, he received $51.60; how many were they of each 3 Ans. 24 chickens. 36 turkeys. SOLUTION. Had all the fowls been turkeys and sold at $1.10 each, he would have received ($1.10 × 60), $66.00; but because some were chickens, and sold at 50 cents each, he only received $51.60, hence the difference, ($66–$51,60) = $14.40 is the deficit by reason of having sold some chickens, and as the turkeys were sold at $1.10, and the chickens at 50 cents, there is a deficit on each chicken of ($1.10 — 50c. = 60); therefore there were as many chickens as $14.40, are equal to 60c. which is ($14.40 - 60c.) = 24; and as there were 60 in all, it is clear that 60 — 24 = 36, is the number of turkeys. 51. A planter being asked how many head of mules he had, replied, that if he had as many more, one-half as many more, and seven mules and a half, he would then have 100. How many had he? Ans. 37. SOLUTION. According to the conditions of the question the 100 head is the result of as many more, one- half as many more, and 8even and one-half added to the true number; and hence it is evident, that by deducting 7% from 100 we shall have in the remainder the true number, plus as many more, and one-half as many more. 100 — 7# = 92%; now as 92% is the true number plus the true number, and one-half of the true number, it is evident that it is 2% or 3 times the true number, and hence 92+ — 23 = 37, the true number. 52. A father bought a certain number of oranges, which he divided among the members of his family, as follows: To his wife he gave # of all that he had, and # an orange more; to his daughter he gave # of the remainder, and # an orange more; to his son he gave # of the second remainder, and # an orange more; and to his servant, he gave 1, and had 1 left for himself; how many oranges did he buy, and how many did each receive? SOLUTION. (Problems of this character are most easily solved by working in the reverse direction or order from which the occurrences of the problem took place). It is evident, on inspection, that he had 2 oranges after having made the third division (with his son), and as he gave his son # an orange more than 3 of what he had before, making the third division, it is evident that he must have had 2 times 2 (the remainder) plus 1, which is 5; then as 5 was what he had before making the third division, it is what he had after making the second division; and as he gave in the second division 3, an orange more than ; of what he had before making the second division, it is also evident that he had 2 times 5, plus 1, which is 11. Then as 11 was what he had left before making the second division, it is what he had after making the first division; and as he gave in the first division 3 an orange more than % of what he had before making the first division, it is likewise evident that he had 2 times 11, plus 1, which is 23. Then 23 - 2 = 11} + 4 = 12, the wife's part. 23 — 12 = 11; 11 — 2 = 5% + 3 = 6, the daughter's part. 11 – 6 = 5; 5 - 2 = 2+ + 4 = 3, the son's part. 5 – 3 = 2; 2 — 1 = 1, the servant's part. 2 — 1 = 1 remaining, the father's part. 53. A. and B. had the same income; A. saved of his, but B. by spending $50 more per annum than A. found himself at the end of ten years $300 in debt; how much was the income of each. Ans. $160. SOLUTION. B. having spent $300 in 10 years, at $50 per year, it is plain that he spent ſº of it in ºne ear; ºn of $360 is $30. Now as B. spent $50 more per year than A., the difference between what }. spent per year, and $50 is what A. saved. This difference is 50 – 30 = $20. \ - 60 Therefore $20 is 4 of their yearly income, and # or the whole income is 8 times $20, which is $160, x. MISCELLANEOUS PROBLEMS. 961 54. There is a fish whose head is 9 inches long, his tail is as long as his head and half of his body, whilst the body is as long as his head and tail both; what is the length of the fish? Ans. 72 inches. SOLUTION. Since the tail is as long as the head and # the body, it is equal to 9 inches + 4 of the body, but as the body = the head and tail, it (the body) = 9 inches + 9 inches + 4 of the body. That is the body = 18 inches + # the body. Hence # the body is 18 inches, and # or the whole body 36 inches; and as the body is ; the length of the fish, 3 or the whole length is 2 times 36 inches, which is 72 inches. 55. A hare is 40 leaps before a hound and takes 4 leaps while the hound takes 3, but 2 of the hound's leaps are equal to 3 of the hare's; how many leaps must the hound take to catch the hare ? Ans. 240. SOLUTION. Since 2 leaps of the hound = 3 leaps of the hare, 1 leap of the hound = 1% leaps of the hare; then, 3 leaps of the hound = 4% of the hare; hence, in making 3 leaps the hound gains # leap of the hare, and in making 1 leap, (; – 3) # ; then 6 leaps of the hound gain 1 leap of the hare, and 6 × 40 = 240, number of leaps. 56. A general formed his men into a square, that is, an equal number in rank and file, and found that he had 59 men over; and increasing the number in both rank and file by 1 man, he wanted 84 men to complete the square; how many men had he? Ans. 5100. SOLUTION. To increase the number in both rank and file by 1 man, it is evident, that it would require twice the number of men in rank or file at first, plus 1 (for the man occupying the corner). And as it require to effect this, 59 -- 84.5- 143 men; then (143 – 1) – 2 = 71, the number of men in rank and file at first. Therefore, 71* + 59 = 5100 men in the army. 57. Three footmen, A., B. and C. start together and travel in the same direc- tion around an island 72 miles in circumference; A. travels 6 miles a day, B. 8, and C. 12. In what time will they all be together again? Ans. 36 days. SOLUTION. 72 – 6 = 12 days required for A. to travel around the island. 72 - 8 - 9 { % é & B. & 4 { { & 4 72 —- 12 - 6 { { & 4 C. { { & 4 & 4 Then the least common multiple of 12, 9 and 6 is the number of days required. 3 ) 12 9 6 2 ) 4 3 2 2 3 1 3 × 2 × 2 × 3 = 36 days, Ans. or, thus: B. gains on A, 2 miles a day, and hence will pass A. every 73 days, = 36 days. C. gains on A. 6 miles a day, and hence will pass A. every # days = 12 days...Therefore, the least common multiple of 35 and 12 will give the number of days that B. and C. will first pass A, together. 12 ) 36 12 3 1 12 × 3 = 36 days, Ans. 962 SouLE's PHILOSOPHIC PRACTICAL MATHEMATICs. * 58. A lake is 56 miles in circumference. A. traveling 30 miles per day, B. 24, C. 16, D. 12 and E. 10, all commence to travel at the same time and in the same direction around the lake. In how many days will they all be together again? Ans. 28 days. SOLUTION. A. gains on E. 20 miles per day. 2 ) {} }} hºº º B. ** E. 14 “ é & C. 44 E. 6 “ ( { 3 ) 21 30 70 210 D. & 4 E. 2 & 4 & 4 56 - 20 = 23 = 2+} = {} 56 - 14 = 4 = 4 = }} 56 - 6 = 9+ = 9 * = *** 56 - 2 = 28 = 28 = 4% * 7 ) 7 10 70 70 10 ) 1 10 10 10 1 1 1 1 2 × 3 × 7 × 10 = ** = 28 days, Ans. 59. How many gallons in a tank of the following description and dimen- sions: The upper portion is in the form of a vertical rectangular prism, 13 ft. 5 inches deep, 11 feet 9 inches long, and 9 feet 3 inches wide. The bottom is a segment of a horizontal cylinder, the chord being 9 feet 3 inches, and the height or versed sine 4 feet Ans. 13372.404 gallons. NOTE.—For the solution of this problem, see page 399. 60. A live stock dealer sold two horses, each at the same price. On one horse he gained 25 per cent, and on the other he lost 25 per cent. His net loss on the two sales was $12. What was the cost of each horse? Ans. $72 cost of 1st horse, $120 cost of 2d horse. NOTE.-For the solution of this problem, see page 477. 61. A man, at his death, left an estate amounting to $35000, to his wife and two children, a son and a daughter. His children being absentin Europe, he directed by will, that if his son returned his wife should have of the estate, and the son the remainder; but if the daughter returned, his wife should have 3 and the daughter the remainder. Now it so happened that they both returned. What sum should each receive? Ans. $20000 son, $10000 wife. $5000 daughter. NOTE.-For the solution, see page 876. 62. A man had 2 sons and 3 daughters; to the youngest son he gave 4 of 14 times $3600, which was # of the elder son's share, and what the elder son received was # of g of the whole estate; the balance was divided among his 3 daughters in reciprocal proportion to their ages, 8, 10, 12 years; what was each daughter's share 7 Ans. $7114.87 for the one 8 years old. $5691.89 “ (: 10 44 $4743.24 “ “ 12 46 Nore.—For the solution, see page 874, º ºf MISCELLANEOUS PROBLEMS. 963 63. A., B. and C. were to receive $6000, in proportion to 3, 4 and #; but B. having died, it is required to divide the money between A. and C. What sum should each receive # Ans. A. $4000. C. $2000. NOTE.-For the solution, see page 875. 64. A. and B. paid $600 ($300 each) for 300 acres of land. In dividing the land according to quality, it was agreed that A. should have the best quality, and pay therefor 75 cents per acre more than B. How many acres did each receive, and what price per acre did each pay ? Ans. A. received 122.8 acres and paid $2.443 per acre. B. { % 177.2 4 { % 1,693 (4 NOTE. – For the solution, see page 878. 65. A merchant receives from France, an invoice valued at port of shipment at 4550.25 francs. In the invoice are 100 dozen pairs of kid gloves that cost 18.50 francs per dozen. Freight and insurance on the whole invoice to New Orleans, amounted to $87.82. Duty was paid on the gloves at 33% per cent. For what price per pair must the merchant mark these gloves so that he may discount 10 per cent from his asking price and yet gain 163 per cent on full importing cost 3 Ans. 55-H 2. OPERATION INDICATED. % of charges on invoice. $ import cost 1 dozen gloves. 87.82 s 4550.25 | 18.50 .193 || 100 *º- $3.57 10% practically. 1.19 = duty at 33}. .36 = proportion of charges at 10%. $ selling price per pair. - 5.12 $5.12 full N. O. cost per dozen. 6 || 7 9 : 10 12 |ºre 66. August 30, 1895, No. 7, coffee was quoted in Rio at 138850, exchange 103d. What is the equivalent value per pound in New Orleans, the rate of English Exchange being $4.88, no allowance being made for time, freight or insurance? Ans. 9.24+ 2. FIRST OPERATION. SECOND OPERATION. 13$850 13$850 × 10#d = 147.15625d. 32.38 10; 240 || 4.88 147.15625 - 32.38 = 4.545 – d per pound. 9.24-H c. Ans. (4.545 – d x $4.88) — 240 = 9.24-H c. Ans. Explanation.—The rate, 138850, is 13 milries and 850 reis per arroba, which is equal to 32.38 pounds avoirdupois. The 10#d, exchange is the English gold value of a milreis of silver of Brazil including exchange. The $4.88 is N. O. gold rate of exchange on England. 67. A. and B. together agree to dig 100 rods of ditch for $100. The part of the ditch on which A. was employed was more difficult of excavation than the part on which B. was employed; and it was therefore agreed that A. Should receive for each rod 25 cents more than B. received for each rod that he dug. How many rods must each dig and at what price so that each may receive just $50? Ans. B. receives $ .8903 -H per rod, and digs 56.16 rotis, A. receives $1.1403 per rod, and digs 43.84 rods. NOTE.—For the solution, see page 878. 964 soul E's PHILOSOPHIC PRACTICAL MATHEMATICS. * 68. A person being asked the hour of the day, said, the time past noon is # of the time from now to midnight. What is the hour? Ans. 3 hrs. 25 m. 42% s. OPERATION. 12 hrs. = 1 + # = % of time to midnight. If 12 = }, + = } X 12 and # = } X 12 = 24 hrs. *# = 3% hrs. = 3 hrs. 25 m. 42% s. Ans. Ea:planation.—Since the time past noon is # of the time to midnight, we may consider the time to midnight as a whole, or #, and the two periods of time taken together = } + # = }. Ther. as the time from noon to midnight = 12 hrs., we have 12 hrs. = % of the time to midnight; and if # = 12 hrs., + = } x 12, and # = } x 12 = ** = 3 hrs. 25 m. 42% s. Ans. 69. What time after 5 o'clock will the hour and minute hands of a watch be together ? Ans. 27 m. 16++ S. OPERATION. Tºr x 60 = 5 ºr m. = number minute spaces passed by hour hand at first crossing. 5 × 5*r = 27 ºr m. = number minute spaces after 5 crossings, i. e., between 5 and 6 o'clock. 27 ºr m. = 27 m. 1614, s. Ea:planation.—Since the hands are together at 12 M., the beginning of the hour circle, and cross each other 11 times during the passage of the hour hand (with a uniform motion) once around the circle of 60 minute spaces, we see that the first crossing of the hour hand will require h of the 60 spaces, or 5*, spaces, and at the fifth crossing, occurring between 5 and 6 o'clock, the hour hand will have passed over 5 times 5 #1 m. = 27.4 m. = 27 m. 16#, s. Ans. 70. On the dial of a town clock the end of the minute hand requires 4 minutes and 42 seconds to pass over a space of 6 inches. What is its length ? Ans. 12.19 + inches. OPERATION. 42 s. = ## = ſo of a minute. Ea:planation.—We first find what part of the 41% dial circle is 4 m. and 42 s. The circle being = fl O = #7 - divided into 60 spaces (i. e., minutes) we find it 41% m. F5 – 500 of a circle. to be gºſo of the whole. This fraction represents 6 inches of length of perimeter. If 6 ſo = 6 in., 47 600 gºry = 4% of 6 and {}}}, or whole circle, - 600 sing 6 times A, of 6 or 90%; g. Having thus found the circumference, we find the radius, i. e., the | 12.19056 in. Ans. minute hand, by dividing by 3.1416 and taking # the quotient. 71. A lady being asked how old she was at the time of her marriage, replied, that the age of her oldest son was 13 years; that he was born 2 years after her marriage; that when she married, the age of her husband was three times her own, and that now her husband was twice as old as herself. How old was she and her husband when married? Ans. 15 and 45. SOLUTION. Since the son, aged 13, was born 2 years after the marriage, she is 15 years older than when married. Her husband's age then was to her’s as 3:1; with 15 years added to both it was as 2 : 1, or 4: 2, i. e., she is now twice as old as when married. But as the addition of 15 produced this s {- g wº tº- tº- tº * . s $800 “Having paid the interest in advance for 100 months, or eight years and four months, of course he has no interest to pay in monthly installments, as by the Gross Plan; and there being no premium to pay or deduct, there remains but the $8 per month dues to pay on the eight shares he has borrowed upon. He therefore pays no more per month as a borrower than he would pay as a non-borrower. If the shares should mature in eight years, the borrower having paid four months' interest in advance of the time of winding up, the four months’ excess of interest would be returned to him with his satisfied mortgage. On the other hand, if the shares should exceed eight years and four months in running out, and reach as far as, say nine years, the borrower would, on and after the 101st month, have to pay monthly interest in addition for eight months longer. “If one borrows on an old series, he is allowed a deduction or rebate from his interest for each month of the past age of the series. “If he desires to repay a loan, he is allowed a return of interest, paid in advance, for each unexpired month. “The foregoing is an outline of the plan of working. “The apparent gain of the association by this method results from the greater amount of profit deducted at once on making a loan as compared with the Gross Plan. “For instance: On a loan, under the new plan, on eight shares at 100 months' interest in advance, $800 would be retained as a profit, the borrower receiving the net sum of $800; while under the Gross Plan of deducting premiums on a loan of five shares at 20 per cent premium, but 982 soulE's PHILOSOPHIC PRACTICAL MATHEMATICs. ºr $200 will be retained as profit, while the borrower receives the same net amount of money ($800) as the borrower under the new plan receives. “It will thus be seen that the association apparently gains the advantage of $800 immediate. profit under the new plan, against $200 under the Gross Plan, while the borrower receives the same amount of money in each case; and, under the Interest in Advance Plan, pays but $8 a month, against $10 a month under the Gross Plan.” Other advantages are also claimed for this plan. STATE AND UNITED STATES LEAGUES. 1545. A Building and Loan Association League is a chartered organization or confederation of different associations, for the promotion of joint interests, com- mon purposes, and mutual Support. - 1546. State Leagues are those formed by associations domiciled within the respective States. 1547. A United States League is one formed by associations domiciled in the various States. These leagues are of great benefit to the cause in which they serve. Through them, all matters relating to the cause are discussed by the ablest, minds engaged in the service, and the interest of each is made the good of all. NATIONAL BUILDING ASSOCIATIONS. 1548. The name “Nationals” is applied to a class of Building and Loan: Associations that is patterned, organized, and conducted somewhat according to the general methods of the original or parent Building and Loan Associations as: above described. They are a growth (branches or children, as it were) of the parent, institutions, and have individualized themselves by incorporating some new features, the practical advantages of which are seriously questioned by the original Associations. The new leading features of the Nationals are the establishment of agencies in different cities and States and the loaning of money to whomsoever may wish to borrow on their terms. By reason of these features, they are denomin- ated “Nationals” in contradistinction to the parent associations which loan money only in their respective localities, and as a rule only to their respective members, and are hence denominated “Locals.” The “Nationals” are of quite recent birth, and in pressing their claims for patronage have, in Some cases, introduced non- ethical methods in their management, and used decoying statements through their agents and in their circular and advertising literature. In many cases, they masquerade and do business on $20,000,000 or $50,000,000. authorized capital, when in truth they have not as many thousand in net assets. To open offices throughout the country, they incur expenses for salary of agents, rent of offices, commission of solicitors, and advertising that consume much of, and sometimes, all of the profits and all the dues, and then dishonorable bankruptcy follows. The leading special features of the Nationals make some of them of greater service to their army of officials than to their subscribers. These officials. * BUILDING AND LOAN ASSOCIATIONS. 983 of the “Nationals” are often protected by a special expense fund, which is raised by a direct tax on the members. It is believed by those who have had the largest experience with Co-operative Building Associations, that the business of loaning money is conducted on too small a margin of profit to admit of the various expenses necessary to carry on Building Associations on the expensive agency plan of the Nationals. Great prudence should be exercised by parties who wish to receive the benefits of a well and ethically conducted Building Association. “All is not gold that glitters.” And experience proves that some locals as well as many Nationals have proved unfaithful. PLANS OF LOANING. MONEY OR PREMIUM PLANS. 1549. The six thousand Building and Loan Associations in the United States adopt different plans with various combinations, of loaning their money. The Commissioner of Labor, Washington, D.C., in his Ninth Annual Report on Building and Loan Associations, 1894, gives 68 different plans covering 38 pages. It is not the function of this work to present these plans. Those who are interested in the subject, we refer to the report of the Commissioner of Labor, to be found at almost every Building and Loan Association. PLANS OF WITH DRAWAL. 1550. Different Associations adopt different plans for the withdrawal of members. This subject is also fully treated by the Commissioner of Labor, in his Ninth Annual Report, 1894, wherein he presents 12 plans, with over 100 variations. We refer those interested in the subject to said report. PLANS @F DISTRIBUTION OF PROFIT. 1551. To adjust the profits and to find the value of each series and of each share at the close of each gain declaring period, is one of the most important and complex operations which arises in the accounting Work of Building and Loan Associations. As in the case of premium plans and plans of withdrawing, there are various plans for the distribution of profits. Of these numerous plans, the Com- missioner of Labor, in his Ninth Annual Report on Building and Loan Associations, 1894, presents 25 plans with numerous variations, covering 35 pages. We give space and credit for the 1st, 8th and 9th plans presented in said report: PLAN 1-(Partnership Ptan). “This plan apportions the profits among series just as profits among partners are apportioned in a firm where the partners enter at different dates, each series representing a partner, 984 SOULE's PHILOSOPHIC PRACTICAL MATHEMATICs. * 1. Multiply the dues paid in on the shares in force in each series by the equated time of investment. 2. Take the sum of these products and then find what fractional part each product is of the SUllrl. 3. These fractions are the parts of the total net profits belonging to each series. To illustrate the rule, let us suppose that an association whose monthly dues are $1 per share had three series in force at the end of the third year, and that the number of shares in each series and their value per share were as follows: º First series, 500 shares, value per share $38.87; second series, 600 shares, value per share $25.27; third series, 400 shares, value per share $12.32; that a fourth series of 500 shares is then issued; the net profits for the fourth year are $3,000, and the total net profits for the four years are $5,325. Required: The value of a share of each series at the end of the fourth year. The first series above alluded to has run four years, or forty-eight months. Forty-eight $1 payments have therefore been made on each share of stock. The first dollar paid has been invested forty-eight months; the second dollar paid, forty-seven months; the third dollar paid, forty-six months, etc., the last dollar of the forty-eight having been invested one month. The times of investment thus form a decreasing arithmetical series, with forty-eight for the first term, one for #he last term, and forty-eight for the number of terms. The total investment is thus equal to $1 invested for 1,176 months (the sum of the series), equivalent to $48 invested for 24; months. Treating the other series in the same way, we find that $36 paid per share in the second series has been invested for 184 months; $24 paid per share in the third series, for 12# months; and $12 paid per share in the fourth series, for 64 months; then— $48 × 500 × 24% = $588,000, first series’ investment for one month. $36 × 600 × 18} = $399,600, second series’ investment for one month. $24 × 400 × 12# = $120,000, third series’ investment for one month. $12 × 500 × 6+ = $39,000, fourth series’ investment for one month. $1,146,600, total investment for one month. Bence the total net profits are divided as follows: ******, or ºr of the total profits belong to the first series. *% or ºr of the total profits belong to the second series. ******, or ºr of the total profits belong to the third series. T####30 or TÉr of the total profits belong to the fourth series. Total profits to be divided are $5,325. *r of $5,325 = $2,730.77, first series' share of the profits, Mººr of $5,325 = $1,855.81, second series' share of the profits. *ºr of $5,325 = $557.30, third series' share of the profits. T#r of $5,325 = $181.12, fourth series' share of the profits. $2,730.77 – 500 = $5.46, profit of a share of the first series. $1,855.81 – 600 = $3.09, profit of a share of the second series. $557.30 – 400 = $1.39, profit of a share of the third series. $181.12 – 500 = $0.36, profit of a share of the fourth series. $48.00, dues paid, + $5.46, profit, = $53.46, value of a share of the first series. $36.00, dues paid, + $3.09, profit, = $39.09, value of a share of the second series. $24.00, dues paid, +- $1.39, profit, - $25.39, value of a share of the third series. $12.00, dues paid, +- $0.36, profit, - $12.36, value of a share of the fourth series. In the above example, all the net profits made during the four years have been apportioned to the several series, but some associations apportion only each year's profits in this way. Other associations using this plan simplify the process, but obtain the same results, by dividing the total Investment for one month (or for one week, as the case may be), into the profits to be apportioned, for the profit on $1 invested for one month, which is then multiplied successively by the sum of the number of weeks, months, or other periods of time for which each dollar of dues in each series has been invested. The products will be the amount of the profits belonging to a share in each series. * BUILDING AND LOAN ASSOCIATIONS. 985 M A few associations have been found that arrive at the same results by using the following method, which is known as Clark's plan: wº 1. Multiply the number of shares in force in each series by the quotient obtained by dividing the sum of the number of weeks, months, or other periods of time for which each dollar of dues in each series has been invested by the product obtained by multiplying the dues paid in on one share during the first year by the average time of investment, for the equalized results for each series. 2. Take the sum of these results and divide it into the total profits since the beginning of the associations, for the rate per cent of profit. 3. Multiply the quotients already found by the rate per cent of profit, for the profit of a share in each series. We have before seen that $48 dues per share in the first series have been invested for 244 months, which is equal to $1 invested for 1,176 months. In like manner $36 dues per share in the second series have been invested for 184 months, which is equal to $1 invested for 666 months; $24 dues per share in the third series, for 124 months, which is equal to $1 invested for 300 months; and $12 dues per share in the fourth series for 64 months, which is equal to $1 invested for 78 months. The average time of first year's payments is simply half the time of investment, which is 6 months. Twelve dollars invested for an average period of 6 months is equal to $1 invested for 72 months. Then : & 1,176 – 72 = 16.333. 666 – 72 = 9.250. *. 300 – 72 = 4.166. 78 – 72 = 1,083. * 500 x 16.333 = 8,166.50, equalized result for the first series. 600 × 9.250 = 5,550.00, equalized result for the second series. 400 × 4.166 = 1,666.40, equalized result for the third series. 500 × 1.083 = 541.50, equalized result for the fourth series. *E**-ºº-ºº. 15,924.40, equalized result for all series. $5,325, the total profits, – 15,924.40 = 33.4392, the rate per cent of profit. $0.334392 × 16.333 = $5.46, profit of a share of the first series. $0.334392 × 9.250 = $3.09, profit of a share of the second series. $0.334392 × 4.166 = $1.39, profit of a share of the third series. $0.334392 × 1.083 = $0.36, profit of a share of the fourth series. For the value of each share, add the dues as above. There is a modification of plan 1, which follows the same general method as that shown in the first illustration, but differs in certain particulars and gives a different result. The modifica- tion is as follows: Instead of finding the exact equated time of investment, many associations arrive at an approximate equated time by taking one-half the number of months a series has run. Using the same data as in the above illustrations, we get 24, 18, 12, and 6 as the average number of months the series have run. It is this modification that is commonly, but erroneously, called the partnership plan. ILLUSTRATION. $48 × 500 × 24 = $576,000, first series’ investment for one month. $36 × 600 x 18 = $388,800, second series’ investment for one month. $24 × 400 × 12 = $115,200, third series’ investment for one month. $12 × 500 × 6 = $36,000, fourth series’ investment for one month. $1,116,000, total investment for one month. 986 soul E's PHILOSOPHIC PRACTICAL MATHEMATICS. * The total net profits are then divided in proportion to each series' investment for one month, thus: #7% or #9, of $5,325 = $2,748.39, first series' share of the profits. #% or ºr of $5,325 = $1,855.16, second series' share of the profits. tºº, or ºr of $5,325 = $549.68, third series' share of the profits. Tºg}} or rāg of $5,325 = $171.77, fourth series' share of the profits. $2,748.39 - 500 = $5.50, profit of a share of the first series. $1,855.16 -i- 600 = $3.09, profit of a share of the second series. $549.68 - 400 = $1.37, profit of a share of the third series. $171.77 - 500 = $0.34, profit of a share of the fourth series. This modification of plan 1 has been simplified, the principle consisting in casting out come mon factors in the process of multiplication. The first series has run 48 months; the second, 36 months; the third, 24 months; and the fourth, 12 months. The average time of investment, as we have before seen, is 24, 18, 12, and 6. respectively. Then we proceed thus: 48, age in months, x 24, average time, × 500 shares. 36, age in months, x 18, average time, × 600 shares. 24, age in months, x 12, average time, X 400 shares. 12, age in months, x 6, average time, × 500 shares. It will be readily seen that 12 is a factor common to all the numbers of the first column, and that 6 is a factor common to all the numbers in the second column. Casting out these factors, We have— t 4 × 4 × 500 = 8,000. Hence #% of the total profits belong to the first series. 3 × 3 × 600 = 5,400. Yºr of the total profits belong to the second series. 2 × 2 × 400 = 1,600. Tºr of the total profits belong to the third series. 1 × 1 × 500 = 500. Tºg of the total profits belong to the fourth Series. Total, 15,500. Briefly put, then, the simplification is as follows: Multiply the number of shares in force in each series by the square"of the time of investment expressed in terms or periods corresponding to the intervals between the series, and then divide the profits in proportion to these products. The foregoing simplification has been still further simplified by finding the profit of a share in each series directly, instead of finding each series' share of the profit, as follows: 1. Multiply the number of shares in force in each series by the square of the time of invest- ment expressed in terms or periods corresponding to the intervals between the series. 2. Divide the sum of these products into the results obtained by multiplying the total net, profit by the square of the time of investment expressed as above. The total of the products, as in the last illustration, is 15,500; then— $5,325, total profits, × 4 × 4 + 15,500 = $5.50, profit of a share of the first series. $5,325, total profits, x 3 × 3 – 15,500 = $3.09, profit of a share of the second series. $5,325, total profits, x 2 × 2 - 15,500 = $1.37, profit of a share of the third series. $5,325, total profits, × 1 × 1 — 15,500 = $0.34, profit of a share of the fourth series. A share of the first series receives 16 times as much profit as a share of the fourth series; a share of the second series, 9 times as much; and a share of the third series, 4 times as much. This method, therefore, reveals the fact that, by multiplying the number of shares in force in each series by the square of the time each series has been invested, expressed in years, half years, quarter years, etc., corresponding to the intervals between the series, a correct basis of calculation is reached. These simplifications, however, are practicable only where series are issued at regular intervais, as fractions complicate the operation. This simplification is known as Rice's rule. BUILDING AND LOAN ASSOCIATIONS. 987: A few associations arrive at the same results by dividing the total investment for one month into the profits, for a rate per cent of profit, and then applying the rate to each series' investment for one month for each series' share of the profits. The process is also varied in the following manner: Find what annual rate of interest the profits are equivalent to on the amount of dues paid for one-half the time that all the dues have been invested, and apply this rate on the dues paid per share for one-half the time of investment, for the profit of a share in any series. Other variations are the following: 1. The profits are distributed on the amount of dues actually paid in on the shares in force in each series (not what the regular payments should have amounted to), multiplied by one-half the time of investment. 2. The profits are distributed on the total amount of dues standing to the credit of the shareholders in the loan fund multiplied by one-half the time of investment. 3. The series are not allowed to participate in the profits for the term in which they were issued. 4. The profits are distributed on the amount of dues actually paid in on all shares in force that are three months old or over, multiplied by one-half the time of investment, shares less than three months old not participating. 5. The profits are distributed to the free shares only, dues on shares borrowed on being credited on loans. 6. Profits arising from withdrawals are divided equally among the shares of the respective series from which the shares were withdrawn. 7. Profits arising from entrance fees are divided equally among the shares of the respective Series in which the shares are taken. 8. A profit of $1 is given to all shares six months old or over. The remainder of the profits is distributed on the dues paid in on the shares in force six months old or over multiplied by one- half the time of investment. 9. Profits arising from premiums are divided equally among all the shares in force at the end of the period during which the loans were made. Profits from all other sources are distributed in accordance with the modified rule. - 10. A fixed rate of interest is given on the total amount of dues paid on the shares in force at each apportionment. This interest is deducted from the profits for the term and the remainder distributed according to the modified rule. 11. A fixed rate of interest is given on the value of the shares in force as declared by the last report. This interest is deducted from the profits for the term and the remainder is distributed according to the modified rule. 12. A portion of the total amount of premiums received by and due the association is arbitrarily determined upon, and held in reserve to be applied in future dividends; the amount thus determined upon is deducted from the total profits, and the remainder of the profits is dis- tributed as follows: The interest and dividends allowed on free shares withdrawn are added to the dues paid in on such shares, and the sum total of said interest and dividends is deducted from the amount of distributable profits; the balance is distributed among all the shares in accordance with the foregoing modified rule. * There is still another modification of plan 1, as follows: Multiply each series' investment (that is, the dues paid in on the shares in force) by one-half the number of months invested plus one and apportion the profits in proportion to these products. Using the same data as before we proceed as follows: ILLUSTRATION. $48 × 500 × 25 = $600,000, first series' investment for one month. $36 × 600 × 19 = $410,400, second series' investment for one month. $24 × 400 × 13 = $124,800, third series' investment for one month. $12 × 500 × 7 = $42,000, fourth series’ investment for one month. $1,177,200, total investment for one month. 988 SouLE's PHILOSOPHIC PRACTICAL MATHEMATICs. & Then, proceeding as before, we find that— #9% or §§ of $5,325 = $2,714.07, the first series' share of the profits. Yº, or # of $5,325 = $1,856.42, the second series' share of the profits. ***ś or ### of $5,325 = $564.53, the third series' share of the profits. T#####5 or ºr of $5,325 = $189,98, the fourth series' share of the profits. $48, dues, + ($2,714.07 – 500) = $53.43, value of a share of the first series. $36, dues, + ($1,856.42 – 600) = $39.09, value of a share of the second series. $24, dues, + ( $564.53 - 400) = $25.41, value of a share of the third series. $12, dues, + ( $189,98 - 500) = $12.38, value of a share of the fourth series.” NoTE. 1.-1280 local associations and 81 Nationals throughout 45 States adopt this plan with some of its variations. NOTE: 2. —1 association in Louisiana adopts this method with variations. & PLAN 8.—(Brooks' Plan). 1. Find the legal rate of interest on the values of the shares as declared by the last report and deduct it from the profits of the term for the net profit. * 2. Divide the net profit by the sum of the dues paid in during the term and the interest on the previous series, for the rate per cent of profit. 3. Multiply the sum of the interest and dues for the term on one share of each series by the rate per cent of profit, for the profit on one share. 4. Add the previous value of a share of each series, the legal interest on this value, the dues paid in during the term, and the profit on the share to find the present value of a share of each series. Let the legal rate of interest be 6 per cent, and using the same data as in the preceding illustrations, we proceed thus: ILLUSTRATION. $38.87 × .06 = $2.33, interest on a share of the first series. $25.27 × .06 = $1.52, interest on a share of the second series. $12.32 × .06 = $0.74, interest on a share of the third series. $2.33 × 500 = $1,165, interest on first series. $1.52 × 600 = $912, interest on second series. $0.74 × 400 = $296, interest on third series. $2,373, total interest on old series. $3,000, the profits for the term, - $2,373, - $627, the net profits. $12 × 2,000, the number of shares in force, = $24,000, the dues paid during the term. $24,000 + $ 2,373 = $26,373, the active capital. $627 –– $26,373 = 2.3774, the rate per cent of profit. ($12 + $2.33) × .023774 = $0.34, the profit of a share of the first series ($12 + $1.52) × .023774 = $0.32, the profit of a share of the second series. ($12 + $0.74) × .023774 = $0.30, the profit of a share of the third series. $12 × .023774 = $0.29, the profit of a share of the fourth series. $38.87 -- $2.33 + $12 + $0.34 = $53.54, value of a share of the first series. $25.27 -i- $1.52 -- $12 + $0.32 = $39.11, value of a share of the second series. $12.32 + $0.74 + $12 + $0.30 = $25.36, value of a share of the third series. $12 + $0.29 = $12.29, value of a share of the fourth series. Some associations using this rule vary it by allowing less than the legal rate of interest on the old values, but no separate classification of such associations has been made. Another variation from the rule consists in allowing the legal rate of interest, not only on the values of the old series at the beginning of the term, but also on the equated amount of dues paid in on all the shares during the term. Yºr BUILDING AND LOAN ASSOCIATIONs. 989 Another variation from the rule is as follows: After the legal rate of interest for the term on the values of the old series has been deducted from the profits for the term. the remainder of the profit is divided by the sum of the value of all shares at the beginning of the term, the dues paid in during the term, and the interest for the rate per cent of profit. This rate is then applied to the sum of the previous value of each share, the dues paid in during the term, and the interest on the share for the profit on one share. NOTE.-25 associations adopt this plan. PLAN 9. The profits are divided equally among all the shares in force. This plan is used principally by terminating associations. The following variations of this rule have been found: 1. The profits for each term are divided equally among all the shares in force. 2. The profits for each term are divided equally among all shares three months old or over, six months old or over, or nine months old or over, as the rules of the association may provide. In such cases shares less than three months, six months, or nine months old do not participate in the profits. 3. Shares less than a year old are given such portion of the profits as the board of directors may allow. The remainder of the profits is divided equally among all the shares one year old or OWOI’. 4. The profits for each term are divided equally among the free shares. 5. Shares one year old or over receive equal amounts of the profits for the term. Shares nine months old receive three-fourths as much as those a year old; shares six months old one-half as much, etc. 6. The profits made by each series are kept separate, and are divided equally among all the shares of the series. 7. Profits arising from premiums and fines are divided equally among the shares of the series in which such profits were made. All other profits are divided equally among all the shares. NOTE: 1.—1062 locals and 2 Nationals adopt this plan with variations. NOTE 2.—2 associations in Louisiana adopt this method with variations. PROBLEM BY THE GROSS PLAN. 1552. To elucidate, in detail, the manner of adjusting profits and finding the value of the different series and the separate shares by the Gross plan, we present the following problem and work the same in full, according to the method of Edmund Wrigley. PROBLEM, FIRST TERM. An association which issues yearly series and loans on the Gross plan, Coms mencing business with an issue of 500 shares of $200 each, and collecting dues 25 cents per week, would, during the first fiscal year, receive dues amounting to $6500. Bresuming that the dues had been loaned at 25 per cent premium, that the gains by interest etc., were, at the close of the fiscal year, $400, and the expenses 990 soul E's PHILOSOPHIC PRACTICAL MATHEMATICS. * . $300, what would be the net gain for the term, the value of the series, and the value per share ? Ans. See operation. *. < OPERATION. 500 shares at 25 cents for 52 weeks = $13 per share, = $6500 dues. $6500 loaned at 25 per cent premium = * * * m = * - $1625 prem. Gains by interest , etc. tº gº º ºs sº s ſº tº ſº * 400 Total gain, tº a gº sº tº º * , sº s & gº - $2025 Less expenses, say, - - º gº {- se tº-8 &= 300 Net gain for the term, sº gº sº tº º ºs - - - 1725 $8225 Total value of first series, * * sº gº *- tº- $º wº $8225 – 500 shares = $16.45 value per share. SECOND TERMI. The association issued at the beginning of the second year or term, a second series of, say, 800 shares. The following statement shows the business of the second term or year: Passive capital $8225. Dues on first series for second year $6500. Dues on second series for first year, 52 weeks at 25 cents on 800 shares, = $10400. Total dues $16900. Interest on $8225 passive capital or value of first series at 6 per cent for second term; $493.50. Premium on $6500 dues of first series at 25 per cent $1625. Premium on $10400 dues on second series at, say, 30 per cent, $3120. Total premium $4745. ſº Average interest on active capital, $22138.50, at, say 6 per cent for 3 term or average time, $664.15. Total gain, $5902.65, expenses, say $402.65. Net gain for second term, $5500. What is the value per share of each series and what is the value of each series? OPERATION. Dues for Second Term. e ** Dues for second term on first series, 500 shares, 52 weeks at 25 cents = s tº § wº $6500 Dues for first term on second series, 800 shares, 52 weeks at 25 cents = - - - - 10400 Total dues for both series, second term, sº 3- tºº tº tº es sº º $16900 Gains for Second Term. Interest on $8225, passive capital, at 6 per cent for second term, - tºº $493.50 Premium on $6500, at 25 per cent for second term, - - - - - 1625.00 Premium on $10400, at 30 per per cent for first term, tº E tº gº 3.120.00 Average interest on active capital, $22138.50 at 6 per cent for 3 term, y or average time, sº tº gº tº ſº- * - sº e- s º 644.15 Total gain, - - - - - - - - - - - - - $5882.65 Less expenses, *- s sº s sº sº dº sº gº sº tº . sº- * 402.65 - - - - $5480.00. Net gain for second term, - º sº - - sº- g X- BUILDING AND LOAN ASSOCIATIONS. 99 I PRESENT VALUE OF ONE SHARE OF FIRST SERIES AND TOTAL VALUE OF FIRST SERIES. Value of one share, first series, at beginning of term, $16.45. Interest on same at 6 per cent for the term is 98.7 cents. .987 × 500 shares first series = $493.50, as shown above. $5480 met gain — $493.50 = $4986.50 gain to be divided among the 1300 shares of both series. $4986.50 – 1300 = $3.83576-H per share. Dues on one share of first series for two years, $26 Gain on one share, first term, ($1725 – 500) - 3.45 Gain by interest, second term, - .987 4.82276-H Gain by interest, first term, - 3.83576–H $ — Total gain in two terms or years, - $34.27276-H value of each share. $34.27276 × 500 shares = $17136.38 value of first series. PRESENT VALUE OF ONE SHARE OF SECOND SERIES AND TOTAL VALUE OF SECOND SERIES. Dues on one share, second series, for one term or year, * - $13. Gain per share for second term, as above, - º - -> - 3.83576–H Value of each share of second series, - - sº - wº- - $16,83576–H $16.83576 × 800 shares = $13468.61 value of second series. $17136.38 + $13468.61 = $30604.99 total value of both series, 1300 shares, at the close of second term or year. NOTE: 1.-257 associations adopt this plan with variations. The variations are shown on page 437 of the report of the Commissioner of Labor, 1894, and pages 83 to 100 of Wrigley’s How to Manage Building Associations. NOTE 2.—8 associations in Louisiana adopt this method with variations. METHODS AIDOPTED BY SOME OF TELE NEW ORLEANS BTUILDING º AND LOAN ASSOCIATIONS. 1553. In response to inquiries made of several Building and Loan Associations in New Orleans, regarding their method of determining and adjusting profits, we received replies from four of the most popular, prosperous and ably managed associations in the city. They are as follows: OFFICE OF MUTUAL BUILDING AND HOMESTEAD ASSOCIATION, No. 337 (Old No. 87) ST. CHARLES STREET, MASONIC TEMPLE. NEW ORLEANs, Aug. 2, 1895. GEO. SOULF, Esq., City. IXEAR SIR: Replying to your favor of the 29th ult, I am pleased to furnish you with the information requested by you, and hope it may prove serviceable. Our profits are apportioned upon the partnership plan, and the dividend is a percentage based 992 SOULE's PHILOSOPHIC PRACTICAL MATHEMATICS. * upon the amount each shareholder has invested for the time it is invested. While we issue shares in what we nominally term series, said shares do not mature at the one time, because the amounts deposited thereon may exceed the minimum installment, and hence the accumulated capital invested, and consequent profits vary and cause shares to mature at uneven dates. The rule we follow may be formulated thus: 1. Find the net profits to be apportioned. 2. Find the cash value at the commencement of the period just ended, of all shares then ir force, which are still in force. 3. Find the total of installments paid in on all shares within the period Just ended. 4. Add to item (2), the amount of item (3), reduced to its equivalent capital for an invest- ment covering the whole period. e 5. Divide item (1), by item (4), and the result will be the per cent of profit. 6. At the rate found apportion each share (or batch of shares under one account) with its individual profit, following in each case the method described above. EXAMPLE. On 10 shares, the dues for 52 weeks at $2.50 equals, - - - - - - - - $130.00 These being paid monthly, it is the equivalent of Sixty-five (65) Dollars invested for one year, and if profit is six (6) per cent, the dividend equals, gº tº gº tº * = tº 3.90 Cash value of shares at second year, - {º tº sº. sº tº se *- & wº - $133.90 Add dues for year, - gº gºs tº tº sº tº $º º ſº gº e- & tº- º ſº 130.00 The capital then invested is, - - - tº gº º ºs - - - $133.90 Plus average of $130, tºº tº- sº s dºe * * tº- - * sº 65.00 Equivalent capital for year, º gº tº Eº tº tº - - - $198.90 On which a six (6) per cent dividend would be, - sº tº 4-3 sº tº * > tº º º 11.93 Cash value of shares at third year, tº gº *s gº * º sº * tº gº gº $275.83 Add dues for year, - tº- tº * gº $º wº e- tº- * Ǻs * = sº gº 4-º 130.00 Following same method, $275.83, plus $65, equals $340.83 at six (6) per cent, - tº-e tº 20.44 Cash value at fourth year, º *-i- dº * = fº tº gº * * &= } tº tº iºs $426.27 And so on. Very respectfully, ESPY W. H. WILLIAMS, Secretary. P. S.—This company also issues shares on the Dayton plan. These shares are $100 face value, payable in installments of not less than 25 cents per week, but as much more as desired by holder. The shares are issued at any time, and begin to participate in profits at once. Full paid shares are issued on payments of $100 each, and dividends are paid thereon in cash every six months. Loans are also made without any premium or bonus, and without bidding, simply upon application filed. E. W. H. W., Sec'y. —cºe— OFFICE OF UNION HOMESTEAD ASSOCIATION, NEW 336 CARON DELET STREET. NEW ORLEANs, August 7, 1895. Col. GEO. SOULE, City. * DEAR SIR:—In response to your circular letter of July 29th, I note with interest, that you intend giving representation in your forthcoming work on “Philosophic Practical Mathematics,” to the problems of Building and Loan, or Homestead Associations. * * * †. º $ As to our own method of distributing profits to members, I reply as follows: The member stands to us in the relation of a series, of which you speak. Our system con- * BUILDING AND LOAN ASSOCIATIONS. 993 templates the calculation of dividend on every pass-book, independently of every other. No two need be exactly the same. We do not issue stock in series, except nominally; we call all the issue of any one month the series of that month, and give it a number; but the books or shares in that series are each independent of every other. For example: In series 38, issued last January, books 521, 529, and 536, each call for 10 shares, on which the regular deposit would be $5.00 per month. But on June 30th, the standing of the three books was, respectively, $75, $30 and $70. We do not declare a dividend of so much per share, but a percentage dividend, this time 5 per cent for the six months. This enables us to give to each of these books its fair (right) portion of earnings. Where the deposits are made regularly, month by month, we take half the amount as the average investment; so on book 529, on which 6 payments of $5 were made, total $30, we have an average investment of $15 at 5 per cent dividend, 75 cents. On book 521, there was a deposit of $50 in February, on which we allow interest from the time it was put in, and give that book $2.68 dividend. This method of averaging is not mathematically exact, but is close enough for all practical purposes, especially in view of the fact that the surplus or undivided profit, is held and belongs to the stockholders, and may be subsequently divided among them, when ordered by the directors. In case of a book in force prior to January, for example, No. 412, issued July 1894, it showed a value, Jan. 1, of $76,85, (6 × 12.50 plus dividend) and during the six months six payments of $12.50 were made. We add to the capital, at the beginning of the term, half the deposits of the term, $37.50, and (dropping fractional parts of $1) have an invested capital for the term of $114, at 5 per cent, gives dividend $5.70, and cash value of the book, July 1, 1895, $157.55. In case but 5 monthly payments had been made during the term, I would allow but 5/6ths of the regular dividend. This method is simple, and can be made as accurate as desired. Its beauty lies in its flexibility, adapting itself to the needs of every book, or every share, or every series, if applied to the series capital. It can be made as exact as any method in use, if one wishes to determine to a minute fraction the rate of earnings or dividend. It is applicable to any so-called “permanent” associations, the specific name adopted by most associations issuing stock continuously, and treating each pass-book independently. Trusting, that from the foregoing, you will understand clearly our method of division of profits, and holding myself in readiness to answer promptly any questions you wish to ask in further connection with this subject, I am, Very truly, JNO. E. HUFFMAN, Secretary. —sºe— THE SECURITY BUILDING AND LOAN ASSOCIATION, No. 5 Connrnercial Place, Roorn 1. NEW ORLEANs, August 12, 1895. GEORGE SOULF, Esq. City. DEAR SIR: Your favor, 29th, was duly received, and allow me to apologize for not answering sooner. Our association, the Security Building and Loan Association does not issue stock in series. The shares are $100 each, and are issued at any time to suit the subscriber. Our method of divid- ing the profits is much more simple than that of serial associations, as we conduct our business on the cash basis, or Dayton plan. The following extract from Article 4, of our Charter will help to explain: “On January 1, and July 1, of each year, the Board of Directors shall ascertain the earnings 994 SOULE's PHILOSOPHIC PRACTICAL MATHEMATICs. * of the association for the preceding six months, and out of such earnings they shall (1) deduct all the expenses of the association for such time; (2) set aside to the fund for contingent losses such an amount as they may deem best, until such contingent fund equals 5 per cent of the entire out- standing mortgage loans, which fund shall be invested as directed by the board; (3) the balance of said earnings shall be declared as a dividend to the shareholders, in proportion to the value of stock of each member, at the time the preceding dividend was declared, and all payments made between such dividend periods shall begin to earn dividends from the beginning of the next semi- annual term.” “The dividends declared on ‘running stock’ shall be credited on the pass books of the members semi-annually, and dividends on “paid up stock’ shall be paid in cash on the fourth Monday of January and July of each year.” The Security has declared its third semi-annual dividend of 4 per cent. I beg to enclose copy of our charter, also our last semi-annual statement. The serial system is not being adopted by the new associations; it is complicated and not easy for the average stockholder to understand. Any further information that I can furnish will be cheerfully given. Very truly, W. E. DODSWORTH, Secretary. —eſpe— OFFICE OF THIRD DISTRICT BUILDING ASSOCIATION, CORNER ROYAL STREET AND LAFAYETTE AVENUE. NEw ORLEANs, August 13, 1895. PROF. GEO. SouLé, City. DEAR SIR: Enclosed please to find example of the method of distribution of profits as adopted by this association. Trusting same will be of use to you, I remain, Yours truly, JOS. L. SPORL, Secretary. EXAMPLE, Entitled to full Series No. Shares. Paid in on dividend for Pro rata on A. 708 $9204.00 12 months or 52 weeks, one year, - * * * $9204.00 B 109 981.00 9 4 & ** 39 “ # of year, tº a º 735.50 C 327 2125.50 6 “ “ 26 “ 4 “ gº tºº 1062.75 D 841 2943.50 3 & 4 “ 13 “ + “ * 735.87 Total paid in $15254.00 $11738.12 The net profits for distribution are $2969. Divide net profits $2969 × 100 by amounts prorated, $11738, gives 25.29-H 96, multiply $9204 amount paid in by series A by 25.29%, gives $2327.69 gain for the whole number of shares of series A. $2327.69 - 708 shares of series A = $3.28 dividend per share of series A. Then to find the dividend per share for each of the other series, we proceed thus: series B $ 735.50 prorated value x 25.29% = $186.01 – 109 No. shares = $1.71 dividend per share. 48 C 1062.75 “ “ × 25.29% = 268.77 – 327 “ “ = .82+ “ & 6 48 D 735.87 “ “ × 25.29% = 186.10 - 841 “ “ = .22+ “ é & NoTE.—The payments on stock in this company are 25 cents per week, or $13 per year, of 52 weeks. Hence the different series are credited as follows: º: BUILDING AND LOAN ASSOCIATIONS. 995 series A is credited with the full $13.00 on each share as paid in. * B “ 4 & “ only 9.00 * * * * ‘‘ ‘‘ “ instead of $9.75 because some are tº C « {{ ** the full 6.50 “ (: & & ( & & 4 [delinquent. “ D tº & 4 & 4 3.50 < * * * & ( & & “ instead of $3.25 because some are [paid in advance. According to the foregoing work, the value of the stock or series at the end of the year would be as follows: Series A, amount paid in $9204.00+ dividend $2327.69 = <--> - & - - º $11531.69 “ B, 4 & “ “ 981.00-- * { 186.01 = º - gº º - Gº 1167.01 “ C, { { “ “ 2125.50+ { { 268.77 – º - sº - - º 2394,27 “ D, { { “ “ 2943.50+ ( & 186.10 = * - = - - tºs 3129.60 Total value, tº º sº; - * * --> © - - - - - $18222.57 Balance undivided by reason of the non-exactness of the gain per cent, tº .43 $18223.00 MISCELLANEOUS PRACTICAL PROBLEMS. To find the rate per cent of interest received by a non-borrowing member on maturing shares. 1554. What rate of interest is gained on shares of building associations, the maturity value of which is $200, the dues being $1 per month, supposing the series to mature in 84 years? Ans. 22.39 — ‘ſ. oPERATION, 84 years = 102 months at $1 = $102 dues paid. $200 maturity value of share — $102 = $98 gain on the monthly investment of $1. 102, first term + 1, last term = 103. 103 × 51, half the number of terms = 5253 months or (5253 – 12) 437; years, equated time of interest on $1; or conversely, interest on $437# for 1 year. Then since $437% gain $98 in 1 year, the rate per cent is 437; ; 98 : : 100 : 22.39 – 96, Ans. Or thus, to find the rate per cent: $ 98 437; 100 22.39 – 9%, Ans. SECOND OPERATION TO FIND RATE PER CENT. Interest on $437# for 1 year at 1 per cent is $4.37%. Then as $4,374 : $98 : : 1 per cent : 22.39 – 96 Ans. Explanation.—Since $1 dues were paid each month for 102 months, the first dollar paid bears interest in favor of the investor, for 102 months; the second payment is at interest 101 months, etc., to the last payment, which is at interest for 1 month. Hence, the interest on the different payments is equivalent to the interest on $1 for a number of months equal to the sum of an arith- metical series, whose first term is 102, last term 1, and number of terms 102. This number is, as shown in the operation, 5253 months, or 437# years. Then since the interest on $1 for 437+ years is the same as the interest on $437# for 1 year, we may use either statement in computing the interest or in finding the rate per cent. 996 SOULE's PHILOSOPHIC PRACTICAL MATHEMATICs. * To find the rate of interest paid by a borrower, on the Net Plan. 1555. A member bought a loan in a building association conducted on the net plan, and paid $60 premium on exchange of 8 shares, new issue. What will be his rate per cent of interest if the series expires in 84 years, and 6 per cent interest is charged on the loan 3 Ans. 6.25+ 7%. OPERATION. - 8 shares at $200, = - * - $1600 par of shares. 8 shares at 60 premium = - - 480 premium. Amount of loan, - * - $1120 Interest on $1120 for 1 month is enºw, = $5.60 Dues, 1 month on 8 shares at $1, = º - 8.00 Monthly payment is - - - - - $13.60 $13.60 × 100 months period of loan = $1360 total payment. Then the interest on the $13.60 monthly payments for the different periods, is equivalent to the interest on $1 for a number of months equal to the sum of an arithmetical series, whose first term is 100, last term 1, and number of terms 100, which is, 100 + 1 = 101 × 50, half the number of terms, 5050 months. Interest on $13.60 for 5050 months is $343.40 + $1360 = $1703.40, aggregate payments. $1703.40 — $1120 = $583.40 interest on the loan for 84 years. $583.40 - 8+ = $70.008 interest for 1 year. Then what per cent of $1120 is $70.008? It is ($70.008 × 100) -- $1120 = .0625-H or 6.25-H% Ans. NotE.—By the net plan the premium is deducted from the par value, and interest is charged on the net amount of the loan. To find the gain per cent realized by a Building and Loan Association, on its invest- ments of dues, at simple interest. 1556. If the dues of a Building and Loan Association are $1 per month, and the shares mature in 83 years, what would be the annual average gain per cent of the association, provided it invested monthly the dues as received, at 6 per cent simple interest? - Ans. 6.06 — ?). * OPERATION. 84 years = 102 months at $1 = $102 dues received in 83 years. The interest on the $1 monthly dues, or receipts for the 102 months, is equivalent to the interest on $1 for the equated time, which is (102 + 1 = 103 × 51) 5253 months or 437# years. Interest on $1 for 437+ years, at 6 per cent, is $26.26}. This $26.264 interest is the gain on $1 monthly dues for 84 years. Then $26.264 – 84 years = $3.09 gain per year on $102 dues, or average investment of ($102 - 2) $51. g $3.09 – $51 = $.0606 — or 6.06 – 96 average gain per annum. Or thus, to find the per cent as per page 599: Interest on $51 for 1 year, at 1 per cent, is 51 cents. X} BUILDING AND LOAN ASSOCIATIONS. 997 Then since 1 per cent for the time gives 51 cents interest, conversely 51 cents required 1 per cent; and since 51 cents interest required 1 per cent, 1 cent will require the 51st part and 309 cents will require 309 times as much, which is 6.06 – 96. Or thus, 51 : 100 :: 3.09: 6.06 – 96. PROOF. Interest on $51 for 84 years at 6.06 – 96 = $26.27 —. To find the per cent realized by a Building and Loan Association on its investments of dues, at simple interest invested weekly. 1557. If the dues of an association are 50 cents a week, and the shares mature in 43 years, what would be the annual average gain per cent to the association, if it invested weekly the dues received at 6 per cent, simple interest? Ans. 6.03 — 7%. OPERATION. 44 years × 52 weeks = 234 weeks at 50 cents = $117 dues received in 44 years. The interest on the 50 cents weekly dues for the 234 weeks is equivalent to the interest on the 50 cents for the equated time, which is (234 + 1 = 235 × 117) 27495 weeks, or 52833 years of 52 weeks each. The interest on 50 cents for 27495 weeks, at 6 per cent, is $15.864. This $15.864 interest is the gain on 50 cents monthly dues for 43 years. Then 15.8625 – 4} = $3.525 gain per year on $117 dues or average investment of (117 –– 2) $58.50. Then $3.525 -- $58% = .0603 — or 6.03 – 96 average gain per annum of 52 weeks. Or thus, 584 : 100 : ; 3.525 : 6.03 – 96. NOTE.—In the above operation 52 weeks are used as a year. To find the average annual gain per cent realized on weekly payments, simple interest being allowed on the weekly payments for each year, and the annual amounts to be put at compound interest to the close of the period. 1558. Using the figures of the preceding problem, what would be the com. pound interest, and the average annual gain per cent, allowing 6 per cent simple interest on the weekly payments for each year of 52 weeks, and the annual amounts to be put at compound interest at 6 per cent, to the close of the period or the maturity of the shares, 44 years of 52 weeks each? -*. OPERATION. The dues being 50 cents per week, the sum received and loaned is $26 for the first year, at 6 per cent. The interest on the 50 cents weekly payment of dues for 52 weeks, of the first year, is equiva- lent to the interest on 50 cents for the equated time, which is (52 + 1 = 53 × 26) 1378 weeks. The interest on 50 cents for 1378 weeks, at 6 per cent, is 79.5 cents. Then 26 + 79.5 cents = $26,795 yearly value of the 50 cents weekly payments. This $26,795 is a yearly recurring amount to be placed at compound interest; thus forming a series of annuities, each term of which is $26,795. 998 soul E's PHILOSOPHIC PRACTICAL MATHEMATICs. º: Then to find the sum of these annuities, we have the following formula: S = A X Rºl — A = S = $26,795 × 1.06% – $26,795 $26,795 × 1.2624770 – $26,795 R – 1 -p .06 - .06 *- $117.21 = the sum of an annuity of $1 at compound interest for 4 years at 6 per cent. NOTE.—The 1.2624770 is the fourth power of 1.06, and is obtained from the Compound Interest Table, page 612. To this $117.21 we must add: (1). The interest thereon for 6 months, at 6 per cent, which is $3.51. (2). The $13 dues for 26 weeks. (3). The interest on the $13 dues for the equated time, which is 20 cents. Thus making a total sum of $133,93. See the close of the first solution for the full details of finding gain and gain per cent. SECOND SOLUTION BY COMPOUND ANNUITY TABLE. $4.374616 26,795 $117.217835.720 3.51 13.00 .20 $133,927–1– To find the total cost of a loan to a borrower at 6 per cent simple interest, Gross plan, monthly payments, $1 per share. 1559. A member of a building association bought a loan on 10 shares in a new series, at $15 and the stated premium, with 6 per cent interest. What will be the full cost of his loan if the series matures in 110 months? Ans. $2810.50. OPERATION. NOTE.-By the Gross and Installment Plans, the dues and the interest at 6 per cent on each share of $200, are each $1 per month. $1 dues + $1 interest = $2 monthly payment per share. $2 × 10 shares = $20 monthly installments to be made and which earn interest as follows: The first installment will earn interest for 110 months, the second installment, 109 months, and so on, until the payment of the last installment which earns no interest. Hence the interest on the 110 installment payments for the different periods of time is equivalent to the interest on $20, one installment for the equated time which is (110 – 1 = 111 × 55) 6105 months. The interest on $20 for 6105 months, at 6 per cent, is $610.50. $20 each payment x 110 payments = $2200, sum of payments. $2200 + $610.50 = $2810.50 total cost of loan to the borrower. NOTE.-This method is known as the 6 per cent method. To find the actual premium charged on loans and refunded on their payment. 1560. A member bought a loan on 8 shares of stock 1 year and 4 months old, and paid $80 premium. What is the actual premium if the value of a share was $18 at the last annual report 3 Ans. $71.20. º: BUILDING AND LOAN ASSOCIATIONS. - 999 OPERATION. $18, value of 1 share at last report + $4 dues for four months = $22 present accumulated value of each share. $200 unaccumulated value — $22 accumulated value = $178 unaccumulated value. Hence ### of the par is unaccumulated; therefore #$$ of the $80 premium bid is the actual premium paid by the borrower. §§ of $80 = $71.20, actual premium. NOTE 1.--To find the met loan, deduct the actual premium. Nors 2.—To find the total cost of a loan, multiply monthly payments by the number of months. To find the average annual gain per cent realized on monthly payments, simple interest being allowed on the monthly payments for each year, and the annual amounts to be put at compound interest to the close of the period. 1561. 1. If the dues of a Building and Loan Association are $1 per month, and the shares mature in 84 years, what would be the compound interest and the average annual gain per cent allowing 6 per cent simple interest on the monthly payments for each year, and the annual amounts to be put at compound interest at 6 per cent to the close of the period or the maturity of the shares? Ans. Compound interest $30,42; 7.016+ %. REMARKS.—The solution of this problem forms the basis for solving many problems of interest to Building'and Loan Associations, and to all parties concerned in loaning or borrowing money. We therefore give three solutions of the problem in order to fully elucidate the principles involved, and to make clear the methods of Solution. FIRST SOLUTION, 1st year. 12 months at $1 = $12 dues received 1st year. One dollar the first receipt was at interest for 12 months, the second receipt of $1 was at interest for 11 months, etc. Hence the interest on the receipts for dues for the 12 different periods, is equivalent to the interest on $1 for a number of months represented by the sum of an arithmetical series, whose first term is 12, last term 1, and number of terms 12. Thus 12 + 1 = 13 × 6, half the number of terms, = 78 months equated time of interest on $1 at 6 per cent gives, * , - - $ .39 Dues received and loaned the 1st year, * * & gº * - wº * * * fº ę 12.00 Amount to loan the 2d year, - - - " - - - - - - - $12.39 £d year. Interest on $12.39 for 12 months, at 6 per cent, * * * * * * = .74 Dues 2d year and interest for 78 months, equated time, on $1 as in the first year, - 12.39 Amount to loan the 3d year, - gº gº * > tº i- * º º tº tº - $25.52 3d year. Interest on $25.52 for 12 months, at 6 per cent = - tº sº º tº º ºs 1.53 Dues the 3d year and interest, 78 months, on $1 as in the first year, - e- ge tºº 12.39 *Amount to loan the 4th year, º $º tº- gº * : sº tº- tº † = tº º - $39,44 4th year. Interest on $39.44, 12 months, at 6 per cent = * tºº º * * = tº 2.37 Dues and interest, 4th year, as in the first year, - tº º tº tº gº sº sº E- 12.39 Amount to loan, 5th year, sº tº - - - - - - - - - - $54.20 Bth year. Interest on $54.20, 12 months, at 6 per cent = tº ſº * * * * * 3.25 Dues and interest, 5th year, as in first year, * > tº a ſº tº e tº * , : º tº 12.39 Amount to loan, 6th year, gº tºº & Cº º º • * * - - - $69.84 IOOO SOULE's PHILOSOPHIC PRACTICAL MATHEMATICS. 3. 6th year. Interest on $69.84, 12 months, at 6 per cent, e - se º - tº - $ 4.19 Dues and interest, 6th year, as in first year, - * - - tº - tº - 12.39 Amount to loan, 7th year, º - - tº - -e - - º - º ºg $86.42 7th year. Interest on $86.42, 12 months, at 6 per cent, sº - º - - * - 5.19 Dues and interest, 7th year, as in first year, - eº - tº- tº * sº {- 12.39 Amount to loan, 8th year, º - º - - º - º - - &= - $104.00 8th year. Interest on $104.00, 6 months, at 6 per cent, - º - º tº - º - 6.24 Dues and interest, 8th year, as in first year, - sº -* - º - tº - 12.39 Amount to loan, one-half of 9th year, sº - º - - º - eme - $122.63 84th year. Interest on $122.63, 6 months, at 6 per cent, º - tº - 3.68 Dues and interest, for 6 months are equivalent to the interest on $1 for the equated time, which is (6 -- 1 = 7 × 3) 21 months. Interest on $1 for 21 months, at 6 per cent, is $.11 + $6 dues = - tº- - * -* t- º tº- 6.11 Compound amount of $102 for 84 years, at 6 per cent, - - - -> - " - $132,42 I)ues deducted, º - º - - - - --> - º º - º - 102.00 Interest received in 84 years, - - sºme - * 4- º tº- º & - $30,42 $30.42 + 84 = $3.5782–H gain per year on an average investment of (102 − 2) $51. 51 : 100 : : 3.5782–H : 7.016+ 9% average gain. SECOND SOLUTION. By the Compound Interest Table, page 611. The payments being $1 per month, the sum received and loaned is $12 for the first year. The interest on the $1 monthly dues or receipts for the 12 months of the first year is equivalent to the interest on $1 for the equated time, which is, as fully shown in the first solution, (12 + 1) = 13 × 6 = 78 months. The interest on $1 for 78 months at 6 per cent, is 39 cents. $12 + 39c. = $12,39 the yearly value of the monthly payments. This $12.39 is a yearly recurring amount to be placed at compound interest; thus forming a series of annuities, each term of which is $12.39. Then to find the sum of these annuities, we have the following formula: S = A × Rn — A Or S = $12.39 × 1.06% – $12.39 $12.39 × 1.5938481 – $12.39 - R – 1 *s .06 - .06 *- $122.62962852 the sum of an annuity of $1 at compound interest for 8 years, at 6 per cent. A NotE.—The 1.5938481 is the 8th power of 1.06, and is obtained from the Compound Interest Table, page 612. To this $122.63 we must add: (1). The interest thereon for 6 months at 6 per cent, which is $3.68. (2). The $6 dues for 6 months. (3). The interest on the $6 dues for the equated time, which is, as shown in the first solution, 11 cents. Thus making a total sum of $132.42. See the close of the first solution for the full details of finding gain and gain per cent. NoTE.—See the remarks on Annuities, on page 921. THIRD SOLUTION. By the Annuity Table, page 925. We first produce the annuity of $12.39 as shown in the first and second solutions, then to find the sum of the annuities at the end of the given time, we have but to multiply the amount of the annuity of $1 per annum, at compound interest for 8 years, by the annual annuity $12.39. jºr BUILDING AND LOAN ASSOCIATIONS. IOOI OPERATION. $9.897468 = sum of an annuity of $1 per annum at compound interest for 8 years at 6 per 12.39 [cent. See Table, page 925. $122.62962852 = the sum of an annual annuity of $12.39 for 8 years, at 6 per cent compound interest. To this sum $122.63, we add interest and dues for 6 months, as shown at the close of the first and second solutions. NOTE 1.-See the close of the first solution, for the full details of finding gain and gain per cent. NOTE 2.-See the remarks on Annuities, page 921. FOURTH SOLUTION By Logarithms. All problems of this kind can be worked by the Algebraic formula for finding the amount of Sinking Funds, or the amount of Annuities. The formula is A = sº-0. Applying this formula to the above problem, and using figures instead of letters, we have amount = $12.39 gº — 1) To raise $1.06 to the 8th power by the use of Logarithm Tables proceed as follows: (1). Find the log of the number 1.06 = .025306. (2). Multiply this log by 8 = .202448. (3). Look for this log in the Table of Logarithms. If you cannot find it exactly, find the log just less than it (.202216) and the one just greater than it (.2024.88). The difference between the two extremes is 272, and the difference between the given log (.202448) and the lesser log (.202216) is 232. Write these differences into a fraction as, #}}. (4). Find the number corresponding to log .202216 = 1.59300; find the number corresponding to log .2024.88 = 1.59400. (5). Take the ### part of 100 = 85+ and add it to 1.59300, writing it in the place of the naughts = 1.59385. Multiply difference between 1.59385 and 1 = .59385 by 12.39 = 7.3578015. (6). Divide this product by .06 = $122.63 = amount for 8 years. Now having the compound amount for 8 years, we proceed for the remaining 6 months as shown in the first and second solutions. If the monthly dues in the above problem were to be compounded monthly, the following would be the formula for finding the amount at the end of the 102nd month : _ S(Rn--1–1) $1 (1.005”—1) _S (Rn–1) $1 (1.005%–1) A = ** – 4–S = .005 — $1, or A = * º ,005 = $132.63, to this amount add interest for one month = 66% = $133.29. OPERATION FOR FIRST FORMUL.A. USING LOGARITHMS TO FACILITATE MULTIPLICATIONS. Log 1.005 = .002166 × 103 = log .223098. The number corresponding to this log is $1.67147. From this subtract $1 = $.67.147. Divide this amount by $.005 = $134.29. Less $1 = 133.29, final value. OPERATION BY SECOND FORMUL.A. USING LOGARITHMS TO FACILITATE MULTIPLICATION. Log 1.005 = .002166 × 102 = log .220932. The number corresponding to this log is $1.66315. Subtract $1 = $.66315. Divide this by $.005 = $132.63. Add + per cent on $132.63 = 66c. =$133.29. final value. IOO2 SOULE's PHILOSOPHIC PRACTICAL MATHEMATICs. * 2. A Land and Investment Company sells to B. a piece of property for $4000, at 8 per cent interest, on condition that B. shall pay for the property at the rate of $90 per month, which monthly payments are to include the interest on the successive monthly balances. How long will it take B. to pay for the property, the first payment being made one month after the sale? How many payments did B. make 3 What was the total interest? What was the average rate per cent, per annum on the $4000? What did the company gain and B. lose in excess of the 8 per cent interest charged on the successive monthly balances? Answers. (1). It requires 53 months to pay for the property. (2). B. made 53 payments, 52 of $90 each and 1 of $79.66. (3). B. paid $4759.66. (4). Total interest was $759.66. (5). The average rate per annum on the $4000 was 4.3 per cent. (6). The company gained and B. lost in excess of the 8 per cent on the successive monthly balances which bore interest, the interest (simple, annual or compound, according to manner of making investments), on each $90 monthly payment, except the last payment of $79.66, from the time each payment was made until the expiration of 53 months. PARTIAL OPERATION By the United States Partial Payment Method. OPERATION OPERATION For the first two payments. For the last two payments. Amt. at 896 int. for the 1st month, - $4000.00 | Bal. at 896 int. for the 52d month, - $168.01 Int. on same for the 1st month, - º 26.67 || Int. on the same for the 52d month, - 1.12 Amount due at close of 1st month, - $4026.67 || Amt. due at the close of 52d month, - $169.13 1st payment for the 1st month, - gº 90.00 52d payment, e * * º tº gº 90.00 Bal. at 896 int. for the 2d month, - $3936.67 | Bal. at 89% int. for the 53d month, - $79.13 Int. on same for the 2d month, - e 26.24 Int, on same, for the 53d month, - s .53 Amount due, at close of 2d month, - $3962.91 Amt, due at the close of 53d month, - $79.66 2d payment, - sº tº º sº º º 90.00 53d payment, - * * tº e iº tº º 79.66 Bal. at 896 int. for the 3d month, - $3872.91 * We have omitted in this partial operation the several items of interest, amount, payments and balances, from the third item of interest to the 51st payment, both inclusive. The beginning and closing work of the problem which we have presented is sufficient to clearly elucidate the same. A. made 52 payments of $90 each = $4680, and 1 payment, the last, of $79.66, making a total of $4759.66, and 53 payments in 53 months or 4 years and 5 months. The interest paid was $759.66. $759.66 interest on $4000, for 53 months = 4 3 per cent per annum, as per Article 1166, page 599. Though the average annual interest is but 4.3 per cent on the $4000, the company made and B. lost 8 per cent on the successive monthly amounts that bore interest. And as the $90 monthly payments were received by the company and paid out by B. the use or interest on said payments to the company would be equal to about 4.3 per cent, thus making nearly 8.6 per cent. * BUILDING AND LOAN ASSOCIATIONS. IOO3 It will be remembered that by the United States system of partial payments, the shorter the intervals of time between payments, the greater the gain is to the party receiving the payments. 3. A Loan and Land Company effects loans or sells lands on the following conditions: 1. To charge 8 per cent interest on the monthly principal loaned, or on the amount due for land. 2. To divide the sum loaned or the amount due for land by the number of months the loan is to run, and to take notes, secured by mortgage payable in successive months, each for an amount equal to the quotient obtained by this division, plus the monthly interest on the successive principals loaned. According to this method, what would be the face of each note for a loan of $3600 for 6 years at 8 per cent? Ans. See partial operation. PARTIAL OPERATION. $3600 loaned -- 72 months = $50 partial face of each of the 72 notes. Amount at interest the first month, - - - - - - - - - - - $3600.00 Interest on same, 8 per cent, 1 month, * = º º is sº º sº * * 24.00 Amount due at the close of the first month, sº nº me tº º ºs - - - $3624.00 Face of first note, $50 + $24 = $74, - - - - - - - - - - - 74.00 Balance at interest the second month, tº ºs º ºs º º sº - - - $3550.00 Interest on same, 8 per cent, 1 month, * * * * * * * * * * 23.67 Amount due at the close of the second month, - tº tº tº * * tº gº tº - $3573.67 Face of second note, $50 + $23.67 = $73.67, sm tº º ºs º º º sº. gº 73.67 Balance at interest the third month, - ſº gº {º tº gº { } { } - - - $3500.00 Interest on same, 8 per cent, 1 month, ſº * , dº ſº. tº gº gº ºn tº tº ºr 23.33 Amount at the close of the third month, tº º tº e tº s tº tº º - - - $3523.33 Face of third note, $50 + $23.33 = $73.33, - - - - - - - - - - 73.33 Balance at interest the fourth month, - - - - - - - - - - $3450.00 And in like manner we continue the operation to find the face of the 72 notes. It will be observed that the operation involves the United States method of partial payments as elucidated in the preceding problem. To find the interest and the rate per cent of interest realized by a Loan and Investment Company on certain conditions. 1562. 1. A Loan and Investment Company loans $3000 for 5 years, at 8 per cent per annum. The interest ($1200) is added in advance, and the Whole amount, $4200, is made payable in 60 notes of $70 each, payable monthly, without interest. Allowing that the company may re-invest the monthly collections of the notes at 8 per cent simple interest, what would be the total interest received by the company, and what would be the rate per cent per annum ? Ans. $2026 total interest; 13% rate per cent per annum. NotE.—For the solution of this and the two following problems, see page 675. IOO4. soul E's PHILOSOPHIC PRACTICAL MATHEMATICS. Yºr 2. Suppose in problem 1, above, that the monthly note and interest collec- tions were re-invested at simple interest as therein stated, what would have been the respective interest gain for each year? tº 3. Suppose in problem 1, above, that the monthly collection of interest on the notes and the interest on such interest were used, at the close of each year, as a part of the dividend fund, what would be the amount of interest earned yearly 3 4. April 8, 1894, a party purchased from a real estate holder a house and lot for $7500, on the following terms and conditions: $300 was paid in cash. 36 notes of $200 each bearing 7 per cent interest were given; three of the notes were made payable in 3 months, July 8, 1894. The other 33 notes were made payable monthly after July 8, 1894. After the three notes payable July 8, and 19 notes payable monthly after July 8 had been paid, the holder sold the 14 remaining notes at 7 per cent per annum discount. Counting 30 days to the month without grace or discount day, what were the proceeds of the 14 notes, Bankers' Discount Method Ans. $3136.74. NotE.—For the solution of this problem, see page 590. 5. A party holds $2700 of stock, which pays $80 per month revenue, (35% per cent interest per annum). The stock has 27 years to run. What is the present value of the stock, the current rate of interest being 6 per cent, allowing 6 per cent interest on all the monthly payments of the revenue or income 3 Ans. $12614.02. NoTE.—For the solution of this and the following problem, see page 591. 6. Suppose in the above problem, interest is allowed only on the annual pay- ments of the monthly revenue instead of the monthly payments, what would be the present value of the stock # Ans. $12575.34. 7. A Homestead Association sold to Levin Cooper Soulé, a house and lot for $5000, on the following conditions of payment: Cash $1000; the remainder in 80 notes of $50 each, bearing 7 per cent interest and payable monthly, as the months expire, in 80 consecutive months. In this transaction, what is the rate per cent interest that the homestead makes and what per cent interest does Levin Cooper Soulé pay on the $4000? Ans. The homestead makes and Levin Cooper Soulé loses, 3.543% + 3.45% = 7% + .5512+ ſº use of inter- est on interest payments, = 7.5512+ 7%. NOTE. 1.-For the solution of this problem, see pages 592 to 596. NoTE_2.—The following are valuable works on Building and Loan Associations: How to Manage Building Associations, by Edmund Wrigley. Treatise on Building Associations, by N. C. Thompson. H. S. Rosenthal’s Manual for Building and Loan Associations. G. A. Endlish on the Laws of Building Associations. Ninth Annual Report of the Commissioner of Labor. ==". 6. wº 3% , CHECK FIGURE SYSTEM OF PROVING POSTINGS. IOO5 *- Multiples of 18. TEIE THIRTEEN CHECK FIGURE SYSTEM OF PROW- To be memorized ING POSTINGS OR TRANSFERS. or used on a card. * gº GENERAL DIRECTIONS FOR. USING THIS SYSTEMI. 13 351 689 26 364 702 *E---> ; § # 1. Separate the number when consisting of more than three figures, 65 403 741 into periods of threes. º § # 2. When there is more than one period of odd or of even periods in the 104 442 780 number, add the like periods together. 117 455 793 130 468 806 3. When the number contains less than, or only three figures, then #. # § subtract therefrom some multiple of 13 which is less than the number. ; § #: 4. When the odd period or the sum of the odd periods, is more than 13 less 195 533 871 than the even period or the sum of the even periods, then increase the odd period or ; : ; periods by adding thereto some multiple of 18, 80 as to produce a sum not exceeding 234 572 910 13 greater than the sum of the even period or periods. Then subtract the even % ; ; period from the odd and the difference will be the Check Figure. 273 611 949 g = i e g 286 624 962 5. When the even period is more than 13 less than the odd period, then add 299 637 975 some mutiple of 13 thereto 80 as to produce a sum less than the odd period and ; § 988 which, when subtracted from the odd period, will leave a remainder less than 13, 338 676 which will be the Check Figure. A SHORT PRACTICAL METHOD OF FINDING TEIE 13 CHECK FIGURE. Multiples of 13 In practice it is necessary to memorize only the first ten multiples of 13, To be memo- and to acquire quickness in seeing the products thereof when multiplied by rized. the numbers from 2 to 10. By this knowledge, the various multiples of 13 13 may be seen instantly by the mind and applied in a manner as shown in the ; following illustrative examples: 52 1st. When there is but one period, mentally see what multiple of 13 ; must be subtracted from it to leave a remainder less than 13; this remain . der is the Check Figure. 104 Thus: To find the Check Figure of 57, we subtract 52 from 57 and obtain # a remainder of 5, which is the Check Figure. The Learner should also 2d. When there are two periods, odd and even, mentally see memorize the products of the what multiple of 13, if any, must be added to the lesser period so most of these multiples by that the difference between them will be less than 13. 2, 3, 4, 5, 6, 7, 8, 9 and 10. Thus: 1st. To find the Check Figure of 27453, we mentally Thus: 65 × 3 195 : add 416 to 27, the even period, making 443, which we subtract from 78 × 4 313: 453, the odd period, and produce 10, as the Check Figure. gig 5 iss; Or 2d. To find the Check Figure of 352865, we add 520 to the 52 × 10 520 'ete even period, making 872; from which we subtract the odd period * y * and produce 7 remainder; then as 872 was larger than the odd period, we subtract 7 from 13 and produce 6 as the Check Figure. If we had added 507 to the even period, the operation would have been shorter. But to produce 507 from the table of the first 10 multiples of 13, more mental work is required than when we use 520. * * Find the Check Figure of 242,723,492,168. (891)= sum of odd periods, 734 = sum of even periods, plus 156 (2 times 78) = 890. 1 Check Figure. IOO6 SOULE's PHILOSOPHIC PRACTICAL MATHEMATICs. * Find the Check Figure of the following numbers by mental operations. No figures to be written except the Check Figure. NOS. OPERATIONS. (1) 13 13 from 13 - * , - & Check Fig. (2) | 15 13 from 15 - *- - - {{ {& (3) 29 26 from 29 -> e- 3- - {{ {{ (4) | 84 78 from 84 •º {{ {{ (5) 142 130 from 142 - º º 1 {{ {{ (6) 568 520 (10 times 52)+39=559 from 568 = {{ {{ (7) || 794 780 (10 times 78)+13=793 from 794 F % - 4% (8) 907 780 (10 times 78)+117=897 “ 907 F 1 {{ {{ or thus: 910 less 807=3, and 3 from 13 = l {{ {{ (9) 2,640 652 (10 times 65+2 in. 2d period), less 640=12, and 12 from 13, - - 44 or thus: 520-H2+ 117 =639 from 640 { % {{ (10) || 6,028 13+6=19 from 28 - º F. {{ {& (11) || 2,972 910–H 65+2=977 less 972=5 & 5 from 13 = {{ {4 or thus: 912–H52=964 from 972 F {{ {{ (12) 53,415 351 (3 times 117)+53 in 20 period=404 {{ {{ {{ 66 {{ & 4 and 404 from 415 ge -> -> (13) | 178,213 178+26–204 from 213 - ſº ſº (14) 249,007 234 (2 times 117)+7-H13=254 less 245 or thus: 234-H 7=241 from 249–8 and 8 from 13 gºs †- - - º or thus: 260—13-H17 =254 less 249 (15) || 7,210,586 |390 (10 times 39)+210, in 2d period, =600 less 593 (sum of odd periods) 1. :;i {{ 44 {{ {{ =7 from 13 - º º * = • 6 {{ {{ or thus: 351 (3 times 117)+210+26 =587 and 587 from 593 - º - s 6 {{ {{ (16) 324,808,410|734 (sum of odd periods) + 78 = 812 less even period, º tº tº- * = s 4 {{ {{ (17) | 1,540,017 | 18 (sum of odd periods) + 520 (10 times * 52)=538 from 540=2; and 2 from 13 = - 11 {{ {{ Or thus: 18+533=551 less 540 - = • 11 {{ {{ (18) 96,244,503 |599 (sum of odd periods) less (244+351) 595 from 599 - - - º - - 4 {{ {{ (19) 25,000 26 (2 times 13) less 25 (even period) - - 1. {{ {{ (20) | 1,302 312 (4 times 78)+1=313 less 302=11 and 11 from 13 - - tº- º := • 5 {{ {{ (21) 100 91 from 100 sº - - - º - * 9 {{ {{ (22) $ 3.60 |351 s & - tº- tº = 9 cf. (23) | $ 360.00 39 * * - * - es = 3 cf. (24) $ 425.00 |455+42=497 - - tº = 3 cf. (25) $ 204.50 |416+13+20=449 from 450 - = 1 cf. (26) $6028.15 (208-1-602) from 815 º = 5 cf. (27) $ 400.00 52 less 40 - - - - = 12 cf. (28) $ .43 39 from 43 - tº tº = 4 Cf. By memorizing perfectly the first 10 multiples of 13, and becoming quick in producing mentally the products thereof, by the numbers 2 to 10, any person, who can add and subtract with facility, may see mentally and instantly the successive steps which produce the Check Figure. An hour's practice per day, for a few days, on the foregoing, and on similar problems which may be assumed or taken from any book of account, will enable any bookkeeper or clerk to write the Check Figure almost at sight. ffi This knowledge of the 13 Check Figure system is of great value to all bookkeepers and to Oil Cô Iſle Il. sk CHECK FIGURE SYSTEM OF PROVING POSTINGS, ETC. roo? APPLICATION OF THE 13 CHECK FIGURE METHOD OF LOCATING ERRORS AND OF PROVING POSTINGS AND TRANSFERS. Post or transfer the following items from the Sales Book to the Ledger: Ck. SALES BOOK ITEMS. Fig. LEDGER ACCOUNTS. J. P. Hayter, tº- - $ 120 00| 1 J. P. HAYTER. D. M. Smith, - - - 2400 00 7 CK F. B. Denson, 17284 50| 9 Fig, J. C. Rimes, - - - 79| 1 $120 00 || 1 | Total, - - $19805 29, 18 D. M. SMITH. Check Fig. of the total is tº-e 5 F. 66 ** ** 18, the Sum of the Check Figures is tº 5 which proves the cor- * $2400 00 7 rectness of the addition. F. B. BENSON. By inspection, we observe that the #. respective sums of the Check Figures in $17284 50 | * the Sales Book and on the debit of the Ledger are the same, 18, and that 5 is the Check Figure of each. J. C. RIMES. This proves that the items were posted §: to the correct side of the Ledger Ac- .79 || "i" COuntS. 18 sum of C. fs. In like manner the posting of the Invoice Book, Cash Book, Journal or any other book that is used as a posting medium, would be made and the work proved by the 13 Check Figure. NotE.--When posting, set the Ledger check figures on a slip containing debit and credit check figure columns, and make this test for each book posted. TO PROVE ADDITION BY TEIE 13 CHECK FIGURE METEIOD. Pirst. Find the check figure of each of the numbers added. 2d. Find the check figure of these check figures. 3d. Find the check figure of the sum of the numbers. If the check figure of the sum of the numbers is the same as the check figure of the check figures of the different numbers, the work is correct as far as this method of proof can determine. PROBLEMI. 345 = 7 c. f. 87 = ** f \ 21 — 13 = 8 c.f Prove the addition of the follow- 4357 = 2 c. f. — 13 = 8 c. f. V 796 = 3 C. f. ing numbers by the 13 Check Figure method. 5585 = 8 c. f. V TO PROVE SUBTRACTION BY THE 13 OHECK FIGURE METEIOD. First. Find the check figures of the minuend, subtrahend and remainder. 2d. Subtract the check figure of the subtrahend from the check figure of the minuend and if the difference is the same as the check figure of the remainder, the work is correct as far as this method of proof can determine. º NOTE: 1.-Increase the check figure of the minuend by 13, when it is less than the check figure of the subtrahend. NOTE: 2. —By adding the check figures of the subtrahend and the remainder, the sum should equal the check figure of the minuend, and thus prove the work. When this sum is in excess of 13, then subtract 13 theref, on and the difference will equal the check figure of the minnend. PROBLEMIS. Prove the subtraction Minuend, - 857 = 12 c. f. $230 50 c. f. =1 of the following numbers Subtrahend, 465= 10 c. f. 87 25 C. f. = 2 by the 13 Check Figure tºmºmºsºms - gºmºmº- method. Remainder, 3.92= 2 c. f. $143 25 = 12 c. f. }12 C. f. IOO8 SOULE's PHILOSOPHIC PRACTICAL MATHEMATICS. ºr TO PROVE MULTIPLICATION BY THE 13 CHECK FIGURE METHOD, First. Find the check figure of the multiplicand, the multiplier, and the product. 2d. Find the check figure of the product of the check figures of the multiplicand and the multiplier. If this check figure is the same as the check figure of the product, the operation is correct, as far as this proof can determine. PROBLEMS, Prove the multipli-|Multiplicand, 6428 = 6 451 = 9 — as - cation of the follow- Multiplier, . 4357 = 2 } =12 C. f. 64 = 12 } =108–4 c. f. ing numbers by the 13 Check Figure method Product, 28006796 = 12 C. f. 28864 = 4 c. f. TO PROVE DIVISION BY THE 13 CEIECK FIGURE METHOD. First. Find the check figures of the divisor, dividend, quotient, and remain- der, if any. 2d. Multiply together the check figures of the divisor and quotient, and find the check figure of the product. 3d. To this check figure add the check figure of the remainder and find the check figure of this sum; if this check figure is the same as the check figure of the dividend, the work is correct, as far as this method of proof can determine. PROBLEMI. Prove the division of DIVISOR. I)IVII) END. QUOTIENT. the following number by (9 c. f.) 3207 )467582 = (11 c. f.) (145 = 2 c. f. the 1 3 Check Figure method. Remainder, 2567 = 6 c. f. 9× 2 = 18 = 5 c. f. -- 6 = 11, same as c. f. of the dividend. NOTE.-Tbere are certain errors that cannot be detected by the 13, the 11 and the 9 Check Tigure methods of proof. THE CENT ET. UNE, OR 101 METHOD FOR FINDING THE CHECK FIGURE IN POSTING. Separate the given number into periods of two figures or places each, from the right. From the sum of the odd place periods, subtract the sum of the even place periods, adding 101 to the sum of the odd place periods, when smaller than the sum of the even place periods. If the remainder contains three or more figures, proceed as before, until there remains not more than two figures. Then subtract the tens figure from the units figure, adding 10 to the units figure when the smaller, as in ordinary subtraction. The remainder, or final differential, will be the check figure required. The two places or figures for cents must always be considered the first odd period. THE MILLE ET UNE, OR 1001 METHOD FOR FINDING THE CHECK FIGURE IN POSTING. Separate the given number into periods of three figures each, from the right. Then from the sum of the odd place periods, subtract the sum of the even place periods, the difference will be the remainder, if the whole number had been divided by 1001. Then reduce the remainder by the 9, 11, 13, or 101 method for the final differential or check figure. THE 99 METHOD of FINDING THE CHECK FIGURE. Separate the number into periods of two figures from the right. Then add all the periods and reduce the sum by the same method to two figures; then reduce to one figure by adding, as for 9’s. The final result will be the check figure. THE 999 METHOD OF FINDING THE CHECK FIGURE. Separate the number into periods of three figures from the right. Then add all the periods, and reduce the sum to two figures by the 99 method, and reduce this result by the 9 method, for the final check figure. See pages 590 to 596 of Soulé's New Science and Practice of Accounts for full explanation and application of the 9 and 11 check figure systems. º: EQUATION OF PAYMENTS WITH SPECIAL FEATURES. IOO9 PROBLEM 1. Jones bought of Brown the following bills of goods on 60 days credit and 2% discount in 10 days, with the privilege of 60 days additional credit, in case the discount privilege is not availed of: Jan'y 4, '03, $300; Jan'y 26, $150; Feb'y 10, $800; Mar. 20, $1200; Apr. 1, $90; May 25, $500. If payment of this account is made May 30, '03, what is the amount due to Brown, the rate of interest being 6% Ans. $3019.73, NOTE.—See pp. 627 to 653, for full elucidation of Equation of Accounts. FIRST OPERATION.—FIRST STEP To equate the sales and find when the time expires for the 2% discount privilege. 1903. Jan'y 4. $ 300 × 0 = 0 4; 26. 150 × 22 = 3300 Feb'y 10. 800 × 37 = 29600 Mar. 20, 1200 x 75 = 90000 Apr. 1. 90 × 87 = 7830 May 25, 500 x 141 = 70500 $3040 ) 201230 (66 ds SECOND OPERATION.—FIRST STEP To find the Equated Date of the account, allow- ing 120 days credit on all sales. 1903. Jan'y 4. $ 300x120= 36000 46 26. 150 × 142= 21300 Feb'y 10. 800 × 157 = 125600 Mar. 20. 1200 x 195– 234000 Apr. 1. 90 × 207 = 18630 May 25. 500 × 261 = 130500 $3040 ) 566030 (186 ds. 566030 ds. -- $3040 = 186 days after Jan'y 4, '03, gives July 9, '03, as the Bquated Date for the payment of the account without discount. NotE.-By adding 120 days to the 66 days, equated date of the sales, as shown in the sec- ond step of the first operation, we could have found the 186 days credit on the whole account, without making the second operation. SECOND STEP OF FIRST OPERATION. 201230 ds. -- $3040 = 66 days. 66 ds. equated date of sales after Jan. 4, 1903. credit on each sale. discount privilege. 60 ds. 10 ds. 136 ds. after Jan'y 4, '03, is May 20, '03, the Equated Date for the pay. ment of all the purchases at 2% discount. But as pay- ment was not made until May 30, '03, the 2% discount is not allowed, but 60 ds. additional credit is allowed in place thereof. Ea:planation.—From the above we see that the discount privilege ceased May 20, '03; that the equated date of the account, allowing 120 days credit on each sale, is July 9, '03; and that the account was settled May 30, '03. Hence Jones is entitled to interest on the amount purchased from May 30 to July 9, 03, which is 40 days. SECOND STEP OF SECOND OPERATION To find the interest and balance due to Brown. $ 3040 $3040 Amount. 60 | 40 20.27 Int. | $20,263=int. $3019.73 due Brown. IOIO SOULE's PHILOSOPHIC PRACTICAL MATHEMATICS. * PROBLEM 2. A. sold to B. the following invoices of goods: May 10, $500; May 20, $700; June 1, $400, July 3, $1000. The terms are 60 days credit and 2% discount in 10. days, or 60 days additional credit, in case the 2% discount is not availed of. B. desires to settle the account on July 20, 1904. What amount does he owe A. on this date 3 AnS. $2536.96. 1st Operation to equate the sales and find the 2d Operation to find amount due July 20, '04, date that the 2% discount privilege ceases. when the settlement was made. 1904. By the above work we see that the May 19. 3500 x 9 = 0 - discount privilege extended to Aug. 15, me". * . § = §. '04, and as the settlement was made July 3. 1000 × 54 = 54000 July 20, '04, B. is entitled to the 2% dis- count, and to the interest on the balance. $2600 ) 69800 (27 days | from July 20, to Aug. 15, '04. Thus: To this 27 days credit after May 10, '04, we add 60 days credit on each sale, and also 10 days credit for dis’nt option and thus — & 2 TAS ſº 2.548 Balance. produce 97 days credit after May 10, '04, § 2.) g which gives Aug. 15, '04, as the date OIl 11.04 hº º which the 2% discount privilege expires. o, "U4, ays at 6%. $2600 Amount of purchases. 52=2% discount. * NOTE. –By adding 120 days credit to the 95 above 27 days, we produce 147 days credit after $2536.96 Net, balance due A. July May 10, which gives Oct. 4, '04, as the equated 20, '04. date of the whole account, which, however, is not required in this problem, as the settlement was made within the 2% discount limit. PROBLEM 3. Suppose in the preceding problem that B. had settled the account August 28, '04, what amount would have been due to A.3 Ans. $2583.97. Explanation.—From the work of the preceding problem, we see that the privilege of 2% dis- count expired Aug. 15, '04. Then, by the terms of the contract, B. was entitled to 120 days credit. which gave Oct. 4, '04, as the equated date of the account, without considering the element of 2% discount. Hence, if he settles the account Aug. 28, '04, he is entitled to the interest thereon from Aug. 28, to Oct. 4,-37 ds, a 6% = $16.03 from $2600 = $2583.97 balance. PROBLEM 4. The firm of E. P. Singeltary, sold to A. Barbier & Co., the following invoices of merchandise : 1904. The terms for each sale are as follows: 60 days. Feb. 19, $372.30 credit and 2% discount in 10 days, or 60 days additional “ 23, 53.41 credit in case the 2% discount privilege is not accepted Mar. 3, 181.39 by the purchaser. “ 16, 365.79 A settlement of this account was made Sept. 1, 1904. “ 25, 240.25 (1). What is the date on which the 2% discount privi- “ 29, 109.12 lege expires 3 (2). What is the average date of the Apl. 7, 57.09 account ? (3). What is the amount due to E. P. Sin- “ 20, 130.00 geltary on settlement, allowing 6% interest ? e-sºmsºmº sºmºsºmº Ans. 1, May 22/04. Ans. 2, July 11/04. $1509.35 Ans. 3, $1522.43. :}: EQUATION OF PAYMENTS WITH SPECIAL FEATURES. IOIII PROBLEM 5. Smith sold to Jones on 60 days credit, with the privilege of 3% discount in 10 days, the following invoices of goods: May 10, $500; May 20, $700; June 1, $400; July 3, $1000. Jones has paid on account, as follows: June 9, $800; Aug. 25, $900. A final settlement was made Sept. 15, '04. Allowing interest at 6%, and 360 ds. as an interest year, what does Jones owe Smith on final settlement 3 Ans. $877.57. FIRST SOLUTION, OPERATION Or thus : 20 OPERATION. To find the Equated Date of Sales and the time tº e 1904 limit of the di t privilege. o loo." O e discount privilege May 10. $ 500 × 70 – 35000 May 10. $ 500 × 0= 0 4 20. 700 x 80 = 56000 {{ 20. 700 × 10= 7000 June 1. 400 x 92 = 36800 June 1. 400 × 22= 8800 July 3. 1000 x 124 = 124000 July 3. 1000× 54= 54000 - $2600 ) 251800 (97 ds. $2600 ) 69800 (27 ds. af. ter May 10. after May 10, '04, = August 15, '04, as the Equated Date of the sales, allowing 60 days credit and 10 days discount, privilege. 27 ds. after May 10, '04. 60 ds. Cr. On each sale. 10 ds. Dis. privilege. 97 ds. after May 10, = Aug. 15, '04, = limit of dis. privilege. OPERATION TO EQUATE THE ACCOUNT, BOTH DEBIT AND CREDIT ITEMS. 1904. 1904. May 10. $ 500 x 60= 30000 June 9. $800 x 30 = 24000 4 20. 700 × 70= 49000 Aug. 25. 900x107 = , 96300 June 1. 400x 82= 32800 July 3. 1000×114=114000 $2600 225800 $1700 120300 1700 120300 $ 900 ) 105500 (1173 ds. after May 10, '04, = Sept. 4, '04, as the Equated date or time for the payment of the balance. By inspection, we see that the $800 paid June 9, was paid before the expi- ration of the discount privilege, August 15, and hence the debtor is entitled to discount thereon, which, at 3%, is $24. This deducted from the $900 balance of account, leaves a net balance of $876 due Sept. 4%, ’04. As settlement was not made until Sept. 15, '04, the creditor is therefore entitled to interest on the net balance, from Sept. 4%, ’04, to Sept. 15, '04, 10% days at 6% = $1.57 interest. $876 + $1.57 = $877.57, amount due Sept. 15, '04. NOTE: 1. –In practice, it is not the custom to use the exact fraction of a day, but to state the day to the nearest unit. In this problem we have used the fraction of a day in order to obtain the exact interest, so as to compare the same with the interest produced by the solution of the problem, by the Account Current and Interest Account Method, which is given on the following 3,976. pag NOTE: 2. —Since the purchaser is allowed 3% discount on all payments paid within the dis- count time privilege, his discount credit on the $300 payment would be in strict accuracy and justice, $24.74, i. e. in the ratio $100 credit for every $97 paid. But this degree of accuracy is not observed in calculations of this character, (Continued. ) IOI 2 SOULE'S. PHILOSOPHIC PRACTICAL MATHEMATICS. jºr NOTE 3.—Problems containing conditions of credit and of discount, like the five preceding problems, are of quite recent origin, and have given rise to considerable discussion among account- ants. These discussions result from the different interpretations given to the conditions, and these interpretations are often reached because a solution in accordance there with will give a financial result in favor of the party holding such an interpretation. As an example of this point note the following operation of the above problem. VARIATION OF PROBLEM. EQUATION OF THE ACCOUNT ALLOWING 70 DAYS CitFDIT ON EACH ITEM OF PURCHASE. 1904. 1904. May 10, $ 500 x 70– 35000 June 9. $800 × 30 = 24000 64 20. 700 × 80= 56000 Aug. 25. 900 × 107 = 96300 June 1. 400 × 92= 36800 July 3. 1000 × 124=124000 $2600 251800 $1700 120300 1700 120300 $ 900 ) 131500 (146 ds. after May 10, '04, = Oct. 3, '04, as the Equated Date for the payment of the balance. This equation is made on the interpretation that the debtor has really 70 days credit on each item of purchase, because he had 10 days discount privilege. This, we think, is an erroneous construction of the conditions of the contract. The 10 days discount privilege were conditioned on the payment of cash as the respective sales came due. But as the cash payments were not made, the 10 days privilege cannot, in justice, be claimed. A difference of opinion exists among accountants and also among business men, regarding the proper manner of adjusting accounts like, or similar, to the above. In some cases, special agreements between the parties, and in Some cases the custom of the trade will govern the method of adjustment or settlement. SECOND SOLUTION OF PROBLEM 5. BY THE PARTIAL PAYMENT METHOD, OR BY MAKING PERIODIC PARTIAL SETTLEMENTS CORRESPONDING WITH THE DATES OF PAYMENTS, 1904. 1904. May 10. $500×60 = 30000 June 9. $800 × 30=24000 “ 20. 700 × 70= 49000 June 1. ‘ 400 x 82= 32800 $1600 111800 800 24000 $ 800 ) 87800 (1093 =110 ds. after May 10 = Aug. 28, '04. 3% dis’t, 24 Balance, $776 1904. 1904. June 28, $ 776 × 80=62080 Aug. 25. $900 x 77=69300 July 3. 1000x84=84000 $1776 146080 900 69300 $ 876 ) 76780 (87 ds. after June 9, '04, = Sept. 4, '04. . NotE.—The $776 balance from the first equation is not due until August 28, '04. Hence, as the Focal date 1s June 9, there is 80 days credit on this balance. By this method of solution, we obtain the same equated date as in the first solution, Sept. 4, 1904. The remaining part of the operation, to find the balance due to the creditor on the day of settlement, is performed in the same manner as in the first solution. * * Account cURRENT AND INT. Account WITH SPECIAL CONDITIONS IOI3 THIRD SOLUTION BY THE ACCOUNT CURRENT AND INTEREST ACC’T METHOD, joNEs in Account Current and Interest Account at 6% to SEPT. 15, '04, with SMITH. 60 ds, credit is allowed on all sales, and 3% discount is allowed on all payments made within 10 days after the expiration of the 60 days credit. * I)”, Or. When When Ds || Prodc't| Amt. Ds.|Prod'ct Amt. due, due. 1904. * 1904. ººm--> May |10|Mdse. 60 ds. &|July 9, 68|| 34000|50000 June 9|Cash, - - June 9| 98|| 78400 80000 3% dis. in 10 ds. g “ 20Mdse. “ “ || “ |19||58||40600|70000|Aug. 25. “ tº º Aug.25 21|| 18900 90000 June 1. “ { % { % ** 31|| 46|| 18400|| 40000 July 3. “ & 4 ‘‘ ||Sept | 1 14|| 14000||100000 Dr. Int. on $1. To Bal. of ds on 107000 *== $1, which di- 97.300 Ds. Int, on $1. 97300 vided by the *-*E* 6% int. divisor 9700 6000, gives int. $1.617 Less int. on 2400 $24 for 10; Sept|4}|By3%dis.on $800 4. ds. = .043 Sept. 7; By ićiance due 87.757 *º-º- 157 the creditor, tº-º Int. due the *E*m. I ºm- creditor $1.574 2601|57 2601|57 Explanation—By this method of working the problem, interest is computed on all debit and credit items from the time due to date of settlement; hence interest has been computed on the $900 balance of account, from the EQUATED DATE to the date of payment, which is 105 days shown in the solution above by the equation method. This $900 balance of account is entitled to a credit of $24, the same being 3% discount on $800 paid within the discount period, thus leaving a net debit balance of $876, which bears interest from the equated date to the date of settlement. Now, as interest has been computed on $900. instead of $876 from the equated date to the date of settlement, it is evident, therefore, that the interest produced by this method of solution is in excess to a sum equal to the interest on the amount of DISCOUNT CREDIT, (in this case $24) from the equated date to the date of settlement. This interest, in this problem, amounts to $,043, and is deducted from the full amount of interest, $1.617, as is shown in the operation. NotE 1.--To find the limit of time of the discount privilege, equate the sales as shown in the preceding solution. NOTE 2.-For a full elucidation of Equation of payments, see pages 627 to 653. For a full elucidation of Accounts Current and Interest Accounts, see pages 653 to 670, le renew the tharge, book must be broughl io ſhe desk. D0 NOT RETURN B00 ($ 0N SUNDAY DATE DUE Form 7079 a e- _Tºº!!!Nºt It is the HIGHEST STANDARD of AUTHORITY for BOOK-KEEPERS and BUSINESS MEN. Ate Á SCIENCE AND …ſº PRACTICE OF Tº 726 Pages, 8x11 Inches, F: { Price, $4.00. (, CCOUNTS Filliºl||Ili iſ 'I'll turnºullur !!! III;: li III, i lºi II iſ ºff Joint Stock Company Book-keeping and Expert Accounting. INDEX AND SPECIMEN PAGES SENT FREE. i —J [E - - -* ---2. *N. O SIMILAR WORK ever published contains such a Revelation of Stock Comº and Expert Accounting, Complex Partnership Adjustments, Short Methods, Claf Accounts, Locating §3. Proof of Ledgers, Higher Work of Closing Books, f Commercial Firms to Stock Companies, Liquidating Estates, Duties of Auditors.” Commissioners, Etc. ,' It is an Exhaustive Treatise, replete with Advanced Thought and New Y Important Questions pertaining to Accounts and Complex Financial Affairs’ any other similar treatise. It is no catch penny work of a single thought; it covers, the ly It is the ROYAL ROAD for all who would possess the highest.” loftiest planes in the realms of Accounting. / - It is the ROYAL ADVISOR for business men on questions of y’ and Liquidation of Partnerships and Corporations, and also on V3' JOINT STOCK BOOKKEEPING embraces 187 pages, sh9 ments of writing, in full. . The Opening, Conducting, Čonsoli’ Companies are presented, and Treasury Stock, Préferred/ Capital, Etc., are discussed. It contains forms of Rooks for New Business Pr; Banking, Commission, Trades, Farming, Hotelling,” Department System, Cost Accounting, Columnar Sy” READ and HEED. I the Location of Errors by the Reverse Posting,/ Charters, By-Laws, Etc. tº - 2 bu ſl ſl-ſl ſl-ſl ſl ſl. u / This book contains forty years of study, experience ap largest, highest and best work ever published on the sup’ - Z en tº = ~ *. / ,”2. - §§ º- zº §§§º-NYR= 2 ~E. §. %; § … ==}\ $ §§ § ...” #Aſº Fº tº E: - ... .- - §s §§§3. * t §§§ R- - ... • -- § ###" ~. HIS work J. w ºl §§: A: of Account - - - JIl º Auditing, and It presſ . - ations arising in - -- . ... before publish, cors often made * - . . - A (alsehood it. y == - - - - ARLES STREET_2 ~. ...EANS, LA, SoulB' - ..., $2.00. SO’ - - - - - - on), $1.00. ſae) {--. .-.-.-.--- ' !", ºſſ. №ººººººººº,,,。、、。ºſº ºſſºſººººº Œ º : * · · · * *** ¿?,t. -·) (, ***·~~ ~ ~ .* -ae),